You are on page 1of 722

é *

>

" CHEMISTRY
CLASS-XI tn
ps 4 |
PART 1 & I
STRICTLY ACCORDING TO THE NEW SYLLABUS ‘elAEytra
INDIA'S FIRST SMART BOOK Dr. $.P. Jauhar
According to new syllabus prescribed by Central Board of Secondary Education (CBSE), New Delhi and State
Boards of Uttarakhand, Karnataka (Ist Year PUC), Chhattisgarh, Jharkhand, Punjab, Haryana, Himachal,
Kerala, Mizoram, Meghalaya, Nagaland, Assam, Manipur and other States following CBSE & ISC curriculum.

MO DE RN S

For Class XI
strictly ee
: ccordan
en the Lates) wus PART-II
yes and Sa
Guide sued by ee
n.c.E R.1

By
Dr. S.P. JAUHAR
Formerly Professor of Chemistry
Department of Chemistry
Panjab University, Chandigarh

3 valUd... REVISED AND UPDATED EDITION 2019-2020

MODERN PUBLISHERS
(Cad Kole 6 [or] sme) @IUT-l
llavam (> duelele)
<9
OUR ADDRESSES IN INDIA
New Delhi: MBD House, Gulab Bhawan, 6, Bahadur Shah Zafar Marg Ph. 23317931, 23318301
Mumbai: A-683, T.T.C. Industrial Area, M.1.D.C. Off. Thane-Belapur Road, Navi Mumbai Ph. 32996410, 27780821, 8691053365
Chennai: No. 26 B/2 SIDCO Estate, North Phase, Pataravakkam Ambattur Industrial Estate, Ambattur Ph. 26359376, 26242350
Chennai: Plot No. 3018, Old Y Block, 3rd Street, 12th Main Road, Anna Nagar West, Chennai Ph. 23741471
Kolkata: Satyam Building, 46-D, Rafi Anmed Kidwai Marg Ph. 22296863, 22161670
Jalandhar City: MBD House, Railway Road Ph. 2458388, 2459046, 2455663
Bengaluru: 124/31, 1st Main, Industrial Town (Near Chowdeshwari Kalyan Mantap),
West of Chord Road, Rajajinagar Ph. 23103329, 23104667
Hyderabad: 3-4-492, Varun Towers, Barkatpura Ph. 27564788, 9985820001
Ernakulam: Surabhi Building, South Janatha Road, Palarivattom Ph. 2338107, 2347371
Nagpur: Plot No. 231, Lendra Park, Behind Krims Hospital, Ramdaspeth, Nagpur Ph. 0712-2447306, 2447330, 7410017938
DOODO
DOOOOODO
Ahmedabad: Godown No.10, Vedant Prabha Estate, Opp. ONGC
Pumping Station, Sarkhej Sanand Road, Sarkhej Ph. 26890336, 7600024542
Cuttack: Badambadi, Link Road Ph. 2367277, 2367279, 2313013
Guwahati: Chancellor Commercial, Hem Baruah Road, Paan Bazar Ph. 7637014051
Lucknow: 173/15, Dr. B. N. Verma Road, Old 30 Kutchery Road Ph. 4010992, 4010993
Patna: Ist Floor, Annapurna Complex, Naya Tola Ph. 2678732, 2678994
Bhopal: Plot No. 137, 138, 139, Sector-l, Special Industrial Area, Govindpura Ph. 2581540, 2601535
Goa: H. No. 1644, Plot No. 100, Kranti Nagar, H.B. Colony, Nr. Nana Nani Park, Porvorim, Tal: Bardez, Dist. North Goa
DOOOODO
Ph. 0832-2413982, 7028912261
C) Jaipur: C-66A, In front of Malpani Hospital, Road No. 1, V.K. Industrial Area, Sikar Road Ph. 4050309, 4020168
C) Raipur: Behind Aligarh Safe Steel Industries, Vidhan Sabha Road, Avanti Bai Chowk, Lodhi Para Pandri Ph. 2445370, 4052529
C) Karnal: Plot No. 203, Sector-3, HSIDC, Near Namaste Chowk, Opp. New World Ph. 2220006, 2220009
Cl Shimla (H.P.): C-89, Sector-l, New Shimla-9 Ph. 2670221, 2670816
C) Jammu (J&K): Guru Nanak College of Education, Jallo Chak, Bari Brahmana Ph. 2467376, 9419104035
C) Ranchi (Jharkhand): MBD Group, Shivani Complex, 2nd Floor, Jyoti Sangam Lane, Upper Bazar Ph. 7260813703, 7260813710
C) Sahibabad (U.P.): B-9 & 10, Site IV, Industrial Area Ph. 3100045, 2896939
C) Dehradun (Uttarakhand): Plot No. 37, Bhagirathipuram, Niranjanpur, GMS Road Ph. 2520360, 2107214
DELHI LOCAL OFFICES
Q) Delhi (Shakarpur) : MB 161, Street No. 4 Ph. 22546557, 22518122

C) Delhi (Daryaganj): MBD House, 4587/15, Opp. Times of India Ph. 23245676
QO Delhi (Patparganj): Plot No. 225, Industrial Area Ph. 22149691, 22147073

MODERN'S OUTSTANDING TEXTBOOKS FOR JEE-Main, JEE-Advance


FOR CLASS XI & Medical (NEET)
Modern's abc + of Physics Modern's abc of Objective Physics
Modern's abc of Practical Physics
Modern's abc of Objective Chemistry
Modern's abc + of Chemistry
Modern's abc of Objective Mathematics
Modern's abc of Practical Chemistry
Modern's abc of Objective Biology
Modern's abc + of Biology
Modern's abc of Practical Biology KIND
ATTENTION
: DEAR CUSTOMER
Modern's abc + of Mathematics To avoid duplicates, a HOLOGRAM has been pasted
Solutions of Modern's abc of Mathematics on the cover of this book. If this hologram is missing,
; _ please do not purchase the book as it is likely to be
Modern's abc of Computer Science C++ a duplicate book. Any such duplicacy if noted may
please be intimated to the Publishers.

| We are committed to serve students with best of our knowledge and resources. We have taken utmost care and paid much attention
| while editing and printing this book but we would beg to state that Authors and Publishers should not be held responsible for unintentional
| mistakes that might have crept in. However, errors brought to our notice shall be gratefully acknowledged and attended to.

I © All rights reserved. No part of this publication may be reproduced, stored in a retrieval system, or transmitted in any form or byany means, |
| electronic, mechanical, photocopying, recording, or otherwise without the prior written permission of the publisher. Any breach will entail legal |
| action and prosecution without further notice.
| _Lessee veces eS sear Ee ee Fen neraeeEWONT revs vn eve ee ON esTE ne NPNEN Sse RET SPS ese E NNN SREN Fone ORDO ay more Slee RRS eR nae meer rem nee ee era ene ee ane Seen eweee

Published by: Balwant Sharma


MODERN PUBLISHERS
MBD House, Railway Road, Jalandhar
. E info@mbdgroup.com
Printed at: |§ M.GULAB SINGH & SONS We aawanbdciolicecoh
B-5/14, Site IV, Industrial Area Sahibabad (U.P.) Toll Free No. : 1800 200 2233
CONTENTS)
PART-II

Redox Reactions 8/1 — 8/79

Hydrogen 9/1 — 9/59

10. s-Block Elements (Alkali and Alkaline Earth Metals) 10/1 — 10/61

EL, Some p-Block Elements 11/1 — 11/65

11A. p-Block Elements 11A/1 — 11A/61

12. Organic Chemistry: Basic Principles and Techniques 12/1 — 12/160

13. Hydrocarbons 13/1 — 18/152

14. Environmental Pollution 14/1 — 14/44

e MOCK TEST —2 1-2

e MOCK TEST — 3 (According to Completely CBSE Pattern) 3-4

e Solution File (Unit Practice Tests and Mock Tests) 5-9

e Appendices 11-23

e Logarithm and Anti-logarithm Tables (t) —(v)


OBJECTIVES
Building on..... Assessing..... Preparing for Competition.....
@ Understanding Text 1 @ Quick Memory Test with ¢ Topicwise MCQs 63
¢ Conceptual Questions 42 Answers 57 @ Competitive Examination Qs
CHAPTER SUMMARY & QUICK °* HOTS &AdvanceLevel —=§ > AIPMT & Other State Boards’
CHAPTER ROUND UP 44 Questions with Answers 58 — Medical Entrance 65
NCERT FILE REVISION EXERCISES 60 > JEE (Main) & Other State Boards’
@ In-text Qs & Exercises with - Hints & Answ des for Engineering Entrance 66
none 4G Revision Exercises 62 | |
| > JEE (Advance) for ITT Entrance 69
nas oe UNIT PRACTICE TEST 79 * | RPE E AS
Answers & Solutions (Subjective) 54 (Objective Questions) 72
¢ Hints & Explanations for Difficult
Questions 74

Vee oxidation and reduction reactions or redox reactions are very


common in our daily life. A large number of phenomena both
physical as well as chemical and biological are concerned with
c redox reactions. Chemical reactions like burning of coal,
sulphur, cooking gas, rusting of iron, generation of electricity,
digestion of food, etc are examples of redox reactions. Many
Conventional current industrial processes such as electroplating, production of
\ caustic soda, potassium permanganate, extraction of metals
like sodium, iron and aluminium, etc are also common
examples of redox reactions. In fact, redox reactions keep
| us alive. They help to capture the energy of the sun by the
photosynthesis process by plants which is used by us.
Recently, environmental issues such as hydrogen economy
_ i.e., use of liquid hydrogen as fuel and development of ozone
eran
hole are considered as redox phenomena.
In the present unit, we shall classify the reactions into
two classes, oxidation and reduction reactions and discuss
recreate
ert ele ctr och em ica | ce ||c

OXIDATION AND REDUCTION REACTIONS


Classical Concepts of Oxidation and Reduction
Oxidation
According to classical (earlier) views, oxidation is defined as
the addition of oxygen or any other electronegative element or removal of hydrogen or any other
electropositive element.
For example, the oxidation reactions are :
(i) he Co ee
(ii) 2Mg + 0, ——> 2Mg0 [“0u#on oF oxygen
(iit) Zn+S ——> Zn5 — ea
(iv) Mg + Cl, ——> MgCl,
MODERN'S abc + OF CHEMISTRY-XI

The above reactions involve addition of oxygen or electronegative element (5 and Cl,) and therefore, are
oxidation reactions.

(v) 2H,S +O, ——— 2H,0 + 28 Removal of


hydrogen
(vi) MnO,+ 4HCl ——> MnCl, + Cl, + 2H,O
Removal of
(vil) 2KI + Cl, ——-> 2KClI + I, electropositive
element
(vit) 2K, | Fe(CN),| + H,O, ——> 2K, [Fe(CN),] + 2KOH

These reactions involve removal of hydrogen or In these examples, reactions involve removal of
some electropositive element (potassium) and, oxygen or electronegative element (chlorine) and
therefore, are oxidation reactions. therefore, are reduction reactions.
In these reactions, the substances shown coloured The substances shown coloured have undergone
have undergone oxidation. reduction.
Oxidising Agent or Oxidant Reducing Agent
A substance which gives oxygen or removes
A substance which provides hydrogen or
hydrogen is called an oxidising agent or removes oxygen is called a reducing agent or
oxidant.
reductant.
For example, in examples (7) and (ii), O, is an
oxidising agent because it causes the oxidation of For example, in examples (i) and (ii), H, and
C and Mg. Similarly, in reactions (iii) and (iv) 5S H.,S act as reducing agents because these cause
and Cl, act as oxidising agents because they add reduction of Cl, and Br,. Similarly, in reactions
electronegative element to Zn and Mg. Similarly, (iii) and (iv), Fe and SnCl, act as reducting agents
Cl, and MnO, act as oxidising agents in reactions because these add electropositive elements Fe and
(v) and (vi) because they remove hydrogen while, Cl, Hg. Similarly, in reactions (v) and (vi), H, and CO act
and H,O, act as oxidising agents in reactions (vii) and as reducing agents because these remove oxygen
(vill) because they remove electropositive element from CuO and Fe,0O,, respectively. In reactions
(potassium). (vii) and (viii), H, and Na act as reducing agents
because these remove electronegative element (Cl)
Reduction
from FeCl, and SiCl,.
According to classical concept, reduction is
defined as Oxidation-Reduction Reactions are

the addition of hydrogen or any other Complementary : go side by side


electropositive element or removal of oxygen Oxidation-reduction reactions are complementary
or any other electronegative element. reactions. Whenever any substance is oxidised,
For example, the reduction reactions are : another substance is always reduced at the same time
(Z) Cl,+H, — 2HCl and vice versa. In other words, the oxidation reduction
Addition ofhydrogen reactions always occur simultaneously 1.e., they always
(it) Br, +H,S — >/2HBr+8
70 hand in hand or side by side. For example, consider
Addition of
(iit) 2FeCl, + Fe —->_ 3FeCl, electro- the reaction :
. at | positive
(tv) 2HgCl, + SnCl,—>Hg.Cl, + SnCl, ae 2HgCl,+SnCl, —— > Hg,.Cl,+5nCl,
In the above examples, reactions (1) and (ii) involve In this reaction, stannous chloride (SnCl,) has
addition of hydrogen and reactions (iii) and (iv) involve been oxidised to stannic chloride (SnCl,) because
addition of electropositive elements (Fe and Hg) and of addition of electronegative element chlorine to
therefore, are reduction reactions. Similarly, it. At the same time HgCl, is reduced to Hg,Cl,
(v) CuO + H, ——~ Cu + H,O because of the addition of mercury to mercuric
Removal of
chloride. Thus, SnCl, is oxidised by HgCl, and HgCl,
(vt) Fe,O, + 3CO — 3 2Fe + 38CO, oxygen
is reduced by SnCl,. Hence, HgCl, acts as an oxidising
(vit) 2FeCl,+ H, ——~>2FeCl, + fee of agent and SnCl, acts as a reducing agent in this
electronegative reaction.
(vit) SiCl, + 4Na ——> 81+ 4NaCl element

WWW.JEEBOOKS.IN
REDOX REACTIONS

Thus, oxidation and reduction reactions are Na —— Nat+e7


collectively called redox reactions. Some more Mg —— Meg + 2e-
examples of redox reactions are given below : Sn*+ ——> Sn* + 2e-
fn Oxidaner Fe*+ —— > Fe* +e-
(it) Loss of electrons resulting in decrease of
2H, +0, —— 2H,0 negative charge.
(R.A.) (O.A.,) MnO,* ———> MnO, +e
[Fe(CN),|* ——» [Fe(CN),]* + e-
| Reduction
2017 CL, + 2e7
Zz Oxidation
S7- ———> § + 2e-
Reduction . According to electronic concept,
Fe,0, + 2Al oe Al,O, + 2Fe
oe (R.A.) reductionis
a process which involves gain of electrons by
Reduction an atom or group of atoms.
Due to gain of electrons there is increase of
a Oxidation |
negative charge or decrease of positive charge of an
atom or ion undergoing reduction. For example,
H,S + 2FeCl,—— 2FeCl, + 2HC1+S
(RA) (O.A) | (1) Gain of electrons resulting in decrease of
positive charge.
Reduction | Fe*+ + e- ———> Fe?+
Sn*+ + 2e- ———> Sn?
Sb*+ + 2e- ———> Sb*+
(ii) Gain of electrons resulting in increase of
H,S + Cl, ——~ 2HC1+S
negative charge.
(R.A.) (O.A,) | Cl,+ 2e- ——> 2Cl
| Reduction S +2e- ——— S*
MnO, + e- ——— Mn0O,*
Oxidation
p wr’ [Fe(CN),]*- +e ——— [Fe(CN),]*
MnO, + 4HCl ———> MnCl, + Cl,+H,O
OXIDATION-REDUCTION REACTIONS
(O.A.) (R.A.) | AS ELECTRONS TRANSFER REACTIONS
Reduction As already learnt, oxidation process involves loss
of electrons whereas reduction process involves gain
Here O.A. and R.A. stand for oxidising agent of electrons. During transfer of electrons in a reaction,
and reducing agent respectively. the species which loses electrons is said to be oxidised
and the species which gains electrons is said to be
Electronic Concept of Oxidation and Reduction
reduced. Since there cannot be a net loss or gain of
Oxidation and reduction reactions are now used
electrons in a chemical reaction, this means that all
in a much broader and general sense. These are
chemical changes involving the loss and gain of
defined in terms of electron transfer between the
electrons occur simultaneously l.e. one species loses
reactants, known as electronic concept of oxidation and
reduction. electrons in oxidation while the other gains electrons
during the reduction. Thus, both these changes take
Oxidation. According to electronic concept,
place side by side 1.e., oxidation takes place at the
oxidation is
cost of reduction and vice versa.
a process which involves loss of electrons by an
atom or group of atoms. In other words, oxidation and reduction reactions
As a result of loss of electrons, there is increase in vo hand in hand. This may be illustrated by the
positive charge or decrease in negative charge of the following reactions:
atom or ion undergoing oxidation. 4Na(s) +O,g) —— 2Na,O0(s)
For example, 2Na(s) + Cl(g) —— 2NaC\l(s)
(1) Loss of electrons resulting in increase tn positive
2Na(s) +S(s) —— Na,d(s)
charge.
MODERN'S abc + OF CHEMISTRY-XI

All these reactions are redox reactions because in the number of electrons gained during reduction half
each of these reactions sodium is oxidised due to reaction, the two half reactions are multiplied by
addition of either oxygen or more electronegative suitable integers so that when the two half equations
elements such as chlorine or sulphur to sodium. are added, the electrons get cancelled out in the final
Simultaneously oxygen, chlorine and sulphur are redox reaction. For example,
reduced because electropositive element sodium has Na(s) —> Nat+e|]x4
been added to each of these. We have also learnt in Oxidation half reaction
chemical bonding that sodium oxide, sodium chloride O,(g) + 4e7 —> 2)-
and sodium sulphide are ionic compounds and may be
Reduction half reaction
written as :
Sodium oxide : (Na*),O7-, Sodium chloride : 4Na(s) + O,(g) —~> 2(Nat),O% or 2Na,O
Na* CI, Sodium sulphide : (Na*), 82, Overall redox reaction
These ionic charges suggest that the above redox Similarly, we may write other reactions as :
reactions may also be written as : Na(s) — Nat+e'|x2
Loss of 4e~ : oxidation Oxidation half reaction
Clg) + 2e—-§ —> 2Clr
4Na(s) +O,(g) Mmm 2(Na*), O%(s) Reduction half reaction

es | 2Na(s) + Cl.(g)—> 2Nat*tCIl(s) or 2NaCl


Gain of 4e— : reduction Overall redox reaction
Loss of 2e— : oxidation Na (s) — Nat+e |x 2
Oxidation half reaction
S(s) + 2e7 —> S*
2Na(s) + ClL(g) —— 2Nat* CI (s)
Reduction half reaction
|
Gain of 2e~ : reduction 2Na(s) + S(s) —— (Na*), S*(s) or Na,s(s)
Loss of 2e- : oxidation Overall redox reaction

In all the above reactions, electrons are transferred


2Na(s) +S(s) ———3 (Na*), S$(s) from sodium to O,, Cl, and 8. Thus, oxidation-

|S reduction t.e., redox reactions are regarded as


electron transfer reactions. Moreover, we observe
Gain of 2e~ : reduction
that O,, Cl, and 5 have gained electrons only if there
For the sake of convenience, each of the above
is another substance which can donate electrons. This
redox reaction may be considered as the sum of two
means that reduction can occur only if oxidation is
half reactions; one involving loss of electrons 1.e.
occurring side by side and vice versa. In these
oxidation and the other involving gain of electrons
reactions, sodium loses electrons and has been oxidised
i.e. reduction as
to sodium ion, Nat. This is brought about by the
Na(s) —m—— Nat+e
acceptance of electrons by oxygen, chlorine or sulphur,
Oxidation half reaction which get reduced. Thus, oxygen, chlorine and sulphur
O, + 4e— »-——> 202- bring about the oxidation and are called oxidising
Reduction half reaction agents or oxidants,
These two half reactions are called oxidation half Thus, according to electronic concept,
reaction and reduction halfreaction respectively. The
oxidising agent is a substance which can accept
sum of the half reactions give the overall reaction. It
one or more electrons.
may be noted that the overall reaction is so written
that the electrons are balanced i.e., the number of Similarly, in the above example, oxygen, chlorine
electrons lost and gained become equal. This means and sulphur are reduced to oxide ion (O*-), chloride ion
that the two half reactions are simply added if the (Cl-) and sulphide ion (S*>) respectively. This has been
number of electrons lost during oxidation half reaction possible by furnishing electrons by sodium. Thus,
is equal to the number of electrons gained during sodium is areducing agent or reductant. Thus,
reduction half reaction. But if the number of electrons reducing agent is a substance which can give
lost during oxidation half reaction are different from one or more electrons.

WW.JEEBOOKS ||
REDOX REACTIONS

the bottom of the beaker.


Oxidation is a process in which one or more (iit) The blue colour of the solution gradually fades
electrons are lost. away.
Reduction is a processin which oneor more electrons (iv) The reaction is exothermic and proceeds with
are gained. the liberation of heat energy.
Oxidising agent or oxidant is a substance which (v) The solution remains electrically neutral.
can accept one or more electrons. Let us try to explain the above reaction. In aqueous
Reducing agent or reductant is a substance which solution, CuSO, dissociates into Cu?* and SO,” ions.
can give one or more electrons. When a strip of zinc is dipped in CuSO, solution, zinc
In aredox reaction, oxidising agent is reduced by metal loses electrons and gets oxidised whereas Cu?+
ions accept electrons and get reduced. Since zinc is
accepting electrons and reducing agent is oxidised
by losing electrons. getting oxidised and converted to Zn** ions which go
into the solution, the weight of zinc plate gradually
TYPES OF REDOX REACTIONS decreases. On the other hand, Cu?* ions present in the
The redox reactions may be classified into two types: solution are accepting the electrons given by zinc and
1. Direct redox reactions. The reactions in which are getting reduced to copper. As a result, copper metal
the oxidation and reduction take place in the same either gets deposited on the zinc plate or gets precipitated
vessel are called direct redox reactions. at the bottom of the beaker [Fig. 1(b)]. Since Cu?+ ions
2. Indirect redox reactions. The reactions in from the solution are changing to Cu(s), the blue colour
which oxidation and reduction take place in different of the solution (which is due to Cu?* ions) slowly fades.
vessels are called indirect redox reactions. These are The reaction may be written as:
very important reactions because these form the basis Zn(s) + Cu** (aq) + SO,?" (ag) ——>
of electrochemical cells. Cu(s) + Zn”* (aq) + SO,
DIRECT REDOX REACTIONS: Formation of Zn?* ions among the products can
REDOX REACTIONS IN AQUEOUS easily be tested when the blue colour of the solution
SOLUTIONS due to Cu”* ions has disappeared. If we pass, hydrogen
Redox reactions are very common in our daily life. sulphide (H,S) gas through the solution, formation of
Whenever you use a battery, a redox reaction occurs. white precipitate of zinc sulphide (ZnS) on making
A simple example of an oxidation-reduction reaction the solution alkaline with ammonia indicates the
is the reaction between zinc metal and copper (II) salt presence of Zn?* ions in the solution.
in aqueous solution : In this reaction, sulphate ions, sO,?- do not
Zn(s) + Cu*(aq) ——— Zn**(aq) + Cu(s) participate and we may write the reaction as :
Let us study this reaction. Loss of 2e~ : Oxidation
Take a strip of zinc metal and clean it with a sand
paper. Place this strip in a solution of copper sulphate Zn(s) + Cu®+ (ag) ——————> Zn** (aq) + Cu (s)
in a beaker [Fig. 1]. We observe that a spontaneous
reaction takes place and the following observations (;ain of 2e- : Reduction
are made :
(t) Zine metal starts dissolving and it loses its or Zn(s) + Cu?* (ag) ———> Zn** (aq) + Cu(s)
weight gradually. The reaction is commonly described as zinc having
(iz) The copper metal starts getting either displaced Cu? ions from the CuSO , solution.
deposited on the zinc plate or settles down at

Blue coloured
copper sulphate Intensity of blue Finally Cu®+ is
Cu deposited
solution colour decreases
decreases deposited as Cu
———> on Zn strip

Initial stage Intermediate stage Final stage


(a) (c)
Fig. 1. Redox reaction occurring in a beaker.
MODERN'S abc + OF CHEMISTRY
-XI

In this case, zinc acts as a reducing agent while At equilibrium, chemical tests show that both
Cu” ions act as oxidising agent. Ni**(aqg) and Co?*(ag) are present in moderate
In this redox reaction, the transference of electrons concentration. This means that in this case, neither
from Zn metal to Cu** ions is occurring directly because the reactants [Co(s) and Ni?*(aq)] nor the products
the two arein direct contact. The energy released appears [Co?*(aq) and Ni(s)| are greatly favoured.
as heat and therefore, it is an exothermic reaction. Thus, we observe that there is a competition of
Let us study the state of equilibrium for the above loss or gain of electrons by metals. This is similar to
reaction. For this purpose, place a rod of copper in zinc the competition between various acids to lose a
sulphate (ZnSO,) solution. You will notice that no proton (Ht) in water. This similarity suggests that
visible reaction occurs. If we pass H,5 gas through the we can develop a table in which the metals and their
solution, there will be no black precipitate of copper ions are listed on the basis of their tendency to lose
sulphide indicating that no reaction has occurred. In or gain electrons similar to the table in which
various acids are arranged according to their acid
other words, the following reaction :
strengths. For example, we have learnt in above
Cu(s)+Zn50O,(aq) —~>Cus0O,(aq)+ Zn(s)
experiments that zinc loses electrons to copper and
does not occur. Thus, we conclude that the state of
copper releases electrons to silver. Therefore, the
equilibrium for the above reaction between zinc and electron releasing tendency of these three metals is
copper sulphate greatly favours the products over the in the order :
reactants.
Zn > Cu > Ag
Similarly, when we dip a copper strip in a silver
nitrate solution, copper gets oxidised and goes into the Similarly, by comparing the relative tendencies of
other metals, we can build a table to arrange the
solution whereas Agt ions accept electrons and get
metals in their decreasing or increasing order of losing
reduced (Fig. 2). The reaction may be written as :
electrons. This series is called activity series or
Cu(s) + 2Ag* (ag) ———> _2Ag(s) + Cu** (aq)
electrochemical series and is discussed later.
Loss of 2e~ : Oxidation
Some other examples of common redox reactions

Cu (s) + 2Ag* (ag) —————>_—- Cu" (aq) + 2Ag (s) When a metal reacts with acids, hydrogen is
liberated. For example metals like Zn, Mg, Ni etc.
Gain of Ze : Reduction
can easily displace hydrogen from dil. HCI.
Zn + 2HCl ——~ZnCl, + H,
Thus, copper is oxidised to Cu?* and Ag* is reduced
This reaction occurs as redox reaction in which
to Ag(s). Therefore, copper acts as an oxidising agent or
metal is oxidised and hydrogen is reduced as :
oxidant while silver acts as a reducing agent or reductant.
Zn ———>7Zn** + 2e- Oxidation
The equilibrium greatly favours the products
Cu?*(ag) and Ag(s). 2H* + 2Cl- + 2e~-——_>2CI + H, Reduction

For the purpose of comparison, let us study the Zn + 2H* + 2Cl! ——+>Zn** + 2Cl-+ H,
reaction between cobalt metal and nickel sulphate In electrolytic process also, redox reaction takes
solution. For this, place a rod of cobalt metal in nickel place. For example, during electrolysis of aqueous
sulphate solution. The following reaction occurs : solution of copper chloride, oxidation and reduction
Loses 2e- : Oxidation occur as:
PF
DL CuCl, —= Cu? + 2Cl-
Co(s) + Ni?+(ag)——> Co*t (ag) + Ni(s)
At cathode : Cu?* + 2e- ——> Cus) Reduction
me
Tf fF
Gains 2e~ : Reduction At anode: 2C]-/- ———> Cl, + 2e- Oxidation

Silver nitrate Blue colour starts Intensity of blue


developing colour increases silver deposits
solution
—_———— > on the strip

Initial stage Intermediate stage Final stage

Fig. 2. Redox reaction between AgNO, and copper.


REDOX REACTIONS
37 |
Potassium permanganate is a strong oxidising Loses electrons : oxidation
agent in the presence of dil. H,SO,. In the presence of
dil. H,SO,, it gives nascent oxygen which oxidises H,S (g) + Cl,(g)——> 2HCl (g) + Sts)
ferrous ammonium sulphate [FeS5O,(NH,),50,] to
ferric sulphate as: gains electrons : reduction
2KMnO, + 3H,SO, —>K,SO,+ 2MnSO, +3H,0 +50 (it) Aluminium is oxidised because oxygen is added to
it.
2FeSO,(NH,),SO, + H,SO, + O—> Ferrous ferric oxide (Fe,O,) is reduced because oxygen
Fe,(SO,),+ 2(NH,),SO, + H,O] x 5 has been removed from it.
In terms of electronic concept :
2KMnO, + 10FeSO0,(NH,),.5O, + 8H,SO,—> loses electrons : oxidation
K,50, + 5Fe,(S0,),+ 2MnsO, + 10(NH,),.50, + 8H,0
The ionic equation for the reaction may be written 3Fe,O, (s) + 8Al(s) ——> 9Fe(s) + 4A1,0,(s)

as:
MnO, + 8H* + 5e7 —>»Mn?* + 4H.O gains electrons : reduction

Fe2+ —> Fe#+ + e-] x 5 (iii) In this case, sodium has been oxidised and hydrogen
has been reduced, This becomes more clear in terms of
5Fe2+ + MnO 4 +8H*——> 5Fe*+ + Mn?2+ + 4H,O electronic concept :
loses electron : oxidation
Similarly, acidified KMnO, oxidises oxalic acid
(C,H.O,,) solution. In this reaction, acidified KMnO,
2Na(s) + H,(g) ——> 2Na*H(s)
liberates nascent oxygen which oxidises oxalic acid to
carbon dioxide as: gains electrons : reduction
2KMnO, + 3H,SO,—>K,SO, + 2MnSO, +3H,0 +50 NaH is ionic compound and exists as Na*H~
C,H,0,+ O —>2CO, + H,O] x 5 L) Example 2
Identify the oxidant and reductant in the following
2KMn0O, + 5C,H,O, + 3H,SO, —> reactions :
K,SO, + 2MnSO, + 10CO, +8H,O
The ionic equation for the reaction may be written (a) = Zn(s) + =0, (g) ——> ZnO (s)
as: (b) CH, (g) + 4Cl, (g) ——— CCl, (g) + 4HCI (g)
[MnO,- + 8H* + 5e —>Mn”* + 4H,O] x 2 (c) I, (aq) + 25,07 (aq) ———> 2F (aq) + SF (aq)
C,0,2-—>2C0, + 2e-] x5 (d) Zn (s) + 2H* (ag) ———> Zn** (aq) + H, (g)
2MnO,- + 5C,0,2-+ 16H*> 2Mn** + 10C0,48H,O Solution:(a)
B |
Zn (s) + 902 (2) —— > ZnO (s)

In this reaction, zinc donates electrons to O to give


= — #10) Bee Se zinc ions and oxide ions. Thus, Zn acts as reductant
while oxygen acts as oxidant.
lL] Example 1 (b) CH, (g) + 4Cl, (g) ——> CCl, (g) + 4HCl (g)
Identify the species undergoing oxidation and CH, is oxidised and acts as reductant while Cl, is
reduction in the following reaction : reduced and acts as oxidant.

(i) H,S(g) +Cl(g)— 2HCI (g) +S (s) (c) I, (aq) + 25,0,7" (aq) ——— 2I- (aq) + 8,0,” (aq)
I, gains electrons and is reduced. Therefore, it acts as
(tt) 3Fe,0,(s) + 8Al(s) —> 9Fe(s) + 4Al,0,(s) oxidant. 8,0,7- acts as reductant.
(ii) 2Na(s)+Hg)— 2NaH(s) (d) Zn (s) + 2H* (ag+—>Zn** (aq) + H, (g)
Solution: Zinc loses electrons and gets oxidised. Therefore, it
acts as reductant. Hydrogen gets reduced and acts as
(1) H,S is oxidised because a more electronegative
an oxidant.
element, chlorine is added to hydrogen (or a more
electropositive element hydrogen has been removed lL) Example 3
from H,5). Justify that the reaction :
Chlorine has been reduced because hydrogen is added 2Na(s) + H.(g) ——— 2NaH (s) ts a redox reaction.
to it.
In terms of electronic concept, we can say,
Solution: Sodium hydride is an ionic compound and
it exists as Na* H.
MODERN'S abc + OF CHEMISTRY-XI

2Na + H, ——> 2N a” H (s) atom without electron has a unit positive charge.
The reaction may be split up into two half reactions: Hence, oxidation number of hydrogen in hydrogen
Na(s) —m——> Nat+e7|x2 Oxidation chloride is + 1.
H,(g) + 2e"° —— 2H Reduction The oxidation number is a fictitious charge in case
of covalent species. The oxidation number can have
2Na(s) + H,(g) ——> 2 NaH
positive, zero or negative values depending upon their
In this reaction, Na is oxidised to Na* and hydrogen is state of combination.
reduced from H, to 2H~ ion. Therefore, it is a redox
Rules for the Determination of Oxidation Number
reaction.
of an Atom
OXIDATION NUMBER The following general rules are used for the
The concept of electron transfer can easily explain calculation of oxidation number of an atom in a
the redox reactions in case of ionic substances. molecule :
However, we cannot easily explain the redox changes 1. The oxidation number of an element in the
in terms of electron transfer in case of covalent free or elementary state or in any of its
compounds. Now-a-days, to understand oxidation- allotropic forms is always zero. For example,
reduction reactions, systematically, scientists assign oxidation numbers of helium in He, hydrogen
an oxidation number or oxidation state to each in H,, oxygen in O, or Os, iron in Fe, carbon in
chemical species (ionic or covalent) according to a set diamond, helium atom in He, bromine in Brg,
of rules. The concept of oxidation number has been phosphorus in P,, sulphur in 8,, are zero.
very useful in balancing of redox reactions. The 2. The oxidation number of an element in a single
oxidation number is defined as (monoatomic) ion is the same as the charge
the charge which an atom of the element has in on the ion. For example, oxidation number of
its ion or appears to have when present in the K+ is +1, of Ca?* is + 2, of Al®* is +3. Similarly,
combined state with other atoms. the oxidation numbers of Cl-, SO? and PO,?-
are —1, —2 and —3 respectively.
Oxidation number also gives the charge which
3. In binary compounds of metal and non-metal,
an atom appears to have when all other atoms are
the oxidation number of metal is always
removed from it as ions. During the removal of atoms,
positive while that of the non-metal is negative.
the electrons on the atoms are counted according to
For example, in NaCl, the oxidation number of
the following two arbitrary rules : sodium is +1 and that of chlorine is —1.
(1) Electrons shared between two like atoms are 4. In compounds formed by the combination of
divided equally between the sharing atoms. In case of non-metallic atoms, the atom with higher
hydrogen molecule, for example, the electron pair is electronegativity is given negative oxidation
equally shared between the two hydrogen atoms. number. For example, in HCl, the oxidation
Therefore, one electron is counted with each hydrogen number of chlorine in HCl is —1 because of its
atom as shown below : high electronegativity.
H-|-H Similarly, N is given an oxidation number of —3,
Now, each hydrogen atom is having one electron when it is bonded to less electronegative atom as in
while its nucleus has one proton. Therefore, there is NH, and NI., However, it is given an oxidation number
no net charge on each atom of hydrogen. In other of +3 when it is bonded to more electronegative atom
words, oxidation number of hydrogen in hydrogen as in NCI,.
molecule is zero. Note. Rules (3) and (4) are based on the fact that atoms
(it) Electrons shared between two unlike atoms having high electronegativity have a tendency to form
are counted with more electronegative atom. For negative ions (anions) while atoms having low
example, in hydrogen chloride molecule, chlorine is electronegativity have great tendency to form positive ions
more electronegative than hydrogen. Therefore, the (cations).
shared pair is counted towards chlorine atom as shown 5. In all compounds of hydrogen, the oxidation
below : number of hydrogen is +1 except in hydrides
of active metals such as LiH, NaH, KH, MgH,,
H|:Cl: CaH,, etc., where hydrogen has the oxidation
numoer of —1.
As a result of this, chlorine gets one extra electron 6. The oxidation number of oxygen ts —2 in most
and acquires a unit negative charge. Hence, oxidation of the compounds. However, there are two
number of chlorine is — 1. On the other hand, hydrogen exceptions. The first exception is peroxides
REDOX REACTIONS

and superoxides in which oxygen atoms are


directly linked to each other. In peroxides (e.g., SOLVED EXAMPLES ———
H,O,, Na,O,), each oxygen atom is assigned L} Example4
an oxidation number of —I and in superoxides Calculate the oxidation number of sulphur in the
(e.g., KO,, R6O,) each oxygen atom ts assigned following molecules /ions :
an oxidation number of —1/2. The second
exception is found in compounds in which
(a) HS (b) H,SO, (c) SO2-
oxygen ts bonded to fluorine. For example, (d) Na,S,03 (e) S,07 (P H,SO,
in OF , (oxygen difluoride) the oxidation
Cop ae
number of oxygen ts +2 and in O,F ,(dioxygen Solution: (a) H,S. The oxidation number of hydrogen
difluoride) the oxidation number of each is +1. Let the oxidation number of sulphur be x.
+1 ag
oxygen is +1. This is because of the fact that
H, 5
fluorine being most electronegative element
known has always an oxidation number of —1. (1 x 2) (x)
1x24+x=0 ; x = —2
7. The most electronegative element, fluorine has
oxidation number —1. For other halogens, the Oxidation number of 5 in H,S is —2.
oxidation number ts generally —1, but there (6) H,SO,. Oxidation number of hydrogen is +1
are exceptions when these are bonded to a and that of oxygen is —2. Let the oxidation number
more electronegative halogen atom or oxygen. of S be x.
For example, in HI, the oxidation number of I +1 c —2
is —1 but in IF,, it is +5 and in IF,, it is +7. H, S O,
8. For neutral molecule, the sum of the (+1 x 2) x (-2
x 3)
oxidation numbers of all the atoms is equal to (41x 2)4+x+4+(-2x3)=0
zero. For example, in NH, the sum of the or 424+2x-6=0
oxidation numbers of nitrogen atom and 3
or ax = +4
hydrogen atoms is equal to zero.
Oxidation number of 5 in H,SO, is + 4,
For a polyatomic ion, the sum of the oxidation
(c) SO Paz The oxidation number of oxygen is —2 and
numbers of all the atoms is equal to charge on
let the oxidation number of S be x.
the ion. For example, in SO fam ion, the sum of
i
the oxidation numbers of sulphur atom and 4
oxygen atoms must be equal to —2.
5 O4
It may be noted that oxidation number is also
frequently called as oxidation state. For example, in x (—2 x 4)
H,O, the oxidation state of hydrogen is +1 and the x +(—2x4)=-2 or x=2+8
oxidation state of oxygen is —2. Similarly, in CO,, the or x = +6
oxidation number of carbon is +4, which is also its Oxidation number of S in SO?- is +6.
oxidation state and the oxidation number of oxygen is (d) Na,S,O,. The oxidation number of Na is +1 and
—2, which is also its oxidation state. This means that that of oxygen is —2. Let the oxidation number of 5 be
oxidation number gives the oxidation state of an x.
element in a compound. +1 x —2
Procedure for calculation of oxidation numbers. Nas S» O.
By applying the above rules, we can calculate the (+1 x 2) (x x 2) (—2
x 3)
oxidation numbers of elements in the molecules/ions (+1 x 2) + (x x 2) + (—2x 3) = 0
by the following steps : +24+2x—-6 = 0 or 2x =4
x = +2
(i) Write down the formula of the given molecule/ion
Oxidation number of 5 in Na,S,0, is +2,
leaving some space between the atoms.
(ii) Write oxidation number on the top of each atom. (e) $,0,7-. The oxidation number of oxygen is —2 and
In case of the atom whose oxidation number has to let the oxidation number of sulphur be x.
be calculated write x.
x —-2 =
(tit) Beneath the formula, write down the total oxidation
numbers of each element. For this purpose, Sg Oy
multiply the oxidation numbers of each atom with (x x 2) (—2 x 7)
the number of atoms of that kind in the molecule/ ux24+(—2x7) = -2
ion. Write the product in a bracket. 2X —2+14
(iv) Equate the sum of the oxidation numbers to zero 2 x =6
for neutral molecule and equal to charge on the ion.
Oxidation number of S in =O oe is + 6.
(v) Solve for the value of x.
MODERN'S abc + OF CHEMISTRY-XI

(f) H,SO,. The oxidation number of hydrogen is +1 and


(zv) N in (NH,),SO ‘ Let the oxidation no. of N be x.
that of oxygen is —2. Let the oxidation number of S be x
x +1
+1 x —2
(NH, ),S02 2x + 2 (+1 x 4) +(-2)=0
H, S O,
(+1 x 2) x (2 x 4) 2x+6=0
(41x 2)4+x4+(2x4)=0
2x =-—Gorx= -2=-3
or 4244-8=0
or x=+6
Oxidation number of 5 in H,SO, is + 6.
KEY NOTE
(g) $,0,*. The oxidation number of oxygen is —2 and
let the oxidation number of sulphur be «x.
PARADOX OF FRACTIONAL OXIDATION

0,
_42-
x -—-2 > For some compounds, the general rules give fractional
oxidation number to some atoms. For example, the
(x x 2) (-2 x 4) oxidation number of Fe in Fe,O, comes out to be + 3:
xx24+(-2x4) = -2
2x-8=-2 ..xe = 43
This is strange because electrons are never shared or
transferred in fractions.
.. Oxidation number of 5 in 8,0 i is +3.
Actually, this fractional oxidation number is the
L) Example5. average oxidation number of the element in the
What is the oxidation number of the underlined compound. This can be explained by the fact that Fe,O,
atoms in each of the following molecules /tons ? contains Fe atoms of both + 2 and + 3 oxidation number.
(a) CIO.; (b) BrF, (c) CH, (d) CH,0, (e) Na,B,O, It is a stoichiometric mixture of ferrous (FeO) and ferric
(Fe,O0,) oxides combined as FeO . Fe,O,. Therefore, the
(f) Na,lFe(CN),] (g) NoH,. oxidation number found in such cases is average
Solution: Suppose x be the oxidation number of the oxidation number.
underlined atom : Similarly, Mn,O, is regarded as a mixture of MnO
(a) ClO,” x +(—2x3) =-1 and MnO, having the composition 2MnO-MnQ,.
Therefore, the oxidation number of Mn in MnO is +2
x—6=—-lorx = +5
and that in MnO, is +4. Hence, average comes out to be
(6) BrF, x+(-lx3)=0 [2(4+2) + 1(+4)|/3 = 8/3.
x—-d=O0orx = +3 Red lead, Pb,O, is regarded as a mixture with
(c) CH, x+(1x4)=0 composition of 2PbO-PbO,. In PbO, the oxidation number
x+4=Oorx =-4 of Pb is +2 while in PbO,, the oxidation number of Pb is
(d) CgH,.0, 6xx+1x12+(2x6)=0 +4, The average value is [2(+2) + 1(+4)]|/3 = 8/3.
6x +12—-—12=O0orx
=0 Similarly, in ferriferrocyanide, Fe,[Fe(CN),l,
molecule, there are four iron atoms with oxidation
(e) Na,B,O, (+1 x 2) +(x x 4)+ (2x7) =0
number + 3 and three iron atoms with oxidation number
+2+4%-—14=Oo0rx= +3 + 2, The oxidation number of iron in this molecule is the
(f) Na,[Fe(CN),| (+1 x4)+x4+(-1 x6) =0 average of all these atoms equal to 18/7 (strange number).
4+44x—-—6=o0rx
= 42 Thus, the average oxidation number indicates average of
(g) N,H, 2xx¢+(1x4)=0 atoms in different bonding situations in a compound.
2x+4=Oorx=-—2 > In some compounds, the knowledge of chemical
lL) Example6. bonds or structures is essential to calculate
oxidation numbers. For example,
Calculate the oxidation number of (i) Fe in (a) Caro’sacid, H,SO, (peroxymonosulphuric acid)
Fe 0, (it) S in NagS,O, (iii) Pb in Pb,0, The O.N. can be calculated as
(iv) N in (NH,),SO,. 2x (41) +x +4 5(-2)=0, x =+8
Solution: But this is wrong because the maximum oxidation
(z) Fe in Fe,O,. Let the oxidation number of Fe be x.
number of S cannot be more than +6 because it
has only six electrons in its valence shell. In this
x —2 ax—8 = 0
structure, two oxygen atoms are bonded by a
Fe, O, 3x = 8 peroxide bond as
(x x 3) (2 x 4) x = 8/3 = 2.67
(iz) S in Na,S8,O,. Let the oxidation number of 5 be x.
QO
+1 x —2 244-12 =0
|
ee
Na, 5,4 Og 4y = 10
10
(+1x2) (xx4) (2x6) x = = 2.5 In the peroxide bond each O atom has —1 oxidation
(iit) Pb in Pb,O,. Let the oxidation no. of Pb be x.
state
x -2 ax—8 = 0 H,S(0,)0,
Pb. 0, ox = 8 2(4+1) +x + 2(—1) + 3(-2) = 0 or
x= +6
. 8
(xx3) (2x4) or => = 2.67
*=3 O.N. of Sin H,SO, is +6
REDOX REACTIONS

Similarly in $,0,? ion,


Cr
HNO, (peroxynitric acid)
The oxidation number of N comes out to be +7, This
Lae
a =
I There is one peroxide
is not possible for N. In fact, HNO, is peroxynitric
acid which contains one peroxide group [- O—O-|*.
| bond The formula may be written as HNO,(O,). In
O O this case, the oxidation number comes out to be
Be oes. 2x + 2(—1) + 6(—2) =-—2 or 2x = 12 orx=+6 (414+x-4-2=Oorx=4+5)+5.
(6) Oxidation number of Cr in CrO,
By conventional method — Fe) A 0
CrO, x + 5(—2) =0 . x=+10 lL] Example 7
But this is wrong because maximum O.N. of Cr is
+6 (3d°4s!) because it has six electrons in its Calculate the oxidation number of carbon in the
valence shell, The structure of CrO, is following compounds :
C,H.,, CO, C,H, CH,OH,HCOOH, CH,O.
Ts O
Solution: In all these compounds, the O.N., (oxidation
y ik number) of oxygen is —2 and that of hydrogen is +1.
So, oxidation number of carbon assuming x can be
easily calculated as :
It contains two peroxide bonds and may be written A: A : 2x+2x1=0 2x=-2 ..x=-1
as Cr(O,),0 CO, > x-(2x2)=0 “ x=+4
O.N. can be calculated as C,H, : 2n+(1x6)=0 2x=-6 .x=-3
x + Ix (-2) + 4(—1)=0 CH,OH : x +(1x4)-2 es x=-—?

(for Cr) (forone O) (for O—O bond) HCOOH ©: x 4(@x1)+(2x2)=0 x-2=0.. x=+42
x —2-4=0 CH,O = x4 (1 x 2) + (2) =0 . x=0
x=+6
Oxidation number of Cl in bleaching powder,
CaOCl,
Give examples of substances where carbon can
exhibit oxidation states from 4 to + 4 and nitrogen
Average O.N. of Cl in CaOCl, is
from -3 to +5.
+2—-2 42% =0
x= Solution: Carbon Nitrogen
However, it may be noted that the composition of —4 CH, —3 NH,
bleaching powder is Ca2+ (OCI-)CI-. In this O.N. of -3 C,H, ~2 NH,
Clin OCI is + 1 and in Cl is — 1 and the average —2 C,H, —1 N,H,
of two oxidation numbers 1 x (+ 1)+1x(—1)=0 —1 C,H, ON,
00.7.0. +1N,0
Calculated oxidation number of C +2 CO +2 NO
= 3x + 2 (— 2) =0orx == +3 N,O,
+4 CO, +4 NO,
Actual oxidation numbers of different C atoms are :
+5 N,O.
+2 O +2
O=C=—=C*—C=O Example 9
(the element marked with asterisk (*) shows Calculate the oxidation number of all the atoms
different oxidation number from rest of the atoms in the following compounds and tons :
of the same element). CO., Si0,, PoSsQ,, ClO,

Br,O, (tribromooctaoxide) Solution: CO, : Oxidation number of each oxygen


Calculated oxidation number of Br atom = —2
= Bx+ 8(- 2) = Oor x= — Oxidation number of carbon (say x)
x —2
Actual oxidation number of different Br atoms are :
C O,
| O
Oy +6 +4 460 x+(-2x« 2)=Oorx=44
OQ =Br —Br—-Br=0 .. Oxidation number of C = +4, O =-2
a y SiO, : Oxidation number of each oxygen
O O O
S,0,7 (tetrathionate ion) atom = —2
Calculated oxidation number of S
Oxidation number of silicon (say x)
x —2
= 4x + 6(—2)=-2orx= - Si O,
Actual oxidation number of different 5 atoms are: x +(-2x 2)=Oorx =+44
O Oxidation number of Si = + 4, O =-— 2
0
lk-s # PbSO, : Oxidation number of each oxygen
O=— x“ S =-2
I Oxidation number of lead (Pb) = +2
O
Oxidation number of sulphur (say x)
MODERN'S abc + OF CHEMISTRY
-XI

+2 x —2 5.0.2
Pb S O, 2x + (-2 x 6) +(-1 x 2)=-2
424x4(2x4)=Oorx=+6 2x =—2+14=+412
Oxidation number of Pb = + 2, 5 = + 6, “x = +6,
O=-2
Thus, by the application ofrules, we can calculate
ClO,- : Oxidation number of each oxygen
the oxidation number of the desired element in a
=-2
compound or in an ion. Metals have positive oxidation
Oxidation number of chlorine (say x)
states while non-metals may have positive or negative
x 2] oxidation states. The atoms of transition elements
Cl on (d-block elements) usually exhibit several oxidation
x + (—2 x 4) =-lorx=+7 states. The highest positive oxidation state of a
*. Oxidation number of O = 2, Cl = +7 representative element is the group number for the
lL] Example 10 first two groups (s-block elements) and the group
apa tachi the oxidation number of sulphur in number minus 10 for other groups, t.e. p-block elements
S,0 i (except for noble gases). Thus, the highest value of
oxidation state exhibited by an atom of an element
a re In 8, O- -, there
is one peroxide bond
(—O—O0— ) therefore, two oxygen atoms, oxidation
cenerally increases across period in the periodic table.
number is — 1 (i.e.,05 2-) and for the other six oxygen For example, in the third period, the highest value of
atoms, the oxidation Tuner is = 2. oxidation state increases from +1 to +7 inthe compounds
of the elements, as shown below :

17
Element Cl
Compound HClO,
Oxidation state of the silk
group element in the
compound

= Duals:
©5. -1,41,4+7,0(42 -2-2x=0 -..x=0)4+4
Ol. A is the oxidation number of nitrogen in
©6. K(+1), Mn(+7), O(-2), K(+1), Cr(+6), O(-2), K(+1),
(a) nitric acid (6) nitrous acid (c) nitric oxide
Cl (+7), O(-2)
(d) nitrous oxide (e) ammonia (/) N,
©7 +2,0,-1,-1
Q 2. What is the oxidation number of the underlined
atoms in the following? ©8. -3,-2.5, + 2, +4 and +4
KMnO,, Na,Cr,0.,
quae 7? Fe,O,, PO,?- CCl,
amg 4? KC1O,, a4? REDOX REACTIONS IN TERMS OF OXIDATION
. What are the oxidation numbers of the following ? NUMBER
(a) Cr in CrO,? (b) C in C,H,,0,
(c) Lin IF, (d)O in O, Based upon the concept of oxidation number,
oxidationis
. Determine the oxidation number of the atom in bold
in the following species : achemicalchangeinwhich thereis an increase
BH,, BF, BrO,-, HPO =. SAO, SiH, in oxidation number.
0 5. Determine the oxidation number of Cl in HCl, On the other hand, reductionis
HClO, ClO,- and Ca(OCl) Cl and Cl1O,. achemical change in which there ts adecrease
0 6. Calculate the oxidation number of all the atoms in inthe oxidation number.
the following well known oxidants :
For example, consider the following reactions:
KMn0O,, K,Cr,0,,
eal i KCIO,
QO 7. Calculate the oxidation sabe of oxygen in the (2) Reduction
following : OF,, O,, Na,O, and CH,COOH
0 8. Calculate the oxidation number of C in the
p
0 -1 —l 0
following :C,H,, C,H,,,
2° 6 ~ 40°10?
CO, CO, and HCO,~ Cl, (g) + 2Br-(aqg) —___, 2Cl (aq) + Br, (aq)

Answers to Practice Problems Oxidation


©l. (a) +5, (6) +3, (c) +2, (d) +1, (e) —3; nitrogen is Here oxidation number of bromine increases from
more electronegative than H, (/) 0. —l1 to 0 and that of chlorine decreases from 0 to —1.
© 2. +7, +6, +2.66, + 5, + 5, + 4(Clis more Therefore, bromide ion is oxidised to Br, and Cl, is
electronegative than C) reduced to CI ions.
© 3. (a) + 6, (b) 0, (c) + 7, (d) 0
REDOX REACTIONS

(ZL) Reduction In this case, MnO, is reduced (oxidation number


of Mn decreases from +4 to 0) and Al is oxidised
0 ms ee 42-1 0. (oxidation number increases from 0 to +3). Thus, Al
fn + 2HCl(aq) » “ZnCl, +H, acts as a reducing agent and MnO, acts as an
eer, . oxidising agent.
Oxidation Consider an ionic reaction :
In this reaction, oxidation number of zinc Reduction
increases from 0 to +2 and therefore zinc is oxidised.
The oxidation number of hydrogen decreases from +1 +7 =] +2 0)
to 0 and therefore, H, is reduced. The oxidation 2MnO, + 10Cl + 16H*__, IMn2* + 5Cl, + 8H,O
number of chlorine remains unchanged. | }§£3so)+
Oxidation
(zit) Reduction
In this case, MnO , is reduced (oxidation number
+4-2 +1 -l +2 —-l 0 +1 -2 of Mn decreases from +7 to +2) and Cl is oxidised
MnO, + AHCI , MnCl, + Cl,+ 2H,O (oxidation number increases from —1 to 0). Thus MnO,-
acts as an oxidising agent and Cl acts as a reducing
Oxidation agent.
In this reaction, the oxidation number of The various terms studied in this chapter can be
manganese decreases from +4 (in MnQO,) to +2 (in summed up as :
MnCl,) indicating that manganese dioxide undergoes
reduction. On the other hand, the oxidation number of Oxidation Loss of electrons or Increase
chlorine increases from—1(in HCI) to0 (in Cl, )indicating in oxidation number.
that hydrochloric acid undergoes oxidation. Reduction Gain of electrons or Decrease
Similarly, oxidising and reducing agents can be in oxidation number.
defined as follows :
Oxidisingagent =: Electron acceptor : Substance
Oxidising agent is whose oxidation number
a substance which increases the oxidation
decreases.
number of other substance in a chemical
reaction. Reducingagent -: Electron donor : Substance
Alternatively, the oxidation number of oxidising whose oxidation number
agent decreases. On the other hand, reducing agent increases.
is a substance which decreases the oxidation
number of other substance in a chemical : SOLVED EXAMPLES =
reaction.
L} Example 11
Thus, the oxidation number of a reducing agent
increases. Identify the oxidising and reducing agent and the
atoms undergoing oxidation and reduction in the
TO SUM UP:
following reactions :
An increase in oxidation number corresponds to
(J) HS+HNO, —— NO+S+H,0
oxidation of the element in the given compound
that acts as a reductant or reducing agent. A (b) Zn + CuSO, ——> Cu+ZnSO,
decrease in oxidation number corresponds to (c) I, (g) +H,S (g) ——> 2HI (g) + S(s)
reduction of the element in a compound and it (d) 4Zn (s) + 10H* (aq) + NO, (aq) ——>
behaves as an oxidant or oxidising agent. 4Zn** (aq) + NH (aq) + 3H,O(1)
In the above example (iz), manganese dioxide acts Solution: (a)
as an oxidising agent (oxidation number of Mn Reduction
decreases from +4 to +2) while hydrochloric acid acts
as a reducing agent (oxidation number of Cl increases
+1-2 +1+5-2 +2 -2 0 +1 -2
from —1 to 0). Similarly, consider the reaction :
H,S - HNO, __, NO + S + H,O
Reduction
Oxidation
+4 0 0 43 We observe that oxidation number of § increases from
3Mn0O, + 4Al —_, 3Mn+ 2Al,0, —2 (in H,S) to 0 (in 8) which means that H,S is being
oxidised or H,S is acting as a reductant. The oxidation
Oxidation number of nitrogen decreases from +5 (in HNQ,) to +2 in
(Oxidation numbers of only those atoms are shown (NO) indicating that HNO, is being reduced or HNO, is
which undergo change). acting as an oxidant.

WWW.JEEBOOKS.IN
| as MODERN'S abc + OF CHEMISTRY
-XI

Reduction L) Example 13
Justify that the reaction :
0 42 +6 -2 0 42 46 -2 2Cu,O(s) + Cu,S(s) ——— 6Cu (s) + SO,(g)
(6b) Zn + CusO, ————% Cu + AsO, is a redox reaction. Identify the species oxidised,
reduced, acts as an oxidant or a reductant.
Oxidation
Solution: N.C.E.R.T.
We observe that oxidation number of zinc increases from
Let us write the oxidation number of each element :
0 (in Zn) to +2 (in ZnSO,) which means that Zn is being
oxidised or zinc is acting as a reducant. On the other hand, Reduction
the oxidation number of copper decreases from +2 (in CuSO,,)
to 0 (in Cu), indicating that copper is being reduced or CuSO, +1 —2 +1 -? 0 +4 —2
is acting as an oxidant. 2Cu,O(s) + Cu,S(s) —— > 6Cufs) +85 O,(g)
[| =———s Reduction
0 a =| 0 Oxidation
(c) I,() + HS) —— 2HI (g) + S (s) In this reaction copper is reduced from +1 state (in Cu,O
or Cu,5) to 0 (in Cu) whereas sulphur is oxidised from —2
Oxidation
state (in Cu,5) to +4 (in SQ,), Therefore, the above reaction
is a redox reaction.
Oxidation number of iodine decreases from 0 to —1. In the reaction, Cu,O helps sulphur in Cu,5 to increase its
Therefore, it is reduced and acts as an oxidant. The oxidation oxidation number from —2to +4 and therefore, Cu,Ois oxidant.
number of 8 in hydrogen sulphide increases from —2 to 0. Alternatively, sulphur in Cu,S helps copper in both
Therefore, it is oxidised and hence acts as a reductant. Cu,O and Cu,S to decrease its oxidation number (i.e., get
| Reduction i reduced) and therefore, Cu,S is reductant.
0 +1 +5 +2 =o +1 0
(d) 47n(S)+10H*(aq) + NO,- (aq+—4Zn**(aq) + NH Pu (aq) + 3H,OW)
LoS
Oxidation
. Identify the oxidants and reductants in the
Oxidation number of zinc increases from 0 to +2. following reactions :
Therefore, it is oxidised and acts as a reductant. The (a) CH,(g) + 4CL(¢) —— CCL,(g) + 2HCI (g)
oxidation number of N decreases from +5 to —3. and,
(b) C,H,O,(aq) + 2H* + MnO,(s) —>
therefore, it is reduced and acts as an oxidant. Mn**(aq) + 2CO, (g) + 2H,0 (J)
L) Example 12. (c) I,(aq) + 8,0,?-(aq) —— 2I-(aq) + S,0,?(aq)
Determine the change in the oxidation number of S (dq) Cl, (g) + 2Br- (aq) —— 2CIl (aq) + Br,(aq)
in H,S and SO, in the following industrial reaction: Answers to Practice Problems
2H,S (g) + SO, (g) —— 3S (s) + 2H,O (g)
© 9. Oxidants and reductant are respectively
Solution:
(a) Cl,, CH,, (b) Mn0O,, C,H,0,, (c) I,, Oem

(d) Cl,, Br-.


2 +4 0
2H, S(g) + SO, (g) __, 3S (s) + 2H,0 @) Oxidation Number and Valency
There have been confusion regarding the usage of
the terms oxidation number and valency. The two
Change in O.N. of S in H,S from — 2 to 0
terms have different meanings as discussed below :
Change in O.N. of S in SO, from + 4 to 0.

Valency is the combining capacity of an atom. It is Oxidation number is the residual charge which an
expressed as the number of hydrogen atoms or double atom has or appears to have when all the atoms are
the number of oxygen atoms with which an atom of the removed as lions.
element combines. Since it refers to charge, it can be positive, negative or
Since it refers to combining capacity, it is a whole zero. For example, in NHg, nitrogen has oxidation
number only. As such it does not carry any plus or minus number of —3 and that of hydrogen is +1.
sign. For example, in NH, the valency of nitrogen is 3 Oxidation number of an element can be zero. For
and that of hydrogen is 1. example, oxidation number of C in CH,Cl, is zero.
Valency of an element cannot be zero (except for noble Oxidation number may have fractional values. For
gases). example, oxidation number of 5 in Na,S,O, is +2.5.
Since atoms always combine in whole numbers, valency Even the elements C, N and S$ exhibit variable valency.
of an element is always a whole number. For example, the oxidation number of N (given in
. In general, elements like C, N and S exhibit constant brackets) in its compounds vary from +5 to —3 as:
valency. Thus, valency of N in all its compounds is three. N,O,;(+5), NO,(+4), N,O,(+3), NO(+2), N,O(+1), N,(0),
N,H,(-2) and NH,(—3)
REDOX REACTIONS

OXIDATION NUMBER AND NOMENCLATURE Solution:


When an element forms two monoatomic cations Knowing oxidation of metallic element, these
(representing different oxidation states), the two ions compounds may be represented as :
are distinguished by using the ending -ows and ic. The +3
suffix -ous is used for the cation with lower oxidation @) HAuCl, HAu(IIDCl,
state and the suffix -ic is used for the cation with higher +1
oxidation state. For example, (ii) T1,0 T1(DO
Cut (oxidation number +1) cuprous : Cu?+
+2
(oxidation number +2) cupric. (tut) FeO Fe(II)O
However, Albert Stock proposed a new system
+3
known as Stock system. In this system, the oxidation (tv) Fe,O, Fe,JIDO,
states are indicated by Roman numeral written in
parentheses immediately after the name of the +1
element. For example, (v) Cul Cul(DI

Cu,O Copper (1) oxide +2


(vi) CuO Cu(IDO
snO Tin (IT) oxide
+2
FeCl, Tron (II) chloride
(vit) MnO MnO
FeCl, Iron (II) chloride
+4
CuO Copper (II) oxide (viii) MnO, Mn(IV)O,
snO, Tin (IV) oxide TYPES OF REDOX REACTIONS
AuCl Gold (1) chloride The redox reactions may be classified into the
AuCl, Gold (IIT) chloride following types :
Hg,Cl, Mercury (I) chloride 1. Combination reactions
The chemical reactions in which two or more
HgCl, Mercury (II) chloride
substances (elements or compounds) combine to form
Many more examples are taken for illustration : a single substance are called combination reactions.
A combination reaction may be expressed as :
Mn,O, Manganese (VII) oxide
A+B— 3C
K,Cr,0, Potassium dichromate (V1)
For a combination reaction to be a redox reaction
Na,CrO, Sodium chromate (VI) one or both A and B must be in the elementary form.
V0; Vanadium (V) oxide All combustion reactions in which elemental oxygen
Stock system is not used for non-metals. is used and all other reactions which involve
elements other than oxygen are redox reactions.
SOLVED EXAMPLES = Some common combination redox reactions are:
0 0 +4 —2
lL} Example14 C(s) + O, (g) ——> CO,
Write formulas for the following compounds ~441 0 +4 -2 +H 2
CH, (g) + 20g) ——> CO,(g) + 2H, OD)
(i) Mercury(ID]) chloride (ti) Nickel (II) sulphate
0 0) +2 —2
(iu) Tin(TV) oxide (iv) Thallium (I) sulphate 2Mg (s) + O,(g) —— 2MgO(s)
(v) Iron (IID) sulphate (vi) Chromium (IID) oxide. 0 0 +2 -—3
3Mg (s) + N.(g) ——> Mg.N.(s)
Solution:
0) 0 +2—2
(1) HgCl, (iz) NiSO, (17) SnO, Fe (s) + S(s) —— __FeS(s)
(tv) TLSO, (v) Fe,(SO,), (vi) Cr,O, 2. Decomposition reactions
lL) Example 15 The chemical reactions in which a compound
Using stock notation, represent the following breaks up into two or more simpler substances are
compounds : called decomposition reactions. The decomposition
reactions are the opposite of combination reactions.
(i) HAuCl, (wi) T1,0 (ut) FeO (ww) FeO, For a decomposition reaction to be a redox reaction,
(v) Cul (vt) CuO (vu) MnO (vit) MnO, at least one of the component formed after the break
down of the compound must be in the elemental state.
MODERN'S abc + OF CHEMISTRY
-XI

Some common examples of redox decomposition (6) Non-Metal displacement reactions. The
reactions are: non-metal displacement redox reactions are mainly
+1 -—2 0 0 hydrogen displacement or oxygen displacement
2H,0() —#t 5 2H,(g) + O,(g) reactions. For example,
+1 -1 0) 0 (1) All alkali metals and some alkaline earth metals
QNaH(s) —Ht_, 2Na(s) + H,(g) (e.g, Ca, Sr, Ba) which are good reducing agents will
42 =-2 0 0 displace hydrogen from cold water.
2HgO(s) —**, 2Hg(l) + O,(g) 0 +1 ~2 $1241 0
2Na(s)+ 2H,O() ——> 2NaOH(aq) + H,(g)
+1 +5-2 +1 -1 0
2KCIO.(s) ey 2KCI(s) + 30,(g) 0 +1 -2 +2 -2+1 0

It may be noted that all decomposition reactions


Mg(s)+ 2H,O() -——— Mg(OH),(s) + H,(g)
are not redox reactions. For example, decomposition 0 +1 -2 +2 —2 +1 +0)

of calcium carbonate is not a redox reaction because it Ca(s)+2H,O() ——> Ca(OH),(aq)+ H.(g)
does not involve change in oxidation number of (it) Less active metals such as magnesium and iron
elements. react with steam to produce dihydrogen gas.
+2 +4-2 +2 -2 +4 —2 0 fis—2 42 241 0
CaCO,s) —“> CaO(s) + C O,(g) Mg(s) + 2H,O(g) —He@t-, Mg(OH), + H,(g)
(Bieta
3. Displacement reactions
The reactions in which one ton (or atom) in a +1—2 +3 -2 0
compound is replaced by an ion (or atom) of other 2 Fe(s) + 3H,07) "45 Fe,0, + 3H,(g)
element are called displacement reactions. These (iit) Many fretals including phone which donot
reactions are very common in chemistry in which a react with cold water displace hydrogen from acids.
more active metal displaces or removes another atom For example,
from a compound. In general, a displacement reaction 0 +1-1 +2 —] 0
may be expressed as : VA + HCl (aq) —— ZnCl,(aq) + ne)
XY+4 —> X+ ZY 4146-2 2+6—-2
In this reaction, the atom X from the compound zn)+ H,SO,(aq) ——> ZnSO4iaq) + i,
(8)
XY has been displaced by another atom Z.
Mo(s) + 2HClaq) ——> MgCl,(aq) + i(g)
The displacement reactions are of two types : 0 1-1
(a) Metal displacement and 2A\(s) + 6HCKaq) ——> 2AiCl,(aq) + aH
(6) Non-metal displacement 0 +1-1
Fe(s) + 2HCl(ag) —— FeCl (aq) + H(g)
(a) Metal displacement reactions. In these
Metals like cadmium and tin which donot react
reactions, a metal in a compound can be displaced by
with steam also react with acids to displace dihydrogen
another metal in the uncombined state. These have
gas:
already been discussed earlier.
0
Some common examples of metal displacement +] =]
Cd(s) + 2HCl(aq) ———>
+2 —1
CdCl,(aq) + H,(g)
0

reactions are :
Sn(s) + ZHGl(ag) ——> 8nCl,(aq) + ,(@)
| +1 - +2 -]
+2 +6 -2 0 0 +24+6-2
CuSO, (aq) + Zn(s) ——> Cu(s) + ZnSO,(aq)
(itv) Very less reactive metals such as silver (Ag),
+2 +6 -2 0 +246-2 0 cold (Au), ete which occur in the native state donot
NiSO,(aq) + 2Co(s) ——~+ CoSO,(s) + Ni(s) react even with dilute hydrochloric acid.
+3 -2 0 0 +3 -2 The above reactions may be used to prepare
Fe,0.(s) + Al(fs) —— > 2Fe(s) + Al,0.(s) dihydrogen gas in the laboratory. The reactivity of the
43 -2 0 +3 -2 0 metals can be predicted from the rate of evolution of
Cr,0.(s) + 2Al(s) Heat, Al 20,(s) + 2Cr/(s) H, from water or aqueous acids . For example, sodium
reacts with water very readily (fastest rate),
+4-1 0 +2 =1
magnesium reacts slowly while iron reacts at the
TiCL,(@) + 2Mg(s) Heat . Tis) + 2MgCl,(s)
slowest rate while silver and gold donot react at all.
+5 -2 0 +2 -2 We have already learnt that metals such as zinc
V,0,(s) + 5Ca(s) —Heat_, ovis) + 5CaO(s) (Zn), copper (Cu) and silver (Ag) though have the
In each of these reactions, the reducing metal is a tendency to lose electrons, but their reducing tendency
better reducing agent than the metal being reduced is in the order Zn > Cu > Ag.
(e.g., Cu, Ni, Fe, Cr, Ti, V, etc.). This is because the Reactivity of non-metals. Like metals, there is
reducing metal has greater tendency to lose electrons also an activity series for non-metals, therefore, their
as compared to the one which is reduced. reactivity depends upon their oxidising power. For
REDOX REACTIONS

example, oxidising power of halogens decreases as oxidation state is simultaneously oxidised and reduced.
we move down the group 17 from fluorine to iodine. In other words, one reacting substance in a
Thus, fluorine is the strongest oxidising agent. It can disproportionation reaction always contains an
displace in their aqueous solutions Cl, from Cl ions, element that can exhibit at least three oxidation states.
Br, from Br- ions and I, from I ions. Thus, fluorine The element in the form of reacting substance is in
can show displacement reactions as : the intermediate oxidation state and both higher and
2X(aq) + F (aq) ——~X,(g) + 2F (aq) lower oxidation states exist for that element. The most
common example is the decomposition of hydrogen
where X-= CI, Br- and -
peroxide,
In fact, fluorine is so reactive that it even attacks
water and displaces the oxygen of water. 2H,0,(aq) ——> 2H, O(/) + O,(g)
2H,O(/) + 2F',(¢) ——— 4HF(aq) + O,(g) In this reaction, oxidation number of O decreases
It is because of this reason, the displacement from — 1 to — 2 (in H,O) and increases from — 1 to 0
reactions of chlorine bromine or iodine using fluorine (in Oy).
are usually not carried out in aqueous solutions. This may be simply represented as
On the other hand, chlorine can displace bromide Reduced
and iodide ions in aqueous solution as shown below :
0 1 cal +1 -1 0 Aa +1 -2 0
Cl, + 2K Br(aqg) ——> 2K Cl (aq) + Br,(/) HO, (ag) ——> H,O(/) + O,(g)
0 +1-1 41-1
Cl, + 2K I (aq) ——> 2KCl (aq) + LI,(s) Oxidised
Since bromine and iodine are coloured and dissolve Some other examples are :
in CCI, or CS,, these can be easily identified from the
({) Phosphorus and sulphur undergo dispropor-
colour of their solution. The above reactions may be
tionation in alkaline medium as shown below:
written in ionic from as :
P,(s) + 30H (ag) + 3H,O () > PH, (g) + 3H,PO, (aq)
2X (aq) + Cl,(g) ——> X, + 2CI (aq) (X =Br,I)
=i 0 0 —1 In this reaction, oxidation number of P increases
e.g., 2Br- (aq) + Cl,(g) ——> Br,(l) + 2CI(aq) from 0 to + 1 (in H,PO,”) and decreases from 0 to — 3
= | 0 0 —1
(in PH,).
21 (aqg)+ Clg) ——> I,{s) + 2CI (aq)
Reduced
The above two reactions form the basis of identifying
bromide (Br) and iodide (T-) ions in the laboratory [|
by carbon tetrachloride (CC1,) or carbon disulphide 0 -3 +1
P, (s)+ 30H" (ag) ——~ PH,(g) + 3H,PO; (aq)
(CS,). This test is popularly known as ‘layer test’.
Similarly, bromine can displace iodine from iodide
ion in the aqueous solution : Oxidised
(tt) S.(s) + 1ZOH- (aq) —> 4S?" (ag) + 28,02 (aq)
Br,(/)+ 21-(aq) ——_. Br (ag) + L(s) + 6H,O
The above halogen displacement reactions have In this reaction, oxidation number of 5 increases
been widely used in the laboratory and industry. The from 0 to + 2 (in S,0,7 ion) and decreases from 0 to
halogens can be recovered from the corresponding ~2 (in S* ion).
halides by using suitable oxidising agents according Reduced
to the reaction :
2X- (aq) Oxidising agent
Hg + Ze fd
0 —-2- +2 |
(where X = halogen element) S,(s) + 1ZOH (aq) —48*(aq) + 28,03 (aq) + 6H,O
It may be noted that although a number of oxidising
agents such as KMnO,, K,Cr,0,, MnCl,, etc are
Oxidised
available to oxidise Cl-, Br~, and I ions to liberate
Cl,, Br, and I, respectively, no oxidising agent is (iit) Chlorine also undergoes disproportionation in
available to oxidise F- ions to F,. This is because, alkaline medium as :
fluorine itself is the strongest oxidising agent. Reduction
Therefore, the only method to prepare fluorine is
to oxidise F~ ions electrolytically.
Pd+1
0 = |
4, Disproportionation reactions Cl(g) + 20H (ag)——> ClO“(aq)+ CI (aq) + H,O(/)
These are the reactions in which an element tn one
Oxidation
MODERN'S abc + OF CHEMISTRY
-XI

This reaction forms the basis of preparing | 0


household bleaching agents. The hypochlorite (ClO) (vi) sTIt+) —-——— 2Tl +TI*
ion acts as a bleaching agent by oxidising the colouring QO Example 18
matter (or stains) to colourless compounds.
Classify the following redox reactions giving reasons:
Like chlorine, bromine and iodine also undergo
similar disproportionation reactions. However, (a) N(g) + O.(g) —> 2NO()
fluorine does not show this behaviour. This is because
fluorine is the most electronegative and strongest
(b) 2Pb(NO,) (3) —> 2PbO(s) + 2NO,(g) + “0.(g)
oxidising agent and therefore, it does not show positive (c) NaH(s) + H,O() —> NaOH (aq) + H,{g)
oxidation states. Therefore, fluorine reacts in different (d) 2NO,(g) + 20H-(aq)—-NO,
(aq) + NO,- (aq)+ H,OWD
way forming oxygen difluoride as :
Solution:

oF» (g) +20H (aq) 2 F- (aq) + OF, (g) + H,O() 0) 0) +2 -—2


(a) N(g) + Og) 2NO(g)
It may be noted that F, also reacts with water to
The compound nitric oxide is formed by the combination
produce some oxygen. of elemental substances (N, and O,). Therefore, it is a
There is another important disproportionation combination redox reaction.
reaction. Copper (I) disproportionates to copper (II)
+2 +5 —2 +2 —2 +4 -2 4 0
and copper as:
(6J2Pb(NO,), (s) Heat, 2PbO(s) + 2NO,(g) + 9 Ox)
2Cu* ——> Cu+Cu*
Reduction Lead nitrate decomposes to form three products and

|
|
therefore, it is a decomposition redox reaction.
+1-1 +1 -2 +1 —2+1 0
+1 () +2 (ec) NaH(s) + H,OW) —> NaOH (aq) + 2 H,(g)
2Cu*t ——> Cu + Cu?+ Hydrogen of water has been displaced by hydride ion
to form dihydrogen gas. Therefore, it is displacement redox
reaction.
Oxidation
+4 +3 +5
Therefore, Cu(1) is unstable and only found in the (d) 2NO,(g) + 20H (aq)—— NO(aq) + NO, (aq) + H,OW)
solid state. In this reaction, the +4 oxidation state of nitrogen
(in NO,) decreases to +3 (in NO,-) and increases to +5 (in
| SOLVED EXAMPLES = NO,-). Therefore, it is disproportionation redox
L} Example 16 reaction.
Which of the two ClO, or ClO, show KEY NOTE
disproportionation reaction and why ?
Write reaction for the species that disproportionates. Pb,O, reacts with HCl and HNO, quite differently as:
Solution: The oxidation state of Clin ClO,” is + 3 and in Pb,O, + 8 HCl —-> 3PbCl, + Cl, + 4H,O
ClO,-, itis + 7. So, chlorine is present in highest oxidation Pb,O, + 4 HNO, —> 2Pb(NO,), + PbO,+ 2H,O
state of + 7 in ClO,” and it cannot increase its oxidation Why does it react differently ? This is an
state. Hence ClO, does not disproportionates. interesting observation.
The disproportionation reaction of ClO, is Pb,O, is not a simple compound. It is a mixture of 2
+3 —1 +5 mol of PbO and 1 mol of PbO, as 2 PbO. PbO,.
3010, =a 2C10, In PbO,, lead is present in + 4 oxidation state,
L] Example 17 whereas the stable oxidation state of lead in PbO is
Write the disproportionation reactions of the + 2. Therefore PbO, can act as an oxidising agent
following species : and can oxidise Cl ion in HCI to chlorine.
(i) CLO- (ii) ClO, (tit) Cl- (iv) ClO, It may be noted that PbO is a basic oxide. Thus the
(vy) ClO, = (vi) TH reaction :
Solution: Pb,O, + 8 HCl —> 3PbCl, + Cl, + 4H,O
+1 —1 +5
can be splitted into the following two reactions:
(1) 3ClO- ——> 2Cl + ClO,- 2PbO + 4HC1—— 2PbCl, + 2H,O: acid-base reaction
+4 =) +2 0
+5 —l +7 PbO, + 4HCl —— PbCl, + Cl, + H,O : redox reaction
(ii) 4Cl0O, ——> Cl + 3Clo;
However, nitric acid is itself an oxidising agent and
(zit) Cl- does not disproportionates.
therefore, no reaction occurs betwen PbO, and HNO,,.
+5 -1 +5 But the acid-base reaction between PbO and HNO,
(iv) 3Cl0,5 ——> Cl + 2ClO0, may occur as :
(v) ClO,” does not disproportionate because in this
2PbO + 4HNO, ———> 2Pb (NO,), + 2H,O
oxoanion, chlorine is present in its highest
oxidation state i.e., + 7.
REDOX REACTIONS

The complete reaction may be written as : Step 7. Finally, balance hydrogen and oxygen. For
Pb,O, + 4 HNO, —> 2Pb (NO,), + PbO, + 2H,O balancing for oxygen atoms add water molecules to
Thus, it is the passive nature of PbO, against HNO, the side deficient in it. Balancing of hydrogen atoms
which results in different reactions of Pb,O, with depend upon the medium (acidic or basic) as explained
HNO, than those with HCl. below :
(i) For reactions taking place in acidic solutions,
BALANCING OXIDATION-REDUCTION add H* ions to the side deficient in hydrogen atoms.
REACTIONS
(ii) For reactions taking place in basic solutions,
Though there are a number of methods for
add H,O molecules to the side deficient in hydrogen
balancing oxidation-reduction reactions, two methods
atoms and simultaneously add equal number of OH-
are very important. These are :
ions on the other side of the equation.
1. Oxidation number method
Step 8. Finally balance the equation by cancelling
2, Ion-electron method or half equation method. common species present on both sides of the equation.
1, Oxidation number method.
Let us discuss the above method stepwise with
We have learnt that during oxidation, there is the help of reaction between zinc and hydrochloric acid.
always increase in oxidation number and during Step 1. The skeleton equation is :
reduction there is always decrease in oxidation
Zn + HCl ——> ZnCl, + H,
number. Therefore, during a redox reaction, the fotal
Step 2. Oxidation number of various atoms
increase in oxidation number must be equal to total
involved in the reaction :
decrease in oxidation number. This is the basic
principle for balancing the chemical equations. In 0 +1-1 42 1 0
Zn + HCl —— ZnCl, + H,
addition, the law of conservation of mass should not
be violated, i.e., the number of atoms of each kind on Step 3. The oxidation number of zinc has
one side of the equation must be equal to the number increased from 0 to +2 while that of hydrogen has
of atoms of the corresponding elements on the other decreased from +1 to 0. However, the oxidation number
side. The following steps should be followed : of chlorine remains the same on both sides of the
Steps for Balancing Redox Equations by Oxidation equation. Therefore, zinc is reducing agent while HCl
Number Method is oxidising agent in this reaction and the changes are
shown as :
Step 1. Write the skeletal redox equation for all
the known reactants and products of the reaction.
0 +1-1 +2 —-l 0
Step 2. Indicate the oxidation number of all the
fn + HCl —— ZnCl, + H,
atoms in each compound above the symbol of the
element.
Step 4. The increase and decrease in oxidation
Step 3. Identify the element or elements which number per atom can be indicated as :
undergo a change in oxidation number. Usually only
O.N. increases by 2 per atom
two elements will be involved, one whose oxidation
number increases (i.e. reducing agent) and the other [
0 +1 +2 0
whose oxidation number decreases (1.e. oxidising
/n + HCl ——} ZnCl, + H,
agent). ee.
Step 4. Calculate the increase or decrease in O.N. decrease by 1 per atom
oxidation numbers per atom. If more than one atom
of the same element is involved, find out the total Step 5. The increase in oxidation number of 2 per
increase or decrease in O.N. Then multiply this atom can be balanced with decrease in oxidation
number of increase/decrease of O. N. with the number number of 1 per atom if Zn atoms are multiplied by 1
of atoms which are undergoing the change. and HCl by 2. Consequently the equation will be :

Step 5. Equate the increase in oxidation number Zn + 2HCl ——> ZnCl, +H,
with decrease 1n oxidation number on the reactant side Step 6. Balance all other atoms on both sides of
by multiplying the formulae of the oxidising and the equation. By chance, they are equal, otherwise
reducing agents. equate them.
Step 6. Balance the equation with respect to all Hence, balanced chemical equation is:
other atoms except hydrogen and oxygen. Zn + 2HCl ——> ZnCl, +H,
MODERN'S abc + OF CHEMISTRY-XI

3. To balance increase or decrease multiply C by 3


— SOLVED EXAMPLES and Fe,O, by 1 as:
L] Example 19 Fe,0,+3C ——— Fe+ CO
Copper reacts with nitric acid. A brown gas is
formed and the solution turns blue. The equation 4, Balancing all other atoms, we get
may be written as : Fe,O,
+ 3C ———> 2Fe+3CO
Cu +NO,- ——> NO, + Cu** (ii) Fe?++Cr,0,7 +Ht ———> Cr*+ + Fe'++H,O
Balance the equation by oxidation number method.
1. The skeleton equation along with oxidation
Solution: Step 1. Skeleton equation
number of each atom
Cu +NO,- ——> NO, + Cu**
Step 2. Writing oxidation numbers of each atom 42 +6 -2 +1 +3 +3 +1-2
0) +5-2 +4—2 +2 Fe?* + [Cr,0.]¥ > Ht =, Fe** + Cr** + H,O
Cu + NO, ——> NO, + Cu2+ 2. The oxidation number of chromium decreases from
Step 3. +6 in Cr Gaga to +3 in Cr**. The total decrease for
two chromium atoms in Cr,O,7- to Cr** is 6. On
0 +5 +4 +2
the other hand, the oxidation number of iron
Cu + NO, ——> NO, + Cu increases from +2 (in Fe?+) to +3 (in Fe®+),
Therefore, the equation can be written as:
{|
The oxidation number of copper has increased from O.N. increases by 1 per atom
0 to +2 while that of nitrogen has decreased from +5 to +4.
Step 4. Show the increase/decrease of oxidation number +2 +6 +3 +3 +1
O.N. increases by 2 per atom
Fe** + [Cr,0,/* + H*——— Fe* + Cr°++H,O
[
0 +5 +4 +2
ee
O.N. decreases by 6 for 2 Cr atoms
Cu + NO, —— NO, + Cu2+
3. To balance increase and decrease of oxidation
LS numbers, multiply Fe?* by 6 and Cr,O,* by 1. Then
O.N. decreases by 1 per atom
we get
Step 5. Balance the increase/decrease in oxidation
number by multiplying NO,” by 2 and Cu by 1. 6Fe** + Cr,0,7- + Ht ——> Fe** + Cr** + H,O
Cu + 2NO; ——> NO, + Cu? 4, On counting and equating the atoms on both sides,
Step 6. Balance other atoms except H and O as we get the balanced equation as:
Cu +2NO; ——> 2NO, + Cu** 6Fe*+ + Cr,0,2> + 14H+ ———> 6Fe** + 2Cr** + 7H,O
Step 7. Reaction takes place in acidic medium, so add (iit) Zn + HNO, ———> Zn(NOQ,), + NO, + H,O
H* ions to the side deficient in H+ and balance H and
O atoms: 1. The skeleton equation along with oxidation
Cu + 2NO; + 4H+ ——> 2NO, + Cu** + 2H,O number of various atoms is
L) Example 20.
0 +14+5-2 +2 +6-2 +4 -2 +1 —2
Balance the following equations : Zn +HNO, ———> Zn(NO,),+NO, + H,O
(i) FeO, + C ——} Fe+CoO
O.N. increases by 2
(ii) Fe?* + Cr,07- + Hey Fe* + Cr+ +H,O
(Luu) Zn + HNO, —> Z£n (NO3). + NO, [
+ H,O 0 +5 +2 +4
(iv) C,H, + 0, —>—> CO, + H,O 2. Zn + HNO, ———~ 4n(NOQ,), + N O, + H,0
Solution:(i) Fe,0,4+ C —— > Fe+CO Lt
O.N. decreases by 1
1. The skeleton equation along with oxidation
number of each atom is : 3. Multiply HNO, by 2
+3 -2 0 0 +2 2 Zn + 2HNO, ———> Zn(NO,), + NO, + H,O
Fe,O, + GC ——> Fe + C O
It has been observed that part of nitrogen
2. The oxidation number of C increases from 0 to +2
while that of Fe decreases from +3 to 0. Therefore, undergoes no change in oxidation number in
the equation may be written as forming Zn(NO,),. However, in order to balance
O.N. increases by 2 per atom additional two NO, ions, add 2HNO, more on
the left hand side as :
+3 0 +2
An + 4HNO, —— > Yin(N 05). + NO, + H,O
Fe, O, + Cc — Fe + CO
| O.N. decreases by 3 per atom é 4. Balance all other atoms.

3 x 2 per 2 atoms Zn + 4HNO, ———> Zn(NO,), + 2NO, + 2H,O


REDOX REACTIONS

(iv) C,H, +O, ——— CO, +H,0 (ut) Cr(OH),+10,,5 ——> I + Cro?--
1. The skeleton equation along with oxidation (in basic medium)
number of each atom : Solution: (1) Fe5,+0O, ———» Fe,0, + 5O,
1. The skeleton equation with oxidation number is :
-1 +1 0 +42 +1=2
C, H,+O, ——— CO, + H,0 42-1 O +3 -2 +4-2

O.N. increases by 5 per atom FeS,+0, ———> Fe,0, + SO,


(Note : Oxidation number of 5 in FeS, is —1 and
or|ox 6 per 6 atoms i not —2).
Ze -1 0 +4 -3
2. The oxidation number of iron as well as sulphur
increases from +2 to +3 and —1 to +4 respectively.
O.N. decreases by 2 per atom Since Fe and 5 must maintain their atomic ratio,
or 2 x 2 per molecule the change in oxidation number will be considered
3. Multiply C,H, by 2 and O, by 15 to balance together. On the other hand, the oxidation number
increase and decrease in O.N. of oxygen decreases from 0 to —2. Thus,
2 C,H, + 150, ———> CO, + H,O O.N. increases 1 per Fe and 5 per 5 atom
ie. 1+2x5=11 per Fes,
4, Balance all atoms
2C,H, + 150, ———> 12CO, + 6H,O
lL) Example 21. +2—1 0 +3 $42
Feige: “O./—— Fe,0O, + SO
Permangnate ton reacts with bromide ton in basic 2

medium to give managanese dioxide and bromate | O.N. decreases by 2 per atom #
ton. Write the balanced chemical equation for the or 4 per O,
reaction. 3. To balance increase and decrease of oxidation
number, multiply FeS, by 4 and O, by 11.
Solution:The chemical equation is :
4FeS, + 110, ———> Fe,O, + SO,
MnO, + Br: ——> MnO, + BrO,~ 4, Balancing all atoms
1. The skeleton equation with oxidation number of 4FeS, + 110. ——— 2Fe,0, + 850,
each atom is:
(ti) MnO, + Fe** + H+> Mn”* + Fe** + HO
+7 -2 -1 44-2 +5 -2 1. The skeleton equation along with oxidation
MnO,” + Br — MnO, + BrO, number of each atom is as:
O.N. decreases by 3 per atom +7 -2 +2 +2 +3 +1 -2
Mn O, + Fe?* + Ht ——— Mn? + Fe** + H,O
+7 =] +4 +5
O.N. decreases by 5 per atom
MnO,- + Br ———> MnO, + BrO,-

O.N. increases by 6 per atom +7


PY +2 +2 +3
2. To balance increase and decrease in oxidation MnO, +Fe?* + Ht-———> Mn?* + Fe*++ H,O
numbers, multiply MnO,- by 2.
2MnO, + Br» ———~ Mn0O, + BrO,~ O.N. increases by 1 per atom
3. Balance all atoms other than H and O. 2. The oxidation number of Mn decreases from +7 to
2MnO, + Br- ——~> 2Mn0, + BrO,- +2 while that of Fe increases from +2 to +3.

4, Since the reaction occurs in basic medium, add OH- 3. Multiply Fe?* by 5 and MnO, by 1 to balance
ions to right hand side. increase and decrease in oxidation number,
2MnO,- + Bro ——— 2Mn0O, + BrO,- + OH- MnO,” + 5Fe?* + Ht ———> Mn?+ + Fe*+ + H,O
5. To add H atoms, add H,O molecules on the left 4, Balance all atoms (except H and O)
hand side and equal number of OH” ions on the
MnO,” + 5Fe?* + Ht ———> Mn?* + 5Fe** + H,O
right hand side
This reaction takes place in acidic medium. The
2MnO st Br- + H,O———> 2MnO, + BrO.~ + OH-
O and H atoms are balanced as given below :
+ OH-
or 2MnO, + Br’ + HzO ——> 2Mn0O, + BrO,” + 2OH- 5. Balance O-atoms by adding equal number of H,O
molecules as :
L} Example 22.
MnO, + 5Fe2+ + H+ — Mn?+ + 5Fe?* + H,O + 3H,O
Balance the following reactions by oxidation
6. Balance H-atoms by adding H* ions on the side
number method :
deficient in H-atoms.
(i) eS, + O. ——> Fe,0, + SO,
MnO,” + 5Fe?* + H+ + TH+ — Mn?+ + 5Fe** + 4H,O
(i) MnO, + Fe?+ ———> Mn** + Fe**
(in acidic medium) or MnO, + 5Fe*+ + 8H* > Mn?t + 5Fe** + 4H,O
MODERN'S abc + OF CHEMISTRY-XI

Let us sum up the methods for balancing O and (t1L) P+ HNO, ———-. Hi,PO, +H,O+ NO,
H-atoms in oxidation-reduction reaction taking
Solution: (/)As,S, + NO, —> AsO} + NO+S
placing in acidic medium.
1. The skeleton equation with oxidation number of
1. Balance O-atoms by adding H,O molecules each atom is :
on the side deficient in O-atoms. +48 —-2 +5-2 +5 —2 +2-2 0
2. To balance H-atoms, add H* ions (available As,S,+ NO, —> AsO,>+NO+S
in acid solution) on the side deficient in
2. The O.N. of As increases from +3 to +5 and that of
H-atoms.
S increases from —2 to 0. The O.N. of N decreases
(i) Cr (OH), + 105, ——> I + Cro,> from +5 to +2, This means that both As and 5 have
(in alkaline medium) been oxidised while NO,” has been reduced. Since
As and § must maintain their atomic ratio of 2:3
1. The skeleton equation along with oxidation
(as inAs,Sq), therefore, the change in O.N. of these
number of each atom is:
two atoms must be considered together. Thus,
+3 -2 +1 +5-2 -l +6 -2
Cr(QH), + IO. —— >IT + CrO, 2 As and2 per 5 atom
O.N, increases by 2 per
ie 2 x 2+ 2x 3=10 per As,5,
O.N. increases by 3 per atom

+3 —2 +5 +5 3- +2 0
+3 +5 -l +6 As,Sy+ NO, ——> As0,++ NO+ S
Cr(OH), + 1O,- — [- + CrO0 =
b7F
O.N,. decreases by 3 per atom
O.N. decreases by 6 per atom
3. To balance increase and decrease of O.N., multiply
2. To balance increase or decrease in oxidation As,S, by 3 and NO, by 10.
numbers, multiply Cr(OH), by 2 and IO,” by one.
3AS5, + 1ONO,.——> AsO yg +NO +8
2Cr(OH), + 10,5“ ———> I + CrO,?- 4, Balance all atoms (except H and O)
3. Balance all atoms (other than H and OQ). 3As,S, + 10NO,,———> 6AsO,* + 10NO + 9S
2Cr(OH), + 10,5“ ——— I + 2Cr0O,> This reaction takes place in acidic medium. The O
and H atoms are balanced as :
4, Reaction takes place in basic medium, add OH™
5. Balance O atoms by adding equal number of H,O
to right hand side to balance O-atoms.
molecules as :
2Cr(OH), + 10,5, ——~> T+ 2CrO,? + OH- 3As,S,+10NO,- +4H,O —> 6AsO,?"+ 10NO +98
5. To balance H-atoms, add H,O molecules on right 6. Balance H atoms by adding H* ions on the side
hand side and equal number of OH~ on left hand deficient in H atoms :
side, 3As,9, + 1ONO,- + 4H,0 —~> 6As0,° + 10NO +
2Cr(OH), + 10,- + 50H- > [I> + 2Cr0,> + OH- 95 + 8H*.
+ 5H,O (iL) MnO, + C,H,O, —> Mn?* + CO, + H,O
or 2Cr(OH), + 10,- + 40H- > I-+2Cr0,? +5H,O 1. The skeleton equation with O.N. of each atom is :
+7 -2 43 +1 -2 +2 +4-2 +1 -2
Let us sum up the method for balancing O and H
atoms in oxidation-reduction reactions taking MnO," + C,H,O0,—> Mn?++ CO, + H,O
place in basic medium. 2. The O.N. of C increases from +3 to +4 and the O.N.
of Mn decreases from +7 to +2. Thus,
1. Balance O atoms by adding OH™ ions
(available from basic solution) on the side O.N. increases by 1 per atom
deficient in O-atoms (step 5) or 2 per molecule

2. Tobalance H atoms, add one molecule of water


for each H-atom needed on the side deficient +7 +3 +2 +4
in H atom. At the same time add an equal MnO, + C,H,O, ——> Mn +CO,+ H,O2
number of OH" ions to the opposite side.
lL] Example 23 O.N. decreases by 5 per atom

Balance the following equations by oxidation 3. To balance increase and decrease in O.N. multiply
number method : MnO, by 2 and C,H,0, by 5.

i) —_ As,S,(s) + NO,(aq) ——> 2MnO, + 5C,H,O,


> 2Mn?”* +CO, +H,O
AsO Figag) + NO(g) + S(s) (acidic medium) 4, Balance all atoms (except H and O)
(ii) MnO (aq) + C,H,O aq) ——> 2MnO,- + 5C,H,O,—> 2Mn** +10CO, + H,O
Mn** (aq)+ CO,(g) + H,OW (acidic medium) This reaction takes place in acidic medium. The
O and H atoms are balanced as :
REDOX REACTIONS

5. Balance O atoms by adding equal number of H,O Cr,0,7> +3S0,7" + 8H* ———> 2Cr** + 380," +
molecules as : 4H,O

precise prem
2MnO, +5C,H,O,——> 2Mn?* +10CO, + 8H,O This is a balanced equation.
6. To balance H atoms add H* ions on the side
deficient in H atoms :
2MnO,- + 5C,H,O, + 6H* —> 2Mn** +10CO, + Q 10. Balance the following equations by oxidation
8H,O number method :
(7) P+ HNO, —> H,PO, + NO, + H,O (a) MnO, + HCl ———> MnCl, + Cl, + H,O
1. The skeleton equation with O.N. of each atom is: (6) HS + KMnO, + H,SO, —— KHSO,+MnSO,+5
0 4+14+5-2 4145-2 +4-2 +1-2 + H,O
P+HNO,—> H,PO, + NO, + H,O (c) CuO + NH, —-Cu+N,+H,0O
2. The O.N. of P increases from 0 to +5 and the O.N. Q@ 11. Balance the following ionic equations by oxidation
of N decreases from +5 to +4. Thus, number method :
O.N. increases by 5 per atom (Z) Fe*+ + H,O, ——— Fe*+ + H,O
[
0 +5 +5 +4 (iL) Zn + NO, ——>
(acidic medium)
Zn?* + NH ‘
P + HNO,——> H,PO,+ NO,+ H,O (basic medium)
(ii) H,S + MnO, ——>S + Mn** + H,O
O.N. decreases by 1 per atom
(acidic medium)
3. To balance increase and decrease in O.N. multiply
(iv) Cu + NO, + H* > NO, + Cu*+
HNO, by 5.
(v) MnO, + H,C,0, t——> Mn?* + CO, + H,O
P + 5HNO,——— H,PO, + NO, + H,O
(acidic medium)
4, Balance all atoms (except H and O)
Q 12. Magnesium reacts with nitric acid to give
P + 5HNO,——— H,PO, + NO, + H,O
magnesium nitrate in aqueous solution, nitrous
This is balanced equation.
oxide gas and liquid water. Write balanced
lL} Example 24. equation for this redox reaction.
Write the balanced tonic equation for the reaction Q@ 13. Balance the following equations by oxidation
of potassium dichromate (VI), K, Cr,0, with number method :
sodium sulphite, Na,SO, in acid medium to give (i) H,S (aq) + Cl, (g¢) ——> 8 (s) + Cl (aq)
chromium(LD ion and sulphate ton. (acidic medium)
Solution : The skeleton equation is : (iz) Cl,O., (g) + H,O, (¢) ——> ClO,- (aq) + O, ()
Cr,0,2> + SO,”
—> Cr#* + S02 (basic medium)
1. The skeleton equation with oxidation number is : (ii) P (s) + OH- (aq) ——> PH, (g) + H,PO,- (aq)
+6 -2 +4-2 +3 +52
(basic medium)
Cr,0? + SO7 —A@cr* + S02 (wv) NH, (ge) + ClO (aq) —> NO (g) + 4ClF (g)
a O.N. increases by 2 per atom
(v) MnO,(aq) + C,H,O,—> Mn?*(aq) + CO,(g) +H,0 (1)
(vt) MnO,‘(aq) + C,H,OH— Mn?*(aq) + CH,COOH
+6 +4 +3 +6 (aq)
Cr,0,> + SO,”—> Cr+ + S02
Answers to Practice Problems
O.N, decreases by 3 x 2 per molecule © 10. (a) MnO, + 4HCl ——> MnCl, + Cl, + 2H,O
3. To balance increase and decrease multiply SO,” (6) 5H, 5 + 2KMn0O, + 4H,50, ——> 2KHSO, + 2MnSO,
by 3 + 5S + 8H,O
Cr0" - 3S0,7°—_> Cr** + so, (c) 38CuO + 2NH, ———> 3Cu +N, + 3H,O
4, Balance all atoms (except O and H) © 11. (i) 2Fe”* + H,O, + 2H* ——-> 2Fe** + 2H,O
Cr,0,7 + 3S0,2———> 2Cr*+ + 380,7- (ii) 4Zn + NO, + 7H,0 ——— 4Zn** + NH{,+ 10 OH-
This reaction occurs in acidic medium, O and H @ii) 5H,S + 2MnO7 + 6H* ——-> 5S + 2Mn”* + 8H,O
atoms are balanced as :
(iv) Cu + 2NO,- + 4H*+ ——> 2NO, + Cu + 2H,0
5. Balance O atoms by adding H,O molecules as :
(v) 2MnO,- + 5H,C,O, + 6H* ———> 2Mn?*+10CO, +8H,O
Cr,0,*- - 380,» ——> 2Cr3+ + 3802 + 4H,O
© 12. 4Mg(s) + 1OHNO, (aq) —> 4Mg(NO,), (aq) + N,O (g)
6. Balance H atoms by adding H* ions on the side + 5H,O()
deficient in H atoms :
MODERN'S abc + OF CHEMISTRY
-XI

Solution:
13.1) H,S (aq) + Cl, (g) —— 8 (s) + 2CI (aq) + 2H* (aq)
Step 1. Separate the equation into two half
(ii) Cl,0, (g) + 4H,0, (aq) + 20H-(ag) —> reactions :
2Cl0,- (aq) + 40, (g) + 5H,O (0)
The oxidation number of various atoms are
(tit) 4P (s) + 30H” (aq) + 3H,O (/) ——> shown below :
PH, (g) + 3H,PO,- (aq) +6 -2 +2 +1 +3 +3 +1 -2
(iv) 83N,H, (s) + 4C10,~ (ag/——> Cr,0,? + Fe?* + Ht > Cr*+ + Fe*+ + H,O
6NO (g) + 4Cl (aq) + 6H,O (2) In this case, chromium undergoes
(v) 2MnO,(aq) + GH*(aq) + 5C,H,O,(aqg) —> reduction, oxidation number decreases
2Mn**(aq) + 10CO,(g) +8H,0 (1) from +6 (in Cr,O0,7-) to +3 (in Cr**)
(vi) 4MnO, (aq) + 5C,H,OH(aq) + 12H*(aq) — Fe*+ (O.N. = +2) changes to Fe#* (O.N. = +3).
4Mn**(aq) + 5CH,COOH (aq) + 11H,O The species undergoing oxidation and
reduction are :
B. Ion-Electron Method (Balancing Equations Oxidation : Fe** ———> Fe®+
Using Half Reactions) Reduction : Cr,0,2- ———> Cr**
The balancing of chemical equation by ion-electron Step 2. Balance each half reaction separately as :
method (using half reactions) is done by the following (a) Fe?+ ———> Fe®+
steps : (1) Balance all atoms other than H and O.
Step 1. Find the elements whose oxidation This step is not needed, because, it is
numbers are changed. Choose the substance which already balanced.
acts as an oxidising agent and one that acts as a (11) The oxidation number on left is +2 and
reducing agent. on right is +3. To account for the difference,
the electron is added to the right as :
Step 2. Separate the complete equation into two
half reactions, one for the change undergone by the Fe2+ ———> Fe*+ + e-
oxidising agent and the other for the change undergone (iz) Charge is already balanced.
by the reducing agent. (iv) No need to add H or O.
Step 3. Balance half equations by the following The balanced half equation is :
steps : Fe?* ——> Fe**4+e- ...(i)
(t) balance all atoms other than H and O. Consider the second half equation
(ii) calculate the oxidation number on both sides Cr,0,7- ———> Cr**
of the equation. Add electrons to whichever side is (1) Balance the atoms other than H and O.
necessary, to make up the difference.
Cr,0,2-. ———> 2Cr**
(iit) balance the half equation so that both sides
(it) The oxidation number of chromium on
cet the same charge. the left is +6 and on the right is +3. Each
(iv) add water molecules to complete the balancing chromium atom must gain three electrons.
of the equation. Since there are two Cr atoms, add 6e™ on
the left.
Step 4. Add two balanced half equations. Multiply
one or both half equations by suitable numbers so that Cr,0,2- + 6e- ———> 2Cr3+
on adding the two equations, the electrons are balanced. (111) Since the reaction takes place in acidic
medium add 14H* on the left to equate the
It may be noted that redox reaction may take place
net charge on both sides.
in all the three media /.e., acidic or basic or neutral. If H*
ions appear on either side of the equation, the reaction Cr,0,2- + 6e- + 14H* ——> 2Cr3+
takes place in acidic medium. If OH- ions appear on (iv) To balance H atoms, add 7H,O
either side of the equation, the solution is basic. Ifneither molecules on the right.
H* nor OH" ions are present, the reaction occurs in Cr,0,7- + 6e- + 14H+ ——> 2Cr?+ + 7H,O
neutral solution. For balancing redox reactions involving (iL)
acidic and basic media, the method has to be modified This is the balanced half equation.
slightly as illustrated in the examples given ahead :
Step 3. Now add up the two half equations.
Multiply eq. Gi) by 6 so that electrons are
= SOLVED EXAMPLES :
balanced.
L} Example 25 Fe?+ ——> Fe**+ +e¢]x6
Consider the reaction between tron (II) tons and Cr,0,7 + 6e~ + 14H*+ ———> 2Cr** + 7H,O
dichromate tons in acidic solution as :
Cr,0,7- + Fe?* + Ht ——>Cr** + Fe** + H,O 6Fe2* + Cr,0,7- + 14H*+ ——> 6Fe** + 2Cr°* + 7TH,O
REDOX REACTIONS

The balanced equation is : (iit) & (iv). As the reaction is proceeding in acidic
6Fe”* + Cr,0,> + 14H* ———> 6Fe** + 2Cr** + 7H,O medium add 14H* on the left to equate the
net charge on both sides. At the same time,
Note. If we carefully see step 2 part (iii) and (iv) we
get a method for balancing redox reaction in acidic add 7H,O to right.
medium. Charge is balanced by adding H* and then Cr,0,?- + 6e~ + 14H*+ ———> 2Cr*+ + 7H,O
appropriate number of H,O molecules are added on Step 5. Adding the two half reactions.
the other side. Now add up the two half reactions to get
the final equation. But before doing so,
L}) Example 26
multiply the oxidation half reaction by 3
Balance the following equations by ion electron
to equate the number of electrons.
method :
C,H,0 + H,.O ———»,€,H,0, + 2e- + 2H*)] x 3
(a) Cr,07- + C,H,0 ——C,fH1,0, + Cr+
Cr,O,7 + 14H* + 6e= ———>, 2Cr** + 7H,O
(in acidic medium)
(b) Zn + NO, —_ >Zn** + NH ;* 3C,H,O + Cr,0,2- +8H+t —— 8C,H,O, + 2Cr®* + 4H,O
(in. basic medium)
(b) in Nog =—> Zn?* + NH,*
Solution: Step 1. Write oxidation number of each atom and
Step 1. Separate the equation into the half split the equation into two half equations :
reactions : 0 +5 -2 +2 3 +1
Cr,0,7- + C,H,O ——> C,H,0, + Cr+ Zn
+ NO, ——— Zn** + NH}
Write the oxidation number of the various In this reaction, zinc undergoes oxidation
atoms, and NO, undergoes reduction :
+6 -1 0 +3 Oxidation : Zn —— > Zn**
[Cr,0,7-] + C,H,O ———> C,H,O, + Cr** Reduction: NO, ——— NH,*
The species undergoing change in Step 2. Balance each half reaction separately as :
oxidation number are : (a) Zn ———> 7Zn?*
—l 0 (i) Balance all atoms other than H and O
Oxidation : C,H,O ——— C,H,0O, (already done)
(iz) Add electrons to make up the difference in
+6 +3
oxidation number,
Reduction : Cr,0,7- ———> Cr** Zn ———> Zn?* + 2e-
Step 2. Balance each half reaction separately as : ..(Z)
(a) C,H,O —— C,H,0, (ii) Balance the charges (already done)
(1) The equation is already balanced in terms (iv) No need to add water.
of carbon atoms. The balanced oxidation half reaction is :
(ii) The O.N. of carbon on the left is —1 while Zn ———> Zn?* + 2e-
on the right, it is 0. To account for the (b) ho —— Na
difference in oxidation number, add two (i) Balance all atoms other than H and O
electrons on the right. (already done).
C,H,0O°———>_.C,H,0, + 2e- (iz) Nitrogen changes its oxidation number
(iz) & (iv) Since the reaction is taking place in from +5 to —3 and there is difference of 8
acidic medium, and 2H* on the right anda electrons. To balance the electrons :
H,O molecule on the left NO,- + 8e ——— NH,*
C,H,04 B30 ——> (C,H,0, + 2e7 + 2H* (zit) Balance the charges. Since the reaction
(6) Balancing of reduction half reaction : takes place in basic medium, add 10 OH-
ions on the right side.
Cr.0.7- ———> Cr**
NO,” + 8e— ———> NH,* + 10 OH-
(t) Balance the Cr atoms on both sides of the
(iv) Add water molecules to complete
equation
balancing,
Cr,0,7- ——> 2Cr*
NO,- + 8e— + 7H,0 ——— NH,* + 10 OH-
(iz) The O.N. of Cr atom on the left is +6 and
AEE)
on the right is +3. Thus, each Cr atom has
Step 3. To add up the two half reactions, multiply
gained three electrons t.e. total number of
eq. (1) by 4as:
electrons gained is six. Add six electrons
Zn ———> Zn?* + 2e"| x 4
on the reactant side to account for
difference in O.N. NO, + 7H,O + 8e° ———> NH,* + 10 OH”
Cr0.7 + 6e— ——> 2Cr°* 4Zn + NO,” + 7H,0 ———> AZn2* + NH,* + 10 OH-
sc
L}) Example 27 +7
MODERN'S abc

+2
+ OF CHEMISTRY
-XI

Balance the following redox equations by half Reduction: MnO,- —— Mn*


reaction method : +4 +6

(a) Cr(OH),+ 10, ——>I + Cro ?- Oxidation: SO,* ——> SO,*


These can be balanced by following the
(in. basic medium)
steps as given above.
(5) MnO; +SO, —— Mn** + SO ,-
(i) MnO, ———> Mn**
(in acidic medium)
MnO; + 5e= ———> Mn?*
(c) Br, + H,0, ——— BrO,; + H,O0
MnO; + 5e ———> Mn?* + 4H,O
(in acidic medium)
MnO,- + 8H* + 5e~ ———> Mn** + 4H,O
Solution: (a) Cr(OH), + 10,5 ——— [ + Cr
(ti) ao." +> SO,
Step 1. Separate the equation into two half
SO,*>— SO,?- + 2e-
reactions :
Cr(OH), + 10,5 ———> [T+ CrO,>
SO,7 + HO ——> SO,? + 2e-
Write the oxidation number of various SO,?> + H,0 ——— SO,? + 2e- + 2H*
atoms. (zz) Adding the two half reactions by equating
+3 +5 -1 +6
the number of electrons
Cr(OH), +10,, ——> I + Cr0,2 MnO,- + 8H* + 5e ———> Mn? + 4H,O | x 2
The species undergoing change in SO,7 + H,O ———> SO,? + 2H* + 2e-] x5
oxidation number are :
2MnO,- + 5SO,?> + 6Ht ———> 2Mn** +5SO,2-+3H,O
Oxidation: Cr (OH), ———> ‘CrOi (c) Br, + HO, ——> BrO, + H,O
+5 —l 0 +1 -1 +5 -2
Reduction: lIOo.- —— T Br, + H,O, (Br O,) + H,O
Step 2. Balance each half reaction separately as : Separating the equation into two half
(i) Cr(OH), ——> Cr0,* reactions :
The O.N. of Cr on the left is +3 and on the 0 +5
right it is +6. To account for the difference Oxidation: Br, ———> BrO,~
add 3 electrons on the right.
= 2
Cr(OH), ———> CrO J + 3e7 Reduction: H,O0, ———> H,0
Since, it is occurring in basic medium Balancing the two half reactions separately
Cr(OH), + OH" ———> CrO,7> + 3e>
0 +5
Cr(OH), + OH- + 40H- ———> CrO ? + 4H,O + 8e- (z) Br, ———> BrO,-
or Cr(OH), + 50H- ———> CrO,*- + 4H,0 + 3e- Since each Br loses 5 electrons
(iL) l0,, ——/@ Br, ——— 2BrO,- + 10e~
The oxidation number of iodine on the left Br, + 61,0 ———> 2BrO,- + 10e-
is +5 and on the right is —1. To account for
Br, + 61,0 ———> 2BrO, + 10e~ + 12H*
the difference, add 6 electrons on the left.
I0,~ + 6e fae ail 2
Other steps are : (it) H,O, ——— H,0
Each O atom gains one electron
I0,- + 6e—- ——> I + 30H-
I0,- + 3H,0 + 6e~— ———+ I + 30H + 30H =i 2
H,O, + 2e- ———> H,O
or 10,” + 3H,O + 6e° ——— Il + 60H"
Step 3. Add the two half reactions. To equate the H,O, + 2e- ———> 2H,O
number of electrons, multiply the oxidation H,O, + 2e~ + 2H+ ———> 2H,O
half reaction by 2. (iit) Adding the two half reactions by equating
Cr(OH), + 50H- ———>CrO,? + 4H,O + 3e" |x 2 the number of electrons.
10,- + 3H,O + 667 ———>I + 60H” Br, + 6H,0 ——~ 2BrO,- + 12H* + 10e-
H,O, + 2H* + 2e- ———> 2H,O | x 5
2Cr(OH), + 10,- + 40H- ——>2CrO ¢ +5H,0+1
(65) The equation is : Br, + 5H,0, ———> 2BrO,” + 4H,O + 2H*
MnO, + SO,2> ———>Mn** + SO, L}) Example 28
(in acidic medium) In passing chlorine gas through a concentrated
The equation can be separated as :
solution of alkali we get chloride and chlorate ions.
REDOX REACTIONS

Obtain balanced chemical equation for this or 4P + 30H-+ 3H,O—— PH, + 3H,PO,-
reaction (ii) Jon electron method
Solution: The chemical equation for the reaction is 1. Oxidation number of each atom :
Cl, +OH- ——> Cl +ClO,- + H,O 0 —3 +1

Separating the equation into two half P + OH-———> PH, + H,PO,-


reactions : In this P undergoes oxidation to H,PO,~ and
0 —2 +1 =] +5 -2 +1 -—2 reduction to PH,
Cl, + OH- ———> Cl + ClO,~ + H,O Oxidation P —— H,P0O,.-
0 +5 Reduction : RK, —— PH,
Oxidation : Cl, ———> Cl0,- 2. Balance each reaction separately
0 =] +1
Reduction : Cl, ——> Cl- (a) P —Sab.
4 PO?
Balancing the two half reactions separately, (i) Add electron to make up the difference in
0 +5 oxidation number
(1) Cl, ——— ClO, P= _A,PO,- + e-
Each Cl atom loses 5 electrons (it) Balance OH>™ ions to balance charge
Cl, ———> 2Cl0,- + 10 e- P+20H- ——>H,PO, +e _.....i)
Cl, +6H,O ———> 2ClO,°+ 10e 0 —3
Cl, + 6H,O + 120H- ———> 2ClO,- + 10e-+12 H,O (b) P ——» PH,
(it) Clh —— Cl (1) Add electrons to balance oxidation state
Each Cl atom gains one electron. P +3e ———> PH,
Cl, + 2e° ———> 2CIl- (it) Add OH” ions to balance charge
(az) Adding the two half reactions by equating the number P+3e° —— PH, + 3O0H-
of electrons :
(ut) Add 3H,O molecules on L.H.S. to balance O
Cl, + 6H,O + 120H” ———> 2ClO, + 12H,O + 10e7 and H atoms
Cl, + 2e- —— 2Cl|x5d5 P+3H,O + 3e— —— PH, + 30K ....(ii)
3. Multiply eqn (z) by 3 and add to eqn (iz)
6Cl, + 120H- ———> 2Cl0,- + 10 Cl + 6H,O 4P + 3H,O + 60H- ——-— PH, + 30H + 3H,PO,-
or 3Cl, + 6(OH- ———> ClO, + 5Cl- + 3H,0 or 4P + 3H,O + 30H” ——— PH, + 3H,PO,-
L) Example 29. lL} Example 30
Balance the following equation by oxidation Permanganate (VII) ion, MnO, in basic solution
number as well as ion-electron method : oxidises todide ion, I- to produce molecular iodine
P(s) + OH (aq) ——> PH,(g) + H,PO,(aq) (I,) and manganese (IV) oxide (MnO,). Write a
balanced ionic equation to represent this redox
Solution: (1) Oxidation number method :
reaction. N.C.E.R.T.
1. The skeleton equation with oxidation number is
0 —2 +1 —3 +1 +1 +1 -2 Solution: The skeleton equation for the reaction is :
P+QOH —> PH, +H,PO,- MnO,(aq) + (aq) —~ Mn0O,/s) + 1, (s)
O.N. increases by 1 per atom 1. Separating the equation into two half
reactions :
—] 0)
0) -3 +1
Oxidation : - — |
P+ OH-———3 PH, + H,P O,-
+7 +4

O.N. decreases by 3 per atom Reduction: MnO, ——> MnO,

2. To balance increase and decrease, multiply H,PO,~


Balancing the two half reactions separately
by 3.
—1 0
P + OH-——> PH, + 3H,PO,.-
(i) I- — >» I,
3. Tobalance all atoms (other than H and O) multiply Each I ion loses one electron each
P by 4
2I- —— I, + 2e
4P + OH-———> PH, + 3H,PO,-
+7 +4
4. To balance O atoms add OH™ ions to L.H.S.
(ii) MnO,- ——> MnO,
4P + OH-+ 50H-———> PH, + 3H,PO,- MnO, + 38e-— ——> MnO,
5. To balance H atoms add H,O molecules on L.H.S.
Since the reaction occurs in basic medium, add OH™
and equal number of OH™~ ions on other side
ions on R.H.S. to balance O atoms
4P + 6OH-+ 3H,0O——> PH, + 3H,PO,-+ 30H- MnO,” + 3e—° ——> MnO, + 20H"
MODERN'S abc + OF CHEMISTRY-XI

To balance H atoms add H,O molecules on L.HL.S. APPLICATIONS OF REDOX REACTIONS


and equal number of OH™ ions on the other side
Redox reactions are the basis of many applications
MnO, + 3e7 + 2H,O ——> MnO, + 20H + 20H in chemistry. Some of the important applications of
Adding the two half reactions by equating the redox reactions are :
number of electrons :
1. Photosynthesis. Photosynthesis is used to
2 —— 1,+2e|]x3 convert carbon dioxide and water by chlorophyll of
MnO, + 2H,O + 3e~° ———~ MnO,+ 40H™ | x 2 creen plants in the presence of sunlight to
carbohydrates.

A verification of this equation shows that it is 6CO,(g) + 6H,0 (1) GSB > C,H,,0, (aq) + 60,(g)
balanced in respect to the number of atoms and
charges on both sides. In this case, CO, is reduced to carbohydrates and
water is oxidised to oxygen. The light provides the
=Pral
Ses
alle energy required for the reaction. This reaction is a
source of food for plants and animals.
Q@ 14. Balance the following equations by ion electron 2. In metallurgy. Many metal oxides are
method : reduced to metals by using suitable reducing agents.
(a) NO,- + Bi(s) ——~ Bi*+ + NO, For example, Al,O, is reduced to aluminium by
(in acidic medium) cathodic reduction in electrolytic cell. Fe,O, is reduced
to iron in a blast furnace using coke. Other metals
(6) MnO, +H,C,0, ———> Mn2* + Co,
such as magnesium, calcium, strontium, barium,
(in acidic medium) sodium, potassium, etc., are produced commercially
(c) Cr,0.> 4+ Fett —— > (Crt + Fett by electrolysis methods.
(in acidic medium) 3. Production of chemicals. Many chemical
Mn*t 4+ Br,
compounds of our common need are commercially
(d) MnO,” + Bre ——>
prepared by redox reactions. For example, sodium
(in acidic medium) hydroxide, Cl,, F,, KOH, etc. are produced by
(e) MnO, + SO,7 ——> Mn?*+S0,?> electrolysis which is based on redox reactions.
(acidic medium) 4, Oxidation of fuels. Oxidation of fuels is an
important source of energy which satisfies our daily
(f) Cl,O,(¢g) + H,O,(g¢) ——-> ClO," (aq)+ O,(g) need of life. These are used for obtaining energy for
Q 15. Starting with correctly balanced half reactions, domestic, transport or industrial purposes. They are
write the overall net reactions for the following also used for digestion of food in animals.
changes: Fuels + O, ———> CO, + H,O + Energy
(a) Nitrous acid (HNO,) oxidises, I” to I, in acidic In living cells, glucose (C,H, ,O,) is oxidised to CO,
medium. and H,O in the presence of oxygen and energy is released.
(6) Chloride ion is oxidised to Cl, by MnO,” in acidic C,H,,0, (aq) + 60,.(¢) ——— 6CO,(g) + 6H,O()
medium. + Energy
5. Electrochemical cells or batteries. The
_Answers to Practice Problems electrochemical cells or batteries are most important
© 14. (a) 3NO,- + Bi + 6H* ——> 3NO, + 3H,O + Bi** and are contributing significantly in different fields.
They provide a source of electricity in many motors in
(b) 2MnO,- +5H,C,0, + 6H* ——> 2Mn** + 8H,O small and large equipments. Our vehicles cars, trucks,
+ 10CO, buses, etc. are moving on the storage cells based on
(c) Cr,O,?> + 6Fe?* + 14H+* ——> 2Cr** + 7H,O redox reactions. The electrochemical cells involving
+ 6Fe** reaction between hydrogen and oxygen using hydrogen
and oxygen electrodes in fuel cells meet our demand
(d) 2MnO,- +. 10 Br- + 16H* ——> 2Mn?* + 5Br, of electrical energy in space capsule.
+ 8H,O 6. Quantitative relationships and
(e) 2MnO,- + 5SO,7"+ 6H* +=——> stoichiometry. Redox reactions also play significant
2Mn** + 5 SO,?- + 3H,O role in quantitative analysis of different species by
redox titrations. These titrations involve the reactions
() Cl,0, + 4H,O,+20H- —— between oxidising and reducing agents and help to
2Cl,~ + 40,+5H,O calculate the amount of unknown species in solutions.
© 15. (a) 2I- + 2NO,- + 4H* ——— I, + 2NO + 2H,O REDOX TITRATIONS : REDOX REACTIONS AS
THE BASIS FOR TITRATIONS
(b) 2MnO, + 16H* + 10Cl-_ ———> 5Cl, + 2Mn**
The titration of an acid with a base or of a base
+ 8H,O with an acid is very convenient method for the
REDOX REACTIONS

measurement of quantities of the substances in MnO, + 8H* + 5e ——> Mn?”* + 4H,0] x 2


solution. The titration is carried out by adding a COO-
solution of the reagent of known concentration —> 2CO, + Ye-|x 5
(standard solution) taken in a burette to unknown COO-
solution taken in a flask known as titration flask. The
COO-
solution taken in a burette is called titrant and the
2MnO,- +5 | + 16H*+—> 2Mn** + 10CO, + 8H,O
other solution taken in the titration flask is called COO-
analyte. The titration is continued until the
2. Potassium dichromate titrations
stoichiometric amounts of the two reagents (titrant
and analyte) have been mixed. For example in a Acidified potassium dichromate K,Cr,0O, is also
titration of a strong acid such as HCl and a strong used as an oxidising agent in redox titrations. It is a
base such as NaOH, the following reaction occurs. relatively weak oxidising agent as compared to KMnQO,.
HCI (aq) + NaOH (ag) ——- NaCl (aq) + H,O () However, unlike KMnO,, it does not act as a self indicator
The stoichometric point corresponds to the because no dramatic auto-colour change (as with MnO,~
neutralisation of HCl and NaOH. It is also called titrations) occurs 1n these titrations. In these titrations,
equivalence point or end point. indicators such as diphenylamine, N—phenylanthranilic
There are several methods for determining the acid (and external indicator potassium ferricyanide,
equivalence point. The simplest method is to add K,[Fe(CN),] are commonly used. For example, K,Cr,O0,
to the solution a dye that shows change in colour at oxidises the indicator diphenylamine just after the
the equivalence point. Such dyes are called indicators. equivalence point to give intense blue colour indicating
The indicator helps to determine the end point by the end point. The chemical equation showing Cr,0,7-
showing change in colour. For example, when as an oxidising agent is :
phenolphthalein is added to a basic solution, it gives Cr,0,? + 14 H* + 6e-——> 2Cr** + TH,O
pink colour. When this solution is titrated against an
acid solution, the end point is indicated by Oxidation of ferrous salts with K,Cr,O, may be
disappearance of pink colour. written as:
The same principles apply to redox titrations. 6Fe** + Cr,0,2 + 14H* —> 2Cr** + 6Fe** + 7H,O
However, the indicators used in acid-base titrations lodimetric and Iodometric titrations
are not used for redox titrations. There are a number Iodine, I, also acts as a mild oxidising agent in
of oxidising agents such as acidified KMnO,, acidified solution according to the reaction :
K,Cr,0,, iodine solution, potassium iodate etc. which
can be titrated against reducing agents such as ferrous I, + 2e —— 27°
suphate (or Fe** ions), oxalic acid, sodium thiosulphate, It may be noted that iodine being nonpolar is
potassium iodide etc. For example : very slightly soluble in water. However, it
1. Potassium permanganate titrations dissolves in water in the presence of potassium
Potassium permanganate, KMn0O, is a versatile iodide due to the formation of KI, as :
and strong oxidising agent used in estimation of many KI + 1,—— KI,
reducing agents such as Fe?* ions, oxalates ions etc. Iodine can be used for titrating strong reducing
In these titrations, no indicator is used because KMnO, agents like sodium thiosulphate according to the
is itself intensely coloured (dark purple), So, KMnO, reaction :
acts as a self indicator and when it is titrated against
I, (aq) + S,0,7- (ag) ———> 2T (aq) + S,0,* (aq)
a reducing agent (e.g. Fe*, C30;7-) its colour
disappears. As soon as the reducing agent has been These titration in which iodine is directly titrated
used up, the additional a single drop of potassium against a reducing agent are called iodimetric
permanganate gives a tinge of pink colour to the titrations.
solution. For example, pink colour appears at MnO,- There is another method in which iodine is
concentration as low as 10° mol L~. This ensures a liberated from potassium iodide by some oxidising
minimal ‘overshoot’ in the colour beyond the agent and liberated iodine is titrated against standard
equivalence point i1.e., the point where the reductant sodium thiosulphate solution. These titrations in
and oxidant are equal in terms of their stoichiometry. which liberated todine from an oxidising agent is
The reaction for titration of MnO, against Fe?+ titrated against a standard solution of sodium
ions is: thiosulphate are called todometric titration. For
MnO, + 8H+ + 5e ———~> Mn? + 4H,O example, Cut ions (e.g. CuSO,) is estimated by
Fe?+ —— > Fe#++e]x5 oxidising I- ions (from KI) to liberate I,, which is
titrated against Na,S,O..
MnO, + 5Fe?* + 8H* ——>Mn** + 5Fe*+ + 4H,O
2Cu?* (aq) + 4I-(ag) ——> Cu,I,(s) + I, (aq)
Similarly, the reaction for the titration of MnO,-
against oxalate ions 1s 2Na,8,0, + 1, ——> 2Nal + Na,5,0,
MODERN'S abc + OF CHEMISTRY-XI

In these iodine titrations, end point can be detected


; SOLVED EXAMPLES &
by adding freshly prepared starch solution. It reacts
with iodine in the presence of iodide to form intensely L} Example 31
blue coloured complex. Iodine appearance or How many grams of potassium dichromate are
disappearance can be identified very sharply by this required to oxidise 20.0 g of Fe** in FeSO, to Fe**
method. if the reaction ts carried out in an acidic solution ?
STOICHIOMETRY OF REDOX REACTIONS Solution: When dissolved in water, K,Cr,O,
The quantitative relationships between different dissociates into K* and Cr,O,?- ions and FeSO,
chemical species in redox reactions are derived exactly dissociates into Fe?*and SO,?- ions. The skeleton
equation is :
in the same way as discussed in Unit 1 for general
Fe** + Cr,0,7- ———> Fe** + 2Cr*+
chemical reactions. To perform stoichiometric
The balanced equation is
calculations, we need a balanced redox equation for
the reaction because the coefficients in the equation Cr,O,7 + 6Fe** + 14H* => 6Fe** + 2Cr**+ + 7H,O
From the balanced equation, it is clear that
tell us the ratio by moles in which the reactants
combine and in which the products are formed. If we 1 mol K,Cr,0, = 6 mol FeSO,
know the molarities of the reductant and oxidant and Formula weight of
their volumes used in titration and n, and n, are their K,Cr,0, = 2x 39+2x52+7x 16 = 294
stoichiometric coefficients, then molarity equation may Formula weight of FeSO,
be written as : 56+ 32+4x 16= 152
1 mol K,Cr,0, = 6 mol FeSO,
M,Y, _ M.V, for volumetric estimation. 294 6 x 152
Nn, Ny 6 x 152 g of FeSO, require K,Cr,O, = 294 g
For example, in the titration of FeSO, with
20 g of FeSO, require K,Cr,O,
KMn0O,, the balanced equation is :
10FeSO, + 2KMnO, + 8H,SO, —> 5Fe, (SO,), = 794 x20=6.45¢
6152
+ K,SO, + 2MnSO, + 8H,O K,Cr,O, required to oxidise 20 g of FeSO,
If FeSO, is considered as reagent 1 and KMnO, = 6.45g
is reagent 2, then molarity equation becomes :
MV, _ MV; lL) Example 32
10 °»©Q How many millilitres of 0.125 M KMnO, are
required to react completely with 25.0 mL of
or J
M,V M.V.
0.250 M FeSO, solution in the acidic medium ?
This relation can be used to calculate unknown Solution: The balanced chemical equation is :
quantity if other three quantities are known. MnO, + 5Fe?* + 8H* t———> Mn” +5Fe** + 4H,O
It may be noted that normality equation can also From the balanced equation, it is clear
be used to determine the stoichiometry of redox 1 mol KMnO, = 5 mol FeSO,
reactions. The normality equation is 158 5 x 152
Moles of FeSO, present in 25 mL of 0.250 M solution :
ING, = N,V,
0.250
If we know the normality (N,) of any solution of = 000 * 25 = 0.00625 mol
an oxidising agent and volume (V,) of this solution
Let us determine the number of moles of KMnO,
which completely oxidises the known volume (V,) of that must react
the solution of a reducing agent then normality of 5 mol of FeSO, react with KMnO,
reducing agent can be calculated or vice-versa. = 1 mol
Once the normality of the reducing (or the 0.00625 mol of FeSO, will react with KMnO,
oxidising) agent is known, its strength can be 1
calculated as : = 5 * 0.00625 = 0.00125 mol
Strength = Normality of the solution x Now, we are to calculate the volume of 0.125 M
Equivalent weight of the oxidising KMn0O, solution which contain 0.00125 mol.
(or the reducing) agent. According to definition of molarity, 0.125 M solution
However, it may be noted that molarity equation means that
is preferred over normality equation. The basic rules 0.125 mol of KMnO, is present in
for calculations are same as discussed in Unit 1. = 1000 mL
0.00125 mol of KMnO, is present in
The main important step is to write the balanced
1000
redox equation. = 0125 x 0.00125 =10 mL
REDOX REACTIONS

This can also be calculated as: Solution: The balanced chemical equation is :
. “Moles
x 1000 2MnO, + 5C,0,?> + 16H+ ——-> 2Mn?* + 10CO, + 8H,O
Molarity = <a Applying molarity equation,
0.00125 x 1000
v1 = Volume
[AY Ma%s)
0.00125 x 1000 ™% Jxmmo, \ "2 Jo,07
Volume = 0125. =10 mL
1. 30 _ -M,
x 20
50 2 ~ 5
L}) Example 33
How many millilitres of 0.025 M K,Cr,0O,
80x 5
are required to react completely with 25.0 mL of
Me = 20 Miaeag = O75M
Molecular wt. of C,0,?> = 88
0.20 M solution of FeSO, ?
Wt. of C,0,2> = 0.075 x 88 = 6.6 g
Solution: The balanced chemical equation is
Amount of C,0¢- in 250 mL = — x 250 = 1.65¢
6Fe* + 6Cr,0,7 + 14H* ———> 6Fe**++ 2Cr*+ + 7H,O
. 1.65
1 mol of K,Cr,O0, = 6 mol of FeSO, % Purity = 35 x 100 = 91.7%
Moles of FeSO, present in 25.0 mL of 0.20 M solution
Oh Example 36
070 «25.0Oe =0.005
in
mol 2.48 g of hydrated sodium thiosulphate
Now 6 mol of FeSO, require K,Cr,0. (Na,S,O.-xH,O) was dissolved per litre of the
= 1 mol solution. 25 mL of this solution required 12.5 mL
1 of M/100 todine solution. Determine the value
= 6 x 0.005 = 0.000833 mol Oty.
Let us calculate the volume of K,Cr,O, solution Solution: The balanced chemical equation is :
containing 0.000833 mol 2Na,5,0, + lL——— Na,58,0,+ 2Nal
0.025 mol of K,Cr,O, are present in
Let the molarity of Na,S,O0,-xH,O be M,
= 1000 mL
Applying molarity equation,
0.000833 mol of K,Cr,O, are present in
_ 1000
x0.000833
0.025 = 33.3 mL
[MM [MX
M Jyas0, \ Me It z
LJ Example 34 Mx _ 1 125
Determine the volume of — KMnO, solution 2 = T00* 1
required to react completely with 25.0 mL of M/5 (12.52
oxalic acid solution. M, = To0x25 001M
Solution: The balanced chemical equation is Molecular wt. of Na,S,0,-«H,O
=2xK254+2x524+3x164+xx18
2KMn0O, + 5(COOH), + 3H,SO0, ———>
= 158 + 18x
K,S0, + 2MnSQO, + 10CO, + 8H,O Amount of Na,5,0,-«H,O = 0.01 x (158 + 18x) g
From the balanced equation, it is clear that But actual amount = 2.48 ¢
2 mol of KMnO, = 5 mol of (COOH), . 0.011158 + 18x) = 2.48
Applying molarity equation - 2.48
(158 + 18x) = —— = 248
18x = 248 — 158 = 90
™ IKMnO, M5 COOH), 90
x= 18 = 5.

Ly Min 1, 25.05
10° 2 ~5*
25.0x10x 2
or Vi = x5 = 20.0 mL.
@ 16. How many millilitres of 0.125 M K,Cr,O, are required
) Example 35 to react completely with 25.0 mL of 0.250 M
1,80 g of impure sample of oxalate was dissolved FeSO, solution in acidic medium?
in water and the solution made to 250 mL. On 017. A 16.4 ml volume of 0.14 M KMn0O, solution is
titration 20 mL of this solution required 30 mL required to oxidise 20.0 ml of FeSO, solution in acidic
of M/50 KMnO, solution. Calculate the medium. What is the concentration of FeSO,
percentage purity of the sample. solution ?
MODERN'S abc + OF CHEMISTRY-XI

@ 18. How many millilitres of 0.150 M KMnO, solution are . 0.014 mol of KMnO, is present in solution
required to react completely with 35.0 ml of 1000 |
0.150 M SnCl, solution, given the chemical equation = op * 0.014 = 280 mL.
for the reaction :
5 Sn*+ + 2 MnO; + 16H* > 5 Sn* + 2Mn**++ 8H,O INDIRECT REDOX REACTIONS:
Q@ 19. Calculate the concentration of hypo (Na,5,0,-5H,O) ELECTROCHEMICAL CELLS
solution in gL~! if 10.0 mL of this solution In the direct redox reaction, the transference of
decolourised 25 mL of M/50 iodine solution.
electrons is limited to very small distances and,
@ 20. Calculate the volume of 0.05 M KMnO, solution therefore, no useful electrical work could be obtained.
required to completely oxidise 3.20 g of oxalic acid
In these reactions, chemical energy appears as heat.
(H,C,O,) in acidic medium.
However, if the transference of electrons from zinc
to copper ions is allowed to occur through some
=== Answers to Practice Problems == metallic wires, useful electrical work could be
© 16. The balance equation is performed. Such redox reactions are called indirect
6Fe** + Cr,0,7 + 14 Ht —> 6Fe* + 2Cr*+ + 7H,O redox reactions and electrical energy is produced
From the equation, it is clear during such spontaneous reactions instead of heat
energy. Let us consider the above redox reaction
M,V, = MLV,
(ny, Pe 7 —The bases between Zn metal and Cu?* ions performed indirectly
in separate vessels.
0.125 x V, _ 0,250 x 25.0 The arrangement consists of two beakers, one
— 6
of which contains 1.0 M solution of zinc sulphate
0.250 x 25.0 and the other 1.0 M solution of copper sulphate. A
= 8.33 mL
eles 0.125 x 6
zine rod is dipped into ZnSO, solution while a copper
rod is dipped into CuSO, solution. These rods are
© 17. MnO, + 5Fe2+ + 8 Ht —> Mn?+ + 5Fe®* + 4H,O
known as electrodes. The metallic rods in the beaker
are connected to the ammeter by means of an
MY, _ MV, insulated wire through a key having an on or off
1 5
switch. The solutions in the two beakers are
or Lit
x164 _ 20.0
20.0 xM,
0.14 x 16.4
1 5 connected by an inverted U-tube containing
concentrated aqueous solution of some electrolyte
(0.14 x 1645
M, =——x) = 0.574 mL such as KCI, KNO,, or NH,NO, which does not
undergo a chemical change during the process. The
© 18. 5Sn?* + 2MnO,- + 16 H* —> 5Sn** + 2Mn** +8 H,O
two openings of the U-tube are plugged with some
MY, _ MV, porous material such as glass wool or cotton (Fig. 3).
5 2
The U-tube which connects the two glass beakers is
35.0 x 0.150 _ 0,150 x Vy called a salt bridge. Its function will be discussed
: 5 2 separately.
35.0 x 0.150 x 2
Ve = 0350xTan Oe
© 19. 28,0,7 + 1, —> S8,0,? + 2I- ay
Key 1 —— =
M, 9x10 =
1an 25 or M, =0.1M
mmeter Current

Salt bridge
Conc. of Na,S,0,-5H,O 0.1 x 248 = 24.8 gI-,
© 20. 2KMnO, + 5(COOH), + 3H,SO, —>
K,S0, + 2MnSO, + 8H,O + 10CO,

Moles of oxalic acid = = 0.035


90)

5 moles of (COOH), = 2 moles of KMnO,


“1M ZnSO, (aq)
0.035 moles of (COOH), = = x 0.035 = 0.014 mol.
Fig. 3. Indirect redox reaction between zinc and copper
Now, 0.05 mol of KMnO, is present in solution sulphate.
= 1000 mL.
REDOX REACTIONS

When the switch is in the off position, no reaction


takes place in either of the beakers and no current
flows through the metallic wire. When the switch is
in the on position, the following observations can be
made :
(t) There is a flow of electric current through the
external circuit.
(it) The zinc rod loses its weight and the copper
plate gains weight.
(zit) The concentration of ZnSO, solution increases
and that of CuSO, solution decreases.
(iv) The solutions in the two beakers remain Oxidation
?7n— fn ‘+ 2a Cun*+ 22 — Cu
electrically neutral.
From these observations, it is clear that the overall
reaction is : Fig. 4. Working of an electrochemical! cell.
Zn(s) + Cu2*+ (ag) ———> Zn?* (ag) + Cu(s)
During the reaction, zinc is oxidised to Zn* ions neither involved in any electrochemical change nor
which go into the solution. The electrons released do they react chemically with the electrolytes
Zn(s) ———> Zn?* (aq) + 2e7 electrically connected by them. Generally salts like
at the electrode move towards the other electrode KCI, KNO., NH,NO,, etc. are used. For preparing a
through outer circuit. These electrons are accepted by salt bridge, gelatin or agar agar is dissolved in a hot
Cu2+ ions of CuSO , solution which are reduced to aqueous solution of KCI, KNO, or NH,NO, and filled
copper. The metal gets deposited on the copper in a U-tube. On cooling, the agar agar solution becomes
solid and is used as salt bridge.
electrode.
Cu?* (ag) + 2e7- ———> Cu(s) The important functions of the salt bridge are :
Thus, zinc plate gradually loses its weight while (i) Salt bridge connects the solutions in the two
copper plate gains in weight. half cells and completes the cell circuit. For the flow
The redox reaction occurring in this indirect of electric current through the circuit, we know that
arrangement is the same as that taking place in a single there should not be broken contacts throughout the
beaker. The basic difference between the two is that in circuit. As already explained, the solutions of the half
the direct redox reaction, energy is liberated in the form cells are connected by a salt bridge and their electrodes
of heat energy whereas in indirect redox reaction, by means of a wire. Therefore, the salt bridge
completes the circuit.
energy is liberated in the form of electrical energy. Thus,
indirect redox reactions lead to the production of (it) Salt bridge keeps the solutions in the two
electrical energy. The arrangement for carrying out half cells electrically neutral. In order to understand
indirect redox reactions is called electrochemical cell. this, let us consider Zn—CuSO, cell. In the oxidation
Thus, an electrochemical cell is half cell, zinc atoms lose electrons and change into
Zn**+ ions. The positively charged Zn?* ions pass
a device used to convert chemical energy
into the solution. After some time, this results
produced in a redox reaction into electrical
into accumulation of extra positive charge in the
energy. solution around the anode. Similarly, in the reduction
The electrochemical cells are also called galvanic half cell, Cu?* ions accept electrons and get reduced
cells or voltaic cells after the names of the scientists to Cu. After some time, the solution around cathode
Luigi Galvani (1780) and Alessandro Volta (1800) who will acquire extra negative charge due to excess of
were the first to device such experiments for 50 ions. The accumulation of positive charge
converting chemical energy into electrical energy. around zinc rod will prevent the further flow of
electrons from the zinc rod. Similarly, the
Salt Bridge and its Functions accumulation of negative charge around copper
A salt bridge has an important role to play in an electrode will prevent the flow of electrons to the
electrochemical cell. It is usually an inverted U- tube copper ions. Thus, in the absence of salt bridge the
filled with concentrated solution of inert electrolytes flow of electrons will occur only momentarily and the
(Fig 4). An inert electrolyte is the one whose ions are cell will stop working.
MODERN'S abc + OF CHEMISTRY-XI

However, the accumulation of charges in the two Oxidation half Cu(s) ——> Cu?*(aq)+2e7
half cells is prevented by using a salt bridge. When reaction
the concentration of Zn?* ions around anode increases, Reduction 2Ag* (ag) + 2e~7 ———> 2Ag(s)
sufficient number of Cl ions migrate from the salt reaction
bridge to the anode half cell. Similarly, to neutralise
Net reaction Cu(s) + 2Ag* (ag)———> Cu** (aq) + 2Ag(s)
the excess negative charge due to the additional So,
ions in cathode half cell, sufficient number of K* ions The electrons lost in the oxidation half reaction
migrate from the salt bridge to this half cell. Thus, must be equal to the electrons gained in the reduction
the salt bridge provides cations and anions to replace half reaction. In order to equate them, the ionic
the ions lost or produced in the two half cells to equations may be multiplied suitably if necessary.
maintain electrical neutrality.
| SOLVED EXAMPLES
It may be noted that the salt bridge in a cell can be
replaced by a porous partition which allows the migration L) Example37
of ions without allowing the solutions to intermix. Write the half reactions for the following redox
The working of the electrochemical cell is reactions :
represented in Fig. 4. (a) 2Fe**(aq) + 2I- (ag) ——>2Fe**(aq) + I,(aq)
Half Cells and Half Cell Reactions (b) Zn(s)+2H (aq) ——> Zn?*(aq) + Hg)
In the indirect redox reactions, the oxidation and
(c) Al(s) + 3Ag*t (aq) ———>Al?*(aq) + 3Ag(s)
reduction occur separately and these are called half cells.
The reactions occurring in half cells are called half cell Solution:
reactions. The half cell in which oxidation (loss of (a) 2Fe*+(aq) + 21- (aq) ——>2Fe**(aq) + I,(aq)
electrons) occurs is called oxidation half cell and the
Oxidation half reaction :
reaction taking place in it is called oxidation half cell
reaction. Similarly, half cell in which reduction (gain of 21 (aq) ——— I,(aq) + 2e7
electrons) occurs is called reduction half cell and the Reduction half reaction :
reaction taking place in it is called reduction half cell
2Fe*+(ag) + 2e- (ag) ——>2F
e**(aq)
reaction. The two half cells constitute a redox couple. A
redox couple is defined as having together the oxidized (b) Zn(s) + 2H" (ag) ——> Zn?**(aq) + Hyg)
and reduced forms of a substance taking part in an Oxidation half reaction :
oxidation or reduction half reaction. Zn(s) ———> Zn**(aq) + 2e-
The net reaction is the sum of two half cell reactions.
Reduction half reaction :
This can be illustrated by the following examples :
Reaction between Zn metal and Cu** ions (CuSO, 2H* (aq) ——— H,(g)
solution) may be written as: (c) Al(s) + 3Ag* (ag) ——> AI**(aq) + 3Ag(s)
Oxidation half Zn(s) ———> Zn?* (aq) + 2e7 Oxidation half reaction :
reaction
Reduction half Cu?* (ag) + 2e- ———> Cu(s) Al(s) ———> Al#*(aq) + 3e7
reaction Reduction half reaction :
Net reaction Zn(s) + Cu?* (ag) ———> Zn?* (aq)
+ Cu(s) 3Ag* (aq) ——— 3Ag (s)
Similarly, in case of aluminium metal displacing
silver ions from a solution of silver nitrate we get the REPRESENTATION OF AN
half reactions as : ELECTROCHEMICAL CELL
Oxidation half Al(s) ———> Al** (aq) + 3e7 We have seen that an electrochemical cell or
reaction
galvanic cell consists of two electrodes i.e., anode and
Reduction half Ag*(aq) + ee ———> Ag(s) cathode. The electrolyte solutions containing these
reaction electrodes are called half cells. When these two half
To balance the number of electrons, we multiply cells are combined, a cell is formed. The following
reduction half reaction by 3 and then add so that net universally accepted conventions are used for
reaction becomes representing an electrochemical cell.
Al(s) + 3Agt(ag) ——— Al**(aq) + dAg(s) 1.A galvanic cell is represented by writing the anode
Similarly, reaction between Cu metal and Ag* ions (where oxidation occurs) on the left hand side and

(AgNO, solution) may be written as : cathode (where reduction occurs) on the right hand side.
REDOX REACTIONS

2. The anode of the cell is represented by writing Sometimes negative and positive signs are also
metal first and then the electrolyte. The two are put on the electrodes. Anode is a negative pole while
separated either by a semicolon (;) or by a vertical line. cathode acts as positive pole. The electrons flow from
The electrolyte may be written as ionic species or by the negative pole (anode) to the positive pole (cathode)
the formula of the whole compound. For example, in the external circuit. On the other hand,
Zn; Zn** or Zn | Zn** conventionally, the current is said to flow in the
opposite direction.
For further information, the concentration of the
electrolyte is also mentioned within bracket after the The important generalisations in an electro-
cation, 1L.e., chemical cell may be summed up as :
Zn;Zn7*(1M) or Zn | Zn** (1M)
Oxidation occurs at anode.
In case of gas electrodes such as hydrogen
Reduction occurs at cathode.
electrode, inert metal which serves as the electric
contact for the electrode is also written as: Cathode is positive and anode is negative in a
galvanic cell.
Pt, H, (1 atm); H* (1 M)
In this case, Pt, H, (1 atm) stands for hydrogen Electrons flow from anode to cathode in the
and platinum together as solid phase, in which H, is external circuit.
absorbed under 1 atm pressure. Comma is used to Chemical energy is converted into electrical
indicate that they form one phase. energy.
3. The cathode of the cell is represented by writing
the electrolyte first (or cation of the electrolyte) and ELECTRODE POTENTIAL
then metal. For example, When an electrode say copper, is immersed in
Cu2*;Cu or Cu**+!Cu or Cu?*(1M)! Cu contact with a solution of its ions, then either of the
Cl (0.1 M); Cl, (1 atm), Pt following three possibilities can take place :
4. The salt bridge which separates the two half (a) The metal ions (Cu2+) may collide with the
cells is indicated by two vertical lines. For example, electrode and do not undergo any change.
Zn | Zn?* (1M) ||Cu?* (1M) | Cu (b) Cu?* ions may collide with the electrode, gain
The complete formula of a cell may be represented electrons and get converted into metal atoms (i.e., the
as follows : tons are reduced).
Electrons Cu*t + 2e7- ——> Cu (Reduction)
+ (c) Cu atoms on the electrode may lose electrons

Sf to the electrode and become Cu** ions and go into the


solution.
Cu —— Cu?*t + 2e- (Oxidation)
Zn | Zn?+ (1M) | Cu2+ (1M) | Cu
These possibilities are shown in Fig. 5.
Anode Cathode
Now, if the metal has relatively high tendency to
Oxidation occurs get oxidised, its atoms will lose electrons readily and
form Cu?+ ions, which go into the solution. The

COPPER ROD COPPER ROD

Fig. 5. Concept of electrode potential.


MODERN'S abc + OF CHEMISTRY
-XI

electrons lost on the electrode would be accumulated Since the potential depends upon the concen-
on the metal electrode and the electrode acquires a tration of the ions in solution we adopt a standard as
slight negative charge with respect to the solution. 1 mole of tons per litre of solution. This is called
Some of the Cu”* ions from the solution will take up standard electrode potential.
electrons and become Cu atoms. After some time, an The absolute value of the electrode potential
equilibrium will be established as : cannot be determined experimentally because
Cu(s) === Cu? + 2e neither the oxidation nor the reduction reaction can
Electrode Solution (on electrode) occur by itself. This difficulty can be solved by
selecting one of the electrodes as a reference
When such an equilibrium is attained, it results
in separation of charges (negative on the electrode with
electrode and arbitrarily fixing the potential of this
electrode as zero. For this purpose, reversible
respect to the solution).
hydrogen electrode has been universally accepted
Similarly, if the metal ions have relatively greater
as reference electrode. It is called standard hydrogen
tendency to get reduced, they will take electrons from
electrode (S.H.E.) or normal hydrogen electrode
the electrode. As a result, a net positive charge will be
(N.H.E.).
developed on the electrode with respect to the solution.
This will also result into separation of charges (positive Standard Hydrogen Electrode
on the electrode with respect to the solution). It consists of platinum wire sealed in a glass
Due to separation of charges between the electrode tube and has a platinum foil attached to it.
and the solution, an electrical potential is set up
The foil is coated with finely divided platinum
between metal electrode and its solution. The electrical
and acts as platinum electrode. It is dipped into an
potential difference set up between the metal and its
acid solution containing H* ionsin 1 M concentration
solution is known as electrode potential. Thus, the
(1 M HCl). Pure hydrogen gas at 1 atmospheric
electrode potential is a measure of tendency of an
pressure is constantly bubbled into the solution at
electrode in a half cell to gain or lose electrons. The
constant temperature of 298 K. The surface of the
electrode potential may be of two types :
foil acts as a site for the reaction.
@ Oxidation potential. The tendency of an
electrode to lose electrons or to get oxidised is called its
oxidation potential. Thus, oxidation potentials give
the tendency of an electrode to lose electrons, 1.e.,
M(s) ——— M" (aq) + new —<— H, gas at
1 atm
e.8., Zn(s) ———=> Zn** (ag) + 2e7
H, —— 2H* (aq) + 2e7 Bubbles of :
escaping Hp
(ii) Reduction potential. The tendency of an
electrode to gain electrons or to get reduced is called its
reduction potential. Therefore, it measures the
tendency of an electrode to gain electrons i.e.,
M"+ (ag) + ne~7 .[a
<==
J —M(s) Platinum
foil
e.8., Cu” (ag) + 2e°- iS——_—NC
_
ul)
2H* (aq) + 2e7 _
—=——~ _H,f(g) 1M HCI solution
It is evident that the oxidation potential is the
reverse of reduction potential. For example, if Fig. 6. Standard hydrogen electrode.
reduction potential of Zn is — 0.76 volts, its oxidation
potential is + 0.76 volts. The following reactions occur in this half cell
According to the present convention, the half cell depending upon whether it acts as an anode or asa
reactions are always written as reduction half cathode.
reactions and their potentials are represented as It S.H.E. acts as anode
reduction potentials. Hg) ——— 2Ht + 2e-
The electrode potential depends upon It S.H.E. acts as cathode
(t) the nature of the metal and its tons 2H*t + 2e~- ——— H,(g)
(11) concentration of the tons in the solution and The standard hydrogen electrode is also
(uu) temperature. regarded as a reversible electrode.
REDOX REACTIONS

H,(g) ——_- 2H + 2e™ Now, ifthe cell potential and the electrode potential for
one of the half cell reactions were known, the electrode
The electrode potential of an electrode can be
potential of the other electrode can be calculated. To
determined by connecting its half cell with a
solve the problem, therefore, a standard hydrogen
standard hydrogen electrode. The electrode
electrode has been chosen and it has been assigned a
potential of the standard hydrogen electrode is
assigned zero electrode potential. The electrode standard reduction potential of zero volts. All other
potential of a metal electrode as determined with half cell reactions can be compared with this standard
respect to a standard or normal hydrogen electrode and a set of E’ values is obtained.
is called electrode potential (E). If the e In case, the electron accepting tendency of the metal
concentration of the ions is 1 mol L~ solution, the electrode is more than that of a 8.H.E., its standard
electrode potential is also called standard reduction potential gets a positive sign
electrode potential (E°). e In case the electron accepting tendency of the metal
Standard hydrogen electrode is represented electrode is lesser than that of S.H.E. its standard
as : Pt, H,(g) |H* (aq) and has zero potential at all reduction potential gets a negative sign. It must be
temperatures. remembered that according to latest convention, all
standard potentials are taken asreduction potentials.
E.M.F. OR CELL POTENTIAL OF A CELL The electrode at which reduction occurs with respect
As discussed earlier, an electrochemical cell to S.H.E. (7.e., weaker reducing agent than H*/H, couple)
has +ve reduction potential.
consists of two half cells. One of the electrodes in a
half cell has a higher value of reduction potential as The electrode at which oxidation occurs with respect
to $.H.E. (7.e., stronger reducing agent than H*/H, couple)
compared to the other. As a result of this potential
has — ve reduction potential.
difference, there is a flow of electrons from the
electrode with a lower reduction potential (or higher The standard electrode potentials are very
oxidation potential) to the electrode with higher important and give a lot of useful information. The
reduction potential (or lower oxidation potential). The basic point regarding the series is the stability of
difference between the electrode potentials of the two oxidized form or reduced form. If the standard
electrodes constituting an elecrochemical cell is known electrode potential is more than zero, then its reduced
as electromotive force or cell potential of a cell. form will be more stable compared to hydrogen gas.
In other words, it will be readily reduced in comparison
The electromotive force is commonly abbreviated
to hydrogen and therefore, acts as strong oxidizing
as E.M.F. and is expressed in volts. The E.M.F. of a
agent. On the other hand; if the standard electrode
cell may be expressed in terms of the difference in the
potential is negative than hydrogen, then hydrogen
reduction electrode potential as :
gas is more stable than the reduced form of the species.
E.M.F. = D sabetanee reduced — E.ubstance oxidised In other words, tt will be readily oxidized in comparison
to hydrogen and therefore, acts as strong reducing
agent. Similarly, we can compare two species. The
or E.M.F. = BD thud 7 EB node species having higher electrode potential will be
where E, and E, represent electrode potential of readily reduced (acts as oxidising agent) in comparison
electrode on right hand side and electrode on left hand to the other.
side respectively. It depends upon the nature of the ELECTROCHEMICAL SERIES
electrodes and the concentrations of the solutions in
In the previous section, we have discussed the
the two half cells. For example, the E.M.F. of a Daniell
measurement of the standard reduction potentials (E°)
cell in which the concentrations of aqueous solutions of of a few electrodes. In a similar manner, the E° values
CuSO, and ZnSO, in the two half cells is 1 M at 298 K of other electrodes can also be measured. When the
is 1.10 volts. standard reduction potentials of the electrodes are
It may be noted that it is not possible to measure arranged in an increasing order, a series called
experimentally the absolute value of a single electrode electrochemical series is obtained.
because a half cell reaction cannot take place The arrangement of elements in order of
independently. We can measure only the difference decreasing reduction potential values ts called
between the electrode potentials of any two half cell electrochemical series.
reactions. Then, a question arises as to how we can The electrochemical series, also called activity
obtain the reduction potential of any single electrode. series, of some typical electrodes is given in Table 1.
Tae Table 1. Standard reduction electrode potentials at 298 K.
MODERN'S abc + OF CHEMISTRY-A&I

Electrode Electrodereaction E* (V)


(Oxidized form +ne~ —— Reduced form)

Ba lie Strongest oxidizing F, (g) + 2e° —> 2F (aq) Weakest reducing 2.87
Co** | Co*+* agent Co* +e —> Co* agent 1.81
H,O, | H,O H,O, + 2H* + 2e- —» 2H,O 1.78
MnO,, H+ | Mn?+ MnO, (s) + 4H* + 2e" —> Mn?* + 2H,O 1.61
Au*+ | Au Au®+ + 3e-5 ——> Au (s) 1.50
MnO,- | Mn** MnO,- + 8H* + 5e ——> Mn** + 4H,O 1.49
Cl, |Cl Cl, (g) + 2e- —> 2C 1.36
Cr Oo ne ier Cr,0,2- + 4H*+ + 6e> —> 2Cr** + 7H,O 1.33
O,, H* | H,O O, + 4H* + 4e- —> 2H,0 1.23
Br, | Br~ > Br, @) + 2e° ——> 2Br- = 1.09
NO,-, H+ | NO Z NO,- + 4H* + 3e- —» NO(g)+2H,0 Z 0.97
He**' | Hg.** C5 2Hg** + 2e7 —> Hg,** S 0.92
ClO- | Cl- ze ClO- + H,O + 2e°- —> Cl +20H- is 0.89
Hg*+ | Hg Hg** + 2e° ——> He Zz 0.85
Agt | Ag = Agt+e— — > Ag 2) 0.80
Hg,-* | Hg - Hg,** +e —> 2Hg = 0.79
Fe®+ | Fe*+ iS Fe*+ +e- —+>_Fe**+ a 0.78
MnO,- | MnO,?- “i MnO, +e 455 Mn0,4 = 0.56
ja -) I, + 2e7 —> 21; ) 0.54
Cu’? | Cu ee Cu* +e —> Cu = 0.52
Cu** | Cu ds) Cu*t + 2e- —3) Cu ) 0.36
AgCl |Ag a AgCl +e —5 /Ag+Ch 0.22
Cu* | Cut e Cu #e- —4 Cut 0.15
AgBr | Ag wD AgBr + e7 — > Ag+ Br- er) 0.10

Ht | H,
Fe?+ | Fe Fe?+ +3e- —— Fe —0.04
Pb2* | Pb z Pb2+/+ 2e- —» Pb z 0.13
Sn?* | Sn 7 Sn** + 2e- —> Sn a —0.16
Ni | Ni S Ni2+ + 2e- —> Ni s 0.25
Fe*+ | Fe Ps Fe*+ + 2e-° —— + Fe js —0,.44
Cr*+ | Cr Z Cr*+ + 3e-° — > Cr Z —0).74
Zn** | Zn Zn*+ + 2e- — > Zn —).76
Al?+ | Al Al*+ + 3e-§ ——> +Al —1.66
Mg**+ | Mg Mg?* + 2e- —> Mg —2.36
Ce*+ | Ce eee fee ee ~2.48
Nat | Na Nat +e ——> Na —2.71
Ca2+ | Ca Ca** + 2e- —-> Ca ~2.87
Ba?*+ | Ba Ba** + 2e- ——> Ba Se |
Cs* | Cs Cst +e ——> Cs —2.92
Kt | K Weakest oxidizing tee Strongest —?2.93
Lit | Li agent Lit+e — Li reducing agent §=—_s 95
It may be noted that the standard oxidation are given below :
potential will have the same magnitude but opposite 1. Relative strengths of oxidising and
sign. For example, the standard oxidation potential of
reducing agents.
first electrode, Li, Lit is + 3.05 V and standard oxidation
potential of the last electrode F’, F, is — 2.87 V. The electrochemical series helps us to pick out
Applications of Electrochemical Series substances that are good oxidising agents and those
The main applications of the electrochemical series which are good reducing agents. In the electrochemical
REDOX REACTIONS

series, the substances are arranged in the decreasing 3. Calculation of the E.M.F. of the cell
order of reduction potential i.e. decreasing tendency
The E.M.F. of the cell which is the difference
for reduction to occur or power as oxidising agent.
between the reduction potential of the cathode and
Therefore, the elements at the top of the table have anode is determined by the following steps :
maximum tendency to get reduced and consequently,
they will act as good oxidising agents. On the other Step I. Write the two half cell reactions in such a
hand, the substances at the bottom of the table have way that the reaction taking place at the left hand
electrode is written as an oxidation reaction and that
lower reduction potential, therefore, they have least
tendency to get reduced. Consequently, they may be
taking place at the right hand electrode is written as
reduction.
oxidised and act as good reducing agents. Thus, the
substances which have lower reduction potentials are Step IT. Multiply one of the equations if necessary
stronger reducing agents while those which have by suitable number to equate the number of electrons
higher reduction potentials are stronger oxidising in the two equations. However, it may be noted that
agents. For example, a very high negative reduction electrode potential values, E° are not multiplied.
potential of lithium electrode indicates that Lit cannot Step III. The E.M.F. of the cell is equal to the
accept electrons easily from other species. In other difference between the standard electrode potential
words, lithium can readily lose electrons and, of the right hand side electrode (E%,) and the standard
therefore, behaves as strong reducing agent. Thus, all electrode potential of the left hand side electrode (HE ).
the substances appearing at the bottom of the series Thus,
behave as good reducing agents. For example,
Ee = ER- Ey
Li, K are the best reducing agents.
The electrode potentials of both the electrodes
Similarly, the substances at the top of the table are takentobereduction potentials.
have high reduction potentials and they can be easily Step IV. Ifthe E.M.F. of the cell is +ve, the reaction
reduced and act as oxidising agents. For example, is feasible in the given direction. But if E.M.F. of the
fluorine has the highest electrode potential indicating cell is —ve, the cell reaction is not feasible in the given
that fluorine is very powerful oxidizing agent. From direction. The reaction must be occurring in the
the position of the electrodes in the table we can reverse direction. Thus, to get positive value for the
conclude that H+ is a better oxidising agent than Zn?* E..M.F. of the cell, the electrodes are to be reversed.
but Cu? is a better oxidising agent than H*. Thus,
good oxidising agents are the substances appearing SOLVED EXAMPLES
at the top of the table.
L}) Example 38
2. Comparison of reactivities of metals
Write the half cell reaction and the overall cell
The reduction potential values indicate the
reaction for the electrochemical cell :
relative ease with which the various species of metals
and ions may be oxidised or reduced. The metals Zn |Zn** (1.0 M) |]Pb?* (1.0 M) |Pb
having lower reduction potential, will not be reduced Calculate the standard e.m.f. for the cell if
easily. On the contrary, such metals would be easily standard electrode potentials (reduction) for
oxidized to their ions losing electrons. These electrons Pb** | Pb and Zn?* | Zn electrodes are — 0.126 V
and — 0.763 V respectively.
would reduce the other metals having higher
reduction potentials. In other words, a metal having Solution: Zn electrode acts as anode while Pb electrode
acts as cathode and, therefore, oxidation occurs at zinc
smaller reduction potential can displace metals
electrode and reduction occurs at lead electrode. The
having larger reduction potentials from the solutions half cell reactions are:
of their salts. For example, copper has lower Oxidation half reaction :
reduction potential than silver in the electrochemical Zn(s) ——> Zn?+ (aq) + 2e-
series, therefore, if copper foils are added to AgNO, Reduction half reaction :
solution, silver is displaced from the solution. In
Pb?+ (aq) + 2e- ——> Pb(s)
general a metal occupying lower position in the series
can displace the metals lying above it from the Overall cell reaction :
solutions of their salts. In other words, we can say Zn(s) + Pb?* (ag) —> Zn?* (aq) + Pb(s)
that the metals occupying lower positions in the
electrochemical series are more reactive in displacing
E.M.F. = E°,-E°
the other metals from the solutions of their salts.
— 0.126 — (-0.763)
= 0.637 V.
MODERN'S abc + OF CHEMISTRY
-XI

Note. The EMF of the cell can also simply be


calculated as :
= SOLVED EXAMPLES :

Be = Ep - BE’, () Example 39
If the value of E°_,,, is positive, then the cell is Iodine (I,) and bromine (Br,) are added to a
correctly represented, i.e., oxidation occurs at left solution containing todide (I) and bromide (Br-)
electrode (anode) and reduction occurs at the right ions. What reaction would occur if the
electrode (cathode). On the other hand, if the value concentration of each species is 1M ? The
of E°.,,, comes out to be negative, then the cell is electrode potentials for the reactions are :
wrongly represented. Its order is reversed to get fot 2e—-§ — 2 Ee =+0.54 V
positive value for E°
Br, + 2e0°§ ———> 2Br> be=+ 7.02 V
4, Predicting feasibility of a redox reaction. Solution: Since the reduction potential of Br, is more
than that of I,, it means that bromine can readily be
The electrochemical series helps in finding out
reduced. Therefore, > will be oxidised to I, and this
whether a given redox reaction is feasible or not from
reaction should be written as oxidation. Therefore,
the E° values of the two electrodes. the following reactions will occur :
In general, a redox reaction is feasible only if the Br, + 2e7 ———> 2Br-
species which has higher reduction potential is reduced 2I- ——> I, + 2e
i.e., accepts the electrons and the species which has
Br, + 2- ——~> 2Br +I,
lower reduction potential is oxidised t.e., loses the
electrons. Otherwise, a redox reaction is not feasible. Alternatively, the problem can also be solved by
In other words, the species to release electrons must considering the half cell reactions.
have lower reduction potential as compared to the (z) I, + 2e7- ——> 2]- B°=+0.54V
species which is to accept electrons. (12) Br, + 2e°9 ———> 2Br-_ E° =+1.02 V
The electrochemical series (Table 1) gives the Since for the feasibility of the reaction, the e.m.f. should
decreasing order of electrode potentials (reduction) of be +ve, therefore to get + ve value for the overall cell
reaction, subtract the equation representing lower
different electrodes on moving down the table. This
value of E° from the equation representing the higher
means that the species to accept the electrons (getting
value of E°. Thus, subtract Eq. (7) from (iz).
reduced) must be higher in the electrochemical series
Br, + 2- ——> 2Br +I,
as compared to the other which is to lose electrons
(getting oxidised). E° = 1.02-0.54 = 0.48V

Let us illustrate this by predicting the feasibility lL} Example 40


of the reaction : What will be the spontaneous reaction between
Cu?* (aq) + 2Ag(s) ———> Cu(s) + 2Ag* (aq) the following half cell reactions?
From the electrochemical series (Table 1), E° (i) Cr** (aq) + 3e°-—> Cr(s) k° =-0.74V
value of Cu = +0.34 V and the E® value ofAg = + 0.80 V. (it) MnO, (s) + 4H* + 2e°-—> Mn** (aq) + 2,0 (D)
Since the reduction potential of Ag is more than that Eb° = 1.28 V
of Cu, this means that silver has greater tendency to
Calculate £5 ell
get reduced in comparison to copper. Thus, the reaction
Solution: Since the reduction potential of reaction
Ag* (aq) + e —— Ag(s) (ii) is more than that of reaction (i) ; reaction (i) will
occurs more readily than the reaction occur as reduction. Therefore, reaction (1) should be
written as oxidation. To obtain the net reaction, we
Cu?* (ag) + 2e7 ——> Cu(s)
multiply by appropriate coefficients so that electrons
Similarly, since reduction potential of copper is get cancelled.
less than that of Ag, this means that copper will be MnO, (s) + 4H* (ag) + 2e~ ———> Mn?* (aq) + 2H,O
oxidised in comparison to Ag. Thus, the reaction, ()| x 3
Cu(s) ——~> Cu?* (aq) + 2e- Cr(s) ———> Cr+ (ag) + 3e7] x 2
occurs more readily than 2Cr(s) + 3 MnO, (s) + 12H* (ag) ———> 2Cr** (aq)+3 Mn**
Ag (s) ——> Ag?* (aq) + e~ (aq) + 6H,O
go

Therefore, silver will be reduced and copper will Fell ~ es der reduced — Es pstance oxidised
be oxidised and the above reaction is not feasible. 1.28 — (-0.74) = 2.02 V
Rather the reverse reaction, UW Example 41
Cu(s) + 2Agt (ag) ———> Cu?+ (aq) + 2Ag(s) can The standard electrode potential corresponding
occur. to the reaction : Au** (ag) + 3e- ——> Au (s) is
1.42 V. Predict (i) if gold can be dissolved in 1M
REDOX REACTIONS
oat
HCl solution and (it) on passing hydrogen gas
through gold salt solution, metallic gold will be
precipitated or not.
=Dnakhile Wee
Solution: The standard reduction potential is greater
than zero, therefore, reduced form of gold is more stable Q 21.Calculate E° For the cell :
than hydrogen. In other words, gold cannot be oxidized by Al | Al§+(1M) Il Cu2+ (1 M)!ICu
hydrogen to give hydrogen. Hence gold does not dissolve in Given E° (Al*+ | Al) =—1.66 V and
1 M HCI solution. On the other hand, if hydrogen gas is
E°(Cu?+ | Cu) = 0.34 V
passed through the salt solution of the gold, it will reduce
Au** ion and metallic gold will be precipitated. @ 22. The standard electrode potentials of some electrodes
are :
LJ) Example 42 Electrode Zn2*|Zn Cd?+!Cd Ag*lAg Fe?*|Fe
Is it possible to store E° (V) —0.76 —0.40 O0.80V —-0.44V
(tL) Copper sulphate in a zinc vessel ? Which of the following cells are feasible and give their
(it) Copper sulphate in a silver vessel ? Er oat
(tit) Copper sulphate in a nickel vessel ? (i) Zn|Zn2* || Cd?2+1Cd = (ai) Fel Fe?* Il Zn?+ |Zn
(tv) Copper sulphate in a gold vessel ? (iii) Cdl Cd?* ll Agt|Ag = (iv) Fel Fe?* ll Ag+ |Ag
Solution: (2) From Table 18.1, it is clear that electrode Q 23. An iron wire is immersed in a solution containing
potential of copper (Cu 2+ | Cu = 0.34) is more than ZnSO, and NiSO,. When the concentration of each
zine (Zn2*+ | Zn = —0.76 V), therefore, it will be readily salt is 1 M, predict giving reasons which of the
reduced to copper and zinc will be oxidized. As a result, following reactions is likely to proceed ?
zine vessel will dissolve.
(1) Iron reduces Zn?* ions
(iz) It is possible to store copper sulphate in a silver
vessel because silver vessel will not dissolve (ii) Iron reduces Ni** ions. Given
(Agt| Ag = 0.80V). E? (Zn?* |Zn) = —0.76V, E° (Fe?+| Fe) = —0.44V,
(iit) Itis not possible to store copper sulphate in nickel and E° (Ni?* |Ni) = —0.25V
vessel because Ni will dissolve (Ni2*+ | Ni . Can a solution of 1 M CuSO, be stored in a vessel
=—0.13 V). made of nickel metal ? Given E®° (Ni** | Ni) = —0.25V
(tv) It is possible to store copper sulphate in gold and E° (Cu?+!Cu) = 0.34V
vessel (Au®*+ | Au = 1.61 V).
. Can chlorine gas be stored in a copper cylinder?
lL} Example 43 Given E° (Cu2*| Cu) = 0.34V and E° (CI-! Cl) = 1.36V.
Give two examples each of oxidants which can Q@ 26. Why does blue colour of copper sulphate get
oxidize discharged when zinc rod is dipped in it ?
(i) Cl-(aq) to Cl, (g) Given E° (Cu?+|Cu) = 0.34V and E®° (Zn?*1Zn)
(ii) Fe (s) to Fe** (aq)
= —0.76V.
(tit) I” (aq) to I, (aq)
Solution: @ 27. A copper wire is dipped in silver nitrate solution in
beaker A and a silver wire is dipped in a solution of
(i) Au, F, (zt) Ni, Sn
copper sulphate kept in beaker B. If the standard
(iii) Fe, Ag
electrode potential for
5. Predicting whether a metal can liberate
Cut* + 2e~ ———> Cu is +0.34 and for
hydrogen from the acid or not. The metals like zinc,
magnesium and nickel can liberate hydrogen from the Ag* + e~ ——> Ag is + 0.80 V.
acids like HCl, HaSO, etc., but metals like copper and Given E° (Ni?+| Ni) = —0.25V and E?® (Cu?+!|Cu)
silver cannot do so. In general, only those metals can = 0.384V
liberate hydrogen from the acids which have negative Predict in which beaker the ions present will get
values of reduction potentials t.e., —E° values. Obviously, reduced ?
hydrogen will have greater tendency to get reduced
(accept electrons) and the metal can lose electrons (get
oxidised) and hydrogen gas is liberated.
Answers to Practice Problems
M —-> M* (ag) +e © 21. 2.0V
(Monovalent) © 22. (i) Feasible, + 0.36 V_ (ii) not feasible, -0.32 V
H* (aq) + ee———> Hor ~ H, (iit) feasible, +1.20 V (iv) feasible, +1.24 V.
© 23. Iron reduces Ni** ions.
Metals like copper and silver have + E° values
© 24, No.
i.e., electron accepting tendencies. Their atoms are not
in a position to lose electrons to H* ions of the acid. © 25. No.
Therefore, hydrogen gas is not liberated. © 27. In beaker A, Ag* will be reduced.
MODERN'S abc + OF CHEMISTRY
-XI

Conceptual
Q.1. Arrange the following molecules in the decreasing order of oxidation state (+ve to
-ve) of nitrogen : NO,, NH, HN, NO,, N,H,.
Ans. Oxidation states of N are: .
1
NO, = +4, NH,=-3, HN,=->3
3 3
NO,- = +3, N,H, = ~-2
Decreasing order of oxidation state :
NO, > NO,- > HN, > N,H, > NH,
Q. 2. Can the reaction: |
Cr,0,7> + H,O —> 2CrO,” + 2H*
be regarded as a redox reaction ?
Ans. In this reaction,
Oxidation number of Cr in Cr,O,?" = +6
Oxidation number of Cr in CrO,- = +6
Since during this reaction, the oxidation number of Cr has neither increased nor decreased, therefore,
the above reaction cannot be regarded as a redox reaction.
. Calculate the oxidation number of Fe in
(i) Fe,O, (ti) Fe,[Fe(CN) gl,
Fe,0, 3x —8 = 0 or x = 8/3 (fractional)
Actually the composition of Fe,O, is Fe0.Fe,0,
The O.N. of Fe in FeO is +2 and in Fe,O,, it is +3.
On the average O.N. comes out to be 8/3.
Fe,|Fe(CN),|.. In this case the O.N. of Fe in the bracket is +2 and that of Fe outside the brackets is +3.
Fe 4 [Fe*7(CN ela
. Split the following redox reactions in the oxidation and reduction half reactions:
2K (s) + Cl, (g) —> 2KCI (s)
2Al (s) + 3Cu** (aq) —— > 2Al**(aq) + 3Cu (s)
Oxidation half reaction =: 2K (s) — 2K*(aq) + 2e7
Reduction half reaction: Cl, (g) + 2e7 —: 2Cl-
Oxidation half raction : 2Al (s) —> 2Al** (aq) + 6e™
Reduction half reaction: 3Cu2* + 6e- —_— 3Cu (s)
Q. 5. Nitric acid acts only as an oxidising agent while nitrous acid acts both as an oxidising as well as
reducing agent. Explain.
Ans. (i) HNO,. Oxidation number of N in HNO, is +5.
Maximum oxidation number of N is +5 because it has five electrons in the valence shell (2s?2p°).
Minimum oxidation number of N is —3 because it can accept 3 more electrons to get noble gas
configuration.
Since O.N. of N in HNO, is maximum, therefore, it can only decrease. Thus, HNO, can act as an
oxidising agent.
(wz) HNO,. Oxidation number of N = + 3
Maximum O.N. of N=45
Minimum O.N. of N =— 3
Therefore, the O.N. of N can increase by losing electrons or can decrease by accepting electrons. Thus,
HNO, can act both as an oxidising as well as a reducing agent.
Q. 6. Calculate the oxidation number of the underlined element in
(a) VO,* (b) UO,”* (c) Ba,XeO, (d) K,P,0, (e) KS
Ans. The O.N. of underlined atom is represented by x
(a) VO,* x + 2 (-2) = +1 or x=+5
(b) UO,** x +2 (-2) = +2 or x=+6

WWW.JEEBOOKS.IN
REDOX REACTIONS

(ec) Ba,XeO. +2(2)


+x +6 (2) =0 or x= +8
(d) K,P,0, 1x4+2x + 7(-2)=0 or x= +5
(e) K,5 Ix2+x=0 or x= -—2
Q. 7. Which of the following equations represent oxidation reduction reaction ? Identify each oxidising
agent and each reducing agent.
(a) KOH + H,0, — KHO, + H,O
(b) Cr,0,7 + 2 OH- —_—> 2CrO,> + H,O
(c) K+0O, — KO,
(d) Ca(HCOQ,), —— CaCO, + CO, + H,O
Ans.(7) a, 6b, and d are not redox reactions.
c is a redox reaction. The oxidising agent is O, and reducing agent is K.
Q. 8. Identify the oxidising and reducing agent in the following reaction :
Fe** + 2H* + NO,- —— Fe** + NO, + H,O
Ans. +2 +5 o+ | +4
Fe** + 2H*+NO,- ——> Fe** + NO, + H,O
Fe?* is oxidised to Fe** by NO,~ ion and therefore, NO,~ is an oxidising agent. NO, is reduced from N(+5)
to N(+4) in NO, by Fe** and therefore, Fe** is a reducing agent.
. Nitric acid is an oxidising agent and reacts with PbO but it does not react with PbO,. Explain
Why?
- PbO is a basic oxide and reacts with. HNO, and simple acid base reaction takes place.

2PbO + 4HNO, —— 2Pb(NO,), + 2H,O


But in PbO,, lead is in +4 oxidation state and cannot be further oxidised. Therefore, it does not react with
HNO..a
. Predict the maximum and minimum oxidation states for (1) Cl (11) Ti
. Maximum oxidation state for Cl = +7
Minimum oxidation state for Cl =—1
Maximum oxidation state for Ti = +4
Minimum oxidation state for Ti = 0
For metals minimum O.N. is zero but for non—metals, it is group number minus 8.
. The electrode reduction potentials of four metallic elements A, B, C and D are respectively +0.79,
- 0.74, 1.08 and - 0.31 V. Arrange these in order of decreasing electropositive character.
. Higher the electrode reduction potential, lower is its tendency to lose electrons and therefore, lower is
the electropositive character of the metal. So, the metals can be arranged as: B, D, A, C
. An iron rod is immersed in a solution containing NiSO, and ZnSO,. When the concentration
of each salt is 1M, predict giving reasons which of the following reactions is likely to
proceed ?
(i) Iron reduces Zn** ions (ii) Iron reduces Ni** ions
Given : E’z,.2+)7n) = - 0.76 V, E’qe2ipe) = — 0.44 V and F*qyj2+\ni) = - 0.25 V
Ans. (1) The reduction potential of iron is more than that of zinc. Therefore, iron will be reduced. In other
words, Zn?* will not be reduced by iron.
(ii) The reduction potential of Ni2* is more than that of iron. Therefore, Ni** will be reduced by iron.
Q. 13. Which of the following reactions are not feasible ?
(a) Zn(s)+ 2Ag*(aq) — Zn**(aq) + 2Ag(s)
(b) I,(s) + 2Br-(aq) ———) 2I-(aq) + Br,
(c) 2Fe**(aq) + 2I-(ag) —— I,(aq) + 2Fe**(aq)
(dq) 2Ag + 2H*(ag) — H, + 2Ag*(aq)
Ans. (a) and (c) are feasible.
Q. 14. At what concentration of Cu** (aq) will its electrode potential become equal to its standard
electrode potential ?
Ans. At 1 M concentration.
a 8/44 MODERN'S abc + OF CHEMISTRY-XI

Q.15. How does Cu,0 act as both oxidant and reductant ? Explain with proper reactions showing the
change of oxidation number in each case.
Ans. In Cu,O, copper is in +1 (Cu*) oxidation state and it undergoes disproportionation to form Cu and Cu2+
as:
2Cut(ag) —> Cu(s) + Cu?*(aq)
Thus, Cut or Cu,O acts both as an oxidant as well as a reductant.
For example,
(1) When heated in air, Cu,O is oxidised to CuO

+1 qd +2
Cu,0 + 2 V2 — +2Cu0

Thus, Cu,O acts as a reductant and reduces O, to O--


(it) When heated with Cu,5, it oxidises S2- to SO, and hence Cu,O acts as an oxidant.
+1 +1 0
2Cu,0 + Cu,s —> 6Cu+ S50,
Q. 16. Asolution of silver nitrate was stirrred with an iron rod. Will it cause any change in the concentration
of silver and nitrate ions ?
Ans. The E® of Fe2* | Fe (— 0.44 V) is lower than that of Ag+ | Ag (+0.80 V), therefore, Ag+ gets reduced and Fe
gets oxidised. As a result, the concentration of Ag*t ions decreases and that of NO,” ions remains unchanged.
2 Ag* (aq) + 2 NO, (aq) + Fe(s) —> 2 Ag(s) + Fe** (aq) + 2 NO, (aq)
Q. 17. What is the oxidation number of N in HNO,?
Ans. By conventional method, O.N. ofN is: +1 +x + 4(— 2) =0orx = +7.
However, N cannot have O.N. more than +5, Therefore, O.N. of N in HNO, (peroxynitric acid) must be
calculated from its structure. The structure of HNO, is H—O—O—N—— O. It contains one peroxide

linkage and therefore, each of the two oxygen atoms of peroxide bonds has an O.N. of —1.
Thus, +1—1-—1+x-—2-2=0
org= #5.
Q. 18. Calculate the oxidation number of nickel in Ni(COQ),, iron in Fe(CO), and carbon in CH,O.
Ans. Ni(CQ), :x + 4(0) =0 ork =%
Fe(CQ), : x + 5(0) = 0 pr Tab
CH,O :x+2(41l-—2=8 orx=0-
Q. 19. At what concentration of Zn** (aq) will its electrode potential become equal to its standard electrode
potential ?
Ans. At 1 M concentration.
Q. 20. The E® (Al**| Al) and E® (Cu**!|Cu) are-1.66 V and + 0.34 V respectively. Is the cell :
Cul Cu”* (1M) II Al®** (1M) 1Al feasible or not feasible?
Ans. Not feasible.

Oxidation is a process which involves loss of electrons or increase in oxidation number.


Reduction is a process which involves gain of electrons or decrease in oxidation number.
Oxidation number is the charge which an atom appears to have when all other atoms are removed from it as ions.
Oxidising agent is a substance which accepts one or more electrons or its oxidation number decreases.
6
8
@#
xe Reducing agent is a substance which loses one or more electrons or its oxidation number increases.
REDOX REACTIONS
feus]
QUICK CHAPTER ROUND UP
UP

Oxidation & Reduction

Electronic concept In terms of oxidation number


Oxidation : a process in which one or more electrons Oxidation: a process which involves increase in oxidation
are lost. number.
Reduction : a process in which one or more electrons Reduction: a process which involves decrease in oxidation
are gained. number.
Oxidising agent or oxidant : a substance which can Oxidising agent: a substance whose oxidation number
accept one or more electrons. decreases,
Reducing agent or reductant : a substance which can Reducing agent: a substance whose oxidation number
give one or more electrons. increases.

Oxidation and reduction always occur side byside.

Oxidation number ® Combination reaction


for elementary state, O.N is zero. 2Mg + O, —> 2MgO
e.g., He(O.N. = 0), H, (O.N.=6}, $,(0.N. =0) ® Decomposition reaction
oxidation number of hydrogen is + 1 except in hydrides 2KC1O, ——> 2KCl + 30,
NaH, LiH, CaH, (—1).
® Displacement reaction
Oxidation number of oxygen is always —2 except
Ccuso, + Zn-—>+Cu + Zn050,
in peroxides, H,O,, Na,OQ, (—1)
in superoxides KO, (— 1/2) sn + 2HCl—~> Sncl, + Hy
in OF, (+2) Br, + 2I- —426r- + 1,
OOP taal

® Disproportionation reactions
The reactions in which the oxidation number of an element both increases
and decreases.
eg. 2H,O, >> 2H,0 + O,
In this case the oxidation number of O decreases from —1 to —2 (in
HO) and increases from —1 to 0 [in O.,).
» Cie /2OH- —> OCIT + Cl” + H,O
©.N. of Cl increases from 0 to +1 (in OCI”) and decreases from 0 to
— jin Cl~),
» 3CIO™ —> CIO,” + 2Cl™ is also a disproportionation reaction
because O.N. of Cl increases from +1 (in ClO™) to +5 (in ClO,~)
and decreases from +1 to —1 (in Cl”).

Oxidation and reduction reactions can be blalanced by oxidation number method and half reaction method,

WWW.JEEBOOKS.IN
MODERN'S abc + OF CHEMISTRY
-XI

TW Solved
Textbook Exercises i
Assign oxidation number to the underlined But oxidation number cannot be fractional. In fact
elements in each of the following species: KI, exists as : K* [J-l «— I} in which a coordinate
(a) NaH,PO, (b) NaHSO, bond is formed between I, molecule and I. In this
the oxidation number of two iodine atoms forming
(c) H,P,0, (d) K,MnO, I, molecule is zero and that of iodine forming the
(e) CaQ, (f) NaBH,
coordinate bond is —l. Thus, the actual oxidation
(g) H,8,0, (h) KAI(SO,),.12H,O number of atoms are :
Ans. Let the oxidation number of underlined atom be x. + 0 -1
+1 +1 x -2 Kf, I
(4) NaH, PO, (6) H,S,0,. The oxidation number may be calculated
1(4+1) + 2(41) +x%44(23=0 wx = +5
as :
Oxidation number of P in NaH,PO, is + 5 +1 x =-2
. +1 41 x -2
Hy Sz O¢
(6) NaH SO,
1(+1) + 1(41) +x + 4 (2) =0 2(+1)+4x +6(-2)=0 neat
x—-6=0 a
Since oxidation number cannot be fractional,
+1 x -2
the actual oxidation number of different S atoms
(c) H, P, O,
are:
4(+1) + 2(x) + 7 (2) =0
2x —10 =0 OQ O
+5 0 0 +5
d) K, Mn 0, H—O—S—s—S— S—O—H
2(+1) +x + 4-2) = 0
x—-6=0
(c) Fe,O,. The oxidation number may be calculated
+2 as :
(e) Ca O, x <2
+2 +2 (x) =0 . x==<—] Fe, O,
+1 -1
() NaBH, ax +4 (-2)
=0 or .. x=
00
69]
1(+1) + x + 4-1) = 0 In fact, Fe,O, exists as a mixture of FeO and
x-d=0 Fe,O, as :
+1 x -2 42-2 +3 -3
(g) H, S, O, FeO. Fey Og
2(+1) + 2 (x) + 7(-2) =0 Fe has oxidation number of +2 and +3.
2x —12 =0 x=+6 (d) CH,CH,OH — By conventional method, the
. +1 +3 * =o +1 —2
oxidation number may be calculated as :
(A) K Al (S O,4)9.12H, O x +1 «x +1 -2 +1
CH, C H, O H
1(+1) + 1 (43) + 2x + 8 (2) +12 (2x 1-2) =0
x +3 (41) + x + 2(41) + 1(-2) + (+1) =0
2x —12 =0 x=+6
2x +4=0 . x=-2
What are the oxidation number of the underlined
In fact, ethanol may be written as :
elements in each of the following and how do you
rationalise your results ?
(a) KI, (b) H,S,0, ale beOH
(c) Fe,0, (d) CH,CH,OH
(e) CH,COOH hou
Ans. (a) In KI, since the oxidation number of K is +1, C,, is attached to three H atoms (less electronegative
therefore, the average oxidation number of iodine than carbon) and one CH,OH group (more
electronegative than carbon). Therefore,
2
is -—. O.N. of C, : 3 (+1) + x +1(-1) = 0 or x = -2
C, is attached to one OH (O.N. = —1) and one
+1 x 1423 0 ; ‘1
E1, + 1+ 3x)= = — CH,(O.N. = +1) and two H-atoms
O.N. of C7,=+14+2(41)+x241-D=0 .x=-2
REDOX REACTIONS

(e) CH,COOH. By conventional method Fluorine reacts with ice and results in the
x +l x -2-2+1 change:
CH, COOH H,O(s) + F,(¢) —> HF(g) + HOF(g)
x +3 (+1) + x« + (-2) + (-2) + 1/41) =0 Justify that this reaction is a redox reaction.
2x +0=0 , x=0 +1 -2 () +1 -l1 +1 -2 +1
Ans. H, O(s)+F,(g) ——> H F+ HOF
But according to structure,
H O In this case, F, is reduced to HF and oxidised to
HOF. Therefore, it is redox reaction. HOF is unstable
Hee a OH and decomposes to form O, and HF.
+1-2 +1 +1 -l 0
C, is attached to three H atoms (less electronegative
2HO F—> 2H F +0
than C) and one-—COOH group (more electronegative In this reaction, F of HOF is reduced while O of HOF
than C),. is oxidised. Therefore, it is a redox reaction but not
a disproportionation reaction. If oxygen of HOF
O.N. of C, = 3 (41) +x + 10-1) =90 an Xx = —3
disproportionates, then oxygen must show three
C, is attached to one oxygen atom by a double bond,
oxidation states. If we assume that oxidation state
one OH-group (O.N. =—1) and one CH, (O.N. = +1).
of O is zero in HOF, then its oxidation state decreases
O.N. of C, = + 14x + 1-2) +1(C1) =0 to —2. HOF is reduced to H,O and increases to +2
x-2=Qorx = +2 if if HOF gets oxidised to OF,.
Q.3. Justify that the following reactions are redox Therefore, the possible reaction is
reactions: +1 0, -1 +1 -2 +2 -1
(a) CuO(s) + H,(g) —> Cu(s) + H,O(g) 2HO F——>H, O + OF,
(6) Fe,O,(s) + 3CO(g) —> 2Fe(s) + 3CO,(g) Calculate the oxidation number of sulphur,
(c) 4BCL,(g) + 3LiAH, (s) —> 2B,H,(g) + chromium and nitrogen in H,SO,, Cr,0?7- and
SLICI(s) + 3AICI,(s) NO,. Suggest structure of these compounds.
(d) 2K(s) + F,(g) —> 2K*F(s) Count for the fallacy.
Ans, (i) H,SO;. By conventional method
(e) 4NH,(g) + 50,(¢) —> 4NO(g) + 6H,O(g)
4 2 0
+1 x -2
0 +1 -2
Ans. (2) Cy O(s)+ Ho( g)——>Cu(s)+ Hy, O(g) H, 5 O;
2(+1) +x%+5 (-2) =0 or x=+8
In this case, O.N. of Cu decreases from +2 (in
CuO) to 0 (in Cu) and that of H increase from This is not possible because S cannot have O.N.
0 (in H,) to +1 Gn H,O). Therefore, CuO is more than 6. The fallacy is overcomed if we
reduced to Cu while H, is oxidised to H,O. calculate its O.N. from its structure as :
Thus, this is a redox reaction. O
|
+3 -2 +2 —-2
(6) Fe, O, +3CO ——>2Fe(s) + 3C0,
0 +4 -2
H—O—S—O—O—H
O.N. of Fe decreases from +3 (in Fe,O,) to 0 (in
Fe) and that of C increases from +2 (in CO) to 2x(+1)
+x + 2-1) + 3x(BZ=O
+4 (in CO,). Therefore, Fe,O, is reduced while (for HH) (forS) (forO—O) — (for other
CO is oxidised. Thus, this is a redox reaction. O atom)
+3 -1 +1 +3 -l —d +1 #1 -l orx =+6
(c) 4B Cl,(s) + 3Li AlH,(s) — 2B, He(g)+ 3LiCl(s) (ii) Cr,O2-
+3 -l 2x + T(—-2) = —2
+ 3AI1Cl,(s)
2 = 10 x=+6
O.N. of B. decreases from +3 (in BCl,) to —3 in This is correct and there is no fallacy.
B,H, and that of H increases from —1 (in LiAlH,)
(zit) NO. By conventional method,
to + 1 (in B,H,). Therefore, BCI, is reduced and
x + 3(-2) =—-1 or x =4+5
LiAIH, is oxidised. Hence, it is a redox reaction.
From the structure,
0 0 +1 —1
(d 2K(s) + F(g) —>2K*+F (s) O
— nm
rn

O.N. of K increases from 0 (in K) to +1 (in K*) and sg)


that of F decreases from 0 in (F,) to — 1 (in F*).
Hence, K has been oxidised while F, has been x+1(—1)4+ 1(-2)+1(-2)=0 or x=+5
reduced. Therefore, it is a redox reaction. (for O-) (for =O) (for 30)
el 0 +2 -2 a -2 Hence, there is no fallacy about O.N. of N in
(e) 4NH,(g) +50,(g¢) ——> 4N O(g)+6H, O(g) NO,” as + 5 whether we calculate by
In this case O.N. of N increases from— 3 (in NH,) conventional method or by chemical bonding.
to +2 (in NO) and that of O decreases from 0 Q.6. Write formulas for the following compounds:
(in O,) to — 2 (in H,O). Therefore, NH, has been (a) Mercury (ID) chloride
oxidised while O, has been reduced. Hence, it (6b) Nickel (II) sulphate
is a redox reaction. (ec) Tin (IV) oxide
MODERN'S abc + OF CHEMISTRY
-XI

(d) Thallium (I) sulphate Q. 11. Whenever a reaction between an oxidising agent
(e) Iron (IIT) sulphate and areducing agent is carried out, a compound
(f) Chromium (IIT) oxide of lower oxidation state is formed if the reducing
Ans. (a) HgCl, (6) NiSO, (c) SnO, agent is in excess and a compound of higher
oxidation state is formed if the oxidising agent
(d) TI,SO, = (e) Fe,(SO,), — (f) Cr,O4
is in excess. Justify this statement giving three
Q. 7. Suggest a list of the substances where carbon illustrations.
can exhibit oxidation states from - 4 to + 4 and
Ans. (i) C is a reducing agent while O, is an oxidising
nitrogen from — 3 to + 5.
agent. If excess of C is burnt in a limited supply of
Refer Solved Example 8. (Page 11). Q,, CO is formed in which the oxidation state of C
While sulphur dioxide and hydrogen peroxide is +2, However, if O, is excess, the initially formed
can act as oxidising as well as reducing agents CO gets oxidised to CO in which the oxidation state
in their reactions, ozone and nitric acid act only of C is +4.
as oxidants. Why ? +2
Refer HOTS Questions, Q.1 (Page 58). 2C(s), + O.(g)——2C0
(excess
Consider the reactions :
+4
(a) 6CO,(g) + 6H,O(/) —> C,H, ,O,(aq) + 60,(g) C(s) + Oo(g) CO,
(b) O,(g) + H,O,() —> H,O(D) + 20,(¢) (excess)

Why it is more appropriate to write these (1) Sodium is a reducing agent while O, is an
reactions as : oxidising agent. When excess of Na is used, sodium
(a) 6CO, + 12H,O(1) —> oxide is formed in which the oxidation state of O is
—2. If however, excess of O, is used, Na,O, is formed
C,H,,0,(aq) + 6H,O(W) + 60,(¢)
in which the oxidation state of O is —1, which is
(6) O,(g) + H,O,(@) —> H,O() + O,(g) + O,(g) higher than —2.
Also suggest a technique to investigate the
path of the above (a) and (6) redox reactions. -2
4Na(s) + O,(g) —— 2 Na,O(s)
Ans. It is believed that the photosynthesis reaction occurs (excess)
in two steps. In the first step, H,O decomposes to
-1
give H, and O, in the presence of chlorophyll and 2Na(s) + Oo(g) ——Na,0,(s)
the H, produced reduces CO, to C,H,,O, in the (excess)
second step. During the second step, some H,O (tit) P, is a reducing agent while Cl, is an oxidising
molecules are also produced and therefore, the agent. When excess of P, is used, PCl, is formed in
reaction occurs as : which the oxidation state of P is +3. However, if
(Z) 12H,O (1) —> 12H,(¢) + 60, (g) excess of Cl, is used, then initially formed PCl,
(uw) 6CO,(g) + 12H,(g) —> C,H,.0, (s) + 6H,0M reacts further to form PCl,, in which the oxidation
state of P is +5.
(iit) 6CO,(g) + 12H, 0) —>
+3
C,H,,0,(s) + 6H,0W/) + 60,(¢) P,(s) + 6Cl,——>4PCl,
Therefore, itis more appropriate to write the reaction (excess)
for photosynthesis as (ili) because it means that 12 +5
molecules of H,O are used per molecule of P,(s) + 10Cl, ——>4PCl,
(excess)
carbohydrate and 6H,O molecules are produced per
molecule of carbohydrate during the process. Q. 12. How do you account for the following obser-
vations ?
(ii) O, is written two times in the product which
(a) Though alkaline potassium permanganate
suggests that O, is being obtained from the two
and acidic potassium permanganate both are
reactants as:
used as oxidants, yet in the manufacture of
On(g) =>), O,(g) + Of)
benzoic acid from toluene we use alcoholic
H,0O, (1) + O (g¢) ——> H,0O (J) + O,{g) potassium permanganate as an oxidant.
O.(g) + H,O, (2) —-> H,O) + O,(g) + O,(g) Why ? Write a balanced redox equation for the
The path of the reaction can be studied by using reaction.
H,O’8 in reaction (a) or by using H,O,'* or O,'° in (b) When concentrated sulphuric acid is added to
reaction (6). an inorganic mixture containing chloride, we get
colourless pungent smelling gas HCl, but if the
Q. 10. The compound AgF, is unstable compound.
mixture contains bromide then we get red vapour
However, if formed, the compound acts as a very
of bromine. Why ?
strong oxidising agent. Why ?
Ans. When conc. H,SO, is added to an inorganic mixture
Ans. In AgF,, oxidation state of Ag is +2 which is very
containing chloride, a pungent smelling gas HCl is
unstable. Therefore, it readily accepts an electron to
produced because a stronger acid displaces a weaker
form a more stable oxidation state of +1 as
Ag** + en » Agt
acid from its salt.
2NaCl + 2H,SO, —> 2NaHSO, + 2HCl
Therefore, AgF, if formed will act as a strong
2 HCl+H,SO, —-< Cl, + 50, + 2H,O
oxidising agent.
REDOX REACTIONS

Since HCl is a weak reducing agent, it cannot reduce Q. 14. Consider the reactions :
H,SO, to SO, and therefore, HCl is not oxidised to 2S,0,7(aqg) + L,(s) —> S,0,7(aq) + 2I-(aq)
chlorine. S,0,* (aq) + 2Br,(J) + 5H,O(1) —> 2SO,* (aq)
However, if the mixture contains bromide ion, the + 4Br-(aqg) + 10H*(aq)
initially produced HBr is a stronger reducing agent. Why does the same reductant, thiosulphate
Therefore, it reduces H,SO, to SO, and itself gets react differently with iodine and bromine ?
oxidised to produce red vapour of Bry. The O.N. of S in Se0,7 is +2, in S,O,?" it is +2.5
2 NaBr + 2H,SO, —> 2NaHSO, + 2HBr (average) andin SO rl itis +6. Since Br, is a stronger
2HBr + H,SO, ——> Br,+ SO, + 2H,O oxidising agent than I,, it oxidises S of S,0O,* to
Q. 13. Identify the substance oxidised, reduced, oxidising SO,* (with higher O.N. of +6). However, I, being a
agent and reducing agent for each of the following weaker oxidising agent oxidises S of S,0,?> into a
reactions: lower O.S., of +2.5 in $,0,?-. Therefore, S,O? reacts
(a) 2AgBr(s) + C,H,O,(aq) —> 2Ag(s) + differently with Br, and I,.
2HBr(aq) + C,H,O,(aq) Q. 15. Justify giving reactions that among halogens,
(b) HCHO(/) + 2[Ag(NH,),|*(aq) + 30H (aq) — fluorine is the best oxidant and among hydrohalic
2Ag(s) + HCOO-(aq) + 4NH,(aq) + 2H,O(W) compounds, hydroiodic acid is the best reductant.
(c) HCHO() + 2Cu?*(aq) + 50H-(aqg) —> The oxidising power of halogens decreases in the
Cu,O(s) + HCOO-(aq) + 3H,O(l) order : F, > Cl, > Br, > I,. Therefore, F’, can oxidise
(d) N,H,() + 2H,0O,(2)) —3N,(g) + 4H,O() Cl-, Br- and I to respective Cl,, Brg, I,.
(e) Pb(s) + PbO,(s) + 2H,SO,(aq) —> F(g) + 2Cl(aq) ——> 2F (aq) + CL/{g)
2PbSO,(s) + 2H,O(D) F,(g) + 2Br(aq) ——>_ _2F(aq) + Br,(/)
Ans. Reduction F.(g) + 2b(aq) ——>_—s- 2CI-(agq) + I,{s)
Cl, can oxidise Br to Br, and I to I,
+1 0 Cl,(g) + 2Br(aqg) ——> 2Cl(aq) + Br,(0)
2AgBris) + C,H,O,(ag) —-> 2Agis) + 2HBrlag) + C,H,O,(aq) 2ClL,(g) + 2I-(aqg) ——> __2CI(aq) + I,(s)
However, Cl, cannot oxidise F~ to F,.
Oxidation Br, can oxidise only I> (ag) to L, and not F- or Cl-
AgBr : reduced, C.gH,O, (aq) : oxidised to F, or Cl,.
AgBr : Oxidising agent, C,H,O, (aq) : reducing agent Br,(/) + 2I-(aqg) —— 2Br‘(agq) + L(s)
Reduction Thus, F, is the best oxidant.
Among hydrohalic acids, the reducing power
decreases as : HI > HBr > HCl > HF. For example,
(b) HCHO() + 2[Ag(NH,).I*(aq) + 30H- —> 2Agis) HI and HBr reduce H,SO, to SO, while HCl and HF
+ HCOO- (ag) + 4NH,(aq) + 2H,OW
donot.
Oxidation 2HBr + H,SO, —— Br, + SO, + 2H,O
[Ag(NH,),I* (ag) : reduced, HCHO(/) : oxidised 2HI + H,SO, —— I[,+50, + 2H,O
[Ag(NH,),]* (aq) : oxidising agent, HCHO(/): reducing Similarly, ” reduces Cu** to Cut but Br- does not.
agent 2Cu* (ag) + 4° (ag) ——> CuI, (s) + L,(aq)
Reduction Further among HCl and HF, HCl is a stronger
reducing agent than HF because HCl can reduce
(c) HCHO() + 2Cu**(aq) + 50H-(ag) = Cu,O(s) + HCOO~(aq) MnO, to Mn?*.
+ 3H,0 MnO,(s) + 4HCl (aq)> MnCl,(aq) + Cl,(g¢) + 2H,O
But HF does not reduce MnO,. Therefore, among
Oxidation
hydrohalic acid HI is the strongest reducing agent.
Cu** (ag) : reduced, HCHO(I) : oxidised
Cu** (aq) : oxidising agent, HCHO(/) : reducing agent Q. 16. Why does the following reaction occur ?
Reduction XeO,* (aq) + 2F- (aq) + 6H*(ag) —> XeO,(g) +
F,(g) + 3H,O(D)
(d) N,H,() + 2H,0,(1) —> N,(g) + 4H,0(@) What conclusion about the compound Na,XeO,
(of which XeO,* is a part) can be drawn from
the reaction?
Oxidation
H,O, : reduced, N.H,(/) : oxidised +8 A . “1 ; +e
XeO, (aq)+2F (aq) +6H" (ag) ———>
H,0, : oxidising agent, NH, (/) : reducing agent
Reduction +6 0
Xe O,(s) + Fj(g)+3H,O(/)
0 +4 +2 O.N. of Xe decreases from +8 (in XeO,*) to +6 (in
(e) Pb(s) + PbO,(s) + 2H,SO,(aqg) —> 2PbSO,(s) + 2H,O(2) XeO,) and that of F increases from —1 (in F~) to 0
(in F,). Therefore, XeO,* is reduced while F- is
Oxidation oxidised.
| The reaction occurs because Na,XeO, 6 (or
PbO, : reduced Phis) : oxidised XeO,*) is a stronger oxidising agent than F,.
PbO, (s) : oxidising agent, Pb(s) : reducing agent
MODERN'S abc + OF CHEMISTRY
-XI

Q. 17. Consider the reactions : SO,(g) + 2H,O() ——> HSO; (aq) + 3H* + 2e-
(a) H,PO,(aq) + 4AgNO,(aq) + H,O() —> To balance reduction half reaction.
H,PO,(aq) + 4Ag(s) + 4HNO,(aq) MnO, (ag) ——> Mn** (aq)
(6) H,PO,(aq) + 2CuSO,(aq) + H,O(W) —> (1) Balance all atoms other than H and O (already
H,PO,(aq) + 2Cu(s) + H,SO,(aq) done).
(c) C,H,CHO(/) + 2[Ag(NH,),l(aq) + 30H (aq) (iz) Manganese changes its oxidation number from
—> C,H,COO (aq) + 2Ag(s) + 7 to + 2 and there is difference of 5 electrons. To
+ 4NH,(aq) + 2H,O(/) balance the electrons,
(d) C,H.CHO() + 2Cu**(aq) + 50H-(ag) —> MnO; (aq) + 5e- ——> Mn?* (aq)
No change observed. (iii) Since the reaction takes place in acidic medium
What inference do you draw about the add 8H* on the left to equate the net charges on both
behaviour of Agt and Cu”* from these ? sides and 4 H,O to the right to balance oxygen atoms.
Ans. Reaction (a) and (6) infer that H,PO, MnO; + 8 H* + 5e ——> Mn”
(hypophosphorus acid) is a reducing agent and MnO; + 8H* + 5e ——> Mn** + 4H,O
reduces both AgNO, and CuSO, to Ag and Cu Adding two half reactions by equating the number
respectively, This also means that both AgNO, and of electrons.
CuSO, act as oxidising agent and oxidise H,PO, to MnO, (aq) + 8H* + 5e° ——> Mn?*(aq) + 4H,O(/)| x 2
H,PO, (phosphoric acid). SO,(g) + 2H,O ——> HSOj (aq) + 3H*(aq) + 2e-] x 5
Reaction (c) indicate that [Ag(NH,),|* oxidises
C,H:CHO (benzaldehyde) to C,H,COO™ (benzoate 2MnO; (aq) + 5SO,(g) + 2H,O(/) + Haq) —> 2Mn?*(aq)
ion) but reaction (d) indicates that Cu?* ions cannot + 5 HSO,- (aq)
oxidise CgH.CHO to C,H-COO-. Thus, we can infer
ea +2 +3 -2
from the above reactions that Ag* ion is a stronger (c) H, O2(aq)+Fe** (ag) _Fe** (aq) + H, O()
oxidising agent than Cu?* ion.
in acidic medium.
Q. 18. Balance the following redox reactions by ion
In this reaction, the oxidation number of Fe increases
- electron method :
from + 2 (in Fe*t) to +3 (in Fe**) and therefore, Fe?*
(a) MnO,(aq) + I-(aq) —> MnO,(s) + I,(s)
has been oxidised. The oxidation number of oxygen
(in basic medium)
has decreased from —1 (in H,O,) to — 2 in H,O and
(6) MnO, (aq) + SO,(g¢) —
therefore, H,O, has been reduced.
Mn?*(aq) + HSO, (aq)
Oxidation half reaction : Fe**(ag) ———> Fe**(aq)
(in acidic solution)
Reduction half reaction : H,O,(aq) ———> H,O(aq)
(c) H,O,(aq) + Fe**(aq) —> Fe**(aq) + H,O(l)
These can be balanced by following the steps of
(in acidic solution)
balancing as :
(d) Cr,0,> + SO,(g) — Cr**(aq) + SO ¢ (aq)
Oxidation half reaction :
(in acidic solution)
Fe?+(ag) ———> Fe**(aq)
Ans. (a) Refer Solved Example 30. (Page 27).
Fe2+(aq) ——> Fe**(aq) + e7 AL)
+7 +4 +2 +6 Reduction half reaction
(b) MnO, (aq) + SO2(g)—— Mn**(aq)+ HSO4 (aq)
H,0,(aq) ——> H,O(/)
In this reaction, manganese undergoes reduction
H,O,(aq) + e —— H,0O()
because oxidation number of Mn decreases from
H,O,(aq) + 2H* (aq) + 2e° ——> 2H,O(/) (UL)
+7(in MnO7z) to +2 (in Mn**), SO, gets oxidised
because oxidation number of sulphur increases from Multiply eg (2) by 2 and add it to eq. (iz) to balance
+ 4 (in SO,) to + 6.in (HSO,>). electrons.
Oxidation half reaction : SO,(¢) ——-~ HSO,- (aq) H,O,(aq) + 2Fe*+ (aq) + 2H*t ——-> 2Fe** (aq) + 2H,O(W)
Reduction half reaction : MnO (ag) ——> Mn**(aq) _. +6 od +o +6
Balancing each half reaction separately as : (d)Cr,
O;* (aq) + SOo(g— Cr*(aq)+ $O,* (aq)
50,(¢) ——> _HSO, (aq) (in acidic medium)
(t) Balance all atoms other than H and O (already Oxidation half reaction : SO,(¢) ——> SO? (aq)
done). Reduction half reaction : Cr (aq) ———> Cr**(aq)
(iz) Sulphur changes its oxidation number from + 4 Balancing the oxidation and reduction half reactions
to + 6 and there is a difference of 2 electrons. To
separately following the steps of balancing.
balance the electrons,
Oxidation half reaction :
SO,(¢) ——> HS0O,° (aq) + 2e7
SO,(g) ——> SO,* (aq)
(ii) Since the reaction takes place in acidic medium
SO,(¢) ——> SO, (aq) + 2e
add 3H* on the right hand side and two H,O
molecules on the left hand side. SO,(g) + 2H,O() ——> SO, + 2e° + 4H*(aq) __...{i)
REDOX REACTIONS

Reduction half reaction : Reduction half reaction


Cr,0,7> (aq) ——> Cr**(aq) : 3
Cr,0,7> ——> 2Cr** (aq) P,——» PH, (g)
Cr,0,7> (aq) + 6e- ——> 2Cr** (aq) P,(s) ——> 4PH.,(g)
Cr,0,7" (aq) + 6e~ + 14 H*(aqg) ——> 2Cr**(aq) P,(s) + 12e- ——-> 4PH.,(g)
Cr,0,7 (aq) + 6e~ + 14H*(aqg) ——> 2Cr**(aq) P,(s) + 12e- ——> 4PH,(g) + 12 OH" (aq)
+ 7H,O...(7) P,(s) + 12H,O (/) + 12e- ———> 4PH,(g) + 12 OH" (aq)
Multiply eq. (z) by 3 and add it to eq (iz) to balance (EL)
electrons : To balance electrons, multiply eq. (4) by 3 and add
Cr,0,*" (aq) + 3S0,(g) + 2H*(aqg) ——> 2Cr** (aq) to eq. (it).
- 380, (aq) + H,O 4P,(s) + 12O0H™ (ag) + 12H,OV) ——> 4PH.,(g)
Q. 19. Balance the following equations in basic medium + 12 H,PO,- (aq)
by ion-electron method and oxidation number or P,(s) + 30H” (aq) + 3H,O() ——> PH,(g¢)
methods and identify the oxidising agent and the + 3H,PO, (aq)
reducing agent.
Oxidation
(a) P,(s) + OH (aq) —> PH, (g) + H,PO, (aq)
(6) N,H,(D) + ClO,-(aqg) —> NO(g) + Cig) —2 +5 +2 —1
(b) N,H,(D) + .C1lOs- (ag) ——-> NO(g) + Cl (gz)
(c) C1,O,(g) + H,O,(aq) —> ClO,(aq) + O,(g)
+ Ht
Reduction
Reduction

. Oo a —3+1 +1 In this reaction, NH, gets oxidised and acts as a


Ans. (a) P4(s) + OH (ag) —— PH,(g) + HyPOZ reducing agent and ClO,” gets reduced and acts as
an oxidising agent.
Oxidation
Oxidation number method
P, gets oxidised to H,PO, and reduced to PH, and
therefore, P, acts as reducing agent as well as QO.N. increases by 4 per N atom
oxidising agent. | or 4x 2 per NoH, molecule
Oxidation number method -2
NjH,@%) +
+5
ClO, (ag)——-> NO)
-2
+
-1
Cl (ag)
O.N. decreases by 3 per atom
O.N. decreases by 6 per Cl atom
0 —3+1 +1
P,(s) + OH (ag)—> PH,(g) + HyPO, (aq) To balance the increase or decrease in oxidation
O.N. increases by 1 per atom number, multiply N,H, by 3 and ClO; by 4 and add
or 1x4per P, molecule 3N,H,(2) + 4C10,~ (ag) ——> NO(g) + Cl (aq)
Balance the increase or decrease in oxidation number Balance N and Cl atoms on R.H.S.
by multiplying H,PO; by 3 and PH, by 1. 3N,H,() + 4ClO0,~ (ag) ———> 6NO(g) + 4Cl(aq)
P,(s) + OH’ (aq) ——> PH,{g) + 3H,PO, (aq) To balance O atoms add 6H,O molecules on R.H.S.
To balance O atoms, multiply OH” by 6, 3N,H,(2) + 4Cl0,~ (aq) ——> 6NO(g)+ 4CI (aq) + 6H,O
P,(s) + 60H (ag) ———> PH.(g) + 3H,PO,- (aq) Since H atoms are balanced automatically, the above
To balance H atoms, add 3 H,O to L.H.S. and 30H equation represents balanced chemical equation.
on R.H.S, we have Ion electron method
P,(s) + 6OH” (aq) + 83H,07) —— PH,{(g) + Oxidation half reaction: N,H,(/) ——~+ NO()
3H,PO,- (aq) + 30H-(aq) Reduction half reaction : ClO,~ (aq) —— > CI(aq)
or P,(s) + 80H (aq) + 3H,O(/) ——~ PH.(g) + These can be balanced separately as :
3H,PO, (aq) Oxidation half reaction:
Ion electron method N,H, (2) ——> NOj)
The equation can be split up as N,H,() ——> 2NO@)
Oxidation half reaction : P,(s) ———~ H,PO, (aq) 2
Reduction half reaction : P,(s) ——— PH.(g) Ne H, (2)——> 2NO(g) +8e7
These can be balanced separately as : N,H,) + 80H" (aq) ——> 2NO(g) + 8e-
Oxidation halfreaction (balancing OH™ ions)
0 +1
NH,(2) + 80H-(aq) ———> 2NO(g) + 6H,O(/) + 8e-
CZ)
P,(s) ———> 4H,PO, (aq)
Reduction halfreaction
P,(s) ———> 4H,PO,- (aq) + 4e-
ClO; (aq) ——> CF (aq)
P,(s) + 80H (aq) ———> 4H,PO, (aq) + 4e- ___...(Z)
MODERN'S abc + OF CHEMISTRY-XI

+5 -1 Q. 20. What sorts of informations can you draw from


C10, +6e —— Cl(aq)
the following reaction ?
ClO; (aq) + 6e™ ——+ Cl (aq) + 60H (aq) (CN),(g) + 20H-(aq) —> CN-(aq) + CNO-(aq) +
ClO; (aq) + 3H,O(/) + 6e- ——> Cl (aq) + 6 OH-(aq) H,0O()
Ad) (i) Itis a disproportionation reaction.
To balance electrons, multiply eq (1) by 3 and eq (iz) (iz) The reaction occurs in basic medium.
by 4
(wi) O.N. of N in (CN), is —-3 and that in CN™ is
3N,H,() + 4Cl0;(aq) ——> 6NO() + 4 Cl(aq) + 6H,O() —2 and in CNO° it is —5.
Reduction
(iv) Cyanogen, (CN), gets simultaneously reduced
+7 -1 +3 0 +
to CN~ ion as well as oxidised to cyanate,
(c) C1,0,(g) + H,0,(ag) ——-~ ClO, (aq) + Oo (g)+H CNO> ion.
Q. 21. The Mn** ion is unstable in solution and
Oxidation undergoes disproportionation to give Mn**,MnO,,
In this reaction, Cl,O, gets reduced and acts as an and H* ion. Write a balanced ionic equation for
oxidising agent while H,O, gets oxidised and acts the reaction.
as a reducing agent.
The reaction may be written as :
Oxidation number method
Mn**(aq) —> Mn**(aq) + MnO,(s) + H*(aq)
O.N. decreases by 4 per atom
This may be balanced as :
or 4 x 2 per molecule y
+7 ms| +3 0 Oxidation half equation :
Cl,O,(g) + H,O,lag)——~— ClO, (ag) + On (g) a+ +1
O.N. increases by 1 per atom Mn** (aq) ——> Mn O,,(s)
or 1 x 2 per molecule Balance O.N. by adding electrons
To balance the increase or decrease in oxidation +4
number, multiply H,O, by 4 and add Mn** (ag)
> Mn0,,(s)+ e7
Cl,O(g) + 4 H,O, (aq) ——> ClO, (aq) + O,(g) Balance charge by adding H* ions
Balance Cl atoms and O, Mn** (ag) ———> Mn0O,(s) + 4H* + e~
Cl,O, (g) + 4H,O,(aqg) ——— ClO,~ (aq) + 40,(g) Balance O atoms by adding H,O
To balance Cl multiply ClO; by 2. Mn**(ag) + 2H,O(2) —> Mn0O,(s) + 4H* (aq) + e&
Cl,O. (g) + 4H,0,(ag) ——> 2ClO, (aq) + 40,(¢) vevee(L)
To balance O atoms, add 3H,O molecules on R.H.5. Reduction half equation
Cl,0.(g) + 4H,O, (ag) ——> 2Cl0,~ (aq) + 40,(¢) Mn*+(aqg) ———> Mn?*(aq)
+ 3H,O(/) Balancing O.N. by adding electrons.
To balance H atoms, add 2H* on R.H.S. since the Mn®*(aq) + e- ———> Mn2*(aq) __.... (it)
medium is acidic Adding eq. (z) and eq. (it)
Cl,0.(g) + 4 H,O, (aq) ———> 2Cl0,- (aq) + 40,(¢) 2Mn** (aq) + 2H,O(2) ——>
+ 3H,OW) + 2H*(aq) MnO,(s) + Mn?*(aq) + 4H*(aq)
Ion electron method Q. 22. Consider the elements :
1 Cs, Ne, I and F
Oxidation half reaction : H,O2 (aq) O> (g) (a) Identify the element that exhibits only
Reduction half reaction : Cl,O,.g) ——> ClO, (aq) negative oxidation state.
These can be balanced separately as : (6) Identify the element that exhibits only positive
Oxidation halfreaction oxidation state.
=] (c) Identify the element that exhibits neither the
H, O, (ag) ——> O,(g) negative nor does the positive oxidation
=| state.
H, Og (aq) ——— O,(g) + 2e7 Ans. (a) F : being most electronegative; shows only a —ve
Since the reaction occurs in acidic medium add 2H* oxidation state of —1.
on R.H.S. to equate charge, (6) Cs : Alkali metals have only one electron in their
H,O,(aqg) ———= O,(g) + 2e~ + 2H* (aq) w(L) valence shell and hence exhibits only +1 oxidation
Reduction half reaction state.
ClL,O.(¢) ——> ClO,- (aq) (c) I : because of the presence of 7 electrons in its
Cl,0O.(g) ——> 2Cl0,- (aq) valence shell, I shows O.S of —1 (to have stable
ClL,O.(g) + 8e— ——> 2ClO,° (aq) noble gas configuration) and +ve oxidation states
Cl,O.(g) + 8e— + 6H*(aq) ——> 2ClO,- (aq) of +1, +8, +5 and +7 because of the presence of
d-orbitals.
(medium is acidic)
C1,O,(g) + 6H*(aqg) + 8e- ——> 2C10,- (aq) + 3H,0 ...(iii) (2) Ne : It is an inert gas and therefore, does not
exhibit —ve or +ve O.5.
To balance electrons, multiply eq. (i) by 4 and add
to eq (iL) Q. 23. Chlorine is used to purify drinking water.
Cl,O.(g) + 4 H,O, (ag) ——— 2Cl0, (aq) + 40,(¢) Excess of chlorine is harmful. The excess of
chlorine is removed by treating with sulphur
+ 3H,O(/) + 2H* (aq)
REDOX REACTIONS

dioxide. Present a balanced equation for this O, is a limiting reagent.


redox change taking place in water. 160 g of O, produces NO = 120 g
Ans. The skeletal equation is : (120
Cl(g) + SO,(aq) + H,OW) —> Cl(aq) + SO,*(aq) 20 g of O, will produces NO = i60 * 20
Reduction half equation: =15¢
Cl(aqg) —> Cl(aq) Q. 26. Using the standard electrode potentials given in
Balancing Cl atoms the Table 8.1, predict if the reaction between the
Cl,(aq) —> 2CIl-(aq) following is feasible:
Balancing O.N. by adding electrons : (a) Fe**(aq) and I-(aq)
0 (6b) Agt(aq) and Cu(s)
Cl(aq) + 2e- —> 2Cl(aq) ast) (ec) Fe** (aq) and Cu(s)
Oxidation half equation: (d) Ag(s) and Fe**(aq)
SO,(aq) —> SO rae (aq) (e) Br,(aq) and Fe**(aq).
Balancing O.N. by adding electrons (a) It is clear from the table that electrode potential
+4 +6 Fe** |Fe (0.77V) is more than that of I, |I- (0.54V),
SO0,(aq) —> SO,2- (aq) + 2e- therefore, Fe®*+ will be readily reduced and the
Balancing charge by adding H* ions
following reaction is feasible.
SO,(aq) —> SO, (aq) + 2e- + 4H*(aq) Fe**(aq) + I- (ag) —> Fe?*(aq) + =1A)
Balancing O atoms by adding H,O (b) The electrode potential of Ag* | Ag (0.80V) is
SO,(aq) + 2H,OWU) —> SO,* (aq) + 2e- + 4H*(aq) more than that of Cu?* | Cu (0.34V) and therefore,
ohbE) Ag* will be reduced by copper. The following reaction
Adding equations (7) and (11) we get is feasible.
Cl,(aq) + SO,(aq) + 2H,OW) —> 2Agt(aq) + Cu (s) —> 2Ag(s) + Cu?*(aq)
2Cl-(aq) + SO?(aq) + 4H*(aq) (ec) The electrode potential of Fe*+ | Fe (0.77V) is
Q. 24. Refer to the periodic table given in your book and more than that of Cu?* |Cu (0.34V), therefore, Fe*+
now answer the following questions: can be reduced . The following reaction is feasible.
(a) Select the possible non-metals that can show Fe?+(aq) + Cu (s) ——> Fe?*(aq) + Cu?*(aq)
disproportionation reaction. (dq) The electrode potential of Ag | Ag* (0.80V) is
(b) Select three metals that can show more than that of Fe** | Fe (0.77V) and therefore,
disproportionation reaction. Agt will not be reduced by Fe*+. Therefore, the
Ans. (a) Non-metals showing disproportionation reactions reaction will not be feasible.
are : P,, Cl, and Se Ag(s) + Fe?+ (aq) ——> 2Ag(aq) + Fe?*(aq)
0 -3 +3 (e) The electrode potential of Br |Br- (1.09V) is more
P,(s) + 30H (aq) + 3H,0(/) > PH,{g) + 3H,PO,- (aq) than that of Fe** |Fe** (0.77V) and therefore, Br will
0 —1 +1 be able to reduce by Fe**. Therefore, the following
Cl,(aq) + 20H-(ag) —> Cl-(aq) + ClO” (aq) + HOC) reaction is feasible.
0 —2 +2
S,(s) + 120H-(aqg) —> S(aq) + 28,0,” + 6H,O(/) =Br,(aq) + Fe?* (aq) —> Br-(aq) + Fe**(aq)
(b) Metals exhibiting disproportionation reactions Q. 27. Predict the products of electrolysis in each of the
are : following:
Cut, Ga* Int (i) An aqueous solution of AgNO, with silver
+1 +2 0) electrodes
2Cut(aq) —> Cu?*(aqg) + Cu(s) (i) An aqueous solution of AgNO, with platinum
+1 +3 0 electrodes
3Ga*(aq) —> Ga**(aq) + 2Ga(s) (ii) An aqueous solution of H,SO, with platinum
+1 +3 0
electrodes
3In*(ag) ——> In**(aq) + 2In(s)
(wv) An aqueous solution of CuCl, with platinum
Q. 25. In Ostwald’s process for the manufacture of nitric
electrodes
acid, the first step involves the oxidation of
Ans. (z) Electrolysis of aqueous solution of AgNO, using
ammonia gas by oxygen gas to give nitric oxide
silver electrodes :
gas and steam. What is the maximum weight of
nitric oxide that can be obtained starting only AgNO,{(s) +nH,O —- Agt (aq) + NO,” (aq)
with 10.00 g. of ammonia and 20.00 g of oxygen H,O H+ + OH-
»
At cathode: Agt ions have lower discharge potential
Ans. 4NH.(g) + 50,(g) —> 4NO(g) + 6H, OW) than H* ions.
4x17 5 x 32 4x 30 Hence Ag* ions will be deposited as silver (in
preference to H* ions).
160 g of O, will react with 68 g of NH, At anode : Since silver electrode is attacked by
NO,” ions, Ag anode will dissolve to form Ag* ions
20 g of O, will react with NH, = —— x 20 in the solution.
= 8.0g Ag —> Ag* + e&
MODERN'S abc + OF CHEMISTRY-XI

(11) Klectrolysis of aqueous solution of AgNO, using Thus copper will be deposited on the cathode and Cl,
platinum electrodes : will be liberated at anode.
At cathode : same as above. Q. 28. Arrange the following metals in the order in
At anode: Since silver is not attackable, out of OH- which they displace each other from the
and NO,” ions, OH” ions have lower discharge potential solution of their salts.
and hence OH ions will be discharged in preference Al, Cu, Fe, Mg and Zn.
to NO,. The OH™ will decompose to give O,. Ans. Mg, Al, Zn, Fe, Cu
OH (ag) ——> OH + & Q. 29. Given the standard electrode potentials,
40H (aq) —> 2H,O(/) + O,) K* |K = -2.93V, Ag* |Ag = 0.80V,
(111) Electrolysis of H,SO, with Pt electrodes : Hg** |Hg = 0.79 V
Mg** |Mg = -2.37V, Cr**| Cr = -0.74V
H,SO, (ag) —> 2Ht(aq) + SO,? (aq)
arrange these metals in their increasing order
H,O H* + OH- of reducing power.
At cathode : H* + e* —->H Ans. Ag*| Ag, Hg** |Hg, Cr** |Cr, Mg”* |Mg, Kt |K
H +H —> H,fg) Q. 30. Depict the galvanic cell in which the reaction
At anode: OH-—=> OH +e Zn(s) + 2Ag*(aq) ——> Zn** (ag) + 2Ag(s) takes
40H —> 2H,0 + O,(g) place. Further show :
(tv) Electrolysis of aqueous solution of CuCl, with (¢) which of the electrode is negatively charged,
platinum electrodes : (ii) the carriers of the current in the cell, and
CuCl, (s) + (aq) —> Cu?*(aq) + 2Cl-(aq) (tii) individual reaction at each electrode.
Bw H* + OH- Ans. The cell may be depicted similar to Fig. 8.3. The cell
may be represented as
At cathode : Cu?* will be reduced in preference to
Zn(s) |Zn** (aq) || Agt(aq) |Ag(s)
H* ions.
(1) Zine electrode (anode)
Cu? + 2e°- —> Cu
(ii) Current will flow from silver to zinc in the
At anode: Cl- ions will be oxidised in preference external circuit.
to OH” ions (iii) At anode : Zn (s) —> Zn?*(aq) + 2e-
Cl —> Cl +e At cathode : Agt (ag) + e~-—> Agis)
Cl + Cl—> Cl,

Exemplar Problems //
Subjective Questions

Short Answer Questions <4


3MnO,” + 4H* ——> 2MnO, + MnO, + 2H,O
Q. 1. The reaction Cl,(g) + 20H (aq) — ClO(aq) +
Cl'(aq) + H,O(!) represents the process of Here O.N. of Mn increases from +6 (in MnO," ) to +7 (in
bleaching. Identify and name the species that Mn0O,,) and decreases to +4(in MnO,).
bleaches the substances due to its oxidising Q. 3. PbO and PbO, react with HCI according to
action. following chemical equations :
Ans. 0) —2 +1 +1 —2 —] +1 —2 2PbO + 4HCI —— 2PbCl1, + 2H,O
Cl,(g) + 20H (aq) —— Cl O'(aq) + Cl(aqg) + H,O(W) PbO, + 4HCl —— PbCl, + Cl, + 2H,O
In this reaction, O.N. of Cl increases from 0 (in Cl,) Why do these compounds differ in their
to +1(in ClO”) and decreases to —L(in Cl"). Therefore, reactivity ?
Cl, is both oxidised to ClO” and reduced to CI. +2 +2
Since Cl” ion cannot act as an oxidising agent
Ans. (7) 2PbO + 4HC]l —> 2PbCl, + 2H,O
(because it cannot decrease its O.N. lower than
+4 +2 0
—1), therefore, Cl, bleaches substances due to
oxidising action of hypochlorite, ClO’, ion. PbO, + 4HCl —> PbCl, + Cl, + 2H,O
MnO,” undergoes disproportionation reaction In reaction (1), O.N. of none of the atoms undergo a
in acidic medium but MnO, does not. Give change. Therefore, it is not a redox reaction. It is an
reason. acid-base reaction, because PbO, is a basic oxide
Ans. In MnO,,, Mn is in the highest oxidation state (+7). which reacts with HCl acid.
Therefore, it cannot be oxidised further and hence does The reaction (i) is a redox reaction in which PbO,
not undergo disproportionation. However, in MnO fan gets reduced and acts as an oxidising agent.
Mn is in +6 oxidation state. Therefore, it can increase Q. 4. Nitric acid is an oxidising agent and reacts with
its O.N to+ 7 and decrease its O.N. tosome lower value. PbO but it does not react with PbO,. Explain
Thus, it undergoes disproportionation as : why ?
REDOX REACTIONS

Ans. PbOisabasicoxide and therefore, it reacts with nitric (ii) I, + NOg- —> NO, + IO,”
acid. It is a simple acid-base reaction between PbO
and HNO,,. (iii) 1, + S037 —>T + S40,
2PbO + 4HNO, —> 2Pb(NO,), + 2H,O (iv) MnO, + C,0,2> —> Mn** + CO,
(Acid-base reaction) Ans. Try yourself. Balanced equations are
On the other hand in PbO,, lead is in +4 oxidation state
and cannot be oxidised further. Therefore
(i) 6Fe”* + Cr,02- + 14H* —>
PbO, is passive towards HNO, and no reaction takes 2Cr** + 6Fe** + 7H,O
place. (ii) 1, + 10ONO3 + 8H*— > 10NO, + 2103 + 4H,O
Q. 5. Write balanced chemical equations for the
(iit) I, + 28,03, —~2T + 8,06"
following reactions :
(i) Permanganate ion (MnO,°) reacts with (iv) MnO, + C,027 + 4H* —> Mn”* + 2CO, + 2H,O
sulphur dioxide gas in acidic medium to Q.9. Identify the redox reactions out of the
produce Mn”* and hydrogen sulphate ion. following reactions and identify the oxidising
(Balance by ion electron method) and reducing agents in them.
(i) Reaction of liquid hydrazine (N,H,) with (t) 3HCl(ag) + HNO,(aq) —>
chlorate ion (CIO3;) in basic medium CL,(g) + NOCI(g) + 2H,O(J)
produces nitric oxide gas and chloride ion (it) HgCl,(aq) + 2KI(aq) —> Hgl,(s) + 2KCl(aq)
in gaseous state. (Balance by oxidation (iii) Fe,O0,(s) + 3CO(g) —4_, 2Fe(s) + 3CO,(g)
number method) (iv) PCL,(1) + 3H,0V) —> 3HCl(aq) + H,PO,(aq)
(iit) Dichlorine heptaoxide (C1,O,) in gaseous (v) 4NH, + 30,(¢) —> 2N,(g) + 6H,O(g)
state combines with an aqueous solution Ans. (i) Writing the O.N. of on each atom,
of hydrogen peroxide in acidic medium +1 -1 +1 +5 -2
to give chlorite ion (Cl1O,) and oxygen 3HCl(aq) + H N O,(aqg) —>
gas. (Balance by ion electron method) 0 +3 -—2 -1 +1 -2
Ans. (i) MnO, (aq) + SO,(¢) —> Mn?*(aqg) + HSO 4 (aq) Clg) + N O Cl) + 2H,O(W)
Here, O.N. of Cl increases from —1 (in HCl) to
in acidic medium
O(in Cl,). Therefore, Cl is oxidised and hence
Refer NCERT Textbook Exercises Q. 18(b).
HCl acts as a reducing agent.
(it) NoH,(@) + ClO, (aq) —> NO(g) + CI (aq) The O.N. of N decreases from + 5 (in HNO.) to
Refer NCERT Textbook Exercises Q. 19(b). +3 (in NOCI) and therefore, HNO, acts as an
(iii) ClOAg) + H,O,(aq) —> ClO, (aq) + O,(g) + H* oxidising agent.
Refer NCERT TextBook Exercises Q. 19(c). Thus, reaction (1) is a redox reaction.
Q.6. Calculate the oxidation number of phosphorus +2 —1 +1 -l +2 —1 +1 -1
in the following species.
(it) HgCl(aq) + 2K I(aqg) —> Hg I,(s) + 2K Cl(aq)
(a) HPO3 and (6) POY Here O.N. of none of the atoms undergo a change
Ans. (a) Let O.N. of P be x and therefore, this is not a redox reaction.
H P 0,2 +3 —2 +2 —2 0 +4 -2
+1 +x +0(-2) =/ 22 or x = +d (iii) Fe,O0,(s) + 3COlg) —A_, 2Fe(s) + 3CO,(g)
(b) PO,” Here, O.N. of Fe decreases from +3 (in Fe,O,) to
x +4(-2) 2-3 or x = +5 0 (in Fe) and therefore, Fe,O., acts as an oxidising
Q. 7. Caleulate the oxidation number of each agent.
sulphur atom in the following compounds : O.N. of C increases from +2 (in CO) to +4 (in
(a) Na,S,O, (6) Na,S,0, CQ,) and therefore, CO acts as a reducing agent.
(c) Na,SO, (d) Na,SO, Thus, this is a redox reaction.
Ans. Let O.N. of S be x +3 — 1 +1 -2 +1 -1 +1 +3 —2
(a) Na, S, O, (iv) PCI,(/) + 3H,O0W7) —> 3HCl(aq) + H, P O,(aq)
2(+1) + 2x) + 38-2) = 0 or 2x = +4 or x = 42
Here O.N. of none of the atoms undergo a change
(b) 2.5, Refer Solved Example 6 (ii) page 10.
and therefore, it is not a redox reaction.
(c) Na, SO,
(+1) + x + 32) = Oorx = +4
3 +1 0 0 +1 -2
(d) Na, SO, (v) 4NH.(g) + 30,(¢) ——> 2N,(g) + 6H, Olg)
(+1) + x + 4-2) = Oorx = +6 Here O.N. of N increases from —3 (in NH,) to 0 in
Q. 8 Balance the following equations by the (N,) and therefore, NH, acts as a reducing agent.
oxidation number method. O.N. of O decreases from 0 (in O,) to —2(in H,O)
and therefore, O, acts as an oxidising agent.
(i) Fe** + H* + Cr,0%
—> Cr** + Fe** + H,O
Thus, reaction (v) is a redox reaction.
MODERN'S abc + OF CHEMISTRY-XI

Q. 10. Balance the following ionic equations MnO; +5e ——> Mn?*

(i) Cr,02- + Ht + IF—> Cr** +1, + H,O MnO; + 8H* + 5e= =~——> Mn”
(ii) Cr,02- + Fe?* + Ht —> Cr** + Fe®* + H,O MnO; + 8H* + 5e— ——> Mn**+4H,O _ ...ai)
(iii) MnO + SOZ” + H*—> Mn* + SO?” + H,O To balance the electrons, multiply eq. (1) by 5 and
eq. (ii) by 2 and add
(iv) MnO; + H* + Br: —> Mn* + Br, + H,O 2MnO; + 5SO2-+ 6H*——> 2Mn** + 5SO2-+ 3H,O
Reduction Reduction
[
Of li
(+6 -1 +3 o - +7 7 -1 +2 9 0
Ans. (i) Cr,02 + H* + I>-——+Cr*+I, + H,O (wv) MnO; + H’ + Bro > Mn+ Bro + H,O
a |
Oxidation Oxidation

Dividing the equation into two half reactions : Dividing the equation into two half reactions :
Oxidation half reaction : 1 ——-> I, Oxidation half reaction : Br~ ———> Br,
Reduction half reaction : Cr,O7 ——> Cr** Reduction half reaction : MnO; ——> Mn?+
Balancing oxidation and reduction half reactions Balancing oxidation and reduction half reaction
separately as : separately as :
Oxidation half reaction Oxidation half reaction
rF —> |
Br —— Br,
2- ——- |,
2Br —— Br,
2- —— = I, + 2e AL)
2Bre =——_—i Bry, + 2e™ et
Reduction half reaction
Cr, 02 ——> Cr* Reduction half reaction
Cr,0? ——> 2Cr** MnO,” —— + Mn*
Cr,02-+6e— ——> 2Cr** MnO, +5e MM Mn?
Cr,07-+ 14H* + 6e=§ ——> 2Cr** + 7H,O ..(ii) MnO, + 8H* + 5e° ——> Mn?
(acidic medium) MnO, + 8H* + 5e ——> Mn?* + 4H,0 © ...(ii)
To balance the electrons, multiply eq(z) by 3 and To balance the electrons, multiply eq.(z) by 5 and
add to eg (iz) eq.(iz) by 2 and add
Cr,0?-+ 14H* + 6I- ——> 2Cr** + 3L/#7H,O
2MnO; + 10Br- + 16H* ——-> 2Mn?* + 5Br, + 8H,O
(it) Refer Solved Example 25 (Page 908).

ee #7
Reduction

+4 +2 +6
»> Long Answer Questions [Carving Smart <4
(iii) MnO; + SO2 ——> Mn**++ S02> + H,O
Q. 11. Explain redox reactions on the basis of electron
Oxidation transfer. Give suitable examples.
Dividing the equation into two half reactions : Ans. Refer Text (Page 3-4).
Oxidation half reaction : ~~. SO a Q. 12.0n the basis of standard electrode potential
Reduction half reaction : MnOj7 ——> Mn?** values, suggest which of the following reactions
Balancing oxidation and reduction half reactions would take place ? (Consult the book for E°
separately as : value).

Oxidation half reaction (i) Cu + Zn** —> Cu + Zn


SO,7 — > s0; (ii) Mg + Fe?* —-+» Mg” + Fe
SO,7" ——> so? + 2e7 (iii) Br, + 2Cl- —> Cl, + 2Br
Since the reaction occurs in acidic medium, (iv) Fe + Cd?* —> Cd + Fe**
sO, ——> SO? + 2e- + 2H* Ans. (i) Cu + Zn** —> Cu?* + Zn
SO2-+H,O —— SO} + 2H* + 2ce-....@) E°(Cu?* |Cu) +0.34V, E(Zn**| Zn) = — 0.76V
o
Reduction half reaction cell
E*(cathode) — E*(anode)

— 0.76 — 0.384 =— 1.10V


MnO; —— Mn?*
Since Ey, 1s —ve, the reaction does not occur.
REDOX REACTIONS

(ii) Mg + Fe?* —> Mg** + Fe increasing order of oxidation number of


E°Mg"*|Mg) = — 2.37V, E°(Fe?*|
Fe) = — 0.74V chlorine : NaClO,, Na,CO,, NaClO, KCIO,, Cl,0.,
C1O,, CLO, NaCl, Cl,, ClO,.
cell = — 0.74 — (— 2.37) = +1.63V
Which oxidation state is not present in any of
the above compounds ?
Since E7., is +ve, the reaction will take place.
Ans. Let oxidation state of Cl be x
(wt) Br, + 2Cl —> Cl, + 2Br-
NaClo, +14+x + 4(-2) =0 orx =+7
a
cell
= +1.08 — 1.86 = — 0.28V NaClO, +1+x+4+3(-2)=0 or x = +5
NaClo +1l+x4-2=0 orx=+1
Since Ey.) is —ve, this reaction does not occur.
KCIO, +14+% +4 2(-2) =0 orx = +3
(iv) Fe + Cd?* —-> Cd + Fe**
ClO, 2x + 7(-2) =0 orx = 47
Ee,cell = — 0.44 —(- 0.74) = + 0.30
ClO, x + 3(-2) = 0 or x = +6

Since Eft, 1s +ve, therefore, this redox reaction Cl1,O 2x —2=0 orx
= +1
will occur. NaCl +1+x=0 orx =—-l
Q. 13.Why does fluorine not show disproportionation Cl, 2x = 0 ors =)
reaction ?
Clo, x + 2(-2) = 0 or x = +4
Ans. Fluorine is most electronegative and strongest
oxidising agent and therefore, it does not show
None of the compounds have an O.N. of +2.
positive oxidation states. Q. 16.Which method can be used to find out the
Q. 14.Write redox couples involved in the reactions strength of reductant/oxidant in a solution ?
(z) to (iv) given in Q. 12. Explain with an example.

Ans. (i) Cu2*|Cu and Zn?* | Zn Ans iJ The method to find out the strength of reductant/
oxidant in a solution, is to connect the redox couple
(ii) Mg?*|Mg and Fe?* | Fe of that species to normal hydrogen electrode and
(iii) Br, |Br- and Cl, | Cl- measure its electrode potential.
(iv) Fe?*|Fe and Cd?* | Cd If E° is +ve, the electrode acts as reductant and if it
is —ve, the electrode acts as oxidant. In this way by
Q. 15.Find out the oxidation number of chlorine in
the following compounds and arrange them in measuring the value for different redox couples, the
comparative strength can be determined.

ae

atno
Memory TEST W

“a Say True or False ia Complete the missing links


1. The oxidation state of oxygen is always —2. 1. The oxidation number of chromium in CrO; 1B occcsacrees
2. Oxidation number of an element can be zero but
valency is never zero. . Oxidant is a substance in which the oxidation number
The decomposition of calcium carbonate to calcium of one of the atoms
oxide and carbon dioxide is a redox reaction.
. When the oxidation number of an element is
Nitrous acid can act both as an oxidising as well as
reducing agent. maximum, it can act only as ...........
The reaction : Cr,02- +H,O0= 2CrO02- + 2H* cannot . The oxidation number of oxygen in sodium peroxide
be regarded as a redox reaction.
1S wcccccucnss
The oxidation number of each iron atom in Fe,O, is
same. . In an electrochemical cell acts as a negative
The reaction : V,O, + 5Ca —> 2V + 5CaO is a metal pole and acts as a positive pole.
displacement reaction.
The oxidation number of carbon in CH,Cl, is +4. . A compound in which oxidation number of oxygen is
Oxidation number of Mn in MnO, is more than in +2 is
MnQ,. . In an electrochemical cell, oxidation occurs at ..........
The oxidation number of hydrogen in LiH is —1. and reduction occurs at
MODERN'S abc + OF CHEMISTRY
-XI

8. Stock notation of chromium trioxide is .......... 4, Oxidation number of phosphorus in Na,PO, is + 3/+5.
9. Oxidation number of N in ammonium sulphate is 5. The oxidation number of each terminal S atom in
tetrathionate ion is +5/+2.5.
6. In the reaction : F’, + 2NaOQH —> 2NaF + OF, + H,O,
F, undergoes only reduction/both oxidation and
reduction.
7. Copper metal can/cannot be oxidised by Zn?* ions.
1. Inanelectrochemical cell reduction occurs at cathode/
anode.
8. Iodine oxidises Na,S,O, to Na,S,O,. This statement
is true/false.
2. In an oxidation reaction, oxidation number of an atom
9. Reductant is a substance in which oxidation number
increases /decreases.
of one of the atoms decreases /increases.
3. Oxidation number of C in CO, is more/less than in
CH,Cl,,. 10. In the reaction Zn(s) + PbCl,(aqg) —> Pb(s) +
ZnCl,(aq), zine acts as oxidising /reducing agent.

Memory Test “\\


say True or False Be Complete the missing links

1. 5 2. decreases
1. False 2. True
3. oxidising agent 4, -1
3. False : It is not a redox reaction because oxidation
5. anode, cathode 6. OF,
number of none of the atoms changes.
7. anode, cathode 8. Cr, (IDO,
4, True 5. True 9.-—3 10. +7
6. False : It exists as FeO-Fe,O, having oxidation number
+2 and +3.

7. True 8. False : It is zero. 1. cathode 2. increases 3. more


4, +5 5. +5 6. only reduction
10. True.
7. cannot 8. true 9. increases
10. reducing

a Advanced Level
QUESTIONS WITH ANSWERS

Q. 1.While sulphur dioxide and hydrogen peroxide B®413+;a) = —1-66 V, FE agtlAg =+0.80 V.
can act as oxidising as well as reducing agents in their
reactions, ozone and nitric acid act as oxidising agents. Ans.Since reduction potential of silver is more than that
Why ? of hydrogen ‘a , Pt = 0), silver vessel will be suitable to
Ans.The oxidation state of 5 in SQ, is +4. It can be store 1M HCl. On the other hand, E° Al?*| ays less than that
oxidised to +6 oxidation state or reduced to +2. Therefore, it
acts as a reducing agent as well as oxidising agent. Similarly, of hydrogen (EH ,Pt) so that hydrogen will be liberated
the oxidation state of oxygen in H,O, is —1. It can be if stored in aluminium vessel.
oxidised to O, (zero oxidation state) or reduced to H,O or Q. 3.Can Fe** oxidize Br- to Br, at 1 M concen-
OH-(—2 oxidation state) and therefore acts as reducing as tration?
well as oxidising agent. E°(Fe** |Fe**) = 0.77 V and E°(Br |Br-) = 1.09 V
However, both ozone and nitric acid can only decrease Ans.E°(Fe** | Fe*+) is lower than that of EBr | Br”).
their oxidation number and therefore, act only as oxidising Therefore, Fe2* can reduce Br, but Br~ cannot reduce Fe*+,
agents, Thus, Fe** cannot oxidise Br to Bry.
Q. 2.0ut of aluminium and silver vessel, which Q.4. Is it safe to stir 1M AgNO, solution with a
one will be more suitable to store 1 M HCI solution copper spoon ? Given:
and why ? E° esti = 0.80 V, E° Cu2+!Cu = 0.34V.

WWW.JEEBOOKS.IN
REDOX REACTIONS

Ans. No, copper spoon will dissolve as Cu? ions because (difference of electrode potentials of the cathode and anode)
copper has more tendency to get oxidised than silver. becomes zero.
Q.5. Consider the following table of standard Q.9. (a) What is the maximum and minimum
reduction potentials : oxidation states of nitrogen in its compounds ? Give one
example each.
(b) What is the oxidation number of N in each of the
following:
(t) NH, (it) NoH, (it) HN, (iv) NO.” (v) NO (vi) HCN
(vit) N, (vitt) NH,OH (ix) HNO, (x) NQ,.
(c) What is the oxidation state of hydrogen in each
of the following :
(a) Which substance is (t) H* (tit) H, (tit) LiAIA, (iv) HC! (v) Li
(i) strongest oxidising agent ? Ans. (a) +5 equal to group number (e.g. N,O.) and —3
(iz) strongest reducing agent ? equal to the group number minus 8 (e.g. NH,)
(b) Which substance can be oxidised by B** ? (b) (4) —3 (it) —2 (ait) —1/3 (tv) +3 (v) +1 (vi) -3 (vit) 0
(c) Which substance can be reduced by C ? (viit) —1 (ix) +5 (x) +4
(d) Write a balanced chemical equation for the (ce) (Z) +1 (zi) O (zit) —1 (iv) +1 (Vv) -1
overall cell reaction that gives the highest cell voltage Q.10. (a) Use the following reactions to arrange
and calculate E° for the reaction. the elements A, B, C and D in order of their redox
Ans. (qa) (i) A** is the strongest oxidising agent. reactivity :
(iz) D is the strongest reducing agent. (@) A+ B*— AT+B
(b) B** can oxidise C and D. (iz) B+ D*—> B*+D
(c) C can reduce A*+ and B?*. (i) C* + D—S No reaction
(d) A+ + 2D ——> At + 2D* (jv) B+ C* 35 B*+C
K° = 1.36 — (-1.42) = 2.78 V (6) On the basis of above redox activity series,
Q.6. Copper dissolves in dilute HNO, but not in predict which of the following reactions would you
dilute HCL. Explain. expect to occur.
Ans, Since E° of Cu?* |Cu electrode (+0.34 V) is higher (j) A*+C —3A+C* (ti) A*+D—3G9A+D*
1 Ans. (a) The electrochemical series or redox activity is
than that of H*| pile (0.0V), therefore, H+ ions cannot
based on the decreasing order of reduction potentials. This
oxidise Cu to Cu?* ions and consequently, copper does not means that the species which gets reduced is higher in the
dissolve in dil. HCl. On the other hand, the electrode potential
electrochemical series as compared to the other which is to
of NO,” ion, NO, |NO (+0.97V) is higher than that of copper get oxidised (lose electrons).
electrode and hence, it can oxidise Cu to Cu** and hence
In reaction (1), B* gets reduced by A and therefore B is
copper dissolves in dil. HNO.
higher than A in electrochemical series.
Q.7. Element A will reduce the cation of element B
(B*) but will not reduce the cation of element C (C*). Will In reaction (11) D* gets reduced by B and therefore, D is
higher in electrochemical series than B. In reaction (zz) Ct
element C reduce the cation of element B? Explain.
does not get reduced by D, therefore, C is lower than D in
Ans. Since A will reduce B*, B has higher reduction
electrochemical series. But according to reaction (iv) C* gets
potential than A and therefore, B lies above A in the
reduced by B and therefore, C is higher in electrochemical
electrochemical series. Now, A will not reduce C*, C has lower
series than B. Thus, the correct order is
reduction potential than A and therefore, C lies below A in
the electrochemical series. Therefore, C must be below B in D>C>BrA
the series and hence C will reduce Bt. (6) Both reactions donot occur because A cannot be
Q.8.Why does an electrochemical cell stops reduced by C as well as D.
working after some time ? Q.11. 40.05 mL of 1.0 M Ce* are required to titrate
Ans. The electrode potential of any electrode depends 20.0 mL of 1.0 M Sn?* to Sn**, What is the oxidation
upon the concentration of ions. With time as the cell state of cerium in the reduction product ?
operates, the concentrations of ions in different Ans. The reactions occuring are :
compartments change and therefore, the electrode potentials Ce* + ne- —> Celt *)+
also change. For example, in Zn—Cu cell, Sn*+ ——> Sn* + 2e-
Zn(s) + Cu**(aqg) —> Zn?* + Cu(s) To balance the equations, (the no. of electrons lost = no.
the concentration of the Zn** ions in the anode compartment of electrons gained) multiply eq. (i) by 2 and eq. (ii) by n and
increases and hence the electrode potential of anode also add
increases. At the same time, the concentration of Cu2* ions
2Ce* + nSn*+ —> Ce + Sn*#
in the cathode compartment decreases and hence the
Moles of Ce** in 40.05 mL of 1.0 M solution,
electrode potential of the cathode also decreases. Ultimately,
the electrode potential of the two electrodes become equal
- a x 40.05 = 40.05 x 10-3 mol
and at this stage, the cell stops working because E,,,, 1000
MODERN'S abc + OF CHEMISTRY-XI

Now 2 mol of Ce** will oxidise n mole of Sn?* 11.70 = 19.5-—3.90 n


40.05 x 10-° mol of Ce** will oxidise Sn?+ or 490n = 19.5-11.7=7.8

- 5% 40.0510 mol = 20.02 nx 103 mol


.. Value of n = 2
But moles of Sn?2* in 20.0 mL of 1.0 M solution Q. 13. 15.0 mL of 0.05 M SeO, reacts with 30.6 mL
of 0.1 M CrSO, solution. If during the reaction CrSO,
= 1.0 x 20.0 = 20.0 x 10- mol gets oxidised to Cr,(SO,),, to what oxidation state
1000
does selenium get converted ?
20.02 n x 10-% mol = 20.0 x 10-° mol Ans. Moles of Se** ions present in 15.0 mL of 0.05
n=1
0.05 —4
Hence 1 mol of electrons are required in the reduction M = T000" 15.0=7.5*10~ mole
of each mol of Ce** ion.
Moles of Cr2* ions present in 30.6 mL of 0.1 M solution
‘ Ce* + e- —> Ce**
Ce** is the reduction product. = 1 30.6 =8.06x10~ méle
Q.12. 3.90 x 10-3 moles of a solution containing an 1000
ion A”* require 2.34 x 10° moles of MnO, for the The oxidation reduction reactions are :
oxidation of A”* to AO,” in acidic medium. What is Cr4+ f2— Cre wld)
the value of n ?
Se*t + nevy—> _~Selt+ (it)
Ans. Oxidation and reduction reactions are :
Reduction : Multiply eqn. (i) by n and add to eqn. (ii)
MnO, + 8H* + 5e- —> Mn?* + 4H,O AL) nCr**+ Se** —> 4Cr*+ + Set-)+
Oxidation : According to the reaction
A™* + 3H,O0—> AO, + 6H* + (5—n)e~ ...()
n mole of Cr?*+ will reduce 1 mole of Se**
In a redox reaction, no. of electrons lost must be equal
to no. of electrons gained, therefore, multiply eq. (1) by 3.06 x 10 mole of Cr?* will reduce Se**
(5-n) and eq(iz) by 5 so that 7 ,
(6—n)MnO, = 5A™ = —x3.06x10~ mole
n
“. (5 —n) moles of MnO,- will oxidise 5 moles of A™*
or 2.34 x 10-3 moles of MnO 4 will oxidise A"* But Se** actually reduced = 7.5 x 10 mole
5 | C0GHIO? -3 pe
x2.34x107 moles ape
—n
But number of moles of A"* actually oxidised = 3.90 x Ii

10-8 moles
3.06 x107
x 2.34x10° 3.90 x 10°
= fh 7.5x10
or Bx 2.4 = (5-—n)
x 3.90 Reduction of 1 mole SeO, requires 4 electrons.
Hence, Se** is reduced to Se®.

Revision Exercises
9. Calculate the oxidation number of lead in Pb,O,.
»»> Very Short Answer Questions <4
10. Identify the oxidising and reducing agent in the
. Define oxidant and reductant. following reaction :
. Define oxidation and reduction.
3CuO + 2NH, ——-> 3Cu + N,+ 2H,O
11. Give an example of a compound in which the oxidation
. What is the oxidation number of Mn in MnO, ? number is fractional.
bo.
CF
-~ In the following reaction, predict the oxidising agent 12. Calculate the oxidation number of phosphorous in
and reducing agent Mg.P.0..
13. Indicate the oxidizing and reducing agents in the
Cr,0,> + 6 Fe** + 14 Ht ——-> 2Cr°* + 6Fe®+ + 7H,O
following reaction :
5. What is the average oxidation number of carbon in
acetone (CH,COCH,) ?
2Cu?* + 4I- —> 2Cul +L,
14, Calculate the oxidation number of N in NO,°.
6. Can oxidation number be zero ? Illustrate.
15. The oxidation number of B in Na, B,O, is +3. Is the
7. Will oxidation number increase or decrease in a
statement correct ?
reduction reaction ?
16. What is the oxidation number of Cr in CrO,Cl, ?
8. What is the oxidation number of oxygen in H,O,?
17. Calculate the oxidation number of C in C,H, ,0¢.
REDOX REACTIONS

18. Identify oxidant and reductant in the following 9. State the maximum and minimum oxidation state of
reaction : each of the following :
I, (aq) + 25,0?" (aq) —> 2I- (aq) + 8,0," (i) Ta (iz) V (i) 5 (ww) N
19. In the reaction : 10. Calculate the oxidation number ofthe underlined atom:
MnO, + 4HCl —> MnCl, + Cl, + 2H,O, which species @K MnO, (i) PoO, (it) Fe,O, (iv) Xe OF,
is oxidized ?
(v)S,O,2- = (vi) Cr, O,*.
20. Name a compound each in which the oxidation state of
11. In the following reactions, label the oxidising agent and
O is (i) +2 and (ii) —2.
the reducing agent :
21. Name the type of reactions that occur at cathode in an
electrochemical cell. Give one example.
(a) MnO, + 4HCl_ ——> MnCl, + Cl, + 2H,O
22. The reduction potentials are : (b) PbS(s) + 4H,O,(aq) — > PbSO,(s) + 4H,O(/)
Cl, + 2e- = 2 Cl E° = 1.36 V (c) 2Al + 3F,(g) ——— 2AlF,(s).
F, + 2e- = 2 F- BE’ = 2.87V 12. How will you differentiate between oxidation number
Which is a better oxidising agent ? and valency ?
23. What is the oxidation number of § in the reactants and 13. Arrange the following in the decreasing order of
the products : oxidation number of Mn :
2Na,5,0,+1,——> Na,§S,0,
246 + 2Nal
(1) KMnO, (i) MnO, (iii) Mn,O,
24, Can we store 1 M AgNO, in a copper vessel ? Given (iv) Mn (v) K, MnO,
E°cu*i cu = + 0.34 V, E°agtiag = + 0.80 V 14, Define oxidation and reduction according to electronic
concept. Comment “oxidation and reduction always go
side by side”.
>» Short Answer Questions <4
15. Balance the following redox reaction by oxidation
1. Define the terms : oxidation, reduction, oxidising agent number method :
and reducing agent according to electronic concept. Cr(OH), + 10,5 ——> I’ +CrO0,2- (basic medium)
2. Comment on the statement : oxidation and reduction 16. Consider the reaction :
go side by side.
2HBr + Cl, ——> 2HCl + Br,
3. Which of the following reactions is oxidation and which
Identify the substance
is reduction ?
(z) 2H,0 +2e —— 20H +H, (i) getting reduced (iz) getting oxidised
(ii) Al —— AIl+ + 3e- (iz) acting as reducing agent
(iii) Fe?+ ——» Fe?+ —e- (iv) acting as oxidising agent.
(iv) 207 +C — > CO, + 4e7
»»> Long Answer Questions <4
(v) Br —— 1/2 Brote™
(vi) 2H,O0 —- 4e- ——— 4H*++0O, 1. Balance the following equations using oxidation number
4. In the following reactions, explain which reactant is method :
oxidised and which is reduced. Give reasons for your (i) MnO, + Ht + Fe?* ——>» Mn? + Fe*+ +H,O
answer : (ii) Zn + NO, + Ht ——> Zn2* + H,O
(1) 2H,S + SO, ———> 2H,0+ 35 (zi) H,SO,+1,+H,Q —— H,SO, + HI
(it) MnO, + 4HCl ———> MnCl, +H,0 + Cl,
(iv) HNO, +I, —— HIO, + NO, + H,O
(iL) 2KI + Cl, —— 2KCl + I, (v) MnO, +H,O, ——> MnO,? +0,
(iv) CuO + CO. —=-> CO, + Cu
(in alkaline medium)
5. Calculate the oxidation number of :
2. Balance the following equations by oxidation number
(i) Sin Na,S,O, (iz) Clin HCIO, method :
(zit) Mn in MnO, (1v) Boron in Na,B,O, (7) HS +HNO,
—— H,0+NO+5
(v) Crin K,Cr,0,
(iz) NH, + O, ——~> NO+H,0
6. What is the oxidation number of 5 in the following :
(iii) Cu + HNO, ———> Cu(NQ,), + NO + H,0.
(7) SO, (it) Nags. (iii) S*
3. Balance the following equations by ion electron (half
(iv) CS, (v) S,Cl, reaction) method :
7. What is the oxidation number of hydrogen in the
following: () H,S+MnO, + Ht ——-> S+Mn?*+H,0
(1) Ht (it) NaH (iii) Hy (wv) LiAlH, (ii) Cr,0,*- + Ht + Fe** ———> Cr** + Fe** + H,O
8. Calculate the oxidation number of underlined atom in
(iit) AsO, + 10,> ——~> AsO rf. +1-
the following :
(2) NaAuCl, (ii) OF, (iii) Ca(C1O,), (iv) snO, + C ——> 5n+CoO.
(iv) NH,OH (v) SeO,?- 4, What are redox reactions ? Discuss with examples. Give
important applications of redox reactions.

WWW.JEEBOOKS.IN
MODERN'S abc + OF CHEMISTRY-XI

5. Write short notes on: dioxide gas in acidic medium to produce Mn** and
hydrogen sulphate ion.
(a) Electrochemical series
(Balance by ion electron method)
(b) Redox titrations

(c) Abnormal oxidation number and structures of


(iz) Reaction of liquid hydrazine (N,H,) with chlorate
compounds. ion (C1O,) in basic medium produces nitric oxide
gas and chloride ion in gaseous state.
. Identify the redox reactions out of the following
reactions and identify the oxidising and reducing agents (Balance by oxidation number method)
in them. (uit) Dichlorine heptaoxide (Cl,O,) in gaseous state
(i) 8HCl(aq) + HNO, (ag) ——> combines with an aqueous solution of hydrogen
peroxide in acidic medium to give chlorite ion
Cl, (g) + NOCI (g) + 2H, (1) (Cl1O;) and oxygen gas.
(it) HgCl, (ag) + 2KI (ag) ———Hegl, (s) + 2KCl (aq) (Balance by ion electron method)
(zit) FeO, (s) + 3CO (g) _4_, 2Fe (s) + 38CO, (g) 8. Balance the following equations by the oxidation
(iv) PCI, (1) + 3H,O () —> 3HCl (aq) + H, PO, (aq) number method.

(v) 4NH, + 30, (¢) ——> 2N, (g) + 6H,O (g) (i) Fe* +H*+Cr,07 —— Cr* + Fe* + H,O
7. Write balanced chemical equation for the following (i) I+ NO>—— NO, + 10;
reactions :
(iii) I, + SOF ——I +8,0%
(i) Permanganate ion (MnO7) reacts with sulphur
(iv) MnO, + C,O{ ——> Mn* + CO,

€ =
=, im,
\ Hints & Answers
, =
Revision Exercises

Very Short Answer Questions Long Answer Questions


3. +4 5. 4/3 6. Yes 7. decrease
1. (v) MnO + 8H + 5Fe* —> Mn? + 5Fe* + 4H,0
8. -—1 9. + 8/3 11. Na,S,0, 12. +5
(it) 4Zn + NO, + 10H+ —>4Zn** + NH,* +3H,O
1 8. Cu: Oxidizing agent, I- : reducing agent
14. +5. 15. Correct 16. +6 (iit) H,SO, +1,+H,O0 ——H,SO, + 2HI
(iv) 10HNO, +1, —— 2HIO,+10NO,+4H,O
Short Answer Questions
(v) 2MnO,- +20H- +H,0, —>2Mn0,2-+0, +2H,O
oe Reduction reaction (Z), (1/2) and oxidation reaction
2. (i) 3H,S + 2HNO, — 2NO + 35 + 4H,O
(iz), (tv), (v), (vt)
(ii) 4NH, +50, —~4NO +6H,O
4, (t) SO, is reduced and H,S is oxidised.
(ii) MnO, is reduced and HCl is oxidised. (il) 3Cu + 8HNO, —> 3Cu(NO,),+2NO+4H,0
(iii) Cl, is reduced and KI is oxidised. 3. (i) 5H,S + 2MnO; + 16H*+ —>5S + 2Mn?+ + 8H,O
(iv) CuO is reduced and C is oxidised.
(ii) 6Fe** + Cr,0,2- + 14H* —-> 6Fe** + 2Cr** + 7H,O
o. (1)+2 (ut) +7 (ut) +4 (wv) +3 (v) +6
10. (¢) +7 (a) +5 (it) +3 (iv) +6 (v) +2 (vl) +6
(iii) 3AsO7 +10,> —>3AsOf +I
13. KMnO,, K,MnO,, MnO,, Mn,O,, Mn (iv) SsnO, + 2C ——>Sn + 2C0O

WWW.JEEBOOKS.IN
REDOX REACTIONS

(Competition Filo
«= |OBJECTIVE TYPE QUESTIONS /5
All. The oxidation states of V and Br in V(BrQO,), are
respectively
A xo with only one correct answer
(a) 2 and 2 (6) 2and 1
Select the correct answer: (c) 4and2 (d) 2 and 3.
Al12. The oxidation state of N in HN, is
Oxidation, Reduction and Oxidation Number
(a) +8 (b)-3 ~~ (e)-1/8— (d) +1/8
A1. Reduction involves : Al13. In which of the following 5 has highest oxidation state?
(a) gain of electrons (a) Na,S,0, (b) 5,Cl,
(6) addition of oxygen (c) 8, (d) H,SO,
(c) increase in oxidation number Al4, Which of the following rules for oxidation number is
(d@) loss of electrons. not correct ?
A2. Oxidation number of P in PO ion is: (a) The algebraic sum of all the oxidation numbers
(a)-—3 (b) +7 in a compound is zero.
(c)+5 (d) +3. (6) An element in the free or the uncombined state
A3. Oxidation number of Mn in MnO, ion is : bears oxidation number zero.
(a) +1 (6) -7 (c) The oxidation number of hydrogen is always +1.
(c) -1 (d) +7. (d) In all its compounds, the oxidation number of
A4, Oxidation number of C in CH,OH, CH,O, HCOOH and fluorine is —1.
C,H, is respectively :
Al15. In the reaction :
(a) —2, 0, +2, —1 (5) +2, 0, +2, -2
(c) —2, 0, +2, 0 (d) —2, -4, +2, -2 3CuO0 + 2NH,—> N, + 3H,O + 3Cu
A5. What is the oxidation state of 5S in Na,5, ? the change of NH, to N, involve
(a) +1 (b) -2 (a) Loss of 6 electrons per mol of N,
(ec) -1 (d) 0. (6b) Loss of 3 electrons per mol of N,
A6. The oxidation state of S in Caro’s acid (permono (c) Gain of 6 electrons per mol of Ny
sulphuric acid) H,SO, is (d) Gain of 3 electrons per mol of Ny
(a) +8 (b) +6
Al16. Which of the following statement is not correct ?
(c) +5 (d) +4
(a) Oxidant is a substance which increases the
AZ. Oxidation number of sodium in sodium amalgam
oxidation number of other substance.
(Na/Hg) is
(6) Reductant is a substance which decreases the
(a) 0 (b) +1
oxidation number of other substance.
(ce) =1 (dq) +2
(c) The oxidation number of oxidant decreases.
A8. In the reaction : Cl, + 20H —— OCI + Cl + H,O
(@) In oxidation there is decrease in oxidation number,
(a) OH" is oxidising and Cl~ is reducing agent
A177. When phosphorus reacts with caustic soda, the products
(6) Cl, is oxidising and OH~ is reducing agent
are PH, and NaH, PO,. The reaction is an example of
(c) OH~ is both oxidising and reducing agent
(d) Cl, is both oxidising and reducing agent. (a) oxidation (6) reduction
AQ. The oxidation state of S in $,0,” is (c) disproportionation (d) none of these.
(a) +2 (b) +4 Al18. Which of the following is not an example of redox
(c) +6 (dq) +7 reaction ?
A10. In which of the following compounds, the oxidation (a) CuO+H, ——>Cu+H,0O
number of carbon is not zero ?
(6) Fe,O, + 3CO ——-> 2Fe + 3CO,
(a) Cy H5.0), (6) HCHO
(c) 2K + F,——> 2KF
(ec) CH,CHO (d) CH,COOH.
(d) BaCl, + H,SO, —— BaSO, + 2HC1

Ariswes
Al. (a) A2. (c) A3. (d) Ad. (a) A5. (c) A6. (5) AZ. (a) <A8&. (d) AY (c) Al0. (ce)
All. (d) A112. (c) A183. (qd) Al4, (ec) A115. (a) A17%. (c) A18. (d)

WWW.JEEBOOKS.IN
MODERN'S abc + OF CHEMISTRY-XI

Al19. The oxidation state of Cr in Cr(CO), is . On the basis of standard electrode potential of redox
(a) 0 (b) +2 couples given below find out which of the following is
(c) -2 (d) +6. the strongest oxidising agent.
. The oxidation state of oxygen in H,QO, is E°values : Fe* |Fe? = + 0.77; I(s) IT = +0.54;
(a) —1 (b) +2 Cu*!Cu = + 0.34; Ag* |Ag = + 0.80V
(c) -2 (d) +1. (a) Fe* (6). I,(s)
. The oxidation state of phosphorus in Ba(H,PQO,), is (c} Cu (d) Ag”
(a) +8 (b) +2 A31. The coefficients x, y and 2 in the following balanced
(c) +1 (d) -1 equation :
The oxidation number of 5 in S,, 5,F, and H,5
x Zn + yNO,-—> zZn** + NH,’ (in basic medium) are
respectively are :
(a) 0, +1,-2 (6) +2, +1,—2 (a) 4,1,4 (b) 2,2,2
(c) 0,41, +2 (7d) +2, +1,—2 (c) 4,2,4 (d) 4,4,4
. Inthe reaction: A382. The following four colourless salt solutions are placed
3Br, + 6CO,? + 3H,O—— 5Br + BrO,- + 6HCO,~ in separate test tubes and a strip of a copper is placed
(a) Br, is oxidised and carbonate is reduced. in each. Which of the following solutions will finally
(6) Bromine is reduced and water is oxidised. turn blue ?
(c) Bromine is neither reduced nor oxidised (a) NaCl (6) AgNO,
(qd) Bromine is both reduced and oxidised. (c) ZnSO, (d) Cd(NO,),.
. Oxidation state of Fe in Fe,O 4 is In a standard hydrogen electrode, the concentration
(a) +2 (b) 43 of H* is:
(c) +8/3 (d) +2/3 (a) 0.1M (6) 1M
. In Br,O, compound, oxidation number of bromine is (c) 10M (d) Not fixed.
(a) 16/13 (b) 26/3 A34, In a galvanic cell, which of the following statement
(c) 24/3 (d) 16/3 is correct ?
A26. Which is the best description of the behaviour of bromine (a) anode is negatively charged
in the reaction (5) cathode is positively charged
H,O + Br, —> HOBr + HBr (c) reduction occurs at anode
(a) Proton acceptor only (d) standard e.m.f of the cells is always zero.
A365. For the redox reaction :
(6) Both oxidised and reduced
MnO, + Fe?+ + H*—> Mn** + Fe** + H,O
(c) Oxidised only
The correct coefficients of the reactants in the balanced
(d@) Reduced only reaction are :
A27. The oxidation number of H in LiAlH, is MnO, Fe? Ht
(a) —1 (b) +1 (c) O (d) +3 (a) 1 5 8
A28. The oxidation states of sulphur in the anions SO? , (b) 2 5 8
S,0,?- and S,0,?> follow the order : (c) 4 5 16
(a) S,Of <SO}-<S,0> (d) 2 5 16
(b) SO? <S,07'<S,0?" A36. The standard reduction potential values of the three
metallic cations X, Y and Z are 0.52, —3.03 and —-1.18
(c) S,Of <S,02"<SO? V respectively. The order of reducing power of the
(d) S,O? <S,07-<SO? corresponding metal is ?
Balancing Redox Reactions and Electrochemical Cells (a) Y>4>X (b) X>Y>Z
(c) Z>Y>X (dq) Z>X>Y
A29. E°values of some redox couples are given below. On
A377. A gas at 1 atm is bubbled through a solution containing
the basis of these values choose the correct option.
1M Y~- and 1 M & at 25° C. If the reduction potential
E*values : Br,| Br = + 1.90; Ag" |Ag(s) = + 0.80 of Z> Y >Z, then
Cu*!Cu(s) = + 0.34; L(s) IT = + 0.54 (a) Y will oxidize X and not 74
(a) Cuwillreduce Br (6) Cu will reduce Ag (6) Y will oxidise Z and not X

Answer
(ec) Cuwillreducel (d) Cuwill reduce Br, (c) Y will oxidize both X and 4%
(d) Y will reduce both X and Z.

A19. (a) A20. (a) A21. (c) A22. (a) A23. (d) A24, (c) A26. (b) A27. (a) A28. (a)
A30. (d) A311. (a) A33. (b) AS4, (c) AS6. (a) A37. (a)
REDOX REACTIONS

A38. Standard electrode potentials are : (c) Fe?+/Fe?+ remains unchanged


Fe2+| Fe (E° = —0.44 V), Fe?+ | Fe*+ (E° = 0.77 V) (d) Fe?* decreases
Fe?+, Fe*+ and Fe blocks are kept together, then . For decolourization of 1 mole of KMnQ,, the moles of
(a) Fe** increases H,O, required is
(b) Fe*+ decreases (a) 1/2 (b) 3/2
(c) 5/2 (d) 7/2
Araswers
38. (b) 39. (c)

B6. In the redox reaction


MULTIPLE CHOICE QUESTIONS xKMnO, +NH,—> yKNO, + MnO, + KOH + H,O
from competitive examinations (a) x =Agy = 6 eld) x = 8, y =6
(c) x=3,y=8 (d) x=8,y=8
AIPMT & Other State Boards’ (DUMET 2009)
Medical Entrance B7. Which of the following have been arranged in
decreasing order of oxidation number of sulphur ?
Bl. Which of the following is a redox reaction ?
(a) Na,S,O, > H,S,0, > Na,S,0, > S,
(a) 2CuSO, + 4KI —> Cu,I, + 2K,S0, + I,
(6b) SO, + H,O —> H,SO, (6b) H,SO, > SO, > H,S > H,S,0,
(ec) Na,SO, + BaCl, —> BaSO, + 2NaCl (c) SO > SO{ > SO? > HSO,;
(d) CuSO, + 4NH, —=> [Cu(NH,),|SO, (dq) H,SO, > H,SO, > SCI, > H,S
(A.M.U. Med 2010)
(e) C990), + HJO—> C,H .0¢ + CoH 1.0, B8. The average oxidation state of sulphur in Na,8,0, is
(Kerala P-M.T. 2007) (a) +2.5 (6) +2
B2. Oxidation number of iodine in IO-,, [O-,, KI and [, (c) +3.0 (dq) +3.5
respectively are (e) +4 (Kerala PMT 2010)
(a) —1,—1, 0, +1 (6b) +3, +5, +7,0
B9. Standard electrode potential of three metals X, Y and
(c) +5,+7,-1,0 (d) -1,-5,-1,0
“ are — 1.2 V, +0.5 V and — 3.0 V respectively. The
(e) —2,-5,-1, 0 (Kerala PMT 2008) reducing power of these metals will be
B3. In the balanced chemical equation,
(a) ¥>4>X (6) YoXRoZ
I0,- +al- + b6H* —> cH,0 + dl,
a, b,c and d respectively correspond to (c) Z>X>Y (dq) X>VY>o4 (ALPMT 2011)

(a) 5, 6, 3,3 (6) 5, 3, 6.3 B10. In which of the following compounds, nitrogen exhibits
(c) 3,5, 3,6 (d) 5, 6/5, 5 highest oxidation state ?
(AFMC 2009) (a) NH, (b) NH,
B4. Number of moles of MnO, required to oxidise one (c) N,H (72) NH,OH (ALP.M.T. 2012)
mole of ferrous oxalate completely in acidic medium B11. A mixture of potassium chlorate, oxalic acid and
will be :
sulphuric acid is heated. During the reaction which
(a) 0.6 moles (b) 0.4 moles element undergoes maximum change in the oxidation
(c) 7.5 moles (d) 0.2 moles number?
(CBSE Med. 2008) (a) S (6) H
B5. On the basis of following E° values, the strongest
(c) Cl (d) C (A..P.M.T. 2012)
oxidising agent is
[Fe(CN),]* —> [Fe(CN),|> + e7 E° =— 0.35 V B12. In acidic medium, H,O, changes Cr0,- toCrO, which
Fe*+ —> Fe* + e- E° =— 0.77V has two (—O—O—) bonds. Oxidation state of Cr in
CrO; is
(a) [Fe(CN),/* (6) Fe*
(c) Fe (d) [Fe(CN),)- (a) +5 (b) +3

Answer
(CBSE Med 2008) (c) +6 (d) -—10 (A.LP.M.T. 2014)

Bl.(a) B2. (c) B3. (a) B4. (a) BS. (c) B6. (d) B7. (2d) BS8. (a) BS. (ec) B10. (©)
Bll. (c) B12. (c)
MODERN'S abc + OF CHEMISTRY-XI

B18. Choose the disproportionation reaction among the (@q) a=1,b=4,¢=6ands=2,y7=6,2=3


following redox reactions. (e) a=1,b=6,c=4and*=6,y=2,2z=3
(a) 3Mg(s) + N,(g) —> Mg.N,(s) (Kerala P_E.T.2004)
(6) P,(s) + 3NaOH(aq) + 3H,OW) —> PH,(g) + B19.Consider the following E° values
3NaH,PO,(aq) E° (Fe?+| Fe*+)=+0.77V:; E° (Sn2+|1Sn) =—0.14 V
(c) CL (g) + 2KI(aq) —> 2KCl(agq) + I,(s) Under standard conditions, the potential for the
(d) Cr,O,(s) + 2Al(s) —> Al,O,(s) + 2Cr(s) reaction:
(e) 2NaH(s)——> 2Na(s) + H,(g) (Kerala P.M.T. 2015) Sn (s) + 2Fe?+ (ag) ——> 2F e**(aq) + Sn?* (aq) is
B14, Which of the following processes does not involve (a) 0.19 V (b) 0.140 V
oxidation of iron?
(c) 1.68 V (d) 0.63 V (A.LE.ELE. 2004)
(a) Formation of Fe(CO),. from Fe
B20. In a cell that utilizes the reaction :
(6) Liberation of H, from steam by iron at high
Zn(s) + 2H* (aq) ——> Zn?*(aq) + H,(g)
temperature.
addition of H,SO, to cathode compartment will
(c) Rusting of iron sheets.
(a) increase the E and shift equilibrium to the right
(d) Decolourisation of blue CuSO, solution by iron
(A.LL.P.M.T, 2015) (b) lower the E and shift equilibrium to the right
B15. Assuming complete ionization, same moles of which of (c) lower the E and shift equilibrium to the left
the following compounds will require the least amount (d) increase the E and shift equilibrium to the left
of acidified KMnO, for complete oxidation?
(ALEELE, 2004)
(a) FeC,0, (b) Fe(NQ,), B21. a K,Cr,0., + 6 KCl+ c H,SO, —>
(ec) FeSO, (d) FeSO, (A.LP.M.T. 2015) x CrO,Cl, + yKHSO,
+ zH,O
B16. For the redox reaction The above equation balances when
MnO; + C,07 + Ht > Mn*t + CO, +H,O (a) a=2,b=4,c=6andx=2,y=6,2z=8
(6) a=4,b=2,c=6andx=6,y=2,2=3
The correct coefficients of the reactants for the balanced
(c) a=6,b=4,c=2andx=6,y =3,2=2
equation are
(7) a=1b=4,c=6andx=2,y=6,2z=8
MnO; C,0% H+ () a=1,6=6,c=4andx=6,y=2,z=3
(a) 16 5 2 (Kerala P.E.T: 2004)
(b) 2 5 16 B22.What products are expected from the dispro-
(c) 2 16 5 portionation reaction of hypochlorous acid ?
(dq) 5 16 2 (a) HClO and HClO,
(NEET 2018) (6) HCl and Cl,O
(c) HCl and HClO,
JEE (Mains) & Other State Boards’
E gin eering Entrance (d) HClO, and Cl,0 (A.LE.E_LE. 2006)
B28.What is the oxidation number of carbonyl carbon in
B17. When KMn0O, acts as an oxidising agent and ultimately
methanal ?
forms MnO,?, MnO,, Mn,O. and Mn?*, then thenumber
of electrons transferred in each case is (a) +3 (b) +2
(c) +4 (d) 0 (Gujarat C_E.T. 2007)
(a) 4,3,1,5 (bh. 1, 5) 3, 7
B24.The standard oxidation potentials of Zn, Cu, Ag and Ni
(c) 1,3,4,5 (d) 3,5,7,1 (A.LEELE.2002) electrodes are +0.76, —0.34, —0.80 and +0.25 V
B18. aK,Cr,O, + bKCI + cH,SO,—> respectively. Which of the following reaction will
xCrO,Cl, + yKHSO, + 3H,O provide maximum voltage ?
The above equation balances when (a) Cu + 2Agt(ag) —> Cu**(aq) + 2Ag
(a) a=2,b6=4,c=G6andx=2,y=6,2z=3 (b) Zn + 2Agt(ag) —> Zn?*(aq) + 2Ag
(®) @=4,b5=2, c=fand+=6,7=2,2=3 (c) Hi + Ni**(aqg) —> 2H*(aq) + Ni
(ce) @=6,b=4,c=2andx=6,y=3,z=2 (d) Zn + Cu2*(aqg) —> Zn?*(aq) + Cu

Arwen
(e) Zn + 2H*(aqg) —> Zn?*(aq) + H,
(Kerala CLE.T. 2007)

B13. (5) B14. (a) B15. (c) B16. (6) B17. (c) B18. (d) B19. (a) B20. (a) B21. (d) B22. (c)
REDOX REACTIONS

B25.Oxidation number of Mn atom in KMnO, is B33.The oxidation number of Fe in brown ring


(a) 6 (b) 7 [Fe(H,O).NO}?* is
(a) O (b) +41
(c) 14 (d) 5 (B.H.U. 2007)
(c) +2 (d) +3 (Orissa JEE 2008)
B26.When sulphur dioxide is passed in an acidified K,Cr,O,
B34.Amount of oxalic acid present in a solution can be
solution, the oxidation state of sulphur is changed
determined by its titration with KMnO, solution in
from
the presence of H,SO,. The titration gives
(a) +4 to +6 (b) +6 to +4 unsatisfactory results when carried out in the presence
(c) +4 to 0 (dq) +4 to +2 of HCl because HCl
(Karnataka C.E.T. 2008) (a) oxidises oxalic acid to carbon dioxide and water
B27.Hydrogen gas is not liberated when the following metal (b) gets oxidised by oxalic acid to chlorine
is added to dil. HCl (c) furnishes H* ions in addition to those from oxalic
(a) Mg (b) Sn acid.
(c) Ag (d) Zn (Karnataka C.E.T: 2008) (d) reduces permanganate to Mn2+ (AIEEE 2008)
B28. Which one of the following reactions involves B35.The oxidation number of sulphur atoms in
disproportionation? peroxomonosulphuric acid (H,SO,) and peroxodi-
(a) 2H,SO, + Cu—— CuSO, + 2H,O + SO, sulphuric acid (H,5,0,) are respectively
(b) As,O, + 3H,S— As,S, + 3H,O (a) +8 and +7 (b) +3 and +3
(c) 2KOH + Cl, —> KCl + KOC] + H,O (c) +6 and +6 (d) +4 and +4 (J & K 2009)
B36.Oxidation states of Fe in Mohr's salt is
(d) Ca,P, + 6H,0 —-> 3Ca (OH), + 2PH,
(a) +1 (b) +2
(e) 4NH,+30,—> 2N,+6H,O (Kerala PET 2008)
(c) +3 (d) +4 (Orissa-JEE 2009)
B29.In which of the following the oxidation number of oxygen
B37.Oxidation state of S in §,0,?- from left to right are
has been arranged in increasing order?
(a) OF, < KO, < BaQ, < O, O O
(6b) BaO, < KO, < OQ, < OF,
-o_4_s_s_4_o-
(c) BaO, < O, < OF, < KO,
(d) KO, < OF, < O, < BaO,
( (
(e) OF, < O, < KO, < BaO, (Kerala PET 2008) (a) +6, 0, 0, +6 (b) +5, 0, 0, +5
B30.Electrode potential of hydrogen electrode is ... volt (c)} +3, +1, +1, +3 (d) +4, +1, +1, +4
(a) O (6b) +1 (Orissa-JEE 2009)
(c) -1 (d) None of these B38.Oxidation number of P in PO,*, S in SO,?- and that of
Cr in Cr,O,* are respectively
(Orissa JEE 2008)
(a) +3,4+46and+5 (6) —3, +6 and +6
B31.The reaction
(c) +5,4+3 and+6 (d) +5, +6 and +6
P, + 3 NaOH + 3H,O —~ 3NaH,PO, + PH,
(Orissa-JEE 2009)
is an example of
B39. The oxidation state of S in H,SOQ, is
(a) Disproportionation reaction (a) +2 (b) +4
(6) Neutralisation reaction (c) +6 (d) +7 (Orissa JEE 2010)
(c) Double decomposition reaction B40. In chromite ore, the oxidation number of iron, and
(dq) Pyrolytic reaction (Orissa JEE 2008) chromium are respectively
(a) +3,4+2 (6b) +3,4+6
oO O (c) +2,+6 (d) +2,4+3
B32.Oxidation state of chromium in p< is (Karnataka CET 2010)

> B41. Which of the following species can function both as


oxidizing as well as reducing agent ?
(a) +10 (6) +6 (a) Cl (b) ClO;
(c) +3 (d) +2 (Orissa JEE 2008) (ec) C1lO- (d) MnO;
(e) NO; (Kerala PET 2010)

B25. (6) B26. (a) B27. (c) B28. (c) B29. (5) B30. (a) B31. (a) B32. (6) B33. (6) B34. (d)
B35. (c) B36. (6) B37. (c) B88. (d) B39. (c) B40. (d) B41. (ce)
MODERN'S abc + OF CHEMISTRY-XI

B42. If the molecular wt. of Na,S,0, and I, are M, and M, The cellvoltage will be
respectively, then what will be the equivalent wt. of (a) 2.71 V (6b) 2.038 V
Na,5,0, and I, in the following reaction ?
(c) —2.71 V (d) —2.03 V
2S,02- +1,—> 8,077 + 21 (JK. C.E.T. 2013)
B47. Given
(b) M,,M,/2
E°(Cr3+|Cr) =— 0.74 V, E%(MnOj!Mn2+) = 1.51 V
(a) M,,M,

(c) 2M,,M, (d) M,,2M, E°Cr2o7 |Cr3+ = 1,383. VjeE® CLICI =1.36 V
(WB JEE 2011) Based on the data given above, strongest oxidising
B43.In the reaction : H,S + H,O, —> S + 2H,O agent will be
(a) Cl (6) Cr*
(a) H,S is an acid and H,O, is a base. (c) Mn* (qd) MnO Fs
(b) H,5 is a base and H,O, is an acid. (JHE. Main 2013)
(c) H,S is an oxidising agent and HO, is a reducing B48. Consider the following reaction :
agent
(d) H,S is a reducing agent and H,O, is oxidising x MnO, + C,O} + zH*—> x Mn + 2y CO, + 5 H,O
agent.
The values of x, y and z in the reaction are
(e) H,S is hydrolysed to 8. respectively
(Kerala PET 2011) (a) 5,2 and 16 (6) 2,5 and 8
(c) 2,5 and 16 (d) 5,2 and8
B44, 1M solution each of Cu(NOQO,),, AgNO,, Hg,(NO,),
and Mg(NO,), is electrolysed using Pt-electrodes. (JEE. Main 2013)
The values of standard electrode potentials in B49. In the redox reaction,
volts are :
_ _ ‘5
MnO, +8H*+5Br —> Mn?* + 4H,O + 3 Br,
Agt| Ag = + 0.80 V, Cu*!1Cu
= + 0.34V
Hg,” |Hg = + 0.79 V, Mg** |Mg =— 2.37 V Which one is the reducing agent?
The sequence of deposition of metals on the (a) Ht (6) MnO, —
cathode will be (c) Bro (d) Mn?+
(a) Mg, Ag, Cu (b) Mg, Cu, Ag (A.M.U. Engg. 2014)

(c) Ag, Hg, Cu (d) Cu, Hg, Ag B50. Potassium dichromate is a good oxidizing agent, in
acidic medium. The oxidation state of chromium
(A.M.U. Engg. 2012)
changes by
B45. In the reaction, (a) 2 (b) 3
2FeSO, + H,SO, + H,O, — > Fe,(S0,), + 2H,O, (c) 4 (d) 5
the oxidising agent is (M.H-C_E.T. 2015)
(a) FeSO, B51. One mole of hydrazine (N,H,) loses 10 moles of
electrons in a reaction to form a new compound X.
(b) H,SO, Assuming that all the nitrogen atoms in hydrazine
(c) H,O, appear in the new compound, what is the oxidation
(d) both H,SO, and H,O, state of nitrogen in X?
(Karnataka C.E.T. 2013) (Note: There is no change in the oxidation state of
B46. An electrochemical cell has two half cell reactions hydrogen in the reaction).
as: (a) =—1 (6) =—3
A** + 2e—> A; B®, = 0.34 V (c) +3 (d) +5
(e) +1 (Kerala PET 2016)
X — > X** + 2e° ; BX, =-— 2.37 V

Answer
B42. (b) B43. (d) B44, (c) B45. (c) B46. (a) B47. (d) B48. (c) B49. (c) B50. (5) B51. (ec)
REDOX REACTIONS

B52. 3C1O™ (ag) —> ClO; + 2CI is an example of titration. Some half reactions and their standard
(a) oxidation reaction potentials are given below :
(6) reduction reaction MnO, (aq) + 8H* (aq) + 5e° ——> Mn?*(aq) + 4H,O (J)
(c) disproportionation reaction E°=1.51V
(d) decomposition reaction (Karnataka CET 2017)
Cr,0,?(aq)+ 14H*(aq) + 62° ——> 2Cr**(aq) + 7H, O10)
B53. How many electrons are involved during the
oxidation reaction of KMnO, in acidic medium? E°=1.38 V
(a) 1 (b) 3 Fe**(aq) + e-——> Fe** (aq) E° =0.77V
ic) 6 (d) 4 Cl,(g) + 2e°- ——> 2CP (aq) K° = 1.40 V
(J.K. CET 2018)
Identify the only incorrect statement regarding the
B54. For the redox reaction:
quantitative estumation of aqueous Fe(NQ,),.
xMnO; + yH,C,0, +zH* — mMn* + nCO, +pH,0.
The values of x, vy, m and n are (a) MnO, can be used in aqueous HCl
(a) 10,2,5,2 (6b) 2,5, 2, 10 (b) Cr,0,*> can be used in aqueous HCl
(c) 6,4, 2,4 (d) 3, 5,2, 10 (c) MnO, can be used in aqueous H,SO,
(Karnataka CET 2018)
(d) Cr,0,* can be used in aqueous H,SO,
B55. Iodine reacts with concentrated HNO, to yield Y along
with other products. The oxidation state of iodine in Y, is
(1.1.T. 2002)
(a) 7 (6b) 1 B58. The pairs of compounds having metals in their
(c) 5 (dq) 3 (JEE Main 2019) highest oxidation state is
(a) MnO,, FeCl,
JEE (Advance) for ITT Entrance
(6) MnO,, CrO,Cl,
B56. For the electrochemical cell M| Mt || X-! X,
(c) [Fe(CN),|*-, Co(CN),
ESs+) = 0.44 V, EY,y- = 0.33 V
(d) [NiCl,]?-, [CoC],
(a) M+X-s M* +X is the spontaneous reaction
(L1.T. 2004)
(6) Mt +X-——> M + Xis the spontaneous reaction
B59. Which ordering of compounds is according to the
(ec) Ein = 0.77V. decreasing order of the oxidation state of nitrogen ?
(dq) E°n = —0.77V. (a) HNO,, NO, NH,CLN,
(LLT. 2000) (6) HNO,, NO, N,, NH,Cl
B57. Standard electrode potential data are used for (c) HNO,, NH,Cl, NO,N,
understanding the stability of an oxidant in a redox (d) NO, HNO,, NH,Cl, N, (107. JE. 2012)

Arisvets
B52. (c) B53. (c) B54, (6) B55. (c) B56. (b) B57. (a) B58. (b) B59. (b)

C2. In which of the following the oxidation number of atom


with more than one correct answers
is/are correctly given ?

C1. Which of the following are redox reactions ? (a) C,H,,0, : C=0
(a) BaCl, + H,SO,—> BaSO, + 2HCl (b) Na,[Fe(CN),.]: Fe =+3

(6) Zn + 2HCl —-> ZnCl, + H, (c) HCOOH: C=4+4


(c) 6CO, +6H,O—- C,H,,0, + 60, (dq) HCHO:C=0

(d) KCN + AgCN ——> KlAgi(CN),]

Aeswecs
C1. (b,c) C2. (a, d)

WWW.JEEBOOKS.IN
MODERN'S abc + OF CHEMISTRY-XI

C3. Which of the following act both as an oxidising as well C7. Which of the following statements are wrong ?
as reducing agent ? (a) Reduction involves gain of electrons.
(a) HNO, (6) H,O, (c) HNO, (d)H,S (6) The oxidation number of reducing agent decreases.
(c) Oxidising agent helps to increase the oxidation
C4, In which of the following the oxidation number of the number of reducing agent.
underlined atom is maximum ?
(d) Oxidation involves gain of electrons.
(a) H,P,0, (6) KAMSO,),.12H,O C8. Consider the redox reaction :
(c) K,MnO, (dq) Na,SO, 28,07- +1,—> S,0F + 2h
C5. Oxidation number of Cr in CrO, is same as of 5 in (a) 2S,0* gets oxidised to S,OF
(6) I, gets oxidised to I-
(a) H,SO, (6) Na,S,O, (c) Na,S,0, (d) H,SO,
(c) there is increase in oxidation number of iodine
C6. Which of the following statements are not correct for during the reaction
the following reaction :
(d) The total increase in oxidation number of sulphur
2MnO, +61 + 4H,O —> 2Mn0O, + 31, + 80H- is +1 during the reaction
(a) Oxidation number of Mn in MnO, and MnO, are C9. For the reaction:
same I~ + C10," + H,50,
—> Cl + HS0O;, + I,
(6) MnO, acts as an oxidising agent The correct statement(s) in the balanced equation is/are
(c) H,O has been reduced (a) stoichiometric coefficient of HSO, is 6
(d@) Oxidation number of iodide has increased from I- (b) iodide is oxidised
to I,.
(c) sulphur is reduced

Answers (d) H,Ois one of the products. (JHE advance 2014)

C3. (b,c) C4. (b,c) C5. (a,c) C6. (b, d) C7. (b, d) C8. (a, d) C9. (a, b, d)

(6) Bromine is neither oxidised nor reduced


(c) Bromine is oxidised and water is reduced
ion
based on the given passage/comprehens (dq) Bromine is both reduced and oxidised
D3. When copper is treated with concentrated nitric acid,
Passage I.
nitric oxide and nitrogen dioxide are produced in equal
Oxidation and reduction reactions ocurring simultaneously volumes according to the following reaction :
are called redox reactions. Oxidation involves loss of xCu + yYHNO, —> Cu(NQ,), + NO + NO, + H,O
electrons while reduction involves gain of electrons. As a
result, oxidation number of an atom increases during The coefficients x and y are
oxidation and decreases during reduction. The redox reaction (a) land3 (6) 2 and 3
may involve combination of atoms or molecules, (c) 2 and 6 (dq) 8 and 8
displacement of metals or non-metals and disproportiona- p4, A compound contains atoms of three elements A, B
tion reactions. Redox reactions may be balanced by oxidation
and C. If the oxidation number of Ais +2, B is +5 and
number method or ion-elecron method.
that of C is —2, then the possible formula of the
Answer the following questions : compound is

D1. Which of the following represents oxidation ? (a) A,(BC,), (b) Aq(B,C),
(a) [Fe(CN),- +e — [Fe(CN),]* (c) ABC, (d) A,(BC,).(CBSE PM.T.2000)
(6) S+2e-—> §* D5. In which of the following the change of oxidation
states is not correctly represented ?
(ce) 2Clr — 2e° —> Cl,
(d) Sb®+ + 2e——> Sb3+
(a) KMnO, + 5e° —> Mn”
D2. In the reaction : (6) KMnO, + 2e—> =Mn,0,
3Br, + 6CO> + 3H,0 — 5Br + BrO, + 6HCO,
(c) KMnO, + 3e°—> MnO,
(a) Bromine is oxidised and carbonate is reduced
(d) KMnO, + e-—> Mn0O?-
Aaswecs
Passage I. D1. (c) D2. (d) D3. (d) D4, (d) D5. (d)

WWW.JEEBOOKS.IN
REDOX REACTIONS 8/74 >=

Passage II. (a) O, oxidises Mn?* to Mn*+


(b) O, oxidises both Mn?* to Mn** and Fe* to Fe*+
Redox reactions play pivotal role in chemistry and biology.
(c) Fe** oxidises H,O to O,
The values of standard redox potential (E”) of two half-cell
reactions decide which way the reaction is expected to (d) Mn** oxidises H,O to O,
proceed, A simple example is Daniel cell in which zinc goes Ds. Sodium fusion extract, obtained from aniline, on
into solution and copper gets deposited. Given below area treatment with iron (Il) sulphate and H,SOQO, in the
presence of air gives a prussian blue precipitate.
set of half-cell reactions (acidic medium) along with their
Hence, the blue colour is due to the formation of
E° (V with respect to normal hydrogen electrode) values.
: ; (a) Fe,[Fe(CN)gl, (6) Fe,[Fe(CN),l,
Answer the following questions : (c) Fe,[Fe(CN)gl, (d) Fe,[Fe(CN),l,
I,+2e-° ——> a E° = 0.54 (LL.T. 2007)
Passage III.
Cl,+2e— Mm—> 2Cl~ E° = 1.36
oe Bleaching powder and bleach solution are produced on a
Mn?++e> ——> Mn+ E° = 1.50 large scale and used in several household products. The
effectiveness of bleach solution is often measured iodometry.
Fe?++e- —— Fe E° = 0.77
. Answer the following questions :
O, + 4H" + 4e > 2H,O Ee = 1.23 D9. Bleaching powder contains a salt of an oxoacid as one
Using these data, obtain the correct explanation for the of its components. The anhydride of that oxoacid is
following questions. (a) ClO (6) ClO, (ec) ClO, (d) Cl,O,
D6. Among the following, identify the correct statement. D10. 25 mL of household bleach solution was mixed with
(a) Chloride ion is oxidised by O, 30 mL of 0.50 M KI and 10 mL of 4 N acetic acid. In
(b) Fe2+ is oxidised by iodide | the titration of the liberated iodine, 48 mL of 0.25 N
Na,5,0, was used to reach the end point. The molarity
(c) lodide ion is oxidised by chlorine of the household bleach solution is
(d) Mn? is oxidised by chlorine (a) 0.48 M (b) 0.96 M
D7. While Fe** is stable, Mn** is not stable in acid solution (c) 0.24 M (d) 0.024 M (LLT 2012)
because ) oe —_

Passage Il. D6. (c) D%. (d) D8. (a) Passage III. D9. (a) D10. (c)

2. Assertion : Oxidation number of C in HCHO is zero.


Assertion Reason Type Questions Reason — : Formaldehyde is a covalent compound.
The questions given below consist of an Assertion 3. Assertion : Oxygen has oxidation state of —2 in both
O, and Ox.
and the Reason. Use the following key to choose the
Reason =: Oxygen is assigned an oxidation state of
appropriate answer.
— 2 in almost all its compounds.
(a) If both assertion and reason are CORRECT and 4 Apcatinn : Oxidation number of phosphorus in P, is
reasson is the corect explanation of the assertion. zero.
(5) If both assertion and reason are CORRET, but reason Reason —: Phosphorus has oxidation state zero in
is NOT THE CORRECT explanation of the assertion. all its compounds.
(c) Ifassertion is CORRECT, but reason is INCORRECT. EF Assertion : Redox reactions are aleo called
(d) If assertion is INCORRECT, but reason is CORRECT. neutralisation reactions.
(e) If both assertion and reason are INCORRECT. Reason =: The number of electrons gained or lost in
1. Assertion : Oxidation state of hydrogen in H,O is +1 the reaction are balanced.
and in CaH, it is —1. 6. Assertion : 3ClO°-—— ClO,” + 2CIl is an example of
Reason’: CaH, is metal hydride and for hybrides, dissociation reaction.
= ae is assigned the oxidation state Reason =: ClO~ gets oxidised as well as reduced.

Arswecs
1. (a) 2. (6) 3. (d) 4. (c) 5. (d) 6. (d)

WWW.JEEBOOKS.IN
MODERN'S abc + OF CHEMISTRY-XI

7. Assertion : A substance which gets reduced can act as Reason : Oxidation potential of zinc anode
reducing agent. decreases and that of copper cathode
Reason — : An oxidising agent itself gets oxidised. increases. (AHMS 2013)
8. Assertion : Copper sulphate solution is not stored in 10. Assertion : In iodometric titrations, starch is used as
zinc vessel. an indicator.
Reason — : Zinc forms complex with copper sulphate. Reason — : Starch is a polysaccharide.
9. Assertion : The Daniell cell becomes dead after
sometime.

Arasvets
7. (e) 8. (c) 9. (a) 10. (b)

Integer Type Questions . The oxidation number of Sin H,SO,; is +x. The value of
x 18

Integer Type : The answer to each of the following . The number of compounds among the following having
question is a single-digit-integer ranging from O.N, of S as +6 is
0 to 9.
1. The difference in oxidation number of two types of 5,07 Na,8,0., H,S0,, HSO;, (NH,), SO,, H,S0,,
sulphur atoms in Na,S,O, is HSO3, 5,0,"
(L1.T. 2011) - Reaction of Br, with Na,CO, in aqueous solution gives
sodium bromide and sodium bromate with the
2. How many moles of ‘iodine’ are liberated when 2 moles
evolution of CO, gas. The number of sodium bromide
of KMnO, react with KI in acidic medium?
molecules involved in the balanced chemical equation
is (1.1! 2011)
3. The value of n in the molecular formula Be, Al, Sig O., is
U.I.T. 2010)

Answers
1. (5) 2. (5) 3. (3) 4, (6) 5. (4) 6. (5)

NCERT Exemplar Problems TT |


»>» Multiple Choice Questions (Type-l) <@
Objective Questions
1. Which of the following is not an example of redox below find out which of the following is the strongest
reaction ? oxidising agent.

(a) CuO + H, —> Cu + H,O E° values : Fe**| Fe** = +0.77: I,(s) | =+ 0.54,

(b) Fe,O, + 3CO —> 2Fe + 3CQ, Cu** |Cu = +0.34; Ag* |Ag = + 0.80V
(a) Fe** (b) L{s)
(c) 2K +F,— 2KF
(ec) Cu?* (d) Ag*
(d) BaCl, + H,SO, —> BaSO, + 2HCI1 3. E*° values of some redox couples are given below. On
2. The more positive the value of E°, the greater is the
the basis of these values choose the correct option.
tendency of the species to get reduced. Using the E° values : Br, |Br” = + 1.09; Ag* |Ag(s) = + 0.80
standard electrode potential of redox couples given Cu** |Cu(s) = + 0.34; I,(s) |IT = + 0.54
(a) Cuwillreduce Br” (5) Cu will reduceAg

Answers
(ec) Cuwillreducel” (d) Cu will reduce Br,

1. (d) 2. (d) 3. (d)


REDOX REACTIONS

4. Using the standard electrode potential, find out the (c) An element in the free or the uncombined state
pair between which redox reaction is not feasible. bears oxidation number zero.
E° values : Fe** |Fe** = + 0.77; I, | =+ 0.54; (d) In all its compounds, the oxidation number of
Cu** |Cu = + 0.34; Ag*| Ag = + 0.80 V. fluorine is —1.
(a) Fe** andI- (b) Ag* and Cu In which of the following compounds, an element
exhibits two different oxidation states ?
(c) Fe®+ and Cu (d) Ag and Fe**
(a) NH,OH (b) NH,NO,
Thiosulphate reacts differently with iodine and
te) NH, (d) NH
bromine in the reactions given below :
Which of the following arrangements represent
28,02 +1, —> 8,07- +2
increasing oxidation number of the central atom ?
5,02- + 2Br, + 5H,0 —> 2507 + Br + 10H*
(a) CrO3, ClO3 ,Cr027, MnO;
Which of the following statements justifies the above
dual behaviour of thiosulphate ? (b) ClOzZ, CrO* ,MnO7z, CrO3Z
(a) Bromine is a stronger oxidant than iodine. (c) CrO3Z, ClOZ ,MnOz, CrO=-
(6) Bromine is a weaker oxidant than iodine. (d) CrO#7, MnO7, CrO35, C103
(c) Thiosulphate undergoes oxidation by bromine and The largest oxidation number exhibited by an element
reduction by iodine in these reactions. depends on its outer electronic configuration. With
which of the following outer electronic configurations
(qd) Bromine undergoes oxidation and iodine undergoes
the element will exhibit largest oxidation number ?
reduction in these reactions.
(a) 8d'4s? (b) 3d%4s* (c) 3d°4s! (d) 3d°4s?
The oxidation number of an element in a compound is
10. Identify disproportionation reaction
evaluated on the basis of certain rules. Which of the
(a) CH, +20,— > CO, + 2H,O
following rules is not correct in this respect ?
(b) CH, + 4Cl, ——> CCL, + 4HCl
(a) The oxidation number of hydrogen is always +1.
(c) 2F,+20H —> 2F + OF, +H,O
(6) The algebraic sum of all the oxidation numbers (d) 2NO, + 20H —> NOs; + NO3 +H,O
in a compound is zero.
11. Which of the following elements does not show
disproportionation tendency ?
Answers (a) Cl (b) Br (c) F (d) I

5. (a) 6. (a) 7. (b) 8. (a) 9. (d) 10. (d) 11. (e)

»>
configuration of its atom. Atom(s) having which of the
Multiple Choice Questions (Type-tIl) following outermost electronic configurations will
12. Which ofthe following statement (s)is/are not true about exhibit more than one oxidation state in its compounds.
the following decomposition reaction. (a) 3s' (b) 3d'4s"
2KCIO, —> 2KCl + 30, (c) 3d74s” (d) 3s"3p*
(a) Potassium is undergoing oxidation 15. Identify the correct statements with reference to the
(6) Chlorine is undergoing oxidation given reaction
(c) Oxygen is reduced P,+30H + 3H,O —> PH, + 3H»_PO5
(d) None of the species are undergoing oxidation or (a) Phosphorus is undergoing reduction only.
reduction (65) Phosphorus is undergoing oxidation only.
13. Identify the correct statement (s) in reflection to the (c) Phosphorus is undergoing oxidation as well as
following reaction : reduction.
(qd) Hydrogen is undergoing neither oxidation nor
Zn + 2HCl —> ZnCl, + H,
reduction.
(a) Zine is acting as an oxidant 16. Which of the following electrodes will act as anodes,
(6) Chlorine is acting as a reductant when connected to Standard Hydrogen Electrode ?
(c) Hydrogen ion is acting as an oxidant
(a) Al/Al*+ E° =—1.66V
(d) Zinc is acting as a reductant
(b) Fe/Fe2* E°=-0.44V
14, The exhibition of various oxidation states by an
element is also related to the outer orbital electronic (ce) Cu/Cu2* R°=+0.34V
(d) F,(g/2F- (aq) E° =+2.87V
Aswets
12, (a, 5, ¢, d) 13. (c,d) 14, (b,c, d) 15. (c, d) 16. (a, b)

WWW.JEEBOOKS.IN
MODERN'S abc + OF CHEMISTRY
-XI

n Fue
Q. 18. Match the items in Column I with relevant
Matching Type Questions
items in Column II.
Q. 17. Match Column I with Column II for the Column I Column IT
oxidation states of the central atoms.
(i) Ions having positive charge (a) +7
Column I Column IT (71) The sum of oxidation number of (6) —1
all atoms in a neutral! molecule
i Cn07
(iii) Oxidation number of hydrogen (c) +1
(ii) MnOjz (b) +4 ion (H*)
(iii) VOR (c) +5 (iv) Oxidation number of fluorine in (d) 0
(iv) FeFs7 (d) +6 NaF
(e) +7 (v) Ions having negative charge (e) Cation
(f) Anion

Aaswiets
17. (%) — (a); (tz) — (e); (tat) — (e); Gv) — (@) 18. (i) —(e); (i) — (d); (iii) — (e); (iv) — (b), (vr) —P)

>» Assertion and Reason Type Questions <q 20. Assertion (A) : In the reaction between potassium
permanganate and potassium iodide, permanganate
In the following questions a statement of ions act as oxidising agent.
assertion (A) followed by a statement of reason Reason (R) : Oxidation state of manganese changes
(R) is given. Choose the correct option out of from +2 to +7 during the reaction.
the choices given below each question.
21. Assertion (A) : The decomposition of hydrogen
(a) Both A and R are true and R is the correct peroxide to form water and oxygen is an example of
explanation of A. disproportionation reaction.
(6) Both A and R are true but R is not the correct
Reason (R) : The oxygen of peroxide is in —1 oxidation
explanation of A.
state and it is converted to zero oxidation state in O,
(c) Ais true but R is false.
and —2 oxidation state in H,O.
(d) Both Aand R are false.
22. Assertion (A) : Redox couple is the combination of
19. Assertion (A) : Among halogens fluorine is the best oxidised and reduced form of a substance involved in
oxidant.
an oxidation or reduction half cell.
Reason (R) : Fluorine is the most electronegative
atom. Reason (R) : In the representation Bie /Fe2 and
E 0
, Fe**/Fe?* and Cu?*/Cu are redox couples.
Cu**/Cu

Aaswets
19, (5) 20. (c) 21. (a) 22. (b)

: a tions for
Hints & Explan Difficult Objective Type Questions

A. meq with only one correct answer A6. (6): H,SO, has one peroxide bond as

AZ. (c) : x -—2x4=-3 x= 45 O


I]
AS. (d):
x —8=-1 “x=47 ZoO-O0-H
Ad, (a): CH,OH: x+3—-241 “x= 2 OQ OH
CH,0O: x+2-2 x= So, it may be written as
HCOOH: +14+x-4+4+1 “xX=42 H,S(O,)0.
C,H, : 2x + 2 x=-l 42 4x—2 4 3(-2) =0 2 £=46
A5. (c) : Na,S, +2+4+2x =0 “x=-l A%. (a): Sodium exists in elemental form.
REDOX REACTIONS

A&8. (d): Cl, + 2 OH-—> OCI + Cl +H,0


B. meq from competitive examinations
O.N. of Cl changes from 0 to +1 (in OCI-) and —1 in
(Cl). BS. (a): IO, + 5 + 6H* ——> 3H,0 + 3I,
AS. (c) : S,0,7" has structure B4, (a): 5FeC,O, + 3MnO, + 24 H* ——> 5Fe* + 10CO,
O O + 3Mn** + 12H,O
|
ct oe i
|
Oo—o—S—O
5 mol of FeC,0, = 3 mol of MnO,-
3
. 1 mol of FeC,0, = 5 mol of MnO,”
= 0.6 mol of MnO,~
2 O atoms have peroxide linkage so it may be B5. (c): Higher the reduction potential, larger is its
written as §,(0,) O,?- tendency to get reduced and hence stronger is the
. 2x —2 + 6(-2) =-2 . x=+4+6 reducing agent.
+8 od +2 =2
Al10. (c) : In CH,CHO, 2x +4-2=0 x=] B7. (d): H,50,>H,50,>SCl,>H,85
All. (d): V(+2), Br +3 in (BrO,) so it exists as V" (Br'O,), BS. (c): As the electrode potential decreases, reducing
Al12. : HN, 1+3x=0 “x=-L/3 power decreases.
Al3. (d): a (+2.5), b (+1), ¢ (0), d (+7) Z>X>Y
B10. (c): NoHy => 2x/+ 4 (+ 1) or 2n+4=0 -.x%=-2
+3 0
Al15. (a): CuO + 2NH, —— Ng + 3H,O + Cu NH, = x/+ 3 (41) =Oorx+3=0
..x=-3
N3H => 3x + 1 (+1) =O0o0r 3x +1=0 x =-1/3
—Ge- NH,OH = x +2+(2)+1=0o0rx+1=0
Loss of 6e~ for 1 mol of N, a x =-1
Al6. (d): In oxidation, there is increase in O.N. Thus, highest oxidation state is —1/3.
Al7. (c): O.N. of P changes from zero to —3 in PH, and +1 B11. (c):
in NaH,PO,. Therefore, it undergoes dispropor- +1+5-2 COOH +6 +6 -1
KClO3+ | + Hy 504 —_ Ky 504+ KCl+ CO, + H,O
tionation.
COOH
$2497
=0 1x =-1 Maximum change in oxidation number of chlorine
(ce): Ba(H,PO,), Le., from + 5 to — 1.
+2 +2(4+2 +x—4)=0 B18. (5) : P, + 3NaOH + 3H,0 —> pq, + 3NaH, PO,
42444 297-—8=0 ."x=+4l1
In this reaction, P is oxidised to NaH,PO, and
. (d): The O.N. of bromine decreases from 0 to —1 (in Br~) reduced to PH,. This is disproportionation
and increases from 0 to +5 (in BrO,),
reaction,
. (d) : The calculated O.N. of Br in Br,O, is
B14, (a): Oxidation of iron in Fe(CO), is zero.
ax + 8-2) =O0orx = is There is no change in oxidation number of Fe in
Actual O.N. of different Br atoms is Fe + 5CO —> Fe(CO),
Hence, it does not involve oxidation of iron.
O
S46 I. Sa B16. (5): The correct balanced equation is
sae a — Br— Br=
2MnO; +5C,0?" + 16H*—>
o* I N6
2Mn** + 10CO, + 8H,O
A26. (b): The O.N. of Br increases from 0 to +1 in (HOBr) +7 +6
and decreases from 0 to —1 (in HBr). Therefore it B17. (c) : KMnO,+e —> MnO, (le)
is oxidised as well as reduced.
+7 +4
A28., (a): O.N. are SO,?-(+4), S,0,2°(+3), S,0,-7(+5) KMnO,+3e — > MnO, (de)
.. Correct order
+7 +3
8,0,7 < so,” < S,0,7
KMn0O, + 4e7 Mn,0,(4e)
AS1, (a): The balanced equation is
+7 +2
4 Zn + NO, + 7H,0 — 4Z7n** + NH,* + 10 OH” KMnO,+5e- —> Mn**(5e)
A3S5. (a): The balanced equation is B18. (d): The balanced equation is :
MnO; + 5Fe?* + 8H*——> Mn** + 5Fe*+ + 4H,O K,Cr,0, + 4KCl + 6H,50,—> 2CrO,Cl, +
A37. (a): Since the reduction potential of Y is greater than 6KHSO, + 3H,O
that of X but lower than that of Z. Therefore, Y
B19. (a): E° cell
_,, ~= E° (Fe*+ |Fe2+) —E° (Sn2* |Sn)
will oxidize X but not Z.
= (0.77 — (-0.14) = +0.91 V
MODERN'S abc + OF CHEMISTRY-XI

B20. (a): For the reaction : B38. (6): Here NO is in +1 oxidation state so that
Zn (s) + 2H* (aq) —— Zn** (aq) + H, (g) x+5(0)+1=42
x=+1
E-F°- RT |Ont" | Pas } B34, (d): KMnO, can oxidise HCl also into Cl, and itself
nF ra gets reduced to Mn?*.
B40. (d): Chromite ore is FeCr,O.,.
On adding H,SO,, the [H*] increases. This will
increase the value of E and, therefore, reaction Change in O.N. per mol
proceeds more to the right. =]lx2=2

B21. (d): K,Cr,0, + 2H,SO, —> 2KHSO, + 2CrO, + H,O +i 2 0 4+2.5 =]

2KCl + H,SO, —> 2KHSO, + 2HCI ] x 2


CrO, + 2HCl —> Cr0,Cl, + H,O | x 2 Change in O.N. per mol
K,Cr,0, + 4KCl + 6H,SO, —> 2CrO,Cl, + Sao x 2a}

6KHSO, + 3H,O M
Equivalent wt. of Na,S,O, = a
B22. (c): HClO disproportionates as :
“Mo
+1 -l1 +5 Equivalent wt. of I, = 5
3HOC!] —> 2HCl + HCIO,
Reduction (Oxidising agent)
8)

B48. (d):H,S + H,O, —> S + 2H,0


— = =1 0 —2
B23. (d) : nH or HCHO

+l+x—-24+1=0
Oxidation (reducing agent)
orx=0
B44, (c) : Metals get deposited after undergoing reduction
B24. (b) :For (a) E° = — 0.34 + 0.80 = 0.46 V on cathode. Higher the value of reduction potential,
(6) E° = 0.76 + 0.80 = 1.56 V more is the tendency to get reduced. Order of E°
(c) B° =— 0.25 V values : Ag > Hg > Cu.
(2d) E° = 0.76 + 0.384 = 1.00 V Hence, sequence of deposition : Ag, Hg, Cu.
(e) E° = 0.76 V B45. (c) : H,O, gets reduced to H,O and acts as an oxidising
agent.
Correct answer (6).
B46. (a) : Boo = Eoasia — EXetix
+4 +6 = 0.34 — (- 2.37) = 2.71 V
B26. (a): K,Cr,0,+H,SO, +35 O, > K,SO, + Cr,(S0,),.+ HO
B47. (d) : MnO,” is the strongest oxidising agent because it
Oxidation number of S changes from +4 to +6. has highest reduction potential value.
B48. (c) : [MnO, + 8H* + 5e —> Mn + 4H,O] x 2
Oxidation
C07 —> 2CO, + 2e-] x 5
0 -1 +1 2MnO; + 5C,0; + 16H* —> 2Mn* + 10CO, + 8H,O
B28. (c): 2KOH + Cl, —> KC] + KOC] + H,O B49. (c) : O.N. of Br has increased from —1 to 0 and it is
|Reduction | oxidised. Hence, it acts as a reducing agent.
B50. (b) : Cr,O,7 + 14H* + 6e> —> Cr*+ + 7H,O
Oxidation Oxidation state of Cr in Cr,O,?- = +6
[
eFC
fe
0 +1 —3 Oxidation state of Cr in Cr*+ = +3
B31. (a) :P, + 3NaOH +3H,O—-> 3NaH,PO, + PH, Change in oxidation state = 3
Reduction
B51. (c) : Oxidation state of N in hydrazine:
In this, the same substance undergoes oxidation N,H,
as well as reduction and therefore, it is an example 2x +4=0o0rx =-2
of disproportionation reaction.

p<
1 mol of N,H, loses 10 mol of electrons. Therefore,
one nitrogen loses 5 electrons, Thus, the oxidation
To Oo state of N in new compound is = -2+5=+4383
B32. (5):
Reduction

+1 +5 —l
B52, (c): sclo-§ ——> ClOZ; + 2CI”
. x= +6 |
_____—#
Oxidation
REDOX REACTIONS

The oxidation state of Cl decreases from +1 to —1 DS. (a) : Na+C+N — NaCN


(reduction) and increases from +1 to +5 (oxidation). Sodium fusion extract
It is a disproportionation reaction. Fe?* +6CN” — > _ [Fe(CN),|*
B53. (c) : MnOz + 8H* + 5e° —> Mn?* + 4H,O In the presence of air, Fe?* ions get oxidised to
B54. (6): The balanced reaction is : Fe** ions. |
2MnO; + 5H,C,0, + 6H* —> Fe** — > Fe*+e— E°=-0.77V
2Mn** + 10CO, + 8H,O
O, + 4H* + 4e— — > 2H,0 E° = -1.23 V
Therefore, x = 2, y=5,m=2,n=10
B55. (c) : Conc. HNO, oxidises [, to iodic acid (HIO,) 4Fe?+ + 4H+ +O, —> 4¥Fe*+ + 2H,O
Oxidation state of iodine in HIO, is +5 Eon = + 0.46 V
B56. (6b): M* +e-———> M E°=+044V a... (Z)
Fe** ions then combine with [Fe(CN),|* ion to form ferric
X+e—> & E° =+0.33 V sectceaulb th)
ferrrocynide which has prussian blue colour.
Subtracting eqn. (72) from eqn. (7)
M* + X- > M+ X E° = +0.11V 4Fe** + 3[Fe(CN)gI* —> Fe, [Fe(CN)gl,
Since E° is positive, the reaction is spontaneous. Prussian blue
B57. (a): MnO,- will oxidise Cl- ion according to the reaction. D9. (a) : The active constituent of bleaching powder or
2KMnO, + 16H* + 16 Cl 2Mn?+ + 8H,O +5 Cl, bleaching solution is Ca(OCl),. The anion of this
The corresponding cell is salt is OCI and therefore, the corresponding acid
Pt!Cl, (g) ICl (aq) Il MnO,(aq), Mn?+, H+ | Pt is HOCI. The anhydride of this acid is Cl,O as
E° 1, = E° (cathode) — E° (anode) 2HOC] —> H,O + Cl,O
=151-140=0.11V D10. (c) : Applying normality equation,
Since E® _), is +ve, the above reaction is feasible.
N,V, — N.Vo
Therefore, MnO, will oxidise both Fe?+ and Cl-
ions in aqueous medium. Thus, quantitative
estimation of Fe(NO,), is not suitable. N, x 25 = 0,25 x 48
B58. (b) : MnO,-(+7) and CrO,Cl,(+6) have highest oxidation
states. N = eoxss _049N
B59. (b) : Oxidation state of nitrogen in compounds is 25
HNO, (+5), NO(+2), N,(0), NH,Cl (—3). The reaction is
Ca(OCl), + 4H+ + 2e- —> Ca*t + 2H,O + Cl, ltl
C. mcq with more than one correct answer
2KI + Cl, — > 2KCl + I,
C9. (a, b, d). The balanced chemical equation is: 2N a,5,0. + 1 —> Na,S nop + 2Nal
ok 2 z +6 0
6I + ClO, + 6H, 80, CI + 6HS80;+ 31, + 3H,O Ca(OCl), + 4H* + 2e— + 2KT + 2Na,5,0,
—>

Ca?+ + 2H,O + 2KCl + Na,S,O, + 2Nal_ ...(iz)


O.N. increases (oxidised)
Since the eqn. (7) involves change of 2e-,
(a) Stoichiometric coefficient of HSO; is 6.
“Mol. wt
(b) O.N. of F increases from —1 to 0 and hence it is oxidised. Eq, wt. of Ca(OCl, = =
(dq) H,O is one of the products.
Molarity = “Normali
Normality
D. mcq based on comprehension 2
D6. (c) : On calculating the e.m.f. of the cells, we find that = 0.48
= =0.0.24
only the e.m.f. of the cell involving oxidation of I
ion by Cl, is +ve and option (c) is correct. Molarity of household bleach = 0.24 M
o- —Saae/ E° = -0.54V
Clh+2e= —> 2Cl- E*° = + 1.36V
Integer Type Answers

Clh+2I- — > 2Cl +I, E® = 1.36 — 0.54 1.(5): ON. of different S atoms in Na,S ni” is
= 0.82 V
D7. (d): Only the e.m.f. of the cell involving the oxidation
of H,O to O, by Mn** is +ve.
Mn?++e- —> Mn2]x4 E° = +1.50V
2H,0 —> 4H++0,4+4e E® =-1.23V
4Mn** + 2H,O —> 4Mn?+ +0, +4Ht Difference in O.N.=5-0=5
B® =+0.27 V
MODERN'S abc + OF CHEMISTRY-XI

2. (6): 2MnO, + 10 l- + 16H* —> 2Mn** + 51, + 8H,O 11. (c): Fluorine being most electronegative exhibits only
Thus, 2 mol of KMnO, react with KI to liberate 5 —1 oxidation state.
moles of I,,.
» (3): Be, Al, Sig Ors NCERT Exemplar Problems : MCQs Type-II
ri(+ 2) + 2(+ 3) + 6(+ 4) + 18(-— 2) = 0
+145 -2 +1 -1 0
2n +6+24-—36=0 or 2n=6 n=3 12. (a, b,c, d) : 2KClO, —> 2KCl + 30,
(a) Since O.N. of potassium does not change, it neither
O
undergoes oxidation nor reduction.
H— O—S —O— O—H (b) O.N. of Cl decreases from +5 (KCIO,) to —1 (in KCI),
4,.(6):
therefore, it is undergoing reduction.
(c) O.N. of O increases from —2 (in KCI1O,) to zero (in O,),
therefore, O is oxidised.
In peroxide bond, the oxidation state of O is — 1
(d) This statement is not correct because Cl is undergoing
H,S(O,)0, reduction and O is undergoing oxidation.
2(+ 1) +a” 4 2(-1) + 38/2) =0 x=+6 13. (c,d): Zn+2HCl——>ZnCl, + H,
5. (4): O.N. of Sis + 6in S,03- HSO;, (NH,),50,, H,SO, (a) The O.N. of Zn increases from 0 to +2 (in ZnCl.)
and therefore, Zn acts as a reductant and not as
6. (5): 3Br, + 3Na,CO, —> 5Nabr + NaBrO, + 3CO,
an oxidant. Hence, option is not correct.

NCERT Exemplar Problems : MCQs Type-I (b) The O.N. of Cl does not change and therefore, it
neither acts as a reductant nor an oxidant. Hence,
2. (d): Ag*|Ag is most positive and therefore, Ag* is the option is not correct.
strongest oxidising agent. (c) The O.N. of H decreases from +1 in H* to 0 in H,,.
. (d): Copper will reduce Br, because E°(1.09 — 0.34 Therefore H* acts an oxidant. This option is
= 0.75V) is +ve. correct.
2Cu + 2Br, —> 2Cu Br,
(d) Zine acts as reductant because its O.N. changes
» {d): The reaction : Ag + Fe** —> Agt + Fe? is not
from 0 to +2. This option is correct.
feasible because E° (0.77 — 0.80 = — 0.03) is
14.(6, c,d): The elements having b, ¢ and d configuration
negative. can exhibit variable oxidation states.
. (a): O.N. of hydrogen is not always +1. The O.N. of
15. (c,d): P. +30H- + 3H,O —> PH, +3H,PO,
—2 +1 +1 -2 341

hydrogen in hydrides is —1 and in H,, it is zero.


. (Bb): NH,NO, exists as NH; and NO3 ions, The O.N. Because O.N. of P increases from O(P,) to +1 (H,PO,)
of Nin NHj is—3 and NO3, O.N. is +5. and decreases from 0(P,) to —3 (PH,), therefore, P
has undergone both oxidation as well as reduction.
+3 +5 +6 +7 Option (c) is correct. Option (d) is also correct because
. (a): CrO5 , ClO3Z , CrO{” and MnO;
O.N. of H remains +1 in all the compounds and hence
» {dt}: The element having electronic configuration 3d°4s? hydrogen is undergoing neither oxidation nor
can exhibit maximum O.N, of +7. reduction.
10. (d): 2NO, + 20H — > NOs + NOs + HO isa 16.(a,b):The electrodes having negative electrode
disproportionation reaction because N is both potentials are stronger reducing agents than H,
gas and therefore, will act as anodes.
oxidised as well as reduced. O.N. of N increases
from +4 (in NO,) to +3 (in NQ35 ).

WWW.JEEBOOKS.IN
REDOX REACTIONS

Unit Practice Test Examination


Time Allowed: 1 Hr. Maximum Marks: 25

], Calculate the oxidation number of sulphur in $,0,?-. (1)


2. Identify the oxidising and reducing agent in the following :
9Cu*t + 41-—3 2Cul + I, (1)
Give one example each in which the oxidation state of oxygen is—1 and +2. (1)
Define a disproportionation reaction. (1)
_ Can the reaction:
Cr,0,7 + H,O—> 2CrO,? + 2H*
be regarded as a redox reaction or not ? (T)
Out of Al and Ag vessel, which one will be more suitable to store 1M HCI solution and why?
E°(Al** |Al) = -1.66 V, E°(Ag* |Ag) = +0.80 V (2)
So that the following reactions are redox reactions :
(1) FeO, + 3 CO—> 2F e(s) + 8CO,
(ti) 4NH,(g) + 50,(¢) —> 4NO(g) + 6H, O(g)
The compound AgF, is unstable compound. However, if formed, the compound acts as a very
strong oxidising agent. Why? (2)
Consider the reactions :
28,0,” (aq) + I,(s)—> 8,0,” (aq) + 2I-(aq)
S,0,”" (ag) + 2Br,() + 5H,O0(l)
—> 280,” (aq) + 4Br(ag) + 10H*(aq)
Why does the same reductant, thiosulphate react differently with iodine and bromine. (2)
10. Depict the galvanic cell in which the reaction:
Zn(s) + 2Ag*(ag) —> Zn?*(aq) + 2Ag(s)
takes place. Further show
(i) which electrode is negatively charged
(ii) individual reaction at each electrode. (3)
11. Permanganate (VII) ion, MnO, in basic solution oxidises iodide ion, I” to produce molecular iodine
(I,) and manganese (IV) oxide (MnO,). Write a balanced ionic equation to represent this
redox reaction. (3)
12. Balance the following equations by oxidation number method
(i) Fe?* + Cr,0,%* #H*=— Fe** + Cr+ + H,O
(ii) C,H, + G@, —=>,CO, +H,0 (3)
13. (a) Fluorine reacts with ice and results in the following reaction :
H,O(s) + F,(¢) —> HF(g) + HO(g)
Justify that this reaction is a redox reaction.
(b) Calculate the oxidation number of sulphur in H,SO,.
Suggest structure and account for the fallacy. (5)

To check your performance, see HINTS and SOLUTIONS to some questions at the end of
Part II of the book.

WWW.JEEBOOKS.IN
WWW.JEEBOOKS.IN
HYDROGEN

ODI OMA DS
Building on..... Assessing..... Preparing for Competition.....
@ Understanding Text 1 @ Quick Memory Test with + Topicwise MCQs a 48
Conceptual Questions 32 Answers 42 Competitive Examination Qs
CHAPTER SUMMARY & QUICK ¢ HOTS&AdvancedLevel 7” ail Lhe State Boards’ o
CHAPTER ROUND UP 33 Questions with Answers 43 —_ 7 oe
> Main ,
NCERT FILE REVISION EXERCISES 46 ~ ), rae fo9 shia tie =
@ In-text Qs & Exercises with @ Blancs Avewers tor | ia aaa
Solutions 35 ee ee 48 » JEE (Advance) for IT Entrance 53
# NCERT Exemplar Problems with ¢ NCERT Exemplar Problems
- ae ) (Objective Questions) 54
Anawers & Solutions (Sulyective) 38° UNEF PRACTICE YaPT Ok /, Fints & Explanations for Difficult
Questions 56

H ydrogen is the first element in the periodic table and is the lightest
element known. It exists as a diatomic moleculeH,and is called
dihydrogen. The atomic form of hydrogen exists only at high
temperatures. It was discovered by Henry Cavendish in 1766.
He prepared the gas by treating iron with dil. H,SO,. Its name
x hydrogen was proposed by Lavoisier because it produces
9 \ water on burning with oxygen (In Greek :hydro means water,
\ gene means producing). Hydrogen forms a very large
| number of compounds and is of great industrial
importance.
UNIQUE POSITION OF HYDROGEN IN THE
| PERIODIC TABLE
| Hydrogen is the first element in the periodic table.
/ Its atomic number is 1. It has one proton and one electron
/ and it has the electronic configuration 1s*. Its position in
7T the periodic table is shown in Fig. 1. In the normal
. ,
elemental form, it exists as a diatomic molecule (H,). That
/ iswhy, it is also called dihydrogen. It resembles both alkali
~ metals and halogens and, therefore, its position is anomalous.
a a Some chemists have argued for its placement with alkali metals
———_—— while some other chemists have proposed its inclusion with halogens.
Resemblances with Alkali Metals
Hydrogen resembles alkali metals such as lithium (Li), sodium (Na), potassium (K), rubidium (Rb), cesium
(Cs) and francium (Fr) of group I of the periodic table in the following respects :
1. Electronic configuration. Hydrogen like alkali metals has one electron in its valence shell. For example,
H (Z= 1) 1s?
Li (Z = 3) [He] 2s!
Na(Z=11) : [Ne] 3s"
K (Z = 19) [Ar] 4s1, ete.

WWW.JEEBOOKS.IN
MODERN'S abc + OF CHEMISTRY-XI

Halogens Group 18

Fig. 1. Position of hydrogen


2. Electropositive character. Hydrogen like alkali of group 17 of the periodic table. Some of its
metals is electropositive because it can lose one characteristics which show resemblance with halogens
electron readily to form hydrogen ion (H*) as are given below :
H ——- H*+e 1. Electronic configuration. Like halogens,
1s! 1s° (proton) hydrogen has only one electron less than the nearest
Na —— Na*t+e noble gas configuration.
[Ne] 3s? [Ne] no: i He : 1s?
3. Valency and oxidation state. Like alkali F : 1s? 2s? 2p® Ne : 1s? 2s? 2p®
metals, hydrogen also shows an oxidation state of +1 Cl : 1s? 2s? 2p° 3s? 3p® Ar : 1s? 2s” 2n® 3s? 3p®
in its compounds. For example, 2. lonisation enthalpy. The ionisation enthalpy
H'Ccr Nat Cl K*cl of hydrogen is quite close to that of halogens, e.g.,
Hydrogen Sodium Potassium H : 1312 kJ mol™* F : 1680 kJ mol
chloride chloride chloride
Cl : 1255 kJ mol
4, Combination with electronegative elements. Br : 1143 kJ mol
Like alkali metals, hydrogen forms binary compounds
The ionisation enthalpy of hydrogen and
with electronegative elements such as halogen, oxygen
halogens are much higher than those of alkali metals:
and sulphur. For example,
Halides HCl NaCl, KCl, ete. Li : 520 kJ mol
Sulphides : H,5S Na,», K,5, ete. Na : 496 kJ mol?
Oxides H,O Na,O, KO, etc. K :419kJ mol
With oxygen, hydrogen also forms hydrogen 3. Electronegative character. Hydrogen like
peroxide (H,O,). Alkali metals also form peroxides e.g. halogens has tendency to accept electron to form
Na,O, (sodium peroxide), ete. monovalent anion. By accepting an electron hydrogen
5. Liberation at cathode. When the fused salts of as well as halogens acquire noble gas configurations :
alkali metals are electrolysed, metal ions are liberated H+e —— > 8H (He gas configuration)
at the cathode. In a similar manner, hydrogen is Ist 1s”
liberated at the cathode on subjecting the aqueous Pee —4 F (Ne gas configuration)
solutions of its salts to electrolysis : [He] 2s?2p° [He] 2s72p°
2HCl (ag) SEeeieelyss oH, + Cl, Clee ———— cy (Ar gas configuration)
(At cathode) [Ne] 3s?3p* [Ne] 3s?3p®
2NaCl (molten) —Hect's8_,oNa + = Ci 4, Oxidation state. Like halogens, hydrogen
(At cathode) exhibits oxidation state of —1 in its compounds.
6. Strong reducing agent. Like alkali metals, Na* H- Na* Cl
hydrogen acts as a strong reducing agent. Sodium hydride Sodium chloride

Fe,0,+4H, — “> Heat


3Fe + 4H,O 5. Diatomic nature. Hydrogen molecule (H,) is
diatomic like halogen molecules (F,, Cl,, Br, I,).
B,0,+6K —%, 2B + 3K,0 6. Liberation at anode. When fused alkali metal
Resemblances with Halogens hydrides such as lithium or sodium hydride is
Hydrogen also resembles with halogens i.e., electrolysed, hydrogen is liberated at the anode. This
fluorine (F), chlorine (Cl), bromine (Br), and iodine (1) is in the same way as halogens are liberated at anode
HYDROGEN

when fused alkali metal halides are electrolysed. hydrogen atoms do not possess any unshared electron
Electrolysis _ pairs whereas there are three unshared electron pairs
2NaH (molten) —————> 2Na(J)+ H, (g) on each halogen atom in its molecule.
. At anode
2NaCl (molten) ——"** > 2Na (1) + Cl, (g) H—H Cl—Cl:
Hydrogen molecule Chlorine molecule
At anode
7. Non-metallic nature. Like halogens, hydrogen 3. Nature of oxides. The oxides of halogens are
is non—metallic in nature. acidic while hydrogen oxide (H,O) is neutral.
8. Nature of compounds, Like halogens, hydrogen CLO, H,O
combines with non-metals such as carbon, silicon, Acidic Neutral
vermanium, etc. to form covalent compounds. For From the above discussion, it is clear that
example, hydrogen is unique in its behaviour because it
CH,, SiH, GeH, Compounds of hydrogen resembles alkali metals as well as halogens.
CCL,, SiCl,, GeCl, Compounds of halogens Therefore, it is difficult to assign any definite
Though hydrogen resembles alkali metals and position to it in the periodic table. Thus, the
halogens in many respects yet it is different from these position of hydrogen in the periodic table is
in some respects. Hydrogen has none of the metallic anomalous and it is best placed separately in the
characteristics* like alkali metals and its resemblance periodic table of elements.
to halogens is only slight. The proton, H* has very OCCURRENCE OF HYDROGEN
small size about 1.5 x 10 pm compared to normal Hydrogen is the most abundant element in the
atomic and ionic sizes of 50 to 220 pm respectively. As universe (about 70% of the total mass of the universe)
a result, H* does not exist freely and is always and is also common on earth. In the free state, hydrogen
associated with other atoms or molecules. is present in some volcanic gases and in the outer
Some of its differences from alkali metals and atmosphere of the sun and other stars of the universe.
halogens are discussed below. About half the mass of sun and some other stars is due
Differences from Alkali Metals to hydrogen. Jupiter and Saturn planets consist mainly
Hydrogen differs in some of its characteristics from of hydrogen. The extremely high temperature of the
alkali metals. Some of these differences are : sun and stars enables nuclear fusion of hydrogen atoms
1. Ionisation enthalpy. Ionisation enthalpy of liberating large amount of energy.
hydrogen is very high in comparison to alkali metals. 43H —— 3He + 2ie + Energy
For example, Positron
IE of H = 1312 kJ mol? It is the third most abundant element on the
IE of Li = 520 kJ mol™, IE of Na = 495 kJ mol’, surface of the earth. On the earth, hydrogen is present
IE of K = 418 kJ mol in the form of various compounds mainly with oxygen
2. Non-metallic character. Alkali metals are in the form of water and in combination with carbon,
typical metals while hydrogen is non-metal. nitrogen, sulphur and halogens in the form of organic
3. Atomicity. Hydrogen is diatomic (H.,) whereas matter in plants and animal tissues, carbohydrates,
alkali metals are monoatomic. proteins, etc. Apart from this, hydrogen is also
4, Nature of compounds. The compounds of distributed in the mineral kingdom e.g., in coal,
hydrogen are predominantly covalent while those of petroleum, oil and natural gas. It constitutes 15.4% of
alkali metals are ionic. For example, HCl! is covalent earth’s crust and oceans. It is not found in our
while NaCl is ionic. atmosphere because earth does not have enough
The oxides of alkali metals are basic while eravitational pull to retain light hydrogen molecules.
hydrogen oxide (H,O) is neutral. In the crustal rocks, itis tenth in the order of
Li,O, Na,O, K,O, ete.: basic; H,O : neutral. abundance (about 0.15% by weight).
Differences from Halogens
ISOTOPES OF HYDROGEN
Though hydrogen resembles halogens in many
Isotopes are the different forms of the same
respects, yet it differs also in certain respects. Some element which have the same atomic number but
of the important differences between hydrogen and different mass numbers. Hydrogen has three
halogens are given below : isotopes with mass number 1, 2 and 3 respectively.
1. Less tendency for hydride formation.
These are called protium CGH ), deuterium (?H or D)
Hydrogen has less tendency to take up electron to form
hydride ion (H ) as compared to the halogens which and tritium (9H or T). Hydrogen occurring in nature
form halide ions (X~) very easily. is mainly composed of atoms of protium. It contains
2. Absence of unshared pairs of electrons. The only 0.0156% of deuterium mainly in the form of HD.
hydrogen and the halogens are diatomic in nature in However, the third isotope, tritium is radioactive and
which the atoms are held by covalent bonds. However, emits low energy f-particles (half life of tritium
*Recent studies have shown that hydrogen under very very high pressures is expected to behave as a metal.
MODERN'S abc + OF CHEMISTRY
-XI

Table 1. Isotopes of hydrogen.

Property Protium Deuterium Tritium


(H) (D) (T)
1 2 3
Symbol 1H or H Hl or D 1H or T
Atomic number 1 1 1
Mass number 1 2 3
Relative atomic mass 1.007825 2.014102 3.016049
Nuclear spin
quantum number Wp i Vy
Radioactive stability Non-radioactive Non-radioactive Radioactive, emits
stable stable particles (¢,, = 12.33 y)

= 12.33 y). Its concentration is about 1 atom per 1018 It is present in heavy water (D,O) and can be recovered
atoms of protium. The symbols, mass numbers, by fractional electrolysis. It has also one solitary
relative abundance and radioactive nature of three electron outside the nucleus.
isotopes of hydrogen are given in Table 1. (iii) Tritium (*H or ‘T). It is the rarest isotope
All the three isotopes of hydrogen have one
electron in their outer shell and only one proton in
of hydrogen (19-15%) due to the instability and
the nucleus. However, they differ with respect to the radioactive nature of its nucleus. It is formed in the
number of neutrons as shown below : upper atmosphere by certain nuclear reactions which
are induced by cosmic rays. Its nucleus has one proton
and two neutrons (mass no. = 3). It is represented as
Tor ase It is prepared artificially by the bombardment
of nitrogen or an isotope of lithium with deutrons :

WN + on —— 7C + *H (or T)
Protium Deuterium Tritium
oLi - jn — *He + °H (or T)
‘Hor H ‘Hor D sHor T Thus, the three isotopes of hydrogen differ only
in the number of neutrons in the nucleus. Since the
Fig. 2. Isotopes of hydrogen. isotopes have the same atomic number and electronic
A brief description of the isotopes of hydrogen is configuration, they are expected to have similar
civen below : chemical characteristics.
(i) Protium or ordinary hydrogen (+H). It is the The common physical properties of H,, D, and T,
are given in Table 2.
most common and most abundant isotope of hydrogen.
It constitutes 99.985% of the total hydrogen available DIHYDROGEN
in nature. The nucleus of hydrogen has one proton Preparation of Dihydrogen
and no neutron (mass no. = 1). It is represented as iH Dihydrogen can be prepared by the following methods:
or H. There is a solitary electron outside the nucleus. 1. By the action of water on metals
(ti) Deuterium or heavy hydrogen ?H or Al ). Various metals differ in their reactivity towards
It constitutes only 0.0156% of the total hydrogen water. For example,
occurring in nature. Its nucleus has one proton and one (a) Very active metals like Na, K and Ca react
neutron (mass no. = 2), It is represented as D or ‘H. with water at room temperature evolving dihydrogen.
Table 2. Physical constants of H,, D, and T,.

Property H, D, T,

Molecular mass 2.016 4.028 6.03


Melting point (K) 13.96 18.73 20.62
Boiling point (K) 20.39 23.67 20.0
Density (gmL~‘) 0.09 0.18 We eaty
Enthalpy of fusion (kJ mol!) 0.117 0.197 0.250
Enthalpy of vaporisation (kJ mol-!) 0.904 1.226 1.393
Enthalpy of dissociation (kJ mol!) 435.88 443.35 446.9
Internuclear distance (pm) 74,14 74,14 —
HYDROGEN

2Na + 2H,O —> 2Na0H + H, Zn+2NaQOH —— > Na,4n0O, + H,


2K +2H,0 — > 2KOH +H, Sodium
Ca + 2H,O —— > Ca(OH), + H, zincate
The reaction is very vigorous and exothermic so 4, By the action of metals on acids
that the hydrogen evolved catches fire. In order to slow
down the reaction, amalgam of these metals with Active metals like zinc, magnesium and iron which
mercury are generally used. In the amalgam, only a lie above hydrogen in the electrochemical series, can
small surface of the metal is exposed to water and displace dihydrogen from dilute mineral acids such
therefore, the reaction is slow. as hydrochloric acid and sulphuric acid. For example,
(6) Less active metals like Zn, Mg, Al, etc. can
Zn + H,SO, ——> ZnSO, + 4H,
liberate hydrogen from water only upon heating.
Mg + 2HCl —- MgCl, + H,
Mg+H,O —<, MgO + H, Fe + 2HOIG —e FeCl, + 4H,
Zn+H,O —Heatt, ZnO + H, 5. Laboratory preparation of dihydrogen
2A1 + 3H,0 —, ALO, + 3H, In the laboratory, dihydrogen can be prepared by
(c) Metals like tron and nickel can react only by the action of dil. H,SO, on granulated zinc.
passing steam since they are very little reactive.
Zn + H,SO, (dil.) —— ZnSO,+H,
3Fe
+4H,O (steam) '3K
7 _, Fe, + 4H,
Ferroso-ferric oxide
dil. H, SO,
(Magnetic oxide of iron) HYDROGEN
The reaction between iron and steam is used for the GAS
largescale preparation of dihydrogen by Lane's process.
2. By the electrolysis of water ats

ital
Dihydrogen of high purity is usually prepared by Hitt
a
the electrolysis of using platinum electrodes in the ty
Lagpity
TLL
presence of a small amount of acid or alkali. A small heii
i,t,l TL
j!a'y
quantity of acid or alkali (15-25%) is added to water
to make it a good conductor.
Electrolysis =—S——-—---M-e

2H, OW) “traces


ofacid/base 2H,(g) + O.(g)
= = ae 2S 2S SS SE SS Se a
J = ee a

GRANULATED ZIN
During electrolysis, dihydrogen is collected at
Fig. 3. Laboratory preparation of dihydrogen.
cathode and dioxygen is liberated at anode. The
following reactions take place during the electrolysis
In the preparation of dihydrogen in the laboratory,
of acidulated water :
eranulated zinc is taken in a Woulfe’s bottle. Dilute
H,O ——=—=—$= H* + OH™
sulphuric acid is poured dropwise through thistle
At cathode
funnel on granulated zinc pieces (Fig. 3). The gas
Ht+e —— H
evolved is collected over water by the downward
H+H —— > H,@)1
At anode displacement of water.
4OH —4e —— 40H It has been observed that sometimes the bubbles
40H ——~» 2H,0(/)+0,(g) T of hydrogen formed stick to the surface of zinc metal
and this prevents the further reaction of the acid on
The hydrogen prepared by this method is of very high
the metal. However, this can be prevented by adding
purity. However, this method is not commonly used
because it is very expensive. This method can be used a few crystals of copper sulphate to the reaction
only at those places where the electricity is cheap. mixture.

3. By the action of alkalies on metals


Metals like beryllium, aluminium, zinc and tin
react with strong alkalies like NaOH and KOH and The following points should be remembered for the
liberate dihydrogen. laboratory preparation of dihydrogen :
(i) Pure zine is not used in the preparation of
Be+2NaOH —Heat_, Na,BeO, + H,
dihydrogen because its reaction with sulphuric acid is
Sodium
slow. The presence of some impurities increases the rate
beryllate
of reaction due to the formation of electrochemical couples.
2Al+ 2NaO0H + 2H,0 —— 2NaAl0, + 3H, (it) Concentrated sulphuric acid is also not used
Sodium for the preparation of dihydrogen because zinc reacts
meta-aluminate with concentrated acid to form SO, gas instead of
Sn+2NaOH+H,O —— Na,SnO, + 2H, dihydrogen.
Sodium Zn + 2H,SO, (conc.) ——> ZnSO, + SO, + 2H,O
stannate

WWW.JEEBOOKS.IN
MODERN'S abc + OF CHEMISTRY
-XI

Preparation of Pure Hydrogen 3. Hydrocarbon steam process. Dihydrogen is


Very pure hydrogen (more than 99.95% purity) also prepared by the action of steam on hydrocarbon
may be obtained by the following methods : at high temperatures.
(1) By the action of pure dil. H,SO, on magnesium C,H, ,9+nH,O —@@*.. nCO+(2n+1)H,
ribbon.
Mg + H,SO,(dil.) ———> Mgs0O,+H,
e.g.,CH,(g) + H,O(g) B® , CO) + 3H,(g)
(steam)
(iz) By the action of water on sodium hydride.
NaH+H,O0 ——> NaOH+H, CH, (g) + 3H,0 (g) meee 3CO(g) + 7H, (g)
(zit) By the action of potassium hydroxide on scrap The CO and H, mixture obtained is called syn gas.
aluminium.
Carbon monoxide is oxidised to carbon dioxide by
2Al + 2KOH + 2H,O ——> 2KAIO, + 3H, passing the gases and steam over an iron chromate
This method is known as Uyeno's method and is used at 673 K resulting in the production of more H,.
for preparing hydrogen for military purposes.
(tv) By the electrolysis of warm solution of barium CO +H,O——“*> CO, + H,
hydroxide [Ba(OH),] using nickel or platinum
electrodes. The CO, is removed either by scrubbing the
mixture with sodium arsenite or by passing the mixture
Commercial Preparation of Dihydrogen through water under 30 atm pressure, when CO,
Commercially dihydrogen is prepared by the dissolves leaving behind H,, which is collected.
following methods :
The whole process of obtaining dihydrogen from
1. Electrolysis of water. This is the best method
for the manufacture of dihydrogen, where electricity hydrocarbons (or natural gas) is called steam
is cheap. This has already been discussed (general reforming process.
methods of preparation, 2). 4, Lane’s process. Hydrogen is prepared by
2. By the reaction of steam on coke, Dihydrogen passing alternate currents of steam and water gas over
is prepared by passing steam over red hot coke at red hot iron. The method consists of two stages :
1270 Kin the presence of Nias catalyst. In this reaction, (i) Oxidation stage. Super heated steam is passed
first a mixture of CO and dihydrogen is formed. over iron filings heated to about 1025-1075 K when
hydrogen is produced and magnetic oxide of
C(s) + H,O(g) —2* > CO) + H,(g)-121.3kd
| iron (Fe,O,) is left behind. This is carried out for half
Coke Steam Water gas an hour.
The mixture of CO and H, is called water gas. It
is an important industrial fuel because it is easy to 3Fe
+4H,O —U2-
Ok FeO, + 4H, + 161kJ
make and it burns producing a lot of heat. Since the Iron (Steam) Macneue
filings oxide of iron
mixture of CO and H, is used for the synthesis of
methanol and a number of hydrocarbons, it is called (it) Reduction stage. When the whole iron has
synthesis gas or syngas. been oxidised, the supply of steam is stopped and
The process of producing syngas from coal or coke stream of water gas (H, + CO) is passed to reduce
is called coal gasification. Fe,O, to iron.
Nowadays, syngas is produced from saw dust Fe,0, + 4CO —— > 3Fe + 4CO,
sewage, scrap wood, news papers, etc.
Fe.0, + 4H, —— 3Fe + 4H,O
It may be noted that earlier a 1:1 mixture of CO and These reactions also take place at 1025—1075K.
H, was called water gas but now all mixtures of CO and
H, irrespective of their composition are called syngas. In actual process, iron is heated in a tube over a furnace
maintained at the required temperature and steam
It is difficult to obtain H, from water gas or syngas and water gas are passed over it alternately.
because CO is difficult to remove. To increase the
production of dihydrogen, the mixture of CO and H, 5. From methanol. Relatively small quantities of
is mixed with more steam and the gaseous mixtureiis dihydrogen are obtained by passing a 1: 1 molar
heated in the presence of iron chromate (FeCrO,) as mixture of vaporised methanol and water over a
catalyst at 673 K. special type of catalyst ‘base metal chromite type’ at
673 K 673 K. The mixture of dihydrogen and carbon
,H, + CO | + H,O FeCrO4 CO, + 2H, monoxide obtained is made to react with steam to give
Water gas Steam CO, and more dihydrogen.
This chemical reaction in which carbon monoxide
673K
of the syngas reacts with steam to form carbon dioxide CH,OH ———~
Catalyst
CO + 2H,
and more dihydrogen is called water-gas shift
reaction. The CO, thus produced is removed by co+H,o —2%.
673 K
Catalyst
co,+ H,
scrubbing with sodium arsenite solution. Alternatively,
the resulting mixture is compressed to about 30 6. By-product in the manufacture of caustic
atmospheric pressure and is passed through water. soda. Dihydrogen is also produced as a by-product of
Carbon dioxide dissolves in water under pressure while the brine electrolysis process for the manufacture of
dihydrogen is released and is collected. sodium hydroxide and chlorine.
HYDROGEN
97 |
During electrolysis, the following reactions take is not very reactive. However, it combines with many
place : elements or compounds.
at anode : 2CI (aq) —> CL(g) + 2e7 1. Action with metals. Dihydrogen combines with
at cathode : 2H,O (/) + 2e— H,(g) + 20H (aq) metals like sodium, potassium and calcium upon
heating to form corresponding hydrides which are of
Overall reaction
ionic nature (salt-like). For example,
2Cl- (aq) + 2H,O(/) —> C1,
(@) + H, (g) + 20H" (aq)
or 2Na* (aq) + 2Cl (aq) + 2H,O(/) 2M(s) + H,(g) ——— 2MH(s)
(M = alkali metal)
— ClL(g) + H, (g) + 2Na* (aq) + 20H (aq)
(adding Na’ ions on both sides) eg.,2Na+H, —~“*+ 2NaH
Sodium hydride
Presently about 77% of the industrial hydrogen
produced is from petrochemicals, 18% from coal, Ca +H, _ tps CaH,
4% from electrolysis of aqueous solutions and 1% Calcium hydride
from other sources. With transition metals (elements of d-block) such
as platinum, palladium, nickel, etc. dihydrogen forms
Properties of Dihydrogen
interstitial hydrides in which the small molecules of
Physical properties
dihydrogen occupy the interstitial sites in the crystal
The important physical properties of dihydrogen
lattices of these hydrides. As a result of formation of
are :
interstitial hydrides, these metals adsorb large volume
(z) It is a colourless, tasteless and odourless
of hydrogen on their surface. This property of
gas.
adsorption of a gas by a metal is called occlusion. The
(it) It is shghtly soluble in water.
occluded hydrogen can be liberated from the metals
(it) It is highly combustible and therefore
by strong heating.
should be handled carefully.
(iv) It is lightest substance known. For 2. Reaction with non-metals. Dihydrogen reacts
example, one litre of hydrogen at N.T.P. with non-metals under specific conditions. Some of the
weighs only 0.0899 g. reactions are summed up below :
970K
(v) Its density is about 1/14th of that of air. Dioxygen 2H,(g) + O,(g) or Are 2H,O(/)
The physical constants of atomic and molecular discharge Waiar
hydrogen are given in Table 3. This reaction is strongly exothermic (AH =
It may be noted that the term dihydrogen is used for ~285.9 kJ mol*)
H, molecule while referring to the isotopic mixture with Fe, Mo
natural abundance for H and D. However, in terms of Dinitrogen N,(g) + 3H8) Gisk oo0atm ZN;
isotopes, the correct term for H, is diprotium. Similarly, Ammonia
H* is named as proton and hydron is used in relation to
This reaction is also exothermic (AH = —92.6 kJmol!),
isotopic mixture.
Here Fe acts as a catalyst and Mo acts as a
Chemical properties
promoter.
Dihydrogen is quite stable and dissociates into
This reaction is the basis for the manufacture of
hydrogen atoms only when heated above 2000 K.
ammonia by the Haber process.
H,(g) 2000 K Hig) +H (g)
Above 2000K, the dissociation of hydrogen into
Halogns H, + F, —“ > 2HF
H, 4 Cl, Diffused sunlight IHC]
atoms is only 0.081% which increases to 95.5% at
5000 K.
Its bond dissociation energy is very high.
H, + Br, _°@Kpressue . 2HBr
H, ——> H-4H; AH =435.9 kJ mol H.¢ i ==. ont
The H—H bond dissociation enthalpy is the In general,
highest for a single bond between two atoms of any
element. Due to its high bond dissociation enthalpy, it H,(g) 2 X,(g) — > 2HX(g)
(X = F, Cl, Br, D
Table 3. Physical constants of atomic and molecular hydrogen.

Atomic properties Molecular properties


Atomic radius (pm) = a7 Melting point (K) = 13.8
Ionic radius of H- ion (pm) = 210 Boiling point (K) = 20.4
Ionisation enthalpy (kJ mol!) = 1312 Density (gcm™) = 0.0899
Electron affinity (kJ mol) = 172.8 Bond length (pm) = 74.2
Electronegativity a | Bond dissociation
Enthalpy (kJ mol!) = 435.9
MODERN'S abc + OF CHEMISTRY-XI

The reactivity of halogen towards dihydrogen RCH = CH, + H,+ CO Gia


[Co(CO)4 lo
RCH,CH,CHO
temp. and pressure
decreases as:
Olefin Aldehyde
F, > CL > Br, > I,
This reaction is called hydroformylation or oxo
As a result, fluorine reacts in dark, chlorine in
process. The aldehyde thus formed is further reduced
the presence of sunlight, bromine reacts only upon
to alcohol by the reaction :
heating while the reaction with iodine occurs in
the presence of a catalyst. RCH,CH,CHO + H, ————+ RCH,CH,CH,OH
Aldehyde Alcohol
700 K (it) Aldehyde Hydrogenation of oils. The
Sulphur H, + § ——_ HS
Carbon 2 on. —<23)5 Cy, hydrogenation of vegetable oils which are
polyunsaturated give solid edible fats (margarine and
Methane
vanaspati).
2c + H, ik C,H, This reaction is used in the hydrogenation or
Electric spark Acetylene
hardening of oils. The vegetable oils such as groundnut
3. Reaction with metal oxides. Dihydrogen acts oil or cotton-seed oil are poly-unsaturated in nature
as a strong reducing agent and therefore, it reduces because they contain double bonds in their molecules.
oxides of metals (less active than iron) to metals. The C = C bonds in oils can easily undergo oxidation
and the oil becomes rancid i.e., unpleasant in taste.
M,0,(s) + yH, (g) —— xM(s) + yH,O (J)
Hydrogenation reduces the number of double bonds but
For example, does not completely eliminate them. Under controlled
Cu0+H, —Heat_, Cu + H,O conditions, dihydrogen is passed through the oils at
Copper about 473 K in the presence of nickel catalyst to form
solid fats.
ZnO+H, —H## , Zn + H,O Ni
AInC
Vegetable oil + 4H, 173K Fat
PbO+H, ———> Pb + H,O (liquid) (solid)
Lead
Edible oils and margarine can be prepared by
Fe,0,+4H, —2 , 3Fe + 4H,O hydrogenation of vegetable oils like soyabean, corn,
Tron cotton seed oils etc. The vanaspati ghee such as Dalda,
It may be noted that the oxides of strongly Rath ete. are usually prepared on a large scale in India
electropositive metals as those of alkali and alkaline by this process.
earth metals are not reduced by this method.
Uses of Dihydrogen
Dihydrogen also reduces some metal ions in
aqueous solution. For example, (tz) Itis used as a reducing agent in the laboratory
and industry. In metallurgical processes, it is used to
Pd** (aq) +H,(g) ————> Pd(s) + 2H* (aq) reduce heavy metal oxides to metals.
Cu*(aqg) + H,(g) ———> Cu(s) + 2H*(aq) (it) It is used in the manufacture of vanaspati fat
4, Reaction with carbon monoxide Dihydrogen by the hydrogenation of poly-unsaturated vegetable
reacts with CO at 700 K in the presence of a catalyst oils like soyabean, cotton seeds etc.
ZnO/Cr,O, to form methanol.
(iit) The most important use of dihydrogen is in
700 K, 200 atm
CO + H, ZnO, Cr,0, CH.OH the manufacture of ammonia by Haber process.

N, (g) + 3H, a= _-2NH, (g)


673K, 200 atm
Methanol
5. Reaction with unsaturated hydro-carbons. (1v) It is used in the preparation of many
Dihydrogen reacts with unsaturated hydrocarbons compounds such as metal hydrides, hydrogen chloride,
such as ethylene and acetylene to give saturated water gas, methanol and fertilizers such as urea,
hydrocarbons. ammonium sulphate, etc.
H,C(C== CH, + H, — ee.
Ni or Pt or Pd
==. H,C —— CH,| (v) Dihydrogen is used in the manufacture of bulk
Ethylene Ethane organic chemicals such as methanol.

CO (g) + 2H, CH,OH (i)


Cobalt catalyst
HC=CH +2H, SNiorPtorPd , 4.C—CH, 700K, 200 atm
A73 K :
Acetylene Ethane (vi) It is used as a rocket fuel for space research in
The hydrogenation of many organic compounds the form of liquid hydrogen and liquid oxygen.
using homogeneous and heterogeneous catalysis (vit) It is used in the manufacture of synthetic
constitute important industrial processes. For petrol.
example : (vit) It is used in the atomic hydrogen torch and
(1) Hydroformylation of olefins to aldehydes. oxy hydrogen torches for cutting and welding.
Olefins react with carbon monoxide and dihydrogen Dihydrogen is dissociated with the help of an electric
in the presence of octacarbonyl dicobalt [Co(CO),], as arc and the hydrogen atoms produced are allowed to
catalyst under high temperature and pressure to form recombine on the surface to be welded. High
aldehydes. temperature of about 4000 K is generated.
HYDROGEN

(ix) Dihydrogen is also used in bubble chambers as (tv) Even at very high temperatures, the ratio
liquid hydrogen for the study of high energy particles. of ortho to para hydrogen can never be more
(x) Dihydrogen is used in fuel cells for generating than 3: 1.
electrical energy. It has many advantages over Thus, it has been possible to get pure para
conventional fossil fuels and electric power. It does hydrogen by cooling ordinary hydrogen gas
not produce any pollution and releases larger energy to a very low temperature (close to 20 K) but
per unit mass of fuel in comparison to gasoline and it is never possible to get a sample of
other fuels. hydrogen containing more than 75% of ortho
hydrogen.
Learning Plus The physical properties of dihydrogen are not
ORTHO AND PARA HYDROGEN affected significantly by nuclear spin isomerism.
However, some of the following are some notable
A molecule of dihydrogen contains two atoms.
differences :
The nuclei of both the atoms in each molecule of (i) The thermal conductivity of para-hydrogen
dihydrogen are spinning. Depending upon the is about 50% greater than that of ortho-hydrogen,
direction of the spin of the nuclei, the hydrogens are and
of two types : (it) The melting point of para hydrogen is
(a) Molecules of hydrogen in which the spins of 0.15 K below that of hydrogen containing 75% ortho
both the nuclei are in the same directions (parallel hydrogen.
nuclear spins) are called ortho hydrogen.
(b) Molecules of hydrogen in which the spins of
both the nuclei are in the opposite directions (anti-
ATOMIC HYDROGEN
parallel nuclear spins) are called para hydrogen. Atomic hydrogen. The hydrogen obtained by the
These two types of hydrogens are called nuclear dissociation of hydrogen molecules is called atomic
spin isomers and are shown below : hydrogen. Because of high H—H bond enthalpy, atomic
hydrogen is produced only at a high temperature in an
electric arc or under ultraviolet radiations.
Nuclei It is obtained by passing dihydrogen gas at
atmospheric pressure through an electric arc struck
between two tungsten rods.
The electric arc maintains a temperature around 4000
Ortho hydrogen — 4500°C. As the molecules of dihydrogen gas pass through
(Parallel nuclear spins) the electric arc, these absorb energy and get dissociated
into atoms as
H,(g) —“ > 2H(g) : AH = 435.90 kJ mol
The atomic hydrogen is stable only for a fraction of a
second and is extremely reactive. The life time of atomic
hydrogen is 0.3 sec and therefore, it immediately gets
Para hydrogen converted into the molecular form liberating a large
(Antiparallel nuclear spins) amount of energy which is used for cutting and welding
purposes.
These two isomers have 1 and 0 nuclear spin This arrangement is also called atomic hydrogen
respectively. The conversion of one isomer into torch. :
\_ Jungesten rod
another isomer is a slow process. Ordinary
dihydrogen is an equilibrium mixture of ortho and
para hydrogen.
Ortho hydrogen =——— > Para hydrogen
The amount of ortho and para hydrogen varies
with temperature. The ratio of ortho to para hydrogen
increases with increase in temperature upto a Tu ngesten rod

temperature of about 300 K and thereafter it becomes ATOMIC HYDROGEN TORCH


almost constant. It has been observed : Since the atomic orbital of hydrogen is incomplete
(i) At OK, hydrogen contains mainly para with 1s! electronic configuration, it needs only one electron
to achieve stable noble gas configuration. Therefore,
hydrogen which is more stable. atomic hydrogen ts very reactive and combines with almost
(ii) At the temperature of liquefaction of air, the all elements. It reacts in three different ways:
ratio of ortho and para hydrogen is 1 : 1. (t) by loss of its only one electron to form H*.
(ii) by gain of one electron to form H~ and
(iit) At the room temperature (300 K), the ratio
(iit) by sharing its electron with other atoms to form single
of ortho to para hydrogen is 3 : 1. covalent bond.
MODERN'S abc + OF CHEMISTRY
-XI

=Datick
Dina ? 1. lonic hydrides or saline hydrides
These are formed by the combination of hydrogen
with metals which are highly electropositive in
Q 1. Which isotope of hydrogen character. These include elements of s-block elements
(1) does not contain neutron i.e. of group 1 (alkali metals), group 2 (alkaline earth
(iz) is radioactive ? metals with the exception of Be and Mg) as well as
Q@ 2. Which gaseous compound on treatment with some other elements such as lanthanum.
dihydrogen produces methanol ? These are formed by the transfer of electrons from
Q@ 3. Why is dihydrogen not preferred in balloons these metals to hydrogen atom and contain hydrogen as H™
days ? ion. For example, lithium hydride (Li*H~), sodium
@ 4. Which of the following metals can be used for hydride (Na*H), calcium hydride (Ca”*H,), ete.
liberating hydrogen from dilute hydrochloric acid?
Ionic hydrides are prepared by the direct
(i) Zine (it) Copper (iii) Iron (iv) Silver
(v) Magnesium combination of the metals with hydrogen at high
Q@ 5. Name the constituents of water gas. temperatures usually close to 1025 K.
Q@ 6. Name one industrial method for the preparation 2Li s+ HH, —— +#£2LiH
of dihydrogen. (Molten)

Q@ 7. Arrange H,, D, and T, in the decreasing order of


their
2K/@: e/_S@* . okH
(4) boiling points (i) heat of fusion
Ca + ri, Wisk , CaH,
Q@ 8. Namethe products obtained when hydrogen reacts
under suitable conditions with Sr + H, eS srH,
(4) Nitrogen (72) Carbon monoxide (zi) Lead oxide General Chal Aeristics of Ionic Hydrides
Q 9. Does hydrogen support combustion ?
The general characteristics of ionic hydrides are
Q10. Name one compound each in which hydrogen exists given below :
in (1) positive oxidation state, and (ii) negative
oxidation state. 1. lonic hydrides are generally white or light grey
non-volatile and non-conducting crystalline
—— Answers to Practice Problems — = solids. They behave like salts and are also
© 1. (2) Protium (7) Tritium referred to salt like hydrides. However, Bef, and
© 2. Carbon monoxide MgH, have covalent polymeric structures.
© 3. Because it is combustible in nature. 2. They have high melting and boiling points.
© 4. (z), (vt) and (v) 3. The binary hydrides of alkali metals (LiH, NaH,
© 5. Carbon monoxide and hydrogen. KH, RbH and CsH) have rock salt like structure.
© 6. Bosch process. 4, They conduct electricity in fused state liberating
© 7. @T,>D,>H,@)T,>D,>H, hydrogen at the anode. The negative charge on
© 8. (i) Ammonia (ii) Methanol (iii) Lead the hydrogen in these hydrides is proved by
electrolysis of molten lithium hydride at 973 K.
© 9. No, it does not support combustion.
During electrolysis, lithium metal deposits at the
©10. (@) HCl (i) NaH
cathode and hydrogen is liberated at the anode.
HYDRIDES Li Li* + H-
Hydrogen under certain conditions combines with At cathode: Pinte
almost all elements, metals and non-metals (except
Livi +e — Li
noble gases). The binary compounds of hydrogen (deposited)
with other elements are called hydrides. These At anode:
hydrides have the formula EH, (e.g., MgH,) or EH,
(e.g., B,H,) and are formed by H' -e ——> H

(i) all main group elements except the noble gases


H+H —— > H,?
and probably indium and thallium This shows the existence of the hydride (H_) ion.
(ii) all lanthanoids and actinoids 2H" (melt) —— > H,(g) + 2e7
(zit) transition elements such as Sc, Y, La, Ac, Te, 5. Their densities are higher than those of metals.
“Zr, Hf and to lesser extent V, Nb, Ta, Cr, Cu and Zn. 6. LiH, CaH, and SrH, are the most stable hydrides
Among the transition metals, palladium hydride has of this class, others undergo thermal
been extensively studied.
decomposition above 775 K. Enthalpies of
The hydrides are classified into three main types : formation of hydrides of group 1 and 2 are given
1. onic or salt like or saline hydrides in Table 4. These values give the relative stabilities
2. Covalent or molecular hydrides of these hydrides. Larger the enthalpies of
3. Metallic or interstitial hydrides. formation, greater are their stabilities.
HYDROGEN

Table 4. Enthalpies of formation of hydrides of group 1 The reaction of these hydrides with water is very
and 2. explosive. The fires so produced cannot be
extinguished by carbon dioxide because it gets
Hydride Enthalpies of Temperature of
reduced by the hot metal hydride. To extinguish
formation dissociation
the fire, only sand can be used because itis a solid.
(kJ mol) (K) 9. They react with protonic solvents liberating
Group 1 hydrogen.
LiH 90.3 823 LiH + CH,0H——> LiOCH,+H,1
NaH 58.1 483 NaH +NH;,———> NaNH,+H,1
KH 58.9 473 10. They act as powerful reducing agents, especially
RbH 96.5 443 at high temperatures. For example,
CsH 54.4 443 NaH reduces CO and CO, to formates on heating.

Group 2 2CO + NaH —*"**_5 HCOONa + C


CaH, 188.7 1158 K CO; +NaH —““> HCOONa
SrH, 176.9 858 K Sodium formate
BaH, Darya ea 5038 K They reduce sulphates to sulphides
PbSO, + 2CaH, ——> PbS + 2Ca(OH),
The thermal stability of the alkali metal hydrides They also reduce some halides and oxides as :
decreases from LiH to CsH.
Fe,0, + 4NaH —Heat_, 3Fe + 4NaOH
LiH > NaH > KH > RbH > CsH
SiCl, + 4NaH ——> SiH, + 4NaCl
The order of stability in alkaline earth metal
2BF, + 6NaH _459K_, BH, + 6NaF
hydrides is :
Diborane
CaH, > SrH, > BaH,
They are capable of reducing organic acids to
7. The ionic hydrides are oxidised by air to form alcohols.
metal oxide and water. RCOOH + 2LiH ——> RCH,OH + Li,O
2EH +O, ——>_ E,0 + H,O 11. Some hydrides are used to prepare complex
hydrides of other metals. The complex hydrides
RbH and CsH burn spontaneously at room
also act as reducing agents. For example,
temperature whereas LiH is unreactive at
LiH reacts with aluminium chloride in ether
moderate temperatures. solution or diborane to form lithium aluminium
8. They are hydrolysed by water with the formation hydride, LiAlIH, (complex hydride) or lithium
of hydroxide and liberation of hydrogen. borohydride (LiBH,,).
LiH + H,OQ——> LiOH + H, 8LiH + Al,Cl, ——> 2LiAlH, + 6LiCl
Lithium aluminium
NaH + H,O——~> NaOH + H, hydride
Therefore, their aqueous solutions are basic. 2LiH + B,H, Pryether ,2LiBH,
Diborane Lithium borohydride
NaH reacts with diborane (B,H,) to form sodium
[Sa ore interesting facts borohydride as : -
F WEL LLL AIP 2NaH + B,H, —““* 2NaBH,
I, 1 Proton H* has very small size (about 10-” m Sodium borohydride
E. radius) and intense electric field. It binds to a The hydrides of lanthanides are also regarded as
water molecule to form hydrated proton or ionic hydrides. However, there is some doubt about
hydronium ion H,O*. This species has been their true ionic character. Their heats of formation are
detected in many compounds such as H,O*CI- quite high suggesting ionic character but these are
in the solid state. In aqueous solution, non-stoichiometric like metallic hydrides (discussed
hydronium ion is itself strongly hydrated in later on).
solution to give species like H,O,*, H,O,* and Uses of Ionic Hydrides
H,0,.. The ionic hydrides and their complexes (LiAIH,,
1 Heavy hydrogen or deuterium (D,) was LiBH,, NaBH,, etc.) are used as reducing agents. Since
separated from liquid hydrogen by fractional they evolve hydrogen when heated and, therefore,
evaporation by Harold C. Urey. For his ignite spontaneously, and are used as solid fuels.
contribution, Urey was awarded Noble Prize in 2. Molecular or covalent hydrides
Chemistry in 1934. These hydrides are formed by the combination
of elements of comparatively higher electronegativity
MODERN'S abc + OF CHEMISTRY
-XI

as of p-block elements. The common elements forming (iv) By reduction of certain compounds with
molecular hydrides are B, C, N, O, F, Si, P, 5, Cl, Ga, suitable reducing agents like LiAIH,, NaBH, (in ether
Ge, As, Se, Br, In, Sn, Pb, Te, I, Tl, Pb, At, etc. The solution).
bonds formed are mostly covalent in character. But in 4BCl, + 3LiAIH, ——~> 2B,H,+3AlICl,+3LiCl
some cases, as for example in HF, the bond may be Diborane
partly ionic in character as well. These hydrides,
therefore, have molecular lattices. The molecules are SiCl, + LiAIH, ——> SiH, + AICI, + LiCl
held together by weak van der Waals forces. In some Silane
cases, hydrogen bonds are also formed. In many cases
but not in all cases, the general formula for covalent SnCl, + LiAlH, Ether.
— LiCl + AICI, + SnH,
hydrides is EH, | where E stands for the symbol of a Stannane

metal and n is the group number.


GeCl, + LiAIH, —“**_, LiCl + AICI, + GeH,
The systematic names of these hydrides are Germane
usually formed from the name of the element and the
suffix -ane. For example, phosphane (PH,), silane 4H.AsO, + 3NaBH, ———> 4AsH, + 3H,BO,
(SiH,), germane (GeH,), etc. However, these are + 3NaQOH
popularly known by their common names such as General Characteristics of Covalent Hydrides
phosphine (PH,), water (H,O), ammonia (NH,), etc. It Some important general characteristics of
may be noted that according to systematic molecular hydrides are discussed below :
nomenclature water should be named as oxidane, 1. The molecular hydrides are soft.
ammonia as azane, etc. However, the scientists are
2. They have low melting and boiling points and,
reluctant to use these systematic names which are
are generally, volatile in nature.
popularly known for such a long time in chemistry.
Preparation of Covalent Hydrides 3. They have low electrical conductivity.
Covalent hydrides are prepared by the following 4. The properties of the covalent hydrides depend
methods : upon the electronegativity difference between
(.) By direct combination of elements, e.g., hydrogen and the atom bonded to it. For
example, the hydrides become increasingly
H,+F, —“ > 2HF acidic in character on moving from left to right
Electric discharge
2H, 4 O, es 2H,O along a given row in the periodic table. Thus,
while NH, is weak base, H,O is neutral and
Fe, Mo
HF is acidic. Similarly, in the next row while
3H, + No aK 20am 2NH,
PH, is a weak base, H,5 is a weak acid and
(it) By hydrolysis of metal borides, carbides, HCl is highly acidic.
nitrides, phosphides, etc.
Molecular hydrides are further classified according
Mg,B,+6H,O ———>» 3Mg(OH), + B,H,
to the relative number of electrons and bonds in their
Diborane
Lewis structures. These are electron deficient, electron
Ca,P, + 6H,O ——— 3Ca(OH), + 2PH,
precise and electron rich molecular hydrides.
Phosphine
Mg,N, +6H,O ———> 3Mg(OH), + 2NH, (i) Electron deficient molecular hydrides.
Ammonia
These have less electrons for writing their
AL,C, + 12H,O ———> 4Al(OH), + 8CH, conventional Lewis structures. For example, the
Methane
hydrides of group 13 are electron deficient compounds
(tit) By the action of acids on suitable binary and exist in polymerised forms such as B,H,
compounds : (diborane), (AIH,),, etc.

Al,C, + 12HClL_——> 3CH,+ 4AlCl, (it) Electron precise molecular hydrides. These
Aluminium Methane have sufficient number of electrons required for their
carbide conventional Lewis structures. For example, the
2Mg,B, + 4H,PO, —— B,H,, + 2Mg,(PO,), +H, hydrides formed by group 14 elements are electron
Magnesium Tetraborane precise molecular hydrides such as CH,. It has
boride tetrahedral structure.
FeS + H,sO, ——— H,5 + FesO, (zit) Electron rich molecular hydrides. These
Iron Hydrogen
have excess electrons as required by conventional
sulphide sulphide
Lewis structures. The excess electrons are present as
Ca,P, + 3H,SO, ——> 2PH, + 3CaSO, lone pairs of electrons. The hydrides of group 15-17
Calcium Phosphine form such compounds. For example, water has two
phosphide
and ammonia has one lone pair of electrons.
HYDROGEN

H :0: H :N: H transition metal lattices. For example, palladium


H H metal adsorbs hydrogen to form palladium hydride.
In some cases, the metals are used as cathodes in the
The presence of lone pairs on the highly
electrolysis of their aqueous solutions so that they
electronegative atoms like nitrogen, oxygen and fluorine
adsorb hydrogen during electrolysis. PdH, is prepared
result in the formation of hydrogen bonds in these
by this method.
molecular hydrides. Because of the hydrogen bonds in
General Characteristics of Metallic Hydrides
these molecules, they exist as associated molecules
such as (NH,),, (H,O),, (HF),, etc. As a result of 1. They are hard, have a metallic lustre, conduct
ageregation, they have abnormally high boiling points electricity and have significant magnetic properties.
and high permittivities. For example, NH,, H,O and 2. The densities of these hydrides are lower than
HF have abnormally high boiling points as compared those of metals from which they are formed because
to the boiling points of hydrides of each group (i.e., the crystal lattice expands due to inclusion of H,.
PH,, H,S and HCD. 3. These compounds are non-stoichiometric, and
3. Metallic or Interstitial Hydrides their composition varies with temperature and
Most of the d-block elements (i.e., transition pressure. The compositions of hydrides of titanium and
elements) and f-block elements form metallic hydrides zirconium, for example, are represented by TiH,
on reacting with hydrogen. Among the d-block (x = 1.5 to 1.8) and ZrH, (x = 1.30 to 1.75), respectively.
elements, the metallic hydrides of groups 3, 4 and 5 Some other examples are NiH, (x = 0.6—0.7), PdH,
are very common. In group 6, chromium alone forms (x = 0.6 — 0.8), VH)-., etc. The density of each
CrH. The metals of groups 7, 8 and 9 do not form compound is less than that of the metal itself and the
hydrides. This region of periodic table from group 7 to properties are not much different from that of the
9 is referred to as the hydride gap. The common metal.
examples of hydrides of groups 3 to 5 are: 4. They give out hydrogen easily and are strong
ScH,, YH,, YH,, LaH,, LaH,, TiH,, ZrH,, HfHi,, reducing agents. This suggests that hydrogen is
VH, VH,, NbH, NbH,, TaH. The /-block elements form present in the atomic state. These compounds are used
hydrides of limiting compositions of EH, and EH,. as industrial reducing agents.

Beryllium and magnesium of s-block elements also The f-block elements (i.e., lanthanoids and
form metallic hydrides. In these hydrides, hydrogen actinoids) also form non—stoichiometric interstitial
exists in the atomic rather than ionic form. These are hydrides, e.g., LaH,.,,YbH,-.,CeH,64,TbH,97.These
interstitial compounds as the hydrogen atoms occupy hydrides have lower densities than their respective
interstitial positions in the metal lattices. This is due metals.
to small size of hydrogen atoms compared to the metal Metallic hydrides can be used as hydrogen storage
atoms. Some workers regard them nearly as solid media. This property has potential for hydrogen
solutions. storage and as a source of energy for vehicles.
These are prepared by adsorbing hydrogen directly The characteristics of different hydrides are listed
at appropriate temperatures into the interstices of in Table 5.
Table 5. Characteristics of different hydrides.

Examples

Ionic Salt like solids, good Largely ionic LiH, NaH, CaH,
conductors in fused state
Molecular or Gaseous or volatile Covalent HF, H,0, CH,, H,S
covalent liquids, non-conductors
Metallic or Metallic appearance Partly ionic and ZrH, g; TiH 1.73? LaH
, 76

interstitial brittle, conductors, metallic bond


semiconductors
Sens MODERN'S abc + OF CHEMISTRY-XI

4LiH + AIC] —““— LiAlH, + 3LiCl


(anhydrous)
Polymeric Hydrides or Complex Hydrides
On an industrial scale, it is prepared by direct
In addition to above three main types of hydrides, combination of constituent elements under high
some hydrides are regarded as polymeric hydrides
pressure in the presence of ether.
and complex hydrides. These are briefly discussed
below : Li + Al+ 2H," LiAlH,
(1) Polymeric hydrides. These are formed by Properties. (i) It is a white, non-volatile
elements having electronegativity range between crystalline solid which is freely soluble in ethers. It
1.4 and 2.0. These hydrides consist of molecules held is stable upto 120°C.
together in two or three dimensions by hydrogen
(ii) It decomposes on heating liberating hydrogen.
bridges. These are generally solids. The common
examples of polymeric hydrides are (BeH,),,(MgH,),, 2LiAlH/gep—""» /2LiH + 2Al + 3H,
(AlH,),, (InH,),, (SiH), etc. The preparation and (iii) It is readily attacked by water and ethanol
properties of beryllium hydride are discussed giving hydrogen gas
below :
LiAIH, + 4H;0-> LiOH + Al(OH), + 2H,
Preparation. It was first prepared by the
reduction of BeCl, with LiH. LiAIH, #3C,H,OH ——> Li[AIH(OC,H,),] + 3H,
BeCl, + 2LiH ——— BeH, + 2LiCl (itv) It reduces compounds of boron, aluminium,
It can also be prepared by heating tertiary butyl silicon, tin, etc. to form hydrides.
beryllium at 100°C. SiCl, + LiAIH, —“*— SiH, + LiCl + AICI,
Be[(CH,),C], —“* > BeH, + 2(CH,),C= CH, SnCl, + LiAIH, —““ SnH, + LiCl + AlCl,
Isobutylene
AIC], + 3LiAIH, ———— 4AlH, + 3LiCl
Properties. (i) It is amorphous white solid. It is
stable upto 250°C and when heated above 250°C, it (v) It reduces many organic compounds. For
begins to evolve hydrogen gas. example, compounds containing carbonyl group
(iz) It is moderately stable in air or water but it (—CQ) are reduced to alcohols.
is rapidly hydrolysed by acids giving Hg. CH,CHO +2H —“":_, CH,CH,OH
BeH, + 2HCl ———Be(Cl, + 2H,
Acetaldehyde Ethanol
Structure. BeH, appears to be highly
Uses. (1) It is extensively used in organic
polymerised by means of BeHBe, three centre bonds
and inorganic chemistry as a versatile reducing
as shown below :
agent.
(it) It is used to prepare hydrides of elements
like B, Al, Sn, etc.

SOLVED EXAMPLES
(it) Complex hydrides. These are the compounds lL] Example 1.
which contain hydride ions co-ordinated to metal
Would you expect the hydrides of N,O and F to
ions. These are common in both the transition and
have lower boiling points than the hydrides of
non-transition metals. The most common examples
of complex hydrides are formed by the elements of their subsequent groups ¢ Explain. \N.C.E.R.T,
group IIJA such as B, Al, Ga, etc. For example, Solution:
lithium aluminium hydride (LiAIH,), sodium and The hydrides of N, O and F are NH,, H,O and HF
lithium borohydrides (LiBH,, NaBH,), etc. These respectively. On the basis of molecular masses of NH,, H,O
are briefly discussed below : and HF, their boiling points are expected to be lower than
Lithium aluminium hydride those of the subsequent group members (i.e., PH,, H,S,
HCl). However, due to high electronegativity of N, O and
Preparation. It is prepared by reacting lithium
F, their hydrides have strong intermolecular hydrogen
hydride with anhydrous aluminium chloride in ether.
bonding in their molecules.
HYDROGEN

As a result of hydrogen bonding, the boiling points of Distribution of water over the earth’s surface
NH,, H,O and HF are higher than the hydrides of their
subsequent group members, which have almost negligible
Water is the principal constituent of earth’s
hydrogen bonding. surface. The distribution of water over the earth’s
surface is not uniform. There is no permanent
L} Example 2.
surface water in desert regions while the oceans
Comment upon the reactions of dihydrogen with cover vast areas of 78% of the earth surface 1.e.,
(i) Chlorine 1.61 x 10° km?. They contain 97% of the available
(i) Sodium and water. The fresh water amounts to only 2.7%, most
(iit) Copper (IDoxide \N.C.E.R.T, of it is locked in Antarctic ice cap and in the Arctic.
Antarctic ice cap covers 1.5 x 10’ km? and contains
Solution:
2.5 — 2.9 x 10’ km? of fresh water.
(i) Dihydrogen reduces chlorine to chloride (Cl-) ion
and itself gets oxidised to H* ion in HCl. An electron pair
Fresh water lakes contain 1.25 x 10° km? of
is shared between H and Cl to form a covalent molecule water and more than half of which is in the four
of hydrogen chloride. largest lakes :
H,(g) + Cl,(g) ——— 2HCI (g) Baikal (26,000 km*), Tanganyika (20,000 km’),
(iz) Sodium reduces dihydrogen to form hydride (H~) Nayassa (13,000 km®) and Superior (12,000 km*).
ion and itself gets oxidised to sodium (Na*) ion. During this Fresh water is also present in many other smaller
reaction, an electron is transferred from Na to H leading lakes, springs and rivers. The purification and
to the formation of an ionic compound sodium hydride, recycling of water has now become major industry.
NatH. The production of fresh water from sea and other
saline water has been commercialised.
2Na(s) + H,(g) —He24, 2Na*H-(s)
(zi) Hydrogen reduces copper (II) oxide to copper metal
(in zero oxidation state) and itself gets oxidised to H,O, The estimated world water supply is
which is a covalent molecule. Source Percentage of total
41-2 0 0 +1 -2 Oceans 97.33
CuO + H,(g) ——> Cu(s) + H,O(/) Saline lakes and Inland seas 0.008
Polar ice and glaciers 2.04
lL] Example 3.
Ground water 0.61
Can phosphorus with outer electronic
Lakes 0.009
configuration 3s*3p* form PH gf IN.C.E.R.T)
Soil moisture 0.005
Solution:
Atmospheric water vapour 0.001
Though phosphorus exhibits +3 and +5 oxidation states,
it cannot form PH,. This is because of high enthalpy of
Rivers 0.0001
atomisation (AH) (435.89 kJ mol”) of dihydrogen and slightly Apart from water, hydrogen and oxygen form
negative electron gain enthalpy, A, (-72.8 kJ mol) of another oxide namely hydrogen peroxide which has
hydrogen, it acts only as a weak oxidising agent. Therefore, it the molecular formula H,O,.
can oxidise P to +3 oxidation state but not to its highest
oxidation state of +5. Thus, P forms PH, but not PH,.
Structure of Water Molecule and its Aggregates
The molecule of water consists of two hydrogen
WATER atoms bonded to oxygen atom by covalent bonds.
Water is essential to all forms of life. It is most Oxygen atom (Z = 8, 1s? 2s? 2n*) has six electrons in
common, abundant and easily obtainable of all chemical its valence shell. Two of these valence electrons form
compounds. It is a solvent of great importance and is bond pairs with two H atoms while the remaining four
electrons remain as two lone pairs. Therefore, the
regarded as a universal solvent. It is most studied of
oxygen atom in water is surrounded by four electron
all chemical compounds. It can be easily transformed
pairs and has tetrahedral geometry in which two
from liquid to solid and to gaseous states. It is the
positions are occupied by lone pairs as shown in
principal constituent of earth’s surface. It is an
Fig. 4 (6). Due to the presence of lone pairs, the
important component of animal and vegetable matter
ceometry of water is distorted and the
and plays a vital role in their life processes. Water
H—O—H bond angle is 104.5° which is less than the
constitutes about 65% of our body and is an essential
normal tetrahedral angle (109.5°). The geometry of
for its growth. We can live without food for some days the molecule is regarded as angular or bent. The
but cannot live without water. Some plants have about O—H bond length in water is 95.7 pm.
95% water.
MODERN'S abc + OF CHEMISTRY-XI

atoms; two by covalent bonds and two by hydrogen


bonds. Experimental studies have shown that liquid
water and ice consists of aggregates of varying number
LONE PAIR of water molecules held together by hydrogen bonds.
H:0:H The resulting structure of ice is open structure having
TWO BOND PAIRS a number of vacant spaces. Therefore, the density of
TWO LONE PAIRS ice is less. When the ice melts, some of the hydrogen
bonds are broken and the water molecules go in
(a) between the vacant spaces in the structure. As a result,
the structure of liquid water is less open than structure
of ice. Therefore, the density of water is more than
Fig. 4. Structure of water molecule.
that of ice. In other words, the density of ice is less
than that of water and ice floats over water.
Now, in water each O—H bond is polar because
of the high electronegativity of oxygen (3.5) in It may be noted that water has maximum density
comparison to that of hydrogen (2.1). The resultant (1 g cm) at 4°C (277 K). This property is very helpful
dipole moment of water molecule is 1.84D. for aquatic life. In severe cold, the upper layer of the
sea water freezes. The heavier water (density more
Hydrogen Bonding in Water and Ice
than that of ice) is present below the surface of ice.
Because of the polar nature of water molecule, The ice layer formed on the surface of a lake in winter
the water molecules are held together by intermolecular does not sink to the bottom. In fact, this provides a
hydrogen bonds in water. As a result, the water thermal insulation for the water below it. The sea
molecules are joined together in an extensive three animals can live safely in water under these conditions.
dimensional network (Fig. 5). In this arrangement, each Thus, it ensures the survival of the aquatic life.
oxygen is tetrahedrally surrounded by four hydrogen

Tetrahedral
a arrangement
around oxygen

Fig. 5. Hydrogen bonding in water and ice.

Physical Properties of Water (iii) Water has strong hydrogen bonding in its
Many of the properties of water are due to molecules and exists as associated
hydrogen bonding in their molecules. The important molecules, (HO)...
physical properties of water are : (iv) Water is polar in nature having dipole
moment of 1.84D. It has solubility for a
(i) Water is liquid with freezing point of
273.2 K and boiling point of 373.2 K. variety of substances and is regarded as a
universal solvent.
(ii) Water has maximum density of 1.00 g em™
Some of the important physical properties of
at 277 K (4°C).
water (H,O) and heavy water (D,O) containing isotope
of hydrogen are listed in Table 6.
HYDROGEN

Table 6. Some physical constants of H,O and D,O at 298 K.


Constant Ordinary water Heavy water
(H,O) (D,O)
Molecular mass (g mol") 18.0151 20.0276
Maximum density (g cm) at 298 K 1.0000 1.10859
Viscosity (centipoise) 0.8903 1.107
Melting point (K) 213.2 276.8
Boiling point (K) ofd.2 374.4
Enthalpy of fusion (kJ mol!) at 273K 6.01 6.28
Enthalpy of vaporisation (k.J mol~') at 373K 40.66 41.61
Enthalpy of formation (kJ mol") —285.9 —294.6
Ionisation constant 1.008 x 10-4 1.95 x 1074
Dielectric constant 78.39 78.06
Electrical conductivity (293 K) (ohm~! cm~*) 5.7 x 10% —

Some of the important observations of formation (AjH = —285.9kJ mol"). At about 1500 K,
regarding the physical properties of water are : its decomposition to elements is only 2%.
(i) The freezing point, boiling point, heat of
fusion and heat of vaporisation of water are higher However, when electricity is passed in the presence
as compared to the hydrides of the other members of a small amount of an acid or an alkali water
of same group (16) such as H,S, H,Se, H,Te, etc. decomposes to its elements.
This ts due to the presence of hydrogen bonding in 2H,,0( 1) Electric current 2H, (g) 4 O,(g)
H,O molecules which is absent in the hydrides of
(at cathode) (at anode)
the rest of the members.
2. Acid-base character : Amphoteric nature of
(11) Water has the capacity to dissolve most of water. Water is a weak electrolyte and therefore, ionizes
the inorganic substances and is, therefore, regarded to a small extent to give H,O"(aq) and OH (aq) ions.
as a universal solvent. Most of the inorganic Therefore, pure water has a small but measurable
substances which are of ionic nature dissolve in electrical conductivity and it dissociates as :
water. Even few organic substances such as urea,
H,O + HO ————- H,0"_— ++ OH”
alcohols, carbohydrates, etc. also dissolve in water.
Hydronium ion
This is because of the presence of intermolecular
hydrogen bonding in the molecules of the solute K,, = 1.0 x 10-* mol” L? at 298K.
and water. Under very high pressure and Water can act both as an acid and a base and is
temperature, water behaves as a non-polar solvent said to be amphoteric. According to Lowry Bronsted
and dissolves organic compounds while common concept, it can act as an acid by losing a proton and as
inorganic compounds are not dissolved. a base by accepting a proton. For example,
When water freezes, it forms ice which is the It behaves as a base towards hydrochloric acid or
crystalline form of water. Nine crystalline forms of hydrogen sulphide (H,5).
ice are known. At atmospheric pressure, ice HCl (aq) + HO (2) => CF (aq) + H,0* (aq)
crystallises in the normal hexagonal form. (acid) (Base)
However, at very low temperature, it condenses to H,S (aq) + HO () ==>. _H,,0* (aq) + HS (aq)
cubic form. Many salts crystallise as hydrated salts (acid) (Base)
from aqueous solutions. This type of association of
While it behaves as an acid towards ammonia,
water is essentially of five types such as
co-ordinated water, hydrogen bonded water, H,0 (J) + NH, (aq) =———> NH, *(aq) + OH (aq)
lattice water, zeolite water and clathrate (Acid) (Base)
water. In general, water can act as a base towards acids
stronger than itself and as an acid towards bases
Chemical Properties of Water stronger than it. The auto protolysis of water may be
Water displays a versatile and unique chemical represented as :
behaviour. It behaves as an acid, as a base, an oxidant, H,0 (J) + H,0 (2) => H,0* (aq) + OH (aq)
a reductant and as ligand to metals. Some of its
Acid, Base : Acid, Base :
characteristics are given below :
It is therefore of great importance in acid base
1. Stability of water. Water is quite stable and
chemistry as discussed in Unit 7.
does not dissociate into its elements even at high
temperatures. This is due to its high negative enthalpy However, water is neutral towards litmus and its
pH is 7.
MODERN'S abc + OF CHEMISTRY-XI

3. Oxidising and reducing nature. Water can SO, (g) + H,O (1) ——> H,SO, (aq)
act both as an oxidising and a reducing agent in its Sulphuric acid
chemical reactions. Therefore, aqueous chemistry is
very interesting for oxidation-reduction or redox PBr,(/)+3H,O(@) ——~ H,PO.(aq) + 3HBr(aq)
reactions. Phosphorous
(1) As an oxidising agent: Water can be reduced acid

to H, by highly electropositive metals (having E° value SiCL,(Z) + 2H,OW@) ——— 3810,(s) + 4HCl(aq)
(M™* |M) below — 0.88 V). Hydrochloric acid
2H,OW) + 2e ——> 20H (aq) + H,{g) AICI, (s) + 6H,0 (1) ——> [AI(H,0),]** (aq) + 8CT (aq)
E° = — 0.83 V for [OH] = 10°’ M (it) Water hydrolyses the carbides, hydrides, nitrides
Therefore, with active metals (highly and phosphides of some metals.
electropositive) such as Na, Ca etc. (whose electrode CaC, + 2H,0 +> Ca(OH), + C,H,
potential is less than — 0.83 V), water can be easily Cal. carbide Acetylene
reduced to dihydrogen : Al,C, + 12H,O.——> 4A\OH), + 3CH,
2Na(s) + H,O(W) ——> 2Na0OH (aq) + H,(g) Aluminium Methane
Oxidising agent carbide
Thus, it is a great source of dihydrogen. CaH, (s) + 2H,O (1) ———> Ca(OH), (aq)
+ 2H, (g)
Similarly, when steam is passed over red hot coke Cal. hydride Calcium hydroxide
(at 1273K), syn gas is formed. Ca,P, + 6H,0 ——> 3Ca(OH), + 2PH,
C(s) + H,O@) #8%, CO) + Hyg) Cal. phosphide Phosphine
Syn gas Ca,N, (s) +6H,O (1) ——> 3Ca(OH), + 2NH,
In this reaction also, water acts as an oxidising Calcium nitride Ammonia
agent and itself gets reduced to dihydrogen. Mg,N, + 6H,O ——> 3Mg(OH), + 2NH,
(it) As a reducing agent : Water can also be Magnesium nitride Ammonia
oxidised according to the reaction : AIN + 3H,O ——> AKOH), + NH,
O,(g) + 4H* (aq) + 4e° ——> 2H, O(/) E° = + 1.23 V for Aluminium nitride Ammonia

[H*] = 1077 M 5. Hydration reactions. Water has strong ability


Therefore, water reacts with highly to form compounds with some metal salts known as
electronegative elements behaving as reducing agent. hydrates. These hydrates can be classified into three
For example, with fluorine, water acts as a reducing types :
agent and hence itself gets oxidised to O, or Os. (t) Water molecules are co-ordinated to metal ton
2F, (g) + 2H,O (1) ——> 4H*(aq) + 4F (aq) + O,(g) in a complex.
Reducing agent Ni?* + 6H,O0 ——> [Ni(H,0),|**
or 3F, (g) + 3H,0 (1) ——>0, (g) + 6H" (aq) + 6F (aq) Some other examples are [Cr(H,O),|Cl,,
Water reacts with chlorine and bromine similar [Fe(H,O),ICI1,, ete.
to reaction with fluorine. (it) Water molecules may be hydrogen bonded to
Another typical example of this type of reaction certain oxygen containing anions. For example, in
are oxidation of H,O to O, during photosynthesis. CuSO, . 5H,O, four H,O molecules are co-ordinated
to Cu** ion while the fifth H,O molecule is hydrogen
6CO, (g) + 12H,O0) gan > CeH,,0,(aq) + bonded to SO ra ion.
6H,O(/) + 60,(g)
2+

ae O-H,
4, Hydrolysis reactions. Water has very strong
solvating power due to its high dielectric constant.
Therefore, it acts as an excellent solvent. Because of H—O HO O
ereat affinity of many elements for oxygen, water can
easily hydrolyse many ionic and covalent compounds \ i’ |a / eaoan VA
of non-metals. For example,
a 2 bond UA
(1) Water hydrolyses oxides and halides of non-
metals forming their respective acids.
CaO (s) + H,O (1) ———~ Ca(OH,) (aq) H H
Calcium hydroxide
SO, (g) + H,O () ——> H,SO, (aq) (uit) Water molecules may occupy the interstitial
Sulphurous acid sites (voids) in the crystal lattice. For example,
P,O,, (s) + 6H,0 (1) ——> 4H.PO, (aq) BaCl, . 2H,O.
Phosphorus Phosphoric acid
pentoxide
HYDROGEN

HEAVY WATER 4, Action with carbides, nitrides, phosphides,


Chemically heavy water is deuterium oxide (D,O). It arsenides, etc. The reactions of D,O with these
compounds are similar to those of water. For
was discovered by Urey in 1932. He showed that ordinary
example :
water contains small proportion (about 1 part in 5000) of
D,O. Itcan be prepared by the prolonged electrolysis of CaC, +2D,0 ——> Ca(OD),+ C,D,
water or by fractional distillation. When water is Calcium carbide Deutero acetylene
electrolysed, H, is liberated much faster than D, and the Al,C, +2D,0 ——> 4Al(OD),+ 3CD,
remaining water becomes enriched in heavy water, D,O. Aluminium carbide Deutero methane
If the process is continued until only a small volume Mg,N, + 6D,0 ——» 3Mg(OD), + 2ND,
remains, then almost pure D,O is obtained. It has been Magnesium nitride Deutero ammonia
estimated that about 29,000 L of water must be Ca,P, +6D,0 ——> 3Ca(OD), + 2PD,
electrolysed to get 1L of D,O that is 99% pure.
Calcium phosphide Deutero phosphine
PHYSICAL PROPERTIES Na,As + 3D,0 ———~ 3NaOD +AsD,
Sodium arsenide Deutero arsine
Like ordinary water, it is colourless, odourless and
tasteless liquid. The physical constants of heavy water 5. Electrolysis. On electrolysis, heavy water
are generally slightly higher than ordinary water (H,O). decomposes to give deuterium and oxygen, which are
This is quite expected because the molecular mass of liberated at cathode and anode respectively.
heavy water is more than ordinary water. The dielectric 2D,0 ——— 2D, + O,
constant of D,O is slightly lower than that of H,O and Deuterium
therefore, ionic compounds are less soluble in D,O than 6. Exchange reactions. When compounds
in H,O. Some of the important physical constants of containing hydrogen are treated with D,O, hydrogen
heavy water are compared with water in Table 6. undergoes exchange for deuterium.
CHEMICAL PROPERTIES NaOH + D,0 ——> NaOD + HOD
NH,Cl + 4D,0 ——~ ND,Cl + 4HOD
Heavy water differs only slightly from ordinary
water in its chemical properties. However, the HCl +D,0 =— DCl +HOD
reactions of D,O are slightly slower than those of H,O. Deutero chloride

Some of the important chemical reactions of heavy CHCl, +D,0 == CDCl, + HOD
water are discussed below : Chloroform Deutero chloroform

1. Action with metals. D,O reacts with alkali and 7. Formation of deuterates. Like ordinary water,
alkaline earth metals slowly liberating deuterium. heavy water combines with many compounds as heavy
water of crystallization. These heavy water hydrates
2Na + 2D,0 ——— 2Na0D +, A) 2
are also called deuterates. For example,
Sodium
deuteroxide CuSO,.5D,0, MgSO,.7D,0, NiCl,.6D,0, CoCl,.6D,0,
Ca + D,O ——> Ca(OD), + D9 Na,5O,.10D,0, ete.
Calcium 8. Biological properties. Heavy water is injurious
deuteroxide to plants, animals and human beings because it slows
2. Action with metallic oxides. Basic oxides of down the rates of reactions occurring in them.
metals like Na, Ca, etc. dissolve in D,O forming heavy Thus, heavy water does not support life so well
alkalies. as does ordinary water.
Na,O + D,O0 »———> 2Na0OD Uses of heavy water:
Sodium deuteroxide 1. Asamoderator. Heavy water has been finding
CaO + D,O ~-——> Ca (OD), use in nuclear reactors as a moderator
Calcium deuteroxide because it slows down the fast moving
neutrons and therefore, helps in controlling
3. Action with non-metallic oxides. Non-
the nuclear fission process.
metallic oxides such as phosphorus pentoxide (P,O,),
2. Asatracer compound. Heavy water has also
sulphur trioxide (SO,) etc. readily dissolve in heavy
been used as a tracer compound to study
water forming their corresponding deuteroacids.
mechanism of many chemical reactions.
P,0,+3D,0 —> 2D,PO, 3. Innuclear magnetic resonance. Heavy water
Deutero phosphoric acid is used as one of the reference in nuclear
§0O,+D,0 —~> DSO, magnetic resonance spectroscopy.
Deutero sulphuric acid
MODERN'S abc + OF CHEMISTRY-XI

4, For the preparation of deuterium. Heavy (b) Permanent hardness. This hardness is due to
hydrogen or deuterium can be obtained by the the presence of chlorides and sulphates of calcium and
electrolysis of heavy water or by its magnesium dissolved in water. Since the hardness
decomposition by sodium metal. cannot be easily removed, it is therefore, called
permanent hardness. The permanent hardness is also
HARD AND SOFT WATER
called non-carbonate hardness.
Natural water contains dissolved salts. Depending
Softening of Water
upon its behaviour towards soap solution, water may
The process of the removal of hardness from
be classified as soft water or hard water.
water is called softening of water.
(a) Soft water. Water which produces lather with
I. Removal of temporary hardness. Temporary
soap solution readily is called soft water. For example,
hardness of water is due to the presence of
distilled water, rain water and demineralised water.
bicarbonates of calcium and magnesium. It can be
(6) Hard water. Water which does not produce lather removed by the following methods :
with soap solution readily is called hard water. For
1. By boiling. The temporary hardness of water can
example, sea water, river water, well water and tap water.
be removed by boiling the hard water in large boilers.
Cause of Hardness of Water During boiling, the bicarbonates ofcalcium and magnesium
The hardness of water 1s due to the presence of decompose into insoluble carbonates and give CO,. The
the bicarbonates, chlorides and sulphates of calcium insoluble carbonates can be removed by filtration.
and magnesium. These salts dissolve in water when
Ca(HCO,), —#£2*-> CaCO,! + CO,1 + H,O
it passes through the grounds or rocks. Hard water
Cal. bicarbonate ppt
does not produce lather because the cations (Ca** and
Meg**) present in hard water react with soap to form Mg(HCO,), —“““>MgCoO,\ + CO,T + H,0
insoluble precipitates. Soaps are sodium salts of higher Mag. bicarbonate ppt
fatty acids like stearic acid (C,,H,.COOH), oleic acid 2. Calcium hydroxide method (Clark's method).
(C,, H,,COOH) or palmitic acid (C,,H,, COOH). When This process is used on a commercial scale. In this
soap is added to hard water, these anions combine with process, calculated amount of lime (calcium hydroxide)
Ca?*+ and Mg** ions to form calcium and magnesium is added to temporary hard water. The soluble
salts which are insoluble in water. bicarbonates present in water are converted into
M?* + 2C,,H,,;COONa
—> (C,,H,,COO), MA + 2Na* insoluble carbonates which settle at the bottom of the
From Sodium stearate Metal stearate tank and are removed by filtration.
hard water (soap) (precipitate) Ca(HCO,), + Ca(OH), ——— 2CaCO, + 2H,O
where M = Ca or Mg (Soluble) Lime (Insoluble)

Therefore, no lather is produced until all the Mg(HCO,), + Ca(OH), t——> MgCO,J + CaCO,1
calcium and magnesium ions are precipitated. This (Soluble) Lime (Insoluble) + 2H,O
also results into wastage of lot of soap. In this method, excess of lime should not be
Therefore, hard water is unsuitable for laundry added, otherwise water will again become hard due
washing and dyeing. In addition hard water is also to absorption of CO, from the atmosphere by reacting
harmful for steam boilers. The inner surface of the with unused excess calcium hydroxide. Calcium
boiler gets crusted with scale known as boiler scale. It bicarbonate formed will make water hard.
is mainly calcium sulphate, calcium carbonate and
magnesium oxychloride. The deposition of scale
II. Removal of permanent hardness, Permanent
decreases the efficiency of the boiler and also damages
hardness cannot be removed by boiling. It can be
it. Therefore, hard water should not be used in boilers.
removed by the following methods :
Type of Hardness of Water
1. By washing soda method. In this method,
The hardness of water is of two types : temporary water is treated with a calculated amount of washing
hardness and permanent hardness. soda (Na,CO,) which converts the chlorides and
(a) Temporary hardness. Thistype of hardness in sulphates of calclum and magnesium into their
water is due to the presence of bicarbonates of calctum respective carbonates which get precipitated.
and magnesium dissolved in it. The hardness is called CaCl, + Na,CO, ———> CaCO, + 2NaCl
temporary because it can be very easily removed by
ppt
simply boiling the hard water for some time. Temporary
hardness is also called carbonate hardness.
MgSO, + Na,CO, ——~> MgCO,J + Na,SO,
ppt

WWW.JEEBOOKS.IN
HYDROGEN

Thus, water becomes free from the Ca?* and Mg? The calcium and magnesium ions remain attached
ions and becomes soft. to the zeolite and the water becomes soft. The soft
2. lon exchange method. This is a modern method water contains sodium salts (which are soluble in
employed for the softening of hard water. In this water). These sodium salts do not precipitate the soap
method, the ions responsible for the hardness of water and do not interfere with lather formation.
are exchanged by certain less damaging ions present
Regeneration of Permutit. As the process
in some chemical compounds called ion-exchangers.
These exchangers are of two types : continues, the zeolite gets exhausted because the
(A) Inorganic cation exchangers (Permutit or whole of sodium zeolite gets converted to calcium or
zeolite method). These are complex inorganic salts like magnesium zeolite. It may be regenerated by passing
hydrated sodium aluminium silicate, Na,Al,Si,O, .« H,O 10% solution of sodium chloride through it. The
which have interesting property of exchanging cations following reactions take place :
such as calcium and magnesium ions in hard water with CaZ + 2Nat———> Na,Z + (Catt
sodium ions. These complex salts are known as zeolites.
(exhausted (regenerated
These can be either naturally occurring or artificially zeolite) zeolite)
synthesised substances. Their technical name is
permutit. For artificial synthesis of permutit, a mixture MgZ + 2Nat——> Na,Z + Mg”
of soda ash (Na,CO,), sand (Si0,) and alumina (Al,O.) (exhausted (regenerated

is fused. The product is washed with water to remove zeolite) zeolite)

soluble impurities leaving behind a porous mass of The regenerated sodium zeolite can be used again
permutit. for the softening of water.
(B) Synthetic resins or organic ion exchangers
Hard Water —+ ——- 7 +—_ NaCl Solution (ion exchange resins). These are insoluble polymeric
solids having giant hydrocarbon network containing
reactive acidic or basic groups. These are
manufactured in bead form and are permeable to
== 2-2 —- —
—— i
—=——_— oo ——
=—-=-2-—->-—_—- — ——_— i i
—_— =] =| = = = —— =

water. These are called ion exchange resins. These


-— — SS — = =- = =—-— =
_— a ee eee
— ee eee ee

Tg lydglopleglig! bYgllyE ESE


——j—-->—_—
—ee ee ee Soft Water
-= = » © @ 8 =
=—<—
=
8
©
5
=< —-—
&
5
§
#8
8

#
&
8
are superior to zeolites because they can remove all
= © = © © @ = 5 &® 5 8

types of cations as well as anions present in water.


Ce a a: =» & # & @& &
Lt : 8 8 @ @ # i * it #«# # @
* & #&# & & & : f+ © @& @& @
i ' @#, + = @&
. *« « i s @

The resulting water is known as demineralised or


= 8s ¥ 8 8 #
Cr | |
= 8s fF 8s 8 8) fF @ 8 4 8 8
= 8 #8] © 8 8 | fF &e 8 @#]) ee mo oe OU
=» & & 5 & &
* & & & & & - + = © & @
= 5 #& 5 @ # &

deionised water.
. os 8s 8 @ 8
= 8 8 8 8 © * 6 #& & & &
= 8 84, © @ - ff

Ce, ee ee eee a]
= 8 @8]8 8 # 8
= 8 fF 8 8 8) fF 8 8 4 8 8

These are essentially of two types :


= 8 8 8 #8 8 | fF &e 5 # 5 © & @

SE werc eae? Rec etsoe —marE Permutit


= = wl es 8 @ = = wle
@ #8 @
= 8 © 8s 8

(1) Cation exchange resins. These contain giant


= 8 #8 | & @& #& @
Lt i} ' i} * i i i i
Lt ' Lt i = *#& 6 #) i & & @

=e
:
Ss 8S, hydrocarbon framework attached to acidic groups such
= ert et
as —COOH (carboxyl) or —SO,H (sulphonic acid).
lea
| Sr
ae =.
le ——" v. These are capable of exchanging the H* with the
hosse
[Seleeessia eecss ee, .
cations of the hard water and are called cation
atMint
5 a
ne
5505
-
‘a >
Waste Peeseaeeac et
: ee MOOS

exchange resins or cation exchangers. These may


Fig. 6. Permutit method for softening of hard water. be represented as resin —H”*.

The permutit is loosely packed in a big tank over (11) Anion exchange resins. These resins contain
a layer of coarse sand (Fig. 6). Hard water is introduced giant hydrocarbon framework attached to basic groups
into the tank from the top. Water reaches the bottom such as —OH™ (hydroxide) or —NH, (amine) group.
of the tank and then slowly rises through the permutit These groups are usually present in the form of
layer in the tank. The cations present in hard water substituted ammonium hydroxides as
are exchanged for sodium ions.
Resin — NH,(s) + H,OW/) —- Resin — NH,"OH (s)
Na,Z + Ca’* ——>» CaZ +2Nat* Therefore, these may be represented as resin
Sodium (from hard Calcium
—NH,*OH™ or simply as Resin —OH’. These resins
zeolite water) zeolite
are capable of exchanging the OH “ions with the anions
Na,Z + Mg** ——>» MgZ_ +2Na*
of hard water and are called anion exchange resins
(from hard Magnesium
or simply as anion exchangers.
water) zeolite

where / = Al,Si,0, Xx H,O


MODERN'S abc + OF CHEMISTRY-XI

Method of removal of hardness by organic The exhausted resin in the second tank is
exchanger regenerated by treatment with moderately
The hard water is first passed through abed of cation concentrated solution of sodium hydroxide.
exchange resin (Fig. 7). All the cations present in hard Resin-NH,*Cl’ + NaOH ——>
water will exchange with H” ions of the resin as : Exhausted resin Resin-NH,*OH™ + Nat+Cr

2Resin — H* + 2M*(aq) —> M (Resin), + 2H* Regenerated resin

(Exhausted resin) 3. By Calgon’s process (Sequestration). This is


(M = Ca*t, Mg**) the most modern method of the softening of water. In
e.g., 2Resin — H+ + Ca** + 2Cl--—> this method, the harmful Ca** and Mg** ions present
(in hard water) Ca(Resin), + 2H* + 2Cl- in hard water are sequestered (rendered ineffective)
(Exhausted resin) in the form of soluble complexes by adding some
2Resin — H* + Mg”* + SO,277,—> substances. For example, hard water is treated with
(in hard water) Mg(Resin),+2H* + SO? sodium polymetaphosphate (NaPO,), (where n is as
(Exhausted resin) high as 1000), commercially known as calgon
Therefore, water which comes out from the bottom (meaning calcium gone). The commonly used calgon
of the first tank will be richer in H* ions. This water is is sodium hexametaphosphate, Na,P,O,, or
then passed through a bed of anion exchange resin Na, [Na,(PO,),]. Calgon combines with calcium and
which exchanges its OH” with CI ions and SO ram ions magnesium ions to form complex ions which are
present in hard water as : soluble in water as :
Resin — NH,* OH” + Cl—— Resin NH,*CI" + OH™ Na, [Na,(PO,),] + 2Ca** ——> Na, [Ca, (PO,),] + 4Na*
in hard water Exhausted resin Calgon Complex salt (soluble)

2Resin — NH,* OH" + SO,7;—> Na, [Na, (PO,),] + 2Mg’*—> Na, [Mg, (PO, ),] + 4Na*
(Resin-NH,"*), SO,” + 20H7 Calgon Complex salt (soluble)

Thereafter, the H* ions (formed in the first tank) The complexed calcium and magnesium ions do
combine with the OH ions (formed in the second tank) not form any precipitate with soap. Therefore, water
to produce water. can easily form lather with soap.
H* + OH —— H,0 HYDROGEN PEROXIDE
Thus, water obtained by this method is free from
Hydrogen peroxide was discovered by a French
all types of cations as well as anions and is called
chemist J. L. Thenard in 1818. Its molecular formula
deionised water.
is H,O,.
Regeneration of resins. The exhausted resin in Preparation of hydrogen peroxide.
the first tank is regenerated by treatment with
1. Hydrogen peroxide can be prepared in the
moderately concentrated hydrochloric or sulphuric acid.
laboratory by the action of cold, dilute sulphuric acid
(Resin),Ca + 2HCl—~> CaCl,+2 Resin—H* on sodium or barium peroxide.
(exhausted resin) (regenerated resin)

H*, Cl’. SO,” ——>


Hard water

Dil. HCI for Dil. NaOH for


—>
regeneration
on regeneration
——_ | — SS SES HS SS SS
a ne ea a

Cation _ i

_ ee Anion exchange
=——_ S| S| S| HS SH SS SS =
—— eee eee

exchange resin
tigi resin
S——_—_
S-— SY SH SH SS SK SKS =
eeee ee

v
Waste —- Soft water

Fig. 7. Removal of hardness by the organic ion exchanger.


HYDROGEN

(a) From sodium peroxide (Merck’s process).


Hydrogen peroxide is prepared by adding calculated
2H,SO, “55 _,HO,S00S80,H (aq)
amounts of sodium peroxide to ice cold dilute (20%)
or H,S,O, (aq) + H,(g)
Peroxodisulphuric acid
solution of sulphuric acid. The addition is carried out
slowly in small amounts with constant stirring. The electrolysis occurs as :

Na,O, + H,SO, ——— Na,SO, + H,O, H,SO, ——> HSO, + Ht


Anode Cathode
Sodium peroxide
At cathode
Upon cooling, crystals of Na,5O,.10H,O separate
out. The crystals of Na,SO,.10H,O are decanted 2H* + 2e° —+~— 2HMoor H,
leaving behind solution of hydrogen peroxide. In this At anode 2HS0O,” &——= H,8,0¢, + ae
method, 30% hydrogen peroxide is obtained. Peroxodisulphuric acid
(b) From barium peroxide. Hydrogen peroxide Peroxodisulphuric acid (formed around anode) is
was first prepared by J.L. Thenard in 1818 by drawn off from the cell and hydrolysed with water to
acidifying barium peroxide and removal of excess give hydrogen peroxide.
water by evaporation under reduced pressure. In this
H,S,0, +2H,O ——> 2H,SO, +H,0,
method, a paste of hydrated barium peroxide is
prepared in ice cold water and is treated with about The resulting solution is distilled under reduced
20% ice cold solution of sulphuric acid. pressure when hydrogen peroxide gets distilled while
sulphuric acid with high boiling point, remains
BaO,.8H,O(s) + H,SO,(aq) —> undistilled.
Hydrated barium BaSO 4(S)+ H,0, (ag )+ 8H,O )
Recent modification. The above method has been
peroxide (ppt)
modified by using equimolar mixture of sulphuric acid
The white precipitate of barium sulphate is and ammonium sulphate for electrolysis.
removed by filtration leaving behind about 5% solution
(NH,),SO, + H,SO,——> 2NH,HSO,
of hydrogen peroxide. In this method, anhydrous
Ammonium Sulphuric Ammonium
barium peroxide is not used because barium sulphate sulphate acid hydrogen
forms a protective layer around unreacted barium sulphate
peroxide which prevents its further participation in 2NH,HSO, =——— > 2NH,SO, + 2Ht*
the chemical reaction.
Anode Cathode
Limitation of the method. The hydrogen peroxide Atcathode 2H*+2e —M—-> 2HorH,
solution prepared by this method contains some heavy
Ba”* ions. These catalyse the decomposition of hydrogen
Atanode 2NH,SO, ——~> (NH,), S,0, + 2e7
Ammonium persulphate
peroxide and, thus, the solution cannot be stored for a
Ammonium persulphate is withdrawn and
long time. To prevent this, sulphuric acid is replaced
distilled with water to get hydrogen peroxide
by phosphoric acid. In this method, barium phosphate
(NH,). 5,0, + 2H,0 ———~ 2NH,HSO, + H,O,
gets completely precipitated and the solution does not
contain any free Ba”* ions to catalyse the decomposition This method is now being used for the
of hydrogen peroxide. laboratory preparation of D,O,.
K,S,0,(s) + 2D,0(1) —— 2 KDSO,(aq) + D,O,(2)
3BaO, .8H,O + 2H,PO,——> Ba,(PO,), + 24H,0 + 3H,0,
(6) From 2-Ethyl anthraquinol. On an
(ppt)
industrial scale, hydrogen peroxide can be obtained
Manufacture of Hydrogen Peroxide by the auto-oxidation of an organic compound, 2-ethyl
On a commercial scale, hydrogen peroxide can be anthraquinol.
prepared by the following methods : | O, (air)
2-Ethyl! anthraquinol Oxidised product + H,O,_
(a) By electrolysis of 50% H,SO, solution. In this H,/Pd
method, a 50% solution of sulphuric acid is electrolysed The method involves the following steps :
at high current density in an electrolytic cell when (1) 2-Ethyl anthraquinone is dissolved in benzene
peroxodisulphuric acid is formed at the anode. and hydrogen gas is passed through the solution in
MODERN'S abc + OF CHEMISTRY-XI

the presence of paladium (Pd) catalyst when the Further the decomposition of H,O, is catalysed by
compound is reduced to 2-ethyl anthraquinol. presence of heavy metal ion impurities, dust and rough
and uneven surfaces. The concentration may be done
by the following steps :
(t) Evaporation on a water bath. The dilute
solution of hydrogen peroxide is transferred to an
evaporating dish and warmed carefully on a water bath.
In this process about 30% solution of H,O, is obtained.
(it) Dehydration in a vacuum desiccator. The
above solution of hydrogen peroxide is placed over
2-Ethyl anthraquinone concentrated sulphuric acid in a vacuum desiccator
(Fig. 8). The water vapours are absorbed by
concentrated sulphuric acid and, thus, about 90%
concentrated solution of hydrogen peroxide is obtained.
(iii) Distillation under reduced pressure. The
90% solution of hydrogen peroxide is then distilled
under reduced pressure (10-15 mm). During this
OH process, water distils over at 303-313 K and 99% pure
2-Ethyl anthraquinol
hydrogen peroxide is left behind.
(11) The reduced product is dissolved in a mixture (tv) Removal of last traces of water. The 99%
of benzene and cyclohexanol and upon passing air, it solution of hydrogen peroxide is cooled in a freezing
is oxidised back to 2-ethyl anthraquinone and mixture of solid carbon dioxide and ether. As a result
hydrogen peroxide is produced. of this, crystals of hydrogen peroxide separate out
OH which are removed, dried and remelted. This gives
CoH completely pure hydrogen peroxide.
Storage of hydrogen peroxide. The following
O, precautions must be taken while storing hydrogen
(From air) peroxide :

OH
2-Ethyl anthraquinol

Hydrogen
peroxide
Vacuum desiccator

Conc. H,SO,

O
2-Ethyl anthraquinone Fig. 8. Vacuum desiccator.
The 2-ethyl anthraquinone formed can be used
(a) It must be kept in wax lined coloured bottles
again in the first step. In short, in this process only
because the rough glass surface causes its decomposition.
dihydrogen and atmospheric oxygen are used up to
form hydrogen peroxide and 2-ethyl anthraquinone is (6) A small amount of phosphoric acid, glycerol or
not used up. acetanilide is generally added which retard the
The net reaction is decomposition of H,O,. These are also called negative
H,+0, ——> H,0O, catalysts.
This process is, therefore, very cheap and is widely Properties of Hydrogen Peroxide
used for the manufacture of hydrogen peroxide.
Physical Properties
Concentration of Hydrogen Peroxide (1) Pure hydrogen peroxide is a thick syrupy
The hydrogen peroxide obtained (about 19%) by the liquid with pale blue colour.
above methods is extracted with water and the
(it) It is more viscous, less volatile and dense
aqueous solution is concentrated. It may be noted that
than water.
the concentration of the solution cannot be done by
simple boiling because, hydrogen peroxide decomposes (iii) Its density is 1.44 g cm~.
below its boiling point. (tv) Its melting point is 272.4 K and boiling point
2H,0, —> 2H,0 + O, is 358 K at 68 mm of Hg pressure.
HYDROGEN

(v) It is completely miscible with water, alcohol 3. Oxidising and reducing character.
and ether in all proportions. Itformsa hydrate Hydrogen peroxide behaves as an oxidising agent as
with water as H,O,.H,O (m.p. 221 K). well as reducing agent in both acid and alkaline
solution. The oxidation state of oxygen in hydrogen
The important physical characteristics of H,O,
peroxide is —1. It can, therefore, be oxidised to O, (zero
are given in Table 7.
oxidation state) or reduced to H,O or OH” (—2 oxidation
Table 7. Physical characteristics of H,O, state for oxygen). However, tt ts a powerful oxidising
agent but a weak reducing agent. For example,
Physical property Value
Oxidising character.
Melting point (K) 272.4
Hydrogen peroxide acts as an oxidising agent both
Boiling point (K) 423 in acidic as in alkaline solutions as :
Vapour pressure 19
In acidic medium
(at 298 mm Hg)
HO, +290 +20 fe 2H,O
Density (solid at 268.5 K) 1.6434
In basic medium
1gcm™*
HO, ¢ OBS+ 22, /———> 30H
Density (liquid at 1.4425
or nO, a” —— 20H”
298 K) 1 gem™ Some important reactions in which H,O, acts as
Viscosity (290 K) cP. 1,245 an oxidising agent are given below :
Dielectric constant a (a) Oxidising action in acidic medium.
(288 K) In the presence of an acid, H,O, can accept
Electrical conductivity 5.1 x 10" electrons and, thus acts as an oxidising agent.
(at 298 K) ohm~! em~ H,O, + 2H* + 2e ——— 2H,O
(t) H,O,oxidises ferrous sulphate to ferric sulphate
Chemical Properties
H,O, —— H,0+0
1. Decomposition. Pure hydrogen peroxide is an
2FeSO, + H,SO,+O —— Fe,(SO0,), + H,O
unstable liquid and decomposes into water and oxygen
either upon standing or upon heating. 2FeSO, + H,SO,+H,O, ——— Fe,(SO,), + 2H,O
2H,O,(/) —> 2H,O(/) + O,(g); AH =-196.0 kJ or 2Fe*+H,O,+2H* ——> 2Fe* +2H,0O
In H,O,, the oxidation state of oxygen is —1 which (it) HO, oxidises acidified potassium ferrocyanide
is intermediate between the values for O, (zero) and to potassium ferricyanide
HO (—2). Therefore, aqueous solution of H,O, readily H,O, ——> H,0+0
disproportionates (decomposes). 2K,[Fe(CN),] + H,SO, + O——~+ 2K,[Fe(CN),|
The decomposition is further accelerated by the + K,50, + H,O
presence of metals like platinum, gold, metal oxides 2K,[Fe(CN),] + H,SO, + H,O, ——> 2K,[Fe(CN)¢]
(MnO,) or by certain metal ions such as Fe** ions. Pot. ferrocyanide Pot. ferricyanide
Even rough surface favours its decomposition. + K,S0O, + 2H,O
It is also decomposed by exposure to light. It is or 2[Fe(CN) ,.]* + H,O, + 2H* ——> 2[Fe(CN),1*
+ 2H,O
in the presence of stabilizers like urea. It is kept away
(uit) HO, liberates todine from acidified potassium
from dust because dust (even in small quantities)
iodide solution.
induces explosive decomposition.
2. Acidic nature. Pure hydrogen peroxide is a weak H,O, —— H,0+0
acid (K, = 1.55 x 10-” at 298 K) and dissociates as : 2KI + H,50,+0 —— K,s80,+1,+ H,O
H,O, + HQ —— H,0° + HO, 2KI + H,S5O,+H,O, ——— K,50,+I1,+2H,O
Hydroperoxide ion
or 21 + H,O, + 2H* —— I,+2H,0O
HO, +H,O ——> H,O* + 0O,*
(tv) HO, oxidises ice cold acidified potassium
Peroxide ion
dichromate solution (containing some ether) to
It forms two types of salts. For example,
chromium pentoxide.
NaOH + H,O, ———> NaHO, + H,O
Sod. hydroperoxide H,O, —> H,O+0]x4
(Acidic salt) K,Cr,O, + H,SO, + 4 [0] —> K,SO, + 2CrO, +H,O
2NaOH + H,O, ——> Na,O, + 2H,O K,Cr,0, + H,50, + 4H,0,—- K,SO,+2CrO, + 5H,O
Sod. peroxide Chromium
(Normal salt) pentoxide
MODERN'S abc + OF CHEMISTRY-XI

Chromium pentoxide dissolves in ether producing MnsO, + H,O, + 2NaOH ———> MnO, + Na,SO,
blue colouration. It is actually a peroxo compound Manganese Manganese + 2H,O
having the structure: sulphate dioxide
Mn** + H,O, + 20H- ——— MnO, + 2H,O
or Mn* + H,O, ——> Mn* +20H™
tle | (it) In alkaline medium also, H,O, oxidises ferrous
tons to ferric ions :
Therefore, it is written as CrO(O,), and is also 2Fe** + H,O, ——> 2Fe** + 20H-
known oxodiperoxochromium (VI). (iit) In alkaline medium, H,O, oxidises chromium
salts to chromates
Cr,(SO,), + 3H,0, + 1ONaOH ——— 2Na,CrO,
Oxidation state of Cr in CrO, Chromium sulphate Sod. chromate
The structure of CrO, shows that it has two + 3Na,SO, + 8H,O
peroxo groups. The oxidation state of each peroxo or 2Cr** + 3H,O, + 10 OH-——> 2Cr0O, 2 8H,0O
group O,” is —2 so that (iv) Hydrogen peroxide oxidises nitrites, sulphites
x + 1x(-2) + 2x(-2) and arsenites to nitrates, sulphates and arsenates
(for Cr) (for oxygen) (for peroxo) respectively in alkaline medium.
x-2-4=-0 x=+6 NaNO, + H,O, ——> NaNO, + H,O
Oxidation state of Cr in CrO, is +6 Sod. nitrite Sod. nitrate
Na,SO, + H,O, ——— Na,SO, + H,O
(v) H,O, oxidises H,SO, to H,SO, Sod. sulphite Sod. sulphate
H,O, —— H,O+0 Na,AsO, + H,0, ——~ Na,AsO, ++ H,O
H,SO, +O ——~» H,SO, Sod. arsenite Sod. arsenate

H,SO, + HO, ———~ H,S0O,+H,0 Reducing nature


or SO,” + H,O, ——> SO,*> +H,O Hydrogen peroxide has also a tendency to take up
(vt) HO, oxidises sulphides to sulphates oxygen from strong oxidising agents and thus, acts as
H,O, ——> H,O0+0O]x4 a reducing agent.
PbS + 4[0] ——~ PbsSO, H,O, + O —— H,0+ O 2
From oxidising agent
PbS + 4H,O, ——> PSO, 7G It can act as a reducing agent in acidic as well as
or S> + 4H,0, —_—+ so ,*7 + 4H,0O basic medium.
This property of
hydrogen peroxideis quite useful In acidic medium
in restoring the original white colour of the lead H,O, ——> 2H" +0, + 2e7
paintings. The traces of hydrogen sulphide in the In alkaline medium
atmosphere blacken the lead paintings due to the H,O, + 20H” ——> 2H,0 + O, + 2e7
formation of lead sulphide (black). To restore the
Some important reactions of hydrogen peroxide
colour of lead paintings, it is dipped in aqueous
behaving as a reducing agent are given beolw :
solution of hydrogen peroxide when the lead sulphide
is oxidised to lead sulphate which is white in colour. (a) Reducing action in active medium
(t) H,O, reduces acidified potassium permanga-
(vii) Hydrogen peroxide oxidises hydrogen sulphide nate solution to colourless manganese sulphate. As a
to sulphur and sulphurous acid to sulphuric acid. result of this reaction, the pink colour of KMnO, gets
HS + H,O3,——> 2H,0+ 8 discharged.
H,SO, +H,0, ——> H,SO,+ H,O 2KMnO, + 3H,SO, ——— K,SO,+Mns0O,
Sulphurous acid Sulphuric acid + 3H,O + 5(O)
(vill) Hydrogen peroxide oxidises mercury to H,O, +O —— H,0+0,]x5
mercuric oxide in acidic medium.
Hg + H,O, #24 _, HgO + HO 2KMn0O, + 3H,SO, + 5H,0, ——~ K,S0O, + MnsSO, +
8H,O + 50,
(6) Oxidising action in alkaline solution
or 2MnO, + 6H* + 5H,0, ——> 2Mn”* + 8H,O
Hydrogen peroxide can also accept electrons in the
+50,
alkaline medium and acts as an oxidising agent.
(it) It reduces acidified potasstum dichromate
H,O, + OH” + 2e° ——> 30H”
solution. As a result the orange colour of K,Cr,O,
(1) In alkaline medium, it oxidises manganese changes to green due to the formation of chromium
sulphate to manganese dioxide. salt.
HYDROGEN

K,Cr,0, + 4H,SO, ——> K,SO, + Cr,(SO,), Similarly,


+ 4H,0 + 3(O) I, + H,O, + 20H” ———> 2I + 2H,0 +0,
H,O,+0 ——> H,0+0,]x3 (vi) It reduces hypohatlites to halide
NaOBr + H,O, ——— NaBr+H,0+0,
K,Cr,0, + 4H,50, + 3H,0, ——> K,SO, +Cr,(S0,),
Sod. hypobromite
+ 7H,O + 30,
CaOClL, + H,O, ——> CaCl, + H,O +0,
or Cr,O,” + 8H* + 3H,0,——> _2Cr** + 7H,O
Bleaching powder
+30,
(iit) It reduces manganese dioxide to manganese 5. Bleaching action. Hydrogen peroxide acts as
sulphate in the presence of dil. H,SO,. a bleaching agent due to the release of nascent oxygen.
MnO, + H,SO, ——> Mns0O,+H,0+0 H,0, —— H,0+0
H,0, +O —— H,0+0, Thus, the bleaching action of hydrogen peroxide
is permanent and is due to oxidation. It oxidises the
MnO, + H,SO, + H,O, ———> Mns0O,+2H,0+ O, colouring matter to a colourless product.
(iv) It reduces ozone to dioxygen Colouring matter + OQ -———> Colourless matter
O, —— 0,+0 Hydrogen peroxide is used to bleach delicate
H,0, +O —— H,0+0, materials like ivory, silk, wool, feathers, etc.
H,0O,+ O, ——~ H,0 + 20, 6. Addition reactions. Hydrogen peroxide adds
(v) [t reduces hypohalous acid to halide ion in acid to alkenes to form glycols.
medium CH,OH
HOCI|(aq) + H,O,(aq) ———~> H,O0* +Cl+0O, CH, = CH, + H,O, ———>
(b) Reducing action in alkaline medium Ethylene CH,OH
(1) ft reduces potassium permanganate to Ethylene glycol
manganese dioxide in alkaline medium Uses of Hydrogen Peroxide
2KMn0O, (aq) + 3H,O, (aq) ——— 2Mn0O, + 30, 1. It is used in industry as a bleaching agent for
+2H,0O (/) + 2KOH(aq) textiles, paper, pulp, straw, leather, oils, fats, etc.
or 2MnO, + 3H,O, —> 2MnO, + 30, + 2H,O + 20H 2. Domestically, it is used as a hair bleach and asa
mild disinfectant.
(ii) It reduces ferric (Fe**) salts to ferrous (Fe**)
salts 3. It is used in the manufacture of many inorganic
compounds such as sodium perborates, and
2Fe**(aq) + H,O,(aq) + 2OH- ——-> 2F e”* (aq) percarbonates which are important constituents
+ O, (g) +2H,0 (7) of high quality detergents.
(zit) It reduces alkaline potassium ferricyanide to
4, It is used as an antiseptic for washing wounds,
potassium ferrocyanide
teeth and ears under the name perhydrol.
2K,[Fe(CN),] + 2KOH ——~ 2K,[Fe(CN),]
5. It is used for the production of epioxides,
+H,0+0 propylene oxide and polyurethanes.
H,O, +O ——+>_H,0+0, 6. It is used for the synthesis of hydroquinone,
pharmaceuticals (cephalosoporin), food products
2K.[Fe(CN),] + 2KOH + H,O, ——— 2K,[Fe(CN),|
like tartaric acid.
Potassium ferricyanide Pot. ferrocyanide
+ 2H,O + O, 7. It is used as an antichlor in bleaching.
ea). Itis used for restoring the colour of lead paintings.
or [Fe(CN),|* + 20H + H,O, ———> [Fe(CN )gl*
+ 2H,0 + O, 9. It is used for preserving milk, wine and other
(iv) The oxides of metals are reduced to metals. liquors.
For example, silver oxide is reduced to silver 10. Recently, H,O, is used in environmental
AgoO +H,O, ——— 2Ag+H,0+0, chemistry such as in pollution control treatment
Silver oxide Silver
of domestic and industrial effluents; oxidation of
cyanides and restoration of aerobic conditions to
However, lead dioxide is reduced to lead monoxide. sewage wastes.
PbO, + H,O, ———__ PbO + H,O oo O, Because of its fast growing uses, there has been
Lead dioxide Lead tremendous increase in industrial production.
monoxide Structure of hydrogen peroxide.
(uv) [t reduces halogens to halide tons Hydrogen peroxide has a non-planar structure in
Cl,+ H,O, + 20H” ——+> 2CI°+2H,0 + O, which two H atoms are arranged in two directions
Since hydrogen peroxide can reduce chlorine, it almost perpendicular to each other and to the axis
acts as an anticholor in bleaching by destroying the joining the two oxygen atoms.
unreacted chlorine.
MODERN'S abc + OF CHEMISTRY-XI

The O—O linkage is called peroxide linkage. The According to the equation, 1 mole of oxygen (or
structure of H,O, in the gas phase and solid phase are 22.4 L of oxygen) at N.T.P. is liberated from 2 mol
shown in Fig. 9. In the solid phase, the dihedral angle (or 68 g) of H,O,.
is reduced to 90.2° from 111.5° in the gas phase. 22.4 L of oxygen at N.T.P. is produced from
H,O, = 68 g
1 L of oxygen at N.T.P. is produced from
68 —
4102= 994 §
20 L of oxygen at N.T.P. are produced from
68
H,0, = 99.4 x 20 ¢ = 60.7 g

1 L of H,O, solution of 20 volume = 60.7 g H,O,


Strength of 20 volume H,O, = 60.7 g/litre.
= SAE ASR eS a -|
| !I SOLVED EXAMPLES
! I lL} Example 4
| i
i
| Calculate the strength and normality of 10 volume
| ‘i :
solution of hydrogen peroxide. \N.C.E.R.T,
| 98.8 pm | Solution: 10 volume H,O, solution means that 1 L of
| . this solution will liberate 10 L of oxygen at N.T.P. Let us
er 145.8 pm | 90.2 °
e—) calculate the amount of H,O, (in grams) which gives 10 L of
\ 101,9° 7 oxygen at N.T.P. This amount will be present in 1 L of 10
\ i > volume solution of H,Q,.
™ > H,O, decomposes as :
~y ,
= N 2H,O, —— 2H,O + QO,
N N 2x 34=68 g 22.4 L at N.T.P.
~\ ».
22.4 L of O, at N.T.P. is produced from H,O, = 68 g
eree eae LY
(6b) Solid phase 10 L of O, at N.T.P. is produced from H,O,= a Zz * 10
Fig. 9. Structure of H,O, in (a) the gas phase and = 30.357 g
(b) solid phase (at 110 K). Strength of H,O, in 10 volume H,O, = 30.357 g/litre
Now, normality = Gram equivalents dissolved
Strength of hydrogen peroxide
per litre of solution
The strength of hydrogen peroxide is expressed Gram equivalent of H,O0,
in terms of weight or volume as : ~ solution
Volume of in litres
(i) As weight percentage. The weight percentage 30.357
Gram equivalents of H,O, = 7, = 1.79
of hydrogen peroxide gives the weight of HO, in 100
(Eq. wt. of H,O, = 17)
¢ of solution. For example, a 40% solution by wt. means
1.79
40 g of hydrogen peroxide are present in 100 g of Normality = —— = 1.79 N.
i|
solution.
lL} Example5
(ti) As volume. The strength of hydrogen peroxide
is commonly expressed as volume. This refers to the Calculate the strength in volumes of a solution
volume of oxygen which a solution of H,O, will give. containing 30.36 g /litre of H,0..
For example, a “20 volume” of hydrogen peroxide Solution: Let us calculate the volume of O, which is
means that 1 litre of this solution will give 20 litres of liberated by 30.36 g HO, at N.T.P. This will correspond to
oxygen at N.T LP. volume strength of H,Q,.
To illustrate this, let us calculate the strength of Hydrogen peroxide decomposes as :
20 volume of hydrogen peroxide in gram per litre. 2 H,O, — > 2H,0O + O,
20 volumes of H,O, means that 1L of given 2xd34=68 ¢ 29.4
L at
solution of HO, will give 20 L of oxygen at N.T.P. Let N.T.P.
us calculate the amount of H,O, present in 1 Lof H,O, 68 g of HO, produces O, at N.T.P. = 22.4 L
which gives 20 L of oxygen. 30.36 g of H,O, will produce O, at N.T.P.
H,O, decomposes as : 22.
22.4
2H,0, ——> 2H,0+ 0, = Ee x 30.36 = 10L

2xd4=68 g 22,400 mL The given solution of H,O, produces 10 LofO, at N.T.P.


at N.LP. Volume strength = 10 volumes.
HYDROGEN

lL} Example6 22.400


= 68 x 0.0136g
Calculate the volume strength of2N H,O, solution.
= 4.48 ml of S.T.P.
Solution: Strength = Normality x Equivalent weight
1 cm® of H,O, at S.T.P. gives 4.48 ml of O,.
Normality = 2N, Equivalent weight of H,O, = 17
: Strength of H,O, = 4.48 volumes.
Strength = 2 x 17 = 34g/L L] Example 9.
Hydrogen peroxide decomposes as : Calculate the volume strength of a 3% solution of
2H,0, —_——_} 2H,O + O,
2 x 34 = 68g 22.4L at N.T.P. Solution: 100 mL of solution contains H,O, = 3 g
68g of HO, produce O, at N.T.P. = 22.41 1000 mL of solution will contain H,O,
3
34g of H,O, will produce O, at N.T.P. = —— = Too x 1000
= 30g

=11.2L H,O, decomposes :


The given solution of H,O, produces 11.2L of O, at N.T.P. 2H,O, — > 2H,O~ + 0,
68 g 22.4 at N.T.P.
Volume strength = 11.2 volumes
68 g of H,O, give O, at N.T.P. = 22.4 L
lL} Example 7 .. 30 gofH,O, will give O, at N.T.P.
What is the mass of hydrogen peroxide present in 22.4
1 litre of a 2M solution ? Calculate the volume of = G3 * 30
oxygen at S.T.P. liberated upon the complete = 9.88 L
decomposition at 100 cm? of the above solution. = 9880 mL
Solution: Molecular mass of H,O, = 2 + 32 = 34 g But 30 g of H,O, is present in 1000 mL of H,O,
1000 mL of H,O, solution gives O, at N.T.P. = 9880 mL
Now, 1 litre of 1 M solution contains H,O, = 34 g
1 mL of H,O, solution will give O, at N.T.P.
. 1 litre of 2 M solution contains H,O, = 34 x 2 = 68 g
9880 L
H,O, decomposes as : = 000 = 9.85 m
2H,O, ———> 2H,O + 0, Hence volume strength of 3% H,O, = 9.88
2x 34=68¢ 22400 cm? () Example 10
Mass of H,O, in 100 cm? of the solution
30 mL of H,O, solution after acidification required
_ —- x 100 =6.8 g 30 mL of N/10 KMnO, solution for complete
Now, according to the equation oxidation. Calculate the percentage and volume
68 g of H,O, at S.T.P. evolve oxygen = 22400 cm® strength of H,O, solution.
6.8 g of HO, at S.T.P. evolve oxygen Solution: To calculate the normality of H,O, solution,
apply normality equation,
= — x 6.8 = 2240 cm®, N,V, =N,V.
lL} Example 8 H,O, KMn0O,
A 5.0 cm? of H,O, liberates 0.508 g of iodine from For H,O,, V, = 30 mL, N, =?
an acidified KI solution. Calculate the strength of
H,O, solution in terms of volume strength at For KMn0,, V, = 30 mL, N, = —
Die. N
Solution: H,O, reacts with acidified KI solution as : N, x 30 = | Io * 30
2KI + H,SO, + H,O, =—>"K,80, + I, + 2H,O 1 30
34 ¢ 254 g N, = 10 30 =l1/10N=0.1N

5 em® 0.508 g Strength = Equivalent wt. x Normality


254 g of I, is hberated by H,O, = 34 g =17x 01 =1.7 g/L

0.508 g of I, is liberated by H,O, = —— x 0.508 % of strength =


la
1000 x 100 = 0.17%
= 0.068 g H,O, decomposes as :
5 cm® of H,O, contains = 0,068 g 2H,0, —> 2H,O + iO,
68 g 22.4L at N.T.P.
1 cm? of H,O, contains = — = 0.0136 g
68 g of H,O, give O, at N.T.P. = 22.4L
Now 2H,O, ——— 2H,O + O, 1.7g of H,O, will give O, at N.T.P.
68 g 22400 ml at 8.T-P.
68 g of H,O, give O, at 8.T.P. = 22400 ml = ais x 1.7 = 0.56 L
68
1 cm? or 0.0136 g of H,0, give O, at 8.T.P. Since 1.7g of H,O, is present in 1L,
.. Volume strength = 0.56 volumes.

WWW.JEEBOOKS.IN
MODERN'S abc + OF CHEMISTRY-XI

= ES SS
_———— ERODES Tie ere as = cae
= 0.759 g.
Q@ 11. wees is ane significance of 15 volume of H,O, ©17. Black colour in paintings is due to
RUE ; lead sulphide (PbS). This is oxidised to lead
Q12. Whatis the trade name of hydrogen peroxide used sulphate by hydrogen peroxide.
as an antiseptic ?
Q13. Does hydrogen peroxide act as ©18. 25,0, :
2H,0 + O,
(i) strong acid (i) bleaching agent ? 68g 22.4 L at N.TP.
Q@14. Name two compounds which retard the 0.91 x 1000
decomposition of hydrogen peroxide. Amount of H,O,iniL = 10 =91¢g
@15. 0, the normality of 20 volume solution of 68g of H,O, give O, at N.T.P. = 90a Tt

Q@16. Calonlate the amount of hydrogen peroxide present 912 of H.O. will civ@O atNTP
= 22.4 91
in 10 ml of 25 volume solution of H,O,. lie ee | ee
Q17. Old paintings of lead are Senerally waakied with = 30
dilute solution of hydrogen peroxide in order to Be 91 g of HO, is present in 1L,
regain its colour. Why ? Volume strength= 30 volume
Q18. 10 ml of given H,O, solution contains 0.91 g of © 19. Streuf of H,O, = 1.6 x 17 = 27.2g/L
H,O,. Express its strength in volumes. . 68o/af e@mees
tS O. atN TP -224L
Q@19. Calculate the volume strength of 1.6 N H,O, _ a |
solves: 27.2 of H,O, will produce O, at N.T.P.
“ 22.4
Answers to Practice Problems = 5X 27.2 = 8.96L
©11. It means that 1 L of this solution gives The given solution of H,O, produces 8.96L of O,
15 L of oxygen at N.T-P. at N.T-.P.
© 12. Perhydrol. .. Volume strength = 8.96 volume.

ee
© 14.
acetanilide, glycerol.
LIQUID HYDROGEN AS
|
A FUEL _. |
© 15. 20 volume of H,O, solution means that 1L of this Fuels burn to release large quantities ofenergy.
solution will liberate 20L of O, at N.T.P. The energy released by combustion of fuels such
2H,0, —> 2H,O + O, as dihydrogen, methane, LPG and gasoline (octane)
68¢ 22.4 at N.T.P. as compared in terms of the same amounts in mole,
22.4L of O, at N.T-.P. is produced from H,O, = 68 g mass and volume are given in Table 8.
20L of O, at N.T.P. is produced from H,0, It is clear from the above table that on mass to
_ 88 . o9_ 60 71g mass basis, dihydrogen can release more energy than
22.4 a telat os even gasoline i/.e., octane (about three times). Moreover,
Strength of H,O, = 60.71 g/L the pollutants in the combustion of dihydrogen will be
7 less than petrol. The only pollutant will be oxides of
60.71 ae
Gram equivalents of H,O, = oa 3.57 dinitrogen (due to the presence of impurity with
357 dihydrogen). However, this can be reduced by injecting
.. Normality = =i = 3.57 N small amounts of water into the cylinder to lower the
©16. 2H,0, 2H,0 + 0, temperature so that the reaction between dinitrogen
2 and dioxygen may not occur.
68 g 22.4L at N.T.P. a ; |
| Liquid hydrogen is used as an important rocket
22.41 of O, at N.T.P.is produced from H,O,=68 = fue], A primary condition for a rocket fuel is that its
25 L of O, at N.T-P.is produced from H,0, mass be as small as possible for a given amount of
_ 8 £05 =75.9¢ energy produced. Since hydrogen has low mass and
hen 2) ee Eis high enthalpy of combustion, therefore, it was
Strength of 25 volume H,O, = 75.9 g/L considered as an ideal rocket fuel. It was used in

Table 8 Energy released by combustion of various fuels in moles, mass and volume

Energy released Dihydrogen Dihydrogen LPG CH, gas Octane


on combustion (kJ) (in gaseous (in liquid (in liquid
state) state) state)
per mole 286 285 2220 880 5511
per gram 143 142 50 53 AT
per litre 12 9968 25590 35 34005
HYDROGEN

Saturn—V rocket that enabled the first astronauts to nuclear power can also not meet all our energy
land on the moon and it has been the main fuel in the requirements, particularly for small mobile units such
space shuttle rockets. Both the hydrogen and the as cars, etc. The search for alternative sources of
oxygen needed to burn the hydrogen are carried on energy has given a solution as “hydrogen economy”
the rocket in liquid form. whereby energy is transported and stored in the
The important advantages of hydrogen asa fuel form of liquid or gaseous hydrogen. Hydrogen is
for rockets are : considered to be a possible source of clean energy.
1. It releases greater energy per unit weight of The advantage of hydrogen economy is that energy is
the fuel. transmitted in the form of dihydrogen and not as
electric power. The major problem is to make hydrogen
2. The only product of combustion is water.
cheaply. Various methods of obtaining hydrogen are
3. There is no emission of environmental
being explored.
pollutants such as CO, CO,, SO,, oxides of
If the problems of its safety and its economically
nitrogen (NO,), hydrocarbons, aldehydes and
production are solved, it may eventually be delivered
lead compounds, etc.
to homes and industry by pipelines as natural gas.
4. Internal combustion engines can easily be Hydrogen might even be used as fuel to run vehicles
modified for use of hydrogen as fuel. and jet planes and produce electricity. For this purpose
5. Fuel cells for generation of electric power with it might be stored as a liquid at very low temperature
conversion efficiency of 70-85% have been or as solid metal hydride that when heated decomposes
successfully operated commercially. to produce hydrogen.
Hydrogen is so abundant in the form of water in Better ways of storing hydrogen are worked out.
the oceans that it could meet all our energy needs, In USA, technology exists for producing hydrogen
and provide inexhaustible supply. However the main electrically and storing it in bulk. For example, large
problem is to extract it from water. This is because quantities of liquid hydrogen are routinely stored in
decomposition of water to hydrogen and oxygen : vacuum insulated cryogenic tanks for US space
programme. It can also be stored in underground tanks
H,O(l) ——> H,(g) + - O,(g) AH=286kdJ. and transported by pipelines. Smaller storage units
requires 286 kJ per mole of water. Therefore it has made of metal alloys like Ti-TiH,, NaNi,, Mg-MgH,,
been difficult to carry out this reaction economically. etc. are used. The advantages for hydrogen to be used
Because of large amount of electricity required, as an automobile fuel are the larger energy released
electrolysis is an expensive method. It can be carried per unit mass of fuel and the absence of emission of
out only where electricity is cheap. pollutants such as CO, CO,, oxides of nitrogen, 5O,,
However, the main problems for the use of hydrocarbons and lead compounds. The product of
hydrogen as a fuel are: combustion is only water. Several conventional
(4) Hydrogen is highly combustible and is difficult internal combustion petrol engines have already been
to handle safely. Because of its combustible modified to run on hydrogen.
nature, it burns with explosion. It is for the first time in the history of India that a
(it) It is not easy to store or transport hydrogen pilot project using dihydrogen as fuel was launched
safely and compactly from one place to another. in October 2005 for running automobiles. Initially
A cylinder of compressed dihydrogen weighs about 5% dihydrogen has been mixed with CNG for
about 30 times as much as a tank of petrol use in four wheeler vehicles. The percentage of
containing the same amount of energy. dihydrogen would be gradually increased to reach the
Moreover, dihydrogen gas is converted into optimum level.
liquid state by cooling to 20 K. This would Nowadays, hydrogen is also used in fuel cells for
require expensive insulated tanks. the generation of electric power.Fuel cells using
Tanks made of metal alloys like Ti—TiH,, hydrogen for the generation of electricity have been
Mg—MghH,, NaNi,, etc. are in use for storage of successfully operated with a conversion efficiency of
dihydrogen in small quantities. about 70-85%.
(iit) Itis an expensive fuel because cost of production The production of non-electrolytic hydrogen is
is high. drawing the attention of many scientists in recent
HYDROGEN ECONOMY years. The main principle involved in the process is
With the growth of modern civilization, the need the mode of transferring free energy to a water
of energy has been continuously increasing at a very molecule for decomposing it. Fortunately, some success
fast rate. The natural reserves of the world for energy has been achieved in this field and it is expected that
such as coal and oil are finite. The continuous use of it will be possible to develop economically viable and
these sources may exhaust them completely and this safe sources of hydrogen for its use as a general source
may cause a serious problem for getting energy. The of energy in future.
MODERN'S abc + OF CHEMISTRY
-XI

Conceptual
. Which isotope of hydrogen is used commoly as a tracer in organic reactions ?
Both deuterium and tritium can be used as tracers in organic reactions. However, tritium is not only
radioactive but is also least abundant hydrogen isotope and therefore, deuterium is commonly used as
tracer to study the mechanism of organic reactions.
Q. 2. Why is dihydrogen gas not preferred in balloons ?
. Dihydrogen gas is combustible in nature. Therefore, it may react with oxygen highly violently. Thus, it
is not used in balloons.
Q. 3. Acetanilide checks the decomposition of H,O,. What is this compound called ?
Ans. Acetanilide is a neutral compound which suppresses the decomposition of H,O, into H,O and Q,. It is
called an inhibitor.
Q. 4. Consider the reaction of water with F, and suggest, in terms of oxidation and reduction
which species are oxidised or reduced.
Ans. 2F (g) + 2H,O(aq) ———> 4H" (aq) + 4F" (aq) + O,(g)
In this reaction, water is oxidised to oxygen by F,. Fluorine has been reduced to F’ ion.
Q. 5. What is meant by autoprotolysis of water ?
Ans. Autoprotolysis is a reaction in which two same molecules react to give ions with proton transfer, For
example, in water, a proton from one H,O molecule is transferred to another water molecule leaving
behind OH” ion and forming a H,O* ion.
H,O(/) + H,OW) H,O* (aq) + OH” (aq)
Q. 6. How will you prepare a sample of ND, ?
Ans. ND, is prepared by reacting magnesium nitride (Mg.N,) with heavy water as :

Mg.,N, + 6D,0 ——> 3Mg(OD), + 2ND,


. Classify the following as covalent, ionic or polymer interstitial hydrides :
(i) BeH, (ii) CaH, (iii) ROH (iv) GeH, (v) Ti,
(i) BeH, Polymeric [exists as (BeH,)_]
(it) CaH, Ionic
(iii) RbH : Ionic
(iv) GeH, Covalent
(v) TiH, Interstitial
Q. 8. Name the radioactive isotope of hydrogen.
Tritium (}T ).
Q. 9. Give one example of a reaction in which dihydrogen acts as a reducing agent and an
oxidising agent.
. () Dihydrogen acts as a reducing agent :
Cu0 +H, —> Cu+H,0
(iz) Dihydrogen acts as an oxidising agent :
2Na+H, —> 2NaH
Q. 10. How is heavy water obtained from ordinary water ?
Ans. Heavy water is obtained from ordinary water by repeated electrolysis in the presence of
3% NaOH.
Q. 11. Name one compound in which hydrogen occurs as (i) +1 and (ii) -1 oxidation state.
Ans. (i) +1 in HCl
(ii) —1 in NaH

WWW.JEEBOOKS.IN
HYDROGEN

. What type of elements form interstitial hybrides ?


. Elements of d— and f— block.
. Which of the two is heavier 1 kg of ice or 1 kg of water?
Both have same weight.
. Among NH,, H,O and HF, which would you expect to have highest magnitude of hydrogen
bonding and why ?
. HF will have maximum hydrogen bonding because it has highest electronegativity among N, O and F.
Complete the following equations :
(i) Fe(s)+H,O (¢)—
(ii) PbS (s) + H,O, (aq) —>
(iii) MnO, (aq) + H,O, (aq) —>
(i) 3Fe (s) + 4H,0 (g) —> Fe,O, (s) + 4H, @&)
(it) PbS (s) + 4H,O, (aq) —> PbSO, (s) + 4H,O
(iii) 2MnO,- (aq) + 5H,0, (aq) + 6H* —> 2Mn* + 8H,O + 50,
Q. 16. Complete the following reactions :
(i) CuO (s) + H, (g) —
(ii) CO (g) + H, (g) —
. (i) CuO (s) +H, (g)—>H,0 + Cu
Cobalt
(ii) CO (g) + 2H, (8) —Gatalyst’? CH,OH ()
Give an example each of an ionic hydride and a covalent hydride.
Ionic hydride : NaH
Covalent hydride : NH,
How many hydrogen bonded water molecule(s) are present in CuSO,.5H,O ?

. InCuSO,.5H,O, there is one hydrogen bonded water molecule which is outside the coordination sphere.
The other four molecules of water are coordinated.
Complete the following reactions :
(i) CaO (s) + H,O (1) —>
(it) Na,O (s) + H,O (1) —>
. (d) CaO (s) + H,O (/) —> Ca(OH), (aq)
(ii) Na,O (s) + H,O (1) —> 2NaOH (aq)
. What happens when heavy water is added to calcium carbide ?
. Deutero acetylene is formed
CaC, + 2D,0 —> Ca(OD), + C.D.

® Coal gasification. The process of producing a mixture of carbon monoxide and hydrogen from coal.
® Softening of water. The process of the removal of hardness from water.
® Zeolites. Hydrated sodium aluminium silicates Na, Al, Si, Og . x H,O.
L984 MODERN'S abc + OF CHEMISTRY-XI

cHaPresGTIDT
rT |
HYDROGEN |
Hydrogen is the first element in the periodic table.
It has atomic number | and its electronic configuration is 1s’.
Hydrogen resembles alkali metals as well as halogens in its properties.
Hydrogen exists in three isotopes Le., protium (‘H), deuterium (JH or D) and tnitium ((H or T). Tritium is radioactive and
emits f-particles (t,,, = 12.33 years).
Electrolysis of an aqueous solution of HC! liberates H, at the cathode and the electrolysis of NaH (fused) liberates H,,
at the anode.
Ordinary hydrogen exists as ortho and para hydrogen. The nuclei of ortho hydrogen spin in the same direction while
that of para hydrogen spin in the opposite directions.
At room temperature, the ratio of ortho to para hydrogen is 3: 1, at the temperature of liquefaction of air, the ratio is
1: 1 and at O K, hydrogen contains mainly para hydrogen, which is more stable.
Alkali metals and alkaline earth metals react with cold water evolving H, gas.
» Mg, Zn and Al decompose boiling water evolving H, gas while Fe decomposes steam producing H, gas.
Hydrogen forms compounds by losing, gaining or sharing of electrons and exhibits oxidation states of +1, —1 and 0.
Hydrogen reacts with metals and non-metals
2Na + H, ——® 2NaH
N, + 3H, —» 2NH,
H, + XX, ——> 2ZHK (X = F, Cl, Br, land reactivity of halogen is F, > Cl, > Br, > I,)
Hydrogen acts as a powerful reducing agent and reduces CuO, ZnO, PbO, Fe,O, etc.
ZnO + H, ——> 21 YEO

Ionic : formed by combination of H with highly Water is angular or bent molecule and H-—O—H bond is
electropositive metals. 104,5°
e.g. LiH, KH, CaH,, STH, e Water can act as a base towards acids stronger than itself
Molecular or covalent : formed by combination of elements and as an acid towards bases stronger than it.
of higher electronegativity. e Water which produces lather with soap solution readily is:
e.g. HF, HO, NH,, BjH,, CH, Fis, PH, called soft water.
Interstitial: formed by elements of d- and Fblock. e Water which does not produce lather with soap solution,
e.g. ZTH,,, TiH,,, LaH,7 readily is called hard water.
Complex : LiAIH,, NaBH, « The hardness of water is due to the presence of the
i‘- s bicarbonates, chlorides and sulphates of calclum and
Hydrolith is the common name of CaH,. Magnesium.
e Water acts aS an oxidising agent towards highly active
metals and itself gets reduced to H.,.
EY EN PEROXIDE | 2H,O(1) + 2e- ——» 20H™~(aq) + H,(g)
Itacts as a reducing agents towards highly electronegative
« It is prepared in the aE as elements and itself gets oxidised to O, or O,,.
Na,O, + H,50,— Na5sO, + HGS O,(g) + 4H* (aq) + 4e— ——» 2H,O(l)
BaO,,.8H,O(s) + H,SO,(aq) — BaSO,(s) + H,O, + 8H,O ag. 2P, + ZEA —» 40° -- 4" +O,
e On commercial scale, it is prepared by or a, t 30 —— 0, t+ +
— electrolysis of 50% H,SO, solution
— 2-Fthyl anthraquinol
« H,O, decomposes as In CuSO 7 9H,9, Cu is coordinated to four H,O molecules
2H,0, Se~ ZFRO 7 O,,
while the fifth H,O molecule is hydrogen bonded to SO ag
Its decomposition is retarded by adding glycerol, phosphoric |
ion,
acid or acetanilide (negative calalysts).
e HO, is a weak acid and forms two types of salts.
HEAVY WATER
e.g. NaOH + 4,0, ——» NaHO, + H,O
2Z2NaQOH +S Gis # Na,O, + 2H,0
¢ 1,0, acts as an oxidizing agent as well as a reducing agent Heavy water is D,O which is obtained by prolonged
in both acidic and basic medium. hydrolysis of water or by fractional distillation.
e The bleaching action of HO, is due to oxidation and is e The reactions of D,O are slower than those of H,O.
permanent. D,O is used in nuclear reactors as a moderator

strength of H,O,
strength of hydrogen peroxide is usually expressed as volume. For example, a 20 volume HO, means that 1 litre of this
solution will give 20 litres of oayyen at BM. T.P.
e Strength of 10 volume H,O, = 30.35 g/L
e Normality of 10 volume H,0, = 1.79 N
¢ Molarity of 10 volumeF
H,O, = =0.89 M

WWW.JEEBOOKS.IN
HYDROGEN

2 SINGER Solved
Textbook Exercises //
Q.1. Justify the position of hydrogen in the periodic
(iii) C,H,(g)+ 3H,0@) Seaun? 3CO(g) + 7H,(g)
table on the basis of its electronic configuration.
Ans. Refer Text Page 1. (iv) Zn(s) + 2NaOH(aq) #2245 Na,ZnO,(aq) + H,(g)
Q.2. Write the names of isotopes of hydrogen. What Sodium zincate
is the mass ratio of these isotopes ? Discuss the consequences of high enthalpy of
Protium, +H, deuterium “H or D, tritium *H or T. H—H bond in terms of chemical reactivity of
Mass ratio of protium, deuterium and tritium dihydrogen.
sl:2: 3. Ans. The bond dissociation enthalpy of H—H bond is
Q.3. Why does hydrogen occur in a diatomic form very high (435.9 kJ mol‘). Due to high bond
rather than in a monoatomic form under enthalpy, itis not very reactive at room temperature.
normal conditions ? However, at high temperatures or in the presence
of catalysts, hydrogen combines with many metals
Ans. Hydrogen atom has only one electron in its valence
shell (1st) and has only one electron less than the and non-metals to form hydrides.
stable noble gas configuration of helium. Therefore, What do you understand by (i) electron-deficient,
it shares its single electron with electron of other (ii) electron-precise, and (iii) electron-rich
H-atom to achieve stable inert gas configuration of compounds of hydrogen? Provide justification with
He. Therefore, it readily forms diatomic molecule suitable examples.
and exists as H, rather than monoatomic. Ans. (i) Electron deficient hydrides are those which donot
Q. 4. How can the production of dihydrogen, obtained have sufficient number of electrons to form normal
from ‘coal gasification’, be increased ? covalent bonds. For example, hydrides of group
Ans. The process of producing a mixture of carbon 13 (BHg, AIH,, etc). To make up their deficiency
monoxide and hydrogen from coal is called coal they generally exist in polymeric forms such as
gasification. This mixture is also called synthesis B,H,, Al,H,, ete.
gas or syn gas. : (it) Electron precise hydrides are those which have
C(s) + H,O(g) => cow) + Hye) sufficient number of electrons required for forming
The production of hydrogen can be increased by covalent bonds. For example, hydrides of group
reacting carbon monoxide of syn gas with steam in 14 (CH,, SiH,, GeH,, SnH,, PbH, etc). These have
the presence of iron chromate as catalyst at 673K. tetrahedral shapes.

CO(g) + H,0(g) ga"? CO,(e) + He) (iit) Electron rich hydrides are those which have excess
electrons as required to form normal covalent
steam
bonds. For example, hydride of group 15 or 17
The CO, thus produced is removed by scrubbing (z.e., NH,, PH,, H,O, HS, H,Se, H,Te, etc). The
with a solution of sodium arsenite. excess electrons in these hydrides are present as
Describe the bulk preparation of dihydrogen by lone pairs.
electrolytic method. What is the role of an What characteristics do you expect from an
electrolyte in this process ? electron-deficient hydride with respect to its
Ans. Refer Text Page 6. structure and chemical reactions ?
Q. 6. Complete the following reactions : Ans. Electron deficient hydrides donot have sufficient number
of electrons to form normal covalent bonds. Therefore,
(i) H,(g) + M,O,(s) —4-, to make up their deficiency, they generally exist as
polymeric species e.g., ByH,, B,H,9, (AIH), ete.
(ii) CO(g) + Hi) Gaya? The electron deficient hydrides act as electron
acceptors i.e., Lewis acids and thus form complexes
(iii) C5H,(g) + 3H,O0(g) ———>
Catalyst with Lewis bases. For example :
—He8t, BH, + 2NMe, ——> 2BH, «— NMe,
(iv) Zn(s) + NaOH(aq)
Diborane Trimethylamine
Ans. (i) H,(g) + M,,O,(s) 4, mM(s) + 0H, O(/) (Lewis base)
BH, + 2CO ——} 2BH, — CO
aa A
(ii) CO(g) + 2H,(g) “Gaeta? CH,OH(/) Diborane Carbon monoxide
Methanol (Lewis base)
MODERN'S abc + OF CHEMISTRY-A&I

Q. 10. Do you expect the carbon hydrides of the type Ans. All these molecules undergo hydrogen bonding due
(C_H,,,,.) to act as ‘Lewis’ acid or base ? Justify to greater electronegativity of N, O and F.
your answer.
Ans. Carbon hydrides of the type C,H,.,. are electron
precise hydrides. They have exact number of electrons
required to form covalent bonds. Therefore, they
have no tendency to either gain or lose electrons and
hence they do not act as Lewis acids or Lewis bases.
Q. 11. What do you understand by the term “non-
stoichiometric hydrides” ? Do you expect this
type of the hydrides to be formed by alkali metals ?
Justify your answer.
Ans. The hydrides in which the ratio of the metal to
hydrogen is fractional and does not correspond to hand aw.
he
bat Se
=

actual formula are called non-stoichiometric


hydrides. Their composition varies with temperature
and pressure. For example, the hydrides of titanium
or zirconium are represented as Ti H, (x = 1.8 to 2)
and Zr H, (x = 1.30 to 1.75) respectively. This type
of hydrides are formed by elements of
d-and f-block.
Alkali metals are very strong reducing agents and
they transfer their lone electron to H-atom to form
H” ion. In other words, alkali metal hydrides are
a7
ionic in which H~ ions occupy holes of lattice. Since
hydride ion is formed by the transfer of an electron, pA >

therefore, the ratio of metal to hydrogen is always


140° “s, 6+
ri
va i
fixed and hence alkali metals form only
a 7 Nae
stoichiometric hydrides. Thus, alkali metals donot
form non-stoichiometric hydrides.
Since F is most electronegative and therefore, the
Q. 12. How do you expect the metallic hydrides to be
magnitude of negative charge on F and positive
useful for hydrogen storage ? Explain.
charge on H will be highest and therefore, H-bonding
Ans. Metallic hydrides can be used as hydrogen storage.
will be strongest in HF.
This is because in metallic hydrides, hydrogen is
absorbed as H-atoms. Due to inclusion of H-atoms,
Q. 15. Saline hydrides are known to react with water
the metal lattice expands and therefore becomes violently producing fire. Can CO,, a well known
fire extinguisher, be used in this case ? Explain.
less stable. Therefore, when metallic hydride is
heated, it decomposes to form hydrogen and very Ans. Saline hydrides (e.g., NaH, CaH, etc) react with
finely divided metal. water violently to form corresponding metal
The hydrogen thus produced can be used as a fuel. hydroxide with the evolution of dihydrogen as,
Thus transition metals or their alloys can be used NaH(s) + H,O (2) ——> NaOH (aq) + H,(g)
to store and transport hydrogen to be used as a fuel. CaH,(s) + 2H,O (J) ———> Ca(OH), (aq) + 2H,(g)
This is called hydrogen economy. These reactions are very exothermic and the evolved
Q. 13. How does the atomic hydrogen or oxy-hydrogen H, catches fire. The fire produced cannot be
torch function for cutting and welding extinguished by CO, because it gets reduced by hot
purposes ? Explain. metal hydride.
Ans. Atomic hydrogen is produced when molecular NaH + CO, ——> HCOONa
hydrogen is passed through an electric are struck
However, sand is used because it is highly stable
between tungsten electrodes at 3773-4273K.
solid.
Hy sas?
Electric are
— 2H AH = 435.9 kJ mol
= ‘ =1 . Arrange the following
(t) CaH,, BeH, and Til, in order of increasing
The life time of atomic hydrogen is very very small (only
electrical conductance.
0.3 sec) and hence it immediately gets converted into
(a) LiH, NaH and CsH in order of increasing
molecular form by liberating a large amount of energy
ionic character.
which may be used for cutting or welding purposes in
the form of atomic hydrogen torch. (See page 9). (az) H—H, D—D and F—F in order of increasing
bond dissociation enthalpy.
Q. 14. Among NH,, H,O and HF, which would you
expect to have highest magnitude of hydrogen (iv) NaH, MgH, and H,0O in order of increasing
bonding and why ? reducing property.
HYDROGEN

Ans. (i) BeH, < CaH, < TiH, Ans, (i) PbS (s) + 4H,O,(ag)——> PbSO,(s) + 4H,O (J)
(ii) LiH < NaH < CsH (ii) MnO,(aq) + 5H,O,(aq) + 6H* (aq) —>
(iz) F—F < H—H < D—D 2Mn?*(aq) + 8H,O(/) + 50,(g)
(tv) HO < MgH, < NaH (it) CaQ(s) + H,O(g) —> Ca(OH), (aq)
Q. 17. Compare the structures of H,O and H,O, (iv) AICI,(g) + 3H,O07) —> Al(OH),(s) + 3HCl(aq)
Ans. In water, O atom is sp* hybridised and there are two (v) Ca,N,(s) + 6H,O0V7) — > 3Ca(OH),(aq)
O—H bonds and two sp® hybrid orbitals occupy lone + 2NH,(aq)
pairs of electrons. Due to stronger lone pair—lone Q. 21. Describe the structure of the common form of
pair repulsions than bond pair—bond pair repulsions, ice.
the HOH bond decreases from 109.5° to 104.5%, Ans. Ice has a three dimensional network in which each
Therefore, water molecule is a bent or angular O atom is tetrahedrally surrounded by four hydrogen
molecule. H,O, has non-planar structure. In this atoms; two by covalent bonds and two by hydrogen
structure, two O—O oxygen atoms are bonded to bonds. Ice has open structure having a number of
each other by a single covalent bond and each O
vacant spaces. Refer Fig. 5 (Page 16).
atom is further bonded to a hydrogen atom by a Q. 22. What causes the temporary and permanent
hardness of water ?
single covalent bond. The two O—H bonds are in
Ans. The temporary hardness is due to the presence of
different planes. The dihedral angle between two
bicarbonates of calcium and magnesium i.e.,
planes is 111.5° in the gas phase (Fig. 9).
Ca(HCO,), and Mg (HCO,), in water. On the other
Q. 18. What do you understand by the term ‘auto- hand, permanent hardness is caused by the
protolysis’ of water ? What is its significance ? presence of chlorides and sulphates of calcium and
Ans. Auto-protolysis is a reaction in which two same magnesium i.e., CaCl,, CaSO,, MgCl,, MgSO, in
molecules react to give ions with proton transfer. water.
For example, in water, a proton from one water Q. 23. Discuss the principle and method of softening of
molecule is transferred to another water molecule hard water by synthetic ion-exchange resins.
leaving behind OH ions and forming H,0* ion : Ans. Refer Text; Page 21.
Q. 24. Write chemical reactions to show the
H,OW) + H,OW) == — H,O0*(aq) + OH (aq)
amphoteric nature of water.
Because of auto-protolysis, water is amphoteric in Ans. Water is amphoteric in character and it behaves
nature i.e., it reacts with acids as well as bases. It both as an acid as well as a base. With acids stronger
acts as a base towards acids stronger than itself and than itself (e.g., H,S) it behaves as a base and with
as an acid towards bases stronger than itself. For bases stronger than itself (e.g., NH.) it acts as an
example : acid.
H,O(/) + NH,(ag) —» NHj(aq) + OH-(aq) (1) As a base
acid, base, acid, base, H,OW) + H,S(aq) —> H,O*(aq) + HS (aq)
base, acid, acid. base,
H,O() + H,S(aqg) —> H,O*(aq) + HS (aq)
(iz) As an acid
base, acid, acid, base,
H,O() + NH,(aq) —> NH,*(aq) + OH (aq)
It helps to understand Bronsted concept of conjugate acid, base, acid, base,
acid-base pairs.
Q. 25. Write the chemical reactions to justify that
Q. 19. Consider the reaction of water with F, and hydrogen peroxide can function as an oxidising
suggest, in terms of oxidation and reduction, as well as reducing agent.
which species are oxidised/reduced. Ans. Hydrogen peroxide behaves both as an oxidising
Ans. 2F(g) + 2H,OU) —> O,(g) + 4H"*(aq) + 4F (aq) agent as well as reducing agent both in acidic and
Oxidant Reductant alkaline medium. The oxidation state of O in H,O,
or 3F,(g) + 3H,OU) —> O,(g) + 6H*(aq) + 6F (aq) is —1. Therefore, it can be oxidised to O,(zero
Oxidant Reductant
oxidation state) or reduced to H,O or OH™
(—2 oxidation state). Some reactions of H,O, are :
Water acts as a reducing agent and gets oxidised to
(1) Oxidising action in acidic medium.
O, or Os.
2Fe**(aq) + H,O,(aq) + 2H"(aqg) —> 2Fe**(aq)
F, acts as an oxidising agent and itself gets reduced
+ 2H,O
to F’,
(iz) Oxidising action in basic medium.
Q. 20. Complete the following chemical reactions.
Mn?*(aq) + H,0,(aq) + 20H” —> Mn0O,{s)
(i) PbS (s) + H,O,(aq) —>
+ 2H,O(/)
(ii) MnO;j(aq) + H,O,(aq) — (zz) Reducing action in acidic medium
(ui) CaO(s) + H,O(g) —} 2MnO, (aq) + 6H*(aq) + 5H,0,(aq) —>
(ww) AICI,(g) + H,OW) —> 2Mn**(aq) + 8H,O(/) + 50,(g)
(v) Ca,N,(s) + H,OW) — (iv) Reducing action in basic medium
Classify the above into (a) hydrolysis, (6) redox 2Fe**(aq) + H,O,(aq) + 20H (aq) — 2Fe**(aq)
and (c) hydration reactions. + O,(g) + 2H,O,(/)
MODERN'S abc + OF CHEMISTRY-XI

Q. 26. What is meant by ‘demineralised’ water and Ans, Heavy water is injurious to plants, animals and
how can it be obtained ? human beings because it slows down the rates of
Ans. Water which is free from all cations and anions is reactions occurring in them. Thus, heavy water
called demineralised water. It is obtained by does not support life so well as does ordinary water.
passing hard water first through cation exchange Q. 31. What is the difference between the terms
resin and then through anion exchange resin. For ‘hydrolysis’ and ‘hydration’?
detail, Refer Text (Page 21). Ans. Hydrolysis is the interaction of H* and OH™ ions of
Q. 27. Is demineralised or distilled water useful for water with the anion and cation of a salt respectively
drinking purposes ? If not, how can it be made to form acid and base. For example,
useful? Na,CO.(s) + 2H,O —=> 2NaOH + H,CO,
Ans. Demineralised or distilled water is not useful for Salt Base Acid
drinking purposes since it does not contain even Hydraton is the interaction of water with the salts
useful minerals. Therefore, to make water useful to forms coordinated or hydrated ions or hydrated
for drinking purposes, useful minerals in proper salts.
amounts should be added to demineralised or Na*Cl(s) + nH,O —> Na‘*(H,O), + CI(H,O),
distilled water.
Ni* + 6H,0V7)—— Ni(—H,0), (aq)
Q. 28. Describe the usefulness of water in biosphere CusO,(s) + 5H,0W)—> CuSO,(s). 5H,O0
and biological systems.
Hydrated salt
Ans. Water is essential to all forms of life. Water
constitutes about 65% of the body weight of animals
Q. 32. How can saline hydrides remove traces of water
from organic compounds?
and plants and is essential for their growth. We can
live without food for some days but cannot live
Ans. Saline hydrides (e.g., NaH, CaH,, etc) react with
water forming their corresponding metal hydroxides
without water. It is also a principal constituent of
with the liberation of H, gas. Thus, traces of water
earth’s surface. In comparison to other liquids, it
present in organic solvents can be easily removed by
has characteristic properties such as high specific
distilling them over saline hydrides when H, gas
heat, thermal conductivity, dipole moment, surface
tension, dielectric constant, etc. which make water
escapes into the atmosphere and metal hydroxide is
left in the flask while dry organic solvent distils over.
to play a key and significant role in biosphere. The
high heat of vaporisation and heat capacity help it Q. 33. What do you expect the nature of hydrides is, if
for moderation of the climate and body temperature formed by elements of atomic numbers 15, 19, 23
of living beings. It is an excellent solvent for and 44 with dihydrogen? Compare their behaviour
transportation of minerals and other nutrients for towards water.
plant and animal metabolism. This also plays an (z) Element with Z = 15 is a non-metal (7.e., P) and
important role in photosynthesis in plants which hence form covalent hydride (PH,).
releases oxygen in the atmosphere. (iz) Element with Z = 19 is an alkali metal (K) and
. What properties of water make it useful as a hence form ionic or saline hydride (K*H).
solvent ? What types of compound can it (iit) Element with Z = 23 is a transition element
(t) dissolve, and (ii) hydrolyse ? (i.e., V) of group 5 and hence forms metallic or
Ans. Water has a high dielectric constant (78.39) and interstitial hydride (z.e., VH,¢).
high dipole moment (1.84D). Because of these (iv) Element with Z = 44 is a transition metal (Ru)
properties water has the capacity to dissolve most of group 8 and therefore, it does not form any
of the inorganic substances, ionic as well as covalent hydride.
and is, therefore, regarded as a universal solvent. Behaviour towards water. Only ionic hydrides
Ionic compounds dissolve in water due to ion-dipole (e.g., K*H~) react with water evolving H, gas as
interactions or solvation of ions whereas covalent 2KH (s) + 2H,O() ——— 2KOH (aq) + 2H, (2)
compounds such as urea, glucose, amines, alcohols, Water
sugar, etc. dissolve in water due to hydrogen bonding.
. Do you expect different products in solution when
Water can easily hydrolyse many ionic and covalent aluminium (IIT) chloride and potassium chloride
compounds of metals and non-metals. In these treated separately with (i) normal water (ii)
reactions, H* and OH ions of water interact with acidified water, and (iti) alkaline water ? Write
anions and cations of compounds to form acids and equations wherever necessary.
bases. For exmaple, Ans. AICI, is a salt of a weak base Al (OH), and a strong
CaOQ(s) + HOW) — Ca(OH),(aq) acid HCl. Therefore, in normal water, it undergoes
SO0,(g) + HOW) — H,SO,(aq) hydrolysis as :
P,O,,(s) + 6H,OW — 4H.,PO,(aq) AICI,(s) + 3H,OW@) —> Al(OH).(s) + 3H" (aq)
CaH,(s) + 2H,O(/) —> Ca(OH),(aqg) + 2H,(g) + 3CI (aq)
CaC,(s) + 2H,O() — Ca(OH),(ag) + C,H,
In acidic water, H” ions react with Al(OH), to form
acetylene
Al®*(aqg) ions and water. Thus, in acidic water AICI,
For more reactions, Refer Text.
exists as Al°* (ag) and Cl (aq) ions.
Q. 30. Knowing the properties of H,O and D,O, do
acidified
you think that D,O can be used for drinking
AICI,(ss) water
Al**(ag) + 3CI(aq)
purposes?
HYDROGEN

In alkaline water, Al(OH), reacts to form tetra action of H,O, is due to the oxidation of colouring
hydroxoaluminate complex or metaluminate ion : matter by nascent oxygen. It is used for bleaching
Al(OH),(s) + OH (ag) ——> materials like ivory, silk, wool, feathers, etc.
[Al(OH),! or AIO, + 2H,O(/) . What do you understand by the terms :
Tetrahydroxo Metaaluminate (4) hydrogen economy, (ii) hydrogenation,
aluminate ion ion (fiz) syn gas, (iv) water-gas shift reaction
The complete reaction may be written as :
(v) fuel cell.
AICI,(s)2328225
water
[AMOH),F (aq) + 3Cl (aq) Ans. (i) Refer Text Page 31.
OH” (it) Refer Text Chemical properties of H,; Property
5(i1); Page 7.
AlO; + 2H,O()) + 3CI(aq)
(111) Refer Text Commercial preparation of hydrogen;
Potassium chloride is a salt of a strong acid and a Page 6.
strong base. It does not undergo hydrolysis in normal
water. It dissociates to K*(aq) and Cl (aq) ions as: (1v) Refer Text Commercial preparation of pure
hydrogen; Page 6.
KCl(s 22> K*(aq) + CI-(aq) (v) Fuel cell: It is a device which converts the
Therefore, the aqueous solution of KCl is neutral energy produced during the combustion of a fuel
and hence the ions donot react in acidified or alkaline directly into electrical energy. Dihydrogen is
water. used in hydrogen-oxygen fuel cell for generating
Q. 35. How does H,O, behave as a bleaching agent ? electrical energy. It has many advantages over
Ans. Hydrogen peroxide acts as a bleaching agent due to conventional fossil fuels and electric power. It
the release of nascent oxygen : does not produce any pollution and releases
H,O, ———> H,O + O larger energy per unit mass of fuel in comparison
The nascent oxygen combines with colouring matter to gasoline and other fuels.
which in turn gets oxidised. Thus, the bleaching

Exemplar Problems //
Subjective Questions
Ans, H,O : Covalent or molecular hydride
»> Short Answer Questions <4
B,H, : Electron deficient molecular hydride
Q.1. How can production of hydrogen from water NaH : ionic hydride
gas be increased by using water gas shift . If same mass of liquid water and a piece of ice
reaction ? is taken, then why is the density of ice less than
Ans. Water gas is produced when superheated steam is that of liquid water ?
passed over red hot coke or coal at 1270 K in the
Ans. Since water expands on freezing, therefore, volume
presence of nickel as catalyst.
of ice for the same mass of water is more than liquid
C(s) + H,Og)—> i,
CO(g)+H,(g)
et
— 121.3 kJ water. Hence mass per unit volume /.e., density of ice
Steam Water gas
is lower than liquid water.
Q.5. Complete the following equations:
Pure H, from water gas cannot be obtained (a) PbS (s) + H,O, (aq) —>
easily because it is difficult to remove CO. Therefore,
to increase the production of H, from water gas, CO is (6), CO (g) + 2H, (g)) —Sobatt
Cobalt
oxidised to CO, bymixing it with more steam and passing
the mixture over heated FeCrO, catalyst at 673 K. Ans. (a) PbS (s) + 4H,O, (aq) —> Pb SO,(s) + 4H,O()
H, +CO +H,O Seago CO, + 2H,
Se
,
= Cobalt
Water gas
The CO, produced is removed by scrubbing with Methanol

sodium arsenite solution.


Q.2. What are metallic/interstitial hydrides ? How do
Q.6. Give reasons :

they differ from molecular hydrides ? (t) Lakes freeze from top towards bottom.
Ans. Refer Text; Page 13 and 11. (i) Ice floats on water.
Q.3. Name the classes of hydrides to which H,O, Ans. (1) Refer HOTS, Q.13 (Page 44),
B, H, and NaH belong. (it) Refer Q.4.
MODERN'S abc + OF CHEMISTRY-XI

Q.7. What do you understand by the term ‘auto H,O D,O


protolysis of water ? What is its significance ? Melting point / K 373.0 374.4
Ans, Refer NCERT Qs. Q.18.
Enthalpy of vapourisation40.66 41.61
Q.8. Discuss briefly de-mineralisation of water by
at (373 K)/kJ mol!
ion exchange resin.
Ans, Refer Text; Page 21. Viscosity / centipoise 0.8903 1.107
Q.9. Molecular hydrides are classified as electron On the basis of this data explain in which of
deficient, electron precise and electron rich these liquids intermolecular forces are
compounds. Explain each type with two stronger?
examples. . Intermolecular forces of attraction are stronger in
Ans, Refer Text; Page 11-12. D,O than in water.
Q.10. How is heavy water prepared ? Compare its Q.15. Dihydrogen reacts with dioxygen (O,) to form
physical properties with those of ordinary water. Write the name and formula of the product
water. when the isotope of hydrogen which has one
Ans, Refer Text; Page 19-20. proton and one neutron in its nucleus is treated
Q.11. Write one chemical reaction for the preparation with oxygen? Will the reactivity of both the
of D,O,. isotopes be the same towards oxygen ? Justify
Ans, D,0O, can be prepared by the reaction of D,SO, your answer.
dissolved in water over BaQ,. Ans. The isotope of hydrogen containing one proton and
BaO, + D,SO, —> BaSO, + D,O, one neutron is deuterium (D).
Q.12. Calculate the strength of 5 volume H,O, 2D,(g) + O,(g) —} 2D,0
solution. Deuterium oxide
Ans. 5 volume H,QO, solution means that 1 L of
Since D — D bond is stronger than H — H bond,
5 volume H,O, on decomposition gives 5L of O, at
therefore, D, is less reactive towards oxygen than H,.
NTP.
Q.16. Explain why HCl is a gas and HF is a liquid.
H,O,
— 2H,0 + O,
Ans. Refer text. (because of hydrogen bonding).
68 g 22.4 L at NTP
Q.17. When the first element of the periodic table
22.4 L of O, at NTP is produced from H,O, = 68 g is treated with dioxygen, it gives a compound
whose solid state floats on its liquid state.
68
5 L of O, at NTP is produced from H,O, = x5 This compound has an ability to act as an acid
29.4 as well as a base. What products will be
= 15,18 ¢ formed when this compound undergoes auto
ionisation?
But 5L of O, at NTP is produced from 1 L of 5 volume
Ans. The first element is hydrogen and its molecular form
H,0,
is dihydrogen (H,). It reacts with oxygen to form
Strength of H,O, in 5 volume H,O, = 15.18 g/L water whose solid state is ice which is lighter than
water and floats over water.
15.18 Water is amphoteric in nature /.e., it acts as an acid
Percentage strength of H,O, solution = nin x 100
in the presence of strong bases and acts as a base in
the presence of strong acids.
= 1.518 %
As an acid
Q.13. ({) Draw the gas phase and solid phase structure
of H,0,,. H,OW) + NH,(aq) — NH; (ag) + OH (aq)
(i) H,O, is a better oxidising agent than water. acid base
Explain. As a base
Ans, (1) Refer Fig. 9; Page 28. H,O (/) + H,S (aq) —> H,O* + HS™
(ii) H,O, is better oxidising agent than water because base acid
Due to amphoteric nature, water undergoes self
(a) it oxidises an acidified KI solution to I, which
ionization as
gives blue colour with starch solution but water
H,O + H,O —> H,O* + OH”
does not.
Q.18. Rohan heard that instructions were given to
(b) H,0, turns black PbS to white PbSO P but water the laboratory attendent to store a particular
does not. chemical i.e., keep it in the dark room, add some
Q.14. Melting point, enthalpy of vapourisation and urea in it, and keep it away from dust. This
viscosity data of H,O and D,O is given below : chemical acts as an oxidising as well as a
HYDROGEN
vat
reducing agent in both acidic and alkaline concentrated solution of hydrogen peroxide be
media. This chemical is important for use in obtained ?
the pollution control treatment of domestic Ans. H,O, cannot be concentrated simply by heating
and industrial effluents. because it decomposes much below its boiling point.
(¢) Write the name of this compound. Therefore, concentration of H,O, is carried out in a
(zi) Explain why such precautions are taken for number of stages. Refer Text; Page 24.
storing this chemical.
Q.30. Why is hydrogen peroxide stored in wax-lined
Ans. (1) Hydrogen peroxide, H,Q,. bottles ?
(it) H,O, decomposes in the presence of light and
Ans. Hydrogen peroxide is decomposed by the rough
dust particles. Therefore, it is stored in wax-lined surfaces of glass, alkali oxides present in it and light.
glass or plastic vessels in the presence of
Therefore, to prevent its decomposition, H,O, is
stabilizers like urea. It is also kept away from usually stored in coloured paraffin wax coated plastic
dust.
or teflon bottles.
Q.19. Give reasons why hydrogen resembles alkali Q.31. Why does hard water not form lather with
metals ? soap ?
Ans, Refer Text; Page 1.
Ans. Refer Text; Page 20.
Q.20. Hydrogen generally forms covalent compounds.
Q.32. Phosphoric acid is preferred over sulphuric
Give reason.
acid in preparing hydrogen peroxide from
Ans, Refer Text; Page 2.
peroxides. Why ?
Q.21. Why is the ionisation enthalpy of hydrogen
Ans. H,SO, acts as a catalyst for decomposition of H,O,.
higher than that of sodium ?
Therefore, some weaker acid such as HPO, is
Ans, The size of hydrogen is smaller than that of sodium
preferred over H,SO, for preparing H,O, from
and therefore, the ionization enthalpy of H is higher
peroxides
(1312 kJ mol’) than that of Na (496 kJ mol™’).
Q.22.Basic principle of hydrogen economy is 3BaO, + 2H,PO, — > Ba,(PO,), + 3H,O,
transportation and storage of energy in the form Q.33. How will you account for 104.5° bond angle in
of liquid or gaseous hydrogen. Which property water?
of hydrogen may be useful for this purpose ? . Refer Text (because of presence of 2 lone pairs); Page
Support your answer with the chemical equation 15-16.
if required. Q.34. Write redox reaction between fluorine and
Ans. Basic property of hydrogen economy is that it can be water.
converted into liquid by cooling under high pressure
and therefore, can be easily transported, Refer Text; Ans. 2F (g) + 2H,O(W) ——> 4H"(aq) + 4F
(aq) + O,
Page 993. Q.35. Write two reactions to explain amphoteric nature
Q.23. What is the importance of heavy water ? of water.
Ans, Refer Text; Page 19. Ans. Refer Text; Page 17 and Q. 17.
Q.24. Write the Lewis structure of hydrogen
peroxide.
»> Long Answer Questions [Caraina Fmarto C4
Ans. Q.36. Atomic hydrogen combines with almost all
elements but molecular hydrogen does not.
Explain.
Q.25. An acidic solution of hydrogen peroxide behaves
as an oxidising as well asreducing agent. Illustrate Ans. Atomic hydrogen is highly unstable and hence very
it with the help of a chemical equation. reactive. Therefore, it combines with almost all the
Ans, Refer Text page 25. elements. However, molecular hydrogen has large
With the help of suitable examples, explain the bond dissociation enthalpy (458.5 kJ mol!) and
Q.26.
property of H,O, that is responsible for its therefore, less reactive.
bleaching action ? Q.37. How can D,O be prepared from water ? Mention
Ans, Refer Text; Page 27. the physical properties in which D,O differs
from H,O. Give at least three reactions of
Q.27. Why is water molecule polar ?
D,O showing the exchange of hydrogen with
Ans, Due to difference in electronegativity between O and
deuterium.
H and angular shape. Refer Text; Page 16.
Ans. Refer Text; Page 19.
Q.28. Why does water show high boiling point as
compared to hydrogen sulphide ? Give reasons for Q.38. How will you concentrate H,O,? Show
your answer. differences between structures of H,O, and
Ans, Due to hydrogen bonding. Refer Text; Page 16. H,O by drawing their spatial structures. Also
mention three important uses of H,O,.
Q.29. Why can dilute solutions of hydogen peroxide
not be concentrated by heating. How can a Ans. Refer Text; Page 24, 27-28.
MODERN'S abc + OF CHEMISTRY-XI

Q.39. (i) Give a method for the manufacture of (i) For structure Refer Fig. 9; Page 28.
hydrogen peroxide and explain the reactions (ii) 2H,0, —U82, 2H.0 + O,
involved therein.
Q.42. An ionic hydride of an alkali metal has
(zi) Illustrate oxidising, reducing and acidic significant covalent character and is almost
properties ofhydrogen peroxide
with equations. unreactive towards oxygen and chlorine. This
Ans . Refer Text; Page 23, 24,25. is used in the synthesis of other useful hydrides.
Q.40 . What mass of hydrogen peroxide will be present Write the formula of this hydride. Write its
in 2 litres of a 5 molar solution ? Calculate the reaction with Al,Cl,.
mass of oxygen which will be liberated by the Ans. Since the ionic hydride of alkali metal has significant
decomposition of 200 mL of this solution. covalent character, therefore, it is LiH. Since LiH is
Ans. Molar mass of H,O, = 34 g mol very stable, therefore, it is almost unreactive towards
O, and Cl,.
1L of 5M solution of H,O, will contain H,O,
It reacts with Al,Cl, to form lithium aluminium
= 34 x 5g
hydride
2L of 5M solution of H,O, will contain H,O,
8LiH + Al,Cl, —~ 2LiAlH, + 6LiCl
=2x 34x 5= 340 g
Q.43. Sodium forms a crystalline ionic solid with
200 mL of 5M solution will contain H,O, dihydrogen. The solid is non-volatile and non-
_ 340 conducting in nature. It reacts violently with water
x 200 = 34 g
~ 2000 to produce dihydrogen gas. Write the formula of
2H,0O, —> H,0 + O, this compound and its reaction with water. What
68 g of H,O, on decomposition will give O, = 32 g will happen on electrolysis of the melt of this
solid?
.. 34 g of H,O, on decomposition will give O,
Ans. Sodium reacts with dihydrogen to form sodium
ue 34 == 16 hydride which is a crystalline ionic solid.
=
68
_—_
. :

2Na + H, —> 2NaH or 2Na*H™


Q.41. A colourless liquid ‘A’ contains H and O elements It reacts with water to produce H, gas.
only. It decomposes slowly on exposure to light.
2NaH + 2H,O ——> 2Na0OH + H,(g)
It is stabilised by mixing urea to store in the
Although Na*H™ does not conduct electricity in the
presence of light.
solid state, but electrolysis of its melt give H, gas at
(t) Suggest possible structure of A. anode and Na at cathode.
(ii) Write chemical equations for its decomposition 2NatH- Electrolysis
reaction in light.
SO Na ll) + H,(g)
At cathode At anode
Ans. The liquid A is hydrogen peroxide (H,O,).

QUICK Memory TEST W


B. Complete the missing links
Say True or False |
. The electrolysis of molten sodium hydride liberates
1. Decomposition of hydrogen peroxide is an exothermic
hydrogen gas at
reaction.
. Heavy water was discovered by ...........
2. Electrolysis of HCl(aq) gives hydrogen at cathode. . The hardness due to the presence of chlorides and
3. Syngas is a mixture of H, and CQ,. sulphates of calcium and magnesium dissolved in it
is called hardness.
pe . Dihydrogen reacts with CO at 700K in the presence
. Heavy water is used as ........... in nuclear reactors.
of a catalyst ZnO/Cr,O, to form formaldehyde.
. Hydrogen peroxide oxidises hydrogen sulphide to
. Magnesium boride reacts with water to give boric acid.
. Water has maximum density at 277K. . 11.2 L of O, at N.T.P. is produced from
H,,.
. Heavy water was discovered by Urey.
. Among HF, NH, and H,O, maximum hydrogen
1.
oo
OF
S&S The bleaching action of H,O, is permanent and is due bonding occurs in ...........
to oxidation. . The isotope of hydrogen commonly used as a tracer
. The permanent hardness of water is due to chlorides in organic reactions is ...........
and sulphates of calcium and magnesium. . Bleaching action of H,O, is due to its ........... nature.
10. Hydrogen gas is liberated by the action of aluminium
10. Calgon is Na,[Na,(PO,)a].
with concentrated solution of ..........
HYDROGEN

5. Calcium and magnesium ions in water combine with


uf
‘635 Choose the correct alternative
Calgon to form soluble /insoluble complex ions.
1. Most of the d- and f-block elements form metallic/ 6. MnO, /acetanilide checks the decomposition of H,O,.
covalent hydrides.
7. The bleaching action of H,O, is due to reduction/
2. Methane is an electron precise /electron rich molecular oxidation.
hydride.
8. Viscosity of D,O is more/less than H,O.
3. At OK, hydrogen contains mainly para/ortho
9. Electron deficient molecular hydride is B,H,/NH,.
hydrogen.
10. Molecules of hydrogen in which the spins of both the
4, The hardness in water due to the presence of
nuclei are in the same direction is called ortho/ para
bicarbonates of calcium is called permanent/
hydrogen.
temporary hardness.

Memory Test “\
A, BREVA More goles 5. sulphur 6. 34
7. HF 8. deuterium
1. True 2. True
3. False : Syngas is a mixture of H, and CO. 9. oxidising 10. sodium hydroxide
4. False : It forms methanol. C, BG iti emaiteec wate linswitiokus
5. False : [t forms diborane.
6. True 7. True 8. True 9. True 1. metallic 2. electron precise
10. False : Calgon is Na,[Na,(PO,)<]. 3. para 4, temporary
5. soluble 6. acetanilide
7. oxidation 8. more
1. anode 2. Urey
9. BH, 10. ortho.
o. permanent 4, moderator

& Advanced Level | —


QUESTIONS WITH ANSWERS
Q.1. Why is hydrated barium peroxide used in Q. 3.Which element can oxidise water to O,?
the preparation of hydrogen peroxide instead of Justify.
anhydrous barium peroxide ? Ans. Fluorine being more electronegative than oxygen
Ans. If anhydrous barium peroxide is used in the can oxidise water to oxygen. In this, water acts a reducing
preparation of H,O,, the barium sulphate formed during agent and hence itself gets oxidised to O,.
the reaction forms an insoluble protective coating on the
2F (g) + 2H,OW) —> 4H*(aq) + 4F-(aq) + O,(g)
surface of solid barium peroxide.
Q.4. Dilute solution of hydrogen peroxide cannot be
BaO,(s) + H,SO,(aq) —> BaSO,(s) + H,O,(aq)
heated strongly for its concentration. Explain.
Insoluble
Ans. Dilute solution of H,O, cannot be concentrated by
This prevents further action of acid and therefore, heating because it decomposes into H, and O, on heating.
stops the reaction. If, however, hydrated barium peroxide
(in the form of thin paste) is used, the water of crystallisation H,O, —“ H, +0,
does not allow BaSO, to deposit on the surface of BaO, and Q.5. Can sodium bicarbonate make water hard ?
the reaction goes to completion. Explain.
Q.2.Hydrogen is not prepared by the reaction of Ans. Sodium bicarbonate cannot make water hard
because soaps themselves are sodium salts of fatty acids
concentrated sulphuric acid on zinc. Explain.
which are soluble in water.
Ans. Concentrated sulphuric acid is not used for the Q.6. Why is phosphoric acid preferred to sulphuric
preparation of hydrogen because it reacts with zinc to form acid in the preparation of H,O, from barium peroxide?
SO, gas instead of hydrogen. Explain.
Zn + 2H,SO,(cone.) —> ZnSO, + SOQ, + 2H,O Ans. The aqueous solution of H,O, prepared by the
action of dil. H,SO, on hydrated BaO, contains impurities

WWW.JEEBOOKS.IN
MODERN'S abc + OF CHEMISTRY-XI

of heavy metal ions such as Ba?*, Pb?*, etc. These catalyse lower temperature floats over the water below it. Due to
the decomposition of H,O, and therefore, the solution cannot this the freezing of water into ice occurs continuously from
be stored for a long time. To prevent this, phosphoric acid is top towards bottom.
preferred to sulphuric acid because impurities of heavy Q.14. Amixture of H,O, and hydrazine with copper
metal ions get completely precipitated as insoluble (II) is used as a rocket propellant. Why ?
phosphates and the solution does not contain any free heavy
Ans. The reaction between hydrazine and H,O, in the
metal ions to catalyse the decomposition of H,O,. Therefore,
it can be safely stored for a longer time. presence of Cu(II) is highly exothermic and is accompanied
by a large increase in volume of the products. Therefore,
Q.7.Statues coated with white lead on long exposure
this mixture is used as a rocket propellant.
to atmosphere turn black and their original colour can be
restored on treatment with H,O,. Why ? 2H,0,(/) + H,NNH,(/) Cul),
— Nu(g)T + 4H,O(@)T
Ans. On long exposure to atmosphere, white lead gets Hydrazine
converted to black PbS due to action of H,S present in the Q.15. Water cannot be used to extinguish petrol fires.
atmosphere. Why ?
PbO, + HS — PbS + 2H,O Ans. Water is used to extinguish most fires because it
from atmosphere Black lowers the temperature of burning material. However, in case
As a result, statues turn black. On treatment of these of petrol fires, petrol being lighter than water, floats over
black statues with H,O,, the black PbS gets oxidised to water and hence fire spreads instead of being extinguished.
PbSO, and their colour is restored. Q.16. Ferric chloride is reduced when zinc and
PbS + H,O, —> PbSO, + 4H,O hydrochloric acid are added to its solution and not by
Q.8. Which of the two : hydrogen or deuterium passing H, gas through its solution. Explain
undergoes reactions more rapidly and why ? Ans, Ordinary H, is less reactive and therefore, it does
Ans.Hydrogen undergoes reactions more rapidly than not reduce acidified FeCl, solution. However, when zinc is
deuterium. This is because of mass differences. For example, added to acidified FeCl, solution, nascent hydrogen is
reaction between hydrogen and chlorine is about 14 times produced which has enormous energy. It is more reactive
faster than between deuterium and chlorine. than ordinary H, and reduces acidified FeCl, solution.
Q. 9. Why can't sea animals live in distilled water ? FeCl, + H, —— No reaction
Ans. Distilled water does not contain dissolved Oy. Zn + H,SO,—> ZnSO, + H,°
Therefore, sea animals cannot live in distilled water because nascent hydrogen
of the absence of dissolved oxygen for breathing. 2FeCl, + H,” — > 2FeCl, + 2HCl
Q.10. Concentrated sulphuric acid cannot be used Green
for drying H,. Why ? Q.17. Hard water is softened before using in boilers.
Ans. Conc. H,SO, on absorbing water from moist H, Explain.
produces so much heat that H,, catches fire. Ans. Hard water on boiling forms precipitates of
Q.11. Can we remove completely temporary hardness MgCO,, CaCO, and CaSO, which form scales in the boilers.
due to Mg(HCO,), by boiling? As of result of these scales in the boilers, the boiler gets
Ans. Temporary hardness of water due to deteriorated due to over heating. Moreover, these scales are
Mg (HCO,), can be completely removed by boiling because non-conducting and therefore, more fuel is consumed.
soluble Mg(HCO,), is converted into insoluble MgCO, which Therefore, in order to prevent the formation of scales, hard
can be easily removed by filtration. water is softened before using in boilers.
Mg(HCO,), —2%-» MgCO, + CO, + H,O Q.18. Why is hydrogen peroxide stored in wax-lined
Soluble Insoluble plastic coloured bottles?
Q.12. H,O, acts as an oxidising agent as well as Ans. The decomposition of H,O, occurs readily in the
reducing agent. Why ? presence of rough surface (acting as catalyst). It is also
decomposed by exposure of light. Therefore, wax-lined
Ans. In H,O,, oxygen has -1 oxidation state which lies
between maximum (0 or +2 in OF,) and minimum —2, smooth surface and coloured bottles retard the
Therefore, oxygen can be oxidised to O, (zero oxidation state) decomposition of H,O,.
acting as reducing agent or can be reduced to H,O or OH- Q.19. Presence of water is avoided during the
(—2 oxidation state) acting as an oxidising agent. preparation of H,O, from Na,O,. Why ?
—l 0 Ans. Water present during the reaction, reacts with
O, —— _O, + 2e Na,O, to form NaOH which tends to decompose H,QO,.
Reducing agent Na,O, + 2H,O —> H,0, + 2NaOH
-] 2
2H,O, —82#-;2H,0 + 0,
O, + 2e°-—— 20
Q.20. An aqueous solution of an inorganic compound
Oxidising agent
(X) shows the following reactions :
Q.13. Why do lakes freeze from top towards
(t) It decolourises an acidified KMnO, solution
bottom ?
accompanied with evolution of oxygen gas.
Ans. The density of ice is less than that of liquid water.
(it) It liberates iodine from acidified potassium iodide
Therefore, ice floats on the surface. Thus, the ice layer at
solution.
HYDROGEN

(riz) It gives brown precipitate with alkaline


KMnO, solution with evolution of oxygen gas. Normality of H,O, = a =1.79N
(iv) It removes black stains from old oil paintings. Applying normality equation,
Identify (X) and give chemical reactions for steps
(i) to (iv). N,V, N, Vz
H,O, | KMnO,
ee. ee a — a

Ans. Since X removes black stains from old oil


paintings, it is hydrogen peroxide H,O,. (H,O,) KMnO,
(1) 5H,0, + 2KMnO, + 3H,SO, ——> Lid x Vv, = 2x 200
(X) K,SO, + 2MnSO, + 8H,O + 50,
2x 200
(iz) H,O, + 2KI + H,SO, ——> I, + K,SO, + 2H,O Vi a 1 7 = 223.5 mL

(wi) 3H,O, + 2KMnO, ——>


Q.23. 0.5 L each of three samples of H,O, labelled
(X) 2 MnO, + 2 KOH + 30, + 2H,O 10 vol, 15 vol and 20 vol are mixed and then diluted with
Brown ppt equal volume of water. What is the strength of resultant
(iv) 4H,0, + PbS ——— PbSO, + 4H,O H,0, solution ?
(black) white Ans. Volume strength of H,O, solution
Q.21. To a 25 mL H,O, solution, excess of acidified = 5.6 x Normality
solution of KI was added. The iodine liberated required Vol. strength
20.0 mL of 0.3 N Na,S,O, solution. Calculate the volume Normality = RG
strength of H,O, solution. ; 10
Normality of 10 vol of HO, = Be
Ans. Applying normality equation,

N,V, _ Ny Vo (15
Normality of 15 vol of H,O, = Reo
Pith. Na,8,0.

N, x 25 = 0.3 x 20 —— 20
Normality of 20 vol of H,O, = en
N, =2:3%20 _ 9 o4N
25 Let 500 mL of each solution is mixed and then total
volume of mixture becomes = 1500 mL. Since it is diluted
Strength of H,O, = 0.24 x 17 = 4.08 g/L
with equal volume of water, total volume becomes 3000 mL.
2H,0, —- 2H,O + QO,
Applying normality equation
2x34=68¢ 22400 mL
68 g of H,O, produce O, at N.T.P. = 22400 mL
4.08 g of H,O, will produce O, at N.T-P. 10x500 15500 20500
+ + - )
5.6 5.6 56 7Ng x 3000
_ 22400 x4.08 = 1344 mL
N= os g39 N
4-5 6x3000
Now 4.08 g of H,O, are present in 1000 mL of solution. Volume strength of resulting solution = 1.339 x 5.6 = 7.5
.. 1000 mL of H,O, solution give 1344 mL of O, at N.T-P. Q.24. The degree of hardness of a given sample of
(1344 hard water is 60 ppm. If the entire hardness is due to
.. 1mL of H,O, solution will give = ana 1.344 mL of
0 MgsO,, how much of MgSO, is present per kilogram of
O, at N.T.P. hard water ?
.. Volume strength of H,O, = 1.444 Ans. Degree of hardness of water = 60 ppm
Q.22. Calculate the volume of 10 volume H,0O, Since degree of hardness is the number of parts of
required to neutralise 200 mL of 2N KMn0O, in acidic calcium carbonate or equivalent to calcium and magnesium
medium. salts present in a million parts of water by mass,
Ans. 2H,0, —> 2H,O + QO, ‘. 10° g of water contain 60 g of CaCO,.
68 g 22.4 L at N.T.P. Now 1 mol of CaCO, = 1 mol of MgSO,
22.4 L of QO, at N.T.P. is produced from H,O, = 68 g 100 g of CaCO, = 120 g of MgSO,
60120 _
10 Lof O, at N.T.P. is produced from H,O, = — x10 10® g of water contain MgSO, = 72
100 6
= 30.357 g 72
10° g of water will contain MgSO, = 7 10° = 0.072 g
30.357 -1.79
Gram equivalents of H,O, =
.. 1 kg of water contains MgSO, = 72 mg
MODERN'S abc + OF CHEMISTRY-XI

Vilelem cen el tiie] eam Carrying 2 or 3 marks


<4
. Name the three isotopes of hydrogen. . How is H,O, manufactured by the electrolysis of
. What is meant by 10 volume hydrogen peroxide ? water ? Give its structure.
. What are the ways in which the water molecules
. Name the element which when treated with dilute
bonded to anhydrous salt to form a hydrate ?
H,SO, gives pure hydrogen.
. Name the class of hydrides to which NaH, H,O,, B,H,
. Give a compound in which hydrogen exists in the
and LaH, belong. What is understood by hydride
negative oxidation state.
gap ?
. Give two advantages of using hydrogen as fuel over
. Compare the structural properties of H,O and H,Q,.
gasoline.
. What causes the temporary and permanent hardness
. Explain why H,O, cannot be stored for prolonged of water ?
periods? . Explain why hydrogen is best placed separately in the
. Why does H,O have a higher melting and boiling points periodic table of elements.
as compared to H,S? . Discuss the principle and method of softening of hard
. What is heavy water ? What is the mass of 1 mole of water by synthetic ion exchange resins.
heavy water ? . Discuss the structure of common form of ice.
. Name two metals which can displace hydrogen from . Show how can hydrogen peroxide function both as an
acids. oxidising and reducing agent.
10. Which isotope of hydrogen is radioactive in nature ? 10. Complete the following reactions :
11. Can sodium bicarbonate make water hard ? (7) AIN + H,Q ——> (wz) CaC, + H,Q ——>
12. What is meant by autoprotolysis of water ? 11. How does H,O, react with (7) potassium ferricyanide
(iz) lead sulphide ?
13. Which of the two Cl, or Br, more reactive towards
12. What is the structure of hydrogen peroxide ? Give its
dihydrogen?
four uses.
14, How does H,O, react with lead dioxide ?
13. Why is anhydrous barium peroxide not used in the
15. The bleaching action of H,O, is due to oxidation or preparation of H,O,? How does hydrogen peroxide
reduction? react with :
16. What happens when peroxydisulphuric acid is (z) chlorine (22) moist silver oxide
hydolysed with water ? (111) sodium carbonate ?
17. Give an example each of ionic hydride and covalent 14, How is hydrogen peroxide manufactured ?
hydride. Explain why H,O, cannot be stored for long periods ?
18. Name the gases produced when water reacts with : 15. Complete the following reactions :
(i) Aluminium nitride (ii) Aluminium carbide. (a) Cl,O. + H,Q ——>
19. Complete the reaction : (6) Sn + NaOH + H,O ——>
SiCL, + H,O —> 16. Compare the structures of H,O and H,O,.
20. Complete the reaction ; 17. What are metallic or interstital hydrides ? How do they
differ from molecular hydrides ?
PbS + H,O, —>
18. Describe some unusual properties of water.
21. Complete the following reactions :
19. How is heavy water prepared from normal water ?
CaO (s) + H,O @) —> Describe.
Na,O (s) + H,O (1) —- 20. Write chemical reactions to show the amphoteric
22. Why do lakes freeze from top towards bottom ? nature of water.
23. Explain why water has higher melting and boiling 21. Hydrogen forms three types of bonds in their
points than H,S., compounds. Describe each type of bonding using
suitable examples.
24, Account for the amphoteric character of water.
22. Explain the following :
25. What is meant by hydrogenation ?
(a) A solution of H,O, cannot be stored for a long
26. How is temporary hardness in water removed? period.
27. Which cations cause hardness in water? (6) Anhydrous BaQO, cannot be used to prepare H,O,
28. Which anions cause hardness in water? by reacting with acid.
(c) Liquid hydrogen cannot be used easily as a fuel.
HYDROGEN

23. Explain 48. What are the advantages of using hydrogen as a fuel?
(a) Use of liquid hydrogen as a fuel. 49. Ionic hydrides are frequently used to remove traces of
(b) Uses of H,O,. water from organic compounds. What is the underlying
24, What is heavy water ? How does it react with : basis of this process ?
(1) sodium metal (i) Na,O (ut) CaC, 50. What is meant by demineralized water and how can it
25. How is hydrogen prepared commercially ? Give brief be obtained?
account of isotope of hydrogen.
51. What is the difference between hydrolysis and
26. Give four uses of dihydrogen.
hydration?
27. What happens when H,0O, is treated with :
(i) acidified potassium iodide. 52. Write chemical reaction to justify that hydrogen
(iz) lead sulphide. peroxide can function as an oxidising as well as
reducing agent?
(iit) alkaline solution of potassium ferrocyanide ?
28. Give structure and four uses of hydrogen peroxide.
53. How does H,O, behave as a bleaching agent ?
29. Give equations for the preparation of dihydrogen »> Long Answer Questions Carrying 5 marks <4
(i) from water using reducing agent.
(iz) by Lane’s process. 1. Discuss the position of hydrogen in the periodic table.
30. What is the differences between hydrolysis and 2. Describe the industrial uses of hydrogen which depend
hydration ? upon :
31. Describe the importance of heavy water in nuclear (t) the heat liberated when it burns.
reactors. (iz) the ability to react with vegetable oil in the
32. Explain presence of a catalyst.
(iii) its ability to unite with nitrogen.
(i) lonic hydrides are commonly used to remove traces
of water from organic compounds. 3. (i) How would you prepare dihydrogen from water
using a reducing agent?
(iz) D,O resembles H,O chemically yet it is toxic
substance. (71) How would you prepare dihydrogen from a
substance other than water ?
33. What are covalent and ionic hydrides ? Give one
(iii) How would you prepare pure dihydrogen in the
example of each.
laboratory ?
34. Explain briefly the laboratory preparation for pure
4, What is the mass of H,O, present in 1 litre of a 2M
hydrogen.
solution ? Calculate the volume of oxygen at 8.T\P.
35. What are interstitial hydrides ? Give examples. liberated upon the complete decomposition of 100 cm?
36. Give the preparation and two uses of LiAlH,. of the above solution.
37. What are saline hydrides ? Give examples. 5. When hydrogen is passed over a black solid compound A,
38. What are polymeric hydrides ? Give the preparation the products are grey metal B and a colourless liquid C.
and structure of beryllium hydride. The metal 5 displaces copper from copper sulphate solution
39. What is heavy water ? Give its important uses. and reacts with dilute hydrochloric acid to give hydrogen.
40. Discuss the structure of common form of ice. Why does (i) Name A, B and C.,
ice float over water ? (it) Give the chemical equations involved.
41, Is it correct to say that hydrogen can behave as a (iit) How will you confirm the identity of C ?
metal ? State the conditions under which such 6. How is dihydrogen prepared :
behaviour is possible. (t) from water by using a reducing agent.
42, Klements with atomic numbers 17 and 20 form (iz) in the laboratory in the pure form.
compounds with hydrogen. Write the formulae of these (iit) from hydrocarbons.
two compounds and compare their chemical behaviour. 7. How is hydrogen peroxide commercially prepared by
43. Write the formulae of the compounds of hydrogen with electrolysis of H,SO, solution ? How is it concentrated ?
elements having atomic numbers 9, 11, 12 and 17. Explain its oxidising and reducing character.
Compare their chemical behaviour. 8. Complete the following reactions :
44, Explain : (a) H,S + H,O, —>
(1) Why water has higher boiling and melting points (6) H,O, + 20H” —>
as compared to H,S ?
(c) CaOCl, + H,O, —
(ii) Why do lakes freeze from top towards bottom ?
(d) C,H, + H,O, —>
45. Name the isotopes of hydrogen. What is the importance
(e) K,Cr,0, + H,SO, + H,O, —>
of heavier isotopes of hydrogen ?
9. Write short notes on.
46. Name the class of hydrides which H,O, B,H,, NaH,
(a) Concentration of hydrogen peroxide.
and LaH, belongs. What is understood by hydride gap?
(b) Use of liquid hydrogen as fuel.
47. Although D,O resembles H,O chemically yet it is a
toxic substance. Explain. (c) Heavy water and its uses.

WWW.JEEBOOKS.IN
MODERN'S abc + OF CHEMISTRY-XI

10. Explain the following terms : 12. How is hydrogen peroxides prepared ? Show by proper
(i) diprotium (ii) dihydrogen chemical reactions how can it function both as an
(iii) proton (iv) hydron. oxidising and a reducing agent ? Explain its
11. Describe the industrial applications of hydrogen importance in nuclear reactors.
dependent on
13. What do you understand by the terms :
(i) the heat liberated when its atoms combine on the
surface of a metal. (i) Hydrogen economy (ii) Hydrogenation
(i) its effect on unsaturated organic system in the (iii) Syn gas (iv) Water-shift reaction
presence of a catalyst.
(v) Fuel cell.
(iii) its ability to combine with nitrogen under specific
conditions.

f,
,

Revision Exercises

Very Short Answer Questions Short Answer Questions

2. It means that 1 ml of H,O, will give 10 ml of 10. (i) AIN + 3H,O ——> Al(OH), + NH,
(ii) CaC,+2H,0 —— Ca(OH), + C,H,
oxygen at N.T\P.
32. (Z) H-isastrong Bronsted base which has high
3. Magnesium 4, NaH reactivity with water.
8. 20 9. Zinc, Magnesium (ii) Itis because of slow rate of D* compared to
H* in reactions involved in enzymatic
10. Tritium 13. Cl, catalysis.
14, Oxidation 17. NaH, B,H, 42, HCl (molecular hydride), CaH, (ionic hydride).
43. HF, NaH, MgH, and HCl
18. (i) Ammonia (ii) Methane
27. Ca*,Me™* 28. SO,’, Cl Long Answer Questions

4, 68g, 2240 cm?

Competition Fil Objective Questions

AS. Which of the following metals give hydrogen with very


dil HNO, ?
with only one correct answer (a) Al (b) Mg
(c) Au (d) Sn
Hydrogen
A4, Which of the following is most reactive towards H, ?
Al. The radioactive isotope of hydrogen is (a) Cl, (b) F,
(a) Hydrogen (b) Protium (c) Br, (d) I,
A5. The least abundant isotope of hydrogen is
(c) Deuterium _ (d) Tritium
A2. Which of the following metals cannot liberate hydrogen (a) }H (b) D
from dilute hydrochloric acid ?
(c) a7 (d) both (a) and (6)
(a) Zn (b) Mg A6. Which of the following is not an example of ionic
(c) Fe (d) Cu hydride ?
(a) LiH (b) CaH,
(c) CsH (d) GeH,

Araswets
Al. (d) A2. (d) AS. (b) Ad. (6) AB. (oe) A6. (d)
HYDROGEN

AZ. Hydrogen combines with other elements by


(a) CH, (g) + H,O (g) —“*— CO (g) + H, (g)
(a) losing an electron
(5) gaining an electron (b) CO (g) + H,O (g) 2% > CO, (g) + H, &)
(c) sharing an electron
(d) losing, gaining and sharing of an electron. (c) C,H,,,5 +nH,0 (g) 225 > n CO + (Qn + DH,
A8. In which of the following pair, both the hydrides are i eas Cobalt,
not of the same type ? A18 ae oa re i i Se as df,
(b) CH,, H,S ae : e following compounds
pe re is used for water
(a) LaH,, TiH,

AQ.
Babe VBR
Ortho and para
2) BOE aEy
hydrogen differ in
(a) Cag (PO4),
(c) NagP,Oj.
(b) Na,PO,
(d) Na,HPO,
(a) Atomic number A19, Heavy water is obtained by
(6) Mass number (a) boili
— a) boiling water
(c) Electron spin in two atoms (B tind Time CO
(d) Nuclear spin in two atoms (0) reacting Lau :
A10. Which of the following hydrides is electron-precise (c) prolonged electrolysis of HO
hydride ? (d) heating H,0,.
(a) Bs (b) NH, A20.Water gas is a mixture of
(c) H,O (d) CH, (a) CO, and H,O (b) CO and H,O
All. Hydr
ydrogen resembles halogens in i manylyrrespects for (c) CO and H, (d). CO and N,
which several factors are responsible. Of the following
factors which one is most important in this respect ? Hydrogen Peroxide
(a) Its tendency to lose an electron to form a cation. A21.Hydrogen peroxide can be prepared from
(6) Its tendency to gain a single electron in its valence aa |
shell to attain stable electronic configuration. (a) NaOH (6) BaO,.8H,O
(c) Its low negative electron gain enthalpy value. (¢) Ca(OH), (d) Na,O
(d) Its small size. A22.What is the mass of hydrogen peroxide in 1 L of 3M
A112. Dihydrogen react with CO at 700K in the presence of solution ?
a catalyst ZnO/Cr,O, to form | |
(a) CH, (b) HCHO (a) 10.2 ¢ ©) 102 ¢g
(c) CgH, (d) CH,0H (c) 11.3 g (d) 68 g
Water A23.Decomposition of hydrogen peroxide is prevented by

A138. Calcium phosphide gets hydrolysed and give (a) NaOH (b) MnO,
(a) Ca,(PO,)y (6) PH, (c) glycerol (d@) oxalic acid
(c) H,PO, (d) (HPO), . The oxidation number of oxygen in hydrogen peroxide
Al4, Which of the following is used as a moderator in is
nuclear reactors ? (a) +1 (b) —1
(a) Hard water (b) Heavy water (c) +2 (d) —2
(c) Deuterium (d) Deionized water . The strength of 20 volume of H,O, is
Al15. One mole of calcium phosphide on reaction with excess (a) 18.6 g/litre (6) 60.7 g/litre
of water gives (c) 160 g/litre (d) 20.2 g/litre
(a) one mole of phosphine . Hydrogen peroxide is used as an antiseptic under the
(b) two moles of phosphoric acid name
(c) two moles of phosphine (a) Bleaching powder (6) Perhydrol
(d) one mole of phosphorus pentoxide (c) Nessler's reagent (d) Catechol
A16. The salt responsible for permanent hardness of A9Q7 . 25 volumes of H,0, means
water is (a) 25% H,O,
(a) K,SO, (6) MgCl, (b) 25 cm® of the solution contains 1g of H,O,
(c) Mg(HCO,), (d) NaCl (c) 1 cm? of solution liberates 25 cm? of O, at N.T-P.
A17. Which of the following reaction is an example of use of (d) 25 cm? of the solution contains 1 mole of H,0,,.
water gas in the synthesis of other compounds ? A28. The volume strength of 1.5 N H,O, solution is

§ (a) 4.8 (b) 8.4


Arswes (c) 3.0 (d) 8.0
AZ. (d) A8. (d) <A (d) AlO. (d) All. (6) Al2. (d) Al8. (6b) Ald. (6) Al5. (ec) Al6. (bd)
A1L7.(d) <Al8. (c) A119 (a) A2O. (c) AZl. (6) A222. (6b) As. (c) A224. (6b) AS. (6b) AE. (5)
A27.(c) A28. (bd)
MODERN'S abc + OF CHEMISTRY
-XI

. The oxide which gives H,O, on treatment with dilute A33. Commercial 11.2 volume H,O, solution has a molarity
acid is of
(a) PbO, (b) Na,O, (a) 1.0 (b) 0.5
(c) MnO, (d) TiO, (c) 11.2 (7) 1.12
A30. The structure of H,O, is A34, H,O, acts as an oxidising agent in
(a) Planar (6) Non-planar
(a) neutral medium
(c) spherical (d) linear
(6) acidic medium
A31. The O—O-H bond angle in H,O, is ?
(a) 106° (b) 109°.28' (c) alkaline and neutral medium
(c) 120° (d) 101.9° (d) acidic and alkaline medium
A382. Which of the following equation depicts reducing A35. What is false about H,O, ?
nature of H,O, ? (a) It acts as both oxidising and reducing agent
(a) 2[Fe(CN),I" + 2H* +H,O, ——> 2[Fe(CN), |” + 2H,O (b) Two OH bonds lie in the same plane
(c) It is pale blue liquid
(6) 1, + H,O, + 20H” —— 2r + 2H,O +O,
(d) It can be oxidised by O,
(c) Mn?* +H,0, ——> Mn** + 20H™
(d) PbS + 4H,O, ——> PbSO, + 4H,O

Answer
A29.(b) A30. (b) A31. (d) A382. (b) A33. (a) A34. (d) A35. (b)

B4,. The strength of H,O, (in g/litre) in 11.2 volume solution


of H,O, is
from Competitive Examinations (a) 17 (b) 51
(c) 34 (d) 85
AIPMT & Other State Boards’ Medical Entrance (e) 68 (Kerala P.M.T: 2012)
Bl. The degree of hardness of water is usually expressed B5. The inorganic compound obtained by the auto-
in terms of oxidation of 2-alkylanthraquinol is
(a) ppm by weight of MgSO, (a) H,O (b) H,O,
(6) g/L of CaCO, and MgCO, present (c) H, (d) O,
(c) ppm by weight of CaCO, irrespective of whether it (e) H,SO, (Kerala P.M.T. 2014)
is actually present B6. (i) H,O, + O, —> H,0 + 20,
(d) ppm of CaCO, actually present in water (it) H,O, + Ag,O —> 2Ag + H,O + 20,
(A.M.U. Med. 2010) Role of hydrogen peroxide in the above reactions is
B2. Permanent hardness of water is due to the presence of respectively
(a) bicarbonates of sodium and potassium (a) oxidising in (z) and reducing in (iz)
(6) chlorides and sulphates of sodium and potassium (6) reducing in (z) and oxidising in (iz)
(c) chlorides and sulphates of calcium and magnesium (c) reducing in (7) and (iz)
(d) bicarbonates of calcium and magnesium (d) oxidising in (z) and (iz) (AIPMT 2014)
(e) phosphates of sodium and potassium B7. Which of the following statements about hydrogen is
(Kerala PMT 2011) incorrect?
B3. Syngas is a mixture of (a) Hydrogen has three isotopes of which tritium is
(a) CO, + Hy (b) CO + Hy the most common.
(c) CO + CO, (d) CO + N, (6) Hydrogen never acts as cation in ionic salts.
(e) CO+ 0, (c) Hydronium ion, H,O* exists freely in solution.
(Kerala P.M.T: 2012) (d@) Dihydrogen does not act as reducing agent.
(NEET 2016)

Answer
Bl. (c) B2. (c) B3. (6) B4. (c) BS. (b) B6. (c) B77. (a,d)
HYDROGEN

JEE (Main) & Other State Boards’ (c) CO is removed by absorption in aqueous Cu,Cl,
Engineering Entrance solution
B8. Which one of the following processes will produce hard (d) H, is removed through occlusion with Pd.
water? (AIEEE 2008)
(a) Addition of Na,SO, to water B18. The strength of 10 vol. H,O, solution is
(6) Saturation of water with CaCO, (a) 10 (b) 68
(¢c) Saturation of water with MgCO, (c) 60.70 (d) 30.36
(d) Saturation of water with CaSO, (A..E.E.E. 2003) (A.M.U. Engg. 2009)
B19. Ortho- and para-hydrogen have
B9. The reagent commonly used to determine hardness
of water titrimetrically is ? (a) identical chemical properties but different physical
properties
(a) Oxalic acid (6b) Disodium salt of EDTA
(6) identical physical and chemical properties
(c) Sodium citrate (d) Sodium thiosulphate
(c) identical physical properties but different chemical
(A.LE.E.E. 2003)
properties
B10. Which of the following is a true peroxide?
(d) different physical and chemical properties
(a) NO, (b) MnO, (West Bengal JEE 2010)
(c) BaQ, (d) SO, (D.CLE. 2005) B20. Why do calcium ions make water hard but sodium ions
B11. One mole of magnesium nitride on reaction with excess do not ?
of water gives (a) Calcium forms insoluble compounds with stearate
(a) one mole of ammonia ions present in soap
(6) one mole of nitric acid (6) Sodium forms insoluble compounds with stearate
(c) two moles of ammonia ions present in soap.
(2) two moles of nitric acid (ALLELE. 2004) (c) Calcium forms soluble compounds with stearate
B12. A commercial sample of hydrogen peroxide is labelled ions present in soap.
as 10 volume. Its percentage strength is nearly (2) Both calcium and sodium form insoluble
(a) 3% (b) 1% compounds with stearate ions present in soap.
(JK. CET 2010)
(c) 9% (d) 10%
(Karnataka C.E.T. 2007) B21. Which one of the following undergoes reduction with
hydrogen peroxide in an alkaline medium ?
B18. Pure water does not conduct electricity because it is
(a) basic (6) almost not ionized
(a) Mn?* (b) HOCI
(c) PbS (d) Fe?*
(c) decomposed easily (d) acidic (JK. CLE.T. 2007)
(e) I, (Kerala PET 2010)
B14, Al,C, on hydrolysis gives .............. gas
B22. The normality of 30 volume H,O, is
(a) CH, (6) CoH,
(a) 2.678 N (b) 5.3386 N
(c) CH, (d) C,H, (Orissa J.E.E. 2007)
(c) 8.0384 N (d) 6.685 N
B15. Which of the following is formed by the action of water
on Na,O, ?
(WB JEE 2011)
(a) H, (b) O, B23. Very pure hydrogen (99.9%) can be made by which of
the following processes ?
(c) No (d) CO, (B.H.U. 2007)
(a) Mixing natural hydrocarbons of high molecular
B16. In transforming 1 mole of PbS to PbSO, the volume of weight.
‘10 volume’ H,O, required will be
(6) Electrolysis of water.
(a) 11.2 mL (6b) 22.4 mL
(c) Reaction of salt like hydrides with water.
(c) 33.6 mL (d)44.8 mL (WB-JEE 2008)
(ad) Reaction of methane with steam.
B17. In context with the industrial preparation of hydrogen
(A.LELELE, 2012)
from water gas (CO + Fi,), which of the following is
the correct statement? B24, Commercial sample of H,O, is labeled as 10 V. Its %
strength is nearly
(a) CO is oxidised to CO, with steam in the presence
of a catalyst followed by absorption of CO, in alkali (a) 3 (b)6

(b)CO and H, are fractionally separated using (ec) 9 (7d) 12


differences in their densities (WB. JEE 2014)

Answer
B8. (d) BY. (6) B10. (c) Bil. (c) B12. (a) B13. (b) B14. (a) B15. (b) B16. (d) B17. (a)
B18.(d) B19. (a) B20. (a) B31. (5) B22. ©) B23. (c) B24. (a)
MODERN'S abc + OF CHEMISTRY-XI

B25. In which of the following reactions, H,O,, acts as a (c) There is extensive intramolecular hydrogen
reducing agent? bonding in the condensed phase.
(1) H,O, + 2H* + 2e- —> 2H,O (d) lee formed by heavy water sinks in normal water.
(2) H,O, — 2e° —> O, + 2H* (JHE Main 2015)
(3) H,O, + 2e°- —> 20H" B30. In the manufacture of hydrogen from water gas
(4) H,O, + 20H- — 2e- —> O, + 2H,O (CO + H,), which of the following is correct statement?
(a) (2), (4) (b) (1), (2) (a) CO is oxidised to CO, with steam in the presence
(ce) (3), (4) (d)(1), (38) (JEE Main 2014) of a catalyst followed by absorption of CO, in alkali.
B26. The molecular formula of a commercial resin used for (b) H, is removed by occlusion with Pd.
exchanging ions in water softening is C,H.SO,,Na (Mol. (c) Hydrogen is isolated by diffusion.
wt. 206). What would be the maximum uptake of Ca?* (d)CO and H, are separated based on difference in
ions by the resin when expressed in mole per gram their densities.
resin?
(Karnataka CET 2017)
9 4 B31. Water gas is produced by
@) Oa
(a) passing steam over red hot coke.
(b) passing
steam and air over red hot coke.
(ec) (d) + (WJEE Main 2015)
103 206 (c) burning
coke in excess air.
B27. From the following statements regarding H,O,, choose (d) burning
coke in limited supply of air.
the incorrect statement. (e) both (@)
and (5).
(a) It has to be stored in plastic or wax lined glass (Kerala PET 2017)
bottles in dark. B32. The volume of oxygen liberated at STP from 15 mL of
(6) It has to be kept away from dust. 20 volume H,O, is
(c) It can act only as an oxidizing agent. (a) 100 mL (6) 150 mL
(d) It decomposes on exposure to light. (c) 200 mL (2) 250 mL
(e) 300 mL (Kerala PET 2017)
(JEE Main 2015)
B33. Which of the following ions will cause hardness in
B28. The various types of hydrides and examples of each
water?
type are given below:
Hydride type Compound (a) Ca?* (b) Na*
(ec) Cl (d) K*
(A) Electron deficient (i) LiH (Karnataka CET 2018)
(B) Saline (ii) CH, B34. H,O, is
(C) Electron precise (uit) NH, (a) an oxidising agent
(D) Interstitial (iv) BH, (6) a reducing agent
(E) Electron rich (v) C,H (c) both oxidising and reducing agent
Choose the correct matching from the codes given (d) neither oxdising nor reducing agent.
below: (Karnataka CET 2018)
(a) Ait), (Biv), = (Hv), (DH), =CE)Hi1) B35. Hydrogen peroxide oxidises [Fe(CN gl to [Fe(CN),]>-
(b) Av), =(B)H2), (CHa), (Dv), (E)uz) in acidic medium but reduces [Fe(CN na to [Fe(CN),]*
(c) A-aiv), (By), (Cv), (Doi), CH 7) in alkaline medium. The other products formed are,
respectively:
(dq) AHv), (Bi), (C)Hiv), (DH), (H)H2)
(a) (H,O + O,) and H,O
(e) AHiv), (B)-(v), (C)}@), (DHE), (EH)
(b) (H,O + O,) and (H,0 + OH”)
(Karala PET 2016)
(c) H,O and (H,O + O,)
B29. Which one of the following statement about water is
false? (d) H,O0 and (H,O + OH") (JEK Main 2018)
(a) Water is oxidised to oxygen during photosynthesis. B36. The volume strength of 1M H,O, is (molar mass of
H,O, = 34 g mol)
(5) Water can act both as an acid and as a base.
(a) 16.8 (6) 11.35

Answer
B25. (a) B26. (b) B27. (c) B28. (6) B29. (c) B30.
(c) 22.4

(a) B31. (a) B32.


(d) 5.6

(e) B33.
(JEE Main 2019)

(a) B84. (c)


B35. (c) B36. (b)
HYDROGEN

JEE (Advance) for IIT Entrance (a) form soluble complexes with anionic species
(5) precipitate anionic species
B37. Amongst H,O, H,S, H,Se and H,Te, the one with the
highest boiling point is (c) form soluble complexes with cationic species
(a) precipitate cationic species (LET. 2002)
(a) H,O because of hydrogen bonding
(6) H,Te because of higher molecular weight B39. Hydrogen peroxide in its reaction with KIO, and
(c) H,S because of hydrogen bonding NH,OH respectively, is acting as a
(d) H,Se because of lower molecular weight. (a) reducing agent, oxidising agent
(LET. 2000) (b) reducing agent, reducing agent
B38. Polyphosphates are used as water softening agents (c) oxidising agent, oxidising agent
because they (d) oxidising agent, reducing agent [JHE (advance) 2014/

Ansel
B37. (a) B38. (c) B39. (a)

(d) Heavy water has lower boiling point than orinary


water.
(NCERT Exemplar Problem)
C1. Which of the following has larger value for D,O than C7. Permanent hardness is due to the presence of
for H,O ? (a) Chlorides of Ca and Mg in water.
(a) Molecular mass _—_— (b) Dielectric constant (5) Sulphates of Ca and Mg in water.
(c) Viscosity (d) lonisation constant (c) Hydrogen carbonates of Ca and Mg in water
C2. Molecular hydrides are (d@) Carbonates of alkali metals in water
(a) NaH (6) H,S (NCERT Exemplar Problem)
(c) LaH (d) HF C8. Which of the following statements is correct ?
C8. The hydrides of which elements are non stoichiometric? (a) Metallic hydrides are deficient of hydrogen.
(a) Zirconium (6) Titanium (6) Metallic hydrides conduct heat and electricity.
(c) Calcium (d) Sulphur
(c) lonic hydrides do not conduct electricity
in solid state.
C4, When zeolite, which is hydrated sodium aluminium
silicate, is treated with hard water, the sodium ions (dq) lonic hydrides are very good conductors of
are exchanged with electricity in solid state.
(a) H* ions (b) Ca** ions C9. Which of the following statements about hydrogen
(c) SO,” ions (d) Mg** ions are correct ?
(e) OH” ions (a) Hydrogen has three isotopes of which protium is
C5. Which of the following statements are not correct ? the most common.
(a) Aluminium carbide reacts with heavy water to give (6) Hydrogen never acts as cation in ionic salts.
deutero acetylene. (c) Hydrogen ion, H’, exists freely in solution.
(6) In hydrated copper sulphate, four water molecules (d@) Dihydrogen does not act as a reducing agent.
are coordinated to Cu** ion
C10. Which of the following statements is correct ?
(c) P,O,, reacts with water to give phosphoric acid.
(a) Elements of group 15 form electron deficient
(d) H,O, acts both as an oxidant as well as reductant.
hydrides.
C6. Which of the following statements (s) is/are correct in
the case of heavy water ? (6) All elements of group 14 form electron precise
hydrides.
(a) Heavy water is used as a moderator in nuclear reactor.
(5) Heavy water is more effective as solvent than (c) Electron precise hydrides have tetrahedral
ordinary water. geometries.
(c) Heavy water is more associated than ordinary water. (d) Electron rich hydrides can act as Lewis acids.

Answer
C1. (a, c) C2. (6,d) C3. (a,b) C4, (6,d) C5. (b,c) C6. (a,c) Ci.(a,d) C8.(a,6,c) C9.(a,6) C10. (b,c)
MODERN'S abc + OF CHEMISTRY-XI

Exemplar Problems //
Objective Questions
Which of the following statement is correct about
»>» Multiple Choice Questions (Type-l) << H,O, with reference to these reactions ? Hydrogen
peroxide is ..........
1. Hydrogen resembles halogens in many respects for
(a) an oxidising agent in both (A) and (B)
which several factors are responsible, Of the following
(6) anoxidising agentin(A) and reducing agent in (B)
factors which one is most important in this respect ?
(c) areducing agent in (A) and oxidising agent in (B)
(a) Its tendency to lose an electron to form a cation. (d) a reducing agent in both (A) and (B)
(6) Its tendency togainasingle electron in its valence The oxide that gives H,O, on treatment with dilute
shell to attain stable electronic configuration. H,SO, is
(c) Its low negative electron gain enthalpy value. (a) PbO, (6) BaO,-8H,O
(d) Its small size. (c) MnO, (dq) TiO,
2. Why does Ht ion always get associated with other Which of the following equation depicts the oxidising
atoms or molecules ? nature of H,O, ?

(a) lonisation enthalpy of hydrogen resembles that (a) 2MnO; +6H*+5H,0,—> IMn2* + 8H,0 +50,
of alkali metals.
(b), 2Fe** + 2H* + H,0, —> 2Fe*+ + 2H,0 + O,
(6) Its reactivity is similar to halogens. (c)_ 21- + 2H* + H,O, —> I, + 2H,O
(c) It resembles both alkali metals and halogens. (dq) KIO, + H,O, ——> KIO, + H,O + O,
Which of the following equation depicts reducing
(d@) Loss of an electron from hydrogen atom results nature of H,O, ?
in a nucleus of very small size as compared to
(a) 2[Fe(CN),|* + 2H+ + H,O, —> 2[Fe(CN),|*
other atoms or ions. Due to small size it cannot
+ 2H,O
exist free.
(6) I, + H,0, + 20H-——> 2° + 2H,0 + 0,
3. Metal hydrides are ionic, covalent or molecular in (c) Mn?* + H,O, —> Mn* + 20H-
nature. Among LiH, NaH, KH, RbH, CsH, the correct
(d) PbS + 4H,0, —> PbSO, + 4H,O
order of increasing ionic character is
10. Hydrogen peroxide is ..........
(a) LiH > NaH > CsH > KH > RbH
(a) an oxidising agent
(b) LiH < NaH < KH < RbH < CsH (6) a reducing agent
(c) RbH > CsH > NaH > KH > LiH (c) both an oxidising and a reducing agent
(dq) NaH > CsH > RbH > LiH > KH (qd) neither oxidising nor reducing agent
4, Which of the following hydrides is electron-precise 11. Which of the following reaction increases production
hydride ? of dihydrogen from synthesis gas ?
(a) B,H, (b), NH, (a) CH,g) + H,O(g) 24> CO) + 3H,(g)
(c) H,O (d). CH,
5. Radioactive elements emit @, $ and y rays and are (b) C(s) + H,O(g) —2*_, cog) + H,(g)
characterised by their half-lives. The radioactive
isotope of hydrogen is (c) CO(g) + H,O(g) ar os CO,(g) + H,(g)
(a) Protium (6) Deuterium
(c) Tritium (7d) Hydronium
(d) C,H, + 2H,0 ———>
1270K
2CO + 5H,
6. Consider the reactions 12. When sodium peroxide is treated with dilute
sulphuric acid, we get ..........
(A) H,O, + 2HI —— I, + 2H,O
(a) sodium sulphate and water
(B) HOC! + H,O, —> H,Ot + Cl + O, (6) sodium sulphate and oxygen
(c) sodium sulphate, hydrogen and oxygen
(d) sodium sulphate and hydrogen peroxide

Answer
1. (5) 2. (d) 3. (5) 4. (d) 5. (c) 6. (b) 7. (6) 8. (e) 9. (6) 10. (ce)
11. (c) 12. (d)
HYDROGEN

13. Hydrogen peroxide is obtained iy the electrolysis of (6) Heavy water is more effective as solvent than
ordinary water.
(a) “water (6) sulphuric acid
(c) Heavy water is more associated than ordinary
(ec) hydrochloric acid (d) fused sodium peroxide
water.
14. Which of the following reactions is an example of use (qd) Heavy water has lower boiling point than
of water gas in the synthesis of other compounds ?
ordinary water.
(a) CH,g) + H,O(g) >> cow) + Hyg) 22. Which of the following statements about hydrogen are
correct ?
675K 575
(6) COlg) + H,0(8) —Gitaiyss? COW) + HAG) (a) Hydrogen has three isotopes of which protium is
the most common.
(c) C,H, ,9 +H,O(g) “C5 nCO + (2n + DH, (6) Hydrogen never acts as cation in ionic salts.
(c) Hydrogen ion, H’, exists freely in solution.
(d) CO(g) + 2H,(g) > CH,OH) (d) Dihydrogen does not act as a reducing agent.
15. Which of the following ions will cause hardness in 23. Some of the properties of water are described below.
water sample ? Which of them is/are not correct ?
(a) Ca* (6) Nat (a) Water is known to be a universal solvent.
(ec) Ch (dq) Kt
(b) Hydrogen bonding is present to a large extent in
16. Which of the following compound is used for water liquid water.
softening ? (c) There is no hydrogen bonding in the frozen state
(a) Ca,(PO,), (6) Na,PO, of water.
(c) Na,P.O (d) Na,HPO,
6 6” 18 (qd) Frozen water is heavier than liquid water.
17. Elements of which of the following group (s) of periodic 24, Hardness of water may be temporary or permanent.
table do not form hydrides. Permanent hardness is due to the presence of
(a) Groups 7, 8, 9 (6) Group 138
(a) Chlorides of Ca and Mg in water
(c) Groups 15, 16, 17 (d) Group 14
(6) Sulphates of Ca and Mg in water
18. Only one element of ......... forms hydride.
(c) Hydrogen carbonates of Ca and Mg in water
(a) group 6 (6) group 7
(d) Carbonates of alkali metals in water
(c) group 8 (dq) group 9
295. Which of the following statements is correct ?
»>» Multiple Choice Questions (Type-Hl) << (a) Elements of group 15 form electron deficient
hydrides.
19. Which of the following statements are not true for
(5) All elements of group 14 form electron-precise
hydrogen ?
hydrides.
(a) It exists as diatomic molecule.
(6) It has one electron in the outermost shell. (c) Electron-precise hydrides have tetrahedral
(c) It can lose an electron to form a cation which can geometries.
freely exist. (d) Electron rich hydrides can act as Lewis acids.
(d) It forms a large number of ionic compounds by 26. Which of the following statements is correct ?
losing an electron. (a) Hydrides of group 13 act as Lewis acids.
20. Dihydrogen can be prepared on commercial scale by (6) Hydrides ofgroup 14 are electron deficient hydrides.
different methods, In its preparation by the action of (c) Hydrides of group 14 act as Lewis acids.
steam on hydrocarbons, a mixture of CO and H, gas (dq) Hydrides of group 15 act as Lewis bases.
is formed it is known as 27. Which of the following statements is correct ?
(a) water gas (5) syngas (a) Metallic hydrides are deficient of hydrogen.
(c) producer gas (d) industrial gas (6) Metallic hydrides conduct heat and electricity.
21. Which of the following statement (s) is/are correct in (c) Ionic hydrides do not conduct electricity in solid
the case of heavy water ? state.
(a) Heavy water is used as a moderator in nuclear (d) Ionic hydrides are very good conductors of
reactor. electricity in solid state.

Answer
13. (5) 14. (d) 15. (a) 16. (c) 1%. (a) 18. (a) 19. (a, ©) 20. (a, b) 21. (a, c) 22. (a, b) 28. (ce, d)
24. (a, b) 25. (b,c) 26. (a,d) 27. (a, b,c)
MODERN'S abc + OF CHEMISTRY-XI

Matching Type Questions generate high


temperature
28. Correlate the items listed in Column I with those (iv) Heavy water is used in (d) reducing agent
listed in Column II. Find out as many correlations (v) Atomic hydrogen (e) hydrogen and
as You Can. oxygen
Column I Column II
31. Match the items in Column I with the relevant
(i) Synthesis gas (a) Na,{Na,(PO,)¢] item in Column IL.
(ii) Dihydrogen (6) Oxidising agent
Column I Column IT
(iit) Heavy water (c) Softening of water
(iv) Calgon (d) Reducing agent (1) Hydrogen peroxide is used (a) zeolite
(v) Hydrogen peroxide (e) Stoichiometric compounds of asa
s-block elements. (it) Used in Calgon method (b) perhydrol
(zt) Permanent hardness of (c) sodium
(vi) Salt like hydrides (f) Prolonged electrolysis of water
hard water is removed by hexametaphosphate
(g) Zn + NaOH (d) propellant
(h) Zn + dil. H,SO,
(i) Synthesis of methanol >» Assertion and Reason Type Questions <q
(7) Mixture of CO and H,
In the following questions a statement of Assertion
29. Match Column I with Column II for the given (A) followed by a statement of Reason (R) is given.
properties/applications mentioned therein. Choose the correct option out of the choices given
Column I | Column IT below:
(i) H (a) Used in the name of perhydrol. (a) Statements A and R both are correct and R is
the correct explanation of A.
(uz) Hy (6) Can be reduced to dihydrogen by NaH.
(6) Ais correct but R is not correct.
(wi) H,O (c) Can be used in hydroformylation of olefin.
(c) A and R both are correct but R is not the correct
(tv) H,O, ~~‘(d) Can be used in cutting and welding.
explanation of A.
380. Match the terms in Column I with the relevant (2) A and R both are false.
item in Column II. 32. Assertion (A) : Permanent hardness of water is
removed by treatment with washing soda.
Column I .
Column I
Reason (R) : Washing soda reacts with soluble
(i) Electrolysis of water produces (a) atomic reactor magnesium and calcium sulphate to form insoluble
(ii) Lithium aluminium hydride is (6) polar molecule carbonates.
used as 33. Assertion (A) : Some metals like platinum and
(tit) Hydrogen chloride is a (c) recombines on palladium can be used as storage media for hydrogen.
metal surface to

Answer
Reason (R) : Platinum and palladium can absorb
large volumes of hydrogen.

28. (1) — (i, J); (a)— (0, d, e, g, h, 0); (ai) — (f); Gv)— (a, ©); (v)
— (6, d); (vi) —(e) =29. (0) — (d); (it)
— (e); (aii)
— (6); (iv)
— (a)
30. (i) — (e); (ii) — (d); (ii) — (8); (iv) — (a); (v) — (e) 31. (@) — (6, d); (i) — (c); Giz) — (a,c) $2. (a) 33. (a)

nations for
Hints & Expla Difficult Objective Type Questions
A22. (6): 1 Lof 1M solution of H,O, contain 34 g
A. meq with only one correct answer
1 Lof3 M solution of H,O, contain = 3x34 = 102g
A2, (d): Cu cannot displace H, from dilute hydrochloric A24, (6): H,O,: 1x 2+ 2x=0 i oF =-—]

acid because it is below hydrogen in


A25. (b): 2H,O, ————— 2H,0+0,
electrochemical series.
2x 34=68 22.4 L at N.T.P.
A12. (d): CO + 2H, “5 CH,OH 22.4 L of O, at N.T-P. is produced from 68 g H,O,
Al3. (6): Ca,P,+6H, — > 3Ca(OH), + 2 PH, 20 L of O, at N.T.P. is produced from H,O,
Al5. (c) : Ca,P,+ 6H,O —— 3Ca(OH), + 2PH, _ 68x 20
= 60.7g
1 mol 2 mol 22.4
. Strength of H,O, in 20 volume H,O, = 60.7 g/L
HYDROGEN

A28. (b): Strength of H,O, = 1.5 x 17 = 25.5g 2H,0, ——>» 2H,0 + 0,


2H,O,
—> 2H,0 + O, 2x 34=68¢ 22.4L
68 g 22.4 L 22.4 L of O, is obtained at N.T.P. from H,O,
68 g of H,O, giveO, = 22.4L
=- 68¢
22.4 10 L of O, will be obtained at N.T.P. from H,O,
25.5 g of H,O, give O, = 63. * 25.5

= 84L = “a x 10
pat
Volume strength = 8.4 L = 8.4 volumes.
A33. (a): 2H,O, ——> 2H,0 + O, = 30.36 g
68g 22.4 L .. Strength of 10 volume H,O, = 30.36 g
22.4 L of O, at N.T-.P. is produced from H,O, = 68 g B21. (6): HOC] + HO, ——~ HCl + H,O+ 0,
11.2 L of O, at N.T.P. is produced from H,O, B22. (b): 21,0, aef> 2H,0 “ O,
68 22.4 L at NTP
ce ee
oY eee 22.4 L of O, at N.T.P. is produced from H,O,

Moles of H,0, = 34 = 1 mol


=68¢
34 30 L of O, at N.T.P. is produced from H,O,
1
Molarity = 1% 1M 68
= 32.4 *°0
= 91.07
B. meq from competitive examinations
md
‘. Strength of H,O, in 30 volume H,O, = 91.07 g/L
Bé4. (c): 11.2 volume solution means that 1L of this = 91.07 g
solution liberates 11.2 L of O, at N.T.P.
Gram equivalents of H,O, = “aq
91.07 = 5.34
2H,0, —> 2H,0O+ O,
68g 22.4L
22.4 L of O, at N.T.P. is produced from H,O, Normality = “= = §,.34 N.
= 68 ¢
11.2 L of O, at N.T.P. is produced from H,O, B23. (c) : Very pure H, (99.9%) is prepared by the action of
— «668 water on salt hydrides (e.g. NaH)
= 904 11.23 = 34 £
NaH + H,OQ ——> NaOH + H,.
Strength = 34 g/L
B24. (a) 2H,O, —> 2H,O + O,
B6. (c) : H,O, acts as reducing agent in all these reactions
in which QO, is evolved. 68 g 22.4 L at N.T.P.
B7. (a, d) : Hydrogen has three isotopes of which tritium 22.4 LO, at N.T.P. is obtained from H,O, = 68 g
is radioactive and rare, Dihydrogen acts as
10 L O, at N.T.P. is obtained from H,O,
reducing agent.
B11. (c) : Mg,N, + 3H,0 —> 2NH, + 3MgO 68 “x10
22.4
B12. (a): 2H,0, ——> 2H,0 /+ O,
= 30.357 g
2 x 34 = 68¢ 22.4 Lat N.T.P.
10 vol. H,O, means 1 L H,O, gives 10 L O, at N.T.P.
22.4 L of O, at N.T.P. is produced from H,O, = 68 g
30.35 g of H,O, are present in 1 L (1000 mL) of
10 L of O, is produced at N.T.P. from H,O,
HO,
68x10 anne 30.35
"2a See Gp Strength = ooo 100 = 3.035

Strength of H,O, in 10 volume H,O, = 30.35 g/L


or 3.035%
B25. (a): The reducing agent itself gets oxidised (the O.N.
increases).
B14. (a): Al, C, + 12H,O —> 4A\(OH), + 3CH, =| —2
(1) H,0O, + 2Ht + 2e-—— 2H,O
B15. (5): 2Na, O, + 2H,O —> 4NaOH + O,
-] 0
B18. (d): 10 vol HO, means that 1 mL of H,O, gives 10 (2) H,0O, — 2e- — 0, + 2H
mL of O, at N.T.P.
MODERN'S abc + OF CHEMISTRY-XI

-1 a
(3) H,O, + 2e° —» 20H- NCERT Exemplar Problems: MCQ Type I
—1 = 0
(4) H,O, + 20H —2e-—> 0, + 2H,0 3. (b): Ionic character increases as the size of the atom
B26. (b): 2C,H.SO,Na + Ca** —> (C,H,SO,), Ca + 2Na” increases.
2 mol 1 mol LiH < NaH < KH < RbH < CsH
2 mol of C,H,SO,Na require 1 mol of Ca** A. (d): Electron precise hydrides contain exact number
of electrons to form normal covalent bonds 1.e.,
2 x 206 g of C,H.SO,Na = 1 mol of Ca?*
CH,. Thus, option (d) is correct.
1
1 g of C,H,5SO.,Na = TD mol : Nucleides with n/p (neutron-proton) ratio >1.5 are
usually radioactive.
1
.. Maximum uptake = mol per gram For example, tritium (n = 2, p = 1).
412
B27. (c): H,O, can be reduced or oxidised. Hence, it can +1 -1 —l 0, +1-2
act as an oxidising agent as well as reducing 6. (5): H,O0, + 2HI — I, + 2H,0
agent. O.N. of oxygen is decreased from — 1(H,O,) to —
B28. (5): Electron deficient — B,H, 2(H,O), therefore, it is reduced and acts as an
Saline — LiH oxidising agent.
Electron precise — CH, = | 0
Interstitial — CrH HOC! + H,O, —> H,0* + Cl + O,
Electron rich — NH, O.N. of oxygen is increased from —1(H,,O,) to 0(O,),
B29. (c): There is extensive intermolecular hydrogen therefore, it is oxidised and acts as a reducing
bonding in the condensed phase instead of agent.
intramolecular hydrogen bonding.
7. (b): The oxides such as BaO,, NaO, which contain
B31. (a): C + HO — >» H,+CO peroxide linkage ((O—O~ i.e. O,?) on treatment
En
Red hot Steam Water gas with dil. H,SO,, give H,O, but dioxides
coke (O = M = O where M is a metal) such as PbO,,
B32. (e): 1 L of 20 volume H,O, liberates O, = 20 L MnO,, TiO, etc. donot give H,O, on treatment
or 1000 mL of 20 volume H,0, liberates O, with dil. H,SO.,.
= 20,000 mL 8. (c): Only in reaction (c) O, is not evolved, and
«15 therefore, it represents the oxidising action of
15 mL of 20 volume H,0O, liberates O, = 20,000
1000 H,0,.
= 300 mL 9. (b): Only in reaction (6), O, is evolved, therefore, it
B33. (a): The hardness of water is generally caused by the represents reducing action of H,Q,.
presence of carbonates, chlorides and sulphates 12. (d): Na,O, + dil. H,5O,——> Na,SO, + H,O,
of calcium and magnesium.
B35. (c): 2[Fe(CN),.\* +H,0,+2H* —> 2[Fe(CN),)* + 2H,O 17. (a): The elements of groups 7, 8 and 9 donot form
2[Fe(CN),]*- + H,O, + 20H> ——> 2[Fe(CN),|* + hydrides and hence, it is called hydride gap.
2H,O + O, 18. (a): Only one element of group 6 i.e., Cr forms
B38. (c) : Polyphosphates such as sodium hexametaphosphate hydride.
combines with cations (Ca?* or Mg**) to form soluble
complexes : NCERT Exemplar Problems: MCQ Type IT
2Ca** + Na,[Na,(PO,),] —>
20.(a, b): Mixture of CO and H, is called water gas or
4Na* + Na,[Ca,(PO,)¢| syngas.
Soluble complex 21.(a,c): Heavy water is used as moderator in nuclear
B39. (a): Reducing agent reactor and is more associated than water.
P: Decrease
L j in O.N
* * x: 22.(a, b): Hydrogen has three isotopes. H* being small in
size, never exists as cation in ionic compounds.
KIO, + H,O, ——— KI0,+H,0 +0, 23.(c, d): There is hydrogen bonding even in frozen state
of water and ice is lighter than liquid water.
+1 +5
25.(6, c): Elements of group 14 form electron precise
NH,OH + 3H,O, ————>HNO, + 4H,O0 hydrides, which have tetrahedral structure.
Increase in O.N. 27.(a, b,c): Statement (d)is not correct. All other statements
Oxidising agent are correct.
HYDROGEN

t 4
Unit P ractice Tes Examination

Time Allowed: 1 Hr. Maximum Marks: 25

jala What is the trade name of hydrogen peroxide used as an antiseptic ? (1)
Concentrated sulphuric acid cannot be used for drying hydrogen. Why ? (1)
What happens when heavy water is added to calcium carbide? (1)
What is the mass ratio of different isotopes of hydrogen? (1)
fFoN
a . Knowing the properties of H,O and D,O do you think that D,O can be used for drinking
purposes? (1)
. Write chemical reactions to show the amphoteric nature of water, (2)
. Compare the structure of H,O and H,Q,. (2)
. Calculate the amount of hydrogen peroxide present in 10 mL of 25 volume solution of H,Q,. (2)
co.
ss
oo
& Discuss the consequences of high enthalpy of H—H bond in terms of chemical reactivity of
dihydrogen. (2)
10. What do you understand by (7) electron deficient, (ii) electron-precise, and (iii) electron-rich
compounds of hydrogen ? Provide justification with suitable examples. (3)
12. Complete the following reactions :
(a) Zn (s) + NaOH (ag) —>
(b) Li + AICI, Ether,
(c) Ca,N, + HO —> (3)
12. (a) Write chemical equations to justify that hydrogen peroxide can act as an oxidizing as well
as reducing agent.
(b) How does H,O, behave as a bleaching agent ? (3)
13. (a) What causes the permanent hardness of water?
(6) Anhydrous BaO, cannot be used to prepare H,O, by reacting with acid. Explain.
(c) What is meant by water — gas shift reaction? Explain. (3)

To check your performance, see HINTS AND SOLUTIONS TO SOME QUESTIONS at the end
of Part II of the book.

WWW.JEEBOOKS.IN
WWW.JEEBOOKS.IN
-BLOCK ELEMEN”
~~ (ALKALI AND
ALKALINE EARTH
METALS)
O55) Os AAD
Building on..... Assessing..... Preparing for Competition.....
@ Understanding Text 1 @ Quick Memory Test with # Topicwise MCQs AQ
@ Conceptual Questions 18,35 Answers 43 @ Competitive Examination Qs
CHAPTER SUMMARY & QUICK ° pee mira eral 4s » AIPMT & Other State Boards’
CHAPTER ROUND UP Ye danas uaioeaekias) . We eee eee ce
NCERT FILE altace EXERCISES‘ 45 a ses
Aim er | DO

Hints & Answers for gineering Entrance b2


@ In-text Qs & Exercises with ae 4g » JEE (Advance) forIITEntrance 54
Solutions 38 @ NCERT Exemplar Problems
@ NCERT Exemplar Problems with UNIT PRACTICE TEST 61 (Objective Questions) 56
Answers & Solutions (Subjective) 41 ¢ Hints & Explanations for Difficult
Questions 58

T he s-block elements are those in which the last electron enters the
outermost s-orbital. As the s-orbital can accommodate only two
electrons, this block includes only two groups; group 1 (or IA) and
group 2 (or Il A). They have one or two electrons in their outermost
s-subshell respectively. These are placed on the left hand side
_ of the Long Form of the periodic table.
The elements of group 1 are called alkali metals and those of
group 2 are called alkaline earth metals. Therefore, alkali
metals and alkaline earth metals constitute s-block.
ABUNDANCE AND OCCURRENCE
The alkali and alkaline earth metals are the most
reactive of all the metals. These are readily oxidised
and therefore, are not found in the free state in nature.
/ These are found in the combined form with halide, oxide,
sulphate, carbonate, silicate ions, etc.
The abundance of alkali and alkaline earth metals in
| the earth’s crust shows a good range. Among these, calcium
is the fifth most abundant element in the earth’s crust and hence,
the third most abundant metal after aluminium and iron. Vast
sedimentary deposits of CaCO, occur over large parts of the earth’s surface. Magnesium is the sixth and
sodium is the seventh most dbandant element in the crustal rocks. These are fourth and fifth most abundant
metals after Al, Fe and Ca. Potassium (eighth) is the next most abundant element after sodium. Barium is
fourteenth and strontium is the fifteenth most abundant element by weight in the earth’s crust. Sodium and
Magnesium are also present in relatively large amounts in sea water, brine wells and a few salt lakes. Calclum
and potassium also occur in sea water but to smaller extents. The occurrence of other metals in the earth’s
crust is very poor. Francium and radium are radioactive and have rare terrestrial abundance. Radium was

conf
Tio 1 2 13 14
MODERN'S abe + OF CHEMISTRY

#+15 #16 #17~«18


-XI

(I-A) (I-A) (III-A) (IV-A) (V-A) (VI-A) (VILA) (VII-A)


s-BLOCK
ra ELEMENTS

p-BLOCK ELEMENTS
d-BLOCK ELEMENTS

f-BLOCK ELEMENTS
Fig. 1. s-block elements.
isolated by Pierre and Marie Curie by processing belonging to different groups. This is called diagonal
uranium ore pitch blende. Francium is highly relationship.
radioactive and its longest lived isotope 7“°Fr has a Thus,
half-life of only 21 minutes. It is formed during the similarity in properties of elements present
radioactive decay of actinium. diagonally is called diagonal relationship.
This is shown in Table 1. Three important
227 Ac —— 223 By + He diagonal pairs are:
Beryllium is rare and radium is the rarest of all (i) Lithium — magnesium
(it) Beryllium — aluminium
comprising only 10-" per cent of igneous rocks.
(zit) Boron — silicon.
ANOMALOUS PROPERTIES OF Cause of diagonal relationship. The cause of
FIRST ELEMENT IN EACH GROUP diagonal relationship is the similarity in properties
The elements in a group show similar physical and such as electronegativity, ionisation enthalpy, size or
chemical properties. However, the first member of each charge/ radius ratio, etc. between the diagonal elements.
croup differs from its succeeding members (called For example, on moving from left to right across a
congeners). For example, lithium shows an period, the electronegativity increases and while moving
anomalous behaviour as compared to sodium and rest down a group electronegativity decreases. Therefore,
of the family members of the alkali metal family. It is on moving diagonally, the two opposing tendencies
mainly due to the following reasons : almost cancel out and the electronegativity values
(1) Small size of the atom and its ion. remain almost same as we move diagonally. Thus, the
(ii) High ionisation enthalpy and electronegativity. diagonal pairs have many similar properties.
(ui) High polarizing power of its cation.
Electronegativity increases
(iv) Non-availability of d-orbitals.
The specific characteristics differences between
lithium and sodium (group 1) and beryllium and
magnesium (group 2) are discussed later in the
discussion of groups.
DIAGONAL RELATIONSHIP
It has been observed that some elements of second decreases
Electronegativity
period show similarities with the elements of the third Similar electronegativity
period present diagonally to each other, though
Table 1. Diagonal relationship in the periodic table.
Group 1 Group 2 Group 13 Group 14
Second Li Be )
period

Third
period Na Mg Al S1
s-BLOCK ELEMENTS (ALKALI AND ALKALINE EARTH METALS)

Rubidium and cesium are obtained as a by product


CHEMISTRY OF ALKALI METALS of lithium processing. Francium being radioactive
does not occur appreciably in nature.
The elements belonging to group 1 of the periodic GENERAL CHARACTERISTICS OF ALKALI
table are called alkali metals. They constitute the six METALS
elements namely, lithium (Li), sodium (Na), potassium
The electronic configurations and important atomic
(K), rubidium (Rb), cestum (Cs) and francium (Fr). These
and physical constants of alkali metals are given in
are called alkali metals because they readily dissolve
Table 2 and 3 respectively. Some of these trends are
in water to form hydroxides which are strongly alkaline
in nature. They also form alkaline oxides. The element,
discussed below.
francium is radioactive. 1. Electronic Configurations
OCCURRENCE All the alkali metals have one electron in their
Lithium is the 35th most abundant element by outermost s-orbitals preceded by the noble gas
weight in the earth's crust. It occurs mainly as silicate configuration. Thus, the general configuration of alkali
minerals such as spodumene [LiAl(Si0,),], lepidolite metals may be written as [Noble gas] ns: where n
[(Li, AL,(S10,), (F, OH),I, ete. represents the valence shell. The electronic
Sodium and potassium together make up over 4% configurations of alkali metals are given in Table 2.
by weight of the earth’s crust. Compounds of sodium 2. Atomic and ionic radii.
and potassium have been known from ancient times. Alkali metals have the largest atomic and tonic
Sodium and potassium are seventh and eighth most radii in their respective periods of the periodic table.
abundant elements by weight in the earth’s crust. NaCl On moving down the group, the atomic and tonic radit
and KC] occur in large amounts in sea water. Rock increase.
salt (NaCl) is the major source of sodium. Explanation. The alkali metals are the first
Other important minerals of sodium are: elements of each period. As we move in a period, the
Na,B,0O,.10H,O (borax), Na,CO,.NaHCO,.2H,O atomic radius and ionic radius tend to decrease due to
(trona), NaNO.(chile salt petre) ete. increase in the effective nuclear charge. Therefore, the
Potassium occurs mainly as deposits of KCl alkali metals have the largest atomic and ionic
(sylvite), a mixture of KC] and NaCl (sylvinite) and radii in their respective periods. On moving down
double salt KC]. MgCl,.6H,O (carnallite) and as
K,0.Al1,0,.6510, (feldspar).
Table 2. The electronic configurations of alkali metals.

Element Symbol Atomic No. Electronic configuration

Lithium Li 3 1s?2s* or [He]2s?


Sodium Na 11 1s*2s72n°3s* or [Ne]3s?
Potassium K 19 1s?2s72p%3s73p%4s' or [Ar]4s1
Rubidium Rb 37 1s72572p°3s?3p°3d"94s74p%5st or [Kr]5s*
Cesium Cs 55 As*2672p°3s"3p "3d 4374p *4d5s"5p "6s! or [Xe]6s!
Francium Fr 87 1372979. °3s73p 53d" 4574p 84d 9445975 p"5d™6s*6p°7s! or [Rn]7s!

Table 3. Some physical properties of alkali metals.

Property Li Na K Rb Cs Fr

Atomic number 3 11 19 37 55 87
Atomic mass (g mol") 6.94 22.94 39.10 85.47 182.91 223
Atomic radius (pm) 162 186 A | 248 265
Ionic radius (pm) 76 102 138 152 167 180
Ionization enthalpy (kJ mol)
IE, 520 496 419 A03 376 —375
Tks 7298 4562 3051 2633 2230
Hydration enthalpy (kJ mol!) -506 —406 —330 —310 —276
Electronegativity 0.98 0.93 0.82 0.82 0.79
Density (g cm™~) 0.53 0.97 0.86 1.53 1.90
Melting point (K) 454 371 306 312 302
Boiling point (K) 1615 1156 1032 961 944.0
E® value (V) —3.04 —2.714 — 2.925 — 2.930 — 2.927
[M* (aq) + e' —+> M(s)]
Occurrence in lithosphere(ppm) 18 27700" 18400* 78 2.6 see
“These can also be expressed as percentage by wt; Na (2.27% by wt), K (1.84% by wt), etc.
| 104 MODERN'S abe + OF CHEMISTRY-AI

the group, there is increase in the number of shells (mass/volume) ofalkali metals gradually increasefrom Li
and, therefore, atomic and ionic radii increase. to Cs. However, potassium is hghter than sodium probably
3. Ionisation enthalpies due to abnormal increase in atomic size of potassium.
(1) Alkali metals have the lowest ionisation 3. Electropositive or metallic character.
enthalpy ineach period. Within the group, the tonisation All the alkali metals are strongly electropositive or
enthalpies of alkali metals decrease down the group. metallic in character.
Explanation. The atoms ofalkali metals are largest Explanation. The electropositive character of an
in their respective periods and therefore the valence element is expressed in terms of the tendency of its
electrons are loosely held by the nucleus. By losing the atom to release electrons :
valence electron, they acquire stable noble gas M ——~> Mtoe
configurations. Therefore, they readily lose electrons As alkali metals have low ionisation enthalpies,
and have low ionisation enthalpies. their atoms readily lose their valence electron. These
elements are, therefore, said to have strong electro-
On moving down the group, the atomic size increases
positive or metallic character. Since the ionisation
and the magnitude of screening effect (number of inner
enthalpies decrease down the family, the electron
shells) also increases and consequently, the ionisation
releasing tendency or electropositive character is
enthalpy decreases down the group.
expected to increase down the family.
(it) The second ionisation enthalpies of alkali metals
4, Oxidation states.
are very high.
All the alkali metals exhibit an oxidation state of
Explanation. When an electron is removed from + 1 (unipositive tons) in their compounds.
the alkali metals, they form monovalent cations which Explanation. The alkali metals have only one
have very stable electronic configurations (same as electron in their valence shell and therefore they can
that ofnoble gases). Therefore, it becomes very difficult lose the single valence electron readily to acquire the
to remove the second electron from the stable noble gas stable configuration of a noble gas. Thus, they form
configurations and hence their second ionisation monovalent ions, M*(e.g., Lit, Nat, K*, Rb*, Cs*). Since
enthalpy values (IE,) are very high. the second ionisation enthalpies are very high, they
Physical Properties cannot form divalent ions. Thus, alkali metals are
All the alkali metals are silver white, soft and light univalent and form tonic compounds.
metals. Lithium is harder than sodium. The important 5. Characteristic flame colouration.
characteristic physical properties (Table 3) are All the alkali metals and their salts tmpart
discussed below : characteristic flame colourations.
1. Melting and boiling points. Explanation. The alkali metals have very low
ionisation enthalpies. The energy from the flame of
All these metals are soft and have low melting and
bunsen burner is sufficient to excite the electrons of
boiling points.
alkali metals to higher energy levels. The excited state
Explanation. The alkali metals have only one
is quite unstable and therefore when these excited
valence electron per metal atom and therefore, the
electrons come back to their original energy levels, they
energy binding the atoms in the crystal lattice of the
emit extra energy, which falls in the visible region of the
metal is low. Thus, the metallic bonds in these metals
electromagnetic spectrum and thus appear coloured.
are not very strong and consequently, their melting and The following colours are given by alkali metals:
boiling points decrease on moving down from Li to Cs.
i Na K Rb Cs
2. Density.
Crimson Yellow Pale Violet Bluish
The densities of alkali metals are quite low as red violet
compared to other metals, Li, Naand K are even lighter X(nm) 670.8 589.2 766.5 780.0 455.5
than water. The densities increase on moving down the
group. However, K is lighter than Na. The different colours of the alkali metals can be
explained on the basis of amount of energy absorbed
Explanation. The densities of metallic elements
for excitation of the valence electron. Therefore, the
depend upon the type of packing of atoms in metallic
alkali metals can be detected by the respective flame
state and also on their size. Because of the large size of
tests and can be determined by flame photometry or
alkali metal atoms, they have less closely packed
atomic absorption spectroscopy.
metal atoms in their lattice. Therefore, they have low
densities. As we move down the group from Li to Cs, 6. Photoelectric effect.
there 1s increase in atomic size as well as atomic mass. The alkali metals emit electrons when radiation
But the increase in atomic mass is more than the strikes on their surfaces. This phenomenon of emission
increase in atomic size. As a result, the densities of electrons when electromagnetic radiation strikes
s-BLOCK ELEMENTS (ALKALI AND ALKALINE EARTH METALS)

against them is called photoelectric effect (Unit 2). halogens. Some of their characteristic chemical
Therefore, alkali metals exhibit photoelectric effect. properties are given below :
Explanation. Alkali metals have low ionisation
enthalpies and therefore, the electrons are easily 1. Reactivity towards air or oxygen
ejected when exposed to light. Among alkali metals, All alkali metals get tarnished in dry air due to the
cesium has lowest ionisation enthalpy and hence it formation of their oxides.
can show photoelectric effect tothe maximum extent. When heated in excess of air or oxygen, they burn
Because of strong photoelectric effect, cesium is vigorously forming different types of oxides depending
frequently used in solar cells. upon the nature of metal.
Lithium forms monoxide (Li,O),
7. Lattice enthalpies.
sodium forms peroxide (Na,O,) and
The alkali metal salts consist of cations and anions
the other elements form superoxides (MO, :
which are held up by strong electrostatic forces of
attraction. Therefore, the lattice enthalpies of alkali M = K, Rb, Cs).
metal salts are very high. The lattice enthalpy is ATi+ O, ___¥ 211,0
defined as the amount of energy required to break one Lithium monoxide
mole of an ionic compound into its free ions. 2Na t+ U5
575K
Na,O,
MX (s) Lattice enthalpy Mt (g) ‘ 4 “(g)
Sodium peroxide
Therefore, lattice enthalpy gives a measure of the Mint ee ——>
7 MO, (M = K, Rb, Cs)
forces of attraction between the ions. Larger the forces
Metal superoxide
of attraction, the greater will be lattice enthalpy. The
lattice enthalpy also depends upon the size of the ion and Thus, alkali metals form three types of oxides:
its charge. For the cation of same valency, the lattice (i) normal oxides containing O7 ion (oxide ion)
enthalpy of tonic solids having the same anion decreases (ii) peroxides containing O,” ion (peroxide ion)
with increase in size of the cation due to decrease in forces (ii) superoxides containing O,. ion (superoxide ion)
of attraction between them. For example,
Now question arises as to why lithium forms only
LiCl > NaCl > KCl > RbCl > CsCl
normal oxide, sodium forms peroxides while others
form superoxides.
Salt Lattice enthalpy (kJ mol!)
The formation and stability of these oxides can be
explained on the basis of size of the cation (alkali metal
LiCl 802.6 Z ion) and the anion. It is known that smaller cation can
NaCl 758.7 y stabilize smaller anion and larger cation can stabilize
larger anion. This is the basis of lattice energy
KCl 681.4 = effects. The normal oxides contain O? ion, the
RbCl 660.6 A
peroxides contain O,7- [or -O—O-]*- ion and superoxides
CsCl 618.7
contain O,- [:0-:0: }- ion.
Similarly, the lattice enthalpy of tonic solids having Lithium ion is very small ion and, therefore, has
the same cation decreases with increase in size of the strong positive field around it. Therefore, it can stabilize
anion as given below : only asmall anion, O*-. In other words, because of small
NaF > NaCl > NaBr > Nal size of lithium ion, the spreading of negative charge
towards another oxygen atom is prevented and,
Salt Lattice enthalpy (kJ mol-')
therefore, it cannot combine with another oxygen atom
NaF $94.5 to form peroxide ion, O,7-. Consequently, higher oxides
are not formed. Sodium ion is large and it can stabilize
NaCl 758.7
large anion. The positive field around sodium ion is
NaBr 714.8
comparatively weaker and does not prevent the O* ion
Nal 668.8 to combine with another oxygen atom to form peroxide
ion, O,7-. On the other hand, because of large size of
Chemical Properties K*, Rb* and Cs* ions, the positive field is very weak and
allows even peroxide ion, O,* to combine with another
The alkali metals exhibit high chemical
reactivity. This is due to oxygen atom to form superoxide ion, O,.
O
(i) their low tonisation enthalpies or 2%, 0,224 20,
(it) low enthalpy of atomisation.
Oxide Peroxide Superoxide
As the value ofionisation enthalpy decreases down 2. Reactivity towards air and moisture
the group from Li to Cs, therefore, the reactivity of
alkali metals increases from Li to Cs. All alkali metals on exposure to atmosphere (air
The alkali metals are highly reactive towards the and moisture) get converted to their oxides, which react
more electronegative elements such as oxygen and with moisture to form hydroxides and finally to
L106,
carbonates. Therefore, alkali metals get tarnished
MODERN'S abc + OF CHEMISTRY-XI

hydrides decreases from Lil to CsH.


when exposed to air and moisture. (ii) The stability of hydrides decreases from Li to
4M+0O, ——> 2M,0 Cs. This is due to the fact that as the size of alkali metal
M,O+H,O —» 2MOH increases from Lito Cs, the M—H bond becomes weak.
Therefore, the stability of hydrides decreases.
29MOH+CO, ——> M,CO,+H,O (iv) Thehydrides behave as strong reducing agents
Lithium shows exceptional behaviour because it and their reducing nature increases down the group.
reacts directly with nitrogen of air to form nitride, (v) Since these hydrides contain the hydride ion
Li,N. (H-), therefore, they hberate hydrogen at the anode
Thus, alkali metals cannot be kept either in air or on electrolysis.
in water. They are normally stored in inert (vi) They react with proton donors such as alcohols,
hydrocarbon solvents like kerosene oil, petroleum gaseous ammonia and alkynes liberating H, gas.
ether or benzene, hexane, etc which prevent them LiH(s) + H,OW@) ——+> Li(OH)\(aq) + Hf)
from coming in contact with air and moisture.
NaH(s) + ROH() ——> RONa(s)+ Hg)
3. Reactivity towards water
Alcohol Sodium alkoxide
Alkali metals react with water to form hydroxides,
MOH and hydrogen gas is evolved. NaH(s) +NH,(g) —*> NaNH,(s)+H,(g)
2M+H,O — > 2M*+20H +H,
2KH(s)+ HC =CHy) —4, KC=CK+2H,(g)
(where M is an alkali metal)
Acetyline Potassium acetylide
eg., 2Na+H,O —— > 2Nat+20H'+H,
5. Reactivity towards halogens
2K+H,O —— > 2Kt+20H +H,
The alkali metals readily combine with halogens to
It may be noted that although lithium has most
form ionic halides M*X e.g.
negative E° value (Table 2), its reaction with water is
less vigorous than that with sodium, which has least E° oM+X, ——> 2MX (X = halogen)
value among the alkali metals. Lithium reacts 9Na+Cl, ——> 2NaCl
somewhat slowly. This behaviour of lithium is attributed The reactivity ofalkali metals towards a particular
to its small size and very high hydration energy. But halogen increases on moving down the group.
sodium and the other members of the family react so
explosively with water that the hydrogen gas evolved Li < Na < K < Cs Li
immediately catches fire. This is due to decrease in ionisation
enthalpy or increase in electropositive Na
i R.U. Curious... a character as we move down the group.
All the metal halides are ionic K
| ) Sodium fire in the laboratory should not be crystals. However, Lilis slightly covalent
extinguished by pouring water! because ofthe polarization(Li*beingthe Rb
| » Sodium reacts violently with water producing H, smallest cation has maximum polarizing A
WITH
REACTIVITY
power and iodideionbeinglargestanion ©§ INCREASES
HALOGEN
gas which also catches fire. Therefore, the fire
can be polarised to the maximum extent).
spreads instead of being extinguished. Hence, water
6. Solutions in liquid ammonia
should not be used for extinguishing sodium fire.
Alkali metals dissolve in liquid ammonia giving
| For extinguishing sodium fire, pyrene (CCl1,) should deep blue solutions which are conducting in nature. As
|_be used. the concentration increases, the colour changes to
bronze and their conductivity decreases.
The hydroxides of alkali metals are strongly basic Explanation. In solution, the alkali metal atom
and the strength of the base increases down the group. readily loses the valence electron. Both the cation and
4, Reactivity towards hydrogen the electron combine with ammonia to form
All the alkali metals react with hydrogen at about ammoniated cation and ammoniated electron.
673 K (except Li which reacts at 1073K) to form M + (+ y) NH, t——> [M(NH,)]* + le(NH3) 1"
hydrides which are ionic in nature (M*H_). Ammoniated Ammoniated

2M +H,—"“*> oM* H> (M=Na, K, Rb or Cs) cation


The ammoniated electron is responsible for the
electron

Metal hydride blue colour of the solution. When ordinary light falls on
e.g. 2Li + LS 21iH these ammoniated electrons, they get excited to higher
Lithium hydride energy levels by absorbing energy corresponding to red
673K
region of the visible light. As a result, the transmitted
2Na + H, ———+> 2NaH light is blue which imparts blue colour to the solution.
Sodium hydride The electrical conductivity is due to the ammoniated
(i) The reactivity of alkali metals with hydrogen cation as well as ammoniated electron.
decreases from Li to Cs as: The dilute solutions are paramagnetic because
Li> Na>K > Rb> Cs they contain free ammoniated electrons.
(iu) Allthe alkali metal hydrides are tonic solids with When the concentration increases, the ammoniated
high melting point. However, the tonic character of the metal ions may get bound by free unpaired electrons.
s-BLOCK ELEMENTS (ALKALI AND ALKALINE EARTH METALS)

This is described as expanded metals. The solutions M(g) ——> M?* (g)+e7 lonisation enthalpy
become bronze in colour and paramagnetic character However, oxidation potential is the property when the
decreases. The decrease in paramagnetic character metal goes into the solution as M* (aq) ions :
suggests that in concentrated solutions the ammoniated
M(s) ———> M*(aqg)+e- Oxidation potential
electrons also associate to form electron pairs.
2e~ (NH3), —> |[e~ (NH), |, The process of oxidation may be thought to proceed
as :
Thus, blue coloured solutions (less concentrated)
are paramagnetic while bronze coloured solutions (2) M(s) ———> M(g) AH sublimation
(high concentration) are diamagnetic. (iL) Mig) ——— M?*(g)+e7 Ionisation enthalpy
On standing, the solutions slowly liberate hydrogen (uit) M*(g)+ H,Q0 ———> Mi(aq) Hydration enthalpy
and therefore, are unstable withrespecttoamide formation. The overall tendency for the change depends
M* (am) + e- (am) + NH, (2) ——> M NH, (am) + -Hi, upon the net effect of three steps. Now, we know that
Li* has the smallest size and is hydrated to maximum
where am stands for solution in ammonia.
extent. Therefore, large amount of energy (called
2M + 2NH, —— 2MNH, + H,O hydration enthalpy) is released in the third step. The
Metal amine amount of energy released (in third step) is so large
However, under anhydrous conditions and in the that it compensates the higher energy needed to remove
absence of catalytic impurities (transition metals such electron (in second step). The net effect is that it has
as Fe, Pt, Zn, etc.) solutions are stable and can be ereater tendency to lose electrons in solution than
stored for several days. other alkali metals and the reaction :
Li(s) —— Li*(ag) + e occurs easily
7. Hydration of ions
The alkali metal ions are highly hydrated. The Therefore, lithium is the strongest reducing agent
smaller the size of the ton, the greater ts the degree of because of its greater hydration energy.
hydration. Thus, Lit ion gets much more hydrated GENERAL CHARACTERISTICS OF
than Na* ion which is more hydrated than K* ion and THE COMPOUNDS OF ALKALI METALS
so on. Therefore, the extent of hydration decreases The compounds of alkali metals are mostly ionic in
from Li* to Cs*. As a result of larger hydration of Lit nature. Due to the low ionization enthalpies and large
ion than Na? ion, the effective size of Lit ion is more atomic sizes, the atoms of alkali metals form cations
than that of Na* ion and the ionic radii in water
(called hydrated ionic radii) decrease in the order : Partial covalent character of lithium halides
Lit > Nat > K* > Rb* > Cst The salts of alkali metals are the most ionic salts
known. Although lithium is an alkali metal, yet its
Ion Lit Nat Kt Rb* fats" compounds, particularly halides, are slightly covalent in
Ionic radius (pm) 76 102 188 152/67 167 nature. This is because the Li* ion has small size and has
Hydrated radius (pm) 340 276 2382 228 226 maximum tendency to withdraw the electrons towards itself
Ionic mobility 33.56 43.5 645 Aib5 “Gas from the negative ion. In other words, it distorts the electron
(ohm! em? mol!) cloud of the anion towards itself. This distortion of the electron
cloud of the negative ion by the positive ion is known as
As aresult, the hydrated Li* ion being largest polarisation. As a result, the charges on the ions become
in size, has the lowest mobility in water. On the less because some of their charges get neutralized.
other hand, the hydrated Cs* ion being smallest in
Polarized anion Unpolarized
size has the highest mobility in water. 7 e anion
Lit has highest degree of hydration and therefore,
lithium salts are mostly hydrated e.g., LiCl. 2H,O.
8. Reducing nature
Alkali metals are strong reducing agents. This is
due to their greater ease to lose electrons. This is also
@)
Polarizing cation
indicated by the very large negative values of their
reduction potentials (Table 3). All of them are better This leads to covalent character. Thus, ifpolarisation
reducing agents than hydrogen (E° = zero). Therefore, is more, larger will be the covalent character of the bond. The
these metals react with compounds containing acidic polarisation depends upon
hydrogen atoms such as alcohols and acetylene (i) The polarisation increases with decrease in size of
the cation.
liberating hydrogen.
(it) The polarisation increases with increase in size of
2Li+2C,H,OH —— 2C,H,OLi +H,
the anion because, the anion gets polarised. If the anion is
2Na+HC=CH —— NaC=CNa+H, large, its electron cloud will be weakly held by the nucleus
Acetylene Sod. acetylide and can be easily deformed towards the cation.
Lithium, the first element of the group ts expected (ii) The polarisation increases with increase in charge
to be the weakest reducing agent due to its very high of the ion (cation or anion).
tonisation enthalpy. However, it is the strongest Thus, Li* being small in size, polarises the anion and
reducing agent as indicated by its reduction potential results in decrease of the positive charge on Lit ion. Therefore,
value (3.04 V). This anomaly can be explained as : lithium halides are covalent in nature. For example, Lil is
As we have learnt that ionisation is the property covalent.
of isolated atoms in the gaseous state.
MODERN'S abc + OF CHEMISTRY
-XI

readily by losing the valence electrons. Consequently, The hydroxides are obtained by the reaction of
they form ionic bonds with non-metals of p-block. alkali metals or their oxides with water.
General characteristics of some of the compounds 2Na+2H,O0 — > 2NaOH + Ho
of alkali metals are discussed below.
2K+2H,O0 ——> 2KOH + H,
1. Oxides and hydroxides
Na,O+H,O — > 2Na0H
Alkali metals react with oxygen to form three types
of oxides, namely oxides, peroxides and superoxides These are white crystalline solids. The hydroxides
depending upon the metal. of alkali metals are strongly basic and_ their basic
Lithium forms only oxide, LiO, and some peroxide strength increases down the group.
(Li,O,), sodium forms the peroxide, Na,O, (plus some Explanation. The M—OH bond in the hydroxides
superoxide Na,O) while potassium, rubidium and of alkali metals is very weak and it can easily ionise to
cesium form the superoxides (MO,). form M* and OH™ ions.
Under appropriate conditions, pure compounds,
MOH(aq) == M*(aq) + OH (aq).
M,O, M,O, or MO, may be prepared. It may be noted
that superoxide ion, (:(Q:::(Q; ) has three electron bond This accounts for their
l.e., it has one unpaired electron. Therefore, basic character. Since the LiOH .
superoxides are coloured. For example, LiO, and NaO, ionisation enthalpy decreases S
are yellow, KO, is orange, RbO, is brown and CsQ, is down the group, the bond NaOH ras
orange. These are also paramagnetic. Sodium peroxide between metal and oxygen =e
is also yellow in colour probably due to the presence of becomes weak. Therefore, the KOH s:
some amount of superoxide in it. However, the normal basic strength of the O04
: . RbOH a
oxides of alkali metals are colourless and diamagnetic. hydroxides increases <
It may be noted that the oxides of the formula M,O, accordingly. Thus, NaOH isa CsOH
called sesquoxides have also been prepared. These stronger base than LiOH and
are dark coloured paramagnetic powders. These are So on.
prepared by thermal decomposition of MO, (M = K, Numerous hydrates have been prepared from the
Rb, Cs). These can also be prepared by oxidation of aqueous solutions of the heavier alkali metal
liquid ammonia solutions of metals or by controlled hydroxides such as NaOH.nH,O (where n = 1, 2,3, 4,5
oxidation of the peroxides. These are considered to and 7) but little is known about their structures.
be peroxide disuperoxides having the formula All the hydroxides are highly soluble in water and
[(M*),05(02).]. thermally stable except lithium hydroxide which
Alkali metals also form oxides of the formula MO, decomposes on heating to lithium oxide.
(called ozonides); M = Na, K, Rb and Cs. These 2LiOH —*> Li,O + H,O
ozonides on standing decompose to superoxide and
oxygen. 2. Halides
2MO, ——— 2MO, + O, The alkali metal halides of the formula, MX (where
M is an alkali metal and X is a halogen) are known.
The normal oxides are basic because they dissolve These are conveniently prepared by the reaction of
in water to form alkali metal hydroxides. Therefore, appropriate oxide, hydroxide or carbonate with aqueous
the alkali metal oxides are basic in nature. hydrohalic acid (HX). For example,
M,O + H,O ——> 2M* + 20H- M,O+2HX —> 2MX+H,0
Metal oxide
Metal oxide
e.g, Na,O + H,O ——> 2NaOH (aq)
MOH+ HX — > MX+H,0
Peroxides and superoxides give hydrogen peroxide
Metal hydroxide
also.
M,O, + 2H,O0 ——> 2M* + 20H + H,O, M,CO,+2HX —~> 2MX+CO,+H,0
Metal peroxide Metal carbonate

e.g., Na,O, + 2H,O ——-— 2NaOH + H,O, where, M = Li, Na, K, Rb or Cs and X =F, Cl, Br or I
2MO, + 2H,O —-> 2M* + 20H + H,O, + O, e.g., NaOH + HCl—-+ NaCl+H,O
Metal super oxide Na,CO,+2HCl —~ 2NaCl+CO,+H,O
e.£., 2KO, + 2H,O ——— 2KOH + H,O, + O, The alkali metal halides are high melting colourless
The peroxides and superoxides also act as oxidising crystalline solids. All these halides have high negative
agents because they react with water and form H,O, enthalpies of formation (A-H"). The enthalpies of
and O, respectively. Sodium peroxide is widely used formation for fluorides become less negative as we go
as an oxidizing agent in inorganic chemistry. down the group while the reverse is true for A,H° for
s-BLOCK ELEMENTS (ALKALI AND ALKALINE EARTH METALS)

chlorides, bromides and iodides. For a given metal, Lithium being small in size polarises a large CO,”
the enthalpy of formation becomes less negative from ion leading to the formation of more stable Li,O and
the fluoride to the iodide. Thus, fluorides of alkali CO,.
metals are most stable while iodides of alkali
metals are least stable. Lithium does not form solid bicarbonate though it
does exist in solution. All the other bicarbonates on
Table 4. Enthalpies of formation (kJ mol!) of
centle heating undergo decomposition to form
alkali metal halides. carbonates with the evolution of carbon dioxide.
Element MF MCI MBr MI 2MHCO, —#28t_, M{CO, + CO, +H,O
Li —612 —398 —350 —271
Na —569 —400 —360 —288 ANOMALOUS BEHAVIOUR OF LITHIUM
K —563 428 —392 —328 Though lithium belongs to the alkali metals group
Rb —549 423 —389 —329 (Group 1) and has most of the characteristic properties
Cs —531 4124 —395 —337 of the metals of Group 1. However, it differs from the
remaining members of the group in many respects.
The melting and boiling points of the alkali This is mainly due to the following reasons :
metal halides decrease with the increase in atomic (1) The size of lithium atom and its ton is very
mass of the halide as: small.
Fluoride > Chloride > Bromide > Iodide (ii) It has high ionisation enthalpy and least
For a given halide ion, the melting and boiling points electropositive character.
of LiX are always less than that of NaX and their values (ii) The polarising power of Li* ion is quite high
decrease from Na to K and Rb to Cs. The values of NaX due to its small size which results in the covalent
are Maximum (with the exception of the m.p. of KI) for character of its compounds.
each series. (iv) Ithas no vacant d-orbitals in its valence shell.
All the alkali metal halides are soluble in water Some of the important characteristics of
with the exception of LiF. However, LiF is insoluble in distinction between lithium and the rest of family
water due to its high lattice energy because of small members are given below :
cation and small anion size. The CsI has also low (i) Lithium isthe hardest ofall the alkali metals.
solubility due to smaller hydration energy of its two (ii) The melting and boiling points of lithium are
ions. It may be noted that in solubilities lithium salts much higher than those of other elements of group 1.
resemble those of Mg** salts. Other halides of lithium (iii) Lithium is less reactive as compared to other
are soluble in ethanol, acetone, ethyl acetate etc. LiCl metals and it does not get tarnished readily in air.
is soluble in pyridine.
(iv) Lithium is deliquescent and crystallizes as a
3. Salts of oxo acids hydrate, LiC].2H,O whereas, other alkali metal
Alkali metals are electropositive metals and chlorides do not form hydrates.
therefore, they form salts with oxo acids. Oxo acids are (v) Lithium bicarbonate is not obtained in the
those in which the acidic proton is on a hydroxyl group solid form while all other elements of this group form
with an oxo group attached to the same acid. The solid bicarbonates.
common examples are, carbonic acid, H,CO, or (vi) It forms only monoxide, Li,O with oxygen
OC(OH),, sulphuric acid, H,SO, or O,S(OH),, while sodium forms peroxide, Na,O, and other elements
phosphoric acid, H,PO, or OP(OH),, etc. The alkali form superoxide, MO,,.
metals form salts with all the oxo acids. These are (vii) Thesalts of lithium have lower ionic character
generally soluble in water and are thermally stable. than salts of other alkali metals. This is because ofhigh
Their carbonates (M,,CO,) and in most cases hydrogen polarizing power of Li* ion.
carbonates or bicarbonates (MHCO,) are highly stable
(viii) Itreadily reacts with nitrogen forming nitride
to heat. As the electropositive character increases down
while other metals do not react.
the group, the stability of the carbonates and
6Li+N, —— 2Li,N
bicarbonates increases.
(ix) Only lithium combines with carbon and silicon
The behaviour of hthium is unusual because its forming the carbide and the silicide.
carbonate is not so stable to heat and its bicarbonate (x) Lithium hydroxide and carbonate are unstable
does not exist as a solid. Li,CO, decomposes readily and decompose on heating while the corresponding
because it is not very stable. compounds of the rest of the family members are stable
and do not decompose :
Li,cO, —“> Li,O + CO, 2Li0OH ——> Li,O+H,O
S Tiono li,cO, ——> 1li1,0+CO, respective groups.
MODERN'S abc + OF CHEMISTRY-XI

(xt) Upon heating, lithium nitrate gives nitrogen (wv) Both LiOH and Mg(OH), are weak bases.
dioxide and oxygen. (v) Unlike the other members of the group,
lithium reacts with N, to form lithium nitride.
4LiNO, —“"> 2Li,0 + 4NO, + O, Magnesium also reacts in a similar way.
Nitrogen dioxide
Sodium nitrate and potassium nitrate upon strong
6Li +N, —“"> eLi,N
Lithium nitride
heating form corresponding nitrites and evolve oxygen.
3Mg + N, —=“2" 5 Mg.N,
2NaNO, —“* » 2NaNO, + 0, : Magnesium nitride
(vi) Both lithium and magnesium combine with
(xii) Lithium does not react with ethyne oxygen to form monoxides. Other members form
(acetylene) to form lithium acetylide while all other peroxides and superoxides.
alkali metals react to form corresponding acetylides.
For example, 4li+ 0, f=», #0
_
2Na + HC=CH
Liq. NH
Se Na* C=C Na* + H,
2Mg + O, —H82!_, 2MgO
The oxides Li,O and MgO do not combine with
Ethyne Disodium acetylide excess O, to form peroxides or superoxides.
(vit) Both react slowly with cold water. Their
However, when lithium is heated with carbon,
oxides and hydroxides are much less soluble. The
it forms lithium acetylide (or lithium carbide) whereas hydroxides of both these metals decompose on strong
other alkali metals do not react with carbon directly. heating to form respective oxides.
2Li + 20 —He8_, Li CHC Li* 2LiOH —“**_, Li,O + H,O
Dilithium acetylide
(xiit) The hydride of lithium (LiH) is more stable as (vill) The carbonates of both these metals
compared to the hydrides of other members of the decompose on heating to form oxide and evolve CO,,.
family.
[100,- "5 11,04-CO,
(xiv) Lithium chloride and nitrate are soluble in
alcohol, while the salts of sodium are insoluble in
MgCo, —““**_ MgO + CO,
alcohol.
(ix) Solid bicarbonates of both lithium and
magnesium are not formed.
(xv) Lithium hydroxide is much less basic than (x) Both LiCl and MgCl, are deliquescent and
the hydroxides of other metals. crystallise from aqueous solution as hydrates,
(xvi) Lithium fluoride, carbonate, hydroxide, LiCl.2H,O and MgCl,. 8H,O.
oxalate are sparingly soluble in water. The corres- (xi) Both LiCl and MgCl, are soluble in ethanol.
ponding salts of sodium and potassium are readily (xii) The nitrates of both lithium and magnesium
soluble.
evolve nitrogen dioxide and oxygen on heating.
(xvit) The Li* ion and its compounds are more
heavily hydrated than those of sodium. 4LiNO, —~“"> 21i,0 + 4NO, + 0,
DIAGONAL RELATIONSHIP OF LITHIUM
2Mg(NO,),—~"> 2MgO + 4NO, + O,
(xiit) Lithium hydroxide, fluoride, phosphate,
WITH MAGNESIUM
oxalate and carbonate are much less soluble than the
As already learnt, hthium resembles with
corresponding sodium and potassium salts. However,
magnesium due to diagonal relationship.
the solubilities are comparable to those of the
Similarities of lithium and magnesium corresponding magnesium compounds.
Some common characteristics of lithium and
Magnesium are : = SOLVED EXAMPLES =
(i) The atomic radius of lithium (152 pm) is close
to that of magnesium (160 pm). The ionic radii are LJ Example 1
also similar (Lit = 76 pm, Mg** = 72 pm). What makes lithium to show properties
(iz) Both have almost similar electronegativities uncommon. to rest of the alkali metals ¢
(Li = 1.0, Mg = 1.2) Solution: The uncommon properties of lithium as
(uit) Both Li and Mg are quite hard. They are compared to other alkali metals is due to
harder and lighter than other elements in the (i) The exceptionally small size of its atom and ion.
s-BLOCK ELEMENTS (ALKALI AND ALKALINE EARTH METALS)

(ii) High polarising power because of high charge/ radius strength 1s:
of its ion. Li > Na’ >I >Ag >CtI.
LI Example 2 LY Example 7
When a cation is highly polarising ¢ Which alkali Why is KO, paramagnetic? N. C.E.R. T.
metal ion has the highest polarising power ?
Solution: The superoxide ion, O, may be represented
Solution: A cation is highly polarising if its charge / as:
radius ratio 1s very high. Li* 10n has the highest polarising
power among the alkali metal ions because it has highest
charge/ radius ratio.
[:626:]
It contains one unpaired electron and therefore, it 1s
LI Example 3. paramagnetic. The M.O. electronic configuration of O, is:
Arrange the following alkali metal ions in KK(o2s)?(o'2s)"(o2p,)" (n2p,P(n2p,)"(n
2p, 2,"
decreasing order of their mobility : Li’, Na’, K’, This also shows that it has one unpaired electron and
Rbt, Cs* hence is paramagnetic.
Solution: Cs > Rb’ > K >Na’ >Li USES OF ALKALI METALS
LJ Example 4 Some of the important uses of alkali metals are
Arrange the following in order of increasing listed below :
covalent character : Uses of lithium
MCl, MBr, MF and MI (where M = alkali metal) The important uses of lithium are :
Solution: With increasing size of the anion, covalent
character increases and, hence, the order is : (i) It is used in the manufacture of various alloys
such as lithium-lead alloy, ithium- aluminium
MF <MCl < MBr < MI.
alloy, ithium—magnesium alloy etc.
LJ Example 5.
¢ Its alloy with magnesium (14% Li) lithium-
What is the oxidation state of magnesium is extremely tough and corrosion
(i) lithium in L1,0. resistant. It is used for armour plate and
aerospace components.
(it) sodium in Na,O,.
¢ The lithium-lead alloy (0.05% Li) (called white
(iti) potassium in KO,.
metal) is used for making toughened bearings
Solution: (i) Lithium in Li,0. for motor engines and sheets for cables.
Oy + (—2)=0 “x= 4+] ¢ Lithium-aluminium alloy has high tensile
Oxidation state of Li in Li,O is +1. strength and elasticity like that of mild steel.
(ii) Sodium in Na,O,. The oxidation state of peroxide It is used for aircraft construction.
ion, O,7 is -2. (iz) Lithium is used for refining copper and
“, Qe + (—2) =0 “x= +l nickel.
Oxidation state of Na in Na,O, is +1. (i) Lithium is used for producing thermo-
(iit) Potassium in KO, nuclear energy required for propelling
The oxidation state of superoxide ion, O,- is —1 rockets and guided missiles.
-*+(-1)=0o0rx=+1 (iv) Lithium is used to make both primary and
Oxidation state of K in KO, is +1 secondary batteries. There is great interest
in Li/S batteries which could power battery
LJ Example 6. cars in the future.
The E° values are :
(v) Lithium compounds also find extensive uses
Cl’/Cl, = + 1.36V, I/II, = +0.58V such as:
Ag*/Ag= +0.79V, Na*/Na=-2.71V (a) Lithium carbonate is used for making
Lit /Li = -3.04V special variety of tough glass which is
Arrange the following tonic species in decreasing order of weather proof and is very strong.
reducing strength: (6) Lithium chloride is usedin air-conditioning
I’Ag", CI, Li‘, Na’ plants to regulate the humidity.
Solution: The lesser the E° value, the stronger is the (c) Lithium hydride is used to generate
reducing agent. Therefore, the decreasing order of reducing hydrogen for meteorological purposes.
S Tionz (d) Lithium hydroxide is used to absorb CO,
MODERN'S abe + OF CHEMISTRY-AI

potassium compounds such as


from exhaled air in closed quarters like ¢ Potassium chloride is used as a fertilizer.
submarines and space vehicles.
¢ Potassium hydroxide is used in the
(e) Lithium bromide is used in medicines as manufacture of soft soap. It is also used as an
sedative. excellent absorbent of carbon dioxide.
() Lithium stearate is used in making ¢ Potassium nitrate is used in ceramics,
automobile grease. colour TV tubes and fluorescent light tubes.
(g) Lithium bicarbonate and lithium Cesium is used in devicing photoelectric cells.
salicylate have been used for treatment
SOME IMPORTANT COMPOUNDS OF
of rheumatism because the lithium
SODIUM
urate formed is soluble in water.
Sodium forms many industrially important
(h) Lithium aluminium hydride (LiAIH,) is compounds. Among these sodium carbonate, sodium
used as a reducing agent in synthetic hydroxide, sodium chloride and sodium bicarbonate
organic chemistry. are very important and are discussed below :
Uses of sodium 1. Sodium carbonate (washing soda),
The important uses of sodium are: Na,CO,.10H,O.
(4) An alloy of sodium with mercury (sodium Sodium carbonate is generally prepared
amalgam) is used for the preparation of many commercially by Solvay process or ammonia-soda
organic compounds. process.
(it) Liquid sodium or its alloy with potassium is In principle, sodium carbonate may be simply
used as a coolant in fast breeder nuclear prepared as :
reactors. It transfers heat from the reactor to 2NaCl + CaCO, ——> Na,CO, + CaCl,
turbines, where the heat produces steam However, this reaction is reversible and creates
which is used to generate electricity. other problems in the system. Therefore, other
(iit) Sodium with lead alloy is used to prepare chemicals are required to get the desired product. It
tetraethyl lead (C,H.), Pb, which acts as an can be prepared on the basis of the following
antiknock compound for petrol. About 60% of principle.
the world production of sodium is used to Principle of the process. When carbon dioxide
make (C,H.),Pb. cas is passed through a brine solution (about 28% NaCl),
(iv) Itis used in sodium vapour lamp. saturated with ammonia, it gives sodium bicarbonate.
(v) Sodium is used as reagent to detect the 2NH, + H,O + CO, ——> (NH,),CO,
presence of nitrogen, sulphur and halogens in Ammonium carbonate
organic compounds (Lassaigne extract). (NH,),CO, + H,O + CO, ———~ 2NH,HCO,
(vt) Sodium is used for the preparation of useful Ammonium bicarbonate
reagents such as NaNH, (sodamide), Na,O, NH,HCO, + NaCl ———> NaHCO, + NH,Cl
(sodium peroxide) and NaCN (sodium cyanide). Brine Sodium
(vit) Sodium is used as a reducing agent in the bicarbonate (ppt.)
extraction of boron and silicon. The precipitated sodium bicarbonate is filtered and
(viit) It is used as a reagent in Wurtz reaction for dried. It is ignited to give sodium carbonate.
the synthesis of many organic compounds. It
is also used in drying organic solvents.
2NaHCO, —“"> Na,CO, + CO, +H,O
Sodium
(ix) Sodium is largely used in industry for the carbonate

production of artificial rubber, dyes, drugs, etc. Raw materials. The raw materials for the process
are :
(x) Because of its lightness and high thermal
conductivity, it is used for filling exhaust (i) Sodium chloride (NaC]).
valves of aeroplane engines. (i) Limestone (CaCO,) for carbon dioxide.
Uses of potassium (wi) Ammonia gas (NH,).
Details of the process. The plant used for the
The important uses of potassium are :
manufacture of Na,CO, is shown in Fig. 2. It involves
(i) Itis used as a fertilizer in agriculture. the following steps :
(ii) It has vital role in biological systems. 1. Ammonia absorber-saturation of brine
(ui) It is used in the preparation of many useful with ammonia. About 30% brine solution is pumped
s-BLOCK ELEMENTS (ALKALI AND ALKALINE EARTH METALS)

NH, + small CO,

®
2 tower
2
6
o
&
Ammonia
recovery
L
rs]
‘Ss
brine
Ammonical
= Stea
<<

Filter
Fe ee FT] ey 6]a NH,Cl + NH, HCO,
i— oa ie ome a
i i ie a) et
Hota A eee
Sodium bicarbonate for ignition

Fig. 2. Solvay process for the manufacture of Na,CQ,.

into ammonia absorber. A mixture of ammonia and a vacuum pump to separate sodium bicarbonate.
small amount of carbon dioxide 1s bubbled through the The filtrate containing ammonium chloride and
brine solution so that it gets saturated with ammonia. small amount of ammonium bicarbonate is pumped to
The impurities of magnesium or calcium salts (if the top of the ammonia recovery tower.
present in brine) are precipitated as their
4. Calcination of sodium bicarbonate. Sodium
carbonates.
bicarbonate recovered from the carbonation tower is
2NH, + CO, +H,O ——> (NH,),CO, heated strongly (calcined) in a kiln to form sodium
CaCl, +(NH,),CO, ——> CaCO, + 2NH,Cl carbonate.

MgCl, +(NH,),CO, t——>


(ppt.)
MgCo, + 2NH,Cl
2NaHCo, —@* »Na,CO, +H,0+CO,
5. Ammonia recovery tower. The filtrate in
(ppt.) step-3 (containing ammonium chloride and ammonium
The ammoniated brine is filtered to remove the bicarbonate) is mixed with calcium hydroxide and
precipitates of calcium and magnesium carbonate. heated with steam in ammonia recovery tower with
2. Carbonation tower. The ammoniated brine calcium hydroxide.
is introduced from the top of a tower known as
NH,HCo, —22!_, NH, + CO, + H,O
carbonation tower. It is packed with a number of
horizontal perforated plates each having a hole covered 2NH,Cl + Ca(OH), “22> CaCl, + 2NH
by a perforated plate. In this tower, carbon dioxide is Ammonium chloride + 2H,O
admitted from the base of the tower, which rises The ammonia along with a small amount of carbon
through the small perforations. The brine solution dioxide is pumped to the top of the brine saturation
slowly trickles down and meets the upgoing vapours of tower as shown in the figure.
carbon dioxide to form insoluble sodium bicarbonate. 6. Lime kiln. The carbon dioxide required in
carbonation tower is prepared by heating limestone
NH, +71 een? f/fNEH,* + OH”
(CaCO,) to about 1300 K in a lime kiln. Lime formed
Nat + Cl + NH,*+0OH-+ CO, —~+Na*+HCO,-
is dissolved in water to get calclum hydroxide which is
+ NH,* + Cl
transferred to the ammonia recovery tower.
Nat + HCO.” —+> NaHCO,
(as precipitate) Cac0, "8 5Ci0 4 CO;
Calcium oxide (lime)
NH,*+Clr —+ NH,Cl
(in solution)
CaO +H,O ——-> Ca(OH),
or NaCl + NH, + H,O + CO, —> NaHCO, + NH,Cl Properties
ppt. in solution (4) Sodium carbonate is a white crystalline solid
3. Filtration. The milky solution from the which exists as a decahydrate (Na,CO,. 10H,O). This
carbonation tower is filtered with the help of rotary is also called washing soda.

WWW.JEEBOOKS.IN
S Tiona MODERN'S abc + OF CHEMISTRY
-XI

Flow sheet diagram for Solvay process :

30% Brine
solution

Ammonia
absorber

a (NH,),CO,
CaCO,, MgCo,

Filtered

(NH,),CO,
Heat

Carbonation |
:
tower

Heated in |
‘On | ammonia NaHCo, CO, CaO (s) |
Ca(OH), NH,Cl
U | recovery tower
Treat with
Filtered eee

C a(OH),
NaHCO
|
Heat Sent to ammonia
recovery tower

NEROOR
(it) It is readily soluble in water. (ii) A mixture of Na,CO, and K,CO, is used as a
(iii) Action of heat. On heating, the decahydrate fusion mixture.
loses the water of crystallisation to form monohydrate (iv) It is used in textile industry and also in
(Na,CO, . H,O). Above 373 K, the monohydrate petroleum refining.
becomes completely anhydrous and changes to a white (v) It is used for the manufacture of soap, glass,
powder called soda ash. paper, borax, caustic soda, etc.
(vi) It is used as a reagent in the laboratory.
Na,CO,.10H,o —°“* , /Na,CO,.Hj,0+9H,O (vii) Itis used for paint and dye stuffs.
Na,CO,.H,O 2338 _,Na,cO,+H,O (viii) It is used for metal refining.
Soda ash (ix) It is used for the manufacture of sodium
(iv) Hydrolysis. It gets hydrolysed by water to compounds such as borax, caustic soda,
form an alkaline solution. sodium phosphate, etc.
CO,* +H,O --“@> Ire, + OH- 2. Sodium chloride, NaCl
(v) Action of acids, It reacts with dilute mineral
acids evolving carbon dioxide gas. Preparation. Sea water is the major source of
Na,CO, + 2HCl ——» 2NaCl+H,O + CO,T sodium chloride. The sea water contains about
(vi) Reaction with milk of lime. With hot milk 2.7—2.9% of NaCl. In tropical countries like India,
of lime, Ca(OH),, sodium carbonate reacts to form common salt is generally obtained by evaporation of
sodium hydroxide. sea water. In India, about 50 lakh tons of salt are
Ca(OH), +Na,CO, ——> CaCO,| + 2NaOH produced annually by solar evaporation. The
(vit) Reaction with carbon dioxide. On passing evaporation of sea water gives crude sodium chloride
CO, through aqueous solution of sodium carbonate, which contains impurities of calcium sulphate, sodium
sodium hydrogen carbonate is formed. sulphate, calcium chloride and magnesium chloride.
Na,CO, + H,0 +CO, ———> 2NaHCO, Calcium chloride, CaCl, and magnesium chloride,
Uses of sodium carbonate MgCl, are impurities because they are deliquescent
The important uses of sodium carbonate are : and absorb moisture easily from the atmosphere. For
(i) It is used for softening of hard water. purification, the crude salt is dissolved in minimum
(i) It is used for washing purposes in laundary. amount of water and filtered to remove insoluble
s-BLOCK ELEMENTS (ALKALI AND ALKALINE EARTH METALS)

impurities. Then HCl gas is passed through the It also lowers the electrical resistance.
solution to get saturated solution. Due to common ion
effect, the crystals of pure sodium chloride separate
out. The salts of chlorides of calcium and magnesium
being more soluble in water remain in the solution.
Properties.
(i) Itis white crystalline solid.
(ii) It has m.p. 1081K and b.p. 1713K.
(Zit) Itis soluble in water and its solubility is 36 ¢
per 100 g of water at 273K.
(iv) Pure sodium chloride is non-hygroscopic but
due to the impurities of CaCl,-2H,O and
MgCl,-2H,0O, it behaves as hygroscopic.
Uses of sodium chloride
(t) It is used as a table salt or common salt for
domestic purposes, an essential constituent Moving mercury cathode (—)
of our food.
(ii) It is used in the manufacture of Na,0O,,
Na,CO,, NaOH, Cl,, ete. Fig. 3. Castner Kellner cell.
(iit) It is used for salting out soap.
(iv) It is used in freezing mixture. (6) Diaphragm cell. In Castner Kellner cell,
(v) It is used in tanning and textile industries. there is one problem. The chlorine gas
evolved at the anode tends to dissolve and react with
3. Sodium hydroxide or caustic soda, NaOH
sodium hydroxide forming sodium hypochlorite as :
Sodium hydroxide is manufactured by the
2NaOH + Cl, — > NaCl+NaOCl + H,O
electrolysis of a saturated solution of sodium chloride
(brine) in the following types of cells. or Cl, 2 + 20H —~> OCI + H,
(a) Mercury cathode cell or Castner Kellner Hypochlorite
Cell. The cell consists of a rectangular iron tank in If H, and Cl, gases mix and react, the reaction
which mercury flows along the bottom of the cell and may be explosive. In the presence of sunlight, a
is made cathode. The brine solution flows in the same photolytic reaction may take place which produces
direction and the anode consists of a number of chlorine atoms leading to an explosive chain reaction
eraphite blocks. When electric current is passed with hydrogen. Therefore, it is essential to prevent
through the cell, the following reactions take place at the contact of chlorine with the solution. For this, an
different electrodes. asbestos diaphragm is used which separates the anode
NaCl —> Na*+Cl and cathode compartments. This reduces the chances
that NaOH produced in the cathode compartment can
At anode Clr-e-~— — > Cl
mix and react with Cl, produced in the anode
Cl+Cl —— 7/201, compartment. Thus, it reduces the chances of the side
or 2CL (> ee reaction producing hypochlorite.
At cathode Nat +e- —~>Na
Na+Hge ——>NaHg
Sodium amalgam Carbon anode (+ve)
Since the cathode is mercury, the sodium atoms
produced dissolve in mercury and form an amalgam.
The sodium amalgam flows out and is reacted with
water to give NaOH and H, gas is evolved.
2NaHg+2H,O —> 2Na0OH+2Hg+H,
The clean mercury is recycled back to the
electrolysis tank. Thus during electrolysis,
hydrogen is evolved at cathode and chlorine is evolved i a eea tea oe —

| Asbestos
arte DEE OF ieoe al re
i ee ee) ate rt pee OS ee, Pn) ge ed ha
at anode. Hydrogen and chlorine are two important oe
.
ae ea
—_——————————————————
a ee al
——:
diaphragm
by products. wire gauze
It may be noted that originally the anodes were
cathode (-)
made of graphite. But because of traces of oxygen
produced in a side reaction, they may become pitted
due to the formation of CO,. Therefore, these days
anodes are made of steel coated with titanium
because, titanium is very resistant to corrosion and Fig. 4. Diaphragm method for manufacture of NaOH.
it prevents the problem of pitting and forming CO,.
_ 10/16 MODERN'S abe + OF CHEMISTRY-XI
When the current is passed through carbon (+ve) AIC], +3NaQOH —~ Al(OH), +3 NaCl
anode and wire gauze (—ve) cathode, electrolysis begins. White ppt
| -Blectrolysis Al(OH), +30H- ——~ [Al(OH),]*
2NaCl (aq) + 2H,O (1) ——————> 2Na0OH (aq) + Soluble complex
H.(g) + CL) or 2Al + GOH- + 6GH,O —> 2[Al(OH),]* + 3H,
Chlorine gas is produced at the carbon anode and Similarly,
sodium hydroxide is formed at the outside edge of the Zn (NO,), + 2NaOH —-> Zn(OH), + 2NaNO,
cathode. ppt
Zn (OH), + 20H- —> [Zn(OH),]*
Nafion Membrane Cell
or Zn + 20H- + 2H,0 —> [Zn(OH),]? + H,
There has been considerable interest in using
Uses of sodium hydroxide
thin synthetic plastic membranes as diaphragm in
(4) Sodium hydroxide is used in the manufacture
place of asbestos. These membranes are made of a
of soap, paper, artificial silk and a number of
polymer called nafion supported on a teflon mesh.
chemicals.
Nafion is a copolymer of tetrafluoroethylene and a
(it) It is used in the petroleum refining.
pentafluoro sulphonyl ethoxy ether. These plastic
(iii) It is used in the purification of bauxite (ore of
membranes have lower resistance than asbestos.
aluminium).
In this cell, the natural brine is electrolysed in a
membrane cell in which the anolyte and the (iv) Itisused inthe textile industry for mercerising
catholyte are separated by a nafion membrane. The cotton fabrics.
electrolysis occurs as : (v) Itis used as cleansing agent for machines and
metal sheets.
2NaCl (aq) + 2H,O (2) SE _,oNaOH (aq) (vi) Itis used for the preparation of pure fats and
+ H,(g) + Cl,@) oils.
In India, about 1.35 million tonnes of caustic (vii) Itis used for the preparation of artificial silk.
soda is produced annually. Presently, nafion 4, Sodium hydrogen carbonate or Baking
membrane method is used in place of diaphragm. soda, NaHCO,
Preparation.
Properties
Sodium bicarbonate is obtained as an intermediate
(4) Sodium hydroxide is a white crystalline,
product in Solvay’s process for the manufacture of
deliquescent solid. It melts at 591 K.
sodium carbonate.
(it) It is readily soluble in water to give alkaline It can also be prepared by saturating a solution of
solution. It also neutralises acids forming salt sodium carbonate with carbon dioxide. Sodium
and water. bicarbonate being less soluble separates out as white
NaOH + HCl ——-> NaCl + H,O crystals.
2NaOH + H,SO,——Y> Na,SO, + 2H,O Na,CO, + H,O + CO, —> 2NaHCO,
(iii) The crystals of sodium hydroxide are Properties.
deliquescent t.e., they absorb moisture from (i) Itis a white crystalline solid.
air. On prolong exposure, sodium hydroxide (it) It is only sparingly soluble in water.
absorbs CO, resulting in the formation of
(iit) Upon heating to 373 K, it decomposes to form
white crust of solid hydrated Na,CO, at the
sodium carbonate.
surface.
2NaOH + CO, ——-> Na,CO, + H,O 2NaHCO, 2K _,Na,CO, + H,O + CO,
(itv) Its aqueous solution is soapy to touch and has (iv) Its aqueous solution is alkaline in nature.
a strong corrosive action on the skin.
NaHCo, + H,O ——+NaOH + H,0 + CO,
(v) An aqueous solution of sodium hydroxide
Uses of sodium hydrogen carbonate
contains large concentration of hydroxy] ions
(OH-) and precipitates insoluble metal (i) Sodium hydrogen carbonate (or baking soda)
hydroxides from solutions containing is mainly used in making baking powder.
metallic ions. The hydroxides of aluminium, Baking powder contains sodium hydrogen
zinc, lead and tin dissolve in excess of sodium carbonate and an acid like tartaric acid or
hydroxide giving clear solutions due to the citric acid. When baking powder (mixed in the
formation of soluble complexes. These soluble dough for preparing cakes or bread) is heated,
complexes can also be obtained when these sodium hydrogen carbonate decomposes to
metals react with concentrated solution of give carbon dioxide and sodium carbonate.
NaOH. For example, Carbon dioxide bubbles out causing the bread
s-BLOCK ELEMENTS (ALKALI AND ALKALINE EARTH METALS)

and cakes to rise leaving holes in cakes or in the concentration of Na* and K* ions found on the
pastries. This makes them light and fluffy. opposite sides of cell membranes. For example, in
Tartaric acid or citric acid present in the blood plasma, Nat* ions are present to the extent of
baking powder reacts with sodium carbonate 143 m mol L! while K* ions are present only upto
and neutralises it. It may be noted that if 5 m mol L! within the red blood cells. However, these
tartaric acid or citric acid is not present in concentrations change to 10 m mol L~! (Nat) and
baking powder, it will taste bitter due to the 105 m mol L7?(K*) in the cell fluids. These ionic gradients
presence of sodium carbonate. control the development and the functioning of the
nerve cells.
It may be noted that baking powder must be
This has been explained in terms of a mechanism
added during the preparation of cakes,
pastries, etc. If, however, baking powder is
known as sodium-potassium pump. Because of large
concentration gradient inside and outside the cells, the
not added, the cake obtained will be
transport of Na‘ ion into the cell is favoured. Therefore,
comparatively hard and small in size.
the Na* ions entering the cell must be pumped out
(it) Baking sodais used in medicines as an antacid again from the cell to maintain concentration
to remove acidity of the stomach. Sodium gradients. The maintenance of these large
hydrogen carbonate is a mild antiseptic for concentration gradients provide a driving force to carry
skin infections. out important transport processes involving amino
(zit) Itis used in the manufacture of aerated water acids, sugars, nucleotides, etc. For this transport, some
(soda water). metabolic energy 1s expanded to maintain the
(iv) Sodium bicarbonate is used in fire concentration gradients of Nat and K* ions inside and
extinguishers. Soda acid fire extinguisher outside the cells. This process is known as sodium-
contains a solution of sodium hydrogen potassium pump which operates across the cell
carbonate and sulphuric acid. These can be membrane. The energy for the transport of these ions
brought in contact with each other either by is provided by the hydrolysis of energy rich molecules
pressing a knob or by inverting the ATP. Thus, both sodium and potassium ions are
extinguisher. Carbon dioxide is liberated and essential ions for living organisms.
it forces a steam of effervescing liquid on to The Na* ions are :
the fire. As aresult, carbon dioxide surrounds (z) major component of blood plasma.
the combustible substances and cuts off the (ii)Important in transmission of nerve signals and
supply of air. Thus, it helps to put out the fire. function of heart.
(iii) important in activation of some enzymes.
BIOLOGICAL IMPORTANCE OF SODIUM AND
(iv) important in regulating the flow of water
POTASSIUM
across cell membranes.
Sodium and potassium are most common cations (v) intransport of sugars and amino acids into cells.
in biological systems. Though Na* and K* ions are The K* ions are required in the cells for
chemically quite similar but surprisingly they differ (i) activating many enzymes.
ereatly in their biological activities. Their distribution (ii) participating in the oxidation of glucose to
in biological systems is also different. A typical man produce energy rich ATP molecules.
weighing 70 kg contains about 90 g of sodium and (iii) proteins synthesis
170 g of potassium as compared with only 5g of iron (iv) participate with Na* ions in sodium —
and 0.06 g of copper. The most salient feature is that potassium pump for the transmission of nerve
K* ions are present in higher concentrations inside the signals.
cells than Na* ions while their relative concentrations In general, more than 1/3 of the ATP is used by a
are reverse outside the cells. In other words, K* ions resting animal and about 15 kg per 24 hr in a resting
are concentrated inside the cells while Na* ions are human for the operation of Na—K pump.
concentrated outside the cells. Sodium ions are In 1957, Jens Skou discovered that membranes
primarily found in blood plasma and in the interstitial with sodium pump were found to contain an enzyme
fluid which surrounds the cells. These ions participate adenosin triphosphatase (ATPase), which is
in the transmission of nerve signals, in regulating the responsible for the action of sodium pump.
flow of water across cell membranes and in the
transport of sugars and amino acids into the cells. On SOLVED EXAMPLES
the other hand, K* ions are most abundant cations
within cell fluids, where they activate many enzymes, LJ Example 8
participate in the oxidation of glucose to produce ATP Why tis that on being heated in excess supply of
and with Na* ions are responsible for the transmission air, K, Rb, and Cs form superoxides in preference
of nerve signals. There is very considerable variation to oxides and peroxides ?
S Tione. Solution: K*, Rb* and Cs* are large cations and
MODERN'S abc + OF CHEMISTRY-XI

superoxide ion, O,- is larger than oxide (O7-) ion and Gio —2a* 5
peroxide (O,?-) ion. Alarger cation can stabilize a large Git) Li + HC = CH ——__>
anion and therefore, these metals form superoxides in (tv) Na + O,——>
preference to oxides and peroxides. Q@4. Can we store sodium in water or not ?
UL) Example 9 @ 5. Lithium compounds are slightly covalent because
What happens when KO, reacts with water ? Write of its greater ............ . Complete the statement.
the balanced chemical equation for the reaction. @ 6. What compounds are formed when Li, Na and K
Solution: When KO, reacts with water an alkaline combine with oxygen ?
solution containing H,O, 1s formed and oxygen gas 1s Q 7. Which is most abundant metal among alkaline
evolved. earth metal ?
2KO, + 2H,O > 2KOH + H,0, + O, Q@ 8. Which is the most abundant element among alkali
or 20, + 2H,O —— 20H’ +H,0, +O, metal ?

Ll Example 10. @ 9. Which among the alkali metals form nitride ?


Write balanced equations for reactions between : Q@ 10. Which of the following alkali metals is most
(t) Na,O, and water (ut) KO, and water electropositive ?
(iii) Na,O and CO, Li, Na, K, Rb or Cs.
Solution: = Answers to Practice Problems
(i) Na,O, + 2H,O ——> 2NaOH + H,O,
(ii) 2KO, + 2H,O ——-> 2KOH + H,0, + O, Oo1@Mh (ii) Cs
(iii) _ + CO, ——> Na,CO,,. (iw) La (iv) la
©2. KOH

Pr © 3. (i) 2Mg(NO,), ——> 2Mg0+4NO,+0,


——< Dapllew
=p (ii) 2LiOH no ere
Q@1. Among the alkali metals which element has (iii) 2Li1+ HC =CH ——> LiC=ClLi+H,
(i) highest melting point (iv) 2Na+0O, —- Na,O,
(ii) most electropositive character © 4. No, sodium can be stored in kerosene.
(iit) lowest size of ion _
(iv) strongest reducing character © 5. polarising power.

Q@2. Out of LiIOH, NaOH and KOH, which is the © 6. Li,O,Na,0,,KO,


9
strongest base ” oe Calan.
Q@ 3. Complete the following reactions :
© 8. Sodium
Heat ee
(1) Mg(NO,), ————> © 9. Lithum

© 10. Lithium

add Of Conceptu QU CYUCSTIONS


Q.1. Sodium metal is kept under kerosene. Explain.
Ans. Sodium metalisveryreactive. When exposed to air, it reacts with oxygen, moisture and carbon dioxide present
in the air.
4Na(s) —2+> 2Na,O
Sodium oxide
2Hy0
2Na,0 ——> 4NaQOH
Sod. hydroxide

4Na0OH —=“2, 2Na,CO, +2H,0


Sod. carbonate
Therefore, to prevent these reactions and protect the metal, sodium is kept under kerosene.
s-BLOCK ELEMENTS (ALKALI AND ALKALINE EARTH METALS) 10/19 ~~»

Q. 2. LiClis soluble in organic solvents. Explain


Ans. Li‘ has very high polarizing power and, therefore, LiCl is covalent in nature. Being covalent in nature, it 1s
soluble in organic solvents.
Q. 3. Name the chief factors responsible for anomalous behaviour of lithium.
Ans. The anomalous behaviour of lithium is because of
(t) small size of atom and ion,
(ii) high ionization enthalpy, and
(iii) absence of d- orbitals in its valence shell.
Q. 4. Name the alkali metals which form superoxides when heated in excess of air.
Ans. Potassium, rubidium and cesium.
Q. 5. Name the metal which floats on water without any apparent reaction with it.
Ans. Lithium
Q.6. Predict giving reason, the outcome of the reaction :
Lil + KF ——_>
Ans. Lil + KF ——-> LiF + KI
Large cation (Kt) can stabilize large anion (T).
Q. 7. Why does the following reaction :
SC - Cl + MF —_> SC -F+MCl
proceed better with KF than with NaF.
Ans. This is because the larger cation can stabilize larger anion.
Q.8. What compounds are formed when Li and Na combine with oxygen. Give the hydrolysis reactions
of compounds formed. N.C.E.R.T.
Ans. Li and Na combine with oxygen to form Li,O and Na,O,. The hydrolysis reactions are :
Li,O + H,O —+ 2Li0H
Na,O, + H,O —> 2NaOH + H,O,
@. 9. Explain why sodium is less reactive than potassium.
Ans. Potassium is more reactive than sodium because of its large size and low ionisation enthalpy in comparison
to sodium.
Q. 10. Why are potassium and cesium, rather than lithium can be used in photoelectric cells ?
Ans. Potassium and cesium have low ionisation enthalpy values, so electrons can be easily ejected from these
metals when light of suitable frequency strikes on their surfaces. Lithium has high ionization enthalpy and
therefore, it does not show photoelectric effect and cannot be used in photoelectric cells.
Q. 11. Why are lithium salts commonly hydrated and those of other alkali ions usually anhydrous ?
Ans. Because of small size, Li* has high charge density and therefore, lithium salts are extensively hydrated.
Q. 12. Why is LiF almost insoluble in water whereas LiCl is soluble not only in water but is also in
acetone?

Ans. Both Li* and F- have small size and therefore LiF has very high lattice enthalpy. Although hydration
enthalpy of LiF is also high but it is less than lattice enthalpy (Lattice enthalpy of LiF = —1045 kJ, hydration
enthalpy of LiF = —1034 kJ mol“). But LiCl has lower lattice enthalpy than corresponding hydration
enthalpy and therefore, is soluble in water. LiCl has also some covalent character because of greater
polarization hence it 1s also readily soluble in nonpolar solvents such as acetone.
Q. 13. Lithium is the only alkali metal to form nitride directly. Explain.
Ans. Li* being small in size can easily stabilize small nitride ion, N*-. Other alkali metal ions of larger size cannot
stabilize small nitride ion.
Q. 14. An aqueous solution of sodium carbonate gives alkaline tests. Why ? N.C. E. R.T.

Ans. Sodium carbonate in aqueous solution gets hydrolysed to give OH ions and, therefore, gives alkaline tests.
CO,2-+H,O > HCO, + OH-.
ST020 Q. 15.
MODERN'S abe + OF CHEMISTRY-AI

Why is Li,CO, decomposed at lower temperature whereas Na,CO, at higher temperature ?


Ans. Li* ion being small in size, cannot stabilize large CO,” ion therefore, Li,CO, is unstable. On the other hand
sodium is larger in size and can easily stabilize large CO,” ions and form stable Na,CO,. This is because
of lattice energy effects.
Q. 16. LiH, LiF and Li,N show exceptional thermal stabilities. Explain.
Ans. Li‘ is small in size and has exceptionally high charge—radius ratio. Therefore, its salts with small anions
such as F-, H- and N* are exceptionally stable due to their high lattice enthalpies.
Q. 17. Which out of Li or Na has greater value for the following properties :
(4) Hydration enthalpy (i) Stability of hydride
(iii) Stability of carbonate (jv) Basic character of hydroxide
(v) lIonisation enthalpy.
Ans. (i) Li (ii) Li (iii)Na (iv) Na (v) Li
Q. 18. Which out of the following and why can be used to store an alkali metal ?
H,O, C,H.OH, Benzene
Ans. Benzene can be used to store an alkali metal because other substances react with alkali metals as :
Na+H,O oe NaOH + 42H,
Na+ C,H,OH an C,H,ONa + 4H,
Q.19. Why are alkali metals not found free in nature? N.C.E.R.T.
Ans. Alkali metals are highly reactive and therefore, arenot found free in nature. They are present in the combined
state in the form of halides, oxides, silicates, borates, nitrates, etc.
Q.20. Why can cesium be used in photoelectric cell but lithium cannot?
Ans. Cesium has the lowest while lithium has the highest ionization enthalpy. Hence, cesium can lose electrons
very easily and can be used in photoelectric cells.
. Complete the following reactions :
(i) OF +H,O— (ii) OF +H,O—>
Ans, (i)07°-+2H,O—-+> 20H +H,0,
(tz) 20, + 2H,O+ 20H + H,O, + O,
Q.22. Can potassium carbonate like sodium carbonate be prepared by Solvay process ? Explain.
Ans. Potassium carbonate cannot be prepared by the Solvay-ammonia process like sodium carbonate. This is
because unlike NaHCO, which is sparingly soluble in water, KHCO, is fairly soluble in water. When CO, is
passed through ammoniated brine, NaHCO, gets precipitated but KHCO, does not get precipitated when CO,
is passed through an ammoniated solution of potassium chloride. Therefore, K,CO, cannot be prepared by
the Solvay ammonia process.

CHEMISTRY OF ALKALINE EARTH METALS by common basaltic mineral olivine [(MgFe),Si10,,] and
by soapstone (talc) [Mg,81,0,,(OH).], ete. Calcium is
fifth most abundant element in earth’s crust. It is found
The group 2 of the periodic table consists of the in vast sedimentary rocks of CaCO, existing as whole
elements beryllium (Be), magnesium (Mg), calcium mountain ranges of limestone, marble and chalk.
(Ca), strontium (Sr), barium (Ba) and radium (Ra). Other important minerals are gypsum (CaSO,,. 2H,0),
These elements are known as alkaline earth metals. anhydrite CaSO,, fluorite (CaF) and fluoroapatite
OCCURRENCE [3Ca,(PO,),.CaF,].
Magnesium like its heavier congeners occurs in Strontium occurs as celestite (SrSO,) and
crustal rocks mainly as insoluble carbonates, sulphates strontianite (SrCO,). The main ore of barium is
and silicates. It also occurs to about 0.13 % in sea water baryte (BaSO,). It also occurs as witherite (BaCO.,).
as chloride and sulphate. Magnesium is sixth most Beryllium is less abundant like Li. It is found as
abundant element following calcium. It occurs as surface deposits of beryl in pegmatite rocks. Radium
dolomites (MgCO,.CaCO,), magnesites (MgCO.,), occurs only in association with uranium having
epsomite (MgSO,.7H,0). As silicates it is represented terrestrial abundance of approximately 10 ppm.
s-BLOCK ELEMENTS (ALKALI AND ALKALINE EARTH METALS)

GENERAL CHARACTERISTICS OF ALKALINE metals. On moving down the group, the radii increase
EARTH METALS due to gradual increase 1n the number of the shells and
The general relationships throughout the group the screening effect.
are similar to those in Group 1. Table 5 and 6 give the 3. lonisation enthalpy. The alkaline earth
electronic configurations and atomic and physical metals have low tonisation enthalpies due to fairly
properties of Group 2 elements and these are discussed large size of the atoms. Since the atomic size Increases
below : down the group, the ionisation enthalpy decreases
Electronic Configurations and Atomic (Table 5). A comparison of the ionisation enthalpies of
Properties the members of groups 1 and 2 shows that the members
present in the second group have higher values as
1. Electronic Configurations. These elements
compared to those of group 1 because they have smaller
are called alkaline earth metals. The atoms of these
size and electrons are more attracted towards the
elements have two electrons in their valence shell
nucleus of the atoms.
preceded by the noble gas configuration. Their general
configuration may be written as [Noble gas] ns” It may be noted that although IE, values of
where n represents the valence shell (Table 5). alkaline earth metals are higher than those of alkali
metals, but the LE, values of alkaline earth metals are
2. Atomic and ionic sizes. The atomic and
much smaller than those of alkali metals. The ionisation
tonic radi of the members of the family are smaller
enthalpy values ofsodium (alkali metal) and magnesium
than the corresponding members of the alkali
(alkaline earth metal) are given below :
metals.
Explanation. The alkaline earth metals have a Element IE, IE,
higher nuclear charge and therefore, the electrons are
attracted more towards the nucleus. As a result, their Na 496 kJ mol"! 4562 kJ mol
atomic and ionic radii are smaller than those of alkali
Mg 737 kJ mol 1450 kJ mol

Table 5. The electronic configuration of alkaline earth metals.

Element Symbol Atomic No. Electronic


Configuration

Beryllium Be 4 [He] 2s?


Magnesium Mg 12 [Ne]3s?
Calcium Ca 20 [Ar] 4s?
Strontium Sr 38 [Kr] 5s?
Barium Ba 56 [Xe]6s?
Radium Ra 88 [Rn]7s?

Table 6. Physical constants of alkaline earth metals.

Be Mg Ca Sr Ba Ra

Atomic number A 12 20 38 56 88
Atomic radius (pm) 113 160 197 215 229 —
Ionic radius M?* (pm) 31 fe 100 118 135 148
Density (g cm-*) 1.84 1.74 1.55 2.63 3.09 (5.5)
Ionisation enthalpy (kJ mol)
Li 899 737 590 D49 p03 p09
IE, 1757 1450 1146 1064 965 979
Electronegativity L.57 1.31 1.00 0.95 0.89 0.90
Melting point (K) 1560 924 1124 1062 1002 O73
Boiling point (K) 2745 1363 1767 1655 2078 (1973)
Hydration enthalpy (kJ mol) -2494 -1921 ~1577 ~1443 ~1305 =
E° (V) M?* |M -1.97 —2.36 —2.84 —2.89 —2.92 —2.92
Occurrence in lithosphere (ppm) rae 27640* 46600* 384 390 ilige
Values in brackets represent uncertain values
* These are expressed as 2.76 (Mg) and 4.66% (Ca) by wt. respectively.
S Tioz2Explanation. In case of alkali metals (i.e., Na)
MODERN'S abe + OF CHEMISTRY-AI

3. Electropositive and metallic character.


the second electron is to be removed from a cation Because of the low ionisation enthalpies of alkaline
which has already acquired a noble gas configuration. earth metals, they are strongly electropositive in
On the other hand, in the alkaline earth metals (i.e. nature. However, these are not as strongly
Mg), the second electron is to be removed from a electropositive as the alkali metals of group I because
monovalent cation, Mg* : (1s?2s?2p°3s!) which still has of comparatively higher tonisation enthalpies. The
one electron in the outermost shell. Thus, the second electropositive character increases down the group
electron in Mg can be removed easily. i.e., from Be to Ba.
Nag) —2, Nat(g) —™> Na?*(g) 4. Characteristic flame colouration. Except
1s?2s72p%3s1 “es 1572522 % 1s22s22p° for Be and Mg, the alkaline earth metal salts impart
Stable configuration characteristic colours to the flame.
TE; ; TEs Be Mg Ca or Ba Ra
Mag(g) en : Mg *(g) Le? Meg** (g )
— — Brick Crimson Grassy Crimson
1s72s72p°3s" 1s22522n6351 15225226 red red ereen red
Not very stable
Beryllium and magnesium do not give any
The IE, of Mg will be very high because the
characteristic flame colours.
electron is to be removed from stable noble gas
configuration. Explanation. The alkaline earth metals give
characteristic flame colours because their ionisation
4. Hydration enthalpy. The hydration
enthalpies are low. Therefore, the valence electrons in
enthalpies of alkaline earth metal ions are larger than
these atoms can be easily excited to higher energy
those of the corresponding alkali metal ions. This is due
states by the energy of flame of bunsen burner. When
to smaller size ofalkaline earth metal ions as compared
these excited electrons (excited state is unstable state)
to corresponding alkali metal ions of the same period. come back to ground state they emit radiations which
Due to smaller size, the alkaline earth metal ions are fallin the visible region. Therefore, they give colours to
strongly attracted by polar water molecules and hence the flame. On the other hand, beryllium and magnesium
have larger hydration enthalpies. Thus, the compounds atoms are comparatively smaller and their tonisation
of alkaline earth metals are more extensively hydrated enthalpies are very high. Hence, the energy of the flame
than those of alkali metals. For example, MgCl, and is not sufficient to excite their electrons to higher energy
CaCl, exist as MgCl,.6H,O and CaCl,.6H,O while levels. These elements, therefore, do not give any colour
NaCl and KC] donot form such hydrates. The number in Bunsen flame.
of molecules of water of crystallisation decreases as The flame test for calcium, strontium and
the ions become larger. For example, BaCl, exists as barium is helpfulin their detection in qualitative
BaCl,.2H,O. analysis.
Like alkali metals ions, the hydration enthalpies 5. Electrical and thermal conductivities. The
of alkaline earth metal ions decrease with increase in alkaline earth metals have high electrical and thermal
tonic size down the group. conductivities which are typical characteristics of
Be** > Mg?* > Ca?* > Sr** > Ba** metals.
This decrease in hydration enthalpies down the 6. Tendency to form bivalent ions. The
croup is due to increase in lonic size of the ion. alkaline earth metals exhibit a valency of +2 as they
Physical Properties can lose two electrons and form bivalentions. Inspection
of Table 5 reveals that the second ionisation enthalpy
1. Physical appearance. The alkaline earth
(IE.,) of alkaline earth metals is greater than the first
metals, in general are silvery white, lustrous and
ionisation enthalpy (IE,). It, thus appears that if
relatively soft. However, these are harder than the
ionisation enthalpy is the only factor involved in the
corresponding alkali metals. Beryllium and
formation ofdivalentions, we would expect thatalkaline
magnesium appear to be somewhat greyish.
earth metals would prefer to form +1 tons (E*) rather
2. Melting and boiling points. Alkaline earth than + 2 ions (E**). Actually, they predominatly show
metals have low melting and boiling points which are +2 valency, e.g., Mg?*, Ca**, Ba**, etc.
higher than the corresponding alkali metals in the
Explanation. This can be explained as :
same period due to comparatively smaller size. However,
melting and boiling points do not show regular trends. (iz) Divalent ions have the stable noble gas
configuration.
Explanation. The atoms ofalkaline earth metals
have smaller size as compared to alkali metals. (ii) In solution, the divalent ions of alkaline earth
Therefore, they are more closely packed in their crystal metals are extensively hydrated and the high hydration
lattices and hence, they have higher melting and energies of E?* ions make them more stable than E*
boiling points. ions. It is observed that the amount of energy released
s-BLOCK ELEMENTS (ALKALI AND ALKALINE EARTH METALS)

when E* ion is dissolved in water is much more that that for E* ion. This large amount of extra energy released
in the hydration of + 2 tons compensates the second tonisation enthalpy required for the formation of such tons.
(iii) In the solid state, the divalent cations form stronger lattices than monovalent cations and therefore, a
lot of energy called lattice enthalpy is released. It is the greater lattice enthalpy of E** ion which compensates for
the high second ionisation enthalpy and is responsible for its greater stability as compared to E* ion.

This can be easily understood by considering the overall energy changes for the formation of MgCl and
MgCl, in water with the help of Born Haber cycle. The various steps are :
Mg(s) ——> Me(g) AH iinntian = 150 Kelprmol*

Mgig) —— Meg(g)+e AH,, = 737 kJ mol!

YCL(g) ——> Cl(g) AH. ication = foe) Mil}

Cl(g) —— CI(g) AH = — 348 kJ mol

Mg*( g) + Clg) +nH,O ——> Meg*(aq) + Cl(aq) AH rpdratien —735 kJ mol!


Net energy change = -74 kJ mol"
Mg(s) ——~> Mg(g) AH, @>, Wreeti50 kJ mol

Me(g) ——> Mg++ 2e- AH,; = 2188 kJ mol

Cl.(g) —— > 2Cl AH atonmusation


= 2x 122 kJ mol?

2C1 + 2e— ——> 2CI AH... = —2 x 348 kJ mol!

Mg?*( g) + 2CI( g) + nH,O——> Meg?*(aq) + 2Cl(aq) AH = —2670 kJ mol

Net energy change = -784 kJ mol"


Thus, it is clear that during the formation of MgCl, in water, more energy is released and therefore, it
is more readily formed.
Similary, it can be shown that net energy change for the formation of MgCl,(s) is -642 kJ mol and that
for the formation of MgCl(s) is —125 kJ mol-*. This shows that MgCl,(s) is more stable as compared to MgCl(s)
Thus, the alkaline earth metals prefer to form divalent ions rather than monovalent ions.

Chemical properties Heat


3Be +N, ———>_ Be,N,
Because of their low ionisation enthalpies and
from air Beryllium nitride
high electropositive character, the alkaline earth metals
have strong tendency to lose valence electrons. Magnesium is more electropositive and burns
Therefore, they are very reactive. The reactivity of with dazzling brilliance in air to give a mixture of MgO
these elements increases on going down the group. and Mg.N..
However, alkaline earth metals are in general less 4Mg + air (O,, N,) “> MgO + Mg.N,
reactive than alkali metals. The chemistry is mainly Calcium, strontium and barium are readily
dominated by the dipositive oxidation state (E**). attacked by air to form oxide and nitride.
Some of the general trends are discussed below :
Thus, the reactivity with oxygen increases as we
1. Reactivity towards air or oxygen. move down the group due to increasing electropositive
The alkaline earth metals being less character of the elements. Therefore, calcium, barium
electropositive than alkali metals, react with air or and strontium are stored in paraffin but beryllium and
oxygen slowly upon heating to form oxides, MO. magnesium are not stored in paraffin because they
Beryllium and magnesium are kinetically inert form a protective layer of oxide on their surface .
towards oxygen because of the formation of a 2. Reactivity towards hydrogen.
film of oxide on their surface. However, powdered
All the elements except beryllium combine with
beryllium burns brilliantly on ignition to give BeO and
hydrogen upon heating to form ionic hydrides, MH,
Be.N,.
2Be + O. ———>
Heat
2BeO Ca +H, ——~—
Heat
CaH,
air Beryllium oxide Calcium hydride
BeH.,,, however, can be prepared by the reaction
of BeCl, with LiAlH,,.
Toa MODERN'S abc + OF CHEMISTRY
-XI

2BeCl, +LiAIH, ——> 2BeH, + LiCl + AICI,. alkaline earth metal salt, BaSO, is insoluble. The
Both BeH, and MgH, are covalent compounds lattice enthalpy of Na,SO, is less than the hydration
having polymeric structures. MgH, is covalent while enthalpy whereas the lattice enthalpy of BaSO,
all other hydrides are ionic. (because of bivalent charge) is very high so that
hydration enthalpy released is not sufficient to
Calcium hydride is known as hydrolith.
break the lattice and it remains insoluble.
The hydrides are highly reactive towards water
4, Action with acids.
and form hydroxides and liberate hydrogen.
Alkaline earth metals react with acids and liberate
CaH, + 2H,0 ———> Ca(OH), + 2H,
hydrogen. However, Be reacts slowly because it is
3. Reactivity towards water. rendered passive by concentrated HNO,. This is
The alkaline earth metals have lesser tendency to because conc. HNO, is a strong oxidizing agent and it
react with water as compared to alkali metals. They forms a very thin layer of oxide on the surface of the
combine with water slowly to form hydroxides. metal, which protects it from further attack by acid.
Beryllium and magnesium are kinetically inert to M + 2HCl ——~+ MCl, +H,
water. Beryllium does not react with water or steam
(M = Be, Mg, Ca, Sr, Ba)
even at red heat.
Mg + 2HC]l ——-> MgCl, + H,
Calcium, stronium and barium react with water
with less difficulty. Be is amphoteric because it also reacts with
NaOH giving H, and sodium berrylate.
Ca+2H,0 —!* Ca(OH), + H, Be + 2NaOH + 2H,0 ———> Na, [Be(OH),] + H,
Like alkali metals, the hydroxides are basic
or NaBeO,. 2H,0O + H,
in nature and basic strength increases down the
family. Sodium beryllate
Mg, Ca, Sr and Ba do not react with NaOH and
Lattice enthalpy versus hydration enthalpy
are purely basic. This shows that basic properties
The solubility of a salt in water depends upon increase on descending the group.
two factors :
5. Reactivity towards halogens.
(i) lattice enthalpy
All the alkaline earth metals combine with
(it) hydration enthalpy. halogens at higher temperatures forming their halides.
Lattice enthalpy is the amount of energy M +X, ——> MX, (X=F, Cl, Br, D
required to break one mole of a solid salt into its
The metal halides can also be obtained by the
tons as:
MX(s) Lattice enthalpy M+ (g) 1X (g) action of halogen acids on metals, their oxides,
carbonates and hydroxides.
Greater the lattice enthalpy, more will be the MO + 2HX ——+ MX, +H,0O
energy required to break the lattice.
M(OH), + 2HX ———> MX, + 2H,O
Hydration enthalpy is the amount of energy
BeF, is conveniently prepared by thermal
released when one mole ofa solid salt gets dissolved
decomposition of (NH,), BeF , while BeCl, can be made
in water (hydrated). Therefore, the net effect will be
from the oxide as :
the energy of solution.
Net enthalpy = Lattice enthalpy — Hydration BeO + 2NH, + 4HF ——>(NH,),[BeF,] <>
enthalpy. BeF,,+2NH,F
Now we know that for the feasibility of a BeO +C+Cl, 870 — 1070K
BeCl, + CO
process i.e., solubility of the salt, the net energy Beryllium chloride can also be prepared by heating
change should be negative i.e., energy should be beryllium oxide with carbon tetrachloride.
released. Thus, it becomes clear that 2BeO + CCL, —““* 5 2BeCl, + CO,
» if hydration enthalpy is more than lattice 6. Reactivity with ammonia.
enthalpy; salt will dissolve
Like alkali metals, alkaline earth metals dissolve
» if hydration enthalpy is less than lattice in liquid ammonia to give deep blue black solutions
enthalpy; salt will not dissolve. forming ammoniated ions.
This can easily explain the increasing solubility M + (x + y) NH, ——~> M(NH,)?* + 2e- (NHs3),
of alkaline earth metal hydroxides on moving down
From these solutions, the ammoniates of the
the group. The lattice enthalpy decreases with
general formula [M(NH,),] 2+ may be recovered.
increase in size of the cation and therefore, solubility
7. Tendency to form complexes.
increases from Be to Ba.
The group 2 elements have tendency to form
This can also explain easily as to why alkali
some stable complexes. Among these, beryllium and
metal salt, say Na,SO, is soluble in water whereas
s-BLOCK ELEMENTS (ALKALI AND ALKALINE EARTH METALS)

Magnesium have maximum tendency to form Metal oxide BeO MeO CaO SrO- BaO
complexes. This is due to their small size and higher
AH? (kJ mol-!) 550 590 623 590 £545
charge density. For example, beryllium forms
complexes of the type [BeF.,]* Among the oxides, BeO is amphoteric while oxides
BeF, + 2F- ——> [BeF,]” of other elements are basic in nature.
Chlorophyll is an important complex of BeO MgO CaO, SrO, BaO
magnesium. Amphoteric Weakly basic Basic
8. Reducing power. The amphoteric character of BeO is supported by
Like alkali metals, the alkaline earth metals are the fact that it reacts with acids as well as alkalies.
strong reducing agents. This is indicated by large BeO + 2HCl —__> BeCl, + H5O (Basicnature)
negative values of their reduction potentials (Table 4). BeO + 2NaOH ——-> Na,BeO, + H,O (Acidic nature)
Beryllium has less negative value as compared to Sod. beryllate
other alkaline earth metals. However, its reducing
The decrease tn basic character of oxides from Be
nature is due to large hydration enthalpy associated
to Ba is because of decrease in polarizing power with
with the small size of Be** ion and relatively large
increase in Lonic size.
value of the enthalpy of atomization of the metal.
Magnesium cation is more easily reduced than the The oxides of Be and Mg (BeO, MgO) are almost
other heavier cations of the group because its small insoluble in water while the oxides of rest of the metals
size leads to a relatively large negative value for its dissolve in water to from hydroxides.
hydration enthalpy. CaO +H,O -——Y+> Ca(OH), + Heat
GENERAL CHARACTERISTICS OF The insolubility of BeO and MgO in water is due
to their large lattice enthalpies.
COMPOUNDS OF ALKALINE EARTH METALS
All these oxides except BeO react with water to
The alkaline earth metals form compounds having
form sparingly soluble hydroxides :
dipositive ions, MX,. Due to the increased nuclear
charge and smaller ionic size, the alkaline earth metals MO+H,O ——> M(OH),
are expected to form less ionic compounds than the The solubility, thermal stability and the basic
corresponding alkali metals. The tendency to form ionic character of these hydroxides increase with increasing
compounds increases as we move down the group. For atomic number from Mg(OH), to Ba(OH),.
example, the oxides and other compounds of Be and Mg The basic character of hydroxides is due to the
are more covalent than those formed by the heavier and low ionisation enthalpies of these metals. Because of
the large sized members (Ca, Sr, Ba). low ionisation enthalpies, the M—O bond in MOH is
weak and it can cleave to give OH” ions in solution.
The general characteristics of some compounds of
Since, the ionisation enthalpy decreases down the
alkaline earth metals are discussed below :
family, therefore, the basic strength of the hydroxides
1. Oxides and hydroxides increases. For example, Be(OH), is amphoteric,
All the alkaline earth metals form oxides of the Mg(OH), is mildly basic while others are strong bases.
formula MO (Mis alkaline earth metal). These can be Beryllium hydroxide is amphoteric in nature
prepared by reacting the metal with oxygen : because it reacts with acids as well as alkalies.
2M +O, —““> 2MO. (M=Be, Mg, Ca) Be(OH), + 2HCl + 2H,O —-> [Be(OH),]Cl,
(Monoxide) Be(OH), + 20H-—- [Be(OH),]*
These monoxides can be best prepared by the Beryllate ion
calcination of their carbonates. Hydroxides of Alkali and Alkaline earth metals
MCO, —— +> MO + CO,
LiOH Be(OH),
(M = Be, Mg, Ca, Sr or Ba) Amphoteric
The decomposition temperatures for these
carbonates are :
NaOH Mg(OH), :
Mild base 52
BeCO, MgCO, CaCO, SrCO, BaCO, KOH Ca(OH), a
Decomposition Strong base FS pS
temperature 523 K 813K 1173 K 1562 K 1633 K RbOH Sr(OH), q=
Strong base eal
All these alkaline earth metal oxides except BeO CsOH Ba(OH),
have rock salt (NaCl type) structure. The BeO is Very strong base
essentially covalent in nature. The enthalpies of
All alkali metal hydroxides are strong bases
formation of these oxides are very high and therefore,
they are very stable. BASIC CHARACTER DECREASES
T0725. MODERN'S abc + OF CHEMISTRY-XI

The hydroxides of alkaline earth metals are less (vit) Being covalent BeCl, is soluble in organic
basic than alkali metals of the corresponding periods. solvents. Of the other halides MgBr, and Mgl, are
The lesser basic strength of hydroxides of alkaline soluble in organic solvents.
earth metal hydroxides is due to their (iv) The anhydrous halides of alkaline earth
(1) high ionisation enthalpies metals are hygroscopic i.e., they absorb water
molecules and form hydrates such as BeCl, .4H,O,
(it) small tonic size and
MgCl, . 6H,0, CaCl, . 6H,O. The tendency to form
(ut) dipositive charge on the tons. halide hydrates gradually decreases down the group
As aresult, the M—O bond in these hydroxides is e.g., MgCl,.8H,O, CaCl, . 6H,0, SrCl, . 8H,O and
relatively stronger than that of corresponding alkali BaCl, . 2H,O. Because of this property, anhydrous
metals and therefore, does not break. Therfore, they calcium chloride is used as a dehydrating agent. BeCl,
are less basic than corresponding alkali metals. gets hydrolysed by moisture and, therefore, gives
The hydroxides of alkaline earth metals are less fumes of hydrochloric acid.
soluble than alkali metal hydroxides. However, the BeCl, + 2H,Q,» ———» Be(OH), + 2HCl
solubility of hydroxides in water increases with increase The hydrated chlorides, bromides and iodides can
in size of the metal. Be(OH), and Mg(OH), are almost be dehydrated by heating but those of magnesium and
insoluble, Ca(OH), is sparingly soluble while Sr(OH), beryllium get hydrolysed.
and Ba(OH), are fairly soluble. This trend is due to (v) The fluorides of alkaline earth metals are
increase 1n size of the cation on moving down the group sparingly soluble in water. The solubility increases
so that their lattice enthalpies decrease. Due to decrease slightly with increase in cation size.
in lattice enthalpies, the hydration enthalpies become (vi) Except for BeCl, and MgCl,, the chlorides of
more and more and the solubility of hydroxides in the other members impart characteristic flame colours.
water increases. CaCl, SrCl, BaCl,
2. Halides Brick red Crimson Grassy green
The halides of alkaline earth metals can be Be and Mg chlorides do not give flame colourations
obtained either by directly reacting with halogens or because of their high ionisation enthalpies. The energy of
by the action of halogen acids on metals, their oxides, bunsen flame is not sufficient to excite the outermost electron
carbonates or hydroxides. in Be and Mg (already discussed). Of all the fluorides of
alkaline earth metals, CaF, is very important because
M+X, ——> MX, (X =AGE,, Br.) fluorspar (CaF,) is the only large scale source of fluorine.
MO + 2HX ——-> MX,+H,0O
M(OH), + 2HX ———> MX, + 2H,O Beryllium chloride.
The important characteristics of halides of It is prepared by heating beryllium oxide with
alkaline earth metals are: chlorine vapours in the presence of carbon.
(i) Beryllium halides are covalent while all other BeO+CL+cC ——“*, Bec, +co
halides are ionic and are readily soluble in water. BeCl, can also be prepared by direct high
(ii) BeCl, is relatively low melting solid and temperature chlorination of metallic beryllium or Be,C.
volatile while others have high melting points. BaCl,
has very high melting point.

Fig. 5. Polymeric structure of BeCl.,,.

ON _

Cl——Be ~~ . 2 1 Cl—_Be—_C
Monomer

Dimer
Fig. 6 Dimer (vapour phase) and monomer of BeC].,.
s-BLOCK ELEMENTS (ALKALI AND ALKALINE EARTH METALS)

Structure of BeCl.,,. In the solid state, beryllium is slightly soluble in water while BaCO, is completely
chloride has a polymeric chain structure as shown insoluble.
in Fig. 5. However, all these carbonates become more soluble
In this case, each Be atom is terahedrally in the presence of carbon dioxide in water due to the
surrounded by four Cl atoms. Two of the chlorine formation of bicarbonate ion (HCO,-). But
atoms are bonded by covalent bonds while the other bicarbonates have not been isolated in the pure state.
two by coordinate bonds. The polymeric structure of (b) Sulphates. The sulphates of alkaline earth
BeCl, is due to its electron deficient nature. It has only metals are all white solids and stable to heat. These
four electrons in the valence shell and can accept two can be formed by the action of dil H,SO, on metals,
electron pairs (from neighbouring Cl atoms forming metal oxides, metal hydroxides or metal carbonates.
coordinate bonds) to complete its octet. Ca+H,SO, —-> CaSO, +H,
In the vapour phase, BeCl, exists as dimer
CaO + H,SO, — > CaSO, + H,O
(Fig. 6 )which dissociates into the linear monomer at
Beryllium, magnesium and calcium sulphates
1200 K.
crystallize in hydrated form as : BeSO,. 4H,0, MgSO,.
3. Salts of oxo acids
7H,O, CaSO,. 2H,O. The sulphates of strontium and
Alkaline earth metals also form salts of oxoacids. barium crystallize without water.
Some of these are :
The sulphates of alkaline earth metals are less
(a) Carbonates. The carbonates of alkaline earth soluble than the corresponding salts of alkali metals.
metals can be prepared by passing carbon dioxide in BeSO, and MgSO, are readily soluble, and their
limited supply through the solutions of their hydroxides. solubilities decrease on going down the group. This is
Ca(OH),+CO, —+ CaCO,+ H,O due to higher lattice enthalpies of alkaline earth metal
The alkaline earth metal carbonates being sulphates than alkali metal sulphates (as already
insoluble in water can be precipitated by the addition explained). On moving down the group, the hydration
of a sodium or ammonium carbonate solution to a enthalpy decreases with increase in size of the metal
solution of soluble salt of these metals. ion and consequently their solubilities decrease. The
Most beryllium salts of strong oxoacids crystallize greater hydration enthalpies of Be** and Mg** ions
as soluble hydrates. Beryllium carbonate is very prone overcome the lattice enthalpy factor and therefore,
to hydrolysis and can be precipitated only in the their sulphates are soluble.
atmosphere of carbon dioxide. (c) Oxalates. The oxalates of Ca, Sr and Ba are
All carbonates decompose on heating to give sparingly soluble and solubility increases from Ca to
carbon dioxide and metal oxide. Ba. Beryllium oxalate is highly soluble.
Caco, —*** , CaO + CO, (d) Nitrates. The alkaline earth metals form
nitrates by dissolution of carbonates in dilute HNO,.
The stability of these
; BeCO Aa Magnesium nitrate crystallizes with six water
carbonates increases down « Ss
molecules (MgNO,.6H,O) while barium nitrate
the group. For example, Mgco (-—
crystallizes as anhydrous salt. All these nitrates
BeCO, is least stable while ° <7
decompose on heating to give oxide.
BaCoO, gist
is the most stable. The y Wal, on S
solubilities of the metal z= 2M(NO,), —Heat_, 2MO + 4NO, +0,
carbonates in water SrCO aa (M = Be, Mg, Ca, Sr, Ba)
decreases down. the ° = Strontium and barium nitrates are used in
family. For example, BeCO, BaCO, = pyrotechnics for giving red and green flames.

Sparingly soluble salts Soluble salts

Mg decrease in increase in solubility is


solubility is due to due to decrease in
wo decrease in hydra- lattice enthalpy.
o
a
4s)
tion enthalpy.
ah)
Oo

o
=
Decrease
S T03 Solubility and thermal stability of oxo salts
MODERN'S abc + OF CHEMISTRY-iI

() Beryllium is harder than magnesium.


As already studied the solubility of salts depends (ii) It possesses higher boiling and melting points
upon two factors : as compared to magnesium.
(i) Lattice enthalpy of the salt to break into ions. (ui) Beryllium does not react with water even at
(ii) Hydration enthalpy released when ions are elevated temperatures. On the other hand, magnesium
hydrated. reacts with boiling water.
The carbonates and sulphates of alkaline earth Mg + 2H,0 ——-> Mg(OH), + H,
metals are less soluble than corresponding alkali metal (iv) Beryllium does not combine with hydrogen
salts. Their solubility decreases down the group due directly to form hydride whereas magnesium does.
to decrease in the hydration enthalpy. For example, (v) Because ofhigh ionisation enthalpy and small
MgCO, is slightly soluble in water whereas BaCO, is size, beryllium forms covalent compounds while
almost insoluble. Similarly, magnesium sulphate is magnesium forms ionic compounds. Because of covalent
soluble in water, calcium sulphate is slightly soluble character, salts of beryllium are easily hydrolysed. For
while SrSO, and BaSO, are insoluble. example,
Among the hydroxides there is increase in BeCO, + 4H,0 ——— [Be(H,0)]**+CO,?-
solubility as we go down the group. This is due to (vi) Beryllium does not exhibit coordination
decrease in the lattice enthalpy. However, alkaline number more than four because its valence shell has
earth metal nitrates do not show regular order. The only four orbitals. The other members of the group can
solubility increases from Mg to Ca and then decreases have coordination number of six by making use of d-—
from Sr to Ba. orbitals. Beryllium carbide with water gives methane
Thermal stability of the salts depend upon their while magnesium carbide gives acetylene.
lattice enthalpy. It depends upon two factors : Be,C + 2H,0 ——-> 2BeO + CH,
(i) The smaller the ions, the more closely they can Methane
approach in the lattice and the higher is the lattice MgC, + 2H,0 ——+ Mg(OH), + C,H,
enthalpy. Acetylene
(it) The greater the charges on the ions, the larger (vii) The oxide and hydroxide of beryllium are
is the attraction between them and higher is the lattice amphoteric while those of other elements in the group
enthalpy. are basic.
For example, the carbonates of alkaline earth
DIAGONAL SIMILARITIES OF
metals are relatively less stable towards heat and BERYLLIUM AND ALUMINIUM
decompose to CO,. However, their stability increases Beryllium shows some similarities with aluminium
as we move down the group. The temperatures at due to diagonal relationship. Some common examples
which these carbonates decompose are are :
({) Both Be and Al have same electronegativity
BeCO, MgCO, CaCO, Srco, BaCo, values (Be=1.5, Al=1.5) and their charge/radius ratios
<373K 813K 1173K 1563K 1635 K (Be = 0.064 and Al = 0.060) are similar indicating
similar field strengths.
Thermal stability increases
(ii) Both Be and Al form covalent compounds.
(ui) Beryllium and aluminium are resistant to
Nitrates and hydroxides also decompose to
the action of acids due to the formation of a protective
corresponding oxides. The O* ion is more highly
film of oxide on the surface.
charged than the NO, and OH. Therefore, E?*O? have
(tv) Both form soluble complexes by action of
high lattice enthalpies than nitrates and hydroxides.
alkalies such as beryllates [Be(OH),]*" and aluminates
2Mg (NO,), (s) ———_> 2Mg0O (s) + 4NO, (g) + O, (g) [Al(OH),1-.
Mg (OH), (s) ———> MgO (s) + H,O (/) (v) BeO and Al,O, are amphoteric 1.e., they
dissolve in acids as well as in bases.
DIFFERENCES BETWEEN BERYLLIUM
BeO + 2HCl ———> BeCl, + H,O
AND MAGNESIUM
BeO + 2NaOH ——-+ Na,BeO, + H,O
Beryllium, the first member of the alkaline earth Sod. beryllate
metals family shows anomalous behaviour as Al,O, + 6HCl1 ——> 2A1Cl1, + 3H,O
compared to magnesium and the rest of the members. Al,O, + 2NaOH ——> 2NaAlO, + H,O
It is mainly because of the following reasons : Sodium meta-aluminate
(tL) small size of the atom or its ton (vi) The carbides of both B and Al liberate methane
(uw) highiontsation enthalpy and electronegativity with water.
(tut) absence of d-orbitals. Be,C +2H,O ———> 2BeO+CH,
Some of the important differences between Al,C,+6H,O ——> 2Al,0,+3CH,
beryllium and magnesium are given below :
s-BLOCK ELEMENTS (ALKALI AND ALKALINE EARTH METALS)

(vit) Both the metals are weakly electropositive (iv) Which element in the group 2 1s radioactive ?
in nature. (v) Which out of Mg**, Ba?*, Ca?* has maximum ionic
(viii) Beryllium and aluminium have strong mobility in water ?
tendency to form complexes. They form fluoro complex Answers to Practice Problems
anions, BeF ,?- and AIF,,* in solution. The other group
©ll. (i)Be (ii)Ca (it) Be (iv) Be.
2 metals do not form stable fluoro complexes in solution.
©12. [E,:Na<Mg;lIE,:Na>Mg
(ix) Beryllium dissolves in alkalies to give beryllate
ion [Be(OH),]*- while aluminium dissolves to give © 13. (1)BeCl,, MgCl,, CaCl,, BaCl,
(it) Mg (OH),, Ca (OH),, Sr(OH),, Ba(OH),
[Al(OH),]*-.
(1it)BeO, MgO, CaO, BaO
(x) BeCl, like Al,Cl, has a bridged polymeric (1v)Mg(OH),, Ca(OH),, Ba(OH),
structure in vapour phase as shown below. © 14. (i)CaH, (ii) Be,Mg (ii)NaOH (iv) Radium
Cl (v) Ba**.
=
Cl—Be
ae Be—Cl USES OF ALKALINE EARTH METALS
The important uses of alkaline earth metals are
summarized below :
Uses of beryllium
Sa San
Cl Cl
The important uses of beryllium are :
(i) Beryllium is used in the manufacture of many
a ee ie alloys. For example, copper—beryllium alloys
are used in the preparation of high strength
Both the chlorides are soluble in organic solvents springs.
and are strong Lewis acids. They are used as Friedel (it) Metallicberyllium is used for making windows
Craft catalysts. of X-ray tubes.
(xt) Salts of these metals form hydrated ions e.g., Uses of magnesium
[Be(H,O),]?* and [Al(H,O),]** in aqueous solutions. The important uses of magnesium are :
(xii) Because of similarity in charge/size ratios both
(.) Magnesium is the lightest constructional
have strong tendency to form complexes. For example, metal in industry having a density less than
Be forms tetrahedral complexes such as BeF f° and two thirds of aluminium.
[Be(C,O oe and Al forms octahedral complexes like
(it) Magnesium is an important alloying metal.
AIF? and [Al(C,0,)3]*-. Some of the alloys of magnesium contain more
(xiii) Halides of both beryllium and aluminium than 90% Mg and 2—9% Al and 1% Zn. These
have similar solubilities. alloys are used in automobile engines, making
balances and in aircraft constructions. Upto
Q@ 11. Which member of the alkaline earth metals family 5% magnesium is added to most commercial
has aluminium alloys (e.g. duralumin, magnalium)
(i) least reactivity (ii) lowest density to improve its resistance to corrosion,
(iii) highest boiling point (iv) maximum reduction mechanical properties and weldability.
potential? (iz) Magnesium is used for cathodic protection of
Q 12. Compare the first and second ionisation enthalpies other metals as oxygen scavenger.
of sodium and magnesium. (iv) It is used as a reducing agent in metallurgy
Q 13. Arrange the following in the order of property for the production of certain metals such as
mentioned : Ti, Zr and Hf.
(i) BaCl,, MgCl,, BeCl,, CaCl, (v) Magnesium powder is used in flash powders
increasing ionic character and bulbs, incendiary bombs and signals.
(it) Mg(OH),, Sr(OH),, Ba(OH),, Ca(OH), (vi) Magnesium is used for the ignition of thermite
increasing solubility in water
charge in aluminothermy.
(iii) BeO, MgO, BaO, CaO
increasing basic character (vit) In the form of Grignard reagent, which is
(iv) Mg(OH),, Ba(OH),, Ca(OH), alkyl magnesium halide (RMgX), magnesium
increasing basic character is useful in preparing a number of organic
Q 14. Answer the following : compounds.
(i) What is hydrolith ? (vit) The compounds of magnesium such as
(ii) Which elements of alkaline earth metals family magnesium hydroxide and magnesium
do not give characteristic flame colouration ? carbonate are used in making tooth pastes.
(iii) Which out of NaOH or Mg(OH), is stronger (ix) Magnesium hydroxide (milk of magnesia) is
base ? used as an antacid to neutralise excess of acid
in stomach.
Too (x) Magnesium is used as a deoxidizer for
MODERN'S abe + OF CHEMISTRY-AI

The reaction with water is highly exothermic :


removing last traces of oxygen from copper, CaO (s) + H,O (1) —> Ca(OH), (aq)
steel, etc. ; AH = —- 64.5 kJ mol
Uses of calcium (iii) Itis usually obtained in the form of hard lumps.
The important uses of calcium are : When heated in oxyhydrogen flame, it emits a brillant
white ight known as lime light.
(i) Calcium is a powerful reducing agent and
(tv) The addition of limited amount of water breaks
therefore, it is used for the extraction of
the lump of lime. This process is called slaking of
metals from their oxides in metallurgy.
the lime and the product is called slaked lime.
(ii) Itis used to remove traces of air from vacuum Quicklime slaked with soda gives solid, sodalime.
tubes since it has strong affinity for both (v) It combines with acidic oxides at high
nitrogen and oxygen. temperatures :
(iii) It is used to remove last traces of moisture CaO + S10, ——> CaSi0,
from alcohol. 6CaO + P,O,, ——~ 2Ca, (PO,),
(tv) Compounds of calcium such as limestone and Uses of calcium oxide
gypsum are used as constituents of cement
(i) Itisanimportant primary material and is the
and mortar. cheapest form of alkali.
(v) When alloyed with lead, it is used for cable (ii) It is used in the manufacture of sodium
coverings. When alloyed with aluminium, it carbonate from caustic soda.
is used in casting and forging. (iii) It is used as a flux in metallurgy.
Radium salts are used in radiotherapy such (iv) It is used in the purification of sugar and in
as for treatment of cancer. the manufacture of dye stuffs.
SOME IMPORTANT COMPOUNDS OF CALCIUM (v) It is used in drying gases and alcohol.
Alkaline earth metals form many compounds. The (vi) Calcium oxide is used as a building material.
compounds of calcium are industrially important 2. Calcium hydroxide or Slaked lime, Ca(OH),
compounds. Some of these are discussed below : Preparation
1. Calcium oxide or Quicklime, CaO (i) From quicklime. Calcium hydroxide is
Preparation prepared by adding water to quicklime, CaO. This
It is prepared on a commercial scale by heating process is called slaking of lime.
limestone (CaCO,) in a special type of rotary kiln at CaO +H,O ——+ Ca(OH),
1070 — 1270K. (it) From calcium chloride. It can also be
CaCO, Heat
Ca0+CO,, AH=179.9kJ obtained by treating caustic soda with calcium chloride.
CaCl, + 2NaOH——-+ Ca(OH), + 2NaCl
The necessary conditions for the good yield of quick
Properties
lime are :
(t) Itis a white amorphous powder and is sparingly
(z) Since the reaction is reversible, therefore, the
carbon dioxide formed must be removed as soon as it soluble in water. The aqueous solution of calcium
is formed so that the reaction remains in the forward hydroxide is called lime water. The suspension of
direction. In other words, it will not make the backward slaked lime in water is called milk of lime.
reaction possible. It is only above 1100 K that the gas (it) It is strongly basic and therefore, reacts with
escapes freely and therefore, a temperature of 1273 K acids and acidic gases forming salts.
is Maintained. (iii) It reacts with chlorine to form bleaching powder.
(it) The temperature should not be allowed to rise Ca(OH), +Cl, ———> CaOCl, + H,O
above 1273 K otherwise silica (Si0,) present as Bleaching powder
impurity in limestone will react with calcium oxide to (iv) When carbon dioxide is passed through lime
form calcium silicate. water, it becomes milky due to formation of calcium
carbonate.
CaO + SiO, —
Above 1273 K
CaSiO, Ca(OH), + CO, ———> CaCO, + H,O
Calcium silicate Lime water Milky
Properties The milkiness disappears on passing the gas in
(4) Itis a white amorphous solid having melting excess because calcium carbonate changes to a soluble
point about 2870 K. bicarbonate.
(it) On exposure to atmosphere, it absorbs CaCO, + CO,+H,O ———>+ Ca(HCO,),
moisture and carbon dioxide. (Soluble)

Ca0+H,O ——> Ca(OH), Milkiness disappears

CaO +CO, ——> CaCO, (v) It dissolves in dilute hydrochloric acid.


Ca(OH), + 2HCl ———> CaCl, + 2H,O
s-BLOCK ELEMENTS (ALKALI AND ALKALINE EARTH METALS)

(vt) Milk of lime reacts with chlorine to form (vi) Itis also used as an antacid, mild abrasive in
hypochlorite, which is a constituent of bleaching powder. tooth paste, a constituent of chewing gum and
2Ca(OH), + 2CL, ———> CaCl, + Ca(ClO), + 2H,O a filler in cosmetics.
However, it is not soluble in dilute sulphuric acid 4, Plaster of Paris,
because calcium sulphate formed is water insoluble salt. (CaSO,), . H,O or CaSO,. —H,0
Uses of calcium hydroxide
Preparation
(z) Calcium hydroxide is used in the preparation
It is prepared by heating gypsum to 393 K.
of mortar, a building material.
(ii) Itisused as a whitewash due toits disinfectant 2(CaSO,.2H,O) —**-, (CaSO,),-H,O + 3H,O
nature. Thus, it contains one molecule of water for every
(iit) It is used for the softening of hard water. two calcium and two sulphate ions. It may be noted
(tv) Itis used for the detection of carbon dioxide as that the temperature should not be allowed to rise
lime water. because at 437K the whole of water is lost and the
anhydrous salt left is called dead burnt plaster. It has
(v) It is used for absorbing acidic gases.
no setting properties.
(vi) It is used for preparing ammonia from
Properties
ammonium chloride.
(¢) It is a white powder.
(vit) It is used in glass making, tanning industry,
(it) On mixing with water, it forms a plastic mass
for preparation of bleaching powder and for
which sets into a hard solid in 5-15 minutes. This is
purification of sugar.
called setting of plaster of paris. The setting is due
3. Calcium carbonate or Limestone or to hydration of plaster of paris into gypsum.
Marble, CaCQ,. (CaSO,),.H,0+3H,0 ——> 2CaSO,.2H,O
Calcium carbonate occurs in nature alongwith Gypsum
magnesium carbonate. During setting, there is a slight increase in volume.
Preparation This helps plaster of paris to take the shape of any
(1) From slaked lime. It can be prepared by mould in which it is added.
passing carbon dioxide through slaked lime in limited Uses of plaster of paris
amount. (i) Plaster of paris is used for producing moulds
Ca(OH), + CO, ———> CaCO, + H,O for pottery, ceramics, etc.
It may be noted that excess of carbon dioxide (ii) It is used for making statues, models and
should be avoided because this leads to the formation other decorative materials.
of water soluble calcium hydrogen carbonate. (iii) It is used in the building industry as well as
plasters.
(it) From calcium chloride. It can be prepared
(iv) It is used in surgical bandages known as
by adding aqueous solution of sodium carbonate to
plasters for setting broken and fractured bones
calcium chloride.
in the body because it immoblises the affected
Na,CO, + CaCl, ———> CaCO, + 2NaCl part of the organ where there is a bone
Properties fracture or sprain.
(i) Itisawhite fluffy solid andis almost insoluble (v) Itis also used in dentistry.
in water.
(i) When heated to 1200 K, it decomposes to ; SOLVED EXAMPLES
evolve carbon dioxide. L) Example 11.
Caco, Aa ®, CO +0, The enthalpy of formation of hypothetical CaCl (s)
(iit) It reacts with dilute acids to liberate carbon theoretically found to be —188 kJ mol! and AT °
for CaCl, (s) is -795 kJ mol. Calculate AH” for
dioxide.
the disproportion reaction :
CaCO, + 2HC]l ——+> CaCl,+H,O +CO, 2CaCl (s) —> CaCl, (s) + Ca (s)
CaCO, +H,SO, —— CaSO, + H,O + CO, Solution: The given equations are :
Uses of calcium carbonate (i) Ca(s) + CL, (g) —+ CaCl, (s)
(4) Calcium carbonate is used as a building A” =— 795 kJ mol"!
material in the form of marble.
(iz) Ca(s)+ =Cl, (g) — > CaCl (s)
(it) It is used in the manufacture of quicklime.
(iii) It is also used as a raw material for the A,H° =- 188 kJ mol
(uit) or 2Ca(s)+Cl,(g) —~> 2CaCl (s)
manufacture of sodium carbonate in Solvay
AH® =—376 kJ mol
process.
Subtracting (iii) from (i)
(itv) It is used as a flux in the extraction of metals A H° for disproportion reaction is
such as iron. 2CaCl (s) —> CaCl, (s) + Ca(s)
(v) Specially precipitated calcium carbonate is AH° =-795 —(-376)
extensively used in the manufacture of high = -419 kJ
quality paper.
Toe
L) Example 12
MODERN'S abe + OF CHEMISTRY-AI

Solution: In carbonates and sulphates, the size of the


What happens when anion is much larger as compared to cations and the lattice
(i) Magnesium ts burnt in air enthalpy remains almost constant within a particular group.
Since with the increase in size of the cation down the group,
(i) Quicklime is heated with silica
the hydration enthalpies decrease, the solubility will also
(ui) Calcium nitrate is heated decrease. Therefore, solubility of alkaline earth metal
(iv) Chlorine reacts with slaked lime carbonates and sulphates decreases down the group.

Solution: (i) When magnesium is burnt in alr, it gives Why does solubility of alkaline earth metal
brilliant flame forming magnesium oxide and hydroxides increase down the group?
magnesium nitride.
2Mg+O, —~> 2MgO
Solution: If the cation and anion are of comparable
3Mg+N, ——> Mg.N,
size, the anion being common, the radius of cation will
(it) When quicklime is heated with silica, it gives calclum influence the lattice enthalpy. In the group, both the lattice
silicate. enthalpy and hydration enthalpy decrease with increase in
CaO + SiO, 2" ¥E* , CaSiO, size of the cation. But the decrease in lattice enthalpy is
much more than the hydration enthalpy with increasing ionic
Cal. silicate
size and hence, solubilty will increase as we go down the
(zit) When calcium nitrate is heated, it gives CaO, NO, group. Therefore, the solubility of alkaline earth metal
and Q,. hydroxides increases down the group.
2Ca(NO,), — > 2Ca0+4NO, +0, LJ) Example 18
(iv) Chlorine reacts with slaked lime to form calcium When water is added to compound (A) of calcium,
hypochlorite, Ca(OCl), which is a constituent of solution of compound (B) is formed. When carbon
bleaching powder. dioxide is passed into the solution, it turns milky
2Ca(OH), + 2Cl, —> CaCl, +Ca(OCl), + 2H,O due to the formation of compound (C). If excess of
_—.. SS —.
carbon dioxide is passed into the solution
Bleaching powder milkiness disappears due to the formation of
LU) Example 138 compound (D). Identify the compounds A, B, C
Arrange the following in the decreasing order of and D. Explain why the milkiness disappears in
the property mentioned : the last step. (NCERT Exemplar Problem)
(i) Lit, Nat, K+, Rb+ onic mobility) Solution: The compound A is calcium oxide, CaO
(i) Be, Mg, Ca, Sr (Melting point) and B is calcium hydroxide, Ca(OH),.
(ut) BeO, MgO, CaO (Enthalpy of formation) The compound C is calcium carbonate, CaCO, and
(iv) Be, Mg, Ca (Metallic radius) the compound D is calcium bicarbonate, Ca(HCO,),.
Solution: Calcium bicarbonate Ca(HCO,), is soluble in water and
(i) Rb*>K*>Nat> Lr (ii) Be>Ca>Sr> Mg therefore, milkiness of solution disappears on passing
(iii) CaQ>MgO> BeO (iv) Ca>Mg > Be excess of carbon dioxide into the solution.
LI Example 14
Ca(OH), + CO, —___+ CaCO, + H,O
Name one reagent or one operation to distinguish
(Milkiness)
between :
() BeSO,and BaSO, — (w) Be(OF), and Ba(OH), CaCO, + CO, + H,O —__+ Ca(HCO,),
Solution: (i) BeSO, is soluble in water while BaSO, is soluble
insoluble. (Milkiness disappears)
(ii) Be(OH), dissolves in NaOH while Ba(OH), is
insoluble.
LJ Example 15
Complete the following equations for the reaction @ 15. What is (i) dolomite (ii) milk of lime ?
between : @ 16. Name any two ores of magnesium.
i) Ca+H,O (ii) Ca(OH), + Cl, O17. What is dead burnt plaster ?
(iii) BeO+NaOH (iv) BaO, + H,SO, Q@ 18. What is the formulae of
Solution: (i) Plaster of Paris (ii) Gypsum ?
(i) Ca + 2H,O Heat, Ca(OH), + H, Q19. What is magnesia ?
(iz) Ca(OH), + Cl, ———+ CaOCl, + H,O @ 20. Why does a tube containing lime water turns milky
Bleaching powder when exhaling is made into it ?
(tit) BeO + 2NaOQH ——-+> Na,BeO, + H,O @ 21. Name three forms of calcium carbonate.
Sodium Beryllate Q@ 22. What happens when gypsum is heated to 390 K?
(iv) BaO,+H,SO, ———> BasSO,+H,0,.
LI Example 16
@ 23. What happens when carbon dioxide 1s passed through
lime water ?
Why does the solubility of alkaline earth metal
@ 24. Element A burns in nitrogen to give anioniccompound
carbonates and sulphates decrease down the
B. Compound B reacts with water to give C and D. A
group? solution of C becomes milky on bubbling carbon
dioxide. Identify A, B, C and D.
s-BLOCK ELEMENTS (ALKALI AND ALKALINE EARTH METALS)

Answers to Practice Problems Mg(HCO,), + Ca(OH), ——> MgCO, | +CaCo, |


© 15. (i)CaCO,.MgCoO, (ii) Suspension of calctum hydroxide
In water.
+ 2H,O
© 16. Magnesite (MgCO,), Dolomite (MgCO, . CaCO,) MgCO, + Ca(OH),—> Mg(OH), | + CaCO, J , ete.
©17. Itis anhydrous calcium sulphate (CaSO,).
© 18. (i) (CaSO,), . H,0 (ii) CaSO,. 2H,O (v) Lime is used in scrubbers to remove SO, and
© 19. Magnesium oxide (MgO) H,S from stack gases of fossil fuels in power generating
© 20. Due to the formation of calcium carbonate. stations and metallurgical smelters.
© 21. Limestone, chalk, marble. (vit) Lime is used in chemical industries such as
© 22. Plaster of paris is formed as manufacture of calcium carbide (for obtaining
2CaSO,.2H,O 39K , (CaSO,),.H,0+3H,O acetylene), cyanamide and other chemicals.
© 23. Milkiness appears (due to CaCO,) which disappears (vit) Lime is also used in the manufacture of glass.
on passing excess of CO, [(due to the formation of (viit) Lime is used for manufacture of insecticides,
Ca(HCO,),]
calcium arsenate and fungicides such as Bordeaux
© 24.
mixtures [CuSO,/Ca(OH),].
3Ca+N, —> Ca,N,—%° . 3Ca(OH), + 2NH, (ix) Paper and pulp industries consume large
(A) ae (B) (C) (D)
quantities of Ca(OH), and CaCO,. Bleaching liquor,
CAE s<n
calcium hypochlorite is obtained by reacting lime and
(C)
chlorine. It is used for bleaching paper pulp.
INDUSTRIAL USES OF LIMESTONE AND LIME Specially precipitated CaCO, is extensively used in
Limestone rock is the common form of calcium the manufacture of high quality paper. This is obtained
carbonate, which also occurs as chalk, marble, calcite, by calcining limestone and collecting the carbon dioxide
argonite, etc. It also occurs as dolomite with and calcium oxide separately. The latter is then hydrated
magnesium. Limestone and dolomite are widely used and re-carbonated to get the desired product
as building materials on a large scale throughout the CaCO, —““> CaO + CO,
world. CaCO, is a major industrial chemical and is CaO +H,O ——+ Ca(OH),
important for the production of quicklime (CaO) and Ca(OH), + CO, ———> CaCO, +H,O
slaked lime [Ca(OH),]. These chemicals have wide (x) CaCO, has domestic and pharmaceutical
applications in chemical, metallurgical and applications. Some of these are : as an antacid, mild
construction industry. Some of the important abrasive in tooth paste, a constituent of chewing gum,
industrial uses of lime and limestone are given below: source ofcalcium enrichment in diets, a fillerin cosmetics.
(i) Lime is an important constituent of Portland (xi) Lime also finds applications in dairy industry.
cement which is obtained by roasting limestone and
CEMENT*
sand with clay.
Cement is one of the most important building
(it) Large quantities of lime are used in the steel
materials. It was first introduced in England in 1824
industry, where it is used as a flux to remove P, §, Si,
by Joseph Aspdin. It is also called Portland cement
Mn, etc. Lime is also used as a lubricant in steel wire
because it resembles with the famous building stone
drawing and in neutralising wastes.
found near Portland in England. In India, the cement
(iit) It has also been used in the production of
industry started in 1914. Now Indiais producing above
magnesium in Dow’s process.
100 million tonnes of cement annually (1999-2000).
(iv) Lime is used in the chemical treatment of Chemically, it is a finely ground mixture of calcium
potable and industrial water supplies. It is also used
silicates and aluminates which set to a hard mass
with iron or alum to coagulate suspended solids and
when treated with water.
remove turbidity. It is also commonly used in water
Composition of Cement
softening to remove temporary hardness. The common The average composition of cement is
reactions occurring during water softening are : CaO = 50 — 60% MgO = 2—3%
S10, = 20—25% Fe,O, = 1—2%
Ca(HCO,), + Ca(OH), ——> 2CaCO, | +2H,O Al,O, = 5—10% SO, = 1—2%

* Not in the syllabus of CBSE


MODERN'S abc + OF CHEMISTRY
-XI

\ } Charge

screw conveyer
Rotary furnace Burner
oal dust hopper

am
[=e q— Air blower
al 9 a

Dust chamber

Fig. 7. Manufacture of cement.

For a good quality cement the ratio of silica (Si10,) is finely powdered and the powder is called Portland
and alumina (Al,O,) should be between 2.5 to 4.0. cement. The gypsum slows down the setting of cement
Similarly, the ratio of lime (CaO) to the total oxide so that its gets sufficiently hardened.
mixtures consisting of Si0,, Al,O, and Fe,O, should be
The important compounds present in Portland
roughly 2: 1. If lime is in excess, the cement cracks
cement are :
during setting. On the other hand, if lime is less than
the required, the cementis weak in strength. Therefore, Dicalcium silicate : (Ca,SiO,) : 26%
a proper composition of cement must be maintained to Tricalcium silicate (Ca,Si0,.) : 51%
get cement of good quality. Tricalctum aluminate (Ca,Al,O,) : 11%
Manufacture of Cement Setting of cement. When cement is to be used,
The main raw materials for the manufacture are it is always mixed with water. It is an exothermic
limestone and clay. The former supplies lime while reaction and the cement reacts with water to form a
the latter is the source of silica, alumina and iron oxide. gvelatinous mass which slowly sets into a hard mass
The process involves the following steps : having —Si—O—Si— and—Si—O—A]— chains.
(.) The powdered limestone (3 parts) and clay Uses. Cement is used as an important construc-
(1 part) are mixed. tional material. It is used in concrete and reinforced
concrete, in plastering and in construction of buildings,
(ii) The mixture is heated at about 1770-1870 Kin
bridges and dams.
a rotary kiln (Fig. 7). In this kiln, the temperature
varies between 1100-1800 K. In this process, the Next to iron and steel, it has become as a
following changes occur : commodity of national necessity for any country.
(_) Limestone decomposes to give lime and CO,. In the present industrial age, demand of cement
CaCO, ————> CaO +CO, is increasing day by day. Huge structures of building
cannot be erected without cement. Attempts have been
(ii) Lime (obtained from limestone), alumina and made to find a substitute of cement. One of these 1s fly
silica (constituents of clay) react together to form a ash which is a waste product from the steel industry.
mixture of calcium silicate (2CaQ. SiO,, 3CaO. Si0,) This can be added to the cement to reduce its cost but
and calcium aluminate (8CaO Al,O,, 4CaO.Al,O.). at the same time it does not degrade its quality. In
The reactions taking place are : many countries, cement is made from rice husk with
2CaO + S10, ——> 2CaO . S10, high silica contents.
Dicalcium silicate BIOLOGICAL IMPORTANCE OF
3CaO +810, ——+ 3CaO . 510, CALCIUM AND MAGNESIUM
Tricalcium silicate On an average an adult body contains about 25 g of
3CaO +Al,O, ———> 3CaO.Al,O, Mg and 1200 g of Ca as compared to only 5 g ofiron and
Tricalcium aluminate 0.06 g of copper. The daily requirement in the human
4CaO + Al,O, + Fe,O, ——> 4CaO . ALO, . Fe,O, body has been estimated to be 200-300 mg.
Magnesium and calcium play important role in
Tetracalcium aluminoferrite
biological processes. Mg** ions are concentrated in
Due to very high temperature in this zone, about
animal cells and Ca** are concentrated in body fluids
20-30% mass melts and combines with solid mass to
outside the cells in much a similar way that K*
form pebbles known as cement clinkers.
The hot clinkers are cooled and are mixed with concentrates inside the cells and Nat outside.
2 to 3 per cent by weight of gypsum (CaSO, .2H,O). It Allenzymes that utilise ATP in phosphate transfer
require magnesium as the cofactor. Mg** ions form a
s-BLOCK ELEMENTS (ALKALI AND ALKALINE EARTH METALS)

complex with ATP and are constituents of maintained by two hormones calcitonin and parathyroid
phosphohydrolases and phosphotransferases, which hormone. It is important to know that bone is not an
are enzymes for reactions involving ATP and energy inert and unchanging substance but it is continuously
release. They are very essential for the transmission of being solubilized and redeposited to the extent of 400
impulses along nerve fibres. Mg** ions are also present mg per day in man. All this calcium passes through the
in chlorophyll which is the green colouring pigment plasma. In addition to skeletal role, Ca?* ions play
present in plants. Chlorophyll absorbs light from the many biochemical roles in neuromuscular function,
sun and carries the process of photosynthesis in plants. interneuronal transmission, cell membrane integrity
Ca** ions play a significant role in skeletal blood coagulation, etc. Some of these are :
formation.With phosphorus in the mineral (i) It acts as a messenger for hormonal action.
hydroxyapatite, Ca,(PO ,), OH itis the major constituent (i) It acts as a trigger for muscular contraction.
of bones, teeth and shells. About 99% of body calcium (iit) It acts as initiator for blood clotting.
is present in bones and teeth. The calcium concentration (iv) It plays a role in the stabilization of protein
in plasma is regulated at about 100 mgL-?. It is structure.
(n) Tthelns in the maintenance of rhvthm ofheart.

Conceptuat CYUCSTIONS 342

. Mg?* is much more heavily hydrated than Na*. Why ?


The extent of hydration depends upon the charge density. Mg** 10n 1s smaller in size than Na* and has twice
the charge than that of Na*. Hence, Mg?* has higher charge density than Na* and therefore, is heavily
hydrated.
. Why are halides of beryllium polymeric ?
The halides of Be are electron deficient as their octets are incomplete. Therefore, to complete their octets,
the halides polymerize.
. What is difference between quicklime, slaked lime and lime water ?
Quicklime is CaO, slaked lime is Ca(OH), and lime water is an aqueous solution of Ca(OH),.
. What is dead burnt plaster ? How is it obtained ?
Dead burnt plaster is anhydrous CaSO,,. It is obtained by heating plaster of Paris.

CaSO,. =H,0 ie, Comet =H,0


Plaster of Paris Dead burnt plaster
. Magnesium metal burns in air to form a white ash. On treating the white ash with water, odour of
ammonia is detected. Explain.
Ans. Mg burns in air to form MgO and Mg.N,.
2Mg¢g+0O, oe 2MgO
3Mg+N, — Mg.N,
Magnesium nitride on hydrolysis given ammonia.
Mg.N, +6H,O —> 3Mg(OH), + 2NH,

5 A piece of burning magnesium continues to burn in SO, Explain.


A piece of magnesium continues to burn in SO, because it reacts with it to form MgO and 8.
2Mg+sS0, —> 2Mg0 +5
This reaction is exothermic and it keeps the piece of magnesium burning.
Q.7. BeCl, can be easily hydrolysed. Explain.
BeCl, can be easily hydrolysed because it is electron deficient compound.
BeCl, + 4H,O ee [Be(H,O),JCL,
Q.8. Arrange the alkaline earth metal carbonates in the decreasing order of thermal stability.
The decreasing order of thermal stability is
BaCO, > SrCO, > CaCO, > MgCO, > BeCO,
Q.9. Which of the following has highest solubility in each of the following:
(i) BaSO,, MgSO,, CaSO, (ii) Mg(OH),, Ba(OH),, Ca(OH),
(1) MgSO, (it) Ba(OH),
Q.10. Arrange the following sulphates of alkaline earth metals in order of decreasing thermal stability:
BeSO,, MgSO,, CaSO,, SrSO,.
srsO, > CaSO, > MgSO, > BeSO,,.
Q.11. Why does beryllium show similarities with aluminium ?
MODERN'S abc + OF CHEMISTRY-XI

Beryllium shows similarities with aluminium because of same electronegativity (1.5) and very similar
polarizing power i.e., charge/radius ratio (Be** = 2/31 = 0.064, Al** = 3/50 = 0.060) of their ions.
. The second ionization enthalpy of calcium is more than that of first and yet calcium forms CaCl,
not CaCl Why? |
The higher enthaply of lattice formation of Ca?* is more than compensates the second ealactian eubaley
required for the formation of divalent Ca** ions.
. Explain why alkali and alkaline earth metals can not be obtained by chemical reduction methods?
Alkah and alkaline earth metals are themselves very strong reducing agents and, therefore, cannot be reduced.
Beryllium and magnesium donot give colour to flame whereas other alkaline earth metals do so why?
Beryllium and magnesium atoms in comparison to other alkaline earth metals are comparatively smaller
and their ionisation enthalpies are very high. Hence, the energy of the flame is not sufficient to excite their
electrons to higher energy levels. These elements, therefore, do not give any colour in Bunsen flame.
. Mg?*is much hydrated than Na*. Why?
The extent of hydration depends upon the charge densityi.e. , charge/size. Me** ion is smaller in size than Nat ion
and has twice the charge than Na’. Therefore, 1ts charge density isi high and hence is extensively hydrated.
. Anelement of group 2 forms covalent oxide which is amphoteric in nature and dissolves in water to
give an amphoteric hydroxide. Name the element.
Beryllium.
. Namean element of group 2 which forms an amphoteric oxide and water soluble sulphate. To which
period, does it belong?
Beryllium. It belongs to second period.
. Whatis epsom salt? What is the action of heat on it?
Epsom salt is MgSO,. 7H,O.

Action of heat: MgSO,.7H,O —37-5—> MgSO,.H,O —277-4> MgSO, —He#t_, Mg0 +80,
Q.19. How will you distinguish between BaSO, and BeSO,?
Ans. BeSO, is soluble in water but BaSO, is insoluble in water.
Q.20. Which of the two: MgCO, or CaCO, is more stable?
Ans. CaCO, .

@ s-block elements. The elements in which the last electron enters the outermost s-orbital.
® Diagonal relationship. The similarity in properties of elements present diagonally in different groups.
® Polarisation. ihre gistopuor of ape electron cloud of the negative ion by positive ion.

QUICK CHAPTER ROUND UP


s-Block Elements
[Noble gas]ns'™ *

Group I: Alkali metals : [Noble gas]ns! Group II: Alkaline earth metals : [Noble gas]ns*
li, Na, K, Rb, Cs, Fr Be, Mg, Ca, Sr, Ba, Ra
show +1 oxidation state show +2 oxidation state
give characteristic flame colourations due to except Be and Mg, they give characteristic flame
their low ionisation enthalpies colourations.
exhibit photoelectric effect (except Li) Basic character:
Basic character: Be(OH), < Mg(OH), < Ca(OH), < Sr(OH), < Ba(OH),
liOH < NaOH < KOH < RbOH < Cs0OH

WWW.JEEBOOKS.IN
s-BLOCK ELEMENTS (ALKALI AND ALKALINE EARTH METALS)

® get tarnished in moist air and are kept under kerosene Alkaline earth metals react with air or O, slowly upon
oil, benzene, hexane heating to form oxides MO.
On burning lithium forms monoxide (Li,O), sodium Powdered Be burns brilliantly to give BeO and Be.N,,.
forms the peroxides (N a,O,), the other elements form 2Be + O, (air) —> 2BeO
oBe + N, (air) —> Be,N,
superoxides (MO,: M = K, Rb, Cs).
Alkaline earth metals react with acids and evolve H,.
cAI SUN — 1,0 (monoxide)
Be is amphoteric and reacts with HCl and NaOH.
2Na + O, —* EmOm (peroxide)
Solubility decreases :
M+ 0, —> MO 5 (superoxide)
BeSO, > MgSO, > CaSO, > SrSO, > Cas0O,;
(M = K, Rb, Cs) BaCO, > MgCO, > CaCO, > SrCO, > BaCO,
This is because small cation can stabilize small anion and
Oxides are
large cation can stabilize large anion.
BeO MgO CaO, SrO, BaO
e Jonic radii of alkali metals :
Amphoteric Weakly basic Basic
Cs* > Rb* > K* > Na* > Li*
BeCl, is covalent while MgCl, and CaCl, are ionic.
® Enthalpy of hydration :
Beryllium halides are covalent while all other halides are
Lit > Nat > K* > Rb* > Cs*
ionic and are readily soluble in water.
® Jonic radii of hydrated alkali metal ions : BeCl, in vapour phase exist as dimer which dissociates
Li* > Na* > K* > Rb* > Cs* to linear monomer at 1200 K.
Tonic radii of hydrated Li* is largest because the value of In solid state, BeCl, has polymeric chain structure
charge/size for Lit is highest. Hence, it is hydrated to the
greatest extent.
Compounds of alkaline earth metals
® lonic mobility in water :
® Calcium oxide or Quick lime (CaO)
Lait < Nat < K* < Rb < Cat
obtained by heating limestone
* Solutions of alkali metals in liquid NH, are highly
Heat
conducting and deep blue in colour. CaCO, == CHO 1 el
® The blue colour is due to the presence of ammoniated Calcium hydroxide or slaked lime [Ca(OH),]
electrons. On adding water to CaQ, calcium hydroxide is formed.
CaO + FLO — Ca(OH),
® Alkali metals are strong reducing agents and reducing
It is called slaking of lime.
character increases down the group (except Li, which
is strongest reducing agent due to its high oxidation When CO, is passed through lime water, milkiness is
potential). formed (due to CaCQ,) and on further passing CO, gas,
® Fluorides of alkali metals are most stable while iodides milkiness disappears [due to Ca(HCQ,),].
are least stable. Calcium carbonate or limestone (CaCO,)
Ca(OH), + CO, —> CaCO, + H,O
CL)Compounds of alkali metals
Na,CO, + CaCl, —~> CaCO, + 2NaCl
® Sodium carbonate (washing soda) all
Na,CO,.10H,O Plaster of paris estes ee

Preparation : Solvay process a(CaSO,.2H,0) —> (CaSO,), H,0 + 3H,0


aN + HO + CO, —fty (ib.Of Gypeum

(NH,), CO,+H,O+CO, Fo 2NH,HCO, On mixing with water, it forms a plastic mass which sets
into a hard solid mass (gypsum). This is called setting of
NH,HCO, + NaCl ee eZ O, + NH,Cl
Plaster of Paris.
2NaHCO, —Heat_, .Na,CO, + CO, + H,O 1
2CaSO, .> H,O + 3H,O —> 2CaSO,.2H,0
Sodium chloride (NaCl) Gypsum
Obtained by solar evaporation of sea water.
Sodium hydroxide or caustic soda (NaQH) Diagonal relationship
commercially obtained by electrolysis of an aqueous Li—Mg and Be—Al

solution of sodium chloride using mercury cathode and e BeCl, and AIC], behave as strong Lewis acids and are

graphite anode, (Castner Kellner Cell) soluble in organic solvents.

Sodium bicarbonate (baking soda) NaHCo,. *¢ Both Be and Al form fluoro complex ions i.e. BeF,* and

prepared by saturating sodium carbonate solution with AIF, in solution.


le
Na,CO,+H,O+CO, —>» 2NaHCO,

Nat, K+, Mg** and Ca* are found in large proportions in biological fluids. These ions perform important biological functions
such as maintenance of ion balance and nerve impulse conduction.
Toe. MODERN'S abc + OF CHEMISTRY
-XI

) SING FILE \ Solved


Textbook Exercises iii
Q.1. What are the common physical and chemical Explain why can alkali and alkaline earth metals
features of alkali metals? not be obtained by chemical reduction
Ans. Refer text page 344. methods?
Q. 2. Discuss the general characteristics and Alkah and alkaline earth metals are themselves very
gradation in properties of alkaline earth metals. strong reducing agents (high reduction potential
Ans. Refer text page 21. values). Therefore, they cannot be isolated by reduction
Q.3. Why are alkali metals not found in nature? of their oxides or other compounds.
Ans. Alkali metals are highly reactive and therefore, are Why are potassium and cesium, rather than
not found in nature. They are present in the combined lithium used in photoelectric cells?
state in the form of halides, oxides, silicates, borates, Potasstum and cesium have much lower ionization
nitrates, ete. enthalpy than that of lithium. Therefore, these
Q. 4. Find out the oxidation state of sodium in Na,O,,. metals on exposure to light, easily emit electrons but
Ans. Let x is the oxidation state of sodium. Since Na,O, lithium does not. Therefore, K and Cs rather than Li
contains a peroxide linkage in which oxygen has are used in photoelectric cells.
oxidation state of -1, Q. 10. When an alkali metal dissolves in liquid
x —l ammonia the solution can acquire different
colours. Explain the reasons for this type of
Na, 0.
colour change.
9n+2(-1)=0 orx=+1
Ans. The dilute solutions of alkali metals in liquid
Q. 5. Explain why is sodium less reactive than ammonia are blue. As the concentration increases,
potassium. the colour changes to bronze. The blue colour of
Ans. The ionisation enthalpy of potassium (496 kJ mol-*) alkali metals is due to the ammoniated electrons.
is less than that of sodium (520 kJmol™) and M +(x +.y) NH, ——> [M(NH,)]* + leCNH,) J~
therefore, potassium is more reactive than sodium.
Alkali metal Ammoniated electron
This is also indicated by more negative standard
However, when the concentration increases, the
electrode potential of potassium (—2.925 V) as
ammoniated metal ions may get bound by free
compared to sodium (—2.714V).
electrons and the colour becomes copper bronze.
Compare the alkali metals and alkaline earth
metals with respect to (1) ionisation enthalpy 2e-(NH,), ——> le(NH,),],
(ii) basicity of oxides and (iii) solubility of . Beryllium and magnesium donot give colour to
hydroxides. flame whereas other alkaline earth metals do
so. Why?
(i) lonisation enthalpy. Ionisation enthalpies of
alkaline earth metals are higher than those of alkali Refer Conceptual Questions [2Ja.14. (Page 36)
metals. This 1s because of smaller size of alkaline earth Discuss the various reactions that occur in the
metal corresponding to alkali metal ofthe same period. Solvay process.
(ii) The oxides of alkali and alkaline earth metals In Solvay process, carbon dioxide is passed through
dissolve in water to form their respective a brine solution (about 28% NaCl) saturated with
hydroxides. These oxides are strong bases. ammonia to give sodium bicarbonate.
However, the oxides of alkali metals are more 2NH, + H,O + CO, ——> (NH,),CO,
basic than those of alkaline earth metals. This is Amm. carbonate
because the ionization enthalpy of alkali metals is (NH,),CO, + H,O + CO, ———> 2NH,HCO,
lower or the electropositive character of alkali Amm. bicarbonate
metals is higher than that of corresponding alkaline
earth metal so that M—OH bond in alkali metals NH,HCO, + NaCl ——-> NaHCO, + NH,Cl
can more easily ionize. Brine Sod. bicarbonate (ppt.)
MOH ——-> M* + OH™ The precipitated sodium bicarbonate 1s filtered and
(iii) Because of small size and high charge, the dried. It is ignited to give sodium carbonate.
lattice enthalpies of alkaline earth metals are much 2NaHCO, —!, Na,CO, + CO, + H,O
more than those of alkali metals and therfore, the
sodium carbonate
solubility of alkaline earth metal hydroxides is less
than that of alkali metals. The carbon dioxide required for the reaction is prepared
Q. 7. In what ways lithium shows similarities to by heating limestone (CaCO,) to about 1300 K in a
magnesium in its chemical behaviour? lime kiln. Lime formed is dissolved in water to get
Ans. Refer text page 10. calclum hydroxide which is transferred to the
ammonia recovery tower.
s-BLOCK ELEMENTS (ALKALI AND ALKALINE EARTH METALS)

CaCO, —Heat_, CaO + CO, MCO,——> MO +CO,


(M = Be, Mg, Ca, Sr, Ba)
Calcium oxide (lime)
However, their stability increases as we move down
CaO + H,O ——-> Ca(OH),
the group. The temperature at which these carbonate
Ammonia needed for the purpose is prepared by decompose are:
heating ammonium chloride obtained above with
calcium hydroxide. BeCO, MgCO, CaCO, SrCO, BaCO,
2NH,Cl+ Ca(OH), ———> 2NH, + CaCl, + 2H,O <373 K 813K 11738K 1563K 1635K
Therefore, the only by product ofthe reaction1sCaCl. Thermal stability increases ———>
Q.13. Potassium carbonate cannot be prepared by
Solvay process. Why? 1i,CO, decomposes similar to MgCO, because of
Ans, Potassium carbonate cannot be prepared by Solvay diagonal relationship.
process because potassium bicarbonate being more
soluble than sodium bicarbonate does not get
Li, FQ» “> 110 + CO,
All other alkali metal carbonates are stable and
precipitated when CO, is passed through a
donot decompose readily.
concentrated solution of KC] saturated with NH.
(c) Sulphates of alkaline earth metals decompose on
Q.14. Why is Li,CO, decomposed at a lower
temperature whereas Na,CO, at higher
heating and give SQ,.
temperature? Mso, 4+ MO+SO0,
Ans. Lithium is less electropositive than sodium and The stability of sulphates of alkaline earth metals
therefore, carbonate of lithium is less stable than increases as the electropositive character of the
that of sodium. Li,CO, is not so stable to heat and metals increases.
therefore, decomposes at lower temperature. This is BeSO, MgSO, CaSO, Srso,
because lithium being very small in size polarises a
T7I3K 1168K 1422K 1644K
large CO,* ion leading to the formation of Li,O and
CO,. On the other hand, Na,CO, is very stable and
Among alkali metal sulphates Li,SO, decomposes
decomposes at higher temperature. like MgSO, to give SO,.
Q. 15. Compare the solubility and thermal stability of
Li,so, —4> Li,O + So,
the following compounds of the alkali
metals with those of the alkaline earth metals.
(a) Nitrates (6) Carbonates (c) Sulphates.
MgSO, “> MgO +80,
Other alkali metal sulphates are stable to heat and
Solubility. (a) Nitrates, carbonates and sulphates of
donot decompose easily.
alkali metals are soluble in water. Their solubility in
water increases on moving down the group because Q.16. Starting with sodium chloride how would you
their lattice enthalpies decrease more rapidly than proceed to prepare (i) sodium metal (iz) sodium
the hydration enthalpies. hydroxide (111) sodium peroxide (iv) sodium
carbonate?
(6) On the other hand, nitrates of alkaline earth
metals are soluble in water. However, their solubility Ans. (i) Sodium metal is manufactured by electrolysis of
decreases down the group because their hydration a fused mixture of NaCl (40%) and CaCl, (60%) in
enthalpies decrease more rapidly than the lattice Down cell at 873K using graphite anode and iron
enthalpies. cathode. Sodium is liberated at the anode while Cl,
is evolved at cathode
The carbonates and sulphates of alkaline earth
metals are less soluble than corresponding alkali At cathode : Na* (melt) + e* ——-> Na (I)
metals. Their solubility decreases down the group due 2Cl(melt) ——-+ Cl, + 2e-
to decrease in hydration enthalpy. (ii) Sodium hydroxideis manufactured by electrolysis
Thermal stability. (a) Nitrates of both alkali metals of sodium chloride (brine) in Castner Kellner cell,
and alkaline earth metals decompose on heating. using carbon anode and mercury cathode. Sodium
Alkaline earth metals nitrates on heating give metal metal discharged at the cathode combines with
oxide, NO, and oxygen. The nitrates of Na, K, Rb and mercury to form sodium amalgam while Cl, gas is
Cs decompose to give metal nitrites and oxygen. evolved at the anode.
Cathode : Nat + e ——-> Na
2M(NO,), —“ > 2MO + 4NO, + O, 2Na + Hg ——> NaHg (amalgam)
(M = Be, Mg, Ca, Sr, Ba)
Anode : 2Cl ——-> Cl, + 2e7
2MNO, —4_, 2MNO, + 0, The amalgam reacts with water to give NaOH and
(M = Na, K, Rb, Cs) H, gas is evolved.
Lithium nitrate decomposes like Mg(NO,), to form 2NaHg + 2H,O ——-> 2NaOH + 2H¢ + H,
metal oxide, NO, and O,. (iii) Sodium peroxide is manufactured by heating
4LiNO, —4_, 2Li,0 + 4NO, +O, sodium in excess of oxygen. The initially formed
sodium oxide reacts with more oxygen to form Na,O,.
This is because Li* being small in size cannot
stabilize large NOJ ion.
2Na + -O, ——> Na,O
(b) The carbonates of alkaline earth metals are
relatively less stable towards heat and decompose to 2Na,0 + O, ——-> 2 Na,O,
CQ,.
Too (iv) Sodium carbonate is obtained by Solvay process.
MODERN'S abe + OF CHEMISTRY-AI

(iii) Plaster of Paris. It is used in building industry


For details refer answer to Q. 12. as well as in plasters. It is used for making statues,
Q.17. What happens when models and other decorative materials. It is used in
(4) magnesium is burnt in air surgical bandages known as plasters for immoblising
(41) quick lime is heated with silica the fractured bases in the body.
(iit) chlorine reacts with slaked lime Q.22. Why are lithium salts commonly hydrated and
those of the other alkali metal ions usually
(iv) calcium nitrate is heated
anhydrous?
Ans. Refer Solved Example 12. (Page 32)
Ans. Because of smallest size of Li* ion among alkali
Q.18. Describe two important uses of each of the
metal ions, it can polarize water molecules more
following: (i) caustic soda (ii) sodium carbonate
easily than other alkali metalions. Therefore, lithium
(zit) quick lime.
salts get bonded to water of crystallisation. For
Ans. (a) Caustic soda example, lithium chloride exists as LiC].2H,O.
(i) Itisusedin the manufacture ofsoap, paper, Q.23. Why is LiF almost insoluble in water whereas
artificial silk and other chemicals. LiCl soluble not only in water but also in
(ii) It is used in the textile industry for acetone?
mercerising cotton fabrics. Refer Conceptual Questions, Q. 12. (Page 19)
(b) Sodium carbonate
Q. 24. Explain the significance of sodium, potassium,
(i) Itis used for softening hard water. magnesium and calcium in biological fluids.
(ii) Itisused for washing purposes in laundary.
Refer Text page 11-12.
(c) Quick lime
Q.25. What happens when
(i) It is used in the manufacture of sodium
(i) sodium metal is dropped in water?
carbonate from caustic soda.
(ii) Itis used as a flux in metallurgy. (ii) sodium metal is heated in free supply of
air ?
Q.19. Draw the structures of (i) BeCl, (vapour) and (iii) sodium peroxide dissolves in water?
(it) BeCl, (solid). Ans. (i) When sodium metal is dropped in water,
Ans. In vapour state, BeCl, exists as dimer hydrogen gas is evolved which catches fire due
to exothermic nature of the reaction.
ee 2Na(s) + 2H,O(7) ——+ 2NaOH(aq) + H,
CI—B Be—Cl
=. C1 (ii) Sodium a and sodium oxide is formed.
2Na(s) + =0,(¢) —> Na, O(s)
In solid state, BeCl, exists as polymeric structure (Minor ama)
with chlorobridges.
Na,O(s) + 1/20,(¢) ——- Na,O,(s)
ae ee > <a ae [IS Ch.
(Major product)
(iii) When sodium peroxide is dissolved in water,
~— ~. cr hydrogen peroxide is formed.
—s Yo
Na,O,(s) + 2H,O(/) ——- 2NaQOH (aq) + H,0,(2)
Q.20. The hydroxides and carbonates of sodium and
potassium are easily soluble in water while the Q.26. Comment on each of the following observations:
corresponding salts of magnesium and calcium (a) The mobilities of the alkali metal ions in
are sparingly soluble in water. Explain. aqueous solution are
Ans. Sodium and potassium have large size as compared Li* < Nat < K* < Rb* < Cs*.
to magnesium and calcium. Therefore, the lattice (b) Lithium is the only alkali metal to form a
enthalpies of hydroxides and carbonates of sodium nitride directly.
and potassium are much lower than those of (c) E’for M?**(aq) + 2e- > M(s) (where M = Ca,
hydroxides and carbonates of magnesium and calcium. Sr or Ba) is nearly constant.
As aresult, the hydroxides and carbonates of sodium (a) The alkali metal ions are highly hydrated. The
and potassium are easily soluble in water whereas smaller the size of the ion, the greater is the degree
the corresponding salts of magnesium and calcium of hydration. Thus, Li* ion gets much more hydrated
are sparingly soluble in water. than Na* ion which is more hydrated than K* ion and
Q.21. Describe the importance of the following : so on. Therefore, the extent of hydration decreases
(z) limestone (i) cement (iii) plaster of paris. from Lit to Cs*. As a result of larger hydration of Li*
Ans. (i) Limestone. It is used as a building material in ion than Nat ion the effective size of Li* ion is more
the form of marble. It is used in the preparation of than that of Nat ion and ionic radii in water decrease
quick lime and as araw material for the manufacture as:
of sodium carbonate in Solvay process. It is also used Li* > Nat > K* > Rb* > Cst
as a constituent of tooth paste and filler in cosmetics. Therefore, the mobility of Li* ion is slowest and that
(it) Cement. Itis animportant construction material. of Cs* ion is maximum. Thus, mobility ofions in water
It is used in concrete and reinforced concrete, in increases as:
plastering and in the construction ofbuildings, bridges,
Lit < Nat < Kt < Rbt < Cst
dams, etc.
s-BLOCK ELEMENTS (ALKALI AND ALKALINE EARTH METALS)

(b) Refer Conceptual Questions Q@. 13 (Page 19). (tt) BaO is soluble but BaSO, is insoluble in
(c) E° value of M?*/M(s) electrode depends upon three water,
factors: (iii) Lil is more soluble than KI in ethanol.
(i) enthalpy of vaporisation Ans. (i) BeO is insoluble in water because of its high
lattice enthalpy. It is also covalent in nature and
(i) ionization enthalpy and
therefore, insoluble in water. On the other hand,
(iii) enthalpy ofhydration. The combined effect of these BeSO, is ionic. Its hydration enthalpy is much more
factors is approximately the same for Ca, Sr and Ba than its lattice enthalpy and hence it is soluble in
so that their electrode potentials are nearly same. water.
Q. 27. State as to why (ii) Both BaO and BaSO, are ionic compounds.
(a) a solution of Na,CO, is alkaline? However, the size of O*-10nis much smaller than that
(6) alkali metals are prepared by electrolysis of of the SO7 ion. Since a bigger anion stabilizes a bigger
their fused chlorides? cation more than a smaller cation stabilizes a bigger
(c) sodium is found to be more useful than anion, therefore, the lattice enthalpy of BaOis smaller
potassium? than BaSO,. Therefore, BaO is soluble while BaSO,
(a) Refer Conceptual Questions [7]e. 14. (Page 19) is insoluble in water.
(b) The discharge potentials of alkali metals are (iit) Lit is smaller in size than K* ion. Therefore,
much higher than that of hydrogen and therefore, according to Fazan rules, Li* ion can polarize bigger
when the aqueous solutions of alkali metal chlorides I ion to a greater extent than K* ion. As a result,
are subjected to electrolysis, H, is evolved preferably Lilismore covalent than KI and henceismoresoluble
than the alkali metal at the cathode. Therefore, to in organic solvents like ethanol.
prepare alkali metals, electrolysis of their fused Q. 30. Which of the alkali metal is having least melting
chlorides is carried out. point?
(c) Sodium ions are found primarily in the blood (a) Na (6) K (c) Rb (d) Cs
plasma and in the interstitial fluid which surrounds (d) With increase in size, the strength of metallic
the cells. On the other hand, potassium ions are bonding decreases and therefore, melting point also
present within the cell fluids. Sodium ions help in the decreases.
transmission of nerve signals, regulating the flow of Q. 31. Which one of the following alkali metals gives
water across cell membranes and in the transport of hydrated salts?
sugars and amino acids in the cells. Thus, sodium is (a) Li (6) Na (c) K (d) Cs
more useful than potassium.
(a) Li* being the smallest has highest charge density
Q. 28. Write balanced equations for reactions between and hence maximum hydrated.
(a) Na,O, and water Q.32. Which one of the alkaline earth metal
(6) KO, and water carbonates is thermally the most stable?
(c) Na,O and CO,,. (a) MgCO, (b) CaCO, (c) SrCO, (d) BaCO,
Ans Refer Solved Example 10. (Page 18) (cd) As the electropositive character of alkaline earth
Q. 29. How would you explain the following metals increases down the group, their thermal
observations? stability increases. Hence BaCO, is the most stable.
({) BeOisalmostinsoluble but BeSO, is soluble
in water,

* NCERT Exemplar Problems //


§ Objective Questions from Exemplar Problems are Subjective Questions
given in Competition File, page 56
Ans. Lithium forms only oxide (Li,O), sodium forms mainly
peroxide (Na,O,) and potassium forms superoxide
7 Telam clei meOlictadfelipe carrying 2 or 3 marks <4
Q. 3.
(KO,). Refer text page 5-6.
Complete the following reactions
Q. 1. How do you account for the strong reducing
power of lithium in aqueous solution ? (i) 02- +H,O —>
Ans. Lithium has most negative electrode potential (—3.04V) (ii) O05; +H,O —>
among alkali metals and hence it is the strongest
reducing agent. This is mainly due to very high enthalpy . (Z) O; +2H,O0 —> 20H +H,0,
of hydration in its aqueous solution. For reasons, Refer (ii) 20;+2H,0 —> 20H-+H,0, +0,
text page 10-11. . Lithium resembles magnesium in some of its
.2. When heated in air, the alkali metals form properties. Mention two such properties and
various oxides. Mention the oxides formed by give reasons for this resemblance.
Li, Na and K. . Refer diagonal relationship, page 10.
S Tioae
Q. 5. Name an element from Group 2 which forms
MODERN'S abc + OF CHEMISTRY-XI

has seven electrons and hence its O.N. is —1. The


an amphoteric oxide and a water soluble
average O.N. of each O atom is =(O-1)=-1/2
sulphate.
Ans. Beryllium forms amphoteric oxide, BeO. It reacts Q.11. Why do beryllium and magnesium not impart
with acids as well as bases : colour to the flame in the flame test ?
BeO + 2HC] —-> BeCl, + H,O (basic nature) . Refer NCERT Qs. Q. 11.
BeO + 2NaOH —-> Na,BeO, + H,O (acidic nature) Q. 12. What is the structure of BeCl, molecule in
It also forms highly soluble BeSO,. gaseous and solid state?
. Discuss the trend of the following :
. Refer Text and Fig. 5 and 6.
(4) Thermal stability of carbonates of Group 2
elements.
(t2) The solubility and the nature of oxides of
»> Long Answer Questions Carrying 5 marks <4
Q. 13. The s-block elements are characterised by their
Group 2 elements. larger atomic sizes, lower ionisation enthalpies,
Ans. (i) The stability of carbonates of Group 2 elements invariable +1 oxidation state and solubilities of
increases down the group.
their oxosalts. In the light of these features
describe the nature of their oxides, halides and
(iz) All the elements of Group 2 form oxides (MO). oxosalts.
The solubility of these increases down the group.
Among the oxides, BeO is amphoteric and other Ans. Refer Text (Page 8).
oxides are basic in nature. Refer text page 25. Q. 14. Present a comparative account of the alkali and
. Why are BeSO, and MgSO, readily soluble in alkaline earth metals with respect to the
water while CaSO,, SrSO, and BaSOQ, are following characteristics :
insoluble? (i) Tendency to form ionic/covalent compounds
Ans. Th hydration enthalpies of BeSO, and MgSO, are (it) Nature of oxides and their solubility in water
quite high because of small size of Be?* and Mg”** ions. (iii) Formation of oxosalts
These hydration enthalpy values are higher than their (tv) Solubility of oxosalts
corresponding lattice enthalpies and therefore, BeSO,
and MgSO, are highly soluble in water. However, (v) Thermal stability of oxosalts
hydration enthalpies of CaSO,, SrSO, and BaSO, are Ans. Refer Text (Page 7-21).
not very high as compared to lattice enthalpies and Q. 15. When a metal of group 1 was dissolved in liquid
hence these are insoluble in water. ammonia, the following observations were
. All compounds of alkali metals are easily soluble obtained:
in water but lithium compounds are more soluble (4) Blue solution was obtained initially.
in organic solvents. Explain.
(tz) On concentrating the solution, blue colour
Ans. Lithium compounds have covalent character and changed to bronze colour.
therefore, more soluble in organic solvents. The other
How do you account for the blue colour of the
alkah metal compounds are ionic in nature and are
solution ? Give the name of the product formed
soluble in water.
on keeping the solution for some time.
. In the Solvay process, can we obtain sodium
carbonate directly by treating the solution Ans. Refer Text page 6.
containing (NH,),CO, with sodium chloride? Q. 16.The stability of peroxide and superoxide of
Explain. alkali metals increase as we go down the group.
Explain giving reason.
Ans. In Solvay process, we cannot obtain directly sodium
carbonate by treating the solution containing Ans. Refer Text page 5.
Q.17.When water is added to compound (A) of
(NH,),CO, with NaCl because both Na,CO, and
calcium, solution of compound (B) is formed.
NH,Cl are highly soluble and equilibrium will not
When carbon dioxide is passed into the solution,
shift in the forward direction.
it turns milky due to the formation of compound
(NH,),CO, + 2NaCl = Na,CO, + 2NH,Cl (C). If excess of carbon dioxide is passed into
Q. 10. Write Lewis structure of O35 ion and find out the solution milkiness disappears due to the
formation of compound (D). Identify the
oxidation state of each oxygen atom? What is the
compounds A, B, C and D. Explain why the
average oxidation state of oxygen in this ion?
milkiness disappears in the last step.
Ans. When CO, gas is passed through solution of
Ans. Lewis structure is 0 =. O:
compound B, it turns milky, therefore, solution of
compound B is Ca(OH),.
Since O having no charge on it has six electrons, its
O.N. is zero. The O.N. of oxygen carrying —ve charge The milkiness is due to the formation of CaCO,(C).
Since compound B is obtained by dissolving
s-BLOCK ELEMENTS (ALKALI AND ALKALINE EARTH METALS)

compound (A) in water, compound A must be quick Q. 20. Ions of an element of group 1 participate in the
lime, CaO. The reactions are : transmission of nerve signals and transport of
CaO + £#H,O —> Ca(OH), sugars and aminoacids into cells. This element
Calcium oxide Lime water imparts yellow colour to the flame in flame test
and forms an oxide and a peroxide with oxygen.
(A) (B)
Identify the element and write chemical
Ca(OH), + CO, — > CaCO, + H,O
reaction to show the formation of its peroxide.
Calcium carbonate (C) Why does the element impart colour to the
(Milkiness)
flame ?
Milkiness disappears on passing excess CO, because Ans. Since the alkali metal imparts yellow colour in the
of the formation of soluble calcium bicarbonate. flame test, it must be sodium.
CaCO, + CO, + H,O — > Ca(HCQ,), It reacts with O, to form sodium peroxide and sodium
Soluble (D) oxide.
Milkiness disappears
Q. 18. Lithium hydride can be used to prepare other 4Na+O, ——+ 2Na,0
useful hydrides. Beryllium hydride is one of Sodium oxide
them. Suggest a route for the preparation of
beryllium hydride starting from lithium 2Na +0, “=> Na,O,
hydride. Write chemical equations involved in Sodium peroxide
the process.
Ans, 8LiH + Al,Cl, —> 2LiAlH, + 6LiCl 2Na,0 + Op —"> 2Na,0,
2BeCl, + LiAIH, —~> 2BeH, + LiCl + AICI, Sodium peroxide
Q. 19.An element of group 2 forms covalent oxide So mainly sodium peroxide is formed.
which is amphoteric in nature and dissolves Why does sodium impart yellow colour to the flame ?
in water to give an amphoteric hydroxide. The ionization enthalpy of sodium is low. When sodium
Identify the element and write chemical metal or its salt is heated in Bunsen flame, its outermost
reactions of the hydroxide of the element with electron gets excited to higher energy levels by absorption
an alkali and an acid. of energy. When the excited electron returns to the ground
Ans. The element is Be and its covalent oxide is BeO. Its state, it emits the extra energy which falls in yellow region
reaction with acids and bases are : of the visible spectrum. Therefore, sodium imparts yellow
BeO + 2HCl ——> BeCl, + H,O colour to the flame.
Heat
BeO + 2NaOQH ——— Na,BeO, + H,O
Sodium beryllate

age

ra
a0
eltaX Memory TEST WN

|A. | Say True or False |


1:30) Complete the missing links
1. Sodium is stronger reducing agent than magnesium.
2. The solubilities of metal carbonates in water decreases 1. The radioactive alkali metal is ...........
down the family. 2. Lithium shows diagonal relationship with ...........
3. Chlorophyll is an important complex of calcium. 3. The bicarbonate of alkali metal which does not exist
4, Potassium has higher density than sodium. in the solid state is ...........
5. Beryllium and magnesium donot give characteristic 4, Oxidation state of sodium in sodium peroxide is
colour to the flame.
6. LiF is almost insoluble in water whereas LiCl is 5. Lithium reacts readily with nitrogen to form ..........
soluble. 6. Solvay process is used for the manufacture of ..........
7. Lithium floats on water without any apparent reaction
7. Alkali metals dissolve in liquid ammonia to form
with it.
ere coloured solutions.
8. KO, is diamagnetic oxide.
8. The reactivity of alkali metals .......... from Li to Cs.
9. Sodium nitrate on heating gives nitrogen dioxide and
oxygen. | ers gives golden yellow colour to the flame.
10. Beryllium resembles more with aluminium than with 10. Plaster of paris is obtained by heating ..........
magnesium.

WWW.JEEBOO |
MODERN'S abc + OF CHEMISTRY
-XI

6. Among alkali metals cesium can show photoelectric


effect to the maximum /minimum extent.
1. Lithium forms only oxide /peroxide. 7. The superoxide ion is diamagnetic /paramagnetic.
2. Among the peroxides of alkaline earth metals, MgO, 8. The alkali metal hydrides behave as strong reducing/
is the least/most stable. oxidising agents.
3. The hydration energy of Li* ion is more /less than that 9. Among the alkali metal halides MF, MCl, MBr and
of Na® ion. MI, covalent character increases /decreases.
4, Mg(OH), is less/more basic than Ca(OH),. 10. Alkali metals can be stored in benzene /ethanol.
5. K,CO, can /cannot be prepared by Solvay process like
Na,COQ,.

elife4 Memory Test “\


Say True or False

1. True 2. True 5. Lithium nitride (Li,N)


3. False : Chlorophyll is a complex of magnesium. 6. Na,CO,-10H,O (sodium carbonate)
4, False : Potassium has lower density than sodium. 7. blue 8. decreases
5. True 6. True 7. True
9. sodium 10. gypsum
8. False : KO, is paramagnetic.
9. False : Sodium nitrate decomposes as :
Choose the correct alternative
2NaNO, —=22"+ 2NaNO, + O,
1. oxide 2. most 3. more
10. True.
4, less 5. cannot 6. maximum
Es Complete the missing links 7. paramagnetic 8. reducing 9, increases
1. francium 2. magnesium 10. benzene
3. LiHCO, 4, +1

oe Advanced Level }——


QUESTIONS WITH ANSWERS
Q.1. Name the element which is invariably bivalent stable because Cs* being large cation can stabilize large I~
and whose oxide is soluble in excess of NaOH and its 1on.
dipositive ion has noble gas configuration. Q. 5. Describe the difference in structure between
Ans. Beryllium BeO dissolves in NaOH to form sodium CaH, and BeH,,.
beryllate. Ans. CaH, is an ionic solid consisting of Ca** and H-
BeO + 2NaQH —-> Na,BeO, + H,0. ions having slightly distorted hep arrangement. On the other
Q. 2. BeClL, gives an acidic solution when dissolved hand, BeH, has polymeric bridged like structure (BeH,).
in water. Why ?
Ans. BeCl, gets hydrolysed in water forming H* ions Be _ a Be C
and therefore, the solution is acidic.
BeCl, (s) + 2H,0 ——> Be(OH), + 2H* + 2CI- Q. 6. LiF is insoluble in water whereas other alkali
Q. 3. Salts of alkaline earth metals are colourless metal fluorides are soluble.
and diamagnetic. Explain. Ans. Both Li* and F- ions have small size and therefore,
Ans. Alkaline earth metals form dipositive ions in their LiF has high lattice energy. Although the hydration energy of
salts. These dipositive ions have noble gas configurations LiF is also high but, it is less than lattice energy (hydration
with no unpaired electrons. Therefore, these compounds energy of LiF = —1034 kJ, lattice energy = —1045 kJ).
are colourless and diamagnetic. Therefore, it is insoluble.
Q. 4. Lil, is less stable than CsI,. Why? On the other hand, all other alkali metal fluorides have
Ans. This is due to lattice energy effects. The large cation lower lattice energies than corresponding hydration energies
stabilizes a large anion in its lattice. Therefore, CsI, is more and therefore, these are soluble.
s-BLOCK ELEMENTS (ALKALI AND ALKALINE EARTH METALS)

Q.7. Why are alkali metals soft and have low melting dissolves in NaOH forming sodium beryllate :
points ? Be(OH), + 2NaOH —-> Na,BeO, + 2H,O
Ans. Alkali metals have only one valence electron per On the other hand, Mg(OH), is basic and does not
metal atom. As a result, the binding energy of alkali metal dissolve in NaOH.
ions in the close-packed metal lattices are weak. Therefore, (b) The beryllium halides are electron deficient covalent
these are soft and have low melting points. compounds because they have only four electrons in the
Q.8.LiH, LiF and Li,N show exceptional thermal valence shell. Therefore, to complete their octets, they
stabilities. Explain. undergo polymerisation.
Ans. Li* is very small in size. Therefore, its salts with Q.14. Mg,N, when reacted with water gives off
small anions such as F-, H- and N* are exceptionally stable ammonia but HCl is not obtained from MgCl, on
due to their high lattice energies. reaction with water at room temperature. Explain.
Q.9. When burnt in air, lithium forms normal oxide,
Ans. Mg,N, is a salt of a strong base Mg(OH), and a
Na the peroxide and K the superoxide. Why ?
weak acid NH, and therefore, gets hydrolysed giving NH,.
Ans, Li‘ is very small in size and has strong field around On the other hand, MgCl, is a salt of strong acid HCl and a
it. It can stabilize only small sized oxide O* ion and strong base Mg(OH), and therefore, does not undergo
therefore, forms only Li,O. It cannot stabilize other large hydrolysis to give HCl.
anions such as peroxide (O,”-) and superoxide (O,,).
Q.15. Li is more stable than NaH. Explain.
Na* being larger in size can stabilize large peroxide ion
(O,?-) and therefore, forms peroxides. Ans. Both Lit and H’ have small size and their
K* being large can also stabilize large superoxide ion combination has high lattice energy. Therefore, LiH is stable
(O,-) forming superoxides. as compared with NaH.
Q.10. Li,CO, is unstable while all other alkali metal Q.16. Sodium fire in the laboratory should not be
carbonates are relatively stable. Explain. extinguished by pouring water. Why ?
Ans. Li* ion being small in size, cannot stabilize large Ans, Sodium reacts violently with water producing
CO,” ion and therefore, Li,CO, is unstable. Other alkali hydrogen which also catches fire. Therefore, the fire increases
metal ions are large in size and can easily stabilize large on adding water instead of getting extinguished. Hence,
CO,* ions. This is because of lattice energy effects. water should not be used for extinguishing sodium fire. It can
Q.11. Which out of the following and why can be be extinguished by pyrene (CCL).
used to store an alkali metal ? Q.17. Can you dissolve sodium hydride in water ?
H,0, C,H,OH, Benzene
Ans. Benzene can be used to store an alkali metal
Ans. No, sodium hydride cannot be dissolved in water
because it gets hydrolysed with brisk evolution of hydrogen
because other substances react with alkali metals as : gas.
Na + H,O ——> NaOH + 4H, NaH + H,O —>H, + NaOH
Na + C,H.OH ——> C,H,ONa + 44H, Q.18. The crystalline salts of alkaline earth metals
contain more water of crystallization than the
Q.12. Mg** is much more hydrated than Na*. Why?
corresponding alkali metal salts. Explain.
Ans. The extent of hydration depends upon the charge
Ans. Alkaline earth metals have smaller size and
density (charge/size). Mg** ion is smaller in size than Nat
higher nuclear charge than alkali metals. Therefore,
and has twice the charge than Na. Therefore, its charge
density is high and hence is extensively hydrated. alkaline earth metals have a higher tendency to attract
water molecules and hence contain more water of
Q.13. Explain the following:
crystallisation molecules than alkali metals. For example,
(a) Be(OH), dissolves in sodium hydroxide but
MgCl,.6H,O. However, among alkali metals only lithium
Mg(OH), does not.
because of its small size forms hydrated salts such as
(6) Beryllium halides are polymeric in nature. LiCl.2H,0.
Ans. (a) Be(OH), is amphoteric and therefore, it

= Revision Exercises
5. What is the chemical formula of Plaster of Paris ? Give
>» Very Short Answer Questions <4 its one use.
1. Name the groups which constitute s-block elements. 6. What is the cause of diagonal relationship among
elements ?
2. Write the general configuration of s-block elements.
3. What is the position of Na and K in the order of 7. How does the solubility of hydroxides of alkaline earth
metals vary as we move down the group ?
abundance among elements ?
4, Why does the basic character of hydroxides of alkali 8. Give two main reasons for the differences of properties
metals increase down the group ? of Li and Na.
S Ti06_
9. Give the chemical formula of limestone and Plaster of
MODERN'S abe + OF CHEMISTRY-A4I

13. Describe a method for the preparation of quicklime.


Paris. What happens when water is poured over quicklime ?
10. Alkali and alkaline earth metals cannot be obtained by 14, Differentiate between
chemical reduction method. Explain. (i) Quicklime (it) Lime water
11. Name the compound which can be obtained by Solvay’s (iii) Slaked lime
process.
15. Contrast the action of heat on the following and explain
12. Why do we store sodium metal in kerosene oil and not
your answer :
in water ?
(i) Na,CO, and CaCO,
13. What is quicklime ? What happens when we add water
to quicklime ? (ii) MgCl,.6H,0 and CaCl,.6H,O
14, What is dead burnt plaster ? (iii) Ca(NO,), and NaNO,
15. Why cannot sodium be prepared by the electrolysis of its 16. State as to why ? N.C.E.R.T.
aqueous solution ? (i) Sodium is found more useful than potassium
16. Why does lithium form monoxide while sodium form (ii) Solution of Na,CO, is alkaline
peroxides ? (iii) Alkah metals are prepared by the electrolysis
17. How does the basic character of hydroxides of alkali of their fused chlorides
metals vary down the group ? 17. In what ways, lithium shows similarities to magnesium
18. Compare the action of heat on LiNO, and NaNO,. in its chemical behaviour ?
19. Which out of Li or Na forms nitrides ? Why ? 18. State the effect of heat on
20. Give two uses of Na,CO,. (i) gypsum (iz) limestone (iii) epsom salt.
21. Name radioactive elements of group 1 and 2.
22. Why does magnesium not give flame colouration ?
19. What is cement ? Give the chemistry of manufacture of
23. Which is the most abundant element among alkaline
cement.
earth metals ?
20. Write three general characteristics of elements of
24, Which out of Nat or Kt ion has higher concentration
s-block of the periodic table which distinguish them
within the cells ?
25. Which out of MgSO, or BaSO, is more soluble in water? from the element of the other blocks.
21. What happens when
»»> Vilelem cpitte7 meOlictid(elem carrying
2 or 3marks << (i) Sodium metal is dropped in water ?
(ii) Sodium metal is heated in free supply of air ?
1, What are the common physical and chemical features of (iii) Sodium peroxide dissolves in water ?
alkali metals? 22. Compare the solubility and thermal stability of the
2. Why do not Be and Mg give characteristic flame following compounds of the alkali metals with those of
colouration while others do ? the alkaline earth metals :
3. Discuss the general characteristics and gradation in
(i) Nitrates (ii) Carbonates
properties of alkaline earth metals.
(iit) Sulphates.
4, How do the following properties vary among alkali
metals ? 23. Describe the principle involved in the electrolysis of
(i) Atomic radius molten sodium chloride.
(it) Ionisation energy 24, Explain the chemistry of Down’s process for the
(iii) Metallic character. extraction of sodium.
5. Name the alkali metals which form superoxides when 20. Describe the preparation of each of the following starting
heated in excess of air. with limestone :
6. Alkali metals are good reducing agents. Explain. (i) Plaster of paris (ii) Slaked lime.
7. Lithium differs from other alkali metals. Explain with 26. In what way does lithium show similarities to
examples. Magnesium in its chemical behaviour ?
8. Compare the alkali metals and alkaline earth metals 27. Account for the thermal stability and solubility of
with respect to oxosalts of alkaline earth metals.
(i) ionisation enthalpy
28. Beryllium exhibits some similarities with aluminium.
(ii) basicity of oxides
Point out four such properties.
(iii) solubility of hydroxides
9, Explain the extraction of sodium from sodium chloride. 29. Explain thetrends in solubility of sulphates and
10. Give the chemical formulae of the following ores : hydroxides of alkaline earth metals.
(i) Dolomite (ii) Gypsum oO). Draw the structure of
(iii) Epsom salt (iv) Carnallite (1) BeCl, (vapour) (i) BeCl, (s)
11. Name the chief factor responsible for the anomalous ol. Starting with sodium chloride how would you proceed
behaviour of lithium. N.C.E.R.T. to prepare (state the steps only)
12, Why is it necessary to add gypsum in the final stages (i) sodium metal (ii) sodium hydroxide
of the preparation of cement ? (iii) sodium peroxide (iv) sodium carbonate.
s-BLOCK ELEMENTS (ALKALI AND ALKALINE EARTH METALS)

oo. “The chemistry of beryllium is not essentially ionic”. ol. Lithium forms only monoxide, sodium peroxide and
Justify the statement by making a reference to the potassium superoxide. Explain.
nature of oxide, chloride and fluoride of beryllium. ay Discuss the various reactions that occur in Solvay
oo. Why does the following reaction : process.
oo. How is Plaster of Paris prepared ?
>C—Cl + MF — SC—F+MCl
54. Like lithium in Group 1, beryllium shows anomalous
proceed better with KF than with NaF ? |N.C.E.R.T. behaviour in Group 2. Write three such properties of
. Why is it difficult to extract alkali metals by usual beryllium which make it anomalous in the group.
methods ”? ede ‘The chemistry of beryllium is not essentially ionic’.
. Give three uses each of lithtum and sodium. Jusfify the statement by making a reference to the
SH. Give the chemistry of Solvay process for the manufacture nature of oxide, chloride and fluoride of beryllium.
of Na,CO,. 56. Why is that the s-block elements never occur free in
od. The second ionization energy of alkaline earth metals is nature ? What are their usual modes of occurrence and
creater than the first ionisation energy, yet they prefer how are they generally prepared ?
to form +2 ions. Explain. ov. When an alkali metal is dissolved in liquid ammonia
. Explain the following properties of alkaline earth metals : the solution acquires different colours. Explain the
(i) Formation of oxides
reason for this type of colour change. E
(it) Formation of hydroxides.
oo. Give one method of preparation and two uses of each of 58 Discuss the trend of the following :
the following : (i) Thermal stability of carbonates of Group 2
(i) Slaked lime (ii) Limestone elements.
(iii) Plaster of Paris. (it) The solubility and the nature of oxides of Group
. Describe the importance of the following : 2 elements. (NCERT Exemplar Problem)
(i) Cement (ii) Plaster of Paris 59. What is the structure of BeCl, molecule in gaseous
(iii) Limestone. and solid state ? (NCERT Exemplar Problem)
41. Explain the following : . All compounds of alkali metals are easily soluble in
(i) The mobilities of the alkali metal ions in water but lithium compounds are more soluble in
aqueous solutions are : organic solvents. Explain.(NCERT Exemplar Problem)
Lit < Nat < K* < Rb* < Cs*

»> <4
(ii) Lithium is the only alkali metal to form nitride
directly
Long Answer Questions Carrying 5 marks
(iit) LiF is least soluble among the fluorides of alkali
metals. . What are alkali metals ? Explain their occurrence and
(iv) E®° for M?* (aq) + 2e-——> M (s) (where M = Ca, Sr discuss their important chemical properties.
. Explain the following properties of alkaline earth metals :
or Ba) is nearly constant. N.C-E.R.T) (t) Tendency to give characteristic flame colouration
. What happens when
(i) calcium nitrate is heated
(ii) Action with air
(ii) chlorine reacts with slaked lime
(iii) Action with water
(iii) quicklime is heated with silica (iv) Thermal stability of sulphates.
(iv) magnesium is burnt in alr. . What is diagonal relationship ? What is its cause ?
45. Give an account of industrial uses of lime and limestone. Discuss any five points to show diagonal relationship
. The alkali metals follow the noble gases in their atomic between lithium and magnesium.
structures. What properties of these metals can be . Name the important ores of lithium. Explain its
predicted from this information. extraction from its ores. Give important properties and
45. Give the chemistry of extraction of magnesium by uses of lithium.
Down's process ? . Name the important ores of sodium. How 1s it extracted
. Describe three industrial uses of caustic soda. Describe from its ores ? Explain important properties and its
one method for the manufacture of sodium hydroxide.
What happens when sodium hydroxide reacts with
. The first member of each group differs from its congeners.
(i) S10, (i) CO, (ii) Al metal
Why ? Discuss by taking a specific example.
AT. Commercial aluminium always contains some
magnesium. Name two such alloys of aluminium. . Describe in detail the manufacture of sodium carbonate
What properties are imparted by the addition of by Solvay process. State the principles involve in the
magnesium to these alloys ? process.
. List four properties of Li in which it differs from rest . How does magnesium occur in nature? How is
of the family members. Magnessium metal obtained by the electrolysis
49, List four properties of Be in which Be differs from rest method?
of the family members. . Name a few important uses of the following compounds:
. Give two important uses of each of the following :
(i) Quicklime (ii) Caustic soda (1) Sodium carbonate (iz) Epsom salt
(iit) Sodium carbonate. (iii) Quick lime (iv) Plaste of Paris
STi08
10. Describe the preparation of each of the following
MODERN'S abe + OF CHEMISTRY-AI

(iii) Tendency to give flame colours by alkali metals.


starting from limestone. (iv) Importance of lime and limestone.
(z) Slaked lime (iz) Calcium carbide 16. How would you explain :
(tit)Plaster of Paris (i) BeO is insoluble but BeSO, is soluble in water.
11. What is cement ? What is its composition ? How is it (ii) BaO is soluble but BaSO, is insoluble in water.
manufactured ?
(iii) Lil is more soluble than KI in ethanol.
12. What are s—block elements ? Discuss their occurrence.
17. How do the following properties change on moving from
Compare the following properties of alkali metal and
eroup 1 to Group 2 in the periodic table ?
alkaline earth metals :
(i) Atomic size (ii) Ionization enthalpy
(i) Ionization energy (ii) Basicity of oxides
(iii) Density (tv) Melting point
(iii) Basicity of hydroxides.
13. What are s—block elements ? Compare and contrast N.C.E.R.T,
the chemistry of group 1 metals with those of group 2 18. State as to why
metals with respect to (i) lithium on being heated in air mainly forms the
(i) nature of oxides monoxide and not the peroxide.
(ii) polarizing power of cations (ii) an aqueous solution of sodium carbonate gives
(iii)solubility and thermal stability of carbonates alkaline tests.
(iv) reactivity and reducing power. (iii) sodium is prepared by electrolytic method and not
14, Write balanced equations for the reactions between : by chemical method.
(1) Na,O and CO, (it) KO, and water 19. Present a comparative account of the alkali and
(211) BeO + H,SO ri (iv) Ca(OH), - cl alkaline earth metals with respect to the following
characteristics :
(v) BeO + NaOH (vi) Na,O, and water
(i) Tendency to form ionic/covalent compounds
(vit) Ca + H,O (it) Nature of oxides and their solubility in water
15. Give a brief account of the following : (iii) Formation of oxosalts
(t) Blue coloured solutions of alkali metals in liquid (iv) Solubility of oxosalts
ammonia (v) Thermal stability of oxosalts.
(ii) Tendency to form divalent compounds of alkaline (NCERT Exemplar Problem)
earth metals than monovalent.

Hints & Answers


Revision Exercises

Very Short Answer Questions (iz): NaCl (aq) Electrolysis


Na*(aqg)+ Cl(aq)
o : [Noble gas] ns}? H,O —— H*t+0OH-
3 : Sodium is the 7th and potassium is 8th most
abundant element by wt 1n earth’s crust. Na*+e— ——+ Na
te : Increases as we go down the group Na+ Hg ——+> Na/Hg
1 2Na/Hg + 2H,O ——— 2Na0H +H, + Hg
9, : CaCQ,, (CaSO,). 4 H,O
(zit): NaC] ——_> Nat+ Cl
11. : Sodium carbonate
At cathode : Na* + e ———> Na(s)
Li. : Increases down the group
2Na + O, ——-> Na,O,
23. : calcium
24, : K* ion (iv): NH,+H,O+CO, ——+ (NH,HCO,
20. : MgSO, NaCl +NH,CO, ——» NaHCO, +NH,Cl
Short Answer Questions 2NaHCo, > Na,CO,+CO,+H,O
42, (i): 2Ca(NO,), ——> 2CaO+4NO, +0,
21. (i) : 2Na + 2H,O ——> 2Na‘*+ 20H+ H, (ii): 2Ca(OH), + 2Cl, ——> CaCl, + Ca(OCl,
(il): 2Na+O, ——> Na,0O,
+ 2H,O
(wi): Na,O,+2H,O, ———> 2Na* + 20H +H,0,
(tit): Ca0+Si0, 4, CaSi0,
ol. (i) : NaC] S28 _,Nate Cl
Molten (iv): 2Mg+O, ——> 2Mg0O

At cathode : Na* + ee ——> Na(s) 3aMg +N, ——> Mg,N,

At anode: 2Cl — 2e°-——-+ Cl,


s-BLOCK ELEMENTS (ALKALI AND ALKALINE EARTH METALS)

Competition Filo Objective Questions

; of OBJECTIVE TYPE QUESTIONS ;>


STIONSAll. The correct order of mobility of alkali metal
Ate
<0V with only one correct answer
aqueous solution is
ions in

(a) Nat > Kt > Rb* > Lit


Alkali metals (6) Kt > Rbt > Na? > Lit
Al. Which of the following is most basic ? (c) Rb*t > Kt > Na*> Lit
(a) CsOH (b) KOH (d) Lit > Kt > Na* > Rbt* (C.B.S.E. Med. 2006)
(c) LiOH (d) RbOH A12. When sodium burns in excess air, the main product
A2. Lithium shows diagonal relationship with formed is
(a) Beryllium (b) Magnestum (a) sodium oxide
(ec) Calcium (d) Boron (b) sodium peroxide
A3. Down’s process is used for the extraction of (c) sodium superoxide
(a) Na (6) Li (d) sodium oxide and sodium nitrate
(c) Ba (d) Mg
Alkaline earth metals
A4, Carnallite is :
(a) KC] MgCl,.6H,O (6) Na,AIF, A138. Which of the following alkaline earth metals does not
impart characteristic colour to the flame ?
(c) Ca,B,O,,. 2H,O (d) Ca,Mg,Si,0,,(0H),
(a) Be (b) Ca
A5. Solvay process is used for the manufacture of (c) Ba (d) Sr
(a) NaOH (b) Na,CO, Al4. Which of the following alkaline earth metal ion has
(c) NH, (d) NaCl smallest mobility in aqueous solution ?
A6. Which of the following is not an ore of lithium? (a) Mg (b) Ca**
(a) Petalite (b) Triphylite (c) Sr* (d) Ba**
Al15. Beryllium exhibits diagonal relationship with
(c) Albite (d) Spodumene
(a) Boron (b) Aluminium
A7. Which of the following 1s radioactive alkali metal ? (c) Magnesium (d) Silicon
(a) Fr (b) Ra A16. Which of the following is most stable ?
(ec) At (d) Rn (a) BeCO, (b) MgCO,
A8. The stability of the following alkali metal chlorides (c) SrCO, (d) CaCO,
A17. Magnesium is present in
follows the order :
(a) Haemoglobin (6) Chlorophyll
(a) LiCl > KCl > NaCl > CsCl
(c) Vitamin B,, (d) Ascorbic acid.
(b) CsCl > KCl > NaCl > LiCl
A18. Quicklime is:
(c) NaCl. KCl. LiCl . CsCl
(a) Ca(OH), (b) CaCO,
(d) KCl > CsCl > NaCl = LiCl (c) CaO (d) CaSO,
A9. The metallic lustre exhibited by sodium is explained A19. Slaked lime reacts with chlorine to give
by (a) CaCl, (b) CaO
(a) diffusion of sodium ions
(c) CaOCl, (d) CaCO,
(b) oscillation of loose electrons A20. The by-product in Solvay ammonia process 1s
(c) excitation of free electrons (a) Carbon dioxide (b) Ammonia
(d) existence of body centred cubic lattice (c) Calcium chloride (d) Calcium carbonate
Al10. On dissolving moderate amount of sodium metal in . Epsom salt is
liquid ammonia at low temperature, which one of the (a) Na,SO,.10 H,O (b) FeSO,.7H,O
following does not occur ?
(c) MgSO,.7H,O (d) MgCl,.7H,0
(a) Blue coloured solution is obtained
. Which of the following has largest solubility in water ?
(b) Nat ions are formed in the solution
(a) Mg(OH), (b) Ca(OH),
(c) Liquid ammonia becomes good conductor of electricity
(c) Ba(OH), (d) Sr(OH),
(d) iquid ammonia remains diamagnetic.

reswees
Al. (a) A2. (5) AS. (a) Ad4d.(a) AB. (6) AG. (c) AZT. (a) A8&. (a) AM (b) AlO. (d) All. (e)
Al2, (b) Al3. (a) Al4, (a) Al15. (6) Al16, (c) A117. (6) A18. (c) Al9. (c) A20. (c) A21, (c) A22, (c)
ST00. MODERN'S abe + OF CHEMISTRY-AI

EU

A23. Which of the following is not an ore of magnesium ? A27. The following compounds have been arranged in order
(a) Epsom salt (b) Dolomite of their increasing thermal stabilities. Identify the
(c) Asbestos (d) Gypsum correct order :
A24, Which of the following readily forms nitride ? K,CO,(1), MgCO,(ID, CaCO,(II), BeCO, (TV)
(a) K (b) Mg (a) l<I<IT<IV (6) IV<I<II1<I
(c) Ba (d) Ca (c) WeT<I1l<Ii (7) W<IV<I1<I
A25. Gypsum on heating to 390 K gives . Which of the following statements about alkaline
(a) CaSO,.2H,O (b) CaSO, earth metals are correct?
1. yuna enthalpy of Sr** is greater than that of
(c) CaSO,. 5H,O (d) SO, and CaO Be?*
A26. Among the alkaline earth metals, the element forming 2. CaCO, decomposes at a higher temperature than
predominantly covalent compound is BeCO,
(a) Barium (b) Strontium 3. Ba(OH), is a stronger base than Mg(OH),
(c) Calcium (d) Beryllium 4. SrSO, is less soluble in water than CaSOQ,.
Select the correct answer using the codes given below:

ATHENS
(a) 4only (6b) land3
(c) Band4 (d) 3and4

A238. (d) A24. (b) A25. (c) A26.(d) A227. (6) A228. (d)

(a) CaO (b) Si0,


MULTIPLE CHOICE QUESTIONS (c) BeO (dq) B,O, (C.B.S.E. PMT 2009)
from competitive examinations Which of the following does not react with water even
under red hot condition ?
AIPMT & Other State Boards’ Medical Entrance (a) Na (b) Be (c) Ca (d) K
(e) Sr (Kerala PMT 2009)
Bl. The sequence of ionic mobility in aqueous solution is :
(a) Kt > Na* > Rb* > Cst B7. Which of the following on thermal decomposition yields
a basic as well as acidic oxide?
(b) Cs* > Rb*+ > Kt > Nat
(c) Rb* > Kt > Cst > Nat
(a) KCIO, (b) Na,CO,
(d) Na* > K* > Rb* > Cs* (C.B.S.E, Med. 2008)
(c) NaNO, (d) CaCO,
B2. Thealkali metals form salts like hydrides by the direct
(ec) NH,NO, (Kerala PMT 2008)
synthesis at elevated temperature. The thermal B8. Which among the following is kinetically inert towards
stability of these hydrides decreases in which of the water ?
(a) Na (b) Be
following order ?
(c) Ca (d) K (e) Sr (Kerala PMT 2010)
(a) CsH > RbH > KH > NaH > Li
B9. The property of alkaline earth metals that increases
(b) KH > NaH > LiH > CsH > RbH with their atomic number is
(c) NaH > LiH > KH > RbH> CsH (a) Electronegativity
(d) LiH > NaH > KH > RbH > CsH (b) Solubility of their hydroxides in water
(C.B.S.E. Med. 2008) (c) Solubility of their sulphates in water
B3. Equimolar solutions of the following were prepared in (d) Ionization enthalpy (C.B.S_E. Med. 2010)
water separately. Which one of the solutions will B10. Which of the following alkaline earth metal sulphates
record the highest pH? has hydration enthalpy higher than the lattice
(a) SrCl, (b) BaCl, enthalpy ?
(c) MgCl, (d) CaCl, (C.B.S.E. Med. 2008) (a) SrSO, (b) CaSO,
B4, In case of alkali metals, the covalent character (c) BeSO, (d) BaSO, (C.B.S_E. Med. 2010)
decreases in the order Bll. The increasing order of the density of alkali metals is
(a) MF > MCI > MBr= MI (a)Li<K<Na<Rb<Cs
(b)Li<Na<K<Rb<Cs
(b) MF > MCl > MI > MBr
(c) Cs< Rb<Na<Ke<Li
(c) MI > MBr > MCl > MF (d)Cs<Rb<K<Na<Li
(d) MC] > MI > MBr > MF (C.B.S.E. PMT 2009) (e) Liz Na<Rb<K<Cs (Kerala PMT 2011)
B5. Which of the following oxides is not expected to react B12. The alkali metal halide that is soluble in pyridine is
with sodium hydroxide ?
(a)NaCl (6) LiCl (c) KCl (d) CsI

B1. (5) B2. (d) B38. (b) B4, (c) BS. (a) B6. (e) B7. (d) B8. (6) B9. (6) B10. (c)
Bll. (a) B12. (bd)
s-BLOCK ELEMENTS (ALKALI AND ALKALINE EARTH METALS)

B18. When Br, is treated with aqueous solutions of NaF, (c) Na,SO, (d) 11,50,
NaCl and Nal separately ? (e) NaNO, (Kerala P.M.T, 2015)
(a) F,, Cl, and I, are liberated B22. Solubility of the alkaline earth metal sulphates in
(b) only F, and Cl, are liberated water decreases in the sequence:
(c) only I, is liberated (a) Sr > Ca > Mg > Ba
(d) only Cl, is liberated (6) Ba> Mg > Sr> Ca
(e) only Cl, and I, are liberated (Kerala PMT 2011) (c) Mg >Ca>Sr>Ba
B14. Which one of the following is present as an active (d) Ca>Sr>Ba> Mg (A..P.M.T. 2015
ingredient in bleaching powder for bleaching action ? B24, The function of "sodium pump" is a biological process
(a) CaOCl, (b) Ca(OC), operating in each and every cell of all animals. Which
(c) CaO,Cl (d) CaCl, (AIPMT 2011) of the following biologically important ions is also a
constituent of this pump?
B15. Which of the following compound has the lowest
melting point ? (a) K* (b) Fe?*
(a) CaCl, (6b) CaBr, (ce) Ca?* (d) Mg** (A..P.M.T. 2015)
(c) Cal, (d) CaF, (AI PMT 2011) B25. On heating which of the following releases CO, most
easily?
B16. Choose the incorrect statement in the following
(a) BeO is almost insoluble but BeSO, is soluble in water. (a) MgCO, (6) CaCO,
(b) BaO is soluble but BaSO, is insoluble in water. (c) K,CO, (d) Na,CO, (A.L.P.M.T. 2015)
(c) Lil is more soluble than KI in ethanol B26. Which of the following statement is false?
(a) Ca** lons are not important in maintaining the
(d) Both Li and Mg form solid hydrogen carbonates.
regular beating of the heart.
(e) Both LiCl] and MgCl, are deliquescent. (b) Mg** ions are important in the green parts of the
(Kerala P.M.T, 2012) plants.
B17. The correct formula of plaster of Paris is (c) Mg** ions form a complex with ATP.
(a) CaSO,.2H,O (6) CaSO,.H,O (dq) Ca** ions are important in blood clotting.
(c) Ca,Si0,.2H,O (d) Ca,Si0,.H,O (NEET 2016)
1
B27. The suspension of slaked lime in water is known as
(e) CaSO,. 3 H,0 (Kerala P.M.T. 2012) (a) lume water (b) quick lime
(c) milk of lime
B18. Which one of the alkali metals, forms only, the normal
oxide, M,O on heating in air ? (d) aqueous solution of slaked lime. (NEET 2016)
(a) Rb (b) K B28. In context with beryllium, which one of the following
statement is incorrect?
(c) Li (d) Na (A.I_P.M-T, 2012)
(a) It is rendered passive by nitric acid
B19. Which of the following is the weakest base?
(6) It forms Be,C
(a) Ca(OH), (b) KOH (c) Its salts rarely hydrolyse
(c) Li(OH) (d) Sr(OH), (A.M.U. Med. 2013) (d) Its hydride is electron—deficient and polymeric
B20. Which one of the following is a strong base? (NEET 2016)
(a) NaOH (b) KOH B29. Ionic mobility of which of the following alkali metal
(c) Ca(OH), (d) Mg(OH), (A.M.U. Med. 2013) ions 1s lowest when aqueous solution of their salts are
B21. The element responsible for the neuromuscular put under an electric field ?
function in the body is (a) K (6b) Rb
(a) calclum (b) magnesium (e) Li (d) Na (NEET 2017)
(c) potassium (d) sodium B30. Which of the following oxides is most acidic in nature?
(ec) Manganese (Kerala P.M.T. 2015) (a) MgO (b) BeO
B22. The salt of an alkali metal gives yellow colour in the (c) BaO (d) CaO (NEET 2018)
flame test. Also its aqueous solution gives an insoluble B31. Among CaH,, BeH,, BaH.,, the order of ionic character
white precipitate with barium chloride in acidic 15
medium. The salt is (a) BeH, < CaH, < BaH,
(a) NaCl (b) K,SO, (b) CaH, < BeH, < BaH,
(c) BeH, < BaH, < CaH,
Araswets (d) BaH, < BeH, < CaH, (NEET 2018)

B13. (c) B14. (b) B15. (c) B16. (d) B17. (ec) B18. (c) B19. (c) B20. (6) B21. (a) B22. (c)
B23. (c) B24. (a) B25. (a) B26. (a) B27. (c) B28. (c) B29. (c) B30. (b) B31. (a)
Sloe MODERN'S abe + OF CHEMISTRY-AI

(c) SrCO, > BaCO, > CaCO, > MgCO,


JEE Main & Other State Boards’
Engineering Entrance (d) CaCO, > MgCO, > BaCO, > SrCO,
B32. Beryllium and aluminium exihibit many properties (Orissa JEE 2009)
which are similar. But, the two elements differ in B41. Which of the following oxides is most acidic in nature ?
(a)
forming covalent halides (a) BeO (b) MgO
(b)
forming polymeric hydrides (c) CaO (dq) BaO (A.M.U. Engg. 2009)
(c)
exhibiting maximum covalency in compounds B42. Formula of gypsum is
(d)
exhibiting amphoteric nature in their oxides.
(A.LELELE. 2004) (a) CaSO,2H,O (b) CaSO,.=H,0
B33. One mole of magnesium nitride on reaction with excess (c) 3CaSO,.H,O (d) 2CaSO,.2H,O
of water gives (Orissa JEE 2010)
(a) one mole of ammonia B43. Which of the following metals has the largest
(b) one mole of nitric acid abundance in the earth's crust ?
(c) two moles of ammonia (a) Aluminium (b) Calcium
(d) two moles of nitric acid (ALLELE. 2004)
(c) Magnesium (d) Sodium
. Based on the lattice enthalpy and other considerations
(West Bengal JEE 2010)
which one of the following alkali metal chlorides is
expected to have highest melting point. B44, The metal that produces red-violet colour in the non-
(a) LiCl (b) NaCl luminous flame is
(c) KCl (d) RbCl.(ALBEE. 2005) (a) Ba (b) Ag (c) Rb (d) Pb
(e) Zn (Kerala PET 2010)
B35. The ionic mobility of alkali metal ions in aqueous
solution is maximum for B45. The correct order of reducing character of alkali
metals is
(a) Rb* (b) Lit
(a) Rb <Ke< Na<Li (b) Lic Na<KeRb
(c) Nat (d) Kt (A.LE_ELE. 2006)
(ce) Na<Ke<Rb<Li (dq) Rb<Na<KeLi
B36. Which of the following is formed by the action of (J.K. CET 2011)
water on Na,O, ?
B46. Among the following, the compound that is readily
(a) H, (b) O, soluble in water is
(c) N, (d) COQ, (B.H.U, 2007) (a) BeSO, (b) CaSO,
B37. Which one of the following processes is used for the
manufacture of calctum?
(c) SrSO, (d) BaSO, (JK. CET 2011)
B47. Which of the following represents the composition of
(a) Reduction of CaO with carbon Carnallite mineral ?
(6b) Reduction of CaO with hydrogen
(ce) a of a mixture of anhydrous CaCl, and
(a) K,0-Al,0,'6Si0,
(b) KNO,
(d) Electrolysis of molten Ca(OH), (WB-JEE-2008)
(c) K,SO,MgSO,-MgCl,-6H,O
B38. RbO, is (d) KCl-MgCl,-6H,O (West Bengal JEE 2011)
(a) Peroxide and paramagnetic B48. Which one of the following on hydrolysis, gives the
corresponding metallic hydroxide, H,O, and O,?
(b) Peroxide and diamagnetic
(c) Superoxide and paramagnetic
(a) Li,O (b) Na,O,
(c) NaO, (d) Na,O
(d) Superoxide and diamagnetic (Orissa JEE 2008) (e) BeO (Kerala PET 2011)
B39. Which pair of the following chlorides do not impart B49. Be and Al exhibit diagonal relationship. Which of the
colour to the flame? following statements about them is/are not true ?
(a) BeCl, and SrCl, (i) Both react with HC] to liberate H,.
(b) BeCl, and MgCl, (ii) They are made passive by HNO,.
(ec) CaCl, and BaCl, (iii) Their carbides give acetylene on treatment with
(d) BaCl, and SrCl, water.
(e) MgCl, and CaCl, (Kerala PET 2008) (iv) Their oxides are amphoteric.
B40. Correct order of stability of group II Ametal carbonates (a) (tii) and (iv) (b) (i) and (iii)
1s (c) (z) only (d) (ii) and (iit)
(a) MgCO, > CaCO, > SrCO, > BaCO, (e) (iii) only (Kerala PET 2011)
(b) BaCO, > SrCO, > CaCO, > MgCO,

nswees
B32. (c) B33. (c) B34. (5) B37. (c) B38. (c) B39. (6) B40. (5) B41. (a)
B42. (a) B44, (c) B45. (c) B46. (a) B47. (d) B48. (c) B49. (e)
s-BLOCK ELEMENTS (ALKALI AND ALKALINE EARTH METALS)

B50. Which one of the following order presents the correct The salt is
sequence of the increasing basic nature of the given (a) KSO, (b) KCl
oxides ? (c) Na,SO, (d) K,CO,
(a) Al,O, < MgO < Na,O < K,O (e) Li,SO, (Kerala PET 2014)
(b) MgO < K,O < Al,O, < Na,O B59. What is the product of reaction between calcium
(c) Na,O < K,0 < MgO < ALO, carbide and water?
(d) K,O <Na,O < Al,O, < MgO (AIFEE 2011) (a) Ethylene (b) Acetylene
(c) Methane (d) Benzene (J.K. CET 2014)
B51. The product obtained on heating LiNO, will be
B60. The correct statement for the molecule CsI, is
(a) Li,0+NO,+0, (b) Li,N+0O, (a) It contains Cs‘, I and lattice I, molecule
(c) i,O+NO+0, (d) LiNO,+0, (AIEEE 2011) (b) It is a covalent molecule.
B52. The strongest base among the following is (c) It contains Cs* and I> ions
(a) NaOH (b) KOH (d) It contains Cs* andIT-ions. (JEE Main 2014)
B61. The decreasing order of basic character of K,O,
(c) LiIOH (d) CsOH (A.M.U. Engg 2012)
BaO, CaO and MgO is
B53. Among the following compounds, the one that gets (a) KO > BaO > CaO > MgO
hydrolysed to form metallic hydroxide, hydrogen (b) K,0 > CaO > BaO > MgO
peroxide and oxygen is (ec) MgO > BaO > CaO > K,O
(a)Na,O (6)Na,O, (c) Li,O (d) 11,0, (d) MgO > CaO > BaO > K,O (WB JEE 2015)
(e) KO, (Kerala P.E.T. 2012) B62. Match the flame colours of the alkaline earth metal
salts in the Bunsen burner.
B54. The alkaline earth metal with the least density value is
(p) Cale1um 1. Brick red
(a) Mg (b) Be (g) Strontium 2. Apple green
(c) Sr (d) Ca (r) Barium 3. crimson
(e) Ba (Kerala P.E.T. 2012) (a) p—1, g-3, r-2 (b) p-3, g-l, r-2
B55. The reaction between sodium and water can be made (c) p-2, q-3, r-1 (d) p-1, g-2, r-3
less vigorous by (WB JEE 2015)
B63. Which one of the following alkaline earth metal
(a) lowering the temperature
sulphates has its hydration enthalpy greater than
(b) adding a little alcohol its lattice enthalpy?
(c) amalgamating sodium (a) BaSO, (6) SrSO,
(d) adding a little acetic acid (c) CaSO, (d) BeSO, (WB JEE 2015)
B64. Which of the following arrangement is correct in
(Karnataka C.E.T, 2012) respect of solubility in water?
B56. Alkali metals have negative reduction potential and (a) CaSO, > BaSO, > BeSO, > MgSO, > SrSO 4
hence they behave as (6b) BeSO, > MgSO, > CaSO, > SrSO, > BaSO,
(a) oxidising agents (6b) Lewis bases (c) BaSO, > SrSO, > CaSO, > MgSO, > BeSO,
(c) reducing agents (d) electrolytes (d) BeSO, > CaSO, > MgSO, > SrSO, > BaSO,
(WB JEE 2016)
(Karnataka C.E.T. 2013)
B65. Which halide of magnesium has highest ionic
B57. Which of the following statement is false regarding character?
alkali metals? (a) Chloride (b) Bromide
(a) Alkali metals are soft and can be cut with the help (c) Iodide (d) Fluoride (MH CET 2016)
of a knife. B66. The low solubility of LiF and that of CsCl in water are
(6) Alkali metals donot occur is free state in nature. respectively due to which of the properties of the alkali
metal ions?
(c) Alkali metals are highly electropositive elements.
(a) Higher hydration enthalpy of Li‘, higher lattice
(d) Alkali metal hydrides are covalent in character. enthalpy of Cs*
(J.K. CET 2013) (6) Smaller hydration enthalpy of Li‘, higher lattice
. The salt of alkali metal gives violet colour in the flame enthalpy of Cs*
test. Its aqueous solution gives a white precipitate (c) Smaller lattice enthalpy of Li-, higher hydration
with barium chloride in hydrochloric acid medium. enthalpy of Cs*
(d) Smaller hydration enthalpy of Li*, smaller
lattice enthalpy of Cs*

Answers
(e) Higher lattice enthalpy of Li*, smaller hydration
enthalpy of Cs*. (Kerala PET 2016)

B50. (a) B51. (a) B52. (d) B53. (e) B54. (d) B55. (c) B56. (c) B57. (d) B58. (a) B59. (b)
B60. (c) Bol. (a ) B62. (a) B65. (d) B66. (e)
S Tiose
B67. The second ionization enthalpy of which of the following
MODERN'S abe + OF CHEMISTRY-AI

B74. Portland cement does not contain


alkaline earth metals is the highest? (a) CaSi0, (6) CaSi0,
(a) Ba (b) Mg (ec) Ca,AL,0, (d) Ca,(PO,),
(c) Ca (d) Sr (e) Both (c) and (d) (Kerala P.E.T. 2018)
(e) Be (Kerala PET 2016) B75. Which of the following is least thermally stable ?
B68. The main oxides formed on combustion of Li, Na and (a) MgCO, (b) CaCO,
K in excess of air are, respectively
(c) SrCO, (d) BeCO,
(a) Li,O, Na,O and KO,
(W.B. J.E_E. 2018)
(b) LiO,, Na,O, and K,O
B76. A metal on combustion in excess air forms X. X upon
(c) Li,O,, Na,O, and KO,
hydrolysis with water yields H,O, and O, alongwith
(d) Li,O, Na,O, and KO, (JEE Main 2016)
other product. The metal is
B69. Plaster of Paris is represented as
(a) Rb (6) Li
(2) CaSO,.2H,O —_(b) CaSO,.H,O
(c) Mg (d) Na (JEE Main 2019)
1 B77. The correct statement(s) among I to III with respect
(ce) CaSO,.5H,O —(d) CaSO, to potassium ions that are abundant within the cell
(Karnataka CET 2017)
fluids is/are :
I. They activate many enzymes
B70. Which of the following is covalent?
II. They participate in the oxidation of glucose to
(a) NaCl (b) KCl
produce ATP.
(c) BeCl, (d) MgCl, III. Alongwith sodium ions, they are responsible for
(e) CaCl, (Kerala PET 2017)
the transmission of nerve signals.
B71. One mole of an unknown compound was treated with
(a) I, II and III (b) I and III only
excess water and resulted in the evolution of two moles
(c) II only (d) I and II only
of a readily combustible gas. The resulting solution
(JEE Main 2019)
was treated with CO, and resulted in the formation of
B78. The amphoteric hydroxide is
white turbidity. The unknown compound is
(a) Ca(OH), (b) Be(OH),
(a) Ca (6b) CaH,
(c) Ca(OH), (d) Ca(NQ,),
(ec) Sr(OH), (d) Mg(OH),
(JHE Main 2019)
(e) CaSO, (Kerala PET 2017)
B72. Both lithium and magnesium display several similar JEE (Advance) for ITT Entrance
properties due to the diagonal relationship; however,
the one which is incorrect Is : B79. Which of the folowing process is used in the extraction
(a) Both form basic carbonates of metallurgy of magnesium ?
(6b) Both form soluble biocarbonates (a) fused salt electrolysis
(c) Both form nitrides (b) salt reduction
(d) Nitrates of both Li and Mg yield NO, and O, on
(c) aqueous solution electrolysis
heating (JEE Main 2017)
(d) thermite reduction. (LL.T. 2002)
B78. Dead burnt plaster is
B80. Ammoniacal solution of MgSO, m presence of NH,Clis
(a) CaSO,,. (6) CaSO, =H,0 heated with Na,HPO,, a white precipitate is formed of
(a) Mg(NH,)PO, (6) Mg,(PO,),
(c) CaSO,.H,O (d) CaSO,.2H,O
(Karnataka CLE.T. 2018) (c) MgSO,.MgCl, (d) MgSO,.MgPO, (LIT. 2006)

Arawets
B67. (e) B68. (d) B69. (c) B70. (c) B71. (b) B72. (a) B73. (a) B74. (d) B75. (d) B76. (a)
B77. (a) B78. (6) B79. (a) B80. (a)

(c) forms sodium amide


(d) produces H, gas
with more than one correct answers
C2. Which of the following is not correct order of the
C1. Highly pure dilute solution of sodium in hquid ammonia property mentioned?
(a) shows blue colour (a) Thermal stability: MgCO, < CaCO, < BaCO,
(6) exhibits electrical conductivity (6) Reactivity with O,: Be <Mg<Ca

Arswcs
(c) Solubility in water : MgSO, < CaSO, < SrSO,
(d) Enthalpy of formation : CaO < SrO < BeO

Cl. (a, 5) C2. (c, d)


s-BLOCK ELEMENTS (ALKALI AND ALKALINE EARTH METALS)

C3. Which of the following donot impart colour to the C8. Which of the following statements is/are correct.
flame? (a) Berylliumis not readily attacked byacids because
(a) Be (b) Mg i) Ga (d)Ba So of an oxide film on the surface ofthe
metal.
C4, Cal d be usedfor drying of
, P =< —S es ee (b) Beryllium sulphate is readily soluble in water as
(a) Hy (0) 3 the greater hydration enthalpy of Be” overcomes
(ec) CO, (d) Cl, the lattice enthalpy factor.
C5. Which of the following statements are correct? (c) Beryllium exhibits coordination number more

(a) Sodium is most abundant among alkali metals. than four.


(6) Among group I elements, radium is the only (d) Beryllium oxide is purely acidic
in nature.
radioactive element. C9. The alkali metals are characterised by which of the
(c) Lattice energy of Nal is more than that of NaBr rouawIn Le ge ?
(d) The mobility of Lit in water is less than Nat. (a) High as poy
C6. Which of the following statements are incorrect? (0) Hen gensity
(a) CaH, is also called hydrolith (c) High nega 4 aaa eiarote poental
(b) BeCl, in vapour phase exists as polymeric (d) pause i ail .
: : — C10. Which of the following compounds of sodium are used
(c) MgsoO, 1s less soluble than CaSO, 1s water eextilaiAdustry ?
(d) a, decomposes at lower temperature than (a). Na,CO, (b) NaHCO,

C7. Which of the following compounds are readily soluble (c) BYAOH | (¢) NaCl
in water ? C11. The compound (s) formed upon combustion of sodium
(a) BeSO, (b) MgSO, metal in excess of al ae (are)

(c) SrSO, (d) BaSO, a) ee on ea


fi (c) NaO, (d) NaOH (LIT. JEE 2009)

C3. (a,b) C4. (a,b) C5. (a,d) C6. (b,c) C7. (a,b) C8. (a,b) C9. (c,d) C10. (a,c) Cl1l1.a, db)

- ; 3. Assertion : Al(OH), is amphoteric in nature.


Assertion Reason Type Questions Reason _: Al-O and O-H bonds can be broken with
equal lease in Al(OH).
The questions given below consists of an Assertion and 4, Assertion : Barium is not required for normal
the Reason. Use the following key to choose the biological function in human.
appropriate answer. Reason _ : Barium does not show variable oxidation
(a) Ifboth assertion and reason are CORRECT and reason state. (A.LILM.S. 2003)
is the CORRECT explanation of the assertion. 5. Assertion : Li resembles Mg.
(b) Ifboth assertion and reason are CORRECT, but reason Reason — : Li has same size as Mg”.
is NOT THE CORRECT explanation of the assertion. 6. Assertion : Alkali metals impart colour to the flame.
(c) Ifassertion is CORRECT but reason is INCORRECT. Reason _: Their ionization energies are low.
(d) Ifassertion is INCORRECT but reason is CORRECT. 7. Assertion : Sulphur is estimated as BaSO, and not
(e) If both assertion and reason are INCORRECT. rm MgSO,
Reason — : The ionic radius of Mg is smaller than
1. Assertion : LiClis predominantly covalent compound. that of Ba2«.
Reason — : The electronegativity difference between 8. Assertion : BeCl, is covalent whereas MgCl, and
Li and Cl is too small. CaCl, are ionic.
2. Assertion : Na,SO, is soluble in water while BaSO, Reason — : Beryllium is the first member of the
is insoluble. group.
Reason — : Latticeenergyof barium sulphate exceeds 9. Assertion : Beryllium can form BeF, while
its hydration energy. aluminium can also form AlF,* ion.
Reason : Beryllium does not have vacant
Araswecs d-orbitals in its outermost shell.

l(c) 2. (a) 3. (a) 4, (b) 5. (a) 6. (a) q (6) 8. (b) 9. (a)


ee 10/56 MODERN'S abe + OF CHEMISTRY-XI

10. Assertion : K, Rb and Cs can also form superoxides. 13. Assertion : Beryllium carbonate is kept in the
Reason — : Their ionic radii increase in the order atmosphere of carbon dioxide.
Cs* < Rbt < Kr’. Reason — : Beryllium carbonate is unstable and
11. Assertion : Cu,Cl, is more covalent than NaCl. decomposes to give beryllium oxide and
Scliong _ :' carbon dioxide.
Reason —: Cu*1on 1s more polarizing than Na* ion.
14, Assertion : The carbonate of lithium decomposes
12. Assertion : Be gives characteristic flame colouration.
— 4 orm easily on heating to form lithium oxide
Reason — : lomzation energy of Be is high. and CO
a
Reason — : Lithium beingverysmallin size polarises
large carbonate ion resulting the
A nswets formation of more stable Li,O and CO.,,.

10. (e) 11. (a) 12, (d) 13. (a) 14. (a)

Matrix Match Type Questions 2; Match the compounds given in Column I with their
uses given in Column II.
1. Match the element in Column I with the properties
mentioned in Column II
(A) CaCO, Dentistry, ornamental work
(B) Ca(OH), Manufacture of sodium
carbonate from caustic soda
(A) Sodium (p) belong to third period
(C) CaO r) Manufacture of high
(B) Magnesium | (q) form insoluble sulphate
quality paper
(C) Bartum (r) give characteristic flame colours (D) CaSO, s) Used in white washing
(D) Lithium (s) form stable nitride

(aaa ~
1. (A) —-(p), (r);
(B) -(p), (s) ; (C) —(@), (r) ; (D) -(s) 2. (A) -(r); (B) -(s) ; (C) -—(q); (D) -(p).

Exemplar Problems //
»>» Multiple Choice Questions (Type-l) Obj ective Qu estions

1. The alkali metals are low melting. Which of the 3. The reducing power of a metal depends on various
following alkali metal is expected to melt ifthe room factors. Suggest the factor which makes Li, the
temperature rises to 30°C ? strongest reducing agent in aqueous solution.
(a) Na (b) K (a) Sublimation enthalpy
(c) Rb (d) Cs (b) Ionisation enthalpy
2. Alkali metals react with water vigorously to form (c) Hydration enthalpy
hydroxides and dihydrogen. Which of the following (d) Electron-gain enthalpy
alkali metal reacts with water least vigorously ? 4, Metal carbonates decompose on heating to give metal
(a) Li (b) Na oxide and carbon dioxide. Which of the metal
(c) K (d) Cs carbonates is most stable thermally ?
(a) MgCO, (b) CaCO,
Arasogaes (c) SrCoO, (d) BaCO,

1. (d) 2. (a) 3. (c) 4, (d)

WWW.JEEBOOKS.IN
s-BLOCK ELEMENTS (ALKALI AND ALKALINE EARTH METALS) Uys >

5. Which of the carbonates given below is unstable in air 15. Suspension of slaked lime in water is known as
andis kept in CO, atmosphere to avoid decomposition. (a) ee
(b) MgCo, (b) quick lime
(a) BeCO, (c) milk of ime
(c) CaCO, (d) BaCO, (d) aqueous solution of slaked lime
6. Metals form basic hydroxides. Which of the following 16. Which of the following elements does not form hydride
metal hydroxide is the least basic ? by direct heating with dihydrogen ?
(a) Mg(OR), (b) Ca(OH), (a) Be (b) Mg
(c) Sr(OH), (d) Ba(OH), (c) Sr (d) Ba
7. Some of the Group 2 metal halides are covalent and 17. The formula of soda ash is
soluble in organic solvents. Among the following (a) Na,CO,.10H,O (6) Na,CO,.2H,O
metal halides, the one which is soluble in ethanol is (c) Na,CO,.H,O (d) Na,CO,
(a) BeCl, (6) MgCl, 18. A substance which gives brick red flame and breaks
(c) CaCl, (d) SrCl, down on heating to give oxygen and a brown gas is
8. The order of decreasing ionisation enthalpy in alkali (a) Magnesium nitrate (b) Calcium nitrate
metals is (c) Barium nitrate (d) Strontium nitrate
(a) Na>Li>K-=>Rb (b) Rb<Na<Ke<Li 19. Which of the following statements is true about
(c) Li>Na>K>Rb ~~ (d) Li<Na<K<Rb Ca(OH),¢
9. The solubility of metal halides depends on their (a) Itis used in the preparation of bleaching powder.
nature, lattice enthalpy and hydration enthalpy of the (b) It is a light blue solid.
individual ions. Amongst fluorides of alkali metals, (c) It does not possess disinfectant property.
the lowest solubility of LiF in water is due to (d) Itis used in the manufacture of cement.
(a) Ionic nature of lithium fluoride 20. A chemical A is used for the preparation of washing
soda to recover ammonia. When CO, is bubbled
(6) High lattice enthalpy through an aqueous solution of A, the solution turns
(c) High hydration enthalpy for lithium ion milky. It is used in white washing due to disinfectant
(d) Low ionisation enthalpy of lithium atom nature. What is the chemical formula of A ?
10. Amphoteric hydroxides react with both alkalies and (a) Ca(HCO,), (0) CaO
acids. Which of the following Group 2 metal (c) Ca(OH), (d) CaCO,
hydroxides is soluble in sodium hydroxide ? 21. Dehydration of hydrates of halides of calcium, barium
and strontium i.e., CaCl,-6H,O, BaCl,-2H,O,
(a) Be(OH), (6) Mg(OH), SrCl,:2H,0O, can be achieved by heating. These bacorne
(c) Ca(OH), (d) Ba(OH), wet on keeping in air. Which of the following
11. In the synthesis of sodium carbonate, the recovery of statements 1s correct about these halides ?
ammonia is done by treating NH,Cl with Ca(OH),. (a) Act as dehydrating agent
The by-product obtained in this process is (b) Can absorb moisture from air
(a) CaCl, (b) NaCl (c) Tendency to form hydrate decreases from calctum
(c) NaOH (d) NaHCO, EEE
12. When sodium is dissolved in liquid ammonia, a (d) All of the above
solution of deep blue colour is obtained. The colour of
the solution is due to
(a) ammoniated electron Multiple Choice Questions (Type-Hl)
(b) sodium ion
(c) sodium amide 22. Metallic elements are described by their standard
(d) ammoniated sodium ion electrode potential, fusion enthalpy, atomic size, etc.
13. By adding gypsum to cement The alkah metals are characterised by which of the
(a) setting time of cement becomes less. following properties ?
(b) setting time of cement increases. (a) High boiling point
(c) colour of cement becomes light. (b) Hieh aa oe ce eee eeeee
id): Ghisiag wariaes is obusied, igh negative standard electrode potentia
: | (c) High density
14. Dead burnt plaster is 1 DL 7
(a) CaSO, (b) CaSO, 5H,O (d) Large atomic size
(c) CaSO,H,O (d) CaSO,2H,O

¢mswecs
5. (a) 6. (a) 7. (a) 8. (c) 9. (b) 10. (a) 11. (a) 12. (a) 13. (6) 14. (a)
15. (c) 16. (a) 1%. (d) 18. (5) 19. (a) 20. (c) 21. (d) 22. (b, d)
S T0e. MODERN'S abc + OF CHEMISTRY
-XI

on File
23. Several sodium compounds find use in industries. Column I Column II
Which of the following compounds are used for textile (i) Li (a) Insoluble sulphate
industry ?
(ii) Na (b) Strongest monoacidic base
(a) Na,CO, (6) NaHCO, (iit) Ca (c) Most negative E° value among alkali
(c) NaOH (d) NaCl metals
Which of the following compounds are readily soluble (iv) Ba (d) Insoluble oxalate
in water ? (e) 6s” outer electronic configuration
(a) BeSO, (6) Mgso,
(c) BaSO, (d) SrsO, 30. Match the compounds given in Column I with their
25. When Zeolite, which is hydrated sodium aluminium uses mentioned in Column II.
silicate is treated with hard water, the sodium ions are Column I Column IT
exchanged with which of the following ion(s)?
(a) H* ions (b) Mg** ions
() Caco, (a) Dentistry, ornamental work
(ii) Ca(OH), (6) Manufacture of sodium carbonate from
(c) Ca?* ions (d) SO ions
caustic soda
26. Identify the correct formula ofhalides ofalkaline earth
metals from the following. (iii) CaO (c) Manufacture of high quality paper
(a) BaCl,.2H,O (6) BaCl,.4H,O (iv) CaSO, (d) Used in white washing
(c) CaCl,.6H,O (d) SrCl,.4H,O 31. Match the element given in Column I with the colour it
27. Choose the correct statements from the following. imparts to the flame given in Column II.
(a) Beryllium is not readily attacked by acids because Column I Column IT
of the presence of an oxide film on the surface of the
(i) Cs Apple green
metal.
(ii) Na (b) Violet
(6) Beryllium sulphate is readily soluble in water as
(tii) K (c) Brick red
the greater hydration enthalpy of Be** overcomes
the lattice enthalpy factor. (iv) Ca (dq) Yellow
(c) Beryllium exhibits coordination number more (v) Sr (e) Crimson red
than four. (vi) Ba (f) Blue
(d) Beryllium oxide is purely acidic in nature.
28. Which of the following are the correct reasons for »»> fissertion and Reason Type Questions <q
anomalous behaviour of lithium ?
32. Assertion (A) : The carbonate of lithium decomposes
(a) Exceptionally small size if its atom. easily on heating to form lithium oxide and COQ,.
(b) Its high polarisig power.
Reason (R) : Lithium being very small in size
(c) It has high degree of hydration. polarises large carbonate ion leading to the formation
(d) Exceptionally low ionisation enthalpy.

>> Le <a
of more stable Li,O and CO,,.
oo. Assertion (A) : Beryllium carbonate is kept in the
atmosphere of carbon dioxide.
29. Match the elements given in Column I with the
Reason (R) : Beryllium carbonate is unstable and
properties mentioned in Column II. decomposes to give beryllium oxide and carbon dioxide.

Answers
23. (a,c) 24. (a,b) 2h (b,c 26. (a,c) 27. (a,c) 28. (a,b,c)
29. (1) —(c); (a) — (6); (it) — (d); (iv) — (a, e) 30. (i) —(e); (ii) — (d); (iii) — (6); (iv) — (a)
31. (7) —(/); (iz) —(d); (tii) — (6); (iv) — (©), (Vv) — (e); (vt) — (a) 32. (a) 33. (a)

nations for
Hints & Expla Difficult Objective Type Questions

A. meq with only one correct answer Al14, (a): Because effective (hydrated) size of Mg** ion is
large.
A10. (d): It forms paramagnetic solution. A19. (c) : Ca(OH), + Cl, ——> CaOCl, + H,O
A138. (a): Be does not give flame colouration because of very
high ionization enthalpy. A25. (c) : CaSO,.2H,O_2%K_, CaSO,.%H,0+ %H,0
s-BLOCK ELEMENTS (ALKALI AND ALKALINE EARTH METALS)

BeH, < CaH, < BaH,


B. meq from competitive examinations
B32. (c) : Be can have maximum covalency of 4 while Al can
Bl. (b): In aqueous solution, because of hydration the order exhibit covalency more than 4 also.
of effective size is : B33. (c) : Mg,N,+3H,O > 3Mg(OH), +2NH,
Na® > K* > Rb’ > Cs" B40. (5): On moving down the group, the size of cation
Greater is size lesser is mobility. increases and therefore, tendency to stability large
BS. (b): Basic character increases down the group. cO,7- ion increases. Hence, stability increases.
B4, (c) : According to Fajan rules, larger the size of anion,
B41. (a): On moving down a group, basic character
greater is the covalent character.
increases and acidic character decreases.
Order of size of anions : I> > Br~- > Cl> > F-
B45. (c) : The reduction potentials of alkali metals are :
“. Covalent character : MI > MBr > MCl > MF
B77. (d): CaCO, ——> CaO + CO, Li(—3.04), Na(—2.72), K(—2.92), Rb(—2.93)
Basic oxide Acidic oxide *. Correct order is Na < K < Rb < Li.
Bil. (a): Density gradually decreases on moving down a B46. (a): Solubility decreases down the group and therefore,
group. However, K is lighter than Na. BeSQ, is readily soluble.
li< K<Na<Rb<Cs B48. (c) : 2NaO, + 2H,O —> 2NaOH + H,O, + O,
B12. (b): LiCl being covalent is soluble in pyridine. B49. (e) : The carbides of both Be and Al react with water
B18. (c) : Br, reacts with Nal only to give I,
and liberate methane.
2Nal + Br, —> 2NaBr + I,
Be,C + 2H,O ——> 2BeO + CH,
B15. (c) : Covalent character increases and melting point
decreases : Al,C, + 64,0 —> 2Al,0, + 3CH,
CaF, > CaCl, > CaBr, > Cal,
B16. (d): Lithium does not form solid bicarbonate, though
B51. (a): LiNO, —“* >Li,O +NO, +0,
it exists in solution. B53. (e) : Superoxides (KO,) react with water to form
B18. (c) : Only lithium forms monoxide on heating in air. hydroxides, hydrogen peroxide and oxygen.
B21. (a): Calcium is responsible for neuromuscular 2KO, + 2H,0 ———> 2KOH + H,O, + O,
function in the body. B57. (d): Alkali metal hydrides are ionic in character.
B22. (c) : Na,SO, + BaCl, ——> BaSO, + 2NaCl B58. (a) : Potassium and their salts give violet colour in the

Insoluble white ppt flame.


Na,SO, gives yellow colour in flame test. K,SO, + BaCl, —> BaSO, + 2KCl
white ppt
B23. (c) : Solubilities of alkaline earth metal sulphates
B59. (5): CaC, + 2H,O —> Ca(OH), + C,H,
decrease down the group. This is because, on
Acetylene
moving down the group, hydration enthalpy
B60. (c) : Cs cannot show +3 oxidation state. Therefore,
decreases with increase in size of the metal ion
CsI, contains Cs* and I, ions.
and concequently, their solubilities decrease.
B61. (a): Alkali metal oxides are strongly basic in nature.
B24, (a): K* is constituent of sodium pump.
Down the group in alkaline earth metals, the
B25. (a): MgCO, —*» MgO +O, basic character increases due to decrease in
Thermal stability follows the order: ionization enthalpy. So, the correct order of basic
K,CO, > Na,CO, > CaCO, > MgCO, character is:
B26. (a): Ca?* ions are important and perform important K,O > BaO > CaO > MgO
biological functions such as maintaining the B63. (d): Be* being smaller in size has maximum hydra-
regular beating of a heart. tion enthalpy which exceeds its lattice enthalpy.
B28. (c) : Beryllium salts rarely hydrolyse due to absence B64. (5): The solubility of alkaline earth metal sulphates
of d-orbitals. decreases down the group because the size of
B29. (c) : Li* is the smallest among these and has metal cation increases due to which hydration
maximum electron density. Therefore, it will be enthalpies decrease.
heavily hydrated and its effective size in aqueous B65. (d): Smaller is the anion, less is the polarisation,
solution will be maximum among all alkali metal more is the ionic character.
ions. Thus, its ionic mobility will be lowest. B68. (d): Li forms oxide, Li,O, Na forms peroxide, Na,O,
B30. (b): Basic character of oxides of group II increases down and K forms super oxide, KO.,.
the group. Hence, BeO is most acidic oxide. B70. (c): Among these BeCl, is covalent and all other are
sla): Smaller the size of cation, more will be its ionic compounds.
polarising power and larger will be its covalent B71. (5): Calcium hydride (CaH,) on reaction with excess
character. Hence, BeH, is least ionic and ionic water evolves two moles of readily combustible
character increases as : H, gas and one mole of calcium hydroxide. The
T0060 MODERN'S abe + OF CHEMISTRY-AI

solution containing Ca(OH), on treatment with BeCO, —— BeO + CO,


CO, gas forms white turbidity due to the Since the decomposition reaction is reversible,
formation of CaCO,. therefore, to increase the stability of BeCO, i.e. to
CaH,(s) + 2H,O——+ Ca(OH), + 2H,(g) reverse the equilibrium, BeCO, is kept in an
Combustible gas atmosphere of CO.,.
Ca(OH),(aq) + CO,(g)
—> CaCO,(s) + H,O . (a): Basic character of the hydroxides of alkaline earth
Calcium carbonate metals increases down the group.
(white turbidity)
. (ar: Due to small size and large electronegativity and
B72. (a): Mg forms basic carbonate like high ionization enthalpy of Be, BeCl, is covalent
3MgCO,. Mg(OH),.3H,O but Li forms only and therefore, more soluble in organic solvents
carbonate (Li,CO,) which is very less basic. such as ethanol.
B73. (a): Anhydrous CaSO, is known as dead burnt plaster. fe) Ionisation enthalpy decreases as atomic size
B74, (d)-: Portland cement does not contain Ca,(PO,),. increases. Thus Li > Na > K > Rb.
B75. (d): For alkaline earth metals, thermal stability of (b): Due to small size of Lit and F- ions, the lattice
metal carbonates increases down the group.
enthalpy of LiF is much higher than hydration
enthalpy and hence it is least soluble among
Therefore, BeCO, is least thermally stable.
alkali metal fluorides.
B76. (a): Rb + O, ——> RbO,
10. (a): Be(OH), is amphoteric and therefore, is soluble in
excess
NaOH.
2RbO, + 2H,O——> 2RbOH + H,0, +O,
11. (a): 2NH,Cl + Ca(OH), —-> 2NH, + CaCl, + 2H,0.
B78. (b): Be(OH), is amphoteric while all other are basic. Therefore, CaCl, is the by product.
B80. (a): Mg** + NH, + HPO,2, ——-> Mg(NH,) PO, 16. (a): Being least reactive, Be does not form hydrides by
White ppt direct heating with dihydrogen.
18. (b): Calcium gives brick red colour and its nitrate
C. mcq with more than one correct answer
breaks down to give O, and brown gas, NO,,.
C11. (a,6): 4Na+ O, —~+ 2Na,O0
2Ca(NO,), —28t_, 2CaO + O, + 4NO,
4Na + 20, ——> 2Na,0,

NCERT Exemplar Problems: MCQ Type I NCERT Exemplar Problems: MCQ Type IT
. (d)- Melting point decreases as the strength of metallic 22.(b,d): Alkali metals have high negative standard
bonding decreases with increasing size of the atom.
electrode potential and large atomic size.
Ns The reactivity of alkali metals increases down the
eroup so that Li is least reactive. 24. (a,b): Solubility decreases down the group because
» (ce): Due to small size of Li* ion, 1ts hydration enthalpy hydration enthalpy decreases but lattice
is very high. enthalpies remain almost constant. Thus, BeSO,
and MgSO, are soluble.
. (d): Thermal stability of metal carbonates of alkaline
earth metals increases down the group. 28.(a, b, c) Li has exceptionally small size, high polarizing
. (a): Due to least electropositive character or least power and high degree of hydration.
basicity of Be(OH),, BeCO, is less stable and hence
decomposes to give BeO and CO,,.
s-BLOCK ELEMENTS (ALKALI AND ALKALINE EARTH METALS)

Unit Practice Test jomBoajdyExamivation

Time Allowed: 1 Hr. Maximum Marks: 25

rm"a Name the radioactive elements of s-block. (1)


LiH is more stable than NaH. Explain. (1)
Name the alkali metal which floats on water without any apparent reaction with it. (1)
fb
~ Which out of the following can be used to store an alkali metal?
C.H.0H, CHO (1)
ch What is the oxidation state of sodium in Na,O,? (1)
6. Potassium carbonate cannot be prepared by Solvay process. Why ? (2)
7. Why do beryllium and magnesium not give colour to flame whereas other alkaline earth metals
do so ? (2)
8. Draw the structure of BeCl, in vapour and solid state. (2)
9. Explain the following :
({) Sodium is found to be more useful than potassium.
(ii) Alkali metals are prepared by the electrolysis of their fused chlorides. (2)
10. Compare the stability and thermal stability of the following compounds of the alkali metals with
those of alkaline earth metals :
(1) Carbonates
(ii) Nitrates
(iii) Sulphates (3)
11, What is diagonal relationship? Why does lithium differ from its family members but resembles
magnesium? (3)
12. Explain the following :
(a) KO, is paramagnetic.
(b) Lithium forms oxide, sodium forms peroxide while potassium and rubidium form superoxides.
(c) BaO is soluble but BaSO, is insoluble in water. (3)
13. (a) Giving reasons arrange the following in the decreasing order of ionic mobility :
Li', Nat, Rot, K*
(6) Explain the various reactions that occur in the Solvay process for the manufacture of
Na,CO,. (3)

To check your performance, see HINTS and SOLUTIONS to some questions at the end of
Part II of the book.

WWW.JEEBOOKS.IN
WWW.JEEBOOKS.IN
SOME p-BLOCK
ELEMENTS

Building on..... Assessing..... Preparing for Competition.....


@ Understanding Text 1 @ Quick Memory Test with @ Additional Useful Information 52
| : Answers 48 @ Topicwise MCQs 53
Conceptual Questions 19,40 Bcoucifive E men (Os

CHAPTER SUMMARY & QUICK ¢ HOTS & Advanced Level ® AIPMT & Other State Boards’
CHAPTER ROUND UP Al Questions with Answers 49 Medical Entrance 55
| — > JEE (Main) & Other State Boards’
NCERT FILE REVISION EXERCISES 51 po cern Fntrance ar
¢ In-text Qs & Exercises with i ae acneees _
Solutions 42 UNIT PRACTICE JaptT ‘Q5s/” IEE Advance) for IT Entrance 58
¢ NCERT
|
Exemplar Problems
pears
with GRE essayite arss
(Objective Questions) 61
Answers & Solutions (Subjective) 46 ¢ Hints & Explanations for Difficult
Q uestions 63

The elements in which the last electron enters the p-subshell of


their outermost energy level are called p-block elements. Since a
p-Block Elements p-subshell has three degenerate p-orbitals, each of which can
18 accommodate two electrons, therefore, there are six groups of
p-block elements in the periodic table. The elements belonging
13 14 15 16 vA \ to groups 13 to 18 constitute p-block elements. The elements

8/6]
|Ole ine
C LN OF INS
boron, carbon, nitrogen, oxygen, fluorine and helium head
these groups respectively. These involve the addition of

Alsi]
PS |Gla
one (ns*np'), two (ns*np?), three (ns*2/p%), four (ns*2p*),

clalgieele
Al | Si | P #4S@ CPVAr five (ns*np?) and six (ns*np®) electrons respectively in p-
orbitals, and s-orbitals are already filled.
Ga | Ge | As). Se

in|Siso|
te|1|xe.
/ GENERAL CHARACTERISTICS OF p-BLOCK

HEHE
50 7 Wn! Cews 5S | 54
ELEMENTS
The general electronic configuration of p-block elements
. | Tl | Pb | Bi} Po | At | Rn is ns?np!* (where n varies from 2 to 7). The inner core of
~ ~ the electronic configuration may however, differ. The difference
in inner core of elements results in variations in their physical
properties (such as atomic and ionic radii, ionisation enthalpy, electron
gain enthalpy, electronegativity, etc.) as well as chemical properties. Therefore, a lot of variation in properties
of a group in p-block elements is observed. Apart from these, in each group of the p-block elements, the first
element differs in many respects from the other elements of the same group. Some of the significant characteristics
of p-block elements are discussed ahead.
Tia
Oxidation states
MODERN'S abe + OF CHEMISTRY-AXI

Metallic and non-metallic character


The oxidation states of elements are closely related The p-block contains metallic and non-metallic
to the number of electrons in the valence shell of their elements. It is very interesting to note that the non-
atoms. The maximum (or the highest) oxidation state metals and metalloids exist only in the p-block of the
of these elements is numerically equal to the total periodic table.
number of valence electrons (1.e., the sum of the The non-metallic character increases along a
s and p-electrons) or the group number minus period but decreases down a group.
10. This is also called group oxidation state.
In fact, the heaviest element in each p-block group
Clearly, the number of possible oxidation is the most metallic in nature. Therefore, the elements
states increases towards the right of the periodic with most metallic character are located mostly in the
table. For example, the elements of group 13 show lower left portion while those with most non-metallic
highest oxidation state of 3 (13 —10), while those of group character are present at the top right portion of the
14 show highest oxidation state of 4(14 — 10) and so on. periodic table.
In addition to group oxidation state, p-block elements
In between these, there are some elements which
may show other oxidation states, which normally but
show characteristics of both metals and non-metals and
not necessarily differ from the total number of valence
are called metalloids. The common metalloids in
electrons by unit of two.
p-block elements are B, Al, Si, Ge, As, Sb, Te, Po, At,
On descending the group, a lower oxidation state etc. This change from non-metallic to metallic brings
which is two units less than the highest oxidation state significant diversity in the chemistry of these elements.
becomes more stable in groups 13 to 16. This trend is
In general, non-metals have higher ionization
due to inert pair effect. For example, the highest
enthalpies and higher electronegativities than metals.
oxidation state for the elements of group 13 is + 3.
Therefore, in contrast to metals which readily form
However, in addition to + 3 oxidation state, these
cations, non-metals readily form anions. The compounds
elements also show + 1 oxidation state which becomes
formed by combination of highly reactive non-metals
more stable on moving down the group. In fact, for the
with highly reactive metals are generally ionic in nature
last element, thallium, + 1 oxidation state is more stable
because of large diferences in their electronegativities.
than + 3. Similarly, for group 14, the group oxidation
On the other hand, compounds formed between non-
state is + 4, but + 2 oxidation state becomes more and
metals themselves are largely covalent in character
more stable on going down the group. For example,
because of small differences in their electronegativities.
the last element, lead + 2 oxidation state is more stable
This can be understood in terms of their oxides.
than + 4 oxidation state.
e Oxides of non-metals are acidic or neutral
This trend of occurrence of oxidation state
whereas
two units less than the group oxidation state is
called inert pair effect and becomes more and ¢ Oxides of metals are basic in nature.
more prominent as we move down the group. ¢ The oxides of metalloids are amphoteric.
This has been discussed later. Furthermore, the more electropositive the metal, the
The common oxidation states displayed by the more basic is its oxide and the more electronegative the
p-block elements are given in Table 1.

Table 1. Common oxidation states of p-block elements.

General electronic
configuration
Group oxidation
state

Various oxidation
states P, As
+3, +5, Sh -1,+1,+3
+5, +7
Ga, In, Tl Sb, Bi
+3, +1 +3, +5
SOME p-BLOCK ELEMENTS
non-metal, the more acidic is its oxide. Therefore, in the next members belonging to third or higher periods
ep
p-block elements, acidic character of the oxides have vacant d-orbitals. For example, the elements of
increases or basic character decreases along a third period of p-block with the electronic configuration
3s? 3p* has vacant 3d-orbitals lying between 3p
period.
and 4s levels of energy. Therefore, they can easily
Similarly, the basic character of the oxides increases expand their octets and can show covalence above four.
or acidic character decreases down the group. For example,
Variation in physical properties (t) Boron forms only BF, (coordination number
four) whereas aluminium forms AIF > (coordination
As we shall learn in the chapter, that there is a
number six).
diverse nature of elements in p-block. However, there
are some general useful trends in their properties as (ii) Carbon can form only tetrahalides (CX,, X=F,
summed up below : Cl, Br, 1) whereas other members can form hexahalides,
Electronegativity, SF,, SiClj” ete
Ionization enthalpy, (iii) Nitrogen forms only NF, (upto octet) while
Oxidizing power phosphorus forms pentahalides, PF, PCl,, etc.
(iv) Fluorine does not form FCI, (more than octet)
while chlorine forms CIF, (extends octet).
(6) Reactivity. Due to availability of d-orbitals of
elements of third period, they are more reactive than
elements of second period which do not have d-orbitals.
For example, tetrahalides of carbon are not hydrolysed

Senocic [pecans
® Covalent radius,
® van der Waals radius,
by water whereas tetrahalides of other elements of
croup 14 are readily hydrolysed. The hydrolysis
@® Metallic character

rfeelesilcsepAN]|||
involves the nucleophilic attack of water molecule.
(c) Tendency to form multiple bonds. Because
of the combined effect of smaller size and availability
Enthalpy of atomization, of d-orbitals, the first member of each group shows
(except for N,, O,, F,) greater tendency to form pt — pt multiple bonds either
Metals IMmcrease along period e Ionization enthalpy, with itself (such as C = C, C=C, N=N, O=O)or with
® Oxidizing power. other members of the second period of elements (such
as C=0,C=N,N =O, etc). The other members of the
Covalent Paine, croup donot have strong tendency to form 1- bonding.
Van der Waals radius, The heavier elements do form t-bonding but they
Enthalpy of atomization involve d- orbitals and form d1- pt or dr - dm bonding.
(upto group 14), For example, the bonds between sulphur and oxygen
in oxides of sulphur (SO, and 5O,) are much shorter
Metallic character
than might be expected for a single bond. In these
molecules, in addition to normal o bond, a 1 bond is
Differences in behaviour of first element of formed by the sidewise overlap of a filled 2p-orbital
each group on the oxygen with a vacant 3d-orbital on the sulphur
The first member of each group of p-block differs (Fig. 1). This is called pa-dnx bond and results in
in many respects from its succeeding members (called bringing the two atoms closer and thus accounts for
congeners) of their respective groups. For example, the shorter bond length of S—O bond.
boron shows anomalous behaviour as compared to rest
dn—pn Bond
of the members of the 18 group elements. The main
reasons for the different behaviour of the first member
as compared to other members is because of
(i) small size of the atom and its ion
(ti) high electronegativity and
(tit) absence of d-orbitals in their valence shell
These factors have significant effect on the
chemistry of first element as compared to other
Vacant d-orbital Filled 2p-orbital
elements (specially second). For example ,
(a) Covalence upto four. First member of each Fig. 1. Overlapping of p and d orbitals to form pr-dt bond.
eroup belongs to second period elements and have only Because the d-orbitals are of higher energy than
four valence orbitals t.e., one 2s and three 2p orbitals the p-orbitals, they contribute less to the overall
available for taking part in chemical combinations. stability of molecules than does the pa-pn bonding of
They donot have vacant d-orbitals in their valence second row elements. However, the coordination
shell. Therefore, they may have maximum covalence number in species of heavier elements may be higher
of four (using one 2s and three 2p orbitals). In contrast, than those of first element in the same oxidation state.
| 1s MODERN'S abc + OF CHEMISTRY-XI

For example, both nitrogen and phosphorus form () Bauxite : Al,O,.2H,O. (It may also be
oxoanions in + 5 oxidation state as NO, (three represented as AlO.(OH),..where 0 <x < 1).
coordination with m bonding using one p-orbital of N) (i) Cryolite (Sodium aluminium fluoride) :
and PO; (having four coordination using s, p and d Na, AlF..
orbitals contributing to the m-bonding). (iz) Orthoclase (feldspar) : KAISi,O,
The first member of 13 group (boron) shows (tv) Mica (muscovite) : KAI,(Si,Al0,,) (OH),
diagonal relationship with silicon (of group 14). (v) Beryl : Be,Al,Si,0,.
(vt) Corundum (anhydrous alumina): Al,O,
ROUP 13 ELEMENTS : BORON FAMILY Bauxite is commercially the most important ore
from which aluminium metal is extracted.
Group 13 of the periodic table consists of the elements Gallium, indium and thallium are very much less
boron (B), aluminium (Al), gallium (Ga), indium (In)
abundant and occur in traces in sulphide minerals
and thallium (Tl). Except boron, which is classified as
rather than oxides. Small amounts of gallium are also
non-metal, all other elements of this group are metals.
found in the ores of elements adjacent to it in the
Aluminium is a metal but shows many similarities to
periodic table (Al, Zn and Ge). Highest concentration
boron (non-metal) and gallium, indium and thallium are
(0.1-1%) of Ga is found in the rare mineral germanite
almost exclusively metallic in character. Aluminium is
the most common metallic element in the earth’s crust. which is a complex sulphide of Zn, Cu, Ge and As.
It is also very important element of this group because Traces of In and Ti are found in ZnS and PbS ores.
of its various industrial uses. The abundance of these elements in earth's crust are
given below :
OCCURRENCE

Farad
Boron is fairly rare element and it occurs to a very Element Abundance in Relative
small extent (0.0001% by mass) in earth’s crust. It
occurs in two allotropic forms, }°B (19%) and “B (81%).
It does not occur in free state but occurs as borates or B 9 38
orthoboric acid. The main ores of boron are : Al 8.3 x 104 3
(1) Borax : Ga 19 33
Na, [B,O,(OH),].8H,O or Na,B,O,.10H,O In 0.24 63
(11) Kernite:
Tl 0.5 60
Na, [B,O,(OH),] or Na,B,O,.2H,O
(111) Colemanite : GENERAL CHARACTERISTICS OF GROUP 13
Ca,[B,0,(OH).],.2H,O or Ca,B,O,,.5H,O ELEMENTS
(tv) Orthoboric acid : HBO, A. Electronic Configurations
The deposits of boron are found in California The elements of group 13 have the general
(U.S.A.) and in Turkey. In India, boron occurs as borax electronic configuration ns2@np! i.e., one electron in the
in Puga Valley of Ladakh region in Kashmir and also outermost p-orbitals and two electrons in the s-orbital.
in Sambhar lake of Rajasthan. Their electronic configurations are given in Table 2.
Among the elements of group 13, aluminium is
most abundant metal. It is third most abundant It is clear from Table 2 that boron and aluminium
element (8.3% by mass) in the earth's crust after have noble gas core, gallium and indium have noble
oxygen (45.5%) and silicon (25.7%). It is a major gas plus 10 d-electrons core while thallium has noble
constituent of many common aluminosilicate rocks gas plus 14 f-electrons and 10 d-electrons core. Thus,
such as feldspars and micas. In India, aluminium is the electronic configurations of elements of group 13
found as mica in Madhya Pradesh, Karnataka, are more complex than those of groups 1 and 2. This
Orissa, and Jammu.The important minerals of
difference in electronic configurations affects the other
aluminium are :
properties and consequently the chemistry of this group.

Table 2. Electronic configurations of the elements of group 13

Boron B 5 [He] 2s? 2p!


Aluminium [Ne] 3s? 3p!
Gallium [Ar] 3d!° 4s? 4p
Indium [Kr] 4d? 5s? 5p!
Thallium [Xe] 5d!° 6s? Gp!
SOME p-BLOCK ELEMENTS

Table 3. Physical constants of Group 13 elements.

Property ee ee ee ee ee ee
Atomic number 5C 13 31 49 81
Atomic mass 10.81 69.72 204.38
Atomic radius (pm) 85 135 170
Ionic radius (pm) M** 27 62.0 88.5
Ionic radius, M* (pm) - 120 LA 150
Density (g cm~) 5.90 11.85
Ionization enthalpy (kJ mol-')
IE, o79 ) 590
IE, 1979 1971
IE, 2962 2877
Density (g cm at 293 K) 5.90 11.85
Electronegativity 1.6 l. 1.8
Density (g em™ at 293 K) 5.90 11.85
Melting point (K) 303 a) 576
Boiling point (K) 2676 1730
E° (M+/M) (V) — 0.56 + 1.26
K° (Mt / M) (V) — 0.79 (acid) — 0.34
— 1.39 (alkali)

B. Atomic and Physical Properties effective nuclear charge in Ga becomes more than in
Al and its atomic radius, therefore, decreases slightly.
The important atomic parameters and physical
constants of group 13 elements are given in Table 3 2. Ionisation enthalpies
and their general physical characteristics are
discussed below : The first tonisation enthalpies of group 13 elements
are less than the corresponding members of the alkaline
1. Atomic and ionic radii
earths.
The atomic and ionic radii of group 13 elements Explanation. We know that the first electron in
are smaller as compared to corresponding elements of the case of group 13 elements (ns@np') is to be removed
group 2. from p-orbital while in case of elements of group 2,
the electron has to be removed from s-orbital. Since
Explanation. This is due to the increase in p-orbitals are at slightly higher energy than the
nuclear charge when we move from element of group s-orbitals, the electron in the atoms of group 13
2 to group 13 in the same period. As we move from left elements is weakly held by the nucleus and, therefore,
to right in the period, the magnitude of nuclear charge the first ionisation enthalpy is less. However, second
increases but the electrons are added to the same shell. (IE,) and third (IE,) ionisation enthalpies are quite
Since the electrons in the same shell do not screen high. As expected, the order of ionisation enthalpies
each other, therefore, the electrons experience greater is AH, < AH, < AH,. The sum of the first three
nuclear charge. In other words, effective nuclear ionisation enthalpies for each of these elements is very
charge increases and thus, size decreases. Therefore, high. This will be affecting the chemical properties of
the elements of this group have smaller size than the these elements as discussed later. When one electron
corresponding elements of second group. is removed from outermost p-subshell, the resulting
ion has completely filled s-orbital (ns”). Therefore, it
On moving down the group both atomic and tonic
becomes difficult to remove the second electron.
radu are expected to increase due to the addition of
new shells. However, the observed atomic radius of Al On moving down the group, there is a sharp
(143 pm) ts slightly more than that of Ga (135 pm). decrease in first ionisation enthalpy (A, ,) from B to
Al and then first tonisation enthalpy of Ga is
Explanation. While going from Al (4 = 18) to
unexpectedly slightly higher (about 2 kJ mol-") than
Ga (Z = 31) there are also ten elements of the first
that of Al. Then again the first tonisation enthalpy
transition series of d-block from (4 = 21 to 30) which
decreases from Ga to In while that of Tl is much higher
have electrons in the inner d-orbitals. The d-orbitals than those of Al,Ga and In. Thus, amongst the elements
do not screen the nucleus effectively because of their of group 13, B has the highest and In has the lowest
shapes and poor penetration power. As a result, the AH.
Tie MODERN'S abc + OF CHEMISTRY-XI

Boron exists in four common allotropic forms under


Explanation. The sharp decrease in ionisation
enthalpy from B to Al is due to increase in atomic size different conditions. The transition between different
and screening effect which is more than to compensate forms is extremely slow process. All these boron
the effect of increase in nuclear charge. Consequently, allotropes are built up of basic building B,, icosahedral
the electron becomes less and less tightly held by the units made up of polyhedron having 20 faces as shown
nucleus as we move down the group. Hence, ionisation in Fig. 2. These are amorphous boron (dark brown
enthalpy decreases down the group from B to Al. In case colour) and crystalline boron (black metallic lustre).
of Ga, there are ten d-electrons in its inner sub shell.
Since the d-electrons shield the nuclear charge less
effectively than the s- and p-electrons, and therefore,
due to poor shielding of ten d-electrons the outer
electron is held fairly strongly by the nucleus. As a
result, the ionisation enthalpy increases slightly inspite
of the increase in atomic size as we move from Al to Ga.
Further, on moving down the group from Ga to In,
although nuclear charge increases but the total shielding
effect of 3d- and 4d- electrons outweighs the effect of
increased nuclear charge and therefore, A,H, of In is
lower than that of Ga. On further moving from In to TI,
there is increase in A,H, similar to that from B to Al.
This is due to the presence of 14 f-electrons in inner Fig. 2. B,, icosahedral unit of boron.
electronic configuration of Tl which have very poor
shielding effect. As a result, effective nuclear charge 5. Electropositive (or metallic) character
acting on Tl is much higher than that on In and hence Due to high tonisation enthalpies, the elements of
A, of Tl is much higher than that of In even higher group 13 are less electropositive as compared to
than those of Ga and Al. Thus, the anomalous behaviour elements of group 2. On moving down the group, the
in general decrease down the group 13 elements is due to electropositive (metallic) character increases because
the presence of ten d-electrons after Al and 14 f-electrons ionisation enthalpy decreases. For example, boron is a
after In which have poor shielding effect. non-metal while the other elements are typical metals.
3. Electronegativity C. Oxidation States
The electronegativities of group 13 elements are The atoms of these elements have three valence
more than those of elements of group 1(alkali metals) electrons, two in s-subshell and one in p-subshell.
and group 2 (alkaline earth metals). In group 13, the Therefore, all these elements can show maximum of
electronegativity first decreases from B to Al and then +3 oxidation state.It may also be noted that due to the
increases down the group. small size of boron, the sum of its first three ionisation
Explanation. As we move from B to Al, the atomic enthalpies is very high. Therefore, it cannot form +3
size increases considerably. Therefore, attraction of the ions and can only form covalent compounds. But as
nucleus for the electrons decreases and hence we move from B to Al, the sum of the first three
electronegativity decreases. On further moving down ionisation enthalpies of Al considerably decreases and
the group, although the size increases (except that of is therefore, able to form Al** ions. In fact, aluminium
Ga), yet the effective nuclear charge increases because is highly electropositive metal. However as we move
of poor shielding of the inner d- and /- electrons. As a down the group, due to poor shielding effect of
result, the force of attraction of the nucleus for the intervening d and f-orbitals, the increased effective
electrons increases and hence the electronegativity nuclear charge holds ns electrons very tightly
increases from Al to Tl. (responsible for inert pair effect) restricting their
4. Melting and boiling points participation in bonding. As a result, only p-orbitals
The melting and boiling points decrease on moving may be involved in bonding. Thus, the elements of
down the group. However, the decrease in melting point croup 13 exhibits +3 and +1 oxidation states.
is not as regular as in boiling points. Boron shows only +3 oxidation state in its
Boron is non-metallic in nature. Itis extremely hard compounds. Except boron, the other elements also show
and black coloured solid. Boron has very strong + 1 oxidation state. The + 1 oxidation state becomes
crystalline lattice and therefore it has very high melting more stable as we move down the group from
point. Rest of the members are soft metals with low aluminium to thallium.
melting point and high electrical conductivity. Gallium Al < Ga<In< Tl.
has very low melting point (303 K) and therfore could In fact, in Ga, In and TI, both +8 and +1 oxidation
exist in liquid state during summer. It melts in our hands states are observed. However, in Ga and In, +3
in summer because of its low melting point. Its high oxidation state is more stable than +1 oxidation state.
boiling point (2676 K) makes it a useful material for As a result, both Gat and In* salts disproportionate in
measuring high temperatures. water 1.e., undergo self oxidation and reduction as:
SOME p-BLOCK ELEMENTS

+1 0 +4 34 :
The first three ionisation enthalpies of boron are
3 Ga X (s) —+2Ga + Ga’ (aq) + 3X (aq) quite higher than the corresponding values for the
0 +3. 7 other elements of group 13. Therefore, the total energy
3In X (s) —>2 In + In®” (aq) + 3X (aq)
needed to form B** ion is much greater than that
In case of last element, thallium, + I oxidation state compensated by lattice energies (in solids) or hydration
has been found to be more stable than + 3 oxidation energies (in aqueous solutions). As a result, ionic
state in aqueous solution. This ts evident from the redox compounds of boron are not easily formed. Boron can
potential as : readily form covalent compounds by sharing.
Unlike boron, the other atoms of group 13 form
TI** (aq) + 2e- —— TI (aq) E® =41.25V ionic compounds in aqueous solution. For example,
It may be noted that gallium, the third element of the compounds of aluminium are both ionic and
the group appears to show +2 oxidation state in some covalent. For example, AlCl, is covalent while
of its compounds such as GaCl,. However, it has been [Al(H,O),1°* is 10n1c.
shown that this compound has the structure Except thallium, all other elements form
Ga*[GaCl,]- which contains gallium in +1 (Ga*) and compounds in trivalent state. In trivalent state, the
+3(GaCl,~) oxidation states. number of electrons around the central atom in a
Explanation. This is explained on the basis of molecule of these elements will be only six. For
inert pair effect. The elements of group 13 have three example, boron forms boron halides such as BF ,, BCl,,
electrons in their valence shell (ns*np+) and, therefore, BBrz, BI, etc. in which boron has only six electrons
exhibit, oxidation state of + 3. However, it has been around it. Such compounds have two electrons less
observed that in addition to + 3 oxidation state, they than the octet and therefore, are called electron
also exhibit oxidation state of +1. The +1 oxidation deficient molecules. These have tendency to accept
state becomes more and more stable as we go down a pair of electrons to achieve stable electronic
the group from B, Al, Ga, In to Tl. For example, the configuration and therefore, behave as Lewis acids.
+1 oxidation state of Tlis more stable than +3 oxidation For example, BCI, readily combines with donors
state. Thus, thallous compounds such as TIOH and (Lewis bases) such as amines, ethers, etc. For example,
TIC] are more stable than their corresponding thallic BCl, readily accepts a lone pair of electrons from
compounds. This is due to the fact that in the case of ammonia forming BCl,. NH, as
last element, after the removal of one electron from p- 1
orbital, the remaining ns? (e.g. 6s”) electrons behave
like stable noble gas and do not take part in compound
BCl,
Lewis acid ¥ eee cr a
formation. This reluctance of the s-electron pair Cl
to take part in chemical combination is called The Lewis acid character of these compounds
inert pair effect. As a result of inert pair effect, the decreases down the group as the size of the element
heavier elements of p-block show oxidation states two increases 1.€.,
units less than the group oxidation state. This helps BX, > AIX, > GaX, > InX,
to explain the stability of lower oxidation states for where X = F, Cl, Br, I
the heavier elements of a group. AICI, achieves stability by forming dimer AlI,Cl,
The decreasing stability of the higher oxidation (discussed later). Most of the compounds in trivalent
state (+3) with increasing atomic number arises state being covalent are readily hydrolysed by water.
because of the decrease in bond energy with size from For example, trichlorides of boron on hydrolysis with
Al to Tl. As a result, the energy required to unpair the water give tetrahedral [M(OH),\ species. In this case,
ns” electrons is not compensated by the energy the hybridisation of M is sp®. On the other hand, AICl,
released in forming the two additional bonds. in acidified solution gets hydrolysed to form octahedral
D. Chemical Properties [Al(H,O),|3* ion. In this complex, 3d orbitals of Al are
Except boron, the other elements of group 13 show
also used involving sp? d? hybridisation.
metallic character which increases down the group. Let us discuss some common chemical properties
Boron is non-metallic and the crystalline form is very of group 13 elements.
hard, inert and non-conducting. The amorphous form 1. Reactivity towards air
of boron is more reactive. Aluminium is also reactive Pure boron is almost unreactive at ordinary
metal but it usually gets covered with a protective temperature. Aluminium reacts readily in air, even
oxide layer which makes it inert to acids, although it at ordinary temperature forming a thin oxide layer
is attacked by alkalies. Gallium, indium and thallium on the surface which protects the metal from further
are relatively soft and reactive metals which readily attack. Amorphous boron and aluminium metal on
dissolve in acids. heating in air form boron trioxide and aluminium
oxide. With nitrogen, they form nitrides.
lie MODERN'S abc + OF CHEMISTRY-XI

4E (s)+30,(g) —“*> 2E,0, (s) However, concentrated nitric acid makes


e.g., 4B(s)+30,(g) > 2B,0,(s) aluminium passive by forming a protective layer of its
oxide (Al,O.) on the surface.
4 Al(s)+30, (g) —> 2Al,0, (s)
2 Al(s) + 6HNO.(cone.) ——> Al,O, + 6NO, + 3H,O
2E (s)+N,(g) —“> 2EN (s)
This protects it from further action.
e.g., 2B (s) +N, (g) —*> 2BN (s) Gallium, indium and thallium dissolve in dilute
2 Al(s)+N,(g) —““> 2AIN(s) acids liberating hydrogen. Gallium is amphoteric like
aluminium and it dissolves in aqueous NaOH
The nature of oxides varies down the group. Boron liberating H, and forming gallates.
trioxide is acidic and reacts with basic (metallic) oxides
2Ga(s) + 2NaOH(aq) + 6H,0() ——>
forming metal borates. Aluminium and gallium oxides
are amphoteric and those of indium and thallium are 2Na* [Ga(OH),\ (aq) + 3H,(g)
basic 1n nature. Sodium tetrahydroxogallate (IIT)

2. Reactivity towards water In and TI, however, donot react with alkalies.

Pure boron does not react with water. Aluminium 4. Reactivity towards halogens
is also not attacked by cold water. However, aluminium These elements react with halogens to form
amalgamated with mercury becomes very reactive and trihalides.
decomposes even cold water liberating hydrogen.
2.Al+6H,O ——>2 Al(OH), + 3H, 2E(s) + 3X, ——> 2 EX,(g)
E = Element of 13 Group, X = F, Cl, Br and I
Gallium and indium are attacked neither by cold
water nor by hot water unless oxygen is present. e.g., 2B 4+ 3X, ——> 2BX, (X = F, Cl, Br, D
Thallium is a little more reactive than gallium and
forms an oxide on the surface. IMPORTANT TRENDS AND ANOMALOUS
PROPERTIES OF BORON
3. Reactivity towards acids and bases Let us discuss some of the chemical reactivity
Boron does not react with acids and alkalies even trends of elements of group 13.
at moderate temperature. However, it is attacked at 1. Formation of hydrides
high temperatures by strong oxidising acids such as a
The group 13 elements form hydrides of the
mixture of hot conc. H,SO, and HNO, (2 ; 1) forming
general formula EH,. The thermal stability of these
boric acid.
hydrides decreases as we go down the group.
B(s) + 3HNO,(aq) Ome #50., 1 BO.(aq) + 3NO,(g) Boron forms a number of hydrides having the
general formula BH, and B_H__.. These compounds
Boric acid
of boron and hydrogen are called boranes. For
Boron also reacts with fused caustic alkalies like
example, B,H,, B,H,,, BsH,, etc. These contain special
NaOH and KOH forming corresponding borates and
types of bonds known as multicentre bonds. The
liberates dihydrogen.
simplest among the boranes is diborane (B,H,).
2B(s) + GBNaOH(s) => 2Na,BO,(s)+ 3H,(g) Aluminium forms polymeric hydrides of the
general formula (AlH,),. Other members form less
Sodium borate
stable hydrides.
Boron also dissolves in fused Na,CO, and NaNO, These hydrides are weak Lewis acids and readily
mixture at 1123 K. form adducts with strong Lewis bases (:B) of the type
2B(s) + 3Na,CO, (s) + 3NaNO.(s) ——> EH, : B (where E = Al or Ga). Therefore, they form
2Na,BO,(s) + 3NaNO, (s) + 3CO,(g) complex hydrides like lithium aluminium hydride
Sodium borate Li[AIH,], lithium boron hydride Li[BH,] and lithium
gallium hydride LilGaH,]|. The formation of these
Aluminium dissolves in mineral acids as well as
hydrides is due to the presence of vacant p-orbital in
aqueous alkalies and this shows its amphoteric character.
their outermost shells which readily takes electron
2Al(s) + 6GHCl (ag) ——> 2Al8*(aq) + 6Cl-(aq) + 3H,(g)
pair from hydride ion (H~), and form tetra hydrido
2Al(s) + 2NaOQH(aq) + 6H,0 (1) —_> anions such as [EH,|-
2Na* [AIOH),/ (aq) + 3H, (g) XH, +H ——-> [XH,]
Sodium tetrahydroxoaluminate (IIT) ion
where X = B, Al, Ga.
SOME p-BLOCK ELEMENTS

The complex hydrides act as powerful reducing As already discussed, there is one 2p-vacant
agents. Li[AIH.,] is also prepared by the action of Li orbital on B atom in BF, and 2p-orbital of each F
with AICI, in ether as : atom is fully filled. Since the energies of these two
4LiH + AIC], —““> Li[AlH,] + 3LiCl 2p-orbitals (of B and F) are almost similar, one of
the 2p filled orbital of F overlaps sidewise with the
It is a white crystalline solid and soluble in diethyl vacant 2p-orbital of B atom resulting in the
ether. It is very powerful reducing agent. transference of electrons from F atom to vacant 2p
2. Formation of halides orbitals. Thus, B—F bond acquires some double bond
All the elements of group 13 form trihalides of (or pt—prt bond) character. The type of bond is called
the general formula EX, (except TII,). The halides of dative or back bonding.
boron are covalent and monomeric in vapour phase.
In these halides, boron involves sp? hybridisation using
one (2s) and two (2p) orbitals. The sp” hybrid orbitals
adopt trigonal planar structure. Each of the sp?
or Sete B
hybrid orbital overlaps with the 2p-orbital of halogen
to form B—X bonds. A 2p-orbital is left vacant in the i F
valence shell (Fig. 3). a

All the halides of boron are volatile, highly reactive Back bonding
and monomeric. BF, and BCl, are gases at room Fig. 4. Formation of pn—pz back bonding in
temperature. BBr, is a volatile liquid while BI, is a one of the B—F bonds in BF.,.
solid. In these compounds, the octet of boron is not The back bonding in BF, molecule also gets
complete because they have only six electrons around
support from the fact that the observed B—F bond
boron. Therefore, these halides act as Lewis acids
length in BF, (1.30 A) is significantly shorter than
and combine with donors (Lewis bases) such as amines,
the sum of the covalent radii (B = 0.80 A and F =
ethers, sulphides or phosphines. For example,
0.72 A) or normal B—F single bond.
F H H H As a result of the back donation of electrons
from fluorine to boron, the electron deficiency of boron
F:B + :-N:H ——__> F:B <—_:N:H atom gets compensated and, therefore, the Lewis acid
character of BF, decreases. The tendency to form pr—
F isl F H pw bond is maximum in the case of BF, and decreases
Lewis acid Lewis base Addition compound rapidly as we move to BCI, to BBr, and BI,. This is
The relative Lewis acid strength of boron because of the inability of the vacant 2p-orbital of
halides increases as : boron to overlap effectively with the 3p- orbital of
BF, < BCI, < BBr, < Bl, chlorine in B—Cl bond and the 4p-orbital of bromine
in B—Br bond and the sp-orbital of iodine in B—I
This order is just reverse of what may be bond due to the appreciable differences in their
expected on the basis of the electronegativities of the energy levels.
halogen atoms. Since F is most electronegative
Halides of other elements of Group III also
among the halogens, it will withdraw electrons from
behave as Lewis acids. The Lewis acid strength
boron atom strongly. As a result, it will make BF, as
decreases as :
the most electron deficient and strongest Lewis acid.
However, BF, is the least acidic among the trihalides B > Al>Ga> In.
of B. This anomalous behaviour can be explained In contrast to boron halides, aluminium halides
on the basis of the tendency of the halogen atom to exist as dimeric molecules. The halides of Al in the
back donate its electrons to the boron atom. This is vapour state as well as in inert organic solvents such
explained below : as benzene exist as dimers. For example, AICI, exists

sce: @ OOO
25 2p

state B 0 ora
Vacant

Excited B F
0 as Al,Cl,. The formation of dimer is illustrated below :
MODERN'S abc + OF CHEMISTRY-XI

3. Oxides and hydroxides


Cl Cl Cl Cl Cl Cl All the elements of group 13 form oxides of general
i \N 7 Ne 7 formula KO, and hydroxides E(OH),. The oxides of B
Al - Al. —> Al Al and Al are obtained by heating the metal in oxygen.
/ i ™~ J/’N/S\ 4B+30, ——> 2B,0,
Cl Cl Cl Cl Cl Cl 4Al+30, —— 2AL0,
Two AICI, molecules Dimer The hydroxides are generally obtained by dissolving
the oxides of the element in water. However, boron
In AICl,, there are six electrons aroundAl atom
oxide does not react with water even in the form of
and these are two less than the octet. In the dimeric
structure, each aluminium atom completes its octet steam. The basic character of oxides and hydroxides
by accepting a lone pair of electrons from the decreases from aluminium to thallium. The oxides and
chlorine atom of another aluminium chloride hydroxides of B are weakly acidic and therefore,
molecule. The two co-ordinate covalent Cl—AI bonds react with alkalies :
are shown in the figure, in which the arrow indicates B,O, + 2NaOQH ——_> 2NaBO, +H,0
the donation of electrons from chlorine to Sod, metaborate
aluminium. B(OH), + NaOH ——> NaBO, + 2H,O
The dimer structure exists only in vapour state and Sod. metaborate
at low temperature (473 K) but at higher temperature, Aluminium oxide and hydroxide are amphoteric
it dissociates to trigonal planar AICI, molecule. in nature. They dissolve both in acids as well as in
It may be noted that anhydrous AICl, is alkalies.
covalent in nature but hydrated aluminium Al,O,(s)+3H,SO, (aq) ——~+AL,(SO,), (aq) + 3H,O (0)
chloride is ionic. In anhydrous AICl,, Al atom is Al,O, + 2NaOH ——> 2NaAl0, + H,O
linked to three Cl atoms by covalent bonds. This is
Similarly,
due to the fact that a large amount of energy is
needed to convert aluminium atom to aluminium Al(OH), (s) + NaOH (aq) ———> Nal[AI(OH),] (aq)
Sodium
ion (Al*+). Therefore, it prefers to form covalent
tetrahydroxoaluminate (III)
bonds with Cl atoms. However, when aluminium
chloride is dissolved in water, it undergoes hydration or NaAlO,.2H,O
as: Sodium aluminate
AlCl, + nH,O —— 2 [Al (H,O),|°+ Al(OH), (aq) + 3HCI (aq) ——+ AICI, (aq) + 3H,O (Z).
+ 6CI- + Energy Like Al(OH)., Ga(OH), is also amphoteric. Indium
Hydration of Al,Cl, is an exothermic process and hydroxide, In(OH), and thalous hydroxide TIOH are
a large amount of hydration energy is released. basic. TKOH), is insoluble in water while TI(OH) is
Thus, energy liberated during hydration process is soluble and is a strong base. Many of Tl(/) compounds
responsible for the removal of three electrons from are similar to corresponding alkali metal compounds.
Al and forms Al** ion. In fact, in dissolved state, Al
Therefore, the basic character of oxides and
exists as [Al(H,O),]*+. Therefore, the hydrated
hydroxides increases as
aluminium chloride is ionic in nature,
BO. Al,O, Ga,O, In,O, T1,0,
The trihalides of all other elements of group 13
except TI(III) iodide are known. The fluorides are B(OH), Al(OH). Ga(OH), In(OH)., TIOH
generally ionic and have high melting points. The Acidic Amphoteric Basic
chlorides, bromides and iodides all have low melting
points and are mainly covalent. Basic character increases
Characteristic structural difference
Explanation. As we move down the group, the
between trihalides of B and Al
magnitude of the ionisation energy decreases. As a result,
There is a characteristic difference in the
the strength of the M—O bond also decreases accordingly.
structures of trihalides of boron and aluminium. For Therefore, its cleavage becomes easy resulting in the
example, AlCl, exists as dimer Al,Cl, while BCI, increased basic strength down the group. The acidic
exists as monomer. It is interesting to note that both strength of the hydroxides, therefore, decreases.
B and Al are electron deficient compounds and
behave as Lewis acids but aluminium chloride exists 4, Behaviour of Compounds
as dimer whereas boron chloride exists as monomer. Except for boron, the trichlorides, bromides and
This is due to the reason that boron atom is very small iodides of all these elements are hydrolysed by water.
in size and it cannot accommodate four large sized These exist as [M(OH),| tetrahedral and (M(H,0),|°*
halide ions around it to form dimer. octahedral species in aqueous medium.
SOME p-BLOCK ELEMENTS

Aluminium sulphate forms double salts with (vit) Boron compounds are finding use as rocket fuels
sulphates of other metals such as E Al(SO,),.12H,O because of their high energy/mass ratio.
where E is a univalent cation, Na* or Kt. These are (viit) Boron is an essential element in plant metabolism.
called alums. These alums are used in the softening Uses of Aluminium
of hard water and as mordant in dyeing and printing Aluminium is a bright silvery white metal, with
of textiles. The mordant helps to bind the dye to the very high tensille strength. It has high electrical and
fabric. thermal conductivity. Some important applications of
The (+1) oxidation state becomes more and more aluminium are :
stable from Ga to Tl. For example, the mono halides, (1) Aluminium metal is light, soft, non-toxic and
GaX, InX, TIX are known (X = Cl, Br and I). But GaX corrosion resistant. On a weight-to-weight basis,
the electrical conductivity of aluminium is twice
and InX disproportionate in water as :
that of copper. Therefore, it is extensively used
3 EX(s) ——> 2E(s) + E** (aq) + 3X (aq) in household utensils, in chemical plants and as
However, TI(T) is stable. structural metal for aircrafts, ships, cars, heat
USES OF BORON AND ALUMINIUM AND exchangers, etc.
THEIR COMPOUNDS (it) It is also used for making electric power cables.
(iii) Aluminium foil is used for packing chocolates,
Uses of Boron
cigarettes, food items, etc.
Boron being extremely hard refractory solid of
(tv) Aluminium is also used for making cans for
high melting point, low density and very low electrical drinks, beer, dairy products and containers for
conductivity finds many applications. Some important chemicals and tooth paste tubes.
applications of boron are : (v) Alumina is used in making refractory bricks and
(1) It is used as a semi-conductor for making ultra marines and alums are used in dyeing and
electronic devices. paints.
(it) Itisusedin steel industry for increasing hardness (vt) Many of the mechanical properties of pure Al
of steel. Boron has infact replaced expensive are greatly improved by alloying it with Cu, Mn,
metals like Mo, Cr and W in the manufacture of Sn, Ni, Mg, Zn, etc. These alloys can be given
special hard steels. shapes of pipes, tubes, rods, wires, plates or foils
(111) Its compounds such as borax and boric acid are and therefore, have many uses in different fields.
used in glass industry for making heat resistant Some of the alloys of aluminium are given in
glass i.e., pyrex glass, glass-wool and fibre glass. Table 4.
(tv) Borax is also used as a flux for soldering metals, It may be noted that the use of aluminium and its
for heat, scratch and stain resistant glazed coating compounds for domestic purposes is now discouraged
to earthenwares and as a constituent of medical because of their toxic nature.
soaps. An aqueous solution of orthoboric acid is The other metals Ga, In and Tl] have no large scale
generally used as a mild antiseptic. uses. Small amounts of Ga are used to dope crystals for
(v) Boron fibres are used in making bullet proof making transistors. It find uses in solid state devices
vests and light composite materials for aircrafts. as GaAs. It is also used in low melting solder and other
(vi) Metal borides are used in atomic reactors as low melting alloys. Thallium as a metal does not find
protective shields and central rods because of worthwhile uses, though its salts find a number of uses.
the high tendency of B-10 (10B) isotope to Its compounds are used in preparing optical glasses
neutrons. This special property of B-10 isotope because of their high refractive indices. TlBr and TI
absorb neutrons is also of latest interest in the have been used in spectroscopy and as infrared detectors,
possible use of boron compounds in Cancer and as photosensitive diodes. Thallium carboxylates
Chemotherapy. have been used in organic synthesis.
Table 4: Important alloy of aluminium
Alloy Uses

Aluminium Al 95% Light, strong alloy For making coins, utensils,


bronze Cu 5% with golden lustre, picture frames, jewellary etc.
resistant to corrosion
Magnalium Al 95% Very light, tough and hard For making light instruments,
Mg 5% balance beams, pressure
cookers, etc
Duralumin Al 95% Light, strong, ductile, For making bodies of aircrafts,
Cu 4% resistant to water, helicopters, automobile parts
Mg 0.5% corrosion kitchenware, bodies of ships, etc.
Mn 0.5%
Alnico Al, Ni, Co, Fe Highly magnetic in nature For making powerful magnets.
iz
ANOMALOUS PROPERTIES OF BORON
MODERN'S abc + OF CHEMISTRY-XI

The structure of [B,0,(OH),|* ion is given below:


Boron, the first member of group 13, shows ie
anomalous behaviour and differs from rest of the ie

Xo
members of its family. This may be due to
(1) exceptionally small atomic size and ionic size

ee
(ii) its high ionisation enthalpy.
(it) absence of d-orbitals in its valence shell. ona | on
Some important properties in which boron differs
from the rest of the members of its group (especially
the next member, Al) are given below:
YA
(¢) Boron being small is harder than other elements OH
of its group. Preparation of Borax
(it) It has higher melting and boiling points than
other members of its group. 1. From tincal. Borax is obtained from tincal by
boiling it with water. The solution is filtered to remove
(tii) Boron forms only covalent compounds whereas
insoluble impurities of sand, clay, etc. The solution is
all other members of its group form ionic as well
concentrated till the crystals of borax separate out.
as covalent compounds. For example, BF, is
2. From colemanite. Borax can also be prepared
covalent while Al F, is ionic.
from the mineral colemanite (Ca,B,O,,) by boiling with
(tv) The oxide and hydroxide of boron are weakly acidic
sodium carbonate solution to form borax and sodium
and dissolve in alkalies forming metaborates. The
metaborate.
oxides and hydroxides of Al and Ga are amphoteric
while those of In and T! are basic. Ca,B,0,, + 2Na,CO,
=“ >
B,O, + 2NaOH —> 2NaBO, + H,O 2CaCO, + Na,B,O. + 2NaBO,
B(OH), + NaOH —> NaBO, + 2H,O (ppt) Borax Sodium
The oxides of Al are amphotesic and react with metaborate
alkalies and acids. The precipitate of calcium carbonate is filtered off
Al,O, + 2 NaOH —-> 2NaAlO, + 2H,O and the solution upon concentration and cooling gives
Al(OH), + 3HCI —-> AICI, + 3H,O crystals of borax. Sodium metaborate present in the
(v) The trihalides of group 138 being covalent are mother liquor can be converted to borax by passing
hydrolysed by water. The boron trihalides form carbon dioxide through it.
only tetrahedral species [B(OH),\ because of the 4NaBO, + CO, ———> Na,B,0, + Na,CO,
absence of d-orbitals. But the trihalides of Al and Sodium Sodium
other elements, due to the presence of d-orbitals metaborate tetraborate
form octahedral [Al (H,O),]** species. 3. From boric acid. Borax can also be prepared
(vi) The trihalides of boron are monomeric. This is in small quantities from boric acid by neutralizing it
because of its small size, it cannot accommodate with sodium carbonate.
four large sized halogen atoms around it. The 4H,BO, + Na,CO, ———— Na,B,O, + 6H,O + CO,
monomeric trihalides being electron deficient,
behave as strong Lewis acids and therefore, form On cooling crystals of Na,B,O,.10H,O separate out.
complexes with ammonia, alcohols, ethers, etc. Properties of Borax
H,N: + BF, —>H,N,—>/BF, (¢) Itis a white crystalline solid.
On the other hand, the trihalides of Al and other (it) Borax dissolves in water to give alkaline
elements of the group form bridged dimeric structures solutions due to hydrolysis.
in which the metal completes its octet by accepting an Na,B,0, + 7H,0 t——+ 4H,BO, + 2NaOH
Orthoboric acid
electron pair from a halogen atom of the other molecule.
(weak acid) (strong alkali)
SOME IMPORTANT COMPOUNDS OF BORON
(ii) On heating, borax decomposes and swells
Some useful compounds ofboron are discussed below: losing water of crystallisation. On further
1. Borax, Na,[B,0,(OH),|. 8H,O or heating, it melts into a transparent liquid
Na,B,0,.10H,O which solidifies to a transparent glass like
Borax or sodium tetraborate is an important bead which consists of boric anhydride (B,O,)
compound of boron. It occurs naturally as tincal and sodium metaborate (NaBQO,).
(suhaga) in certain dried up lakes of India, Tibet,
Na,B,O,.10H,O a Na,B,O, + 10H,O
Ceylon and California.
Borax contains the tetranuclear’ units Na,B,O, —tieat_, 9NaBO, + B,O,
B,O,(OH) ae and therefore, its correct formula is pod, metaborate Boric anhydride
Na,[B,0,(OH),]|. 8H,0. (Transparent glassy bead)
SOME p-BLOCK ELEMENTS

When the glassy bead is touched with some coloured Na,B,O, + 2HCl+5H,0 ———> 4H,BO, + 2NaCl
cations such as Ni?*, Co?*, Cr?+, Cu?*+, Mn?*, etc. and
Na,B,O, + H,SO, + 5H,0 ———> 4H,BO, + Na,SO,
heated again, it gives characteristic coloured beads of
metaborates of different ions. The test is performed on 2. From colemanite. The powdered mineral
a platinum wire with the salts containing these cations. colemanite is boiled with water. Sulphur dioxide is
Upon heating, the salts are converted into their oxides passed through the solution when boric acid and
which form coloured metaborates with boron oxide. calcium bisulphite are formed.
This is known as borax bead test. Ca,B,0,, + 11H,O —-~> 2Ca(OH), + 6H,BO,
NiO + B,O, —_ Ni(BO,), 2Ca(OH), + 450, — > 2Ca(HSO,),
Nickel metaborate (brown) Ca,B,0,, + 11H,O + 450, —~> 2Ca(HSO,), + 6H,BO,
CoO + B,O, —— Co(BO,), Calcium bisulphite is highly soluble in water while
Cobalt metaborate (blue) boric acid is very sparingly soluble which separates
Cr,0, + 3B,0, —> 2Cr(BO,), out on cooling.
Chromium metaborate (green) 3. From hydrolysis of boron compounds. It
MnO + BO. —— Mn(BO,), can also be prepared by the hydrolysis of boron
Manganese Manganese metaborate (pink) compounds (e.g. halides, hydrides, etc.)
oxide
BCI, +3H,O ———> H,BO, + 3HCl
CuO + BO, —— Cu(BO,),
Cupric Cupric metaborate (blue)
BN + 3H,0 ———> HBO, + NH,
oxide Properties of Boric acid
Certain metaborates are reduced to the free metal (z) Itis a white crystalline solid with a soft soapy
in a reducing flame by the carbon. For example, touch.
(ii) It is sparingly soluble in cold water but fairly
2Cu(BO,),+C ——> 2CuBO, + B,O, + CO soluble in hot water.
Cupric metaborate Cuprous
(Blue) metaborate (iii) On heating, orthoboric acid loses molecules
(colourless) of water in stages at different temperatures
and forms different products.
2CuBO, +C —~ 2Cu+ B,O. +CO
Metal
(a) At 373 K, it forms metaboric acid by losing a
(red)
water molecule.
(iv) On adding calculated amount of sodium H,Bo, —“*> HBO, + H,O
hydroxide to borax, sodium metaborate is formed. Metaboric acid
Na,B,O, + 2NaOH ——> 4NaBO, + H,O (6) When metaboric acid is heated at 433 K,
Sodium tetraboric acid is formed.
metaborate
(v) On adding calculated amount of hot 4HBO, —*#°*
siosietiele H,B,O, + H.O
2
concentrated sulphuric acid to a hot concentrated Tetraboric acid
solution of borax, boric acid is formed.
On further heating to red hot, it swells up giving
Na,B,O, + H,SO, ———~ Na,SO, + H,B,0, a frothy mass and finally forms boron trioxide.
H,B,0, + 5H,0 ——> 4H,BO,
pes Et
H,5,0, ————> 2B,0, + H,O
Boric acid
Boron trioxide
Uses of Borax
The complete action of heat on boric acid may
({) Borax is used in the laboratory for borax bead
be written as:
test.
433 K
(zz) It is used in the manufacture
glazes for earthen pots.
of enamels and
H,BO, ELSE
373Kk
1108 a
(iii) It is used in the candle industry Boric acid Metaboric acid
(iv) It is used in the preparation of medicinal soaps B,O, Red hot H,B,0,
due to its antiseptic character.
Boron oxide Tetraboric acid
(v) It is used in making optical glass.
(vit) Itisusedin softening of water and cleansing agent. (tv) Acidic character. Boric acid is a weak
2. Orthoboric acid, H,BO, or B(OH), monobasic acid (K, = 1.0 x 10-°). It may be noted that
Boric acid is trivial name for orthoboric acid. boric acid does not act as a protonic acid (i.e. proton
donor) but behaves as a Lewts acid by accepting a
Preparation of Boric acid
pair of electrons from OH™ ion.
1. From borax. It is usually prepared by
acidifying an aqueous solution of borax. The resulting H-COH+ B(OH), —> [B (OH),|+H
solution upon concentration and cooling gives crystals
of boric acid. B(OH), + 20 -O—H —— [B(OH),} + H,0*
cE (v) Action with ethyl alcohol. With ethyl 3. Boron Hydrides
MODERN'S abc + OF CHEMISTRY-XI

alcohol, in the presence of H,SO,, boric acid reacts to Boron forms a number of hydrides having the
form ethyl borates.
general formula BH, ,, and BH, ,.. These compounds
H,BO, + 8C,H,OH ———~> B(OC,H,), + 83H,0 of boron and hydrogen are called boranes. For
Ethyl borate example,
The vapours of ethyl borate burn in air with green
(t) B,H,,, boranes: B,H, (diborane), B.Ho
flame. This is the basis for detecting borates and boric
(pentaborane —9), B,H,, (hexaborane—10),
acid in qualitative analysis.
B,, H,, (decaborane-14) etc.
Uses of Boric acid
(z) Boric acid is used in the manufacture ofenamels (iL) =mins boranes: B,H,, (tetraborane),
and pottery glazes. B-H, , (pentaborane—11), BH, (hexaborane—12),
(it) Itisused asa food preservative in foodindustry. B,H,, (octaborane —14), etc.
(tit) It is used in the manufacture of pigments and These contain special types of bonds known as
borax. multicentre bonds. The simplest among the boranes
(1v) It is used in medicines as an eye wash. is diborane (B,H,), as discussed below :
Structure of Boric acid Preparation of diborane. In the laboratory,
Boric acid has the unit BO,*- ions. These have diborane is prepared by the reaction of iodine on
trigonal planar structure. Boron atom in ground state sodium borohydride in a high boiling solvent such as
has the outer electronic configuration as 2s*2p1. In polyether. In this case sodium borohydride is oxidised
the excited state, one of the electrons from 2s-orbital with iodine.
is promoted to 2p-orbital making three unpaired
electrons. The boron atom undergoes sp” hybridisation 2NaBH,+1, —““*— BH, +2Nal+H,
and each sp” hybrid orbital overlaps with 2p-orbital
It can also be prepared by the treatment of boron
of O- forming three B—O™ bonds.Therefore, BO, has
trigonal planar structure as shown in Fig. 5. trifluoride or boron trichloride with LiAIH, in diethyl
ether.
25 2p
4BF, + 3LiAIH, —““"""_, 2B.H, + 3LiF + 3AlF,
2
eee ona ORIDISATION 4BCl, + 3LiAIH, —“"““" _. 2B,H, + 3AICl, + 3LiCl
On an industrial scale, diborane is prepared by
-62...__ fa
the reaction of BF, with lithium hydride or sodium
FORMS THREE B—O” BONDS hydride at 450 K.
Fig. 5. Trigonal planar structure of BO,*- ion.
2BF,+6LiH —“°* , B,H, + 6LiF
Orthoboric acid has a layer structure in which
BO,* units are linked through hydrogen bonds 2 BF, + 6 NaH —“*— BH, + 6NaF
forming a larger structure as shown in Fig. 6. It is The higher boranes are formed when diborane
clear from the figure that each B-atom is bonded to (BH,) is heated at 373-523 K.
three oxygen atoms and each oxygen atom is bonded
to a hydrogen atom. Properties of diborane
(,) Diborane is a colourless, highly toxic gas with
boiling point of 180 K and melting point of
108 K.
(it) It catches fire spontaneously upon exposure
to air. It burns in oxygen releasing an enormous
amount of energy.
B,H,+ 30, ——— B,O, + 3H,O

? *a
So aA AH ° =-1976 kJ mol
It has higher heat of combustion per unit mass
H
than other substances except hydrogen. Therefore, it
oe" = a N is used as a high energy fuel.

r (zit) Diborane is stable only at low temperatures.


When heated at temperatures between 373 K and
Fig. 6. Structure of orthoboric acid (dotted lines 523 K, it changes to a number of higher boranes.
represent hydrogen bonds.
BH, —Hett_,
Heat
B,H,o, BsH» B;H;,, li? BgHyy ete
SOME p-BLOCK ELEMENTS

The careful control of temperature, pressure and (vit) Diborane gives addition reactions with Lewis
reaction time enables the yield of different boranes. bases (L) to give borane adducts BH,L.
For example, B,H,, is best prepared by storing B,H, BH, +2COQO —-> 2BH,. CO
under pressure at 298 K for 10 days. Borine carbonyl
2B,H, 8388
353—d63 K
.BH, + H2 B,H,+2NMe, ———> 2BH,.NMe,
200 atm, 5 hr
(15% yield) (viit) Diborane dissolves in strong alkali solutions
(tv) Boranes are readily hydrolysed by water (NaOH or KOH) giving metaborates and hydrogen is
liberating H, gas. evolved.
B,H,+6H,O —M——> 2H,BO, + 6H, BH, + 2KOH + 2H,O oo 2KBO, +. 6H,
Boric acid Potassium
metaborate
(v) With alkali metal hydrides in diethyl ether,
boranes form complex borohydrides. (ix) Diborane reacts with halogen acids to give
2li1H + B,H, ~=——> 2Li[BH,] halodiboranes and evolving hydrogen. The order of
Lithium borohydride reactivity of halogen acids is HI > HBr > HCl. HI reacts
(vi) Diborane reacts with ammonia and give at about 323 K in the absence of catalyst while HBr
different products depending upon the conditions. and HCl react in the presence of their respective
Initially, it gives B,H,. 2NH, formulated as aluminium halides.
[BH,(NH,),]* BH,. This gives boron or borazine on BH, + HI —** , B,H,I +H,
heating. When treated with excess of NH, at high BH, + HBr —**-» B,H,Br + H,
temperature, diborane gives boron nitride (BN)...
3B,.H,+6NH, > 3[BH,(NH,),|* BH,
B,H, +HCl —~“*> B,H,Cl + H,
(x) Diborane reacts with halogens under different
Diammoniate of diborane
conditions to form halodiboranes.
|Heat, 450 K 298K
BH, +. Cl, > B,H,Cl + HCl
2B.N,H, + 12H, Explosive
Borazine Chlorodiborane

or 3B,H,+6NH, *2* , 2B,N,H, + 12H, '


BH, + Bry aa
373 K
BoH,Br + HBr
Borazine
Bromodiborane
Bl
zs
Excess NH3

High temp.
(BN), BH, . I, ee BHI + HI
Boron nitride lododiborane

Borazine has structure similar to benzene and Structure of diborane


therefore, it is called inorganic benzene. In this The structure of diborane is very puzzling. Boron
structure, both B and N atoms are sp” hybridised. atom in the excited state has the configuration 2s!
Each N atom has one lone pair of electrons while
2p, 2p,’ It has only three electrons available for
each B atom has empty p-orbital. The n-bonding of
sharing. Now, if boron forms all conventional covalent
borazine is dative and it arises because of sidewise
bonds as in ethane (C,H,), 14 electrons are required
overlapping of fully filled p-orbitals of N and empty
(six B—H bonds and one B—B bond). But there are only
p-orbitals of B. The structure of borazine is shown
12 electrons (six from two boron atoms and six from
below alongwith the structure of benzene.
H H hydrogen atoms). Thus, the molecule is short of two
electrons.
H ! Hi

_
H H H H
\nZ ~<a |
H:B ‘4 B:H H:C : C:H
WZ sy Ss H H H 4H
(12 electrons) (14 electrons)
lal
Borazine Benzene Thus, it cannot have structure similar to ethane.
Because of the similarity of borazine and The electron diffraction studies have shown bridged
benzene, borazine is also known as inorganic structure for diborane as shown in Fig. 7 ahead.
benzene. In this structure there are two types of hydrogen
Boron nitride, (BN), is a giant molecule and atoms. Four hydrogen atoms (2 on the left and 2
resembles graphite in its structure. on the right) are known as terminal hydrogens.
aw 11/16 MODERN'S abc + OF CHEMISTRY-XI

(H) = Bridging hydrogen


H =Terminal hydrogen
B—B distance = 178 pm

Bridging hydrogen
Fig. 7. Structure of diborane.

These are different than the other two hydrogen


atoms, which are known as bridged hydrogens. The oe ee ) COO
two boron atoms and the four terminal H-atoms lie in = state)
the same plane while the two bridging H-atoms are in | 5B FT i
a plane perpendicular to the rest of the molecule. There (Excited | Gd) ood Nacantoronal
are two types of bonds in the molecule : ae ee ee ll

po Hybridisatio
(1) The four terminal hydrogens are bonded by A , penn hybrid
normal covalent bonds formed by sharing of one } igre "
electron each by B and H-atoms.
(it) The bridged hydrogens are different than the
normal electron pair bonds. These are bonded by three
centre electron pair bonds, which involve one
electron pair only (two electrons) but binds three
atoms, B, H and B. Such a bond is indicated by
Three Centred two Electron Bonds
H a

H:-B:--H = or (— » > a ~
H H
5 3 ge a
B B
. Pe a
and is known as three centre electron pair bond. H H
There are two such type of bonds in diborane. (b)
This structure can be explained on the basis of Fig 8. Structure of diborane on the basis of hybridisation.
hybridisation. Boron in the excited state has the (ii) It is used as a fuel for supersonic rockets.
electronic configuration : 2s'2p,*2p,'2p.”. The boron
atom undergoes sp® hybridisation involving 2s and all Hydridoborates
the three 2p orbitals including one empty orbital. The Boron also forms a series of hydridoborates and
four sp? hybrid orbitals adopt tetrahedral the most important among these is the tetrahedral
arrangement as shown in Fig . 8. The vacant hybrid [BH,| ion. Tetrahydridoborates of several metals are
orbital is shown by dotted lines, Two hybrid orbitals of known. For example, lithium and sodium
each B atom overlaps with 1s orbitals of two H atoms. tetrahydridoborates, also known as borohydrides, are
Of the two hybrid orbitals left on each B atom, one prepared by the reaction of metal hydrides with
contains an unpaired electron while the other is diborane in diethyl ether.
vacant. The hybrid orbital containing an unpaired 2MH + B,H,——> 2M*[BH,I (M = Lior Na)
electron of one B atom and the vacant hybrid orbital of Both LiBH, and NaBH, are used as reducing
the second B atom overlaps simultaneously with 1s agents in organic synthesis. They are useful starting
orbital of a H atom to form B—H—B bridge bond as materials for preparing other metal borohydrides.
shown in Fig. 8(a). This bond involves three atoms (two These are discussed in detail in unit 9.
B atoms and one H atom) and contains only two electrons
(because one overlapping B orbital is empty). Therefore,
SOLVED EXAMPLES
this B—H—B bond is called three centre electron
pair bonds. Because of the typical shape of this bond, UY Example 1
it is also called banana bond. The structure of Write balanced equation for the reaction of
diborane is shown in Fig. &(b). elemental boron with

Uses of diborane (t) Oxygen (it) Nitrogen (tit) Chlorine at high


(1) Diborane is used for preparing a number of other temperatures.
boron hydrides such as LiBH,, NaBH,, etc. Solution: (i) 4B +30, Heat . oR 90,
(iz) Itis used as a reducing agent in organic reactions. Boron trioxide
SOME p-BLOCK ELEMENTS

Gi) 2B+N, 2, 2BN Solution: Standard electrode potential values for two
half cell reactions suggest that aluminium has high
Boron nitride
tendency to form Al** (aq) ions. Whereas thallium has
(Wit) 2B+3Cl, —> 2BCl,
lesser tendency than Al. Tl** is not only unstable but it
Boron trichloride
is a powerful oxidising agent also. Thus, Tl* is more
Li Example 2 stable in solution than TI**.
Write balanced equation for the preparation of Aluminium being able to form +3 ion easily is more
elemental boron by reduction of BBr, with electropositive than thallium.
dihydrogen. Example 6
Solution: 2BBr,+3H, ——> 2B+6HBr White fumes appear around the bottle ofaluminium
Example 3 chloride. Why ? {
Write balanced equations for: Solution: Anhydrous aluminium chlvvide’sis2 auruaaiy
(i) BF, +LiH ——> hydrolysed with atmospheric moisture and liberates HCl
(ii) NaH + B,H, ——> gas. Moist HCl appears white in colour.
oe Heat
(iit) HBO, >» AICI,(s) + 3H,O ()——-> AI(OH),(s) + 3HCI(g)
(iv) Al+ NaOH ——->
(v) B,H,+NH, ——> Example 7
(vi) BCl, +H,QOQ ——> A certain salt X, gives the following results :
(vii) B,H, + HO ——> (i) Its aqueous solution is alkaline to litmus.
(ii) It swells up to a glassy meterial Y on strong
Solution: (i) 2BF, + 6LiH —“““ 5 B,H, + 6LiF heating.
Diborane (tit) When conc. H,SO, is added to a hot solution of
(ii) 2NaH +B,H, > 2Na[BH,] X, white crystals of an acid Z separate out.
Sodium borohydride Write equations for all the above reactions and
as
(1iL) HBO, 373K
Heat
HBO, + H,O identify X, Y and Z. N.C.E.R.T
Metaboric acid Solution: (i) Since the aqueous solution of salt (X) is
alkaline, it must be the salt of a strong base and a weak
4HBO, 5-5 H,B,O,
"> 28,0, + H,0 acid. X is borax, Na,B,O,.10H,O. It dissolves in water
Boron trioxide to give alkaline solution due to the formation of strong
(iv) 2Al + 2NaOH + 6H,O——> 2NalAl(OH),] + 3H, base (NaOH),
(v) B,H, + 2NH, ——> 2BH,.NH, Na,B,O,.10H,O Water, H,B,O, + 2NaOH + 8H,O
(vi) BCl, + 3H,0 ——> H,BO, + 3HCl Borax (X) Weak acid Strong base
(vii) B,H, + 6H,0 ——> 2H,BO, + 6H, (iz) On heating it swells up because it loses water
Ortho boric acid molecules.
Example 4 On strong heating, a glassy bead is formed.
What happens when
(t) Borax is heated strongly Na,B,O,. 10H,0 "> Na,B,0, 22% 2Na oo
BO, +B,0,
ee
(it) Boric acid is added to water Glassy bead
(iii) Aluminium is treated with HCl. (Y)
(111) When conc. H,SO, is added to a hot solution of X Le.,
Solution: (i) On heating borax first loses water
molecules and swells up. On further heating it turns to borax, white crystals of an acid (4) separate out. The Z
transparent liquid which solidifies into glass like must be the crystals of boric acid.
material called borax bead. Na,B,O,. 10H,O + H,50,—> 4H,BO, + Na, SO, +5H,O
Na,B,0,.10H,O 4225 Na,B,O, "5 2NaBO, + B,O, Boric acid
(Z)
Borax Sodium Boric Example 8
metaborate anhydride
Boron is unable to form BF? ion, Explain.
(iz) It acts as Lewis acid
B(OH), + H,;O —— [B(OH),\ + H,O*
Boric acid Solution: Boron does not have vacant d-orbitals in its
(iit) Liberates hydrogen valence shell. Therefore, it cannot expand its octet.
2A] + GHC] ——> 2Al** + 6Cl- + 3H, Therefore, maximum covalency of boron cannot exceed
four and it does not form BF’,* ion.
Example 5
Standard electrode potential values, E° of Al**/Al Example 9
is -1.66 V and that of Tl°+ /Tl is 1.26 V. Predict Account for the fact that aluminium chloride exists
about the formation of M** ion in solution and as a dimer.
compare the electropositive character of two metals. Solution: In AIClI,, there are six electrons around Al
and are two less than the octet. So, Al in AICI, is electron
cE deficient whereas Cl atom has a lone pair of electrons to
MODERN'S abc + OF CHEMISTRY-XI

Identify (X), (A), (B), (C) and (D). Write suitable


donate. Therefore, each Al atom completes its octet by equations to support their identities. J
accepting a lone pair from Cl atom of another aluminium
Solution: Since the metal X reacts with sodium
chloride molecule. Thus, it forms a dimer. hydroxide to form a white ppt (A), which dissolves in
AL. J. x/\,f excess of NaOH to form a soluble complex (B), metal X
is aluminium. The reactions are :
2Al + 3NaOH —-> Al(OH), + 3Na*

Cl
y Cl
#* Cl (X) White ppt
(A)

Al(OH), + NaQH —-+ Nat [Al(OH),F


Example 10. (A) Sodium tetra
hydroxoaluminate
Why is boric acid considered as a weak acid ? Soluble (B)
Compound (A) is amphoteric in nature and therefore,
is soluble in dil HCl to form compound C, which
Solution: Boric acid is not able to release H* ions of its
must be AICl,.
own. It receives OH~- ions from water to complete its
octet and therefore, releases H* ions. Therefore, it is a
A(OH), +3HCl1 —> AICI, + 3H,O
(A) Aluminium
weak acid.
chloride
Example 11 (C)
Why does boron not form B** tons ? Compound (A) on heating gives (D) which is alumina,
which is used to extract metal.
Solution: Boron has very small size and has very high
2A(OH), ——> AI,O, + 3H,O
sum of three ionisation enthalpies (IE, + IE, + IE,).
(D)
Therefore, it cannot lose its three electrons to form B*
ions. LU) Example 15
Example 12 Suggest reasons why the B—F bond lengths in BF,
Aluminium forms [ALF
,|* but boron does not form (130pm) and BF, (143 pm) differ.
[BF |? ion. Why ? Solution: BF, is a planar molecule in which B is sp?
hybridised. It has empty 2p-orbital. Because of similar
Solution: Boron does not have vacant d-orbitals in its
valence shell and hence cannot extend its octet, It cannot sizes of B and F, back bonding occurs in which a lone pair
show covalency more than four. However, Al has vacant is transferred from F to empty p-orbital of B forming prt-
d-orbitals in its valence shell and can show a covalency pt bond. As a result, B—F acquires some double bond
of six, character. On the other hand, in BF, ion, B is sp*®
hybridised and forms four B—F single bonds. It does not
Example 13
have vacant p-orbital available to accept the electrons
A compound (A) of boron reacts with NMe, to give
from F atom and hence in BF, the B—F bond is purely
an adduct (B) which on hydrolysis gives a compound
single bond. Since double bond is shorter than single
(C) and hydrogen gas. Compound (C) is an acid.
bond, therefore, the B—F bond length in BF, is shorter
Identify the compounds A, B and C. Give the
(130pm) than B—F bond length in BF, (143 pm).
reactions involved. (NCERT Exemplar Problem)
Solution: The compound A is diborane which reacts
with NMe, to form adduct BH,.NMe, (B). The
hydrolysis of compound A gives boric acid,
Q1. What is the formula of colemanite ?
H,BO, (C) and H, gas. The reactions are :
Q@ 2. Are all the B—H bonds in diborane equivalent ?
BH, + NMe, —— B,H,.NMe,
Q3. What is the basicity of orthoboric acid ?
(A) (B)
Q@ 4. Name two ions which can be detected by borax
B,H, + 6H,0 ——> 2H,BO, + 6H, bead test.
Boric acid Q 5. What are the basic units of the larger structure
(C) of orthoboric acid? How are they bonded ?
Example 14 Q@6. Is BF, a Lewis acid or a Lewis base ?
When metal X is treated with sodium hydroxide, a Q 7. Why are boron halides called Lewis acids ?
white precipitate (A) is obtained, which ts soluble
Q8. What is the structure of BO,* ion ?
in excess of NaO8d to give soluble complex (B).
Q 9. What compounds are formed on heating boric acid ?
Compound (A) ts soluble in dilute HCl to form
compound (C). The compound (A) when heated 0 10. What is the oxidation state of boron in orthoboric
acid ?
strongly gives (D), which is used to extract metal.
SOME p-BLOCK ELEMENTS 11/19 ae

Answers to Practice Problems ee


©5. BO,” ions, bonded through hydrogen bonds.
©1. Ca,B,0,, ©6. Lewis acid.
© 2. No, there are two types of bonds in diborane; two ©7. Because they can accept electron pairs.
electron normal bonds and three centred two elec- © 8. Trigonal planar.
tron bonds.
© 9. HBO,, H,B,O, and B,O,.
© a Monobasic.
© 10. +3.
© 4. Ni**, Co”*

Wuestions =
== 1
Q.1. What is the oxidation state of boron in boric acid ? What is the basicity of this acid ?
Ans. Boric acid is H,BOg,.
Oxidation state of boron = +3
Boric acid is monobasic.
Q.2. How does sodium hydride react with diborane ?
Ans. Sodium borohydride is formed.

2NaH + B,H, —“@*t-» 2NaBH,


Q.3. Gallium has higher ionization enthalpy than aluminium. Explain why ?
Ans. Gallium has higher ionization enthalpy than aluminium because of higher effective nuclear charge. This
is due to additional ten 3d electrons which do not screen the nuclear charge effectively so that the outer
electrons are more strongly held.
Molten aluminium bromide is a poor conductor of electricity. Explain.
Aluminium bromide is predominantly covalent compound. Even in the molten state it does not have ions
which can conduct electricity.
Anhydrous AICI, is covalent. From the data given below, predict whether it would remain
covalent or become ionic in aqueous solution (ionisation enthalpy for Al = 5137 kJ mol",
AH, ydration Of Al** = - 4655 kJ mol!) and AH, ydration Of Cl = - 381 kJ mol-!,
The energy released during hydration of Al°+ and 3CI- ions = — 4655 + 3(— 381) = — 5808 kJ mol-!.
The energy released during hydration is greater than the ionisation enthalpy for Al (5137 kJ mol).
Hence AICl, will be ionic in solution.
Boron forms no compounds in unipositive state but thallium in unipositive state is quite stable.
Boron has electronic configuration, 2s? 2p! and therefore forms compounds in trivalent state. However,
thallium prefers to form compounds in +1 oxidation state rather than in +3 oxidation state as suggested
by its group number. This is due to inert pair effect. According to this effect, the 6s? electrons in the case
of heavy metals preferably do not take part in bonding.
Why is B-X bond distance in BX, shorter than the theoretically expected value ?
This is due to pm—pz back bonding of the fully filled p-orbital of halogen (X) into the empty p-orbital of boron.
Is boric acid a protic acid ? Explain. N.C.E.R.T.
Boric acid is not a protic acid because it does not ionise in water to give a proton, But it acts as a Lewis
acid by accepting electrons from hydroxide ion :

(HO),B +20, — (HO),B— "6 _—> [B(OH), | + H*


or B(OH), (aq) + 2H,O —~+ [B(OH),\ (aq) + H,O*(aq)
Thus, boric acid acts as Lewis acid because it reacts with OH (Lewis base) ions rather than acting as a
Bronsted acid.
Q.9. How can you explain the higher stability of BCl, as compared to TICI, ? N.C.E.R.T.
Ans. Boron exhibits +3 oxidation state and can form stable BCl.. Thallium shows oxidation state of +3 as well as
+1 but +1 oxidation state is predominant than +3 because of inert pair effect. Therefore. TICI, is not stable.
It can form stable TIC.
STiia0 Q.10. Explain amphoteric character of aluminium.
MODERN’S abc + OF CHEMISTRY-XI

Aluminium reacts with acids as well as alkalies and therefore, shows amphoteric character.
2Al (s) + GHCl(ag) ——> 2Al** (aq) + 6Cl(aq) + 3H,(g)

2Al(s) + 2NaOH(aq) + 6H,O (1) ——> 2Na*[AL(OH),l(aq) + 3H,(g)


Sodium
tetrahydroxoaluminate (IIT)
Q.11. Why do trihalides of Group 13 elements fume in the moist air ?
They are hydrolysed by water forming hydrogen halides.
MX, + 3H,O——> M(OH), + 3HX.
Q.12. Explain why aluminium, though electropositive metal, finds extensive use as a structural material.
Aluminium is electropositive metal and reacts with oxygen (or air) to form a hard protective layer of Al,O,
which protects it from further action. Therefore, it becomes passive and is extensively used as a structural
material.

Q.13. If B-Cl bond has a dipole moment, why does BCI, has zero dipole moment ?

Ans. BCI, molecule has a symmetrical trigonal structure in which three B—Cl bonds
are oriented at an angle of 120° to one another. The three bonds lie in one
plane and the dipole moments of these bonds cancel one another giving net LY’ iN
dipole moment of zero. Cl Cl
Q.14. Why does gallium undergo disproportionation reactions ?
Ans. Gallium shows both +1 and +3 oxidation states due to inert pair effect. However, its +5 oxidation state
is more stable than +1 oxidation state. Therefore, +1 gallium is less stable than +3 gallium and hence
undergoes disproportionation to form gallium and more stable +3 gallium ions in aqueous solution as
shown below :
Reduced

+1 0 +3
3Ga X(s) MM» 2Gal(s) + Ga®** (aq) + 3X (aq)
LL / ~~ Wf
Oxidised
or 3Gat (aq) —> 2Ga(s) + Ga®* (aq)
Q. 15. Complete the following reactions :
(i) NaBH, + 1, —
i Conc.
(it) B(OH), + C,H,OH Faso,

(itt) NaBO, + CO, —>

Ans. (Gi) 2NaBH, +1, —Vieyme_. BLH, + 2Nal + H,


(ii) B(OH), + 3C,H,OH —S* #259 ,B(OC,H,), + 3H,O
(iit) 4NaBO, + CO, —> Na,B,O, + Na,CO,
Q.16. Unlike In*, TI’ is more stable with respect to disproportionation.
Ans. Both In and Tl can show oxidation states of + 1 and + 3. The + 1 oxidation state of Tl is more stable than its
+ 3 oxidation state because of inert pair effect. But + 3 oxidation state of In is more stable than its + 1
oxidation state. Therefore, in aqueous solution, less stable In* undergoes disproportionation to form more
stable In** but Tl* being more stable does not undergo disproportionation to form T1*.

3 In* (aq) __Disproportionation 9 In(s) + In®*(aq)

3 Tl* (ag) ——————————>. does not undergo disproportionation.


SOME p-BLOCK ELEMENTS

trace amounts in coal and zinc concentrates. Tin occurs


GROUP 14 ELEMENTS mainly as cassiterite, S5nO, and lead is found as
CARBON FAMILY galena, PbS. Other ores of lead are anglesite (PbSO,,)
and cerussite (PbCO,). Small quantities of lead ores
The group 14 of the periodic table contains five occur in Rajasthan. The abundance of the elements in
elements : carbon (C), silicon (Si), germanium (Ge), the earth’s crust by weight is given below :
tin (Sn) and lead (Pb). This group is known as carbon Element Abundance in Relative
family. earth's crust (ppm) abundance
OCCURRENCE C 180 17
Carbon occurs both as the free element (coal, Si 2 Vix 10" 2
eraphite or diamond) and in the combined form (mainly Ge 1.5 54
as carbonates of Ca, Mg and other electropositive Sn 2.1 AQ
elements). It also occurs as CO,, an important Pb 13 36
constituent of atmosphere (0.03%). Carbon is
GENERAL CHARACTERISTICS OF
seventeenth in order of abundance in the earth's crust.
It also occurs widely as hydrocarbons (natural gas, GROUP 14 ELEMENTS
petroleum). Its combination with other elements such A. Electronic Configurations
as dihydrogen, dioxygen, chlorine and sulphur provides The atoms of these elements have four electrons
an astonishing array of materials ranging from living in the outermost shell, two in s and two in p sub-shell.
tissues to drugs and plastics. Organic chemistry is the The general electronic configuration of this group may
branch of chemistry consisting of compounds of carbon. be expressed as ns*np*. The electronic configuration
Naturally occurring carbon contains two stable isotopes of the atoms of this group are given in Table 5.
2C and °C. In addition to these, a third isotope of Table 5. Electronic configuration of group
carbon, '*C is also present which is radioactive and 14 elements.
has half life 5770 years and is used for radiocarbon
dating for determining the age of earth, fossils, etc. Element Symbol | Atomic | Electronic
configuration
Silicon is the second most abundant element
(about 27.7% by mass) in the earth’s crust after [He] 2s? 2p?
oxygen (about 45.5 mass%) as silica (sand or quartz) [Ne] 3s? 3p?
[Ar] 3d!° 452 Ap?
and silicates. In silicates, [Si0,] unit may occur
individually as groups or be linked to form chains, rings, [Kr] 4d!° 5s? 5p?
sheets or three dimensional frameworks. Silicon is very [Xe] 4/145d"° 6s? 6p
important component of ceramics, glass and cement. B. Atomic and Physical Properties
Germanium occurs rarely. Germanium minerals The common physical constants of group 14
are extremely rare but the element is distributed in
elements are given in Table 6.

Table 6. Some physical constants of group 14 elements.

Atomic number 6 14 o2 50
Pa
82
Atomic mass 118.71 207.2
Atomic radius (pm) 140 146
Ionic radius (pm) M* 69 78
118 119
Ionisation enthalpy (kJ mol-*)
IE, 708 715
IK, 1411 1450
IK, 2942 3081
IE, 3929 4082
Electronegativity RE 1.8 1.9
Density (g cm™) 3.51 7.26 11.34
(for diamond)
eee (for B-form)
( for graphite)
m.p. (K) 4373 505 600
b.p. CK) — 2024
Electrical resistivity
(ohm cm at 293 K) 1014 — 1016 2x 10°
Sine 1. Atomic radii C. Oxidation State
MODERN'S abc + OF CHEMISTRY
-XI

The atomic radii of group 14 elements are less than The general valence shell electronic configuration
the corresponding elements of group 18. of elements of group 14 is ns*np*. Therefore, these
The atomic radii of group 14 elements increase elements can achieve noble gas configuration either
down the family. by losing or gaining four electrons forming M* and
M* ions. Since the sum of first four ionisation
Explanation. The decrease in atomic radii is due
enthalpies of carbon is very high, it is not usually
to the increase in effective nuclear charge on going
available in chemical reactions and therefore, C* ions
from group 13 element to group 14 element within
are not easily formed.
the same period. As a result, the outermost electrons
are attracted more strongly towards the nucleus and On the other hand, carbon cannot take up four
electrons to form C* ion. This process is also
therefore, atomic radius decreases. Within the group,
energetically unfavourable because it requires a large
the atomic radii increase on going down the group,
amount of energy for adding four electrons. Therefore,
due to the increase in the number of electron shells.
the compounds in +4 oxidation state are generally
It may be noted that the increase in atomic radii covalent in nature. However, on moving down the
from Si onwards is small. This is due to the presence group the tendency to form covalent compounds
of completely filled d-and f-orbitals which have decreases and the tendency to form ionic compounds
relatively ineffective shielding of the valence electrons. increases. Thus, the elements of group 14 show
2. Ionisation enthalpies tetravalency by sharing four of its valence electrons.
The first ionisation enthalpies of these elements Therefore, they have oxidation state of +4.
are higher than the corresponding members of group In addition, Ge, Sn and Pb also show +2 oxidation
13 elements. state. It has been observed that on going down the group,
Explanation. The higher ionisation enthalpies the +2 oxidation state becomes more stable : Ge < Sn < Pb.
are due to the higher nuclear charge and smaller size For example, for lead compounds, + 2 oxidation
of atoms of group 14 elements. state is more predominant. This is due to inability of
While moving down the group, the tonisation s- electrons of the valence shell of tin and lead to
enthalpies decrease. The first ionisation enthalpies participate in bonding. In other words, the pair of s
(A.H,) of elements of group 14 follow the order : electrons, 6s in lead, behaves as inert. This is known as
C > Si > Ge > Sn < Pb inert pair effect.
This is due to increase in atomic size and screening The relative stabilities of these two oxidation
effect which overweigh the effect of increase in nuclear states vary down the group. Carbon and silicon mostly
charge. Therefore, the outermost electron becomes show +4 oxidation state. Germanium forms stable
less and less tightly held by the nucleus and ionisation compounds in +4 oxidation state and only a few in +2
enthalpy decreases. The observed small decrease in oxidation state. Tin forms compounds in both oxidation
A,H from Si to Ge to Sn and slight increase in A,H from states +4 and +2.
Sn to Pb is due to the combined effects of poor shielding These elements in + 2 oxidation state can act as
effect of intervening d-electrons in Ge and Sn and d reducing agents while in + 4 oxidation state they can
and f orbitals in Pb and the increased atomic size of act as oxidising agents.
the elements. Reducing agent M?+ —-> M4 + 2e-
3. Melting and boiling points Oxidising agent M/‘*+ + 2e-—> M?*+
The atoms of this group form covalent bonds with For example, Sn(II) salts such as SnCl, are widely
each other and therefore, there are strong binding used as reducing agents.
forces between their atoms in both solid and liquid 2FeCl, + SnCl, ——> 2FeCl, + SnCl,
states. Consequently, the melting and boiling points 2HgCl, + SnCl, ——~> Hg,Cl, + SnCl,
of group 14 elements are much higher in comparison Hg Cl, + SnCl, ——-+> 2Hg + SnCl,
to group 13 elements. On moving down the group, the
On the other hand, lead compounds in +2 oxidation
melting and boiling points decrease. 5 state are more stable and in +4 oxidation state behave
4. Metallic character C as strong oxidising agents.
Due to large ionisation For example, Pb (IV) salts such as lead tetraacetate
enthalpies, the elements of this group Pore Pb(OCOCH,), and PbO, are widely used as oxidising
are less metallic than the elements of Si agents.
group 13. On moving down the group, Non-metal
the metallic character increases from Ge PbO, + 4HCI (conc.) —““*> PbCl, + Cl, + 2H,O
carbon to lead. For example, carbon is Wfetalloid CH,OH
typical non-metal, silicon also behaves
as non-metal, but in certain physical Sn H,OH + Pb(OCOCH,), >
properties, it behaves as a semi-metal. Metal Ethylene glycol
Germanium is metalloid while tin and Pb
INCREASE
CHARACTER
METALLIC
'
2HCHO + Pb(OCOCH,), + 2CH,COOH
lead are typical metals. Metal Formaldehyde Lead acetate
SOME p-BLOCK ELEMENTS

In the tetravalent state, the number of electrons Being acidic, CO,, SiO, and GeO, react with bases.
around the central atom in a molecule (e.g., C in CCL,) CO,+2NaO0H —W\> Na,CO, + H,O
is eight. Therefore, these are regarded as electron
CO, + Ca(OH), ———> CaCO, +H,O
precise molecules and hence neither act as electron
acceptors or electron donors. Although carbon SiO, + 2NaOH —- Na,si0, + H,O
cannot extend its covalence more than four, other Sodium silicate
elements of this group can do so because of the presence GeO, + 2NaOH ——-> Na,GeO, +H,0
of d-orbitals in them.
5odium germanate
C. Chemical Properties The dioxides SnO, and PbO, are amphoteric. For
The elements in this group are relatively example, SnO, reacts with acids as well as alkalies.
unreactive, but reactivity increases down the group. snO, + 4HCl ——-+ SnCl, + 2H,O
The M(II) oxidation state becomes increasingly stable
on descending the group. General chemical properties
SnO, + 2NaOH ———> Na,SnO, + H,O
of group 14 elements are : Similarly,
PbO, + 2NaOH ——+> Na,PbO, + H,O
1. Reactivity towards oxygen
Sodium plumbate
All the elements of group 14, when heated in
oxygen form oxides. They generally form two types of PbO, + 4HC] —“@*— PbCl, + 2H,O
oxides i.e. monoxides (MO) and dioxides (MO,). SiO
PbO, is a powerful oxidising agent. Therefore,
exists only at high temperatures. Besides monoxides
it reacts with cone. HNO, or H,SO, to evolve oxygen
and dioxides, lead also forms another oxide called and oxidises HCI to Cl.
trilead tetroxide or red lead (Pb,O,). It is formed by
heating PbO (litharge) with excess of air or O, at 673K. 2PbO, + 4 HNO, —*** + 2Pb(NO,), + 2H,O + O,
6PbO + 0, “==> 2Pb,0, 2PbO, + 2H,SO, —*"**> 2PbSO, + 2H,O + O,
(1) Monoxides. All the elements of this group form PbO, + 4HC] —“* >PbCl, + Cl, + H,O
monoxides of the general formula MO such as CO, SiO,
GeO, SnO and PbO. Among monoxides, CO is neutral, 2. Reactivity towards water
GeO is distinctly acidic while SnO and PbO are Carbon, silicon and germanium are not affected
amphoteric. by water. However, tin reacts with steam to form SnO,
Since +4 oxidation state of carbon is the most
and dihydrogen gas.
stable, therefore, among the monoxides of group 14,
CO is the strongest reducing agent. Therefore, it is used Sn + 2H,O —#* 5 SnO, + 2H,
in the extraction of many metals from their oxides. Steam
Lead is not affected by water probably because of
ZnO + CO Heat, Zn + CO, a protective oxide film.

Fe,0, + 3CO 224, oFe+ 300, 3. Reactivity towards halogens


The elements of group 14 form tetrahalides of the
(ii) Dioxides. All the members of the family form formula EX, and dihalides (EX,) (X = F, Cl, Br, I).
dioxides with the molecular formula MO, e.g. COQ,,
Tetrahalides. All the elements form tetrahedral
SiO,, GeO,, SnO,, PbO,. These dioxides differ
markedly in their structures and properties. Carbon halides, MX,, which are mostly covalent. Except
dioxide 1s a gas. carbon, all other members react directly with halogens
under suitable conditions to form tetrahalides. Si and
On the other hand, silicon dioxide called silica
Ge react readily with all the halogens forming volatile
consists of continuous lattice of silicon and oxygen
halides, 5iX, and GeX,. Sn and Pb are less reactive.
connected by covalent bonds (discussed later, Fig. 12).
Sn reacts with Cl, and Br, in cold and with F, and I,
Solid carbon dioxide is known as dry ice. It is soft, on warming, forming SnX,. Pb reacts with F, in the
white snow like substance and looks like ice. However, it
cold forming PbF, and with chlorine forming PbCl,.
does not wet a piece of cloth or paper because it sublimes
These tetrahalides have tetrahedral structure. The
without melting. On the other hand, ordinary ice wets a
piece of cloth on melting since it does not sublime. Solid central atom in these halides undergoes sp?
carbon dioxide is used as a refrigerant under the hybridisation and the molecule is tetrahedral in shape.
commercial name drikold. Most of the tetrahalides are covalent in nature.
Exceptions are SnF, and PbF,, which are ionic in
Among dioxides, CO,, SiO, are acidic, GeO, is also
nature. PbI, does not exist because Pb—I bond initially
acidic but less acidic than SiO, whereas SnO, and formed during the reaction does not release sufficient
PbO, are amphoterie. energy to unpair 6s? electrons and excite one of them
STiias MODERN'S abc + OF CHEMISTRY-XI

to higher orbitals to have four electrons around the beyond four and therefore, it cannot accept electrons
lead atom. The nonexistence of PbI, may also be from water molecules. On the other hand, SiCl, is
explained due to strong oxidising power of Pb (IV) and readily hydrolysed by water because silicon has vacant
strong reducing power of I .
3d orbitals in its outermost shell. As a result, silicon
The chlorides except CCl, are hydrolysed readily extends its coordination number beyond four. The
by water. For example, vacant 3d orbitals of silicon accept electrons from water
SiCL, + H,OQ ——~+ 8:1(OH), + 4HCI and SiCl, can be hydrolysed as shown below :
Silicic acid
snCl, + 2H,O ——_> 5nO, + 4HCl
CCl, + H,O ——_> No reaction
\ EY \7 -
7" XN ZN, ,
Hydrolysis of silicon tetrafluoride gives silica and
fluorosilicic acid.
3siF, + 2H,O ——+>S8i10, + 2H,siF, Cl Cl
Fluorosilicic acid
Silicon tetrachloride es
Carbon tetrahalides do not form complexes because
carbon does not have vacant d—orbitals in its valence cl OH
shell and therefore, cannot increase its coordination
number. On the other hand, the halides of other Si + HCl
elements form complexes due to the availability of
2) Eee |
vacant d-orbitals in the valence shell. These can
increase the coordination number to six. In other words, One Cl atom is replaced in SiCl, by OH group
they behave as Lewis acids but CCl, does not. forming SiC], .OH. The process continues till all the
Sif, + 2F- ——_+> [SiF,]?- 4 Cl atoms are replaced forming Si(OH), or H,SiO, .
snCl, + 2Cl —+ [SnCl,]?- H,O (hydrated silicic acid). Thus,
PbCl, and PbBr, are unstable while Pbl, is not ol OH
known.
Dihalides. Carbon does not form dihalides while NO 3H,O 3HCl
NS
>1 + 3HC
the other elements form dihalides, MX,. The stability
of dihalides increases down the group. The Cl
~~, Cl /N.
stability of dihalides follows the sequence :
Silicic acid
CX, << SiX, << GeX, << S5nX, < PbX,,.
or H,Si0,.H,O
For example, PbX, is more stable than PbX,. On
the other hand, GeX, is more stable than GeX,. 4, Reactivity with acids and alkalies
Tin(II) chloride is obtained by dissolving tin in conc. C,Siand Ge are unaffected by acids. Sn dissolves
in dilute HNO, forming Sn(NO,),. Lead dissolves
HCl. When the solution is cooled, crystals of SnCl,.2H,O
slowly in dilute HCl forming the sparingly soluble
(dihydrate) separate out. Anhydrous SnCl, is prepared
PbCl, but quite readily in dil. HNO, forming Pb(NO,),
by heating tin in a current of HCl vapour. and oxides of nitrogen,
SnCl, is used as a reducing agent in acid solution. 4Sn +10HNO, —> 4Sn (NO,),+ NH,NO, +3H,O
For example, it reduces Fe** to Fe** state as:
Pb + 4HNO, ——> Pb(NO,), + 2NO, + 2H,O
2Fe** + Sn => Sn* + 2Fe*+
Lead also dissolves in organic acids (e.g., oxalic
Lead (II) halides are formed by adding halide ions acid, acetic acid, citric acid, etc).
to soluble lead salt :
Pb** + 2X- ——>»PbxX, (X= F, Cl, Br or I) Pb + 2CH,COOH + 2 O, —— (CH,COO),Pb + H,O
2 Lead acetate
Pb (IT) halides are colourless solids except PbI, which
is yellow. These are sparingly soluble in water. In Lead does not dissolve in dil H,SO, because a
qualitative analysis, Pb** ion is tested as PbCl, or PbI,. surface coating of PbSO, is formed.
Graphite reacts with hot concentrated HNO,
CCl, cannot be hydrolysed while SiCl, can
forming mellitic acid [C,(COOH),] and with a mixture
be easily hydrolysed. This can be easily explained of HF and HNO, forming graphite oxide. On the
on the basis of absence of d-orbitals in the valence other hand, Si gets oxidized and fluorinated by
shell of carbon. Carbon has only s and p-orbitals. As a concentrated, HF, HNO, mixture. Ge dissolves in hot
result, carbon cannot increase its coordination number concentrated H,SO, as well as HNO,. Sn dissolves in
SOME p-BLOCK ELEMENTS
several concentrated acids. Pb does not dissolve in branched chains and rings of different sizes and lengths
concentrated HCI because a surface coating of PbCl, as shown below :
is formed. However, powdered lead is readily dissolved
by boiling hydrochloric acid due to the formation of
soluble complex, H,[PbCl,].

Pb + 2HCl ——> PbCl, + H,


| | tl
PbCl, + 2HCl ——> H,[PbCl,] — C—

Chloroplumbous acid |
Carbon is unaffected by alkalies. Si reacts slowly |
Straight chain Branched chain) Cyclic /\
or Ring
with cold aqueous solution of NaOH and readily with
hot solutions forming silicates. The tendency to catenation depends upon the
Si + 2KOH + H,O —— K,SiO, + 2H, strength of M—M bond. As we go down the group the
strength of M—M bond decreases due to increase in
It also dissolves in fused sodium carbonate size of the atom. The bond enthalpies of various bonds
displacing carbon. are :
Na,CO, + Si ——~> Na,Si0, + C Bond Bond enthalpy
Sn and Pb are slowly attacked by cold alkali and (kJ mol!)
rapidly by hot alkali forming stannates and C—~C 348
plumbates. Si— Si 297
Sn + 2NaOH + 4H,O ——> Na,[Sn(OH),] + 2H, Ge — Ge 260
Sodium stannate Sn — Sn 240
Pb + 2NaOH + 4H,O —— Na,[Pb(OH),] + 2H,
Sodium plumbate Thus, the tendency to catenation decreases as :
C >> Si>Ge = Sn> Pb.
This also shows amphoteric character of Sn
and Pb. 2.'Tendency to form multiple bonding. Carbon
has also unique tendency to form stable multiple bonds
5. Formation of hydrides
(double and triple) with itself and with other atoms of
All the elements except lead of group 14 form small size and high electronegativity. For example,
covalent hydrides of the type MH,. The number of
C=C, C=C, C=O, C=5 and C=N
hydrides, their thermal stability and their ease of
In these compounds there are pr—pt bonding.
formation decreases from C to Pb.
However, silicon and other elements of the group have
Carbon forms a number of cyclic and acyclic
vacant d-orbitals in their valence shell. As a result,
hydrides known as hydrocarbons. The common are
silicon can form dm—pt double bond with elements like
alkanes (having general formula, C_H,, , ,), alkenes
O and N due to the availability of vacant d—orbitals in
(having general formula C,H, )alkynes (having general
its outermost shell. This bond is formed by the donation
formula, C,H,,_.) and aromatic hydrocarbons. Silicon
of electron pair from filled 2p orbitals of N or O to the
forms only a limited number of hydrides known as vacant 3d orbitals of Si. It is called back bonding.
silanes having general formula Si,H,,,. (n = 1 — 8).
The silanes resemble hydrocarbons in many

Q)
properties.
Germanium hydrides are called germanes having
the general formula Ge,H,, ,,(n = 1-5). Tin forms 7X
two hydrides, SnH, and Sn,H,. On the other hand,
lead is known to form only one hydride, plumbane VY _

PbH,, which is difficult to prepare. Empty 3d Filled 2p orbital dr=pm bond


D. Unique Character of Carbon orbitals of Si of NN

Carbon being the first member of group 14 shows


many unique characteristics as described below : The presence of dt—pt bond helps to explain the
bonding in trimethylamine (CH,),N and trisilylamine
1. Catenation. This is one of the remarkable
N(SiH.,)s.
property of carbon to form bonds with itself. This
The compounds (CH,).N and (SiH,),N have
property of forming bonds with atoms of the same
different structures. In trimethylamine, the N atom
element ts called catenation. Carbon shows maximum
involves sp? hybridisation having one lone pair in one
catenation in its group. On moving down the group 14,
of its tetrahedral position. The geometry is regarded
the catenation tendency decreases. Because of
as pyramidal [Fig. 9(a)]. In the trisilylamine, sp?
catenation, carbon atoms can link with each other to
hybridisation takes place and the molecule has
form a large variety of compounds such as linear chains,
trigonal planar geometry [Fig. 9(b)].
MODERN'S abc + OF CHEMISTRY-XI

Carbon exists in two types of allotropic forms :


? & Figoy 72 SIH, 1. Crystalline. Diamond and graphite are two
f 7 y = oova i crystalline forms of carbon having well defined
, N N < * N- ¢g
/ % dTT= pir ‘. ¥ aut = ptt structure.
Fs ‘, bonds > a bonds
2. Amorphous. There are many amorphous
i CH, ‘Bl oe
H.C a ar CH;
~ \hi/
SiH,
forms of carbon such as coal, wood charcoal, animal
charcoal, lamp black, coke, gas carbon, etc.
Pyramidal Trigonal planar
(sp? hybridisation) (sp* hybridisation) Of these allotropic forms, diamond and graphite
(a) (b) are the most important and are discussed below :
Fig 9. Structure of (CH,),N and (SiH,), N involving Structure of Diamond and Graphite
dx—pz back bonding in (SiH,),N.
Diamond and graphite are two crystalline
The lone pair is present in unhybridized allotropic forms of carbon as shown in Fig. 9. Some of
p-orbital. This overlaps with empty d—orbitals on each the important properties of these are :
of the three Si atoms and results in back pzr—dr ({) Diamond is denser than graphite. The density
bonding. Since the lone pair on nitrogen is involved in of diamond is 3.514 g em? while that of graphite is
back bonding with vacant d—orbitals of silicon, N has 2.226 g cm,
no longer a lone pair of electrons. As a result, the
(ii) Diamond is hard while graphite is soft.
molecule does not have donor properties. Its structure
Graphite has lubricating properties. On the other
is plannar. Such type of pn—dz bonding cannot occur
hand, diamond is the hardest substance known.
in (CH,), N because carbon does not have d-orbitals.
(111) Diamond is a bad conductor of electricity while
ALLOTROPES OF CARBON graphite is a good conductor of electricity.

Elemental carbon exists in many allotropic forms. (1v) Diamond is extremely chemically unreactive
Allotropic forms or allotropes are whereas graphite is quite reactive.

the different forms of the same element The different properties of diamond and graphite
having different physical properties but can be easily explained on the basis of their structures
almost stmilar chemical properties. as described below.
Structure of diamond. Diamond has a network
The phenomenon of existence of allotropic forms
structure consisting of a very large number of carbon
of an element is called allotropy.

(a) Structure of diamond.


SOME p-BLOCK ELEMENTS

atoms bonded to each other. Each carbon atom is sp* (tii) Conductivity. Graphite has delocalised x
hybridised and is bonded to four other carbon atoms by electrons which are relatively free to move under the
single covalent bonds. The structure of diamond is influence of electric field. Therefore, graphite is a good
conductor of electricity. However, in diamond all the
shown in Fig 10 (a). It is clear from the figure that each
valence electrons are involved in forming carbon-
carbon atom lies at the centre of a regular tetrahedron
carbon bonds and does not have free electrons and,
and the other four carbon atoms are present at the therefore, it is a bad conductor of electricity.
corners of the tetrahedron. In this structure all C—C (tv) Reactivity. In diamond, all the four
bond lengths are equal to 154 pm. Therefore, there is a valencies of carbon are satisfied by bonds with other
three-dimensional network of strong covalent bonds in carbon atoms. Therefore, diamond is extremely
diamond. This makes diamond an extremely hard unreactive at room temperature. Graphite, on the
crystal with very high melting point (3843 K). other hand, reacts more readily due to the presence of
Since all the valence electrons of carbon are m bonds.
strongly held in carbon-carbon bonds, diamond is a Uses of Diamond
poor conductor of electricity.
(1) Due to its hardness, it is used for cutting glass,
Structure of graphite. In graphite, each carbon making bores for rock drilling and for making
atom undergoes sp” hybridisation and is covalently abrasives for sharpening hard tools.
bonded to three other carbon atoms by single bonds. (ii) Due to its brilliance, diamond is used in
The fourth electron on each carbon atom results in jewellery as a precious stone. When it is cut at
the formation of z bonds (not shown in figure). In this certain angles and polished, brilliant light is
way, graphite consists of hexagonal rings in two refracted from its surfaces, which makes it a
dimensions [Fig. 10 (6)|. The C—C covalent distance precious gem.
in rings is 142 pm indicating strong bonding. These (iii) It is used for grinding and polishing hard
arrays of ring form layers. The layers are separated materials.
by a distance of 340 pm. The large distance between (iv) Diamond is also used for making dies for
these layers indicates that only weak van der Waals drawing thin wires from metals.
forces hold these layers together. The weak van der (v) It is used in the manufacture of tungsten
Waals’ forces which hold these layers together are filaments for electric light bulbs.
responsible for soft nature of graphite.
Uses of Graphite
Because of the weak van der Waals forces between
(1) Graphite is used in making electrodes.
layers, one layer can move over the other layer and
(it) It is used as a lubricant for heavy machines
this accounts for the slippery nature of graphite.
running at high temperature, where oil cannot
Therefore, graphite is used as /wbricant. The electrons
be used as a lubricant.
forming 7m bonds in the rings of graphite are delocalised
(zit) Itis used in the manufacture of crucibles which
and are relatively free to move under the influence of can withstand high temperatures.
electric field. Therefore, graphite is a good conductor (tv) Mixed with wax or clay, graphite is used for
of electricity. making lead pencils.
Comparison of Properties of Diamond and (v) Graphite is also used as a moderator for fast
Graphite on the Basis of Structural Differences moving neutrons in atomic reactors.
On the basis of structural differences, the Amorphous Forms of Carbon
important properties of diamond and graphite can be
Some important amorphous forms of carbon are:
easily compared.
(1) Coke. It is greyish black hard solid which is
(i) Density. In the case of graphite the different
obtained by destructive distillation (strong heating in
layers are held together by relatively weak van der
the absence of air) of coal. It is commonly used as a
Waals’ forces and the layers are about 340 pm distance
fuel in boilers, engines and it is also used in furnaces
apart. This distance of separation between layers is
as a reducing agent in metallurgy.
more than twice the carbon-carbon distance (154 pm) (1) Charcoal. It is a black, soft and highly porous
in diamond. Because of wide separation in structure substance. It can be obtained in three varieties :
of graphite, its density is lower than that of diamond. (a) Wood charcoal. It is obtained by heating wood
(it) Hard and soft nature. Diamond has three in a limited supply of air. It is highly porous
dimensional network of strong covalent bonds. It is and is therefore used as an adsorbent for gases
very difficult to break extended covalent bonding and and in making gas masks.
therefore, diamond tis the hardest substance on earth. (6) Animal charcoal. It is obtained by destructive
On the other hand, graphite has layer structure and distillation of bones and is also known as bone
the layers are held together by weak van der Waals’ black. It contains about 10% carbon. It is mainly
forces. Therefore, graphite cleaves easily between the used for decolourising sugar syrup in the
layers and this explains its flaky appearance, softness manufacture of sugar and other organic
and lubricating properties. substances.
Sie (c) Sugar charcoal. It is the purest form of
MODERN'S abc + OF CHEMISTRY-XI

amorphous carbon and is obtained by the action


of concentrated sulphuric acid on cane sugar.

CioH 9014
Conc. H,S50,
12C + 11H,0
Cane sugar Sugar charcoal
It is used as black pigment.
(uit) Lamp black. It is a fine black powder and is Buckminster fullerene Ball and stick model

obtained by burning oils (such as kerosene oil, Fig. 11. Structure of fullerene.
turpentine oil, petroleum, natural gas, etc.) in a limited
supply of air. It is mainly used for making printers identical positions. C,, fullerene molecule has 60
ink, black paints and shoe polish. It is also used as a vertices and each one is occupied by one carbon atom.
filler in making rubber tyres. It has a geometry of truncated icosahedron having 20
hexagons (six membered rings) and 12 pentagons
Fullerenes
(five membered rings). Six membered rings are fused
Upto 1985, carbon was considered to have only
both to other six membered rings and five membered
two allotropes graphite and diamond. In 1985,
rings but five membered rings are connected to only
scientists discovered a new family of carbon allotropes
six membered rings. In this case, all the carbon atoms
consisting of cluster of carbon atoms such as Co, Cz,
Coo; Cz Cay, etc. These were called fullerenes. These are equivalent and they undergo sp? hybridisation.
were discovered by H.W. Kroto, R.F. Curl and R.E. Each carbon atom forms three co bonds with other three
Smalley. This exciting discovery has opened a new era carbon atoms. The remaining electron at each carbon
in chemistry. The 1996 Noble Prize in chemistry was is delocalised in molecular orbitals, which give
awarded to the above scientists for their discovery of aromatic character to the molecule. The molecule has
fullerenes, the new form of carbon. Among these the single and double bonds with C—C distance of 145.3
allotrope having the molecular formula C,, is and 138.3 pm respectively.
important. It is named Buckminster fullerene. It was C,, fullerene has 12 pentagonal and 25
named after the name of American architect Robert
hexagonal faces. There are five types of carbon atoms
Buckminster Fuller, who designed ‘geodesic domes’
and thus eight distinct types of C—C bonds which vary
that exactly resembled with it.
from 139 to 154 pm.
Fullerenes are made by heating graphite in an
For example, thin films of C,, have mustard colour
electric arc in the presence of an inert atmosphere of
helium or argon. The sooty material formed by the (dark brown in bulk) and solutions in aromatic
condensation of vapourised C, small molecules consists hydrocarbons have beautiful magenta colour.
of mainly C,, with small amount of C,, and traces of Fullerenes differ from graphite and diamond in their
other fullerenes consisting of even number of C atoms solubilities. Being network solids, both graphite and
upto 350 or more. C,, and C,,. can be readily obtained diamond are insoluble in organic solvents. However,
by extraction with aromatic hydrocarbon solvents such fullerenes dissolve in organic solvents to give coloured
as benzene, toluene and the individual members are solutions. For example, a solution of C,, in toluene is
separated by chromatographic separation over purple whereas that of C,, is orange red.
alumina. The purified fullerenes have very attractive
Fullerene differs from diamond and graphite
colours.
which form lattices whereas it forms discrete
The process may be represented as : molecules. Being network solids both graphite and
. Electric arc diamond are insoluble in liquid solvents. However,
Graphite —7yLorar
fullerene which is molecular can be dissolved in
Vapourised carbon Condensation
suitable organic solvents (such as benzene).
Chromatographic It is interesting to note that graphite is
Fullerene soot
separation over alumina thermodynamically most stable allotrope of carbon
Ceg + Cr
ee |
and, therefore, A-H® of graphite is taken as zero. The
Fullerenes standard enthalpy of formation (A,H°) of diamond and
The structural determination of C,, molecule has fullerene are 1.90 and 38.1 kJ mol respectively.
shown that it is a perfect sphere and looks like a Thus, thermodynamically graphite is more
soccer—ball. Therefore, it is also popularly known as stable than diamond because its enthalpy of formation
bucky-ball (Fig. 11). It consists of five and six is +1.90 kJ mol“! lower than diamond at room
membered carbon rings. It has highly symmetrical temperature and atmospheric pressure.
structure in which all the carbon atoms occupy
SOME p-BLOCK ELEMENTS
USES OF CARBON AND OTHER ELEMENTS and industrial electrolysis. Graphite fibres embedded
Uses of Carbon in plastic material form high strength, light weight
composites. The composites are used in products such
Carbon is used extensively in its different forms.
as tennis rackets, fishing rods, aircrafts and canoes. It
Coal is used as a fuel in boilers, engines, furnaces, etc.
is also used in steel making, metal foundries for
It is also used for the manufacture of coal gas, water
crucibles, as a lubricant and in pencils etc. Crucibles
gas, producer gas and synthetic petrol. Itis largely used
made from graphite are inert to dilute acids and
as areducing agent in metallurgy. Carbon black is used
alkalies. It is also used as the moderator in the cores of
as black pigment in black ink and as filler in automobile
gas cooled nuclear reactors to slow down neutrons.
tyres. Charcoal (activated) being porous is used as an
Diamonds (allotropes of carbon) are cut as gemstones
excellent adsorbent to purify and deodorize sugar and
and used in jewellery and other articles. It is measured
other chemicals. It is also used to adsorb poisonous
in carats (1carat = 200 mg). Itis also used for industrial
gases in gas masks and for removing offensive odour
purposes mainly for making drills or as an abrasive
from the air used in air conditioning processes. It is
powder for cutting and polishing.
also used in water filters to remove organic
contaminators. Graphite being a good conductor of The important uses of different allotropic forms
electricity is used for making electrodes in batteries of carbon are given in Table 6.

Table 6. Uses of various allotropic forms of carbon.


Form of Carbon Uses
Diamond e due to its hardness, used for cutting glass, making bores for rock drilling, making
abrasives
in jewellery as a precious stone, gem stone
for grinding and polishing
for making dyes for drawing thin wires from metals
in industry
Graphite steel manufacture (reducing agent) refractories
electrodes in electrolytic extraction of elements
lubricant for heavy machines
high temperature crucibles and high strength composite materials
moderator in nuclear reactors
for lead pencils
Coke fuel in boilers and engines
in furnaces as a reducing agent in metallurgy (steel manufacture)
Carbon black rubber industry as a filler
pigment in ink, paints and plastics
Activated charcoal decolourizing agent in sugar industry
purification of chemicals and gases by adsorption
as catalyst
Charcoal fuel
reducing agent
Uses of Silicon conductor devices. It is transparent to infra red light
silicon is used as n-type or p-type semiconductors and therefore is also used for making prisms and lenses
when doped with Group15 or Group13 element and windows in infra red spectrometers and other
respectively. Silicon and germanium are extensively scientific apparatus.
used in very pure forms in semiconductor devices, Uses of Tin
which are the basis of the whole electronic industry Because of low strength and high cost of tin, it is
including computer hardware. Silicon is very rarely used by itself but it is used for electroplating
important component of ceramics, glass and cement. and as alloys. Tin plates obtained by electroplating
Silicon is added to steel or iron as such or more usually steel with tin are extensively used for making cans,
in the form of ferrosilicon to increase its resistance to for food and drinks. It is used in the preparation of a
attack by acids. Very pure silicon is used to make number of important alloys such as solder (Sn/Pb),
computer chips. Its alloys such as silicon bronze and bronze (Pb/Cu/Sn), babbitt (Sn/Pb or Cu/Pb/Sn/Sb),
manganese-silicon bronze possess strength and pewter (Pb/Sn/Sb/Cu), type metal (Pb/Sn/Sb), etc.
hardness even greater than steel. Uses of Lead
Uses of Germanium Lead is used for making lead sheets, lead pipes, etc.
Germanium has largest use in transistor It is also used for making telegraph and telephone
technology, in making transistors and other sem1- wires which are to be burried in earth. It is used in
Tio
storage batteries, making bullets. It is commonly used
MODERN'S abc + OF CHEMISTRY-XI

Carbon monoxide is also present in volcanic


in making pigments like chrome yellow, chrome red, gases, gases from the furnaces where coal is burnt,
red lead, white lead, etc. Lead is also used for making exhaust gases from automobile engines, fumes of
important alloys such as type metal, solder, pewter, etc. tobacco, ete.
SOME IMPORTANT COMPOUNDS OF Structure of carbon monoxide
CARBON AND SILICON The electron dot structure of carbon monoxide may
Carbon combines with a variety of other be represented as :
elements to form binary compounds such as oxides, :C::0: or C=O
halides and carbides. However, it is regarded as resonance hybrid of
(1) Oxides of Carbon the following structures :
Carbon burns in air or oxygen to form two
oxides namely (a) carbon monoxide (CQO) and :C::0: <_>:020: —>:€:0:
(6) carbon dioxide (CQ,). The presence of a triple bond between C and O is
(a) Carbon monoxide (CQ) supported by the fact that the carbon—oxygen bond
Preparation length in CO is 113 pm which corresponds to carbon
(1) Carbon monoxide is prepared by incomplete oxygen triple bond.
combustion of carbon or carbon containing compounds Properties
in a limited supply of oxygen. 1. Carbon monoxide is colourless, odourless and
is almost insoluble in water.
C(s) + 50,(g) Hest, COW) 2. Toxic nature. It is highly toxic in nature. The
toxic nature of CO is due to its ability to form a complex
or 2C(s) + O,(g) —Het, 200(g) with haemoglobin in the red blood cells. Haemoglobin
Carbon monoxide is also present in exhaust gases in the blood combines with oxygen and forms
from automobiles due to incomplete combustion of oxyhaemoglobin by a reversible reaction :
carbon or carbon compounds during burning of petrol Haemoglobin + O 9 ——= Oxyhaemoglobin
or diesel.
Oxyhaemoglobin is formed in the lungs and is
(11) Carbon monoxide is prepared by the reduction
carried to the different cells where it gives its oxygen.
of oxides of heavy metals with carbon. However, CO forms complex with haemoglobin more
ZnO +C —> f”n+CO strongly than oxygen. The CO complex with
haemoglobin is about 300 times more stable than the
Fe,0,+3C ——>» 2Fe+3CO oxygen-haemoglobin complex. Therefore, if CO is
(iit) Pure carbon monoxide can be prepared on present, it will form stable compound with
a laboratory scale by heating formic acid (HCOOH) haemoglobin and this will destroy the oxygen carrying
with sulphuric acid at about 373 to 413 K. Sulphuric ability of the blood. As a result, blood will not take up
acid acts as a dehydrating agent. oxygen easily, causing suffocation and finally death.
H2SO4 373-473K 3. Combustibility. Carbon monoxide burns in air
HCOOH CO with a pale blue flame forming carbon dioxide.
—H9O

(iv) On commercial scale, carbon monoxide is CO + . O, —— CO, + Heat


prepared by passing steam over red hot coke. This
4, Reducing character. It is a powerful reducing
reaction produces hydrogen also.
agent and reduces many metal oxides to metals. For
C(s) + H,O) “2%, CO(g)+H,(g) example,
Water gas
The mixture of CO and H, is called water gas or
ZnO +CO —H28t , Zn + CO,
synthesis gas. When air is passed instead of steam, Cu0+CO —H4+ Cu+CoO,
a mixture of CO and N, are formed. This mixture is
called producer gas. Fe,0,+3C0O —t8!, 2Fe + 3C0,
Carbon monoxide reduces almost all metal oxides
2C(s) + On(g)+4No(g) 284% , 2C0(g)+4N,(g) other than those of the alkali and alkaline earth
er Producer gas metals, aluminium and a few transition metals. This
The water gas and producer gas are very property of CO is used in the extraction of many metals
important industrial fuels. Carbon monoxide present from their oxides.
in water gas or producer gas can undergo further 5. Formation of metal carbonyls. In carbon
combustion forming carbon dioxide with the evolution monoxide, there are one sigma and two 7 bonds
of heat. between carbon and oxygen, :C=O-. Because of the
2CO(g) + O,(g) ——~ 2CO,(g) ; presence of a lone pair on carbon, CO molecule acts
A H° = — 566 kJ mol as a donor and reacts with certain metals when heated
SOME p-BLOCK ELEMENTS

to form metal carbonyls. It combines with many (it) In the laboratory, it is prepared by the action
transition metals such as nickel, iron, cobalt, etc. of dilute hydrochloric acid on carbonates such as
forming variety of compounds known as metal CaCO,.
carbonyls. For example, CaCO, + 2HCl ——_> CaCl, + CO, + H,O
Ni+4CO—*"" , Ni(CO), (zit) On an industrial seale, it is obtained as a
Nickel carbonyl by-product in some industrial processes such as
Fe+5CO —— Fe(COQ), manufacture of lime, manufacture of alcohol, etc.
Iron carbonyl
6. Reaction with chlorine. Carbon monoxide
CaCO, —H@* +620 + CO,
Limestone 1600 Lime
readily combines with chlorine in sunlight to give
carbonyl chloride, phosgene. Phosgene is a poisonous C,H,,0, —®™>2C,H,OH + 2CO,
cas. Glucose Ethyl alcohol
CO (g)+ Cl, (g) ——— COC, (g) Structure of CO, molecule
Phosgene
In CO, molecule, carbon undergoes sp
Uses of Carbon monoxide
hybridisation. Two sp hybridised orbitals of carbon atom
(1) Carbon monoxide is used as an industrial fuel in
overlap with two p-orbitals of oxygen atoms to form two
the form of water gas and producer gas :
sigma bonds while other two electrons of carbon atom
(a) Water gas is a mixture of carbon monoxide
are involved in pxr—pr bonding with oxygen atoms. This
and hydrogen. It is obtained by passing steam over
results in linear shape of carbon dioxide molecule. The
red hot coke.
C—O bonds in CO, molecule are of equal length (115
C+H,O(steam) ___, CO+H» pm) and the molecule has zero dipole moment. This is
Water gas explained on the basis of resonance structures as :
Water gas is also known as synthesis gas or 70 =U O:<>:0—C —O3<>:0 = C — Oz
syn gas.
Due to resonance, the C—O bond length is between
(6) Producer gas is a mixture of carbon monoxide double bond (122 pm) and triple bond (110 pm).
and nitrogen. It is prepared by passing air over a bed
Properties
of red hot coke at about 1273 K.
Carbon dioxide has different physical characteri-
2C +0, 2 +4N, 2 00k , 200+4N,
| | stics than carbon monoxide. Some important physical
Producer gas properties of CO and CQ, are given in Table 7.
Both water gas and producer gas contain CO as
Table 7. Some physical properties of CO and CQ,.
their important component which can undergo further
oxidation to form carbon dioxide and liberates
enormous amount of heat. Therefore, these are used Melting point (K) 216.4 (at 5.2atm)
as important industrial fuels. Boiling point (K) h 194.5 (sublimes)
(ii) It is used in some metallurgical processes. For Density (g cm~ at 273 K) 1.977
example, in the metallurgy of iron, it is used as a
C—O bond length (pm) 115
reducing agent.
Heat of formation (kJ mol!) }110.5 | -—393.5
Fe,O, + 83CO faa» 2Fe + 3CQ,
It is used in Mond’s process for the purification of 1. It is a colourless, odourless gas about 1.5 times
nickel by forming nickel carbonyl. heavier than air with density of 1.977 g L“ at 298 K.
(iit) It is used in the manufacture of methyl
2. Unlike CO, it is not poisonous but it does not
alcohol, formic acid, etc. support life in animals and human beings. Animals
(tv) Its compound with iron (iron carbonyl) is used and human beings die in it for lack of oxygen.
in the manufacture of magnetic tapes for tape
recorders. 3. Solubility. Carbon dioxide is slightly soluble
(6) Carbon dioxide (CQ,) in water. Its solubility in water, however, increases
with increase in pressure. Soda water and other
Preparation
aerated soft drinks are solutions of carbon dioxide in
(,) Carbon dioxide is prepared by burning carbon,
water (containing sugar, some flavouring and colouring
hydrocarbons or carbon monoxide in excess of air.
substances) under pressure.
C+O, _Heat_. CO,
4, Carbon dioxide can be readily liquefied under a
CH,+20, _
Heat , CO, + 2H,O pressure of 50-60 atm. at room temperature. When
C.H,.+ 80, —Heat_,. 5CO,+6H,O liquid carbon dioxide is allowed to evaporate rapidly,
it changes to a solid called solid carbon dioxide.
2CO +0, _Heat_, 2CO,
Sie MODERN'S abc + OF CHEMISTRY-XI

absorbed by plants in the presence of chlorophyll (green


The solid carbon dioxide is also called dry ice
because it looks like ice. However, unlike ordinary ice, colouring matter in the leaves) and sunlight to form
it does not wet a piece of cloth because it sublimes glucose, starch and cellulose (which are known as
(changes to vapours without melting). It is a soft, snow carbohydrates).
like substance. It is used as refrigerant for ice-cream
and frozen food. Mixed with ether or acetone, it forms
6CO, + 12H,O Gaoeens” CgH,.0, + 60, + 6H,O
freezing mixture with very low temperature and is 6nCO, + 5nH,O ——-+> (C,H,,0,), +6nO,
used as a coolant for preserving perishable foodstuffs. Starch or cellulose
A mixture of solid CO, and ether can generate a By the process of photosynthesis, the plants prepare
low temperature of about —108°C, which is used in food for themselves as well as for animals and human
the liquefication of other gases. beings. But the increased use of fossil fuels for
5. Non-combustible nature. Ordinary carbon combustion and decomposition of limestone for cement
dioxide is neither combustible nor supporter of manufacture in recent years has considerably increased
combustion. Therefore, it is used in fire extinguishing. the CO, content in the atmosphere. This may lead to
However, certain active metals such as Na, K, Mg, etc. increase in greenhouse effect and thus raise the
continue burning in it. temperature of atmosphere which might result in
2Mg+CO, ——> 2MgO + C serious consequences.
6. Acidic nature. Carbon dioxide turns blue 9. Action of ammonia. When CO, is reacted with
litmus red and is therefore, acidic in nature. It liquid ammonia at 453-473 K under a pressure of 220
dissolves in water forming carbonic acid (H,CO,). atmospheres, it first forms ammonium carbamate
which decomposes to give urea.
CO, + H,O H,CO,
Carbonic acid 2NH, #CO, -“=—<=*
+ NH,COONH, 2 >
Therefore, carbon dioxide is regarded as an Ammonium carbamate
anhydride of carbonic acid. NH,CONH, + H,O
Carbonic acid is a weak dibasic acid and forms Urea
two series of salts namely bicarbonates (hydrogen Uses of carbon dioxide. Carbon dioxide is used :
carbonates) containing the HCO, anion, and the (t) in the preparation of aerated waters like soda
carbonates, containing CO,* anion. water.
H,CO,(aq) ———> H*(aq) + HCO, (aq) (it) as a fire extinguisher because it is a non-
HCO,(aqg) ———> H*(aq) + CO,” (aq) supporter of combustion.
Thus, a solution of carbon dioxide in water is an (iii) in the manufacture of washing soda by
equilibrium mixture of CO,, H,CO,, HCO, and CO,”-. Solvay’s process.
It may be noted that these equilibria play an important (tv) as a refrigerant in the form of solid carbon
role in carbonic acid-hydrogen carbonate ion buffer dioxide (dry ice). Solid carbon dioxide can
system in maintaining the pH of human blood between sublime to the gaseous state without passing
7.26 to 7.42, Furthermore, the rate at which CO, comes through the liquid state and therefore, it is
in equilibrium with H,CO, is slow and has jeniheauit called dry ice. It is also used as coolant for
role in the physiology of Co,. preserving perishable food items such as meat.
7. Action with lime water. On passing carbon Dry ice is also used for curing local burns and in
dioxide gas through lime water, milkiness is produced hospitals for surgical operations of sores.
due to the formation of calcium carbonate. (v carbon
‘—” dioxide is also used in the
Ca(OH), + CO, » CaCO, + H,O manufacture of urea by reaction with
Lime water Milky ammonia.

However, on passing excess of CO, gas through CO, + 2NH, — NH,COONH, et, NH,CONH,
the milky solution, the milkiness disappears due to Ammonium Urea
the formation of calcium bicarbonate which is soluble. carbamate
CaCO, + H,O + CO, » Ca(HCO,), (vt) for artificial respiration (for the victims of
Milkiness disappears carbon monoxide poisoning) as a mixture of
95% oxygen and 5% carbon dioxide (called
8. Photosynthesis. Carbon dioxide, which is
carbogen).
normally present to the extent of about 0.03% by
volume in the atomosphere, is removed from it by the (vit) for the purification of cane sugar juice in the
process known as photosynthesis. It is the process manufacture of sugar.
by which the green plants convert atmospheric CO, (vill) in plants during photosynthesis of glucose,
into carbohydrates such as glucose. Carbon dioxide is starch and cellulose.
SOME p-BLOCK ELEMENTS

Explanation. Carbon atom has a small size and


LEARNING PLUS forms stable multiple covalent bonds because of close
We know that a gaseous substance can be interaction between the 2p—orbitals on carbon and
liquefied by applying pressure only if it is below its 2p—orbitals on oxygen. This is because the energy of
critical temperature. For CO,, the critical pressure is 2p—orbitals of carbon and 2p-—orbitals of oxygen are
73 x 1.013 x 10° Pa and critical temperature is 304 K. nearly the same and they can easily overlap. However,
A substance above its critical pressure is called a silicon has large size and the 3p—orbitals of Si do not
supercritical fluid.Though it is a gas, it is very dense overlap effectively with 2p—orbitals of oxygen.
and, therefore, can behave as a solvent. Therefore, Si=O bonds are not formed. The
In recent years carbon dioxide above its critical tetravalency of silicon is satisfied by the formation of
pressure known as super-critical CO, is used as a O — Si-O bonds which result in a network structure.
solvent to extract organic compounds from their Silica in its normal forms is almost non-reactive
natural sources (e.g., caffeine from coffee beans, because of very strong Si—O bond (bond enthalpy = 368
perfumes from plants or flowers) or to carry out kJ mol). Therefore, it has very high melting point and
industrial important reactions. resists the attack by halogens, dihydrogen and most of
the acids and metals even at elevated temperatures.
(2) Silicon dioxide
However, silica is attacked by F,, HF and NaOH.
Silicon dioxide is also called silica. Most of the SiO, + 2F, ———> Sif, + O,
earth’s crust (95%) is made up of silica and silicates. Silicon tetrafluoride
Silica occurs in several crystallographic forms such as
quartz, cristobalite and tridymite. Each of these
SiO, + 4HF ——+> SiF, + 2H,O
has different structures at high and low temperatures Silicon tetrafluoride

and they are inter convertible at suitable temperature.


SiO, + 2NaOH —“*> Na, SiO, + 2H,O
Among these a-quartz is the most common form and
Sodium silicate
the major constituent of many rocks such as granite
Uses of Silica
and sand stone. Pure SiO, is colourless but traces of
other metals may colour it giving semi-precious gem The different forms of silica have many uses. The
stones such as amethyst (violet), citrine (yellow), most important application of quartz is as a
rose quartz (pink), smoky quartz (brown) and non- piezoelectric material and is used in crystal oscillators
precious materials such as flint (often black due to for radios and computers, filters for frequency control
C), agate and onyx. and in electro-metallurgical devices such as
transducers and pick ups. The silica has made
Silicon dioxide is a covalent, three dimensional
possible to develop extremely accurate clocks, modern
network solid. In this structure (Fig. 12), each silicon
radio and television broadcasting and mobile radio
atom is tetrahedrally surrounded by four oxygen atoms.
communications.The coloured varieties of quartz are
Each oxygen atom in turn is covalently bonded to used as gemstones e.g., amethyst, jespar, opal, etc.
another silicon atoms as shown in the diagram. Fach
Vitreous silica has a low coefficient of expansion,
corner is shared with another tetrahedron. The entire
quite resistant to shock and is quite transparent to
crystal may be considered as giant molecule in which visible and ultraviolet light. Therefore, it is used for
eight membered rings are formed with alternate silicon laboratory glassware and for optical components such
and oxygen atoms. Thus, there are no discrete SiO, as lenses and prisms and cells for samples in UV-
units and it is a network solid. visible spectrophotometers.
GeO,, SnO, and PbO, are all network solids. Silica gel is amorphous and very porous. It is
widely used as a drying agent, a catalyst and in
chromatography. The mineral opal, which is used as
a white or pearl like gemstone is hard (amorphous)
silica gel.
Kieselguhr is another form of SiO, and is a fine
powder. It is used in filtration plants as an abrasive
and an inert filter.
Silica is also used in making refractory bricks for
furnace lining sand (finely ground) and is used in the
extraction of phosphorus from phosphates.
— Si— O-Si-— O-Si —O-Si-
ee ee SILICONES
Silicones are organosilicon polymeric compounds
Fig. 12 Structure of SiO, containing Si—O—Si linkages. These have the general
formula (R,Si0), having (R,SiO—-as a repeating unit.
Tis MODERN'S abc + OF CHEMISTRY-XI

These may be linear, cyclic or cross linked. These


have very high thermal stability and are called
CH, CH;
| | Polymerisation
high temperature polymers (R may be alkyl or nHO—Si—OH + HO—Si—CH, ~ =o
phenyl group). The starting material for the | es
manufacture of silicones is alkyl or aryl substituted CH,
chlorosilanes.
For example, when methyl chloride reacts with
silicon in the presence of copper as catalyst at a Si—0 siCHs
temperature 573 K, various types of methyl
substituted chlorosilanes of the formula CH,SiCl, (or bn,
MesiCl,), (CH,),SiCl, (or Me,SiCl,), (CH, ), Sic] (or Silicone
Me SiCl) alongwith small amount of (CH,\, Si (or
The hydrolysis of monotrichloro silanes, RSiCl,
Me.,>1) are formed. Hydrolysis of dimethyl
gives cross linked polymers. By regulating the
dichlorosilane, (CH,),SiCl, followed by condensation
conditions, the condensation can be stopped at any
polymerisation yields straight chain polymers.
stage and the chains or rings of desired lengths can be
2CH,Cl + Si —“3pe*> ——-(CH), SiC, obtained.
Dimethyl dichlorosilane RCl + Si #2HCM= =~,
Cu, 573K
R SiCl, + H,
CH, CH, R
|
Cl—si—cl +2H,O 2°,
:
yo—si—on
| RSiCl, + 3H,0 —3#C_, O_o
| |
CH; CHs on
R
When two molecules of dimethylsilinol combine, we
nHO—Si_OH Pefseucrisition

‘i fF
eet dimer with the elimination of a molecule of water.

CH, CH,

b 6
ca O—Si—O—Si—O—Si—R
| | :
HO—Si—[OH
+ Hlo—si—oH ——2%>
| |
CH, CH,
a 0—Si—0—si—_o— SiR
CH, CH, R b i
| |
HO—Si—O—Si—OH Cross linked silicone
The commercial silicone polymers are usually
| |
CH, CH, methyl derivatives and to lesser extent phenyl
derivatives.
Dimer
Properties of silicone polymers
Since an active —OH group is left on each end of
1. Silicone polymers are highly stable towards heat.
the chain, polymerisation reaction continues and the
They have generally high thermal stability.
length of the chain increases forming linear silicone as
2. They have high dielectric strength.
3. Low molecular weight silicon polymers are
| | | soluble in organic solvents like ether, carbon
HO—Si—O—Si—OH + HO—Si—OH +-.. tetrachloride, benzene, etc.
| | | 4, They are stable towards chemical reagents. They
CH, CH, CH;
are not affected by weak acids, alkalies and salt
solutions and are resistant to oxidation.
CH, CH, 5. They are water repellants because they are
—- | | surrounded by non-polar alkyl groups..
Polymerisation —_ - —_—_
-H0 O i O if 6. They are good electrical insulators.
7. They are resistant to oxidation. However, when
CH, ).CH, heated in air to 350 to 400°C, silicones are rapidly
Silicone oxidized and this leads to cross linking.
The chain length of the polymer can be controlled Uses of silicone polymers. The important uses
by adding (CH,),SiCl which blocks the ends as shown of silicone polymers are :
in the reaction : (z) Silicone polymers are used for high temperature
oil baths, high vacuum pumps etc.
SOME p-BLOCK ELEMENTS

(ii) Silicone polymers are used as sealant, greases, (v) Silicone rubbers are very useful because they
varnishes and these can be used even at very low retain their elasticity at lower temperatures as
temperatures (of the order of —40°C), compared to other rubbers. They are also mixed in
(111) Because silicones are water repellants and paints to make them damp resistant.
eood insulators, they are used for water proofing of (vl) They are used as excellent insulators for
fabrics and in electrical condensers. electric motors and other electrical appliances.
(1v) These are used as lubricants at both high and (vit) Being biocompatible, they are also used in
low temperatures. surgical and cosmetic plants.

GLASS
Glass is an amorphous and transparent or translucent solid obtained by solidification of a mixture of
silicates of different metals, one of which is always an alkyl metal. Ordinary glass is a mixture of sodium and
calcium silicates and is obtained by heating a mixture of sand (SiO,) with sodium carbonate and calcium oxide
in a furnace at around 1700 K and then rapidly cooling.
CaO + Na,CO, + SiO, >
1700 K
Na,Si0,.CaSi0,.4Si0,
NEN OT a + CO,
Glass
This type of glass is called soda lime glass or soft glass. It has an approximate composition :
Na,SiO, .CaSiO, ASIO, or Na,O. CaO. 6Si0,.
e Addition of seal SGUAts of transition er salts to the glass mixture impart colour to glasses. For example,
addition of Cri(III) compounds imparts green colour, addition of Mn(IV) compounds imparts violet colour,
addition of Co(II) salts imparts blue colour, addition of Fe(III) salts imparts brown colour and so on.
Some other varieties of glass are:
e Lead potash glass or flint glass is obtained by addition of lead oxide and potassium carbonate which
has a high refractive index. It is used for making cut glass objects and lenses for optical purposes.
e Borosilicate glass is obtained by addition of boric oxide or borax to replace part of SiO,. It is heat
resistant glass with a low coefficient of thermal expansion. This type of glass is called pyrex glass,
corning or borosil (trade names), and can withstand sudden changes in temperature and is used for
making laboratory glassware.
e Crook's glass contains cesium oxide (CeO,) and has the properties to cut off ultraviolet rays which are
harmful to eyes. It is used in making lenses.
It may be noted that glass is not a true solid and has no definite melting point. It softens on heating to a
certain temperature. It is vitreous amorphous material. Although glass is made up of SiO,* tetrahedral
units, these are not arranged in any definite or regular pattern as in crystalline silicates or in quartz. Thus,
glass behaves more or less like a fluid. Moreover, since glass is obtained when molten silicates are allowed to
cool rapidly, therefore, glass is regarded as a super cooled liquid.

SILICATES silicon atoms at the centre. This can also be predicated


The silicates are the compounds of silicon and from hybridisation. In this ion, silicon involves sp?
oxygen. A large number of silicate minerals exist in hybridisation. The outer electronic configuration of
nature. Some of these important minerals silicon atom in ground state is 3s23p*. To account for
: feldspars e.g., albite, NaAlSi,O,, zeolites tetravalency, one of the electron from 3s-orbital is
oe. chabazite, Ca,[(Al0,),(Si0,)¢. H ,O], micas promoted to vacant 3p-orbital and its electronic
(muscovite), [KAI,(Si,A10,,)(OH),] cad asbestos
configuration in excited state becomes : 3s* 3p,’ 3p,"
[Mg,(Si,O, (OH), ]. Silicates occur in the earth’s crust
3p,". These orbitals get hybridised forming four sp®
in the form of numerous silicate minerals and alumino-
hybrid orbitals. They form four bonds with oxygen
silicate clays.
atoms giving SiO, unit. Each oxygen atom gets one
All silicates involve silicon-oxygen single bonds.
These are of two types. Terminal —Si—O bonds

® O00
©O00
involving oxygen bonded to silicon and no other atoms Si (Ground state)
and bridging —Si—O—$1 bonds involving oxygen bonded si (Excited state)
to two silicon atoms. The basic building unit of all
silicates is the tetrahedral S10 ll ion "sp?
sp? hybridisation
In Si0,* tetrahedral, the four oxygen ions are electron each form some metal in order to complete
arranged at the four corners of the tetrahedron with its octet resulting SiO am anion. Since, it involves sp?
Tie
hybridisation, it has tetrahedral structure, as given in 3. Cyclic silicates.
MODERN'S abc + OF CHEMISTRY-XI

When two oxygens of each


Fig. 13. si0,* tetrahedron are shared with others, cyclic or
ring structures are obtained as shown in Fig. 15. The
primary unit in these silicates is (10,7" - There are
two types of cyclic ions : [Si,0 9° as shown in Fig. 16 (a)
and cyclic ion [Si,O,,]’* as shown in Fig. 16(5).
[Si,0,|* ion occurs in wollastonite [Ca,Si,0,] and
cyclic [Si,O,,]’* ions occurs in beryl (Be,Al,Si,0,,, ).

Fig. 13. Tetrahedral structure of SiO,,* anion.

For simplicity, the SiO,* unit is represented as shown


in Fig. 14, in which the plane circles represent oxygen
atoms and small solid circle represents silicon atom.

® Silicon

® Silicon oO Oxygen

© oxygen Fig. 16. Structure of cyclic silicates


(a) [Si,O,|* unit and (b) [Si,O,,]'?- unit.
4. Chain silicates. These are also formed by
the sharing of two oxygen atoms of each SiO aa units.
Fig. 14. Representation of SiO,* units. There are two types of chain silicates : single chain
Types of silicates and double chain silicates. Single chain silicates
contain (SiO,).? chain and is shown in Fig. 17. The
In silicates, either the discrete unit is present or a common examples of single chains are pyroxenes such
number of such units are joined together by sharing as diopside [CaMg (SiO,),], enstatite (MgSi0,),
1, 2, 3 or 4 oxygen atoms per silicate unit. When these
spodumene [LiAl(SiO,),]. |
silicate units are linked together, they form chain,
ring, sheet or three dimensional structures. The
negative charge on silicate structure is neutralised by
positively charged metal ions. Depending upon the
linkages of SiO,* tetrahedral units, the silicates may
be classified into the following types :
1. Ortho silicates. These are simple silicates
which contain discrete Si0,* units as shown in
Fig. 13.
The common examples of ortho-silicates are : C) Oxygen e@ Silicon
Zircon (ZrSiO,), Wellimite (Zn,SiO,), Phenacite Fig. 17. Single chain silicates.
(Be, Si0,).
2. Pyrosilicates or islands. When two SiO,* The double chain silicates contain (Si a are
tetrahedral share a corner (common oxygen), island called amphiboles. The common examples of double
structures are obtained. These contain discrete chain silicates are :
[Si,O,]© units as shown in Fig. 15. Since this anion Tremolite, Ca,Mg,(51,0,,),(0H),, asbestos,
carries six negative charges, the number of cations is CaMg,O(Si,0,,).
such that they balance the negative charge. The 5. Sheet silicates. These are formed by sharing
common examples are : of three oxygen atoms of each SiO a unit. The sharing
Thortveitite (Sc,Si,O,), gives an infinite two dimensional sheets and contain
hemimorphite [Zn,(OH),Si,0.] (Si,0,)? type anions. Sheet silicates are shown in
Fig. 18. The common examples are kaolinite
[Al(OH),5i,0,], tale Mg.(OH).,81,0,,. Clay also belongs
to this type affamily containing (Si,0,)? anion.
6. Three-dimensional silicates. When all the
four oxygens of si0,* units are shared, a three-
dimensional network is obtained. Since all the oxygens
are shared the silicate is neutral. The common
Fig. 15. Structure of [Si,O,|* unit in island or pyrosilicates
examples are different forms of silica such as quartz,
SOME p-BLOCK ELEMENTS

ZEOLITES
Zeolites are microporous aluminosilicates having
the general formula M,y, [AlO,], [SiO,]. mH,O. In
zeolites, aluminium atoms replace few silicon atoms
in three dimensional network of silicon dioxide.
The overall structure known as alumino silicates
acquires a negative charge. Cations such as Na’, Kt or
Ca?+ balance the negatives charge.These have open
structure of aluminosilicate frameworks which enables
them to take up loosely bound water or other small
molecules in their structures. The negative charge of
the alumino silicate framework is neutralised by
exchangable cations of valence n. Zeolites have high
Fig. 18. Sheet silicates porosity due to the presence of one, two or three
dimensional network of interconnected channels and
tridymite and cristobalite. These have the general cavities of molecular dimensions. These give them
formula (SiO,).. honey comb like structure. For example, Fig. 19(a)
In some cases, Si** is replaced by Al** in shows a single cavity from 24 linked [SiO,] tetrahedra.
tetrahedral positions in Si0, lattice. In such It is called the b-cage or sodalite cage. Fig. 19(b) shows
cases, the presence of an additional monovalent the zeolite which is formed by linking sodalite cages
cation is necessary to maintain electrical through double four membered rings and Fig. 19(c) is
neutrality. For example, feldspar (KAISi,O,), zeolites formed by linking the sodalite cages through double
(NaAlSi,0,.H,0). six membered rings.

Fig. 19. Structure of zeolites

Uses of Zeolites Solution: CO? +H,O == HCO, +OH


(1) Zeolites are used as molecular sieves and can
separate molecules of different sizes. HCO, +H,O —— H,CO,+0H
(it) These are extensively used as catalysts in
petrochemical inderstries for working of hydrocarbons UY Example 17
and isomerization. For example. ZSM— 5 (a type of What ts the oxidation state of carbon in each of the
zeolite) is used to convert alcohols directly in to gasoline. following :
(iit) Hydrated zeolites are also used as ion- (t) CO, (tt) CO (ait) CaC, (tv) H,CO, (v) HCN.
exchange materials in softening of hard water.
Solution: (i) COQ, +4
SOLVED EXAMPLES (iz) CO +2
(iit) CaC, +1
UO) Example 16.
(iv) H,CO, +4
Carbonic acid ts a weak dibasic acid and solutions
of carbonates or hydrogen carbonates are alkaline. (v) HCN +2
Write balanced equations for the hydrolysis of HCO,”
and CO,7- ions
Tie
LO) Example 18. LU) Example 21
MODERN'S abc + OF CHEMISTRY-XI

What is the state of hybridization of carbon in Write balanced chemical equations for the following
(t) diamond and (i) graphite (til) carbonate ton reactions :
(t) Combustion of C,H, in a limited supply of oxygen
Solution: (i) Diamond : sp?
to form carbon monoxide and water.
(ii) Graphite : sp”
(iii) CO,7 ion : sp (it) The reaction of calcium carbide with water to
form acetylene.
UO) Example 19 (tit) The preparation of calcium cyanamide from
Select the members of group 14 which calcium carbide and nitrogen.
(t) forms the most acidic dioxide. (iv) Formation of hydrogen cyanide from methane
(ii) is commonly found in +2 oxidation state and ammonia.
(tit) used as semiconductor Solution:

Solution:
(i) C,H,
(2) + =0, (g) ——> 4CO(g) + 5H,O(/)
(zt) Carbon (iz) Lead (zz) Silicon and germanium

Li Example 20
(ii) CaC,(s) + 2H,O (1) ——> Ca(OH),(s) + C,H,(g)
Explain the following reactions : (iii) CaC,(s) + N, (g) ——> CaNCN(s) + C(s)
(t) Silicon is heated with methyl chloride at high (iv) CH,(g) + NH, cals! _»HCN(g) + 3H,(g)
Pt. Catalyst

temperature in the presence of copper.

(ti) Silicon dioxide is treated with hydrogen L) Example 22


fluoride. A tetravalent element forms monoxide and dioxide

(tit) CO ts heated with ZnO. with oxygen. When air is passed over heated element
(1273 K), producer gas is obtained. Monoxide of the
(tv) Hydrated alumina is treated with aqueous
element is a poweful reducing agent and reduces ferric
NaOd.
oxide to tron. Identify the element and write formulas of
Solution: (7) Dichlorodimethy] silicon is formed its monoxide and dioxide. Write chemical equations for
2CH,Cl + Si APM, (CH,),SiCl, the formation of producer gas and reduction of ferric
oxide with the monoxide.
(it) Silicon tetrafluoride is formed which dissolves in
(NCERT Exemplar Problem)
HF to form hydrofluoro silicic acid.
Solution: The tetravalent element is carbon
SiO, + 4HF ——— SiF, + 2H,O
which forms monoxide, CO and dioxide CO,,.
Sif, + 2HF ——~+ H,SiF,
On passing air over heated element at 1275 K,
Hydrofluoro
producer gas CO, N, is obtained
silicic acid

(zi) AnO is reduced to zinc. 2C (s) + O,(g)+4N,(g) —=*> 2C0(g)+4N,(g)


|
Fo

ZnO + CO ——-> Zn+Co, air Producer gas

(iv) Alumina dissolves to form sodium meta-aluminate. CO is a powerful reducing agent which reduces
ferric oxide to iron.
Al,0,.2H,O + 2NaOH(aq) 1", 2NaAlO, + 3H,O
Hydrated Sodium Fe,O,(s) + 3CO(g)
—4 > 2Fe(s) + 3CO,(g)
alumina meta-aluminate
LU) Example 23
or Al,O,.2H,O(W) + 2NaOQH(aq) + H,0 (/) Hest, Classify the following oxides as neutral, acidic,
2Na[Al(OH),l(aq) basic or amphoteric :
Sodium
tetrahydroxoaluminate (IIT)
CO, BO, SiO, CO, Al,O, PbO, TIO,
SOME p-BLOCK ELEMENTS
Solution: CO: neutral B,O, _ : acidic Q@ 15. Which can be used as a lubricant—graphite or
diamond ?
S10, : acidic CO, : acidic
Q 16. Which oxide of carbon is an anhydride of carbonic
Al,O, —: amphoteric PbO, — : amphoteric acid ?
TIO, _ : basic Q 17. Give the name of a compound used as a fire
extinguisher under the name pyrene.
O}) Example 24
Q@ 18. Complete the following reactions :
SiF,2- is known but SiCl,2- is not known.
Explain.
()“ HCOOH HoS0,
=
Solution: This is due to the following reasons : Gi) CO+H, - —Ae
200 atm, 675 K
(a) Silicon being small in size can accommodate
6 F atoms but cannot accommodate larger 6 Cl atoms. (iit) SiO, + C ———>

(5) Due to smaller size of F, steric repulsions will be (tv) CaCO,(aq) + CO, (excess) ————>
less in SiF,.
Q 19. Among group 14 elements name
(c ) Interaction of lone pair electrons of F with Si is
(x) the most electronegative element
stronger than that of Cl lone pairs.
(it) most metallic element
O) Example 25. (tii) the elements having highest tendency to form
ptm—pt bonds.
Although diamond is covalent, yet it has very high
melting point. Explain. @ 20. Predict the correct answer :
Solution: (1) more stable out of CH, and SiH,
Diamond has three dimensional network structure (iz) easily hydrolysed out of SiCL, and CCl,
involving strong carbon-carbon bonds. These bonds are (tit) shows stable +2 oxidation state out of Sn and Pb.
difficult to break and therefore, the melting point of diamond
is a very high. Q@ 21. What is the product of

(1) hydrolysis of SiCl,


L}) Example 26
(iz) reaction of Sik’, and F- ion
What are silicones ?
Q 22. What is the basic unit of all silicates ?
Solution:

Silicones are synthetic organosilicon compounds Answers to Practice Problems =


R
containing _ units held by Si—O—Si linkages. ©11. Carbon and phosphorus
© 12. 2CO and 4N,
Fi.
© 13. Calcium carbide, CaC,
The R groups may be alkyl or phenyl occupying the remaining
positions on each silicon. They are hydrophobic 1.e., water © 14. Silicon carbide, SiC
repellent in nature. © 15. Graphite
©16. CO,
©17. Carbon tetrachloride
©19. @)C @)Pb (wi)C.
© 20. (i) CH, (ii) SiCL, (zz) Pb
Q@ 11. Name two elements which exhibit allotropy.
© 21. (i) Silicic acid Si(OH),
Q@ 12. What is the composition of producer gas ?
Q 13. Which carbide is used for preparing acetylene ?
(i) [SiF,)?-
Q@ 14. Name the carbide called carborundum ? © 22. si0,>
a 11/40 MODERN’S abc + OF CHEMISTRY-XI

add Of Conceptu GU CJuestions 42

Q.1. [SiF,]* is known but [CF,]*is not formed. Explain.


Ans. Silicon has vacant d-orbitals and can form complex like (SiF.]?- because Si can extend its coordination number
beyond four. But [CF,]? is not formed because C has no vacant d-orbitals in its valence shell.
Q.2. (CH,),N is basic but (CF,),N is not basic. Explain.
Ans. This is due to different electronegativities of H and F bonded to nitrogen. In (CH,), N, the lone pair on N is
concentrated on N and so it can act as an electron pair donor (Lewis base). However, in (CF), N, the electron
density on N is decreased due to strongly electronegative F atom attached to C. Hence, it cannot act as a
Lewis base.
Q.3. Starting from SiCL,, prepare the following
in steps not exceeding the number given in parenthness
(give reactions only) :
(¢) Silicon (ii) Linear silicon containing methyl groups only (iii) Na,SiO, (LIT. 2001)
. (1) 8SiCl, + 4Al ——> 4AICl, + 385i
(molten)
(iz) 3SiCl, + 4Al ———————> 4 AlCl, + 3Si
; Cu powder .
Si + 2CH,Cl ——
——____—_——__}
(CH,)SiCl,
1

(CH,),SiCl, ——————>,
HOH f
HO—-S—OH
é
CH,

CH, CH, CH;


(CH,), Si(OH) 2 —_______»
penne “Sh .
se : :
ca, <a. A,
Linear silicone

(iii) SiCl, + 4H,0 —___-> SiO, + 4HCI


Ortho
silicic acid

H,sio, ———_——>
A
SiO,
'
+ 2H,O
SiO, + Na,cO, > Na, SiO, + H,0.
Q.4. SiCl, forms SiCl,?> while CCl, does not form CCl,?. Explain.
Ans. Carbon does not have d-orbitals in its valence shell and therefore, cannot extend its octet. Hence CCl,
cannot combine with Cl- ions to form [CCI,]*.
On the other hand, Si has vacant 3d-orbitals in its valence shell and can therefore, combine with Cl- ions
to forms [SiC], }*-.
SICL, + 2Cl —> SiCl,7-.
Q.5. What is the maximum covalency of silicon in its compounds ?
Ans. Silicon shows maximum covalency in its compounds equal to six. For example, Na,Sil’..
Q.6. Account for the following :
(t) PbX, is more stable than PbX, (X = Cl, Br)
(ti) PbCI, is less stable than SnCl, but PbCl, is more stable than SnCl.,.

WWW. JEEBOOKS.IN
SOME p-BLOCK ELEMENTS 11/41 fp
Ans. (t) Due to inert pair effect, lead shows an oxidation state of +2. Hence, PbX, is more stable than PbX,.
(iz) As we move down the group 14, the stability of +4 oxidation state decreases while that of +2
oxidation state increases due to inert pair effect. Therefore, lead shows stable +2 oxidation state
(PbCl,) while tin shows stable +4 oxidation state (SnCl,).
Q.7. What is the importance of ultra pure elemental silicon ? How is it obtained ?
Ans. Ultra pure elemental silicon is used as a semiconductor. It is prepared by the reduction of highly pure silicon
tetrachloride (SiCl,) or silicon chloroform (SiHCI,) with dihydrogen.

SiCL, + 2H,(g) —ss* , Si+4HCl


SiHCl, +H, ———— 51+ 3HCl
It can also be prepared by the pyrolysis of SiH,,

SiH, —., Si+2H,


Q.8. Compare the behaviour of BCl, and CCL, with water.
Ans. BCI, readily gets hydrolysed with water to give boric acid.
BCl, + 3H,0 t———> H,BO, + 6HCl
CCl, does not get hydrolysed.
Q.9. What is dry ice ? Why is it so called ?
Ans. Solid carbon dioxide is called dry ice. Solid carbon dioxide can sublime to the gaseous state without passing
through the liquid stage and, therefore, it is called dry ice.

@ Allotropes are the different forms of the same element having different physical properties but almost similar
chemical properties.
@ Silicones are synthetic organosilicon polymers containing repeated R,SiO units held by Si—O—Si bonds.
@ Silicates are the compounds in which discrete SiO,* tetrahedra units are present.
® Zeolites. These are microporous aluminosilicates having the general formula M,,,, [AlO,], [SiO]. mH,O.
== =

QUICK CHAPTER ROUND UP


GROUP 138 ELEMENTS On heating
® General electronic configuration: ns?np! |
B,Al, Ga, In, Tl Na,B,0,.10H,O —*-> Na,B,O, + 10H,O
@ 5B and Al show an oxidation state of +3 while rest of the a 2NaBO, +B,0,
elements show two oxidation state of +3 and +1. Na,b,O,; ——> Glassy Bead
@ On moving down the group, the stability of +3 oxidation
@ Boric acid (H,BO,) is a weak monobasic acid
state decreases while that of +1 oxidation state
(K =1x 10°). It behaves as a Lewis acid by accepting
increases.
® Boron halides act as Lewis acids and Lewis acid a pair of electrons from OH .
strength decreases as BI, > BBr, > BCI, > BF, ® H,BO, contains triangular BO3” ions in which boron
@® On moving down the group, the acidic character of ig ep? hybridised.
oxides and hydroxides changes gradually from acidic to
amphoteric and their basic character increases. Bop, 2 a SE ge Sey
e B,0, Al,Og, Ga,O, In,0, Tl 20 Boric acid

Pee aes oer ate ae See a @ Diborane (B,H,) is electron deficient compound and
© Borax is Na,B,0,.10H,0 or Na, [B,O,(OH),]. 8H,0. has two ne innit asics pair bonds.

Its aqueous solution is alkaline due to hydrolysis. e B.N.H, is called inorganic benzene or borozole.
MODERN'S abc + OF CHEMISTRY-XI

GROUP 14 ELEMENTS In diamond, carbon undergoes sp* hybridisation and in


® General electronic configuration: ns?np” graphite, carbon undergoes sp” hybridisation.
@® C, Si, Ge, Sn, Pb Family of carbon allotropes consisting of cluster of
® Carbon and silicon show an oxidation state of +4. All carbon atoms such as Cup, C5, Cep,Ca9, Cay, etc. are called
other elements of this group show oxidation states of fullerenes.
+2 and +4. C,, is called Buckminster fullerene and is popularly
@ The stability of +2 oxidation state increases on going known as bucky ball
down the group due to inert pair effect. Pb salts are Silicon dioxide (called silica) has, three dimensional
network solid in which each Si atom is bonded to four
more stable in +2 oxidation state than in +4 oxidation
O atoms which are arranged tetrahedrally around it
state.
and each O atom is attached to two Si atoms by covalent
@ Due to small size and high electronegativity, carbon
bonds.
forms p7-p7 bonding.
Orthosilicates Si0,*7
® Carbon shows maximum catenation tendency which
decreases as C >> Si > Ge = Sn > Ph. Silicates have Pyrosilicates 5i,0,°
® Due to the absence of d-orbitals, carbon cannot extend SiO} basic Cyclic (SiO,)*
its covalency beyond 4. structural units Chain silicates (8i1,0,,),>
@ CCl, cannot be hydrolysed while SiCl, can be easily Sheet silicates (Si,O,)*
hydrolysed because of absence of d-orbitals in carbon. Silicones have repeated R,SiO units held by Si-O-Si
bonds.

WNieadn ceo Solved


Textbook Exercises ||
Q. 1. Discuss the pattern of variation in the oxidation complete its octet. Therefore, it can readily accept a
states of (1) B to Tl and (iz) C to Pb. pair of electrons from Lewis bases such as F’,
Ans, (1) B shows only +3 oxidation state in it compounds (C,H.)O, ete and hence behave as a Lewis acid.
whereas others also show +1 oxidation state. BF, + F- ——> BF,
The +1 oxidation state becomes stable as Q. 4. Consider the compounds, BCl, and CC1,. How
Al < Ga < In < TI. So, boron and Al show only +3 will they behave with water ? Justify.
oxidation state, Ga and In can exhibit +3 and +1
B atom in BCL, has only six electrons in the valence
oxidation states but +1 oxidation state of Tl is
shell and therefore, is an electron deficient molecule.
more stable than +3. The stability of lower
It can readily accept a pair of electrons donated by
oxidation state on moving down the group is due
water and hence undergoes hydrolysis to form boric
to inert pair effect.
acid and HCl.
(it) The elements from C to Pb show +4 and +2
BCl, + 3H,O ——-+ H,BO, + 3HCl
oxidation states. C and Si show mainly +4
oxidation states. Ge and Sn show +4 and +2 In contrast, C atom in CCl, has 8 electrons in its
oxidation state but +4 is more stable than +2. valence shell. It does not have vacant d-orbitals to
However, for Pb, +2 oxidation state in more extend its octet. Therefore, it is an electron precise
stable than +4. So the stability of +4 oxidation molecule and hence neither accepts nor donates a
state decreases as Ge > Sn > Pb while the pair of electrons. Therefore, it does not accept a pair
stability of +2 oxidation state increases as of electrons from H,O molecule and hence CCI, does
Ge < Sn < Pb due to inert pair effect. not undergo hydrolysis in water.
For more detail Refer Text Page 8,9 and 22. Q. 5. Is boric acid a protic acid ? Explain.
Refer Conceptual Questions[I], Q.8 (Page 19).
Q. 2. How can you explain higher stability of BCl, as Q. 6. Explain what happens when boric acid is
compared to TICI,? heated.
Refer Conceptual Questions,1] Q.9 Page 19. Refer Solved Example 3 (ii) (Page 17).
Q. 3. Why does boron triflouride behave as a Lewis Q. 7. Describe the shapes of BF, and BH,. Assign the
acid ? hybridisation of boron in these species.
BF, has only six electrons around B in its valence In BF, , boron is sp” hybridised and therefore, it is
shell and therefore, needs two more electrons to a planar molecule. In [BH,|-, boron is sp? hybridised
and therefore, it is a tetrahedral molecule.
SOME p-BLOCK ELEMENTS

(ii) Pb (+2) oxidation state is more stable than Pb


= H
FO!~ | (+4) because of inert pair effect. Therefore,
Pb([V)CL, is unstable to heat and decomposes to
1!
give Pb(IT)Cl,.
va iy va \ H
Moy PbCl, __Heat_, PbCl, + Cl,
|
! H H (iii) Due to strong oxidising power of Pb** and reducing
power of I” ion, Pbl, does not exist.
Q. 8. Write reactions to justify amphoteric nature of
Q. 14. Suggest reasons why the B-F bond lengths in
aluminium.
BF, (130 pm) and BF] (143 pm) differ.
Ans. Refer Conceptual Questions, [L]Q. 10 (Page 20).
Ans. Refer Solved Example 15 (Page 18).
Q. 9. What are electron deficient compounds ? Are Q. 15. If B—Cl bond has a dipole moment, explain why
BCl, and SiCl, electron deficient species ? BCI, molecule has zero dipole moment.
Explain.
Ans. Refer Conceptual Questions @.13 (Page 20).
Ans. (i) Electron deficient compounds are those in which Q. 16. Aluminium trifluoride is insoluble in anhydrous
the central atom either does not have eight
HF but dissolves on addition of NaF. Aluminium
electrons in the valence shell or it has eight
trifluoride precipitates out of the resulting
electrons but can expand its valency beyond 4
solution when gaseous BF, is bubbled through.
due to the presence of vacant d-orbitals.
Give reasons.
(it) BCl, is an electron deficient species because it
Ans. Anhydrous HF isa covalent compound and is strongly
has 6 electrons in the valence shell of B atom. It
hydrogen bonded. Therefore, it does not give F~ ions
accepts a pair of electrons from NH, to complete
its octet. and therefore, AIF, does not dissolve in HF. On the
other hand, KF is an ionic compound and contains
Cl,.B + NH, ——-> Cl,B <— NH,
F’ ions. Therefore, it combines with AIF, to form the
(iit) In SiCl,, the central Si atom has 8 electrons but
soluble complex as
it can expand its valency beyond 4 due to the
presence of vacant d-orbitals. Therefore, it may AIF, + 3NaF ——-+ Na./AIF,]|
bod, hexafluoroaluminate
also be regarded as electron deficient.
(IIT) soluble complex
Q. 10. Write the resonance structures of CO, and
Because of smaller size and higher electronegativity
HCO... B has much higher tendency to form complexes
o—
:0: O than Al. Therefore, when BF, is added to above
— ++

_ C
| =
|
C
solution, AlF’, gets precipitated.
Na,[AIF,] + 3BF, ———> 3NalBF,]| + AIF.(s)
Sod, tetrafluoroborate
~ Nom a ~
(IIT) soluble complex
Q. 17. Suggest a reason as to why CO is poisonous.
Ans. CO is highly poisonous because of its ability to form
a complex with haemoglobin in the red blood cells.
OH OH Haemoglobin in the blood combines with oxygen
Q. 11. What is the state of hybridisation of carbon in and forms oxyhaemoglobin by a reversible reaction:
(a) CO,” (b) diamond (c) graphite ? Haemoglobin + O, —— Oxyhaemoglobin
Ans. Refer Solved Example 18 (Page 38). Oxyhaemoglobin is formed in the lungs and is carried
Q. 12. Explain the difference in properties of diamond to the different cells where it gives its oxygen.
and graphite on the basis of their structures. However, CO forms bonds with haemoglobin more
Ans. Refer Text page 26. strongly than oxygen. Therefore, if CO is present, it
Q. 13. Rationalise the given statements and give will form stable compound with haemoglobin and
chemical reactions : this will destroy the oxygen carrying ability of the
(¢) Lead (II) chloride reacts with Cl, to give blood. As a result, blood will not take up oxygen
PbCl,. easily, causing suffocation and finally death.
(tz) Lead (IV) chloride is highly unstable towards Q. 18. How is excessive content of CO, responsible for
heat. global warming ?
(ttt) Lead is known not to form an iodide, PbI,. Ans. CQ, is produced during combustion reactions. It is
Ans, (t) Due to inert pair effect, Pb is more stable in +2 used by plants during photosynthesis and QO, is
oxidation state than in +4 oxidation state. released into the atmosphere. However, if the
Therefore, lead (II) chloride is more stable than concentration of CO, in the atmosphere increases
lead (IV) chloride and hence PbCl, does not react beyond a certain limit due to excessive combustion,
with Cl, to give PbCl,. some CQ, will always remain unutilized. The excess
CQ, absorbs heat radiated by the earth. Some of it
ee is dissipated into the atmosphere while the
MODERN'S abc + OF CHEMISTRY-XI

(iit) Graphite has layered structure in which different


remaining part is redirected back towards earth’s layers are held by weak van der Waals forces.
surface which heat up the atmosphere. As a result, Because of weak forces between layers, the layers
temperature of the earth and other bodies on the can slip over one another. Hence graphite acts as
earth increases. This is called green house effect. As a lubricant.
a result of green house effect, global warming occurs (iv) Diamond is very hard and therefore, it is used as
which has serious consequences. an abrasive.
Q. 19. Explain structures of diborane and boric acid. (v) Aluminium alloys such as duralumin is light,
Refer Text Fig. 6-8. tough and resistant to corrosion and hence used
Q. 20. What happens when to make aircraft body.
(a) Borax is heated strongly, (vi) Al reacts with water and dissolved O, to form a

(6) Boric acid is added to water,


thin layer of aluminium oxide.
2Al(s) + H,O(W) + Oof¢)—> Al,0,(s) + H,
(c) Aluminium is treated with dilute NaOH,
A very small amount of Al,O, may dissolve to give
(d) BF, is reacted with ammonia ?
very small amount (a few ppm) of Al*+ ions in the
(a), (6) Refer Solved Example 4 (Page 17).
solution. Since Al** are injurious to health,
(c) Dihydrogen is formed
therefore, drinking water should not be kept in
2Al(s) + 2NaQH(aq) + 6H,O(2) ——> aluminium utensils overnight.
2Na* [Al(OH), Faq) + 3H,(g) (vii) On mass to mass basis, aluminium conducts
(d) BF, being Lewis acid accepts a pair of electrons almost twice as Cu. Therefore, it is used in
from NH, to form coordinate complex. transmission cables.
BF, + NH, ——-> BF,.NH, Q. 23. Explain why is there a phenomenal decrease in
Complex ionization enthalpy from carbon to silicon.
Q. 21. Explain the following reactions Ans. Due to increase in atomic size and screening effect,
(a) Silicon is heated with methyl chloride at the force of attraction of the nucleus for the valence
high temperature in the presence of copper; electron decreases considerably in Si as compared
(6) Silicon dioxide is treated with hydrogen to C. As a result, there is a phenomenal decrease in
fluoride; ionization enthalpy from C to Si.
(c) CO is heated with ZnO; Q. 24. How would you explain the lower atomic radius
(d) Hydrated alumina is treated with aqueous of Ga as compared to Al ?
NaQOH solution. On going from Al to Ga, there are 10 elements of the
Refer Solved Example 20 (Page 38). first transition series of d-block which have electrons
Q. 22. Give reasons : in the inner d-orbitals. The d-orbitals screen the
(¢) Conc. HNO, can be transported in aluminium nucleus less effectively because of their shapes and
container. poor penetration power. As a result, the effective
(it) A mixture of dilute NaOH and aluminium
nuclear charge in Ga becomes more than in Al and
pieces is used to open drain. therefore, atomic radius becomes slightly less
than Al.
(iit) Graphite is used as lubricant.
(tv) Diamond is used as an abrasive.
Q. 25. What are allotropes ? Sketch the structure of
two allotropes of carbon namely diamond and
(v) Aluminium alloys are used to make aircraft
graphite. What is the impact of structure on
body.
physical properties of two allotropes ?
(vt) Aluminium utensils should not be kept in
Refer Text page 26.
water overnight.
(vit) Aluminium
wire is used to make transmission
Q. 26. (a) Classify following oxides as neutral, acidic,
basic or amphoteric :
cables.
CO, B,0,, SiO,, CO,, Al,O,, PbO,, T1,0,
Ans. (t) Aluminium reacts with conc. HNO, and becomes
(6) Write suitable chemical equations to show
passive due to the formation of a very thin film
their nature.
of aluminium oxide on its surface which protects
it from further action. Therefore, aluminium (a) Refer Solved Example 23 (Page 38).
containers can be used to transport conc. HNO,. (6) For chemical equations, pages 24-25.
(it) NaOH reacts with Al and evolve H, gas whose Q. 27. In some of the reactions thallium resembles
pressure can be used to open clogged drains. aluminium, whereas in others it resembles with
2Al(s) + 2NaOQH(aq) + 2H,O(/) —> 2NaAlO,(aq) group I metals. Support this statement by giving
+ 3H,(g) some evidences.
SOME p-BLOCK ELEMENTS

Ans. Thallium exhibits +3 and +1 oxidation states. (it) Laboratory. by action of dil HCl on CaCQ,,
Aluminium shows an oxidation state of +3. Like Al, CaCO.(s) + 2HCl(aqg) ———> CaCl,(aq)
thallium shows +3 oxidation state in some of its + CO,(g) + H,O(/)
compounds such as T1,O., TICI,, etc. Like Al, Tl also Q. 33. An aqueous solution of borax is
forms octahedral complexes such as [TIF,]*~ like (a) neutral
[AIF,,|°-. Group I metals show oxidation state of +1. (6) amphoteric
Like group | elements, Tl also shows +1 oxidation (c) basic
state (due to inert pair effect). It also forms
(d) acidic
compounds like TIC], T1,O, TICIO,, etc. like group |
(c)
oxides, T1,O is strongly basic.
Q. 34. Boric acid is polymeric due to
Q. 28. When metal X is treated with sodium hydroxide, (a) its acidic nature
a white precipitate (A) is obtained, which is (6) the presence of hydrogen bonds
soluble in excess of NaOH to give soluble (c) its monobasic nature
complex (B). Compound (A) is soluble in dilute (d) its geometry
HCl to form compound (C). The compound (A) (6) Boric acid is polymeric because of hydrogen
when heated strongly gives (D), which is used bonding.
to extract metal. Identify (X), (A), (B), (C) and Q. 35. The type of hybridisation of boron in diborane is
(D). Write suitable equations to support their (a) sp (b) sp”
identities. (c) sp? (d) dsp”
Refer Solved Example @.14 (Page 18). (c)
Q. 29. What do you understand by (a) inert pair effect Q. 36. Thermodynamically the most stable form of
(6) allotropy (c) catenation. carbon is
(a) diamond (b) graphite
Ans, Refer Text.
(c) fullerenes (d) coal
Q. 30. A certain salt X, gives the following results :
(b)
(¢) Its aqueous solution is alkaline to litmus.
Q. 37. Elements of group 14
(ti) Itswells up toa glassy material Y on strong (a) exhibit oxidation state of +4 only
heating. (6) exhibit oxidation state of +2 and +4
(tit) Whenconc. H,SO, is added to a hot solution (c) form M2? and M* ion
of X, white crystals of an acid Z separates (dq) form M** and M* ions
out. (Bb)
Write equations for all the above reactions and Q. 38. If the starting material for the manufacture of
identify X, Y and Z. silicones is RSiCl,, write the structure of the
Refer Solved Example 7 (Page 17). product formed.
. Write balanced equations for : Hydrolysis of RSiCl, gives cross linked silicones.
(i) BF, + LiH—> (ii) B,H,+ ne —:
(iit) NaH + B,H,— (wv) H,BO, —> 7 R
| OH
(v) Al + NaOH —+ (vi) B,H, + NH, —> cl|s + 3H,O 2]
—-3HC > OH—Si—
Ans. Refer Solved Example 3 (Page 17).
Cl ae
Q. 32. Give one method for industrial preparation and
one for laboratory preparation of CO and CO,
each. R
Ans. Carbon monoxide nHO—$i—OH Polymerisation,
(1) Industrial. by incomplete combustion of carbon: |
OH
2C(s) + O,(g) HBX4, 2C0(g)
(ii) Laboratory. by dehydration of formic acid. R O
R
| |
HCOOH —504 CO + H,O
HasoO |
Formic
ee

Carbon dioxide
acid
1 of
---O—Si—O— Si—O— Si—R
|
| | |
(¢) Industrial. as a by-product on heating limestone. R R O

CaCO,(s) =a Ca0(s) + CO,(g) Cross linked silicone |


MODERN'S abc + OF CHEMISTRY-XI

Exemplar Problems | /
Subjective Questions
tetrahedrally bonded to four O atoms. Refer Text
Short Answer Questions [Carrying
Carrying 222or
or 33
3marks
marks|
marks
<4 Q. 9.
(Page 30-31).
Ifa trivalent atom replaces a few silicon atoms
. Draw the structures of BCl,-NH, and in three dimensional network of silicon dioxide,
AICI, (dimer). what would be the type of charge on overall
. BCI, «—NH, structure ?
Na aor Ans. If a few tetrahedral Si atoms in a three dimensional
network structure of SiO, are replaced by an equal
ar at aeNor number of trivalent atoms, then one valency of each
Q. 2. Explain the nature of boric acid as a Lewis acid Si atom becomes free. As a result, each substitution
in water. of Si atom by a trivalent atom introduces one unit
Ans. Boric acid acts as Lewis acid in water by accepting a negative charge into three dimensional network
pair of electrons from hydroxy] ion : structure of Si0,. Hence, SiO, becomes negatively
charged.
B(OH), + 2HOH —-> [B(OH),}- + H,O*
Q. 10. When BCI, is treated with water, it hydrolyses
. Draw the structure of boric acid showing
and forms [B(OH),|- only whereas AICI, in
hydrogen bonding. Which species is present in
acidified aqueous solution forms [Al(H,O),]**
water ? What is the hybridisation of boron in
ion. Explain what is the hybridisation of boron
this species ?
and aluminium in these species ?
Ans. Refer Fig. 6 (Page 14)
Ans. Hybridisation of B in [B(OH),]- is sp”.
In water, boric acid is present as [B(OH),|-.
Hybridisation of Al in [Al(H O),J* is
is sp*d?.
Boron is sp® hybridised in it.
Q. 11. Aluminium dissolves in mineral acids and
. Explain why the following compounds behave
aqueous alkalies and thus shows amphoteric
as Lewis acids ?
character. A piece of aluminium foil is treated
(i) BCI, (ii) AICI, with dilute hydrochloric acid or dilute sodium
. BCl, and AIC1, are electron deficient molecules due to hydroxide solution in a test tube and on
incomplete octets of central metal atoms and therefore, bringing a burning matchstick near the mouth
behave as Lewis acids.
of the test tube, a pop sound indicates the
. Give reasons for the following :
evolution of hydrogen gas. The same activity
(4) CCl, is immiscible in water, whereas SiCl,
is easily hydrolysed. when performed with concentrated nitric acid,
reaction doesnot proceed. Explain the reason.
(ii) Carbon has a strong tendency for
catenation compared to silicon. Ans. Conc. HNO, makes aluminium passive by forming
Ans. (i) Refer Text, Page 24. a protective layer of its oxide (Al,O,) on the surface
(it) Refer Text, Page 25, of metal.
Q. 6. Explain the following : 2Al + G6HNO, —- ALO, + 6NO, + 3H,O
(¢) CO, is a gas whereas “aang is a solid. This prevents further action.
(it) Silicon forms SiF,° ion whereas Q. 12. Explain the following:
corresponding fluoro Wi found of carbon (t) Gallium has higher ionisation enthalpy
is not known. than aluminium.
Ans. (i) Refer Text, Page 32-33. (ii) Boron does not exist as B** ion.
(ii) Silicon can extend the octet because of the (iit) Aluminium forms [AIF ,]* ion but boron
persence of vacant d-orbitals but carbon cannot does not form [BF,]* ion.
extend the octet.
(tv) PbX, is more stable than PbX,.
. The +1 oxidation state in group 18 and +2
oxidation state in group 14 becomes more and (v) Pb** acts as an oxidising agent but Sn**
more stable with increasing atomic number. acts as a reducing agent.
Explain. (vt) Electron gain enthalpy of chlorine is more
Ans. Because of inert pair effect. Refer Text (Page 6-7). negative as compared to fluorine.
Q. 8. Carbon and silicon both belong to the group (vit) TI(NO,),. acts as an oxidising agent.
14, but inspite of the stoichiometric similarity, (viit) Carbon shows catenation property but
the dioxides, (i.e., carbon dioxide and silicon lead does not.
dioxide), differ in their structures. Comment. (ix) BF, does not hydrolyse.
Ans. In CO,, carbon is sp hybridised and it is a linear (x) Why does the element silicon not form a
molecule. In SiO,, Si is sp*® hybridised and is graphite like structure whereas carbon
does.
SOME p-BLOCK ELEMENTS 11/47]
Ans. (i) Due to ineffective shielding of valence electrons
»> Long Answer Questions <q
by the intervening 3d-electrons, the effective
nuclear charge on Ga is slightly higher than Q. 15. Describe the general trends in the following
that on Al. Therefore, AH, of Ga is slightly more properties of the elements in Groups 13 and 14.
than that of Al. (i) Atomic size
(it) Because of small size of boron, sum of the first
(zz) Ionisation enthalpy
three ionization enthalpies is high and
(iit) Metallic character
therefore, boron does not lose three electrons
to form B** ions. (iv) Oxidation states
(iit) Al has vacant d-orbitals and hence can expand (v) Nature of halides
its coordination number from 4 to 6 and Ans. Refer Text.
therefore, forms [AIF ,|°. But B does not have Q. 16. Account for the following observations :
vacant d-orbitals and therefore, cannot expand (i) AICI, is a Lewis acid
octet. Hence it cannot form [BF,}*.
(it) Though fluorine is more electronegative than
(iv) Refer Conceptual Qs.[J, Q. 6(7) (Page 40-41).
chlorine yet BF, is a weaker Lewis acid than
(v) Lead shows stronger inert pair effect so that
BCl,.
Pb** ion is more stable than Pb** ion whereas
(tii) PbO, is a stronger oxidising agent than SnQ,.
Sn‘** ion is more stable than Sn** ion.
Therefore, Pb** are more readily reduced to (iv) The +1 oxidation state of thallium is more
Pb** as compared to Sn**. Thus, Pb** acts as a stable than its +3 state.
stronger oxidising agent. Ans. (7) AlCl, is an electron deficient compound and
(vi) Due to small size of fluorine, it has larger therefore, behaves as a Lewis acid.
electron-electron repulsions. Therefore, electron (iz) BF. is a weaker Lewis acid than BCl, because of
gain enthalpy of F is less negative as compared back bonding from filled 2p-orbital of fluorine to
to that of Cl. vacant orbital of B resulting px — pr back bond.
(viz) Due to inert pair effect, Tl in +1 oxidation state Refer Text Page 9.
is more stable than +3 oxidation state. (zit) In PbO, and SnQ,, both lead and tin are present
Therefore, TI(NO,), acts as an oxidising agent in +4 oxidation state. Due to inert pair effect,
and gets reduced to stable +1 oxidation state. Pb** ion is more stable than Pb** ion and
(vil) Refer Text (Page 23-24). therefore, PbO, is readily reduced to PbO and
(ix) BF, does not hydrolyse completely. Instead it therefore, PbO, is a stronger oxidising agent. On
undergoes incomplete hydrolysis to boric acid the other hand, Sn* is more stable than Sn** and
and fluoroboric acid because HF first. formed therefore, Sn** ions are not readily reduced to Sn**
reacts with H,BO,. ions and SnOQ, is not a stronger oxidising agent.
BF, + 3H,0 ——> H,BO, + 3HF] x4 (iv) Due to inert pair effect, +1 oxidation state of
H,BO, + HF —> H* + [BFP + 3H,O] x 3 thallium is more stable than +3. Refer Text
(Page 7).
4BF, + 3H,O —> H,BO, + 3[BF,) + 3H*
Q. 17. When aqueous solution of borax is acidified with
(x) Refer text (Page 25-26). hydrochloric acid, a white crystalline solid is
Q. 13. Identify the compounds A, X and Z in the formed which is soapy to touch. Is this solid
following reactions : acidic or basic in nature ? Explain.
A + 2HCI + 5H,0 —> 2NaC]l + X Ans. When aqueous solution of borax is acidified with
A
HCl, boric acid is formed which is a white crystalline
X a > HBO, > S70K Z solid.

Ans. Na,B,O, + 2HCl


+ 5H, O—> 2NaCl + H,BO,
Na,B,O, + 2HCl + 5H,0 —> 2NaCl + 4H,BO,
oe 4 7
Borax Boric acid
Borax
It acts as a Lewis acid.
H,BO, “> HBO, —s370K— B,O, Q. 18.Three pairs of compounds are given below.
Identify that compound in each of the pairs
A : Na,B,O,; X : H,BO,; Z:B,O,
which has group 13 element in more stable
Q. 14. Complete the following chemical equations :
oxidation state. Give reason for your choice.
Z+ 3LiAlH, —> X + 3LiF + 3AlF, State the nature of bonding also.
X + 6H, — Y + 6H,
(i) TICI,, TIC] (tt) AICL,, AICI
X +30, = B,O, + 3H,O (iit) InCl,, InCl
Ans. BF, + 3LiAlH, —— BH, + 3LIF + 3AlF, Ans. (i) TIC] is more stable and bonding is ionic.
(Z) (X) (it) AlCl, is more stable and bonding is covalent.
B,H, + 6H,0 ——> H,BO, + 6H, (it) InCl, is more stable and bonding is covalent.
(X) (Y) Q.19. BCI, exists asmonomer whereas AICl, is dimerised
B,H, + 30, —> B,O, + 3H,O through halogen bridging. Give reason. Explain
(X) the structure of the dimer of AICI, also.
Ans. Refer Text (Page 9-10).
Tis
Q. 20. Boron fluoride exists as BF, but boron hydride
MODERN'S abc + OF CHEMISTRY-XI

Ans. The non-metallic element of group 13 is boron. It is


doesnot exist as BH,. Give reason. In which form an extremely hard substance and is used in making
does it exist ? Explain its structure. bullet proof vests. This shows maximum covalency
Ans. Refer HOTS. Q. 1. Page 49. of four.
Q. 21.(2) What are silicones ? State the uses of
silicones. H,N + BF, ——> H,N —> BF,
(it) What are boranes ? Give chemical equation Since B in BF, has only six electrons in its valence
for the preparation of diborane. shell and needs two more electrons to complete the
Ans. Refer Text. octet, it acts as a Lewis acid.
Q. 22.A compound (A) of boron reacts with NMe, to Q. 24.A tetravalent element forms monoxide and
give an adduct (B) which on hydrolysis gives a dioxide with oxygen. When air is passed over
compound (C) and hydrogen gas. Compound heated element (1273 K), producer gas is
(C) is an acid. Identify the compounds A, B and obtained. Monoxide of the element is a
C. Give the reactions involved. powerful reducing agent and reduces ferric
Ans. Since compound (A) reacts with NMe, and forms an oxide to iron. Identify the element and write
adduct (B), (A) must be a Lewis acid. Since the adduct formulas of its monoxide and dioxide. Write
(B) on hydrolysis gives an acid (C) and hydrogen gas, chemical equations for the formation of
A must be B,H, and (C) must be boric acid. producer gas and reduction of ferric oxide with
BH, + 2NMe, —— 2BH,NMe, the monoxide.
Diborane (A) Adduct (B) Ans. Producer gas is a mixture of CO and N,. Therefore,
BH,-NMe, + 3H,O —~ H,BO,+ 6H, + NMe, the tetravalent element is carbon and its monoxide
Boric acid (C) and dioxide are CO and CO, respectively.
Q. 23.A non metallic element of group 13, used in
making bullet proof vests is extremely hard 2C(s) + Oo(g)+ 4No(g) —BBE_, 2C0(g) + 4No(g)
solid of black colour. It can exist in many Air Producer gas
allotropic forms and has unusually high The monoxide of carbon is a strong reducing agent
melting point. Its trifluoride acts as Lewis acid and reduces ferric oxide to iron.
towards ammonia. The element exhibits
maximum covalency of four. Identify the
Fe,0,(s) + 3CO(g) —S-» 2Fe(s) + 3CO,(g)
element and write the reaction of its trifluoride
with ammonia. Explain why does the
trifluoride act as a Lewis acid.

MORY TEST
WwW

13. Pbl, is quite stable.


1. Carbon atoms in graphite are sp* hybridised while in
14, TICIO, is more stable than T1(C1O,)..
diamond these are sp* hybridised.
2. BF, is a stronger Lewis acid than BCl,. 15. The aqueous solution of borax is alkaline.
3. PbO, is a stronger oxidising agent than SnQ,,.
fa. The first ionization enthalpy of gallium is lower than
1 . Borate ion has ........... structure.
that of aluminium.
2. SnCl, acts as a ........... agent.
. Orthosilicates contain two SiOj;” tetrahedra units. 3. The basic building unit of silicates Is ...........
. Beryl is an example of pyrosilicates. 4, The most stable oxidation state of thallium is .........
5
. 510, exists as three dimensional network solid. . Pyrosilicates contain .......... anion.
OI
&
os. Diborane structure has two three centre electron pair 6» In N(SiH,),, N atom is .......... hybridised.
bonds. 7. The molecular formula of inorganic benzene is ........ :
Be cedetiente occurs as colemanite.
9. B,H, is also called inorganic benzene.
9. In group 14 elements, ........... shows maximum inert
10. Boric acid is a protonic acid.
pair effect.
11. The stability of +4 oxidation state increases from C to 10. In borax bead test, the bead consists of ........ and
Pb.
12. [SiCl,]>- is known but [SiF 1?” is not known.
SOME p-BLOCK ELEMENTS 11/49 fe

6. Carbon has strong /weak tendency for catenation than


silicon.
1. Graphite is thermodynamically more
/less stable than 7. The anhydride of carbonic acid is CO,/CO.
diamond. : : —— j
: : ‘ — 8. Strong heating of boric acid forms metaboric acid/
2. Reduction of SiCl, with dihydrogen gives Si/SiH,. RarGR ane.
3. +2 oxidation state of Pb is more/less stable than +4. m9 ; ; nae
4. In diamond, C undergoes sp”/sp® hybridisation % or, Same of Hrmue ack’ Eveosiaiaaaa
° ° *B oe cay _ 10. Tale is an example of sheet silicates/chain silicates.
5. Bond length of B—F in BF, is less /more than that in
[BEF

Answers »)

Say True or False Complete the missing links


1. True 1. trigonal planar 2. reducing
2. False : BF, 1s a weaker Lewis acid than BCl,. 3. Si of idra/4, +1 5. Sig oN
3. True 4, False
- 6. sp” 7. B,N3H, 8. Boron
5. False : Orthosilicates contain one SiO; tetrahedral 9. had
unit.
10. Sodium metaborate, boric anhydride
6. False : It is an example of chain silicates. |
7. True 8 Tre Wales Choose the correct alternative

10. False 11. False 1. more 2. Si 3. more


12. False : [SiF,] 2- is known but [SiCl,]>- is not known. 4. sp* 5. less 6. strong
13. False 14, True 15. True. 7. CO, 8. boron oxide 9, CO
10. sheet silicates


——

= Higher Order Thinking Skills ¢


_ Advanced Level |}
“= ==—— QUESTIONS WITH ANSWERS _—

Q. 1. Explain why BF, exists but BH, does not. Ans. Gallium has higher ionisation enthalpy than
Ans. In BF,, the vacant 2p-orbital of boron accepts 7 aluminium because of higher effective nuclear charge. This
electron density by overlapping with filled 2p, orbital of F. is due to additional 3d1° electrons which do not shield the
Therefore, the stability of BF, is due to back bonding. nuclear charge effectively so that the outer electrons are
Va P strongly held.
rR +p =p Q.3. cama sores ne vieouaciigstaota in ee state
. F SS but thallium in unipositive state is quite stable. Why ?
; Ans. Boron has electronic configuration 2s?2p'. It is
iy A always trivalent in its compounds. On the other hand,
<—~ F—B — F==B —_ thallium prefers to form unipositive ions rather than
\ ~ F tripositive ion. This is because of inert pair effect. In case of
Tl ([Xe]4f74 5d" 6s? 6p), the outermost 6s? electrons do not
However, hydrogen atoms do not have free electrons to
form back bonding with boron and therefore, BH, is not take part in bonding and behave like inert gas. Therefore, it
forms stable Tl* compounds.
stable. However, to satisfy the electron deficiency of boron, it
dimerizes to form B,H,. Q.4. Which of the following is not hydrolysed by
water and why ?
Q. 2. Gallium has higher ionization enthalpy than
BF,, BCL, and BBr,
aluminium. Why ?
MODERN’S abc + OF CHEMISTRY-XI

Ans. BF’, is not easily hydrolysed by water. It forms an presence of polyhydroxy compounds like catechol,
adduct BF, .OH, whereas BCl, and BBr, are hydrolysed to mannitol etc. Explain.
boric acid and HCl or HBr respectively. Ans. Boric acid is a weak acid and reacts with NaOH to
This is because the B—F bond in BP, is very strong form sodium metaborate.
due to extensive pn—pmt back bonding. As a result, it is not B(OH). + NaOH ——> Na*BO,~ + 2H,O
hydrolysed by water. The B—F bond energy is far larger The end point during the titration of H,BO, and NaOH
than B—OH bond energy and cannot be compensated. is not sharp and cannot be detected properly. This is because
F ge of the fact that sodium metaborate formed during the
+ 3HF neutralisation reaction undergoes hydrolysis and gives back
p_ Be + H,O — HO.
H,BO, and NaOH.
\F OH
(Not possible) NaBO, + 2H,0O <= B(OH), + NaOH
However, in BCl, and BBr,, the corresponding B—Cl However, when titration is carried out in the presence of
certain polyhydroxy compounds such as catechol [C,H, (OH),I,
and B—Br bond energy is relatively less than B—F because
mannitol [C,H,(OH)<], glycerol [CH,OHCH(OH) CH,OH] etc.
of inefficient pn—pt bonding. Therefore, these get hydrolysed.
boric acid behaves as a strong monobasic acid and end point
Q.5. BF, can act as Lewis acid but CCI, cannot do
can be detected easily. The metaborate ion combines with the
so. Why ? polyhydroxy compound to form a complex.
Ans. BF, molecule can act as a Lewis acid because it
OH

OL
can accept a pair of electrons from electron donor groups and
thus completes its octet. But in CCl,, carbon atom has
+BO,- _-2H,0_,
already got a complete octet so it cannot accept a pair of OH
electrons. Hence, it does not act as a Lewis acid. Catechol

OOM
Q.6. AIF, is high melting solid but AICI, is low
| 0 0
melting. Explain.
Ans. AIF, is high melting solid because it is ionic in
nature. On the other hand, AICI, is covalent in nature and
hence is a low melting solid.
Complex ion
Q.7. Account for the dimeric structure of AICl.. The above complex does not combine with H* ions
Ans. The molecular formula of aluminium chloride is because boron atom has acquired the maximum covalency
AICI,. In aqueous solution, it exists in monomeric form, But of four. Therefore, BO, does not undergo hydrolysis. As a
in vapour state, it exists as a dimer, Al,Cl,. In this structure, result, boric acid ionises to greater extent producing large
the Al atom of one AICI, unit accepts a lone pair of electrons number of H* ions and therefore, it behaves as a strong acid
from a Cl atom of the second AICI, molecule. In this way, and can be easily titrated with NaOH.
the octet of each Al atom is completed as: Q.10. PbO, is a stronger oxidising agent than SnQ,.
Explain.
~\
]
a7 Cl OR Cl
Ans. In PbO, and SnQ,, both lead and tin are present in
a’ av a +4 oxidation state. But lead shows stronger inert pair effect
so that Pb?* ion is more stable than Pb** ion whereas Sn*+
Q.8. Boric acid is monobasic. Explain. ion is more stable than Sn** ion. Therefore, Pb** ions (PbO,)
are more readily reduced to Pb?* ions as compared to Sn‘4*.
Ans. Boric acid B(OH), acts as an acid by accepting OH-
Thus, PbO, acts as a stronger oxidising agent.
ion (Lewis acid).
Q.11. Molten aluminium bromide is a poor
OH OH conductor of electricity. Why ?
B +H,O—> Bo —> [BOH),b Ans. Aluminium bromide is a covalent molecule. Even
in molten state it, ionizes only to a very small extent and
on a HO “|\~oni
by therefore, is a poor conductor of electricity.
+H,O+ Q.12. Like CO, its analog SiO is not stable. Why ?
Boron thus forms [B(OH),| ion and completes its octet. Ans. CO is a resonance hybrid of the following structures:
Therefore, it acts asa monobasic acid because there is no
provision for further reaction since boron atom has
Oo
‘C=O. ———
8
-CcmO:

completed its octet. Therefore, CO has pn—pt multiple bonds. This is because
C being small in size and having high electronegativity forms
Q.9. Boric acid can be titrated against sodium
strong pt—pt bonds. Silicon on the other hand, because of its
hydroxide using methyl orange as indicator only in the
larger size and lower electronegativity has no tendency to
form pm—pt bonds and hence does not form stable SiO.
SOME p-BLOCK ELEMENTS

Revision Exercises
» Very Short Answer Questions <4 » Smee me ciel a Carrying 2 or 3 marks
<4
1. Explain why are boron halides referred to as electron
What is the formula of kernite, an ore of boron ? deficient compounds.
What is borax bead test ? 2. How does boron react with
What is the action of heat on boric acid ? (i) NaOH (zi) N,?
~YS
Why does boron halides form addition compounds with 3. By means of a balanced equation show how B(OH),
amines ? behaves as an acid in water.
Why do boron halides behave as Lewis acids ? 4, What are fullerenes ? How are they prepared ?
5. Write resonance structures for CoO and HCO...
What is the shape of BH; ion ?
6. Account for the toxic nature of carbon monoxide.
Which is the most aboundant element in group 13 ? 7. Explain the differences in the properties of diamond
What are the formulae of bauxite and cryolite ? and graphite on the basis of their structures.
HF
nN
oP Which of the two Al and Ga has larger atomic radius ? 8. Write brief account of water gas and producer gas.
10. Why does AICI1, exists as dimer ? 9, How is borax prepared from colamanite ? How does it
react with water ? Give the chemistry of borax bead
11. Is B,O, acidic or basic oxide ?
test.
12. What happen when borax is heated ?
10. How is boric acid prepared from borax ? Discuss its
13. Name the second most abundant element on the earth’s structure.
crust. 11. How would you explain the lower atomic radius of Ga
14, What type of hybridisation of central atom is possible as compared to Al ?
in [GeCl,]?-? 12. How does BF, react with
15. Name the member of group 14 that forms the most (i) ammonia (iz) lithium hydride
acidic oxide. (iit) water

16. Complete the reaction : 13. Comment on the pr-pz and dz-pz bonding in elements
of group 14.
Sn + steam ——> 14, List some important consequences of absence of
d-orbitals in boron and carbon.
17. Which of the two allotropes : diamond or graphite is
good conductor of electricity and why ? 15. Give four important uses of each of the following :
(¢) Aluminium (iz) Carbon
18. Diamond is a covalent compound, yet it has high melting
point. Why ? 16. In some reactions, thallium resembles aluminium
whereas in others it resembles with group I metals.
19. How is producer gas prepared ? Support this statement by giving some evidences.
20. Write the resonating structures of CO,,. 17. Explain why is there a phenomenal decrease in
21. Write the structure of basic unit of all silicates. ionization enthalpy from carbon to silicon ?
22. Define inert pair effect. 18. Discuss the pattern of variation in oxidation states of
23. Is boric acid a protic acid ? Explain. (i) B to Tl (ii) C to Pb
24, Why does BF, behave as a Lewis acid ? 19. Write reactions to account for amphoteric nature of
aluminium.
20. Explain what happens when boric acid is added to
20. Explain structures of diborane and boric acid.
water ?
21. Suggest reasons why the B-F bond lengths in BF, is
26. Though B-Cl bond is polar yet BCl, has zero dipole
smaller than in BF.
moment. Explain.
22. What is the state of hybridisation of carbon in
27. Suggest reason as to why CO is poisonous.
28. Why do boron halides form addition compounds with (i) CO (ii) diamond
amines ? (iii) graphite (iv) CCl,
29. What is dry ice ? Why is it so called ? 23. Describe the shapes of following :
30. Describe what happens when boric acid is heated ? (i) BEF (ii) BCI,
(iii) AlCl, (iv) SiCl,
Siz
24, How can you explain the stability of BCl, as compared 2. Write short notes on:
MODERN'S abc + OF CHEMISTRY-XI

to TIC], ?
(i) Fullerenes (ii) Structure of graphite
25. Classify the following oxides as neutral, acidic or
(iit) Structure of diborane
amphoteric or basic ?
3. Name the elements of group 14. Discuss the variation
(i) CO (ii) B,O, (ii) SiO, ~—(iv) CO,
in characteristics of the group with reference to
(v) Al,O, (vi) PbO, (vit) T1,O,
(a) Oxidation state (b) Catenation
26. Discuss the variation of following properties in
(c) Metallic character (d) Reactivity with oxygen
group 13:
Explain the following :
(i) reactivity with air
(a) PbCl, is less stable than PbCl,
(ii) reactivity with acids and alkalies
(6) Tl) compounds are more stable than TI(IIT)
(iii) reactivity towards hologens.
(c) CO, is a gas while SiO, 1s a solid.
27. Explain the following :
(d) AICI, exists as dimer but BCl, does not.
(¢) Boron does not form BF,” . (e) BF, is less Lewis acid than BC],.
(ii) Boric acid is a weak acid. (f) Boric acid is not protic acid.
(iii) BF, is weaker Lewis acid than BCl,. How do you account for
(1v) [Sif]? is known but [SiCl,]?- is not. (i) Anhydrous AIC1, is covalent but hydrated AIC1, is
lonic.
28. Discuss the following among group 14 elements :
(ii) BBr, is stronger lewis acid than BF,.
(i) Oxidation states
(tii) Boron does not form B** ions.
(ii) Atomic size
6. Write balanced chemical equations for :
(iii) Reactivity with oxygen
29. Complete the following reactions : (i) SiO, + HF —— >
(i) Na,B,O, 10H,0 —"*_, (ii) AIC1, + NaOH —>
(ii) H,BO, + H,0 ——> (ii) Na,B,O, + H,O ——>
(iii) Fe,O, + CO ——> (iv) BF, + LiAlIH, —>
(iv) HCOOH
Heat (v) BH, +CO —>
Write a brief account of
(v) BN+H,O —> (a) structure of diborane
(6) silicones and their uses
» Long Answer Questions <q (c) uses of aluminium
1. Describe the structure of diborane. What is special
about this structure ?

0,8

tle Additional Useful Information and Objective Questions

ADDITIONAL USEFUL INFORMATION


- CARBIDES Carbides can be classified on the basis of type of bonding
Carbon combines with many electropositive elements in them as:
when heated to high temperature to form carbides. The
(a) Ionic carbides: These are the carbides of strongly
carbides are compounds of carbon with more electropositive
electropositive metals belonging to the alkali and
elements than itself. Some common examples are silicon
alkaline earth metals and aluminium. For
carbide (SiC), iron carbide (Fe,C), calctum carbide (CaC,),
example, CaC,, Al,C,, Mg,C., Be,C, ete.
tungsten carbide (WC), aluminium carbide (AL,C,) ete.

WWW.JEEBOOKS.IN
SOME p-BLOCK ELEMENTS
These are essentially ionic in nature. These are easily Boron carbide is also extremely hard and is used as an
hydrolysed with water or dilute acids at ordinary abrasive and as an shield from radiation. It is prepared by
temperatures forming hydrocarbons corresponding to the the reduction of B,O, with C in an electric furnace at
anions C*(CH,), Cc," (C,H,) and C,*(C gH,). Depending upon ata Ee
the hydrocarbon which they form on hydrolysis, these (c) Interstitial carbides : These are the compounds of
carbides are classified as : carbon with transition metals and some of the lanthanides
(t) Methanides. These contain individual carbon and actinides e.g., WC, TaC, ZrC, V,C, ete.
anions (C*-) and give methane on hydrolysis. In these compounds, the carbon atoms occupy the voids
Be,C+4H,O0 —— 2Be(OH), + CH, in the close packed lattice of metal atoms. These carbides
Al,C,+12H,O —— 4Al(OH), + 3CH, are extremely hard and electrically conducting. Due to their
extremely hardness and inertness, WC and TaC are used for
(ii) Acetylides. These contain the carbide anion C,?-
the manufacture of high speed cutting and drilling tools.
(-C=C-)* and give acetylene on hydrolysis. For
example, MgC.,, CaC, etc. Carbides are generally prepared by heating the element
or its oxide with carbon at a very high temperature. For
CaC,+2H,O —— Ca(OH), + C,H,
example,
Acetylene
Applications of carbides. Carbides have many
(iii) Allylides. These contain C,* anion and give allylene
applications in industry. Some of these are :
on hydrolysis. For example,
(t) Silicon carbide known as carborundum is very hard
Mg,C, + 4H,0——>2Mg (OH), + CH, —C = CH
and is used as an abrasive for sharpening and grinding
Allylene metals and other substances.
CaC,, Al,C,, Mg,Cg, ete. (11) Tungsten carbide is used for making tools and dies.
(6) Covalent carbides : These are the compounds of (iii) Calcium carbide is used for preparing acetylene
carbon with metalloids which have the same which is used for welding and is an important starting
electronegativity as carbon. For example, silicon carbide material for a number of organic compounds such as ethyl
(SiC), boron carbide (B,C). Silicon carbide is known as alcohol, acetic acid, ete.
carborundum and is used for making cutting tools and (iv) Be,C is also very hard and is used as a shield against
abrasives. radioactive radiations.

OBJECTIVE TYPE QUESTIONS


A5. Three centre two electron bond is present is
(a) BF, (b) BoHe
(ec) H,BO, (d) AICI,
Select the Correct Answer : A6. From B,Hg, all the following can be prepared except
(a) B,(CHs), (6) H,BO,
GROUP 13 ELEMENTS (c) B,(CH,),H, (d) B,N.H,
Al. Borax gets hydrolysed to give A7. Among the following the compound which exists as
dimer is
(a) basic medium
(6) acidic medium (a) BCI, (6) AICI,
(c) CHCL, (d) CO,
(c) neutral medium
A8. Which of the following has least Lewis acid character ?
(d) may be acidic or neutral.
A2. Orthoboric acid on strong heating to red hot gives
(a) BI, (b) BCL,
(c) BBr, (d) BF,
(a) Metaboric acid (b) Borax
A9. AICI, dissolves in NaOH to form
(c) Boron trioxide (d) Tetraboric acid.
(a) Al,O, (b) Na[Al(OH),]
AS. The most basic oxide is
(c) Na,[Al(OH),] (d) Na,AlO,
(a) BO, (b) Ga,O, A10. In Al,Cl,, the number o and coordinate bonds are :
(c) 1,0, (d) Al,O, (a) 3,3 (b) 2,4
A4, An aqueous solution of borax is (c) 4, 2 (d) 6,0
(a) bleaching agent (6) alkaline All. The hydrolysis of diborane gives a solution which is
(c) acidic (d) neutral. (a) neutral (6b) basic
(ce) acidic (d) amphoteric

A3. (c) Ad, (b) =A5. (6b) A6. (a) AZ. (6) A8. (d) A (6) AlO. (c)
Tiss MODERN'S abc + OF CHEMISTRY-XI

A12. The type of hybridisation in diborane is . When formic acid is heated, the gas evolved is
(a) sp (b) sp? (a) CO (b) CO,
(c) sp® (d) dsp? (c) H, (d) SO,
A138. Boric acid is polymeric due to . Inert pair effect is predominant in
(a) its acidic nature
(a) Si (b) Pb
(6) the presence of hydrogen bonds
(c) Ge (d) Sn
(c) its monomeric nature
. Silicon shows diagonal relationship with
(d) its geometry
(a) B (b) Be
Al4, B,O, is
(ce) C (d) Al
(a) acidic (b) basic
. The high temperature polymers of silicon are called
(c) neutral (d) amphoteric
(a) Silicates (b) Silicon halides
Al15. Borax is prepared by treating colemanite with
(c) Silanes (d) Silicones
(a) NaNO, (b) NaCl
A27. Which of the following is not the from of silica ?
(c) Na,CO, (d) NaHCo,.
Al6. Which of the following statement about boric acid is
(a) Cassiterite (b) Cristobalite
false ? (c) Tridymite (d) Quartz
(a) It acts as a monobasic acid A28. Which of the following species does not exist ?
(b) It is formed by the hydrolysis of boron halides (a) [SnCl,]?- (b) [SiC] ,]?-
(c) It has planar structure. (c) [CC1,]>- (d) [GeC1,]?-
(d) It acts as a tribasic acid. A29. The basic structural unit of silicates is
Al17. The geometry and type of hybrid orbitals present about (a) Si0, (b) Si**
the central atom in BF, is (c)Si0,* (d)Si0,2-
(a) linear, sp (b) trigonal planar sp? A30. Thermodynamically the most stable form of carbon is
(c) tetrahedral, sp? (d) pyramidal sp* (a) diamond (b) graphite
Al8. Which of the following is not a protonic acid ? (c) fullerenes (d) coal
(a) B(OH), (6) PO(OH), . Elements of group 14
(c) SO(OH), (d) SO,(OH), (a) exhibit oxidation state of + 4 only
Al19. Borax is used as cleansing agent because on dissolving (b) exhibit oxidation state of +2 and +4
in water it gives (c) form M?- and M** ions
(a) alkaline solution
(d) form M?2* and M* ions
(b) acidic solution
. Which of the oxide is neutral ?
(c) bleaching solution
(d) colloidal solution
(a) CO (b) SnO,
. AIL,O, can be converted to anhydrous AlCl, by heating
(c) ZnO (d) SiO,
. In graphite, electrons are
(a) Al,O, with NaCl in solid state
(a) localised on every third carbon
(6) a mixture of Al,O, and carbon in dry Cl, gas
(6) present in antibonding orbitals
(c) Al,O, with Cl, gas
(c) localised on each carbon atom
(d) Al,O, with HCl gas
(d) spread out between the structure
GROUP 14 ELEMENTS . Percentage of lead in lead pencil is
. Carborundum is (a) zero (b) 20
(a) Calcium carbide (6) Boron carbide (c) 80 (d) 60
. Which of the following is electron deficient molecule ?
(c) Aluminium carbide (d) Silicon carbide
. Carbon forms large number of compounds because it
(a) C,H, (b) BH,
has (c) SiH, (d) Al,Cl,
(a) low electron affinity . Supercritical CO, is used as
(b) no d-orbitals in valence shell (a) dry ice (b) fire fighting
(c) a solvent for extraction of organic compounds from
(c) variable valency
natural sources.
(d) property of catenation
(d) a highly inert medium for carrying out various

Ariswts
reactions.

Al2.(c) Al3. (6) Al4. (a) Ald. (c) Al6. (d) A17. (b) Al18. (a) <Al19. (a) A20. (6) A211. (d)
A22. (d) A238. (a) A24. (b) A25. (a) A26. (d) A27. (a) A28. (c) A29. (ec) ASO. (b) ASI. (5)
AS2. (a) A338. (d) A384. (a) A85. (6) A36. (c)
SOME p-BLOCK ELEMENTS

B
B7. B(OH), is a
MULTIPLE CHOICE QUESTIONS (a) Lewis acid (b) Lewis base
from Competitive Examinations (c) Bronsted base (d) Bronsted acid
(e) Neutral compound
AIPMT & Other State Boards’ (Kerala PMT 2010)
Medical Entrance
B8. Germanium of very high purity is obtained by
Bl. Which one of the following anions is present in the (a) liquation (6b) vapour phase refining
chain structure of silicates ?
(c) distillation (d) zone refining
(a) SiO, (b) Si,0, (e) electrolytic refining (Kerala P-M.T. 2012)
(c) (Si,0.2), (d) (Si0,25), B9. The product/s formed when diborane is hydrolysed 1is/
(C.B.S.E. PM.T. 2007) are
B2. The ion(s) that act/s as oxidizing agent in solution is/ (a) B,O, and H,BO, (b) B,O, only
are : (c) H,BO, and H, (d) HBO, only
(a) Tl* and Al** (b) B?* and Al*+ (e) B,O, and H, (Kerala PM.T. 2012)
(c) Tl* only (d) B* only B10. The basie structural unit of silicates is
(e) Ti®* only (Kerala PMT 2008) (a) Si0,7” (b) SiO,”
(ec) Si0O7 (d) SO,7 (NEET 2013)
B3. The stability of +1 oxidation state increases in the B11. Which of these is not a monomer for a high molecular
sequence mass silicone polymer ?
(a) Tle n= Ga<Al (6b) Ine Tl<= Ga<Al (a) Me,SiCl (6) PhSiCl,
(c) Ga<In<Al<TI (d) Al<Ga<In<TI (c) MeSiCl, (d) Me,SiCl, (NEET 2013)
(CBSE-PMT 2009) B12. Three centred two electron bond is present in
B4, The straight chain polymer is formed by (a) NH, (b) BH,
(a) hydrolysis of CH,SiC1, followed by condensation (c) BCI, (d) AICI,
polymerisation (ce) BF, (Kerala PMT 2014)
(b) Hydrolysis of (CH,),5i by addition polymerisation B13. The stability of +1 oxidation state among Al, Ga, In
(c) Hydrolysis of (CH,),SiCl, followed by condensat- and Tl increases in the sequence:
ion polymerisation. (a) Ti<In<Ga<Al (6) n<Tl<Ga<Al
(d) Hydrolysis of (CH,),51Cl followed by condensation (c) Ga<In<Al<Tl (d) Al< Ga<In<TIl
polymerisation. (CBSE-PMT 2009) (AIPMT 2015)
B5. Reaction of diborane with ammonia gives initially B14. Which of the following fluoro-compound is most likely
(a) BJH,NH, (b) Borazole to behave as a Lewis base?
(c) B,H,.8NH, (d) [BH,(NH,),)° [BH (a) BF, (b) PF,
(e) B,N,H,, (Kerala PMT 2010)
(c) CF, (dq) Sik, (NEET 2016)
B6. The correct order of matching of the following compounds is
B15. Boric acid is an acid because its molecule
1
(A) Borazole (i) CaSO,. 3 HO (a) contains replaceable H’ ion
(B) Plaster of Paris (wi) C,, (b) gives up a proton
(c) accepts OH’ from water releasing proton
(C) Boric acid (iii) SiO,
(2d) combines with proton from water molecule
(D) Quartz (wv) B,N,H,
(NEET 2016)
(E) Buckminster fullerence (v) H,BO,
B16. It is because of inability of ns? electrons of the valence
(a) A- (it); B- (i); C- (i); D-(v); E- (iv) shell to participate in bonding that
(b) A-(1);B-(v);C- (it); D- (iv); E- (i) (a) Sn** is oxidising while Pb** is reducing
(c) A-(v);B-@);C-@; D- Gi); E- ww) (b) Sn** and Pb? are both oxidising and reducing
(d) A- (iv); B-(@);C-(v); D- Gi); E- Ga) (c) Sn** is reducing while Pb** is oxidising
(e) A-(iv); B-(@);C-(v); D- i); E- (ui) (d) Sn?* is reducing while Pb** is oxidising
(Kerala PMT 2010) (NEET 2017)

Answers
Bl. dd) B2.(c) BS. dd) B4. (dd) BB. @d) B7. (a) ~=B8. (d) ~—BY. (c) ~—=—«B10. (d)
Bll. (a2) B12. (6) B13. (d) B14. (6) BIS. (c) B16. (d)
Tie MODERN'S abc + OF CHEMISTRY-XI

B17. Which one of the following element is unable to form B25. When PbO, reacts with conc. HNO,, the gas envolved
MF: ion? is
(a) Ga (6) Al (a) NO, (b) O,
(c) B (d) In (NEET 2018) fc) NN, (d) N,O(A.LLE.E.E. 2005)
B18. Thecorrect order ofatomicradiiingroup13elements 396, A metal M forms two chlorides in its +2 and +4
an oxidation states. Which of the following statements
(a) B<Al<n<Ga<Tl about these chlorides is correct ?
(6) B<Al<Ga<In<Tl (a) MCL, is more soluble in anhydrous ethanol than
(c) B<Ga<Al<Tl<In MCL.
(dd) B< Ga<Al<In<TIl (NEET 2018) (6) MCl1, is more ionic than MC1,.
(c) MCL, is more easily hydrolysed than MCl,.
JEE (Main) & Other State Boards’ (d) MCl, is more volatile than MC1,.
Engineering Entrance (A.LE.E.E. 2006)
B19. Graphite is soft and lubricant extremely difficult of melt. B27. The stability of the dihalides of Si, Ge, Sn and Pb
The reason for this anomalous behaviour is that graphite increases steadily in the sequence :
(a2) has carbon atoms arranged in large plates of rings (a) SiX, << GeX, << PbX, << SnX,
of strongly bound carbon atoms with weak interplate (b) SiX, << GeX, << SnX, << PbX,
bonds. (c) PbX, << SnX, << GeX, << Six,
(b) is a non-crystalline substance (d) GeX, << SiX, << SnX, << PbX,
(c) 1s an allotrope form of carbon (A.LELE_E. 2007)
(d) has molecules of variable molecular masses lke B28. Diamond is hard because
polymers. (A.LE.E.E. 2003) (a) All the four valence electrons are bonded to each
B20. Which one of the following has regular tetrahedral carbon atom by covalent bonds
structure ? ae (b) It is a giant molecule
a ae )
a INi(CN) \- (c) It is made up of carbon atoms
| : | (A LE.E.E. 2004) (d) It cannot be burnt
B21. Which of the following oxide is amphoteric in (A.LELELE. 2008)
character : | B29. Which one of the following is the correct statement ?
o =. me OG hen hoor (a) B,H,. 2NH, is known as ‘inorganic benzene’
| ree. ‘ ny 4 86a (b) Boric acid is a protonic acid
B22. Heating an aqueous solution of aluminium chloride _ , ;
to dryness will give (c) Beryllium exhibits coordination number of six
(a) AICI, (b) AL,Cl, (d) Chlorides of both beryllium and aluminium have
| ~ gee bridged chloride structures in the solid phase.
Al,O d) Al(OH)CI1
ae we >o,f (A.LE.E.E. 2008) |
B23. In silicon dioxide sf B30. Which one of the following has highest Lewis acid
' ?
(a) each silicon atom is surrounded by four oxygen ee |
atoms and each oxygen atoms is bonded to two (a) BI, (6) BBr,
silicon atoms (c) BF, (d) BCl,
(6) each silicon atom is surrounded by two oxygen (J & K 2009)
a es each oxygen atom is bonded to two —-Rg 1. The basic structural unit in silicates is
silicon atoms. e eae
(c) silicon atom is bonded to two oxygen atoms id os _ a. ee
(d) there are double bonds between silicon and (c) [51,07] (d) [81,05] |
oxygen atoms (A..LELELE. 20085) (AMU-Engg. 2009)
B24, The structure of diborane (B,H,) contains B32. The products of the following reaction?
(a) four 2c—2e bonds and two 3c—Ze bonds 510, + C _“ are:
(6b) two 2e—2e bonds and four 3c—3e bonds (a) SiC and CO, (b) SiO and CO
(c) two 2e—2e bonds and four 3e—2e bonds.
(c) SiC and CO (d) Si and CO,
(d) four 2e—2e bonds and four 3e—Ze bonds.
(A.LE.E.E. 2005) (Origed JEE 2010)

Answers
B17. (c) B18. (d) B19. (a) B20. (a) B21. (d) B22. (6) B23. (a) B24. (a) B25. (b) B26. (b)
B27. (b) B28. (a) B29. (d)_ B30. (a) B31. (b) B82. (c)
SOME p-BLOCK ELEMENTS

B33. Which of the following is not hydrolysed easily ? B44, For BCl,, AICI, and GaCl, the increasing order of ionic
(a) CCl, (6) SiCl, character is
(c) GeCl, (d) SnCl, (Orissa JEE 2010) (a) BCI, < AICI, < GaCl,
B34. Which of the following is non-existent ? (6) GaCl, < AICI, < BCI,
(a) AIF (b) CoF,” (c) BCl, < GaCl, < AICI,
(c) BF,” (d) SiF2-(AMU Engg. 2010) (d) AICL, < BCI, < GaCl, (W.B.JEE 2013)
B35. Which of the following has least tendency to undergo B45. In borax, the number of B — O — B links and B —- OH
catenation ? bonds present are respectively
(a) C (b) Si (a) five and four (b) four and five
(c) three and four (d) five and five
(c) Ge (dq) Sn (AMU Engg. 2010)
(W.BJEE 2013)
. The strongest Lewis acid among boron halides is B46. Silicone oil is obtained from the hydrolysis and
(a) BBr, (b) BCI, polymerization of
(c) BI, (d) BF, (AMU Engg. 2010) (a) trimethylechlorosilane and dimethyldichloro-
B37. Boron cannot form which one of the following silane
anions ? (b) trimethylchlorosilane and methyltrichlorosilane
(a) BFS (b) BH] (c) methyltrichlorosilane and dimethyldichloro-
(c) B(OH)Z (d) BOs (AIFEE 2011) silane
B38. In diborane (B,H,), there are (d) triethylchlorosilane and diethyldichlorosilane
(a) three 3c — 2e” bonds and three 2c — 2e~ bonds (W.B.JEE 2013)
(b) four 3c — 2e~ bonds and two 2c — 2e~ bonds B47. Among the elements B, Mg, Al and K, the correct order
of increasing metallic character is
(c) two 3c — 2e” bonds and four 2c — 2e— bonds
(a) Be Al<Me<K
(d) none of the above (AMU Engg. 2011)
(6b) B< Mg<Al<K
B39. The stable bivalency of Pb and trivalency of Bi is
(c) Mg<B<Al<K
(a) due to d contraction in Pb and Bi (dq) Mg<Al<B<K
(b) due to relativistic contraction of the 6s orbitals of (fe)
Ke Mg<Al<B (Kerala P.E.T. 2013)
Pb and Bi, leading to inert pair effect B48. The correct formula of borax is
(c) due to screening effect (a) Na,[B,O,(OH),]. 9 H,O
(d) due to attainment of noble configuration (b) Na,[B,O,(OH),].8H,O
(W.B.J.EE. 2012) (c) Na,[B,O,(OH),]. 7 H,O
B40. The correct order of decreasing Lewis acidity is (d) Na,[B,O,(OH),].6 H,O
(a) BF, > BCL, > BBr, > BI, (e) Na,[B,O,(OH).]. 5 H,O (Kerala PE.T. 2013)
(6) BI, > BCl, > BBr, > BF,
B49. The +1 oxidation state of thallium is more stable than
(c) BI, > BBr, > BCI, > BF,
its + 3 oxidation state because of
(d) BCl,>BF,>BBr,>BI, (J-A.C_.E.T 2012)
(a) its atomic size
B41. Boron is unable to form BF? because of
(6) its ionisation potential
(a) high electronegativity of boron
(c) imert pair effect
(b) high electronegativity of fluorine (d) diagonal relationship (JK. C.E.T. 2013)
(c) lack of d-orbitals in boron B50. For the properties mentioned, the correct trend for
(d) less difference in electronegativity between B the different species is in
and F. (J.K.C.E.T. 2012) (a) strength as Lewis acid — BCl, > AlCl, > GaCl,
B42. The most stable oxidation state exhibited by (6) inert pair effect — Al > Ga > In
thallium is (c) oxidising property — Al* > In* > Tl
(a) 0 (b) +1 (d) first ionization enthalpy — B > Al > Tl
(ec) +2 (d) +38 (J.KLC.E.T. 2012) (Karnataka CET 2014)
B43. In diborane, the number of electrons that account for B51. Which one of the following metallic oxides exhibit
bonding in the bridges is amphoteric nature ?
(a) six (a) BaOQ (6) Al,O,
(b) two
(c) Na,O (d) CaO
(ec) eight
(Karnataka CET 2017)
(d) four (W.B.JEE 2013)

Answer
B33. (a) B34. (c) B35. (d) B36. (c) B37. (a) B38. (c) B39. (6) B40. (c) B41. (c) B42. (b)
B43. (d) B44, (c) B45. (a) B46. (a) B47. (a) B48. (0) B49. (c) B50. (a) B51. (5)
Tie MODERN'S abc + OF CHEMISTRY-XI

B52. Addition of mineral acid to an aqueous solution of B59. Aluminium is usually found in +3 oxidation state. In
borax, the following compound is formed: contrast thallium exists in +1 and +3 oxidation states.
(a) boron hydride (b) pyroboric acid This is due to
(c) metaboric acid (d) orthoboric acid (a) lanthanoid contraction
(Karnataka CET 2017) (b) lattice effect
B53. (CH,),51Cl is used during polymerisation of (c) diagonal relationship
organosilicons because (d) inert pair effect (JEE Main 2019)
(a) the chain length of organosilicon polymers can be B60. The electronegativity of aluminium is similar to
controlled by adding (CH,), SiCl (a) Boron (b) Carbon
(b) (CH,), SiC] improves the quality and yield of the (c) Lithium (d) Berylhum
polymer. (JEE Main 2019)
(c) (CH,), SiC1 does not block the end terminal of B61. The element that shows greater ability to form
silicone polymer. pm — pr multiple bond is
(d) (CH,),5iC1l acts as a catalyst during (a) S1 (b) Ge
polymerisation. (Karnataka CET 2018) (c) Sn (d) C (JEE Main 2019)
B54. Al,(SO,), 1s used in the following but not B62. The number of 2-centre-2-electron and 3-centre-
2-electron bonds in B,H,, respectively, are
(a) as a coagulant in treating drinking water and
sewage (a) 2 and 4 (b) 2 and 1
(b) in plastic industry (c) 2 and 2 (d) 4 and 2
(c) as a mordant in dyeing (JEE Main 2019)
B63. The chloride that cannot get hydrolysed is
(d) in paper industry
(a) SiCl, (b) SnCl,
(e) both (c) and (d). (Kerala PET 2018)
(c) PbCl, (d) CCl,
B55. More than four bonds are made by how many elements
in carbon family? (JEE Main 2019)
(a) 1 (b) 2 JEE (Advance) for TT Entrance
(c) 3 (d) 4 B64. B(OH), + NaOH —— Na[B(OH),]
(e) 5 (Kerala PET 2017) To keep the reaction in the forward direction, which
B56. The main reason that SiCl, is easily hydrolysed as reagent should be used ?
compared to CCl, is that (a) cis-1, 2-diol (b) trans-1, 2-diol
(a) Si—C]l bond is weaker than C—Cl bond (c) borax (d) Na,HPO, (LT. 2006)
(6) SiC, can form hydrogen bonds B65. When CO, is passed through water, which of the
(c) SiCl, is covalent following species will be present in water ?
(d) Si can extend its coordination number beyond (a) H,CO,,CO,,CO,7°, HCO,-
four. (WB JEE 2018) (6) HCO,,CO,7-
B57. Which of the set of oxides are arranged in the proper fc) CO? ,CO,
order of basic, amphoteric, acidic? (d) H,CO,, CO, (1.1.7. 2006)
(a) SO,, P,O,, CO (6) BaO, Al,O,, SO, B66. A colourless aqueous solution on adding water and on
(c) CaO, Si0,, Al,O, _ (d) CO,, Al,O,, CO heating gave a white precipitate. This precipitate
(WB JEE 2018) when reacted with NH,Cl and NH,OH in excess
resulted in dissolution of some of the precipitate and
B58. When metal ‘M’ is treated with NaOH, a whiSe
a gelatinous precipitate is obtained. What is the
gelatinous precipitate “X’ 1s obtained, which 1s soluble
hydroxide formed in aqueous solution ?
in excess of NaOH. Compound ‘X’ when heated
(a) Zn(OH), (b) Mg(OH),
strongly gives an oxide which is used in (c) Al(OH), (d) Ca(OH), (LT: 2006)
chromatography as an adsorbent. The metal ‘M’ 1s: B67. The increasing order of atomic radii of the following
(a) Zn (b) Ca eroup 13 elements is
(c) Al (d) Fe (a) Al < Ga < In < Tl (6) Ga <Al<Ine« TI
(JEE Main 2018) (c) Al < In < Ga < TI (qd) Al<= Ga< Tl<In

Araswets (JEE Advance 2016)

"B52. (d) B53.


B52. (d) (a) B54. (b) BSS. (d) BS6. (d) BBS7. (b) BSS. () BS9. (d) B60. (d) B6l.(d)
B53. (a) B54, (b) B55. (d) B56. (d) B58. (ce) B59. (d)

B62. (d) B63. (d) B64. (a) B65. (a) B66. (c) . (b)
SOME p-BLOCK ELEMENTS

Competition File

C7. The correct statement(s) for orthoboric acid is/are


MULTIPLE CHOICE QUESTIONS (a) 1t behaves as a weak acid in water due to self
C with more than one correct answers lonization
C1. Boron trifluoride is (b) acidity of its aqueous solution increases upon
addition of ethylene glycol
(a) electron precise molecule
(c) it has a three dimensional structure due to
(b) electron deficient molecule
hydrogen bonding
(c) used as a rocket fuel
(d) it is a weak electrolyte in water.
(d) Lewis acid (JEE Advance 2014)
C2. Which of the following species are not known ? C8. The crystalline form of borax has
(a) [SiCl,]?- (b) [CF]? (a) tetranuclear [B,O,(OH),]?- unit
(c) [iF] (d) [PbC1,]?- (6) all boron atoms in the same plane
C3. In which of the following elements, the lower oxidation (ec) equal number of sp? and sp? hybridised boron atoms
state is more stable than the higher ? (d) one terminal hydroxide per boron atom
(a) In (6) Pb (JEE Advance 2016)
(c) Sn (d) Tl C9. Select the most appropriate statement. In BF,
C4, Which of the following are amphoteric oxides ? (a) all the bonds are completely ionic
(a) T1,0 (b) Al,O, (b) the B—F bond is partially ionic
(c) Ga,O, (d) B,O, (c) B—F bond has partial double bond character
C5. Which of the following statements are not (d) bond energy and bond length data indicate single
correct ? bond character of the B—F bond
(a) Unlike graphite, diamond is a good conductor of (e) all the bonds are covalent. (Kerala PET 2017)
electricity. C10. The option(s) with only amphoteric oxides is(are)
(6) Graphite can act as lubricant. (a) Cr,O,, BeO, SnO, SnO,
(c) Graphite is a black substance and possesses a (6) ZnO, Al,O,, PhO, PbO,
metallic lustre. (c) NO, B,O,, PbO, SnO,
(d) Cin diamond involves sp? hybridisation. (d) Cr,O,, CrO, SnO, PbO (JEE Advance 2017)
C6. Which of the following examples are correct for Cil. Among the following the correct statement(s) is(are)
silicates ? (a) Al(CH,), has the three-centre two-electron bonds
(a) Orthosilicates : zircon in its dimeric structure
(b) Chain silicates : thortveitite (b) BH, has the three-centre two-electron bonds in
(c) Pyrosilicates : hemimorphite its dimeric structure
(d) Ring silicate : forestrite (c) the Lewis acidity of BCl, is greater than that of
AICl,

Ainsven
(d) AICI, has the three-centre two-electron bonds in
its dimeric structure (JEE Advance 2017)

C1. (6,d) C2. (a,b) C3. (6, d) C4. (6,c) Cd. (a,d) C6. (a,c) CT. (6,d) C8. (a4,¢,d) C9. (ce) C10. GG, d)
Cll. (a, b, c)

MULTIPLE CHOICE QUESTIONS (b) Oxidation state of +4 only


D based on the given passage/comprehension (c) Oxidation state of both +4 and +2
(7) Oxidation state of +4 and +6
Passage-I
D2. The strongest oxidant among the following is
The elements of groups 13 and 14 exhibit two types of (a) GeO, (5) PbO, (c)SnO, (d) S10,
oxidation states which differ by 2 units. The heavier
D3. Which of the following statement is not correct ?
elements exhibit oxidation state which is two units lower
than group oxidation state. (a) PbCl, is less stable than SnCl, but PbCl, is more
stable than SnCl,
Answer the following questions :
(6) PbO, is stronger oxidant than SnO,
D1. Elements of group 14 exhibit
(a) Oxidation state of +2 only (c) (SiH),N is stronger base than (CH,),N.
(d) Sn(II) is a reducing agent but Pb(ID) is not.

(aaa
Passage-I D1. (c) D2. (6b) D8. (c)
Tico MODERN'S abc + OF CHEMISTRY-XI

D4. Which of the following is not correct ?


Answer the following questions :
(a) 3TI* (ag) ——> 2T]1 + TI**(aq)
D5. The precipitate A is because of
(b) 3In*(ag) ——> 2In(s) + In**(aq)
(a) AL,Cl, (b) A(OH), (c) AIC], (d) Al,O,
(c) 2Fe**+ + Sn?* ——-> 2Fe?*+ Sn**
D6. The complex B is
(d) Hg,** +Sn?* ——-+ 2Hg + Sn*
(a) ALO, (6) [AKOH),F (ce) AKOH), (d) Al(OH),
Passage-II
D7. Preicipitate A on heating gives
Aluminium reacts with sodium hydroxide to give white
precipitate (A) which dissolves in excess of NaOH to give (a) Al,O, (b) A(OH), (c) Al (d) Al,Cl,
a soluble complex (B).

Passage-I D4, (a) Passage-II D5. (5) . (b) D7. (a)

Assertion Reason Type Questions Assertion TIC] is more stable than TICI,.
Reason : + 1 oxidation state of heaviest element is
The questions given below consist of an Assertion and more stable than +3.
the Reason. Use the following key to choose the appropriate
Assertion : Carbon dioxide has linear geometry
answer.
(a) If both assertion and reason are CORRECT
involving sp hybridisation of C.
and reason is the correct explanation of the Reason : Dry ice is solid COQ,.
assertion. Assertion : Boron does not form BF. ion while
(b) If both assertion and reason are CORRECT, but AIF?" is known.
reason is NOT THE CORRECT explanation of the Reason : Bdoes not have d-orbitals in valence shell
assertion.
while Al has d-orbitals in valence shell.
(c) If assertion is CORRECT, but reason is
Assertion : Boric acid behaves as weak monobasic
INCORRECT.
acid.
(d) If assertion is INCORRECT, but reason is
CORRECT.
Reason : It has only one ionizable hydrogen.
(e) If both assertion and reason are INCORRECT. Assertion : Glass is not an example of silicates.

1. Assertion : Boron has low electrical conductivity Reason : All silicates have tetrahedral SiOf unit.
Reason : At ordinary temperature, boron behaves 10. Assertion : Graphite is good conductor of heat and
as metal. electricity.
. Assertion : Diborane forms two three centred electron Reason : Graphite has z-electrons which are mobile.
pair bonds. 11. Assertion : Silicons are water repelling in nature.
Reason : B,H, does not have sufficient number Reason : Silicons are organosilicon polymers, which
of electrons to form normal covalent have (—R,Si0—) as repeating unit.
bonds. 12. Assertion : If aluminium atoms replace a few silicon
. Assertion : Graphite is soft and a good lubricating atoms in three dimensional network of
agent.
silicon dioxide, the overall structure
Reason : The successive layers in graphite are held
acquires a negative charge.
together by weak forces of attraction.
Reason : Aluminium is trivalent while silicon is
. Assertion : Diamond is a bad conductor of electricity.
tetravalent.
Reason : All C-C bond lengths in diamond are of
154 pm.

1 te) 2. (a) S3.(a4) 4.(6) 5.(a) 6.(b) 7. (a) 8. (c) 9. (d) 10. (a) 11. (b) 12. (a)
SOME p-BLOCK ELEMENTS

[>> Multiple Choice Questions (Type-l)


Objective Questions
(2) MeSiCl, (b) Me,SiCl,
1. The element which exists in liquid state for a wide (c) Me,SiCl (d). Me,Si
range of temperature and can be used for measuring 9
. Ionisation enthalpy (AH, kJ mol~) for the elements of
high temperature is Group 13 follows the order :
(a) B (6b) Al
(a) B>Al>Ga>In>TIl (6) B<Al<Ga<In<TI
(c) Ga (d) In
(c) B<Al>Ga<In>Tl (qd) B> Al<Ga>n<Tl
Which of the following is a Lewis acid ?
10. In the structure of diborane
(a) AICI,
(6) MgCl, (a) All hydrogen atoms lie in one plane and boron
(ce) CaCl, atoms lie in a plane perpendicular to this plane.
(d) BaCl, (b) 2boron atoms and 4 terminal hydrogen atoms lie
The geometry of a complex species can be understood in the same plane and 2 bridging hydrogen atoms
from the knowledge of type of hybridisation of orbitals lie in the perpendicular plane.
of central atom. The hybridisation of orbitals of central (c) 4 bridging hydrogen atoms and boron atoms lie in
atom in [Be(OH),]- and the geometry of the complex one plane and two terminal hydrogen atoms lie in
are respectively a plane perpendicular to this plane.
(a) sp®, tetrahedral (b) sp, square planar (d) All the atoms are in the same plane.
(c) sp*d’, octahedral (d) dsp’, square planar 11. A compound X of boron reacts with NH, on heating to
Which of the following oxides 1s acidic in nature ? give another compound Y which is called inorganic
(a) B,O, (6) ALO, benzene. The compound X can be prepared by treating
(c) Ga,O, (d) In,O, BF, with lithium aluminium hydride. The compounds
The exhibition of highest co-ordination number X and Y are represented by the formulas.
depends on the availability of vacant orbitals in the
central atom. Which of the following elements 1s not
(a) BH, B.N,H, (b) B,O,, B,N,H,
(c) BF,, B,N,H, (d) B,N,H,, B,H,
likely to act as central atom in MFs~ ?
12. Quartz is extensively used as a piezoelectric material,
(a) B (b) Al
it contains ......... i
(c) Ga (d) In
(a) Pb (b) Si
Boric acid is an acid because its molecule
Co iai (d) Sn
(a) contains replaceable H* ion
13. The most commonly used reducing agent 1s
(b) gives up a proton
(a) AICI, (6) PbCl,
(ce) accepts OH from water releasing proton
(d) combines with proton from water molecule (c) SnCl, (d) SnCl,
Catenation, t.e., limking of similar atoms depends on 14, Dry ice is
size and electronic configuration of atoms. The (a) Solid NH, (b) Solid SO,
tendency of catenation in Group 14 elements follows (c) Solid CO, (d) Solid N,
the order :
15. Cement, the important building material is a mixture
(a) C>Si>Ge>S5n (b) C>>Si>Gee2Sn
of oxides of several elements. Besides calcium, iron and
(c) Si>C>Sn>Ge (dq) Ge>Sn>Si>C
sulphur, oxides of elements of which of the group (s) are
. Silicon has a strong tendency to form polymers like
silicones. The chain length of silicone polymer can be present in the mixture ?
controlled by adding (a) group 2 (6) groups 2, 13 and 14

Aes (c) groups 2 and 13 (d) groups 2 and 14

1. (c) 2. (a) 3. (a) 4, (a) 5. (a) 6. (c) 7. (b) 8. (c) 9. (dd) 10. (6)
1]. (&) 12. (b) 13. (d) 14. (c) 15. (b)
Siva MODERN'S abe + OF CHEMISTRY-AXI

D> Serer kee < ae (c) Graphite is thermodynamically most stable


allotrope of carbon
In the following questions two or more options may be (d) Graphite is slippery and hard and therefore used
correct. as a dry lubricant in machines
16. The reason for small radius of Ga compared to Al is 20. Which of the following statements are correct. Answer
on the basis of figure given below.
(a) poor screening effect of d and f orbitals
(b) increase in nuclear charge
(c) presence of higher orbitals
(d) higher atomic number
17. The linear shape of CO, is due to ..........
(a) sp* hybridisation of carbon
(b) sp hybridisation of carbon
(c) pm—prn bonding between carbon and oxygen
(a) The two bridged hydrogen atoms and the two
(d) sp? hybridisation of carbon
18. Me,SiCl is used during polymerisation of organo boron atoms lie in one plane
silicones because (b) Outofsix B—H bonds two bonds can be described
(a) the chain length of organo silicone polymers can in terms of 3 centre 2-electron bonds
be controlled by adding Me,SiCl (c) Out of six B—H bonds four B —H bonds can be
(b) Me,SiCl blocks the end terminal of silicone described in terms of 3 centre-2 electron bonds
polymer (dq) The fourterminal B—H bonds are two centre-two
(c) Me,SiCl improves the quality and yield of the electron regular bonds.
polymer
(d) Me,SiClacts as a catalyst during polymerisation 21. Identify the correct resonance structures of carbon
dioxide from the following :
19. Which of the following statements are correct ?
(a) Fullerenes have dangling bonds (a) O-C=O0
(6) Fullerenes are cage-like molecules (6) O=C=0
(c) “O=C-OQ

Araswets (d) -O-C=Q*

16. (a, 5) Li Be) 18. (a, b) 19. (b,c) 20. (a,b,d) 21. (b,d)

> LLU <a Column I


(i) Diborane (a) Used
metals
Column II
as a flux for soldering
In the following questions more than one
correlation is possible between options of Column (ii) Galluim (b) Crystalline form of silica
I and Column II. Make as many correlations as (iii) Borax (c) Banana bonds
you can. (iv) Aluminosilicate (d) Low melting, high boiling, useful
22. Match the species given in Column! with the properties for measuring high temperatures
mentioned in Column II. (v) Quartz (e) Used as catalyst in petrochemical
Column I Column IT
industries
24, Match the species given in Column I with the
(i) BFy (a) Oxidation state of central atom is +4
(iz) AICI, (b) Strong oxidising agent hybridisation given in Column II.
(iii) SnO (c) Lewis acid Column I Column IT
(iv) PbO, (7) Can be further oxidised (i) Boron in [B(OH),)- (a) sp*
(e) Tetrahedral shape
(ii) Aluminium in [Al(H,0),]** (b) sp”
23. Match the species given in Column I with properties
(iii) Boron in B,H, (c) sp*d*
civen in Column II.
(iv) Carbon in Buckminster fullerene

Arse
(v) Silicon in si0t
(vi) Germanium in [GeC1,]>-

22. (1) —(e); (it) — (e); (ut) — (d); (wv) — (a, b) 23. (i) —(c); (ii) —(d); (dit) — (a); (iv) — (e); (v) -— ()
24, (i) —(b); (it) — (ec); (it) — (6); (tv) — (a); (Vv) — (B); (vt) — Ce)

WW.JEEBOOKS.IN
SOME p-BLOCK ELEMENTS

(ec) Both Aand R are not correct.


[> Assertion and Reason Type Questions (d) Ais not correct but R is correct.
25. Assertion (A) : [If aluminium atoms replace a few
In the following questions a statement of
silicon atoms in three dimensional network of silicon
Assertion (A) followed by a statement of Reason
dioxide, the overall structure acquires a negative charge.
(R) is given. Choose the correct option out of the
choices given below each question. Reason (R) : Aluminium is trivalent while silicon is
(a) Both A and R are correct and R is the correct tetravalent.
explanation of A. 26. Assertion (A): Silicones are water repelling in nature.
(6) Both A and R are correct but R is not the correct Reason (R) : Silicones are organosilicon polymers,
explanation of A. which have (—R,SiO-—) as repeating unit.

ye
:

Sn (II) is easily oxidised to Sn (IV) and hence


A. meq with only one correct answer Sn (II) is reducing agent.
Al.(a): Na,B,O, +2H,0 —— H,B,O, + 2NaOH Bl17.(c) : Boron cannot form MF* because of the absence of
alkaline d-orbitals in its valence shell.
B18.d) : In group 13, atomic radii increases down the group
A2.(c): H,BO, 8 > HBO, —““> B,0, but Ga has smaller atomic radius than AI.
Boron trioxide Therefore, correct order is
A5. (6): Diborane (B,H,) contains two three centre two
B < Ga < Al<In < Tl
electron bonds (B—H-—B).
B22. (6): Aluminium chloride in aqueous solution exists
All.(c): B,H, + 6H,O 2H,BO, + 6H, as ion pair
——-+ acidic
AICI, + 6H,0 —— [AICL,(H,0),}* (aq) +
A15. (c): Ca,B,O,, + 2Na,CO, —““—> 2CaCO, + Na,B,O,
Colemanite Borax LAICL,(H,0),7
+ 2NaBO, The crystallisation of aqueous AICI, gives an ionic
solid [AICI,(H,O),]* , [AIC],(H,O),|1- which on
A19.(a) : Na,B,O, + 7H,O ——~ 2NaOH + 4H,BO,
further heating gives Al,Cl,,.
Strong base Weak acid
A20.(b) : Al,O, + 3C + 3ClL, ——> 2AlCl, + 3COT [AICI,(H,0),}* [AICL,(H,O),- —22*->
anhyd. Al, Cl, + 2H,O
B27.(b) : The stability of lower oxidation state increases
B. mcg from competitive examinations down the group as
B9c): When diborane is hydrolysed, it forms orthoboric SiX, < < GeX,< < SnX,< < PbX,
acid and hydrogen. B44.(c): Ionic character is inversely proportional to the
BH, + 6H,0-——> 2H 30, + 6H, polarising power of the cation. The polarising
B10.(d) : The basic structural unit of silicates is Si0,*. power decreases with increase in size of cation 1.e.,
B11.(a): It can only form dimer.
B*+ > Ga®* > Al*+, Therefore, BCl, will have the
least ionic character and AICI, will have the
B12.(6) : Diborane, B,H, has three centred two electron
maximum ionic character. The correct order is:
bond.
B14.(6) : PF, has a lone pair of electrons. Therefore, it is AICI, > GaCl, > BCI,
most likely to behave as Lewis base. B45.(a) : The structure of borax ion is
B16.(d) : Because of inability of ns? electrons of the valence OH
shell to participate in bonding on moving down
the group (inert pair effect).
p30.
Pb (II) is more stable than Pb (IV) and HO—B’ 0 B—OH
Sn (IV) is more stable than Sn (ID) 0—R0”%
Therefore, Pb (IV) is easily reduced to Pb (II) and
hence Pb (IV) is oxidising agent.
OH
B — O — B links = 5, B — OH links = 4

WWW.JEEBOOKS.IN
MODERN’S abc + OF CHEMISTRY-XI

B47.(a): Metallic character increases down the group and


decreases along a period. C. mcq with more than one correct answer
B < Al, Mg < K, Mg > Al C7.(b, d). Orthoboric acid (H,BO,) behaves as a weak mono
The correct order B < Al < Mg < K basic acid (Lewis acid). It accepts a pair of lone
pair from OH ion.
B50.(a): Lewis acid strength decreases down the group.
B52.(d) : Orthoboric acid is formed when an aqueous
H—OH + B(OH), —> [B(OH),. + H
solution of borax is treated with mineral acids; On adding ethylene glycol, its acidity increases.
hydrochloric acid or sulphuric acid. C11, (a, 8, c).
Na,B 4 0, + 2HCI + 5H,O —} 4H.BO, + 2NaCl (a) Al(CH,). has two 3-centre 2-electron bonds in its
Na,B,O. + H,SO, + 5H,0 ——> 4H,BO, + Na,SO, Al,(CH,), structure.
(b) BH, has two 3-centre 2-electron bonds in its B,H,
B54.(b) : Aluminium sulphate is mainly used as a structure.
coagulating agent in purification of drinking water BCI, is stronger Lewis acid because of small size
(c)
and in waste water treatment plants, as mordant
of B.
in dyeing and printing textile and also used in
calico printing and in paper industry. (d) Al,Cl, does not have 3-centre 2-electron bonds.

B56.(d) : Silicon has vacant d-orbitals in its valence shell NCERT Exemplar Problems: MCQs Type I
while carbon has only s- and p-orbitals. Therefore,
carbon cannot increase its coordination number 2. (a) AICI, is a Lewis acid because it is electron deficient.
beyond 4 while silicon can. Therefore, SiCl, can be 5. (a) Boron cannot form BF,* because it has no outer
readily hydrolysed by water by extending its octet. d-orbitals and hence cannot extend the octet.
B57.(b) : BaO (basic), Al,O,(amphoteric), SO,(acidic) —.
6. (c) B(OH), +H—OH —> [B(OH),F + Ht
B58.(c) : Al + 3H,O —“2#_, Al(OH), Boric acid
White gelagnous
ppt or B(OH), +2H,O — > [B(OH),- + H,O+
X is soluble in excess NaOH i. Do Catenation decreases rapidly as the strength of
element-element bond decreases due to increasing
size down the group i.e., C >> Si > Ge = Sn.
Sodium meta aluminate
8. (ce) Chain of silicon polymer can be controlled by adding

2AKOH)s eating
Sren ; Al,O.
(CH,), SiCl.

Al,O, is used in column chromatography.


11. (a) BH, + 6NH, ——>
450K
2B,N,H
,N,H, + 12H,
Diborane (X) Inorganic benzene
B60.(d) : Al has similar electronegativity to Be (diagonal
relationship) 4BF, + 3LiAIH, — > 2B,H, + 3Lik + 3AlF,
B62.(d) : In B,H,, there are four 2-centre-2-electron (B—H) 12. (5b) Quartz is made up of silicon.
bonds and two 3-centre-2-electron (B—H-—B) bonds. 18. (d) SnCl, can be easily oxidised to Sn** because +4
B63.(d) : CCl, cannot get hydrolysed due to the absence of oxidation state of Sn is more stable than +2
d-orbitals in the valence shell of carbon. oxidation state.
14. (c) Solid CQ, is known as dry ice.
HO \. Oo H OF 15. (5b): Cement contains elements of group 2 (Ca), group

7 ® CH,—OH _
B64.(a) : 13 (Al) and group 14 (Si).
HO OH
NCERT Exemplar Problems: MCQs Type II
CH,O2 OCH, 2 |. 16. (a,b) Poor shielding of d-electron increases nuclear
charge.
Np% + 2H,0 17. (5, c) The linear shape of CQO, is due to sp hybridisation
CH,O~ OCH, of carbon and px—pt bonding between carbon and
oxygen.
Due to the formation of Lewis acid-base complex 18. (a, b) Me, SiCl is used to control the length of silicone
ion, the equilibrium shifts in the forward direction. polymers by blocking the end terminal of the
silicone polymer.
B65.(a) : H,O + CO, 2
———> H,CO, ——= Ht + HCO,- 21. (b, d) CO, is a resonance hybrid of the following
structures :
HCO,3== H* + CO,”
0=C=0602=C-O
6 0-Cz=0°
B66.(c) : Al(OH), forms gelatinous mass.
NCERT Exemplar Problems:
B67.(5) : The atomic radius of Ga is less than Al due to the
Assertion-Reason Type
presence of d-orbitals in gallium. d-orbitals have
a poor shielding effect as a result of which size 26. (5b): Correct explanation : Silicon atoms in silicones
decreases. are surrounded by non-polar alkyl groups which
repel water molecules.
SOME p-BLOCK ELEMENTS 11/65 fF

jor Examination
Time Allowed: 1 Hr. Maximum Marks: 25

1. Which out of SiCl, and CCl, gets easily hydrolysed?

2. PbCl, is more stable than PbCl,. Why?

3 . What is maximum covalency of silicon in its compounds?


4 . Write resonating structures of CO,.
5. Is boric acid a protic acid?
6. Suggest a reason why the B—-F bond length in BF, (130 pm) and BF, (143 pm) differ.
7. Why does BF, behave as Lewis acid?
8. How would you explain the lower atomic radius of gallium as compared to aluminium ?
9, Explain
(¢) SiF,? is known but CF,*- is not formed.
(ii) (CH,),N is basic but (CF,),N is not basic.
(iii) Although diamond is covalent but it has very high melting point.
10. When metal X is treated with sodium hydroxide, a white precipitate (A) 1s obtained, which is
soluble in excess of NaOH to give soluble complex (B). Compound (A) is soluble in dilute HC] to
form compound (C). The compound (A) when heated strongly gives (D), which is used to extract
metal. Identify (X), (A), (B), (C) and (D). Write suitable equations to support their identities.
11. Complete the following reactions :
(i) BCL, + H,O —>
(ii) SiF, + HF —>
(iii) ZnO + CO—>
(iv) Al,O,. 2H,O + NaOH + H5O —>
(v) Pb + NaOH + H,O —>
(vi) NaH + B,H, —>
12. (a) Account for the fact that aluminium chloride exists as dimer.
(b) Give different products formed when diborane reacts with ammonia.
(c) What are silicones? Explain.

To check your performance, see HINTS and SOLUTIONS to some questions at the end of
Part II of the book.
WWW.JEEBOOKS.IN
0) 575) 1 Os WANA iw
Building on..... Assessing..... Preparing for Competition.....
@ Understanding Text 1 Quick Memory Test with # Topicwise MCQs 50
@ Conceptual Questions 37 Answers 44 Competitive Examination Qs
| ® HOTS & Advanced Level => AIPMT & Other State Boards’
QUICK CHAPTER ROUND UP 40 Questions with Answers A5 Medical Entrance 51
NCERT FILE REVISION EXERCISES 47 => JEE (Main) & Other State
¢ In-text Qs & Exercises with @ Hints & Answers for Revision Boards’ Engineering Entrance 51
Solutions 41 Exercises 49 > JEE (Advance) for ITT
¢ NCERT Exemplar Problems with Entrance 53
Answers & Solutions (Subjective) UNIT PRACTICE TEST 61 ® NCERT Exemplar Problems
43 (Objective Questions) 56
Hints & Explanations for
Difficult Questions 57

reactive element and therefore it does not occur


GROUP 15 ELEMENTS in free state in nature. All its known minerals are
orthophosphates. Major amounts of phosphorus
Group 15 of the periodic table contains five
occur 1n a single mineral family known as apatites,
elements namely nitrogen (N), phosphorus (P), arsenic
(As), antimony (Sb) and bismuth (Bi). This group is which have the general formula, 3Ca,(PO,), CaX, or
regarded as nitrogen family. Collectively, the group Ca,(PO,),.CaX, or Ca,,(PO,).X, where X = F, Cl or
15 elements are also called pnicogens and their OH. The common minerals of phosphorus are:
compounds as pniconides. The name is derived from (t) Phosphorite Ca,(PO,),
the Greek word pnicomigs meaning suffocation. (it) Fluorapatite Ca,(PO,).F
OCCURRENCE or 3Ca,(PO,),.CaF,
The elements of group 15, except phosphorus do not (tit) Chlorapatite Ca,(PO,),Cl
occur very abundantly in nature. Though molecular or 3Ca,(PO,),.CaCl,
nitrogen comprises about 78% by volume of the earth’s (tv) Hydroxyapatite Ca,(PO,),0H
atmosphere, it is not very abundant in the earth’s
or 3Ca,(PO,),.Ca(OH),
crust. It is only thirty third most abundant element
by weight in the earth's crust. Since nitrates are very Large deposits of phosphate rocks are located
soluble in water so these are not widespread in the in North Africa and North America. In India, the
earth’s crust. The only major nitrate minerals are phosphate rocks are mainly found in Rajasthan.
NaNO, (sodanitre, Chile saltpetre) and KNO, (nitre, Phosphorus is essential for life, as a structural
Indian saltpetre). The major deposits of saltpetre material in both animals and plants. It is present in
(KNO,) occur in India. Nitrogen is also an important bones as well as in living cells. About 60% bones and
constituent of proteins, amino acids and nucleic acids teeth are Ca,(PO,), or [8{Ca,(PO,),}.CaF,]. It also
occurs in nucleic acids (DNA and RNA) which control
in plants and animals. The continuous interchange
the hereditary effects in human beings. Phosphorus is
of nitrogen between the atmosphere and biosphere is
also found in ATP (adenosine triphosphate) and ADP
called nitrogen cycle. (adenosine diphosphate) which are of vital importance
Phosphorus is the eleventh element in order for production of energy in the cells. Phospho proteins
of abundance in crustal rocks of the earth. It is very are present in eggs and milk.

aan
|
WWW.JEEBOOKS.IN
saz The elements arsenic, antimony and bismuth
MODERN'’S abc + OF CHEMISTRY-XI

Table 1. Electronic configuration of group 15


are not very abundant. Their important sources elements.
are sulphides occurring as traces in other ores e.g.,
Element Symbol Atomic Electronic
arsenopyrites, FeAss; stibnite, Sb,S,, bismuth glance,
No. configuration
B1,53. These are obtained as metallurgical by-products
from roasting sulphide ores. Nitrogen [He] 2872p?
Phosphorus [Ne] 3873p?
GENERAL CHARACTERISTICS OF
Arsenic [Ar] 3d14s*4p?
GROUP 15 ELEMENTS
Antimony [Kr] 4d 582 5p?
A. Electronic Configurations
Bismuth [Xe] 4/14 5d2° 6s? 6p?
The atoms of group 15 have five electrons in the
outermost shell, two in s and three in p subshell. The B. Atomic and Physical Properties
general electronic configuration of this group may be The important atomic and physical properties of
expressed as ns*np?*. croup 15 elements are given in Table 2.

Table 2. Some important atomic and physical properties of group 15 elements.

Atomic number 15 33 ol 83
Atomic mass (g mol) 30.97 14.92 121.75 208.96
Atomic radius covalent (pm) 110 121 141 148
Ionic radius (pm) 212 cage 76 103
(EB ) (As?) (Sb?*) (Bi?*)
Ionisation enthalpy 1012 947 834 703
(kJ mol) 1903 1798 1595 1610
2910 2736 2443 2466
Electronegativity 3.0 P| 2.0 Lh es
m.p. (K) 63 317.1 1089 904 944.4
(Molecular) (White P) (Grey o-form at 38.6 atm)
b-p. (K) “4 DD3.0 885 1860 18357
(Molecular) (White P) (Sublimation temp.)
Density (g em-?) at 298 K 0.879 1.823 5.778 6.697 9.808
(at 63 K) (Grey o-form)

The important trends of atomic and physical 2. Ionisation enthalpies. The first ionisation
properties are discussed below: enthalpies of the group15 elements are higher than
1. Atomic and ionic radii. The atomic and ionic the corresponding members of the group 14 elements.
radi of group 15 elements are smaller than the atomic On going down the group, the ionisation enthalpies
radi of the corresponding group 14 elements. On going decrease.
down the group, the atomic radii increase with increase Explanation. The larger ionisation enthalpy is
in atomic number. due to greater nuclearcharge, small size and stable
Explanation. The nuclear charge in case of elements configuration of the atoms of group 15 elements. The
of group 15is larger than in the case of elements of group electronic configuration of atoms of group 15 are half
14. Due to increased nuclear charge, the electrons are filled, np,7, np,7, np,’ and are stable. Therefore, they
strongly attracted by the nucleus and therefore, atomic have high ionisation enthalpies.
radii decrease. Thus, the atomic radii of elements of
The decrease in ionisation enthalpy, as we move
eroup 15 are less than those of group 14.
down the group, is due to increase in atomic size and
On moving down the group, the atomic radii
screening effect which overweigh the effect ofincreased
increase due to increase in number of shells because
nuclear charge.
of addition of a new principal shell in each succeeding
As expected, the successive ionisation enthalpies of
element. However, from As to Bi, only a small increase
these elements (Table 2) increase in the order :
in covalent radius is observed. This is due to the
AH, < AH, < AH,
presence of completely filled d- and or f-orbitals in
the heavier members.
p-BLOCK ELEMENTS

3. Electronegativity. The electronegativity


naa)
Hydrazoic acid, HN, tN — whe
values of elements of group 15 are higher than the ss 7 ss

corresponding elements of group 14. On going down Azide ion, Na IN=N=WN:


the group, the electronegativity value decreases. Among the elements of group 15, phosphorus
has the maximum tendency for catenation
Explanation. The elements of group 15 have
forming cyclic as well as open chain
smaller size and greater nuclear charge of atoms and
compounds consisting of many phosphorus
therefore, they have higher electronegativity values.
atoms.
The decrease in electronegativity on going down
Diphosphine (P,H,) has two phosphorus atoms
the group is due to increase in size of the atoms and bonded together.
shielding effect of inner electron shells on going down The lesser tendency of elements of group 15 to
the group. However, amongst the heavier elements, show catenation in comparison to carbon is their low
the difference is not that much pronounced. (M—M) bond dissociation enthalpies.
4. Metallic character. The elements of group
15 are less metallic. However, on going down the Bond C—C N—N P—P As—As
sroup, the metallic character increases from N to Bi. Bond energy 353.3 163.8 201.6 147.4
For example, N and P are non-metallic, As and Sb (kJ mol)
are partly non-metallic while Bi is a metal.
N P As Sb Bi REMEMBER
ell

Non-metals Metalloids The element having maximum tendency for catenation


in the periodic table is carbon.
Metallic character increases Among the elements of group 15, P has maximum
Explanation. Due to increased nuclear charge tendency for catenation.
and higher electronegativity, the elements of group
15 are less metallic than the corresponding elements 7. Allotropy. Except nitrogen and bismuth, all
of group 14. On moving down the group, the atomic other elements of this group show allotropy. For
size as well as the screening effect of the intervening example,
electrons increases. As a result, the ionization enthalpy phosphorus exists as white, black or red
decreases and therefore, metallic character increases phosphorus
down the group. arsenic exists as yellow or grey arsenic
5. Melting and boiling points. The melting antimony exists as yellow or silvery grey
points of group 15 elements first increase from allotropic forms.
nitrogen to arsenic and then decrease to antimony and
C. Oxidation States
bismuth. However, the botling points increase regularly
The elements of group 15 have five electrons in
on going from nitrogen to bismuth.
their valence shell. They exhibit various oxidation
Explanation. The melting points increase down
states from —3 to +5 as discussed below :
the group due to increase in atomic size. However,
(1) Negative oxidation states. These elements
the unexpected decrease in the melting points of Sb
have five electrons in the valence shell (ns*np?) and
and Bi is because of their tendency to form three
therefore, require three more electrons to acquire
covalent bonds instead of five covalent bonds, due to the nearest noble gas configuration. But, the gain
inert pair effect. As a result, the attraction among of three electrons is not energetically favourable
their atoms is weak and hence their melting points because it requires very large amount of energy to
are low. Because of large size of atoms, Bi has still gain three electrons and form M®* ions. However,
weaker interatomic forces than Sb and therefore, has nitrogen being the smallest and most electronegative
still lower melting point. element of the group forms N* (nitride) ion and shows
The boiling points increase down the group from an oxidation state of — 3 in nitrides of some highly
N to Bi because of increase in their atomic size. electropositive metals such as Mg,N,, Ca,N,, etc. The
6. Catenation. The elements of group 15 also other elements of this group form covalent compounds
show a tendency to form bonds with itself (self linking even with metals and show an oxidation state of —3
of atoms) known as ecatenation. All these elements with metals. For example, calctum phosphide (Ca,P,),
show this property but to a much smaller extent than sodium arsenide (Na,As), zinc antimonide (Zn,Sb,),
carbon. For example, hydrazine (H,NNH,) has two magnesium bismuthide (Mg,Bi,). As is evident from the
N atoms bonded together, hydrazoic acid (N,H) has decreasing electronegativities and decreasing ionisation
three N-atoms, azide ion, N, has also three N atoms enthalpies, the tendency of the elements to exhibit
bonded together —3 oxidation state decreases on moving down from
P to Bi due to increase in size and metallic character.
eS. MODERN'’S abc + OF CHEMISTRY-XI

In fact, the last member of the group, Bi hardly forms Nitrogen does not form compounds in +5
any compound in —3 oxidation state. oxidation state such as NF., NCl., etc., because it
In addition to —3 oxidation state, N and P show does not have vacant d-orbitals in its valence shell
oxidation states of —2 in hydrazine (NH,NH,) and which can enable it to extend its octet. The stability
diphosphine (P,H,) respectively. Nitrogen also shows of the highest oxidation state (+5) decreases down the
an oxidation state of —1 in hydroxyl amine (NH,OH) croup. For example, the +5 oxidation state of Biis less
but P does not. stable than that of Sb. This is due to inert pair effect.
(11) Positive oxidation states. All the elements The only well characterised Bi(V) compound is BiF,.
of group 15 also exhibit positive oxidation states of The elements of group 15 form both covalent
+3 and +5. However, on moving down the group, (e.g., NCl,, PCl,; AsCl,, SbCl,) and ionic compounds
the stability of +5 oxidation state decreases (e.g., BiF,, SbF, etc.) in +3 oxidation state. The +3
while that of +3 oxidation state increases due oxidation state becomes more and more stable on
to inert pair effect. moving down the group.
Because of energy considerations, these elements Nitrogen and phosphorus also show oxidation
cannot lose all the five valence electrons. Therefore, state of +4 because of the ability of one lone pair on
they do not form M** ions and all the compounds of NH, and PH, to form dative bonds with Lewis acids.
group 15 elements having +5 oxidation state (i.e., However, nitrogen can exist in various oxidation states
PF., PCl., SbF, BiF., etc.) are essentially covalent from —3 to +5 in its hydrides, oxides and oxoacids as
compounds. shown below :

Compound NH, J lar NH,OH lols NG NO NO; N,O, N,O,


Ammonia Hydrazine Hydroxyl Nitrogen Nitrous Nitric Nitrogen Nitrogen Nitrogen
amine oxide oxide trioxide tetraoxide pentaoxide
Oxidation state —-3 —2 -1 0 +1 +2 +3 +4 +5

Phosphorus also exhibits +1 and +4 oxidation REMEMBER


states in some oxoacids.
e In the last element of a group, after the removal of
(111) Disproportionation: In case of nitrogen, all
electrons from p-orbitals, the remaining ns? (e.g. 6s")
oxidation states from +1 to +4 tend to disproportionate electrons behave like stable noble gas and donot take
in acid solution. For example, part in compound formation. This reluctance of the
+3 + +2
3HNO,.——> HNO, + 2NO + H,0 s-electron pair to take part in chemical combination 1s
called inert pair effect.
Nitrous acid Nitric acid
Nitric oxide NCI, and NF, donot exist.
Similarly, in case of P nearly all the intermediate Nitrogen does not form compounds in +5 oxidation
state such as NCl,, NF. , etc. because of the absence of
oxidation states disproportionate into +5 and —3 both
d-orbitals in its valence shell.
in acid and alkali solutions. For example,
+3 +5 —3
CHEMICAL PROPERTIES AND TRENDS IN
4H,PO, ———— 3H,PO, + PH,
Phosphorous Phosphoric Phosphine CHEMICAL REACTIVITY
acid acid Like first member of other groups, nitrogen differs
However, +3 oxidation state in As, Sb and from rest of members of the group due to its (7) smaller
Bi becomes increasingly stable with respect to size, (i1) high electronegativity, (711) high ionisation
disproportionation because of inert pair effect. enthalpy and (iv) non-availability of d-orbitals in the
Therefore, their tendency to undergo disproportionation valence shell.
decreases. Nitrogen has unique tendency to form
(jv) Maximum covalency : The maximum pt-pt multiple bonds with itself and with
covalency of nitrogen is restricted to four because
other elements having small size and high
it does not have vacant d-orbitals in its outermost
electronegativity (e.g., C and O). However, the
valence shell (n = 2). Therefore, only four (one 2s
heavier elements of this group do not form pa—pt
and three 2p) orbitals are available for bonding and
it cannot extend its valency beyond four (e.g., (NH,*, bonds because their atomic orbitals are so large and
CH,),N*, etc.). That is the reason why nitrogen does diffused that they cannot have effective overlapping.
not form pentahalides such as NF, or NCl,. On the These differences between nitrogen and other members
other hand, phosphorus and all other elements have of the group result into anomalous properties of
vacant d-orbitals in their valence shells and can use nitrogen as explained below :
all their valence orbitals to exhibit covalency of five e Nitrogen is acolourless gas and exists as a diatomic
or six e.g., PF., PCl., AsF., PF, , [SbF,]-, etc. molecule. The two nitrogen atoms are held together
p-BLOCK ELEMENTS

by a triple bond (one o and two m) between two atoms


mAs)
Another factor which affects the chemistry of
and have very high bond enthalpy (941.4 kJ mol"). nitrogen is the absence of d-orbitals in its valence
-N i: N: or > N==N: shell. Nitrogen cannot form pxr—dr bonds
whereas phosphorus and other heavier members of
Nitrogen molecule is inert and has very low reactivity
this group readily form pt—dr multiple bonds. For
because it has triple bond and therefore, has very high
bond enthalpy (941.4 kJ mol). example, phosphorus forms compounds containing
pm—dr bonds such as R,P = O, R,P = CH, (R=
Due to the presence of triple bond, which has alkyl group), POX, (X= F, Cl, Br), etc. Phosphorus
very high bond enthalpy, the nitrogen molecule and arsenic can also form da—dzm bonds with
is inert and has very low reactivity. However, the transition metals where their compounds like
tendency to form multiple bonds (pm—p7) is limited only P(C,H-.), and As(C,H-), act as ligands. Recently, a
to nitrogen. On the other hand, phosphorus, arsenic and few stable compounds of P, As and Sb containing
antimony exist in various forms containing single bonds pt—pnt bonds (e.g., P= C, P= N, P=P, As=As,
as P—P, As—As and Sb—sb. For example, phosphorus etc.) have been synthesized.
exists as tetrahedral P, molecules as shown in Fig. 1.
Let us discuss some trends in chemical reactivity
of group 15 elements and their compounds.
1. Reactivity towards hydrogen (formation
of hydrides)
All the elements of group 15 form gaseous
trihydrides of the formula EH, (where E = N, P, As,
Sb or Bi) such as :
NH, PH, AsH 3 »SbH, Bil,
Ammonia Phosphine Arsine Stibine Bismuthine
The lighter elements also form hydrides of
Fig. 1. Structure of P, molecule. the formula M,H, such as N,H, (hydrazine), P,H,
(diphosphine) and As,H, (diarsine). Nitrogen also
In this case, four P atoms lie at the corners of a forms a special hydride of the formula HN., known
regular tetrahedron. Each P is bonded to three P atoms as hydrogen azide or hydrazoic acid.
by single P—P bonds. However, the single N—N bond
is weaker than the single P—P bond because of high Nitrogen forms a special hydride:
interelectronic repulsions of non bonding electrons HN,: hydrogen azide or hydrazoic acid
due to small bond length (109 pm). As a result, the
Preparation. (i) By hydrolysis of binary metal
catenation tendency is weaker in nitrogen. Therefore,
compounds. The trihydrides can be easily obtained
nitrogen exists as gas while phosphorus exists
by the hydrolysis of their binary metal compounds
as solid.
with water or dilute acids :
@ Since P—P single bond is much weaker than
N==N triple bond, therefore, phosphorus is much more Mg.N, + 6H,O —> 3Mg(OH), + 2NH,
reactive than nitrogen. For example, white phosphorus Magnesium Ammonia
nitride
is more reactive than nitrogen. It catches fire when
exposed to air and burns to form the oxide, P,O,,,. It
Ca,N, + 6H,O —> 3Ca(OH), + 2NH,
is stored under water to prevent it from burning. Red Calcium
nitride
P is stable in air at room temperature but reacts on
heating.
Ca,P, + 6H,O —> 3Ca(OH), + 2PH,
Calcium Phosphine
The reactivity of other elements decreases phosphide
from As to Bi. Arsenic and antimony both occur in
two forms. The most reactive is yellow form which
Zn,As, + 6HCl —+ 3ZnCl, + 2AsH,
Zine arsenide Arsine
contains M, tetrahedral units and resembles white
phosphorus. Arsenic is stable in dry air but tarnishes
Mg,Bi, + 6HCl —> 3MgCl, + 2BiH,
Magnesium Bismuthine
in moist air giving first a bronze and then a black bismuthide
coating on its surface.
Antimony is less reactive and is stable towards (11) By reduction of trichlorides. The trichlorides
water and air at room temperature. On heating in of these elements except that of bismuth give the corre-
air, it forms Sb,O,, sponding hydrides on reduction with Zn/acid or LiAlH,,.
iat el Sb,O, or Sb,O,,.
4™~10° Bismuth forms
Bi,0, on heating. ECl, + 3LiAIH, ——~ EH, + 3LiCl + 3AlH,
(EK = N, P, As, Sb)
iat Structure. All these hydrides are covalent in
MODERN'’S abc + OF CHEMISTRY-XI

basic; PH, is weakly basic; ASH,, SbH, and BiH, are


nature and have pyramidal structure. These involve very weakly basic.
sp* hybridization of the central atom and one of the Explanation. Nitrogen atom has the smallest
tetrahedral position is occupied by a lone pair. The size among the hydrides. Therefore, the lone pair is
structure of NH, molecule is shown in Fig. 2. concentrated on a small region and electron density on
itis the maximum. Consequently, its electron releasing
tendency is maximum. As the size of the central atom
Lone pair increases down the family, the electron density also
decreases. As a result, the electron donor capacity or
N
the basic strength decreases down the group.
(it) Thermal stability. Thermal stability of the
| H hydrides of group 15 elements decreases as we go down
H H
the group. Therefore, NH, is most stable and BiH, is
Fig. 2. Shape of NH, molecule. least stable. The stability of the hydrides of group 15
elements decreases in the order :
Due to the presence of lone pair, the bond angle NH, >/HEL@AeH¢ > SbH, > BiH,
in NH, is less than the normal tetrahedral angle. It Explanation. This is due to the fact that on
has been found to be 107.8°. As we go down the group going down the group, the size of the central atom
the bond angle decreases as: increases and therefore, its tendency to form stable
covalent bond with small hydrogen atom decreases.
NH, PH : AsH, SbH 3 BiH : As a result the M—H bond strength decreases and
107.8° 93.6° 91.8° 91.3" 90° therefore thermal stability decreases.
(111) Reducing character. The reducing character
Explanation. In all these hydrides, the
of the hydrides of group 15 elements increases from
central atom is surrounded by four electron
NH, to BiH. Thus, increasing order of reducing
pairs, three bond pairs and one lone pair. Now,
as we move down the group from N to Bi, the character is as follows :
size of the atom goes on increasing and its NH, < PH, < AsH, < SbH, < BiH,
electronegativity decreases. Consequently, the Explanation. The reducing character depends
position of bond pair shifts more and more away upon the stability ofthe hydride. The lesser the stability
from the central atom in moving from NH, to BiH. of a hydride, the greater is its reducing character.
For example, the bond pair in NH, is close to N in Since the stability of group 15 hydrides decreases from
N—H bond than the bond pair in P—H bond in NH, to BiH, hence the reducing character increases.
PH,. As a result, the force of repulsion between the For example, NH, being most stable among the group
bonded pair of electrons in NH, is more than in PH. 15 hydrides is not a good reducing agent. The other
In general, the force of repulsion between bonded hydrides being unstable act as good reducing agents.
pairs of electrons decreases as we move from NH, However, it may be noted that ammonia at high
to BiH, and therefore, the bond angle also decreases temperatures reduces copper oxide to copper :
in the same order.
3CuO + 2NH, —*5 3cu +N, + 3H,O
(iv) Boiling and melting points. Ammonia
has a higher boiling point than phosphine and then
the boiling point increases down the group because of
increase in size.(Fig. 4).

More repulsion Less repulsion NH, (238.5)


e
—,
-
(bond angle is large) (bond angle is less.) —_
_
£
Fig. 3. Repulsions between electron pairs in NH, o

and PH,.
oS
ch
£

Characteristics of hydrides. The important


characteristics of these hydrides are : PH, (185.5)
(7) Basic strength. All these hydrides have one
lone pair of electrons on their central atom. Therefore,
they act as Lewis bases. They can donate an electron Molecular mass ——~*
pair to electron deficient species (Lewis acids). As we
Fig. 4. Plot of boiling points of the
go down the group, the basic character of these
hydrides of group 15 elements.
hydrides decreases. For example, NH, is distinctly
p-BLOCK ELEMENTS
AT)
Molecule NH, PH, AsH, SbH, BiH, (v) Solubility. Ammonia forms hydrogen bonding
with water molecules while phosphine and other
Boiling point (K) 238.5 185.5 210.6 254.6 290 hydrides do not form hydrogen bonding with water.
Melting point (K) 195.2 139.5 156.7 185.0 — Therefore, NH, is soluble in water while PH, and
other hydrides are insoluble in water.
Similar behaviour is observed for melting points.
The trends in the properties of hydrides of group
Explanation. The abnormally high boiling point
15 are given in Table 3.
of ammonia is due to its tendency to form hydrogen
bonds. The electronegativity of N (3.0) is much higher Table 3. Properties of hydrides of group 15
than that of H (2.1). As a result N—H bonds are polar elements.
and therefore, NH. undergoes intermolecular hydrogen
bonding as shown below:
Melting point (K) 195.2 139.5 _
Boiling point (K) 238.5 185.5 204.6 290
(E — H) distance (pm) 101.7 141.9 170.7 -
HEH angle (°) 107.8 93.6 913 -
A,H® (kJ mol —46.1 13.4 145.1 278
A,,,.41°(E-H) (kJ mol!) 389 322 255

2. Reactivity towards oxygen (formation of


In PH, and other Hydrides of Group 15 oxides)
NH,
hydrides, the intermolecular The elements of group 15 combine with oxygen
forces are van der Waals PH, directly or indirectly to form two types of oxides, E,O,
forces. These van der (trioxides) and KO. (pentaoxides). However, nitrogen
Waals forces increase with AsH, forms a number of oxides with oxidation states ranging
increase in molecular size from +1 to +5 which have no analogues of P, As, Sb
and therefore, boiling points SbH, or Bi. This is mainly because of strong tendency of
increase on moving from PH, nitrogen to form pa—pt multiple bonds with oxygen.
BiH,
to BiH. decreases
character Thermal
Basic decreases
stability The important oxides of group 15 elements along with
increases
character
Reducing
their oxidation states are listed in Table 4.

Table 4. Oxides of group 15 elements in different oxidation states.

Element ———> N
Oxidation state

+1 N,O
Nitrous oxide
+2 NO
Nitric oxide
+3 N,O . Een As,O, Sb,0, Bi,O,
Dinitrogen Phosphorus Arsenic Antimony Bismuth
trioxide trioxide trioxide trioxide trioxide
+4 N,O a ee
Dinitrogen Phosphorus
tetraoxide tetraoxide
+5 NO e Eoin As,O; Sb,0, Bi,O,
Dinitrogen Phosphorus Arsenic Antimony Bismuth
pentoxide pentoxide pentoxide pentoxide pentoxide
sia MODERN'’S abc + OF CHEMISTRY-XI

halogen atoms. The fourth sp? hybrid orbital contains


There is an important difference between the oxides
of nitrogen and other group congeners in their the lone pair of electrons. Therefore, the geometry of
structures. The nitrogen has ability to form trihalides may be regarded as pyramidal.
pz-pa multiple bonds and this is present in ns np
its structures.
On the other hand, the reluctance of P, As, Sb and
E ——=
Bi to form pa—pr multiple bonds leads to the cage sp® hybridisation
structures for their oxides.
Q- Lone pair
The oxides in the higher oxidation state of the elements
x = halogen atom

LS
are more acidic than that of the oxides in the lower
oxidation state. The acidic character of the oxides of
group 15 elements decreases down the group. All the
oxides of nitrogen except N,O and NO and phosphorus
are strongly acidic; oxides of arsenic are weakly acidic; Fig. 5. Pyramidal shape of
oxides of antimony are amphoteric while those of a trihalide of group 15.
bismuth are weakly basic.
The bond angles of the trihalides of an element
The basic nature of oxides increases with increasing increase as the electronegativity of the halogen
atomic number. For example, P (IIT) and As (III) decreases, 1.é.,
oxides are acidic, Sb (III) oxide S amphoteric and Bi
Pie PC. PBr, PI,
(IIT) oxide is distinctly basic. It dissolves in acids to
96.3° 100.4° 101° 102°
form salts.
Bi,0,+ 6HNO, ———> 2Bi(NO,), + 3H,O Properties of trihalides (1) The trihalides of
croup 15 elements are predominantly covalent with
Acidic character of oxides of elements of the ionic character increasing down the group. For
group 15 example, BiF,, is ionic while other halides of Bi, i.e.,
e N,O < NO<N,0, < N,O,< N,O, (N,O & NO are BiCl,, BiBr,, etc. and SbF, are partly covalent and
neutral) partly ionic.
e N,O, > P,O, > As,O, > Sb,O, (As,O, & Sb,O, (iz) Out of all the trihalides, the trihalides of
are amphoteric while Bi,O, is basic.) nitrogen are the least stable. Though NF, is stable,
TO ee410 ake4710 ers, 410 NCl, is explosive. NBr, and NI, are known only as
their unstable ammoniates 1.e., NBr,.NH, and NI,.NH,.
The oxides of nitrogen and phosphorus are
Pure NBr, also explodes even at 173K. The nitrogen
discussed later.
triiodide ammoniate is stable only in the moist state.
3. Reactivity towards halogens (formation
In the dry state, it explodes with noise when struck
of halides)
liberating vapours of iodine.
Group 15 elements form two series of halides
8NI,.NH, ———> 5N, + 91, + 6NH,1
of the type EX, (trihalides) and EX, (pentahalides).
NCi,, NBr, and NI, are unstable because N—X
The trihalides are formed by all the elements while
bond is weak due to large difference in the size of
pentahalides are formed by all the elements except
N and X atoms. However, NF, is stable because of
nitrogen. Nitrogen cannot form pentahalides
small difference in size of N (75 pm) and F (72 pm)
due to the absence of vacant d-orbitals in its
resulting strong N—F bond. As a result NF, is an
outermost shell. Similarly, the last element, Bi
exothermic compound. Because of stability of NF, it
has little tendency to form pentahalides because +5
behaves quite differently from others. It is unreactive
oxidation state of Bi is less stable than +3 oxidation
and does not hydrolyse with water, dilute acids or
state due to inert pair effect.
alkalies. However, it does react only when sparked
(a) Trihalides with water vapour.
All the elements of group 15 form trihalides of the PF, is rather less reactive towards water and is
general formula EX,. All these trihalides are known more easily handled than the other halides.
(X = F, Cl, Br or I and E = N, P, As, Sb, Bi). (iit) The trihalides are easily hydrolysed by water.
Structure. The valence shell electronic However, the products are different in hydrolysis of
configuration of these elements is ns? np,* np," np," different chlorides.
and all these elements undergo sp* hybridisation. The NCl, Ss) + 3H,O — > NH, + 3HCIO
three of the four sp® hybrid orbitals overlap with p Hypochlorous
orbital of halogen atom to form three o bonds with acid
p-BLOCK ELEMENTS
mAs)
(4) As we go down the group, the stability of +5
PCI, + 3H,O —> H,PO, + 3HCl
Phosphorous oxidation state decreases while that of +3 oxidation
acid state increases due to inert pair effect. Therefore, the
pentahalides ofBi are not known except for BiF,.
2AsCl, + 38H,O ——\ As,O, + 6HCI
(it) As the size of halogen atom increases from
Antimony and bismuth trichlorides are only
F to I, the strength of E—X bond decreases and the
partially hydrolysed to form oxychlorides.
steric hindrance increases. As a result, pentabromides
SbCl, + H,O SbOC] (or SbO*Cl) + 2HCI
and pentaiodides are unstable.
Antimony oxychloride
Therefore, the pentahalides of group 15 elements
BiCl, + H,O BiOC]l (or BiO*Cl) + 2HCl1
are less in number than trihalides. For example,
Bismuth oxychloride
pentafluorides of P, As, Sb and Bi are known,
It may be noted that in accordance with Le
pentachlorides of only P, As, and Sb are known while
Chatelier principle, the excess of HC] suppresses the
bromides and iodides of only P are known.
hydrolysis by shifting the equilibrium to the left.
(tv) The trihalides of P, As and Sb (especially Preparation. The pentahalides are prepared as :
fluorides and chlorides) act as Lewis acids and combine 3PCl, + 5AsF, —> 8PF, + S5AsCl,
with Lewis bases : PCl, + Cl,(mca,) —> PCI,
PF, + F, —— PF, 2As,0, |+ 10F, —> 4AsF, + 30,
SbF, + 2F- ——> _ [SbF,J>
SbCl, + 2Ck ——> _ [SbCl,]> 28b,0, + 10F, — > 4SbF, + 30,
The Lewis acid strength however, decreases in
oBi + 5F, —> 4BiF,
the order : Structure. The pentahalides of group 15 elements
PCI, > AsCl, > SbCl, ; PF, > PCl, > PBr, > PI, have trigonal bipyramidal geometry in which the
central atom is sp*d hybridised. The three halogen
REMEMBER
atoms occupy equatorial positions while the other two
In contrast to trihalides of P, As and Sb, the occupy axial positions. This is not a regular structure
trihalides of nitrogen behave as Lewis bases. because all the bond angles are not same. The axial
This is due to the absence of d-orbitals in N and bonds are at 90° while equatorial bonds are at 120°
the presence of lone pair of electrons. NF, has each. The axial positions experience greater repulsion
a little tendency to donate a pair of electrons than equatorial positions by bond pair of electrons.
and hence behaves as very weak Lewis base As a result, axial bonds are usually larger than the
because of high electronegativity of fluorine. equatorial bonds. The structure of PF. is shown in
Fluorine is highly electronegative and has strong Fig. 6 in which P—F axial bond lengths are 158 pm
tendency to withdraw electrons from N. As a while P—F equatorial bond lengths are 153 pm.
result, N-atom acquires partial positive charge
and hence the tendency to donate electron pair
of electrons decreases. Therefore, the Lewis base
"
strength of trihalides of nitrogen increases as the
electronegativity of halogen decreases, i.e.., (excited) Ey
SI ft}
Us | tT
NF, < NCl, < NBr, < NI,.
sp’d hybridisation
» The tendency of trihalides to act as Lewis base
x
decreases from N to Bi for a given halide.

(6) Pentahalides
P, As and Sb form pentahalides of the general
formula EX.; EF. (E = P, As, Sb and Bi), EC,
(HK = P, As and Sb) and PBr, and PI-. As already
studied, nitrogen does not form pentahalides because
of the absence of d-orbitals in its valence shell.
The pentahalides of group 15 elements are
thermally less stable than trihalides. This is
due to the following reasons :
sano Properties (i) Pentahalides are thermally less
MODERN'’S abc + OF CHEMISTRY-XI

(vi) Nitrogen does not form any complex whereas


stable than their corresponding trihalides. PF, is the other members form complexes.
molecular in both the gaseous and solid states. PCl, (vit) Nitrogen can form hydrogen bonds in its
exists as molecules in the gas phase but exists as compounds and rest of the members do not form
[PC1,]* [PCl,]" in the crystalline state. PBr, and PI, hydrogen bonds due to small electronegativities.
also exist in the ionic form as [PBr,]* [PBr,]” and (viit) Nitrogen forms a variety of the oxides while the
[PI,]*I respectively in the solid state. other members form trioxides and pentoxides
(it) All the pentahalides behave as Lewis acids only.
because of the presence of vacant d-orbitals. The (ix) Except NF,, the trihalides of nitrogen (NCIl,,
central atom (except N) can accept a pair of electrons NBr, and NI,) are explosive. The trihalides of
other elements are, however, stable.
thereby expanding its coordination number to 6.
EX, + X ——> [EX,]
During this the hybridisation of the central atom | SOLVED EXAMPLES
changes from sp*d to sp*d?.
LJ] Example 1.
4. Reactivity towards metals
Nitrogen exists as diatomic molecule and
All the elements of group 15 combine with metals
phosphorus exists as tetraatomic molecule, P,.
to form their binary compounds in which the elements
Explain.
show —3 oxidation state. For example, nitrogen forms
\N.C.E.R.T)| (Pb. S.B. 2015, H.P.S.B. 2014, 2016)
nitrides (Mg.N, :magnesium nitride, Ca,P, : calctum
Solution : Thesize of nitrogen atom is quite small. Asa
nitride) arsenic forms arsenides (Na,As : sodium result, two nitrogen atoms can be linked to each other by
arsenide), antimony forms antimonides (Zn,Sb, : three covalent bonds in order to complete the octets of both
zinc antimonide) and bismuth forms bismuthides the nitrogen atoms as : N = N .:. However, phosphorus
(Mg.Bi, : magnesium bismuthide). being large in size has less tendency to form three bonds.
Therefore, P atom completes its octet by sharing its valence
electrons with three other P atoms. As a result, it exists
ANOMALOUS BEHAVIOUR OF NITROGEN as P, molecule.
Nitrogen, the first member of group 15, differs _] Example 2.
from its rest of family members. This may be due to : Though nitrogen exhibits + 5 oxidation state, it
(i) its small size. does not form pentahalides. Give reason.
(iz) its highionisation enthalpy and electronegativity. (A..S.B. 2013, D.S.B. 2015, 2017)
Solution : Nitrogen belongs to second period (7 = 2)
(iit) absence of d-orbitals in its valence shell.
and has only s and p-orbitals. It does not have d-orbitals
Some of the characteristics in which nitrogen in its valence shell and therefore, it cannot extend its octet.
differs from other family members are given below : That is why nitrogen does not form pentahalides.
(4) Nitrogen is a gas at room temperature while the LL] Example 3.
rest of the members of the family are solids. Why is N, less reactive at room temperature ?
(it) Nitrogen does not exhibit pentavalency in (Pb.S.B. 2013, ALLS.B. 2015, H.P.S.B. 2015, 2016,
Meghalaya S.B. 2017)
its compounds while other members show
Solution : In molecular nitrogen, there is a triple
pentavalency. bond between two nitrogen atoms (N = N) and it is non-
(iit) Nitrogen has a tendency to form pz-pz bonds polar in character. Due to the presence of a triple bond,
and exists as N, molecule while other elements it has very high bond dissociation energy (941.4 kJ mol!)
do not have such tendency. Instead, they prefer and therefore, it does not react with other elements under
normal conditions and is very unreactive. However, it may
to form single bonds as P—P, As—As etc.
react at higher temperatures.
(tv) Due to the absence of d-orbitals in the valence _] Example 4.
shell, nitrogen cannot form dz-pz bonds and Phosphine has lower boiling point than ammonia.
therefore, compounds of the type R,N = O and Give reason.
R,N = CH, are not known. However, P and N.C.E.R.T. (A.LS.B. 2008, D.S.B.2018,
arsenic form compounds such as R,P = O, R,P Mizoram S.B.2018,
= CH, containing di-pz bonds. Uttarakhand S.B. 2014,
(v) The hydride of nitrogen (NH,) is more basic FLT. 2018, if. 3b. 20TT)
than the hydrides of the other members of the Solution : Ammonia exists as associated molecule
family. due to its tendency to form hydrogen bonding.
Therefore, it has high boiling point. Unlike NH,,
p-BLOCK ELEMENTS

phosphine (PH,) molecules are not associated through (N,) having triple bond between nitrogen atoms
nant
hydrogen bonding in liquid state. This is because (N==N). Therefore, it is also referred to as dinitrogen.
of low electronegativity of P than N. As a result, It is a typical non-metal with high electronegativity
the boiling point of PH, is lower than that of NH,. next only to fluorine and oxygen.
Preparation of Dinitrogen
_} Example 5.
In the laboratory, dinitrogen is prepared by
Unitke phosphorus, nitrogen shows little tendency
heating an aqueous solution containing an equivalent
for catenation. amount of ammonium chloride and sodium nitrite.
Solution: Nitrogen has little tendency for catenation
because N—N single bond 1s weak. This is because nitrogen
NH,Cl(aq) + NaNO,(ag) —H#&2t,
N, (g) + 2H,O(/) + NaCl(aq)
has small size and the lone pairs on two nitrogen atoms repel
each other. On the other hand, phosphorus is comparatively During the preparation, small amounts of NO
large in size so that lone pairs on P atoms donot repel to the and HNO, are also formed in the reaction. These
same extent. As a result P—P bond is stronger than N—N
impurities can be removed by bubbling the gas through
aqueous sulphuric acid containing a small amount of
bond. Therefore, P has a tendency for catenation because
potassium dichromate.
of high bond enthalpy of P—P bond.
It can also be prepared by
_}] Example 6. (1) Thermal decomposition of ammonium
Out of Bi(V) and Sb(V) which may be a stronger dichromate.
oxidising agent and why ? (D.S.B. 2008) (NH,),Cr,0, 2 cr,0, +N, + 4H,O
Solution : On moving down the group, the stability (it) By the action of ammonia with cupric oxide
of +5 oxidation state decreases while the stability of +3 or bleaching powder.
oxidation state increases due to inert pair effect. Therefore,
+5 oxidation state of Bi is less stable than +5 oxidation state
2NH, + 3Cu0 "5 scu + N, + 3H,0
of Sb. Thus, Bi(V) is a stronger oxidising agent than Sb(V). 2NH, + 8CaOCl, > sCaCl, +N, + 3H,0
Bleaching powder
(iit) Very pure nitrogen can be obtained by the
thermal decomposition of sodium or barium
azide
1. Which of the two : PH, or NH, is more basic ? 2NaN, —“*> 2Na + 3N,
Sodium azide _
2. Which hydride of group 15 elements has lowest boiling Ba(N,), Heat
Ba + 3N,
point ? Barium azide
. Name three allotropes of phosphorus. Which of these REMEMBER
1s most reactive ?
Thermal decomposition of sodium azide is used to
» Which of the two : NH, or PH, is soluble in water ?
inflate the air bags used for safety devices in some
. Arrange BiH,, PH, and NH,, in the increasing order cars.
of their reducing character.
Commercial Isolation of Dinitrogen from Air
Answers to Practice Problems Commercially dinitrogen is prepared by the
01. liquefaction and fractional distillation of air. Air
0 2. PH, contains dinitrogen (b.p. 77.2 K) and dioxygen (b.p. 90
03. Red, black, white or yellow. White P is most reactive. K). When the liquid air is allowed to boil, dinitrogen
with lesser boiling point gets distilled first leaving
0 4. NH,
behind dioxygen. The dinitrogen obtained from air
Q 5. NH, < PH, < BiH,
contains traces of oxygen and some other gases as
impurities.

CHEMISTRY OF NITROGEN AND ITS Properties of Dinitrogen


COMPOUNDS Physical Properties
(i) Dinitrogen is a colourless, odourless and
DINITROGEN tasteless gas.
Nitrogen was discovered by Daniel Rutherford in (ii) It has two stable isotopes 4N and PN.
1772. It is the first member of group 15 of the periodic
(iit) It is a non-toxic gas.
table. It has the electronic configuration 1s? 2s? 2p?
and therefore, has five electrons in its valence shell. (iv) It is shghtly lighter than air and its vapour
In the molecular form, it exists as a diatomic molecule density is 14.
liane (v) It has very low solubility in water (about 2NO + 0, ——>
MODERN'’S abc + OF CHEMISTRY-XI

2NO,
23.2 cm? per litre of water at 1 atmosphere 4. Combination with compounds. Dinitrogen
pressure and 273 K). also combines with certain compounds on strong
(vi) Its melting and boiling points are low; 63.2 heating. For example,
K and 77.2 K respectively. With calctum carbide
Chemical Properties 1300 K
CaC,+ N, —— CaCNn, +_C
Dinitrogen is chemically unreactive at room Calcium rs Calcium Carbon
temperature. It is neither combustible nor it supports carbide cyanamide
combustion. The N—N bond in nitrogen molecule is With alumina in the presence of carbon
a triple bond (N == N) with a bond distance of 109.8 1273K
Al,O, + N, + 6S) 2AIN + 3SCO
pm and bond dissociation enthalpy of 941.4 kJ molt.
Aluminium Aluminium
The low reactivity of nitrogen is due to very small size
oxide nitride
of the molecule and high bond dissociation enthalpy
Both these compounds are hydrolysed on boiling
of the molecule. However, reactivity increases rapidly
with water to give ammonia.
with the rise in temperature. At higher temperature
dinitrogen combines with some metals to form CaCN, + 3H,Q,——~> CaCO, + 2NH,
predominantly ionic nitrides and with non-metals to AIN. + 3H,O0 ——~ AIOH), + NH,
form covalent nitrides. Therefore, calcium cyanamide ts used as a fertilizer
1. Action with litmus. Dinitrogen is neutral under the name nitrolim (CaCN, + C).
towards litmus because it has no action on blue or
red litmus.
R U Curious ...
2. Action with metals. It combines with a
number of active metals on strong heating to form _] How do airbags installed in the dash board
their respective nitrides. For example, of your car work?
Heat
a + NN, = 2L1,N > An airbag is a vehicle safety device. Its purpose
Lithium nitride is to provide protection to the occupants when
they strike interior objects such as steering wheel,
3Mg + N, —Heal, Mg.N, window, dashboard, etc. during accidents. The
Magnesium nitride airbag 1s made ofa thin nylon fabric and contains
3Ca + N, —Ho@, Ca,N, sodium azide (NaN,). When accident occurs,
Calcium nitride the impact sensors provided in the car detect
2A1 + N, —, 2AIN the collision. These sensors send an electrical
Aluminium nitride
signal which overheats and ignites sodium azide
(NaN,) placed in the airbag. Sodium azideisa fast
3. Combination with non-metals. Dinitrogen burning fuel and within about 20—30 milliseconds
combines with non-metals like dioxygen and of impact, it decomposes to produce N, gas.
dihydrogen.
2NaN, ——> 2Na + 3N,
(4) Combination with dihydrogen. Dinitrogen
The N, gas produced passes through filters and
reacts with dihydrogen at about 700 K under a pressure
fills the nylon airbag and inflates it. The airbag
of 200 atm in the presence of iron as catalyst (Haber
then hits the car’s occupant and protects him
process). A small amount of molybdenum is also used from injury. After hitting, the airbag deflates
as a promotor. immediately and the gas escapes through tiny
_ ff GaFe, naMo
fl, INH holes in the fabric (or as itis sometimes called the
N, + 3H, 700 K, 200 atm pressure
Revue sie cushion) and cools. Deployment is accompanied
AH” =~ 461 kJ mol by release of dust like particles usually talcum
powder or corn starch which are used to lubricate
the airbag during deployment. The powder
(74) Combination with dioxygen. Dinitrogen protects the bag from sticking to itself, while it
combines with dioxygen during lightning or at high is folded inside the car. The N, gas released is
temperature of about 2000 K and form nitric oxide. absolutely harmless. The only precaution is to
open the door and windows for good ventillation to
N,+0, 2OR, 9NO AH” = 135 kJ mol avoid the affect of dust particles. It may be noted
Nitric oxide that some airbag manufacturers mix sodium
Nitric oxide is a colourless gas but is very unstable. azide with other chemicals that will react with
It immediately reacts with more of oxygen to form sodium and form less toxic compounds.
nitrogen dioxide.
p-BLOCK ELEMENTS

Uses of Dinitrogen
nana)
2NH,Cl + Ca(OH), #*> CaCl, + 2NH, + 2H,O
(1) Dinitrogen is used in the manufacture
Slaked lime
of compounds like ammonia, nitric acid, calcium
cyanamide, etc. It can
also be prepared by the hydrolysis of
(it) It is used in providing inert atmosphere in magnesium nitride.
iron and steel industries. Mg.N, + 6H,0 ——> 3Mg(OH), + 2NH,
(iit) Liquid nitrogen is used as refrigerant to Ammonia can be dried by passing over quicklime
preserve biological specimens, in freezing food stuffs (CaO). However, it cannot be dried with dehydrating
and also in cryosurgery. agents such as conc. H,SO, P,O0, and anhydrous
(iv) It is used in gas-filled thermometers used CaCl, because ammonia reacts with these compounds.
for measuring high temperatures. Manufacture of ammonia
(v) Dinitrogen is used in filling electric bulbs Ammonia can be manufactured by Haber’s
to reduce the rate of volatilisation of the tungsten process which involves the reaction :
filament.
N,(g) + 3H, (g) 2NH, (g);
Fixation of Nitrogen AH? = — 92.2 kJ mol
The process of conversion of atmospheric dinitrogen or AAT (NH,) =— 46.1 kJ mol
into useful compounds is called nitrogen fixation.
This is a rm” iS) exothermic reaction. The
Fixation of nitrogen is brought about in the following
ways : favourable conditions for high yield of ammonia can
@ During lightning and thunder storms the energy in be understood by applying Le Chatelier’s principle.
lightning allows nitrogen and oxygen to combine (7) Low temperature. Since the forward
form oxides of nitrogen which are washed into the reaction is exothermic, therefore, low temperature
soil and are taken up by the plants as nitrates. As will favour the formation of ammonia. However, an
animals live on plants, they are transferred to the optimum temperature of about 700 K is necessary.
animal bodies where a series of metabolic changes (qi) High pressure. High pressure of the order
take place. of 200 atmospheres or 200 x 10° Pa is required to
The atmospheric nitrogen can be converted into useful favour the forward reaction.
compounds (which can be used by plants) by the action
of certain bacteria and algae. All types of bacteria are (iii) Presence of catalyst. The use of catalyst
not able to fix nitrogen. The symbiotic bacteria present such as iron oxide containing a small amounts of
in the root nodules of leguminous plants (e.g., peas, molybdenum or potassium oxide (K,O) and aluminium
beans, grams, etc.) catch up the nitrogen from the oxide (Al,O,) as promoter, increases the rate of
atmosphere and supplies to the plant, which in turn attainment of equilibrium of ammonia.
give food to the bacteria. The non-leguminous plants
hike Alnus, Ginkgo also fix atmospheric nitrogen. Certain Details of the process
blue-green algae found in paddy fields is also able to The plant required for the manufacture of ammonia
fix nitrogen. These biochemical processes involve an is shown in Fig. 7.
enzyme called nitrogenase in which the active sites
contain molybdenum and iron atoms.
Dinitrogen can be made to take part in different H,—>
chemical reactions under suitable conditions to prepare
useful compounds. This is called artificial fixation.
N.—
|
|
1} :
Compressor
Pump

The common examples of industrial fixation of

|
7 20 MPa N, + H,
nitrogen is the manufacture of ammonia by Haber’s (in the ratio dy
process. In this process, a large amount of atmospheric 3
Condensing |
nitrogen is fixed in the form of NH. pipes
Catalyst chamber
lron oxide
AMMONIA at 700 K Al,O, + K,O

Ammonia is the most important compound of


nitrogen. It is present in small quantities in air and —* t's
soil where it is formed by the decay of nitrogenous (N, + H, + NH,) Liquid NH,
organic matter e.g., urea. Fig. 7. Flow chart for Haber's process for the manufacture of ammonia.

NH,CONH, + 2H,O —-> (NH,),CO, In this method, a mixture of N, and H, in the molar
Urea ————— 2NH, +CO, 2 H,O ratio of 1 : 3 is compressed to about 200 atmosphere
Laboratory Preparation pressure. The compressed gases are then cooled and
passed through soda lime tower to free them from
Ammonia is prepared in the laboratory by heating
moisture and carbon dioxide. Then these are fed
ammonium salt; NH,Cl or (NH,),SO, with a strong
into catalyst chamber packed with iron oxide with
alkali ike NaOH.
small amount of K,O and A1,0, or molybdenum. The
NH,Cl + NaOH —-> NH, + H,O + NaCl chamber is heated siec-cically to a temperature of 700
(NH,),SO, + 2NaOH —-> 2NH, + 2H,O + Na,SO, K when the two gases combine to form ammonia. The
reaction being exothermic, the heat evolved maintains
It can also be prepared by heating ammonium
chloride with slaked lime. the desired temperature and further electrical heating
is not required.
sane The gases which escape from the chamber Ag* (aq) + 2NH, (aq) ——
MODERN'S abc + OF CHEMISTRY-XI

[Ag(NH,),]* (aq)
contain about 15-20% ammonia and the remaining Colourless
are unreacted N, and H,. These are passed through Cu** (aq)+ 4NH, (ag) —— [Cu(NH,),]** (aq)
condensing pipes where ‘ammonia gets liquefied and Deep blue
is collected in the receiver. The unreacted gases are Cd?* + 4NH, —~ [Cd(NH,),]?* (aq)
pumped back to the compression pump where they
Since ammonia forms complexes, the white
are mixed with fresh gaseous mixture.
precipitate of silver chloride dissolves in excess of
Structure of ammonia. Ammonia is expected
to have a tetrahedral geometry because the central ammonium hydroxide to form a soluble complex.
nitrogen atom involves sp® hybridisation as already AgCl + 2NH,OH —— [{Ag(NH,),|Cl + 2H,O
discussed. It has one position occupied by a lone pair. White ppt. Diammunesilver (I)
The lone pair distorts its geometry and the molecule chloride (colourless)
has pyramidal geometry with nitrogen atom at the
Similarly, copper sulphate dissolves in excess
apex. The N—H bond length is 101.7 pm and HNH
of ammonium hydroxide to form deep blue coloured
bond angle is 107.8°.
complex.
CuSO, + 4NH,OH [(Cu(NH,),JSO, + 4H,O
Tetraamminecopper (II)
sulphate (Blue)
The formation of such complex compounds finds
applications in detection of metal ions (Ag*, Cur, etc.)
3. Action with halogens. (i) Chlorine reacts
with ammonia in two ways depending upon whether
Fig. 8. Structure of ammonia. ammonia is in excess or chlorine is in excess.
Excess NH,
Properties of Ammonia 8NH, + 38Cl, —~+ 6NH,Cl + N,
Physical properties (1) Ammonia is a colourless Excess Cl,
gas with a characteristic pungent smell called Mery + 3Cl,— > NCI, + 3HCl
ammoniacal smell. Bromine also reacts in a similar way.
(ii) It is lighter than air (density = 0.68 g/cm?*) Excess of ammonia :
(iit) It is highly soluble in water: one volume 8NH, + Br, —~> 2NH,Br + 3N,
of water can dissolve about 1000 volumes Excess of bromine :
of the gas at 273 K. The high solubility of
NH, - 3Br, —> NBr, + 3HBr
ammonia in water is due to its tendency to
form intermolecular hydrogen bonding. (iz) Iodine reacts with liquid ammonia to form a
dark brown complex.
(iv) Ammonia can be easily liquefied under
pressure. Liquid ammonia has b.p. 239.7 K 2NH, + 31, —~ NH.NI, + $8HI
and f.p. 198.4 K. Like water, ammonia is Nitrogen tri-iodide
also associated in the solid and liquid states ammoniate (brown complex)
through hydrogen bonds. This accounts for The complex is very explosive when dry. It explodes
its higher melting and boiling points than violently when rubbed against a hard surface as
expected on the basis of its molecular mass. follows : =
Chemical properties 8NH.NI, ——~—__5N, + 91, + 6NH,I
1. Basie nature. Ammonia gas is highly soluble 4, Oxidation. Ammonia is oxidised to dinitrogen
in water. Its aqueous solution turns red litmus blue with oxidising agents like CuO, NaClO, Ca(OC)l), ete.
indicating its weakly basic character. The basic For example, when ammonia is passed over heated
character of ammonia is due to the formation of OH copper oxide, it gets oxidised to dinitrogen.
ions in aqueous solution.
8CuO + 2NH, —-> 3Cu + N, + 3H,O
NH, (aq) + H,O (aq) NH,* (aq) + OH (aq)
Similarly, 4NH,, + 3Ca(OCl), —-~ 2N, + 3CaCL, + 6H,O
Being basic ammonia forms salts with both
weak and strong acids such as NH,Cl, (NH,),SO,, When vapours of NH, and O, are passed over
red hot platinum gauze at 1075 K it is oxidised to
(NH,),CO,, etc.
niric oxide.
NH, (ag) + HCl (aq) —~ NH,Cl (s)
2NH,(aq) + H,SO, — > (NH,),SO,(s) ANT + oo,
Pt
ANO + 6H,O
1075K Nitric oxide
2. As a Lewis base : tendency to form
complexes. Ammonia acts as a Lewis base due to This reaction forms the basis of Ostwald's
the presence of lone pair of electrons on the nitrogen process for the manufacture of nitric acid.
atom. Therefore, it can form coordinate bond with
electron deficient molecules (such as BF.) or anumber 5. Combustion. Ammonia is neither combustible
of transition metal cations forming complex compounds. nor supporter of combustion. However, it burns in the
For example, presence of oxygen to form dinitrogen and water.
BF, + NH, — > BF, «NH, 4NH, + 30, —> 2N, + 6H,O
p-BLOCK ELEMENTS 11A/15 =

6. Formation of amides. When ammonia 1s @ With a drop of HCl, it produces dense white fumes of
passed over heated sodium or potassium at 575 K, ATG HC ch ote:
zeros ee eee liberation of dihydrogen. @ Itturns moist red litmus paper blue and moist turmeric
2Na + 2NH, 2% , 2NaNH, + 4H, paper brown.
| Sodamide @ It gives a yellow precipitate with chloro-platinic acid.
2K + 2NH, —“*, 2KNH, + H,_ | H,PtCl,+2NH, —- (NH,),PtCl,
Potassium amide Chloroplatinic Ammonium
7. Precipitation of heavy metal ions from the acid chloroplatinate
aqueous solution of their salts. Heavy metal ions (yellow ppt.)
such as Fe**, Cr**, Al**, etc. are precipitated from their
Uses of ammonia
aqueous salt solutions as hydroxides with ammonia.
(t) It is used in the manufacture of nitric acid
FeCl,(ag) + 3NH,OH(aq) —>
and sodium carbonate.
Fe(OH).(s) + 3NH,Cl(aq)
(it) Liquid ammonia is used as a refrigerant.
Brown ppt.
AICl.(aqg) + 3NH,OH(ag) —> (iit) It is used in the manufacture of rayon.
° ‘ Al(OH).(s) + 3NH,Cl(aq) (iv) It is commonly used for preparing various
White pre : nitrogenous fertilizers such as ammonium
CrCl.(ag) + 3NH.OH(ag) —> nitrate, ammonium sulphate, ammonium
ad , phosphate and urea.
Cr(OH),(s) + 3NH,Cl(aq) f....
Green ppt. (v) It is an important reagent and is used as a
ZnSO,(aq) + 2NH,OH(aq) —> Ivent in
solvent in the the laboratory.
laboratory
Zn(OH), (s) + (NH,),80,(aq) OXIDES OF NITROGEN
White ppt. Nitrogen combines with oxygen under different
Tach at cmon conditions to form a number of binary oxides which
@ Ammonia can be tested by its reaction with Nessler's differ with respect to the oxidation state of the nitrogen
reagent. It gives brown precipitate with Nessler's atom. They range from N,O (oxidation state ofN +1)
reagent (an alkaline solution of K,HglI,) due to the through NO (+2), N,O, (+3), N,O, (+4) to N,O, (5). The
formation of 1odide of Million's base. tendency to form pz—pz multiple bonds dictates the
2K Hel, + NH, + 3KOH —-— Nessler's reagent structures of oxides. The important oxides are NO,
BLN He- O- He 1+ is ge), 4N,0,, NO,, NO, and N,O,.
Iodide of Million's base a | . . :
The molecular formulae of various oxides of
(brown ppt). : :
nitrogen are given in Table 5.

Table 5. Oxides of nitrogen.


Oxidation Physical appearance Properties
state of N acidic or neutral character

a N,O : Colourless gas Unreactive gas


Dinitrogen monoxide = NMontesl
Nitrous oxide or
Nitrogen (I) oxide
NO +2 Colourless Paramagnetic, reactive and
Nitric oxide
or Nitrogen (II) oxide Neutral thermodynamically unstable
N,O, +3 Pale blue solid Unstable in gaseous phase
Dinitrogen trioxide Acidic
or Nitrogen (III) oxide
N,O, +4 Colourless solid Exists in equilibrium with NO,
Dinitrogen tetroxide or liquid both in gaseous and liquid state
or Nitrogen (IV) oxide Acidic
NO, +4 Brown gas Reactive and paramagnetic
Nitrogen dioxide Acidic
or Nitrogen (IV) oxide
N,O, +5 Colourless solid Unstable as gas, in solid state
Dinitrogen pentoxide or gas exists as [NO,]* [NO,]-
or Nitrogen (V) oxide Acidic
sare 1. Nitrous oxide (N,O). It is prepared by heating
MODERN'’S abc + OF CHEMISTRY-XI

Nitric oxide due to the presence of an odd electron,


ammonium nitrate. is very reactive. It readily reacts with oxygen to give
brown fumes of nitrogen dioxide and with halogens
NH,NO, —““*> N,O + 2H,O to give nitrosyl halides.
It is a colourless unreactive gas having faint 2NO (g) + O, (g) —— > 2NO, ig)
pleasant smell. It is also known as laughing gas 2NO + Cl, —— 2NOCIl
because it causes hysterical laughter when inhaled Nitrosyl chloride
in minor quantities. It is a neutral oxide and reacts Nitric oxide readily forms complexes with transition
with sodamide to form sodium azide. metals. For example, Fe** combines with NO to form
N,O + 2NaNH, —> NaN, + NH, + NaOH the complex [Fe(H,O)-NO]** which is responsible for
Sodamide Sod. azide the colour in brown ring test for nitrates.
In small amounts, it acts as an anaesthetic for It is thermodynamically unstable and decomposes
minor operations. into elements at high temperatures (1873 K— 1473 K)
It decomposes into nitrogen and oxygen at 873 K.
2N,0 —— 2N, + O,
Therefore, it supports the combustion acting as 3. Dinitrogen trioxide (N,O,). It is prepared
a source of oxygen. by cooling equimolar quantities of nitric oxide and
nitrogen dioxide to below 253 K.
Nitrous oxide (N,O) is known as laughing gas because
it causes hysterical laughter when inhaled in minor NO(g) + NO,(g) —== N,0,
quantities. It can also be prepared by reacting nitric oxide
and dinitrogen tetraoxide at 250 K.
2. Nitric oxide (NO). It is prepared by the
catalytic oxidation of ammonia at 1100 K in the 2NO +N,0, 8% 2N.0,
presence of platinum. It is a blue solid and is acidic in nature. It is
anhydride of nitrous acid (HNO,).
4NH, + 50, —5—>
1100 K
4NO + 6H,O N,O, + H,O —~> 2HNO,
It can also prepared by the reaction of nitric acid It exists in the pure form only in the solid state
on copper as : at very low temperatures. Above its melting point
3Cu + 8HNO, —-> 3Cu(NO,), + 2NO + 4H,O (273 K) it dissociates to NO and NO,,.
N, O, Above 173 K NO n
It can also be prepared by the reduction of NO,—

sodium nitrite with ferrous sulphate in the presence Blue Colourless Brown
of sulphuric acid. 4. Nitrogen dioxide (NO,). It is prepared by
2NaNO, +2FeSO, +3H,SO, ——> heating dried lead nitrate in a steel reaction vessel.
Fe,(SO,), + 2NaHSO, + 2H,O + 2NO 2Pb(NO,), —~ 2PbO + 4NO, + O,
It is a neutral oxide. It is also an odd electron molecule and in the gas
phase, it exists in equilibrium with N,O, as :
The largest use of N,O is as propellant for NO, 2NO,
whipped ice-cream. Because it has no taste, and Colourless Brown
is non-toxic and therefore, it meets the strict food Above 415 K, it contains mainly NO, and at 250 K,
and health regulations. it consists of mainly N,O,.
Cool
Itis acolourless gas. It has odd number of electrons ~~“
2
Heat a
(11 valence electrons) and therefore, it is paramagnetic
5. Dinitrogen pentoxide (N,O,). It is prepared
in the gaseous state. However, in the liquid and solid
by dehydrating the concentrated nitric acid with
states, it forms a loose dimer in such a way that the
phosphorus pentoxide.
magnetic effects of two unpaired electrons are cancelled
out. The molecule is diamagnetic. 4HNO, + P,O,, —— 2N,0. + 4HPO,
Phosphorus Metaphosphoric
pentoxide acid
N,O, exists as colourless solid below 273K. As
the temperature rises, the colour changes to yellow
due to the partial decomposition of colourless N,O-
to brown NO,,.
2N,0, ——> 4NO, + O,
p-BLOCK ELEMENTS

At 303 K, the crystals melt giving a yellow


nA
its poisonous nature. Its vapours are emitted in the
liquid which decomposes at 313 K to give NO,,. atmosphere during the burning of oil and coal.
N,O. acts as a strong oxidising agent and oxidises
Oxides of nitrogen are emitted into the atmosphere
iodine to I,O,.
during burning of coal and oil in power stations and
The oxides of nitrogen N,O,, NO,, N,O. are acidic automobile engines. These are also present in the
and react with water to give oxoacids of nitrogen. automobile exhausts. These cause air pollution in
For example, N,O, gives nitrous acid (HNO,) while major cities and near power stations. These oxides of
N,O, give nitric acid (HNO,). nitrogen also cause acid rain and photochemical
NOE 2 ne) 2a smog (discussed in class XI in environmental
Nitrous acid chemistry).
NU he i> -2HNo.
Nitric acid
Therefore, N,O, is regarded as anhydride of
'R U Curious ..&
nitrous acid and N,O, is regarded as anhydride of
nitric acid. Nitrogen dioxide (NO,) reacts with water IO You will be surprised to note that though
to give both nitrous acid and nitric acid. nitric oxide (NO) is very reactive and harmful,
2NO, ~ H,O Ss, HNO, + HNO, yet it occurs in biological systems in traces.
Nitrous Nitric
acid acid > It acts as a neuro-transmitter and plays a
Therefore, NO, is a mixed anhydride of nitric significant role in controlling blood pressure
acid and nitrous acid. by relaxing blood vessels. It also protects from
bacterial infections.
Out of various oxides, NO and NO, are used in
the manufacture of nitric acid and nitrate fertilizers.
Liquid N,O, is also used as an oxidiser for the rocket The general methods of preparation of nitrogen oxides
fuels in missiles and space vehicles. However, NO are summed up below:
causes a pollution problem in atmosphere due to

SUM UP

Oxide of nitrogen Common method of preparation

Pe (i) NH,NO, Heat


N,O + 2H,0
(1) HNO, + NH,OH (aq) ——, N,0 + 2H,O
NO (i) 3Cu + 8HNO, ——» 8Cu(NO,), + 2NO + 4H,O
(wi) 4NH, + 50, 1100 K
Pt catalyst
4NO + 6H,O
(ut) 2NaNO, + 2FeSO, + 3H,SO, —— Fe,(SO,), + 2NaHSO, + +2NO + 2H,O

N,O, NO + NO, N,O


cee |

< 250 K 2N,O,


2NO + N,O,

Cool N.O
a 2NO,
Heat es
NO, (i) 2Pb(NQO,), 673K_, 2PbO + 4NO, + O,
(iz) Cu + 4HNO,(conc.) — Cu(NO,), + 2NO, + 2H,O
Jin LE P,O,, + 4HNO, —— 2N,0, + 4HPO,
The molecular and Lewis dot stuructures of oxides of nitrogen are given ahead.
MODERN'’S abc + OF CHEMISTRY-AI

N,O, Nitrous oxide :

wen Lo: N 113 pm mt 119 pm 0


Linear

NO, Nitric oxide:

:N=0:

N,O,, Dinitrogen trioxide:

:O
\ N——N
ge (OF

*
-
*
*
117° Planar
*+

» Nitrogen a
N
<2\®&
wf ~\e 0
134°
Angular
O

N, oe
eee tetraoxide:

Dae
J \

NeO,, Dinitrogen pentoxide:

OBL

YON,
-O: -s

NITRIC ACID (HNO,)


Nitrogen forms numerous oxoacids though several are unstable in the free state and are known only in
aqueous solution or as their salts. The common oxoacids of nitrogen are given below :
Table 6. Oxoacids of nitrogen.
Oxoacids of nitrogen
OH
N =N HO—N =O
Hyponitrous weak acid if
known as its HO
salts Hyponitrous acid Nitrous acid
Nitroxylic highly explosive
acid difficult to get in H O —N.
‘a. HO ———— 7
No
pure state
Nitrous acid HNO, +3 weak acid and Nitric acid Peroxonitric acid
unstable
Nitric acid HNO, +5 weak acid and Out of the oxoacids of nitrogen, nitric acid is the
stable most important. It is very strong oxidising agent and
Peroxonitric HNO 4 +5 unstable and is quite useful. Nitrogen shown an oxidation state of
acid explosive +5 in nitric acid.

WWW.JEEBOOKS.IN
p-BLOCK ELEMENTS

Laboratory preparation of nitric acid. In the 2. Decomposition. Nitric acid decomposes on


mang)
laboratory, nitric acid can be prepared by heating heating giving nitrogen dioxide, water and oxygen.
sodium or potassium nitrate with concentrated 4HNO, (ag) —Heat, 4NO, (g) + O, (g) + 2H,OW
sulphuric acid to about 423-475 K
3. Action on metals. With the exception of gold
NaNO, + H,SO,—~> NaHSO, + HNO,
and platinum (noble metals), nitric acid attacks all
Anhydrous nitric acid can be obtained by
metals forming a variety of products. The product
distillation of concentrated aqueous nitric acid
depends upon the nature of metal, the concentration
with P,O,,. of acid and temperature.
Manufacture of Nitric Acid A. Metals that are more electropositive than
Nitric acid is commonly manufactured by Ostwald hydrogen (Mg, Al, Mn, Zn, Fe, Pb, etc.). In this case
nascent hydrogen is liberated which further reduces
process in which it is prepared by the catalytic
nitric acid.
oxidation of ammonia by atmospheric oxygen. The
M + 2HNO, —~+> M(NO,), + 2H
reaction is carried out at about 500 K and 9 x 10°
(Metal)
Pa (9 bar) pressure in the presence of Pt or Rh gauge
HNO, + H—-} Reduction product + H,O
as catalyst.
In this case, the principal product is NO, with conc.
4NH, (g) + 50,(g) iarSree> 4NO (g) + 6H,O ©) HNO,, N,O with dil. HNO, and ammonium nitrate
(from air) AH = — 90.2 kJ
with very dil. HNO,. For example, Zn reacts as:
Nitric oxide thus formed combines with oxygen
Using concentrated nitric acid (forms nitrogen
to form nitrogen dioxide.
dioxide)
2NO (g) + O, (g) —> 2NO,(g)
Nitrogen dioxide so formed, dissolves in water to
Zn + 2HNO, —> Zn(NO.,), + 2H
give nitric acid. HNO, + H——> NO, + H,O] x 2
3NO, (g) + H,O (1) —~> 2HNO, (aq) + NO (g) Zn + 4HNO, —> Zn(NO,), + 2NO, + 2H,O
The NO thus formed is recycled and the aqueous
Cone. Nitrogen
HNO, can be concentrated by distillation upto about
dioxide
68% by mass.
Using dilute nitric acid (forms nitrous oxide)
Dilute nitric acid is further concentrated by
dehydration with concentrated sulphuric acid to get Zn + 2HNO, —-> Zn(NO,), + 2H] x 4
about 98% acid. 2HNO, + 8H —-> 5H,0 + N,O
Properties of Nitric acid 4Zn + 1OHNO, —- 4Zn (NO,), + N,O + 5H,O
Physical Properties Dilute Nitrous oxide
(i) Pure nitric acid is a colourless liquid. Using very dilute nitric acid (forms ammonium
(it) It has boiling point 355.6 K and freezing nitrate)
point 231.4 K.
Zn + 2HNO, —— Zn (NO,), + 2H] x 4
(iit) Laboratory grade nitric acid contains about
68% of HNO, by mass and has a specific gravity of HNO, + 8H —— NH, + 3H,O
1.504. NH, + HNO, —~> NH,NO,
(iv) The impure acid is generally yellow due to 4Zn + 10OHNO, —-> 4Zn (NO,), + NH,NO, + 3H,O
the presence of nitrogen dioxide as impurity. Nitric
V. dilute Ammonium
acid containing dissolved nitrogen dioxide is known
nitrate
as fuming nitric acid.
(v) Ithas acorrosive action on skin and produces B. Metals which are less electropositive than
painful blisters. hydrogen (Cu, Bi, Hg, Ag). In this case nascent
Chemical Properties hydrogen ts not liberated.
1. Acidic character. It is one of the strongest HNO, —-> Reduction product + H,O + [O]
acids because it 1s highly ionised in aqueous solution Metal + (O) + HNO, —-> Metal nitrate + H,O
giving hydronium and nitrate ions.
The principal product is NO, with conc. HNO, and
HNO,(aqg) + H,O (2) ——>H,O* + NO, (aq) NO with dil. HNO,. For example, Cu reacts as
It turns blue litmus red. It forms salts with metallic Using concentrated nitric acid
oxides, hydroxides, carbonates and bicarbonates giving 2HNO, —-> 2NO, + H,O + [O]
only one series of salts called nitrates.
Cu + O + 2HNO, —> Cu(NO,), + H,O
NaOH + HNO, —-> NaNO, + H,O
Na,CO, + 2HNO,—> 2NaNO, + H,O + CO, Cu + 4HNO, —> Cu(NO,), + 2NO, + 2H,O
NaHCO, + HNO, —> NaNO, + H,O + CO, Cone. Nitrogen
CaO + 2HNO, —> Ca(NO,), + H,O dioxide
ssa
Using dilute nitric acid
MODERN'’S abc + OF CHEMISTRY-XI

(a) Metals lying above hydrogen in electrochemical


2HNO, —-> 2NO + H,O + 3[O] series react with HNO, as:
Cu+O+ 2HNO, —-> Cu(NOQ,), + H,O] x 3 Metal + HNO, ——> Metal nitrate + H
HNO, + H ——> Reduction product + H,O
3Cu + 8SHNO,—> 3Cu(NO,), + 2NO + 4H,O (b) Metals lying below hydrogen in electrochemical
Dilute Nitric oxide series react with HNO, as:
HNO, ——> Reduction product + H,O + [O]
Similarly,
Metal + HNO, +O — >} Metal nitrate + H,O
Hg + 4HNO, — > Hg(NO,), + 2NO, + 2H,O
(cone.) Mercuric nitrate 4. Oxidising nature — Oxidation of non-
metals and compounds. Nitric acid behaves as
6Hg + 8HNO, —-> 3Hg, (NO,), + 2NO + 4H,O
a strong oxidising agent. It has a tendency to give
(dil.) Mercurous nitrate nascent oxygen as :
Ag + 2HNO, —~+ AgNO, + NO, + 2H,O 2HNO,—_> 2NO, + H,O + O
(conc.) (conc.)
3Ag + 4HNO, —> AgNO, + NO + 2H,O 2HNO,—> 2NO +H,0 + 3(O)
3Ag + 4HNO, —> 3AgNO, + NO + 2H,O (dilute)
Therefore, nitric acid oxidises many non-metals
It may be noted that some metals such a chromium
and compounds.
and aluminium donot dissolve in conc. HNO, because
A. Oxidation of non-metals. Dilute nitric acid
of the formation of a passive film of oxide on
their surfaces. has no action on non-metals like carbon, sulphur,
phosphorus, etc. However, concentrated nitric acid
C. Action on noble metals. Noble metals like oxidises many non-metals. For example,
gold and platinum are not attacked by nitric acid. (1) Nitric acid oxtdises sulphur to sulphuric acid
However, these metals are attacked by aqua regia 2HNO,—~> 2NO, + H,O + O] x3
(3 parts conc. HCl and 1 part conc. HNO,) forming ; S,+H,0 +30 —> H,SO,
their chlorides.
HNO, + 38HCl— noc + 2H,0 + 2Cl ~S, + 6HNO, ——> H,SO,+6NO, + 2H,O
— i. Nitrosyl or S,+48HNO, ——> 8H,SO, + 48NO, + 12H,O
Aqua regia (11) Nitric acid oxidises carbon to carbonic acid
chloride
The free chlorine attacks gold, platinum, etc. 2HNO,—~> 2NO, + H,O + O] x 2
Au+3Cl ——+ AuCl, Ct HO +20 —> B00,
Gold chloride C + 4HNO, —+H,CO, + 4NO, + 2H,O
Pt+4Cl ——+ PtCl, (111) Nitric acid oxidises phosphorus to phosphoric acid
Platinum chloride
2HNO, —~> 2NO, + H,O + O] x 5
Chemical Reaction of Metal and HNO, 2P + 3H,O + 50 —> 2H,PO,
It may be noted that during chemical reactions of
metals with HNO,, it not only acts as acid but also 2P + 10OHNO, —_> 2H,PO, + 10NO, + 2H,O
as an oxidising agent. During its reaction, it gives a or P+5HNO, —~> H,PO, + 5NO,+H,O
variety of reduction products. The different stages of or P,+20 HNO, —~+> 4H,PO, + 20NO, + 4H,O
reduction of nitric acid are: Phosphoric acid
+5 = +4 F-. 42 =) eae (iv) It oxidises todine to todic acid.
HNO, ae A NO, —F 1. N,O
2HNO, —> 2NO, - H,O +QO|]x5
eos NH ‘
I. - H,O + 50 —> 2HIO,
The product of the reduction of HNO, depends
upon the L. + 10HNO, —> 2HIO, - 10NO, + 4H,O
(1) nature of the metal Iodic acid
(11) concentration of nitric acid (v) Nitric acid oxidises arsenic to arsenic acid.
(111) temperature. 2HNO, ——+ 2NO, ~ H,O +O])x5
In general, the more the dilution, the greater 2As + 3H,O + 50 —> 2H, AsO,
is the extent of reduction.
The mode of reactions of HNO, and metals can 2 As + 10HNO, —> 10NO, + 2H,AsO, + 2H,O
be divided into two main categories. or As+5HNO,—~> 5NO,+ H,AsO, +H,O
Arsenic acid
p-BLOCK ELEMENTS

Similarly, (vit) Nitric acid oxidises ferrous sulphate


nant
to ferric
sulphate.
Sb + 5HNO,
—> H,SbO, + H,O + 5NO,
Antimonie acid dil HNO, :
6FeSO, + Z2HNO, + 3H,SO,
Nitrous acid versus nitric acid Although nitric
—> 3Fe,(50,), + ZNO + 4H,0
acid acts only as an oxidising agent, nitrous acid acts
both as an oxidising agent as well as reducing agent. cone. HNO, :
e HNO, can be easily oxidised to HNO,, therefore, 2FeSO, + 2HNO, + H,SO, —>
it acts as a reducing agent. Fe,(SO,), + 2NO, + 2H,O
eg ING) is EO 2h Bn
5. Action on organic compounds. Nitric acid
e HNO, can be easily reduced to NO, therefore, it also reacts with organic compounds. For example,
acts as an oxidising agent. sucrose (cane sugar) is oxidised to oxalic acid by
e.g., HS + 2HNO, —> 2H,0 + 2NO+S nitric acid.
2KI + 2HNO, —> 2KOH + I, + 2NO C,5H.0), + 36HNO, —>
SO, + 2HNO, —> H,SO, + 2NO Sucrose
COOH
+ 36NO, + 23H,O
B. Oxidation of compounds. Dilute as well
COOH
as concentrated nitric acid oxidises a number of Oxalic acid
compounds.
In the presence of sulphuric acid, nitric acid reacts
(ut) Nitric acid oxidises hydrogen sulphide to sulphur. with aromatic compounds forming nitro compounds.
This process is called nitration. For example, it reacts
dil HNO, :
with benzene to form nitrobenzene.
3H,5 + 2HNO, —~> 2NO + 4H,0 + 35
H,SO,
conc. HNO, :
C,H, + HNO, —2—+) C,H,NO, + H,O
HS + 2HNO, —~> 2NO, + 2H,O +5 Benzene Nitrobenzene

Similarly, phenol reacts with nitric acid in the


(vit) Nitric acid oxidises sulphur dioxide to sulphuric
acid presence of H,SO, to give 2, 4, 6-trinitrophenol (known
as picric acid).
dil HNO, :
Nitric acid attacks proteins giving a yellow nitro
350, + 2HNO, + 2H,O0 —-> 3H,S50, + 2NO compound known as xantho protein. Therefore,
nitric acid stains skin and renders wool yellow.
conc. HNO, :
Structure. Spectroscopic studies have shown
SO, + 2HNO,——> H,SO, + 2NO, that gaseous nitric acid has planar structure [Fig. 9
(a)]. Nitrate ion, NO, has also planar symmetrical
structure [Fig. 9 (0)).

(a) Nitric acid (6b) Nitrate ion


The structure of NO, is resonance hybrid of the following structures :

20: O- :O;

N <—_>> N
+

q fe‘yy
.
of
hk ‘sf
Siw Brown Ring test for NO,” ion
MODERN'’S abc + OF CHEMISTRY-XI

This lone pair can easily be donated to electron deficient


Nitrates give brown ring test with Fe?* ions in compounds forming complexes. For example, it reacts with
the presence of conc. H,SO,. This is based upon the Cu* ion to form a deep blue complex.
tendency of Fe?* to reduce nitrates to nitric oxide Cu** (ag) + 4NH,(ag) ———> [Cu(NH,),]**
Complex (deep blue)
which reacts with Fe?* to form a brown coloured
L] Example 9.
complex during the qualitative test for nitrates. The
Why does ammonia act as a Lewis base?
test is usually performed by adding dilute FeSO,
N.C.E.R.TA(ALS.B. 2014 Kerala S.B. 2016,
solution to an aqueous solution containing NO, ion
_ Assam S.B. 2017))
and then adding conc. H,SO, slowly along the sides of
the test tube. A brown ring at the interface between Solution : Nitrogen atom in NH, has one lone pair
of electrons which is available for donation. Therefore, it
the solution and sulphuric acid indicates the presence
acts as a Lewis base.
of NO, ion. L] Example 10.
3Fe** + NO,” + 4Ht —> NO + 3Fe* + 2H,O
Write the products of the following reactions (give
Fe** + NO + 5H,0 —> [Fe(H,O), NO]?* balanced chemical equations) :
(Brown complex) (i) Mg,N, +#,0 — (it) I,+ HNO, (conc.)—_~>
The colour of the complex, [Fe(H,O).NO] 2+ is due to (tt) Cu + HNOg (conc) —~+ (iv) Li + N, —>
Solution:
charge transfer i.c., an electron is transferred from
(1) Mg,N, + 6H,O —> 3Mg(OH), + 2NH,
NO to Fe** ion. As a result, this complex formally Ammonia
contains Fe (+1) and NO* (nitrosonium ion). The (zt) I, + 1OHNO, (conc.) —> 2HIO, + 10NO, +4H,O
complex is named as pentaaquanitrosonium iron Todic Nitrogen
(I) (and not as pentaaquanitrosyl iron (II) ion. acid dioxide
(zat) Cu + 4HNO, (cone.) ——>
Cu(NO,), + 2NO, + 2H,O
Uses of Nitric Acid
Copper
(4) Itis used in the manufacture of ammonium nitrate
nitrate for fertilizers. (iv) 6Li+N, —> 2Li,N
(it) It is used in the manufacture of sulphuric Lithium nitride
acid by lead chamber process. LJ] Example 11.
(iii) It is used in the manufacture of explosives NF’, does not have donor properties like ammonia.
such as trinitro toluene (TNT), nitroglycerine, Explain. (Pb. S.B. 2015)
picric acid, etc. Solution : NF, has a pyramidal shape with one lone
pair on N atom. 9 9
(iv) It is used in the manufacture of dyes,

Per oe
perfumes and silk.
(v) It is used for the manufacture of nitrates
for use in explosive and pyrotechnics. H
(vi) It is used in pickling (cleaning) of stainless The lone pair on Ni1s In opposite direction to the N—-F
steel and etching of metals. bond moments and therefore, it has very low dipole moment
(vit) It is also used as an oxidizer in rocket fuels. (about 0.234 D). Thus, it does not show donor properties.
(viii) It is used in the purification of gold and But ammonia has high dipole moment because its lone pair
is in the same direction as the N—H bond moments. Thus,
silver as aqua regia.
it has donor properties.
L] Example 12.
| ee el | Give one reaction in which ammonia acts as a
reducing agent.
_} Example 7. Solution : When ammonia is passed over heated cupric
Why does NO, dimerise ? Explain. oxide, copper is formed as :
IN.C.E.R.T) (D.S.B. 2010)(A.L.S.B. 2014, 3Cu0 +2NH, —“*-, 3Cu+N,+3H,O
Ar. S.B. 2015, Assam S.B. 2017) L] Example 18.
Solution : NO, contains odd number of valence
Complete the reactions:
electrons. It behaves as a typical molecule. In the liquid and (1) NIZLNO, (3) Heat

solid state, it dimerises to form stable N,O, molecule, with i) N,O+NaNH, —>
even number of electrons. Therefore, NO, is paramagnetic, (iii) Cu** (aq) + NH, (aq.) —>
while N,O, is diamagnetic in which two unpaired electrons
get paired. (tv) CuO + NH ~ “2
Solution :
_} Example 8. . Heat .
Ammonia its a good complexing agent. Explain. (2) NH,NO, (s) ———’_N,0 (g) + 2H,0 (2)

(Pb. S.B. 2013) (ii) N,O+2NaNH, ——> NaN,+NH,+Na0H


Solution : Ammonia is a good complexing agent (11) Cu?*(aq) + 4NH, (aq) ——— [Cu(NH,),]?*(aq)
because of the presence of lone pair of electrons on nitrogen. (iv) 3Cu0+2NH, ——> 3Cu+N, +3H,0
p-BLOCK ELEMENTS

_}] Example 14. The reactions taking place are :


Saaz)
What is the covalence of nitrogen in N.O,;. 2Ca,(PO,), + 6Si0, +72*, 6CaSiO, + P,O,,
N.C.E.R.T. (D. SB. 2013) Calcium silicate
Solution : aeO; has the ae
P,O,, + 10c 2%;
1775 K
10C0 +P,
ve The vapour of phosphorus thus produced upon
condensation give white phosphorus, which exists as P,,.
0 QO:
In this structure, N atom has four shared pair of
electrons, two with an oxygen atom to form N =O
bond, one with oxygen atom to form N—O bond and
one with another oxygen atom to form N -> O bond.
Thus, the covalency of N in N,O, is four. It may be
noted that for a coordinate bond, the covalency is
also one.

Q 6. Give ana of compounds in which the oxidation


state of nitrogen is (1) +3 (11) -3 (111) O (tv) +5 Fig. 10. Structure of
Q 7. What does the abbreviation T.N.T. stand for ? white phosphorus
Q 8. What is laughing gas ? How is it prepared ?
Q 9.Which plants mostly help in nitrogen fixing ?
ALLOTROPIC FORMS OF PHOSPHORUS
910. Write the formula of the compound formed during Phosphorus exists in many allotropic forms. Of
ring test of nitrate radical. Give its name. these three main allotropic forms are :
911. Name the oxide of nitrogen (4) White phosphorus
(1) Which is blue solid ? (it) Red phosphorus
(1) Causes pollution problem. (iit) Black phosphorus
Q 12. Which oxide of nitrogen is produced by heating lead 1. White phosphorus. It is the common variety
nitrate ? of phosphorus and is obtained from phosphorite rock
with coke and sand in an electric furnace at 1775 K
Answers to Practice Problems as explained above. It consists of P, units as shown
in Fig. 10. In this case, the four P-atoms le at the
@ 1. (2) NF, (i) NH, (az) N, (iv) N,O, corners of a regular tetrahedron with ZPPP = 60°.
9 7. Trinitrotoluene Each phosphorus is bonded to each of the other three
@ 8. N,O. It is prepared by heating ammonium nitrate. P-atoms by covalent bonds, so that each P-atom
Q 9. Leguminous plants completes its valence shell.
010. [Fe(H,0),.NO]SO,. Properties :
pentaaquanitrosoniumiron(]) sulphate The main characteristics of white phosphorus
are :
0 11. @) N,O, (i) NO
(4) It is soft, translucent white waxy solid with
0 12. NO,
garlic smell.
(iz) It can be cut with a knife.
(iit) It melts at 317 K and boils at 553 K.
(iv) Itisinsoluble in water but soluble in benzene,
carbon disulphide, liquid NHg.
CHEMISTRY OF PHOSPHORUS
(v) It is very poisonous. The vapours of
AND ITS COMPOUNDS
phosphorus, if continuously inhaled, may prove fatal.
(vi) It glows in dark (a property known as
PHOSPHORUS chemtiluminescence).
We have already learnt that phosphorus is widely (vit) White phosphorus, P, molecule are stable
distributed in nature. It occurs mainly in the form of upto 1070 K even in the vapour phase. When heated
phosphate minerals in the crust of earth. above 1070 K, P, molecules begin to dissociate into
Phosphorus is produced by heating bone ash Ps mplecules which have a bond enthalpy of 489.6 kJ
(containing about 58% of calcium phosphate) or mol”.
Pp,4=
above LOTTO K
phosphate rock [Phosphorite, Ca,(PO,),] with silica
(Si0,) and coke in an electric furnace at 1775 K.
sa (viit) White phosphorus is less stable and
MODERN'’S abc + OF CHEMISTRY-XI

This is an example of disproportionation reaction


therefore, more reactive than the other solid phases in which oxidation state of phosphorus decreases from
under normal conditions. This is because of angular 0 in P, to -3 in PH,, while it increases from 0 in P,
strain in the P, molecules where the angles are only to +1 in NaH,PO,,.
60°. Its ignition temperature is very low (303 K) and
therefore, it catches fire in air to form dense white Strike anywhere matches
fumes of P,O,,. Therefore, it cannot be kept in air The sulphide, P,S, is used in strike anywhere
to form dense white fumes of P,O,,. It is generally matches. The matches contain P,S,, KCIO,, fillers
stored under water. and gelatin as a binder. The frictional heat ieaeeen
P, #50, — > PO, the match stick and the sand paper on the side of
(ix) It combines with metals forming their the match box initiates a violent reaction between
phosphides such as Na,P,Mg.P.,, Ca,P,, Ag.P, Cu,P.), ete. P,S, and KClO,. This generates sufficient heat
12Na + P, —+> 4Na,P which makes match stick to burst into flames.
Sodium phosphide
2. Red phosphorus. It is obtained by heating white
12Ag + P, —~+ 4Ag,P
phosphorus in an inert atmosphere (out ofcontact ofair) at
Silver phosphide
573 K for several days.
6Mg + P, —~+ 2Mg,P,
573 K
Magnesium phosphide Ps) inert atmosphere P, (s)
The reaction with sodium or potassium is very White phosphorus Red phosphorus
vigorous. Like white phosphorus, red phosphorus also exists as
(x) It is a weak reducing agent and reduces
P,, tetrahedra but have polymeric structure consisting of
sulphuric acid to sulphur dioxide, nitric acid to nitrogen
P, tetrahedra linked together as shown in Fig 11.
peroxide, etc.
P, + 10H,SO,—~ 10SO, + 4H,PO, + 4H,O
Phosphoric acid
P,+ 20HNO, —-> 4H,PO, + 20NO, + 4H,O
It also reduces solutions of copper, silver and gold
salts to their corresponding metals.
=<)» papas
P,+ 3CuSO, + 6H,0 ——> Cu,P, + 2H,PO.,+ 3H,S0,
Copper Fig. 11. Structure of red phosphorus.
sulphate
In this structure, P, molecules are linked by
Cu,P, + 5CuSO, + 8H,0 > 8Cu + 5H, SO, + 2H,PO,
covalent bonds. The important characteristics of
or P, + 8CuSO, + 14H,O > 8Cu + 8H,SO, + 2H,PO, red phosphorus are :
+ 2H,PO,
(i) It is a hard crystalline solid without any
Similarly,
smell. It posesses iron grey lustre.
P, + 20AgNO, + 16H,O——> 20Ag +4H,PO,+ 20HNO,
(iz) It is non-poisonous in nature.
(xi) It readily combines with halogens to form (iit) It is insoluble in water as well as carbon
trihalides (PX,) and on prolonged treatment forms
disulphide.
pentahalides (PX,). For example, white phosphorus
(iv) It is denser than white phosphorus.
catches fire in chlorine forming phosphorus trichloride
(PCl,) and phosphorus pentachloride (PC1,). (v) It does not glow in the dark.
P, + 6Cl, ———» 4PCl, (vi) Red phosphorus is quite stable and its
P, + 10Cl, ——> 4PCl, ignition temperature is quite high (543 K).
(excess) It, therefore, does not catch fire easily.
(xii) White phosphorus combines with sulphur (vit) Chemically it is less reactive than white
with explosive violence forming a number of sulphides phosphorus.
such as P,5,, P,S,, P,S.,, P,S,, ete. (viit) It is a bad conductor of electricity.
SP, + 35, ——_> 8P,5, (ix) It burns with oxygen at 565 K to form
Tetraphosphorus phosphorus pentoxide.
trisulphide
P,(s) +50,(¢) —— P,O,,(s)
(xiii) On heating with caustic soda solution, it
forms phosphine. (x) Being less reactive than white phosphorus,
P, + 3NaOH + 3H,0 ——+ PH, + NaH,PoO, it reacts with halogens, sulphur and alkali
Phosphine Sodium metals only when heated forming their
hypophosphite corresponding salts.
p-BLOCK ELEMENTS
As)
P, + 6cl, 8%, 4PCl, (iv) It is a moderate conductor of heat and
electricity.
P, + 10Cl —“> 4PCl, (v) Itis thermodynamically the most stable and
4P, + 3S, =, sPs, inactive form of phosphorus. It does not burn
in air upto 673 K.
P, + 12Na 2, 4Na,P Thus, the three allotropes of phosphorus differ in
(xi) Red phosphorus does not react with caustic their chemical reactivity. White phosphorus is the
alkalies. This property is made use in
most reactive form while black phosphorus is
separating red phosphorus from white
the least reactive form. Therefore, white phosphorus
phosphorus.
is stored under water to protect it from air while red
(xii) Red phosphorus can be converted into
white phosphorus by boiling it in an inert and black phosphorus are stable in air.
atmosphere and then condensing the vapours REMEMBER
of white phosphorus formed under water.
3. Black phosphorus. Black phosphorus has two @ White P is most reactive form
forms; o-black phosphorus and f-black phosphorus, while
o-black phosphorus is formed by heating red e Black P is the least reactive form.
phosphorus in a sealed tube at 803 K.
The main characteristics of three forms of
80 3K phosphorus are summed up in Table 7.
Red phosphorus —[aicqiube’ o-Black phosphorus
Table 7. Some physical properties of three
It can be sublimed in air and has opaque monoclinic forms of phosphorus.
or rhombohedral crystals.
Properties White Red Black
B-Black phosphorus is obtained by heating white phosphorus phosphorus phosphorus
phosphorus at 473 K under very high pressure
White but
(4000-12000 atm) in an inert atmosphere. turns
yellow on
A735 K
White phosphorus ——— eS EE
4000-1200 atm. pressure
exposure

B-Black phosphorus State Waxy solid Brittle powder Crystalline


Density 1.84 ¢ em 2.1¢cem? 2.69 gem?
It has a double layered crystal lattice. Each layer is
Ignition 307 K 543 K 673 K
made up of zig-zag chains with P—P—P bond angles of temperature
99° (Fig. 12) and P—P bond distance of 218 pm. Since
stability Less stable at More stable at Most stable
it is highly polymeric, therefore, it has high density. ordinary ordinary
The important characteristics of black phosphorus temperature temperature
are : Very reactive Less reactive Least reactive
(4) It has a black metallic lustre.
(it) It has sharp melting point of 860 K. Its
Phosphatic Fertilizers
specific gravity is 2.69.
To increase the fertility of soil certain chemical
(iit) It has a highly polymeric layered structure compounds known as fertilizers are used. These
and exists in three crystalline and one provide the essential nutrients, potassium, nitrogen and
amorphous forms. Therefore, it has high phosphorus. The most important phosphatic fertilizer is
superphosphate of lime Ca (H,PO,),. This is produced
density.
directly from phosphate rocks by treatment with H,SO,.
This makes insoluble phosphate rock soluble in water to
| | |
PN P
pee P pS P pe improve the release of phosphorus to the soil for uptake
by plants.
ae | | | Ca,(PO,), + 2H,SO, —~> Ca(H,PO,), + 2CaSO,
Fe de i a The phosphatic rock generally contains fluorides which
| | | | react with H,SO, to give HF which in turn forms other side
products. The gaseous side products are removed by washing
pa PPN pe PPN p
PN P
PN with water in a scrubber. About 90% of phosphate rock
mined is used for the production of phosphatic fertilizers and
gy gg gO tO tgp Py the remaining 10% is used for the production of elemental
phosphorus. Treatment of phosphate rock with phosphoric
| | | | acid gives triple super phosphate, Ca(H,PO,).H,O. It is
Fig. 12. Structure of black phosphorus. free from calcium sulphate and therefore, contains larger
percentage of phosphorus :
sa
Ca,(PO,),F + 7H,PO, + 5H,0—> 5Ca(H,PO,),.H,O + HF
MODERN'’S abc + OF CHEMISTRY-XI

The impurity PH, may also be controlled by using


Phosphate Esters alcoholic solution of potassium hydroxide in place of
Phosphate esters are important 1n life processes. The aqueous solution of caustic soda.
most important of these biomolecules are DNA, RNA,
adenosine mono-, di- and tri-phosphates (AMP, ADP and Structure
ATP). These are discussed in Unit 14. These contains
P-O-P links, the hydrolysis of these links releases energy Like ammonia, phosphine has pyramidal structure.
which can be put to useful work. Phosphorus involves sp* hybridisation. Three bonds
are formed by the overlap of sp? hybrid orbitals of
PHOSPHINE
phosphorus with 1s-orbital of hydrogen. One of the
It is hydride of phophorus, PH.
orbital is occupied by a lone pair of electrons. The HPH
Preparation
bond angle in PH, is 93.6° and P—H bond distance
(4) From phosphides. By the action of water or
142 pm.
dilute mineral acid on metallic phosphides (Ca.,P.,),
AIP) etc.)
Ca,P, + 6H,O ——+ 3Ca(OH), + 2PH,
Calcium phosphide
Ca,P, + 6HCl —~— 3CaCl, + 2PH,
Calcium phosphide Ft
Ren

AIP + 3HCl ——} AICI, + PH, 93.6° 4


phosphide H H
(71) From phosphorous acid. Pure phosphine
Fig. 13. Structure of phosphine
can be prepared by heating phosphorous acid at
478 — 483 K.
PROPERTIES OF PHOSPHINE
4H.PO, SiS, —«8H,PO, + PH, Physical properties
Phosphorous acid Phosphoric acid (4) It is a colourless gas with unpleasant smell
(iit) From phosphonium salts. Pure phosphine of rotten fish or garlic.
can also be obtained by heating phosphonium iodide (iz) It is highly poisonous in nature.
with caustic soda solution. (iit) It is heavier than air (vapour density = 17)
PH,I +NaOH -——> Nal + H,O + PH, and is slightly soluble in water.
Phosphonium iodide is obtained from phosphorus. (iv) On cooling to 185.5 K, phosphine condenses
P, + 21, + 8H,0—> 2PH,I + 2HI+ 2H,PO, to a liquid and on cooling to 139.5 K, it
(iv) Laboratory preparation. Phosphine is solidifies.
prepared in the laboratory by heating white phosphorus (v) It is slightly soluble in water. The
with concentrated sodium hydroxide solution in an solution of PH, in water decomposes in
inert atmosphere of carbon dioxide or coal gas. presence of light giving red phosphorus
4P + 3NaOH + 3H,0 —-> 3NaH,PO, + PH, and hydrogen.
Sodium
hypophosphite Chemical properties
It may be noted that pure phosphine is not 1. Action towards litmus. It is feebly basic and
inflammable but the gas may catch fire in air. This reacts with halogen acids to form a phosphonium salt
may be due to the presence of impurity of P,H, which (basic nature).
is spontaneously inflammable. Therefore, a current of PH, + HX — > PH*,x" (X = Cl Br, D
carbon dioxide, coal gas is passed through the flask Phosphonium halide
to displace air. e.g. PH, + HCl —~+ PH,Cl
To get pure phosphine, the gas is passed through Phosphonium chloride
a U-tube placed in a freezing mixture. The liquid P,H, PH, + HEr—-> PH,Br
gets condensed while PH, remains unaffected. Impure
Phosphonium bromide
gas may also be purified by treating it with hydrogen
Po, + Hl. — > Fil
iodide followed by heating with KOH solution.
Phosphonium iodide
PH, + HI ee PH,!
Phosphonium iodide 2. Combustibility. Pure phosphine is not
combustible. But when heated to 423 K in air or
PH,I +KOH —-> PH, + KI + H,O oxygen, it burns forming phosphoric acid.
p-BLOCK ELEMENTS

Amixture of calcium carbide and calcium phosphide


maar
PH, + 20, 28%,
423 K
HPO, is taken in a vessel. A hole is made in it and the vessel
3. Decomposition. By heating in the absence of is Immersed in the sea water near the rock. While
air at 713 K or by passing an electric spark through calcium phosphide liberates phosphine and calcium
it, phosphine breaks into its elements. carbide liberates acetylene.
Ca,P, + 6H,0 —— 3Ca(OH), + 2PH,
APH
3
713 K
absence of air P4 + 6H, CaC, + 2H,O —~ Ca(OH), + C,H,
4. Action with chlorine. When heated in the Phosphine catches fire in air and lights up
atmosphere of chlorine, phosphine burns forming acetylene. This acts as a signal for the approaching
ship.
phosphorus trichloride (PCl1,) or pentachloride (PC1,).
(ii) It is used to prepare Smoke screens in
PE, + 3ClL => PCl, 3 + 8HCl
warfare. Calcium phosphide reacts with water to
PH, + 4Cl, —-> PCl. + 3HCl form phosphine which burns in air to form P,O,,
5. Action with nitrous oxide. When sparked which acts as a smoke screen.
with nitrous oxide, it reduces the latter to nitrogen. Comparision between Phosphine and
PH, + 4N,0 — > H,PO, + 4N, Ammonia
6. Action with nitric oxide. Nitric oxide is Both ammonia and phosphine are the hydrides
reduced to nitrogen. of the elements N and P respectively which belong
PH, + 4NO —-> HPO, + 2N, to the group 15 of the periodic table.
7. Precipitation reactions. When bubbled Thus, these hydrides are expected to have many
through certain metallic salt solutions, phosphides common characteristics. But they differ also in some of
of the metals get precipitated. the properties. The comparison between the properties
8CuSO, + 2PH,—- Cu,P, + 3H,sO, of ammonia and phosphine is given below.
Copper sulphate Cupric phosphide Phosphine
(Black ppt.) Points of
resemblence
3HgCl, + 2PH, —~ Hg,P, + 6HCI
. Colour Colourless Colourless
Mercuric chloride Mercuric phosphide
. Decomposition Decomposes into Decomposes into
3AgNO, + PH,—~+> Ag,P + 3HNO,
by electric spark elements elements
Silver nitrate Silver phosphide
. Action with Reacts violently Reacts with
(Black ppt.)
chlorine to form PCI, chlorine to
Silver phosphide is converted to silver as follows: form nitrogen
Ag,P + 3AgNO, + 3H,O—> 6Ag + 3HNO, + HPO, trichloride
. Actions with Forms Forms
Summary of the chemical halogen phosphonium ammonium salts
properties of phosphine acids salts
. Combustibility Burns in air to Burns in air to
form phosphoric form nitrogen
seQ
acid and water
Points of
differences
1. Smell Unpleasant Characteristic
smell of ammoniacal
rotten fish.
. Density Heavier than smell.
alr.
. Solubility Sparingly Highly soluble in
soluble in water.

. Nature Non-poisonous.
poisonous.
. Action towards |Neutral. Basie.
Uses of Phosphine . Stability Less stable. More stable.
(i) It is used in preparing Holme’s signals for . Stability of salts |Less stable. More stable.
the ships to know about the position of the rocks in
the sea. HALIDES OF PHOSPHORUS
Phosphorus forms two types of halides i.e.,
REMEMBER phosphorus trihalides (covalency of P = 3) PX,(X =
Holme's signals give signals to ships in deep seas F, Cl, Br, I) and phosphorus pentahalides PX.(X = F,
and oceans about danger points. Cl, Br) (covalency of P = 5). With chlorine it forms :
a: 1. Phosphorus trichloride 2PCl, + O2
MODERN’S abc + OF CHEMISTRY-X!I

——. 2POCIl,
2. Phosphorus pentachloride. Phosphorous oxychloride
1. Phosphorus trichloride, PC, 3. Action with sulphur trioxide. It reacts
Preparation. It is prepared in the laboratory by with sulphur trioxide to form phosphorus
oxychloride.
passing dry chlorine gas over heated white phosphorus.
The vapours of PCl, distil over and are collected in a PCL. + 50, -——> POCL + 580Q,
receiver cooled by water. Phosphorus
P, + 6Cl, —— 4PCl, oxychloride
4. Action with chlorine or sulphur. It
The receiver has a calcium chloride tube attached
combines with chlorine or sulphur to form
to it which protects it from the reactions of outside
phosphorus pentachloride or thiophosphorus
moisture. It can be purified by distilling over white
oxychloride.
phosphorus to remove the excess of chlorine.
PCL, + Gi, “Sp PCL.
It can also be obtained by the reaction of thionyl
PCL. +f PSCl,
chloride with white phosphorus
Thiophosphorus oxychloride
PF; + 8SOCl, —-~ 4PCl, + 450, + 25,Cl, 5. Action with sulphur monochloride,
Thionyl chloride thionyl] chloride and sulphury]1 chloride.
Structure It is readily oxidised to phosphorus (V)
Phosphorus in PCl, undergoes sp® hybridisation. derivatives, PCl., POC], and PSCl.,.
Three of the sp? hybrid orbitals overlap with p-orbitals 8PCl,+ 8,Cl, —> PCl, + 2PSCl,
of chlorine to form three P—Cl c bonds while the fourth Sulphur
sp* hybrid orbital contains a lone pair of electrons. monochloride
Therefore, like PH,, PCl, has pyramidal structure as 8PCl, + SOCI, —+POCI, + PSCIl, + PCI,
shown in Fig. 14. Thionyl chloride
PCl, + SO,Cl, —> PCl, + SO, —

Lone pair Sulphuryl chloride


In these reactions, PCl, behaves as a reducing
agent.
P i.42, 6. Action with organic compounds. It reacts
705 » with organic compounds containing —OH
/ ‘go Cl
eroup such as acetic acid, ethyl alcohol, etc.
Cl (4) With organic acid, it forms acid chlorides.
Cl 3CH,COOH + PCl, —> 3CH,COC] + H,PO,
Fig. 14. Structure of phosphorus trichloride. Acetic acid Acetyl chloride Phosphorous acid
(it) With alcohols, it forms alkyl chlorides.
The C1—P—C] bond angle in PCI, is 100.4° which
3C,H.OH + PCl,—~> 3C,H.Cl + H,PO,
is greater than HPH bond angle in PH,(93.6°). This
Ethy alcohol Ethyl chloride Phosphorous acid
is due to stearic crowding of two large Cl] atoms in
7. Action with finely divided metals. Finely
comparison to H atoms. As expected the P—Cl bond
divided metals react on heating to form metal
is much larger (240 pm) than P—H bond (142 pm)
chlorides.
because of larger size of C] atom.
Properties of Phosphorus Trichloride 12Ag + 4PCl,—*" 12AgCl + P,
Physical Properties
(4) Itis acolourless oily liquid fuming constantly
6Na + PCl, —Heat, 3NaCl + Na,P
is the moist air. 8. PCl, undergoes many substitution reactions
(ii) Its specific gravity is 1.6. and is main source of organophosphorus
compounds.
(iit) Its boiling point and freezing point values
are 347 K and 161 K respectively. Uses
(iv) It has a highly pungent smell. It is mainly employed in the organic chemistry as
Chemical properties an important reagent to replace the hydroxyl group
(-OH) by chlorine atom in organic reactions.
1. Action with water. It fumes in moist air
and reacts with water violently to form 2. Phosphorus pentachloride, PC1,
phosphorous acid. Preparation. Phosphorus pentachloride is
PCl, + 3H,O — > H,PO, + 3HCl prepared by the reaction of white phosphorus with
Phosphorous acid excess of dry chlorine.
2. Action with atmospheric air or oxygen. P, + 10Cl, —> 4PCl,
It slowly combines with dry oxygen to form (Excess)
phosphorus oxychloride.
p-BLOCK ELEMENTS

It can also be obtained by the reaction of dry


nA)
POC], + 3H,O —-> H,PO, + 3HCl
chlorine on phosphorus trichloride. Phosphoric acid
PCl, + Cl,— > PCl, 3. Action with compounds containing
OH group. It reacts with compounds
Phosphorus pentachloride can also be prepared
containing hydroxyl groups (—OH) to give the
by the action of SO,Cl, on phosphorus.
corresponding chloro compounds in which each
P, + 10SO,Cl, —> 4PCl, + 10SO, —QOH group is replaced by a chlorine atom.
For example,
Structure (i) With organic acid
In PCl,, phosphorus undergoes sp*d hybridisation CH,COOH + £#ths\—>
and has trigonal bipyramidal structure as shown Acetic acid
in Fig. 15. CH,COCL + POCI, + HCl
Acetyl chloride
(it) With ethyl alcohol
C HOT at BOI. —>
Ethyl alcohol
CF HACL+ POCL + HCl
Ethyl chloride
(iit) With sulphuric acid
HO—SO,—OH + 2PCl, —>
Sulphuric acid
Fig. 15. Structure of PCl,. C1ISO,Cl + 2POCI, + 2HCl
Sulphuryl] chloride
It has three equatorial P—C] bonds and two axial 4, Reaction with phosphorus pentoxide and
P—C]l bonds which are different. Therefore, all the sulphur dioxide. PCl, reacts with P,O,,
five P—Cl bonds are not equal. The axial bonds are forming phosphorus oxychloride and with SO,
larger than equatorial bonds as : forming phosphorus oxychloride and thionyl
P—C] (axial) = 219 pm, P—Cl (equatorial) = 204 pm
chloride.
6PCl. +P,0,,—— 10POCI,
It may be noted that in gaseous and liquid phases, Phosphorus
PCl. has trigonal bipyramidal structure. However, oxychloride
inthesolidstate, it exists as an ionic solid [PCl,]* PCl. + 50, — > _ POCI, + SOCI,
PEC: Phosphorus Thionyl
In this, the cation [PCl1,]* is tetrahedral and the oxychloride chloride
anion [PC],] is octahedral. 5. Reaction with phosphorus pentasulphide.
PCl,. reacts with phosphorus pentasulphide to
In solution, it exists either molecular or ionically form phosphorus thiochloride.
dissociated depending on the nature of the solvent. 6PCl. + Pan —> 10PSCl,
Properties of Phosphorus pentachloride, PC1, Phosphorus Phosphorus
pentasulphide thiochloride
Physical
(4) Itis a yellowish white powder having pungent 6. Reaction with metals. PC], reacts with finely
smell. divided metals on heating to give corresponding
(it) It fumes in moist air due to its strong affinity chlorides.
for water. 2Ag + PCl —~+> 2AgCl + PCl,
(iit) On heating, it sublimes at 433 K and can Sn + 2PCl. ——~ SnCl, + 2PCl,
be melted (m.p. 318 K) only by heating it 7. Reaction with chloride ion acceptors.
under pressure. PCl1, forms addition products with chloride ion
Chemical properties acceptors such as boron trichloride containing
1. Stability. PCl, is thermally less stable than tetrahedral [PCl1,]* species.
PCl,. On heating it sublimes but decomposes on PCl, + BCl, ——> _ [PCl,]* [BCL]
stronger heating into trichloride and chlorine. PCl., + NbCl, ——~ [PCI,]* [NbCl,]-
PCl. =~ PCl, + Cl, 8. Reduction. Phosphorus pentachloride is
reduced with hydrogen to form PC1,.
2. Action with water. In moist air, it
hydrolyses to POCI, and finally gets PCl. + 2H —> PCl, + 2HCl
converted to phosphoric acid. 9. Reaction with potassium fluoride. With KF,
PCI, + H,O —-> POGL - 2HCl PCl. forms potassium phosphorus hexafluoride,
Phosphorus oxychloride K* (PFI.
=| siaon PCl + 6KF ——> K'IPF,J + 5KCI
MODERN'’S abc + OF CHEMISTRY-XI

Potassium phosphorus
hexafluoride
Uses
Itis extremely useful in organic reactions to replace
a hydroxyl group (—OH) by chlorine atom such as for
the synthesis of C,H.Cl, CH,COCI, ete.

REMEMBER
Pentahalides are thermally less stable than trihalides. For Fig. 16. Structure of P,O,.
example, PCl, exists as molecules in the gaseous state
but exists as [PC1,]* [PCl,]- in the crystalline state. PBr, 2. Phosphorus (V) oxide (P,,0,,)
and PI, also exist in the ionic forms as [PBr,]* Br and Preparation
[PI,]*I respectively in the solid state. It is prepared by heating white phosphorus in
excess of alr.
OXIDES OF PHOSPHORUS P,+ 50, (excess) ———+ P,O,,
Phosphorus forms two common oxides namely Properties
(1) phosphorus trioxide (P,O,) and (it) phosphorus (4) It is snowy white solid.
pentaoxide (P,0,,) (iz) Action with water. It readily dissolves in
1. Phosphorus (IIT) oxide (P,,O,) cold water forming metaphosphoric acid
Preparation P,O,, + 2H,O t——+ 4HPO,
Phosphorus trioxide is formed when phosphorus (cold)
Metaphosphoric acid
is burnt in a limited supply of air.
With hot water it gives phosphoric acid.
P, + 30, (limited) ——-> P,O,
P,O,, + 6H,0 ——— 4H,PO,
Properties
(hot) Phosphoricacid
(1) Phosphorus (IIT) oxide is a crystalline solid
Git) Dehydrating nature. Phosphorus
with garlic odour.
pentaoxide has strong affinity for water and
(it) It is soluble in carbon disulphide, ether and
therefore, acts as a powerful dehydrating
chloroform.
agent. It extracts water from many organic
(111) Heating in air. On heating in air, it forms and inorganic compounds including sulphuric
phosphorus pentaoxide.
acid and nitric acid.
PO,+ 20, — > Pa
Phosphorus (V) oxide
(iv) Action of water. It dissolves in cold water
H,SO, “us
P,O
SO,
to give phosphorous acid. P,O,, N,O,
2HNO, inp
P0.+ 6h, — > 4H.PO,
(cold) Orthophosphorous acid 2HCIO, —4-", C10,
It is, therefore, considered as anhydride Perchloric acid = ~—Chilorine (VII) oxide
of phosphorous acid.
With hot water, it gives phosphoric acid and ve P.O,
CH,CONH, —~y79? CH,CN
inflammable phosphine. Acetamide , Methyl cyanide
P,O, + 6H,O (hot) Age % SHPO, + PH,
Phosphoric acid Structure. Its structure is similar to that of P,O,.
In addition, each phosphorus atom forms a double
(v) Action with chlorine. It reacts vigorously
bond with oxygen atom as shown in Fig. 17.
with chlorine to form a mixture of phosphory]
chloride and metaphosphoryl] chloride.
P,O, + 4Cl, a “POCI, + 82PO0,Cl
Phosphoryl Metaphosphoryl
chloride chloride
Structure. Each atom of phosphorus in P,O, is
present at the corner of a tetrahedron (just as P, units
in elemental phosphorus). Each phosphorus atom is
covalently bonded to three oxygen atoms and each
oxygen atom is bonded to two phosphorus atoms.
This is shown in Fig. 16. It is clear from the structure
that the six oxygen atoms le along the edges of the
tetrahedron of P atoms.
p-BLOCK ELEMENTS 11A/31 ——

Phosphorus also forms phosphorus tetraoxide, P,O,. It 3Ba(OH), + 2P, + 6H,0 ——2PH,
Heat
——> + 3Ba(H,PO,),
is prepared from P,O, as |
ppt
4P,O, "440°C, 3P,0, + P, Ba(H,PO,), + H,SO, —— BaSO, + 2H,PO,
Red ce ae Barium sulphate is separated by filtration and
It is a colourless solid. solution is concentrated when hypophosphorous acid
Both P,O, and P,O,, are acidic oxides and they react} separates out.
with aber to form aqueous solutions of phosphorous It has one P=O, one P—OH and two P—H bonds.
acid and phosphoric acid, respectively.
P,O, (s) + 6H,O (1) —> H,PO, (aq) O
Phosphorous acid |
P,O,, (s) + 6H,O (1) —> H,PO, (aq) P
Phosphoric acid
It readily reacts with water to give phosphorous acid % on
(H,PO,) and phosphoric acid (H,PO,). H H
P,O,+6H,O —~+> 2H;PO, + 2H,PO, Phosphinic acid
Phosphorous Phosphoric '
or Hypophosphorous acid
a: ane Basicity = 1
Its structure is similar to P,O, but in addition, two P y=
atoms form coordinate bonds with their lone pairs of Since it has only one P—OH bond, so it has only
electrons to two O atoms. In P,O,,, all the four P atoms} one ionizable hydrogen atom. Therefore, it behaves
form coordinate bonds. as monobasic acid, ionising as :
BO, —— Ht + H,P0,
OXOACIDS OF PHOSPHORUS The other two hydrogen atoms directly bonded to
P are not ionizable. Due to the presence of two P—H
Phosphorus forms a number of oxoacids as given bonds, the acid as well as it salt behave as strong
in Table 8. reducing agents. Therefore, it is used in organic
Table 8. Oxoacids of phosphorus chemistry to reduce arene diazonium salts to arenes.
Oxidation On heating to 313 K it decompose to phosphine
Formula 915K
state of P 3H,PO, ~"=, PH, + 2H,PO,
Phosphinic acid Phosphine Phosphorous acid
(or Hypophosphorous acid) However, on heating above 333K, it decomposes
Phosphonic acid to phosphoric acid and phosphine
(or orthophosphorous acid oH po. 7222K%, HPO + PH
or phosphorous acid) = aii Phosphore rr
Pyrophosphorous acid Phosphine
Hypophosphoric acid (74) Orthophosphorous acid or Phosphonic acid,
Orthophosphoric acid (or H,PO;,.
phosphoric acid) Preparation. It is prepared by the hydrolysis of
Iplingnhamasaa phosphorus trichloride or trioxide.
(Pecrhpechareeee) PCl, + 3H,O —+ H,PO, + 3HCl
P,O, + 6H,0 —~+ 4H,PO,
Metaphosphoric acid
It has one P=O, one P—H and two P—OH bonds.
Peroxophosphoric acid
O
REMEMBER
The P—OH group in oxoacids of phosphorus is ionisable a
and gives proton. Therefore, number of P—OH groups i“ \ H
determines the basicity of the oxoacid. HO OH
. Some important oxoacids of phosphorus are Phogphpnie acid oF
discussed below a Orthophosphorous acid
(7) Phosphinic acid or Hypophosphorous Basicity = 2
acid, H,PO,
Preparation. It is prepared by heating phosphorus Since it has two P—OH bonds, so it has two
with alkalies like barium hydroxide when precipitate 1onisable H atoms. Therefore, it behaves as dibasic
of barium phosphate is formed. The precipitate is or diprotic:
separated and heated with a calculated amount of H,PO, —— H*+H,PO,; K, =1x 10°
sulphuric acid. H,PO; == H*+HPO2 K, =2x 107
Sins It gives two series of salts such as NaH,PO, and
MODERN'’S abc + OF CHEMISTRY-XI

OH OH
Na,HPO,, known as sodium dihydrogen phosphite and
disodium. hydrogen phosphite respectively. |e BE
On heating it decomposes into phosphoric acid
and phosphine :
OH OH
4H,PO, “°“*, 3H,PO, + PH, It contains four P—OH bonds, two P=O bonds
It is readily oxidised by air in the presence of a and one P—O—FP bond. Since it contains four OH
catalyst iodine into croups, it is tetrabasic. It dissolves in water to
phosphoric acid. form orthophosphoric acid.
Iodine
2H,PO, + O, ———> 2H,PO, HP.0, + Ho 3225 2H.PO,
It is a strong reducing agent. Orthophosphoric acid
(iit) Orthophosphoric acid, H,PO, Upon strong heating, it decomposes to form
meta phosphoric acid
Preparation. It is prepared by adding phosphorus
pentaoxide to water and boiling the solution. H,P,0, 22%; oHPo, + H,O
P,O,, + 6H,O —— 4H,PO, Metaphosphoric acid
A small amount of metaphosphoric acid may also (v) Metaphosphoric acid, HPO,
be formed but it changes readily into phosphoric acid It has one P—OH bond and therefore, exists as
on boiling. monobasic.
It can also be prepared by heating phosphorus Preparation. It is prepared by heating
with conc. HNO, or by dissolving PCI, in water orthophosphoriec acid to 525 K.
525 K
P, + 20HNO, —“> 4H,PO, + 20NO, + 4H,O Heeo, HPo, + H,O
Orthophosphoric Metaphosphoric
PCI, + 4H,0 2". H,PO, + 5HCI
acid acid
It has one P=O and three P—OH bonds. However, it may be noted that metaphosphoric
acid does not exist as simple monomer, rather it exists

| as cyclometaphosphoric acid or polymetaphosphoric

O
/>ou r

OH
SNS
Orthophosphoric “| ‘OH
UAN

NO
acid
Basicity = 3

Since it has three P—OH bonds, so it has three A \y


ionisable H atoms. Therefore, it behaves as tribasic
Trimetaphosphoric acid
or triprotic and ionises in three stages :
(HPO,),
H,PO, —— H* + HPO, K, = 7.5 x 10%
H,PO, —__
== H*+HPO,”
+ 2-
K, == 6.2 x 10° O OH O

HPO,” ——= H*+P0, K, “3 =10x 10%


|
Therefore, it forms three series of salts such as
NaH,PO, (sodium dihydrogen phosphate), Na,HPO, \o/ IN AINoIN
(disodium hydrogen phosphate) and Na,PO, (sodium Polymetaphosphoric acid
phosphate). (HPQO,),,
(itv) Pyrophosphoric. acid, H,P,0,
It contains phosphorus in +5 oxidation state.
The basic structural and characteristic
Preparation. It is prepared by heating features of these phosphorus oxoacids are :
orthophosphoric acid to 523K.
(z) All these acids contain phosphorus atom or
atoms bonded tetrahedrally to four other atoms or
oH.po, “88, HPO, + H,O
groups.
Pyrophosphoric
(it) These contain at least one PO unit and one
acid
P—OH group. The P—OH group is ionizable giving
The acid is formed by the loss of a molecule of proton. The number of P—OH groups determines
water from two molecules of phosphoric acid and has the basicity of the oxoacid.
the structure :
p-BLOCK ELEMENTS
a9)
in +8 oxidation state of P tend to disproportionate
i|
P ——.
|
P + H*
to higher and lower oxidation states. For example,
ortho phosphorous acid (or phosphorous acid) on
heating disproportionates to give ortho phosphoric
acid (P = +5) and phosphine, PH, (P= —3).
ner x — \ +3 +5 =
(zit) Some oxoacids have one (or more) P—H 4H.PO, — 3H,PO, ee he
eroups and this bond is non-ionisable. Orthophoesharsus acid Orthostocphacs acid Phnephive
O (vit) Acids which contain P—H bonds have strong
reducing properties. For example, hypophosphorous
P. acid is a good reducing agent because it contains
two P—H bonds and therefore, reduces AgNO, to
metallic silver.
(zv) Some oxoacids have P—O—P or P—P bonds 4AgNO, + 2H,O + H,PO, —> H,PO, + 4Ag + 4HNO 3
obtained by corner sharing of tetrahedra.
It is used in organic chemistry to reduce arene
O O diazonium salts to arenes.
| |
Je
C,H,N,Cl + H,PO, + H,O —>
Benzene
diazonium
chloride C,H, + H,PO, + N, + HCl
Benzene
(v) Peroxo acids have P—O—O—H or P—O—
O—P linkages. REMEMBER
(vt) The oxoacids in which P has lower oxidation Peroxomonophosphoric acid (H,PO,.) and
state (less than +5) contain in addition to P—OH and peroxodiphosphoric acid (H,P, O;) contain —O-—O-
P=O bonds, either P—P (e.g., in H,P,O,) or P—H
linkages.
(e.g., in H,PO,) bonds but not both. These oxoacids

The structures of different oxoacids of phosphorus are given below :

! —
molecules |
FP
||

/\
OH oO
us‘on OH
OH OH
OH

Metaphosphoric Mi WA acid Diphosphoric acid


acid HPO, (P=+5) (or Pyrophosphoric acid)
HPO, (P = +5) (Tribasic) H,P,O, (P = +5)
(Monobasic) (Tetrabasic)

+[O] 9] |40

Lean
OR. —OH 4 OH OHA . on
OH OH
ete Anosphoric:acid prackeanietn acid Hypophosphoric acid
HPO, (P = +5) H,P,0,(P = + 5)
(Tribasic) (Tetrabasic) (Tetrabasic)

| ee
molecules 1 |

Ne / Ne H,0- AW’
VWw™
Hypophosphorous acid ene aby Pyrophosphorous acid
or Phosphinic acid or Phosphonic acid
H,PO, (P = +1) HPO, (P = +3) (Dibasic)
(Monobasic) (Dibasic)

WW.JEEBOOKS.IN
a 11A/34 MODERN'S abc + OF CHEMISTRY-XI

The formulae of oxyacids of P can be remembered as :


The prefix
@ meta acid is used for the acid obtained by the loss of one water molecule.
® pyroacid is used for the acid obtained by heating two molecules with loss of one water molecule.
@ ‘hypo' is generally used for the acid having lower oxygen content than the parent acid.

The main oxoacids of phosphorus acid are summed up below :

SUM UP

Oxoacid Formula Characteristic bonds Method of preparation

Hypophosphorous HPO, OneP—OH P, + 40H +4H,0 —-omeaomy? 4H,PO; + 2H,


or Phosphinic Two P—H
One P=O

Orthophosphorus EP Two P—OH PO. 4 6h Af AH.PO,


or Phosphonic One P—H PU), + dH,0 Sa WE ee SE
One P=O

Pyrophosphorus Lila BE ds ae PCL, + 204088 LP


oO ——

Two P=O
One P—O—P

Hypophosphoric ea cao Four P—OH 2P+ 2NaClO, + 8H,O ——+ Na,H,P,O,.6H,O + 2HCl
Two P=O Sod. salt of
One P—P hypophosphoric acid

Orthophosphoric Als 0 Three P—OH Peg 7cH,O —> 4H,PO,


One P=O P, + 20HNO, Ji Tg Eee aif ae
PCl, + 4H,O Heat . H,PO, + 5HCl

Pyrophosphoric H,P,0. FourP—OH 2H,PO, Se 2 EEO


Two P=O
One P—O—P

Metaphosphoric HPO, Three P—OH [sel mA _525 K HANSJe oe Tal


Three P=O
Three P—O—P
igr (HPO,),

Solution : On heating H,PO, disproportionates to give


SOLVED EXAMPLES phosphoric acid and phosphine.
4H,PO, ———» 3H,PO, + PH,
_} Example 15
L] Example 17.
In what way it can be proved that PH, ts basic in
Write balanced equations for the following reactions:
ture.
ee
Solution : PH, reacts with
ae
acids like HI to form
(a) P,+NaOH+H,O ——>
Tripura S.B. 2016,
(D.S.B. 2009,
Assam S.B. 2016)
phosphonium iodide, PH gl. (Assam S.B. 2018)
PH, + HI ———+ PH,] (b) As,+ Cl,(excess) ——+>
Phosphonium iodide (c) fk dg —=2
This shows that PH, is basic in nature. This basic (d) Ca,P, +H,0 ——>
nature of PH, is due to the presence of lone pair on (D.S.B. 2008, 2014)
phosphorus atom and therefore, it acts as a Lewis base. (e) POU! 45,0 ——>
_}] Example 16 (f) HgCi,+PH, ——>
Write the disproportionation reaction of HPO . | (A.LS.B. 2010)
(Assam S.B.2013) © ear. (A.LS.B. 2014)
p-BLOCK ELEMENTS

Solution : _L] Example 21.


ADS)
(a) P,+3NaOH+3H,O——-> PH, + 3NaH, PO, HPO, is diprotic acid. Explain.
Phosphine
(b) As,+10Cl, ——> 4AsCl, (Ar.S.B. 2005, H.P.S.B. 2015, Pd.S.B. 2013, 2017)

(c) P,O,,+ 6H,O ——-+ 4H,PO, Solution : H,PO, has three H atoms and therefore,

(d) Ca,P,+6H,O ——-> 2PH,+3Ca(OH), it 1s expected to be tribasic. However, in its structure,


(ec) POCI,+3H,0 ——-> H,PO,+3HCl two hydrogen atoms are joined through oxygen atoms
and are ionisable. The third H atom is linked to P and is
() S8HgCl,+2PH, ———-> Hg,P,+6HCI non-lonisable.
(g) 2Ag+PCl,——+2AgCl+ PCl,
_}] Example 18. | H,PO, ——— HPO,2-+ 2H*
Why does PCI, fume in moisture ?
N.C.E.R.T) (Hr.S.B. 2015, 2017, Pa \e
HO H
Kerala S.B. 2016)
L] Example 22.
Solution : PCl, gets hydrolysed in the presence of
moisture and gives fumes of HCl. How do you account for the reducing behaviour of
PC], +3H,O ——> H,PO,+3HCl HPO, on the basis of tts structure.

_} Example 19 N.C.E.R.T. (Assam S.B. 2016)


Solution : H,PO, has one P=O, one P—OH and two
Calculate the volume of 0.1 M
P—H bonds as.

|
NaOH solution required to neutralize the solution
produced by dissolving 1.1 g of P,O, in water.

M1
Solution: P,O, reacts with water to form phosphorous |
5
acid (H,PO,) as :
P,O,+6H,O —— > 4H,PO, ie) HO
HPO, is dibasic and it neutralizes with NaOH as :
Since two H atoms are bonded directly to P atom which
H,PO, + 2NaOQH ——> Na,HPO, + 2H,0 © ..(it) impart reducing character to the acid.
Multiply eqn. (iz) by 4 and add to eqn. (z): _] Example 23.
P,O, + 8NaOH ——-> 4Na,HPO, + 2H,0 What happens when white phosphorus is heated
4x31+6x16 8(23+16+1) with concentrated NaOH solution in an inert
= 220¢ = 320¢ atmosphere of CO, ?
Now 220 g of P,O, require NaOH = 320 g N.C.E.R.T.
1.1 g of P,O, will require NaOH for neutralization Solution : Phosphine is formed.

- 320 44-169 P, + 3NaOH + 3H,O —““> PH, + 3NaH,PO,


220 Phosphine Sodium
hypophosphite
Wt. of NaOH present in 1000 mL of 0.1 M NaOH
— 4 g L] Example 24.

”. 4g of 0.1 M NaOH is present in = 1000 mL What is the basicity of H,PO,?

”. 1.6 g of 0.1 M NaOH is present in N.C.E.R.T\(A.1.S.B. 2014, H.P.S.B. 2017)


Solution : H,PO, contains three P—OH bonds and
1000 x1.6 therefore, its basicity is three.
4 ~~ 7 = 400 mL O
Vol. of 0.1 M NaOH required = 400 mL.
_} Example 20
All the five bonds in PCl, are not equivalent. Justify. OH
aioe ‘OH
(A.I.S.B. 2008, D.S.B. 2012,
Po. 2016, 2015, 2077) L] Example 25.
Solution : PCl, has trigonal bipyramidal structure in What happens when
which there are three P—C] equatorial bonds and two P—C]l
axial bonds. The two axial bonds are being repelled by three (i) (NH),Cr,O, is heated?
bond pairs at 90° while the three equatorial bonds are being (ii) PCl, is heated? (H.P.S.B. 2015)
repelled by two bond pairs at 90°. Therefore, axial bonds
(iti) H,PO, is heated? Write the equations involved.
are repelled more by bond pairs than equatorial bonds and
hence are larger (219 pm) than equatorial bonds (204 pm). (ALS.B. 2015, D.S.B. 2008, 2013, 2017)
save Solution :
MODERN'’S abc + OF CHEMISTRY-XI

() On heating (NH,,),Cr,O., nitrogen gas is evolved.


(NH,),Cr,0, —“ Cr,O, + N, + 4H,O
(zi) On heating, PC1, first sublimes and then decomposes Q 13.Give one example each of oxyacid of P having the
on strong heating : oxidation state
PC], —“*> PCI, + Cl, (1) +4 (11) +3
(iii) On heating, H,PO, disproportionates to give Q 14. What is the chemical formula of laughing gas ? How
orthophosphoric acid and phosphine. is it prepared ?
+3 +5 —3 Q 15. Among the hydrides of group 15, predict the hydride
4H,PO, —— 3H,PO, + Pi. having
Orthophosphoric Phosphine
(¢) most basic character
acid
_}] Example 26 (it) highest thermal stability
(it) Draw the structure of phosphinic acid (H,PO,). (zit) lowest boiling point
(it) Write a chemical reaction for its use as reducing (iv) strongest reducing agent.
agent. (CL.B.S.E. Sample Paper 2011)
Solution: (z) laa aia acid, H,PO,. ® 16. Name the oxide of nitrogen obtained in the following
reactions :
(2) Pb(NO,), —
PL 673 K
OH
(ii) NH,NO, —Heat_,
(ii) H,PO, reduces Ag*t ion to Ag which shows its
reducing nature. (iii) N,O, + NO =
H,PO, + 4AgNO, + 2H,O0 —-> 4Ag + 4HNO, + H,PO, Q 17. What is the oxidation state of phosphorus in the
_} Example 27 following :
(a) Suggest a quantitative method for estimation of i/tl.PO, (zz) PCL,
the gas which protects us from U.V. rays of the sun.
(ii) Ca,P, (tv) Na,PO,
(b) Nitrogen oxides emitted from the exhaust system
of supersonic jet aeroplanes slowly deplete the (v) POF,
concentration of ozone layer in upper atmosphere.
Comment. (C.B.S.E. Sample Paper 2011) 918. Which of the following has maximum P—OH bonds ?
Solution: (a) The gas which protects us from U.V. rays (¢) Orthophosphoric acid
of the sun is ozone. It reacts with T ions to give iodine as : (zt) Pyrophorous acid
O, + 21. + H,O ——> O, + I, + 20H (tit) Pyrophosphoric acid
I, liberated is titrated against sodium thiosulphate
@ 19. Name the oxoacid of P which
solution and amount of O, can be estimated.
(t) exists as polymeric
(6) The release of nitrogen oxides (NO_) into stratosphere
by the exhaust system of supersonic jet aeroplanes deplete (it) has basicity two
the concentration of O, because NO reacts with O, to give O,. Q 20. Name three oxoacids of P having oxidation state of
NO (g) + O, (¢) ——+ NO, (g) + O, (g) P as +5.
Therefore, NO is slowly depleting the concentration of
ozone. Answers to Practice Problems
_} Example 28
13. (i) Hypophosphoric (11) Phosphorus acid
NCl, is readily hydrolysed while NF, does not. Explain.
Solution: In NCl,, Cl has vacant d-orbitals to accept 14. N,O.
the lone pair of electrons donated by oxygen atom of H,O 15. (i) NH, (iz) NH, (diz) PH, (iv) Bi.
molecules. But in NF,, F does not have vacant d-orbitals. 16. (1) Nitrogen dioxide
Therefore, NC1, undergoes hydrolysis but NF, does not.
(it) Nitrous oxide
NCl, + 38H,O0 ——> NH, + 3HOCl
NF, + H,O ——~+ No reaction (111) Dinitrogen trioxide.
_} Example 29 17. (i) +3 (2) +3 (iz) -3 (iv) +5 (v) +5.
Among the hydrides of Group 15 elements, which have the 18. (iit)
(a) lowest boiling point? 19. (1) Metaphosphoric acid
(6) maximum basic character?
(tt) Phosphonic acid
(c) highest bond angle?
(dq) maximum reducing character? (A.J.S.B. 2018) 20. orthophosphoric acid,
Solution: (a) PH, (6) NH, metaphosphoriec acid,
(c) NH, (d) Bil, pyrophosphoric acid
p-BLOCK ELEMENTS nAs7)
add on ULGL KK uestion
s |
Concep
. Draw the structure of P,O,, and identify the number of single and double P-O bonds.

. See text. Double bonds = 4, single bonds = 12.


. Nitric oxide becomes brown when released in air.
. When nitric oxide, NO is released in air, it becomes brown due to the formation of NO, (nitrogen dioxide),
which is a brown gas.
2NO (g) + O, (g) —— 2NO, (g)
(Brown)
- Solid PCl, is ionic in nature. (A.L.S.B. 2016)
. PCl, is ionic in the solid state because it exists as [PC1,]* [PCl,]-in which the cation is tetrahedral and
anion is octahedral.
. Concentrated nitric acid turns yellow on exposure to sunlight. Why ?
. On exposure to sunlight, nitric acid decomposes into NO,, O, and H,O. The presence of NO, in the
partially decomposed nitric acid gives it yellow colour.
4HNO, (1) —> 4NO,(g) + O, (g) + 2H,0 (2)
Q.5. Why does not nitrogen form pentahalides like phosphorus ?
Ans. Nitrogen does not have vacant d-orbitals in its valence shell. Therefore, it cannot extend its valency
beyond 3. Therefore, 1t can form maximum of four bonds and cannot form pentahalides. On the other
hand, phosphorus has vacant 3d-orbitals in the valence shell and therefore, one electron can be easily
promoted from valence 3s-orbital to vacant 3d-orbital. Thus, it can exhibit pentavalency in its compounds.

Qs 2p 3s 3p od

N P OQOOOO #0
AS POOOO sass
Q.6. Determine the oxidation number of nitrogen in (7) N,O (1) NO, (ai) HNO, (iv) NH,.

Ans. (i) N,O:+1 (it) NO,: +4 (111) HNO,: +5 (tv) NH,: — 3

Q.7. How is pyrophosphoric acid related to orthophosphoric acid?


Ans. Pyrophosphoric acid (H,P,O-,) is the anhydride of orthophosphoric acid (H,PO,) and is obtained by the
removal of a molecule of water from two molecules of H,PO 4 as shown below:

prime 527K,
J HPO. + H,O
Orthophosphoric Pyrophosphoric
acid acid

Q.8. N,O supports combustion more vigorously than air. Explain.


. N,O decomposes to give O, which is about 1/3 of the volume of gases produced (2N, + O,).
Heat
2N, ——— 2N,7O,

On the other hand, air contains 1/5th part of O, of its volume. Due to larger content of O,, N,O supports
combustion more vigorously than air.
. On being slowly passed through water, PH, forms bubbles but NH, dissolves. Why?
. N—H bond in ammonia is more polar than P—H bond in phosphine. Therefore, NH, forms hydrogen
bonds with water molecules and hence dissolves in it. On the other hand, PH, does not dissolve and
hence forms bubbles.

WWW.JEEBOOKS.IN
sae Q.10. Write the reaction of thermal decomposition of sodium azide.
MODERN'’S abc + OF CHEMISTRY-XI

Ans. Thermal decomposition of sodium azide gives nitrogen gas.

2ZNaN, ———> 2Na+3N,


Sodium azide
Q.11. Phosphoric acid has high viscosity and high melting point. Why ?
Ans. Phosphoric acid has a tendency to form hydrogen bonding in concentrated solutions. Therefore, it has
high viscosity and is a syrupy liquid and has high boiling point.
Q.12. PCI, exists as [PC],] [PC1,]* but PBr, exists as [PBr,]* [Br]. Explain.
Ans. Both PCI, and PBr, have trigonal bipyramidal geometry. This is not a regular structure and is not very
stable. Therefore, PCl, splits up into more stable octahedral and tetrahedral structures, which are stable
than trigonal bipyramidal.
PCl, == [PCl,) [PC1,]*
On the other hand, PBr, splits up into stable tetrahedral structure as
[PBr,] —— [PBr,]* al
This splitting is different from PCl, because Br atoms are large and six atoms of Br cannot be easily
accommodated around smaller P atom.
» PCl, is known but PI, is not known. Why ?
. Due to small size of Cl atom, five Cl atoms can be accommodated around P atom. But I is of large size
and therefore, five I atoms cannot be accommodated around P atom. As a result, P—I bonds are weak
and prefer to form PI, rather than PI,. Therefore, PI, is unstable.
. Write the structural difference between white P and red P. (D.S.B. 2014)
. White P consists of P, units in which four P atoms lie at the corners of a regular tetrahedron with “PPP
= 60°.
Red P also consists of P, tetrahedra units but it has polymeric structure consisting of P, tetrahedra linked
together by covalent bonds.
. What is liquid nitrogen used for ?
. Liquid nitrogen is used as a refrigerant to preserve biological specimens. It is also used to provide low
temperature.

. Why does iron become passive when dipped in conc. HNO, ?


. Iron becomes passive when dipped in conc. HNO, due to the formation of a thin protective layer of
the metal oxide on its surface. This protective layer corresponds to ferrosoferric oxide, FeO.Fe,O, and
prevents further action of the metal.
. What is calcium cyanamide ? Why is it used as a fertilizer ?
. Calcium cyanamide is CaCN,,. It reacts with water to form ammonia.
CaCN, +3H,O ——>» CaCO, +2NH,
So, 1t can provide nitrogen nutrient to the soil and plants. Therefore, it is used as a fertilizer under the
name nitrolim (CaCN, + C).
Q.18. In the ring test of nitrates, what chemical compound is formed ?
Ans. [Fe(H,O), NOJSO, .
Q.19. What is azote ?
Ans, Azote is the name given to nitrogen by Lavoisier.
Q.20. What is the basicity of H,PO, acid and why ?
(A.LS.B. 2012)
Ans. H,PO, has the structure :

H yo Soe
N

It has only one ionisable hydrogen and therefore, its basicity is one.
p-BLOCK ELEMENTS 11A/39 —

Q.21. Write down the balanced chemical equation representing action of HNO, on sulphur and
iodine.
Ans. (i) Action of HNO, on S

1 (s)+6HNO, ——> H,SO, + 6NO,+ 2H,0


8 Sulphuric acid
(it) Action of HNO, on I
I,(s)+ 10HNO, ——> 2HIO, + 10NO,+4H,O
Todic acid
Q.22. Give examples of oxide of nitrogen
(a) blue liquid below 253 K (6) known as laughing gas
(c) brown gas (d) colourless gas having oxidation state of N equal
to 5
(e) prepared by heating lead nitrate.
Ans. (a) N,O, (6) N,O (c) NO, (d) N,O,; (e) NQ,.
Q.23. How would you prepare a sample of deuterated ammonia, ND, ?
Ans. It is prepared by the action of heavy water on magnesium nitride.
Mg,N, 2
+ 6D,0 ———> 3Mg(OD), + 2ND,
Deuterated ammonia
Q.24, N,O supports combustion more vigorously than air. Explain.
Ans. N,O decomposes to give O, which is about 1/3 of the volume of gases produced (2N, + O,).

2N,O —t» oN, +0,


On the other hand, air contains 1/5th part of O, of its volume. Due to larger content of O,, N,O supports
combustion more vigorously than air.
Q.25. Name the promoter used in Haber's process. (C_.B.S.E. Sample Paper 2017-18)
Ans. Molybdenum.
Q.26. Which one of PCI] and PCI; is not likely to exist and why ?
(D.S.B. 2012)
Ans. PC], is not likely to exist because lone pair on P in PCl, can be donated to Cl* and not to CI.
Q.27. Which is a stronger reducing agent, SbH, or Bil, and why ? (A.LS.B. 2072)
Ans. BiH, is a stronger reducing agent than SbH,. This is because BiH, is less stable than SbH, because of
larger size of Bi than Sb.
Q.28. NF, is an exothermic compound whereas NCI, is not. Explain. (A.LS.B. 2011, 2012)
NF, is an exothermic compound while NCI, is not because :
(¢) The bond dissociation enthalpy of F, is lower than that of Cl,.
(11) Size of F is small as compared to Cl and therefore F forms stronger bonds with nitrogen releasing
large amount of energy.
Therefore, overall energy is released during the formation of NF, and energy is absorbed during the
formation of NC].
Q.29. The bond angles (O—N—O) are not of the same value in NO, and NO,*. (D.S.B. 2072)
In NO, there is one electron on N. Therefore, in NO,*, there is no electron on N atom and there is a
lone pair of electrons on N atom in NO,-.

) nice 9
YN, IN.
NO,* is linear and has bond angle of 180°. Because of repulsions by a lone air of electrons in NO,”, the
molecule has angular shape and bond angle has been found to be 115°. Hence, the bond angles in NO,*
and NO, are different.
MODERN'S abc + OF CHEMISTRY-XI

Q.30. H,PO, is a stronger reducing agent than H,PO,.


(A.LS.B. 2014, 2016)
Ans. The reducing character of the acid is due to H atoms bonded directly to P atom. In H,PO,, there are two
P-H bonds whereas in H,PO,, there is one P—H bond. Therefore, H,PO, is a stronger reducing agent
than H,PO,.
O O

/\u / Na
HO H HO OH
(H,PO,) (H,PO,)
Q.31. Bi(V) is a stronger oxidizing agent than Sb(V). (D.S.B. 2014)
Ans. On moving down the group, the stability of +5 oxidation state decreases while the stability of +3 oxidation
state increases due to inert pair effect. Therefore, +5 oxidation state of Bi is less stable than +5 oxidation
state of Sb. Thus, Bi(V) is a stronger oxidising agent than Sb(V).
Q.32. N—WN single bond is weaker than P—P single bond. (D.S.B. 2014)
Ans. N—N single bond is weaker than P—P single bond because of high interelectronic repulsions of non-
bonding electrons due to small bond length.
Q.33. What happens when orthophosphorous acid is heated? (C.B.S.E Sample Paper 2017-18)
Ans. Phosphoric acid and phosphine are formed.
Heat
4H,PO, ee 3H,PO, + PH,

QUICK CHAPTER ROUND UP).


GROUP 15 ELEMENTS NO, (Brown gas, acidic), N,O, (colourless solid or gas,
acidic)
e General electronic configuration: ns*np*
N,O is called laughing gas and is obtained by heating
e N, P, As, Sb, Bi
NH,NO,.
e Nitrogen differs from other elements of the group due to
: Phosphorus exists as P, in elemental state and exists
O small size,
in several allotropic forms.
0 high electronegativity,
Black P is most stable and least reactive allotropic
O absence of d — orbitals in the valence shell
0 tendency to form pr—pr bonds with itself and with highly form of P. White P is least stable and most reactive.
electronegative atoms (O or C). Black P has polymeric structure.

® Elements of group 15 exhibit two oxidation states +3 P, + 30, (limited) ——-> P,O,
and +5 but +3 oxidation state is formed because of inert ce SUE tres
trsp J
pair effect. P,O, + 6H,0 (cold) ——~+ 4H,PO,
Nitrogen does not form compounds in +5 oxidation state Bee oe nt) tee Pe
such as NCI, and NF... etc, because of the absence of P,O,, + 2H,O (cold) ——-> 4H,PO,
d-orbitals in its valence shell.
Ce oe ot)
In hydrides :
Phosphorus forms a number of oxoacids. Depending
Bond angle : NH, > PH, > AsH, > SbH,
upon the number of P—OH bonds, they have different
Basic character : NH, > PH, > AsH,
basicity.
Boiling point : NH, > PH, < AsH, < SbH,
Hypophosphorous acid, H,PO, (Basicity = 1),
Stability : NH, > PH, > AsH, > SbH,
Phosphonie acid, H,PO, (Basicity = 2),
Reducing character : NH, < PH, < AsHs.
Orthophosphoric acid, H,PO, (Basicity = 3),
e NH, is manufactured by Haber’s process.
Pyrophosphoric acid, H,P,O, (Basicity = 4).
e N also forms hydrides such as hydrazine (N,H,) and
The oxoacids having P—H bonds are good reducing
hydrazoic acid (HN,).
agents.
e N,O (colourless, neutral), NO (colourless, neutral), N,O,
e In solid state PCl, exists as [PC1,]*[PCl],|-.
(pale blue solid, acidic), N,O, (colourless solid, acidic),
p-BLOCK ELEMENTS
a
\ NCERT FILE \| Solved
In-text Questions {/
. Why are pentahalides more covalent than . What is the covalence of nitrogen in N,O,?
trihalides? . Refer Solved Example 14 (Page 23).
. In pentahalides, the oxidation state is more (+5)
. Bond angle in PH, is higher than that in PH,.
than in trihalides (+3). As a result of higher positive
oxidation state of central atom, they have larger
Why?
polarizing power and can polarise the halide ion(X) to . Both PH; and PH, involve sp? hybridisation of
a greater extent than in the corresponding trihalide. P atom. In PH, all the four orbitals are bonded,
Since larger the polarisation, larger is the covalent whereas in PH, there is a lone pair of electrons on
character, therefore, pentahalides are more convalent P. In PH, the HPH bond angle is tetrahedral angle
than trihalides. of 109.5°. But in PH,, lone pair-bond pair repulsion
Q.2. Why is BiH, the strongest reducing agent is more than bond pair-bond pair repulsion so that
amongst all the hydrides of group 15? bond angles become less than normal tetrahedral
Ans. Among the hydrides of group 15, BiH, is least stable angle of 109.5°. The bond angle in PH, has been
because Bi has largest size in the group and has least found to be about 93.6°.
tendency to form covalent bond with small hydrogen
atom. Therefore, it can readily lost H atom and has H +

strongest tendency to act as reducing agent.


Q.3. Why is N, less reactive at room temperature?
Ans. Refer Solved Example 3 (Page 10) Py
Q.4. Mention the conditions required to maximise yy Ao
109.5 yy
P—H
Pas
the yield of ammonia. H H
Ans. Ammonia is formed according to the reaction: Q.8. What happens when white phosphorus is heated
N,(g) + 3H,(g) ————== 2NH,(g) AH’ = - 46.1kJ with concentrated NaOH solution in an inert
mol atmosphere of CO,?
The conditions for maximum yield of ammonia are: . Refer Solved Example 25 (Page 35).
(t) Low temperature of the order of about 700 K Q.9. What happens when PCI, is heated?
(11) High pressure of 200 x 10° Pa (about 200 atm). . Refer Solved Example 25 (Page 35).
(tit) Presence of catalyst such as iron oxide with Q.10. Write a balanced equation for the hydrolytic
small amount of K,O and Al,O,
reaction of PCl. with heavy water.
Q.5. How does ammonia react with a solution of
Cu**?
. PCl, + D,O ——> POCI, + 2DCI
Ans. Ammonia reacts with a solution of Cu** to form deep 11 What is the basicity of H,PO,?
blue coloured complex, tetraamminecopper (II) ion: . Refer Solved Example 24 (Page 35)
Cu”* (aq) + 4NH,(aqg) ———. [Cu(NH,),]?*(aq) Q.12. What happens when H.,PO, is heated?
Tetraamminecopper (II) ion . Refer Solved Example 25 (Page 35)
(deep blue)

Textbook Exercises {/
Q.1. Discuss the general characteristics of group . Nitrogen has a unique ability to form pr—pr multiple
15 elements with reference to their electronic bonds with itself and with other elements having
configuration, oxidation state, atomic size, small size and high electronegativity (e.g., C, O).
ionization enthalpy and electronegativity. Therefore, it has triple bond between two nitrogen
Ans. Refer Text (Pages 2-4). atoms (N = N) and is non-polar. Due to triple bond it
Q.2. Why does the reactivity of nitrogen differ from has very high bond enthalpy (941.4kJ) and therefore,
phosphorus? it does not react with other elements under normal
Sie conditions and 1s very unreactive. On the other hand, Q.9.
MODERN'S abc + OF CHEMISTRY-XI

The HNH angle value is higher than HPH, HAsH


phosphorus forms single bond (P—P) and is reactive and HSbH angles. Why?
In comparison to nitrogen. . All the hydrides of group (NH,, PH,, AsH, and SbH,)
Q.3. Discuss the trends in chemical reactivity of involve sp? hybridisation of the central atom in
group 15 elements. which one of the position is occupied by a lone pair
Ans. Refer Text Page 4. of electrons.
Q.4. Why does NH, forms hydrogen bonds but PH,
does not?
Ans. Because of high electronegativity and small size of
nitrogen, ammonia forms hydrogen bonds. On the (M =N, P, As, Sb)
other hand, P has low electronegativity and large H
size and hence cannot form hydrogen bonds. H H
Q.5. How is nitrogen prepared in the laboratory? Write Now, as we move down the group from N to P to
the chemical equation of the reactions involved. As to Sb, the size of the atom goes on increasing
Ans. In the laboratory, nitrogen is prepared by treating an and its electronegativity decreases. Consequently,
aqueous solution of ammonium chloride with sodium the position of bond pair shifts more and more
away from the central atom in moving form NH,
nitrite.
to SbH,. For example, the bond pair in NH, is
NH,Cl(aq) + NaNO,(aq) ———N,(g) + NaCl(aq) + close to N in N-—H bond than the bond pair in
2H, O(2) P—-H bond in PH,. As a result, the force of repulsion
It may be noted that small amounts of NO and HNO, between the bonded pair of electrons in NH, is more
are also formed in this reaction as impurities. These than in PH,. In general, the force of repulsion between
are removed by passing the gas through aqueous bonded pairs of electrons decreases as we move from
sulphuric acid containing potassium dichromate. NH, to BiH, and therefore, the bond angle also decreases
in the same order.
Q.6. How is ammonia manufactured industrially?
Ans. Ammonia is manufactured industrially by Haber’s
Q.10. Why does R,P = O exists but R,N = O does not
(R = alkyl group). (A.LS.B. 2014)
process.
Ans. RN = O does not exist because nitrogen cannot have
—.*
N,(g) + 3H,(¢) ———— 2NH,(g) covalency more than four. Moreover, R,P = O exists
° = — 46.1 kJ mol? because phosphorus can extend its covalency more
than 4 as well as it can form dz-pn bond whereas
The optimum conditions for the production of ammonia
nitrogen cannot form d1-pn bond.
are a pressure of 200 x 10° Pa (about 200 atm), at
temperature of about 700 K and a catalyst iron oxide Q.11. Explain why NH, is basic while Bill, is only
feebly basic.
with small amounts of K,O and AL,O,.
For detail refer Text (Page 13).
. Both N and Bi have a lone pair of electrons in NH,
and BiH, respectively. They can donate the electron
Q.7. Illustrate how copper metal can give different
pair and therefore behave as Lewis base. In NH,, N
products on reaction with HNO,. has small size and the lone pair is concentrated on a
Ans. Concentrated nitric acid is a strong oxidising agent and small region and electron density on it is maximum.
reacts with metals. The products of oxidation depend Consequently, it has greater electron releasing
upon the concentration of the acid, temperature and the tendency. But the size of Bi is large and the electron
nature of the material undergoing oxidation. For example, density of the lone pair is less. As a result, it has
copper reacts with HNO, giving different products as: lesser tendency to donate electron pair. Hence, NH,
Conc. HNO, gives copper nitrate and nitrogen dioxide. is basic while BiH, is only feebly basic.
Cu + 4HNO, ——>Cu(NO,), + 2NO, + 2H,O . Nitrogen exists as diatomic molecule and
cone. phosphorus as P,. Why ?
. Refer Solved Example 1 (Page 10)
Dilute HNO, gives copper nitrate and nitric oxide.
. Write main differences between the properties
3Cu + 8HNO, ——3 3Cu (NO,), + 2NO + 4H,O of white phosphorus and red phosphorus.
Q.8. Give the resonating structures of NO, and N,O,. . The main differences between white phosphorus and
NO, red phosphorus are:

White phosphorus | Red phosphorus


Colour White but turns Dark red
yellow on exposure
State Waxy solid Brittle powder
Density 1.84 g em? 2.1¢cem?
Ignition 307 K 543 K
temperature
Stability Less stable at More stable at
ordinary ordinary
temperature temperature
Chemical Very reactive Less reactive
reactivity
p-BLOCK ELEMENTS

Q.14. Why does nitrogen shows catenation properties Ans, Phosphorus can show maximum oxidation state of +5
nM)
less than phosphorus? in its compounds. In PCl,, its oxidation state is +5.
Ans. Refer Solved Example 5 (Page 11). Since it cannot increase its oxidation state beyond
Q.15. Give the disproportionation reaction of H,PO,,.
+5, 1t cannot act as a reducing agent. However, it
can act as an oxidising agent by undergoing decrease
Ans. HPO, disproportionate on heating as :
in its oxidation state from +5 to +3. For example, it
4H,PO, ———> 3H,PO, + PH, oxidises silver to AgCl, Sn to SnCl, ete.
Q.16. Can PC, act as an oxidising as well as a reducing +5 +1 +3
agent? Justify. 2Ag + PC], ———> 2AgCl + PCI,

Exemplar Problems [!
Subjective Questions
Calculate the volume of 0.1 M NaOH solution
required to neutralise the acid formed by
dissolving 1.1 g of P,O, in H,O.
. Write a balanced chemical equation for the
Ans, P AY. + 6H,O = 4H,PO,
reaction showing catalytic oxidation of NH, by
atmospheric oxygen. H,PO, can be neutralised with NaOH as :
4H,PO, + 8NaOQH —-> 4Na,HPO, + 8H,O
. 4NH, + 50, Pt/Rh gauge catalyst
500 K. 9 bar 4NO + 6H,O
PyOg + 8NaOQH ——> 4Na,HPO, + 2H,O
. Write the structure of pyrophosphoric acid. 1 mol 8 mol

O O
Moles of P,O, = Soo = 0.005 mol
| ll
: HOS an ia Pyrophosphoric acid
Acid formed by 1 mol of P,O, require NaOH = 8
OH OH mol

. PH, forms bubbles when passed slowly in water


Acid formed by 0.005 mol of P,O, require NaOH
= 8 x 0.005 = 0.04 mol
but NH, dissolves. Explain why?
0.1 M NaOH means 0.1 mol of NaOH 1s present in
. NH, forms hydrogen bonds with water and therefore
1000 mL solution
soluble in water but PH, cannot form hydrogen bonds
1000
with water and therefore, is not soluble in water. It 0.04 M of NaOH 1s present in solution = x 0.04
escapes as gas.
- In PCl1,, phosphorus is in sp*d hybridised state = 400 mL
but all its five bonds are not equivalent. Justify 8. White phosphorus reacts with chlorine and
your answer with reason. the product hydrolyses in the presence of
. It has trigonal bipyramidal geometry in which all water. Calculate the mass of HCl] obtained by
the four bonds are not equal. the hydrolysis of the product formed by the
. Why is nitric oxide paramagnetic in gaseous reaction of 62 g of white phosphorus with
state but the solid obtained on cooling it is chlorine in the presence of water.
diamagnetic? Ans. Phosphorus react with Cl, as :
. In gaseous state NO, exists as monomer which has P, + 6Cl, ——> 4PCl,
one unpaired electron. In the solid state it dimerises PCl, is hydrolysed as :
to N,O, so no unpaired electron is left hence solid
form is diamagnetic. PCl, + 3H,O0 —-+ 3H,PO, + 3HCl1] x 4
. In the ring test of NO; ion, Fe** ion reduces By CL +SHO 7 aH PO + HTC
nitrate ion to nitric oxide, which combines with
Fe** (aq) ion to form brown complex. Write the Moles of white P = Be = 0.5 mol
reactions involved in the formation of brown 1234
ring. 1 mol of white P, produces HCl = 12 mol
» NO, + 3Fe?* + 4H* — > NO + 3Fe** + 2H,O 0.5 mol of white P, will produce HC] = 12 x 0.5 =6 mol
Mass of HCl = 6 x 36.5 = 219.0 g
[Fe(H,0),]?* + NO—-+[Fe(H,0),(NO)]** + H,O
. Name three oxoacids of nitrogen. Write the
(brown complex) disproportionation reaction of that oxoacid of
. P,O, reacts with water according to equation: nitrogen in which nitrogen is in +3 oxidation
P,O, + 6H,O > 4H,PO, state.
a: MODERN'’S abc + OF CHEMISTRY-XI

Ans. Three oxoacids of nitrogen are 12 . Give an example to show the effect of
(4) HNO,, Nitrous acid (it) HNO,, Nitric acid concentration of nitric acid on the formation
(zit) Hyponitrous acid, H,N,O, of oxidation product.
Acid in which oxidation state of N is + 3, is HNO, Ans. Dilute and concentrated nitric acid give different
oxidation products on reaction with metals. For
3HNO, ——SProportionation_, HNO, + H,O + 2NO example, copper reacts with dilute and conc. HNO,
10. Nitric acid forms an oxide of nitrogen as:
on reaction with P,O,,. Write the reaction 8Cu + SHNO, (dil.) ——> 38Cu(NO,), + 2NO + 4H,O
involved. Also write the resonating structures
Cu + 4HNO, (Conce.) —-> Cu(NO,), + 2NO, + 2H,O
of the oxide of nitrogen formed.
. Reaction of nitric acid and P,O,,. 13. PCl, reacts with finely divided silver on
heating and a white silver salt is obtained,
4HNO, + P,O,, —> 4HPO, + 2N,0, which dissolves on adding excess aqueous
Resonating structures of N,O, : NH, solution. Write the reactions involved to
-_
explain what happens.

‘s
La

\yZ bee | 7 \ 7 ::
La

PCl, reacts with silver to form white silver salt


= #

Ans.
7 oe
Ns.

- 3
(AgCl). This dissolves in aqueous ammonia to form
soluble complex.
=

11. Phosphorus has three allotropic forms — (7) PCL, + 2Ag—> 2AeC] + PCI,
white phosphorus (77) red phosphorus and (777) White ppt
black phosphorus. Write the difference between AgCl + 2NH, (aq) ——> [Ag (NH3)9J" Cl
white and red phosphorus on the basis of their Soluble complex
structure and reactivity. 14. Phosphorus forms a number of oxoacids. Out
. For structures of three allotropes of P, refer chapter of these oxoacids phosphinic acid has strong
in Brief. reducing property. Write its structure and
Differences between white P and red P also write a reaction showing its reducing
White phosphorus has discrete P, units. The four behaviour.
P atoms lie at the corners of a regular tetrahedron Ans. Structure of phosphinic acid (Hypophosphorous acid)
with 4PPP = 60°. Each P is bonded to each of the is as follows:
other three P atoms by covalent bonds so that each
O
P completes its valence shell. l
Red phosphorus also consists of P, units but have P
polymeric structure consisting of P, tetrahedra linked a >
i OH
together through P—P bonds to form the chain. H
Reactivity. White phosphorus is much more Reducing behaviour of phosphinic acid is observed
reactive than red phosphorus. This is because in when it reacts with silver nitrate as:
white phosphorus, there is angular strain in P,
4AgNO, + 2H,O + H,PO,——> 4Ag + 4HNO, + H,PO,.
molecules because the bond angles are only 60°.

Memory TEstT W

8. As,O, is amphoteric while Sb,O, is acidic.


“a Say True or False
9, Phosphine is poisonous while ammonia is non-
. The covalence of N,O, is five. poisonous.
. H,PO, is a dibasic acid. 10. Oxidation state of N in peroxonitric acid is +7.
. All the five bonds in PCl, are not equal.
. NH, is less basic than PH,. i:=8| Complete the missing links
a . Both
Oo
fe
Ne pyrophosphoric acid and hypophosphoric acid 1. Calcium phosphide on hydrolysis gives ......... and
have P—O-P linkages. calcium hydroxide.
. White phosphorous is most reactive while black 3. Concentrated nitric acid on dehydration with P,O,,
phosphorus 1s least reactive. GIVES ..........
. Nitrogen cannot form compounds of the type R,N = . The formula of acidic nitrogen hydride ts ..........
O while phosphorus can form compounds of the type . In brown ring test for nitrates, Fe?* ion reduces NO,-
R,P=O. ion to ........ . which reacts with Fe** ion to form a
p-BLOCK ELEMENTS

brown ring complex having the molecular formula 14. Laughing gas is obtained on heating a mixture of
NH,Cl and.......... but nitrogen gas is obtained when
5. PCl, in solid state exists as ionic compound having a mixture of NH,Cl and ......... is heated.
cation ........ . and anion .......... 15. Hypophosphoric acid is ......... basic but
6. Nitrogen is a gas because of its tendency to form pyrophosphorous acid is ......... basic.
16. Orthophosphorous acid on heating gives ......... and

‘| Choose the correct alternative


8. In gaseous state nitric oxide is ........ . while in the
liquid or solid state it is .......... 1. Nitrogen gasis obtained by heating ammonium nitrite/
9. Among group 15 hydrides, ........ . 1s most stable. ammonium nitrate.
10. P,O, and P,O,, dissolve in water to give ......... and 2. White phosphorus is Jess/more reactive than red
phosphorus.
11. Calcium cyanamide is used as fertilizer under the
3. N,O is obtained by heating NH,NO,/ NH,NO..
12. The oxide of nitrogen obtained on heating lead nitrate 4, The polymeric allotrope of phosphorus is black/white
De casinnneans phosphorus.
13. Phosphorus reacts with nitric acid to form ........ .
acid. 5. P,O, dissolves in cold water to give H,PO,/H,PO,.

Memory Test \\
Say True or False > Complete the missing links
1. False. Maximum covalence of nitrogen cannot be more 1. phosphine
than 4 because of absence of d-orbitals. 2. N,O, 3. NH
2. False. H,PO, is monobasic because it has only one 4, NO, [Fe(H,O).NO]?* 5. PCL,*PCl—
lonisable hydrogen atom. 6. multiple bonds
3. True. 7. N,O, NO 8. paramagnetic, diamagnetic
Hee.
False. NH, is more basic than PH,. 9. NH, 10. H,PO,, HPO, 11. nitrolim
5. False. Pyrophosphoric acid has P—O-P linkage but 12. NO, 13. phosphoric
hypophosphoric acid does not have P—O-P linkage. It
14. NaNO,, NaNO, 15. tetra, di
has only P—P linkage.
So . True 7. True
16. phosphoric acid, phosphine
8. False. Acidic character decreases while basic Choose the correct alternative
character increases down the group and therefore,
As,O, is acidic while Sb,O, is amphoteric. 1. ammonium nitrite 2. more. 3. NH,NO,
9. True 10. False : Oxidation state of N is +5. 4, black 5. H,PO,

Higher Order Thinking Skills


& Advanced Level
QUESTIONS WITH ANSWERS
Q.1. Elemental phosphorus does not exist as P, molecule like P==P is not formed. Instead, it prefers
like N,. Why ? to form stable tetra atomic, P, molecules in which each P is
Ans. Nitrogen has a strong tendency to form linked to three other P atoms by three single covalent bonds.
multiple bonds because of its small size and high
The four atoms in P, molecule lie at the corners of a regular
electronegativity. Therefore, it exists as a diatomic
tetrahedron.
molecule, N=N. On the otherhand, phosphorus because of
its large size and small electronegativity does not show any Q.2. H,PO, and H,PO, act as good reducing agents
tendency to form multiple bonds and therefore, diatomic but H,PO, does not. Why?

WWW.JEEBOOKS.IN
sae Ans. The structures of H,PO,, H,PO, and H,PO, are:
MODERN'’S abc + OF CHEMISTRY-XI

Q.7. When a mixture of ammonium chloride and

| | |
potassium dichromate are heated, a stable colourless
gas (A) was evolved which did not support combustion
but magnesium continued to burn in it. The gas (A)

JK OH #F AK AK OH
fs P r reacted with calcium carbide in an electric furnance
forming a solid (B). The compound (B) was slowly
Hy yo OH HO,4 hydrolysed by water forming an insoluble substance
Hypophosphorous Phosphorous Phosphoric (C) and the solution of substance (D) which turned
acid (H,PO,) acid (H,PQ,) acid (H,PO,) Nessler’s reagent brown. Identify (A) to (E) and give
the reactions involved.
Due to the presence of P—H bonds, both H,PO, and H,PO,
act as reducing agents. On the other hand, H,PO, does not Ans. (i) K,Cr,0, + 2NH,Cl ——>. (NH,),Cr,0, + 2KCI
have any P—H bond and hence it does not act as a reduc- Ammoniumdichromate
ing agent. (NH,),Cr,O, __Heat , N, + Cr, + 4H,O
Q.3. Why is nitrous oxide called laughing gas ? (A)
Ans. When nitrous oxide (N,O) is inhaled in minor quan- N,, does not support combustion but Mg burns in it
tities, 1t causes hysterical laughter and therefore, it 1s called
laughing gas. 3Mg+N, —“*', Mg.N,
Q.4. What is the action of heat on (it) Gas (A) reacted with CaC,
(4) Pyrophosphoric acid
(74) Metaphosphoric acid CaCy +N, —“*> CaCN,
(iit) Phosphorous acid (B)
Ans. (i) Pyrophosphoric acid (H,P,0,) on heating gives Calcium cyanamide
metaphosphorie acid. CaCN, + 3H,O ——> CaCO,+2NH,
H,P,0, —— 2HPO, + H,O (C) (D)
Metaphosphoric acid (itt) Dreacts with Nessler’s reagent to give borwn colour as
(ti) Metaphosphoric acid (HPO,) on heating gives phos- NH, + 2K,Hegl, + 3KOH ——-> H, NHgOHgl + 7KI+ 2H,O
phorus pentoxide.
Nessler’s reagent Brown ppt.
2HPO, ——— P,O, + H,O
Phosphorus Q.8. When conc. H,SO, was added to an unknown
pentaoxide salt present in a test tube, a brown gas (A) was
(iit) Phosphorous acid (H,PO,) on heating gives phosphine evolved. The gas intensified when copper turnings
and phosphoric acid. 7 were also added into this test tube. On cooling the
gas A changed into a colourless gas (B).
4H,PO, "°, 3H,PO, + PH,
Phosphoricacid Phosphine (1) Identify the gases (A) and (B).
Q.5. What structures does PC1, adopt in the solid (11) Write the equations for the reactions involved.
state and vapour state ?
Ans. The given saltis a nitrate salt which on reaction with
Ans. In the solid state PCl, is ionic compound consist- cone. H,SO, gives first vapours of HNO, which decompose to
ing of [PC1,]* (tetrahedral) and [PCI,]- (octahedral) ions. In give brown gas (NO,).
vapour state, PCl, is monomeric having trigonal bipyramidal
geometry. 2NaNO, +H,SO, —““> Na,SO,+2HNO,
Q.6. A translucent white waxy solid (A) on heating Colourless
in an inert atmosphere is converted to its allotropic
form (B). Allotrope (A) on reaction with very dilute 4HNO, —“*> 4NO, + 2H,O + O,
aqueous KOH liberates a highly poisonous gas (C)
Brown gas
having rotten fish smell. With excess of chlorine
forms (D) which hydrolyses to compound (E). Identify (A)
compounds (A) to (E). The gas intensified when copper turnings were added due
Ans. (A) The white waxy solid (A) is white phosphorus. to reduction of HNO, by Cu.
When white phosphorus 1s heated in an inert atmosphere Cu+4HNO, => Cu(NO,), + 2NO,1 + 2H,0
at 573K, it changes to red phosphorus. Brown gas
(B) is red phosphorus. (A)
(A) on heating with KOH liberates phosphine (C) which On cooling the gas A changes into a colourless gas (B), N,O,.
1S poisonous gas with rotten fish smell.
2NO, ——N,0,
P, + 3KOH + 3H,0 ——> PH, + 3KH,PO,
Phosphine (A) (B)
(C) Q.9. Why does NCI, on hydrolysis give NH, and
White phosphorus (P,) burns with excess of Cl, to form
HOCI while PCI, on hydrolysis gives H,PO, and HCl.
phosphorus pentachloride (D).
P,+10cl, —““» pci, Ans. This is because N does not have d-orbitals to accom-
modate the electrons donated by O of water. Therefore, attack
(D)
of H,O occurs on Cl atom which has d-orbitals to accommodate
Hydrolysis of (D) gives phosphoric acid (E).
the extra electrons donated by H,O. As a result, C—O bond
PCl, + 4H,0 ——> H,PO, + 5HCl
is formed leading to the formation of NH, and HOCI.
(E)
p-BLOCK ELEMENTS

i
nM
go, FF aswe el
r J Re
H—6: cl—P’ —— > ~~, ee
Cl
| Cl Cl
Cl— N —— 3 HOC] + NH, H
(
(O—H
Na*)-O—H
| |
OH Au OH
+H,0 é; +H,0
H H cl—p —CI—5 > Cl— Ps 5 >HO—P
On the other hand, P and Cl both have d-orbitals to OH OH
accommodate electrons donated by H,O. But P—O bond is
much stronger than Cl—O bond. Therefore, attack of H,O or PCl, +3H,O —» H,PO, + 3HCl
molecules occur preferentially on P of PCl, to form H,PO,
and HCl.

valle Revision Exercises


i
A

D> Vesottueoso IIT


. Why 1s yellow phosphorus kept under water ?
which is obtained when conc. HNO, oxidises P4.
(A.S.B. 2017)
20. Write the formula of the compound of iodine which
: Give one example in which ammonia acts as an is obtained when conc. HNO, oxidises I,.
oxidising agent. (A.LS.B. 2017)
3. Which one of PC]; and PCI, is not likely to exist and 21. Iodine is more soluble in KI than in water. Why?
why? (D.S.B. 2012) (Karnataka S.B. 2018)
4, NH, has higher boiling point than PH,. 22. Complete the following equation:
(Chhatisgarh S.B. 2013)
XeF, + H,O ee rere. ul i
5. Mention the basicity of H,PO,. (Assam S.B. 2016) (Karnataka S.B. 2018)
6. Write the disproportionation reaction of H,PO,. 23. N, is known whereas P, is not known. Why?
(Assam S.B. 2013)
(Meghalaya S.B. 2018)
7. What is the covalency of nitrogen in N,O, ?
(D.S.B. 2013) Short Answer Questions q
8. Draw the structure of H,PO, and explain why it is
1. Describe the trends in the elements of group 15 in order
monobasic. (Mizoram S.B. 2014)
of increasing atomic number:
9. What is the basicity of H,PO,? (D.S.B. 2015)
10. Why is Bi(V) a stronger oxidant than Sb(V) ? (z) Atomic radii (it) Ionisation energy
(A.L.S.B. 2008; D.S.B. 2009) (iit) Oxidation state (iv) Catenation
11. Nitrogen is relatively inert as compared to phosphorus. 2. Complete the following chemical reaction equations:
Why ? (A.S.B. 2010) (z) P,(s) + NaOH(aqg) + H,O() —>
12. Which is stronger reducing agent; SbH, or Bil, and (zi) BiCl, + H,O —-+>
why? (A..S.B. 2012) (zit) Cu + dil. HNO, —>
13. What is the basicity of H,PO, and why ? 3. Draw the structures of white phosphorus and red
(A.LS.B. 2018, 2014) phosphorus. Which one of these two types of phosphorus
14, Why does NH, act as a Lewis base? (AJI.S.B. 2014) is more reactive and why ? (D.S.B. 2010)
15. Why does NO, dimerise? (A.LS.B. 2014) 4, State reasons for each of the following :
16. Out of white phosphorus and red phosphorus, which (1) The N—O bond in NO, is shorter than the N—O
one 1s more reactive and why? (A.L.S.B. 2015) bond in NO,-.
17. On heating Pb(NO,), a brown gas is evolved which (ii) All the P-Cl bonds in PCl, molecule are not
undergoes dimerization on cooling. Identify the gas. equivalent.
(A.LS.B. 2016) 5. (a) Draw the shape of P,O,p.
18. Write the formula of the compound of sulphur (6) What happens when
which is obtained when cone. HNO, oxidises §,,.
(z) PCl, is heated (zi) H,PO, is heated ?
(A.S.B. 2017)
19. Write the formula of the compound of phosphorus Write the reactions involved. (D.S.B. 20183)
a: 6. (a) How is nitric acid manufactured by Ostwald 18.
MODERN'’S abc + OF CHEMISTRY-XI

(a) Name three oxy-acids of phosphorus and draw


process? Write the chemical equations involved. their structures.
(Mizoram S.B. 2014) (6) Discuss the anomalous behaviour of oxygen.
(6) PCl, is known but PI, is not known. Why? (Hr.S.B. 2017)
(a) Why does NH, act as a Lewis base?
19. (a) PH, has lower boiling point than NH,, explain
(6) Bond angle in NH,* is more than that in NH,. why?
(HP. S.B. 2014)
(6) Why does PC], fume in moisture?
Account for the following:
20. Give reasons for the following:
(a) PCl, is more covalent than PCl,.
(6) Bi(V) is a stronger oxidizing agent than Sb(V). (a) NH, acts as a Lewis base.
(c) N—WN single bond is weaker than P—P single (6) NO, dimerises.
bond. 21. Prepare a short write-up on PCl, and PCl, highlighting
(a) Write equations for the manufacture of HNO, by the preparation and chemical properties of PCl, and
Ostwald process. strucrture of PCl,. (Kerala S.B. 2017)
(6) Write the structural difference between white 22. (a) Complete the following reactions :
phosphorus and red phosphorus. (D.S.B. 2014) (i) NH, + 3CL, (excess) —-> (ii) NaN, —“%>
(c) Molecular nitrogen is chemically very inert. Or
Explain why? (H.P.S.B. 2015) What happens when
10. Write short notes on various allotropes of phosphorus.
(1) (NH,),Cr,O, is heated?
Or (zi) HPO, is heated?
Name any five oxoacids of phosphorus and write their Write the equations. (D.S.B. 2017)
formulae. (Hr.S.B. 2015) (6) Draw the structures of the following :
11. Explain the following: iy H,P,0,
(a) Why does PCl, fume in moisture? (zi) HNO, (D.S.B. 2017)
(6) NO, dimerises to N,O,. (Ar.S.B. 2015)
20. How would you account for the following:
12. (a) Give two methods of preparation of NH, and three
important uses of NH,. (4) NCl, is an endothermic compound while NF, is
an exothermic one.
(6) (¢) NH, is a strong base but NF, does not show
any basic property. Why? (ii) Where R is an alkyl group R,P = O exists but
R,N = O does not.
(it) What is laughing gas?
(ii) At room temperature, N, is much less reactive.
135. (a) Why does bond angle decrease in the hydrides of
(A..S.B. 2013)
nitrogen family while going down the group?
24, Give reasons for the following :
(6) Why is ammonia more basic than phosphine?
(1) BiH, is the strongest reducing agent amongst all the
(c) H,PO, is monoprotic acid. Explain.
hydrides of group 15 elements. (A.LS.B. 2013)
14, (a) Complete the following reactions:
(it) Though nitrogen exhibits +5 oxidation state, it
(:) PH,I + NaOH —> (i) P, +50, Cl, —> does not form pentahalide.
(zit) NH,(excess) + Cl, —> (zit) (CH,),P = O exists but (CH,),N = O does not.
(6) Draw the structural formula of H,PO,. How do (1v) HPO, is a stronger reducing agent that H,PO,.
you account for the reducing behaviour of H,PO, (A..S.B. 2014)
on the basis of its structure? 25. Assign reason for the following:
15. (a) Draw the structure of H,PQO,. (1) H,PO, is a stronger reducing agent than H,PO,.
(6) Why does phosphorus exist as P, and nitrogen (11) Red phosphorus is less reactive than white
exists as N, gas? phosphorus.
16. (a) Write the principle for the production of NH, in (iit) N,O, is more acidic than N,O,. (A..S.B. 2017)

Long Answer Questions CC


Haber’s process.
(6) What happens when ammonia reacts with
(1)Na (1)CO, (Jharkhand S.B. 2016) (a) Write down the steps involved in the manufacture
17. (a) Discuss the structure of PF;. of HNO, by Ostwald process.
(6) Write the structural formula of PCl1, in solid state
(6) What are main reasons for anomalous properties
and also indicate the hybridisation of phosphorus
of nitrogen ?
atoms.
(c) How will you prepare HNO, by Ostwald process?
(c) What happens, when sulphur is treated with conc.
(Hr. S.B. 2017, Nagaland S.B. 2017) HNO,? (Meghalaya S.B. 2014)
p-BLOCK ELEMENTS

2. Explain the following facts— (6) Unlike phosphorus, nitrogen shows little tendency
nA)
(1) NH, forms hydrogen bond but PH, does not. for catenation. Why?
(zi) NH, is more basic than BiH. (c) Complete the following chemical equations
(iii) PH, has lower boiling point than NH,. (i) Mg + N, beat, ........
3. Explain Ostwald process for manufacturing nitric acid.
Draw structure of nitric acid and write its uses also. G21 p<OE
SEO carn Paci
Or
(i) (NEL)
Cr, 0, Beat ge necnees H,O
9
(a2) Why does reactivity of nitrogen differ from
phosphorus? (Assam S.B. 2017)
(6) What happens when zinc reacts with (a) Explain why HNO, behaves both as oxidising as
(z) concentrated HNO, (i) dilute HNO, well as reducing agent. (Assam S.B. 2017)
(c) Write formula of phosphine. (Hr. S.B. 2013) Or
4. (a) Describe the manufacture of ammonia by Haber’s (dD) Draw the structure of H,PO,. How do you account
process with favourable conditions. for the reducing behaviour of H,PO, on the basis
(dD) State the hybridisation of the central atom and of its structure ?
draw the structure of PCl.. (c) What happens when (NH,), Cr,O, is heated?
(c) Why is H,PO, tribasic and H,PO, dibasic?
(d) How does ammonia react with a solution of Cu2+?
5. (a) Which allotrope of phosphorus is more reactive
(a Nitric acid forms an oxide of nitrogen on reaction
and why?
a

with Py. Write the formula of the stable molecule


(6) Why does nitric acid act as an oxidising agent?
formed when this oxide undergoes dimerisation.
How it oxidises
(¢) carbon to carbonic acid (6) Write the disproportionation reaction of the
(it) sulphur dioxide to sulphuric acid. oxoacid of nitrogen in which nirtogen is in +3
(tit) sulphur to sulphuric acid. oxidation state.
(iv) ferrous sulphate to ferric sulphate. (c) Draw the molecular structures of the following
6. (a) Arrange the following in the increasing order of compounds :
property mentioned: (i) N,O, (ii) H,PO,
(z) H,PO,, H,PO,, H,PO, (Reducing character) (d@) Account for the following:
(it) NH,, PH,, AsH,, SbH,, Bil, (Base strength) (z) N, is less reactive at room temperature.
(D.S.B. 2016) (ii) Reducing character decreases from NH, to
(6) Why is H,PO, diprotic in nature? Draw structure. BiH.
7. (a) Give a laboratory method of preparation of (e) Which poisonous gas is evolved when white
dinitrogen. (Assam S.B. 2017) phosphorus is heated with conc. NaOH solution?
Write the chemical equations involved.

for Revision Exercises NSS


16. White P is more reactive because of angular strain in
P, molecules where the angles are only 60°
1. Because it catches fire in air. 17. NO 2
3. PClj1s not likely to exist because lone pair on P in PCI,
2Pb(NO3)2 —-—> 2PbO + 4NO, + O,
can be donated to Cl* and not to CI.
4. NH, forms hydrogen bonds but PH, does not.
2NO, —" 5NO,
5. one 6. 4H,PO, ———> 3H,PO, + PH, 18. H,PO, 19.H,SO, 20. HIO,
7. Five 9. tribasic (3) 21. Iodine combines with KI to form a soluble complex.
12. BiH, because BiH, is less stable than SbH, because of KI + ae —__> KI, (soluble)
larger size of Bi than Sb. 22. XeF’,
143K
13. Two, because it contains two P—OH bonds and therefore, XeF, + O,F, ——— XeF, + O,
can give two H* ions.
MODERN'S abc + OF CHEMISTRY-XIl

Objective Questions

AQ. Which of the following is a coloured gas ?


(a) NO, (6) N,O,
(c) N,O, (d) N,O.
Select the Correct Answer: . The oxide of nitrogen obtained by dehydration of nitric
acid with phosphorus pentoxide is
Al. ne brown gas formed when HNO, is reduced by metals (a) NO, (b) N,O,

(c) N,O (d) N,O.


ilo eo ae All. Which dein eon of nitrogen ia teenie when CaCN,
(c) NO, (d) NO. reacts with hot water ?
A2. The hydride of group 15 having largest bond angle (a) NH,
18: (6) N,O
(a) NH, (6) PH, (c) NH,NH,
(c) AsH, (d) BiH,. (d) NO,.
A3. The oxoacid ofP having oxidation state + 4 is : A.12. Among the trihalides of nitrogen, which one is least
(a) Phosphorus acid basic ?
(6) Hypophosphoric acid (a) NF,
(c) Phosphoric acid (6) NCI,
(d) Metaphosphoric acid. (c) NBr,
A4. Nitric acid (conc.) oxidises phosphorus to (d) NI...
(a) H,PO, (6) P,O, A138. On heating ammonium dichromate, the gas evolved is
(c) H,PO, td} FO: (a) Oxygen
A5. Ammonia gas can be dried over (6) Ammonia
(a) CaCl, (b) Cone. H,S0, (c) Nitric acid
(c) PCl, (d) Quick lime (d) Nitrogen.
A6. Ofthe following
hydrides which isthe strongest reducing A114, Which ofthe following is not correct
: White phosphorus
agent ? (P,) has
(a) NH, (b) SbH, (a) six P—P single bonds
(c) AsH, (d) PH,. (6) four P—P single bonds
AZ. Pure nitrogen gas is obtained from (c) four lone pairs of electrons
(a) NH, + NaNO, (d) PPP angle is 60°.
(6) NH,Cl + NaNO, A15, Thenumber of P—O—P bonds in cyclicmetaphosphoric
| acid is
(c) N,O + Cu en
(d) (NH,),Cr,07. | Orne
A8. Calcium phosphide gets hydrolysed and gives ey dae

cei aie (d) four.


rf ee 0 A16, The number of c—bonds in P,O,, 1s
3 4 (a) 6 (b) 16
(d) (HPOs), (c) 20 (d) 7.

Araswets
Al. (ce) A2. (a) A3. (5) Ad. (a) A5. (d) AG. (5) AZ. (6) A8. (5) AS. (a)
Al0. (6) All. (a) A112. (a) Al3. (d) Al4. (6) AS. (ce) AG. (5).
p-BLOCK ELEMENTS
05

|
Bi. The correct order of N-compounds in its decreasing
MULTIPLE CHOICE QUESTIONS order of oxidation states is
from competitive examinations (a) HNO,, NO, N,, NH,Cl
(6) HNO,, NO, NH,CL N,
AIPMT & Other State Boards’
fc) HNO, NHCL NO, N,
Medical Entrance (d) NH,Cl, N,, NO, HNO, (NEET 2018)
Bl. Which of the following statements is not valid for B8. Chlorine reacts with excess of ammonia to form
oxoacids of phosphorus? (a) NH,Cl (6) N, + HCl
(a) Orthophosphoric acid is used 1n the manufacture
(c) N, + NH,Cl (dq) N,+ NCI,
(e) NCI, + HCl (Kerala P.M.T. 2007)
of triple superphosphate.
B9. Which of the following is the correct order of increasing
(6) Hypophosphorous acid is a diprotic acid. enthalpy of vaporisation ?
(c) All oxoacids contain tetrahedral four coordinated (a) NH, <PH,<AsH, (6) AsH, < PH, < NH,
phosphorus. (c) PH, <AsH,<NH, (d) NH, <AsH, < PH,
(d) All oxoacids contain at least one P = O unit and one (e) AsH, < NH, < PH, (Kerala P.M.T. 2007)
B10. The maximum number of P—H bonds are contained in
P—OH group. (A.LP.M.T. 2012)
which of the following molecules?
B2. Strong reducing behaviour of H,PO, is due to: (a) H,PO, (6) H,PO,
(a) High oxidation state of phosphorus ic? EPO, (dq) H,P,0,
(6) Presence of two —OH groups and one P—H bond (A.M.U. Med 2010)
Bll. The basicity of pyrophosphorous acid is
(c) Presence of one -OH group and two P—H bonds
(a) 2 (b) 4 (ec) 1
(d) High electron gain enthalpy of phosphorus (d) 5 (e) 3 (Kerala PMT 2011)
(AIPMT 2015) B12. The oxidation state of phosphorus in
Bs. Nitrogen dioxide and sulphur dioxide have some cyclotrimetaphosphoric acid is
properties in common. Which property is shown by (a) +3 (6) +6 (ce) —3
one of these compounds, but not by the other? (d) +2 (e) -2 (Kerala PMT 2011)
(a) Is soluble in water. B13. The p-block element that forms predominantly basic
(5) Is used as a food preservative. oxide is

(c) Forms ‘acid—rain.


(a) N (6) P
(c) As (d) Sb
(d) Is a reducing agent. (AIPMT 2015)
(e) Bi (Kerala P.M.T. 2012)
» Which is the correct statement for the given acids?
B14. The oxoacid of phosphorus that reduces silver nitrate
(a) Phosphinie acid is a monoprotic acid while into metallic silver is
phosphonic acid is a diprotic acid. (a) H,PO, io) FF .0. ic) HPO,
(6) Phosphinic acid is a diprotic acid while phosphonic (d) H,P,0, (e) (HPO,), (Kerala PMT 2015)
acid is a monoprotic acid. B15. Which of the following oxides of nitrogen contains
(c) Both are diprotic acids. N—O—N bond?
(d) both are triprotic acids. (NEET 2016) (a) Dinitrogen oxide (6) Nuitrogenmonoxide
(c) Dinitrogen pentoxide (d) Dinitrogentrioxide
» The product obtained as a result of a reaction of (e) Dinitrogen tetroxide (e) 2.46
nitrogen with CaC, is (Kerala PMT 2015)
(a) CaCN, (6) Ca,CN
JEE (Main) & Other State Boards’
(c) (CN), (d) CaCN
(NEET 2016) Engineering Entrance
| When copper is heated with conc. HNO, it produces B16. Three reactions involving HPO, are given below :
(i) H,PO, + H,O —> H,O* + H,PO,-
(a) Cu(NO,),, NO and NO,
(iz) H,PO, + H,O —+ HPO? + H,O*
(6) Cu(NO,), and N,O
(iii) H,PO, + OH’ —-> H,PO, + O07
(c) Cu(NO,), and NO, In which of the above does H,PO, act as an acid ?
(d) Cu(NOQO.,), and NO (NEET 2016)
(a) (ut) only (6) (2) only
(c) (it) only (d) (2) and (iz)
(A.LLE.E.E 2010)
Answels
Bl. (6d) B&. (c) Bs. (6) B4. (a) B5. (a) B6. (c) Bi. (a) B8. (a) BS. (a)
B10. (2) Bll. (2) B12. (6) BIS. (e) Bl4. (a) B15. (c) B16. (c)
MODERN'S abc + OF CHEMISTRY-XIl

B17. Which of the following statement is wrong ? B26. An inorganic salt (A) is decomposed on heating to give
(a) The stability of hydrides increases from NH, to two products (B) and (C). Compound (C) 1s a hquid at room
BiH, in group 16 of the periodic table. temperature and is neutral to lhtmus while the compound (B)
(6) Nitrogen cannot form d1-pt bond. is a colourless neutral gas. Compounds (A), (B) and (C) are
(c) Single N — N bond is weaker than the single P — P (a) NH,NO,, N,O,H,O (6) NH,NO,, NO, H,O
bond. (c) CaO, H,O, CaCl, (d) Ba(NO,),, H,O, NO,
(d) N,O, has two resonance structures.
(e) Mg(NO,),, N,O;, H,O (Kerala P.E.T. 2013)
(A.LLE.ELE. 2011)
B27. Oxyacids of phosphorous and the starting materials
B18. The reaction of zine with dilute and concentrated nitric
for their preparation are given below :
acid, respectively produces
(a) N,O and NO, (6) NO and NO, Oxyacid ~ Materials for preparation
(c) NO and N,O (d) NO, and N,O (A) HPO, = (7) Red P + alkali
(JEE Main 2016) (By. FAPO, tw) P,O,, + H,0
B19. The pair in which phosphorus atoms have a formal (C) H,PO, af (iii) P,O,+H,O
oxidation state of +3 is (D) H,P,0, d Gey WHER: aie
(a) orthophosphorous and pyrophosphorous acids
Choose the correct answer from the codes given below:
(6) pyrophosphorous and hypophosphoric acids
(c) orthophosphorous and hypophosphoric acids
(a) (A) —(iv); (B)— (iz); (C) — (ai); (D) —- @
(d) pyrophosphorous and pyrophosphoric acids. (b) (A)—(a); (B) — (ait); (C) — (aa); (D) — (iv)
(JEE Main 2016) (c) (A) —(G@v); (B)— (az); (C) — (2); (D) - zz)
B20. The compound that does not produce nitrogen gas by (d) (A) (ix); (B) — (ait); (C) — (2); (D) — (iv)
the thermal decomposition is: (e) (A) —(aiz); (B) — (2); (C) — (at); (D) — (ev)
(a) Ba(N,), (6) (NH,),Cr,O, (Kerala P.E.T. 2013)
(c) NH,NO, (d) (NH,),SO, B28. The statement that is not correct is
(JEE Main 2018) (a) Hypophosphorous acid reduces silver nitrate to
B21. Which of the following contains P — O — P bond ? silver
(a) Hypophosphorous acid (6) Phosphorus acid (b) In solid state PCl, exists as [PCl1,]* [PCl,]-
(c) Pyrophosphoric acid (d) Orthophosphoric acid (c) Pure phosphine is non-inflammable
B22. NO, is not obtained on heating (d) Phosphorous acid on heating disproportionates
(a) AgNO, (6) KNO, to give metaphosphoric acid and phosphine.
(ec) Cu(NQ,), (d) Pb(NO,), (Karnataka CET 2014)
(W.B. JEE 2011, 2016)
B29. On heating with concentrated NaOH solution in an
B23. aac stable hydride . a elements is inert atmosphere of CO,, white phosphorus gives a
ra Asti a ShTt gas. Which of the following statements is incorrect
ee 3 about the gas?
(e) Bilt, (Rapala BEF 201) (a) Itis more basic than NH,.
B24, Pick out the wrong statement.
(6) Its solution in water decomposes in the presence of
(a) Nitrogen has the ability to form pt-pr bonds with
light.
itself.
(c) Itis less basic than NH,.
(6) Bismuth forms metallic bonds in elemental state.
(c) Catenation tendency is higher in nitrogen when (d) Itis highly poisonous and has smell like rotten
compared with other elements of the same group. fish. (Karnataka CET 2015)
(d) Nitrogen has higher first ionisation enthalpy when B30. What is the basicity of orthophosphorus acid?
compared with other elements of the same group. (a) One (6) Two
(e) Arsenic forms dx-dx bonds with transition metals. (c) Three (dq) Four (MA-CET 2015)
(Kerala P.E.T. 2012) B31. In the solid state, PC], exists as
B25. Nitric acid can be obtained from ammonia via the (a) [PCl,] and [PCI] ions
formation of the intermediate compounds (6) covalent PCl, molecules only
(a) nitric oxide and nitrogen dioxide (c) [PCI Ai and [PCl,] ions
(6) nitrogen and nitric oxide (d) covalent P,Cl,, molecules only. (WB JEE 2016)
(c) nitric oxide and dinitrogen pentoxide B32. Thermal decomposition of ammonium dichromate gives
(d) nitrogen and nitrous oxide (a) N,,H,O and Cr,O, (6) N,, NH, and CrO
(W.B.S ELE. Engg. 2013) (c) (NH,),CrO, and H,O(d) N,, H,O and CrO,
Ans WERS (Kerala PET 2016)

B17. (a) B18. (a) B19. (a2) B20. (d) B21. (c) B22. (6) B23. (e) B24. (c) B25. (a)
B26. (a) B27. (a) B28 (d) B29. (a) B30. (6) B3l. (c) B32. (a)
p-BLOCK ELEMENTS
As
B33. Which is true regarding nitrogen? (a) H,PO, and H,P,0, (6) H,P,O, and H,P,0,
(a) Less electronegative (c) H,PO, and H,PO, (d) H,P,O, and H,PO,
(6) Has low ionisation enthalpy (JEE Main 2019)
(c) d-orbitals are available
(d) Ability to form pa—pt bonds with itself JEE (Advance) for ITT Entrance
(Karnataka C_E.T. 2016)
B34, What will be the resultant products formed when the B40. The reaction of P, with X leads selectively to P,O,.
phosphorus halide, PBr, splits up?
The X is
(a) Dry O,
(a) [PBr,]* and Br- (6) [PBr,]- and [PBr,]* (6) A mixture of O, and N,
fc} [Pel (d) [PBr,]- (c) Moist O,
(J.K. CET 2018) (d) QO, in the presence of aqueous NaOH (J.T. 2009)
B35. Maximum number of covalent bonds formed by N and B41. Extra pure N, can be obtained by heating
P are (a) NH, with CuO (6) NH,NO,
(a) 3,5 (b) 3,6 (ce) (NH,),Cr,0, (d) Ba(N,),(f.1T. JEE 2011)
(c) 3,4,5 (d) 3,4,6
(Kerala PET 2018) B42. The reaction of white phosphorus with aqueous NaOH
gives phosphine along with another phosphorus
B36. Consider the following compounds
containing compound. The reaction type; the oxidation
1. (NH,),Cr,O, 2. NH,NO, states of phosphorus in phosphine and the other product
3. NH,VO, 4. NH,NO, are respectively.
Which compound(s) yield nitrogen gas upon heating? (a) redox reaction; — 3 and—5
(a) land 2 (b) 2and3 (6) redox reaction; + 3 and +5
(c) 3and4 (dq) land4 (c) disproportionation reaction; — 3 and +5
(e) All (Kerala PET 2018) (d) disproportionation reaction; — 3 and +3
B37. Very pure N, can be obtained by (LT. J.B. 2012)
(a) thermal decomposition of ammonium dichromate
. Concentrated nitric acid, upon long standing, turns
(6) treating aqueous solution of NH,Cl and NaNO, yellow-brown due to the formation of
(c) liquefaction and fractional distillation of liquid air (a) NO (6) NO,
(d) thermal decompositon of sodium azide
(Karnataka C_E.T. 2018)
(c) N,O (d) N,O,
(J.L.E. Advance 2013)
B38. Good reducing nature of H,PO, is attributed to the
. The order of the oxidation state of the phosphorus atom
presence of
(a) One P—OH bond (6) One P—H bond
in H,PO,, H,PO,, H,PO,, and H,P,0, is
(c) Two P—H bonds (d) Two P—OH bonds (a) H,PO, > H,PO, > H,PO, > H,P,0,
(JEE Main 2019) (6) H,PO, > H,PO, > H,P,0, > H,PO,
B39. The pair that contains two P—H bonds in each of the
(c) H,PO, > HPO, > H,PO, > H,P,0,
oxoacids is
(d) H,PO, > H,P,0, > H,PO, > H,PO,
Auswets (JEE Advance 2017)

B33. (d) B34. (a) B35. (a) B36. (a) B37. (d) B38. (c) B39. (a) B40. (6) B4l. (d)
B42. (c) B43. (5) B44. (d)
C3. Nitrogen oxide(s) that contain(s) N—N bond(s) is (are)
(a) N,O (6) N,O,
(c) N,O, (d) N,O,;
Cl. White phosphorus has
(LT. JEE 2009)
C4. The nitrogen containing compound produced in the
(a) four P—P bonds (6) bond angle “PPP = 60° reaction of HNO, with P,O,,
(c) six P—P bonds (dq) polymeric structure. (a) can also be prepared by reaction of P, and HNO,
C2. A solution of colourless salt H on boiling with excess (6) is diamagnetic
NaOH produces a non-flammable gas. The gas (c) contains one N—N bond
evolution ceases after some time. Upon addition of Zn (d) reacts with Na metal producing brown gas.
dust to the same solution, the gas evolution restarts. (JEE Advance 2016)
The colourless salt(s) H is (are) C5. White phosphorus P, has the following characteristics
(a) NH,NO, (6) NH,NO, (a) 6 P—P single bonds
(c) NH,Cl (d) (NH,),5O, (.1.T. 2008) (6) 4P—P single bonds

Aries
(c) 4 lone pair of electrons
(d) P—P—P angle of 60° (WB JEE 2018)

C1. (0), (c) C2. (a), (6) C3. (a), (5), (c) C4. (bd), (d) C5.(a), (c), (d)
MODERN'S abe + OF CHEMISTRY-XIl

one

C6. The compound(s) which generate(s) N, gas upon C7. Based on the compounds of group 15 elements, the
thermal decomposition below 300 °C 1s (are) correct statement(s) 1s (are)
(a) NH,NO, (o) (NH )Cr OG. (a) Bi,O, is more basic than N,O,
(c) Ba(N,), (d) Mg.N, (b) NY. is more covalent than Biv.
(JEE Advance 2018) (c) PH, boils at lower temperature than NH,
(d) the NN single bond is stronger than the P—P single
Answers bond. (JEE Advance 2018)

C6. (5), (c) C7. (a), (bd), (c)


Answer the following questions:
D1. The oxyacid of P having tetrabasicity is/are
(a) CandH (6) CandE
Passage I. (c) C and D (7d) Band E
Phosphorus forms a variety of oxyacids. In all these, D2. Which of the acids show reducing properties ?
phosphorus is sp hybridised and is tetrahedrally bonded to
(a) E, F and G (b) E, Gand H
four neighbouring atoms. These contain P—OH bonds, the
hydrogen of which are ionisable giving acidic character to (c) E only (7d) EandF
these compounds. These also contain P—H bonds in which D3. Monobasic oxyacids of phosphorus is/are
hydrogens are not ionisable because P and H have nearly (a) Hand E (6) Aand E
same electronegativity. The presence of P—H groupin these
(c) CandG (d) Eonly
oxyacids is responsible for their reducing properties. The
structures of some oxyacids are given below : D4. Which of these can exist as polymeric ?

0 () (a) Aand E (b) A only


| IF
(c) CandG (d) H only.
P
YN O
D5. Which of these is dibasic as well as reducing ?
(a) E (b) A
HO” Ng lO on
meta-Phosphoric acid ortho-phosphoric acid
(c) G (d) F.
(A) (B) Passage II.

| /|
QO O There are some deposits of nitrates and phosphates in
| P—-—P
earth's crust. Nitrates are more soluble in water. Nitrates
P P are difficult to reduce under the laboratory conditions but
OoA\*o%
bn ea OH ne |Non microbes do it easily. Ammonia forms large number of
OH HO OH OH complexes with transition metal ions. Hybridization easily
Pyrophosphoric acid Hypophosphoric acid explains the ease of sigma donation capability of NH, and
(C) (D) PH,. Phosphine is a flammable gas and is prepared from
i white phosphorus.

Answer the following questions :


iS SoH D6. Among the following, the correct statement is
oY on
Hypophosphorus acid Phosphorus acid (a) Phosphates have no biological significance in
(E) (F) humans

Ld i
O O (6) Between nitrates and phosphates, phosphates
are less abundant in earth’s crust
ag at “A ND O-0H (c) Between nitrates and phosphates, nitrates are
Ho (on OH HO OH less abundant in earth’s crust
Pyrophosphorous acid Peroxomonophosphorie acid (d@) Oxidation of nitrates is possible in soil.
(G) (H)

uswcls
Passage I. D1. (c) D2. (a) D3. (d) D4. (a) D5. (d)
Passage II. D6. (c)
p-BLOCK ELEMENTS 1T1A/55 ss

D7. Among the following, the correct statement is (d) Between NH, and PH,, PH, is a better electron
(a) Between NH, and PH,, NH, is a better electron donor because the lone pair of electrons occupies
donor because the lone pair of electrons occupies spherical ‘s’ orbital and is less directional.
spherical ‘s’ orbital and is less directional. D8. White phosphorus on reaction with NaOH gives PH,
(6) Between NH, and PH,, PH, is a better electron as one ofthe products. This is a
donor because the lone pair of electrons occupies , _—" ;
sp® orbital and is more directional S ae
(c) Between NH, and PH,, NH, is a better electron (6) disproportionation reaction
donor because the lone pair of electrons occupies (c) condensation reaction
sp° orbital and is more directional
(d) precipitation reaction.

Auswets
Passage II. D7. (c) D8. (5)

; : Pp q rr Ss
Matrix Match Type Questions O@O®
Each question contains statements given in two col-
umns, which have to be matched. Statements in Column | (p) (r) (s)
are labelled as A, B, C and D whereas statements in Column
C (r) (s)
IT are labelled as p, g, r and s. Match the entries of Column I
with appropriate entries of Column II. Each entry in Column D (q) (r) (s)
I may have one or more than one correct option from Column
Il. The answers to these questions have to be appropriately Ifthe correct matches are A-g, A-r, B-p, B-s, C-r, C-s and
bubbled as illustrated in the following example. D-q, then the correctly bubbled matrix will look like the
following:

1. Match the compound in Column I with the property in Column II.

(A) Pyrophosphoric acid (p) Dibasic


(B) Thiosulphuriec acid (gq) has oxidiation of +3 of central atom
(C) Orthophosphoric acid (r) has +5 oxidation state of central atom
(D) Phosphoric acid (s) tribasic

2. Match each of the reactions given in Column I with the corresponding product(s) given in Column II.

(A) Cu + dil. H,SO, NO


(B) Cu + cone. HNO, NO;

(C) Zn + dil. HNO, N,O


(D) Zn + cone. HNO, ) Cu (NO,),
Zn(NQ,),
(I.L.T. 2009)

(NBR
(1) :(A)-() (B) -@) (C)— (r), (s) (D) — (Pp), (q)
(2) : (A) —(p), (s) (B) — (q), (s) (C)—(r), © (D) —(q), ©
7 11AI56 MODERN'S abc + OF CHEMISTRY-XII

phoric acid, orthophosphoric acid, diphosphoric


Integer Type Questions acid, metaphosphoric acid, peroxophosphoric acid,
the acids having +3 oxidation state of P are
Integer Type : The answer to each of the following 3. Among the following, the number of compounds
question is a single-digit-integer ranging from 0 to 9. that can react with PCI; to give POCI, is O,, CO,,
SO)... HoB,..6 01. (Li BOTT)
1. Total number of dibasic acids among H,PO,, 4, The difference between the number of lone pairs
BPs FE. A, EPO. TP. H,PO, and P — O bonds in P,O, molecule is
1s 5. The total number of lone pairs of elections in
2. Among phosphonic acid, phosphinic acid, pyrophos- N,O, is (JEE Advance 2015)
phorous acid, hypophosphorous acid, hypophos-

Aaswecs
1. 2. 2. 2. 3. 5 4.4 53.8

NCERT Exemplar Problems //


Objective Questions
»» Multiple Choice Questions (Type-l) <@ (c) It is more basic than NH,.
- ; ae ae (d) It is less basic than NH,.
a single andhas aaah
double bonds aa
are present? aaediliaaaas” 5. Which ofaethe following acids forms ae series of salts?:
ae three

(a) 3 double bonds; 9 single bonds (a) HPO, (6) HBO;


(c) H,PO, (d) H,PO,
(6) 6 double bonds; 6 single bonds
6. Strong reducing behaviour of H,PO, is due to
(c) 3 double bonds; 12 single bonds
(a) Low oxidation state of phosphorus
(d) Zero double bonds; 12 single bonds
(6) Presence of two —OH groups and one P—H bond
2. Which of the following elements can be involved in
pr—dr bonding? (c) Presence
| of one -OH group and two P—H bonds
Ga) Cachan (b) Nitrogen (d) High electron gain enthalpy of phosphorus
(c) Phosphorus (d) Boron 7. On heating lead nitrate forms oxides of nitrogen and
3. Bond dissociation enthalpy of E—H (E = element) oe a ene (b) NO PbO
bonds is given below. Which of the compounds will act (a) NO, Pb! (0) 2?
as strongest reducing agent? (c) NO, PbO (d) NO, PbO,
Compound NH P AsH | SbH
ieeeof the
8. Which following elements does not show

(a) Nitrogen (6) Bismuth


(a) NH, (6) PH, (c) Antimony (dq) Arsenic
(c) AsH, (d) SbH, 9. Maximum covalency of nitrogen is
4, On heating with concentrated NaOH solution in an (a) 3 (b) 6
inert atmosphere of O.. white phosphorus gives a (c) 4 (d) 6
gas. Which of the following statement 1s incorrect 10. Which of the following statements is wrong?
, ?
about ~ oo (a) Single N—N bond is stronger than the single P—P
(a) It is highly poisonous and has smell like rotten bend
fish. — (6) PH, can act as a ligand in the formation of
(6) It's solution in water decomposes in the presence coordination compound with transition elements
of light. . | oe
(c) NO, is paramagnetic in nature.
(d) Covalency of nitrogen in N,O, is four.
Araswecs
1. (a) ke) 3. (d) gee) 5. (c) 6. (c) 7. (5) 8. (a)
Sc} 10. (a)
p-BLOCK ELEMENTS
AT

| _Il.Multiple Choice Questions (Type-ll) €C _


11. A brown ring is formed in the ring test for NO, ion. It
is due to the formation of
(a) [Fe(H,O), (NO)]** (6) FeSO,.NO, Note : In the following questions two or more options may
(c) [Fe(H,0),(NO),}* (d) FeSO,-HNO, be correct.
12. Elements of group-15 form compounds in +5 17. Which of the following is correct for P, molecule of
white phosphorus?
oxidation state. However, bismuth forms only one well
(a) It has 6 lone pairs of electrons.
characterised compound in +5 oxidation state. The
(6) It has six P—P single bonds.
compound is
(c) It has three P—P single bonds.
(a) Bi,O, (6) Bik, (d) It has four lone pairs of electrons.
(c) BiCl, (d) Bi,s, 18. Which of the following statements are correct?
13. On heating ammonium dichromate and barium azide (a) All the three N—O bond lengths in HNO, are
separately we get equal.
(a) N, in both cases (6) All P—Cl bond lengths in PCl, molecule in gaseous
(6) N, with ammonium dichromate and NO with state are equal.
barium azide (c) P, molecule in white phohsphorus have angular
strain therefore white phosphorus is very reactive.
(c) N,O with ammonium dichromate and N, with
(d) PCl, is ionic in solid state in which cation is
barium azide
tetrahedral and anion is octahedral.

>) SII“
(d) N,O with ammonium dichromate and NO, with
barium azide
14, Inthe preparation of HNO,, we get NO gas by catalytic
oxidation of ammonia. The moles of NO produced by Note: In the following questions a statement of assertion
the oxidation of two moles of NH, will be : followed by a statement of reason is given. Choose
(a) 2 (b) 3 the correct answer out of the following choices.
(a) Both assertion and reason are correct statements,
(ce) 4 (d) 6
and reason is the correct explanation of the
15. The oxidation state of central atom in the anion of
assertion.
compound NaH,,PO, will be .
(6) Both assertion and reason are correct statements,
(a) +3 (6) +6 but reason is not the correct explanation of the
(c) +1 (d) -3 assertion.
16. In solid state PCl, is a (c) Assertion is correct, butreasonis wrong statement.
(a) covalent solid (d) Assertionis wrong butreason is correct statement.
(6) octahedral structure (e) Both assertion and reason are wrong statements.
19. Assertion : N, is less reactive than P,.
(c) 1onie solid with [PC1,]* octahedral and [PC1,]~
tetrahedra Reason : Nitrogen has more electron gain enthalpy
than phosphorus.
(d) 1omie solid with [PCl,]* tetrahedral and [PC1,]~
octahedra 20. Assertion : HNO, makes iron passive.

Ase
Reason: HNO, formsa protective layer of ferric nitrate
on the surface of iron.

11. (a) 13. (b) 13. (a) 14. (a) 15. (c) 16. (d) 17. (c) 18. (d) 19. (c) 20. (c)

anations for
Hints & Expl Difficult Objective Type Questions
A8. (6) : Ca,P, + 6H,0 ——> 3Ca(OH), + 2PH,
A. mcg with only one correct answer
A9. (a) : NO, is a brown coloured gas.
A3. (6) : Hypophosphoric acid is H,P,O, -H.0
oxidation state of P = +4.
Al0. (6) : 2HNO, —2"» N,0, +H,O
All. (a) : CaCN, + 3H,O ———> CaCO, + 2NH,
Ad. (a): P, + 20HNO,(conec.) —-+ 4H,PO, + 20NO, +
4H,,0 Al2. (a) : NF,1s least basic because electron pair on N is
AZ. (6) : Ammonium nitrite (NH,Cl + NaNO,) gives N, least available due to high electronegativity of
on heating. Ef.
MODERN'S abc + OF CHEMISTRY-XIl

Pyrophosphorous acid : H,P,O,


A138. (d) : (NH,),Cr,0,
“5 Cr,0, +N,
+4H,O
Oxidation state of P = +3

Al16. (6) : No. of o bonds = 16 B20. ay Ba(N,), —“> Ba+8N,


B. mcg from Competitive Examinations (NH,),Cr,0, —+_N, + 4H,O + Cr,0,
Bl. (b} - Hypophosphorous acid is monoprotic acid. NH,NO, —*~ N, +2H;0
(NH,),SO, “> 2NH,4H,SO,
B22. (b) : NO, is prepared by heating nitrates of heavy
metals and not KNO,,.
2Pp(NO,), —; BPO + 4NO, + O,
B2. (e) : Reducing behaviour of H,PO, is due to the pres- 2Cu(NO,), — 2CuO + 4NO, + O,
ence of one —OH bond and two P—H bonds. 2AgNO, ee 2Ag + 2NO, + O,
Bs. (b) : NO, 1s not used as food preservative but SO, 1s 2KNO, a 2KNO, +0,
used as a food preservative.
B23. (e) : Stability decreases down the group because size
B4. (a) : Phosphinie acid (H,PO,) 1s monoprotic and
of central atom increases and its tendency to
phosphonic acid (H,PO,) is a diprotic acid.
form covalent bond with small atom decreases.
B5. (a) : CaC, + N, —> CaCN, + C B24. : In group 15, catenation tendency of P is higher
(c)
B6. (ce) : Cu + 4HNO, (cone.) —> as compared to other elements of the group.
Cu(NO,), + 2NO, + 2H,O B25. (a) : In Ostwald’s process, the formation of HNO,
B7. (a) : HNO, (+5), NO (+2), N, (0), NH,Cl (-3) OCCUTS 4&5:

Bll. (a) : Pyrophosphorous acid (H,P,O,) 1s a dibasic acid 4NH, + 60, Se


—7o Ptoe
because it contains two P— O—-—H bonds:
ANO + 6H,O + Energy

i
1iin Wiles
2NO + O, —> 2NO,
3NO, + H,O —> 2HNO, + NO
OH OH
B28. (d) ALP. <=) HPO, ¢ FH,
B12. (b) : Cyclotrimetaphosphoric acid is (HPQO,),.
Phosphorous Orthophosphoric Phosphine
Oxidation state of P is acid acid
+14+x%+8(—2)=0 -.¥=46.
B29. (a) : P, +3NaOH+3H,O—“-» PH, + 3NaH, PO,
B13. (e) : Bismuth oxides are most basic. Oxides of N, P
PH, is less basic than NH.
— acidic; oxides of As, Sb — amphoteric, oxides
of Bi—basic. B30. (b) : Basicity of H,PO, is two because it has two
ionizable H atoms
B14. (a): The P—H bond present in H,PO, imparts reduc-
ing property to the acid.
4 AgNO, + 2H5O + H,PO, —~
|
H~ ‘Sow
4Ag + 4HNO, + HPO, OH
B31. (c) : In solid state, PCl, exists as [PCL,]* [PCI,]-
B16. (c) (it) H,PO, because it loses a proton and there-
fore, it is a Bronsted acid. B32. (a) : (NH,).Cr,0, —->N, + Cr,0, + 4H,O
B17. (a) : The stability of hydrides decreases from NH, B33. (d) : Nitrogen has ability to form pz-pnx bonds with
to BiH, due to increase in size of the central itself.
atom.
B34. (a) > PBr, —— [PBr,]* [Br]
B18. (a) : 4Zn + 1OHNO, (dil.) —>
47n(NO,), + N,O + 5H,0O B36. (a): (NH,),Cr,O, —“> Cr,0, +N, + 4H,O
Zn + 4HNO, (cone.) —> NH,NO, —*> N, +2H,O
Zn(NO,), + 2NO, : 2H,0 2NH,VO, —*—> V,O, + 2NH, + H,O
B19. (a) : Orthophosphorous acid: H,PO, NH,NO, —*> N,O + 2H,0
Oxidation state of P = +3
p-BLOCK ELEMENTS
As
B37. (d): 2NaN, —*> 3N,+2Na On ze
Sodium azide
N,0O; na ONS,
B38. (c) : H,PO, is good reducing agent due to the
presence of two P—H bonds. C4, (6, d) : P,O,,+4HNO,—>2N,0, +4 HPO,
O The nitrogen containing compound produced
I above is N,O,.

HO
/\w# H
It cannot be prepared by the reaction of P, and
HNO,.
Itis diamagnetic.
B39. (a) : It does not contain N—N bond.

i
O O O
N,O, 1s decomposed by alkali metals.
N,O, + Na—-+ NaNO, +NO,
/ Sox J So \ Na Brown gas.
H H H OH OH C6. (b,c): NH,NO, ——>N,0+H,0
(H,PO,) (H,P,0,) (NH,),Cr,O, —*~ N, + Cr,0, + 4H,O
B40. (5) : P, gives P,O, when oxygen is in limited supply. Ba(N;), —> 3N, + Ba
Hence a mixture of O, and N, 1s most suitable. |
Mg.N, as°C eile + N,
P, +30, —2~ P,O, Only (NH,),Cr,0, and Ba(N,), give N, on
heating below 300 °C.
B41. (d) : Ba(N,), —@t> Ba +3N,.
C7. (a, 6,c): Bi,O,1s more basic than N,O,
B42. (c) :
+1 3 NF, is more covalent than Bik, because of
lesser
P,(s) + 3NaOH(aq) + 3H,O ——> 3NaH,PO,(aq) + PH,(g) electronegativity difference.
Sodium Phosphine
hypophosphite Boiling point of NH, is more than PH, because of
+1 +5 —3 hydrogen bonding in NH,.
3NaH,PO,(aq) ——>Na,PO, + 2PH, P—P bond is stronger than N—N single bond
In the above reactions, phosphorus undergoes because in N due to smaller size of atoms lone
simultaneous oxidation and reduction to + pair-lone pair repulsions will be more.
5 and — 3 oxidation states respectively. The
reaction in which species undergoes oxidation
and reduction are called disproportionation
D. mcq based on passage/comprehension
reactions. D6. (c) : Nitrates being water soluble and as they get
reduced by microbes so obviously its abundance
B43. (5) : Nitric acid upon long standing turns yellow will decrease.
brown due to decomposition by sunlight into
D7. (c) : NH, is a stronger Lewis base than PH,.
NO,. 0 +1 +3
D8. (5) : P,+3NaOH+ 3H,O0 ——> 3NaH, PO, + PH,
4HNO, —oniett_, 4NO. + 2H20 + Oo
Here P, 1s oxidized to NaH, PO, and it is reduced
B44. (d) - The correct order is: to PH,
+5 +4 +3 +1
H,PO joe Age gs 1G eo > HPU. Matrix Match Type Questions
2. (p, s):3Cu+ 8 HNO,—-> 3Cu(NO,), + 4H,O + 2NOT
C. meq with more than one correct answer (dil.)

(q, s): Cu+ 4 HNO, —-+> Cu(NOQ,), + 2H,O + 2NO,1


Cl. (b,c) : P, has six PP bond and each 4 PPP is 60°.
(conc. )
C3. (a, 6, c):N,O N=N—>0
(r, £) : 44n + 10 HNO, —>
O O (dil.)
NO,
Sy aana rn
4 Zn(NO,), + 5 H,O + N,OT

Ow VP (q, t) : Z1+4HNO,—-> Zn(NO,), + 2H,O + 2NO,T


N,O
a4 ok N—N So
(conc.)
| 11A/60 MODERN'S abc + OF CHEMISTRY-XIl

Integer Type Questions NCERT Exemplar Problems : MCQs Type-I


1. 2 : H,PO, and H,P,O, are dibasic acids. 2. (c) : Phosphorus involves px — dt bonds.
2. 2 : Phosphonic acid (H,PO,), pyrophosphoric
3. (d): Because it has lowest bond dissociation enthalpy
acid (H,P,O0,) have oxidation state of +3 for (E—H bond) and hence strongest reducing agent.

m 2 Pl.
a ce+ SO, FF
|
BOC) SUC.
4. (c) : 4P + . 3NaOH + . 3H,O——> NaH,PO, + PH,
PCl, + H,O POCI, PH, 1s less basic than NHg.
PCl; + H,SO, ——> POCI1, 5. (c) : Basicity of H,PO, is three and it has three ionisable
6PCl, * POGOe = >» 40 POC, OH.
Heat 6. (c) : Strong reducing behaviour of H,PO, is due to the
PCI; + O, ——> POCI, + Cl, presence of two P—H bonds.
4, 4: In the structure of PyOe. 7. (6): 2Pb(NO,), _Heat , 2PbO + 4NO, + O,

9. (c) : Maximum covalency of nitrogen is four.


10. (a): Single P—P bond is stronger than N—N bond.
12. (6): Only Bik’, is known.

13. (a): (NH,),Cr,0, > Cr,O, +N, + 4H,0,


Ba(N,), —“ Ba +3N,
14, (a): 4NH, + 60, —> 4NO + 6H,O
2 mol of NH, gives 2 mol of NO.
Number of lone pairs = 16 15. (c) : NaH, PO, : Here +1 +2 (+1) + x + 2(-2) =0
Number of P — O bonds = 12 or ee os a

pee ee 16. (d): 2PCl —— > [PCL] [PCLJ-


*, ‘Or 86 fOr :0: |
5. 8: .O tetrahedral (octahedral)
NF . +
ys ~——- fp —N NCERT Exemplar Problems : MCQs Type-II
“Os /
17. (6), (d): It has six P—P bonds and four lone pairs of
It has 8 lone pairs of electrons.
electrons.
4
p-BLOCK ELEMENTS
se
Examination

Time Allowed: I hr. Maximum Marks: 26

What is the covalence of nitrogen in N,O,? (1)


Give the disproportionation reaction of H,PO,. (1)
Give one reaction in which ammonia acts as a reducing agent. (1)
What happens when phosphorus is heated with concentrated NaOH is an inert
atmosphere of CO,? (1)
PCl, is known but PI, is not known. Why? (1)
Why does R,P=O exist but R,N=O does not exist (R = alkyl group)? Explain. (2)
Give the structures of P,O, and P,O,). (2)
Explain.
(¢) PCl, exists as [PC1,]* [PC1,]" but PBr, exists as [PBr,]* [Brl-.
(it) NF, does not have donor properties like ammonia. (2)
Explain the following :
(1) NH, is a stronger base than PH.
(ii) Phosphorus (P,) is more reactive than nitrogen (N,).
(tit) HPO, is a diprotic acid. (3)
10. Explain the chemistry of manufacture of nitric acid by Ostwald’s process?
How does it react with iodine? (3)
11 Complete the following chemical quations:
(i) Ca,P, (s) + H,O (@) ——
(@z) P,O,,+H,O —_
(iit) Ca,(PO,), +510, +C —> (3)
12. (a) Nitrogen exists as diatomic molecule and phosphorus as P,. Why?
(6) Write the resonating structures of NO, and N,O,.
(c) Give two examples to show anomalous behaviour of nitrogen.
(d) Why is BiH, the strongest reducing agent amongst all the hydrides of group 15?
Explain.
(e)Give the reaction of zinc with conc. HNO,. (5)
WWW.JEEBOOKS.IN
Ox ORGANIC CHEMIS
OM an BASIC PRINCIPLES
AND TECHNIQUES
0) 5510 OsMAYO
Building on..... Assessing..... Preparing for Competition.....
Understanding Text 1 @ Quick Memory Test with @ Topicwise MCQs 138
Conceptual Questions 92,117 Answers 1382 @ Competitive Examination Qs
SOLUTION FILE ¢ HOTS & Advanced Level > AIPMT & Other State Boards’
¢ Hints & Solutions for Practice Questions with Answers 133 Medical Entrance 142
Problems 119 REVISION EXERCISES 135 » JEE (Main) & Other State Boards’
CHAPTER SUMMARY & QUICK
CHAPTER ROUND UP 120) Ne eres Qh Meee eae 146
NCERT FILE > JEE (Advance) for ITT Entrance 148
@ In-text Qs & Exercises with @ NCERT Exemplar Problems
Solutions 122 (Objective Questions) 150
# NCERT Exemplar Problems with @ Hints & Explanations for Difficult
Answers & Solutions (Subjective) 128 Questions 154

[, the previous unit, we have studied that carbon atom has the
unique property of catenation due to which it forms strong
bonds with other carbon atoms to form chains and rings. It
also forms strong bonds with other elements especially
hydrogen, oxygen, nitrogen, sulphur, phosphorus and
halogens, resulting millions of compounds. These
compounds are studied under a separate branch of
chemistry called organic chemistry.
tal

‘oT,
e

et
* iherd

HC GENERAL INTRODUCTION TO ORGANIC


Simtel:
COMPOUNDS
[i
iz
*“oe
i
A cy
ha
a
oath
ist

Organic compounds are vital for sustaining life on earth.


We depend on organic compounds that occur in nature for
our food, for our clothing, for many of our medicines and for
our energy (natural gas, petroleum, etc). These organic molecules
which make our life possible include molecules like proteins which
constitute essential compounds of our blood, muscles and skin, genetic information bearing deoxyribonucleic
acid (DNA), enzymes, vitamins, lipids, carbohydrates, nucleic acids, etc. Our bodies are also regulated and
defended by complex organic molecules. So, organic chemistry plays a very vital role in our life.

12/1
WWW.JEEBOOKS.IN
a» 12/2 MODERN'S abc + OF CHEMISTRY-XI

VITAL FORCE THEORY

Until early nineteenth century, all the efforts of the chemists failed to synthesise organic compounds in the
laboratory. It was believed that organic compounds cannot be prepared in the laboratory but can only be isolated
from animals and plants. Berzelius, a Swedish chemist proposed that the synthesis of these compounds within
the plants and animals required some mysterious force. This force was called vital force and the theory was
referred to as Vital force theory.
However, the vital force theory received a crippling blow in 1828 when Friedrich Wohler, a German chemist
accidently obtained urea, (NH,),CO, an organic compound found in the urine of mammals. In fact, Wohler
tried to prepare ammonium cyanate, a substance with mineral origin, by heating ammonium sulphate and
potassium cyanate. But under the reaction conditions, ammonium cyanate rearranged to urea, a compound
which was of organic nature.

(NH,),50, + 2KCNO —- 2NH,CNO + K,SO,


Ammonium Potassium Ammonium cyanate
sulphate cyanate (Inorganic compound)

Rearrangement | Heat
|

2NH,CONH,
Urea
(Organic compound)

This chance discovery of Wohler brought about a revolution in the field of organic chemistry. The synthesis
of organic compounds no longer remained a mystery and many organic compounds were prepared in the
laboratory. For example, Kolbe (1845) prepared acetic acid, Hennel (1828) prepared ethyl alcohol, Berthelot
(1856) prepared methane etc. in the laboratory from mineral resources. Acetic acid was prepared from carbon
and hydrogen as :

eC+H, te, Ho=cH ZS


dil H2S
> cH,cHo —“““.
idati
cH,COOH
Acetylene Acetaldehyde Acetic acid
The synthesis of organic compounds altogether changed the very concept of organic chemistry. By the
middle of nineteenth century, the Vital force theory was completely discarded. Chemists then never looked
back and at present about ninety five per cent of the organic compounds are man-made.
Organic chemistry 1s now defined as the chemistry of carbon compounds as all organic compounds contain
carbon as their essential constituent. Thus,
Organic chemistry is the chemistry of compounds of carbon.
The definition appears to be incomplete since there are many carbon compounds which are of inorganic
nature. For example, carbon monoxide (CO), carbon dioxide (CO,), carbon disulphide (CS,), calcium carbonate
(CaCO,) etc., are all inorganic compounds although all of them contain carbon in them.
A detailed study of the structures of organic compounds shows that all of them contain carbon as their
essential constituent and are also invariably associated with hydrogen. Consequently, organic compounds may
be regarded as hydrocarbons. i.e., compounds of carbon and hydrogen. A large number of organic compounds
also contain elements like nitrogen, oxygen, sulphur, halogens, phosphorus, etc. which are derived from
hydrocarbons by replacing one or more hydrogen atoms in their molecules with these atoms. Thus, the modern
definition of the organic chemistry is the chemistry of the hydrocarbons and their derivatives.
TETRAVALENCY OF CARBON
The atomic number of carbon is 6 and its ground state electronic configuration is 1s*2s?2p,* 2p," 2p,°. Since
it has four electrons in its outermost shell, its valency is four. To achieve noble gas configuration, carbon atom
cannot lose or gain four electrons to form C** and C* ions because of the large amount of energy required for
these processes (ionization enthalpy and electron gain enthalpy). Thus, it has a very little tendency to form
ORGANIC CHEMISTRY: BASIC PRINCIPLES AND TECHNIQUES 12/3 =>

ionic compounds. Therefore, carbon achieves the noble gas configuration only by sharing electrons with other
atoms and it forms covalent bonds.
Thus a carbon atom forms four covalent bonds in its compounds. For example, a molecule of methane
(CH,) is formed when four electrons of carbon are shared with four hydrogen atoms as shown below :

H H
. = |
*C- + 4H* ae H iC: H or H—C—H Four C—Hbonds
are
° ™ | formed by sharing of
H H electrons
Methane

In a similar manner, carbon can complete its octet by sharing its valence electrons with the electrons of
other atoms as well. This tendency of carbon atom to form four covalent bonds is known as tetracovalency of
carbon.

Catenation : a unique property of carbon


Carbon atom has unique capacity to form bonds with other carbon atoms. This property of forming
bonds with atoms of the same element is called catenation. Carbon shows maximum catenation in its
eroup (group 14) in the periodic table. This is because of the larger bond strength of carbon to carbon bond as
compared to that of other atoms. For example, C—C bond is very strong (335 kJ mol!) in comparison to
Si—Si bond (220 kJ mol!) or Ge—Ge bond (167 kJ mol"). As a result, carbon atoms can link with each other
to form either linear chains of various lengths or branched chains and even rings of different sizes as shown
below :

Linear chain | Branched chains

a 74
; A “i <
ani > x 4
Rings of different sizes

HYBRIDISATION AND SHAPES OF MOLECULES


We have studied in unit 4 that carbon atom forms four equivalent tetrahedral bonds because of
hybridisation of its valence orbitals. This can explain the shapes of organic molecules. Let us recall the shapes
of simple organic molecules on the basis of concept of hybridisation.
1. sp? Hybridisation and shapes of alkanes
The carbon atoms in alkanes involve sp? hybridisation. As a result, the four bonds formed by each carbon
atom are directed towards the corners of a regular tetrahedron. For example, in case of methane (CH,), the
carbon atom involves sp® hybridisation and forms four sp® hybrid orbitals. Each of these forms sigma bond by

2s 2p
c
(Ground state)
| — |
C
(Excited state) “~" © =
sp® hybridisation
form four hybrid orbitals
overlapping with 1s-orbitals of hydrogen. The four bonds are directed towards the corners of a regular tetrahedron
as shown in Fig. 1 (a). The H—C—H bond angle in this molecule is 109°28' (or 109.5°). Thus, methane molecule
has tetrahedral shape.
MODERN'S abc + OF CHEMISTRY-Al

Fig. 1. Shape of methane.

The structure of methane molecule is also shown in Fig. 1 (6) and (c) .
In ethane (H,C—CH,) molecule, each carbon atom undergoes sp® hybridisation. One of the four sp® hybrid
orbitals of one carbon atom overlaps axially with similar orbital of the other carbon atom to form C—C sigma
bond. The remaining three hybrid orbitals belonging to both the carbon atoms overlap axially with the half-
filled 1s orbitals of hydrogen atoms to form C—H sigma bonds as shown in Fig. 2.

Fig. 2. Shape of ethane.

Thus, in ethane, C—C bond length is 154 pm and each C—H bond length is 109 pm.
2. sp* Hybridisation and shapes of alkenes
Alkenes are planar molecules and the carbon atoms of the C = C bond involve sp*-hybridisation. Carbon
atom has four unpaired electrons in the excited state. The three orbitals (one 2s and two 2p) get hybridised to
form three sp*-hybrid orbitals leaving one 2p, unhybridised orbital.
28 2p

c
(Ground state)

Cc
(Excited state)
sp” hybridisation orbital

For example in the case of ethylene one sp* hybrid orbital of one carbon atom overlaps with sp? hybrid
orbital of the other carbon atom to form C—C sigma bond. The remaining two sp?-hybrid orbitals of both the
carbon atoms overlap with 1 s-orbitals of two hydrogen atoms to form C—H sigma bonds.
The unhybridised 2p-orbital (shown dotted) of one carbon atom overlaps with unhybridised 2p-orbital of
other carbon atom to form pi (z) bond. The orbital structure of ethylene is shown in Fig. 3
UNHYBRIDISED
ORGANIC CHEMISTRY: BASIC PRINCIPLES AND TECHNIQUES 12/5 —_—

3. sp Hybridisation and shapes of alkynes


The two carbon atoms constituting the triple bond are sp-hybridised. In this, carbon undergoes
sp-hybridisation forming two sp-hybrid orbitals. The two 2p-orbitals (2p, and 2p,) remain unhybridised.
Qs 2p
C
(Ground state)

c(t)
igus
Ett
(Excited state)
a oar !
sp hybridisation *_two unhybridised orbitals

For example, in the -—


case of acetylene, one sp- | ian Unhybridised
hybrid orbital of one |(Riee@ 2p-orbitals a iy
carbon atom overlaps \1
with sp-hybrid orbital of
t /i

3
gand2n
106 pm|.
o and ?n n

the second carbon atom | AK agin wy | 180°


and forms C—C sigma UN y 120 pm
bond. The remaining
sp—hybrid orbital of each
C-atom forms sigma bond
Fig. 4, Structure of acetylene.
with H-atom. Each of the
unhybridised orbitals of one carbon atom forms zm-bond with the second carbon atom so that there are two
m-bonds in acetylene molecule. The structure of acetylene (ethyne) is shown in Fig. 4.
Sigma and pi bonds
We have seen that ethylene molecule contains two bonds between carbon atoms, one is sigma(c) bond
and the other is pi (7) bond. Similarly, in acetylene, there is one sigma(c) and two pi (2 ) bonds between
carbon atoms. We have already learnt about these types of bonds in Unit 4.
Sigma (co) bond is formed by the end to end overlapping of bonding orbitals along the
internuclear axis. This overlapping is known as head on overlap or axial overlap. For example, the
overlapping of sp” hybrid orbitals of two carbon atoms in ethylene or sp hybrid orbitals of two carbon atoms
in acetylene.
Pi (x) bond is formed by the sidewise overlapping of the half filled atomic orbitals of bonding
atoms. This overlap is known as sidewise overlap or lateral overlap. In this case, the atomic orbitals
overlap in such a way that their axes remain parallel to each other and perpendicular to the internuclear
axis. The orbital obtained as a result of sidewise overlap consists of two saucer type charged clouds above
and below the plane of the participating atoms.

CK SD Coa Ys) oto

Sigma bonds are stronger bonds than z bonds because during the formation of o bond, the
overlapping of orbitals takes place to a larger extent.
Important features of t-bonds. As already discussed, the double bond in ethylene molecule consists
of a c-bond and a z-bond. The z-bond has some important features as given below :
1. In ethylene, as discussed earlier, the two 2p (unhybridised) orbitals participating in the z-bond are
parallel to each other. For the proper sidewise overlap of these 2p-orbitals, all the atoms in C,H, molecule
must be in the same plane. Thus, the formation of z-bond restricts the molecule into a planar shape.
Therefore, ethylene is a flat or planar molecule.
2. Due to the n-bond formed by sidewise overlap of 2p-orbitals, the rotation of one CH, fragment with
respect to other will be hindered. The rotation of one carbon atom through 90° will break the m-bond because
a» 12/6 MODERN'S abc + OF CHEMISTRY-XI

in that case, the unhybridised 2p-orbitals become perpendicular to each other and no sidewise overlap is
possible. Hence, the rotation about the double bond is restricted or hindered. As a result, there are two
distinct forms of molecules such as C,H,Cl, is shown below :
mo _. ra oN «

SC

es eee ie >
The first one is called the cis form (cis-1,2-dichloroethene) because the two Cl—atoms he on the same side
of the double bond. The other form is called trans (trans-1,2-dichloroethene) because the two Cl—atoms lie
across the double bond from each other. Such type of molecules having same atom-to-atom bonding but different
spatial arrangements are called cis-trans or geometrical isomers.
However, it may be noted that such type of isomerism is not possible in case of ethane (C,H,) because
there is free rotation of one CH, fragment against another.
3. The electrons in the z-bonds are placed above and below the plane of the bonding atoms. These electrons
are more exposed and therefore, are easily available to the electron seeking attacking reagents. Therefore,
the z-bonded compounds are more reactive. In general, x bonds provide the most reactive centres in the
molcules containing multiple bonds.
For example, ethene readily reacts with oxidising agents like potassium permanganate (KMnO,) or
potassium dichromate (K,Cr,O-,) at ordinary temperature. On the other hand, ethane (C,H,) (which has only
c electrons) is totally unaffected by these reagents at normal temperature. In general,
t-bonds provide the most reactive centres in unsaturated molecules.

s SOLVED EXAMPLES & =


LI Example 1.
How many sigma and pi bonds are present in each of the following molecules ?
(a) CH, —C—CH, (6) CH, —CH-CH,-C=CH (c) CH,C=N
Solution : (a) CH, = C = CH,
Sx p= 2C..,=4 Total
c bonds = 6
Toc =2 Total
m bonds = 2
(6) CH, = CH-CH,
-C =f@
So_p =4,59_y=6 Total
o bonds = 10
To_-c=3 Total
x bonds = 3
(c) CH,C =N
Go_c=log_y=3 Go_yH=l Total
c bonds = 5
To _w=2 Total
m bonds = 2
LI Example 2.
What is the type of hybridisation of each carbon in the following compounds ?
() CH,Cl (tt) (CH;),CO (ti) CH,CN
(tv) HCONH, (v) CH,CH = CHCN
sp*
Solution: (i) CH,Cl : gp?
sp? sp”? sp?
(it) Ne ae : sp?, sp”, sp?
O
sp? sp
(iii) CH,C==N : sp%, sp
sp”
(iv) HC NH, : gp?
|
sp* sp sp” sp
(v) CH,CH = CHC=N_: _ sp®, sp, sp*, sp
LJ Example 3.
On the basis of type of hybridisation, predict the shape of the following molecules :
(i) HZC =O (ti) CHF (uu) HC=N

VWW.JEEBOOKS.IN
ORGANIC CHEMISTRY: BASIC PRINCIPLES AND TECHNIQUES qr}

Solution: (i) sp* hybridised carbon and therefore, shape is trigonal planar.
(ii) sp? hybridised carbon and shape is tetrahedral.
(iii) sp hybridised carbon and shape is linear.

Q@ 1. How many o and z bonds are present in each of the following molecules ?
(i) HC=C—CH = CHCH, (uz) CW.
(iii) CH, = C = CHCH, (iv) CH, = C = CH— CH = CH,
Q 2. What is the type of hybridisation of each carbon in the following compounds ?
(i) CH,CH, (ii) CH,Cl
(iii) (CH,),CO (iv) CH, =CN
Q@ 3. Give hybridisation state of each carbon atom in the following molecules :
(i) CH,—CH = CH, (a) CH, —O—@
(iii) CH, = CHCN (iv) HCONH,
Q@ 4. Write the state of hybridisation of all the carbon atoms in CH, = C = CH,.
Q@ 5. Give hybridisation state of each carbon in following compounds :
(i) CH, = C= O (ii) CH,—CH = CH, (zz) (CH,),CO (iv) CH, = CHCN
(v) CH,CH, (vi) CH,CH, (vii) CH,CH,
Q@ 6. Indicate the c and z bonds in the following molecules :
(i) C,H, (ii) ol s Fe (iit) CH,Cl,
(iv) CH,= C = CH, (v) CH,NO, (vi) HCONHCH,

to Practice Problems SS
Answers _
S—
©l. (z) 10, 3 (i) 17, 1 (Guz) 9, 2 (iv) 10, 3

© 2. (i) sp*, sp? (ii) sp? (iii) sp?, sp? (iv) sp”, sp
© 3. (i) sp*, sp”, sp? (ii) sp”, sp (tii) sp”, sp, sp (iv) sp”
© 4. Carbon 1 and 3 are sp? hybridised and carbon 2 is sp hybridised.
© 5. (i) sp?, sp (ii) sp3, sp? sp? (iii) sp? sp2, sp? (iv) sp”, sp?, sp (v) sp? sp? (vi) sp3 sp? (vii) sp? sp?
© 6. (1) 7c (ii) 170, 1m (iii) 4c (iv) 6o, 2x (Uv) Go, 2z (U1) 8c, 1n

STRUCTURAL REPRESENTATION OF ORGANIC COMPOUNDS


Structural Formulae
The atomic composition of a molecule is represented by a molecular formula. The molecular formula gives
the number and types of various atoms present in a molecule. The structures of organic compounds are
represented in several ways. The Lewis structure is the simplest method of representing any molecule by
showing the involvement of electrons between bonds. However, the Lewis structures can be simplified by
representing the two electron covalent bonds by a dash (—). In this representation, a single bond 1s represented
by a single dash (—), a double bond by a double dash (=) and a triple bond by a triple dash (=). The lone pairs
of electrons on heteroatoms (e.g., oxygen, nitrogen, sulphur, halogens, etc.) may or may not be shown. This
representation is called structural formula. For example, the structural formulae of ethane (C,H,), ethene
(C,H,), ethyne (C,H,) and methanol (CH,OH) may be shown as below :
H H H H 7 H
| | || . |
no H—C=—C—H H—C=C—H nn or a

H H H H
Ethane (C,H,) Ethene (C,H,) Ethyne (C,H,) Methanol (CH,OH)
Condensed Formulae
The structural formulae as shown above are time consuming. These may be simplified in a condensed
formula. In this formula, the arrangement of atoms are shown but the bonds (dashes) between (some or all)
may be omitted and the number of identical groups attached to an atom are indicated by a subscript. For example,
a» 12/8 MODERN'S abc + OF CHEMISTRY
-XI

H HH
| | |
H- 7- s- —H may be represented as CH.-CH,—CH, or CH,CH,CH,

H HH
Similarly, ethane, ethene, ethyne and methanol may be written as
CH,CH, H,C = CH, HC = CH CH,0OH
or C,H, or C,H, or C,H, Methanol
Ethane Ethene Ethyne
The long condensed formulae may be further simplified wherever possible. For example,
CH,CH,CH,CH,CH,CH,CH,COOH can be condensed to CH,(CH,), COOH.
Bond-line Structural Formulae
To further simply the formulae, the organic chemists use another way of representation known as bond
line structural representation. In this representation, the carbon and hydrogen atoms are not shown and
the lines between carbon-carbon bonds are shown in a zig-zag manner. For example, hexane has a continuous
chain of six carbon atoms which may be represented as :
CH,—-CH,—CH,—CH,-CH,—-CH,, 4S HF
Hexane
The atoms other than carbon and hydrogen are written. The terminals denote methyl (-CH,) groups
unless indicated otherwise by a functional group while the line junctions denote carbon bonded to
appropriate number of hydrogen atoms required to satisfy the valency of the carbon atoms. For example
the molecule, octanoic acid, CH,CH,CH,CH,CH,CH,CH,COOH may be represented in bond line formula as :

AAA on
Similarly, 3-methyloctane may be represented by the following forms as :

(i) CH,CH,CH CH,CH,CH,CH,CH,


|
CH,
| CH CH CH CH

(it) CH, CH CH} CH,

on
(LiL) \

Terminals represent
methyl groups
Similarly, 3-chloropentane may be represented as
@ CH,CH,CH CH,CH,
Cl
| CH CH
- Lo Se So ~*~
il) CH, an CH,
Cl
(iit)

methyl
groups
ORGANIC CHEMISTRY: BASIC PRINCIPLES AND TECHNIQUES 12/9 >»

Cyclic Compounds or Polygon Formulae


There are many organic compounds in which carbon atoms are joined by rings. These compounds containing
one or more rings are called cyclic compounds. These are represented by drawing the appropriate ring (polygon)
without showing the carbon and hydrogen atoms. In this representation, the corner of the ring (polygon)
represents a carbon atom and the sides of the polygon denote a carbon-carbon bond. The atom or group of
atoms other than hydrogen are shown in the structure. For example,

CH,
ad. = is represented as /\
Cyclopropane
CH,
He CH, :
HC____CH,
| is represented as
. O
CH,
Cyclohexane

Some other examples of cyclic compounds are : —

| ~ J
Cyclobutane Cyclopentane Chlorocyclohexane Cyclohexanol
Three Dimensional Representation of Organic Molecules
The three dimensional (8—D) structure of organic molecules is very difficult to draw on a paper (two
dimensional). Therefore, certain graphic conventions have been proposed. The simplest convention 1s solid and
dashed wedge formula, in which 3—D image of a molecule can be preceived from two dimensional picture. In
this representation, a tetrahedral molecule with four atoms or groups a, b, c and d bonded to it can be represented
by a wedge formula. A solid wedge ( ~a ) (or a heavy line) is used to indicate a bond projecting above the plane
of the paper (i.e. bond projecting towards you) and the dashed wedge (mi) (or a dashed line) is used to depict
the bond below the plane (i.e., bond pointing away from you). The bonds lying in the plane of the paper are
shown by normal lines. For example, in the tetrahedral molecule Cabcd, the two bonds C-a and C—} are shown
by the normal lines and are in the plane of the paper, the C —-c bond connected by solid wedge is intended to be
in front of the plane while C —d_ bond connected by dashed line is behind this plane. The wedge and dash
representation of CH, molecule is also shown below :

Dashed wedge (Bond


a away from observer) H
Bonds in the—————> |
plane of mre C
Min
\
Cu My

oe
Solid wedge .
i
(Bond towards | H
observer) Wedge and dash representation
Projection Formulae
In organic molecules, the three dimensional arrangement of atoms in space can be represented in plane by
projection formulae. These formulae are obtained by actually projecting the 3—D model of a molecule using a
ray of light. The two dimensional image of the molecule is drawn on the paper according to certain conventions.
The projection of the molecule can be obtained by orientating the molecule in different ways with respect to the
plane of the paper. There are three different types of projection formulae. These are Fischer projection, Newmann
projection and Sawhorse projection formulae. These are discussed in the next chapters.
Molecular Models
To have better visualisation and perception of three dimensional shapes of organic molecules, the molecular
models are used. These are made of wood, plastic or metals and are commercially available. Generally, three
types of molecular models are used. These are:
(t) Framework model (it) Ball and stick model and (tit) Space filling model.
a 12/10 MODERN'S abc + OF CHEMISTRY-Al

({)In the framework model, only the bonds connecting the atoms of a molecule are shown. In this
model, atoms are not shown. This model gives the pattern of bonds of a molecule.
(it) In the ball and stick model, both the atoms and the bonds are shown. In this model, balls represent
atoms and sticks represent bonds. The unsaturated compounds containing double bonds or triple bonds (C = C
or C =C) are represented by using springs in place of sticks.
(iii) Inthe space filling model, emphasis is given on the relative size of each atom. In this model, bonds
are not shown. This model also tells the volume occupied by each atom in the molecule.
In addition to these models computer graphics are also used these days for molecular modeling.
These three types of models are shown for methane molecule in Fig. 5.

({) Framework model (ii) Ball and stick model (iii) Space filling model
Fig. 5. Different molecular models for methane (CH,).

The organic molecules may be represented in the following ways :


H H H H H O H H
| | | 1 | |} |
Structural formula H-—-C-—-C—C-—OH H—-C-C-C-C-C-H
| | | | | | |
H HH H H H H
Condensed formula CH,CH,CH,OH CH,CH,COCH,CH,

Bond line structural | OH ~_a


formula J/\f i

Three dimensional formula


H

Lk
|
_ Nu
H

Wedge and dash representation Molecular model (Ball and stick model)

za SOLVED EXAMPLES &


LU) Example 4.
For each of the following compounds write the condensed formula and bond line formula :
i
|
(i) CH 3-H 9CH oCH. CH eH | aa 97 C-O (i) CH. 37 as —-CH oUH

Br CH 3

(LiL) sda lian (iv) HO Og CB CGE CS CH,


[3
OH
(v) NH,- CH, CH, CH, CH, CH, CH, COOH
CH,
Solution : Condensed formula:
(i) CH, (CH,),CHBrCH,CHO (i) (CH,), CHCH,OH (wt) (CN),CHOH
(iv) HO(CH,),CH(CH,)CH(CH,), (v) H,N - (CH,), - COOH
ORGANIC CHEMISTRY:

Bond line formula :


BASIC PRINCIPLES AND TECHNIQUES
an
wo /S/VYN a
Br O

OH
(ii) >
OH

|
(v) aw ~ YY Now

Example 5.
Expand each of the following condensed formulae into their complete structural formulae :
(a) CH,CH = CH (CH,),CH,
(b) HOCH,CH,NH,
(c) CH,(CH,), CH, OH
(d) CH,CH,COCH,CH,
Solution :

(a) a
PERT RRR
ae al ia a ae (b) ae
iaa aa

(c) a
Pee 7 On (d) a
Peat a -¢-C-H
H H H H H H HH

Example6.
Draw all polygon formulae for the molecular formula CH 9.
Solution : The different polygon formulae of the compound having molecular formula C,H,,, are :

(i) Cy @ |) wi ww (iv) x (v)

Example 7.

Expand each of the following bond line formulae to show all the atoms including carbon and hydrogen.
O

(u) (ii)
Cl Br

(iv) oN
OH

WWW.JEEBOOKS.IN
a 12/12 MODERN'S abc + OF CHEMISTRY-XI

Solution :

fs A
O /
1
CH,CH,CH, C CH,CH,
| Br

(ii) 4>C™ CH, CH,—C_CH, CH, CH, CH,


Go Br L

(LLL)
COX

ww ne

° DX
"ANNI

=D
ATnaE
teems

QO 7%. Rewrite each of the following condensed structural formulae, as dash structural (or dash) formulae and as bond line
formulae :
(i) (CH,), C CH,CH, (i) CH, C CH,CH(CH,), (iii) CH,CH(OH)CH,CH,OH

. Write structural formulae for each of the following bond line formulae :

(i) \ on iy (iii) CL (iv) ANN

. Write bond line formulae for the following :


(i) HO (CH,), CH CH, CH (CH,), (i) CH, C= CH CH, CH,
CH, |
© Ans. (i) ao > i) Sa
O10. Write three dimensional (wedge-dashed wedge—line) representation for the following :
(i) CH,Cl (ti) CH,CH,OH
ORGANIC CHEMISTRY: BASIC PRINCIPLES AND TECHNIQUES 12/13 =>

CLASSIFICATION OF ORGANIC COMPOUNDS


The simplest organic compounds containing only carbon and hydrogen are calld hydrocarbons. These
are regarded as the parent organic compounds and all other compounds are considered to be derived from them
by the replacement of one or more hydrogen atoms by other atoms or groups of atoms. They have been broadly
classified as :
1. Open chain or acyclic compounds.
2. Closed chain or cylic or ring compounds.
1. Open chain or acyclic compounds
These compounds contain open chains of carbon atoms in their molecules. The carbon chains may be either
straight chains or branched chains. They are called aliphatic compounds because the earlier compounds of
this class were obtained either from animal or vegetable fats (Greek aliphatos meaning fats).
H H H CH, CH,
| | | |
ame ee = CH,—CH—CH, CH; fp CH—CH=CH—CH,
H H H CH,

rf ey
Methane Ethane Isobutane Neopentane But-2-ene

|
es in cme i lai a= 7 dm H—C—C =O
H | |
H H H H OH
Acetaldehyde Ethyl amine Ethyl alcohol Acetic acid
2. Closed chain or cyclic or ring compounds.
These compounds contain closed chains or rings of atoms in their molecules. These compounds may be
carbocyclic (containing rings of carbon atoms only) or heterocyclic (containing rings of carbon atoms or one
or more heteroatoms). The cyclic compounds are of three types :
(¢) Alicyclic compounds . These compounds contain a ring of three or more carbon atoms in them. They
resemble aliphatic compounds in many of their properties. For example,

2 al OO
CH, wr CH, 2 \ — CH ae
CH i

Hy
C——_ CH, H,C ——— CH,
H,C
2 ™CHy~
CH. a H,C
2 Non
CH
2
Cyclopropane Cyclopentane Cyclohexane Cyclohexene

(iz) Aromatic compounds. These have a cyclic system containing at least one benzene ring. The parent
member of the family is called benzene. Benzene has a homocyclic hexagonal ring of six carbon atoms with
three double bonds in the alternate positions. Benzene can be represented by any of the following structures :
H H
| |
H-C~ . SC—H Le “C—H
H—C~ | |
i AT oo | I 7 CF -
H—C C—H H—C C—H
Ne ScC7 |
| |
H H
Some other examples are :
CHa ia CHO Nie
|
C C Cc C
H-c~ Sc_H Hc“ Sc-H pec OU
Sc_H — ~~
| | l | | |
H—C C—H H—C C—H Cc H—C C—H
NO N oF H—Cw ae H N07
| | |
H H in H
Toluene Phenol Benzaldehyde Aniline
a 12/14 MODERN'S abc + OF CHEMISTRY-XI

These compounds are also called benzenoid compounds and they exhibit special property known as
aromaticity (discussed later). Bicyclic and tricyclic compounds also belong to this category.

ao
Naphthalene
acc Anthracene
ay
Phenanthrene
(Bicyclic) (Tricyclic) (Tricyclic)
Compounds containing two or more benzene rings fused together are called polycyclic aromatic compounds.
Besides, compounds containing fused benzene rings, benzenoid compounds may contain isolated benzene
rings. For example,

Biphenyl or diphenyl Terphenyl


In addition to these, there are some compounds which show aromaticity but donot contain any benzene
ring. These are called non-benzenoids. For example,

A
Tropone
oS
Tropolone Azulene
(tit) Heterocyclic compounds. In these compounds, the ring contains one or more atoms of either nitrogen,
oxygen or sulphur in addition to carbon atoms. The atom other than carbon (such as N, O, S) present in the ring
is called hetero atom. Some common examples are :

TO | TT OP TP
H.C CH HC CH HC af HC
Oe
|
CH | CH
\/ \/ Sg \/ “ sn
| |
Tetrahydrofuran H
(THF) Pyrrole Furan Thiophene Pyridine
Thus, organic compounds may be classified as :
ORGANIC
COMPOUNDS

Acyclic or open Cyclic or closed chain

Homocyclic or — ~ Heterocyclic
carbocyclic compounds compounds

Alicyclic Aromatic
compounds compounds

Benzenoid Non-benzenoid
compounds compounds
ORGANIC CHEMISTRY: BASIC PRINCIPLES AND TECHNIQUES 12/15 —_—_—

FUNCTIONAL GROUP
The organic compounds except alkanes contain some structural features which make it possible to
classify compounds by reactivity. These are called functional groups. These functional groups are responsible
for the characteristic properties of the compounds. A functional group is a group of atoms within a molecule that
has a characteristic chemical behaviour. Thus, functional group may be defined as
an atom or group of atoms that determines the characteristic chemical properties of an organic
compound,
For example, one of the simplest functional groups is the carbon—carbon double bond. It makes a special
class of organic compounds called alkenes.
H H = oft
Paw Functional group : Ze =C.,
H: H
Ethene
In some cases, the carbon bonded to some other atoms such as N, O, S, P, etc. acts as a functional group.
For example, the family of alcohols have its characteristic properties due to the presence of —OH (hydroxy)
croup, called functional group. Similarly, aldehydes owe their characteristic properties to the functional group
—CHO (aldehydic), carboxylic acids have properties due to —COOH (carboxylic acid) group and so on.

Hee 7" HO"


{pf y | a ta“

H—C-LO—H } Hct H—C-ic


| Z ‘

i | 7 i
rd | 7 N
H| ‘ “Sie " % H A *, H ail H = Saar
Aldehyde 7" Functional Carboxylic "rs" Fonctional
Alcohol Functional
acids group
croup
eToup

Some Important Functional Groups and Families


1. Functional groups containing carbon-carbon multiple bonds
Some compounds containing carbon-carbon multiple bonds give characteristic reactions and constitute
different families. These are :

Family Characteristic structural feature or group

ia ri
Alkenes ZX = — Carbon-carbon double bond

Alkynes —C iyC— Carbon-carbon triple bond

ZoN
Arenes —C — Alternate single and double bonds
| in a six membered ring of carbon
Ce » CS atoms. The ring is also called
C aromatic ring.

2. Functional groups with carbon bonded to other atoms like O, N, 8S or P.


Some compounds contain functional groups containing carbon bonded to other atoms like O, N, 5 or P.
These are bonded by single bonds as well as by double bonds. For example, the common families containing
carbon bonded to other atoms by single bonds are :

Compound Structure Functional group

Alkyl halide scx >

Alcohols —c—0H —- OH

Ethers Sc—o—ce Hx

WWW.JEEBOOKS.IN
Slane MODERN'S abc + OF CHEMISTRY-Al

Amines C_—NH, -NH,

Thiols >C—SH _SH

In amines carbon is bonded to N and in thiol carbon is bonded to sulphur.


The families containing carbon bonded to oxygen by double bond are :
Compound Structure Functional group

Aldehydes eFe _
=)

|
Ketones Sc—c—cZ

Carboxylic acids

Esters

Acid halides

Amides
Sey
Bs gag
oe NH,

Nitrile xa
.. ——

‘R’ stands for alkyl groups such as CH,-, C,H.-, C,H,-, ete.
* In nitriles, carbon is bonded to N by a triple bond.

HOMOLOGOUS SERIES
The organic compounds have been classified into various families. Each family contains closely related
organic compounds so far as their structures and chemical properties are concerned. The different families or
classes are known as homologous series. A homologous series may thus, be defined as
a series of similarly constituted compounds in which the members possess the same functional
group and have similar chemical characteristics.
The two consecutive members differ in their molecular formula by —CH, group.
The different members of a series are known as homologues. A few members of the alcohol family are
civen below :

Formula Name

CH,OH Methyl alcohol


C,H,OH Ethyl alcohol
C,H,OH Propyl alcohol
C,H,OH Butyl alcohol

Characteristics of homologous series. The common characteristics of the members present in a


homologous series are :
(i) All the members of a series can be represented by the general formula. For example, the members of
the alcohol family are represented by the formula C,H OH where n may have values 1, 2, 3......... etc.

WWW.JEEBOOKS.IN
ORGANIC CHEMISTRY: BASIC PRINCIPLES AND TECHNIQUES 12/17 =>

(ti) Two successive members differ in their formula by —CH, group or by 14 atomic mass units (12 + 2 x 1).
(iii) Different members in a family have common functional group e.g., the members of the alcohol family
have —OH group as the functional group.
(iv) The members in any particular family have almost identical chemical properties and their physical
properties such as melting point, boiling point, density, solubility etc., show a proper gradation with the
increase in the molecular mass.
(v) The members present in a particular series can be prepared almost by similar methods known as the
general methods of preparation.
DIFFERENT CLASSES OF ALIPHATIC COMPOUNDS AND THEIR NOMENCLATURE
Nomenclature implies assigning proper name to a particular organic compound on the basis of certain
standard rules so that the study of these compounds may become systematic. In case of aliphatic compounds,
two systems of naming are generally used :
(A) Trivial system (B) IUPAC system.
(A) Trivial system.
In earlier days, the organic compounds were named after the source from which they were obtained. For
example, urea got its name because the compound was obtained from the urine of the mammals. Similarly.
methyl alcohol was called wood spirit since it could be obtained as one of the products during the destructive
distillation of wood; formic acid derived its name from the Greek word formicus (red ants) since the acid could
be obtained from red ants. These names are without any systematic basis and are known as common names
or trivial names. Such a system of nomenclature is known as trivial system.
The common names of some popular organic compounds are given in Table 1.

Table 1. Common or trivial names of some organic compounds.

Compound Common name Compound Common name

CH, Methane (CH,CH,),O Diethyl ether


yd Fe Ethylene CHCl, Chloroform
Sol Ee Acetylene CHI. Iodoform
H,CCH,CH,CH, n-Butane CH,CN Acetonitrile
(H,C),CHCH, Isobutane CH,COOH Acetic acid
(H,C),C Neopentane CH,OCH,CH, Ethyl methyl ether
HCHO Formaldehyde tale Benzene
CH,CHO Acetaldehyde CCH, Toluene
(H,C),CO Acetone Ce NE Aniline
CH,CH,OH Ethyl alcohol C,H.OH Phenol
CH,CH,CH,OH n-Propyl alcohol C,H,OCH, Anisole
CH,CONH, Acetamide C,H.COCH, Acetophenone
CH,OCH, Dimethyl ether C,H,CONH, Benzamide

(B) IUPAC system.


In order to rationalise the system of naming, an International Congress of Chemists was held in Geneva in
1892. They adopted certain uniform rules for naming the compounds.
The system of nomenclature was named as Geneva system. Since then the system of naming has been
improved from time to time by the International Union of Pure and Applied Chemistry and the new system is
called IUPAC system of naming. This system of nomenclature was first introduced in 1947 and was modified
from time to time. The most exhaustic rules for nomenclature were first published in 1979 and later revised and
updated in 1993. The rules discussed in the present chapter are based on guide books published by IUPAC in
1979 (Nomenclature of Organic Chemistry by J. Rigandy and 8.P. Klesney) and 1993 (A Guide to TUPAC
Nomenclature for Organic Chemistry by R. Panico, W.H. Powell and J.C. Richer). With the help of this
system, an organic compound having any number of carbon atoms can be easily named.
Though most of the books discuss nomenclature based on the TUPAC recommendations of 1979, we include
here the recommendations of 1993 also to provide LATEST UPDATED RULES.
a 12/18 MODERN'S abc + OF CHEMISTRY-XI

IUPAC System of Naming Organic Compounds


In the IUPAC system, the name of an organic compound consists of three parts :
(t) Word root (it) Suffix (iit) Prefix.
(¢) Word root. The word root denotes the number of carbon atoms present in the chain. For chains containing
upto four carbon atoms, special word roots (meth—C,, eth—C,, prop—C, and but —C,) have been used while
for those containing more than four carbon atoms, Greek numerals have been used to represent the word root.
For example,
Chain length Word root Chain length

Oct
Non
Dec
Undec
Dodec
Icosane
Triacontane

(ti) Suffix. The word root is linked to the suffix which may be primary or secondary or both.
(a) Primary suffix. Jt indicates the nature of linkages in the carbon atoms. For example, if the carbon
atoms are linked by single covalent bonds (C—C), the primary suffix -@ne is used. Similarly, for a double bond
(C = C) between the carbon atoms, the suffix is -eme while for the triple bonded carbon atoms (C = C),
suffix -yne is used. Thus the suffixes are :
ane — for single bonded compounds, C—C
ene — for double bonded compounds,C — C
yne — for triple bonded compounds, C =C
If the parent chain contains two, three or more double or triple bonds, then the numerical prefixes such as
di (for two), tri (for three), tetra (for four), etc. are added to the primary suffix.
It may be noted that extra ‘a’ is added to the word root if the primary suffix to be added begins with
a consonant (other than a, e, i, 0, wu).
For example, for two double bonds, suffix is diene and if it is to be added to word root but (for 4C
atoms), it becomes butadiene.
(b) Secondary suffix. [ft indicates the presence of functional group in the organic compound. Some common
secondary suffixes used to represent functional groups are :

Class of organic compound Functional group Secondary suffix

Alcohols
Aldehydes
Ketones
Carboxylic acids —olc acid
Esters alkyl ...... oate
Acid chlorides —oyl chloride
Acid amides —amide
Nitriles —nitrile
Amines | —amine

It may be noted that while adding the secondary suffix to the primary suffix, the terminal ‘e’ of the primary
suffix (i.e., ane, ene or yne) is dropped if the complete secondary suffix (the suffix plus multiplying affix, if any,
such as di, tri, tetra) begins with a vowel (a, e, i, o, w). However, the terminal —e’ is retained ifthe secondary suffix
begins with a consonant.
For example,
CH.—CH,—CH,—CH,—_OH has word root but
pri. suffix ane
sec. suffix -ol

WWW.JEEBOOKS.IN
ORGANIC CHEMISTRY: BASIC PRINCIPLES AND TECHNIQUES 12/19 =>

So, its name is But + ane + ol = Butanol or Butan-1-ol


[Here, e of ane is dropped because sec. suffix -ol begins with a vowel. Please note 1 before -ol indicates the
position of the sec. suffix discussed later on].
CH.—CH,—_CH—CH,, has word root but
| | pri. suffix ane
OH OH complete sec. suffix diol
So, its name is But + ane + diol = Butanediol or Butane -1, 2-diol
[Here e of ane is not dropped because complete sec—suffix (diol) begins with a consonant; 1—and 2—before
diol indicate the positions of the sec—suffix, -OH groups].
(iit) Prefixes. There are many groups which are not regarded as functional groups in the IUPAC name of the
compound. These are regarded as substituents or side chains. These are represented as prefixes and are
placed before the word root while naming a particular compound. These may be :
(a) Alkyl groups. These groups contain one hydrogen atom less than the alkane. These are named by
substituting the suffix ane of the name of the corresponding alkane by yl.
t.e. alkane — ane + yl = alkyl.
For example,
CH, Methane becomes CH.— : Methyl
CH.CH, : Ethane becomes CH.CH,— > Kthyl
CH, CH,CH, : Propane becomes CH.CH,CH,— : Propyl etc.
(b) Functional groups not regarded as principal functional groups. Ifa compound contains more than
one functional group, then one of the functional group is regarded as principal functional group and is treated
as secondary suffix. The other functional groups are regarded as substituents and are indicated by prefixes. This
is discussed later in this chapter.
Some common prefixes are given below :

Substituent Substituent

Nitroso
Diazo
Methoxy
Ethoxy
Hydroxo

Thus, a complete IUPAC name of an organic compound may be represented as :


Prefix + Word root + Primary suffix + Secondary suffix.
For example, the following compound has a chain of four carbon atoms (word root is but). It contains only
C-—C bond (primary suffix is -ane), It contains functional group —OH at C—1 (secondary suffix is -ol). It also
contains a substituent Cl at C—3 (prefix is chloro).
Functional
4 3 f/f 1 Y tae
CH,—CH—CH,—CH,—OH) — Word root : But
Primary suffix : -ane
Prefix Secondary suffix : -ol
Cl) Prefix : —Chloro

IUPAC name : Chloro + but + ane + ol


3-Chlorobutan-1-ol

The e of —ane is dropped because it is followed by o (vowel). Numbers 1 and 3 represent the positions of suffix
and prefix.
lon 12/20 MODERN'S abc + OF CHEMISTRY-XI

Let us consider the following compound. It has a chain of five carbon atoms having functional group —COOH.
It also contains a double bond at C—2 — a substituent (—Br) at C—4.
Pr]
Yee
certs,
=,

CH Culex —¢ Coy Word root : Pent (five C-C—C—C-C)


Prafz Lyfe ™ en Primary suffix : ene (double bond at C—2)
—~Br) ) Pri. atk Séc. suffix Secondary suffix : oic acid (-COOH group)
nd Prefix : Bromo (—Br group at C—4)
IUPAC name: Bromo + pent + ene + oic acid
or 4—Bromopent-—2-en-1l-oic acid
Representation of Organic Compound
An organic compound may be represented by its molecular formula, structural formula or graphic formula.
For example, propane molecule may be represented as :

C,H, CH,—CH,—CH, ae —
iii C —H
Molecular formula Structural formula |
H H H
Graphic formula
Structural formula is often used to represent an organic compound. Some time the structural formula is written
without bonds e.g., propane may be written as CH,CH,CH.,. This is also known as condensed structural formula.
Before taking up the IUPAC nomenclature of different compounds, let us study common and IUPAC names
of some common classes of organic compounds.
Different Classes of Organic Compounds
1. Hydrocarbons
These are the organic compounds containing carbon and hydrogen atoms only. They may be further classified
into two classes : saturated and unsaturated hydrocarbons.
(a) Saturated Hydrocarbons — Alkanes
General formula = C, H,, . 5 Suffix : ane
These are the organic compounds which contain only carbon—carbon single bonds. These were earlier named
as paraffins (Latin : meaning Jittle affinity) due to their least chemical reactivity. According to IUPAC system,
these are named as alkanes (ane is suffix with word root). Various members of the series are given below :
Formula IUPAC name Common name
CH, Methane Methane
CH,CH, Ethane Ethane
CH,CH,CH, Propane Propane
CH,CH,CH,CH, Butane n-Butane
CH(CH,).CH. Pentane n-Pentane
CH,( CH,) (CLL. Hexane n-Hexane

Classification of carbon atoms in alkanes. The carbon atoms in an alkane molecule may be classified
into four types as primary (1°), secondary (2°), tertiary (3°) and quaternary (4°).
(i) A carbon atom attached to one other (or no other) carbon is called primary carbon
(written as 1° carbon).
(ii) A carbon atom attached to two other carbon atoms is called secondary carbon
(written as 2° carbon).
(ii) A carbon atom attached to three other carbon atoms is called tertiary carbon
(written as 3° carbon).
(iv) A carbon atom attached to four other carbon atoms is called quaternary carbon
(written as 4° carbon).
Hydrogen atoms attached to 1°, 2° and 3° carbon atoms are referred to as primary, secondary and tertiary
H atoms respectively. The following example, clearly differentiates the various carbon atoms.
VCH,
; “| : ‘
Cre 0 = Gun
A oll
SHEL (C18,
ORGANIC CHEMISTRY: BASIC PRINCIPLES AND TECHNIQUES 12/21 »»>

Alkyl radical
The alkanes are also represented by the general formula RH where R is called the alkyl group or radical
and has the formula C_H,,_,. The alkyl group, therefore, contains only one H atom less than the alkane. The
alkyl group is named by substituting the suffix ‘ane’ of the name of the corresponding alkane by ‘yl’. (alkane—
ane + yl = alkyl). For example,
CH,, Methane becomes CH,— Methyl
CH.CH.,, Ethane becomes CH,CH,— Ethyl
Alkyl radical Corresponding alkane
CEL CH 4
CH,CH,— Cu
CH,CH,CH,— C,H,
CH,CH,CH,CH,— Cale
CH,CH,CH,CH,CH,— Cate
CH,(CH,), CH,— Ci pHa,

It may be noted that there is one methyl radical and one ethyl radical. However, there are two propyl
radicals depending upon the nature of H atom removed. There are two types of carbon atoms in propane; the
two end carbon atoms are primary (attached to only one other carbon atom) while the middle carbon is secondary
(attached to two other carbon atoms). The removal of hydrogen may occur as :
(i) The six primary H atoms shown within dotted lines in the formula are equivalent (1°). Removal of any
one of these six H atoms gives an alkyl radical called normal propyl group or simply written as n-propyl.
(it) Removal of one of the two secondary H atoms shown within solid circles in the formula for propane
cives different propyl group called isopropyl group.

raaty ® fH “tn : Ll
—— — a

fod
Ve]
eH
|
et
wg
ae | ee
HH
n-Propyl
ae “Rh OdfA sa
Propane |

aeH
; Isopropyl
Thus, there are two propyl! radicals :
CH,CH,CH,— CH,CHCH,
n-Propyl |
Isopropyl
Similarly, butane gives four butyl radicals (two from n-butane and two from isobutane) as
a 12/22 MODERN'S abe + OF CHEMISTRY-XI
we arl-ss, sa,
e# &4
H H
f Ya oa ™

Be 6
o EL as vas
H— c—ae
ra | . a Nobu, H
(He (fy ieheal
ED
SHE H,

Isobutane tert-Butyl

Thus, there are four butyl groups, namely


CH,CH,CH,CH,— CH,CH,CHCH,
n-Butyl |

rm
sec-Butyl

i i a CH,—C—CH,
|
CH,
Isobutyl tert-Butyl
Similarly, removal of different H atoms in pentane gives the following radicals :
CH, CH,
| |
CH,CH,CH,CH,CH,— CH,CHCH,CH,— CH,CCH,— CH,CHCH,CH.CH, CH,CCH,CH,
-Pentyl
n-Fenty. |
CH, |
CH, | |
Isopentyl Neopentyl sec-Pentyl tert-Pentyl
The prefix sec- or tert-before the name of the group indicate that the H atom was removed from a secondary
or tertiary carbon atom respectively. It may be noted that an iso compound is one which has methyl group
attached to second last carbon atom of the continuous chain (discussed later). For example,

CH,—CH—CH, CH,CH+cH/CH, CH,—CH—CH,—CH,—CH,


| | | : :
CH, CH, CH,
Isobutane Isopentane Isohexane
Thus, the number of different alkyl groups produced from an alkane depends upon how many types of
H atoms (primary, secondary or tertiary) can be replaced.

It may be noted that sec- and fert-compounds are written with a dash between sec or tert and name of the
alkyl group. For example, sec-Butyl, tert-Butyl etc. However, iso or neo compounds are written as one
word. For example, isobutane, neopentane, etc.

Abbreviations for alkyl groups


It may be noted that some of the alkyl groups are also written using abbreviations. For example,
methyl group is written as Me. The common abbreviations for the alkyl groups are :
Alkyl group Abbreviation
CH, — Methyl Me —
CH,CH, — Ethyl Et —
CH,CH,CH, — Propyl Pr —
CH,CH,CH,CH, — Butyl Bu-
. (6) Unsaturated hydrocarbons. These are hydrocarbons which contain carbon to carbon double bonds
(_C = C_) or carbon to carbon triple bonds (—C == C—) in their molecules. These are further classified into two
types :alkenes and alkynes.
(1) Alkenes : General formula: C_H,, Suffix : ene
The molecules containing one carbon-carbon double bond are called alkenes. They have the general formula
C_H,,,. These were previously called olefins (in Greek olefiant—oil forming) because the lower gaseous members
of the family form oily products when treated with chlorine.
ORGANIC CHEMISTRY: BASIC PRINCIPLES AND TECHNIQUES 12/23 ==

In IUPAC system, the name of alkene is derived by replacing suffix ane of the corresponding alkane by
ene. For example,
CH,—CH, H,C = CH,
Ethane (Alkane) Ethene (Alkene)
In higher alkenes, the position of the double bond can be indicated by assigning numbers 1, 2, 3, 4, .... to
the carbon atoms present in the molecule. The position is mentioned by the first carbon of the double bond. The
number is written immediately before the part of the name to which it belongs. For example,
CH, = CH—CH,—CH, CH,—CH = CH—CH,—CH,
But—1l-ene Pent—2-ene
(Not But-—3-ene) (Not Pent—3-—ene)
The IUPAC names of first six members of the family are given below. The common names* are also given
below :

Alkene Common name IUPAC name

CH, = CH, Ethylene Ethene


CH,CH = CH, Propylene Propene
CH,CH,CH = CH, a-Butylene But-1l-ene
CH,CH = CHCH, 6-Butylene But-2-ene
CH, (CH,),CH = CH, a-Pentylene Pent-1-ene
CH,CH,.CH = CH.CH, 6-Pentylene Pen-2-ene
CH,(CH,),CH = CH, a-Hexylene Hex-1-ene
CH,(CH,),CH = CHCH, 6B-Hexylene Hex-2-ene
CH,CH,CH = CHCH,CH, v-Hexylene Hex-3-ene

(ti) Alkynes : General formula : C_H,, , Suffix : yne


Alkynes are the unsaturated hydrocarbons with a triple bond (—C = C—). They are represented by the
general formula C_H,,..
Alkynes are named in the same way as alkenes i.e., by replacing suffix ane of alkane by yne. In higher
members, the position of triple bond is indicated by giving numbers 1, 2, 3, 4, ... etc. to the carbon atom in the
molecule. The numbering of the chain is always done from one end in such a manner that the triple bonded
carbon atom gets the least number, as indicated in alkenes. For example,
CH,—C = CH CH, CH, —C = CH CH,—C =C—CH,
Prop-1-yne But-1l-yne But-2-yne
(Not Prop-2-yne) (Not But-3-yne)
The IUPAC and the common names of a few important members of the family are given below :

Common name IUPAC name


Acetylene Ethyne
Methyl acetylene (Allylene) Propyne
Dimethyl acetylene But—2-yne
Ethyl acetylene But—1—yne
|e hs Se | n—Propyl acetylene Pent—1—yne
H,C—CH,—C = CF, Ethyl methyl acetylene Pent—2—yne
1,C—-+(CH,).—C
Hi, 2/3 = CH n—Butyl
ty acetylene
ty Hex—l—yne
tf
H,C—(CH,), Ge C—CH, Methyl n-propyl acetylene Hex—2-yne
H,C—CH,,—_¢ = Sa 7H, Diethyl acetylene Hex—3—yne

2. Alkyl halides (Halogen derivatives of alkanes)


Halogen derivatives of alkanes are called alkyl halides or haloalkanes
These are obtained from alkanes by the replacement of one or more hydrogen atoms by halogen atoms :
RH —+* ,ex X=FClLBrD
C,H, ———> C,, HCl
—H, + Cl

*The common names of alkenes are derived from corresponding alkanes by replacing the suffix ane of alkane by ylene
as ethylene, propylene, butylene etc. In higher alkenes, sometimes the carbon atoms are indicated as a, B, y, 6...... . so that
the position of double bond can be correctly mentioned. For example,
fe) v 6 OL a) ¥ B a
CH,—CH,—CH = CH, CH,—CH = CH—CH,
a-Butylene B-Butylene

WWW.JEEBOOKS.IN
a 12/24 MODERN'S abe + OF CHEMISTRY-XI

These are further classified as mono-, di -, tri— and tetrahaloalkanes, etc. according as they contain one,
two, three, four etc. halogen atoms respectively in their molecules.
(a) Monohaloalkanes
General formula : C,, Hy, ,X or RX Prefix : Halo
These are derived from alkanes by replacing one hydrogen atom by halogen atom. The common names of
these are derived by adding the word halide (fluoride, chloride, bromide or iodide) to the name of the alkyl
croup i.e. alkyl + halide = alkyl halide.
In the IUPAC system, they are named as halogen substituted alkanes 1.e., haloalkanes. Thus, the name
of halogen compound is given by prefixing fluoro, chloro, bromo or iodo to the name of the alkane. The position
of the halogen atom is also indicated by giving the number of C atom to which halogen is attached.
Structural formula Common name IUPAC name

CH,Cl Methyl chloride Chloromethane


CH,CH,Cl Ethyl chloride Chloroethane
a tala Isopropyl iodide 2-Iodopropane
I
15 6,03 5 ER Os Pe a Sec-butyl bromide 2-Bromobutane
|
Br
CH,CH,CH,CH,I n-Butyl iodide 1-Ilodobutane

(6) Dihaloalkanes
Alkanes containing two halogen atoms per molecule are called dihaloalkanes.
General formula : C, H,,X, Prefix : Dihalo
The dihalogen derivatives have special common names depending upon whether the halogen atoms are
present on the same or different carbon atoms. In IUPAC system, all types of dihalides are called dihaloalkanes,
the positions of the halogen atoms being indicated by lowest possible numbers. These are commonly named
according to IUPAC systems. These can be divided into three types as:
(1) Alkylidene halides : When both halogen atoms are attached to the same C-atom, these are called
alkylidene dihalides or simply alkylidene halides. These are also called gem dihalides or simply
gem halides. CH,

CH.CHCI, CH, —C—Cl CH,CH,CHCI,


Ethylidene dichloride Propylidene dichloride
(gem-dihalide) Cl
Isopropylidene dichloride
IUPAC : 1, 1-Dichloroethane IUPAC : 2, 2-Dichloroethane TUPAC : 1, 1-Dichloropropane
(ii) Alkylene halides ;: When the two halogen atoms are attached to adjacent C-atoms, they are called
alkylene dihalide or simply alkylene halide. These are also called vicinal dihalides or simply
vic-halides. For example,
CH,

CH,—CH, CH,—CH—CH, CH, =Car

Cl CO Cl Cl Br
Ethylene dichloride Propylene dichloride Isobutylene dibromide
(vie-dihalide)
IUPAC : 1, 2-Dichloroethane IUPAC : 1, 2-Dichloropropane IUPAC : 1, 2-Dibromo-2-methylpropane

(tit) Polymethylene dihalides : When the same two halogen atoms are present on the terminal carbon
atoms of alkanes (containing three or more carbon atoms) i.e., @, @ positions of the carbon chain, they
are called polymethylene dihalides. For example,
ClICH,CH,CH,CH,Cl BrCH,CH,CH,Br
Tetramethylene dichloride Trimethylene dibromide
IUPAC : 1, 4-Dichlorobutane IUPAC : 1, 3-Dibromopropane
ORGANIC CHEMISTRY: BASIC PRINCIPLES AND TECHNIQUES

The common and IUPAC names of some dihaloalkanes are given below :

Structure Common name IUPAC name

CH,Cl, Methylene chloride Dichloromethane


‘CH,Br Ethylene dibromide 1, 2-Dibromoethane

2CH,Br
Ethylidine dibromide 1, 1-Dibromoethane

1CHBr,
HS UES Trimethylene dichloride (a, y) 1, 3-Dichloropropane

Cl Cl
CH,CH,CHCI, Propylidene dichloride 1, 1-Dichloropropane

or Propylene dichloride 1, 2 - Dichloropropane

9) Ome6
Cer Isopropylidene dichloride 2, 2 - Dichloropropane

Cl Cl

(c) Polyhaloalkanes
The general formula of trihaloalkanes is C_H,, 2n-l ,X,3 while that of tetrahaloalkanes is C,H,
1 ed
.X,.
There are no systematic common names for tri-and tetrahaloalkanes. However, trihalomethanes are known
by their trivial names i.e. haloforms and tetrahalomethanes as carbon tetrahalides.
These are commonly named according to IUPAC system.
CHCl, CHI, CCl,
Chloroform lodoform Carbon tetrachloride
IUPAC : Trichloromethane IUPAC : Triiodomethane IUPAC : Tetrachloromethane
Fully halogenated hydrocarbons are also called perhalohydrocarbons (per means that all the hydrogens
of the hydrocarbon are replaced by halogen atoms). It may be noted that polyhaloalkanes and other complex
halogen compounds are named according to IUPAC system.
(CF,.—CF,—CF.) : Perfluoropropane IUPAC ; Octafluoropropane
The common and IUPAC names of some tri- and tetrahaloalkanes are given below:

Structure Common name IUPAC name

CHCl, Chloroform Trichloromethane


CHI, lodoform Triiodomethane
CHBr, Bromotform Tribromomethane
CEL. — 1, 1, 1 — Trichloroethane

C1ICH,—CHC1, — 1, 1, 2 — Trichloroethane
CCl, Carbon tetrachloride Tetrachloromethane

Br,CH—CHBr, Acetylene tetrabromide 1, 1, 2, 2 — Tetrabromoethane

3. Alcohols or alkanols
General formula :C, H,,,, OH or ROH
Functional group : —OH Secondary suffix: ol
These are derived by replacing one hydrogen atom from an alkane by one hydroxy] group. In the IUPAC
system, these are called alkanols and their names have been derived by changing -e of the corresponding
alkane by -ol (alkane —-e + ol = alkanol).
Molecules having more than one —OH groups are called diols (2—OH groups), triols (3—OH groups) etc.

WWW.JEEBOOKS.IN
cs Formula Common name
MODERN'S abc + OF CHEMISTRY-XI

IUPAC name

CH,OH Methyl alcohol Methanol


CH,CH,OH Ethyl alcohol Ethanol
CH,CH,CH,OH n-Propyl alcohol Propan-1-ol
ae Isopropyl! alcohol Propan-2-ol

OH
CH,(CH,),CH,OH n-Butyl alcohol Butan—1-ol
CH,CHCH,CH, sec-Butyl alcohol Butan—2?-ol

OH
CH,—CH, Ethylene glycol Ethane—1, 2—diol
|
OH OH

3 ae i le Propylene glycol Propane—1, 2—diol

OH OH
HOCH,—CH,—CH,OH Trimethylene glycol Propane -1, 3-diol

is a aaaCH, Glycerol or Glycerine Propane - 1, 2, 3 - triol


OH OH OH
4, Ethers or Alkoxyalkanes
General formula : C,H, _,OC,H,,,,, or ROR’ m =n orm#n
Functional group : —O— Prefix : alkoxy
These are derived from alkanes by replacing one hydrogen atom by alkoxy group (—OR). For example,
methoxy (OCH,), ethoxy (OC,H.) or propoxy (OC,H.). In an ether molecule, two alkyl radicals are linked on
either sides to a divalent oxygen atom (—O—). They are called simple ethers if R = R’ (e.g., CH,OCH,) and
mixed ethers if R and R’ are different (e.g., CH,O0C,H..). In the IUPAC system, ethers are called alkoxyalkanes.
The alkoxy group is taken with smaller alkyl group.
Ether Common name IUPAC name
CH,OCH, Dimethyl ether Methoxymethane
CHOC, H. Diethyl ether Ethoxyethane
CH,OC,H, Ethyl methyl ether Methoxyethane

It may be noted that the last compound ts not ethoxy methane because alkoxy group is taken with lesser
number of C atoms.
5. Aldehydes or Alkanals
General formula : C,H,, _,CHO (where n can be 0 also e.g., HCHO)

Functional group : Sc = {) Secondary suffix : al


ae
These compounds are obtained by replacing a hydrogen atom attached to the terminal carbon atom in
alkane molecule by an aldehydic (CHO) group. The first member is, however, obtained by replacing a hydrogen
atom in the hydrogen molecule.
These are called alkanals in the IUPAC system of nomenclature and the names of individual members
are derived by changing —e of corresponding alkane by —al (alkane -e + al = alkanal).
Note : In aldehydes, the —CHO group is always present at the end of the chain and there is no need to designate
its position as 1.
While counting the carbon atoms in the parent chain, the carbon of the —CHO group 1s also counted. For example,
the parent chain in CH,CH,CH,CHO corresponds to 4C atoms and not 3. Further, it should be simply named as butanal
and not butan-1-al because —CHO 1s always terminal.

Aldehyde Common name IUPAC name


HCHO Formaldehyde Methanal
CH,CHO Acetaldehyde Ethanal
CH,CH,CHO Propionaldehyde Propanal
CH,CH,CH,CHO n-Butyraldehyde Butanal
ORGANIC CHEMISTRY: BASIC PRINCIPLES AND TECHNIQUES 12/27 >>

6. Ketones or Alkanones
General formula : C,H, ,COC,H,, or RCOR’

Functional group : C=O Secondary suffix : one


=
These are obtained by replacing two hydrogen atoms attached to the non-ferminal carbon atom in alkane
molecule by a divalent oxygen atom. Thus the functional group called ketonic group (_>C = O) contains a
carbon atom joined to the oxygen atom by double bond. To the either sides of the functional group carbon atom,
alkyl radicals are attached.
The ketones are called simple ketones when R = R’ (e.g., CH,COCH,) and mixed ketones if R and R’ are
different (e.g., CH,COC,H.). In the IUPAC system, ketones are called alkanones. The names of individual
members are derived by replacing —e of the corresponding alkane by -one (alkane -— e + one = alkanone).

Ketone Common name

Dimethyl ketone Propanone


(Acetone)

Cho —C—CiLCH, Ethyl methyl ketone Butan-2-one


O
|
Ef a TET Diethyl ketone Pentan-3-one

O
|
CH,—C—CH,CH,CH, Methyl-n-propyl ketone Pentan-2-one

The prefix 2 or 8 indicates the position of the functional group carbon atom which must always get the
lowest possible number.
7. Monocarboxylic acids or Alkanoic acids
General formula : C,H, naj COOH (where n can be zero also)

Functional group : - =) Scondary suffix : oic acid

OH
In these compounds, one hydrogen atom in alkane molecule is replaced by a carboxyl (COOH) group. The
first member is, however, obtained by replacing a hydrogen atom in the hydrogen molecule.
In the IUPAC system, they are called alkanoic acids and are named by replacing the terminal -e of the
corresponding alkane by -oic acid (alkane -e + oic acid = alkanoic acid)
Monocarboxylic acid Common name IUPAC name
HCOOH Formic acid Methanoic acid
CH,COOH Acetic acid Ethanoic acid
CH,CH,COOH Propionic acid Propanoic acid
CH,CH,CH,COOH n-Butyric acid Butanoic acid
CH,CH,CH,CH,COOH n-Valeric acid Pentanoic acid

8. Acid halides or acyl halides or alkanoyl halides


General formula: C,H, ,COX or RCOX (where n can be zero also)

Functional group
i—
: —C X (halocarbonyl) Secondary suffix : oyl halide

These are derivatives of carboxylic acids in which —OH group of a carboxylic acid has been replaced by a
halogen atom. Among these acyl chlorides (RCOC]1) are more common because these are more stable and easy to
handle.
a 12/28 MODERN'S abe + OF CHEMISTRY-XI

The common names are derived from the common names of the acid by replacing te acid by yl halide as
acetic acid-ic acid + yl halide = acetyl halide
The IUPAC names are also derived by replacing -ic acid of the name of acid by yl and adding the name
of the halide. Alternatively, the name is derived by replacing terminal -e of the corresponding alkane by the
suffix oyl halide as :
Alkane - e + oyl halide = Alkanoyl halide
Acid halide IUPAC name
HCOC] (unstable) Formyl chloride Methanoyl chloride
CH,COCl Acetyl chloride Ethanoyl chloride
CH,CH,COCI1 Propionyl chloride Propanoyl chloride
CH,CH,CH,COCI n—Butyryl chloride Butanoyl chloride

9. Acid anhydrides

General formula : R—C—O—C—R' or (RCO),O

where R and R’ may be same or different alkyl groups.

O
Functional group : = — O— — Secondary suffix : anhydride
These are also acid derivatives which are derived from carboxylic acids by the removal of water from two
molecules of the acid.
O O O O
| | | | ||
R—C—lOH+ HIO—c—R —22°.> R-c_0-C_R
The common or IUPAC names are obtained by replacing the word acid of the common name or IUPAC
name of the carboxylic acid by the word anhydride.
Symmetrical anhydrides of substituted carboxylic acids are named by adding the word bis to the name to
indicate that two identical groups are present. However, unsymmetrical anhydrides are named by writing the
names of the two acids alphabetically before the word anhydride.
Acid anhydride Common name IUPAC name
CH,COOCOCH, Acetic anhydride Ethanoic anhydride
CH,CH,COOCOCH,CH, Propionic anhydride Propanoic anhydride
CH,COOCOC,H, Acetic propionic anhydride Ethanoic propanoic anhydride
(CICH,CO),O Bis(chloro acetic anhydride) Bis(chloroethanoic anhydride)

10. Esters
O

General formula : R—C—O—R’ Where R= Hor any alkyl group and R’ is always
an alkyl group
O

Functional group : Sc — OR’ Secondary prefix : alkyl


Secondary suffix : oate

These are the derivatives of carboxylic acids in which —OH group of the carboxylic acid has been replaced
by —OR group (where R is alkyl).

The common or IUPAC names are obtained by prefixing the name of alkyl group (OR’ part) before the
name of the parent acid and changing the suffix ic acid to oate.

WWW.JEEBOOKS.IN
ORGANIC CHEMISTRY: BASIC PRINCIPLES AND TECHNIQUES 12/29 =>

Common name IUPAC name

HCOOCH, Methyl formate Methyl methanoate


HCOOC,H, Ethyl formate Ethyl ethanoate
CH,COOCH, Methyl acetate Methyl ethanoate
CH,COOC,H, Ethyl acetate Ethyl ethanoate

11. Acid amides or alkanamides


General formula : RCONH, (R may be H or any alkyl group)
O

Functional group : —C — NH,(amide) Secondary suffix : amide


These are derivatives of carboxylic acids in which —OH group of a carboxylic acid has been replaced by
—NH, group.
The common names are derived by replacing ic acid from the common name of the corresponding acid by
amide.
The IUPAC names are derived by replacing te acid of the name of the acid by amide or by replacing the
—e of the name of the corresponding alkane by the suffix amide (alkane — e + amide = alkanamide).
Common name IUPAC name
HCONH, Formamide Methanamide
CH,CONH, Acetamide Ethanamide
CH,CH,CONH, Propionamide Propanamide

12, Amines
General formula : C,H, ., — NH,
Functional group : —NH,; ra
Secondary suffix : amine
These are the alkyl derivatives of ammonia. These are formed by replacing one, two or all the three
hydrogen atoms in ammonia with alkyl radicals. In the IUPAC system, these are also called alkanamines. The
name is derived by replacing -e of alkane by amine.
Common name IUPAC name
CHUNEL Methyl amine Methanamine
CH,CH,NH, Ethyl amine Ethanamine
CH,CH,CH,NH, n-Propyl amine Propanamine

13. Alkyl cyanides or alkanenitriles


General formula : R—C=N
Functional group : —C=N (cyano or nitrile) Secondary suffix : nitrile
In the common system, the cyanides are named as alkyl cyanides and the suffix cyanide is added to the
name of the alkyl group. These are also named on the basis of acid produced on their hydrolysis. In this case,
the ending -iec acid of the acid is replaced by onitrile as
acetic acid —ic acid + onitrile = acetonitrile
According to IUPAC system, these are called alkane nitriles and the names are written by attaching the
word nitrile to the name of the alkane. It may be noted that while counting the number of carbon atoms for
a chain, carbon of —-CN group is also counted in the chain.
Compound IUPAC name
CH,CN Methyl cyanide or acetonitrile Ethanenitrile
CH,CH,CN Ethyl cyanide or propionitrile Propanenitrile
CH,CH,CH,CN n-Propyl cyanide or butyronitrile Butanenitrile
CH,CHCN Isopropyl cyanide or isobutyronitrile 2-Methylpropanenitrile

CH a
a 12/30 MODERN'S abe + OF CHEMISTRY-XI

14, Isocyanides or isonitriles or carbylamines


General formula : R—N=C
Functional group : —N==C (isocyanide or isonitrile) Secondary suffix : isocyanide or isonitrile
In the common system, their name are derived by adding the suffix carbylamine or isocyanide to the
name of the alkyl group.
There are no separate TUPAC names for isocyanides or isonitriles.

Compound Common name

Methyl isocyanide or methyl carbylamine or methyl isonitrile


Ethyl isocyanide or ethyl carbylamine or ethyl isonitrile

15. Nitroalkanes
General formula : RNO,

AW
Functional group : —NO, or— “No sroup Secondary prefix : nitro

There are no common names for nitroalkanes. According to IUPAC system, these are named by prefixing
nitro to the name of the parent alkane. The position of nitro group of the carbon chain is indicated by a number.

Compound IUPAC name Compound IUPAC name

CH,NO, :
his ates 3CH,—CH—CH,
2 i Ni:
2-Nitropropane
CH,CH,NO, Nitroethane
$3 2 41 NO,
CH, CH, CH,NO, 1-Nitropropane
4 3 2 1 : 4 3 2 1 :
CH, CH, CH, CH,NO, 1-Nitrobutane ee 2-Nitrobutane

NO,
The different classes of functional groups including multiple bonded compounds and the suffix or prefix
required to name these compounds are given in Table 2. For illustration, one example of each family containing
four carbon atoms is also given.
Table 2. Some of functional groups, classes of organic compounds and their nomenclature.

Class of Functional General Suffix or Prefix Examples


compounds group formula

| | Suffix-ene Replace ‘ane’ of CCH, CH= CH,

Alkenes —C=C— CoH ce But-l-ene


TES alkane by ‘ene
CH,CH = CHCH,
But-2-ene

Suffix-yne HC=CCH,CH,
Alkynes —C =C— Sal Pee Replace ‘ane’ of But-1l-yne
alkane by ‘yne’. |
But-2-yne

Arenes

Benzene

IVW.JEEBOOKS.IN
ORGANIC CHEMISTRY: BASIC PRINCIPLES AND TECHNIQUES

Prefix-halo CH,CH,CH,CH,Cl
named as 1-Chlorobutane
Haloalkanes —X RX haloalkanes Co's
(x= FOL Be i)
cl
2-Chlorobutane

Alcohols —OH R—OH Suffix-ol CH,CH,CH,CH,OH


Replace ‘e’ of Butan-1-ol
alkane by ‘ol’ BCH CHCH,

OH
Butan-2-ol

Thiols —SH R—SH Suffix — thiol CH,CH,CH,CH,SH


Replace ‘e’ of Butan - 1- thiol
alkane by ‘thiol’

Ethers —O— R—O—R' Prefix-alkoxy CH,CH,OCH,CH,


(R’ may be R) named as ‘alkoxy’ Ethoxy ethane
alkane CH,OCH,CH,CH,
Methoxy propane

Aldehyd
pi aia ae
= 0 ee adH Suffix-al CH,CH,CH,CHO
Replace ‘e’ of Butanal
H 7
alkane by ‘al’

Ketones >C=0 RCOR’ Suffix-one


Replace ‘e’ of alkane
|
by ‘one’ CH,CH,CCH,
Butan—2-one

Carboxylic P.: =O RCOOH Suffix-oic acid CH,CH,CH,COOH


acid HO Replace ‘e’ of Butanoic acid
alkane by ‘oie’ acid

Carboxylate —COO- RCOO- Su/fix-oate CH,CH,CH,COO Na*


ion Replace ‘ic’ acid of the Sodium butanoate
corresponding acid by ‘oate’

Carboxylic acid
derivatives \
(i) Acyl halides UL =O RCOX Suffix-oyl halide CH,CH,CH,COCI
x Replace —ic of the Butanoyl chloride
(X = Cl), Br, 2) corresponding acid
by ‘vi’ halide
(ii) Esters >C=0 RCOOR’ Prefixing name of CH,CH,COOCH,
RO
the alkyl group and Methyl propanoate
replacing —ic’ acid
of the corresponding
acid by ‘ate’.
(iit) Amides Ze =O RCONH, Suffix—amide CH,CH,CH,CONH,
9 Replace ‘vic’ acid of Butanamide
corresponding acid
by ‘amide’
CH,C—O—C—CH,
eae
(iv) Acid
0| 0| 0| 0\| Add anhydride to
the name of
| |
verso Le
anhydrides —C—O—C— R—C—O—C—R corresponding acid Ethanoic anhydride
a 12/32 MODERN'S abe + OF CHEMISTRY-XI

Amines —NH, RNH, Suffix—amine CH,CH,CH,CH,NH,


prefix amino
named as ‘amino Butan—1l—amine
alkane’ or ‘alkanamine’.

Cyanides —CN RCN Suffix—nitrile CH,CH,CH,CN


Add nitrile after the Butanenitrile
name of alkane.

Nitro —NO, RNO, prefix nitro CH,CH,CH,CH,NO,


Add nitro before the 1-Nitrobutane
name of alkane

GENERAL RULES FOR NAMING ORGANIC COMPOUNDS


We have discussed the simple rules for naming organic compounds for different families. But many organic
compounds have typical branching or substituents. Due to branching at different positions in a molecule, it is
not possible to name them directly. The simple branched chain compounds can be named according to common
names. In the common system, all the isomeric alkanes (having same molecular formula) have the
same parent name. The names of various isomers are distinguished by prefixes. The prefix indicates the type
of branching in the molecule. For example,
(¢) Prefix n-(normal) is used for those alkanes in which all the carbon atoms form a continuous chain with
no branching.
CH,CH,CH,CH, CH,CH,CH,CH.CH,
n-Butane n-Pentane
(ti) Prefix iso- is used for those alkanes in which one methyl group is attached to the next-to-end carbon
atom (second last) of the continuous chain.
CH,—CHCH, CH,—CH—CH,CH, CH,—CHCH,CH,CH;
CH, CH, CH,
Isobutane Isopentane Isohexane
(tii) Prefix neo is used for those alkanes which have two methyl groups attached to the second last carbon
atom of the continuous chain.

CH,—C—CH,
i‘ y
CH,—C—CH,—CH,
| |
CH, CH,
Neopentane Neohexane

In naming highly branched alkanes, the system becomes complicated. However, they can be easily named
according to IUPAC system as discussed below :
A. Rules for IUPAC Nomenclature of Branched Chain Saturated Hydrocarbons (Alkanes)
The following rules are used for naming the branched chain alkanes :
1. Longest chain rule. Select the longest continuous chain of carbon atoms in the molecule. The selected
chain, containing the maximum number of carbon atoms, 1s regarded as the parent or root chain and it gives
the name of the parent hydrocarbon. Carbon atoms which are not included in the parent chain are identified as
substituents or branched chains.
CH nd Substituent

3—CH—CH 9>—CH 9>—C—CH 5>—CH, 4 Parent chain


tp tt pean ap

CH CH

Substituents /
ORGANIC CHEMISTRY: BASIC PRINCIPLES AND TECHNIQUES 12/33 ~~»

> It may be noted that the longest chain may or may not be straight but it must be continuous. For
example, in the following compound, the parent chain consists of nine carbon atoms (structure I) and not eight
(structure IT). Thus, the compound is named as derivative of nonane.

H, CH,
1
_ 37 3. 4 5 6 7 8 9° | P12 ee eee
| CH, CH CH, CH, CH, CH CH,CH, CH, | CH, CH CH, CH, CH, pac CH, CH,
a aces “7 ae
CHACs | CH, CH, |
I II
Correct chain Wrong chain
(contains 9 C atoms) (contains 8 C atoms)
Similarly, in the following example, the correct parent chain is of 6 C atoms.

a 5 3 4 5
CH _—CH,HCHO
9—CH, eT AT:
|a eeA . ae
iCH,—CH CH, |} CH,—_CH,“
CH,
Cicrect ‘chain t—~™S Wrong chain
(contains 6C atoms) (contains 5C atoms)
Some other examples indicating the proper choice of the parent chain are given below :
ia a) rea - as :
_ CH,—CH, I CH,—CH, I

i aa ale. ; (4= we
saan aa a |
CH _—C—CH ,—C-+CH, CH CH CH+CH, —CH,

CH, CH, | CH,|

| CH,—CH,
(Longest chain consists of six carbon atoms) (Longest chain consists of seven C atoms)
> Iftwo different chains of equal length are possible, the chain with maximum number of side chains or
alkyl groups is selected. For example,
| hile
| |CH,—CH,—CH—CH—CH—CH—CH, |
Dll ay 42 ee a ee ae eee
CH,3|1CH ‘hea CH, CH, CH, CH, CH,
| |
|CH, | CH,
il | |
iCH, | CH,
Not correct Correct
(Longest chain but only 3-substituents) (Longest chain with 4 substituents)
'CH;I
3| bee :
—— _ 1 ———-" y
CH,+CH—CH,| CH,CH—CH, |
||Loe es \| 0 oom
CH—CH—CH,—CH.| | CH—CH—CH, —CH,
Le te of os er _ ——+ |
CH, CH—CH, CH, i\CH—CH,
| | |
CH, |CH,|
Not correct Cat
(Longest chain but only 3 substituents) (Longest chain with 4 substituents)
2. Position of the substituent. Number the carbon atoms of the parent chain as 1, 2, 3... etc. starting
from the end which gives lower number to the carbon atoms carrying the substituents.
For example, if X represents a substituent, then the correct numbering of the carbon chain is as given in
structure A. The numbering of the carbon chain as given in the structure B is wrong because it gives higher
number to the carbon atom carrying the substituent.
a 12/34 MODERN'S abe + OF CHEMISTRY-AI

5 43 lf 1 1 ag4s
= oe pe 5 Se a. C—C CC
A (Correct) B (Wrong)
The number that indicates the position of the substituent or side chain is called locant. The position of the
locant in the above structure is 2. For naming the compound, the substituent group is written first with the
number designating its location on the chain and the parent name is written last. The position number ts written
before the name of the substituent which is separated by a hyphen. For example,
5 4 3 2 1 1 2 3
ie a I aa CH,—CH,— CH—CH,—CH,
4 5 6
nt CH,—CH,—CH;
2-Methylpentane 3-Ethylhexane

| > Jt must be remembered that numbers are separated from the groups bYhyphens and there is no break
between substituent and alkane names i.e., between methyl and pentane or ethyl and hexane.
3. Lowest set of locants. When two or more substituents are present, then end of the parent chain which
gives the lowest set of the locants is preferred for numbering.
This rule is called lowest set of locants. This means that when two or more different sets of locants are
possible, that set of locants which when compared term by term with other sets, each in order of increasing
magnitude, has the lowest term at the first point of difference. This rule is used irrespective of the nature of the
substituent. For example,
6 5 4 3 2 1 fe 2 3 4 & 6
a aie Se ae ae (y/o

CH, CH, CH, CH, CH, CH,


Set of locants : 2, 3, 5 (Correct) Set of locants : 2, 4, 5 (Wrong)
The correct set of locants is 2, 3, 5 and not 2, 4, 5. The first set is lower than the second set because at the
first difference 3 is less than 4. (Note that first locant is same in both sets 2; 2 and the first difference is with
the second locant 3, 4. We can compare term by term as 2-2, 3-4 (first difference), 5-5. Only first point of
difference is considered for preference).
Similarly for the compound :
10 9 8 7 6 5 4 3 2 1
CH,—CH,—CH—CH—CH,—CH.,—_CH.,_CH,—_-CH—CH,,
| | | Set of locants : 2,7,8 (Correct)
CH, GE. CH,
1 2 3. 4 5 6 7 3 9 10
CH,—CH,—CH—CH—CH,—CH,_CH,
CH, -CH—CH,,
| | | Set of locants : 3,4,9 (Wrong)
CH, CH, CH,
First set of locants 2, 7, 8 is lower than second set 3, 4, 9 because at the first point of difference 2 is lower than 3.
Similarly, the compounds given below should be numbered as indicated in structure A and not according
to structure B.
CH, CH, CH, CH,
4l a 9| 1 3 4 ls 6
CH,—C—CH,—_CH—CH, CH,.—C—CH,—_CH—CH,
5 9
CH, CH,
6 1)
CH, CH,
(A) Correct 2,4,4 (B) Wrong
set of locants = 2,4,4 set of locants = 3,3,5
CH, CH, CH, CH,
1 9 a 4| o o l4 o l2 1
CH,—C—-CH—CH—CH, CH,—C— H—CH—CH,
|| ||
CH, CH, CH, CH,
(A) Correct (B) Wrong
Set of locants = 2,2,3,4 set of locants = 2,3,4,4

WWW.JEEBOOKS.IN
ORGANIC CHEMISTRY: BASIC PRINCIPLES AND TECHNIQUES

Lowest sum rule. It may be noted that earlier, the numbering of the parent chain containing two or
more substituents was done in such a way that sum ofthe locants is the lowest. This rule is called lowest
sum rule. For example, the carbon chain of alkanes given below should be numbered as indicated in structures
A and not according to structure B.
CH,—CH, CH =
3 4 5 6 7 Py
ec a i CH, _CH—CH—CH _—CH _—CH,
1
CH,—CH, CH,—CH,
(A) Correct (B) Wrong
Sum of locants =3+4=7 Sum of locants =4+5=9
ele CH, CH, CH,
2 |3 4| 5 = 4 | 3 2 | i
LE = lal —CH—CH, a CH ce Od Coen 0 a

Ole EeCH ae ne CH, — CH Clas


(A) car Sum Sone = 2+ 83+44+4=13
Sum of locants = 2+24353+4+4=11
However, according to latest IUPAC system of nomenclature, the lowest set of locants is
preferred even if it violates the lowest sum rule.
For example, if we consider the compound :
9 3 fi 6 3 4 3 2 af
CH,—CH,—CH—CH—CH,—CH, CH, CH, -CH—CH,, Set of locants = 2, 7, 8
Sum of locants =2+7+8=17
CH CH, CH,
Structure (A)
1 2 3 A 5 6 7 8 9 10
CH oa po el oa op 5 Aa fee Al Set of locants : 3, 4,9
Sum of locants =3+4+9=16
CH CH, CH 3

Structure (B)
This compound is numbered as 2,7, 8 and not as 3, 4, 9 in accordance with latest lowest set of locants rule
even though it violates lowest sum rule.
Similarly, the correct set of locants of the following compound is 2, 7, 8 and not 3, 4, 9 even if it violates
lowest sum rule.
2 3 A 5 6 7 8 9 10
Dey ene a! OS rae ee Set of locants = 2, 7,8
Sum of locants =2+7+8=17
CH, CH, CH,—CH,

8 7 6 5 4 3 2 1
se ay OO de a Set of locants = 3, 4,9
Sum of locants =3+4+9=16
CH, eT CH,—CH,

It must be emphasised that for simple alkanes, both the rules, lowest sum rule and lowest set of locants
rule give the same name. However, differences are observed in case of names of complex or highly branched
alkanes. So, lowest set of locants rule should always be applied even if it violates lowest sum rule.
4, Presence of more than one same substituent. /fthe same substituent or side chain occurs more than
once, the prefixes di (for 2), tri (for 3), tetra (for 4), penta (for 5), hexa (for 6) ..... etc., are attached to the names of
the substituents.
It may be noted that the positions of the substituents are indicated separately and the numerals
representing their positions are separated by commas. For example,
CH, CH,
| 1 2 3 4 5 5 4 3 lo 4
CH,CCH, CH,—CH—CH,—_CH—CH, CH,—CH—CH,—C—CH
|
CH, CH, CH, CH, CH,
2, 2-Dimethylpropane 2, 4-Dimethylpentane 2, 2, 4-Trimethylpentane

WWW.JEEBOOKS.IN
S Tizn6 C,H, CH, CH, CH,
MODERN'S abe + OF CHEMISTRY-XI

1 2 a) 4 5 1 ol 3 4| 5 6 7
Hg C—O Os CH,—_CH—CH,—_CH—CH,, CH—CH,
2, 4, 6-Trimethylheptane
CH.
3, 3-Diethylpentane
5. Naming different substituents. /f two or more different substituents or side chains are present in the
molecule, they are named in the alphabetical order along with their appropriate positions.

It must be remembered that the prefixes di, tri etc. are ignored while comparing the substituents.

CH,CH, CH,H,C CH, CH, CH,


o 4 a Fi 1 1 2 I3 l4 o 6 7 1 9 3|
alice SieUh nae a i a CH,—CHé al — ‘iano.
4
CH,CH, CH: CH: CH,
3-Ethyl-2,
y 3-dimethylpentane
ylp 3-Ethyl-4,
y 4-dimethylhept
methylheptane aCH,—CH,
G
3-Ethyl-2, 2, 3-trimethylhexane
6. Naming different substituents at equivalent positions, // two different substituents are in equivalent
positions from the two ends of the chain (two sets of locants are equally possible), then the numbering of the chain
is done in such a way that the group which comes first in the alphabetical order (written first in the name) gets
lower number.
For example, in the compound given below, the methyl and ethyl groups are at equivalent positions.
1 = 3 4 o 6 7 tT 6 o A 3 = 1
cs enc in mn CH — Figg Pee, CCH, —CH,
CH, CH,CH, CH, CH,CH,
Methyl at C-3 Ethyl at C-3
In such a case, the carbon bearing ethyl] group gets lower position because it is cited first in the name according
to alphabetical order of substituents. So, correct name of the compound is
3-Ethyl-5-methylheptane
(Not 5-Ethyl-3-methylheptane)
Similarly, i aaa |e ee

CH,CH, CH 3
3—Ethyl-6-methyloctane
(Not 3-Methyl-6-ethyloctane)
Some books list the groups in the increasing order of size or complexity (i.e., methyl before ethyl). However,
alphabetical order of listing is most widely used.
7. Naming the complex substituents (or substituted substituents). If the substituent on the parent
chain ts complex (i.e., if is branched) it is named as substituted alkyl group by numbering the carbon atom of this
group attached to the parent chain as 1. The name of such substituent is given in brackets in order to avoid
confusion with the numbering of the parent chain. For example,

— Se a a

5-(1,2-Dimethylpropyl)nonane

WWW.JEEBOOKS.IN
ORGANIC CHEMISTRY: BASIC PRINCIPLES AND TECHNIQUES 12/37 ~~»

1 2 3 4 5 G 7 3 9
CH,—CH,—CH,—_CH,—_CH—CH,—_CH.,,_CH.,_CH,,
Pee 2 2 2 eee

© = lw =
Complex substituent
Complex substituent

CH,Ns

CH,—CH,
5-(2-Ethylbutyl)-3,3-dimethyldecane
In the complex substituent, the numbering of the carbon atoms is also done. The carbon atom attached to the
parent chain is always numbered as 1. So, in the first compound the complex substituent is (1,2-dimethylpropy]l).
Some important notes :
1. The name of the complex substituent is always written in brackets.
2. While deciding the alphabetical order of the various substituents, the name of the complex substituent is
considered to begin with the first letter of the complete name. It may be remembered that in case of simple
substituents, however, the multiplying prefixes are not considered. The names of simple substituents are first
alphabetized and then multiplying prefixes are inserted. For example,

CH,—CH,
1 = 3 4 3 6 l7 8 9 10 11
CH,—CH,—CH,—CH, -CH—CH, —-CH—CH,—CH, —CH,—CH,
GH—OH,
Complex substituent
-_ K &,2-dimethylpropyl)

cc

5-(1,2-Dimethylpropyl)-7-ethyl undecane
It may be noted that dimethylpropyl (a complex substituent) is alphabetized under d and not under m.
Therefore, it is cited before ethyl (e).

an oo
—s 9 oo _CComplex substituent
I CH, CH,—CH, | (1,1-dimethylpropyl)
9 8 C—O 5 4 a 1
CH,—CH,—CH,—CH,—C—CH,—CH, _CH,—CH,
I
a I

| a Complex substituent
CH, | (2-methylpropyl)

5-(1,1-Dimethylpropyl) -5-(2-methylpropyl) nonane

The substituent dimethyl is cited first because it is alphabetized under d.

WWW.JEEBOOKS.IN
a 12/38 MODERN'S abe + OF CHEMISTRY-XI

Similarly,

C,H, CH, CH,


1 2 3 4 6 |i 9 10
CH,—CH,—CH _CH 5 —CH _—CH—CH 5 —CH,—CH,

Complex substituent
CH, (1,1-dimethylpropyl)

4-(1,1-Dimethylpropyl)-3- ey -4,7-dimethyldecane

3. When the names of two or more complex substituents are composed of identical words, priority for citation
is given tothe substituent which has lowest locant at the first cited point of difference within the complex substituent.
For example,

CH, _CH _—CH _—CH _—CH _—CH—CH, ‘HCH CH, —CcH, _cH,

CH,—CH—CH,—CH,
|
Us
nmninmininins
ned
(2-Methylbutyl)

5-(1-Methylbutyl)-7-(2-methylbutyl) dodecane
The substituent (1-methylbutyl) is written first because it has lower locant than the substituent (2-methylbuty]).
4. When the same complex substituent (substituted in the same way) occurs more than once, it is indicated
by the multiplying prefix bis (for two), tris (for three), tetra kis (for four), etc.

irk a
?

S js j i~
T etgy o| ca
5,5-Bis (1,1-dimethylpropyl)-2-methyldecane

Bis is used for two same complex substituents.


8. Cyclic or alicyclic hydrocarbons. These compounds contain carbon chain skeletons which are closed
to form rings. The saturated hydrocarbons with ring of carbon atoms in the molecule are called cycloalkanes.
These have the general formula C,H,
The rules for naming these compounds are discussed later (page 47).

WWW.JEEBOOKS.IN
ORGANIC CHEMISTRY: BASIC PRINCIPLES AND TECHNIQUES 12/39 »»

NOTE
According to the IUPAC system of Nomenclature, certain trivial or semi- systematic names may be used for unsubstituted
radicals. For example, the following names may be used :
(CH,),CH— Isopropyl WES Ble) oe (sil Isobutyl
ee a sec-Butyl (Ch) tert—Butyl

lal
{CH eke Isopentyl (CEC Neopentyl
CH 3

CH,—CH,—C— tert-Pentyl (CH,),CH—CH,—CH,—CH- Isohexyl


|
CH,
However, when these are substituted, these names cannot be used as such. For example:
CH (CH,),
1 9 3 l4 5 6 7 3 9 10
cs es ae caair aye eae ae ee

gh ln og ee

5-sec-Butyl-4-isopropyldecane
| a
CH a

Ells CH-CH,
8 7 6 ee See ee. i 1 2 Ss ee 5 6 7 8
a a aie | a CH.-CH,—CH-CH,—-CH-CH,,-CH,—CH,,
1
i CH, CH,—CH, Ch-C_-CH.
CH, °CH,
3,3-Diethyl-5-isopropyl-4-methyloctane 5-(1-chloro-1-methylethy])-3-isopropyloctane

> It maybe noted that while writing the substituent’s name in alphabetical order, non-hyphenated prefixes (iso,neo)
are considered part of the alkyl group for alphabetizing purposes but the hyphenated prefixes (sec-, tert-) are not. For
example, isopropyl and isobutyl are listed alphabetically under ‘i’, but sec- butyl or tert-butyl are listed under ‘b’.
Therefore, isopropyl (listed under ‘i’) precedes methyl (listed under ‘m’) but tert-butyl (listed under ‘b’) precedes
isopropyl (listed under ‘i’). Thus;
@ di., tri, sec-, tert- are ignored in alphabetizing. However, prefixes sec, tert are not ignored when they are
compared with each other. For example, sec-butyl precedes tert-butyl.
@ Iso, neo, cyclo are not ignored in alphabetizing.

SOLVED EXAMPLES _
LI Example 8.
Give IUPAC names for the following hydrocarbons :
(a) CH / tee AH, (6) CH,—CH,—CH,—CH
C,H, C,H. CH,—C—CH,—CH,
CH—CH,—CH,
CH,—CH,—CH,

H, H,
H,C—————- CH, H,CH,C H,CH,
a 12/40 MODERN'S abe + OF CHEMISTRY-XI

(e) a
CH; CHs CH,

Solution : The structure may be written as


3 4 , | | |
(a) CH,—CH—CH—CH, (6) CH,—CH,—CH,—CH,
CH,—C —— CH,—CH,
9 5
H, CH,
"EH, CH. “CH—CH,—CH,
The longest chain has 6 carbon atoms and is numbered 3 : 1
from lowest left as shown. The two side chains are Peg —CH,
methyl groups at positions 3 and 4 (from both ends). The longest chain contains 9 carbon atoms. The chain is
The name of the compound is 3,4-Dimethylhexane. numbered from lower right to upper left as shown. The side
H, chains are two ethyl groups at positions 4 and 5 and one
| methyl group at position 5. The name of the compound 1s :
—e 4,5-Diethyl-5-methylnonane
() “ i" (d) CH,-CH,
H,C CH,
Methylcyclopentane
Methyl group is attached to cyclopentane ring.
H,CH,C CH,CH,
Thus, the name of the molecule is:
Methylcyclopentane. There are three ethyl groups on cyclohexane ring at positions
(e) 1 2 38 4 5 6 8 1, 3 and 5. Thus, the name of molecule is
CH, CH—CH,—CH,—CH—CH—CH,—CH, 1, 3, 5-Triethylcyclohexane.
CH, CH, CH,
The longest chain contains 8 carbon atoms. The
numbering from left to right gives lowest numbers to
the substitutents. The name of the compound is
2,5,6-Trimethyloctane.

LJ Example 9.
Structures and names of some hydrocarbons are given below. Explain why the names given in parenthesis
are not correct :
(a) CH,CH, CH CH, CH CH, CH, 3- Ethyl-5-methylheptane
| (Not 5-Ethyl-3-methylheptane)
C,H, CH 3
(6) CH, CH — CH-CH,—-CH, 2,3—Dimethylpentane
| | (Not 3, 4- Dimethylpentane)
CH, CH,
(¢) CH, CH CH, CH AGRRRES 2.3,5-Trimethylhexane
| | (Not 2,4,5-trimethylhexane)
CH, CH, CH,
CH,CH,
|
(d) CH, CH,CH,CH,CH,CH-CH,-CHCH,CH, _5-(2-Ethyl butyl)-3, 3-dimethyldecane
|
CH,
CH,—C—CH, Not 5- (2, 2-dimethyl butyl)-3-ethyldecane
CH,CH,
(¢) CH,CH,—CH—CH—CH—CH—CH, 2,3,5— Trimethyl-4-propylheptane
CH, CH, CH, CH, (Not 4-sec-butyl - 2,3-dimethylheptane)

CH,

VWWW.JEEBOOKS.IN
ORGANIC CHEMISTRY: BASIC PRINCIPLES AND TECHNIQUES 12/41 ~~»

Solution: 1 2 3 4 5 6 7
(a) CH,—CH,—CH—CH,—CH—CH,—CH,
|
CH CH,
3-Ethyl-5-methylheptane
Both the substituents ethyl and methyl are in equivalent positions. Lower number is given to the one that comes first
in the name according to alphabetical order.
iI 2 3 4 ta)
(b) CH,—CH—CH—CH,—CH,
| |
CH, CH,
2,3-Dimethylpentane
Numbering is done from the end which gives lower number to the substituents.
6 5 4 8 2 «21
(ec) CH,—CH—CH,—CH —CH—CH,
| | |
CH, CH, CH,
2,3,5— Trimethylhexane
Lowest locant number 2, 3, 5 is lower than 2, 4, 5.
CH, CH,
1 9 8 7 6 a) |
(dq) CH, CH, CH, CH, oa CH, CH CH, CH,
“CH,

ls
ce ee
2 1
CH,CH,
5-(2- Ethylbutyl) -3, 3-dimethyldecane
Larger number of substituents are attached to the parent chain.
7 6 o 4 3 2 I
(e) CH,—CH,—CH—CH—CH— CH—CH,
| | |
CH, CH, CH, CH,
|
CH,
|
CH,
2, 3, 5-Trimethyl-4-propylheptane
Larger number of substituents are attached to the parent chain.
L) Example 10.
The incorrect IUPAC name of a compound is 3,3-diethyl butane. Write its structural formula and correct the
name of the compound.
Solution : 1. As the compound is butane, the parent chain has four carbon atoms :

—C—C—C—C— Parent chain


1 | | | (butane)

2. Number the parent chain from either end. Attach two ethyl substituents at carbon 3 and satisfy the remaining
valencies with hydrogen atoms.

H
1
CH,H
i a a al

| | | |
as 2
CH,
a 12/42 MODERN'S abe + OF CHEMISTRY-XI

3. In this compound, the longest chain is not of four carbon atoms but of five carbon atoms as shown below :

: 5CH 3

cn,
|

CH 3
T
4. Now the correct name of the compound is 3-ethyl-3-methylpentane.
B. Rules for Naming Compounds Containing Double and Triple Bonds
In naming the compounds containing double and triple bonds, the following rules are followed :
1. Select the longest continuous chain containing the carbon atoms involved in the multiple bonds (double
and triple). This gives the parent name of the alkene or alkyne. For example, in the structures given below, the
longest chain has five carbon atoms.

:a CH,—€ =CH,
CH,—CH, Cho CH
Correct chain Wrong chain

Note. It may be noted that the selected chain may or may not be the longest chain in the structure. But
it must contain double or triple bonded carbon atoms. For example, in the above structure, the longest chain
containing double bonded carbon atoms is of five carbon atoms and not of six.

2. While writing the name of the alkene or alkyne, the suffix ‘ane’ of the corresponding alkane is replaced by
‘ene’ or ‘yne’ respectively.
3. The numbering of atoms in the parent chain is done in such a way that the carbon atom carrying the multiple
bonds gets the lowest number. The position of the multiple bond is then indicated by using the number of the first
C-atom of the multiple bond.
Note. It may be noted that we give lowest number to the carbon atom having double or triple bond and
not to any side chain (as in alkanes).

CH,—CH=CH—CH,—CH, CH,—CH—=CH—CH,—CH,
Pent-2-ene (not Pent-3-ene)
The position of the double bond may be indicated by any of the three ways :
2-Pentene or Pent-2-ene or Pentene-2
Generally, the first two methods have been in use. However, according to the latest TUPAC convention
(1993 recommendations), while naming the organic compounds, the locants (numerals and or letters) are
placed immediately before the part of the name to which they relate. This means that in case of unsaturated
hydrocarbons the locant of the double bond or triple bond is placed immediately before the suffix ene or yne
and not before the word root as was practice being followed earlier. For example,
4 3 z i) 4 3 5 1 5 4 3 2 1
CH.—CH=CH—CH, eh lel See) lg ae
But-2-ene But-1-yne Pent-1l-yne
(Formerly 2-Butene) (Formerly 1-Butyne) (Formerly 1-Pentyne)

4, All the rules for naming side chains or substituents are then followed (as in alkanes).
5.1 fthe multiple bond occurs two or more times in the chain, then it is named as diene or diyne, triene or triyne,
etc. The position of multiple bonds are also prefixed with numbers.
Some examples are given below :
1 2 3 4 1 2 3 4 5
CH,—=CH—CH—CH, HC= C—CH,—C = CH
Buta-1,3-diene Penta-1,4-diyne
ORGANIC CHEMISTRY: BASIC PRINCIPLES AND TECHNIQUES 12/43 SS

A 3 = 4
CH,—CH —C—CH, CH,
| | 6 5 4 3 2 1
CH, CH, a ie a Mia= CH

CH, CH,CH,
2-Ethyl-3-methylbut-1l-ene 4-Ethyl-4-methylhex-1-yne
CH, CH,
4 3| 2 1 6 5| 4 3 2 i
CH.—CH—C = CH ca i alii = CH—CH,

3-Methylbut-1-yne CH.CH,
4, 5, 5-Trimethylhex-2-ene

> IMPORTANT NOTES


Ifthe parent chain contains both the double and triple bonds, the following points should be kept in mind.
e The terminal ‘e’ in the name of alkene is dropped if it is followed by the suffix beginning with a, i, 0, u or y.
For example, terminal e of the ene is dropped in the following cases :
en — yne adien — yne
However, terminal e is not dropped in case of ene —diyne because it comes before d of diyne.
@ Numbers as low as possible are given to double bond or triple bond as a set, even though this may at times
give —yne a lower number than —-ene. However, ifa choice is there, preference for lower locant is given to double
bond.
e These compounds are named as derivatives of alkynes rather than alkenes. For example
5 4 8 2 1 6 5A 3 2 1
HC = C—CH,—_CH—CH, HC = C_-CH—CH—CH—CH,
Pent-l-en-4 yne Hexa-1,5-dien-5-yne
(Preference of locant for ene)
i) 2 4 5 54 8B 2 1
HC = C-CH—CH—CH, HC = C—CH=—= CH—CH,
Pent-3-en-1-yne (Not Pent-2-en-4-yne)

The name ofthis compound cannot be pent-2-en-4-yne because lowest set is 1,3 rather than 2, 4.
eee aes 4 5 1 6B 3 4 6 5 4 38 2 1
HC = C—CH—C = CH CH, =C—CH =>=CH, CH=C—CH=CH—CH=CH,
| | Hexa - 1, 3- dien-5- yne
CH, CH,
3-Methylpenta-1,4-diyne 2-Methylbuta-1,3-diene
@ In some cases, all the double bonds or triple bonds present in the molecule cannot be included in the longest
chain. In such cases, the following prefixes are used for double and triple bonded groups acting as substituents.
CH, = CH,CH = CH, = CH- HC =C-
Methylene Ethylidene Ethenyl or vinyl Ethynyl
For example, |
CH,
1 2 wf 4 5 8 7 6 &B Ps 8 2 1
CH,= Se ~ H=C He CH,—C=C—CH,—_CH—CH—CH=CH,,
OH 2 CH =CH,
3-Methylenepenta -1, 4-diene 4-Ethenyl-3-methyloct—l—en—6—yne
or 3-Methyl-4-vinyloct-1-en-6-yne
1 2 a 4 | 5 7 6 5 4 4
lg (ee os Oy) eg Os | CH, = CH—CH=CH—CH—C= CH
6 7 2 1
CH=CH, CH, =CH,
5-Ethynylhepta -1, 3, 6—triene Not: 3-Ethynylhepta-1, 4, 6-triene
(Double bond are given preference (Set of locants are higher)
over the triple bond)

WWW.JEEBOOKS.IN
a 12/44 MODERN'S abe + OF CHEMISTRY-XI

=Dnae
Daaitem
Weer
:

Q11. Write the structural formulae for :


(i) 4-Methyl pent-2-ene (ii) 2,2-Dimethylpropane (iii) 2-Ethylpent-2-ene
(iv) Cyclohexene (v) 1,38-Dimethylcyclohexane
Q@ 12. Give the IUPAC names of :
(1) CH, = CHCH = CHCH = CH,

(ii) CH,—C = cH tH CH, (iii) (C,H,),C = C(C,H,),


H, Hi,

CH, CH,
(iv) a, = C—CH, (v) cu, Ag = CH
oH, cH, CH,
C. Rules for Naming Organic Compounds Containing one Functional Group
The rules for naming an organic compound are exactly same as discussed already for compounds
containing multiple bond. In this case the preference for lowest number is given to carbon atom
containing functional group. The rules are summarised below :
1. Select the longest continuous chain containing the carbon atom having the functional group without caring
whether it also denotes the longest possible carbon chain or not. This is regarded as the parent chain. This
is illustrated below :

CH, OH ae OH
Bm hater ics hl a a

CH,—CH 1C—CH o>—CH >—CH, | | CH,—CH, —C—CH,—CH ee! |

CH, CH,
(Correct) (Wrong)
2. The numbering of atoms tn the parent chain ts done in such a way that carbon atom bearing the functional
group gets the lowest number.
3. All the rules for naming side chains or substituents are then followed as discussed in case of alkanes.
Let us apply these rules to write the names of some compounds.
1

CH,—°C—OH
LS! 2-Methylpropan-2-ol
|
“CH
CH, O
| |
3CH,—?C—'CH,OH a AR aie Re
|
CH, CH,
2, 2-Dimethylpropan-1-ol 5-Methylhexan-3-one

It may be noted that the functional group should get the lowest number even if it violates the lowest
sum rule.

CH,
Is 4 3 2 1
CH,—C—-CH—CH—CH,—CH., 4-Ethyl-5, 5-dimethylheptan-3-ol
6 || |
Filed ie OH
7 |
CH, CH,

WWW.JEEBOOKS.IN
ORGANIC CHEMISTRY: BASIC PRINCIPLES AND TECHNIQUES 12/45 a

3 4 o 2 1
CH, CH, CHCH, CHO 3-Methylpentanal
CH,
It may be noted that if a chain terminating functional group such as —CHO, —COOH, —CONH,,
— COOR, —CN, etc. is present in the molecule, the numbering of the parent chain in such a case must
start from the carbon atom of the functional group. Therefore, this carbon is always given number 1.
However, this number is normally not indicated in the IUPAC name of the compound.

For example,

CH,
a! 3 4 4 3 2
CH,—CH,—_C—_CH,,—_CH, CH, CH,—_CH—CH,
| al
CHO COOH
2-Ethyl-2-methylbutanal 2-Methylbutanoic acid
4 3 2 1 4 3 2 1
C,H, CH, CH, CH, COOH CH,—CH,—_CH,—_C =WNV
4-Phenylbutanoic acid Butane nitrile
If halogen atom is present in addition to functional group, the halogen is treated as substituent and is
written as halo. For example,

4 3 2 1|| 4 3 2 1 4 3 2
CH,—CH,—_CH—C—O0C,H. I—CH,—CH,—CH,—_COOH CH,—CH—CH—CH,
i" 4-Iodobutanoic acid Cl COOH

Ethyl 2-bromobutanoate 3—Chloro—2—methylbutanoic acid


Cl
1 2 3 4 5 5 4 3 P 1
an ie —CH,—_CH, —Cl CH,— CH—CH,—CH, —-CO—NH,
4-Chloropentanamide
5-Chloropentan-2-one
It two or more same functional groups are present, these are indicated by using di (2), tri (3), tetra (4) as
prefixes to the name of the functional group.
o 4 a 2 1
CH,—CH, CH,—CH,—CH, OHC—CH,—CH,—CH,—CHO
| | | | Pentanedial
OH OH OH OH (or Pentane-1, 5-dial)
Ethane-1, 2-diol Propane-1,3-diol (No need to give numbers because —CHO is always terminal)

yoon HOOC—CH,—CH,—COOH
COOH Butanedioic acid
Ethanedioic acid
5 4 3 7 1 2 3
CH,—C—@E—@.—OH, an an
| |
O OH OH OH
Pentane-2, 4-dione Propane-1, 2, 3-triol

LEARNING PLUS
Special note for compounds containing more than two same functional groups
According to latest convention (1993 recommendations for IUPAC nomenclature), if an unbranched chain
is directly bonded to more than two same functional groups, the organic compound is named as a derivative
of parent alkane which does not include the carbon atoms of the functional groups. These are named by use
of suffix such as tricarboxylic acid (for three—COOH groups), tricarbaldehyde (for three—CHO groups) or
tricarbonitrile (for three—CN groups).
a 12/46 MODERN'S abe + OF CHEMISTRY-XI

For example,
5 4 3 2 1
Sa ne a age a a Pentane-1,3,5-tricarboxylic acid

COOH
3 z 1
pe ea aoe Propane-1,2,3-tricarbaldehyde

CHO
1 2 3
N = C—CH,—_CH—CH,—C =N Propane-1,2,3-tricarbonitrile

C=N

D. Rules for Naming Compounds having functional groups, multiple bonds and side chains/
substituents.
If the organic compound contains a functional group, multiple bond, side chain or the substituent, the
following order of preference must be followed :

Functional group > Double bond > Triple bond > Side chain
For example,
4 3 2 1 5 4 a 2 1
CH,—CH = CH—CH,—OH CH.—HC = HC—CH,—CHO
But-2-en-1-ol Pent-3-en-1-al
O CH,
A 4 3 a\| 4 G & A 3. 2 | 1
CH,=CH—CH,—C—CH, CH,—CH,—CH = CH—CH CHO
Pent-4-en-2-one 2-Methylhex-3-en-1-al
5 4 3 2 1 & 7 2 1
OHC—CH,—CH = CH—CHO HOOC—CH = CH—COOH
Pent-2-ene-1,5-dial But-2-ene-1, 4-dioiec acid
(or But-2-enedioic acid)
E. Rules for Naming Organic Compounds containing two or more Functional groups
When an organic compound contains two or more functional groups, one group is regarded as the principal
functional group. The other group is treated as the secondary functional group and may be treated as
substituent. The following order of priority is used for selecting the principal functional group :
Carboxylic acids > sulphonic acids > acid anhydrides > esters > acid halides > amides
> nitriles > aldehydes > ketones > alcohols > amines > ethers
O O O
| | l
—COOH >—SO,H > —C—O—C—>— COOR >— COC] > —CONH, > _CN>H—C—> Yc=0 > —OH >— NH, > —O—
All remaining groups such as halo (fluoro, chloro, bromo, iodo), nitroso (-NO), nitro (- NO,), alkoxy (—-OR),
alkyl(R) are treated as substituent groups.
For example, if an organic group contains —COOH and —OH groups, then —COOH group is regarded as the
principal group and —OH group is the substituent and is called hydroxy.
| ee
=

;
1OH }—substituent
fee a
5 4 3 2 I
CH,—CH—OH,—CH,}6o0H |—~prineipal functional group
4-Hydroxypentanoic acid
The names of secondary groups which are used as (substituents) prefixes are given below :
Functional group Prefix Functional group Prefix
—COOH Carboxy —CHO Formyl
—COOR Alkoxy carbonyl Co ines eta
or Carbalkoxy
—COCI Chloroformyl —OH Hydroxy
—CONH, Carbamoyl —SH Mercapto or sulphanyl
—CN Cyano —NH, Amino
—OR R-oxy = NH Imino
—X Halo —NO, Nitro

WW.JEEBOOKS.IN
ORGANIC CHEMISTRY: BASIC PRINCIPLES AND TECHNIQUES

Some examples are :


5 4 8 2 1
O
5s all 3s 2 4 |
CH,—C—CH, CH,COOH NH, OH
—COOH is principal group —OH is principal group
4-Oxopentanoic acid 4-Aminopentan-2-ol

1 al 2 4 1 el 3 4 |
CH,—C—CH,—CHO CH,—C—CH,—_CH,—_OH
—CHO is principal group (—CO is principal group)
3-Oxobutanal 4-Hydroxybutan-2-one
3 2 1 3 2 1
NC—CH,—CH,—COOH “ge eae
—COOH is principal group
3-Cyanopropanoic acid NH,
—COOH is principal group
2-Aminopropanoic acid
CH, OH
4s] 2 1
CH,—C—C—CH 3
* 7 |
OH O
—CO is principal group
3-Hydroxy-3-methylbutan-2-one —CO is principal group
3-Hydroxycyclohexanone
4 3 2 1 4 3 2 1
ee ee CH,—CH,—CH—COOH

CH, O OH F
—COOH is principal group —COOH is principal group
3-Methyl-2-oxobutanoic acid 2-Fluoro-4-hydroxybutanoic acid
F. Rules for Naming Alicyclic Compounds
The following rules are generally followed for naming alicyclic compounds:
1. The alicyclic compound is named by prefixing cyclo to the name of the corresponding straight chain
hydrocarbon. For example,
H,C——CH,
H,C——CH,
ae a or XY or
H,C——-CH,
Hy C, H,, Cyclobutane
C. H,, Cyclopropane

Hy Hy
C

H,C a CH, H.C


So m\
CH,
| | or
H,C————-CH, | H.C
\G / CH,

C, H,, Cyclopentane
Hp
C, H,. Cyclohexane
For simplicity, the cycloalkanes are also represented by simplified structural formulae without showing C
and H atoms as given above. Similarly,

JO
Cyclobutene Cyclopentene
O
Cyclohexene
©
Cyclooctane Cyclooctene

2. Substituted alicyclic compounds are named by indicating the name of the substituent before the name
of the cycloalkane as alkyl cycloalkanes, halo cycloalkanes, alkyl cycloalkenes, etc.

WWW.JEEBOOKS.IN
ST izs MODERN'S abe + OF CHEMISTRY-XI

CH, CH, Cl

Methylcyclohexane Ethylceyclohexane Chlorocyclopentane


3. If two or more than two alkyl groups or other substituent groups are present in the ring, their positions
are indicated by arabic numerals, t.e., 1, 2, 3, 4, . . etc. While numbering the carbon atoms of the ring, the
substituent which comes first in the alphabetical order is given the lowest number provided it does not violate the
lowest set of locants rule. For example,

CH, CH; (CH,


ae 1
1, CH,CH, 6 -
5 3
CH, : CH,CH,CH,
1.3-Dimethylcyclobutane 1-Kthyl-2-methylceyclopentane 1-Methy1-3-propylcyclohexane 3-Ethyl-1,1-dimethylceyclohexane
(Not 1-Methyl-2-ethylcyclopentane) (Not 5-Methyl-1-propylcyclohexane) (Not 1-Ethyl-3,3-dimethylcyclohexane)
[Alphabetical order of numbering] [Alphabetical order of numbering] [More branched chain gets lower number]

4. (a) If the number of carbon atoms in the ring is equal to or more than the number of carbon atoms in the
alkyl group attached to it (or substituent) then it is named as an alkyl substituted cycloalkane. But if the
numober of carbon atoms in the largest substituent is greater than the number of carbon atoms in the ring then
it is named as cycloalkyl substituted alkane.

CH,
1 5 3 4
But
CH,—CH,—CH,—CH,
Methylcyclopentane 1-cyclopropylbutane
(Chain of 1 C atom, ring of 5 C atoms) (Chain of 4 C atoms, ring of 3 C atoms)

For example,
1 Fi J 4 1 2 a 4 5
CH,—CH,-CH,-CH,-CHé UA_CH-CH,-CH, CH,—CH,—CH—CH,—CH,,

Pentylcyclopentane (2-Butyl) cyclohexane a-Cyclobutylpentane


or (1-Methylpropyl)cyclohexane

<>—cH,—CH, CH=CH 0H,

1-Cyclobutylpentane
(+)
Cyclohexyl cyclohexane
;
Cyclopentyl cyclohexane

CH, CH,
CH—CH—CH,
1 2 a

\CH,CH,
4-Cyclopropyl-3-methyloctane (1,2-dimethylpropyl) cyclohexane 1-Kthyl-2?-methyleyelobutane

WWW.JEEBOOKS.IN
ORGANIC CHEMISTRY: BASIC PRINCIPLES AND TECHNIQUES 12/49 ==
(6) If the side chain contains a multiple bond or a functional group the alicyclic ring is treated as the
substituent irrespective of the size of the ring. For example,

4
CH, O
3 2 1 3 Fs 1 4 a 2| 1
CH,—CH=CH, CH—CH—CH, CH=CH— C—CH,
: : |
OH

3-Cyclopropylprop-l-ene 3-Cyclohexylbutan-2-ol 4-Cyclohexylbut-3-en-2-one

(c) If more than one alicyclic ring is attached to a single chain, the compound is named as a derivative of
alkane irrespective of the number of carbon atoms in the ring or the chain. For example.

1 2 3S

Dicyclopropylmethane 1, 3- Dicyclohexy] propane

5. If a multiple (double or triple) bond and some other substituents are present in the ring, the numbering
is done in such a way that the carbon atoms of the multiple bond get lowest locants 1 and 2 and the substituent
groups get the lower locants at the first point of difference. For example,

CH, CO
J 8
‘ 2
. 1

1-Methylcyclopent-1l-ene 3-Methylcyclohex-1-ene 1,5 Dimethyleyclopent-1l-ene


(Not 2, 8-Dimethylcyclopent-1-ene)

Cl
1 , 1
6
5 3 5 5 2
HC = GH, rf :
3, 5-Dimethylcyclohex-1-ene 1,6-Dichlorocyclohex-1-ene 1,5-Diethyleyclohex-l-ene
(Not 4, 6-Dimethylecyclohex-1-ene) (Not 2, 4-Diethyleyclohex-1-ene)

6. If the side chain contains a functional group and the ring contains a multiple bond, then the ring is
treated as a substituent and the compound is named as derivative of the alkyl chain.

2 1
CH,—CH—CH,OH 4
5 4 1 CH,
3 (8 1 : : P sl 1
4 — CH,—CH,—COOH | . CH, _- CH—CH,—CH,OH
4 a 5 2
a 2
3-(Cyclohex-2-enyl)propanoic acid 2-(Cyclopent-3-enyl)propan-1l-od 3- (4-Methylcyclohex-2-eny])
butan -1-ol

WWW.JEEBOOKS.IN
lon 12/50 MODERN'S abe + OF CHEMISTRY-XI

7. If the ring as well as the side chain contain functional groups, the compound is named as a derivative of
the alicyclic ring or the side chain depending upon whether the ring or the side chain contains the principal
functional group.
For example,
O
4 l 3 2 1
: 2 ill
O— + ,- CH, —CH,— C—OH NO — CH=CH—COOH
5 9 3 2

3-(4-Hydroxycyclohex-2-enyl) propanoic acid 3-(4-Nitrocyclohex-1-enyl) prop-2-en-1-oic acid


—COOH is principal | —COOH is principal group

O
1
1 2 3
; a ie ti

OH
2-(3-Aminomethyl-2-hydroxypropyl) cyclohexan-1-one 3-(6-Oxocyclohex-1-enyl) propanamide
(<0 group is principal group | —CONH, is principal group

However, if the alicyclic ring and the side chain contains the same functional group, the compound is
named as a derivative of side chain or ring according as the side chain or the ring contain larger number of
carbon atoms. For example,

1 2 d 4 a
CH,—CH,—CH_,—CH,,
CHO
4+
=
Hy—CHCH »—CH, 4
2
1

2-(2-Hydroxybutyl) cyelohexan-1-ol | |
CHO
4-(5-Formyl pentyl) cyclohexane-1-carbaldehyde

8 ? a a 4 a pa 1
ao ak —CH,—CH,— CH, —CH, —CH, CH, CHO

7-(3-Formyleyclopenty]) octanal

8. If some functional group along with other substituent group are present in the ring, it is indicated by
some appropriate prefix or suffix and its position is indicated by numbering the carbon atoms of the ring in
such a way that the functional group gets the lowest locant. For example,

> ? 6 CH,

3 5
~~ “CH,
3-Methyleyelohexan-1-ol 3-Hydroxycyclohexan-l-one 5,6-Dimethyleyclohex-2-en-1l-one

9. If an alicyclic ring is directly attached to a carbon containing functional group, the carbon of the
functional group ts not included in the parent name of the alicyclic system. In these systems, the prefixes and
suffixes for the functional groups are commonly used.

WWW.JEEBOOKS.IN
ORGANIC CHEMISTRY: BASIC PRINCIPLES AND TECHNIQUES 12/51 =>

C2 )

5
Cyclohexane carbaldehyde Cyclohex-2-ene-1-carbonitrile

CN O
om <1 COOH

2-Cyanocyclohexane-1l-carboxylic acid 2-Oxocyclopentane-1-carbonitrile 2-Oxocyclohexane-1-carboxylic acid

COOH
O COcl

io 1-COOH

Methy] (2-oxocyclopentane)-1- 2-Formy]-4-oxocyclohexane-1- 2-Chlorocarbonylcyclohexane-1-


carboxylate earboxylic acid earboxylic acid

Ifthe alicyclic ring is directly linked to benzene ring (discussed later), itis treated as cycloalkyl substituent
irrespective of the number of carbon atoms in the ring. Therefore, it is named as derivative of benzene. For
example,

Cyclohexyl benzene Cyclopentyl benzene 1-(2-Methyleyclohexyl)-4-nitrobenzene

Bond Line Notation


Sometimes, the bonds between carbon atoms are represented by lines. For example, n-hexane has a
continuous chain of six carbon atoms which may be represented as :
CH,—CH,—CH,—CH,—CH,—@H, WR/\/
n-Hexane :
In this notation, the carbon atoms are represented by line ends and intersections. It is assumed that the
required number of hydrogen atoms are present wherever they are necessary to satisfy the tetravalency of
carbon. A single line represents a single bond (C—C), two parallel lines represent a double bond (C=C) and
three parallel lines represent a triple bond (C = C). For example,

1- Methyl-3-propylcyclohexane 3-Ethyl-1,1-dimethylcyclohexane 3-Methylcyclohex-1-ene


(Not 1-ethyl-3, 3-dimethylcyclohexane)
1 3
4 2 2 4

\
Pent-1-ene Buta-1,3-diene 1,3-Dicyclohexylpropane
1 3 5
2 4 - 4
WN |
MN/\.,
1-Cyclobutylpentane Penta-1,4-diene 2-Methylbuta-1, 3-diene
(a line at position 2 corresponds to CH, group)
MODERN'S abe + OF CHEMISTRY-XI

3-Cyclohexylbutan-2-ol 1,3-Dimethyl 4-Hydroxy-4-methyl


cyclohex-1l-ene pentan-2-one

; ; OH

6 5
4
3-Ethyleyclohexa-1,3-diene 3,3,5-Trimethylhex-l-en-2-0l 5,6-Dimethylcyclohex-2-en-1-one

2 4

Va
Oo 3 o
;
1-Bromo-3-methylpentan-2-one Hexa-1,3,5-triene
WRITING STRUCTURAL FORMULA FROM THE NAME OF THE COMPOUNDS
We have discussed how to write the name of an organic compound, if its structure is given. Now let us do
the reverse i.e., to write the structure of an organic compound if its name is given. For this purpose, the procedure
is as follows :
(t) Write a straight chain of carbon atoms according to the number of carbon atoms in the parent compound.
At this stage, do not attach any hydrogen atoms to complete the tetravalency of carbon as
C—C—C—C—C—C
(ti) Number the chain of carbon atoms from any end
1 2 838 4 5 6
C—C—C—C—C—C
(iti) In case, the name of the compound contains -ene or -yne, then fill double or triple bond at indicated
positions.
For example, 2-ene may be indicated as
1 {/@ Wins ¢&
C—C =C—C—C—C
(iv) Attach substituents and functional groups at the desired positions.
(v) Make each carbon tetravalent by attaching a suitable number of hydrogen atoms.
Let us illustrate the above procedure by the following examples.

——e= SOLVED EXAMPLES


LJ Example 11.
Write the structural formulae of the following compounds :
(a) But-2-en-1-oic acid
(b) 2, 4, 4-Trimethyl-3-isopropylpent-1-ene.
Solution : (a) But-2-en-1-oic acid
({) The word root but indicates that the compound has four C atoms.
C—C—C—C
(ii) Let us number each carbon atom of the chain as :
12 3 4
C—C—C—C
(iit) Since its name has 2-en, it means double bond starts from carbon atom no. 2.
123 4
C—C
(iv) Since the name of the compound ends in 1-oic acid, hence the first carbon atom will have

a
<7 2 TV)
ee -

O
Xv 2 3 4
C—_Cc=—Cc_c
HO
WWW.JEEBOOKS.IN
ORGANIC CHEMISTRY: BASIC PRINCIPLES AND TECHNIQUES 12/53 ~~»

(v) Attach suitable number of hydrogen atoms.

XLT
SC—C=C =~" or CH,CH = CHCOOH
HO~ H
(b) 2, 4, 4-Trimethyl-3-isopropyl pent-1-ene
(i) As the name of the compound has the word root pent, it has five carbon atoms.
C—C—C—C—C
(72) Let us number each carbon atom of the chain

(iit) Since its name has 1-ene, it means that double bond starts from first carbon atom.
12 38 4 5
C=C—C—C—C
(iv) Attach three methyl radicals at 2, 4 and 4 positions and one isopropyl radical at 3 position.

CH,
12 3 a1 5
eyCH,CHTL
CH,
H,C CH,
(v) Attach suitable number of hydrogen atoms.
H H CH, H
| | | °]

CH, CH CH, H
CH a
CH a
LJ Example 12.
Write the IUPAC names of the following compounds :

(i) CH, CH, - CH, CH, = CH, CH,


OH CH,
O O O
| | |
(ii) CH,;,—CH,—C—CH,—C—CH, _ (iii) CH, -C—CH,—CH,—CH,COOH
(wv) HC =C —CH = CH — CH = CH,

O
(v) Z (vi) .
DON
Cl H

(ix) ANY (x) Cl, CH CH,OH

WWW.JEEBOOKS.IN
Size Solution : O
MODERN'S abe + OF CHEMISTRY-XI

QO
1 2 8 4 58 6 7 8
6 5 4l| 3 a\| 4
(z) CH, CH, CH CH, CH, CH CH, CH, Gi) €n,—bn,—b_én,2b_4n,
H Ha. Hexane-2, 4-dione
6-Methyloctan-3-ol
O
6 ail 4 3 2 1 6 5 A 3 2 1
(ii) CH,—C—CH,—CH,—CH,COOH (iv) HC=C—CH=CH—CH=CH,
5-Oxohexanoie acid Hexa-1,3-dien-5-yne

(v) | GA (vi) _™ [rn


Cl H
or
CH,—CH CH, or
7 6 5 41 3 2141 O
CH,—CH,—CH,—C—C = C—CH, 3 2 1 ||
4-Propylhept-2-yne Cl—CH,—CH,—C—H
3-Chloropropanal

(vit) | CN wid NZ.


~ >
Cl Br

or or
Cl
CH, 7 2 3|
CH,—CH,—C—CH,—CH,,—_CH,
4 5 6 7
CH,
5 4 3| 2 1
CH,—CH,—CH—CH,—CN |
Br
3-Methylpentanenitrile
3-Bromo-3-chloroheptane
2. a
(ix) (x) Cl, CH CH, OH

2.2-Dichloroethanol
1 2 3 4 2 6 T
CH,—CH—CH,—CH,—CH—CH,—CH,
| |
CH, CH,
2,5-Dimethylheptane

U) Example 13
Which of the following represent the correct IUPAC name for the compounds concerned :
(i) But -3-yn-1-ol or But-4-ol-1-yne
(iu) 4-Chloro-2-methylpentane or 2-Chloro-4-methylpentane
(iii) 2, 2,-Dimethylpentane or 2-Dimethylpentane
(iv) 2, 3- Dimethylpentane or 3, 4,-Dimethylpentane
(v) 2, 4, 7- Trimethyloctane or 2, 5, 7-Trimethyloctane.
Solution: 4 o 2 1
(i) But-3-yn-l-ol : HC = C—CH,—CH,OH Functional group (—OH) gets preference in
numbering the carbon chain.
1 3 a 4 o
(ii) 2-Chloro—4—methylpentane : peg ety Chloro substituent gets preference in numbering.

Cl CH

CH,
. | iol 3 4 5
(iit) 2, 2 -Dimethylpentane :H,C—C—CH,—CH,—CH, Position of both the substituents are indicated
separately.
CH a
ORGANIC CHEMISTRY: BASIC PRINCIPLES AND TECHNIQUES 12/55 a

1 2 3 4 5
(tv) 2, 3-Dimethylpentane : CH,—CH—CH—CH,—CH, It has lowest set of locants.
CH, CH,

1 2 3 4 5 6 7 8
(v) 2,4, 7-Trimethyloctane : aan adi Win Wile It has lowest set of locants.

CH, CH, CH,

LJ Example 14.
Write bond line formula for
(i) tert-Butyleyclopentane (it) Isopropylalcohol
(ut) 2, 3-Dimethylbutanal (tv) Heptan-4-one
(v) Cyclohexanone
CH,
|
Solution : (1) H, —CH,

or

,CHO
(ii) > OH (iii) /

O O

(iv) (v)

NOMENCLATURE OF SIMPLE AROMATIC COMPOUNDS


Aromatic compounds are those which contain one or more benzene rings 1n them. An aromatic compound has
two main parts :
(i) Nucleus (ii) Side chain
(i) Nucleus. The benzene ring represented by regular hexagon of six carbon atoms with three double bonds
in the alternate positions is referred to as nucleus. The ring may be represented by any of the following ways :

JON
HC CH O
ca
(z) (iL) (LiL) (iv)
(ii) Side chain. The alkyl or any other aliphatic group containing at least one carbon atom attached to the
nucleus is called side chain. These are formed by replacing one or more hydrogen atoms in the ring by alkyl
radicals t.e., ROR may be —CH,, —C,H,, —C,H. etc.)
_,
/_R/ ———> Side chain

Ring or nucleus
If one atom of hydrogen of benzene molecule is replaced by another atom or group of atoms, the derivative
formed is called monovalent substituted derivative. It can exist only in one form because all the six hydrogens
of benzene represent equivalent positions. For example, C,H.X, where X is a monovalent group.
When two hydrogen atoms of benzene are replaced by two monovalent atoms or group of atoms, the
resulting product can have three different forms. These forms are distinguished by giving the numbers. The
position occupied by one of the substituent is given as 1 and the other position is numbered in a clockwise
direction.
(i) Ortho (or 1, 2-). The compound is said to be ortho (or 1,2-) if the two substituents are on the
adjacent carbon atoms.
(it) Meta (or 1, 3-). The compound is said to be meta or (1,3-) if the two substituents are on alternate
carbon atoms.
(tit) Para (or 1, 4-). The compound is said to be para (or 1,4-) if the two substituents are on diagonally
situated carbon atoms.
Ortho, meta and para are generally represented as o-, m- and p- respectively as shown below :

1, xX 1
2
fi Aky
(1,2-) (1,3-)

Ortho Meta (1,4)


Para
Aryl group. The radicals obtained by removal of one or more hydrogen atoms of the aromatic hydrocarbon
molecules are known as aryl radicals or aryl groups. For example,

| _ CH, CHL CHH =

Phenyl o-tolyl _ Benzyl Benzal Benzo

From nucleus From side chains


Nomenclature of Different Aromatic Compounds
The names of few simple aromatic compounds are given below :
Hydrocarbons
_ CH, _ CH,CH, fw _ CH, CH,

C O Cy CH,
CH,
Benzene Methylbenzene Ethylbenzene 1,2-Dimethylbenzene 1,3-Dimethylbenzene 1, 4-Dimethylbenzene
(Toluene) (o-xylene) (m-Xylene) (p-Xylene)

H, CH.—CH—CH, _ CH=CH, C=CH

x6 CH,
1, 3, 5-Trimethylbenzene (2-Propyl) benzene Ethenylbenzene Ethynylbenzene
(Mesitylene) (Isopropyl benzene (Styrene) (Phenylacetylene)
or Cumene)
The aromatic hydrocarbons may also contain two or more benzene rings condensed together.

Naphthalene Anthracene
ay
Phenanthrene
Halogen derivatives
Nuclear substituted
~—6Cl CH, ~ Cl

Chlorobenzene
on
2-Chlorotoluene
cy
1,2-Dichlorobenzene
(o-Chlorotoluene) (o-Dichlorobenzene)
ORGANIC CHEMISTRY: BASIC PRINCIPLES AND TECHNIQUES

Side chain substituted

_ CH,Cl _ CHCI, r CCl

Phenylchloromethane Phenyldichloromethane Phenyltrichloromethane


(Benzyl chloride) (Benzal chloride) (Benzo chloride)
Hydroxy derivatives
The nuclear hydroxy derivatives are called phenols while the side chain substituted hydroxy derivatives
are called aromatic alcohols.
(4) Phenols
OH CH, _ CH,

on Ci 3
2 3

Li a “~OH

OH
Phenol 2-Methylphenol 3-Methylphenol 4-Methylphenol
(o-Cresol) (m-Cresol) (p-Cresol)
_ OH OH OH
,,OH |

& OH
OH
1,2-Dihydroxybenzene 1, 3-Dihydroxybenzene 1, 4-Dihydroxy- Benzene-1, 2, 3-triol Benzene-1, 2, 4-triol
(Catechol) (Resorcinol) -benzene (Pyrogallol) (Hydroquinol)
(Hydroquinone
or Quinol)
(iz) Aromatic alcohols
2 1 2 i
_ CH,OH
a CH,CH,OH
g = CH;:—C HOH
B o

Phenylmethanol 2-Phenylethanol 1-Phenylethanol


(Benzyl alcohol) (B-Phenyl ethyl alcohol) (a-Phenyl ethyl alcohol)
Ethers
_ OCH, _ OCH,CH,

Methoxybenzene Ethoxybenzene
oo
Phenoxybenzene
(Anisole or Methyl pheny! ether) (Phenetole or Ethyl phenyl ether) (Diphenyl] ether)
Aldehydes and ketones
Nuclear substituted
3 2 1
HO ‘HO _ CH,CHO | CH2CH,CHO
| Of
OH ‘

Benzaldehyde 2-Hydroxy benzaldehyde 2-Phenylethanal 3-Phenylpropanal


(Salicylaldehyde) (Phenyl acetaldehyde) (B-Phenylpropionaldehyde)
1 2 3 4
_ COCH, _ COCH,CH, COCH,
CH, CH,

1-Phenylethan-1-one
ora Diphenyl ketone 1-Phenylpropan-1-one 1-Phenylbutan-1-one
(Methyl phenyl ketone (Benzophenone) (Propiophenone (Butyrophenone or
or acetophenone) or diphenyl ketone or ethylphenyl ketone) Phenyl-n-propylketone)
Size MODERN'S abe + OF CHEMISTRY-XI

Carboxylic acids
Nuclear substituted

_CH,

C
COOH COOH . COOH
cy COOH NH,

Benzoic acid 2-Methylbenzoic acid


co”
2-Hydroxybenzoie acid 9-Aminobenzoie acid
(o-Toluic acid (o-Salicylic acid or o-Hydroxy (Anthranilic acid or
or o-methyl benzoic acid) benzoic acid) o-Aminobenzoic acid)
~ COOH _ COOH COOH
COOH

COOH
COOH
Benzene-1 , 2-dicarboxylic Benzene-1, 3-dicarboxylic acid Benzene-1,4-dicarboxylic
acid (Isophthalic acid) acid
(Phthalic acid) (Terephthalic acid)
Acid derivatives
O
O O }
| |
CCl _. C—NH, _ COOCH, _ COOC,H, oC _ CO

Benzenecarbonyl chloride Benzamide Methyl benzoate Ethyl benzoate Benzoic anhydride


(Benzoyl chloride)

_ OOCCH, COOC,H. COOC,H. CONHCH, _ NHCOC,H,

Phenyl ethanoate Phenyl N-Methylbenzamide N-Phenylbenzamide


(Phenyl acetate) benzoate Ethyl ern cbonzoate (Benzanilide)
Amino derivatives
Nuclear substituted
2 1
_ NH, = 5 _ CH,NH, CH, CH,NH,

O
Aniline or benzenamine
oo”
2-Aminotoluene Phenylmethanamine 2-Phenylethanamine
(Aminobenzene) (o-Toluidine) (Benzyl amine) (6-Phenyl ethylamine)
Sulphonic acids
SO.H SO.H CH 3
1

(:“~SO,H
2

50,H
Benzenesulphonic acid Benzene-1, 3-disulphonic acid 4-Toluenesulphonic acid
(m-Benzene disulphonic acid) (p-Toluene sulphonic acid)
Nitro derivatives
_ NO, _ NO, _ OH
O,N NO,
CO CL vxo,
NO,
Nitrobenzene 1, 3-Dinitrobenzene 2, 4, 6-Trinitrophenol
(m-Dinitrobenzene) (Pierie acid)
ORGANIC CHEMISTRY: BASIC PRINCIPLES AND TECHNIQUES

Cyanides and isocyanides

O C
C=N . CH,C=N N=C

Benzene carbonirile Phenylethanenitrile Phenylisocyanide


(Benzonitrile or (Benzyl cyanide or or Phenyl carbylamine
Phenyl cyanide) Phenyl acetonitrile)

SOME TIPS FOR NOMENCLATURE OF AROMATIC COMPOUNDS


For IUPAC nomenclature of substituted benzene compounds, the substituent is placed as prefix to the
word benzene. It may be noted that common names of many substituted benzene compounds are still universally
used. Some important tips for nomenclature of organic compounds are given below :
1. When the benzene ring is named as substituent on the other molecule, it is named as phenyl group. It
is treated in the nomenclature just like the name of an alkyl group. It is abbreviated as Ph.
For example,
_CH,CH,CHO

or Ph CH,CH,CHO 3-Phenylpropanal

| 4 7 2 2 1
1 > 3 4
|
( )-¢H,—C = C—CH, ()-CH, — a — CH, — CHO
OH
1-Phenylbut-2-yne 4-Phenyl-3-hydroxybutanal
4.5
oH,
3 2 1 2 1
CH = CH—COOH C,H,-C——CH-CH,
C,H, OH
3-Phenylprop-2-enoic acid 3, 3-Diphenyl pentan-2-ol 2, 3-Dibromo-1-phenylpentane
2. Disubstituted, trisubstituted or tetrasubstituted benzenes are named by using the numbers for the
positions of the substituents.
fCl NO,
1 2 Cl 1 ;

> O,N
fk ri NO,
1,3-Dimethylbenzene 1,2-Dichlorobenzene 1,3,5-Trinitrobenzene
3. If different groups are attached to the benzene ring, then the following rules are kept in mind:
(i) When an aromatic compound contains two or more functional groups, it is named as a derivative of the
compound with the principal group fixed as number 1.
(it) The numbering of the chain is done in any direction (clockwise or anticlockwise) which gives lower
number to the substituent.
(iii) The substituents are written in alphabetical order.
For example,
e Fixing the position of principal group as number 1.

_ OH CHO
1, NH, 1 ,

S q ~SOCH,
I OH
2-Aminophenol 4-Iodo-2-methylphenol 4-Hydroxy-3-methoxy 5-Bromo-2-ethylaniline
benzaldehyde
—OH group is —OH group is principal —CHO group is -NH, group is principal group
principal group group principal group
a 12/60 MODERN'S abe + OF CHEMISTRY-XI

CHO _ COOH _ COOH — COOH


2 . 2 . 2 cou

4 7 4 OCH, HO ° 4 NO,
2 NO, 2
4-Aminobenzaldehyde 4-Nitrobenzoic acid 3-Methoxy-4-nitrobenzoic acid 2-Bromo-5-hydroxy-
3-nitrobenzoic acid
—CHO is principal group —COOH is principal group —COOH is principal group —COOH is principal group
e If all the groups present in the benzene ring are different and none of these gives a special name to
the molecule, then these groups are arranged in alphabetical order. The group named first in the
alphabetical order gets the lowest number provided it does not violate the lowest locant rule for all
the substituents. The name of the molecule ends with the word benzene.
Br
1

Ci,

1-Bromo-3-chlorobenzene 1-Chloro-2,4-dinitrobenzene 2-Chloro-1-methyl-4-


(Not: 1-Chloro-3- (Not: 4-chloro-1, -nitrobenzene
bromobenzene) 3-dinitrobenzene) (Not: 4-Methyl
-5-chloro-1-nitrobenzene )
e When one of the substituents gives a special name to the molecule (e.g. toluene, anisole) then the
compound is named as a derivative of that special compound.

C,H
ae
2-Chloro-4-nitrotoluene 2-Chloro-4-methylanisole 3, 4-Dimethylphenol 4-Ethyl-2-methylaniline
IUPAC NAMES FOR SOME TYPICAL COMPOUNDS
Let us write the TUPAC names of a few typical organic compounds from their structural formulae.
CH,
— 1 2| 3
(CH,),C or CH,—C—CH, HCOOCH,
| Methylmethanoate
CH,
2,2-Dimethylpropane
1 (CH,CO),O or CH,—C—O—C—CH,
2 | |
3 OO
CH,CH, Ethanoic anhydride
3-Ethylcyclohexene

1 2 3 4
CH,—CA-SC hae H—CH,—CH, CH,
ll» sl 3 2| 4
CH, CH, °CH, (CH,),;CCHO or CH,—C—CHO
sl 7 |
CH, CH,—CH, CH;
3,4-Diethyl-2-methylheptane 2,2-Dimethylpropanal
CH,—O—CH(CH,),
CH,
or 9

ee (CH,),COH or CH,—C—OH
CH,—O—CH—CH, 7
| CH,
‘CH,
2-Methoxypropane 2-Methylpropan-2-ol
ORGANIC CHEMISTRY: BASIC PRINCIPLES AND TECHNIQUES
5 4 3 2 1
CH,CH,CNH, CH,=C—CH,—CH,—CHO
|
CH,
Propanamide 4-Methylpent-4-en-1-al
Cl
1 2 a4 5 6 1 2 38 4 5
OH OCH CH Cs CH,—C—CH,—_C—CH,
|| |
Br O O
3-Bromo-3-chlorohexane Pentane-2, 4-dione
(C,H,),CH—CH,OH
or
4 3 2 1
ic ce I alien Dea CH, 7/0—CH,CH,

CH,
iy O
Ethyl ethanoate

2-Ethylbutan-1-ol
O
i 23 al 4 5 6 1 2 3 4 5 6 T
CH, an ae le CH, = CH—CH,—CH,—C = C—CH,
Hept-1-en-5-yne
CH
6-Chloro-2-methylhexan-3-one
QO O 6 5 4 3 2 1
CH, CH, CH. CH = CH COOH
H. —H
Hex-2-en-1l-oic acid
Ethanedial
O
1 2 3 4 5 6 7 8 1
a ila Mee CH, CH,

OH CH, ®@:SOH
6-Methyloctan-3-ol 3-Hydroxycyclohexan-1-one
1

eal 3
Cy er
CH, CH,
al 5 Ce OH

“ 2-Cyclohexyl-4-methylpentane Cyclohexane-1, 2-diol


Oo O O
| il
9 L 6 CH, H—C—C—OH
Formyl methanoic acid
oy ty Gm
5,6-Dimethylcyclohex-2-en-1l-one

CH 3 2 1
| °>cH—CH,—CH - CH,
4 a 2 “It
CH, CH = CHCOOH
But-2-en-1l-oie acid CH;
3-Cyclopropylprop-1l-ene
3 2 1 4 3 a 1
CH = CH—CH, OH CH,—CH—CH = CH,

3-Phenylprop-2-en-1-ol 3-Cyclopentylbut-1l-ene

a
O OH
4 all 2 1
CH,—C—CH,—CHO
3-Oxobutanal
(—CHO is principal group) Cyclohex-2-en-1-ol
a 12/62 MODERN'S abe + OF CHEMISTRY-XI

O
4 3 2 dQ
(CH,CH,CO),O CH,—CH—CH—C—C]l

1 CH,Br
Propanoic anhydride 2-(Bromomethyl)-3-chlorobutanoyl chloride
O
4 3 2 1 5 4 8h, 2 1
CH,—CH,—CH,—_CH—OH OHC—CH,—CH,—C—CH, COOH

OCH,CH,
1-Ethoxybutan-1-ol 5-Formyl-3-oxopentanoic acid

Some examples of writing structural formulae from the IUPAC names are given below:
1 2 3 4 5
4-Methylpent-2-yne CH,—C=C—CH—CH.,

ds,
6 5 4 vie 1
Hex-3-enoic acid CH,CH,CH = CHCH,COOH
1 2 3 4 : 6 7
Hepta-1, 5-diene CH,=CH—CH,—CH,—CH = CH—CH,
5 A@:s S /1
4-Nitropent-1-yne CH,CH CH, C = CH
|
NO,
Cl
4a f 2| 1
CH, CH, C CH,OH
2-Chloro-2-methylbutan-1-ol |
CH,
ipa/f 3 4 = «5
5-Chloropent-2-ene CH.CH = CH CH, CH,Cl
1 2 8
1,3-Diaminopropane ia CH, vie

NH, NH,
Is « 4 5
Pentanedial OHC CH, CH, CH, CHO

3-Nitrocyclohexene

= SOLVED EXAMPLES
LJ) Example 15.
Write the structures for the following :
(i) 2-Chlorohexane (iit) Pent-4-en-2-ol
(iii) 3-Nitrocyclohexene (iv) Cyclohex-2-en-1-ol (v) 6-Hydroxyheptanal
Solution:
@ CH,CH,CH,CH,CHCH, (i) CH=CH CH, CH CH,
Cl OH
NO, _ OH
3 1
(iii) ;1 iv) f 3
(v) CH, CHCH,CH,CH,CH,CHO
WWW.JEEBOOKS.IN
ORGANIC CHEMISTRY: BASIC PRINCIPLES AND TECHNIQUES 12/63 >

LJ} Example 16.


The following organic compounds are popularly known by their common names : (i) Neopentane (ii) Acetone
(tii) Vinyl chloride (iv) tert-amyl alcohol (v) iso-butyl bromide (vi) acetonitrile (vii) allyl alcohol
(viii) formaldehyde (ix) acetone (x) glycerol. Write their structural formulas and IUPAC names.
Solution :
(1) Neopentane CH,

CH, HH. 2, 2-Dimethylpropane

CH,
(ii) Acetone CH,
Sco Propanone
CH,

(iii) Vinyl chloride CH, = rt Chloroethene

Cl

(iv) Tert-amyl alcohol in


3 i i 2-Methylbutan-2-ol

OH

(v) Isobutyl bromide Hyon CH,Br 1-Bromo-2-methylpropane

CH,

(vi) Acetonitrile CH,CN Ethanenitrile


(vit) Allyl alcohol CH,=CH—CH,OH Prop-2-en-1-ol
(vii) Formaldehyde HCHO Methanal
(ix) Acetylene HC = CH Ethyne

(x) Glycerol CH,—CH—CH, Propane-1, 2, 3-triol


|
OH OH OH
LI Example 17.
Write the structural formulae of
(a) p-Nitroaniline (b) o-Ethylanisole
(c) 2, 3-Dibromo-1-phenylpentane (d) 4-Ethyl-1-fluoro-2-nitrobenzene
Solution:
_ NH,
| OCH,
(a) (b) CL
C,H,
NO,
NO,
FE
(c) (d) (>
i |
L} Example 18.
Give condensed and bond line structural formulae and identify the functional group(s) present if any; for
(i) 2, 2, 4-trimethyl pentane (ii) 2-hydroxy-1, 2, 3-propanetricarboxylic acid
(tit) Cycloocta-1, 5-diene (iv) Hexanedial
(v) 2-(4-isobutyl phenyl) propanoic acid

WWW.JEEBOOKS.IN
Sizes Solution:
MODERN'S abe + OF CHEMISTRY-XI

Condensed structural Bond line structure Functional group

(i) (CH,),C—CH, CH (CH,),

OH
O OHO
(ii) cH—¢ CH, —COOH (carboxyl), —OH (hydroxyl)
| | |
COOHCOOH COOH HO ono OH
1 2

8 a

(iii) CsHi» a 7 Double bond (—)


6 5

O
(iv) OCH-(CH,),—CHO |
LANA
OH —CHO (aldehyde)
O

(v) CH, CH COOH . O

;K —COOH (carboxyl)

CH,
|
CH—CH,
|
CH,
lL} Example 19.
Identify the functional groups in the following compounds :
CHO NH 2

(a) (> (5) @


~OMe
OH O° ~OCH,CH,N(C,H,), CH=CHNO,
Solution :

(a) CHO

—CHO (aldehyde), —OH (hydroxy or alcohol) and


| OMe

OH —OMe (alkoxy or ether)

(0) 2

—NH, (amino) and

T
OCH,CH,N (C,H,), —C—OCH,CH, N (C,H,),; N, N-dialkylaminoester

(c)
Kz
<<)
CH=CHNO,
—NO, (nitro), C=C (alkene)

or unsaturated nitro group.


ORGANIC CHEMISTRY: BASIC PRINCIPLES AND TECHNIQUES

ISOMERISM
Two or more compounds having the same molecular formula but different physical and chemical
properties are called isomers.
The phenomenon of existence of two or more compounds possessing the same molecular formula
but different properties is called isomerism.
Types of Isomerism
The isomerism is of two main types :
(a) Structural isomerism
(6) Stereo tsomerism
(a) Structural isomerism. The compounds having same molecular formula but different structures 1.e.,
different arrangement of atoms or groups of atoms within the molecules are called structural tsomers and the
phenomenon is called structural tsomerism.
(6) Stereo isomerism. The isomers which have the same structural formula but have different relative
arrangement of atoms or groups of atoms in space are called stereo isomers and the phenomenon is called stereo
isomerism.
The stereo isomerism is of three types :
(1) Geometrical tsomerism,
(ii) Optical isomerism and
(tit) Conformational tsomerism.
Structural isomerism is discussed in the present unit.
A: Structural Isomerism
This can be further sub-divided as :
1. Skeletalorchainisomerism. The compounds having same molecular formula but different arrangement
of carbon chain (skeleton) within the molecule are called chain isomers or skeletal isomers and the
phenomenon is termed as chain isomerism.
For example, the first three members of the alkane family (CH,, C,H, and C,H,) do not exhibit chain
isomerism because they have only one structural formula.
CH, CH.— CH, CH.CH,CH,
Methane Ethane Propane
The next member butane (C 1,5) can have two structural isomers.
1 2 3
CH,—CH,—CH,—CH, or | CH,—CH—CH,
Butane |
CH,
2—Methylpropane

| | JS@l be. lk il
Carbon skeleton Cf
| {ea ae aes

Continuous chain of four Branched chain with three


carbon atoms carbon atoms in long chain

Similarly, pentane (C.H,.) can have three structural isomers. CH,


|
CH,CH,CH,CH,CH, CEC CH,—C—CH,
Pentane |CH,
OH,
2-Methylbutane 2,2-Dimethylpropane
(Isopentane) (Neopentane)
Some other examples are :
(a) C,H,OH has three chain isomers.

CH,—CH,—CH,—CH,OH a Gall e-em


i
Butan-1-ol :
| CH CH,
i a 2-Methylpropan-1-ol 2-Methylpropan-2-ol
(Isobutyl alcohol) (tert-Butyl alcohol)
(6) C gl g has two chain tsomers.
CH,—CH,—CH = CH, CHC = CH,
But-1-ene :
CH,
(Butylene) 2-Methylpropene
(Isobutylene)
2. Position isomerism. The compounds which have the same molecular formula but differ in the position
of the functional group, carbon-carbon multiple bond or substituent group are called position isomers and this
phenomenon is termed as position isomerism. For example, the formula, C,H,OH represents two position
isomers whose structural formulae are given below :
OH
3 2 1 1 a| 3
CH,—CH,—CH,OH and CH,.—CH—CH,
Propan-1-ol Propan-2-ol
(n-Propyl alcohol) (Isopropyl alcohol)

| OH
ls be Ib b hh
an oe ape =a Wied
Functional group is at Functional group is at
carbon atom 1 carbon atom 2

Some other examples of position isomers are :


(a) The compound CA, has two position isomers.
CH,.—CH,—CH=CH, CH,—CH =CH—CH,
But-1-ene But-2-ene
(a-Butylene) (Dimethyl! ethylene or B-Butylene)

(6) The compound C,H, has two position isomers.


4 3 2 1 4 2 f 4
CH.—CH,—C= CH CH.—C=> C—_CH,
But-l-yne _ ™~ But-2-yne
(a-Crotonylene) (Dimethyl acetylene or B-Crotonylene)
(c) The compound C,H, Cl, has two position isomers.
H. H H H
l2 fa lo |a
H—C—C—H H—C—C—C]l
| | | |
Cl A H Cl
1, 2-Dichloroethane 1, 1-Dichloroethane
(Ethylene dichloride) (Ethylidine dichloride)

(d) In aromatic compounds, the position isomers are observed due to the difference in the position of the

& & ¢
substituent groups attached to the benzene ring. For example, C,H,Cl, (dichlorobenzene) has three isomers.
Cl Ci

ortho-Dichlorobenzene meta-Dichlorobenzene para-Dichlorobenzene


ORGANIC CHEMISTRY: BASIC PRINCIPLES AND TECHNIQUES 12/67 a

3. Functional group isomerism. The compounds having same molecular formula but different functional
groups in the molecule are called functional group isomers and this phenomenon is termed as functional
group isomerism.
For example,
(a) Alcohols and ethers. C,H,O represents two isomers having different functional groups.
CH,—CH,OH and CH,—O—CH,

Ethanol Methoxymethane
(Ethyl alcohol) (Dimethyl ether)
(Functional group —OH) (Functional group —O—)
(6) Aldehydes and ketones. C,H,O represents two isomers having the structural formulae :
CH,—CH,—C=0 and CH.—C—CH,
| |
H O
Propanal Propanone
(Propionaldehyde) (Acetone)
(Functional group —CHO) O
|
(Functional group —C—)
It may be noted that unsaturated alcohols and ethers are also functional isomers of aldehydes and ketones.
CH,=CH—CH,OH CH,=CH—O—CH,
Prop-2-en-1-ol Methoxy ethene
(Allyl alcohol) (Methyl vinyl ether)
(c) Carboxylic acids and esters. The formula C,H,O, represents two isomers having the structural formulae:
O O
| |
CH,—C—OH and H—C—OCH,
(Ethanoic acid) Methyl methanoate
(Acetic acid) (Methyl formate)

T |
(Functional group —C—QOH) (Functional group —C—OR)

(d) Dienes, allenes and alkynes. Molecular formula C,H, represents three functional isomers:
1 2 8 4 1 2 38 £44
CH, =CH—CH= CH, CH, =C= CH—CH, CH,CH,C=CH or CH,C =C—CH,
But-1, 3-diene But-1, 2-diene , But-l-yne But-2-yne
(an alkene) (an allene)
(alkynes)
(e) Nitroalkanes and alkyl nitriles. Molecular formula C,H.NO, represents two functional isomers:

Tee
+
Ko
"a or CH,CH,NO,CH,.CH,—O—N==—O or CH,CH,ONO

Nitroethane Ethyl nitrile


(f) Cyanides and isocyanides. Molecular formula C,H,N shows two functional isomers.

CH,—C=N CH,_N==C
Ethane nitrile Methyl isocyanide
(g) Primary, secondary and tertiary amines show functional isomerism. For example, C,H,N represents
three isomers:
CH
H, °
CH,CH,CH,NH, CH,—CH oN CH, —N—CH,

Propan-1l-amine N-Methylethanamine N, N—Dimethylmethanamine


(1°-Amine) (2°-Amine) (3°-Amine)
a 12/68 MODERN'S abe + OF CHEMISTRY-XI

(h) Aromatic alcohols and ethers show functional isomerism. For example, molecular formula C,H,O
represents
. CH,OH OCH, OH OH

o oe on or
CL ‘CH,
or

Benzyl alcohol Anisole o-Cresol m-Cresol p-Cresol


(an alcohol) (an ether) (a phenol) (a phenol) (a phenol)
4, Metamerism. The compounds having same molecular formula but different number of carbon
atoms (or alkyl groups) on either side of the functional group, are called metamers and this phenomenon
is called metamerism. Metamerism is exhibited by the compounds having a bifunctional group i.e. —O—,
—5—, —NH—, —CO— etc. For example,
(a) C,H,,O has two metamers as
CH,CH,COCH,CH, CH,COCH,CH,CH, (—CO has different alkyl groups around it)
Pentan-3-one Pentan-2-one
(6) C,H,,O has two metamers as / CH,
1 2 8 4
CH.CH,O—CH,CH, CH,—O—CH,CH,CH, or CH.—O—CH ,
sae
Ethoxyethane 7 1-Methoxypropane \ CH,
(Diethyl ether) (Methylpropyl ether) 2-Methoxypropane
(Isopropyl methyl ether)
(c) C,H,,N
4°41 has two metamers as
CH.—CH,—NH—CH,—CH, CH,—NH—CH,—CH.,—CH,
N-Ethylethanamine N-Methylpropan-1-amine
(d) C,H,,5 has metamers : : CH
CH,CH,SCH,CH, CH,SCH,CH,CH, or CH, 5 CH _
Diethyl thioether Methyl n-propyl thioether Isopropyl methyl thioether
It may be noted that some compounds exhibit both position isomerism as well as metamerism. For example

CH,CCH,CH,CH, and CH.CH,CCH,CH, are position isomers as


Pentan-2-one Pentan-3-one well as metamers

5. Tautomerism. This is a special type of functional isomerism in which the isomers differ in the
arrangement of atoms but they exist in dynamic equilibrium with each other and this phenomenon ts termed as
tautomerism. For example, acetaldehyde and vinyl alcohol are tautomers which exist in equilibrium as :
H O OH
| |l |
H—C—C—H —————— H—C — C—H
| |
H H
Acetaldehyde Vinyl alcohol
Similarly, Gea: ——_—= CH, — C—CH,

OH
Propanone Prop-1-en-2-ol
oO OH

O
Cyclohexanone
—=<
Cyclohex-1-en-1-ol
Acetyl acetone — m two as:

1 1 ro T
CH,—C—CH,—C—CH, =———>
——=———=— CH, —C = CH—C—CH,
(Keto form) (Enol form)
ORGANIC CHEMISTRY: BASIC PRINCIPLES AND TECHNIQUES 12/69 =>

When one of the tautomers exists in keto form and the other in the enolic form, then tautomerism 1s also called
keto-enol tautomerism.
It may be noted that only those carbonyl compounds exhibit keto-enol tautomerism which have at least one
acidic a-hydrogen i.e. a-hydrogen on a saturated carbon. Thus, acetophenone, butan-2-one, propanal, etc, exhibit
keto-enol tautomerism.
O O
a || a || a a ||
CH,—C—C,H, CH,CH, C CH, CH,CH,C—H
Acetophenone Butan-2-one Propanal
On the other hand, benzophenone, benzaldehyde, etc. do not show keto-enol tautomerism because these do
not have a-hydrogen atom.
O | O

Oi
Benzaldehyde
OX)
Benzophenone
Structural Isomerism in Alkanes
The alkanes show structural isomerism
in which the isomers differ in the skeletons of
carbon atoms. This is also known as chain isomerism. For example, the first three members of the family
(CH,, C,H, and C.H,) do not exhibit chain isomerism because they have only one structural formula :

i rT ma
H H H H H H
Methane Ethane Propane

Butane (C rs can have two structural isomers :

H HH H HH H
HC_¢_¢_C_H wooo
HHH H Pye
or CH,—CH,—CH,—CH, — 7
Butane iH
or CH,—CH—CH,

oH,
2-Methylpropane
The two isomers differ in the chain of carbon atoms as :
C—C—C—C C—C—C
Straight chain (
Branched chain

Similarly, pentane (C.H,.) has three isomers.

Prt et Li it
“Trrty.
H H H H H
meeeH H H
H—C—H
or CH,—CH,— CH, —CH,—CH, |
Pentane
or CH,—CH,— - — CH,
CH,
2-Methylbutane
MODERN'S abc + OF CHEMISTRY
-XI

CH,

or CH, — C—CH,

On,
2-2-Dimethylpropane

Hexane (C,H, ) has five st ructural isomers

PTT dd H H H HH H
HHH HH
PEY TELE
r o
(i) HG_0-0__¢-¢_
Huu |H
H—C—H
i
or CH,—CH,—CH,—CH,—CH,— CH, or CH, —CH,— CH, — a — CH,
Hexane
CH,
2-Methylpentane
H

| |
(iv) a a ie a
H H H
H—C—H
|
H

or CH, —CH,—CH—CH_—CH, or CH,— , — a — CH,

CH, CH, CH,


3-Methylpentane 2,3-Dimethylbutane

H H H—C—H H CH,
or CH,—CH,—C—CH,
H H H—C—H H oH,
2, 2-Dimethylbutane

It is clear that as the number of carbon atoms increases, the number of chain isomers also increases. For
example, C,H,, has nine, C,H,, has eighteen chain isomers. The hydrocarbon C,,H,, has 75 structural
isomers.

B : Stereolsomerism
These are the compounds which have the same constitution (structural formula) and sequence of bonds
but differ in the relative positions of atoms or groups of atoms 1n space. The common example of stereoisomerism
ORGANIC CHEMISTRY: BASIC PRINCIPLES AND TECHNIQUES 12/71 =>

is geometrical isomerism exhibited by alkenes. As will be studied (Unit-13) the compounds containing double
bond (alkenes) show cis-trans isomerism. This is due to the restricted rotation around carbon-carbon double
bond. As a result, the positions of the groups attached to these carbon atoms are fixed in space. This type of
isomerism is also called geometrical isomerism. Stereoisomerism will be discussed in detail in next class.
One of the consequences of geometrical isomerism is steric hindrance. This is due to the fixed positions
of atoms or groups of atoms around carbon-carbon double bond. It is known that two atoms or groups of atoms
in an organic molecule at a distance less than or equal to the sum of the van der Waals radii repel each other due
to spatial crowding. This repulsion is known as steric hindrance or steric strain or van der Waals strain.
Larger is the size of the group, greater will be the steric hindrance. Molecules having steric hindrance are
relatively less stable than those which do not have steric hindrance. For example, cis-But-2-ene has steric
hindrance while trans-But-2-ene does not have any steric hindrance.

Steric hindrance No steric hindrance

(CH,}
f . f%
.CH;} HC )
ee = ee a —_ “Sf
H H H CH,
cis-But-2-ene trans-But-2-ene
(less stable) (more stable)
Similarly, cis-Pent-2-ene is less stable than but-2-ene and hex-3-ene is even less stable than pent-2-ene.
This is due to larger steric hindrance of bulky groups.
CH,CH, fa CH,CH, Jus CH, CH,
‘ =C < = < Now i

of Na “a Na H “a Na
Hex-3-ene Pent-2-ene But-2-ene

Steric effects greatly influence the structure and reactivity of organic compounds.
To Sum Up

STRUCTURAL ISOMERISM STEREOISOMERISM

e Chain isomerism @ Geometrical isomerism


e@ Position isomerism e@ Optical isomerism
@ Functional isomerism
e Metamerism
e Tautomerism

———S Bese 8
Example 20.
Draw the structures of bromomethane, bromoethane, 2-bromopropane and tert-butyl bromide. Arrange these
in order of increasing steric hindrance.
Solution :
H CH, CH, CH,
| |
H—C—Br H—C—Br H—C—Br CH,—C—Br
| | | |
H H CH, CH,
Bromomethane Bromoethane 2-Bromopropane tert-butyl bromide
(No steric hindrance) (Small steric hindrance) (More steric hindrance) (Most steric hindrance)

WWW.JEEBOOKS.IN
a 12/72 MODERN'S abe + OF CHEMISTRY-XI

Thus, the steric hindrance increases as :


bromomethane < bromoethane < 2-bromopropane < tert-butyl bromide.

ELECTRON DISPLACEMENTS IN COVALENT BONDS


Organic compounds mainly consist of covalent bonds. The electron pairs in these covalent bonds may undergo
displacements either of their own or under the influence of other species (atom or group of atoms) or in the
presence of an appropriate attacking reagent. Four types of electron displacements are generally noticed in the
mechanism of organic reactions. These are given below :
1. Inductive Effect
This is a permanent effect which arises whenever an electron withdrawing group (electronegative atom
such as halogen) is attached to the end of a carbon chain. To understand this, let us consider a chain of carbon
atoms having Cl atom at one end:
C—C—C—C—C]
We know that chlorine is more electronegative than C. Due to larger electronegativity of Cl, the electron
pair shared between C, and Cl will be displaced towards Cl atom. As a result of this, Cl acquires a partial
negative charge (5—) and C acquires a partial positive charge (5+). This displacement is however, not limited to
C,—Cl bond but is transmitted to other carbon atoms along the chain. This is because, the small positive charge
on C, will attract the o-electrons of the C,—C, bond towards it. As a result of this displacement, the positive
charge on C, is partially neutralised while a small positive charge is developed on C,. The charge on C, is less
than that on C, (6'+ < 5+). Similarly, C, will acquire a small positive charge 5+" (5+" < 5'+). Such a polarisation
of o-bond caused by the polarisation of adjacent o-bond is called inductive effect. Thus,
the process of displacement of o-electrons along the saturated carbon chain due to the presence
of a polar covalent bond at one end of the chain is called inductive effect and is denoted as I effect.
This is a permanent effect and is generally represented by an arrow with its head in the middle of the
covalent bond pointing in the direction of electron displacement as shown below :
rT nn <n <n
SS SS eae (64'" < 64+" < 6+' < 5+)
However, it may be noted that this effect decreases sharply as we move away from the atoms involved
in the initial polar bond and becomes negligible from the fourth atom onwards.
Types of Inductive Effects
The inductive effect is related to the ability of the substituent to either withdraw or donate electron density
to the attached carbon atom. Based on this ability, the substituents may be classified as electron withdrawing
or electron donating groups relative to hydrogen. For comparing the relative effects, hydrogen is taken as
standard.
The substituents having electron attracting power more than hydrogen are called electron
withdrawing substituents. For example, groups such as halogens, nitro (-NO,), cyano (-CN), carboxy (-COOH),
ester (-COOR), aryloxy (—OAr, e.g., -OC,H.) are electron withdrawing groups.
On the other hand, the substituents having electron attracting power less than hydrogen are called
electron donating groups. For example, alkyl groups such as methyl (-CH,), ethyl (-C,H.), isopropyl
[-CH(CH,),], t-butyl [-C(CH,),] etc. are electron donating groups.
Based on the type of substituents, there are two types of inductive effects designated as -I effect and
+ I effect.
(i) Ifthe substituent attached to the end of the carbon chain is electron withdrawing, the effect is called
-I (or electron withdrawing) effect. For example,
-_
C>Cs C> X (electron withdrawing substituent)
The —I effect of some substituents in the decreasing order w.r.t. hydrogen is:
—NO, > —CN > —COOH > —COOR > —F > —Cl > —Br > —I > —OH > SIE > =. IE >H
ORGANIC CHEMISTRY: BASIC PRINCIPLES AND TECHNIQUES 12/73 >=

(ii) If the substituent attached to the end of the carbon chain is electron donating group, the effect is
called + I (or electron donating) effect. For example,
5-
C<C~<C-~< CH, (electron donating substituent)
The +I effect of some substituents in the decreasing order w.r.t. hydrogen is
(CH,),C— > (CH,),CH— > CH,CH,— > CH,—
It is clear that the more substituted the alkyl group, stronger ts its + I effect.
The inductive effect plays very significant role in organic chemistry in understanding reactions. Carbon of
methyl chloride, for example, acquires partially positive charge due to shift of electrons towards chlorine and
provides a site for attack by a negatively charged species (say OH).

H
—_ lst os
A—Cl ——> CH,OH + Cr
7
2. Electromeric Effect
It is a temporary effect which takes place between two atoms joined by a multiple bond, i.e., a double or a
triple bond. This occurs at the requirements of the attacking reagent and involves instantaneous transfer of a
shared pair of electrons of the multiple bond to one of the linked atoms. However, when the attacking reagent
is removed, the molecule acquires its original electronic condition.

As an example, consider the carbonyl! group, >Cc=0, present in aldehydes and ketones. When a negatively
charged reagent seeking positive site, say :X: approaches the molecule, it causes instantaneous shift of
electron pair of carbonyl group to oxygen (more:-electronegative than carbon). As a result, the carbon becomes
deprived of its share in this transferred pair of électrons and

Nant \8* ny
C—O + :Z: — > c———o —— Sc
7 “ yy ANY
Attacking species

thus acquires positive charge while oxygen which has taken complete control of the electron pair becomes
negatively charged. Therefore, in the presence of attacking reagent, one bond is lost. Then the negatively charged
attacking reagent links to the carbon having positive charge.
The phenomenon of movement of electrons from one atomto anotherina multiple bond at the demand
of attacking reagent is called electromeric effect and is denoted as E effect.
The electromeric shift of electrons occurs only at the moment of the reaction.
Like the inductive effect, the electromeric effect is also classified as :
(t) Positive electromeric effect or + E effect
(ii) Negative electromeric effect or —E effect
(t) Positive electromeric effect (+E effect). When the electrons of the 2-bond are transferred to the atom to
which the attacking reagent finally gets attached, the effect 1s called +E(positive electromeric). For example, the addition
of H*(acids) to alkenes.

Sec?
DC=CK 2+ HOa — > Sis
YCKK
(attacking |
reagent) H
— 12/74 MODERN'S abc + OF CHEMISTRY-XI
(i) Negative electromeric effect (-E effect). When the electrons of the t-bond are transferred to the atom
other than the one to which the reagent gets finally attached, the effect is called -—E (negative electromeric)
effect. For example, the addition of CN to the carbonyl group,

Electromeric effect

1. It is a permanent effect which involves slight . Itisatemporary effect which involves complete
drifting of shared o-electrons towards more transfer of m-electrons to one of the bonded
electronegative atom. atoms.
. It operates only in saturated compounds which 2. It operates in unsaturated compounds which
contain at least one polar bond. contain at least one multiple bond which may
be polar or non-polar.
. It does not need any outside reagent for its . It operates only in the presence of an outside
operation. reagent.
. Itinvolves a partial separation of charges without . It involves complete transfer of electrons from
formation of ions. the reagent to substrate and vice versa. As a
result of this effect, ions are formed.

3. Resonance or Mesomeric Effect


There are many molecules whose behaviour cannot be explained by a single Lewis structure. To explain their
behaviour two or more than two structures may be proposed (Unit IV) and the actual molecule is said to be
resonance hybrid of these structures. This phenomenon is called resonance. Thus
if a molecule can be assigned two or more structures, none of which ts capable of describing all
the known properties of the compound, then the actual structure is intermediate or resonance
hybrid of these structures. This phenomenon is called resonance.
The various structures written are called resonating structures.
The phenomenon of resonance can be explained with the help of CO, molecule. It has been represented
by the structural formula :

:O::C::0: or o=C=0

Though this satisfies the conventional valency requirements, yet it cannot explain some experimental facts.
The actual carbon-oxygen bond in CO, has been found to be 115 pm whereas the normal carbon-oxygen double
bond is 122 pm and that for triple bond is 110 pm. This means that carbon oxygen bond in carbon dioxide is
intermediate between a double and a triple bond. To account for this, two other structures were proposed :

20: CHO: | and:O0#C:60:


The actual molecule is said to be a resonance hybrid of these three structures :

:0=C=O: «<-> :0-C=0: <> :10=Cc-0:


" (i) (ii) (iii)
Let us consider another example of benzene. It may be represented by a cyclic structure having alternate
C—C single and C = C double bonds.

He"
| | or CO
HC. _C—H
ORGANIC CHEMISTRY: BASIC PRINCIPLES AND TECHNIQUES 12/75 >

Therefore, we expect two different bond lengths of carbon-carbon bonds due to C—C single and C = C
double bonds. However, as determined experimentally, benzene has a uniform C—C bond distance of 139 pm.
This value is intermediate between the C—C single (154 pm) and C = C double (134 pm) bonds. Thus, the
structure of benzene cannot be represented adequately by a single structure. The benzene molecule may be
regarded as resonance hybrid of the following two structures :
H H
| |
7 oS
Hc? SoH H—C C—H
| | ~—-> || | or simply as OC} <-> (>
H—C X me H—C~ | oe

| |
H H
The resonating structures of carboxylic acid may be written as :

a O: raO:
ce — RC
6-H Q'-H
Similarly, nitromethane may be represented by the following resonating structures :

O: {oe
CH,— L)

I IT

The structures I and IT have two types of NO bonds. However, it is known that two N—O bonds of
nitromethane are of the same bond length which is intermediate between a N—O single and a N=O double bond.
The actual structure of nitromethane is regarded as the resonance hybrid of structures I and II.
The resonance structures of acetate ion (CH,COO), aniline (C,H.NH,) and phenol (C,H.OH) are shown
below :

Acetate CH,COO™ ion

one | —> CH.—C a -


CO:
.

Aniline C,H,NH, molecule

arene
CNH,

Phenol C,H,OH molecule

to
: OH (OH

It may be noted that the energy of the actual structure of the molecule i.e. resonance hybrid is lower than
that of any of the resonating structures. The difference in energy between the actual structure and the most
stable of the resonating structures is called resonance energy or resonance stabilisation energy. The
different resonance structures contribute to the actual structure in proportion to their stability.
— 12/76 MODERN'S abe + OF CHEMISTRY-XI
Rules for writing resonance structures
The following rules are applied for writing resonance structures :
1. The resonance structures should have same positions of nuclei.
2. All the resonance structures should have same number of unpaired electrons.
3. The contributing structures should have nearly same energy.
4. All the atoms should have octet of electrons (except hydrogen which has duplet). The resonance structures
which violate octet rule should not be considered. For example, in the structures :

CH, (aay
= CH ~ NH, ——— CH, — CH = NH 3
I Il
Structure IT cannot be considered as a resonance structure because it has 10 electrons around nitrogen
and hence violates the octet rule. As we know, nitrogen cannot have more than 8 electrons because it does not
have d-orbitals.
Relative Contribution of Resonance Structures
(t) Structures which are indistinguishable are of equal energy and hence contribute equally towards resonance
hybrid. For example, structures I and II for benzene or IIT and IV for allylic carbocation contribute equally.

O+— én, Con cn We oA — cH ci


I II Il IV

Benzene Allylic carbocation

(it) Structures with more number of covalent bonds contribute more towards the resonance hybrid. For
example, 1,3-butadiene may be represented by the following structures :

on “c_cn-lch, ~——> CH,—CH=CH—CH, > CH,—-CH==CH—CH,


I II Il
Structure I makes more contribution towards resonance hybrid because it has two m-bonds (11 covalent
bonds) than structures IT and IT] which have only one z-bond each (10 covalent bonds).
(ut) The structures with negative charge on the more electronegative atom and the positive charge on less
electronegative atom are of lower energy and hence contribute more towards the resonance hybrid. For example,
ketones may be represented as :
RN (x oe 9 oo Pe Ae a
oe en C—O: — P om
R R~ R:
I II III
In these structures, the structure III contributes least because it has positive charge on the more
electronegative oxygen atom and a negative charge on less electronegative carbon atom. Similarly, among the
structures : #
R—CH R <—> R—CH==C—R

CS: OF
I II
both have same number of covalent bonds and make significant contribution. However, structure II has more
contribution because negative charge is on more electronegative O atom.
(tv) Structures which involve separation of positive and negative charges are of higher energy and contribute
less towards resonance hybrid. For example, in case of 1, 3-butadiene, structures IT and III contribute less than
structure I because these involve separation of charges.

[Mas «——> CH, -CH=CH—CH, «——> CH, -CH=CH—CH,


CH,—CH—CH==CH,
- _ - +

I I Il
ORGANIC CHEMISTRY: BASIC PRINCIPLES AND TECHNIQUES 12/77 >=

Similarly for carboxylic acids, structure IJ involves separation of charges and hence contributes much less
than structure I.
rd :Or
fat ait
R—C©O—H <+——> R—C=0O—H
I II
Carboxylic acid (less stable)
(more stable)
(v) Structures having like charges on adjacent atoms are highly unstable and hence contribute less towards
resonance hybrid. For example, structure I] makes negligible contribution towards the resonance hybrid of
diketone.

RL +> RO—CR
I
1, 2-Diketone (negligible contribution)
(vt) Structures in which the positive charge is delocalised over two or more atoms make important contribution
towards resonance hybrid. For example, the structure (II) makes significant contribution towards the resonance
hybrid of the carbocation (I) because it helps to disperse the positive charge.
+f. re /t
ye +> we (ae

CH, CH,
I II
It may be noted that though structure II has positive charge on electronegative atom, yet its contribution
is significant towards the resonance hybrid because it helps in dispersal of charge.
(vit) Resonance structures in which each atom has complete octet of electrons are more stable and contribute
more to resonance hybrid. For example,

ROC=0: <— R—C= O:


I II
(Less stable) (More stable)
In structure II, the octet of all atoms is complete and therefore, it contributes more towards resonance
hybrid. Similarly, in the following carbocation, structure II makes significant contribution because in it every
atom (except hydrogen) has a complete octet of electrons.
CH, CH,
CH, Ne Oe CH, <«———> CH_c_b_ CH,
ANY H, a4 CH.

I i
(Less stable) (More stable)
Resonance Effect or Mesomeric Effect
Resonance effect is the polarity produced in the molecule by the interaction of two m-bonds or between a
m-bond and a lone pair of electrons present on an adjacent atom. This effect is transmitted through the chain.
This is very significant in case of conjugated systems (having alternate o and z-bonds). The electrons can flow
from one part of the system to the other due to resonance.
There are two types of resonance or mesomeric effect designated as R or M effect :
(i) Positive resonance effect (+ R effect)
(it) Negative resonance effect (—R effect)
(t) Positive resonance effect (+ R effect)
If a substituent has tendency to donate electrons to the double bond or conjugated system, the effect is
called positive resonance effect or + R effect. For example, groups such as
—OH, —OR, A —NHR, —NR,, —Cl, —Br, —l, etc, show + R effect.

e.g., CH,
YY= cut 3: ~—> : CH,—CH=CI:
ee 12/78 MODERN'S abc + OF CHEMISTRY-XI
In this effect, the transfer of electrons is away from an atom or substituent group attached to the conjugated
system. In case of benzene ring, this results into certain positions of high electron densities in the ring. For
example, this effect is shown for aniline :
NH, CNH, ‘NH, ‘NH, ‘NH,
Se —> cS +—> : | ee
{3
Similarly, for alkoxy (-OCH,) group :

OCH, :OCH,

(it) Negative resonance effect (-R effect)


If a substituent has tendency to withdraw electrons from a double bond or a conjugated system towards
itself, the effect is called negative resonance effect or —R effect. For example, groups such as
|
—C=O, —CHO, —CN, —NO,, —COOR, etc, show —R effect.

eg., cu,-Lom olFe5 CH,CH=C=N:


In this effect, the transfer of electrons is towards the atom or substituent group attached to the conjugated
system. In case of benzene ring, this effect results into certain positions of low electron densities in the ring. For
example, this effect is shown by -nitro group in benzene as :

Os mes
by Qs *.-

Withdrawal of electrons from a benzene ring by resonance


Differences between Inductive effect and Resonance effect

This effect involves displacement of o-electrons. The resonance involves displacement of


t-electrons or lone pair of electrons.
It operates in saturated compounds. It operates only in unsaturated conjugated
systems.
This effect moves upto three carbon atoms and becomes | This effect moves all along the length of the
negligible from the fourth carbon atom onwards. conjugated system.
This effect causes slight drift of c-electrons towards the | This results in complete transfer of electrons
more electronegative atom and hence only partial charges | and hence full +ve and —ve charges are
(6+ and 6—) are developed. developed.
The effect of these will be discussed later on.
4, Hyperconjugation
When a H —C bond is attached to an unsaturated system such as double bond or a benzene ring, the sigma
(c) electrons of the H — C bond interact or enter into partial conjugation with the unsaturated system.
The interactions between the electrons of x systems (multiple bonds) and the adjacent o bonds
(single H-C bonds) of the substituent groups in organic compounds is called hyperconjugation.
ORGANIC CHEMISTRY: BASIC PRINCIPLES AND TECHNIQUES 12/79 =>

The concept of hyperconjugation was developed by Baker and Nathan and is also known as Baker and
Nathan effect.
According to this concept, if an alkyl group carrying at least one hydrogen atom is attached to an
unsaturated carbon atom, it releases electrons of carbon - hydrogen single bond towards the multiple bond. For
example, the hyperconjugation in propene is shown below :

i 7
se hea diet — aah ubeilinias <> Ht C=CH-CH, «>»H-C=CH-CH,
i i
H H H H*
I MT Ii] IV
The various hyperconjugation forms of propene are called contributing structures or hyperconjugative
structures. In fact hyperconjugation effect is similar to resonance effect. Since there is no bond between the
a-carbon atom and one of the hydrogen atoms, the hyperconjugation is also called no-bond resonance.
However, it may be noted that although a free proton has been shown in the above structures, tt is still bound quite
firmly to the n-cloud and hence is not free to move.
It is clear from the above structures that hyperconjugation occurs through the H-atoms present on the
carbon atom next to the double bond is a-hydrogen atoms. Therefore, more the number of o-hydrogen atoms,
more are the number of hyperconjugative structures. Thus, hyper conjugation effect is expected to be more due
to CH, — (methyl) group than due to CH, CH, — (ethyl) group because the latter has only two H atoms attached
to the @ - carbon atom. In a similar manner, the hyperconjugation effect will be less if (CH,), CH — (iso-propyl)
group is attached to unsaturated system and there will be no hyperconjugation effect if (CH,).C — (tert-butyl)
eroup is attached to the a-carbon of unsaturated system.
Thus, the order of hyperconjugation effect decreases in the order is :
CH,- > CH,CH,- > (CH,), CH- >(CH,),C-
Orbital concept of hyperconjugation
The orbital concept of hyperconjugation may be explained with the help of propene. In this, the electron pair
of C — H bond (c - bond) is involved in conjugation with the z-electron pair of the double bond. Therefore,
hyperconjugation involves delocalisation of c-electrons of H—C bond through overlapping of p-orbitals of double
bond as shown below:

eo UNSATURATED SYSTEM

Orbital representation of c- m hyperconjugation


Hyperconjugation is also possible in organic compounds in positively charged carbon atom which has an
empty p-orbital. To understand this, let us consider CH,CH,* (ethyl carboation). One of the C—H bonds of the
methyl group can align in plane of the empty p-orbital and the electrons constituting the C—H bond in plane
with this p-orbital can then be delocalised into the empty p-orbital. Therefore, in this case, hyperconjugation arises
due to the partial overlap of a sp®-s (a C—H bond) with the empty p-orbital of an adjacent positively charged
carbon atom.
Hyperconjugation

Pas
—_——_— oo 7
=
&,
\

<I Empty 2p-orbital


— 12/80 MODERN'S abe + OF CHEMISTRY-XI
The hyperconjugation in ethyl carbocation may be represented as :
H H H* H H H H H
x. | | | | | |
H—C ra «—» H-—cC=C «> H*C=C ¢€ > an
| ||
H H " H H H H’ H
This type of overlap stabilises the cation because the electron density from the adjacent
co bond helps in dispersing the positive charge. Thus, in general, larger the number of alkyl groups attached to
the positively charged carbon atom, greater is the hyperconjugation interaction and consequently greater the
stabilisation of the cation. Thus, the stability of the carbocation decreases as :
(CH. ).Ct > (CH), CH. >CHLCH.* > CH
Applications of Hyperconjugation
The concept of hyperconjugation is very useful in explaining stabilities of some organic molecules.
({) Shortening of carbon-carbon single bonds adjacent to multiple bonds. In propene, the carbon-
carbon bond adjacent to double bond acquires some double bond character and hence is little shorter.
H H* H H
mal 2 e — . £f | a
ada al H, <> a ne <> H*C=CH-CH, <> H-C=CH-CH,

H H H H*
Due to hyperconjugation, the bond length of the carbon-carbon bond between C-2 and C-3 in propene is
149 pm, which is less than the normal carbon-carbon bond length (154 pm) in propane.
Similarly, C, —C, single bond adjacent to a triple bond, for example in acetonitrile, is shorter than normal bond.
H H*

“cheLX. — H-C+=-C- N: £4

H H
(it) Relative stability of methylated alkenes. Larger the number of methyl groups linked to carbon-
carbon double bond, more is the number of hyperconjugative C — H bonds and greater is the stability of alkene.
Thus, the stability of alkenes is :
CH A JERS CH, Ju |CH; Pel CH, a At
&aQ > aN > fea > pe = on

CH, CH, CH, H CH, H H H


I II ITT IV
2,3-Dimethylbut-2-ene 2- Methylbut-2-ene 2 - Methylpropene Propene
This order of stability is because of greater number of contributing structures, causing larger delocalisation
and hence stability of alkene. For example, alkene - I has 12 a - hydrogens while alkene-IJ has nine « - hydrogen
atoms. Therefore, there will be larger ee in alkene- I and will be more stable than alkene-II.

FUNDAMENTAL CONCEPTS IN ORGANIC REACION MECHANISM


In an organic reaction, the organic molecule (known as substrate) reacts with an attacking reagent and
leads to the formation of one or more intermediates and finally the products. The general raction may be
represented as follows : + Product (s)

Organic molecule EE, [Intermediate]


(Substrate) slat kta oes By product (s)
The substrate is that reactant which provides carbon to the new bond and the other reactant is
called reagent. However, if both the reactants supply carbon to the new bond then choice of substrate is arbitrary
and in that case, the molecule which we are interested to study is the substrate.
During organic reactions, a covalent bond between two carbon atoms or a carbon atom and some other atom
is broken and a new bond is formed.
The detatled study of stepwise description of a reaction, electron movement, energetics during
bond breaking and bond formation and the details of time required for the steps, when a reactant
is transformed into the product ts called reaction mechanism.
ORGANIC CHEMISTRY: BASIC PRINCIPLES AND TECHNIQUES 12/81 >

The knowledge of reaction mechanism is very important because it helps to understand the reactivity of
organic compounds.
TYPES OF BOND FISSION
Organic reactions usually involve making and breaking of covalent bonds. The fission of bonds can take place
in two ways:
1. Homolytic fission 2. Heterolytic fission
1. Homolytic fission
In homolytic fission, the cleavage of covalent bond between two atoms takes place in such a way that each
bonded atom retains one electron of the shared pair. This is a symmetrical fission and leads to the formation of
neutral species (atoms or groups of atoms) having unpaired electrons. These species are called free radicals.
The free radicals are denoted by putting dot over the symbol of atom or group of atoms. The single electron
movement is shown by “half headed” curved arrows (—). For example,

A-|)-B “fain 7? A+B


Homolytic * .

or ALB ——> A+ B
Free radicals

O O O
| | ll A
e.g., C,H.— ~ofo—c_c, __Heat/light | 2C,H.—C—-O —> 2C,H,. + 2CO,
Free radicals
2. Heterolytic fission
Heterolytic fission is unsymmetrical. In this, the bond breaks in such a way that one of the fragments takes
both the electrons of the shared pair leaving none on the other. This results into two charged particles. One atom
has a sextet of electrons and therefore, has a positive charge while other will have a valence octet with at least
one lone pair and therefore, has a negative charge. The species that has a sextet at the carbon and is positively
charged is called a carbocation (earlier called carbonium ion). The species that has a carbon atom with the
shared pair of electrons and carrying a negative charge is called carbanion. For example,
At+:B- (when B is more electronegative than A)
A: <
A> + Bt (when A is more electronegative than B)
The movement of electrons in organic reactions may be described by curved arrow notation. This shows the
changes in bonding due to electronic redistribution during the reaction. The arrow starts from that point from where
the electron pair is shifted and ends at a location to which electron pair moves. The following notations are used :
— Change in position of a pair of electrons is shown by a curved arrow starting from the electron pair which participates
in the reaction.
— Ifthe electrons being shifted belong to a bond in the reactant then the curved arrow is drawn starting from the
centre of the line representing the bond.
For example,
@ Shifting of electron pair from a z-bond to adjacent bond position:
Ly AR /_ x=
@ Shifting of electron pair from a z-bond to adjacent atom:
fem ff , —x—
@ Shifting of electron pair from atom to adjacent bond position:
2
—xX + —> —xX=
— The nonbonding electrons are shown by pair of dots on the atom.
— The movement of single electron is shown with a single barbed ‘fish hooks’ i.e., half headed curved arrow (—=).
In general, a curved arrow always means that the electron or electrons move from the atom at the tail of
the arrow to the atom at the head of the arrow.
For example, the reaction of hydroxide ion with ethyl bromide forming ethanol may be shown as :

HO: + CH,—Br: ——> CH,OH + :Br:


Similarly, dissociation of chloromethane may be depicted as :

cH, cr ——: 8,24


— 12/82 MODERN'S abe + OF CHEMISTRY-XI
Reaction Intermediates
The species produced during cleavage of bonds are called reaction intermediates. The important reaction
intermediates are :
1, Free radical. A free radical may be defined as an atom or group of atoms having an unpaired electron.
These are produced during the homolytic fission of a covalent bond.
Homolytic = cd
A ‘ B Fasion A + B

Free radicals
These free radicals are very reactive. This is because of the fact that they have strong tendency to pair up
their unpaired electron with another electron from wherever available. These are very short lived and occur only
as reaction intermediates during reactions. For example, dissociation of chlorine gas in the presence of ultra-
violet light produces chlorine free radicals .
cl: cl ——UY.____,, +01
ic Chlorine free radicals
4 _t hv ~
oe Cl Cl homolytic cleavage Cl+Cl
The alkyl free radical may be obtained when free radical chlorine attacks methane.

H-C:H+
i Cl ——~> H-C:
i + HCl

H H
Methyl free radical
The free radicals may be classified as primary, secondary or tertiary depending upon whether one, two or
three carbon atoms are attached to the carbon atom carrying the odd electron :

H— 7
r CHa
i CH,
r: HC—P
T
H H H CH,
Methyl Primary (1°) Secondary (2°) Tertiary (3°)
Stability of free radicals : The order of stability of alkyl free radicals is :
CH,< 1°< 2° < 3°
This order of stability can easily be explained on the basis of hyperconjugation. Larger the number of alkyl groups
attached to the carbon atom carrying the odd electron, greater is the delocalisation of the odd electron and hence more
stable is the free radical. Thus, the tertiary free radical with three alkyl groups attached to the carbon atom carrying
the odd electron is more stable than the secondary free radical containing two alkyl groups and so on.
CH, CH,
CHC >. CH,—CH > CH,—CH, > CH,
tert-Butyl (3°) Isopropyl (2°) Ethyl (1°) Methyl
In contrast, allyl and benzyl free radicals are stabilized by resonance effect. For example,

cH,&cHtu, ———>» CH,—CH=CH,


Allyl free radical stabilized by resonance

S—- GD B—d
°C H, & H, .
C C CH

Ls

Benzyl free radical stabilized by resonance Unhybridised orbital


Structure of alkyl free radical. The carbon atom in alkyl free radicals #77>¢ containing odd
involves sp? hybridisation. Therefore, it has a planar structure. The three , °; electron
hybrid orbitals are used in the formation of three o bonds with three H atoms oy
or alkyl group. The unpaired electron is present in unhybridised p-orbital. ‘a\
2. Carbocation. It is defined as a group of atoms which contain i \ gp Hybridised
positively charged carbon atom having only six electrons (sextet of et carbon
electrons) in tts valence shell. Earlier carbocation was called carbonium ion. Planar structure of alkyl
It is obtained by heterolytic fission of a covalent bond involving carbon atom. free radical
ORGANIC CHEMISTRY: BASIC PRINCIPLES AND TECHNIQUES 12/83 a

| | :
oF x — > -—CO + :X:
|
rx Carbocation
e.g., H,C—Br — > 4H,C* +Br
Methyl
carbocation

The carbocations are also classified as primary (1°), secondary (2°) or tertiary (8°) depending upon whether
one, two or three carbon atoms are attached to carbon bearing the positive charge as :

ave
i ne
i ne
i my,

H H H R
Methyl Primary Secondary Tertiary
Relative stability of carbocation
We know that methyl group has + I inductive effect i.e. electron releasing. The alkyl group attached to +vely
charged carbon atom tends to release electrons towards carbon. As a result, it decreases the +ve charge on the
carbon atom but itself becomes somewhat positive. As a result, the positive charge on the carbon atom gets
dispersed, The dispersal of charge results into stability. Therefore, more the number of alkyl groups, the greater
will be the dispersal of charge and therefore, more stable will be the carbocation.
Thus, tertiary carbocation in which there are three alkyl groups attached to positive carbon is more stable
than a secondary carbocation with two alkyl groups. Similarly, a secondary carbocation is more stable than a
primary carbocation which in turn is more stable than methyl carbocation :
R R H| H| 7
Three alkyl | q

eae ee R>C® > R>C® 5 R>+CO® > H—co ‘(/igpersel


ae
of charge i | | | of ch arge
R H H H
- =. iha CH
Thus, the order of stability is : °
3° > 2° > 1° > CH,*
We also come across other carbocations. For example, benzyl carbocation is primary in nature but it is highly
stable because of resonance stabilization.
*CH, CH, CH, CH, *CH,
Benzyl carbocation <_—> <-> <—> <—->
+

Benzyl carbocation is stabilized by resonance


Thus, more the number of phenyl groups, greater is the stability:
(C,H,),C* > (C,H,), CH* > C,H. CH,”
Triphenyl carbocation, Ph,C* is most stable because of resonance stabilization among three phenyl
groups.
It may be noted that the presence of electron donating groups such as — CH,, - OCH,, —OH, etc. in the
benzene ring tend to disperse the +ve charge and hence increases the stability of the carbocation. On the
otherhand, the presence of electron withdrawing groups such as — NO,, -—CN, -COOH, —COOR, -— Cl, etc.
intensify the positive charge and hence decreases the stability of the carbocation.
Similarly, carbocations in which the + vely charged carbon atom is attached to a double bond are stabilized
by resonance, For example, allyl carbocation is stabilized by resonance as :
+ + Empty unhybridised p-orbital
CH, =CH*CH, ——-> CH,—CH=>=CH, eS
Hy ;
Allyl carbocation is stabilized by resonance
=k 7+
Structure of carbocation. The carbon atom in carbocation is sp? hybridised. 'e
The three sp? hybrid orbitals form three o bonds with hydrogen or other carbon f Te? Tea AEE eg
atoms of alkyl groups. The unhybridised p-orbital of carbon remains vacant. ay, ean
Since it involves sp? hybridisation, its structure is trigonal planar with a bond Ge hiat sori ckneek eacesehion
angle of 120°. |
— 12/84 MODERN'S abc + OF CHEMISTRY-XI
Relative stability of carbocations due to hyperconjugation effect
The relative stability of carbocations and free radicals follows the sequence :
tertary (3°) > secondary (2°) > primary (1°) > methyl
This order of stabilty can be easily explained on the basis of hyperconjugation. In case of tertiary butyl
carbocation (3°), there are nine a-hydrogen atoms and hence nine hyperconjugation structures are possible :

:
Htc <-> ie —C—CH,
i
.< 5 y+
:|
(a es <—_>
\
|
YF“. C—CH,
H CH; H CH, H CH, H* dn,
+ six more such structures from the other two methyl groups.
On the other hand, in isopropyl carbocation (2°), there are six o-hydrogen atoms and hence six
hyperconjugation structures are possible :

H™ H H

SOL <— H—C=C—CH, <—~> H” := C—CH, <—> u—¢ —C—CH,


| | | |
H H H H H sa
+ three more such structures from the other methyl groups.
In a similar way, ethyl carbocation (1°) has only there o-hydrogen atoms and hence only three
hyperconjugation structures may be written as :

Pe a aan
H H H H

H— ie a — H : wae H—C=—C—H
| ar fall
H H H 4H H H HH GH
However, no hyperconjugation structure can be written for methyl, CH,* carbocation. Thus, in general,
greater the number of alkyl groups attached to a positively charged carbon atom, the greater is the
hyperconjugation interaction and stabilisation of the cation. Thus, the order of stability of carbocations is
CH, CH,
| |
Cry “Nn > /CH,;,—CHt+ > CH,CH, > CHg
CH,
Similarly, we can explain the stability of free radicals as :
(CH,),C > (CH,),CH ame /> CH,
In general, the stability of various carbocations decreases in the order :
(C,H,),C* > (C,H,),CH” > (CH,),C* > C,H.CH,* > (CH,),CH* > CH,—CH—CH,,* > RC’=CH, > CH,CH,*
> RCH=CH* > C,H.* > CH,t > HC=C*
3. Carbanion. A carbanion may be defined as a species containing a carbon atom carrying a
negative charge and possessing eight electrons in its valence shell. These are generated by the heterolytic
fission of covalent bond involving carbon atom in which the atom linked to carbon goes without the bonding
electrons. As aresult of this, carbon acquires a negative charge. When group Z attached to the carbon atom leaves
without electron pair, the methyl carbanion is formed.
+

H,C—Z —+> H.C: + Z


Methyl carbanion
For example, removal of hydrogen of methyl part of acetaldehyde by OH ions (base) leaving both the
electrons on carbon :
|H H

OH + Hi: C—CHO ——> : ‘ee + H,O


I

H H
Acetaldehyde Acetaldehyde
carbanion
ORGANIC CHEMISTRY: BASIC PRINCIPLES AND TECHNIQUES 12/85 =>

Carbanions are also very reactive species.


Like carbocations, carbanions are also classified as primary (1°), secondary (2°) and tertiary (3°) depending
upon whether one, two or three carbon atoms are attached to the carbon atom bearing negative charge as :

He
| R—
i
a Ro
i ane
i
H H H R
Methyl Primary Secondary Tertiary
The order of stability of carbanions is reverse of that of carbocations and free radicals.
Ch > les es
This order of stability can be explained on the basis of +1, inductive effect of the alkyl group. Alkyl groups have
electron releasing tendency and therefore, increase the electron density on the negatively charged carbon atom
and hence make it unstable. Larger the number of alkyl groups attached to the negatively charged carbon atom,
greater will be the electron density on the carbon atom and lower will be its stability.
It may be noted that allyl and benzyl carbanions are stabilized by resonance as :

cf, CH ¥oh, <—> CH,—CH=CH,


Allyl carbanion is stabilized by resonance

7?CH, CH, CH, CH, -3CH,


<< x cs
[> <—> Pox) ae 4s
o-
Benzyl carbanion is stabilized by resonance
Thus, more the number of phenyl groups, greater is the stability.
(C.H.).C > A@fH,), Gf > C,H.-CH,
Further the presence of electron withdrawing groups such as —NO,, -CN, —Cl, -COOH, —COOR, etc. in the
benzene ring tend to disperse the —-ve charge and hence increases the stability of the carbanion. On the other hand,
the presence of electron donating groups such as -CH,, -OCH,, etc. tend to intensify the negative charge and
hence decreases the stability of carbanion.
Structure of carbanion. The negatively charged carbon —~ __ §p* Hybrid orbital
atom in carbanion is sp? hybridised. Therefore, it has a tetrahe- containing lone pair
dral structure. Three of the four sp® hybridised orbitals form 3c “
bonds with hydrogen or carbon atom of the alkyl group. The fourth Pa YN
sp® hybrid orbital contains the lone pair of electrons.
Pyramidal structure of carbanion
Thus, we may sum up

Homolytic . : a
fission — +Z sp” hybridisation of carbon

Free radicals
(Stability 3° > 2° > 1° > methyl)
|
— , + :Z7 sp* hybridisation of carbon

Carbocation
(Stability : 3° > 2° > 1° > CH,*)
fission

eee sp® hybridisation of carbon

Carbanion
(Stability : CH, > 1° > 2° > 3°)
ORGANIC CHEMISTRY: BASIC PRINCIPLES AND TECHNIQUES 12/86 ==

4, Carbenes. The carbenes are reactive neutral species in which the carbon atom has six electrons in the
valence shell out of which two are shared. The simplest carbene is methylene (CH,). It is formed when
diazomethane is decomposed by the action of light.
CH, N, __light_, : CH, + Ny,
Diazomethane Methylene carbene
This is formed as:

1.3%
== -= NI: ee H. a
CHy5 : CH, +N,
Carbene

It is very reactive. It reacts with alkenes by adding to the double bond forming cyclopropane.
me “ ~ wo

Alkene Methylene CH,


Cyclopropane
Similarly, when chloroform is treated with sodium ethoxide, dichlorocarbene is formed.
65 (6)Pama laa : CCl,
Dichlorocarbene

Nitrenes
Nitrenes are the nitrogen analogues of carbenes. These are electron deficient monovalent nitrogen species.
In these, nitrogen has a sextet of electrons. The parent compound is N—H (nitrene or azine) and is formed when
hydrazoic acid is treated with UV light.
NH —i
Av
/HN:
Hydrazoic acid _. Nitrene
Thermal or photolytic decomposition of azides give nitrenes.
hh
RN; —.ar” RYN +N,
Allyl nitrene
+ _— —"" | + - ;
or 5 5LSL5 —— 7 re /ty R-N: + Nz,
Alkyl azide Alkyl nitrene
This is the most common method for the generation of nitrenes.
These are very reactive species and cannot be isolated. However, nitrenes can be trapped in reactions.
HN+CH,=—CH, —~ as CH,

|
H
TYPES OF ATTACKING REAGENTS
The attacking reagents are classified into three types :
(i) Free radicals (it) Electrophiles (iit) Nucleophiles
1. Free radicals have already been discussed.
2. Electrophiles. A reagent that takes away an electron pair is called electrophile. These are positively
charged or neutral species which are deficient of electrons and can accept a patr of electrons. These are also called
electron loving (philic) or electron seeking species (KE). These may be positively charged species or neutral
molecules. For example,
H*, H,0*, Cl*, Br*, *, NO* (nitronium ion), R*(carbocation) (Positively charged)
AICI,, BF, SO, (Neutral)
Both Al and B have total of six electrons i.e., two less than the octet. Therefore, these try to complete their
octets and act as electrophiles. According to Lewis concept of acids and bases, these are called Lewis acids.

It may be noted that all positively charged species donot act as electrophiles. Only positively charged ions
which can accept a pair of electrons can act as electrophiles. For example, Nat, Ca?*, NH} etc. cannot act as
electrophiles because all have an octet of electrons in their valence shells.
ORGANIC CHEMISTRY: BASIC PRINCIPLES AND TECHNIQUES 12/87 >=

3. Nucleophiles. A reagent that brings an electron pair is called nucleophile. These reagents contain an
atom having unshared or lone pair of electrons. A nucleophile is electron rich and seeks electron deficient sites,
L.e., nucleus loving or nucleus seeking (Nu). According to Lewis concept of acids and bases, nucleophiles behave
as Lewis bases. These may be negatively charged species or neutral molecules. For example,

H (hydride ion), X ,OH , CN , R (carbanion), RC=C RCOO ,OR ,SR,NH, (Negatively charged)
NH,, RNH,,R,N, H,O:, ROH, RSR, ROR (Neutral molecules)
During a polar organic reaction, a nucleophile attacks an electrophilic centre which is a specific atom or
part of electrophile which is electron deficient. Similarly, the electrophile attacks a nucleophilic centre, which is
an electron rich specific atom or part of the nucleophile. As a result, the electrophiles get electron pair from the
nucleophiles and the two form a bond. A curved-arrow notation is used to show the movement of electrons from
the nucleophile to the electrophile. For example,

E
— “
= »~Nu —_— KE —Nu
Electrophile Nucleophile Bond
It may be noted that in neutral molecules such as alkyl halides, the carbon is bonded to electronegative
halogen atom (X). Due to the polarity of the C—X bond, carbon acquires a partial positive charge. As a result, it
becomes electron deficient or electrophilic centre at which nucleophile can readily attack.
o+(7 _
=k + -Nu — > H,C—Nu+X

Differences between Electrophiles and Nucleophiles

fel Electrophiles Nucleophiles


These are deficient in electrons. These are rich in electrons.

These accept a pair of electrons from the substrate. These donate a pair of electrons to the
substrate.
These behave as Lewis acids. These behave as Lewis bases.
These have at least one empty orbital due to which These have at least one lone pair of electrons
they accept electrons from the substrate molecule. which can be easily donated to the substrate
molecule.
These are either neutral or positively charged These are neutral or negatively charged
chemical species. chemical species.

TYPES OF ORGANIC REACTIONS


Organic reactions can be organised broadly into two ways : by what kinds of reactions occur and by how
reactions occur. These can be broadly classified into the following general types on the basis of kinds of reactions
that take place.
1. Substitution reactions
The reactions in which an atom or group of atoms in a molecule is replaced or substituted by different atoms
or group of atoms are called substitution reactions.For example, in the reaction between bromomethane with
sodium hydroxide, bromine atom is replaced by hydroxide group.

CH,Br + NaOH —> CH,OH + NaBr


In this reaction, a nucleophile (OH-) substitutes the bromide in the substrate and hence this reaction is
also called nucleophilic substitution reaction. Similarly, the substitution may take place by an electrophile
or a free radical resulting electrophilic or free radical substitution reactions. For example, nitration of benzene
is an example of electrophilic substitution. In this case, nitronium ion NO,* acts as an electrophile. The
halogenation of alkanes (e.g. methane) is an example of free radical substitution, in which free radical Cl
attacks the methane molecule.
— 12/88 MODERN'S abe + OF CHEMISTRY-XI
NO,

(6) CH,+Cl,—— CH,Cl+ HCl (His replaced by Cl)

2. Addition reactions
The organic reactions in which two molecules react to form a single product having all the atoms of the
combining units are called addition reactions. For example,
H H H H

H—C=C—H-+ Br,——_~+ H—C—C—H

Br Br
Addition product
Br H
|
CH,CH,CH = CH, + HBr —> CH,CH,CH—CH,

CH,—C=0
i + HCN ——_+ a nny a
[
CN
Addition product

Like substitution reactions, the addition reactions are also called free radical, electrophilic or nucleophilic
addition reactions depending upon the type of attacking reagent.
3. Elimination reactions
The reactions in which two atoms or groups of the molecule are removed are called elimination reactions.
For example,

1H Br}
| alc. KOH
H—C—C—H —W\—+ CH, =CH, (H and Br are eliminated)
Ethene
H H

It may be noted that in these reactions, the two atoms or groups of atoms may be removed from the same
or adjacent atoms of the substance. Depending upon the relative positions of the atoms or groups eliminated,
these reactions are classified as a (alpha), B (beta) or y (gamma) elimination reactions.
(t) a-Elimination. Jn a-elimination reaction, the loss of two atoms or groups occurs from the same atom
(called a-positions) of the substance. This elimination results into electron deficient reactive intermediates,
which further react to form stable products. For example, formation of carbene is an example of a-elimination
reaction.

R,C—C]l —=—> R,C—cl _-"_, Rc:


| + ca” =

Carbene

(it) B-Elimination. In fB-elimination reaction, the loss of two atoms or groups occurs from the adjacent
positions (a, B) of the substance. Elimination of HBr from 1-bromobutane in the presence of alcoholic KOH
civing but-1-ene is an example of B-elimination reaction.
H Br
[B [2 KOH (ale.)
CH,—CH,—_CH—CH, ——_—_> CH,CH,CH = CH, + HBr
But-l-ene

WWW.JEEBOOKS.IN
ORGANIC CHEMISTRY: BASIC PRINCIPLES AND TECHNIQUES 12/89 >

(uit) y-Elimination. Jn y -elimination reaction, the loss of two atoms or groups occurs from three bonds
away (next to adjacent) i.e., a and y-positions. The y-elimination results in the formation of a three membered
ring as shown below :
\ 7
\ B/ C
il
x Si ae >
/\ . + ZnBr,

Br Br Cyclopropane

4, Rearrangement reactions
These reactions involve the migration of atoms or group of atoms to another position within the molecule under
suitable conditions. For example,
AICL
CH,CH,CH,CH, “Rearrangement” CH,—CH—CH,
But:
n-Butane |
|CH,

Isobutane
Similarly, but-1-ene rearranges to but-2-ene by rearrangement in the presence of acid catalyst.
H.C = CHCH,CH, ee EERE a CH.CH = CHCH,
But -1- ene But-2-ene

You will learn in next chapters, a typical rearrangement reaction of migration of -CH, group. Under acidic
conditions, 1,2-migration of -CH, group in 2, 2-dimethylpropan-1-ol gives rearranged product 2-methyl
but-2-ene as shown below :
CH, CH, H CH 3
| | | |
ssste beast
2 Bes 5 CH,—C—CH,—O-H
i af =——2°.> CH,—C—CH,"
CH, CH, CH,
Carbocation
CH,H CH,
|| |
Se
nugration
5 CH,—C—C—H/_
- |
="_(CH,—C = CHCH,
CH, 2-Methylbut-2-ene

5. Condensation reactions
In these reactions, two different or same organic reactants combine with or without the elimination of a
simple molecule to give a product. For example, two molecules of acetaldehyde (CH,CHO) condense in the
presence of a base (NaOH) to form 3-hydroxybutanal. This reaction is called aldol condensation. The product

en
then loses a water molecule to form but-2-enal

CH,—C—H + CH,CHO _#a.Ne0OH_ , CH.—CH—CH,CHO


Two molecules of ethanal 3- Hydroxybutanal

OH
| ;
CH,CH—CH,CHO ———> CH,CH = CHCHO
But- 2- enal

6. Ilsomerisation reactions
The reactions which involve the interconversion of tsomers wherein the molecular formulae and the carbon-
skeletons of reactants and products always remain same are called isomerisation reactions. For example,
as 12/90 MODERN'S abc + OF CHEMISTRY-XI
the interconversion of frans-but-2-ene to cis-but-2-ene is known as geometrical isomerisation reaction.
HC. _A HjC~_ ail
—(C hv. Cc—c:

trans-But-2 ene cis-But-2-ene

SOLVED EXAMPLES
LI Example 21
Write resonance structures of the following showing the movement of electrons by curved arrows :
(i) Acetate ion, CH, ——

(iL) CH, CH =_ CH-CH,

Solution :
O-

(i) CH, oy —— CH, x?


OF SS:

ee
(i) CH,-CH=4CHS-CH, ——> |
CH,—CH—CH=CH,
L} Example 22.
Give reasons why the following two structures I and IT cannot be the major contributors to the reas structues
of CH,COOCH,.
0:7 :O2
HjC—CO-CH, —s H,C_-C==0"—CH,
ri IT
Solution : The structures I and II are less important contributors because they involve ‘charge separation’. Therefore,
they donot contribute substantially towards the resonance hybrid. Further, structure I also contains a carbon atom with an
incomplete octet (sextet of electrons) and therefore, its contribution is lower than that of structure II.
L) Example 25.
Write resonating structures for CH, = CHCHO. Indicate relative stability of the contributing structures :
Solution : The resonating structures for CH, = CHCHO are :
N.C.E.R.T.

09 :0: :0°
1 |! t | _ |
CH,=CH—C—H. —= CH,—CH=C—H —~=S—- :CH,—_CH=C—H
I II ITI
Structure I is most stable because 1t has more number of covalent bonds and each carbon and oxygen atoms has an
octet and no carbon or oxygen atom has a charge. Structure III is least stable because more electronegative O atom
has positive charge and electropositive carbon has negative charge. In structure II, there is positive charge on
electropositive C atom and negative charge on electronegtive O atom. But both structures II and III have charge
separation. Thus, decreasing order of stabilityis: I> II > III
LJ Example 24.
Draw resonance structures for the following compounds :
(i) CH,=CH — ‘=O (ii) CH=CH —Cl: (iii) CH, = CH —CH = CH,

Solution:
H

(i) CH,=C
-
i C—H <> CH, —CH =C—
fF
“s

A
(ii) CH, Yona: <—_ > CH, —CH = Cl:

(iii) on, Lonvelch, <—> CH,— CH = CH—CH,

WWW.JEEBOOKS.IN
ORGANIC CHEMISTRY: BASIC PRINCIPLES AND TECHNIQUES

L) Example 25
Classify the following molecules /ions as nucleophiles or electrophiles :
HS-, BF,, CH,CH,O,, (CH,),N, Cl*, CH,C = O, H,N’:, NO}
Solution : Electrophiles BF,, Cl’, 65 eo O, NO;
In these, the reactive sites have only six valence electrons and therefore can accept electron pairs from a
nucleophile.
Nucleophiles : Hs ,CH,CH,O , (CH,).N, HN:
These species have unshared pair of electrons which can be donated and shared with an electrophile.
LI Example 26.
Identify the electrophilic centre in the following : (i) CH gCN (it) HCI (i) CH 3CHO
Solution : The electrophilic centres are shown by starred carbon (C*) because these will acquire partial positive
charge due to the polarity of the bond :
G)H,CC’=N (ii)H,C—I (iii) CH, HC*=0
LU) Example 27
Which of the following pairs of structures do not constitute resonance structures :

(a) H,C-NZ
en and H,C-O-N=0
O
O: O:
(b) CH, -C ; and CH, -C
cH; &
() (CH,), CO and CH, - ye
CH,
(d) CH,;CH=CHCH, and CH,CHjCH=CH,
Solution :
(a) The two structures differ in the position of atoms and therefore, do not constitute resonance structures.
(b) These constitutes a pair of resonance structures.
(c) Thes are not resonance structures because these differ in the position of atoms
(d) These are not resonance structures because these differ in the position of atoms.
LU) Example 28
Using curved arrows notation, show the formation of reactive intermediates when the following covalent
bonds undergo heterolytic cleavage:
(a) CH,—SCH, (b) CH,—CN (c) CH,—Cu

a —
Solution :

(a) CH, /SCH, Sen CH, + SCH,


(b) CH, C& — wf CH, + CN
(CN is more electronegative than CH,)

(c) an Cu —_—. CH, + Cu


(Cu is more electropositive than CH,)
Li Example 29.
Classify the following reactions according to the reaction type :
(a) CH,CH,Br + HS- —> CH,CH,SH + Br-
(6) (CH,),C = CH, + HCl —-> (CH,),C(CL)—CH,
(c) CH,CH,Br + HO- —> CH, =CH,
(J) H,C=CHCH,CH, —> H,C CH = CHCH,

WWW.JEEBOOKS.IN
— 12/92 MODERN'S abc + OF CHEMISTRY-XI
(ce) CH,—C = NOH —> CH,—C =O
| |
C,H, NHC,H,
(f) (CH,), CCl + OH-—> (CH,),C = CH, + H,O + CT
(g) (CH), C—CH,OH + HBr—-> (CH,),C BrCH, CH,+ H,O
Solution :
(a) CH,CH,Br + HO” — > CH,CH,SH + Br Substitution reaction
(b) (CH,),C = CH, + HS— —» (CH,),C(Cl)—CH, Addition reaction
(c) CH,CH,Br + OH- — >» CH,=CH, Elimination reaction
(d) H,C = CHCH,CH, — H,C CH=CHCH, Isomerisation reaction

(e) a = NOH — +» CH,—C=0O Rearrangement reaction


GH NHC,H,
(f) (CH), CCl + OH- — > (CH,),C =CH,+H,O+Cl Elimination reaction.

(g) (CH,),C—CH,OH+ HBr —-+ (CH,),CBrCH,CH,+H,O | Substitution with rearrangement reaction


L} Example 30.
Classify the reagents shown in bold in the following equations as nucleophiles or electrophiles :

(a) CH,COOH + HO —> CH,COO + H,0


(b) CH,CH,Br+HS — > CH,.CH,SH + Br
(c) C,xH,+ CH,CO* —> C,H,COCH,

OH
|
(d) CH,COCH, +CN” —> CH,C—CH,
|
CN
Solution :
O }
(a) CH, cS +HO —-+CH, aM + H,O (6) CH,CH,Br+HS -—-+ CH,CH,SH + Br
OH a
OH is nucleophile SEH- is nucleophile
OH

(c) CH, + CH,CO* —+ C,H.COCH, (d)] CH;—C\_



+CN ah
—-+> CH, —C—CH,
CH,CO? acts as electrophile CH, CN
CN is nucleophile

CYUCSTIONS 31
Conceptual
Q.1. How many isomers are possible for monosubstituted and disubstituted benzene ?
4
Ans. There is one monosubstituted benzene as
There are three disubstituted benzenes as
xX xX xX

CO.CS
xX
ortho (1, 2-) meta (1,3-) para(1,4-)

WWW.JEEBOOKS.IN
ORGANIC CHEMISTRY: BASIC PRINCIPLES AND TECHNIQUES 12/93 ~~»

Q.2. Write bond line formula for heptan-4-one.


O

me
Q.3.
AAA
Give the IUPAC name of the alkane having the lowest molecular mass that contain a quaternary
carbon.

CH,
|
Ans. ea ee 2, 2- Dimethylpropane
CH 3

Q.4. Name the following compounds :

O _ NO,

(i) | (ii)

O CH,
, 1 2 3. 4b 5 6 7
Ans. (1) én, “81 O_ttob, on,

bsHy

3-Ethyl-4-methylhept-5-en-2-one

NO,
3

(ii) : 3-Nitrocyclohex-1-ene

Q.5. Write structural formulae for the following :


({) 1-Bromoheptane _ (ii) 5-Bromoheptanoic acid
Ans. (i) CH, -—CH,—-CH,—- CH, —CH, — CH, —CH,Br
(i) CH, -CH, ~CH— CH, — CH, —CH, —- COOH
|
Br
Q.6. Which of the following compounds shows geometrical isomerism ?
(i) Pent-1-ene (ii) 2-Methylbut-2-ene (iii) Pent-2-ene
Ans. Pent-2-ene
Q.7. Draw the complete structures of bromomethane, bromoethane, sp rOmOn EoDAne and tert-
butyl bromide. Arrange them in order of increasing steric hindrance.

Ans. (i) Bromomethane, CH.Br : oo


i
H

(ii) Bromoethane, CH,CH,Br : H—C—C—Br

|
(iit) 2-Bromopropane, CH,CHBrCH, : ca i ak fie

WWW.JEEBOOKS.IN
— 12/94 MODERN'S abc + OF CHEMISTRY-XI
H
H HOH
(iv) tert-Butyl bromide, (CH,), CBr : HO _

H
Increasing order of steric hindrance :
Bromomethane < Bromoethane < 2-Bromopropane < tert-Butyl bromide.
Q.8. What is the relationship between the members of the following pairs of structures ? Are they
identical, structural, geometrical or resonance contributors ?

O O C CHCl
AY AX (ii) \ C=C a Se ri
ms Nex, cl cl Nu

*OH OH

(tii)
LEE a
! ? Tr aeOH (iv)
LD ™~,
4 = a
N, i
a — “a

Ans. (i) Structural isomers


(ii) Identical
(iii) Resonance contributors
(iv) Geometrical isomers.
Q.9. How many tetrahedral angles does a molecule of methane have?
Ans. Six tetrahedral angles.
Q.10. Draw structures of all isomeric ethers corresponding to molecular formula C;H,,0.
Ans. The isomeric ethers for C,H,,O are :
(i) CH,CH,CH,CH,OCH, (ii) CH,CH,CH,OCH,CH,
CH,
(iii) a CH, CH, (iv) i tae
CH, CH,
Q.11. What is the hybridisation of each carbon in CH, = C = CH, ?
Ans. sp”, sp and sp?.
Q.12. Show the polarisation of carbon-magnesium bond in the following structure :

CH,CH,—CH,Mgx.
o—- OF
Ans. CH,CH,CH,—Mg—x
Q.13. What type of strurctural isomerism is shown by
CH,
CH,—S—CH,CH,CH, and CH,—S—CH fe
h
CH,
Ans. Metamerism.

WWW.JEEBOOKS.IN
ORGANIC CHEMISTRY: BASIC PRINCIPLES AND TECHNIQUES 12/95 ~~»

Q.14. rian many cyclic and acyclic isomers are possible for the molecular formula C,H,O? Draw
Lnese,

Ans. Seven isomers are possible. These are :


(a) Acyclic isomers :

|
(i) CH,CH,CHO (Gi) CH,CCH,
Propanal Propanone
(it) CH, =CHCH, OH (iv) CH,=CHOCH,
Prop-2-en-1-ol Methoxyethene
(b) Cyclic isomers :

» [9 wo LN CH 3
Oxetane Propylidene oxide

OH H
(vii) /.

Cyclopropanol
Q.15. In which C—C bond of CH,CH, CH, Br, the inductive effect is expected to be least ?
Ans. The inductive effect is least in C,—C, bond because the magnitude of inductive effect decreases as the
number of intervening bonds increases.
3 2 1
CH,—CH,—C H,—Br
Q.16. Draw the structures of
(i) Pent-—4-en-2-ol (ii) Cyclohex—2-en-1-ol
5 4 3 * 4
Ans. (i) CH,=CHCH, CH CH,
|
OH

OH
1
9
(it) 3

Q.17. Draw polygon formulae for the molecular formula C,H,,.


Ans. (i) ie (ii) aa (iit)

(iv) ; (v)

Q. 18. What is the type of hybridisation of each carbon in the following compounds :

(i) CH,CH = CHCN (ii) (CH,),C =O

WWW.JEEBOOKS.IN
ST i26 MODERN'S abe + OF CHEMISTRY-AI

sp? sp” _sp” sp


Ans. (i) CH,—CH=CH—C=N_ : sp®,sp*,sp*,sp
sp? sp?
(ii) aay =O : sp?,sp*,sp*

CH,
sp?

Q. 19. Which bond is more polar in the following pairs of molecules :

(i) H,C—H or H,C—Br (ii) H,C—NH, or H,C—OH

(iti) H,C—OH or H,C—SH


Ans. (i) C—Br because Br is more electronegative than H.

(ii) C—O because O is more electronegative than N.

(iii) C—O because O is more electronegative than 8.

Q. 20. Which is expected to be more stable :


O,N CH, CH,O- and CH,CH,O- and why ?
Ans. O,NCH,CH,O is more stable than CH,CH,O because —NO, group has —I inductive effect and tends to
disperse the —ve charge on the O-atom. This results into stability. However, CH,CH, — group has + I
effect and tends to intensify the —ve charge and therefore, destabilizes it.
Q.21. Write structures of various carbocations that can be obtained from 2-methylbutane. Arrange
these carbocations in order of increasing stability.
Ans. a
CH,

The possible carbocations are :

TR a CH, CH,CH—CH CH, CH, CCH, CH CH,—CHCH, CH,


CH, CH, CH, CH,
I(1*) IT (2°) ITT (3°) I'v (1°)

Order of increasing stability I < IV < II < III.


Q.22. Identity the most stable species in the following sets of ions giving reasons :

(a) CH, , CH,Cl , CHCl, , CCl,


. + + + +
(6) CH, , CH,Br , CHBr, , CBr,

Ans. (a) Ofc is most stable because Cl is more electronegative than hydrogen. On replacing hydrogen by
chlorine, negative charge on C is reduced and the species becomes stable.

(b) CH, is most stable. The replacement of H by Br increases positive charge on carbon atom because Br
is more electronegative than H and consequently the species becomes less stable.
Q.23. Which of the following resonance structure for propenal is more stable ?
+ a
Ans. CH, = CH - CH = 0 <—> CH, -CH = CH-O
(A) (B)

Ais more stable than B.

WWW.JEEBOOKS.IN
ORGANIC CHEMISTRY: BASIC PRINCIPLES AND TECHNIQUES 12/97 SS

Q. 24. Draw resonance structures for the following


(i) C,H,OH (ii) C,H,CHO (iii) C,H,—CH,
:0H +OH +OH

Ans_ (i) CT O

tf if ‘Orae HO 0 HOR VB
C_H C—H Ai io fe
ii) on L+ P

List the following carbocations in the order of decreasing stability :


CH, CHCH,CH3, CH, CH CH,CH3» CH,CH, CHCH3
l H, l

CH,;CHCH,CH, > CH,CHCH,CH3 > CH,CH, CHCH}


Hs ] 1
Explain why (CH,),C* is more stable than CH,CH,* and CH,* is the least stable cation.
Hyperconjugation interaction in carbocations depends upon the number of o-hydrogen atoms. It is greater in
(CH,).C* than in (CH,),CH* because (CH,),C* has nine C—H bonds while (CH,),CH* has only six C—H bonds.
Greater hyperconjugation means greater stability of the cation and hence (CH,),C* is more stable than (CH,),CH".
In CH,*, the vacant p-orbital is perpendicular to the plane in which C—H bonds lie and hence there is no
possibility of overlapping. Thus CH,” lacks hyperconjugation stability and is the least stable carbocation.
. Write resonance structures of

(t) allyl carbocation (fi) i

Ans. (i) CH,=CH—CH,* «—> CH,* —CH==CH,

(ii) || | > | - <> <>


(\ A Ny’ ©

i
N
Ni| |
H| H
H
Q.28. Write IUPAC name of the following:

HC=C—CH,—CH,—CH,
(i) (it) (tit)

Ans. (i) 5-Cyclobutylpent-1l-yne (iz) 1,1-Diethyl-4-isopropylcyclohexane


(tit) 8-Cyclopropyl-1,1-dimethylcyclohexane
Q.29. Arrange the following free radicals in decreasing order of stability :

CH,, (CH,)C, CH,CH,, ¢_\-ch,, CH,=CH—CH,, (CH,),CH


Ans. ¢_\-ch, > CH,=CH—CH, >(CH,),C > (CH,),CH > CH,CH, > CH,
WWW.JEEBOOKS.IN
cs
PURIFICATION OF ORGANIC COMPOUNDS
MODERN'S abc + OF CHEMISTRY-AXI

This process is carried out with the help of a


The organic compounds extracted from the natural filter paper which is placed in the funnel. The mixture
sources or prepared in the laboratory are generally
is dissolved in a suitable solvent in which one
component dissolves. It is then slowly allowed to fall
impure and therefore, it is necessary to first purify
on the funnel as shown in Fig. 6. The liquid containing
them and check their purity. The various methods used
soluble component passes through the filter paper and
for purification are based on the nature of the
the insoluble solid is retained on the filter paper.
compound and the impurities present in it.
For example, a mixture containing
The most common methods used for the
naphthalene and urea can be separated by this
purification of organic compounds are :
technique. Urea dissolves in water while naphthalene
1. Filtration remains insoluble. The residue on the filter paper is
. Recrystallisation naphthalene while urea is in the solution (filtrate).
Urea is recovered from the filtrate by evaporating
. Sublimation
water.
. Distillation
Sometimes the filtration is carried out in a fluted
. Fractional distillation filter paper placed in a glass funnel (Fig. 7).
. Distillation under reduced pressure The use of the fluted filter paper makes the filtration
. Steam distillation rapid.

. Differential extraction
Solution of impure
Hn
fF
WO
om
& . Chromatography.
como compound

After purifying the compounds, it is necessary to


check their purity. The most convenient and commonly 3 Uf_— Fluted filter paper
used physical properties to test the purity ofa substance
in the laboratory are melting points for solids and
boiling points for liquids. Most of the pure compounds
have sharp melting and boiling points. Nowadays, the
purity of an organic compound is determined by new
methods based on different types of chromatographic
and spectroscopic techniques.
Let us briefly learn about different methods for
purifying the organic compounds. Fig. 7. Filtration through fluted filter paper.
1. Filtration
The process of filtration its used to separate If the filtration of a hot solution is required, it is
an insoluble solid component of the mixture carried out in a hot water funnel. This is used when
from the soluble component in a given solvent. the organic compound crystallises during filtration
(Fig. 8). The jacket of the hot water funnel is heated
from outside and this keeps the solution hot in the
glass funnel. This will prevent the formation of crystals
during filtration. The insoluble and suspended
impurities remain on the filter paper and a clear
solution gets collected in the beaker placed below the
funnel. For example, a mixture of anthracene and
benzoic acid is separated by dissolving it in hot water
and filtering the hot solution. Anthracene remains as
an insoluble residue while benzoic acid goes in the
filtrate. It crystallizes on cooling.
At times, the process of filtration is very slow and
takes a long time. In such cases, the filtration 1s carried
out under reduced pressure using Buckner funnel and
a water suction pump as shown in Fig. 9. The suction
pump enables the filtration under reduced pressure
Fig. 6. Process of filtration. and is, therefore, quite fast.
ORGANIC CHEMISTRY: BASIC PRINCIPLES AND TECHNIQUES

Hot solution Solution

Fig. 8. Filtration through hot water funnel. Fig. 9. Filtration under reduced pressure.

2. Recrystallisation through a fluted filter paper placed in a glass funnel as


This is the most common method for purifying described earlier. If the organic compound crystallises
organic solids. This method is based on the differences during filtration, then a hot water funnel is employed
in the solubility of the organic compound and its for filtration (Fig. 8). The jacket of the hot water funnel
impurities in a suitable solvent. This method involves is heated from outside and this keeps the solution hot
the following steps : in the glass funnel.
(a) Preparation of the solution. The given
(c) Crystallisation. The hot filtrate is allowed to
organic substance is powdered and is dissolved in a
cool slowly and undisturbed in a beaker or in a
suitable solvent by heating. The amount of solvent
erystallising dish. After some time the crystals of the
should be just sufficient to dissolve the whole of the
solid on heating. In case of solvents with low boiling pure compound are formed. Sometimes, the sides of
points, the heating must be done carefully by using the vessel are scratched. This helps the process of
either an air condenser or water condenser, so that crystallisation.
the vapour of the solvent may not come in contact (d) Separation of crystals. The crystals formed
with the flame. (step c) are separated from the mother liquor by
Choice of solvent. The choice of solvent is very filtration. The filtration is normally done by use of
important in the crystallisation process. The main Buckner funnel and a suction pump (Fig. 9). This enables
conditions of the solvent are : the filtration under reduced pressure and is, therefore,
(i) The organic substance should be sparingly
quite fast. The crystals are washed 2-3 times with small
soluble at room temperature but appreciably
quantities of pure solvent.
soluble at higher temperature.
(ii) The solvent should crystallise on cooling. (e) Drying of crystals. The crystals are dried by
(iit) The solvent should not dissolve the impurities. pressing between the folds of filter paper and then
(iv) The solvent should not react chemically with placed in a steam or air oven for some time. The
the substance. crystals are finally dried over sulphuric acid or calcium
(v) Removal of solvent becomes easy if it is low chloride in a desiccator.
boiling liquid. (f) Removal of colour. Sometimes the crystals
The choice of a solvent is very crucial. For example, obtained are slightly coloured due to the presence of
suppose we want to purify sugar containing an some coloured impurities. In such cases, the crystals
impurity of common salt. This can be done by treating are redissolved in the same solvent and a pinch of
the mixture with ethanol around 350 K. The sugar will
animal charcoal is added. The solution is boiled and
dissolve whereas common salt remains insoluble. The
then filtered as before. The charcoal adsorbs all the
filtrate on cooling will give crystals of sugar. In this case,
coloured impurities and the crystals now obtained are
we cannot take water as the solvent because both sugar
colourless.
and common salt are soluble in water.
(6) Filtration of the solution. The hot saturated If the substance contains impurities of comparable
solution [prepared in step (a)] is filtered preferably solubilities, it can be purified by repeated crystallisation.
Tian.
3. Sublimation
MODERN'S abc + OF CHEMISTRY-AI

tube connected to a Leibig’s condenser. The liquid to be


Certain organic solids on heating directly change purified is taken in the distillation flask. Afew unglazed
from solid to vapour state without passing through a porcelain pieces or glass beads are added to the liquid
liquid state. Such substances are called sublimable. to avoid bumping. The flask is gently heated on a water
This process is called sublimation. The vapours on bath or oil bath or even directly, depending upon the
cooling change back to the solid form. boiling point of the liquid. When heating is continued,
the liquid starts boiling at a constant temperature and
Heat
Solid ——— Vapours almost whole of it has changed into vapours. When it
Cool passes through the condenser, the vapours get
The sublimation process ts used for the condensed to the liquid form. The pure liquid is collected
separation of sublimable volatile compounds in the receiver kept at the open end of the condenser.
from non-sublimable impurities. The distillate contains pure liquid while the impurities
In this process, the impure solid is taken in a are left behind in the distillation flask.
porcelain dish and covered with a perforated filter
paper. An inverted funnel is placed over it. The stem
of the funnel is plugged with a little cotton and the
sides are kept cooled (Fig. 10).
Distillation

£
ai
D
5
Oo

Perforated filter

st Impure compound
Pure liquid

Fig. 11. Simple distillation.


This method can also be used for separating liquids
having sufficient difference in their boiling points. The
liquids having different boiling points vapourize at
different temperatures. The vapours are cooled and
the liquids so formed are collected separately. The
liquid mixture is taken in a round bottom flask and
heated carefully in an apparatus as shown in Fig. 11.
Fig. 10. Sublimation process. The liquid having lower boiling point will vapourize
The dish is heated gently over sand bath when first and its vapours are condensed using a condenser
the substance volatilises and gets collected on the inner and the liquid formed is collected. The vapours of
cold surface of the inverted funnel. The non-volatile higher boiling component form later and condensed
impurities are left in the dish. The perforated filter separately.
paper allows the vapours to go up but checks the
sublimate from falling back to the dish. Thus, the object of simple distillation is to
The process is generally used for the purification e separate volatile liquids from non-volatile
of camphor, naphthalene, anthracene, benzoic impurities.
acid, etc. containing non-volatile impurities. e separate liquids having sufficient difference in
4, Distillation their boiling points.
This method is used for the purification of liquids
This method can be used to separate a mixture of
which botl without decomposition and contain
non-volatile impurities. The simple distillation involves (i) chloroform (b.p. 334 K) and aniline
the heating of the liquid to its boiling point so that it is (b.p. 457 K)
converted into vapours. On cooling the vapours, pure (it) ether (b.p. 808 K) and toluene (b.p. 384 K )
liquid is obtained and collected separately.
(iti) benzene (b.p. 353 K) and aniline (b6.p.
The apparatus employed for the process is shown
457 K)
in Fig. 11. It consists of a distillation flask having a side
ORGANIC CHEMISTRY: BASIC PRINCIPLES AND TECHNIQUES

5. Fractional Distillation
This process is used to separate a mixture of two
or more miscible liquids which have boiling points close Water outlet
to each other. In these cases, the process of simple
distillation will fail because the vapours of both the Condenser
liquids will be formed simultaneously and the distillate Fractionating
will also contain both of them. To solve the problem, column with Adaptor
packing
the distillation is carried out by using fractionating af aed
columns. The fractionating column is a long tube
Liquid to be water \1
provided with obstructions to the passage of the distilled : ) inlet
vapours moving upwards and liquid moving a Distilled _
downwards. It is a long glass tube with a wide bore liquid
packed with glass beads or porcelain rings or blown
into a number of spherical or pear shaped bulbs.
Because of the obstructions, the fractionating column
increases the cooling surface area. The common types
of fractionating columns are shown in Fig. 12.
The fractionating column is fitted to the distillation Fig. 13. Fractional distillation.
flask.
condensation and vaporisation is called theoretical
plate. These days commercially, columns with
hundreds of plates are available. This method may be
used to separate a mixture of acetone (b.p. 330 K)
and methyl] alcohol (b.p. 338 K).
One of the technological applications of fractional
distillation is to separate different fractions of
erude oil in petroleum industry.
6. Distillation under Reduced Pressure : Vacuum
Distillation
Simple packed
Bubble plate Certain liquids have a tendency to decompose at
column
column a temperature below their boiling points. Such liquids
cannot be purified by ordinary distillation. Therefore,
Fig. 12. Common types of fractionating column. vacuum distillation its used for liquids which
decompose at a temperature below their normal boiling
Purpose of Fractionating Column. The
points. Under reduced pressure, the liquid will boil at
fractionating column provides more surfaces for heat
a low temperature and the temperature of
exchanges and obstructions to the ascending vapours
decomposition will not be reached. For example,
and the descending liquid. When the flask is heated,
glycerol boils with decomposition at 563 K but if the
the vapours of more volatile liquid (low boiling
pressure is reduced to 12 mm, it boils at 453 K without
component) rise up in the fractionating column. Due to
decomposition.
the obstructions in the fractionating column, some of
the vapours condense and fall back in the column. Some The apparatus used for the distillation under
of the condensing liquid in the fractionating column reduced pressure is shown in Fig. 14. The liquid to be
gets heat from the ascending vapours and revaporizes. distilled is taken in a special two necked flask called
As a result, the vapours become richer in low boiling Claisen flask. It is fitted with a long capillary tube in
component. These rise up in the fractionating column the main neck of the flask. The lower end of the tube
and condense while passing through the condenser. is dipping in the liquid and is provided with a screw
They are collected in the receiver. type stop cock so that the flow of air into the flask
may be adjusted as required. The thermometer is
The vapours of the less volatile liquid (high boiling
placed in the other neck of the flask. The side tube of
component) also rise into the column but due to many
the second neck is connected to a condenser, the open
obstructions they get condensed in the column and
end of which goes into a receiver. The receiver is
flow back to the distillation flask. After a series of
further connected to a vacuum pump and a manometer
successive distillations, the remaining liquid in the
which indicates the pressure. The liquid is heated and
distillation flask gets richer in higher boiling
it distils over at a reduced pressure.
component. Therefore, the receiver contains the
fraction rich in more volatile liquid. The process may This technique can be used to separate glycerol
be repeated to get pure liquids. Each successive from spent lye in soap industry.
Tiana, MODERN'S abc + OF CHEMISTRY-AXI

Thermometer

7A
AZ
un

Capillary tube Manometer


to regulate air

Claisen flask
_\_/|mpure | =—> Vacuum pump
== liquid
Oil bath

Water from tap

Pure liquid ,

Fig. 14, Distillation under reduced pressure.

7. Steam Distillation the organic substance that distils over with steam can
The process of steam distillation ts used for the be obtained as :
separation and purification of liquid which is We know from Dalton’s law of partial pressures
appreciably volatile in steam from non-volatile that the partial pressure of different gases in a gaseous
components of a mixture. Thus, the process of steam mixture are in the ratio of their relative number of
distillation is used to purify the substances which moles. Therefore, the ratio of number of moles of
(t) are volatile in steam but are not miscible with organic liquid and water in the distillate will be in the
water. ratio of their partial vapour pressures (in the boiling
(it) possess sufficiently high vapour pressure mixture), 1.e.
(about 10-15 mm Hg) at the boiling point my Py
le Pe ..(L)
temperature of water (100°C).
where n, and n, are the number of moles and p, and p,
(tt) contain non-volatile impurities.
are the partial pressures of organic liquid and water
In this method, the steam from a steam generator
respectively.
is continuously bubbled through impure organic liquid
heated in a flask. Steam heats the liquid but itself If W, and W, are the respective masses of the
gets condensed. After some time, the mixture of the organic liquid and water in the distillate, then
liquid and water starts boiling when the sum of vapour : OW,
M and hg = 73.
pressures due to the organic liquid (p,) and that due
to water (p,) become equal to atmospheric pressure where M is the molecular mass of the organic liquid
(p) L.e., and 18 stands for the molecular mass of water.
Substituting the values of n, and n, in eqn. (i), we get
P=P,TP»5
Since p, is lower than p, the organic liquid Wi,
18 _ Pt
vaporises at lower temperature than its boiling point. M Wp) Pp»
At this temperature, the mixture of steam and the W, py xM -
liquid gets distilled over and collected in the receiver. or Wr p18 Ath)
The components of mixture are then separated by
Equation (ii) gives the relative masses of organic
simple methods. The apparatus is shown in Fig. 15.
liquid and water in the distillate.
Thus, in steam distillation, water and the organic
The process of steam distillation can be applied
substance vapourise together and the total vapour
for the separation of a mixture of o-nitrophenol and
pressure becomes equal to atmospheric pressure, i.e.,
p-nitrophenol. In this process, water vapours carry
Vapour pressure of organic substance
along with them vapours of o-nitrophenol which is
+ Vapour pressure of water more volatile and they get condensed in the receiver ;
= Atmospheric pressure p-nitrophenol with higher boiling point remains in the
This means that in steam distillation, the organic distillation flask.
substance vapourises and gets distilled at lower This method can also be used for the purification
temperature than its boiling point. The proportion of of impure sample of aniline.
ORGANIC CHEMISTRY: BASIC PRINCIPLES AND TECHNIQUES

Safety tube
Steam —
To sink

Steam
generator

Compound
to be
distilled

distilled
liquid

Fig. 15. Steam distillation.

8. Differential Extraction repeated a number of times. Greater the number of


This method ts used to separate a given organic extractions applied, more will be the amount of
compound present in aqueous solution by shaking with substance extracted. For example, benzoic acid from
a suitable organic solvent in which the compound is water may be extracted from its aqueous solution by
more soluble than water. The basic requirement of the using benzene. Then benzoic acid may be finally
organic solvent is that it should be immiscible with recovered from its solution by distilling off the organic
water so that organic and water layers can be easily solvent.
separated. This process can also be used to separate two
For this process, the aqueous solution is mixed organic liquids such as aniline and water.
with a small quantity of organic solvent in a separating It may be noted that when the organic substance
funnel (Fig. 16). The separating funnel is stoppered is less soluble in the organic solvent, a very large
and its contents are shaken for some time and then quantity of the solvent would be required to extract
even a very small amount of the compound. In such
the funnel is allowed to stand for some time.
cases, a technique of continuous extraction is used.
The organic liquid and water form separate layers.
In this method, the same solvent is repeatedly used
The two layers are collected by opening the stop cock.
for the extraction of the compound. The solid mixture
The aqueous layer is again transferred to the containing the compound is taken in a porous tube A
separating funnel and the process is repeated with fitted with a thick filter paper. The solvent is taken in
more organic solvent. The evaporation of solvent from flask B. When the flask B is heated, the vapours of
solvent layer gives the substance. Jt may be noted that the solvent rise up and condense in the condenser. The
to have maximum extraction, the process may be condense liquid falls in tube A and extracts the

Solvent layer Organic compound


in solvent layer

Organic compound Aqueous layer


in aqueous layer

(a) Before extraction (b) After extraction

Fig. 16. Differential extraction with separating funnel.


STianos MODERN'S abc + OF CHEMISTRY-AI

The selective removal of the components may be


due to adsorption or partitioning (distribution)
processes. Based on the principle involved,
chromatography is classified into two categories :
A. Adsorption chromatography
B. Partition chromatography.
A. Adsorption Chromatography
It is based on the fact that different compounds
are adsorbed* on an adsorbent to different
degrees. When a mobile phase is allowed to move over
a stationary phase (adsorbent), the components of the
mixture move by varying distances over the stationary
phase because of the different adsorption tendencies.
In this case, the stationary phase can be held on a
filter paper (paper chromatography), a cylindrical
column of solid (column chromatography) or a thin
layer of the solid spread on an inert surface (thin layer
chromatography).
The following are two main types of
chromatographic techniques based on the principle of
differential absorption :
(1) Column chromatography
(it) Thin layer chromatography
1. Column Chromatography
In this method, a suitable adsorbent like alumina
Fig. 17. Continuous differential extraction of a less (Al,O,), silica (Si0,), magnesium oxide or Fuller’s
soluble compound in a solvent. earth is packed as a column in a burette like long tube
fitted with stop cock near its bottom. A plug of cotton
compound. When the level of the liquid in tube A
reaches up to the top in tube C, it gets siphoned and is or glass wool is placed at the bottom of the column to
collected in the flask which is being heated support the adsorbent. This acts as a stationary phase.
continuously. The solvent is distilled and its vapours The mixture to be separated is dissolved in a
again rise, get condensed and extract compound in suitable solvent and the solution is poured on the top
tube A. Thus, the process of extraction continues. of the column of the adsorbent. Because different
9. Chromatography
This is a modern method used for the separation Mixture
of mixtures into its components, purification of +

compounds and also to test the purity of compounds. solvent


The name chromatography is based on the Greek word Adsorbent 3:7
chroma meaning colour and graphy for writing
because the method was first used for the separation
of coloured substances found in plants. This method
was described by Tswett in 1908.
Principle of Chromatography
The technique of chromatography is based on the Glass wool —
difference in the rates at which the components
of a mixture move through a porous medium
(called stationary phase) under the influence of
some solvent or gas (called moving phase). (a) Adsorbent column (b) Separation of ) !
Thus, this technique consists of two phases—one components :
|
of these is a stationary phase of large surface area
while the second is a moving phase which is allowed
to move slowly over the stationary phase. The (c) Elution
stationary phase is either a solid or a liquid while the Fig. 18. Column chromatography.
moving phase may be a liquid or a gas.
* Adsorption is a phenomenon of higher concentration of molecular species (liquids or gases) on the surface of a solid than in the bulk. The
solid substance on the surface of which adsorption occurs is known as adsorbent (similar to solvent in case of solution). The substance
that gets adsorbed on the solid surface is called adsorbate (similar to solute in case of solution).
ORGANIC CHEMISTRY: BASIC PRINCIPLES AND TECHNIQUES

components of the mixture are adsorbed to different 2. Thin Layer Chromatography (TLC)
extents, they get adsorbed to different parts of the This method involves the separation of the
column. The component which is adsorbed strongly substances of a mixture over a thin layer of an
gets adsorbed at the top. The other components with adsorbent coated over a plate. In this method, a thin
decreasing orders of their adsorbabilities are held up layer (about 0.2 mm thick) of an adsorbent material
at different zones down the column in the form of (silica gel or alumina or cellulose) is spread over a
bands. Consider, for example, a mixture of three glass plate of a suitable size as the stationary phase.
components A, B and C. Let their tendencies to get The plate is called as thin layer chromatography
adsorbed on a solid surface area in the order A> B>C plate or chroma plate. The solution of the mixture
when these are dissolved in a solvent and passed to be separated is applied as a small spot with the
through the column. The component A, which is most help of a fine capillary about 2 cm above one end of
strongly adsorbed by the adsorbent will be held by the the TLC plate. The spot is dried and is placed in a
solid to a greater extent than the others. It will be closed jar containing the solvent (eluant) covered with
adsorbed at the top. As this solution goes down, the a lid. As the solvent moves up the plate, the
concentration of A becomes less and more of B will be components of the mixture move up along the plate to
adsorbed (adsorbing power of B is more than that of different distances depending upon their degree of
adsorption. As a result, separation of the mixture takes
C). After this C will be adsorbed. Thus, a separation of
place within a few minutes.
the mixture takes place.
The plate is taken out and the position of various
After the separation, the substances are extracted
spots are visualised by suitable solvents called
from the adsorbent with suitable solvent. This process
visualising agents. The movement of the substances
of recovery or extraction of the adsorbed components relative to the solvent is expressed in terms of its
from the adsorbent with the help of a suitable solvent retention factor, popularly known as R, value. This
is called elution. The commonly employed eluents are gives the relative adsorption of each component of the
petroleum ether, carbon tetrachloride, benzene, alcohol, mixture. It is related to the movement of the solute
acetone, etc. The eluent acts as a mobile phase. The front relative to the movement of the solvent front as:
eluent dissolves out different components selectively.
The weakly adsorbed component will be eluted more R,=‘Distance moved by the substance from the base line
rapidly than a more strongly adsorbed component. The Distance moved by the solvent from the base line
process of elution is repeated a number of times to The distances are measured from the point of origin
result complete separation. Different components of (usually the centre of the spot is taken as the origin)
the mixture are collected in the form of different as shown in Fig. 19 (b). The Ry defines the movement
fractions in separate conical flasks. The components of a substance relative to the Sveal and it is constant
from the eluent are separated by simple methods such for a given substance (component) under a given set of
as distilling off solvent or separating with the help of conditions. Therefore, it is possible to identify the
separating funnel. various components by determining their R, values.
Applications of the method. This method has It is also possible to estimate the components
quantitatively by measuring the intensity of colours
been used
developed by them on reacting with suitable reagents.
(i) to separate ortho and para nitro-anilines.
(it) to separate blue and red dyes. In case of mixture of substances, two different spots
are obtained as shown in Fig. 19 (c).The Ry values of
(iii) to separate and purify plant pigments and
two components may be calculated.
other natural products.

rl
Solvent front

Spot component
Chromatographic Spots of ,
component

(a) Thin layer chromatography


| Rey,

ase line

(5) Developed chromatogram for (c) Developed chromatogram


apparatus. a compound. for a mixture of substances.
Fig. 19
S Tian0s.
The spots of coloured compounds are visible on
MODERN'S abc + OF CHEMISTRY-AXI

spot on the line. The spot is dried by blowing hot air


TLC plate due to their original colour. In case of over it from a drier.
colourless compounds, which are invisible to the eye The chromatographic paper is then suspended in
but which fluoresce can be detected by putting the a suitable solvent or a mixture of solvents. Aleternatively,
place under ultra-violet light. There is another method the chromotographic paper is placed in a closed chamber
in which the plate is placed in a covered jar containing and the edge of the filter paper is dipped into a suitable
a few crystals of iodine. Spots of compounds which solvent or a mixture of solvents. These two different
adsorb iodine will show up as brown spots. Sometimes methods are shown in Fig. 20 (a) and (0). The solvent
an appropriate reagent may be sprayed on the plate. or the solvent mixture to be used depends upon the
For example, amino acids may be detected by spraying nature of the components of the mixture. The solvent
the plate with ninhydrin solution. rises up the paper by capillary action and flows over
B. Partition Chromatography the spot. The mixture gets separated into its components
It is based upon the differences in the which also move upwards to different extents, because
tendencies of substances to distribute or partition of their different speeds of movement. The paper
between two phases. In this process, the components selectively retains different components according to
of a mixture undergo continuous differential partition their differing partition in two phases. The paper strip
between stationary and mobile phases. so developed is known as a chromatogram. The spots
Paper chromatography is a type of partition of the separated coloured compounds are visible at
chromatography in which the adsorbent column is a differing heights from the position of initial spot on the
paper strip. It is a special type of paper known as chromatogram. The spots of the separated colourless
chromatography paper which contains water trapped compounds may be observed either under ultra-violet
in it. This acts as a stationary phase. light or by the use of an appropriate spray reagent as
discussed in their layer chromatography.
In this method, a line is drawn by a pencil on the
chromatography paper parallel to one edge. Then a In addition to above types of chromatography, ion-
drop of the solution of the mixture is applied as a small exchange chromatography, gas chromatography, high
performance liquid chromatography (HPLC) are used.

Card board lid

Chromatography paper
Chromatographic paper
Jar
Jar
Spot
spot
Base line

Solvent Solvent

Fig. 20. Paper chromatography.

= Pal Fie @ 18. Suggest methods for the separation of following


mixtures :
(i) A mixture of liquid A (b.p. = 356 K) and liquid B
Q 13. A liquid has non-volatile impurities. Which method (b.p. = 365 K)
will you use for its purification ? (ii) A mixture of liquid C (b.p. = 360 K) and liquid D
Q 14. Suggest a method to purify (b.p. = 395 K)
(i) 1odine containing traces of common salt (iii) A liquid ‘EF’ (b.p. = 563 K) having decomposition
(ii) kerosene containing water temperature 450 K.
(iii) benzene containing non-volatile impurities. @ 19. How will you purify an impure sample of (i) camphor
Q 15. How will you separate a mixture of o-nitrophenol (ii) aniline ?
and p-nitrophenol ? Q@ 20. Give two examples of adsorbents used in
Q 16. Who discovered the chromatography technique ? chromatography.
Q 17. A compound “X” has boiling point of 475 K but it @ 21. The R, values of A and B in a mixture determined
starts decomposing at 400 K. Which type of by TLC in a solvent mixture are 0.65 and 0.42
distillation process is suitable for its purification ? respectively. If the mixture is separated by column
ORGANIC CHEMISTRY: BASIC PRINCIPLES AND TECHNIQUES

chromatography using the same solvent mixture as a bulb in the centre (Fig. 21). The other end of the
a mobile phase, which of the two components A or B delivery tube is dipped in a tube containing lime water.
will elute first ? The bulb in the delivery tube is packed with glass wool
Q 22. Name the method of separating naphthalene from containing anhydrous copper sulphate (white). When
kerosene present in a mixture. the mixture is heated, the carbon present in the
compound is oxidised to carbon dioxide which turns
@ 235. A mixture contains 71% of calctum sulphate and 29% lime water milky. The hydrogen present in the organic
of camphor. Name a suitable technique of separation compound is oxidised to water which turns anhydrous
of the components of the mixture.
copper sulphate in the bulb to blue.
Answers to Practice Problems
C + 2Cu0 —“>NCo, + 2Cu
© 13. Distillation From organic (dry)
© 14. (i) Sublimation (ii) solvent extraction method compound
(iii) distillation. CO, +Ca(OE)y —S> CaCO, + H,O
© 15. By steam distillation. Lime water Milky solution
© 16. Tswett. 9H + Cu0 =",
Heat
H,0 + Cu
© 17. Distillation under reduced pressure. From organic
compound
© 18. (i) Fractional distillation (ii) Simple distillation
(iii) Distillation under reduced pressure.
CuSO, + 5H,0 —— > CuSO,.5H,O
Anhydrous Hydrated copper sulphate
© 19. (i) Sublimation (i) Steam distillation copper sulphate (blue)
© 20. Alumina (A1,0,), silica gel. (white)

© 21. A This method is known as copper oxide test.


If the organic compound is volatile liquid or a gas, then
© 22. Fractional extraction.
the vapour of the liquid or gas itself is passed through
© 23. Sublimation method. heated copper oxide and the liberated gases are tested
for CO, (with lime water) and water vapours
QUALITATIVE ANALYSIS OF ORGANIC (with anhydrous CuSO,).
COMPOUNDS
2. Detection of Nitrogen
The qualitative analysis of an organic compound Nitrogen in an organic compound is detected by
involves the detection of all the elements present in the following tests :
it. Carbon and hydrogen are generally present in all (a) Soda lime test. A pinch of an organic
the organic compounds. The other elements which may compound is heated strongly with soda lime
be present 1n organic compounds are oxygen, nitrogen, (NaOH + CaO) in a test tube. Jf ammonia gas evolves,
sulphur, halogens, phosphorus. These can be detected it indicates nitrogen.
by the following tests :
1. Detection of Carbon and Hydrogen
CH,CONH, + NaOH—“°> CH,COONa + NH,
Acetamide
Asmall amount of the dry and powdered substance
Limitation. This method has a limitation. Alarge
is mixed with about double the amount of pure and
number of organic compounds such as nitro (—NO,)
dry copper oxide. The mixture is heated in a well dried
and diazo (-N=N—) compounds do not liberate
hard glass tube provided with a delivery tube having
ammonia on heating with soda lime.
(b) Lassaigne’s test. This is the most reliable
test for the detection of nitrogen. It is carried out as
follows :
A small piece of a dry sodium metal is heated
gently in a fusion tube till it melts to a shining globule.
Organic Then, a small amount of organic substance is added
compound and the tube is heated strongly till it becomes red hot.
The red hot tube is plunged into distilled water
contained in a china dish and boiled for some time. It
is then cooled and filtered. The filtered liquid is known
as sodium extract or Lassaigne’s extract.
The Lassaigne’s extract is usually alkaline. If not,
it may be made alkaline by adding a few drops of a
dilute solution of sodium hydroxide. To a part of sodium
Fig. 21. Detection of carbon and hydrogen in an organic
extract a small amount of a freshly prepared ferrous
compound.
sulphate solution is added and the contents are
Tiare.
warmed. A few drops of ferric chloride solution are
MODERN'S abc + OF CHEMISTRY-AI

The Lassaigne’s extract is divided into two parts


then added to the contents and the resulting solution and following tests are performed :
is acidified with dilute hydrochloric acid. The ({) Sodium nitroprusside test. To one portion
appearance of a bluish green or a prussian blue of the extract, a few drops of sodium nitroprusside are
colouration confirms the presence of nitrogen in the
added. The appearance of violet colouration indicates
organic compound.
sulphur.
The following chemical reactions occur during the
test :
Na,S + Na,[Fe(CN),NO] ———> Na,[Fe(CN),NO.S]
Sod. nitroprusside Violet colouration
Na + C+N
eal Bea m——> NaCN
or S* + [Fe(CN),NO]*,> ———+> [Fe(CN), NOS]*
From organic Violet
compound (it) Lead acetate test. The other part of the
Lassaigne’s extract is acidified with acetic acid and
FeSO, + 2NaCN ———> Fe(CN), + Na,SO, then lead acetate solution is added. Formation of black
Fe(CN), + 4NaCN ———> Na,|[Fe(CN),] precipitate confirms the presence of sulphur.
Sodium hexacyanoferrate (II)
Sodium ferrocyanide Na,S + Pb(CH,COO), ———— PbS + 2CH,COONa
3Na,[Fe(CN),] + 4FeCl, ———> Fe,[Fe(CN),], Lead acetate Black
Tron (III) hexacyanoferrate (IT) 4, Detection of Halogens
Ferric ferrocyanide Halogens in an organic compound are detected by
(Prussian blue) the following tests :
+12 NaCl 1. Beilstein’s test. A clean copper wire is heated
or Fe?+ + 6CN- ———> [Fe(CN),]*- in the non-luminous flame till it does not impart blue
3[Fe(CN),]+ + 4Fe3+ #29 _,Fe, [Fe(CN),],.xH,O colour to flame. The hot end of copper wire is now
touched with the organic substance and is again
Prussian blue
introduced into flame. The reappearance of green or
Note. If the organic substance contains blue colour indicates the presence of halogens in the
nitrogen and sulphur together, sodium thiocynate
organic compound.
may be formed in the sodium extract which gives
Limitation. (i) This test 1s given by substances such
blood red colouration with ferric chloride due as urea, thiourea, etc. which do not contain halogens.
to the formation of ferric thiocyanate.
(it) It does not tell as to which halogen is present.
Na+ C+N+4+5S ——— > NaCNS 2. Lassaigne’s test. Sodium extract is prepared
From organic Sodium as already described. During fusion, sodium will
compound thiocyanate combine with the halogen (from the organic compound)
3NaCNS + FeCl, ———~> Fe(CNS), + 3NaCl to form sodium halide.
Ferric thiocyanate
(Blood red)
Na+X —fusen_, NaX
(X =Cl, Br D
The extract is acidified with concentrated nitric acid
or Fe** + 3CNS ——Y—> [Fe(CNS),]
and boiled to expel the gases. It is then cooled and a
Ferric thiocyanate
(Blood red) small amount of silver nitrate is added. The formation
of white or yellow precipitate confirms halogens.
However, it may be noted that the absence of
blood red colouration in this test does not necessarily (1) A white precipitate soluble in ammonium
mean that sulphur is absent. This is because if hydroxide solution indicates the presence of chlorine
sodium fusion is carried out with excess of sodium, in the organic compound.
the thiocyanate decomposes to yield cyanide and NaCl + AgNO, ————+ AgCl + NaNO,
sulphide. These ions give their usual tests : White ppt.
NaSCN + 2Na —— > NatCN + Na.s (it) A dull yellow precipitate partially soluble in
3. Detection of Sulphur ammonium hydroxide solution indicates the presence
Sulphur in an organic compound can also be of bromine in the organic compound.
detected by Lassaigne’s test. Sodium extract or NaBr + AgNO, ———_> AgBr + NaNO,
Lassaigne solution is prepared as described earlier.
Dull yellow ppt.
The sulphur in the compound reacts with sodium
metal to form sodium sulphide. (iit) A bright yellow precipitate, completely
2Na + 5 —— Na,s insoluble in ammonium hydroxide solution, indicates
From organic the presence of iodine in the organic compound.
compound
ORGANIC CHEMISTRY: BASIC PRINCIPLES AND TECHNIQUES

Nal +AgNO, ———+ Agl + NaNO, @ 26. What precipitate will occur when silver nitrate is
Bright yellow ppt. added to chloroform ?
Function of nitric acid. In case, nitrogen and @ 27. Why is the organic compound fused with sodium
sulphur are present along with the halogens in the metal before testing for N or halogen ?
organic compound, the Lassaigne’s extract contains
sodium sulphide (Na,S) and sodium cyanide (NaCN) Answers to Practice Problems
along with sodium halide. These will form precipitates
with silver nitrate solution. Therefore, nitric acid is added © 24. Urea
to decompose sodium cyanide and sodium sulphide. © 25. Blood red colouration due to Fe(CNS), will be
NaCN + HNO, ———+ NaNO, + HCN produced.
© 26. No ppt.
Na,S + 2HNO, ———> 2NaNO, +H,S
© 27. No ppt.
Special test for bromine and iodine. Boil a
portion of the Lassaigne’s extract with nitric acid. Add QUANTITATIVE ANALYSIS
a few drops of carbon disulphide and then add chlorine
Quantitative analysis involves’ the
water slowly with constant shaking.
determination of the percentages of the various
An orange colouration in carbon disulphide
elements present in a given compound. The common
layer confirms the presence of bromine whereas a
violet colouration in the layer confirms the presence methods are discussed below :
of iodine. 1, Estimation of Carbon and Hydrogen
2NaBr + Cl, ——— > 2NaCl + Br, Carbon and hydrogen in the organic compound are
(turns CS, layer orange) estimated by Liebig’s method.
2Nal + Cl, ———> 2NaCl + I, Principle. A known mass of the given dry organic
(turns CS, layer violet) compound is heated strongly with dry cupric oxide in
5. Detection of Phosphorus an atmosphere of air or oxygen free from CO,. The
To detect the presence of phosphorus, the given carbon and hydrogen of the organic compound are
organic compound is fused with sodium peroxide oxidised to CO, and water vapour as :
(oxidising agent).
2P+5Na,0, Heat
——— > 2Na,PO,+2Na,0
C+2CuO _Heat ,CO, + 2Cu
From organic compound (from compound)
The fused mass 1s extracted with water. The filtrate 2H +CuO _Heat .H,O + Cu
obtained is boiled with a few drops of conc. HNO,. Then (from compound)
the solution is treated with ammonium molybdate. A
In general,
yellow coloured precipitate (due to ammonium
phosphomolybdate, (NH,),PO,. 12MoO, confirms
phosphorus).
CHa + x+2]0
4 Z
__+xCO, Ps +2y H,O2
Na,PO, + 3HNO, ———+> H,PO, + 3NaNO, Carbon dioxide produced is collected in potash
HPO, + 12 (NH,), MoO, + 21LHNO; ———> bulbs (containing KOH solution) while water vapour
Ammonium molybdate are absorbed in anhydrous calcium chloride tubes.
(NH,), PO,. 12 MoO, + 21 NH,NO, + 12 H,O From the amounts of carbon dioxide and water vapour
Ammonium produced, the percentage of carbon and hydrogen can
phosphomolybdate be calculated.
6. Detection of Oxygen Procedure. The apparatus for estimation of
There is no direct method to detect oxygen in an C and H is shown in Fig. 22. The substance is heated
organic compound. However, it can be tested by testing with CuO. The carbon dioxide thus produced is
the presence of oxygen containing functional groups absorbed in a U-tube containing a strong solution of
(such as—OH, —CHO, —COOH, —NO,, etc). The KOH while water produced is absorbed in a U-tube
presence of these functional groups confirms the containing anhydrous calcium chloride. These tubes
presence of oxygen in the organic compound. are weighed before and after the combustion. The

= Pal ie
increase 1n masses after the experiment gives the
amount of water and CO, produced.
Let the mass of organic compound taken = w ¢g
@ 24, Name a compound which does not contain halogen Mass of water formed = m, g
but gives positive Beilstein’s test. (increase in CaCl, U-tube)
Q 25. What will happen during Lassaigne’s test for nitrogen
Mass of carbon dioxide formed = m, g
if the compound also contains sulphur ?
(increase in potash tubes)
Tanto. MODERN'S abc + OF CHEMISTRY-AI

Substance
Cud pellets

Anhydrous CaCl, KOH solution

Fig. 22. Apparatus for the estimation of C and H.

Percentage of Carbon
2B x fess 100 = 54.54
We know that 1 mole of carbon dioxide (44 g) — At J 0.2475 ~- OO
contains 1 gram atom of carbon (12 g)
Percentage of hydrogen = FA % Hot HO x 100
CO, =C 18 Wt. of compound
44 g of CO, contain C = 12g
d 2 . 0.2025 aaa
12 - 18 02475 *°.”
m, g of CO, contain C = qa *™8
O Example 32.
An organic compound contains 69% carbon and
H ence, P ercentage
Lapeaheaton
of carbon ==
77——~ x 100 4.8% hydrogen, the remainder being oxygen.
Calculate the masses of carbon dioxide and water
42 Mass of CO» formed produced when 0.20g ofthis substance is subjected
Percentage of C=—, x ——————————— x« 100 to complete combustion.
44 Mass of compound taken
Solution:
Percentage of Hydrogen Percentage of carbon = 69%
We know that one mole of water (18 g) contains
69
2 gram atom of hydrogen (2 g) Amount of carbon in 0.20 g of compound = 0.2 x Too

H,O = 2H = 0.138 g
18 g of H,O contain H = 2g Percentage of hydrogen = 4.8%

| 2 0.2x4.8
m, g of H,O contains H = Ty xm, g Amount of hydrogen in 0.20 g of compound = jn

= 0.0096 ¢
Percentage
ercentage of
BO o hydrogen
eet =
8 2m
xw x 100 , Now C=CO,
12 g of carbon on combustion give CO, = 44g
9 Mass of HO formed 44
:
Percentage of H=— x —————_———_
18 Mass of compound taken
x 100 0.138) g of carbon on combustion give CO, = 72" 0.138

~ 0.506 g
|SOLVED EXAMPLES 2H = H,0
2 g of hydrogen on combustion give water = 18 g
LI Example 31.
0.2475 g of an organic compound gave on 18
0.0096 g of hydrogen on combustion give water = = x 0.0096
combustion 0.4950 g of carbon dioxide and
0.2025 g of water. Calculate the percentage of — 0.0864 g
carbon and hydrogen tin it. QO Example 33.
Solution: On complete combustion, 0.246 g of an organic
Wt. of organic compound = 0.2475 g compound gave 0.198 g of carbon dioxide and
Wt. of CO, produced = 0.4950 g 0.1014 g of water Calculate the percentage
Wt. of H,O produced = 0.2025 g composition of carbon and hydrogen in the
12
ee
Wt. of CO, — compound.
Percentage of carbon = x 100
44 Wt. of compound
ORGANIC CHEMISTRY: BASIC PRINCIPLES AND TECHNIQUES

Solution: In general,
Wt. of organic compound = 0.246 g
Wt. of CO, produced = 0.198 g
CN, +(x+ 2) Cu0 ——>xCO,+ 5 H,O
Wt. of H,O produced = 0.1014 g
+=2 N,+(2xt+ 22) Cu
12
0.198x100
Percentage of carbon = rr x Procedure. The apparatus for the estimation of
0.246
nitrogen by Duma’s method is shown in Fig. 23.
= 20.95% A known mass of the organic compound is heated with
. . — 9 0.1014
x100 a dry cupric oxide in a combustion tube in the
ercentage of hydrogen= 18 0 246 atmosphere of CO,. The CO, is produced by heating
sodium bicarbonate and the gas is bubbled through
= 4.58% conc. H,SO, to free it from moisture. A roll of oxidised
2. Estimation of Nitrogen copper gauze is placed in the combustion tube which
There are two methods for the estimation of prevents the backward diffusion of the gases evolved
nitrogen : in combustion. The organic compound is oxidised by a
layer of coarse copper oxide that fills about two-third
(1) Duma’s method (it) Kjeldahl’s method
of the combustion tube. On heating, carbon and
(1) Duma’s method. This method is based upon
hydrogen present in the organic compound are
the following principle :
oxidised to carbon dioxide and water vapours
Principle of the method. A known mass of an respectively while nitrogen is set free.
organic compound is heated with dry cupric oxide in During heating a small amount of nitrogen might
an atmosphere of carbon dioxide. The carbon and be oxidised to its oxides but they are reduced back to
hydrogen are oxidised to carbon dioxide and water nitrogen by reduced copper placed at the end of
respectively while nitrogen is set free. If any oxide of combustion tube.
nitrogen is produced during this process, it is reduced
Oxides of nitrogen + Cu ————> CuO +N,
to nitrogen by passing over a heated copper gauze.
The gaseous mixture is collected over an aqueous The gaseous vapours are then passed into Schiff’s
solution of KOH, when all the gases except nitrogen nitrometer tube which contains about 40% KOH
solution. It has a mercury seal at the bottom to check
are absorbed. The volume of nitrogen produced is
the backward flow of the liquid. The volume of nitrogen
measured at room temperature and atmospheric
at the atmospheric pressure is recorded with the help
pressure. From the volume of N, produced, percentage
of reservoir. Both CO, and H,O vapours are absorbed
of nitrogen in the sample can be calculated. The
by KOH solution while nitrogen is collected over it.
following reactions take place during this process :
Its volume is recorded after careful levelling (by
C + 2CuO ——-> CO, + 2Cu making the level of KOH solution in the nitrometer
tube and reservoir the same). The room temperature
2H + CuO ———~ H,0+Cu is recorded and the aqueous tension of water vapours
2N + CuO —- N, + oxides of nitrogen at this temperature is noted from the tables.
Let the mass of organic compound = w g
Oxides of nitrogen + Cu-——- CuO +N,
The volume of nitrogen collected = V mL

Reduced copper
CuO + organic gauze Nitrogen
compound Coarse CuO KOH solution

ee ee
I I
i !
I

CO, ze
eee,
BS| IES
shat ite

urnace
HH

=
=

L Nitrometer
CuO gauze
Slane
Atmospheric pressure (from barometer)= pmm of Hg
MODERN'S abe + OF CHEMISTRY-A&I

quantitatively converted into ammonium sulphate. The


Room temperature = TK resulting solution is then heated with excess of sodium
If temperature is in °C, then T,K = °C + 273 hydroxide. The ammonia gas evolved is passed into a
Aqueous tension at T,K = a mm of Hg known but excess volume of standard acid (HCI or
Pressure of dry nitrogen = (p —a) mm of Hg H,SO,).
) Conc. HaSO4
Let us first convert the volume of nitrogen to
C,H,S Heat
CO, + H,O + SO,
volume at 8.T.P.
Experimental conditions S.T.P. conditions (From organic compound)
Conc. HeSO04
p,=(p —a)mm Hg p. = 760 mm Hg
N Heat
(NH,), SO,
T,=T,K T, = 273K (From organic compound) Ammonium sulphate
V,=VmL ees
(NH,), SO, + 2NaOH— _Heat_, Na,SO,+2NH,+2H,O
Applying gas equation, pia, — Pod:V2
T, Ts, 2NH, + H,SO,—=> (NH,),SO,
or NH, + HCI——— NH,Cl
PV,Te _ (p-a)x Vx273 Cr mL
The acid left unused is estimated by titrating the
Vv. i
760 T, = (say)
° Py Ty solution with standard alkali.
Now, 22,400 mL of N, at S.T.P. will weigh = 28 g
H,SO, + 2Na0QH. ————> Na,SO, + 2H,O
x mL of N, at S.T.P. will weigh = x From the amount of acid left unused, the amount of
22400 . acid used for neutralisation of ammonia can be calculated.
”.Percentage of nitrogen
From this percentage of nitrogen can be calculated.
Mass of nitrogen
= x 100 Procedure. The apparatus is shown in Fig. 24.
‘Mass of organic compound The compound (about 0.5 g) is heated with conc. H,SO,
_ 28 x x 100 in a Kjeldahl’s flask, which is a round bottomed flask
22400 x w having a long neck provided with a loose stopper. A
Percentage of N = small amount of potassium sulphate and a little of
28 Volume of N., at S.T.P mercury (or copper sulphate) are added. The former
= x 100 raises the boiling point of sulphuric acid while the
22400 ~ Mass of compound taken
latter acts as catalyst. The heating is done for 2-3 hours
(ii) Kjeldahl’s method. This method is simpler when carbon and hydrogen present in the compound
and more convenient than Duma’s method. This are oxidised to carbon dioxide and water vapours
method is largely used for the estimation of nitrogen respectively while nitrogen is converted into
in food stuffs, drugs, fertilizers and many other organic ammonium sulphate.
compounds.
Carbon dioxide and water vapours escape through
However, this method cannot be used for the loose stopper whereas ammonium sulphate
(it) organic compounds containing nitrogen in the remains in the flask. The contents of the flask are
ring such as pyridine, quinoline, etc. cooled and transferred to a round bottomed flask.
(ti) organic compounds containing nitro (—NO,) These are diluted with water and a few drops of
and diazo (— N = N —) groups. indicator methyl orange are added. The indicator will
Principle. A known mass of the organic compound give pink colour showing acidic nature of the solution.
is heated with conc.H,SO, so that nitrogen is

NH, + Steam
: ——

Kjeldahl's flask. Loose glass


stopper
Organic compound +

Kjeldahlised
liquid + NaOH in
excess

Known volume of
“standard acid

Heating in Kjeldahl’s flask Distillation in Kjeldahl's flask

Fig. 24, Estimation of nitrogen by Kjeldahl’s method.


ORGANIC CHEMISTRY: BASIC PRINCIPLES AND TECHNIQUES

The flask is fitted with a dropping funnel. About 40% For monobasic acids (basicity = 1)
solution of NaOH is added with the help of dropping Percentage of N =
funnel and the flask is heated. During heating,
ammonia is liberated which passes through the trap 1.4.x Molarity of acid x Volume of acid used
which retains water. The ammonia vapours are Mass of compound taken
condensed and the liquid ammonia formed is absorbed or
in a known excess of standard solution of sulphuric
acid. The unreacted (or excess) acid left after absorption _ 14xNormality
ofacid_x_ Volume ofacidused
Mass of compound taken
of ammonia is estimated by titration with standard
alkali solution. The difference between the initial (« for monobasic acids, molarity = normality)
amount of acid taken and that left after the reaction
gives the amount of acid reacted with ammonia. s SOLVED EXAMPLES&
Calculations Example 34
Let the mass of organic compound = w g In a Duma’s nitrogen estimation, 0.3 g of an
Volume of standard H,SO, of molarity M = V mL organic compound gave 50 mL of nitrogen
Now, excess acid left after absorption of NH, is collected at 300K and 715 mm pressure. Calculate
titrated with NaOH of molarity, M. the percentage of nitrogen in the compound
Let volume of NaOH of molarity M used for (vapour pressure of water at 300 K is 15 mm).
titration of excess of H,SO, = V, mL Solution : Vapour pressure of gas = 715-15
Now, = 700 mm
Vv To calculate the volume of N, at S.T.P.
V, mL of NaOH of molarity M = ot mL of H,50O, V,= 50 mL V,= ?
of molarity M p,= 700 mm p,= 760 mm
.. Volume of H,SO, of molarity M left unreacted T,= 300K T,= 273K
Vv.
ae lew
pV ?D Ne
Applying =.4+ = 2 4
Volume of H,SO, of molarity M used for T, T,

neutralisation of NH, = ( 4) mL
aE vV, = (pViT
Ao
2 pol
Now, | |
| Vv. y. vy, = 100%
50x 273
iV- >) mL of H,SO, of molarity M =2 Vv| 2 760 x 300
= 41.9mL
mL of NH, solution of molarity M _ | | 22400 mL of nitrogen at 8.T-P. weigh = 28 g
| V,
-. Volume of NH, evolved = 2 ian? mL of a 28 x 41.9
41.9 mL of nitrogen at 8.T.P. weigh = 29400
molarity M
Now, 1000 mL of 1 M NH, solution contain 17 g = 0.0524 g
NH, or 14 g nitrogen 4
Percentage of nitrogen = x 100 = 17.46%.
2
ees
ara mL of NH, of molarity M
;
contains O Example 35.
During nitrogen estimation of an organic
14x Mx 2{vd) g compound by Kjeldahl’s method, the ammonia
nitrogen = ———————_ evolved by 0.5 g of the compound neutralised
1000 10 mL of 1 M H,SO,, Calculate the percentage of
nitrogen in the compound.
14x Mx 2{v—~2 gg
Peercentage
t o fN = | wf 7000 2, 100 Solution :
1Mof10mL H,SO, = 1 M of 20 mL of NH,
hoy 1000 mL of 1 M ammonia contain = 14 g nitrogen
1.4xM «x a{v-~2} 14x20
or = 20 mL of 1 M ammonia contain = ——— g nitrogen
1000
Ww
Percentage of N = Percentage
ercentage of nit
of nitrogen = eet
7507p, 6p«19
‘14x Molarity of acid x Basicity of acid x Volume of acid used
Mass of compound taken = 56.0%
QO) Example 36.
(e.g. for H,SO,, basicity = 2)
0.257 g of an organic substance was heated with
conc. H,SO, and then distilled with excess of

WWW.JEEBOOKS.IN
Stan strong alkali. The ammonia gas evolved was
MODERN'S abc + OF CHEMISTRY-AXI

14 ABx1
absorbed in 50 mL of M/10 HCl which required = Th00 To
23.2 mL of M/10 NaO8d for neutralisation at the
1446100
end of the process. Determine the percentage of .. Percentage of nitrogen =
1000 «100.36
nitrogen in the compound.
= 18.4%
M
Solution : Volume of To HCl taken = 50 mL Example 38.
A sample of a 0.50 g of an organic compound was
M heated according to Kjeldahl’s method. The
Volume of To NaOH used for neutralisation of unused
ammonia evolved by absorbed in 50 mL of 0.5 M
acid = 23.2 mL H,SO, The residual acid required 60 mL of 0. 5
Now, M solution of NaOH for neutralisation. Find the
M M percentage composition of nitrogen in the
23.2 mL of 10 NaOH = 23.2 mL of To HCl compound.
M Solution:
-. Volume of To HCl unused = 23.2 mL Mass of organic compound taken = 0.50 g |
M Vol. of 0.6 M H,SO, taken = 60 mL
Volume of To HCl required for neutralisation The residual acid required 60 mL of 0.5 M solution of
NaOH for neutralisation
of NH , = 50-23.2 60 mL of 0.6 M NaOH solution
26.8 mL
60
= ry mL of 0.6 M H,SO, solution
26.8 mL of — HC] =26.8 mL of M/10 NH,
= 30 mL of 0.6 M H,SO, solution
1000 mL of 1 M NH, solution contains nitrogen
Volume of 0.6 M H,SO, used = 50 — 30 = 20 mL
= 14¢
Now
26.8 ml of 10 NH, solution contains nitrogen 20 mL of 0.6 MH,SO, = 2 x 20 mL of 0.6 M NH,
solution
_ 14x 26.8 or = 40 mL of 0.6 M NH, solution
~ 10x 1000 1000 mL of 1 M NH, contain nitrogen = 14 g
14x 26.8 «100 oo 14x
40 x 0.5
Percentage of nitrogen 40 mL of 0.6 M NH, contain nitrogen = —jn00
10 x 1000 x 0.257
14 «400.6
14.6%.
% of N= 7000x050 ~
L} Example 37.
= 56%
0.35 g of an organic substance was Kjeldahlised
and the ammonia obtained was passed into 3. Estimation of Halogens
100 mL of M/10 H,SO,. The excess acid required Halogens are estimated by Carius method.
154mLofM /10 NaOH for neutralisation. Calculate Principle. A known mass of the organic substance
the percentage of nitrogen in the compound. is heated with fuming HNO, in a Carius tube. The
Solution : Vol. of M/10 H,SO, taken = 100 mL silver halide so obtained is separated, washed, dried
Let us first calculate the volume of excess M/10 H,SO, and weighed. From the weight of silver halide formed,
which was neutralised by 154 mL of M/10 NaOH the percentage of halogen can be calculated.
M 7 164 M
164 mL of 10 NaOH = 9 mL of 10 H,sO, X + AgNO, ——> Agx
Halogen
M Carbon, hydrogen or sulphur present in the
“. Volume of To H,SO, left unused = 77 mL
compound will be oxidised to CO,, H,O and H,SO,
M respectively.
Volume of To H,50O, used for neutralisation of NH,
100 — 77 = 235 mL
c+20 —S& , co,
Now, 2H+Q — 5 HO
M
23 mL of 75 H,SO, = 2x 23 mL of
M
> NH; S+H,O+30 —*" , H,SO,
Procedure. Asmall amount (about 0.5 g) of silver
M nitrate and 5 cm® of fuming HNO, are taken in the
46 mL of io NH, Carius tube. A small amount of weighed organic
Now 1000 mL of 1 M NH, contain nitrogen = 14 g substance taken in a small tube, is also placed carefully
into the Carius tube. The tube is placed in an outer
M iron jacket and is heated in a furnace at 553-563 K
46 mL of To NH, contain nitrogen
ORGANIC CHEMISTRY: BASIC PRINCIPLES AND TECHNIQUES

for 5-6 hours when carbon, hydrogen and sulphur tube as described under the estimation of halogens.
present in the compound are oxidised respectively to Upon heating, carbon and hydrogen are oxidised to
carbon dioxide, water vapours and sulphuric acid. The carbon dioxide and water vapours respectively.
halogen (X) present in the compound is converted to Sulphur changes to sulphuric acid. After heating for
silver halide which is precipitated. 5—6 hours, the heating is stopped. The Carius tube is
then broken and the contents are transferred to a
Carius tube |ron tube
beaker. Barium chloride solution is now added
dropwise when sulphuric acid gets precipitated as
Organic substance barium sulphate. The precipitate is filtered, washed,
dried and is finally weighed.
Calculations :
Let the mass of organic compound = w g
Furnace
ee ee ee ee Mass of BaSO, formed = m g
BaSO,=S
Fig. 25. Cartus method for the estimation 137 +32 +64 = 32
of halogens.
= 233
Calculations 233 g of BaSO, contain sulphur = 32 g
32
Let the mass of organic compound be w g m g of BaSO, will contain sulphur = 9 =8
Mass of silver halide formed = mg
Percentage of sulphur
Now , AgX = X
= Masses
of sulphur
108 + X parts by weight of silver halide contain
X parts by weight of halogen (X is its atomic mass). Mass of organic compound
(108 + X) g of silver halide give halogen = X _ 32a 100
m g of silver halide will give halogen 233 Ww
Xx Percentage of S=
(ern 32 Mass of BaSO, formed ~100
Percentage of halogen 233 * Mass of compound taken
Mass of halogen 5. Estimation of Phosphorus
x 100
~ Mass of organic compound
For the estimation of phosphorus, the given
Xxm , 100 organic compound is heated strongly with fuming
_ (108 -+X) w nitric acid. The phosphorus in the compound is
oxidised to phosphoric acid. It is treated with magnesia
mixture (a solution containing magnesium chloride,
ammonium chloride and a little of ammonia). A
Peet ofalge= precipitate of magnesium ammonium phosphate
( Atomic mass! Mass of \ MgNH,PO, is formed. This is filtered, washed, dried
| of halogen } Quiver halide) x 100 and is then ignited to give magnesium pyrophosphate
{108 + At.mass Mass of organic . (Mg,P,O,). From the weight of Mg,P,0_,, phosphorus
of halogen substance can be estimated.
4, Estimation of Sulphur 2P + 50 (from HNO,) —** > P.O,
in organic
Sulphur is estimated by Carius method.
compound
Principle. A known mass of the organic compound
is heated with fuming HNO, in a sealed tube when P,O, + 3H,0 —““> H,PO,
sulphur is quantitatively converted into sulphuric acid. ia acid
It is then precipitated with barium chloride as barium
H,PO, + Magnesia mixture ———> Mg NH,PO,
sulphate. The precipitate is filtered, washed, dried and
(NH,Cl + MgCl,) Magnesium ammo-
weighed. From the weight of BaSO, formed, the
nium phosphate
percentage of sulphur can be calculated. The main
(White ppt)
reactions are :
__
HNO; ,
o+ HO + 30) —— + Bs0, 2MgNH,Po, —He2t_, Heat
Mg,P,0,+ 2NH,
H,SO, + BaCl, ———~+ BaSO, Magnesium + H,O
ppt. pyrophosphate
Procedure. About 0.2 g of organic compound is (white ppt)
heated with about 5 cm® of fuming HNO, in a Carius
Slane.
Calculations : 6. Estimation of Oxygen
MODERN'S abc + OF CHEMISTRY-AI

Let the mass of the organic compound = wg There is no direct method for the estimation of
oxygen in a given organic compound. It is estimated
Mass of magnesium pyrophosphate formed = m g.
by subtracting the sum of the percentages of all other
Mg,P,0, = 2P elements in the compound from 100.
2x 2442x31+7x16=2x3l1
Percentage of oxygen
= 222, 62
= 100 — (% of all other elements)
222 g of Mg,P,O, contain phosphorus = 62 g However, oxygen can also be estimated directly
as explained below :
m g of Mg,P,O, contain phosphorus = 999 *™s A definite amount of an organic compound is
decomposed by heating in a stream of nitrogen gas.
Percentage of phosphorus
The mixture of gaseous products containing oxygen
_ Mass of phosphorus 400 are passed over red hot coke when all the oxygen gets
Mass of organic compound converted to carbon monoxide. The mixture is then
passed through warm iodine pentoxide (I,0,) when
62m ; 100 carbon monoxide is oxidized to carbon dioxide and
- 292 w producing iodine. The percentage of oxygen can be
Percentage of phosphorus calculated from the amount of carbon dioxide or iodine
produced.
62 Mass of magnesium pyrophosphate
|
«100
— Organic compound —tieat _, O, + Gaseous products
~ "299 * Mass of organic substance
2C + O, ———> 2CO
Alternatively, phosphorus may also be Carbon monoxide
estimated as ammonium molybdate. In this method, 1,0, + 5CO ———+ I, + 5CO,
by heating a known mass of an organic compound with
Let the mass of organic compound taken = x g
fuming acid, the phosphoric acid formed is precipitated
Mass of carbon dioxide formed = m g
as ammonium phosphomolybdate, (NH,),PO,.12MnO,
44 ¢g of carbon dioxide contains 32 g oxygen
by adding ammonia and ammonium molybdate.
= 32xm
H,PO, + 12(NH,),MoO, + 21 HNO, —**., m g of carbon dioxide contains oxygen
(Ammonium molybdate)
Percentage of oxygen = 32 %*100
(NH,),;P0,.12Mo0O, + 21NH,NO, + 12H,O 44Axx
Ammonium phosphomolybdate
These days the estimation of elements in an
(yellow ppt)
organic compound is done by using only micro-
The precipitates of ammonium phosphomolybdate quantities of substances with the help of automatic
thus formed are filtered, washed, dried and weighed.
experimental techniques. The elements carbon,
Calculations : hydrogen and nitrogen present in a compound are
Let the mass of organic compound = wg determined with the help of an instrument called
Mass of ammonium phosphomolybdate formed CHN elemental analyzer. In this technique very
= mg
small quantities (about 1-3 mg) of the substance
(NH,),PO,12Mo0O, = P are required and the results are obtained on a screen
3(14+4)+31+4x 16+ 12(96+3 x 16) 31 within a short time. The detailed discussion of these
EAST 31 methods is beyond the scope of the present syllabus.
1877 g of (NH,).PO,. 12MoO, contain phosphorus
=slg
m g of (NH,),PO,. 12MoO, contain phosphorus
= 31 * FPL O Example 39.
1877
0.15 g of an organic compound gave 0.12 g of AgBr
31m 100 by Carius method. Find the percentage of bromine
Percentage of phosphrus = o—
in the compound.
Percentage of phosphorus Solution : Mass of AgBr formed = 0.12 ¢
AgBr = Br
31 : Mass of amm. phosphomolybdate : 188 80
= 100
— 18siTy Mass of organic substance 188 g of AgBr contain bromine = 80 g
80 x 0.12
0.12 g of AgBr will contain bromine = = 0.051 g
188
ORGANIC CHEMISTRY: BASIC PRINCIPLES AND TECHNIQUES

. 0.051 Q 31. 0.2585 g of an organic compound containing iodine


Percentage of bromine = ——— x 100 was heated with excess of strong nitric acid and
0.15
silver nitrate in a Carius tube. The precipitate of
= 34.04% silver iodide was filtered, washed and dried. Its
L} Example 40. weight was found to be 0.3894 g. Calculate the
0.395 g of an organic compound by Carius method percentage of iodine in the compound.
for the estimation of sulphur gave 0.582 g of BaSO,,. Q 32. 0.4 g of an organic compound was kjeldahlised and
Calculate the percentage of sulphur in the compound. the ammonia evolved was absorbed into 50 mL of
M/4 sulphuric acid solution. The residual acid
Solution :
solution was diluted with distilled water and the
Mass of BaSO, = 0.582 g volume was made upto 150 mL. 20 mL of this
We know BaSO, = 58 diluted solution required 31 mL of M/20 NaOH
230 of solution for complete neutralisation. Calculate the
233 g of BaSO, contain sulphur = 32 g percentage of nitrogen in the compound.
0.582 g of BaSO, contain sulphur = — x 0.582 Q 35. An organic compound containing C, H, N and O
gave the following results on analysis :
Wt. of sulphur (i) 0.25 g ofthe compound gave 0.368 g of CO, and
Percentage of sulphur = x 100
Wt. of compound 0.205 g of H,O
(it) The same weight of the compound gave 31.2 ml
82 x 0.582 of N, at 5.T.P.
x 100 = 20.24%
~ 233 x 0.395 Calculate the percentage composition of the
compound.
Q 34. 0.4 ¢g of an organic compound containing phosphorus
gave 0.555 g of magnesium pyrophosphate by usual
Q@ 28. In sulphur estimation, 0.157 g of an organic analysis. Calculate the percentage of phosphorus in
the compound.
compound gave 0.4813 g of barium sulphate. What
is the percentage of sulphur in the compound ? Q 35. 0.12 g of an organic compound containing
phosphorus gave 0.22 gof Mg,P,O- by usual analysis.
Calculate the percentage of phosphorus in the
@ 29. 0.25 g of an organic compound ‘X’ containing carbon, compound.
hydrogen and oxygen was analysed by the
combustion method. The increase in mass of cale1um - Answers to Practice Problems
chloride tube and potash bulbs at the end of the 42.10%
experiment was found to be 0.15 g and 0.1837 g
C = 20.04%, H = 6.66%, O = 73.30%
respectively. Calculate the percentage composition
of the compound. 15.1%

. 0.2325 g of an organic compound was analysed for 81.4%


nitrogen by Duma’s method. 31.7 mL of moist 46.8%
nitrogen was collected at 25°C and 755.8 mm C=40 14%; H=9.11: N= 15.6%: 0 = 35.15%
pressure. Calculate the percentage of nitrogen in the
sample (aq. tension of water at 25°C and 23.8 mm). 38.75%
51.20%
Hints & Solutions on page 119

CYUCSTIONS 32!
Conceptual
How will you separate a mixture of o-nitro-phenol and p-nitrophenol ?
A mixture of o-nitrophenol and p-nitrophenol can be separated by steam distillation. o-nitrophenol
being less volatile distils over along with water while p-nitrophenol being non-volatile remains in
the flask.

How will you separate benzoic acid from a mixture of naphthalene and benzoic acid ?
Benzoic acid can be separated from naphthalene by treating the mixture with hot water. Benzoic acid
dissolves but naphthalene remains insoluble. On cooling, pure benzoic acid crystallises out.
Suggest methods for the separation of the following mixtures:
(a) a mixture of liquid A (b.p. 365 K) and liquid B (b.p. 355 K)
(6) a mixture of liquid C (b.p. 348 K) and liquid D (b.p. 478 K).
(a) Fractional distillation because the boiling points of the two liquids differ by only 10°.
(b) Simple distillation because the boiling points of the two liquids differ much.
Slane. Q. 4.
MODERN'S

Will a precipitate of AgCl be formed on treating CHCl, with AgNO, ? Explain.


abc + OF CHEMISTRY-Al

Ans. The precipitate of AgC] will not be formed because CHCl, is a covalent compound and does not ionize to
give CI ions. Therefore, CHCl, does not react with Ag* ions (from AgNO,) to form ppt. of AgCl.
Q. 5. Name two compounds which are purified by sublimation.
Ans. Camphor, naphthalene.
Q. 6. What is the basic principle of chromatography ?
Ans. The method of chromatography is based on the difference in the rates at which the components of a mixture
are adsorbed on a suitable adsorbent.
Q. 7. What conclusions would you draw if the Lassaigne's extract gives a blood red colouration with
FeCl, ?
Ans. It indicates that the compound contains both N and 8. During fusion, sodium thiocyanate 1s formed which
gives blood red colouration.
38 NaSCN + FeCl,—-> _‘ Fe(SCN), + 3 NaCl
Blood red
Q. 8. Name a method suitable for separating the mixture of iodine and sodium chloride.
Ans. Sublimation is used to separate the mixture of iodine and sodium chloride because iodine sublimes
readily.
Q. 9. Why is nitric acid added to sodium extract before ading silver nitrate solution for testing
halogens ? N.C.E.R. T. |
Ans. Sodium extract is boiled with nitric acid to decompose NaCN and NaS if present.

NaCN+HNO, ——> NaNO, + HCNT


Na,S+HNO, —> 2NaNO, + H,ST
If cyanide and sulphide ions are not decomposed, they will interfere with the test by forming precipitate
of AgCN and Ag,S.

NaCN + AgNO, ——> AgCN+ NaNO,


White ppt.

Na,5S + 2AgNO, —— Ag,S + 2NaNO,


Silver sulphide
(Black ppt).
Q. 10. Why is it necessary to use acetic acid and not sulphuric acid for acidification of sodium extract
for testing sulphur by lead acetate test ?
Ans. For testing sulphur, the sodium extract is acidified with acetic acid because lead acetate 1s soluble and does
not interfere with the test. If H,SO, were used, lead acetate will react with it forming white precipitate
of lead sulphate.

(CH,COO),Pb+H,SO, — > PbSO, + 2CH,COOH


White ppt.
. The R, value ofA and B in a mixture determined by TLC in a solvent mixture are 0.65 and 0.42
respectively. If the mixture is separated by column chromatography using the same solvent
mixture on a mobile phase, which of the two components A or B will elute first ? Explain.
Ans. R, value of A is 0.65, therefore, it is less strongly adsorbed as compared to compound B which has R, value
of 0.42. Therefore, A will be eluted first.
Q. 12. Amixture contains 71 per cent of calcium sulphate and 29 per cent of camphor. Name a suitable
technique of separation of the components of the mixture.
Ans. The mixture can be separated by the process of sublimation. Camphor will sublime whereas calcium
sulphate will be left behind.
Q. 13. Suggest a suitable technique of separating naphthalene from kerosene present in a mixture.
Ans. By differential extraction.
Q. 14. Write the molecular formula of iron (IIT) hexacyanidoferrate (II).
Ans. Fe, [Fe(CN),l..
ORGANIC CHEMISTRY: BASIC PRINCIPLES AND TECHNIQUES

Hints & Solutions for —


SOLUTION FILE Practice Problems
© 28. Molecular mass of BaSO, = 137 + 32464 AgI = I
233 235 g of AgI gives iodide = 127 ¢
233 g of BaSO, contains S 32 ¢ ee 127
0.3894 g of AgI gives iodide = 935 * 0.3894
32
0.4813 g of BaSO, contains S x 0.4813
933 oe 227. 0.3894
= 0.0661 g 0 oflo aeaamNER)* 0.2585
= 81.4%
0.0661
% of Sulphur x 100
0.157 N
© 32. Volume of oO H,S0O, taken = 50 mL
= 42.10%
N
Wt. of CO, formed = Increase in potash bulbs Let volume of > H,sO i left unused = V mL
= 0.1837 ¢ V mL of acid solution was diluted to 150 mL and titrated
Wt. of organic compound = 0.25 against N/20 NaOH
12 = (0.1837 N
%C = — = x 100 20 mL of diluted solution = 31 mL of 20 NaOH
A4 0.25
= 20.04% 30x1 ~= 8
Wt. of H,O produced= Increase in CaCl, Normality of diluted solution = x20
20 20 = 100
tube ol
= 0.15¢ Normality of 150 mL of acid solution = Z00 N

2 = 0.15 N a1 ;
%o H — x —— x 100 V mL of sy H,sS0, =150 mL of a00 % diluted acid
18 0.25
6.66%
150x31x2
isa 23.25 mL
% ofO 100 — (20.04 + 6.66)
73.50% N
Volume of ry H,S0O, left = 23.25 mL
© 30. Calculation of volume of nitrogen at S.T_P. Volume of acid used up for ammonia
Experimental conditions = 50 -— 23.25 mL
= 26.75 mL
P, = 755.8— 23.8 = 732.0 mm
V, = 31.7 mL N N
26.75 mL of > H,SO, = 26.75 mL of > NH,
T, = 25 + 273 = 298 K
solution
S.T.P. conditions N
P, = 760 mm Wt. of nitrogen in 26.75 mL of ey NH,

Me =7
T, = 273 K —2= * 26.75
1000 ~ 0.1872
7° '"8
Applying gas equation, 0.1872
% N= x 100 = 46.8%
0.4

Tae © 33. (1)Percentage of C


-732x31.7 760xV, _12 , _Wt.of
CO, 100
298 273 44 Wt. of compound

_ 2 0.368, 104
or V, = 28 mL

28x Volume of N, at 5.T.P. x 100 44 0.25


f N=
ne 22400 x mass of compound = 40.14%

— 28x 28100 - 9 Wt. of water 100


= 32400x0.2325 = 1%! (ii) Percentage of H = 18 * Wt. of compound

© 31. Wt. of organic compound = 0.2585 g 2 ~~0.205 x100 =9,116¢


eet
Wt. of silver iodide formed = 0.3894 g 18 * 0.25 a1 ie
es 12/120 MODERN'S abc + OF CHEMISTRY-XI

(iit)
iii) Percentage ge of N | orp = : 02, : Wh ofMgsP.r 199
28 Vol. of Ng at 8.T.P. «100 0 0r* = 222 * Wt. of compound
= 22400 ~ Wt. of compound 62 ~=0.555
| = 302 * 940 * 100 = 38.75%
=
28 rE
x —— 100 © 35.
Wi. Wt. of of organic. compound d =_ 0.12g
22400 80.25 2Mg = Mg, P,0,
= 15.6% 62 - 22
Percentage of O = 100— (40.14 + 9.11 + 15.60) 222 g of Mg, P,O, contains Mg = 62
= 35.15% = 62
© 34. 2P = Mg, P,O, 0.22 g of Mg,P,O, contains Mg = 599 * 0.22
62 229 = 0.06144 g
i. P= «100 = 91.20 %

@ Homologous series: A series of similarly constituted compounds in which the members possess the same functional
group and have similar chemical characteristics.
@ Isomers: The compounds which have the same molecular formula but differ in their physical and chemical properties.
e Inductive effect: The process of electron displacement along the chain of carbon atoms due to the presence of a
polar covalent bond at one end of the chain.
e Electromeric effect: The phenomenon of movement of electrons from one atom to another in multi bonded atoms
at the demand of attacking regent.
@ Resonance: If a molecule can be represented by two or more structures, none of which is capable of describing all
the known properties of the compound, then the actual structure is intermediate or resonance hybrid of these
structures.

e Hyperconjugation: Conjugation between electrons of single (H—C) bond with multiple bonds.
@ Electrophiles: The positively charged or neutral species which are deficient in electrons.
@ Distillation: The process is used for the purification of liquids which boil without decomposition and contain
non-volatile impurities.
@ Fractional distillation: The process is used to separate a mixture of two or more miscible liquids which have
boiling points close to each other.
@® Steam distillation is used for separation and purification of liquid which is appreciably volatile in steam, from
non-volatile components of a mixture.
® Differential extraction: The method is used to separate a given organic compound present in aqueous solution by
shaking with a suitable organic solvent in which the compound is more soluble than water.
@ Chromatography: Based on the differences in rates at which the components of a mixture are adsorbed as a
suitable adsorbent.
ORGANIC CHEMISTRY: BASIC PRINCIPLES AND TECHNIQUES qanat}

QUICK CHAPTER ROUND UP)


ORGANIC COMPOUNDS

Homologous series } - Functional group


« series of similar compounds e determines the characteristics
e differ by —CH, group | of a compound

- ISOMERS

Compounds having same molecular formula but different


physical and chemical properties

Structural isomerism Stereoisomerism

* Chain or nuclear Optical isomerism |


CH,CH,CH,CH, & CH,CHCH,
| ‘H,
CH CH : 4 |
Rotate the plane
pl | polarise
larised
CH, No-¢a oc se light in different directions
* Position H” Ny HH” -* CH
CH,CH,CH,OH & CH,CHCH, iy
cis trans
OH But-2-ene
* Functional Conditions
CH,CH,OH & CH,OCH, * should have double bond
® Metamerism * two groups attached to same C atom of
C,H.COC,H. & CH,COC,H, double bond must be different.
« Tautomerism
cE il —____ fa
G tg Electron Displacement in Covalent Bond

Inductive effect Electromeric effect Hyperconjugation


aa st, * occurs between two atoms joined by * interaction between electrons of 1
_ “7, bond system and adjacent o bonds (single
C= C470 77 CS Cl H—C bonds).
o 28, <8, =o,
-I :--NO,> —CN > —COOH > —-COOR >
F > Cl > Br > OH > -OCH,
> C,H, >
+I :(CH3)3C— > (CH,slg CH- > CH vi > CH.—

Aniline C,H;NH,

Phenol C,H,OH

Homolytic fission gives free radicals Heterolytic fission gives


| Carbocations Carbanions
—C:%— 3 —C +7 | | |
| | en eo a cae lad

Free radicals Carbocations Carbanions


Planar : sp* hybridised carbon | | Planar : sp* hybridised C Pyramidal : sp? hybridised C
CH, < 1° < 2° < 3° o so = |’ =. CH, GHA =. 1° = BPs.

(CgH;), Ct > (C,H,), CH* > (C,H, )CH,* (CgH;),C° > (CgH;). CH” > CgH,CH,-

WWW.JEEBOOKS.IN
nes 12/122 MODERN'S abc + OF CHEMISTRY-XI

Methods of Purification Estimation of Elements


e Filtration
= mixture of naphthalene and urea 12 Mass of CO, % 100
e Recrystallisation 44 Mass of compound
= sugar containing common salt
2 ‘ Mass of HO
e Sublimation * 100
18 Mass of compound
= purification of camphor
e Distillation 28 Vol. of N, at STP
= separation of CHCl, and aniline 22400 Mass of compound x 100 (Duma’s method)
e Fractional distillation
= acetone and methanol
1.4 Molarity of acid x Basicity x Vol. of acid used <100NGteldahl’s method)
e Steam distillation Mass of compound
= o-nitrophenol and p-nitrophenol At. mass of X x Mass of AgX
e Differential extraction Cp NM Ee 100)
(halogen) (108 + At. mass of X) x Mass of compound
= aniline and water
« Chromatography % % = 32 Mass of BaSO 4100
= ortho and para nitroanilines 233 Mass of compound

[Lassaigne’s test is used for detection


of nitrogen, halogen and sulphur.

s\ NCERT FILE \pee Solved


Textbook Exercises TT |
Q. 1. What are hybridisation states of each carbon atom in the following compounds?
CH,=C=0, CH,CH=CH,, (CH,),CO, CH,= CHCN, C,H,
sp*
2 a ont 2 sp? sp” sp” : we
sp* sp sp” sp sp Pp sp sp” sp | . oat
Ans. CH,=C=0O, CH,CH=CH,, CH;—0__CH,;, CH,—CHCN, CS each C is sp” hybridised.
!
O
Indicate the o and x bonds in the following molecules :
C,H,, C,H,5, CH,Cl,, CH, = C = CH,, CH,NO,, HCONHCH,

o%
Hyg o Cl
Ans.
H” Nor

| H ‘=e O HH
Ene 1 en FY i
Topo
4/
Tt
dative
| a
litoe |oa Jo| 2
Pn =a Ne HU N—C—H
nH H O H
Q. 3.

Ans. (i) Isopropylalcohol OH (ii) 2, 3-Dimethylbutanal

(iii) Heptan-4-one , | l
ORGANIC CHEMISTRY: BASIC PRINCIPLES AND TECHNIQUES 12/123 —_—

Q. 4. Give the IUPAC names of the following compounds :

(d) C Br (e) Cl (f) CL.CHCH,OH

Ans. (a) Propylbenzene (b) 3-Methylpentanenitrile (c) 2, 5-Dimethylheptane


(d) 3-Bromo-3-chloroheptane (e) 3-Chloropropanal (f) 2,2-Dichloroethanol

Which of the following represents the correct IUPAC name for the compounds concerned ?

(a) 2,2-Dimethylpentane or 2-Dimethylpentane (6) 2,4,7-Trimethyloctane or 2,5,7-Trimethyloctane

(c) 2-Chloro-4-methylpentane or 4-Chloro-2-methylpentane (d) But-3-yn-1-ol or But-4-ol-l-yne.

Ans. (a) 2,2-Dimethylpentane (6) 2,4,7-Trimethyloctane (c) 2-Chloro-4-methylpentane (d) But-3-yn-1-ol

Q. 6. Draw formulas for the first five members of each homologous series beginning with the following
compounds.

(a) H—COOH (6) CH,COCH, (c) H—CH=CH,

Ans. Please try yourself.

Q. 7. Give condensed and bond line structural formulas and identify the functional group(s) present, if any,
for :

(a) 2,2,4-Trimethylpentane

(6) 2-Hydroxy-1,2,3-propanetricarboxylic acid


(c) Hexanedial

Ans. Refer Solved Example 18. (Page 63)


Q. 8. Identify the functional groups in the following compounds :

CHO | NH,

(a) (b) [> (c)


OMe
OH oO” OCH,CH,N(C,H,), CH=CHNO,

Ans. Refer Solved Example 19. (Page 64)

Q. 9. Which of the two : O,NCH,CH,O- or CH,CH,O- is expected to be more stable and why ?
Ans. Refer Conceptual Questions [z]- @.20 (page 96)
Q. 10. Explain why alkyl groups act as electron donors when attached to a t system.
Ans. Alkyl groups act as electron donors when attached to a n-system because of hyperconjugation as shown below :
H . H* H H
iy (4 _ . : | =
H—C—CH=CH, «<—> H—_C—>CH— CH, <«<—> H , = CH— CH, «<— H—C=CH— CH,
| |
H H H Hq”
Propene
a 12/124 MODERN'S abc + OF CHEMISTRY-XI

Q. 11. Draw the resonance structures for the following compounds. Show the electron shift using curved-arrow
notation
+ +
(a)C,H;OH (6) C,H.NO, (c) CH,;CH=CHCHO (d) C,H.—CHO (e)C,H.—CH, (f) CH,CH=CHCH,
Ans. _ parts (a), (d) and (e) Refer Conceptual Suction) Q).24 ae —
‘en — “0 *- ate,0)

WR. VA RYAG AN Os; A a pe

(c) CH, —CH—=CH—CH0: <—> CH,—_CH==


Pr <—— CH, —CH—CH—cCH—6z

(d) ee ae ay <> CH,—CH—CH=CH,


Q. 12. What are electrophiles and nucleophiles? Explain with examples.
Ans. The positively charged or neutral species which are deficient of electrons and can accept a pair of electrons are
called electrophiles. For example, H,O*, Cl*, NOZ, BF,, AICl,, ete.
The species having an atom having unshared or lone pair of electrons and seeking positive sites are called
nucleophiles.
For example, NH,, H,O, ROR, CN’, X, ete.

Q. 13. Identify the reagents shown underlined in the following equations as nucleophiles or electrophiles :
(a) CH,COOH + HO- —-> CH,COO- + H,O (b) CH,COCH, + CN —> (CH,),C(CN) (OH)
(c) C,H, + CH.CO —> C,H,COCH,
Ans. Refer Solved Example 30. (Page 92)
Q. 14. Classify the following reactions in one of the reaction type studied in this unit.
(a) CH,CH,Br + HS" —> CH,CH,SH + Br- (6) (CH,),C = CH, + HCl —> (CH,),CIC—CH,
(c) CH,CH,Br +HO- —> CH, = CH, + H,O + Br- (d) (CH,)C—CH,OH + HBr —-> (CH,),CBrCH,CH,+ H,O
Ans. Refer Solved Example 29. (Page 91)
Q. 15. What is the relationship between the members of following pairs of structures ? Are they structural or
geometrical isomers or resonance contributors ?
0 H D ‘0H OH
@ pole (b) ye = Co (c) H_C_OH H_C_OH
Ans. Refer Conceptual Questions i] Q.8 (Page 94)
Q. 16. For the following bond cleavages, use curved-arrows to show the electron flow and classify each as
homolysis or heterolysis. Identify reactive intermediate produced as free radical, carbocation and
carbanion.

(a) CH,O—OCH,—> CH,O + OCH, (b) >=0 +-OH—> 4=0+H,0


* .

| E
orX —+ k + Br oC) CT,
"
Br

\
Ans. (a) CH,O—OCH, _Homolysis, CH,O + OCH, ~H
(b) on HF Homolysis, —// C=O +H,0
1] Carbanion

(c) NW Hetrolysis : DD» + Br 4 _Homolysis ,


Br
Carbocation
ORGANIC CHEMISTRY: BASIC PRINCIPLES AND TECHNIQUES 12/125 =>

Q. 17. Explain the terms Inductive and Electromeric effects. Which electron displacement effect explain the
following correct orders of acidity of the carboxylic acids ?
(a) Cl,CCOOH > Cl,CHCOOH > CICH,COOH (6) CH,CH,COOH > (CH,),CHCOOH > (CH,),C.COOH
Ans. For definition Refer Text. (Page 72-73, 77)
(a) As the number of halogen atoms decreases, the overall —I effect decreases so that the tendency to withdraw
electrons from O—H bond decreases. Consequently, acidic character decreases.

ON Seow.
CF—-L--L--0--H >
Some yo
|
ne > CEH-CH,-<0-<-0--H
Cl Cl
Inductive effect (-I) decreases.
(6) As the number of alkyl groups increases, the +I effect increases and therefore, acid strength decreases.

I
CH CHC O4-0H > re aan > CH; C--C 0-H
CH; CH;
+I effect increase
Q. 18. Give a brief description of the principles of the following techniques taking an example in each case.
(a) Crystallisation (6) Distillation (c) Chromatography
Ans. Refer Text. (Page 99, 100, 104)
Q. 19. Describe the method, which can be used to separate two compounds with different solubilities in a
solvent 8S.
Ans. Two compounds with different solubilities in a solvent S can be separated by fractional crystallisation. When a
hot saturated solution of these two compounds is allowed to cool, the less soluble compound crystallises out first
while the other more soluble compound remains in the solution. The crystals of less soluble compound are separated
from the mother liquor. The mother liquor is again concentrated and the hot solution is again allowed to cool when
the crystals of second (i.e., more soluble) compound are obtained. These compounds are again filtered and dried.
Q. 20. What is the difference between distillation, distillation under reduced pressure and steam distillation?
Ans. Distillation means the conversion of a liquid into vapours, which on condensation by cooling gives a pure liquid.
This method is used for purification of liquids which boil without decomposition and contain non-volatile impurities.
This method can also be used for separating liquids having sufficient differences in their boiling points.
Distillation under reduced pressure also involves conversion of a liquid into vapours by heating followed by
condensation of the vapours thus produced by cooling under reduced pressure. In this method, the pressure acting
on the system is not atmospheric pressure but is reduced with help of a vacuum pump. This method is used to
purify liquids which decompose below their normal boiling points.
Steam distillation is similar to distillation under reduced pressure even though there is no reduction in the total
pressure acting on the solution. It is used for the separation and purification of a liquid which 1s appreciably volatile
in steam from non-volatile components of a mixture. The mixture of organic liquid and water boils at a temperature
when the sum of the vapour pressure of the organic liquid (p,) and that of water (p,) becomes equal to atmospheric
pressure Le., (p) = p, + Pp.
Q. 21. Discuss the chemistry of Lassaigne’s test. ?
Ans. Refer Text Page 107.
Q. 22. Differentiate between the principle of estimation of nitrogen in an organic compound by (i) Dumas
method and (ii) Kjeldahl’s method
Ans. In Dumas method, a known mass of an organic compound is heated with excess of CuO in an atmosphere of CO,
when nitrogen of the organic compound is converted into N, gas. The volume of N, thus obtained is converted into
N.T.P. and percentage of nitrogen is calculated by using the relation :

28. Vol. of N, at N.T.P.


——$—_—__—<__ * 100
22400 Mass of substance *

In Kjeldahl’s method, a known mass of an organic compound is heated with conc. H,SO, in the presence of
K,SO, (raises the b.p. of H,SO,) and little CuSO, or Hg (catalyst) in a Kjeldahl’s flask when nitrogen present in
the compound is quantitatively converted into (NH,),S5O,. The (NH,),5O, thus obtained is boiled with excess of
NaOH solution to liberate NH, gas which is absorbed in a known excess of standard solution of H,SO, or HCl. The
volume of acid unused is determined by titration against a standard alkali solution. From the volume of acid used,
the percentage of N is calculated from the relation :
Sian. MODERN'S abc + OF CHEMISTRY-AXI

1.4x Molarity of acid x Basicity of acid


oN x Vol. of acid used
‘oO =
Mass of substance taken
Q. 23. Discuss the principle of estimation of halogens, sulphur and phosphorus present in an organic compound.
Ans. Refer Text page 114, 115.
Q. 24. Explain the principle of paper chromatography.
Ans. Refer Text page 106.
Q. 25. Why is nitric acid added to sodium extract before adding silver nitrate for testing halogens ?
Ans. Refer Conceptual Questions Q. 9 (page 118).
Q. 26. Explain the reason for the fusion of an organic compound with metallic sodium for testing nitrogen,
sulphur and halogens.
Ans. The organic compound is fused with sodium metal to convert these elements (N, 5 or halogens) present in the
covalent compound to ionic form.
Q. 27. Name a suitable technique of separation of the components from a mixture of calcium sulphate and
camphor.
Ans. A mixture of CaSO, and camphor can be separated by the following two methods :
(z) Camphor is sublimable while CuSO, is not sublimable. Hence, the mixture can be sublimed to get camphor.
(i) Camphor is soluble in organic solvents such as CCl,, CHC1,, ete. while CaSO, is not soluble. Therefore, when
the mixture is shaken with a solvent, camphor goes into the solution while CaSO, remains as residue. It is filtered
and evaporation of the filtrate gives camphor.
Q. 28. Explain, why an organic liquid vaporises at a temperature below its boiling point in its steam distillation?
Ans. In steam distillation, water and organic substance vaporise together and total vapour pressure becomes equal to
atmospheric pressure 1.e.,
Vapour pressure of organic substance + Vapour pressure of water = Atmospheric pressure.
This means that the organic compound vaporises and gets distilled at lower temperature than its boiling point.
Q. 29. Will CCL, give white precipitate of AgCl on heating it with silver nitrate ? Give reason for your answer.
Ans. The precipitate of AgC] will not be formed because CCl, is covalent compound and does not ionise to give CI ions
to react with AgNO,. CCl, + AgNO, —— > No reaction.
Q. 30. Why is a solution of potassium hydroxide used to absorb carbon dioxide evolved during the estimation
of carbon present in an organic compound ?
Ans. Carbon dioxide is acidic and it reacts with strong base KOH to form potassium carbonate.
2KOH + CO, —~ K,CO,
This results in increase in mass of potassium hydroxide. From the increase in mass of CO, produced, the amount
of carbon in the organic compound can be calculated by using the formula:
rae oe ee 12) Mass of CO, produced
100
44 Mass of substance
Q. 31. Why is it necessary to use acetic acid and not sulphuric acid for acidification of sodium extract for testing
sulphur by lead acetate test?
Ans. Refer Conceptual Questions p | Q. 10 (page 118)
Q. 32. An organic compound contains 69% carbon and 4.8% hydrogen, the remainder being oxygen. Calculate
the masses of carbon dioxide and water produced when 0.20 g of this substance is subjected to complete
combustion,
Ans. Refer Solved Example 31. (Page 110)
Q. 33. A sample of 0.50 g of an organic compound was treated according to Kjeldahl’s method. The ammonia
evolved was absorbed in 50 ml of 0.5 M H,SO,. The residual acid required 60 mL of 0.5M solution of
NaOH for neutralisation. Find the percentage composition of nitrogen in the compound.
Ans. Refer Solved Example 37. (Page 114)
Q. 34. 0.3780 g of an organic chloro compound gave 0.5740 g of silver chloride in Carius estimation. Calculate
the percentage of chlorine present in the compound.
Ans. Mass of substance taken = 0.3780 g
Mass of AgCl formed = 0.5740 g
AgCl = Cl
143.5 30.0
ORGANIC CHEMISTRY: BASIC PRINCIPLES AND TECHNIQUES 12/127 ~~»

143.5 g of AgCl will contain Cl = 35.5 g


«35.5
0.5740 g of AgCl contain Cl] = ——x 0.5740
143.5
= 0.142 g
% Chlorine = 2:142%100 _ 37 570,
0.3780
Q. 35. In the estimation of sulphur by Carius method, 0.468 g of an organic sulphur compound afforded 0.668¢
of barium sulphate. Find out the percentage of sulphur in the given compound.
Mass of organic compound = 0.468 ¢g
Mass of BaSO, formed = 0.668 g
BaSO,=5
Vs esy

233 g of BaSO, contains S = 32 g

0.668 g of BaSO, contains S= — x 0.668

= 0.0917 g

% of sulphur = ee 16h
0.468
= 19.59 %
Q. 36. In the organic compound CH,=CH—CH,—CH,—C=CH, the pair of hybridised orbitals involved in the
formation of : C,—C, bond is :
(a) sp—sp” (b) sp—sp® (c) sp?—sp® (d) sp>—sp*
Ans. When both double bond and triple bond are present in the molecule, the double bond is given preference in
numbering as
1 2 a 4 5 6
CH, = CH—CH,—_CH, —C=CH
C,—C, bond is formed by sp*—sp*
Option (c) is correct.
. In the Lassaigne’s test for nitrogen in an organic compound, the Prussian blue colour is obtained due
to the formation of :
(a) Na,[Fe(CN)¢] (b) Fe,[Fe(CN),],
(c) Fe,[Fe(CN),] (c) Fe,[Fe(CN),l,
Ans. (b) Prussion blue colour is due to the formation of Fe,[Fe(CN),],.
Q. 38. Which of the following carbocation is most stable ?
+ +
(a) (CH,),C.CH, (b) (CH,),C

(c) CH,CH, CH, (d) CH,CH CH,CH,


Ans. Correct choice (b) because order of stability of carbocation is 3° > 2° > 1°.
Q. 39. The best and latest technique for isolation, purification and separation of organic compound is
(a) Crystallisation (6) Distillation
(c) Sublimation (d) Chromatography
Ans. (d) : Chromatography is the latest technique for isolation, purification and separation of organic compound.
Q. 40. The reaction : CH,CH,I + KOH(aq) —~> CH,CH,OH + KI
(a) electrophilic substitution (6) nucleophilic substitution
(c) elimination (d) addition
Ans. Correct choice is (b).
Sian. MODERN'S abc + OF CHEMISTRY-Al

Exemplar Problems TT |
Subjective Questions
Q. 7. Explain, how is the electronegativity of carbon
atoms related to their state of hybridisation in
an organic compound ?
Ans. Electronegativity increases as s character of the
hybridised carbon increases. Therefore, the oder of
electronegativity is sp® < sp” < sp.
I. CH,—CH,—CH,—CH,—OH
Q. 8. Show the polarisation of carbon-magnesium
IL. CH,—CH,—CH—CH, bond in the following structure.
OH
CH, —CH,—CH,—CH,—Mg—X
CH, Ans. CH,—CH,—CH,—CH, <«-Mg—x
Il. CH;—V—CH, Q. 9. Compounds with same molecular formula but
OH differing in their structures are said to be
structural isomers. What type of structural
IV. CH,;—CH—CH,—OH isomerism is shown by
CH,
_— CHs
V. CH,—CH,—O—CH,—CH, CH,—S—CH,—CH,—CH, and CH;—S—CH—__ ogy
Vihar,
VII. CH,—O—CH—CH, Ans. Position isomers.
Q. 10.Which of the following selected chains is
HH;
correct to name the given compound according
© —. Which of the above compounds form pairs of to IUPAC system.
metamers ?
CH,—CH,
. Vand VI or V and VII form pair of metamers because
they differ only in the number of carbon atoms on CH,— none bon
the either side of the ether functional group. CH,
: | beteeeneerernes

. Identify the pairs of compounds which are


functional group isomers. Perce EOE en

. LandV, land VI, I and VII, II andV, I and VI, IT and a Poe COrriyy
.
besese eens eee

VIL; Il and V, ITI and VI, ITI and VII; IV and V, IV and
28S Se eee Te EE

VI and IV and VII are functional isomers.


Pe tte

. Identify the pairs of compounds that represent


position isomerism. errerr tii iiits ttt titi tis

Ans. I and IJ, III and IV and VI and VII. PCE CO ee | LLL

. Identify the pairs of compounds that represent Spt See eee ee Pee eS ee

chain isomerism.
. Tand IU, I and IV, IT and LUI and II and IV. (u,—oH
. For testing halogens in an organic compound
with AgNO, solution, sodium extract nal Janassassseeaal
rrr TTrir
0ee | OT rrr]

(Lassaigne’s test) is acidified with dilute HNQ,. CTELIIIITTTteeeeLiiiiiiiiiiiitliliiiil Gieeeliliilii


ii itt iii iia |

What will happen if a student acidifies the oi


extract with dilute H,SO, in place of dilute CH,—OH
HNO, ?
Ans. The selected chain containing four carbon atoms is
. White ppt of Ag,SO, will be formed and the student correct because it contains both the functional groups
may be mistaken for white ppt. of AgCl due to the i.e., —COOH (carboxyl) and —OH (hydroxyl)
presence of Cl.
Q. 11.In DNA and RNA, nitrogen atom is present in
. What is the hybridisation of each carbon in the ring system. Can Kjeldahl method be used
H,C = C = CH... for the estimation of nitrogen present in these ?
. sp” — sp — sp”. Give reasons.
ORGANIC CHEMISTRY: BASIC PRINCIPLES AND TECHNIQUES

Ans. DNA and RNA have nitrogen in the heterocyclic carbocations in order of increasing stability.
rings. Nitrogen present in the rings (azo group or
nitro group) cannot be completely converted to
se CH HCH CH,
ammonia. Therefore, Kjeldahl method cannot be used CH,
to estimate nitrogen present in DNA and RNA.
The possible carbocations are :
Q. 12.1f a liquid compound decomposes at its boiling
point, which method (s) can you choose for its CH; —CHCH,CH, CH,(H—CHCH,
purification. It is known that the compund is
stable at low pressure, steam volatile and CH, CH,
insoluble in water. I(t") Ii(2°)
Ans. It can be purified by steam distillation.
Note : Answer the questions 13 to 16 on the basis of al Sed ~y “iain
information given below :
H, H,
“Stability of carbocations depends upon the
electron releasing inductive effect of groups IT(3°) IV(1°)
adjacent to positively charged carbon atom Stability of carbocations decreases in the order : 3° >
involvement of neighbouring groups in 2° > 1°. Out of (IV) and I (both 1°), the electron
hyperconjugation and resonance.” withdrawing group of CH, (+I) effect, decreases with
Q. 138.Draw the possible resonance structures for distance and therefore, carbocation (IV) is more
stable than carbocation (1).
CH,O—CH, and predict which of the Order of increasing stability I < IV < II < IIL.
structures is more stable. Give reason for your Q. 17.Three students, Manish, Ramesh and Rajni
answer. were determining the extra elements present
=e - | + in an organic compound given by their teacher.
Ans. CH,—O~CH, <~>CH,—O=CH, They prepared the Lassaigne’s extract (L.E.)
(I) (IT) independently by the fusion of the compound
Structure II is more stable because every atom has
with sodium metal. Then they added solid
complete octet of electrons.
FeSO, and dilute sulphuric acid to a part of
Q. 14.Which of the following ions is more stable ? Lassaigne’s extract. Manish and Rajni obtained
Use reasonance to explain your answer.
@ . prussian blue colour but Ramesh got red colour
CH, as CH, only. They were surprised and went to their

CT : teacher and told him about their observation.


Teacher asked them to think over the reason
A B for this. Can you help them by giving the reason
Ans. Out of these two structures, structure
Ais more stable for this observation. Also, write the chemical
than B. This is because, there 1s resonance in structure equations to explain the formation of
A but no resonance is possible in structure B. compounds of different colours.
Q. 15.The structure of triphenylmethyl! cation is Ans. In Lassaigne’s test, SCN ions are formed due to the
given below. This is very stable and some of its presence of sulphur and nitrogen both. These give
salts can be stored for months. Explain the red colour with Fe** ions.
cause of high stability of this cation. Fe** + 3NaSCN —-> Fe(SCN), + 3Na™
Ferric thiocyanate
(Red colouration)
7 C
This happens when fusion is not carried out in the
excess of sodium. However, if excess of sodium metal
Ans. Due to resonance, the positive charge can move at is used, the sodium thiocyanate initially formed
both the o-and p-positions of each benzene ring and decomposes to form sodium cyanide and sodium
therefore, nine canonical structures are possible. sulphide as :

oR dR =
NaSCN + 2Na —-+ NaCN + Na,S
Sodium cyanide formed gives prussian blue colour
due to the formation of ferric ferrocyanide or iron
(IIT) hexacyanoferrate (ID), Fe,[Fe(CN),]..
Q. 18.Name the following compounds :
O NO,
+) a Other canonical structures
(i) 20 (ii) Oy
Q. 16. Write structures of various carbocations that can
be obtained from 2-methylbutane. Arrange these
hizo. MODERN'S abc + OF CHEMISTRY-Al

Q| CH, (ii) CH,CH = CH CH, NH, does not contain lone


pair of electrons on the N atom in conjugation
Ans. (i) bu, 7h br ‘On fu-bu,— on,
a 1
4 526 7
with the z-electrons of the double bond. Therefore,
resonance is not possible.
CH,
Q. 24.Why does SO, act as an electrophile ?
CH, Ans. Three highly electronegative oxygen atoms are
3-Ethyl-4-methylhept-5-en-2-one attached to sulphur atom. This makes sulphur atom
electron deficient. Due to resonance also, sulphur
NO,
acquires positive charge. Both these factors make
SO, an electrophile.
, os
(iz) : 3-Nitrocyclohex-1l-ene

On yA NY VEN KS
Q. 19. Write structural formulae for the following : o” S070 Gy J*o
(4) 1-Bromoheptane Q. 25. Which of the following resonance structure for
propenal is more stable ?
(tz) 5-Bromoheptanoic acid
+ =
Ans. (1) CH, — CH, - CH, - CH, — CH, - CH, —- CH,Br CH, = CH-CH=0O <——> CH,-CH=CH-O
(uz) CH, —-CH, -CH-— CH, — CH, —- CH, —- COOH (A) (B)
|
Br
Ans. CH, =CH—CH=0
<—> CH, -CH = CH-0
(A) (B)
Q. 20. Draw the resonance structures of the following Ais more stable than B.
compounds : Q. 26. By mistake, an alcohol (boiling point 97°C) was
(i) CH, = CH — Cl: (ii) CH, = CH—CH = CH, mixed with a hydrocarbon (boiling point 68°C).
(iii) CH, = CH—¢ = 0 Suggest a suitable method to separate the two
compounds. Explain the reason for your choice.
H Ans. Simple distillation can be used because the two
compounds have a difference of more than 20° in
Ans. (1) CH,+ CH¥ Ci: <> CH,—CH = Cl:
vis = +
an
their boiling points and therefore, both the liquids
can be distilled without any decomposition.
Gi) cH, £cHNcu LOH, <> Q. 27.Which of the two structures (A) and (B) given
+ =

CH,—CH = CH
_“GHG below is more stabilised by resonance? Explain.
LL ©
CH,COOH and CH;COO
, +
(vit) cu, £cHN —H <> CH,—CH = C—H (A) (B)
re va
| | CH C vf
O
Q. 21. Identity the most stable species in the following
s. CH,—C 4, <> gs,
sets of ions giving reasons :

(@) CH, , CH,Br, CHBr, , CBr, (A) |


H -s

(ii) CH; , CH,Cl , CHCl, , CCl, CH aNY <> CH


a aae
Ans. (a) C-Cl, is most stable because Cl is more i Sg:
electronegative than hydrogen. On replacing (B)
hydrogen by chlorine, negative charge on C is Structure B is more stable because it does not involve
reduced and the species becomes stable. charge separation.

(6) CH, is most stable. The replacement of H by Br
increases positive charge on carbon atom because »> Long Answer Questions Carrying 5 marks <q
Br is more electronegative than H and
consequently the species becomes less stable. Q. 28.What is meant by hybridisation ? Compound
CH, = C = CH, contains sp or sp”? hybridised
Q. 22.Give three points of differences between
carbon atoms. Will it be a planar molecule ?
inductive effect and resonance effect.
Ans. For hybridisation, refer units 5. No, this molecule is
Ans. Refer (Text Page 78) not a planar molecule.
Q. 23. Which of the following compounds will not exist
as resonance hybrid. Give reason for your
answer :
(i) CH,OH (it) R—CONH,
(iit) CH,CH = CHCH,NH,
Ans. (i) CH,OH : It does not contain n-electrons and Central carbon atom is sp hybridised and its two
therefore, cannot exist as resonance hybrid. unhybridised p-orbitals are perpendicular to each
other. The p-orbitals in one plane overlap with one
ORGANIC CHEMISTRY: BASIC PRINCIPLES AND TECHNIQUES

of the p-orbitals of left terminal carbon atom and Liquid B and C will be separated by fractionating
the p-orbital in other plane overlaps with p-orbital column (Fig. 13). On fractional distillation, liquid B
of right side terminal carbon atom. As a result, the will distil over first and then liquid C.
position of two terminal carbon atoms and the Q.32.Draw a diagram of bubble plate type
hydrogen atoms attached to them are fixed in planes fractionating column. When do we require such
perpendicular to each other. Therefore, the pair of
type of a column for separating two liquids.
hydrogen atoms attached to terminal carbon atoms Explain the principle involved in the
are present in different planes and hence molecule separation of components of a mixture of
is not planar. liquids by using fractionating column. What
Q. 29. Benzoic acid is an organic compound. Its crude industrial applications does this process have?
sample can be purified by crystallisation from Ans. The bubble plate type fractionating column is shown
hot water. What characteristic differences in below :
the properties of benzoic acid and the impurity
make this process of purification suitable ?
. Benzoic acid can be purified from hot water because
of the following characteristics :
(_) Benzoic acid is more soluble in hot water but
less soluble in cold water.

(ii) Impurities present in benzoic acid are either Condenser

insoluble in water or are more soluble in water


to such an extent that when a hot saturated
solution of benzoic acid is cooled, the impurities Low boiling
remain in the solution. Since impure benzoic acid x liquid
satisfies both these conditions, it can be purified Bubble
plate
by crystallization.
Q. 30. Two liquids (A) and (B) can be separated by Hi boiling
the method of fractional distillation. The
=e
raction
boiling point of liquid (A) is less than boiling In this, the long tower is divided into a number of
point of liquid (B). Which of the liquids do you compartments by means of shelves having openings.
expect to come out first in the distillate ? The openings are covered with caps called bubble
Explain. caps. Each shelf has an overflow pipe which keeps
Ans. Refer Text, fractional distillation page 101. the liquid to a certain level and then allows the extra
Liquid A with lower boiling point will distil first liquid to trickle down to the lower shelf. This type of
and liquid B with higher boiling point afterwards. column is used for continuous separation of bulk
Q. 31. You have a mixture of three liquids A, B and C. quantities of liquids. For example, this method 1s used
There is a large difference in the boiling points for distillation of fermented liquid for the
of A and rest of the two liquids £.e., B and C. manufacture of rectified spirit.
Boiling points of liquids B and C are quite
Principle of fractional distillation. Refer Text Page 101.
close. Liquid A boils at a higher temperature
than B and C and boiling point of B is lower Applications (i) It is used to separate different
than C. How will you separate the components fractions of crude oil in petroleum industry such as
of the mixture. Draw a diagram showing set gasoline, diesel oil, kerosene oil, lubricating oil, etc.
up of the apparatus for the process.
(iz) It can be used for the separation of acetone and
. Since the boiling point of liquid A is much higher
methanol from pyroligneous acid obtained by
than those of liquids B and C whose boiling points
destructive distillation of wood.
are quite close, therefore, liquid A will be separated
by simple distillation using the apparatus as shown Q. 33. A liquid with high boiling point decomposes
in Fig. 11. Since the boiling points of liquids B and on simple distillation but it can be steam
C are quite close but are much lower than that of distilled for its purification. Explain how is it
liquid A, therefore, mixture of liquids B and C will possible ?
distil together leaving behind liquid A. On further Ans. Refer steam distillation, Text, Page 102.
heating, liquid A will distil over.
S ianse. MODERN'S abc + OF CHEMISTRY-Al

Memory TEST
W

i, (CAC 1s stable than (CH,),C™.


“a Say True or False
8. IUPAC name of CH, = C(CH,) - CHOH — CH,CHO
. Hexane has four structural isomers. ——e
. In CH,=C=CH,, all the carbon atoms are sp" 9. According to IUPAC nomenclature, alkyl cyanides
hybridised. are named as ............
. Sulphur in an organic compound can be estimated by . Camphor, naphthalene, ete are purified by ..........
Dumas method. method.

. (CH,),C* is more stable than CH3CH3.


. -NO, group has —I (inductive) effect. 1. Beilstein test is used for detection of halogens/
. Amides and oximes show functional isomerism. nitrogen.
oF
12
& . The functional isomer of methylmethanoate is
propanoic acid. 2. The IUPAC name of ~JUCN is \3:Methyl
| SO. pentanenitrile /2-Methylbutanenitrile.
. The IUPAC name of “I> is 2-Ethyl-3- 3. Pent-l-ene and pent-2-ene are position/ skeleton
methylpent-2-en-1-al.
isomers.
9. Acetone (b.p. 329K) and methyl alcohol (b.p. 338K)
4, AlCl, behaves as electrophile /nucleophile.
can be separated by simple distillation.
5. In Duma’s method for the estimation of nitrogen, the
10. In Lassaigne’s test, prussian blue colour indicates the
gas collected in nitrometer is NH,/ No.
presence of nitrogen in an organic compound.
6. The boiling point of glycerol is 563K but it decomposes
13) Complete the missing links below 563K, therefore, it is purified by steam
distillation /vacuum distillation.
1. Aldehydes and ketones are ..... 1... 190mMers.
7. Homolytic fission of a covalent bond leads to the
2. Bond angle with increase in s-character of formation of free radicals /carbanions.
the hybridised orbital.
3. Among isomeric butenes, Ue beenheeee is the most stable. 8. The hybridisation state of carbon in CgH,CH2
4, If both N and 8 are present in an organic compound,
colouration is obtained in Lassaigne’s test. carbocation is sp7/sp*.
5. Reactions involving migration of an atom or a group 9. The number of structural isomers for hexane is five/
from one atom to another within the same molecule four.
are called reactions. 10. The Vital force theory about organic compounds was
6. Chemical species bearing a negative charge on carbon proposed by Berzelius/ Wohler.
and possessing eight electrons in its valence shell are
called ..........

Answers » eine” Memory TEST WwW


3. trans but-2-ene 4, blood red
Say True or False
®. rearrangement 6. carbanions
1. False : Hexane has 5 structural isomers. 7. more
2. False : Central C is sp hybridised while the terminal 8. 3-Hydroxy-4-methylpent-4-en-1-al
C atoms are sp” hybridised.
9. alkanenitriles 10. sublimation
3. Fase 4, True
5. True 6. True Choose the correct alternative
7. False : Functional isomer of methyl methanoate is
ethanoic acid. 1. Halogens 2. 3-Methylpentanenitrile
8. True 9. False 3. position 4, electrophile
10. True 5. N, 6. vacuum distillation
ea Complete the missing links 7. free radicals 8. sp”
9. five 10. Berzelius.
1, Functional 2. increases
ORGANIC CHEMISTRY: BASIC PRINCIPLES AND TECHNIQUES

i Higher Order Thinking Skills


& Advanced Level }
QUESTIONS WITH ANSWERS

Q. 1. Why is it necessary to use acetic acid and Ans. The organic compound 1s fused with sodium metal
not hydrochloric acid for acidification of sodium to convert N, 8, P and halogens present in the organic
extract for testing sulphur by lead acetate test ? compound to their corresponding sodium salts.
Ans. For testing sulphur, the sodium extract 1s acidified Q.8.Explain why the name butanol is not specific
with acetic acid because lead acetate is soluble and does whereas the name butanone represents a specific
not interefere with the test. However, if HCl is used, lead compound.
acetate will react with it forming white precipitate of lead Ans. Butanol may have —OH group on the first or the
chloride and therefore, it will interfere with the test. second carbon atom as
(CH,COO),Pb + 2HC] —-+ 2CH,COOH + PbCl, CH,CH,CH,CH,OH or CH,CH,CH CH,
Lead acetate White ppt. |
OH
Q.2. What is the cause of geometrical isomerism Butan-1-ol Butan-?2-ol
in alkenes ? Therefore, butanol does not represent a specific compound.
Ans. Alkenes have a double bond (x bond) in their Butanone cannot have the carbonyl group at any other
molecules and there is a restricted rotation around z-bond position : O
which gives rise to geometrical isomerism. !
CH, CCH,CH,
Q. 3. Will CCl, give white precipitate of AgCl on
heating it with silver nitrate ? Give reason for your Therefore, it represents a specific compound.
answer. Q.9. Which continuous chain hydrocarbon is
Ans. The precipitate of AgC] will not be formed because isomeric with 2-methyl-3-ethyl hexane ?
CCl, is covalent compound and does not ionise to give Cl” Ans. Nonane.
ions to react with AgNO,. Q.10. Arrange the following in order of increasing
CCl, + AgNO, ——> No reaction. stability :
CH,CH, , CH=C’, CH,=CH™
Q. 4. Lassaigne's test is not shown by diazonium
salts, though they contain nitrogen. Why ? Ans. In acetylide ion, CH=C , the carbon atom carrying
the negative charge is sp-hybridised and has 50%,
Ans. Diazonium salts (C,H.N,*X") readily lose N,
s-character; in CH,—CH_ ion, the carbon atom carrying the
on heating before reacting with fused sodium metal.
Therefore, these do not give positive Lassaigne's test for negative charge is sp*-hybridised and has 33.3% s-character
nitrogen. whereas in CH,CH, ion, the carbon atom bearing the
Q. 5. A mixture contains benzoic acid and
negative charge is sp* hybridised and has 25% s-character.
nitrobenzene. How can this mixture be separated into Since the s-electrons, on an average are closer to the
its constituents by the technique of extraction using nucleus than p-electrons, therefore, a carbon atom with
an appropriate chemical reagent ? larger s-character can accommodate or stabilize the negative
Ans. The mixture is shaken with sodium bicarbonate charge better than a carbon atom with smaller s-character.
and extracted with ether or chloroform when nitrobenzene In other words, the stability of carbanion increases as the s-
goes into organic layer. Nitrobenzene is then obtained by character of the carbon atom carrying the negative charge
distillation of the solvent. The filtrate is acidified with dil. increases. Now, the s-character of the carbon atom decreases
HC] and the solution is cooled. Filtration gives benzoic acid. as we move from sp to sp” to sp*, therefore, the relative
Q. 6. A mixture contains two components A and stability of the three carbanions follows the order :
B. The solubilities of Aand 5 in water near its boiling HC=C > CH,=CH > CH,CH,”
point are 30 g per 100 mL and 10 g per 100 mL Q.11. Why is the carbocation F,C* more stable than
respectively. How will you separate A and B from this
mixture ? carbocation FC—C% ‘Explain.
Ans, It can be separated by fractional crystallization.
When the saturated solution of the mixture is allowed to Ans. In F,C—C6, the strongly withdrawing
cool, the less soluble component (i.e.,B) crystallizes at first
leaving the more soluble component (A) in the mixture. F,C— group withdraws electrons of the C—C bond towards
Q.7. Why is an organic compound fused with itself. As a result, the positive charge on C gets intensified
sodium for testing halogens, nitrogen, sulphur and
and therefore, F,C—C <ais less stable. On the other hand, in
phosphorus ?
Tiana
carbocation F,C*— lone pair of electrons on each of the three
MODERN'S abc + OF CHEMISTRY-Al

F-atoms overlaps with the empty p-orbital of the carbon (oct, CH, = or
atom carying the positive charge thereby dispersing the
positive charge and thus stabilizing the carbocation FC*
I I
relative to F,C—C <

—l-effect of the F atoms destabilizes the carbocation by


intensifying the +ve charge.
V
Q.14. Choose electrophiles and nucleophiles from
Filled 2p 4. the following :
orbital *s5 AICI1,, CH,O°", H,CC=CH, H , CH 4CH’CH,, NH.,, H-
Ans. Nucleophiles :CH,O , H,CC=CH, H , NH,
Filled 2p Electrophiles : AICI, CH (CH'CH,
orbital Q.15. Draw geometrical isomers of the following :
(a) 1-bromo-3-chlorocyclobutane
(6) 3, 4-dimethylhept-3-ene
(c) 1, 3-dibromocyclobutane

Ao
(d) 1, 4- poacthyleyclohexane
Filled 2p
orbital

Back bonding from 2p orbitals of F to empty 2p orbital


of C disperses + ve charge
Q.12. Write structural formulae for all the
compounds having molecular formula C,H, .
trans-
CH,
Ans. CH,=CHCH,CH,CH, CH, = Gach CH,CH, CH,CH,CH,
On, (b)

HC
a *\,

Hs
CH, cls-
| CH.CH
CH,CH=CHCH,CH, CH,CH = C— CH, 3 a a

H.Cee Oe
CH,CH,CH,
CH, trans-

CH cH, H,C —— CH—CH, H | Br

7
H,C CH H H
Na Z H,C ——CH, (c) oe H

H,C—— CH—CH,—CHy H,C—CH—— CH—CH, Br


CLs
Br
trans

CH, CH, H H H CH,


Q.13. Write metamers of )- OCH,CH,
H,C CH, H,C H
Ans. The various metamers are I and II, I and III, I and
CcLs- trans-
IV, and I and V
ORGANIC CHEMISTRY: BASIC PRINCIPLES AND TECHNIQUES

[carrying 1markR €4
What is meant by +I and —I inductive effect ?
Who developed the concept of hyperconjugation ?
What is self linking property of carbon known as ? Define.
Define it. 31. Write resonating structures of CH,COO7 ion.
Define a functional group. Give functional groups for 32. Identify the functional groups in the following:
(i) aldehydes
(ii) carboxylic acid families.
What is meant by tetravalency of carbon ?
bo Write the structural formula of 1,5-hexadiene. CH = CHNO,
Give the IUPAC name of the compound : oo. Which of the two : But-3-yn-1-ol or But-4-ol-1-yne
represents correct IUPAC name ?

i
ro
le atten
34, Which of the O,N CH, CH,O” or CH,CH,O7 is
expected to be more stable and why ?
30d. Write bond line structural formula for 2-
CH,—CH,—CH, (4-Isobutyl phenyl) propanoic acid.
Write the correct name of an organic compound whose
incorrect name is 2-ethyl- 2-pentene. 36. Give IUPAC name of
What are primary and secondary suffixes ?
37. Select electrophiles from the following :
Define homologous series.
NO,*, C,H,O, Cl’, BF,, HS-
Write the IUPAC name of freon. 38. Why is (CH,),C* more stable than CH,CH,* ?
10. What is dry ice ? Give its one use. 39. Write structural formula for 2-Phenyl propanal.
11. Name the carbide of silicon which is very hard. 40. Which of the two : trans-but-2-ene-or trans- pent-2-
12. How many secondary hydrogens are present in ene is non polar ?
(i) isopentane and (ii) neo-pentane ? 41. What is sublimation ?
13. Write the IUPAC name of isobutylene. 42. What is fractional distillation ?
43. Name two methods for the estimation of nitrogen in
14, Write the formula of acetophenone.
an organic compound.
15. An alkane has two possible names; 2, 3, 5-trimethyl
4d, How will you separate ether from water ?
hexane and 2, 4, 5-trimethyl hexane. Which of these
45. Will you get any precipitate if you add silver nitrate
is correct ?
solution to chloromethane ? If not, why ?
16. Define inductive effect. 46. What is the principle of chromatography ?
17. What are elimination reactions ? Give one example. 47. Why is Lassaigne’s extract boiled with conc. HNO,
18. Define electromeric efiect. before testing for halogens ?
19. What are carbocations ? Arrange primary, secondary 48. How will you separate a mixture of naphthalene and
sand ?
and tertiary carbocation in the order of their stability.
49. How is nitrogen detected in an organic compound ?
20. What are carbenes ? Give one example.
o0. What is the formula of prussian blue ?
21. What are rearrangement reactions ? Give one ex-
ol. What colour will you get in Lassaigne’s test if the
ample.
compound contains both N and § ?
22. Give one example of a positional isomerism. oz. How will you separate two miscible liquids which
23. What species are formed when a covalent bond gets differ in their boiling points by 10°C ?
fissioned ? oo. Why is it necessary to boil the Lassaigne’s extract
24, Name two functional groups containing C = O group. with conc. HNO, before testing for halogens?
o4, Give the principle of steam distillation.
295. Define hyperconjugation.
od. What is the formula of iron (III) hexacyanoferrate
26. What are electrophiles and nucleophiles ? (IT)?
27. What are carbanions ? Give one example. 56. Will CCL, give white precipitate of AgCl on heating
28. Give the examples of substitution reactions. with silver nitrate ? Give reason for your answer.
S Tianse.
57. Why an organic compound vaporizes at a temperature (i) 1,6-Hexadiene
MODERN'S abc + OF CHEMISTRY-Al

(ii) 2-Ethyl-2-pentene.
below its boiling point in steam distillation ? (b) Write the structural formulae of :
Why is a solution of potassium hydroxide used to (i) 2, 3-dimethylbutane
absorb carbon dioxide evolved during the estimation (ii) 4-Methylpentyne.
of carbon present in an organic compound ? 24, Write IUPAC names of the following :
CH
Short Answer Questions <4 |
3

What is homologous series ? Write its important (i) os a a aeH,—C = CH


characteristics.
What is catenation ? Explain as to why carbon has
CH a

maximum tendency for catenation ? (21) CH,—C =C—C= C—CH,


Write the structural formula of n-butane and (tit) CH,—C = C—CH,—CH,
iso-butane. Write the different types of alkyl radicals
formed from these. (iv) (CH,),CH—CH,—CH = CH,
Name the different types of hydrocarbons. Give 20. Write the structural formula of :
structures of two members of each type.
(i) 2, 3-dimethylbutane
How will you decide whether the two given compounds
are the homologous of the same family or (ii) 2-Methylpentane
not ? (iii) 2, 2,4-trimethylpentane.
What are isomers ? Give one example. 26. Name the following compounds according to IUPAC
gs
oI What does IUPAC name stand for ? Give three system :
compounds with no difference in IUPAC and common CH a
names. |
What family is represented by the general formula CH, CH,
C_H,,,_ 5 ? Write the IUPAC and common names of |
the first three members of this family. (i) CH,—CH,—CH—C—CH,
What are inductive and electromeric effect ? How do
these two effects differ ? CH,—CH,—CH,
10. What is hyperconjugation ? What are its important
consequences ?
(ii) |CH,-CH—CH—CH,
11. What are structural isomers ? Explain different types |
of structural isomers with examples. C,H; C,H;
12. Explain the following reactions with one example (zit) (CH,),C—CH,
each.
(i) Elimination reaction
(ii) Substitution reaction (iv) CHS
l
OCs
13. What are carbocations and carbanions? Explain the
stability of primary, secondary and tertiary
C,H;
carbocations and carbanions. 27. Give the IUPAC names of the following compounds :
14, What is hyperconjugation ? How will you explain
stability of methylated alkenes on the basis of (i) CH, = CH—CH,—CH,—OH
hyperconjugation ? Gi) | CH, —CH,—¢—CH,
OH
15. What are free radicals ? Explain its structure and
stability of different types of alkyl free radicles. CH,
16. What are reaction intermediates ? Explain with
aa
examples.
17. Explain the following types of structural isomers with
one example each.
(iii) 0 ian
(¢) Functional isomers (22) Metamers CH a
(iii) Position isomers.
18. What is tautomerism ? How does it differ from (iv) | CH,—CH—CH,
resonance ? |
19. Write short note on structure and stability of
CH 3
carbocations. 28. Write the IUPAC names of the following compounds:
20. What is resonance ? Explain with two examples.
(i) (CH,),CHCH,CH,
21. What is inductive effect ? What is meant by +I and
—I effect ? (i) (C,H.),CHCH,CH,
22. What are addition and rearrangement reactions ? (til) CB) e00n
Explain with examples.
23. (a) Write out the error and write the correct IUPAC CH 3
names of :

WWW.JEEBOOKS.IN
ORGANIC CHEMISTRY: BASIC PRINCIPLES AND TECHNIQUES

29. The common names of some compounds are given ede How is sulphur detected and estimated in an organic
below. Write their structural formula and IUPAC compound ?
names : a6. A mixture contains benzoic acid and nitrobenzene.
(2) Neo-pentane (it) Crotonaldehyde How can this be separated into its constituents by
(iit) Acetaldehyde (iv) Neo-pentyl alcohol the technique of extraction using an appropriate
(v) Iso-butyl alcohol. chemical reagent ?
30). What are electrophiles and nucleophiles ? Give ov. Suggest a suitable technique of separating
examples. naphthalene from kerosene present in a mixture.
ol. What is the structure of carbanion ? How is it oo. What is the difference between distillation,
formed ? Explain the stability of CH.-, 1°, 2°, 3°. distillation under reduced pressure and steam
oe. Draw the formulae for the first three members of each distillation ?
homologous series starting with og. Differentiate between the principle for estimation of
(i) HCOOH = (i) CH,COCH, nitrogen in an organic compound by
Write stick, condensation and bond line structural (i) Dumas method (ii) Kjeldahl’s method
formulae for all the isomers of the molecular
formulae.
Discuss the principle of estimation of halogen,
sulphur and phosphorus present in an organic
(ie AL... (ti) C,H,O, (iii) C,H,O
compound.
Explain why alkyl group act as electron donors when

» <
attached to a 1-system.
Ree erecam Outil carrying 5 marks
What are reactive intermediates ? How are they
generated by bond fission ? 1. Discuss IUPAC general rules for naming branched
What are electrophiles and nucleophiles ? Explain chain hydrocarbon.
with two examples of each.
2. Write the IUPAC names of the following :
Describe the chemistry of Duma’s method for the
(Z) (i2)
estimation of nitrogen. CH, | a
Name the methods to purify impure sample of
(z) glycerol (iz) benzene. CH,—C — C—CH,
What is the principle of vacuum distillation ? Discuss
this method. OH O
OH
Describe the principle and details of the process of
steam distillation. O
How are carbon and hydrogen detected in an organic
compound ?
-CH,
What is separating funnel ? How does it help in
(iii)
CH
separating a mixture of kerosene, oil and water ? a

Discuss the chemistry of Lassaigne’s test. What is resonance ? Draw the resonating structures
Explain the principle of paper chromatography. for the following molecules showing the electron shift
&
RS Describe the method, which can be used to separate using curved-arrow notation :
two components with different solubilities in a (i) CH, = CHOCH, (ii) C;H,CH,*
solvent X.
(iii) C,H,NO, (iv) C,H,OH
Describe Carius method for the estimation of halogens.
(v) CH,CH=CHCHO (vi) CH,CH = CHCH,*
5S Explain one method each for the (i) detection and
What are structural isomers ? Explain different types
(ii) estimation of sulphur in a compound.
of structural isomers with examples.
Describe the chemistry of the Lassaigne’s test for the
detection of nitrogen. What are different types of reaction intermediates
49. What is the difference between distillation and formed by homolytic and heterolytic fission of a
fractional distillation ? covalent bond ? Explain with examples.
50. Discuss the principle and chemical reactions of Write short notes on
Kjeldahl’s method. (it) Electromeric effect (ii) Inductive effect
ol. How will you estimate bromine in an organic (iii) Hyperconjugation
compound ? How are attacking reagents classified ? Explain with
52. Explain the principle of Duma’s method for examples.
estimation of nitrogen in a compound. Discuss four types of organic reactions.
Do. Discuss the principle of distillation, vacuum
Give condensed and bond line structural formulae
distillation and steam distillation. and identify the functional group or groups present if
How is chromatography used for the purification of any :
organic compounds ? (a) Hexanedial
a 12/138 MODERN'S abc + OF CHEMISTRY-XI
(b) 2- Hydroxy - 1,2,3- propanetricarboxylic acid (i) H—-COOH (ii) CH,COCH,
(c) 2-Hydroxy-1,2-diphenylethan-1-one (iii) H-CH=CH,,

(d) Cycloocta-1,5-diene 13. Explain with an example the following terms :


(e) 2-(4-Isobutyl phenyl) propanoic acid. (i) crystallisation (ii) simple distillation (iii) steam
10. Write IUPAC names for the following : distillation (iv) extraction with a _ solvent
(a) HC=C CH=CHCH=CH, (v) fractional distillation.
(b) CH, CO CH,CH,CH,COOH 14. Write the chemical equations involved in the
detection of nitrogen, sulphur, halogens and
_ OH phosphorus present in an organic compound,
(c) (d) (CH,), CHCH (CH,), 15. Discuss Kjeldahl s method for the estimation of
Cc (CH), nitrogen Ina compound.

O 16. How will you detect the presence of sulphur in a


compound ? Discuss the method for its estimation.
(e) Br 17. Explain the following :
(1) Why is nitric acid added to sodium extract before
11. Drawresonance structures forthe following compounds. adding silver nitrate for testing halogens ?
Show the electro shift using curved arow notation : - : ;
*) C.HLOH (i?) C_H.NO (it) Why is an organic compound fused with sodium
(1) Cgis 652 for testing nitrogen, sulphur and phosphorus ?
si : 7 :
(it) CgH;CH, (tv) CH,CH = CHCHO (iii) Why is it necessary to use acetic acid and not
(v) CH,CH=CHCH,* (vi) C,H,;CHO sulphuric acid for acidification of sodium extract
(vii) CH, = CHO CH, for testing sulphur by lead acetate test ?

12. Draw formulae for the first five members of each (iv) CCl, does not give test with silver nitrate
homologous series begining with the following com- solution.
pounds.
e, 6 e

(ompotition Jule Objective Questions


| ee”
Le FF

A4, The structural formula for 2-ethylprop-2-en-1-ol is :

(a) an = CHOH

Organic Compounds CH,


Al. Anelectrophilic reagent is : (6) C,H;CH = CHCH,OH
(a) Electron deficient species (c) CH,—C = CHCH,OH
(b) Electron rich sp&den Ii,
(c) Negatively charged species
d) CH, = CHCHOH
(d)
A Lewis base. errs |
A2. Which of the following carbocation will be the most C,H,
stable ? A5. Which of the following compound contains amide
(a) (CH,), CH* functional group ?
(b) Ph, C* (a) CH,CH,NHCH,
(c) CH, = CHCH,* (6) CH,CH, —-C — NH,
(d) CH,CH,* |
A3. The shape of carbocation is :
(a) Planar (6) Linear (c) sc) ia
(c) Pyramidal (dq) Tetrahedral NH,
A ragngars (d) CH,N(CH,),

Al. (a) A2. (6) Ad. (a) Ad. (d) AS, (0)

WWW.JEEBOOKS.IN
ORGANIC CHEMISTRY: BASIC PRINCIPLES AND TECHNIQUES

A6. The molecule in which the distance between the Al4, Which of the following is not a cyclic compound ?
neighbouring carbon atoms is least 1s: (a) Anthracene (5) Pyrole

(a) CH, (b) CoH, Al5.


(c) Phenol (d) Neopentane
Which of the following compound does not have only
(c) C,H, (d) C,H, one type of hybrid carbon atoms?
A7. The hybrid orbitals that will form the compound (a) HC=C-C=CH
CH,—C = C—CH,—CH, are : (6) CH,—CH,—_CH,—CH,
(a) sp*,sp (b) sp, sp” (c) CH,—C = C—CH,
(d) CH, = CH—CH = CH,
(c) sp?,sp? (d) sp, sp.
Al6. Which of the following hybridization has maximum
A8. The IUPAC name for the structural formula: s-character ?
(a) sp
CH,— CH — CH — CH, (b) sp?
(c) sp?
C,H; C,H;
(d) All have same s-character
(a) 3, 4-dimethylhexane
Al17. The IUPAC name of the compound :
(b) 2, 3-diethylbutane
CH,—C—CH,—CH,
0H is
(c) 2-ethyl-3-methylpentane

(d) Heptane. (a) 3-oxobutan-1-ol


A9. Among the following, the one having longest chain 1s: (b) 1-Hydroxybutan-3-one
(a) neopentane (b) isopentane (c) 4-Hydroxybutan-2-one
(d) 2-oxobutan-4-ol
(c) 2-methylpentane (d) 2, 2-dimethylbutane
A18. During the reaction :
Al10. The IUPAC name of compound having formula
(CH,),C—CH = CH, is: CH,CONH, —2°5-> CH,CN
the hybridisation state of carbon changes from
(a) 3, 3, 3-trimethyl-1-propene (a) sp? to sp (b) sp? to sp”
(b) 1, 1, 1-trimethyl-3-propene (¢) sp* to sp? (d) sp? to sp
Al19. Which of the following is not an isomer of but-1l-yne ?
(c) 3, 3-dimethyl-1-butene (a) But-2-yne
(d) 1, 1-dimethyl-3-butene. (b) But-2-ene
(c) Buta-1, 3- diene
All. The general formula of cycloalkanes 1s :
(2d) Methyl cyclopropene
(a) C,H, (6) C,H, How many o and z bonds are there in the molecule
of tetracyanoethene ?
(c) CH (d) C,,,H
no 2n+1 nm+23
(NC),C = C(CN),
Al12. The IUPAC name of the compound (a) 96,77 (b) 5c, 9x
CH,—CH = CH—C = CH is (c) 5c, 87 (d) 9c, 97
Which out of the following series contains only
(a) Pent-2-en-4-yne nucleophiles ?
(6) Pent-1-en-4-yne (a) 0. SO”
(b) NH,, H,O, R-O-H
(c) Pent-3-en-1l-yne
(c) NH,, H,0O, AICI,
(d) Pent-2-en-5-yne (d) CN’,SO,, OH™
A138. The IUPAC name of the compound Which of the following species has electron releas-
ing (or +I inductive) effect ?
CH,— CH—CH, is (a) -NO, (b) -COOH
(c) (CH,),CH- (d) C,H.
CH(CH,),
Which of the following is not an electrophile ?
(a) 2-isopropylpropane
(a) Carbocation (b) SO,
(b) isobutane (c) Carbene (d) Thioalcohol
(c) 2, 3-dimethyl butane Which of the following is an electrophile ?
(d) 2, 3-dimethylhexane.

Answer
(a) CN- (b) ROH
(c) AICI, (d) NH,

AG. (b) AT. (a) AS. (a ) AS. (c) Al10. (c) All. (a) Al1?2. (c) AIS. (ce) )§«=60Al4, (d)) «ALB. (ee)
Al6.(a) AL7Z. (e) Al18. (d) A19. (b) A20. (d) A21. (b) A22. (c) A223. (d) A24. (c)
Sian. MODERN'S abc + OF CHEMISTRY-Al

Which of the following is an example of elimination (c) 2-methylpentane-2, 4-diol


reaction ? (d) 1, 3, 3- trimethylpropane-1, 3-diol
(a) Chlorination of methane A31. The structure of 4-methylpent-2-en-1-ol is
(b) Dehydration of alcohol
(a) CH,CH,CH = CHCH, OH
(c) Nitration of benzene
(b) (CH,),C = CHCH,CH,OH
(d) Hydroxylation of ethylene
(c) (CH,),CHCH = CHCH,OH
In which of the following, all are nucleophiles ?
(dq) CH,CHOH — CH = C(CH,),
(a) H,O, NO,*, (CH,),N
A32. The arrangement of
(6) HS, BF,, CH,C* =O
(CH,),C—, (CH,),CH—, CH,CH,—
(c) SH ,C,H,O "»(CH,),N

one
(d) NO,*, BF, Cl’ when attached to benzene or unsaturated group in
The reaction
increasing order of inductive effect is
E (a) (CH,),C— < (CH,),CH— < CH,CH,—
(6) CHACH,— 240 F1,,),CH— < (CH,),C—
represents (c) (CH,),CH— < (CH,),C— < CH,CH,—
(a) nucleophilic substitution (d) (CH,),C— < CH,CH,— < (CH,),CH—
(b) electrophilic addition A338. Inthe compound
(c) electrophilic substitution CH,=CH-CH,—-CH,—-C=CH, the C, — C, bond is of
(d) elimination reaction the type
Which of the following reaction represents a, y (a) sp—sp* (b) sp?—sp?
elimination reaction ? (c) sp—sp* (d) sp?—sp*
A34. The compounds CH,OC,H, and C,H.OC,H, exhibit
on (a) optical isomerism (b). cls-eas isomerism
(c) metamerism (d) chain isomerism
(a) CH,CHO —“=—> CH,CHCH,CHO A35. The IUPAC name of CH,C = CCH(CH,), is
(b) CH,CH,CH,CH,Br —*=*°F, (a) 4-Methylpent-2-yne
. " *¢ = CH, (b) 4, 4-Dimethylbut -2-yne
(c) Methyl isopropylacetylene
(d) 2-Methylpent-4-yne
A36. The IUPAC name of the compound

CN
C |
is
\
(d) BrH,C-CH,-CH,Br 2 dust_ H.C ——CH, — Na ——
The IUPAC name of the compound
(a) 4-Cyano-4-methyloxopentane
CH, = CH — CH(CH,,), is (6) 4-Cyano-4-methylpentan-2-one
(a) 1,1-dimethylprop-2-ene (c) 2, 2-Dimethyl-4-oxopentanenitrile
(6) 3-methylbut-1-ene (d) 2-Cyano-2-methyl-4-oxopentane
(c) 2-vinyl propane A37. The compound
(d) 1-isopropylethene HOOC — CH,CH, — CH — CH,CH,COOH
A30. The IUPAC name for :
CH,CHCH,C(CH,), is COOH
has IUPAC name as:
OH OH (a) 4-carboxyheptanedioic acid
(b) 1,3,5-heptanetrioic acid
(a) 1,1-dimethylbutane-1,3-diol
(c) Pentane-1, 3, 5-tricarboxylic acid
(b) 4-methylpentane-2, 4-diol (d) 3-Carboxy heptanedioic acid

Answets
A25.(b) A226. (c) <A27. (b) A228. (d) A229. (bd) A30. (c) A381. (c) A382. (a) ASB. (d) AB4. (c)
A35.(a) AS6. (c) AST. (c)
ORGANIC CHEMISTRY: BASIC PRINCIPLES AND TECHNIQUES

A38. Name of the compound given below is Ad4, Aniline is purified by


CH, (a) Extraction with solvent
H,C (6) Steam distillation
CH, (c) Vacuum distillation
CH, (d) Sublimation.
(a) 4-Ethyl-3-methyloctane
(b) 3-Methyl-4-ethyl octane Qualitative Analysis of Organic Compounds
(c) 2-3-Diethylheptane
A45. In Lassaigne’s test, when both N and S are present
(d) 5-Ethyl-6- methyloctane
blood red colour is obtained. This is due to the
A39. The name of some compounds are given. Which one formation of
is not in IUPAC system ?
(a) ferricferrocyanide (6) ferric cyanide
(a) CH,CH,CH,CH—CH—CH,CH,
(c) ferric thiocyanate (d) ferric hydroxide.
|
CH,—CH, CH, A46. Lassaigne’s solution gives violet colouration with
sodium nitroprusside. It indicates the presence of
3-Methyl-4-ethylheptane
(6) CH,—CH—CH—CH, (a) nitrogen (6) sulphur
| (c) halogen (d) both N and 8.
OH CH, AAT. The formula of the compound which gives violet
3-Methylbutan-2-ol colour in Lassaigne's test for sulphur with sodium
(c) CH,;—CH,—C— CH—CH, nitroprusside is

CH,CH,
(a) Na, [Fe(CN), NOS]
2-Ethyl-3-methylbut-1l-ene (b) Na, [Fe(CN), NOS]
(d) CH,—C=C—CH(CH,), (c) Na, [Fe(CN), S!
(d) Na, [Fe(CN), S].
4-Methylpent-2-yne
A48. Sodalime test for detection of N cannot be used for

A40. Orbital interaction between sigma bonds of a (a) acetamide (5) urea
substituent group and the neighbouring pi-orbital is (c) diazo compounds (d) thiourea.
known as A49. For which of the following compound, the
Lassaigne’s test for N will fail ?
(a) hyperconjugation
(b) inductive effect
(2) NH, CONHNH,HC1
(c) steric effect
(6) NH,NH,.2HCI
(d) dipole-dipole interactions
(c) NH,CONH,
(d) C,H;N = N-C,H,
Purification Techniques
Quantitative Analysis of Organic Compounds
A4l. The principle involved in paper chromatography
is A50. Percentage of Se in peroxidase anhydrous enzyme is
0.5% by weight (at.wt. = 78.4), then minimum mo-
(a) Adsorption (6) Partition
lecular weight of peroxidase anhydrous enzyme is
(c) Solubility (d) Volatility
(a) 1.568 x 10+ (b) 1.501 x 10?
A42, The boiling point of glycerol is 563 K but it
(c) 15.68 (d) 2.136 x 10°
decomposes below 563 K. It is purified by
A51. Inthe Duma’s method for the estimation of nitrogen,
(a) sublimation (b) vacuum distillation
the gas collected in nitrometer is :
(c) steam distillation (d) fractional distillation.
(a) No (b) NH,
A483. A mixture of acetone and methanol can be separated by
(c) No+CO, (d) COQ,.
(a) steam distillation (}) vacuum distillation
A52. 0.4 9m ofan organic compound gave 0.188 g of silver
(c) fractional distillation bromide by a halogen estimation method. The
(d) sublimation. percentage of bromine in the compound is (at mass
ofAg = 108, Br = 80)

Arse
(a) 39.8% (b) 46.0%
(c) 20.0% (d) 40.0%.

A3s. (a) A389. (a) A40. (a) A411, (b) A42. (6) A43. (c) Ad44, (b) A45. (c) A46. (b) A477. (a)
A48, (c) A49, (b) A5b0. (a) A5l. (a) A522. (c)
Sina MODERN'S abc + OF CHEMISTRY-Al

Inthe Duma’s method for the estimation of nitrogen, (a) 20.24% (b) 35.62%
0.84 g of an organic compound gave 448 ml of (c) 12.24% (d) 40.65%.
nitrogen at S.T.P. The percentage of nitrogen in the
compound is A56. During the estimation of nitrogen by Kjeldahl’s
method, copper sulphate is added to
(a) 33.3% (b) 66.7%
(a) raise boiling point of H,SO,
(c) 50.0% (d) 60%. (b) absorb water formed
In the Kjeldahl's method, the nitrogen
in the organic (c) catalyse the reaction
compound is converted to (d) form ammonium sulphate
(a) Ammonium sulphate (b) Ammonia A57. An organic compound containing C, H and N give
(c) Nitric acid (d) Nitrogen. the following on analysis :
0.395 g of an organic compound by Carius C = 40%, H = 13.33% and N = 46.67%. What would
be its empirical formula ?
method for the estimation of S gave 0.582 g of
BaSO,. The percentage of S in the compound is (a) C,HJN (6b) C,HLN,
(c) CH,N (d) CH,N
Answers
A5S3. (b) A54, (a) A55. (a) <A56. (c) A577. (c)

(c) tertiary-butoxide (d) iso-propoxide


from comp etitive examination (e) n-butoxide (Kerala PMT 2010)
. Homolytic fission of a covalent bond leads to the
AIPMT & Other State Boards’ formation of
Medical Entrance (a) electrophile (6) nucleophile
(c) free radical (d) carbocation
Bl. The IUPAC name of (e) carbanion (Kerala PMT 2010)
. Glycerine is purified by
Cl (a) vacuum distillation (b) simple distillation
(c) steam distillation (d) sublimation
O (e) solvent extraction (Kerala PMT 2010)
(a) 1-chloro-1-oxo-2,3-dimethylpentane B7. The correct IUPAC name for the compound
(6b) 2-ethyl-3-methylbutanoyl chloride
“a
(c) 2,3-dimethyl pentanoyl chloride
is
(d) 3,4-dimethyl pentanoyl chloride
(C_.B.S.E.P.M.T. 2006)
B2. Give the IUPAC name of the alkene (a) 4-ethyl-3-propylhex-1-ene
(b) 3-ethyl-4-ethenylheptane
(c) 3-ethyl-4-propylhex-5-ene
(qd) 3-(1-ethylpropyl) hex-1-ene (AIPMT 2011)
(a) Z-3-methyl-4-propyl-3-octene B8. In Dumas’ method of estimation of nitrogen 0.35 g of
(b) E-3-methyl-4-propyl-3-octene an organic compound gave 55 mL of nitrogen collected
(c) E-4-butyl-3-methyl-3-heptene at 300 K temperature and 715 mm pressure. The
(d) E-2-ethyl-3-propyl-2-heptene percentage composition of nitrogen in the compound
would be (aqueous tension at 300 K = 15 mm).
(e) Z-2-ethyl-3-propyl-2-heptene (Kerala-PMT 2008)
B3. Which of the following compounds will not give (a) 15.45 (6) 16.45
Lassaigne’s test for nitrogen? (c) 17.45 (d) 14.45 (AIPMT 2011)
(a) NH,NH, (b) C,H,.NHNH, B9. The Lassaigne’s extract is boiled with conc. HNO,
(c) PhN = NPh (d) NH,CONH, while testing for halogens. By doing so it
(DUMET 2010) (a) decomposes Na,S and NaCN, if formed
B4. Which one of the following ions is the most resonance (b) helps in the precipitation of AgCl
stabilized? (c) increases the solubility product of AgCl

Arswets
(a) ethoxide (b) phenoxide (d) increases the concentration of NO, ions
(AIPMT 2011)

Bl. (c) B2. (a) BS. (a) B4 (6) Bb. (©) B6. (a) B7. (a) ~—-BS. (0) B9. (a)
ORGANIC CHEMISTRY: BASIC PRINCIPLES AND TECHNIQUES

faa
B10. Aniline is separated from aniline-water mixture by B15. The order of stability of the following carbocations is
(a) crystallization (b) steam distillation . @
CH,
(c) solvent extraction (d) sublimation
® @
(e) differential extraction (Kerala P._M.T. 2012) CH,—CH—CH,; CH,—CH,—CH,;
Bil. Only sp and sp” hybrid orbitals are involved in the
formation of I IT IT
(2) CH,—CH = CH, (b) CH,—CH, (a) TIE >IT> OH (b) TI>II>I1
(c) CH,—C=CH (d) CH,=C=CH, (c) >>I (d) I> 11>I
(ce) CH,—-C=C—CH, (Kerala P-M.T. 2012) (NEET 2013)
B12. A sample of 0.5 g of an organic compound was treated B16. In the Kjeldahl’s method for estimation of nitrogen
according to Kjeldahl’s method. The ammonia evolved present in a soil sample, ammonia evolved from
was absorbed in 50 mL of 0.5M H,SO,. The remaining 0.75 g of sample neutralized 10 mL of 1 M H,SO,. The
acid after neutralisation by ammonia consumed percentage of nitrogen in the soil is
80 mL of 0.5 M NaOH. The percentage of nitrogen in (a) 37.33 (6) 45.338
the organic compound is
(c) 35.33 (d) 43.33 (AIPMT 2014)
(a) 14 (b) 28
B17. Which of the following organic compound has same
(c) 42 (d) 56
hybridization as its combustion product (CO,) ?
(e) 70 (Kerala PM.T: 2012)
B18. Which nomenclature is not according to IUPAC (a) Ethane (b) Ethyne
system? (c) Ethene (d) Ethanol
(a) Br—CH, CH=CH, (AIPMT 2014)
1-Bromoprop-2-ene B18. What is the IUPAC name of the following compound?
CH, O O
|
(b) CH; —CH, — ah iii
OH
(a) Hexa-2,6-dienone-6-ol
4-Bromo-2, 4-dimethylhexane (b) 2-Ketohexanoic acid (c) 5-Ketohexanoic acid
(c) a ed nie (e) 5-Oxohexanoic acid
(d) 2-Oxohexanoic acid
(Kerala PMT 2015)
CH
3 B19. The number of o and z-bonds present in 1, 3-butadiene
2-Methyl-3-phenylpentane
are respectively
(d) CH, CH CH Chas (a) 9 and 2 (6) 8 and 2
a (c)9 and 3 (dq) 9 and 1
2 ene Beat () 8 and 1 (Kerala PMT 2015)
+
aia as B20. Which of the following statement is not correct for a
B14, The structure of isobutyl group in an organic compound nucleophile?
i (a) Nucleophiles attack low electron density sites
(a) CH,—CH,—CH,—CH,— (b) Nucleophiles are not electron seeking
CH (c) Nucleophile is a Lewis acid
(b) CH,—C— (qd) Ammonia is a nucleophile (AIPMT 2015)
CH, B21. Two possible stereo-structures of
() Cy, a a aaaeaiaa which are optically active, are
>CH—CH,— called:
CH; (a) Enantiomers (b) Mesomers
(dq) CH,—CH—CH,—CH, | | (c) Diastereomers (qd) Atropisomers
| (NEET 2013) (AIPMT 2015)

{east
B10. (6) Bll. (d) B12. (6) B13. (a) B14. (c) B15. (a) B16. (a) B17. (6) B18. (e) B19. (a)
B20. (c) B21. (a)
ns 12/144 MODERN'S abc + OF CHEMISTRY-XI

B22. Consider the following compounds: (c)'H, AH (d) HC,


CH, se C—Cl c—Cl
ee
cH.—c—éu—{ :
Pr—C—Pr : CH HC”
3
H,c’ cH
| me (AIPMT 2015)
CH | B27. The pair of electrons in the given carbanion,
(1) (IT) (IIT) CH,C=C_, is present in which of the following orbitals?
Hyperconjugation occurs in (a) sp? (b) sp
(a) IIT only (6) I and Ill (c) 2p (d) sp? (NEET 2016)
(c) only (d) I only B28. The IUPAC name of compound
(AIPMT 2015) ¢) 0
B23. The enolic form of ethyl acetoacetate as shown below |
has Hoe |
: | H H, 1s

HAC TaN p> __s AN LX fp (a) 5-formylhex-2-en-3-one


C C C C (6) 5-methyl-4-oxohex-2-en-5-al
| || || (c) 3-keto-2-methylhex-5-enal
OH OC,H, 0 OC,H, (d) 3-keto-2-methylhex-4-enal (NEET 2017)
(a) 9 sigma bonds and 2 pi—bonds B29. The correct statement regarding electrophile is
(b) Osterman bonds and 1 pi-bond (a) electrophile is a negatively charged species
and can form a bond by accepting a pair of
(c) 18 sigma bonds and 2 pi-bonds electrons from another electrophile
(d) 16 sigma bonds and 1 pi-bond. (AIPMT 2015) (6b) electrophiles are generally neutral species
B24, Given: and can form a bond by accepting a pair of
CH, CH, electrons from a nucleophile
»CH. (c) electrophile can be either neutral or positively
ae CH charged species and can form a bond by
3 j accepting a pair of electrons from a nucleophile
CH; (d) electrophile is a negatively charged species and
O O O can form a bond by accepting a pair of electrons
(I) (II) (IID) from a nucleophile. (NEET 2017)
Which of the given compounds can exhibit B30. Which of the following is correct with respect to —I
tautomerism? effect of the substituents? (R = alkyl)
(a) IT and III (b) I, II and III (a) -NH,<-OR<-F (6) -NR,<-OR<-F
(c) I and II (d) I and II (c)-NH,>-OR>-F (d) -NR,>-OR>-F
(AIPMT 2015) (NEET 2018)
B25. In Duma’s method for estimation of nitrogen, 0.25 g B31. Which of the following carbocation is expected to be
of an organic compound gave 40 mL of nitrogen most stable ?
collected at 300 K temperature and 725 mm pressure. NO NO
If the aqueous tension at 300 K is 25 mm, the : .
percentage of nitrogen in the compound is (a) (b)
(a) 16.76 (b) 15.76 ol
(c) 17.36 (d) 18.20 (AIPMT 2015) _ = Y° “H
B26. In which of the following compounds, the C—C]l bond NO,
ionisation shall give most stable carbonium ion? | NO,
(a) (b) H (c) (d) :
Hcl So—c1 b > k,
.“H
O,NH,C (NEET 2018)

Asie
B22. (a) B23. (c) B24. (b) B25. (a) B26. (d) B27. (d) B28. (d) B29. (c) B30. (a) B31. (ec)
ORGANIC CHEMISTRY: BASIC PRINCIPLES AND TECHNIQUES

JEE (Main) & Other State Boards’ B38. The correct decreasing order of priority for the functional
Engineering Entrance
eroups of organic compounds in the IUPAC system of
nomenclature is
B32. Of the five isomeric hexanes, the isomer which can (a) -CONH,, -CHO, -SO,H, -COOH
give two monochlorinated compounds is (6) -COOH, -SO,H, -CONH,, -CHO
(a) n-hexane (b) 2,3-Dimethylbutane (c) -SO,H, -COOH, —CONH,, -CHO
(c) 2,2-Dimethylbutane (d) -CHO, -COOH, -SO,H, -CONH, (AIEEE 2008)
(d) 2-Methylpentane (A.LLE.ELE. 2005) B39. Methoxy methane and ethanol are
(a) Functional isomers (6) Optical isomers
B33. The IUPAC name of the compound
(c) Position isomers (d) Chain isomers
Cl (AIEEE 2008)
B40. The IUPAC name of the molecule
1s
Br O O
(a) 6-bromo-2-chlorocyclohexene CH, _b-c=c—b_on Is :
(6) 3-bromo-1-chlorocyclohexene
(c) 1-bromo-3-chlorocyclohexene Ou,CH,
(d) 2-bromo-6-chlorocyclohex-1-ene (a) 4-Oxo-2, 3-dimethylpent-2-en-1l-oic acid
(A..LE_LELE. 2006) (b) 2-Carboxy-3-methylpent-2-en-3-one
B34. The increasing order of stability of the following free (c) 4-Carboxy-3-methylpent-3-en-2-one
radicals is (d) 2,3-Dimethyl-4-oxo-pent-2-en-1-oic acid
(J & K CET 2009)
(a) (C,H,),C<(C oH),CH <(CH,),C <(CH,), CH B41. 0.25 g of an organic compound on Kjeldahl’s analysis
gave enough ammonia to just neutralise 10 cm” of
(b) (C.Hs), CH <(Cy H,) C<(CH,) C< (CH,),CH 0.5M H,SO,. The percentage of nitrogen in the
(c) (CH,),CH<(CH,),C < (C,H,),CH< (C,H,),CH compound is
(a) 28 (6) 56 (c) 14 (d) 112
(d) (CH,),CH <(CH,), C< (C,H,),CH < (C,H,),C (e) 42 (Kerala Engg. 2009)
(A.LE.E.E. 2006) . The number of isomers including stereoisomers
possible for the compound having molecular formula
C,H, is
B35. The IUPAC name of adit is (a) one (b) two (c) three (d) four
(JK. CET 2010)
(a)
4,4-Dimethyl-5,5-diethyl pentane . In Kjeldahl’s method, ammonia from 5 g of food
(b)5,5-Diethyl-4,4-dimethyl pentane neutralizes 30 cm® of 0.1 N acid. The percentage of
nitrogen in the food is
(c)3-ethyl-4,4,-dimethyl heptane
(a) 0.84 (6b) 8.4 (c) 16.8 (d) 1.68
(d)1,1-diethyl-2,2-dimethyl pentane (Karnataka CET 2010)
(A.LE.E.E. 2007) . Which one of the following has the most nucleophilic
B36. Which one of the following carbanions is least stable? nitrogen?
(a) CH,CH, (6) HC=C™ 4 ol .. |
(c) (C,H) .C- (d) CH; oC] oY
(e) (CH,),C- (Kerala PET. 2008)
B37. Which of the following does not exhibit tautomerism? NHCOCH, NH,
| O (c) CT (d) Cr
@) 0X0 (b) (Karnataka CET 2010)
O . In Lassaigne’s test for the detection of halogens, the

Sno
sodium fusion extract 1s first boiled with concentrated
nitric acid. This is
(c) O O (a) to remove silver halides
(WB-JEE 2008) (6) to decompose Na,S and NaCN, if present
(c) to dissolve Ag,S
(d) to dissolve AgCN, if formed
(e) because Ag,S and AgCN are insoluble in nitric acid
Answels (Kerala PET 2010)
B32. (b) B33. (b) B34. (d) B35. (c) B86. (ec) B37. (a) B38. (c) B39. (a) B40. (d) B41. (a)
B42. (d) ~=—6B43. (a) ~=B44. (a) ~=B45. (0)
MODERN'S abc + OF CHEMISTRY-XI

B46. Hyper conjugation is most useful for stabilizing which (e) 5,5 and 5 (Kerala PET 2011)
of the following carbocations?
(a) neo-Pentyl (b) tert-Butyl (c) tso-Propyl
(d) Ethyl (e) Methyl (Kerala PET 2010) B53. The IUPAC name of aA OH is
B47. The IUPAC name of the following compound is (a) but-3-enoic acid (6) but-1l-enoic acid
(CH,), CH — CH, CH = CH — CH = CH — ai CH, (c) pent-4-enoic acid (d) prop-2-enoic acid
(Karnataka PET 2011)
CoH, B54. 1.2 g of an organic compound on Kjeldahlization
(a) 1, 1, 7, 7-tetramethyl-2,5-octadiene liberates ammonia which consumes 30 cm? of 1 N HCL.
(6) 2, 8-dimethyl-3,6-decadiene The percentage of nitrogen in the organic compound is
(c) 1,5-di-iso-propyl-1, 4-hexadiene (a) 30 (b) 35
(d) 3, 9-dimethyl-4, 6-decadiene (c) 46.67 (d) 20.8
(e) 2, 8-dimethyl-4, 6-decadiene (Kerala PET 2010) (Karnataka PET 2011)
B48. 29.5 mg of an organic compound containing nitrogen B55. Identify the compound that exhibits tautomerism.
was digested according to Kjeldahl’s method and the (a) 2-Butene (6) Lactic acid
evolved ammonia was absorbed in 20 mL of 0.1 M (c) 2-Pentanone (d) Phenol (AIEEE 2011)
HCI solution. The excess of the acid required 15 mL of B56. Among the following carbocations :
0.1 M NaOH solution for complete neutralization. The (I) Ph,CtCH,Me (II) PhCH,CH,CH*Ph
percentage of nitrogen in the compound is: (II1) Ph,CHCH*Me (IV) Ph,C(Me)CH?
(a) 28.7 (6) 29.5 (e) 59.0 (d) 47.4 the order of stability is
(AIEEE 2010) (a) IV>II>I>IH (6) I>II>TI>IV
B49. The correct order of boiling points of 2, (c) I>I>I1IV>I (d) I>IV>III >I
2-dimethylpropane, 2-methylbutane and n-pentane is (W.B.J.E.E. 2012)
B57. In the estimation of sulphur by Carius method, 0.480
(a) n-pentane > 2, 2-dimethylpropane >
g of an organic compound gives 0.699 g of barium
2-methylbutane
sulphate. The percentage of sulphur in this
(b) n-pentane > 2-methylbutane > 2, 2-dimethylpropane compound is
(c) 2,2-dimethylpropane > 2-methylbutane (Atomic masses : Ba = 137,5 = 32, O = 16)
> n-pentane (a) 20% (b) 15% (c) 35%
(dq) 2-methylbutane = n-pentane > (d) 30% (e) 40% (Kerala PE.T: 2012)
2,2-dimethylpropane B58. The IUPAC name of
(e) 2-methylbutane > 2,2-dimethylpropane CH,CHOHCH,CH(CH, )CHO is
> N-pentane (Kerala PET 2011) (a) 2-hydroxy-4 methylpentanal
B50. The stablest radical among the following is (6) 4-hydroxy-2 methylpentanal
(c) 2-hydroxy-3 methylpentanal
(a) C,H,CH,
—CH, (b) CH, CH, (d) 2-methylpent-4-ol-1-al (AMU Engg. 2012)
B59. The enolic form of acetone contains
(c) CsH,—CH —CH, (dq) CH,—_-CH —CH, (a) 9 sigma bonds, 2 pi bonds and 1 lone pair
(6) 9 sigma bonds, 1 pi bond and 2 lone pairs
fe) CH,—CH,— CH, (Kerala PET 2011) (c) 8 sigma bonds, 2 pi bonds and 2 lone pairs
(d) 10 sigma bonds, 1 pi bond and 1 lone pair
. The compounds CH,CH =CHCH, and CH,CH,CH=CH,
(AMU Engg. 2012)
(a) are tautomers (5) are position isomers
B60. The IUPAC name of the compound X is
(c) contain same number of sp? - sp®, sp? - sp? and sp” —

[ -
sp” carbon-carbon bonds
(d) exist together in dynamic equilibrium

aA Ye IN
X=
(e) are optical isomers (Kerala PET 2011)
B52. In hexa-1,3-diene-5-yne, the number of C—C o, C—C CH, CH, CH,
m™ and C—H o bonds respectively are
(a) 5,4 and 6 (6) 6,3 and 5 (a) 4-cyano-4-methyl-2-oxopentane
(6) 2-cyano-2-methyl-4-oxopentane
(c) 5,3 and 6 (d) 6,4 and 5
(c) 2, 2-dimethyl-4-oxopentanenitrile

(ESAS (d) 4-cyano-4-methyl-2-pentanone (W.B. JEE 2013)

B46. (b) B47. (e) B48. (a) B49. (b) B50. (c) B51. (5) B52. (a) B53. (a) B54. (6) B55. (c)
B56. (b) B57. (a) B58. (6) B59. (b) B60. (c)
ORGANIC CHEMISTRY: BASIC PRINCIPLES AND TECHNIQUES

B61. Structure of the compound whose IUPAC name is (b) Fractional distillation
3-Ethyl-2-hydroxy-4-methylhex-3-en-5-ynoie acid is (c) Steam distillation
OH : (d) Distillation under reduced pressure.
| , | (JEE Main 2016)
i 36 (b)| OOH
B67. Identify the correct statement in the following:
OH (a) Dimethyl ether and ethanol are chain isomers.
: OH : OH (b) Ethanoic acid and methyl methanoate are
position isomers.
(c) n-butane and isobutane are functional isomers.
(c) FY ~eo0n (d) se. (d) Propan-1-ol and propan-2-ol are position isomers.
(Karnataka CET 2017)
(JK. CLE.T, 2013) B68. Lassaigne’s test (with silver nitrate) is commonly
B62. Among the following structures the one which is not a
used to detect halogens such as chlorine, bromine and
resonating structure of others is
iodine but not useful to detect fluorine because the
O O O O product AgF formed is
Me Me (a) volatile (b) reactive
St AY. SoA,
(c) explosive (d) soluble in water
I Il (ge) a liquid (Kerala PET 2017)
_ B69. What is the IUPAC nomenclature of the given
QO O QO O compound?
Mew O BA Me
me y
AK ~
ttl IV (a) 5-Ethynyl-1, 6-heptadiene
(a) I (6) II (6) 3-Ethynyl-1, 6-heptadiene
(e) Il (dq) IV (WB JEE 2014)
(c) 3-Vinyl-hept-6-en-1l-yne
B63. On complete combustion, 0.246 g of an organic (d) 5-Vinyl-hept-1-en-6-yne (JK. CET 2018)
compound gave 0.198 g of CO, and 0.1014 g of H,O. B70. During the fusion of organic compound with sodium
The ratio of carbon and hydrogen atoms in the metal, nitrogen present in the organic compound is
compound is converted into
(a) 1:3 (b) 1:2 (a) NaNO, (b) NaNH,
(c) 2:5 (d) 2:7
(A.M.U. Engg. 2014)
(c) NaCN (d) NaNC
B64. For the estimation of nitrogen, 1.4 g of an organic
(Karnataka CET 2018)
compound was digested by Kjeldahl method and the
B71. The elution sequence of a mixture of compounds
ammonia evolved was absorbed in 60 mL of M/10
containing chlorobenzene, anthracene and p-cresol
sulphuric acid. The unreacted acid required 20 mL
developed on an alumina column using a solvent
M
of - sodium hydroxide for complete neutralisation. system of progressively increasing polarity is
(a) anthracene — chlorobenzene —> p-cresol
The percentage of nitrogen in the compound is
(b) anthracene — p-cresol — chlorobenzene
(a) 6% (b) 10%
(c) chlorobenzene — p-cresol + anthracene
(ec) 3% (d) 5%
(JEE Main 2014)
B65. In Carius method of estimation of halogens, 250 mg (d) chlorobenzene —> anthracene —> p-cresol
of an organic compound gave 141 mg of AgBr. The (e) p-cresol + anthracene > chlorobenzene
percentage of bromine in the compound 1s (at. mass (Kerala PET 2018)
Ag = 108 ; Br = 80) B72. Which of the following compounds will be suitable for
(a) 48 (6b) 60 Kjeldahl’s method for nitrogen estimation ?

CI >
(c) 24 (d) 36
(JEE Main 2015)
B66. The distillation technique most suited for separating (a) (b) Cy
N
glycerol from spent lye in the soap industry is
(a) Simple distillation NO, Nicr

Areswes
(c) (d) Cy
(JEE Main 2018)

B61. (d) B62. (d) B63. (c) B64. (6b) B65. (c) B66. (d) B67. (dd) B68. (d) B69. (b) B70. (ec)
ee 12/148 MODERN'S abc + OF CHEMISTRY-XI

JEE (Advance) for ITT Entrance B77. Hyperconjugation involves overlap of the following
B73. Which of the following compounds will exhibit orbitals
geometrical isomerism ? (a) o-o (6) o-p
(a) 1-Phenylbut-2-ene (c) p-p (d) m-—7 (UT-JEE 2008)
(b) 3-Phenylbut-1-ene B78. The correct stability order for the following species is
(c) 2-Phenylbut-1-ene A, me
(d) 1,1-Diphenylprop-1l-ene (LI.T. 2000) _—,
B74. Which of the following has the lowest dipole moment? | I | I
@

cl =
HWer bee TT IV
| (a) (I) > (TV) > (> CID)
(b) CH,C = CCH, (b) (D) > MI) > MD = Vv)
(c) CH;CH,C = CH | | (c) (I)>@>(MVv)> dip
sae Ta aan Mae eee (d) (> (ID) > dD > dv) (IIT-JEE 2008)
B75. Which of the following represents the given mode of B79. The total number of contributing structures showing
hybridisation sp"—sp ie a from left to hght ? hyperconjugation (involving C—H bonds) for the
(a) H,C=CH—CSN (6) re following carbocation is
(c) H,C=C=C=CH, @)HCA\SZ ~ H,C~_-CH,CH,
(11.7. 2003)
B76. The number of structural isomers for C,H,, is
(a) 3 (6) a: (a) six (6) four
(c) 5 (d) 6 (1.7. 2007) (c) eight (d) three (IT-JEE 2011)

Auswets
B73. (a) B74, (b) B75. (a) B76. (c) B77. (b) B78. (b) B79. (a)

C6. The pairs having only electrophiles are :


(a) BF,, CH,t (b) RCOO-, NH,
with more than one correct answers (c) AICl,,SO, (d) SO,, ROR
Cl. The dipole moment is shown by C7. Which of the following behave both as nucleophile as
(a) 1,4-Dichlorobenzene well as an electrophile ?
(6) cis-1,2-Dichloroethene (a) CH,Cl (6) HCHO
(c) trans-1,2-Dichloroethene (c) CH,CN (d) CH,OH
(d) trans-1,2-Dichloro-1-pentene C8. Which of the following compounds contain all the
C2. Keto-enol tautomerism is shown by carbon atoms in the same hybridisation state?
(a) H—C=C—C=C—H
(a) C,H,CHO (b) C,H.COCH, ee _
(c) C,H,COC,H, (d) C,H,COCH,COCH, ri OR aa ay
C3. Which of the following compounds contain only sp? (d) CH. -c -¢C =
hybridized carbon ? 2 a
(a) Cycloalkfges Wk) TAoched chain alkenes C9. Which of the following groups contain only
electrophiles?
(c) Benzene (d) Cycloalkenes 3 +
C4. Which of the following are not nucleophiles? re wee oe 0 _c=0 mnjag a ee
(a), BF 3 (b) RCOO-
fs i name(s)
C10. The IUPAC a of aethe following
ae compound
ee is(are)
(c) ROR (d) NO,
C5. The species having +I effect are HC < pal
(a) CH,CH, (d) —COOH (a) 4-methylchlorobenzene
(c) —NO, (d) (CH,),C — (b) 4-chlorotoluene
(ec) 1-chloro-4-methylbenzene

Answer
Cl. (6,d) C2. (b,d) C3. (a,b) C4. (a,d) C5. (a,c) C6. (a,c)
(d) 1-methyl-4-chlorobenzene

C7. (6,c) C8 (a,b)


(JEE Advance 2017)

C9. (b,c) C10. (6, c)


ORGANIC CHEMISTRY: BASIC PRINCIPLES AND TECHNIQUES

F [MULTIPLE CHOICE QUESTION CH, CH,


_ based on the given passage/comprehension (c) (d)
Passage
Organic compounds mainly consist of covalent bonds. The siti
electron pair in these covalent bonds may undergo OCH, NO, (D.C.E. 2000)
displacement either of their own or under the influence of D4. The most stable free radical among the following is
other species. The cleavage of covalent bond between two
atoms takes place in homolytic or heterolytic fashion. The
(a) C,H,CH,CH, (6) C,H,CHCH,
homolytic fission results into free radicals while heterolytic (c) CH,CH, (2) CH,CHCH,
fission results into carbocations and carbanions. These are D5 . The most stable carbocation is:
also called reaction intermediates and are attacked by
electrophiles and nucleophiles. The electrophiles seek electron (a) CH," (6) (CH,),C*
rich sites while nucleophiles seek electron deficient sites. (c) CH,CH,CH,; (d) (CH,),CH*
Answer the followings Questions: D6. The hybridisation state of carbon in the carbocation
D1. Which of the following series contain only nucleophiles:
C,H;CH3 is
(a) AICL,, BF,, NH, (6) RCOO”, CH,”, CN™
(c) NH,,SO,,ROR (d) NO,*, BF., Cl* (a) sp (b) sp?
D2. Which of the following is the most stable cation ? (c) sp? (d) sp or sp?
(a) F,C—CH,* (6) (CH,),CH* D7. Which of the following is most stable carbocation ?
(ce) CH,* (d) CF,* (J & K CET. 2004) (a) Ph,CH* (b) PhCH,"
D3. The most stable carbanion among the following is (c) Ph,C* (d) Ph*
CH,CH3; CH; D8. Diazomethane decomposes in the presence of light to
give
(a) QO (b) 0
(a) free radical (6) carbocation
(c) carbenes (dq) carbanion

Arrsies
D1. (db) D2. (db) D3. (d) D4. (6) D5. (6) D6. (5) D7. (c) D8. (c)

: Allyl free radical is more stable than


Assertion Reason Type Questions 1. Assertion
simple free radical.
The questions given below consist of an Assertion and Reason : The allyl free radical is stabilized by
the Reason. Ue the following key to choose the appropriate reasonance. (A.LLM.S. 2001)
answer. 2. Assertion : In allene (C,H,) all carbon atoms are sp
(a) If both assertion and reason are CORRECT and hybridised.
reason is the CORRECT explanation of the Reason : All the three carbon atoms are joined by
assertion. double bond.
(6) If both assertion and reason are CORRECT, but 3. Assertion : Lassaigne’s test is not shown by
reason is NOT THE CORRECT explanation of the diazonium salts.
assertion, Reason : Diazonium salts lose N, on heating much
(c) If assertion is CORRECT, but reason is before than they have a chance to react
INCORRECT. with fused sodium.
(dq) If assertion is INCORRECT, but reason is 4, Assertion : A mixture of o-nitrophenol and
CORRECT. p-nitrophenol can be separated by steam
(e) If both assertion and reason are INCORRECT. distillation.
Reason : p-nitrophenol is steam volatile while

mses
o-nitrophenol is not steam volatile.

1. (a) 2. (d) 3. (a) 4, (c)

WWW.JEEBOOKS.IN
MODERN'S abc + OF CHEMISTRY-XI

. Assertion : The Lassaigne extract is boiled with dil Reason : The resonance energy of benzene is about
HNO, before testing for halogens. 150.6 k.J mol-!.
Reason : Ag,S is soluble in HNO,. 13.Assertion : Simple distillation can help in separating
. Assertion : (CHs),C* is less stable than (CH,),CH,* a mixture of propan-1-ol (boiling point 97°
10n. C) and propanone (boiling point
Reason : (CH,),C* has pyramidal structure. 56°C).
. Assertion : Alkanes having more than three carbon Reason : Liquids with a difference of more than
atoms exhibit chain isomerism. 20°C in their boiling points can be
Reason : All carbon atoms in alkanes are sp®
separated by simple distillation.
hybridized. 14, Assertion : All the carbon atoms in H,C=C=CH, are
. Assertion : Dumas method is more applicable to sp” hybridised.
nitrogen containing compounds than Reason : Inthis molecule all the carbon atoms are
Kjeldahl’s method. attached to each other by double bonds.
Reason : Kjeldahl’s method does not give 15. Assertion : Sulphur present in an organic compound
satisfactory results for compounds in can be estimated quantitatively by
which nitrogen is bonded to oxygen. Carius method.
. Assertion : Butane and 2-methyl butane are Reason : Sulphur is separated easily from other
homologues. atoms in the molecule and gets precipi-
Reason : Butane is a straight chain alkane while tated as light yellow solid.
2-methyl butane is a branched chain 16. Assertion : Components of a mixture of red and blue
alkane. inks can be separated by distributing the
10. Assertion : Methylene has a sextet of electrons. components between stationary and
mobile phases in paper chromatography.
Reason : Methylene behaves as a nucleophile.
Reason : The coloured components of inks migrate
11. Assertion : CH,Ct=O behaves as electrophile.
at different rates because paper
Reason : In this species, O has two pairs of selectively retains different components
electrons. according to the difference in their
12. Assertion : All the C—C bond lengths in benzene are partition between the two phases.
equivalent. 17. Assertion : Pent-1l-ene and pent-2-ene are position
isomers.

arses
5. (c) 6. (e) 4% 10) 8. (b) 9. (6)
Reason

10. (c)
: Position isomers differ in the position of

11. (c)
functional group or a substituent.

12. (6) 13. (a) 14. (d)


15. (e) 16. (a) 17. (a)

NCERT Exemplar Problems ///


Objective Questions
»> <<
(a) 1-hydroxypentane-1, 4-dione
Multiple Choice Questions (Type-l)
(6) 1, 4-dioxopentanol
1. Which of the following is the correct [UPAC name ? (c) 1-carboxybutan-3-one
(a) 3-Ethyl-4, 4-dimethylheptane (d) 4-oxopentanoic acid — Cl
(6) 4, 4-Dimethyl-3-ethylheptane
(c) 5-Ethyl-4, 4-dimethylheptane
3. The IUPAC name for NO,
(d) 4, 4-Bis(methyl)-3-ethylheptane
2. The IUPAC name for
(a) 1-Chloro-2-nitro-4-methylbenzene CH,
| | (6) 1-Chloro-4-methyl-2-nitrobenzene
CH,—C—CH,—CH,—C—OH (c) 2-Chloro-1-nitro-5-methylbenzene

Arswes
(d) m-Nitro-p-chlorotoluene

1. (a) 2. (d) 3. (b)


ORGANIC CHEMISTRY: BASIC PRINCIPLES AND TECHNIQUES

4, Electronegativity of carbon atoms depends upon their (c) 2-sec-butylbutane


state of hybridisation. In which of the following (d) 2, 3-dimethylbutane
compounds, the carbon marked with asterisk is most 11. In which of the following compounds the carbon marked
electronegative ?
with asterisk is expected to have great positive charge ?
(a) CH,—CH,—*CH,—CH, (a) *CH,—CH,—Cl
(6) CH,—*CH=CH—CH, (6) *CH,—CH,—MgtcCl
(¢) CH,—CH,—C="CH (c) *CH,—CH,—Br
(q) CH,—CH,—CH=*CH, (dq) *CH,—CH,—CH,
5. In which of the following, functional group isomerism is 12. Ionic species are stabilised by the dispersal of
not possible ? charge. Which of the following carboxylate ion is the
(a) Alcohols (6b) Aldehydes most stable ?
(c) Alkyl halides (d) Cyanides
6. The fragrance of flowers is due to the presence of some (a) CH,_C—O (6) CI-CH,—C—O"
1
steam volatile organic compounds called essential oils.
These are generally insoluble in water at room tem-
BY -
perature but are miscible with water vapour in vapour (c) F-CH,C—O @) Ff
CH,—C—O
phase. A suitable method for the extraction of these oils
from the flowers is : 18. Electrophilic addition reactions proceed in two steps.
(a) Distillation The first step involves the addition of an electrophile.
(6) Crystallisation Name the type of intermediate formed in the first step
(c) Distillation under reduced pressure of the following addition reaction.
(dq)
Steam distillation H,C—HC=CH, + Ht——?
7. During hearing of a court case, the judge suspected that (a) 2° Carbanion (b) 1° Carbocation
some changes in the documents had been carried out. (ec) 2° Carbocation (d) 1° Carbanion
He asked the forensic department to check the ink used 14 Covalent bond can undergo fission in two different ways.
*

at two different places. According to you which tech- The correct representation involving a heterolytic fission
nique can give the best results ? of CH,—Br is
(a) Column chromatography @ 4
(6) Solvent extraction ai, CH,—Br——
Meek CH, + Br
(c) Distillation
(dq) Thin layer chromatography () ont — CH, + Br®
8. The principle involved in paper chromatography is
(a) Adsorption (6) Partition (c) cu! S,——> CH, + Br°
(c) Solubility (d) Volatility
9. What is the correct order of decreasing stability of the
following cations : (d) fu\ 3.—4 CH, + Br
ae) @ 15. The addition of HC] to an alkene proceeds in two steps.
CH,—CH—CH, .CH,—CH—OCH,
I i The first step is the attack of H* ion to 2c =C < portion
which can be shown as
CH,—CH—CH,—OCH,
Il
(a)
ae |
HY+oSscece
(NE

(a) I>I>I (b) Il>Il>I


(c) Il>I>0 (d) [>>I
(0) HeSC¥cK
a + “

10. Correct IUPAC name for oe Aa


:
15 | ' _ ™ “
(c) H Yo aC.
(a) 2-ethyl-3-methylpentane CoH5 CoHs
(b) 3, 4-dimethylhexane (d) All of these are possible

4, (c) 5. (c) 6. (d) op (d) 8. (b) 9. (a) 10. (5) 11. (a) 12. (d) 13. (c)
14, (b) 15. (b)
MODERN'S abc + OF CHEMISTRY-AXl

r
i j
j

si
;

»> Multiple Choice Questions (Tupe-ll)


16. Which of the following compounds contain all the carbon
I CH,—CH,—CH,—C—H
| QO
atoms in the same hybridisation state ? !
I. CH,—CH,—CH,—C—CH,
(b) CH,—C = C—CH,
(ec) CH,=C=>CH,
Il. CH,—CH,—(—CH;~CH,
(dq) CH, = CH— CH=CH, O
17. In which of the following representations given below
spatial arrangement of group/atom different from the
IV. CH, —CH— Fg a


given in structure ‘A’?
19. Which of the following pairs are position isomers ?
fl CL (a) LTandIl (b) I and LI
i aa es: (c) ILand IV (d) Ul and IV
Br 20. Which of the following pairs are not functional group
(A)
isomers ?

(a) wt C
I (b) 54 at
L (a) Il and Ii
(c) TandIV
(6) Il and IV
(d) Land II
cl“"y “Br H d “Br 21. Nucleophile is a species that should have
H Cl (a) a pair of electrons to donate
re | y (5) positive charge
(c) negative charge
“yo cr" “a
(c) an 1C (ad) ac
(d) electron deficient species
Cl H.C . Hyperconjugation involves delocalisation of
18. Electrophiles are electron seeking species. Which of the (a) electrons of carbon-hydrogen o bond of an alkyl
following groups contain only electrophiles ? group directly attached to an atom of unsaturated
(i) BF,,NH,,H,O system.
(ii) AlCl, SO,No? (6) electrons of carbon-hydrogen o bond of alkyl group
(iii) NOJ, CH;, CH, — Gis
=i) directly attached to the positively charged carbon
atom.
(iv) C.H,;¢CoH,, C,H;
(c) m-electrons of carbon-carbon bond
Note: Consider the following four compounds for answering (dq) lone pair of electrons
questions 19 and 20.

Auswets
16. (a, d) Pr ee a 18. (bf/ GIF b) 20. (a,c) 21. (a,c) 22, (a, b)

»> Matching Type Questions


<q
In the following questions more than one correlation is possible between options of Column I and Column II.
Make as many correlations as you can.
23. Match the type of mixture of compounds in Column I with the technique of separation/purification given in Column II.

Column I Column II

(1) Two solids which have different solubitities in a solvent and which do (a) Steam distillation.
not undergo reaction when dissolved in it.
(ii) Liquid that decomposes at its boiling point. (b) Fractional distillation
(iii) Steam volatile liquid. (c) Simple distillation
(iv) Two liquids which have boiling points close to each other. (d) Distillation under
reduced pressure.
(v) Two liquids with large difference in boiling points. (e) Crystallisation.

(NOE
23. (1) — (e); (iz) — (d); (tit) — (a); (iv) — (8); (v) — (e)

WWW.JEEBOOKS.IN
ORGANIC CHEMISTRY: BASIC PRINCIPLES AND TECHNIQUES

24, Match the terms mentioned in Column| with the terms 26. Match the intermediates given in Column I with their
in Column IT. probable structure in Column II.
Column I Column IT
Column I Column II
(i) Carbocation (a) Cyclohexane and 1-hexene :
(it) Nucleophile (b) Conjugation of electrons of (i) Free radical (a) Trigonal planar
C—H o bond with empty (iz) Carbocation (b) Pyramidal
p-orbital present at adjacent (tit) Carbanion (c) Linear
positively charged carbon. 27. Match the ions given in Column-I with their nature given
(iii) Hyperconjugation | (c) sp? hybridised carbon with
in Column-II.
empty p-orbital.
(zu) Isomers (d) Ethyne Column I Column. IT
(v) sp hybridisation |(e) Species that can receive a pair 7 <oaa
of electrons (1) CH,;—O — CH — CH, | (a) Stable due to resonance
(vi) Electrophile (f) Species that can supply a pair 3 a
of electrons. Ge FC (b) Destabilised due to
25. Match Column I with Column II. inductive effect
Column I Column II (iii) CH, (c) Stabilised by
hypercongugation
(:) Dumas method (a) AgNO, cH,—c° Ser ee he a
(ii) Kjeldahl’s method (b) Silica gel
(iii) Carius method (c) Nitrogen gas CH,
(iv) Chromatography (d) Free radicals , @
(v) Homolysis (ec) Ammonium sulphate (iv) CH,— CH — CH, (d) Asecondary carbocation

Answers
24, (i)—(c, e); (ti) —(); (tit) — (b); (iv)
— (a); (v) — (ad); (vt) —(e) 25. (2) — (e); (it)— (e); (tit)
— (a); (iv)
— (B); (v)
— (ad)
26. (i) —(a); (it) — (a); (iit) — (6) 27. (1) — (a, b, d); (it) — (5); (ait) — (5); (iv) — (e, B)

>» Assertion and Reason Type Questions


“<< 30. Assertion (A): Pent-l-ene and pent-2-ene are
position isomers.
In the following questions a statement of Reason (R) : Position isomers differ in the position of
Assertion (A) followed by a statement of Reason functional group or a substituent.
(R) is given. Choose the correct option out of the 31. Assertion (A): All the carbon atoms in H,C=C=CH,
choices given below each question. are sp” hybridised.
(a) Both A and R are correct and R is the correct Reason (R) : In this molecule, all the carbon atoms are
explaination of A. attached to each other by double bonds.
(6) Both Aand R are correct but R is not the correct 32. Assertion (A): Sulphur present in an organic
explanation of A, compound can be estimated quantitatively by Carius
(c) Both Aand R are not correct. method.
(qd) Ais not correct but R is correct. Reason (R) : Sulphur is separated easily from other
atoms in the molecule and gets precipitated as light
28. Assertion (A) : Simple distillation can help in
yellow solid.
separating a mixture of propan-1-ol (boiling point 97°C)
33. Assertion (A): Components of a mixture of red and
and propanone (boiling point 56°C)
blue inks can be separated by distributing the
Reason (R) : Liquids with a difference of more than
components between stationary and mobile phases in
20°C in their boiling points can be separated by simple
paper chromatography.
distillation.
Reason (R) : The coloured components of inks migrate
29. Assertion (A): Energy of resonance hybrid is equal to
at different rates because paper selectively retains
the average of energies of all canonical forms.
different components according to the difference in their
Reason (R) : Resonance hybrid cannot be presented by partition between the two phases.
a single structure.

Arswes
28. (a) 29. (d) 30. (a) 31. (d) 32. (c) 30. (a)
MODERN'S abc + OF CHEMISTRY-XI

p l a n ations for
Hin t s & E x Difficult Objective Type Questions

A. meq with only one correct answer


ll 0.08 g
% Br =
0.08 x 100
AZ. (b) : Ph,Ct is most sable because of resonance
stabilization among the phenyl groups.
=e %
CH,
A5S. (b) 22400 mL of nitrogen at 5.T.P. weigh
A9.(c) +: (a) CH,—C¢_CH, = “2se
448 mL of nitrogen at 5.T.P. weigh
CH 3

(6) CH,CH CH,CH, Lf 28 _ 448 =0.56¢


22400

CH, % Nitrogen =
0.56 x 100
(c) CH, CH CH, CH, CH, 0.84
= 66.7
CH, A55. (a) 233 g of BaSO, contain 5 = 32 g
(longest chain)
CH, 0.582 g of BaSO, contain 5 = _ x 0.582

(
(d) CH,C—CH,CH, = 0.0799 g
| ee 0.0799 ino
CH,
A20.(d): N=C =N °) = "0.395 *
= 20
ee a : 96 and 9x
Nac Ne =N A57.(c): C:H:N-= 40 | 13,33 | 46.67
ibe 1 14
A38. (d) : When both double and triple bonds are present
= 3.33 : 13.33 : 3.33
in a molecule and if there is choice, the numbering
is done from the end which gives lowest number =] >: 4 31
to a double bond. Empirical formula CH,N
1 2 3 4 5 6
CH, = CH — CH, — CH, — G= CH B. meq from competitive examinations
C, — sp, C,—sp”. 4 3 2 1
A3s. (a) : B1.(c): CH,—CH,CH-CH-—C_Cl
CH 3 | | i
CH, CH, O
1 2 3 4 5 6 f 8 2,3-dimethylpentanoyl chloride
CH,—CH,—_CH—CH—CH,—CH,—_CH,—_CH,,
B3. (a): In hydrazine, nitrogen is not directly attached to
CH,—CH,
4-Kthyl-3-methyloctane carbon. Therefore, it does not give Lassaigne’s test.
B8. (6): Calculating V, from gas equation :
7 6 o 4 5 Ps 1
A39. (a): CH, CH, CH, CH—CH—CH, CH,
PVi = P2V2
CH;—CH, CH, Ty Ts
Correct name: 4-Ethyl-3-methylheptane 700x55 — 760x Vo
300 = 278
A45. (c) : Na+C +N +5 ——> NaCNS
y. _ 100x55 x273
Fe?+ + SCN- ——> [Fe(SCN).]
Ferric thiocyanate
2 ~ 300
x 760
= 46.098 mL
A50. (a): Minimum molecular weight = a 28 46.098
%N = = #100
: 92400 ~ 035.
= 1.568 x 104 = 16.46%
A52. (c) : Mass of halogen in 0.4 g of compound B11. (d): sp? sp? sp”
= ou x 0.188
CH, CH=CH,
188
ORGANIC CHEMISTRY: BASIC PRINCIPLES AND TECHNIQUES

sp? sp?
CH,—CH, B18.
5
6 4 3
sp’ sp. sp
CH,—C=CH 5-Oxohexanoic acid

sp= sp” sp*


CH,—C—CH, B19. (a): PNG, ccliins o alan
lo |e fa H
hn o o

sp" sp sp sp"
H
H H
CH,—C=C—CH,
96 and 2m bonds:
B12. (6) : Unused acid required =80 mL of 0.6 M NaOH B20.
(c): Nucleophile is a Lewis base.
80 mL of 0.6 M NaOH = 40 mL of 0.5 M H,SO, B22. (a): Only structure (III) has H in conjugation with
Volume of acid used = (50 — 40) mL of 0.56 M free radical. Structures I and II donot have any
H,S0, H in conjugation with free radical and hence
= 10 mL of 0.5 M H,SO, donot show hyperconjugation.
% of Nitrogen = B24, (5) : All the three
‘14x Molarity of acid x Basicity x Vol. of acid used (T) In keto-enol tautomerism, ao — H participates

|
Mass of compound taken H;C CH; H;C_ CH,
_14x0.5x2x10 _ -
0.5
3 2 1
B13. (a): Br—CH,—_CH=CH,
O OH
3-Bromoprop-1-ene (II) In this case, a — H participates

|
Ga @
B14, (c) : Isobutyl group is CH,—CH—
CH, — CH, CH,
|
CH, CH
Hs CH,

B15. (a): I. CH,=CH—CH, <> CH, — CH=CH): OH
Stabilised by resonance. (IIT) In this case, y— H participates
+
I], CH, —CH, — CH, ‘Hy /H
: Stabilised by hyperconjugation CH, a» CH,
CH, CH,
m. ¢ Sp CH,<> ey CH, <> OH
B25. (a): Vapour pressure of gas = 725 — 25 = 700 mm Hg
+. = OHS
>C = CH, To calculate volume at S.T.P.
Highly resonance stabilised
P, = 700 mm P, = 760 mm
Greater the number of resonance structures
a = 300K sh = 273K
greater is the stability. Hence correct order
| Bas Avovlvi Pv,
117i . BY
2*2
ppiying Tt “T,
Hi>I1> Il

B16. (a): According to Kjeldahl’s method, = V, _ PVT; _ 700 x 40 x 273


1.4 x Molarity of H,SO, x twice the P,T, 760 x 300
Volume of H,SO, used = 33.53 mL
% of N =
Mass of substance taken
22400 mL of N, at 8.T.P. weigh = 28 g
_ (= x1~x 0)_ 37 950
33.53 mL of N, at S.T.P. weigh = x 33.53
0.75 22400
B17. (5): The hybridisation of ‘C’ in COQ, is sp. Therefore, | = 0.0419 g
the organic compound having same hybridisation
% of nitrogen = eee x 100
as CO, is ethyne. 0.25
= 16.76%
MODERN'S abc + OF CHEMISTRY-XI

CH, 14xN,xV
B48. (a): Percentage of nitrogen =
B26. (d) SNc—a tilly, = Ww

CH,” CH,” _ a = 0.84%


CH, :
(3° Carbonium ion)
B44, (a): In iT ., |, lone pair of nitrogen is taking part in
N
O H
ts 1g 3 4 5 6
B28. (d): H—C — CH—C—CH =CH—CH, aromaticity. In OC. nitrogen is attached to

CH, 0 |
3-keto-2-methylhex-4-enal
NH,
electron withdrawing group. In C) , lone
B31. (c): —NO, group is meta-directing group and will
stabilize an electrophile at m-position. Therefore, pair of nitrogen is in resonance with benzene ring.
Therefore, all these are less nucleophilic.
NO, B47. (e) :
1 2 3 4 5 6 7 8
LO) is most stable
CH, te CH = CERMIIEE ae maT ERNs
Y #¢
B32. (5): CH,—CH—CH—CH, will give CH, °CH,
|
bu, oH, | “CH,

2, 8-Dimethyl-4, 6-decadiene

CICH,—CH—CH—CH, and CH,—C-——-CH—CH, B48. (a) ,

|| || M,V, = M,V,
CH, CH, CH, CH, (HCl) (NaOH)
CH, CH, 0O.1xV=01x 15
6 5 \“/ 3 2 i
B35. (c): CH, —CH, —CH, —C— CH—CH,—CH, Vuc = 15 ml

CH,—CH, ‘. Volume of HCl used by NH,

3-Ethyl-4,4-dimethyl heptane = 20-15=5 ml


B36. (e) : More are the alkyl groups, lesser is the stability ‘14NV
% of N =
of carbanions.
B37. (a): This is because (a) is extra stable due to presence N = Normality of acid used
of conjugated double bond system. V = Volume of acid used
B41. (a): 0.5 M of 10 cm? H,SO, = 0.5 M 20 cm® of NH,
W = wt. of organic compound
1000 cm? of 1 M ammonia contain = 14 g of
nitrogen For HCl, normality = Molarity (" Basicity is 1)

20 cm? of 0.5 M ammonia contain % of N=


14 B49. (5): The boiling points of branched chain alkanes are
— x 20x 0.5 pmitrogen= 0.14¢
~ [000 less than the straight chain isomers.
0.14
Percentage of N = —— x 100 = 28%
t
Tyee
B52. (a):
CH H,C CH
, > «oe
B42. (d) : c—¢ c—¢ H H H H
fo yg
CH, H H H C—C o bonds = 5
trans-but-2-ene cis-but-2-ene C—C rt bonds = 4
H \ CH, a : CH
x a Fas C—H o bonds = 6
C
/ ms
B54, (b) : TN =

H CH, Normality of acid x Vol. used with NH»


1.4x
Wt. of organic compound
H
_ 14x1x 30 _ 35
2
ORGANIC CHEMISTRY: BASIC PRINCIPLES AND TECHNIQUES

B57. (a) : Percentage of 5 =


9 Mass of H,O
32 Mass of BaSO, formed %otH = —x x ———————— _* 100
Xx 100 ° 18 Mass of compound
233 Mass of compound taken

ss oO, Ti
233 ~=-:0.480
BUS = 2 x 0.1014 %100 = 4.58%
18 0.246
& 4 38 +#2 #41
B58. (6): CH, i CH, ‘a CHO Element Percentage At. Relative no. Simple
mass of atoms ratio
OH CH,
4-Hydroxy-2-methylpentanal Cc 21.95 yo EPID _ 13) lore
12
B59. (6): Enolic form of acetone
| 6.
‘oH
“4.58
| 3 een H 4.58 1 is = 4.55 2.5 or 5
CH; —C—CH,, oa
B64: (b): Volume of H,SO, taken = 60 mL
Acetone enol
9 o bonds, 1 m bond and 2 lone pairs.
20:mL of NaOH = 22 mi of { HO
CN 10 9 10 2-4
5 4 3 2| 1
B60. (c) : CH CHaa *. Volume
M —
of Io H,SO , used for neutralisation of

O CH,
ammonia = 60 —10 = 50 mL
2, 2-Dimethyl-4-oxopentane nitrile
" .
Now 50 mL of 55 H,S0,= 2 x 50 mL of NH 3

B61. (d): M
= 100 mLmL of of — NH

1000 mL of 1M NH, contain nitrogen = 14 g

100 mL of — NH, contain nitrogen

= 4 Opaslialed
: : =0.14¢
L000 10

0.14
B62. (d): I, [1 and III are resonating structures of each % of N = x 100
= 10%
14
other.
80 Mass of AgBr formed
B65.(c): % of Bro = —— x «100
—————
vO O O° ? 188 Massof substance
Me~ Me~_ =f
O ra Me <€——~> O : Me
= yey 100
(I) (Ll) 188 250

O O = 24

<>
M ‘ ee, op KA Me B67. (d): CHz,OCH, CH,CH,OH ~ : Functional isomers
Dimethylether Ethanol
(II)
CH,COOH H—C—OCH, : Functional isomers
Ethanoie acid
B63. (ce): SmotC = 12 Mass of CO2 09
Methylmethanoate
te) 44 Mass of compound
CH,CH,CH,CH, CH,—CH—CH; : Chain isomers
= Sey eee x 100 = 21.95% n-Butane |
44 (0.246 CH
Isobutane

CH,CH,CH,OH CH, CH CH, : Position isomers


Propan-1-ol oi

Propan-2-ol
MODERN'S abc + OF CHEMISTRY-XI

B68. (d) : AgF is soluble in water and does not form 12. (d): Maximum dispersal of charge because of two F
precipitate. atoms.
13. (e): Secondary (2°) carbocation is formed as the
1 FJ 3 4 5 6 7
intermediate
B69. (5) :
C
H,C_CHYCH, +H’ —> CH,_CH—CH,
ll 2° Carbocation
CH 14. (b): CH,_Br —> CH, #)Br
3-Ethynyl-1, 6-heptadiene Since Br is more electronegative (2.8) than carbon
B72. (b) : Kjeldahl’s method is suitable for the estimation (2.5), therefore, heterolytic fission occurs in such
of N in aniline. Nitrogen in the ring, -NO, group a way that Br gets the —ve charge and CH, gets
and —Cl group fail to give this test. the +ve charge.
B78. (a): C,H.C=CHCH, shows geometrical isomerism. 15. (5): Double bond is a source of electrons and charge
flows from source of more to less electron density.
H, Therefore, t-electrons of the double bond attack
the proton as
sp” sp? sp sp
B75. (a): H,C=CH—C =N

B78. (b) : (1), (0D, (ID) and (IV) have 6, 5, 3 and 2


hyperconjugating H-atoms. More are
NR
yee
hyperconjugating atoms, more is the stability.
NCERT Exemplar Problems: MCQs Type II
NCERT Exemplar Problems: MCQs Type I sp sp sp sp
16s, 2). (a) H—C =C —C =C —H
4, (c): Electronegativity increases as the state of sp*§ sp sp sp*

hybridisation changes from sp?,to sp? to sp.


Therefore, sp-carbon, CH,CH,C = CH is the most sp? sp sp*
electronegative. (c) CH,==C = CH,
5. (¢): Alkyl halides ‘donot show functional isomerism. sp" ep" sp? sp?
6. (d): Extraction of oils from flowers is done with the (d) C H,==CH — CH=CH,
help of steam distillation. 17. (a, c, d): By making two interchanges of atoms or groups
7. (d): The constituents of ink can be identified and of atoms, bring H below the plane of the paper
separated by thin layer chromatography. and then find out the sequence of the remaining
8. (b): Thin layer chromatography is based on the three atoms or groups in a particular order 1.e.
principle of partition. clockwise or anticlock wise starting from atom
9. (a): CH, >-CH~-CH, _ Stabilized by weak +I effect with highest atomic number (Br) and moving
(1) of two methyl groups towards lower atomic numbers (i.e. Br ——> Cl
— CH,).
CH,—CH —O—CH, Stabilized by strong +R
CH, CH,
dp effect of the -OCH, group
Cc First interchange C
CH,—CH —— CH, —— OCH, Destabilized by —I effect ay a /hn ee Cl—>H
7
Hw]
a -
Say
(TIT) of -OCH, group Br Br
Thus, the correct order of stability of carbocation
CH,
decreases as IT > I > III
Second interchange (Cc Clockwise
4 8 48

10. (bd) : OH yO COs 3,4-Dimethylhexane Br—> Cl qu oP


Cl
"CHA4CH. CH, CH;
|| (a) ‘e| _
First interchange |
C
°CH, *CH, a ; wee
cy / Nap, cl—>H HY l Np
11. (a): CH, — CH, — Cl
H Cl
*CH, — CH, — Mg*tCl
*CH, — CH, — Br CH,

*CH, — CH, — CH,


Second interchange a anti-clockwise
Since Cl has highest electronegativity, structure Cl—> Br Hw ~C]
(a) will have highest positive charge on C*.
Br
ORGANIC CHEMISTRY: BASIC PRINCIPLES AND TECHNIQUES

(b) Clockwise (H is already below the plane


Second interchange
of the paper)
Br GH, $C

ve CH,
| (a), (c) and (d) have different (anticlockwise)
(c) C First interchange
n¢ spatial arrangement of atoms or groups of atoms
ae, NE Br—>H
H wr | Nr
than that given in structure = A (clockwise)
Cl Cl
18. (b,c): In (a) NH, and H,0 are nucleophiles but BF, is
CH, electrophile. In (d) C,H,” is nucleophile but
Second interchange | anticlockwise
yw" Ce H; and C,H.” are electrophiles.
Cl—-> Br
HY Cl
Br 19. (b): II and II are position isomers because they
Br Br differ in the position of Sc=0 group.
@ |
OC First interchange
20. (a,c): Il and III (i.e. option a) and I and IV (option ce)
a6
are not functional isomers.
ch" | H Cl—+H
Hw | Noy
HC CHy

WWW.JEEBOOKS.IN
MODERN'S abc + OF CHEMISTRY
-AXl

Time Allowed : 1 Hr. Maximum Marks : 25

1. Write the bond line formula for heptan-4-one. (1)


2. What are the hybridisation states of each carbon in CH,—CH=CHCN? (1)
3. Which of the following is the most stable carbocation ?
(CH,),CCH", CH,C*HCH(CH,),, (CH,),C* (1)
4, Inthe Lassaigne’stest forN inan organiccompound, prussian blue colour is formed duetotheformation (4)
ee
5. Which of the two :O,NCH,CH,O- or CH,CH,O™ is expected to be more stable and why? (1)
6. Draw resonating structures for : |
(1) C,H,OH (iz) CGH,NO, (2)
7. 0.32 ¢ of an organochloro compound gave 0.628 g of silver chloride in Carius method of estimation.
Calculate the percentage of chlorine present in it. (2)
8. Identify the reagents shown underlined in the following equations as nucleophiles or electrophiles:
(i) CH,COCH, + CN" —> (CH,),C(OH)CN
(iz) CGH, + CH,CO* —~> C,H.COCH,
(iii) CH,COOH + OH- —> CH,COO- + H,O
(iv) CH,CH,Br + SH- —> CH,CH,SH + Br- (2)
9. Discuss the chemistry of Kjeldahl’s method for estimation of N in an organic compound. (2)
10. What is the difference between the following terms :
(i) distillation
(it) distillation under reduced pressure
(iii) steam distillation (3)
11. Give IUPAC names of the following :
(i) HC=C—CH=CH—CH,
CH,—CH, CH, CH,
|
(ii) CH, —CH, —-CH————_ C—CH, —CH, —CH—4CH, ), —CH,
|
CX(CHg),

(iii) ff (3)

12. Write the structural formula of the following :


(i) Pentanedial
(it) 2-(4-isobutylphenyl) propanoic acid
(iii) But-2-en-1-oic acid (3)
13. Explain the following :
(a) CCl, does not give white precipitate with silver nitrate solution on heating.
(6) Nitric acid is added to sodium extract before adding AgNO, for testing halogens.
(c) Alkyl groups act as electron donors when attached to a 1-system. (3)

To check your performance, see HINTS and SOLUTIONS to some questions at the end of
Part II of the book.
HYDROCARBONS

07551
04 WAY DS
Building on..... Assessing..... Preparing for Competition.....
@ Understanding Text 1 @ Quick Memory Test with ¢ Additional Useful Information 124
@ Conceptual Questions 104 Answers 117. @ Topicwise MCQs 126
¢ HOTS & Advanced Level + Competitive Examination Qs
CHAPTER SUMMARY & QUICK : :
@ Questions with Answers 118 +} AIPMT & Other State Boards’
CHAPTER ROUND UP 106 4
anu e REVISION EXERCISES 121 Medical Entrance 128
: : > JEE (Main) & Other State Boards’
+ uae Qs & Exercises with UNIT PRACTICE TEST 152 Engine : Reanteance 133

Solutions 109
> JEE (Advance) for ITT Entrance 137
@ NCERT Exemplar Problems with
@ NCERT Exemplar Problems
Answers & Solutions (Subjective) 113
(Objective Questions) 141
¢ Hints & Explanations for Difficult
Questions 144

AG. compounds which contain only carbon and hydrogen are called
hydrocarbons. These are obtained mainly from petroleum, natural
gas or coal. The hydrocarbons are very important source of energy.
The important fuels like petrol, kerosene, coal gas, oil gas,
\, compressed natural gas (CNG), liquefied petroleum gas (LPG),
liquefied natural gas (LNG), etc. are all hydrocarbons or
their mixtures, Automobiles require fuels like petrol, diesel
and CNG. All these fuels contain mixture of hydrocarbons
which are sources of energy. The automobiles run by petrol
and CNG cause less pollution. Hydrocarbons are also used
for the manufacture of polymers such as polythene,
polypropene, polystyrene, etc. They are also finding use
in manufacture of many dyes and drugs and also as solvents
for paints. Thus, these hydrocarbons play very important role
in our daily life.

CLASSIFICATION OF HYDROCARBONS
Depending upon the types of carbon-carbon bonds present,
hydrocarbons can be classified into three main categories :
1. Saturated hydrocarbons
2. Unsaturated hydrocarbons
3. Aromatic hydrocarbons
1. Saturated hydrocarbons
The hydrocarbons which contain only carbon-carbon single bonds are called saturated
hydrocarbons.
13/1
WWW.JEEBOOKS.IN
a 13/2 MODERN'S abc + OF CHEMISTRY-XI
These include open chain hydrocarbons as well as closed chain (cyclic) hydrocarbons.
If different carbon atoms are joined together to form an open chain of carbon atoms with single bonds, they
are called alkanes.
For example,

CH,—CH, CH,—CH,—CH, CH,—CH,—CH,—CH,


Ethane Propane Butane
These compounds are called saturated because they have maximum number of bonded hydrogens.

2. Unsaturated hydrocarbons.
The hydrocarbons which contain carbon-carbon multiple bonds (double bonds or triple bonds or
both) are called unsaturated hydrocarbons.
Depending upon the multiple bonds (double bond or triple bond) they are further classified as alkenes and
alkynes.
Alkenes. These are hydrocarbons which contain at least one carbon-carbon double bond. For example,
CH, = CH, CH,CH = CHCH, CH, = CH—CH,—CH,
Ethene But-2-ene But-1-ene
Alkynes. These are hydrocarbons which contain at least one carbon-carbon triple bond. For example,
HC=CH CH,C=CH CH,—C=C—CH,
Ethyne Propyne But-2-yne
The saturated and unsaturated hydrocarbons may also contain chains or rings of carbon atoms in their
molecules. These are known as cyclo or cyclic hydrocarbons.
For example,

LAL
Cyclopropane
Lhe
Cyclobutane
Oo
Cyclopentane
*
Cyclohexane

Cyclopropene Cyclobutene Cyclopentene Cyclohexene

3. Aromatic hydrocarbons.
These are hydrocarbons which contain at least one special type of hexagonal ring of six carbon
atoms with three double bonds in the alternate positions.
The ring is called aromatic ring or benzene ring. For example,
CH, CH; CH,

| CO) O cr™
Benzene Toluene Ethylbenzene o-Xylene

The aromatic compounds may also contain more than one benzene rings. For example,

Naphthalene Anthracene

WWW.JEEBOOKS.IN
HYDROCARBONS 13/3 =>

Thus, the types of hydrocarbons may be summarized as :


Hydrocarbon type Characteristic group Example
Saturated hydrocarbons :
Alkanes Only C—C single bonds CH,CH,CH,
Propane

Cycloalkanes Only C—C single bonds Cy


and a cyclic ring Cyclopentane
Unsaturated hydrocarbons :
Alkenes double bond CH,—CH = CH,
Se = C a“ Propene
a % P
Cycloalkenes double bond and ,
a cyclic ring
Cyclohexene
Alkynes triple bond CH,—C =CH
=0 = — Propyne
Aromatic hydrocarbons benzene ring CH,

Methylbenzene

“ CHEMISTRY OF ALKANES :
: =

Alkanes are saturated hydrocarbons containing only carbon-carbon single bonds in their
molecules. They are also called paraffins (from Latin words : parum meaning little and affinis meaning affinity
.e., little affinity or reactivity). This name is justified because under normal conditions of temperature and
pressure, alkanes are inert and do not react with reagents like acids, bases, oxidising agents and reducing agents.
However, under drastic conditions i.e., at high temperatures and pressures alkanes do undergo some reactions.
The alkanes may be divided as :
1. Open chain or acyclic alkanes. These are simple alkanes without any close chains and have the general
formula C,H,,,,. where n is the number of carbon atoms. For example,
CH, CH,CH,CH, CH,CH,CH,CH,
Methane Propane Butane
2. Cycloalkanes or cyclic alkanes. These contain a closed chain or ring in their molecules. They have
the general formula C,H,_. For example,

Q/: A the L
CH, H,C—cCH

H,C CH, H,C —CH,


Cyclopropane Cyclobutane
Methane is the first member of open chain alkanes family. It is a gas found in coal mines and marshy places.
The higher member is obtained by replacing one hydrogen atom of methane by carbon and joining the required
number of hydrogen atoms to satisfy tetravalence of other carbon atoms. For example, next member obtained is

ji i
ethane having molecular formula C,H,.

ney Replace H
Rel Oo or CH,
i) by CH, W Ethane

Similarly, higher members are obtained as C,H, (propane), C,H,, (butane), C-H,, (pentane), C,H, ,(hexane), etc.
a 43/4 MODERN'S abc + OF CHEMISTRY-XI
STRUCTURE OF ALKANES
As already studied, methane, the first member of the family has tetrahedral structure involving sp®
hybridisation of carbon.The four sigma bonds are formed by the overlapping of sp® hybrid orbital of carbon and
1s orbital of hydrogen. In this, carbon atom lies at the centre and the four hydrogen atoms lie at the corners of
a regular tetrahedron, making each H—C—H bond angle of 109.5°.

|
6...
An \/
H H

(c)
Fig. 1. Spatial arrangement of methane.

The structure of methane molecule is also shown in Fig. 1 (6) and Fig. 1 (c). In Fig 1 (c), the thick or dark
lines indicate a C—H bond projecting towards the observer and the dashed line represents
a C—H bond away from the observer (behind the plane of the paper) while the two normal lines indicate C—H
bonds lying on the plane of the paper. In higher alkanes, the tetrahedra are joined together in which C—C and
C—H bond lengths are 154 pm and 112 pm respectively.
However, for the sake of simplicity, the structures are given in two dimensional or planar representation
as shown below . The structures of ethane (C,H,) and propane (C,H,) are also shown.

H H H H H H
HOH H_¢_t_H H—¢_—b_6_H
i Hon hu Ht
Methane Ethane Propane
NOMENCLATURE OF ALKANES
The IUPAC names of some straight chain alkanes are given below:

Methane
Ethane
Propane
Butane
Pentane
Hexane
Heptane
Octane
Nonane
Decane
Undecane
Dodecane
Eicosane
Triacontane

The rules for naming branched chain alkanes have been discussed in the previous unit. Some common examples are:
2 1
CH, CH,CH,
1 2| 3 12383 4 5 6 5 4 3
CH,- C - CH, CH,CHCH,CH,CH, CH,CH,CH,CHCH,
3-Methylhexane
CH 3 CH 3
2,2- Dimethylpropane 2-Methylpentane
HYDROCARBONS

CH. CH CH,
1 2 8 ‘cr 3¢ 6 53 2 1 4
CH,CH CH, CH CH, CH, CH, CH,- C-CH,CHCH,
CH, H, bu,
4-Ethyl-2-methylhexane CH,
(Lowest set of locants and alphabetical 4- Kthyl-2, 4-dimethylhexane
arrangement)

1 2 3 4 65 6 7 1 2 3 4 5 6 7 8
CH,CH,CH,CHCH,CH,CH, CH,CHCH,CHCH,CH,CHCH,
CH, CH CH, CH, CH,CH, CH,
4-(1-methylethyl heptane 4-Ethyl-2, 7-dimethyloctane
or 4-Isopropylheptane

CH, CH, CH,


6 5 4| 3 | 1 1 i 354 5
ne —H,C—O—CH, is H,C—C— CHC CH,

CH, CH, CH, CH,CH,

2, 2, 4, 4-Tetramethylhexane 3-Ethyl—2, 2-dimethylpentane

1 2 3 4 5 6 7
CH,—CH,—CH—CH,—_CH—CH,—CH,
CH,—CH—CH,—CH,
1 2 38 4 5, 6 7 9 10 CH,CH, CH,
CH,—CH,—CH,—CH—CH—CH,—CH,—CH,—CH,—CH, 3-Ethyl-5-methylheptane
(Alphabetical priority order)
CH(CH,),
5-sec—Butyl-4-isopropyldecane
or 4- (1-methylethyl)-5- (1-methylpropyl)decane
1 4 3 4 5 6 7 8 9
(Note : The prefix sec- is ignored while arranging CH, CH, CH, CH, CH—CH,—CH,—CH,
—CH,
alphabetically, but isopropyl is considered as one word).
2
CH,CH, CH,
8 7 6 5 4 tO; 1 3
CH, CH, —CH, CH CH—-C__CH, CH, ———
|
CH CH, CH,CH, CH,
5-(2,2-Dimethylpropyl)nonane
H,C CH,
(Note : Numbering of the complex substituent begins with the
3,3-Diethyl-5-isopropyl-4-methyloctane carbon atom attached to the parent chain).
or 3, 3-Diethyl-4-methyl-5-(1-methylethyl) octane
Cycloalkanes are named by using prefix cyclo-to indicate that the compound contains a ring. As already
discussed in Unit 12 the common rules for numbering the carbons of ring are as follows :
(1) No number is needed if only one group is attached to the ring.
(it) For rings with multiple substituents, begin numbering at one substituent and proceed in the direction
that gives the lowest numbers to the remaining substituents.

For example,
CH
CoH; 1 ACH, 5 ‘
3 7 1 C,H,

3
CH,
Ethylcyclopentane 1, 3-Dimethylcyclohexane 1-Ethyl-2-methylcyclopentane
a 13/6 MODERN'S abc + OF CHEMISTRY-XI
CH,CH,CH, CH;
3
, CHCH, CH, CHyy 3 2 GH,

CH,
1—Methyl—3-propylcyclopentane (1—Methylpropyl) cyclobutane 2—Ethyl—1,4—dimethylcycloheptane

Note : The third compound cannot be named as given below because it will give higher number to the
substituents.

CH,. 4 1-¢.H; CH,. A Sac He


6 1

4 3 6 5

1-Ethyl-2, 6-dimethyl cycloheptane 3-Kthyl-1, 4-dimethyl cycloheptane


T T
Higher Higher

Writing structural formula from the name of hydrocarbon


Just as, it is important to write the correct IUPAC name for a given structure, it is also important to write
the correct structure from the given name. For this, first of all the longest continuous chain of carbon atoms
corresponding to the parent alkane is written. Then after numbering it, the substituents are attached to the
correct carbon atoms and then finally valence of each carbon atom is satisfied by putting the correct number of
hydrogen atoms.
Let us write the structure of 3-ethyl-2,2-dimethylhexane. This involves the following steps :
(i) Draw the chain of six carbon atoms corresponding to the parent compound hexane.
C—C—C—C—C—C
(ii) Number the chain of carbon atoms from any end.
1/2 my 4 6 6
C—C—C—C—_C—C
(iit) Attach ethyl group at carbon 3 and two methyl groups at carbon 2.
CH,
1 2} 3 4 5 6
C—Cc— C—C—C—C
||
CH, CoH;
(iv) Satisfy the valence of each carbon atom (tetravalency) by putting requisite number of hydrogen atoms.
CH,

CH,— CA@gHe.
Cl, —CH,—CH,
(‘e |
CH, C,H,
This is the correct structure of 3-ethyl-2,2-dimethylhexane.

SOLVED EXAMPLES
lL] Examplel.
Assign IUPAC names of the following compounds :
(i) (H,C),C (iL) 2G CH,C(CH,).,

C,H,
(ui) (CH,),CCH,C(CH,), (iv) CH,—(CH,),CH(CH,), CH,

CH,—CH(CH,),
(v) (CH,),C(C,H,), (vi) tetra-tert butylmethane
HYDROCARBONS 13/7 ~~»

Solution: CH,

(z) H,C—C—CH, : 2, 2-Dimethylpropane

3 2 1
(iL) i i i ai : 2, 2, 4, 4-Tetramethylhexane
[5 6
CH,CH, CH,

CH, CH,
1 a 3s a4 5
(iii) CH,—C—CH,—C—CH, : 2,2, 4, 4— Tetramethylpentane
| |
CH ; CH 3
10 9 8 7 6 5 4 3 2 1
(iv) CH,—CH,—CH,—CH,—CH,—CH—CH,—CH,—CH,—CH, : 5- (2- Methylpropyl)decane
1 2 3

3 4 5
(v) CH,—C—CH, —CH, : 8, 3- Dimethylpentane

CH,
|
H,C CH,—C—CH, CH,
1 4 65
(VL) CH, @ §&s #éy : 3,3 -Di-tert-butyl-2,2,4,4-tetramethylpentane
| or 3,3 -Bis (1, 1-dimethylethyl)2,2,4,4-tetramethylpentane
H,C CH,—C—CH, CH,
N.C.E.R.T,
CH,
Example 2.

Write the structural formulas for the following compounds whose IUPAC names are given :
(i) 3-Ethyl-2-methylpentane (ii) 3, 4, 8 - Trimethyldecane
(ii) 3,4, 4, 5 - Tetramethylheptane (iv) 2, 5- Dimethylhexane
Solution: (7) 3 - Ethyl-2-methylpentane

1 2 3 4 5
CH,—CH—CH—CH,—CH,

CH, C,H,
(ii) 3, 4, 8-Trimethyldecane
10 9 | 8 | 7 6 5 4 3 | 2 1
ili Se allie Die Mia ‘all allies Bae
CH, CH, CH,

WWW.JEEBOOKS.IN
Dy 13/8 MODERN'S abc + OF CHEMISTRY-XI
(ii) 3, 4, 4, 5-Tetramethylheptane
CH,
2 3 4 5 6 7
CH,—CH,—CH—-C— CH—CH,—CH,
1

(iv) 2, 5 -Dimethylhexane
1 2 3 4 5 6
CH,—CH—CH,—CH,—CH—CH,
CH, CH,
lL] Example 3.

The names of some compounds are given below :


(i) 5-Ethyl-3-methylheptane
(ii) 4-Isopropyl-5-sec-butyldecane
(iii) 2-Ethylpentane
Why are the given names incorrect ?
Solution: (z) 7 6 5 A 3 9 il
CH,—CH,—CH—CH,—CH—CH,— CH,
CH, C,H,
Ethyl and methyl are at the equivalent positions. The numbering is done from the end which gives
lower number to ethyl group (alphabetically). Hence correct name is :
3-Ethyl-5-methylheptane

(ii) CH(CH,),
1 a 3 4 5 6 7 8 9 10
CH,—CH,—CH,—_CH—CH—CH,—CH,—_CH,—CH,—CH,
CH,—CH—CH,CH,
Prefix sec is not considered as part of name while iso is considered. Hence correct name is
5-sec-Butyl-4-isopropyldecane.
3 4 5 6
(iii) CH, CHC
‘4 1
CH,—CH,
Longest chain is of 6 carbon atoms and not of five. Hence, the correct name is : 3-Methylhexane.

Q@1. Write the IUPAC names of the following structures :

(¢) CH,CH,CH(CH,), CH—CH—CH,CH, (d) (CH,),CCH,C(CH,),


CH, CH, CH,
Ans. (a) 3,3-Diethylpentane (6) 2,3-Dimethylbutane
(c) 3, 4, 8-Trimethyldecane (d) 2,2, 4, 4-Tetramethylpentane

WWW.JEEBOOKS.IN
HYDROCARBONS
hap)
Q 2. Write the structure for the compounds having the IUPAC names :
(a) 3-Ethyl-2-methylpentane (6) 2,3, 5-Trimethylhexane
(c) 2,3, 5-Trimethyl-4-propylheptane

Ans. (a) H,C—CH—CH—CH,CH, (D) OO es

CH, CH,CH, CH, CH, CH,


(c) CH,CH,CH—CH—CH—CH—CH,
CH, CH, CH, CH,
CH,
|
CH,

ISOMERISM IN ALKANES
The alkanes show structural isomerism in which the isomers differ in the skeleton (or chain) of
carbon atoms. For example, the first three members of the family (CH,, C,H, and C,H,) donot exhibit chain
isomerism because they have only one structural formula :

|
H
ian Td
H H H H H H
Methane (CH,) Kthane (C,H,) Propane (C,H,)

Butane (C,H) has two structural isomers :

CH,—CH,—CH,—-CH, may pe es
Butane (n—butane)
b.p. 273K CH,
2-Methylpropane (iso—butane)
b.p. 261K

Similarly, pentane (C,H...) has three isomers.


CH,
11 2 3 4 1 2 | 3
CH,—CH,—CH,—CH,—CH,, an a ia CH,—C—CH,
Pentane (n—pentane) |
b.p.s09K CH, CH,
2-Methylbutane (isopentane) 2,2-Dimethylpropane
b.p. 301K (neo—pentane)
b.p. 282.5 K

It is clear that as the number of carbon atoms increases, the number of chain isomers also increases. For
example, C,H,, has two, C,H,, has three, C,H,, has five, C,H,, has nine, C,H,, has eighteen chain
isomers. The hydrocarbon C,,H., has 75 structural isomers.

Classification of carbon atoms in alkanes.

The carbon atoms in an alkane molecule may be classified into four types as primary (1°), secondary
(2°), tertiary (3°) and quaternary (4°).
_— 413/10 MODERN'S abc + OF CHEMISTRY-XI

(i) Acarbon atom attached to one other (or no other) carbon is called primary carbon (written as 1°
carbon).
(ii) Acarbon atom attached to two other carbon atoms is called secondary carbon (written as 2° carbon).
(ii) Acarbon atom attached to three other carbon atoms is called tertiary carbon (written as 3° carbon)
(tv) Acarbon atom attached to four other carbon atoms is called quaternary carbon (written as 4° carbon).
Hydrogen atoms attached to 1°, 2° and 3° carbon atoms are referred to as primary, secondary and
tertiary H atoms respectively. The following example clearly differentiates the various carbon atoms :

CH,
1° ge 4°] 3° 1°
a a Ir a
wl 1°
CH,CH,

SOLVED EXAMPLES

lL] Example 4.
Write structures of different chain isomers of alkanes corresponding to the molecular formula C,H ,,. Write
their IUPAC names and classify each carbon atom as 1°, 2°, 3° or 4°.

Solution: j- go ge ge ge 1°
(z) CH,—CH,—CH,—CH,—CH,— CH, ; Hexane

1° a? eyo eyo 1°

(1) CH,—CH—CH,—CH,—CH, : 2—Methylpentane


|
1%

CH,

1° eyo 3° go 1°

(LiL) CH,—CH,—_CH—CH,—CH, : 3—Methylpentane


|
* Ole

1° =. -. £ ,
(tv) CH,—CH— CH—CH, : 2, 3—Dimethylbutane
|
CH, 14CH,
1"

CH,
1° 4cl yo ha

(v) CH,—C—CH,—CH, : 2, 2-Dimethylbutane


:

_|
CH,
lL] Example 5.
Write the structures of different isomeric alkyl groups corresponding to the molecular formula C;H,,. Write
IUPAC names of alcohols obtained by attachment of —OH groups at different carbons of the chains.

Solution:
Structures of C,H,, - group Corresponding alcohol
(1) CH,CH,CH,CH,CH,— CH,CH,CH,CH,CH,OH
Pentan-1-ol

(it) CHSC CH CECH, CH, CHCH,CH,CH,

OH
Pentan—2-ol

WWW.JEEBOOKS.IN
HYDROCARBONS 13/11 ~~»

(iii) CH,CH,CHCH,CH, CH,CH, CHCH,CH,


OH
Pentan—3-ol

(w) CH; CHCH,CH,— CH, jee

CH; CH,
3—Methylbutan-1-ol

(v) CH,CH, CHCH, — CH,CH, CHCH,OH


|
CH, H,
2—Methylbutan-1-ol
CH,
| CH,
_ CH,;—C—CH,CH, |
(vi) CH,;—C—CH,CH,
|
OH
2-Methylbutan-2-ol

CH, CH,
7 | |
(vit) CH, —¢— CH, — CH,—C—CH,
—OH
|
CH, CH,
2,2—Dimethylpropan-1-ol

PREPARATION AND PROPERTIES OF ALKANES


Preparation of Alkanes
Alkanes are saturated hydrocarbons with the general formula C,H... Alkanes are mostly obtained from
petroleum and petroleum by products. Petroleum also called crude oil is pumped from wells. The principal
components of crude oil are alkanes. Alkanes are also obtained from natural gas. However, alkanes can also be
prepared in the laboratory. Some of the general methods of preparation of alkanes are discussed below :
1. From Unsaturated Hydrocarbons
The unsaturated hydrocarbons (alkenes and alkynes) are converted into alkanes by catalytic
hydrogenation. In this process dihydrogen gas is passed through alkenes or alkynes in the presence of finely
divided catalysts such as Raney nickel (Ni), platinum (Pt) or palladium (Pd). These metals adsorb dihydrogen
gas on their surfaces and activate the hydrogen-hydrogen bond.
Pd or Pt
or ey Ni
Alkene Alkane
_y Pd or Pt _
eg, H,C—=CH, + 4H, eeRaney Ni CH, —CH,

Ethene Ethane NOTE

CH,CH=CH, + H, -—
+S“ __, CH,CH,CH, Raney nickel is an active form of
: nickel. It is obtained by alloying 50%
Propene Propane Ni and 50% Al (known as Ni-Al alloy).
_ Pd or Pt | The alloy is heated with aqueous
RC=CR + 2 RCH,CH,R |
Hy or Raney Ni a NaOH solution at 50 — 100°C when
Alkyne Alkane Al dissolves and Ni is obtained in the
e.8., -_
HC=CH + 2H, — Pd or Pt
Raneyni? .
CH;-CH; form of very Re fine neyparticles.
Ethyne Ethane

WWW.JEEBOOKS.IN
_— 43/12 MODERN'S abc + OF CHEMISTRY
-XI
Pd or Pt
CH,-C =CH + 2H, or Raney Ni CH, CH, CH,
Propyne Propane
, Ni
CH,=CH,+H, —mgcsax- CH,-CH,
Ethene Ethane.
CH=CH + 2H, gear CH,CH,
Ethyne Ethane
Methane cannot be prepared by this method because starting alkene or alkyne must contain at least two
carbon atoms.
It may be noted that platinum, palladium and Raney nickel catalyse the reaction at room temperature.
However, higher temperature (523—573 K) and pressure are required with nickel catalysts. The hydrogenation
reaction of unsaturated hydrocarbons using nickel at a temperature of 523-573K is commonly known
as Sabatier and Sendern’s reaction or reduction.
2. From Alkyl halides
Alkyl halides (halogen derivatives of alkanes) can be converted into alkanes by the following methods :
(a) Wurtz reaction. When an alkyl halide (usually bromide or iodide) is treated with sodium in dry ether,
a symmetrical alkane containing twice the number of carbon atoms of alkyl halide is obtained.

RIX
+ 2Na + XjR __
Dryether, R-R + 2NaX
D
eg., CH,CH,{1+2Na +I} CH, CH, — CH,CH,CH,CH, + 2Nal
ee etner
Ethyl iodide Butane
This method gives alkane with even number of carbon atoms. If two different alkyl halides are taken for
preparing an alkane with odd number of carbon atoms, a mixture of products is obtained. For example,
D
RX+2Na+XR —- RR+RR +R
etLner
When ethyl bromide and methyl bromide react in the presence of sodium, a mixture of ethane, propane
and butane is obtained. This is because the two alkyl halides not only react with each other but they react
amongst themselves also. For aps)
CH, Br + 2Na + Br| CH,CH, ——~+ _ CH,CH,CH, + 2NaBr
Mena Ethyl fe cullie FA pane
bromide
CH, CH, Br + 2Na + Br: CH, CH, ——> CH, CH,CH, CH, + 2NaBr
Ethyl bromide _____.___ Ethyl Ibromide Satane
CH, iBr +2Na + Bri CH, —— OCH, CH, + 2NaBr
Methyl brodde Methyl browned Ethane
Therefore, Wurtz reaction cannot be used to prepare pure alkanes with odd number of carbon atoms because
it is difficult to separate the mixture obtained.
Two mechanisms have been suggested for Wurtz reaction.
(¢) Through intermediate formation of an organometallic compound :
CH,CH,Br #@iie “= —# CH.CH, Na’ + NaBr
Tam.» Ethyl sodium
CH,CH,Na* + CH,CH,Br ——> CH,CH,CH,CH, + NaBr
Butane
(ii) Through the formation of intermediate free radicals
CH,CH,Br + Na ———> CH,CH, + NaBr
CH,CH, + CH,CH, ——— CH,CH, CH, CH,
Butane

(6) Corey-House Reaction : Wurtz reaction does not give good yields of unsymmetrical alkanes i.e.,
containing odd number of carbon atoms. However, both symmetrical and unsymmetrical alkanes can be prepared
in good yield by Corey-House reaction. In this reaction, the alkyl halide is first treated with lithium metal in dry
ether to form alkyl-lithium which is then allowed to react with cuprous iodide to give lithium dialkyl copper (also
called Gilman reagent). Lithium dialkyl copper thus obtained is treated with suitable alkyl halide to give
desired alkane.
RX + 2Li -Dryether, RT + Lix
Alkyl halide Alkyl lithium
HYDROCARBONS

2R—Li + Cul ———> R,CuLi + Lil


Cuprous Lithium
iodide dialkyl copper

lapse eae [x —— RR’ + RCu + Lix


R
Alkane Alkyl copper
where R and R’ may be same or different alkyl groups. For example,
é.g., CH,CH,Br + 2hLi Dry Aner CH,CH,Li + LiBr
Ethyl bromide Ethyl lithium
2CH,CH,Li+ Cul ———~+ [(CH,CH,),Cu] Li" + Lil
out. ote Lithium diethyl copper
(CHC »)oCu] Li + CH,CH,CH,Br Dry ether , CH,CH,CH,CH,CH, + CH,CH,Cu + LiBr
n-propyl bromide n-Pentane Ethyl copper
(c) Reduction of alkyl halides. Alkyl halides (except fluorides) are reduced to alkanes by suitable reducing
agents. For example,
(t) With zinc and HCl
RX+H, —20/HCI RH
+ HX

e.8., Cities CH af HCl


Chloromethane Methane

CH,CH,Br+2H —“ CH,CH, + HBr


Ethyl bromide Ethane
+
CH,CH,CH,Cl+H, —2%#, CH,CH,CH, + HCl
1-Chloropropane Propane
(ii) With hydrogen in the presence of platinum or palladium (catalytic hydrogenation)
Pdor Pt
RX +H, ——\——~ RH+ Hx

e.8., CH,CH,Br +H, —““-+>


Pd or Pt
CH,CH, + HBr
Ethyl bromide Ethane

(iii) With hydrogen iodide in the presence of red phosphorus

RBr+2Hr Bed RH + HBr + I,


43235K

e.g., CH,CH,Br+ 2HI —**


Red P
, CH,CH, + HBr + I,
Ethyl bromide Ethane

The purpose of red phosphorus is to remove iodine. It combines with iodine forming phosphorus
triiodide (PI,). Therefore, it makes the reaction to proceed in the forward direction. If I, is not removed, it
will convert back alkane into alkyl iodide.
2P + ilk 2PL,
(tv) By zine-copper couple and alcohol
2CH,CH,OH + Zn a (CH,CH,O), Zn + 2H
#n-Cu couple Zinc ethoxide

CH,CH,Br + 2H —M—~ _ CH,CH, + HBr


Ethane
(v) Alkyl halides are also reduced by chemical reagents such as lithium aluminium hydride (LiALH J),
sodium borohydride (NaBH ,), etc.
R—X LiAlH gor
RH
NaBH 4 Alkane
_— 43/14 MODERN'S abc + OF CHEMISTRY
-XI

It may be noted that in the reduction reaction, nascent hydrogen is no longer considered as the reducing
agent. It is believed that there is an electron-transfer from the metal to the substrate followed by the addition
of proton from the solvent (ethanol). Therefore, the reduction of alkyl halides with zinc-copper couple may be
written as:
7n —_—_—> Zn2*
+ Qe™

R — + & ——_— Roe


ee
R:+C,H,OH —> R-—H+-0C,H,
(d) By the use of Grignard reagent. Alkyl halides react with magnesium metal in diethyl ether to form
alkyl magnesium halides. The alkyl magnesium halides are called Grignard reagents after the name of their
discoverer Victor Grignard, who was awarded Noble Prize in Chemistry in 1912 for the discovery of these
versatile reagents. These are highly reactive and can be easily decomposed by water or alcohol to form alkanes.
For example, Dev ether
CH,CH,Br+Mg ——“““> = CH, CH,MgBr
Ethyl magnesium bromide
(Grignard reagent)
CH,CH,MgBr + H,O ———> CH,CH, + Mg(OH) Br
Ethane Hydroxy magnesium bromide
In general, the reaction may be represented as:
RMgX + HOH —— RH + Mg(OH)X
Alkyl magnesium Water Alkane
halide
3. From Carboxylic acids
Alkanes can be prepared from carboxylic acids by decarboxylation reaction and electrolysis.
(¢) Decarboxylation reaction. When sodium salt of monocarboxylic acid is heated with soda lime
(a mixture of NaOH and CaO in the ratio of 3 : 1) at about 630 K, alkane is formed.

RiICOONa
+ Bel NaOlIH
nl AL RO —“°-;
He RH+Na,CO,
Sodium salt “4
of carboxylic acid
In this reaction, a molecule of CO, is removed from carboxylic acid and therefore, the reaction is also called
decarboxylation.
The process of removal of a molecule of CO, from an organic compound is called decarboxylation.
For example,
It may be noted that the alkanes formed contain one carbon atom less than the carboxylic acid.
-—__—__-__ f aaa Cad
CH, | COONa + Na OI — : CH, + Na,CO,
Sodium Methane
ethanoate
CH,CH, |COONa + Na Of]H —“° CH,CH, + Na,CO,
Sodium Ethane
propanoate
It may be noted that sodium hydroxide alone could have been used in the above reaction but soda lime is
preferred because of the following reasons:
Purpose of CaO in sodalime
(i) CaO permits the reaction to be carried out
at a relatively higher temperature so that complete
decarboxylation is ensured.
(it) Sodium hydroxide is hygroscopic (absorbs Soda lime
moisture from air) in nature but CaO present in
soda lime keeps it dry.
For example, methane can be prepared in
the laboratory by heating sodium acetate with soda
lime (in the ratio of 1 : 4) in a copper tube as
shown in Fig. 2. The methane produced is collected
over water because it 1s insoluble in water.
HYDROCARBONS 13/15 =>

(iz) Kolbe’s reaction. When an aqueous solution of sodium or potassium salt of carboxylic acid is
electrolysed, alkane is evolved at the anode.
Electrolysis
2CH,COOK (aq) == — 2CH,COO™ + 2K*

At anode: 2CH,.COO-—
2e— ———-> 2CH.COO
| (unstable)
2CH,COO ———> CH,CH, +2CO,
Kthane
At cathode : K*t as well as H* ions formed by the electrolysis of water will move towards the cathode. But
H* ions will be preferentially discharged at the cathode because the discharge potential of H* ions is lower than
that of K* ions. The H* ions will be discharged to produce H, gas which is liberated at cathode.
Ht + e —m——- H
H + H ——— H,
The Kt ions remain in the solution.
Thus, the reaction may be written as :
2CH.COOK + 2H,O —— CH,CH, + 2CQ, + H, + 2KOH
Ethane
In general,

2RCOOK + 2H,0 Electrolysis .R _R + 200, +H, + 2KOH


Alkane
Methane cannot be prepared by this method.
Several mechanisms have been proposed for the Kolbe’s reaction. The free radical mechanism is the one
now favoured as shown below :

2CH,COO
Na* === 2CH;— C—O + 2Na*
i
At anode

2CH;—_C—O/
1 > 30H; —C—O!
1
Acetate ion Acetate 7
free radical

x O
vA : a= |
2CH,° O40: Uh 2b—0 (2CO,)
. Methyl free radical

CH, + CH, —— CH,CH,


Ethane
At cathode
H,O +e-——> OH + H
. . Free radical
H +H——> H,?
Limitation. Like Wurtz reaction, Kolbe’s electrolysis method can be used to prepare only alkanes with
even number of carbon atoms and not alkanes with odd number of carbon atoms.
(zit) Reduction of carboxylic acids. The carboxylic acids are reduced to alkanes by hydroiodic acid. In
this reaction, the -COOH group is reduced to -CH, group. For example,

RCOOH + 6HI _Reduction | RCH, + 2H,0 + 31,


Alkane
e.8., CH,COOH + 6HI —Reduction_, CH,CH, + 2H,0 + 31,
ie acid thane
é.g., CH, CH, COOH + 6HI __Reduction_, CH, CH, CH, + 2H, O + 3l,
Pranic acid Propane:
_— 413/16 MODERN'S abc + OF CHEMISTRY-XI

Thus, to sum up, carboxylic acids are converted to alkanes as :


R—COONa —@s— RH

R—cooH —“St" , R—CH,


Physical Properties of Alkanes
The important physical properties of alkanes are discussed below :
1. Nature. Alkanes are almost non-polar molecules because of the covalent nature of C—C and C—H
bonds and due to very little difference of electronegativity between carbon and hydrogen atoms. Therefore, the
molecules are held only by weak van der Waals forces (also called London dispersion interactions). The weak
intermolecular forces depend only upon the size (surface area) and the structure of molecules. Due to the weak
forces, the C, to C, alkanes are gases, the next thirteen alkanes from C, to C,, are liquids and the higher
members with more than 18 carbon atoms are solids at 298 K.
2. Boiling point. Alkanes have generally low
boiling points because these are non-polar and the
molecules are held together only by weak van der Waals
forces. With the increase in the number of carbon atoms,
the molecular size increases and therefore, the <
magnitude of van der Waals forces also increases. o =

Consequently, the boiling points increase with increase F


o
in number of carbon atoms. a
a

It has been observed that each carbon added to the =


o
-
chain increases the boiling point by 20-30 K. The boiling |
points of n-alkanes with increase in number of carbon No. of C atoms per molecule
atoms per molecule of the homologous series are shown in Fig/ 8. Variation of boiling points of alkane
Fig. 3. The difference between two lower homologous is “nth sncradae an wiumber or C Atos:
larger than that for the higher ones. . .
The branched chain compounds have lower boiling points than the corresponding straight
chain isomers. For example, the boiling points of isomeric pentanes are :

CH,

CH,CH,CH,CH,CH, aw es

CH, CH,

i ye ao YN ee
No branching
Pentane One branch Two branches
(n-Pentane) 2-Methylbutane 2, 2-Dimethylpropane
(b.p. = 309. K) (Iso-pentane) (Neo-pentane)
(b.p. = 301 K) (b.p. = 282.5 K)
This is due to the fact that branching of the chain makes the molecule more compact and thereby
decreases the surface area. Therefore, the intermolecular attractive forces which depend upon the surface
area, also become small in magnitude on account of branching. Consequently, the boiling points of the branched
chain alkanes are less than the straight chain isomers.

3. Melting point. The melting points of alkanes do not show regular variation with increase in molecular
size. It has been observed that, in general, the alkanes with even number of carbon atoms have higher melting
points as compared to the immediately next lower alkanes with odd number of carbon atoms. For example,

Alkane CH, CyHyy CsHip CoH, CrAig CgHyg CoH CoH.


m.p. (K) 85.5 194.6 1439.3 178.5 182.4 216.2 222.0 243.3
The melting point of a substance depends not only upon the size and shape of the molecules but also upon
the packing of its molecules in the crystal lattice. In n-alkanes, the carbon atoms are arranged in a zig zag
chain. Therefore, in n-alkanes containing an odd number of carbon atoms, the two terminal methyl groups lie
HYDROCARBONS 13/17 =>

on the same side of zig zag chain. On the other hand, in case of n-alkanes containing an even number of carbon
atoms, the two terminal methyl groups lie on the opposite side of the zig zag chain as shown below:

L\/\/\ Z£\S\S
Odd numbered carbon atoms Even numbered carbon atoms

It is clear that the alkanes with even number of carbon atoms have more symmetrical structures and
result in closer packing in the crystal structure as compared to alkanes with odd number of carbon atoms.
Therefore, the van der Waals forces of attraction in the former are more and the melting points are higher as
compared to the alkanes with odd number of carbon atoms.
4. Solubility. The solubility of a compound is governed by the principle that like dissolves like. Therefore,
alkanes being non-polar in nature, are expected to be insoluble in water (polar solvent). They dissolve in non-
polar solvents such as ether, benzene, carbon tetrachloride etc. The solubility generally decreases with increase
in molecular mass. As we know, petrol is a mixture of hydrocarbons and is used as a fuel for automobiles. Petrol
and lower fractions of petroleum are also used for dry cleaning of clothes to remove grease stains. Grease is
non-polar and hence dissolves in non-polar petrol and other lower fractions of petroleum. Hence, grease is
hydrophobic in nature.
5. Density. Alkanes are lighter than water. The density increases with the increase in the number of the
carbon atoms.
The melting points and boiling points of some alkanes are given in Table 1.
Table 1. Melting and boiling points of alkanes
Molecular Name Molecular Boiling Melting point
formula mass point (K)
(u) (K)
CH, Methane 16 111.0 90.5
C,H, Ethane 30 184.4 101.0
C,H, Propane 44 230.8 85.3
C,H, Butane 58 272.4 134.6
C,H, 2-Methylpropane 58 261.0 114.7
C.H,. Pentane 72 309.1 143.3
C,H. 2-Methylbutane ie 300.9 113.1
C,H). 2, 2-Dimethylpropane 72 282.5 256.4
CoH, Hexane 86 341.9 178.5
C,H. Heptane 100 371.4 182.4
C,H. Octane 114 398.8 216.2
C,H, Nonane 128 423.8 222.0
Ci pH Decane 142 447.2 243.3
CopHyo Icosane 282 615.0 236.2

Chemical Properties of Alkanes


Alkanes are saturated compounds and the C—C and C—H bonds are non-polar. Therefore, they are very
less reactive and behave as inert towards acids, bases, oxidising and reducing agents. However, they undergo
some replacement or substitution reactions in which the hydrogen atom of alkanes is substituted by other
atoms or group of atoms.
Some important chemical reactions of alkanes are discussed below :
1. Substitution Reactions of Alkanes
The substitution reactions are those in which an atom or group of atoms in a molecule ts replaced
by some other atom or group of atoms. Alkanes undergo substitution reactions in which one or more hydrogen
atoms are replaced or substituted by different atoms or groups such as halogen atom (Cl, Br or I),
nitro group (—NO,) or sulphonic acid (—SO,H) group. Some of these are discussed below :
(a) Halogenation. This involves the replacement of one or more atoms of alkanes by the corresponding
number of halogen atoms. For example,
_— 413/18 MODERN'S abc + OF CHEMISTRY-XI

Chlorination of alkanes is carried out by treating alkane with chlorine in the presence of diffused sunlight
or ultraviolet light or by heating it to a temperature between 523-773 K.

CH, + Cl —#2> CH,Cl + HC


Chloromethane
(Methyl chloride)
CH,Cl + CL —“> CH,Cl, + HCl
Dichloromethane
(Methylene dichloride)

CH,CL + Cl —™> CHCl + HCl


Trichloromethane
(Chloroform)

CHCl + CL —*»> CCl, + HCl


Tetrachloromethane
(Carbon tetrachloride)
During the reaction, a mixture of different halogenated products is formed and the various halogen
derivatives have to be separated from the mixture.
Bromination occurs similarly but less rapidly.
Iodination is carried out by heating alkane with iodine in the presence of some oxidising agent like
iodic acid (HIO,) or nitric acid (HNO,) or mercuric oxide (HgO) which oxidises HI formed during the reaction.
If HI is not oxidised, being a strong reducing agent, it will make the reaction reversible in nature.
CH 41, CHa “4 HI
Methane Iodomethane
(Methyl iodide)
5HI+HIO, —“t-, /3H,0%8I,
or 2HI+2HNO, ——> 2H,0+2NO, +I,
Fluorination of alkanes takes place explosively resulting even in the rupture of C—C and C—H bonds
forming a mixture of products. Therefore, fluorination of alkanes can be carried out in controlled conditions. It
is carried out by diluting fluorine with an inert gas such as nitrogen or argon. Without special precautions, the
mixtures of fluorine and alkane explode.
Alternatively, alkyl halides are more conveniently prepared by heating suitable chloroalkanes with AsF,,
SbF., Hg,F,, etc. For example,
2CH,CH,Cl 4 Hg —— 2CH,CH,F + Hg,Cl,
Mercurous fluoride Fluoroethane
This reaction iis called WATLEY reaction.

Higher alkanes, on halogenation give different isomers of halogen derivatives. For example, chlorination
of propane gives a mixture of 1-chloropropane and 2-chloropropane as :
CH,CH,CH,Cl 1-Chloropropane (45%)
CL. (n-Propy! chloride)
CH,CH,CH,—
CH,CHCH, 2-Chloropropane (55%)
(Isopropyl chloride)

Halogenation of alkanes proceed through the formation of free radicals. Therefore, it is also called free
radical substitution reaction. The reaction proceeds in the following steps :
1. Chain initiation step. The first step involves the homolytic dissociation of chlorine molecule into two
chlorine atoms or free radicals. The chlorine molecule absorbs light or heat energy, which breaks the bonds
between the atoms. This step initiates the reaction and is called chain initiation step.
HYDROCARBONS 13/19 —_—

dla a’ Cl+ Cl Initiation step


Free radicals
2. Chain propagating steps. The chlorine free radical produced in step (i) attacks methane molecule
and forms HCl molecule and produces methy]| free radical.

|
(Z) H—C—H Oy.+ Cl —_— H—C i 4 HC]
|
Propagation step
H Chlorine 1
free radical Methyl free radical
The methyl free radical further reacts with Cl, molecule to form methyl! chloride (product) and produces
free radical chlorine.

po oh i .
7

(iL) | — A —o + Cl Propagation step


H H Free radical
Methyl chloride
It is clear that steps (Z) and (zi) consume one free radical and generate the other free radical and therefore, they
propagate the reaction. These steps are called chain propagating steps. These are repeated again and again till
whole of CH, and Cl, have reacted to form methyl chloride. Such a sequence of reactions is chain reaction.
3. Chain terminating steps. In addition to above reactions, there are some reactions which remove the
reactive species (free radicals) needed for the propagation of the reaction. These are called chain terminating
reactions. Some of these chain terminating steps are :

aq +a — AC,
CH, + CH, ——> CH,CH,
CH, + cl —— CHCl
Thus , these different steps lead to the formation of methyl chloride. As the reaction progresses, more and
more methyl chloride is formed.
The chlorine free radical can also abstract a hydrogen atom from methyl chloride forming a substituted
methyl free radical. This will react with Cl, molecule to form methylene chloride.

CH,Cl+ cq) —— CHC + HCl


CH,Cl+Cl, /——> WH, Cl, + @ *

Methylene chloride
Similarly, trichloromethane and tetrachloromethane may be formed as represented below :
CHCl + Aaa &—~ CHC, + Hd
CHCl + Cl, —— CHCl, +
Trichloromethane

CHCl, 1 Cl —_— CCl, 1 HCl

co, Sp /ci, — cc, + a


Tetrachloromethane
(b) Nitration, This involves the replacement of a hydrogen atom of alkane with
—NO, group. At room temperature, alkanes do not react with nitric acid. However, nitration of higher alkanes
(hexane or higher) is carried out by boiling alkane with nitric acid.
C,H,, + HNO, ————> ©C,H,,NO, + H,O
Hexane Mi cetegene
The nitration of lower alkanes can be carried out in vapour phase by heating to very high temperature
(723-773 K).
CH, + HNO,—=“*, CH,NO, + H,O
Methane Mitesinathans

CH,CH, + HNO, 723-773K


CH,CH,NO, + H,O
Rihane Nitroethane
_— 43/20 MODERN'S abc + OF CHEMISTRY-XI
However in the vapour phase nitration, the C—C bonds of alkanes may break because of higher temperature
giving a mixture of nitrated products. For example, vapour phase nitration of ethane gives a mixture of
nitromethane and nitroethane.
Vapour phase
CH, — CH, 4 HNO;—seaavaKk CH,CH,N' O, 4 CH,NO, + H,O
Ethane Nitroethane Nitromethane
Higher alkanes give even more complicated mixtures. For example, propane gives the following products:
CH,CH,CH, “255 CH,CH,CH,NO, + CH;—CH—CH, + CH,CH,NO,
Propane 1-Nitropropane Nitroethane (10%)
(25%) NO,
2—Nitropropane
(40%)
+ CH,NO, + other oxidation products (CO,, NO,, H,0O, etc.)
Nitromethane (25%)
The order of reactivity of different hydrogens in this reaction is :
a? Sos 1?
The reaction is believed to occur as:

HONO, —Fomoiyticfision HO + /NY,


673 K

R—H + OX——> R + HO
R+ NO, ——— RNO,
(c) Sulphonation. This involves the replacement of a hydrogen atom of alkane by —SO.H group.
The alkanes undergo sulphonation with fuming sulphuric acid at high temperatures (675-725K). The higher
alkanes (containing six or more carbon atoms) undergo sulphonation to give alkane sulphonic acids. The ease of
substitution is 3° > 2° > 1°.
S03
C,H,, + H,S0O, aes C,H,,50,H + H,O
| Hexane sulphonic acid
CH, CH;
CH,—C—H +H,s0Q,
SO
7/7 ate
(
CH;—C—SO,;H +H,0

A; Hs
2—Methylpropane 2—Methylpropane-2-sulphonic acid
The reaction is believed to occur as:

HO—so,H “© _, HO + SO3H
RH@uO./7— R + HO
R + S O.H A———> R—SO,H
Alkane sulphonic acid
Lower alkanes particularly methane, ethane, etc. do not give this reaction.
2. Oxidation
The alkanes undergo oxidation reactions under special conditions to form a variety of products :
(a) Complete oxidation or combustion. Alkanes on heating in the presence of air or dioxygen are
completely oxidised to carbon dioxide and water with the evolution of large amount of heat. This process is
called combustion.
They burn in air or oxygen giving carbon dioxide, water and liberate large quantities of heat.
CH, (g) + 20,(g) ———> CO,(g)+2H,0@) ; AH® =-890.4
kJ mol"
Methane
2C,H,(g) + 70,(g) ———> 4CO,(g)+6H,0 (J) , AH°= -1580.0 kJ mol!
Ethane ae
C,H,,(g) + = 0,8) ——+ 4CO,(g)+5H,0 (2) ; AH? = -2875.84 kJ mol"
Butane
The general combustion reaction for any alkane may be written as :
sn+*) 0, ———> nCO, +(n+1)H,0
HYDROCARBONS 43/21 >

During the combustion, a large amount of heat is evolved and therefore, the hydrocarbons are used as fuels.
The cooking gas, which is often called L.P.G. (liquefied petroleum gas) is a mixture of propane and butane.
(6) Incomplete combustion. When the combustion is carried out in insufficient supply of oxygen, they
form carbon monoxide and carbon (carbon black or soot). For example,
2CH, (g) + 30,(g) —W™™—5 2CO(g) + 4H,O()
CH,(g) + O,(¢) ———> C\() + 2H,0 ()
limited Carbon black
The carbon black is used in the manufacture of black ink, black pigments, paints, polishes, etc.
(c) Controlled oxidation or catalytic oxidation. The controlled oxidation of alkanes in the presence of
suitable catalysts gives different products :
(i) When a mixture of methane and oxygen in the molar ratio of 9 : 1 is compressed to
about 100 atmospheres and passed through copper tubes at 523 K, methane is oxidised to methanol.
Cu, 523 K
2CH, + O, 100 atm 2CH,OH
Methanol
(Methyl alcohol)
(ii) When methane is mixed with oxygen and passed through heated molybdenum oxide (Mo,O,), under
pressure it is oxidised to methanal.
Mo,O
CH, + O, teat HCHO +, H,O
Methanal
(Formaldehyde)
(vit) Alkanes are oxidised to carboxylic acids by silver oxide (Ag,QO).
2R —CH, + 30, —4#°, 2RCOOH + 2H,0
Alkane Carboxylic acid
e.g., 2CH,CH, +30, ——~
2CH,COOH + 2H,O
Acetic acid
Oxidation of alkanes with manganese acetate also gives carboxylic acids :

2CH,CH, + 30, ““#22"2, 2CH,COOH + 2H,0


Ethane “a Ethanoic acid
(1v) Alkanes having tertiary hydrogen atom can be oxidised to alcohols in the presence of potassium
permanganate (KMnQO,).
cH CH;
CH,—C—H + (AO) “3%, a allan
CH, CH,
2-Methylpropane 2-Methylpropan-2-ol
(Isobutane) (tert-Butyl alcohol)
3. Action of steam. On passing a mixture of steam and methane over heated nickel (supported over
alumina, Al,O,) catalyst at 1273 K, methane is oxidised to carbon monoxide and hydrogen is evolved.
Crh, +/H,O ——> CO + 3H,
Steam
This reaction is used for the industrial preparation of hydrogen from natural gas.
4, Isomerisation. Alkanes isomerise to branched chain alkanes when heated with anhydrous
aluminium chloride (AICI,) and hydrogen chloride gas at 573 K under a pressure of about 30—35 atmosphere.

CH,CH,CH,CH, anhy, — HCl CH,— CH iy CH,

cH
n-Butane Isobutane

CH,(CH,) 4CH, ——— CH,—CHCH,CH,CH, 4 ri i


n-hexane 2-Methylpentane
CH,
3-Methylpentane
The process of isomerisation has been of immense use in petroleum industry for raising the octane number
of a petroleum fraction.
_— 13/22 MODERN'S abc + OF CHEMISTRY-XI
5. Aromatization. The alkanes containing six or more carbon atoms when heated at about 773 K under high
pressure of 10—20 atm in the presence of catalysts like oxides of chromium, molybdenum or vanadium supported on
alumina get converted to aromatic compounds. This process is called aromatization or reforming and is discussed
later. |

__Catalyst Crg03,V205 or Mog03 _,


CH;—(CH,),—CH, = 773K, 10-20 me : ne
n-Hexane Benzene

H,C
Los CH, ty,
| | Crg03 or V905 orMog03/Alg0 Dehydrogenation
or 773 K,10-20 atm “3H :
HC fre Cyclization(—H,)
%~ Benzene
CH, Cyclohexane
n-Hexane

Similarly, heptane gives toluene (methyl benzene),

i
H,C
f* C

CH, |
CH,
| | C or Vo0r or Mog /Alot
773 K,10-20 atm
ma yore Cyclisation, dehydrogenation
Toluene
CH,
n-Heptane

Under similar conditions n-octane gives a mixture of o-, m- and p-xylenes.

CH,—(CH,),—CH, —“"208-%205_cf S0203_/@AH ACH),


773 K, 10-20 atm

n-Octane Mixture of o-, m- and p-xylenes

6. Thermal decomposition or pyrolysis, When higher alkanes are heated to high temperatures (about 700-800 K)
in the presence of alumina or silica catalysts, the alkanes break down to lower alkanes and alkenes. For example,

C,H, ——>—, C,Hy + CH, or C,H, + H,


Propane
The reaction is called fragmentation or cracking or pyrolysis.
Pyrolysis of hexane gives following products :
C.Hj, + H,
Hexene
Pyrolysis
CoH, C,H, + OH
Hexane Butene Ethane
C,H. + C,H, + OH,
Propene Ethene Methane

Pyrolysis of alkanes is believed to be a free radical reaction. Catalytic cracking of petrol or kerosene oil is
employed for the production of oil gas or petrol gas. For example, dodecane a constituent of kerosene oil, on heating
to 973 Kin the presence of platinum, palladium or nickel gives a mixture of heptane, pentene and other products:

Pt, Pd/Ni |
CoH 23k Ci Hi, + C,H, + other products
Dodecane Heptane Pentene
(b.p 489 K) (b.p 371 K) (b.p 309 K)
When methane is heated to a high temperature of the order of 1500 K, it breaks down to its elements.

cH, —""* , Cc + 2H,


Carbon
HYDROCARBONS 13/23 >

The important chemical reactions of methane are summarized below :

cH —%&, cua, —%,. cao, —%, ca,


Methyl Methylene Chloroform Carbon
chloride dichloride tetrachloride
Is * *

HIO, CH,I Methyl] iodide

HNO, *
793 773 K CH,NO, Nitromethane
apour phase

Carbon dioxide
a -

== |e: eatnoy
O
CO, e H,0

Os
Insufficient supp CO or C + H,O Carbon monoxide or Carbon

Cu tub : 00 atm CH,0OH Methanol (Methyl alcohol)

—- HCHO Methanal (Formaldehyde)

4820 HCOOH Methanoic acid (Formic acid)

CONFORMATIONS OF HYDROCARBONS
The carbon-carbon single bond in alkanes is formed when a sp® hybrid orbital of one carbon atom overlaps
with a sp® hybrid orbital of another carbon atom. It is a sigma bond and has cylindrical symmetry. As a result, atoms
involved in single bonds can rotate about the axis of the bond without affecting the overlap of the orbitals that forms
the bond. In other words, rofation about carbon-carbon bond is quite free.
Ss“a

\ / fh, he Fo.
=


Ay,

> <> - iw

Free rotation around


C—Co bond rea er anes

Therefore, a large number of different spatial arrangements of the atoms or groups of atoms attached to the
carbon atoms are possible. These different spatial arrangements are called conformational isomers (conformers)
or rotational isomers (roftamers). Thus,
the different arrangements of atoms in space which can be obtained due to rotation about
carbon-carbon single bond are called conformations.
It may be noted that rotation around a C—C sigma bond is not completely free. It is in fact hindered by an
energy barrier of 1 to 20 kJ mol in different bonds. There is a possibility of weak repulsive interactions between the
bonds or electron pairs of the bonds on adjacent carbon atoms. Such type of repulsive interaction is known as
torsional strain.
Conformations of Ethane
In ethane molecule (CH,—CH.,), the two carbon atoms are bonded by a single covalent bond. If one of the
carbon atoms (methyl group) is kept fixed and the other is rotated about the C—C bond, an infinite number of
arrangements of the hydrogen atoms attached to one
carbon atom with respect to the hydrogen atoms of the
second carbon atom are possible. These infinite number of
different arrangements are called conformations.
To represent these conformations, we can draw three
dimensional pictures. However, chemists represent
conformations in two simple ways as shown in Figs. 4and5.
These are :
(a) Sawhorse representation (6b) Newman projection
(a) Sawhorse representation. In this projection,
the molecule is viewed along the axis of the model from an
oblique angle . The central carbon-carbon bond (C—C) is
drawn as a straight line slightly tilted to right for the sake
_— 13/24 MODERN'S abc + OF CHEMISTRY-XI

of clarity. The front carbon is shown as the lower left hand


carbon and the rear carbon is shown as the upper right Rade cabon
hand carbon. The three bonds around each carbon atom A | os 4
(C—H in ethane or C-C in higher alkanes) are shown by *s ee
three lines (Fig. 4). Front carbon
(6b) Newman projection. This is a simple method to
represent the conformations. In this method, the molecule
is viewed from the front along the carbon-carbon bond axis.
The two carbon atoms forming the o-bond are represented Newman Projection
by two circles; one behind the other so that only the front
carbon is seen. The front carbon atom is shown by a point
Fig. 5. Newman projections of ethane.
whereas the carbon further from the eye is represented by
the circle. Therefore, the C—H bonds of the front
carbon are depicted from the centre of the circle while | H

C—H bonds of the back carbon are drawn from the ee ZH


circumference of the circle at an angle of 120° to each _
other (Fig. 5), |
Let us understand the conformations of ethane. Out of MH q H
all the conformations for ethane, only two extreme |
conformations are important and these are : C
1. Staggered conformation 2. Eclipsed conformation al 4 ra ie
1. Staggered conformation. I[n this Staggered ‘ Eclipsed a
arrangement, the hydrogens of the two carbon atoms conformation conformation
are staggered with respect to one another. As a result,
Fig. 6. Sawhorse representation of staggered and eclipsed
they are at maximum distance apart and have minimum conformations of ethane.
repulsion between them.
2. Eclipsed conformation. Jn this conformation,
the hydrogens of one carbon atom are directly behind those
of the other. Consequently, the repulsion in these atoms is
maximum. The sawhorse projections of these conformations
are represented in Fig. 6. The Newman projections for
staggered and eclipsed conformations of ethane are
shown in Fig. 7. It is clear that when the staggered
conformation is rotated through an angle of 60°, it changes
to eclipsed conformation and similarly, when eclipsed ioe 4
conformation is rotated through the same angle, it gives 60 Staggered Eclipsed
back the staggered conformation. conformation conformation
Relative Stabilities of the Conformations of Ethane Fig. 7. Newman projections for staggered and
The two conformations of ethane differ in their eclipsed conformations of ethane.
relative stabilities. In staggered conformation of ethane,
the electron clouds of carbon—hydrogen bonds are as far
apart as possible and therefore, the staggered
conformation has minimum repulsions between the H-
atoms attached tetrahedrally to the two carbon atoms. On g
the other hand, in the eclipsed conformation the electron Eclipsed (less stable)
clouds of the carbon—hydrogen bonds are closer to each
Energy barrier
other and therefore, eclipsed conformation has maximum
force of repulsion between H-atoms. Therefore, the
staggered conformation is more stable than the eclipsed
conformation. So, when the staggered form changes into
the eclipsed form, the repulsion forces increase and the
molecule will possess more energy and thus loses stability.
These repulsive interactions between the electron clouds s., aHHes,
which affect the stability of a conformation is called ™_Staggered_—
torsional strain. The magnitude of torsional strain 4 (stable)
depends upon the angle of rotation about C—C bond. This . —
angle is also called dihedral angle or torsional angle. |18- 8. The variation of energy versus rotation
Thus, of the conformations of ethane, the staggered form about C—C bond.
HYDROCARBONS wTP43) =>

has the least torsional strain while eclipsed form has maximum torsional strain. Thus, the rotation
around C-—C bond in ethane is completely free. The difference in the energy contents of the staggered and
eclipsed conformations is 12.5 kJ mol". This small barrier to rotation is also called torsional barrier of
the single bond. However, this energy difference is not large enough to prevent rotation. Even at ordinary
temperatures, the molecules have thermal or kinetic energy to overcome this energy barrier. Therefore,
the two conformations of ethane go on changing from one form to another and consequently, it ts not
possible to isolate the different conformations of ethane.
The variation of energy versus rotation about the C—C bond
has been shown in Fig. 8.
The difference in the energy of various conformations
constitutes an energy barrier to rotation.
Conformations of Propane *
The next higher member in alkane series, propane
(CH,CH,CH,) has also two extreme conformations : staggered
conformation and eclipsed conformation. In. Fig. 9, only
Newman projections are shown for simplicity. However, the Staggered propane Eclipsed propane
energy barrier in propane is 14 kJ mol, which is slightly higher Fig. 9. Newman projections of propane.
than that in ethane.
In the eclipsed confirmation of propane, there are two ethane
type H—H interactions and an additional interaction between
C—H bond and C—C bond (of methyl group). The variation of energy
versus rotation about C—C bond as shown in Fig. 10.
Conformations of Butane*
As the alkane molecule becomes larger, the conformation
situation becomes more complex. In butane (CH,CH,CH,CH,),
for example, the rotation about the single bond between two inner 14.0 kJ/mol
carbon atoms (C, and C,) is considered. In this case, all the
Energy
——-»
staggered as well as eclipsed conformations will not have same
stability and energy because of different types of interaction .
3H
between C—C (of methyl) and C—H bonds. The lowest energy ; “Staggered
conformation will be the one, in which the two methy! groups are ; ~H (stable) ;
as far apart as possible 7.e., 180° away from each other.
This conformation will be maximum staggered and is called | Fig, 10, The variation of energy versus rotation
anti conformation (marked I). Other conformations can be about C-C bond in propane.
obtained by rotating one of the C, or C, carbon atoms through
an angle of 60° as shown ahead.
As is clear from the above Newman
projections, the Gauche or Skew
conformations (III and V) are also
staggered. However, in_ these
conformations, the methyl groups are so
close that they repel each other.
This repulsion causes gauche ‘iT
us |
conformations to have about 3.8 kJ mol=| Eclipsed Skew or Gauche
more energy than anti conformation. ,
The conformations II and VI are CH,
eclipsed conformations. These are
unstable because of repulsions. These
are 16 kJ mol less stable than anti
conformation. Conformation IV is also
eclipsed and it is least stable having
energy 19 kJ mol! more than anti IV V
conformation. This is because of Fully eclipsed Skew or Gauche
repulsion between methyl-methyl (same as III) (same as II)
groups which are very close together. It Fig. 11. Different conformations of butane
is called fully eclipsed conformation.
* Conformations of propane and butane are not in CBSE syllabus. These are discussed for competitive examinations.
_— 13/26 MODERN'S abc + OF CHEMISTRY
-XI

The order of stability of these conformations is :


Anti > Skew or Gauche > Eclipsed > Fully eclipsed.
The energy differences between various conformations is shown below :

Fully eclipsed IV

Eclipsed I / | \ Pimees

16 kJ/mol 19 kJ/mol

3.8 kJ/mol

Anti |

Fig. 12, Energy changes during rotation about C,—C, bond of n-butane :

Wey ae 3 4
CH, - CH, - CH, - CH,
LEARNING PLUS
Conformations in Cycloalkanes
Conformational isomerism has also been observed in cycloalkane molecules. Cyclopropane and cyclobutane
are planar molecules —— bond angles of 60° and 90° respectively.

/\
H,C——cH,
Cyclopropane Cyclobutane
Bond angle = 60° Bond angle = 90°
The bond angles in these molecules are quite different from the normal tetrahedral bond angle of 109.5°,
which is anormal angle for sp® hybridised carbon atoms. As a result of deviation from normal bond angle, there
is a considerable sfrain in the molecules of these compounds. Therefore, these are quite reactive. Obviously,
cyclopropane with a bond angle of 60° is more strained and reactive than cyclobutane (bond angle 90°).

Cyclopentane
Bond angle 108° Cyclohexane
Bond angle 120°

In cyclopentane, the bond angle is 108° which is quite close to normal tetrahedral angle. Therefore,
cyclopentane has very less strain and is, therefore, stable. Similarly, the bond angle in cyclohexane ring is 120°
and is expected to be quite reactive and strained. However, cyclohexane is quite stable. The stability of
cyclohexane indicates that the molecule avoids strain by assuming conformation in which all the bond angles
between carbon atoms are close to tetrahedral angle of 109.5°. Therefore, cyclohexane adopts a non-planar
structure. Herman Sachoe proposed two important conformations for cyclohexane based on the fact that the
molecule is non—planar or puckered. These important conformations for cyclohexane are the chair form and
the boat form as shown in Fig. 13. In these structures, the tetrahedral angle is almost retained and the
molecule is free from any angle strain. These structures are also called puckered ring structures. The
names of these forms are due to resemblance of their shapes with chair and boat respectively.
HYDROCARBONS 13/27 —_—

Flagged
a hydrogen ™~#,
aH

Boat conformation
(a) Sawhorse projections

H
Chair conformation a Boat conformation
(b) Newman projections
Fig. 18. Chair and boat conformations of cyclohexane.
Stabilities of the conformations of cyclohexane, The chair conformation of cyclohexane is more
stable than the boat conformation due to the following reasons :
(i) In chair conformation, the adjacent hydrogen atoms on all the neighbouring carbon atoms are quite
staggered and the force of repulsion in them is the minimum. However, in the boat conformation, the
adjacent hydrogen atoms on C,—C, and C,—C, atoms are eclipsed.
(it) The two hydrogen atoms (marked as H,)bonded to the C, and C, atoms in boat conformation are
quite close (distance = 183 pm) and repel each other. These are called flag pole hydrogens. Therefore, the
energy of the boat conformation is more than that of chair conformation.
The energy of the boat conformation of cyclohexane is about 44 kJ mol more than that of chair
conformation. Therefore, cyclohexane exists mainly in the chair form.
Thus, cyclopropane is planar, cyclobutane is almost planar while cyclopentane and cyclohexane are
non-planar.

= SOLVED EXAMPLES

In the alkane H,C—CH,—C(CH.,),—CH,.—CH(CH,)., identify 1°, 2° and 3° carbon atoms and give the
number of H atoms bonded to each one of these.
Solution: 1°, 2° and 3° carbon atoms are labelled as :
he
CH,
1° ge 4°| ge ge 1°

H,C—CH,—C—CH,—_CH—CH,
il 1c |
CH, CH,

No. of H atoms bonded to each C atom :


1° = 3H, 2° = 2H, 3° = 1H and 4° = No H
L] Example 7.
Which of the following has the highest boiling point ?
(t) 2-Methylpentane (i) 2, 3-Dimethylbutane (ut) 2, 2-Dimethylbutane
Solution: 2-Methylpentane has highest boiling point because it has the least branched chain structure as
compared to 2,3-Dimethylbutane and 2, 2-Dimethylbutane. Therefore, it has largest surface area and,
hence has the highest boiling point.
lL] Example 8.
Sodium salt of which acid will be needed for the preparation of propane ¢ Write chemical equation for the
reaction. N .C.E.R.T. |

WWW.JEEBOOKS.IN
_— 13/28 MODERN'S abc + OF CHEMISTRY
-XI

Solution:
Sodium salt of butanoic acid is needed for the preparation of propane.
CH,CH,CH,COONa* + NaOH —“° > CH,CH,CH, + Na,CO,
Propane
lL] Example 9.
Complete the reactions
(i) CH,CH,+ HNO, _ vapour phase_,
a Copper tube
a ——s
(iii) C,H ,COONa + NaOH _£20_,
Solution: (i) CH,CH,+ HNO, —22urP™ase_, CH,CH,NO, + CH,NO, + HO
Nitroethane Nitromethane
ss Copper tube

Methanol

(iii) C,H,COONa + NaOH —“20., C,H, + Na,CO,


Ethane

@ 3. Name the two extreme type of conformations of ethane.


Q 4, Arrange the different type of conformations of butane in the decreasing order of stability.
Q 5. Out of boat and chair form of cyclohexane, which is more stable ?
Q 6. Does the eclipsed conformation of propane has same energy or different as eclipsed conformation of ethane ?
Q 7. Arrange the following in the decreasing order of boiling points :
(7) n-Pentane (ii) Iso-pentane (iit) Neo-pentane
Q 8. Arrange the following alkanes in the increasing order of their melting points :
(t) n-C,H., (it) n-C,H, (tit) n-C,H, (tv) n-C,H,
@ 9. Name the products which may be obtained when a mixture of methyl bromide and ethyl bromide is treated with
sodium metal in ether.
Q 10. Which salt on treatment with soda lime gives ethane ?
Q 11. Name the product obtained by heating butanoic acid with sodalime at 630 K. Name any other acid which also gives
the same product under similar conditions.
Q 12. Draw the structures of the following showing all C and H atoms :
(1) 2-Methyl-3-isopropyl heptane
(it) Dicyclopropyl methane

Answers to Practice Problems


. Staggered, eclipsed /@
. Anti > Skew > Eclipsed > Fully eclipsed
. Chair form is more stable.
. Different energy. Eclipsed conformation of ethane is more stable than that of propane.
« (t) > (it) > (uit)
» (lit) < (t) < (it) < (tv)
ee oo
Oe
2 OM
© . Ethane, propane and butane.
Op
1D
. Sodium propanoate.
6 fb
©- . CH,CH,CH,COOH + 2NaOH —“2°_, CH,CH,CH, + Na,CO, + H,0.
Butanoic acid Propane
2-Methylpentanoic acid also gives the same product (propane).
CH,
HCOOH + 2NaOH —“*2> CH,CH,CH, + Na,CO, + H,0.
CH,
2-Methylpropanoic acid Propane

WWW.JEEBOOKS.IN
— CHEMISTRY OF ALKENES )

Alkenes are unsaturated hydrocarbons containing carbon— Te Unhybridised


carbon double bond (C=C) in their molecules. They have the general es 1 Bond (p-p) —~ 2p-orbital
formula C_H,,. The simplest member of the alkene family is Cee ee
ethene, C,H,. The alkenes are also called olefins (Greek olefiant
meaning oil forming) because the larger members of the series (such
as ethylene, propylene, etc.) react with chlorine to form oily products.
STRUCTURE OF DOUBLE BOND
Carbon—carbon double bond in alkenes consists of one sigma
(o) bond and one pi (m) bond. For example, in the formation of
ethene, one sp” hybridised orbital of carbon overlaps axially (head
on) with the similar sp? hybrid orbital of other carbon atom to O Bond (sp*-sp’)
form a stable o bond. On the other hand, the unhybridized 2p, Fig. 14. Orbital picture of ethene.
orbitals of two carbon atoms overlap sidewise to form 7 bond.
Each of the remaining two sp* hybrid orbitals of each carbon atom
overlaps axially with the half filled 1s orbital of hydrogen to form
o bonds as shown in Fig. 14.
Electron diffraction studies have shown that ethene is a flat
molecule t.e., all the bonds in the molecule lie in a plane.
The H—C—H bond angle is 116.6° and C—C—H bond angleis 121.7°
(Fig. 15). The values of the bond angles are quite close to the values
for sp“ hybridised molecules. The values of the bond lengths for C— . . ee Maar
H and C=C bonds are 110 pm and 134 pm eaeran oy y
ia aes eee poaBuenareen
It may be pointed out that the carbon-carbon bond length in
ethane molecule is 154 pm whereas that in ethene molecule is 134 Tt Bond
pm. This means that C=C bond is shorter than C—C bond. This |
can be explained on the basis of extent of overlapping of orbitals. | 4 c——cu,
Carbon—carbon double bond involves the formation a o bond and °
a 1-bond. For the formation of 1-bond the bonding orbitals have to GO Bond
come close together which shortens the distance between the 3 3 , ae
constituent atoms. Moreover, the effective size of sp? hybrid orbitals Eee eee
is smaller than sp? hybrid orbitals. This also decreases the distance between the constituent carbon atoms in ethene
molecule.
Nature of the double bond
The C == C bond often known as ethylenic double bond, is made up of ‘a sigma (o) bond and a pi (1%) bond (Fig. 16). The
sigma bond is a strong bond having bond dissociation enthalpy of about 397 k.J mol while pi bond is a weak bond having
bond dissociation enthalpy of about 284 kJ mol". This is because o-bond is formed by head on overlapping of orbitals while
the 1-bond is formed by lateral or sidewise overlapping of the orbitals. Since extent of overlapping is less in case of m-bond
than o-bond, therefore, a 1-bond is weaker bond than a o-bond. Due to the presence of weak 1-bond, alkenes are highly
reactive molecules and have a tendency to take part in chemical reactions. Strength of the double bond (bond enthalpy
681 kJ mol) is greater than that of a carbon-carbon single bond in ethane (bond enthalpy 348 kJ mol").
Geometrical Isomerism: Hindered rotation about the double bond.
From the orbital picture of ethene, it is evident that a double bond consists of a o-bond and a 1-bond. The two 2p,
orbitals participating in the 1-bond are parallel to each other. The rotation of one carbon atom through 90° breaks the
t-bond because in that case, the 2p, orbitals become perpendicular to each other and no sidewise overlap is possible.
™ bond

On rotation | |
through 90°

Lobes are perpendicular to each other


—> no overlap possible

But the breaking of m-bond requires 284 kJ mol! of energy which is not provided by the collisions of
molecules at room temperature. Hence, the rotation about the double bond is restricted or hindered
which leads to geometrical isomerism (discussed later).
_— 413/30 MODERN'S abc + OF CHEMISTRY-XI

NOMENCLATURE OF ALKENES
Most alkenes are conveniently named by the IUPAC system. But common names are also sometimes used
for simple compounds. These are already discussed in previous unit. Some of these are :
CH, =CH, CH,CH =CH, CH,CH,CH =CH, CH,CH=CHCH,
Ethylene Propylene a - Butylene 6 - Butylene

CH,
CH.(CH,),CH =CH, CH,(CH,),CH = CH, CH, = C—CH,
a - Pentylene a - Hexylene Isobutylene
According to IUPAC system alkenes are named similar to alkanes with the following modifications :
(i) The longest continuous chain should include both the carbon atoms of the double bond.
(ii) The suffix used for alkene is -ene.
(ii) The chain is numbered from the end that gives the lower number to the first carbon atom of the double bond.
(iv) If there are two or more double bonds the ending ane of the alkane is replaced by adiene or atriene.
For example,
CH3 —CH= CHa CH3CHg —CH—CHo9 CH, —CH=CH—CH;
Propene But-1-ene But-2-ene
CH, 1C
1 2 3 4 6 1 2 3 45 6 2|| 3 4 #5
CH,CH=CH : CH, am a ca CH,CH,—C—CH, CH, CH,

CH, CH, CH,


4-Methylpent-2-ene 3,4,5-Trimethylhex-2-ene 2-Ethylpent-1-ene

= =CH, CH,
1 2 3, 4 5! 6 1 2 3 44
CH; CH,=CH-CH CH,CH CH, CH, = CH—CH—CH,
2-Methylprop-1-ene |
CH, CH,CH, CH;
5-Methyl-3-propylhex—l-ene 3 - Methylbut - 1- ene
The compounds containing two double bonds are named as dienes :
4 3 2 1 1 2 3
CH, = CH—CH=C C = CH—CH—CH,—CH,,—CH Wj 2 VA4
Bila Lo-diens “ “2 4l 5 ° ° :
CH, = CH, 4 3 a)
3-Propylpenta-1, 4-diene 4-Methylpenta-1, 3-diene
ISOMERISM IN ALKENES
Alkenes exhibit the following isomerisms :
1. Structural isomerism
Alkenes show the following types of structural isomerisms :
(¢) Chain isomerism, The isomers differ with respect to the chain of carbon atoms. As in alkanes, ethene
(C,H,) and propene (C,H,) can have only one structure but alkenes higher than propene have different structures.
For example, the alkene having molecular formula C,H, exists as two chain isomers.
4 3 2 1 2 1
CH,— CH,— CH — CH, CH,— C =CH,
But-1-ene |
(o-Butylene) CH,
2-Methylpropene
(Isobutylene)
(uz) Position isomerism. The isomers differ in the position ofthe double bonds. For example,
CH,— CH,—CH = CH, CH,— CH = CH—CH,
But-1l-ene But-2-ene
Thus, there are three structural isomers for the molecular formula C,H.
CH,—=CH—CH,—CH, CH,—CH=CH—CH, ee amie

But-1l-ene But-2-ene CH
2-Methylpropene
HYDROCARBONS

2. Geometrical isomerism
The geometrical isomerism is shown by alkenes.
The compounds which have the same structural
formula but differ in the spatial arrangement of atoms
or groups of atoms about the double bond are called
geometrical isomers and the phenomenon is known
as geometrical isomerism. The geometrical
isomerism is exhibited by certain members of the
alkene family.
Cause of geometrical isomerism. We know
that a double bond consists of a o-bond and a 71-bond.
Due to the presence of m-bond, the groups or atoms
attached to each sp?-hybridised C-atom cannot rotate
about it. They can do so only by breaking the 7-electron
cloud and require energy of the order of 284 kJ. Therefore, the rotation about the double bond is restricted or
hindered. Because of the hindered rotation, the molecule of the type C,A,B, exhibits two space isomers which
are also called geometrical isomers.
The isomer in which similar atoms or groups lie on the same side of the double bond is called cis-isomer
while the other in which they are displaced on opposite sides, is called frans-isomer.
The geometrical isomerism is also called cis-ftrans isomerism.
Conditions for geometrical isomerism. The main conditions for geometrical isomerism are :
1. The molecule must have a double bond.
2. The two atoms or groups attached to the same carbon atom must be different.
For example, but-2-ene (CH,CH = CHCH.,) exhibits geometrical isomerism.
a Same sid \ pp .
£,Opposite .
sides

CH; CH

— _— a Ce
cis-But-2-ene trans-But-2-ene

But its position isomer but-l-ene (CH,CH,CH = CH,) does not show geometrical isomerism because the
two same hydrogen atoms are attached to the double bonded carbon atom.

= am en is same as — aie
a N\ CH, —" we
Not geometrical isomers
Other examples are :
(z) 1, 2-Dibromoethene

\ le ."~< —oae Pu

n/m\ Np! Br Ng
trans

(it) Maleic acid and fumaric acid

H H H COOH
~~ ee ee
Hooc?~ Be HOOC™ “Se
Maleic acid (cis) Fumaric acid (trans)
Geometrical or cis — trans isomerism is also shown by alkenes of the type XYC = CXZ and XYC = CZW.
Geometrical isomers have different physical properties. However their chemical properties
are similar but not identical. These can be distinguished on the basis of many physical properties such as melting
point, boiling point, solubility, dipole moment, refractive index, stability, etc. It may be noted, for example, that
(1) Cis-isomer is more polar than frans-isomer. For example, dipole moment of cis-dichloroethene is 1.90 D
while that of the frans-form is almost zero. Similarly, dipole moment of cis-but-2-ene is 0.85 D while
dipole moment of ¢rans-but-2-ene is almost zero or it can be said that trans-but-2-ene is nonpolar. This

WW.JEEBOOKS.IN
_— 43/32 MODERN'S abc + OF CHEMISTRY-XI

is clear from the figures that in cis form two bond polarities add up to give resultant polarity. But in
case of trans alkenes, the bond polarity being in opposite directions cancel each other giving almost
zero polarity of the molecules.

Hy. 01 CL H
— Ss; a _% Bs Hy ae

p= 190). p= 0.001 D n= 0.33D n=O D


(cis) (trans) cis—But-2-ene trans—But-2-ene

(11) Cis-isomer has higher boiling point than frans-isomer :


CH
\ CH
1 Pw H

a ~~ ne Pe CH,

cis-But-2-ene frans—But-2-ene
(b.p. = 277K) (bp. = 274K)

(iii) In case of solids, the trans- isomer has higher melting point than cis-isomer. This may be attributed
to better symmetry of the trans isomer due to which the trans solids fit very well into the crystal
lattice. Therefore, trans isomers have high melting points.
(iv) Cis-isomer has higher refractive index than trans-isomer.
(v) The stability of trans-isomer is more than cis-isomer.

E, Z-System for Geometrical Isomers


When there are three or four different groups attached to the carbon atoms of a double bond, it becomes
difficult to assign cis and trans designation to isomers. To overcome this problem, a more general system of
geometrical isomers called the E and Z system was devised.
The E and Z system is based on the assignment of priorities to the atoms or groups attached to each
carbon of the double bond. The priorities can be assigned on the basis of the rules given by Cahn-Ingold and
Prelog. The rules are:
1. Ifthe four atoms attached to the double bonded carbon atoms are different, priority depends on the atomic
number. The atom with higher atomic number gets highest priority. For example, if the four atoms
attached are F, Cl, Br and I, the priority becomes :
I>Br>Cl>F
2. If two isotopes of the same element are present, the isotope of higher mass number gets the higher priority.
For example, deuterium (D) gets priority over hydrogen (H).
3. Iftwo atoms are identical, the atomic numbers of the next atoms are considered for priority assignment. For
example, if CH, and C,H, are attached, C,H, gets priority because atoms attached to first C atom are C, H
and H whereas in CH, group, atoms attached are H, H and H only.
4. Where there is a double bond or triple bond, both atoms are considered to be duplicated or triplicated.
For example, A Cc

| | ||
—C= A» equals —C—A and —C=A equals —C—A
| | |
A C A C
O N
I | oc C
eg. R—C—R equals R— 5 —R and R—C Ill =Nequals R— ae Sse
O—C N
E, Z-system. If the two higher priority groups are on the same side of the double bond, the isomer is called
Z (German, Zusammen meaning together) and if the two higher priority groups are on the opposite side of the
double bond, the isomer is called E (German, enigegen meaning opposite).
HYDROCARBONS

high priority high priority high priority low priority


Broup ~~ ve Sroup pone ee
C=C
oe
C—

low priority low priority low priority high priority


group (Z) group group (RE) group
(high priority groups on (high priority groups on
the same side) the opposite side)
CH, a. CH, ~ a

C=C C=C
Sia, ue.
is (Z) Mm | (EK) CY, The atoms or groups
(Priority CH, > H) of higher priority are
For example, Br ; shown bold coloured.
F Cl Br

a ._ a aa
(Z) (E)
(Priority : Cl > F and I > Br)

CH, Ae p weak ae
Ye
ial
— ~ CH,CH, H Cl
(Z) (E)
(Priority :CH, > H and Cl > CH,CH,)

CH; wa ow yor

YK C———C
CH,CH,
Le
(Z)
OH CH,CH,

(KE)
Na
(Priority : C,H, > CH, and OH > H)

F Cl F CH,CH,

H
va
(Z)
\oucu, H
a
(E)
Na
(Priority : F > H and Cl > C,H,;)

SOLVED EXAMPLES
Example 10,
Write IUPAC names of the following compounds :
(i) CH,CH=C(CHs3), (ii) VAN

(iii) CH;-CH=CH—CH,—CH=CH—CH—CH,—CH=CH, (iv) ( )-CHy-CH,—CH=CH,


C,H,
Solution:
4 3 2 1
(i) CH,—CH=C—CH, (ii) CH,—CH—CH==CH,
CH, Penta-2, 4-diene
2-Methylbut-2-ene
_— 13/34 MODERN'S abc + OF CHEMISTRY-XI

1 9 8 7 6 5 4. 8 2 1 . 43 e. 4.
(tit) i aie i i (tv) (CH; -CH, CH=CH,

C,H, 4-Phenylbut-1l-ene
4-Ethyldeca-1, 5, 8-triene

lL} Example 11.

Write IUPAC names of the following compounds and calculate the number of sigma and pi bonds in these
(i) (CH,),CH—CH =CH—CH,—CH =CH— CHCH, (iu) J £<2e>sJ0
|
CoH;

(iii) CH,= C(CH, CH, CH,), (iv) CH;CH,CH, . CH,CH,


CH, = Sg me

Solution:
1 2 38. 4 5 6 7 8
(t) CH,—_CH—CH = CH—CH,—CH = CH—CH—CH,
10
H, “CH, —CH,
2,8-Dimethyldeca-3,6-diene
o-bonds = 33, m-bonds = 2
2 4

® A Nea Nt Nin
Octa-1,3,5,7-tetraene
o-bonds= 17, m-bonds = 4

> -CH,—CH,—CH,
ii) CH, =CC
2-Propylpent-1l-ene
o-bonds = 23, n-bond = 1

10 9 8 7

wv _CH—CH= cu, éCH—CH,


CH,
4-Ethyl-2,6-dimethyldec-4-ene
o-bonds = 41, m-bond = 1
lL} Example 12.
Draw cis and trans isomers of the following compounds and write their IUPAC names
(a) CHCl = CHCl (iL) C,H,C(CH,) = C(CH,)C,H;
Solution:

HA HY. ~~
i of Na
eis-1,2-Dichloroethene trans-1,2-Dichloroethene

WWW.JEEBOOKS.IN
HYDROCARBONS

2 C= Co 5 —
CH,CH, CH,CH, CH,H,C cu,
cis-3,4,-Dimethylhex-3-ene trans-3,4,-Dimethylhex-3-ene

lL} Example 153.


Draw cis and trans8 isomers of hex-2-ene. Which isomers will have higher boiling point and why ?
Solution: oe 4 CH,CH,CH, ae

a oN, a CH,CH,CH,
cis- Hex-2-ene trans-Hex-2-ene
The cis form will have higher boiling point due to more polar nature of the molecule leading to strong
intermolecular dipole-dipole interactions. As a result of stronger intermolecular interactions it requires
more energy to separate the molecules.
lL] Example 14.
Write the structures and IUPAC names of different structural isomers of alkenes corresponding to the molecular
formula CA jo, 7 ,

Solution:
(a) CHy=CH—CH, —CHOH, Pent-l-ene

(b) CHCH= CHC, Ch, Pent-2-ene


2-Methylbut-1l-ene

. a 2 38 4
(d) CH,=CH—CH—CH, 3-Methylbut-1-ene

CH,
3 4
(e) CH,—G=
— CH—CH, 2-Methylbut-2-ene

bn,
lL} Example 15.
Which of the following compounds will show geometrical isomerism ?
(a) H,C = CBr, (0) /(H,C),C = CHCH, (c) C,H,CH = CHCH,
(d) CH,CH. af = CHCH,CH, (e) H,C—CH =CClCH,)

CH,
Solution: (c), (d) and (e) show geometrical isomerism.

lL} Example 16.

rae the following as Z or E isomers :


(a) (b) CH, H

\ a “se i

Cl oN, C,H, J oN OH
(c) (d) Cl Br
>< i
ya we

WWW.JEEBOOKS.IN
_— 13/36 MODERN'S abc + OF CHEMISTRY-XI

(e) H
ee < CN
C“ CH, NH,
Solution: (a)Z (b)2Z4 (e@E (ME (eke

Q13. Which of the following molecules can exhibit geometrical isomerism ? —)


(i) 2-Methylbut-2-en-1-ol (ii) 8-Methylbut-2-en-1-ol (diz) f
‘ * ‘ * ih, ae
Q@ 14. Which of the following alkene does not have E configuration according to E, Z system ?
(i) H ee - i = CH, (iL) Et Sp - wee (zit) — ee H

cH = cH, CH “dH OH Non


@ 15.; the following as E or Z isomers :

3
C,H. (it) HG
56
CH 3 (vit) H Br
“Sc ToCH, = Cac c=c
H,C \H cl———s Xv
(iw) CI Au

a.
- Answers to Practice. Problems
©13. (i)
© 14. (iii)
©15. @)Z (w)Z (ui)E (w)E

STABILITY OF ALKENES
The relative stability of alkenes can be determined by comparing their enthalpies of hydrogenation. Enthalpy
of hydrogenation is the heat evolved when one mole of alkene reacts with hydrogen. For example, both
the geometrical isomers of but—-2—ene consume one mole of H, and produce the same product, butane upon
catalytic hydrogenation in the presence of a catalyst such as ~_— or-

le —2 CH, CH,CH,CH, <<=2—


Pd
oN a
Butane a
cis-But-2-ene a 2-ene

cis—CH,CH = CHCH,
trans-CH,CH=CHCH,(5 kJ mol!

E
N Trans-isomer has 5 kJ mol!
120.0 k.J mol! 115.0 kJ mol-!
E less energy and hence more
stable than cis—isomer
R
G CH,CH,CH,CH,
Y

Fig. 17. Enthalpies of hydrogenation and stability of cis- and trans but-2-ene.
HYDROCARBONS 13/37 >=

It has been observed that enthalpy of hydrogenation of cis-isomer is 120.0 kJ mol! and that of trans-
isomer is 115.0 kJ mol“. Since the product has the same energy in both the reactions, any difference in the heat
produced must be due to the difference in the energies of the starting alkene. The trans-isomer evolves about
5 kJ mol! less heat than the cis-isomer, it means that it contains 5 kJ mol! less energy. In other words, the
trans-isomer is 5 kJ mol! more stable than the cis-isomer as shown in Fig. 17.
This is due to the fact that when large substituents are present on the same side of the double bond, as in
a cis-isomer, their electron clouds can interact with each other resulting steric strain in the molecule. Because
of steric strain, the molecule becomes less stable. On the other hand, when the large substituents are present
on the opposite sides of the double bond, as in the frans-isomer, their electron clouds cannot interact and hence
there is no destabilizing steric strain.

Se aan Pi \
i a a te CO aa
H7 a a 4>: 2 it
BY
ae yy
.
-, C= 5
oer —

—— — nae
9 C a H

cis-But-2-ene Se i
(steric strain) trans-But-2-ene
(no steric strain)

Enthalpies of hydrogenation show that the stability of alkene also depends upon the position of double
bond. For example, enthalpy of hydrogenation of some alkenes are given below :
CH, CH, CH, CH = CH, CH,CH,G =CH, OCH,C=CHCH, CH,C=C—CH,
(monosubstituted)
Pent-1-ene CH, H, CH, CH,
AH® = -126 kJ mol! (disubstituted) (trisubstituted) (tetrasubstituted)
2-Methylbut-1-ene 2-Methylbut-2-ene 2, 3-Dimethylbut-2-ene
AH°=-119.5 kJ mol-! AH° =-112.5 kJ mol-! AH° =-111.5 kJ mol"!
The above data indicate that the alkenes become more stable with increasing substitution. In other words,
the greater the number of alkyl groups atiached to the double bonded carbon atoms, the more
stable ts the alkene.
Thus, the order of stability of alkenes is

PREPARATION AND PROPERTIES OF ALKENES


Preparation of Alkenes
Alkenes can be prepared by the following methods :
1. From alky] halides or haloalkanes, Alkenes can be prepared from alkyl! halides (usually bromides or
iodides) by heating with alcoholic potash (potassium hydroxide dissolved in ethanol). This reaction removes a
molecule of halogen acid (HX) and therefore, the reaction is called dehydrohalogenation.

Ale. KOH
RH,C -CH,-X areas RHC = CH, + HX
A=UlL Br
€.2., CH,CH,Cl + KOH ——> CH, =CH, + KCl +H,O
Chloroethane (alc.) Ethene
(Ethyl chloride)
CH,CH,CH,Br + KOH ———+ CH,CH = CH, + KBr + H,O
Bromopropane (alc.) Propene
(Propyl bromide)
_— 413/38 MODERN'S abc + OF CHEMISTRY
-XI

In this reaction, the hydrogen atom is eliminated from $8 carbon atom (carbon atom next to the carbon to
which halogen is attached). Therefore, the reaction is also called B-elimination reaction.
H
B los 7 = ae =e
H-C¥C-H +KtOH- —a#eKOH + HO + KX
ale. > :
H xX X=Cl, BrorI ‘ieee
The ease of dehydrohalogenation of different halogens is :
Iodine > Bromine > Chlorine
For alkyl groups, the ease of reaction is :
Tertiary > Secondary > Primary
CH,
|
CH._CH—Br + KOH ——>+CH,CH=CH, + KBr+H,0
Isopropyl bromide (alc.) Propene
In case the alkyl halide can undergo dehydrohalogenation in two different ways, that alkene is preferred
which is more alkylated, i.e. have more number of alkyl groups attached to double bonded carbon atoms. This
is known as Saytzeff rule. For example,

CH,
a
— C = CH — CH,
2-Methylbut-2-ene (80%)
CH (I) Major product, (more alkylated)
l 3

cH = Ch CH
|
cnHeat
Br

2—-Bromo—3—methylbutane GH,
CH,— CH — CH=CH,
3-Methylbut-1l-ene (20%)
(IT) Minor product (less alkylated)
The alkene (1) is trialkylated and alkene (II) is monoalkylated. Therefore, alkene (I) is the major product.
2. From dihalogen derivatives (vicinal dihalides). Dihalogen derivatives of alkanes in which two
halogen atoms are attached to adjacent carbon atoms (called vicinal dihalogen derivatives) are converted to
alkenes by heating with zinc dust in ethanol. For example,
CH,Br /
+ Zn ——"__» (CH.=CH. + ZnBr
| Ethanol 2 2 2
r Ethene
1, 2-Dibromoethane
(Ethylene dibromide)

CH, — CH— CH, + Zn —", CH, — CH =CH, + ZnBr,


| | Ethanol Propene
Br Br
1, 2-Dibromopropane
(Propylene dibromide)
This reaction of removal of halogen (Cl, Br, or I,) from a dthaloalkane to form an alkene is
called dehalogenation reaction.
3. From alkynes, Alkynes can be reduced to alkenes using palladium-charcoal (palladised charcoal) catalyst
partially deactivated with poison like sulphur compounds or quinoline. Partially deactivated palladised charcoal
is known as Lindlar’s catalyst. Alkynes can also be reduced to alkenes with sodium in liquid ammonia (called
Birch reduction).
_ Pd/C, Ho
R—C = C—R or Nalliq. NHg R—HC = CH—R
Alkyne Alkene
e.g., HC =CH _FdiC,
He . HLJC=CH,
Ethyne Ethene
HYDROCARBONS RIK} >

CH,C=CH+H, —syeitwa? CH,CH=CH,


Propyne Propene
_ Pd/C, Ho
Cf ., CH,—C = C—CH, or Na/liq. NH3 CH,CH = CH CH,

But-2-yne But-2-ene
It may be noted that reduction of alkynes in presence of palladium supported over BaSO, or CaCO, and
partially poisoned by 5 or quinoline (Lindlar’s catalyst) gives predominantly cis isomer. However, if
alkynes are reduced with lithium or sodium in liquid ammonia (Bireh reduction) gives predominantly
trans alkene.

CH, CH,
H», /Pd/BaSOQ, +5
Lindlar's catalyst ne C= 0S
: eae H
cis- But-2-ene

CH, H
Ae 4
H- /SE
trans-But-2-ene

4. From alcohols. Alkenes are prepared from alcohols by heating with protonic acids such as sulphuric
acid or phosphoric acid at about 443 K. This reaction is called dehydration of alcohols.
CH,CH,OH —24" 45 CH, = CH, + H,O
H,5O0, or HaPO

Ethyl alcohol Ethene

This reaction is also an example of B-elimination reaction because -OH group takes out one hydrogen
atom from the $-carbon atom.
lp | & v4
—f — | sa Conc. Hg504 -
or HgPO4 id C aa + H,O
i |
H OH Alkene
Alkanol

The dehydration of alcohol occurs in the following steps :


(1) Alcohol gets protonated by the acid. The proton from acid attacks the electron rich site of the alcohol and
it gets attached to the O atom forming a protonated alcohol as :

H—
il
(il +Ht =~ H—C—C—H
Lot
@
H»;:O— H H :O—H
H
Protonated alcohol
(it) Protonated alcohol loses a water molecule to give carbocation. This step is slow and hence is the rate
determining step.
H 4H H H

u_b_—¢_u —— u_b_—b_u + H,0


| rt® | =

- 7 HH - Carbocation |
(iii) The carbocation is unstable and it loses a proton from the neighbouring carbon to form alkene (ethene).
H
| | | |
H—C—C—H ——__ H—C =C—H 4 Ht
@ Ethene
_— 13/40 MODERN'S abc + OF CHEMISTRY
-XI

Similarly, propene is obtained by heating n-propyl alcohol with conc. H,SO, at 443 K.
Conc. H,S0,,
CH,CH,CH,OH —
3, CH,CH=CH,
n-Propyl alcohol Propene
It may be noted that secondary alcohols dehydrate under mild conditions while tertiary alcohols are
very easily dehydrated under very mild conditions. Thus, the overall relative ease of dehydration of
alcohols is
3° alcohol > 2° alcohol > 1° alcohol.
ed H»S0,
CH,CHCH, 78K 449K CH,CH == CH,
Propene
OH
Isopropyl alcohol
(2°alcohol)

OH
H.PO,
Cy 373-383 K C)
Cyclohexanol Cyclohexene
(2°alcohol)

CH, CH,
H,SO, | |
CH, —(t —OH 360 K ? CH, —Ce= CH, (easily dehydrated)

CH, 2-Methylpropene
tert-Butyl alcohol
(3° alcohol)

Alkenes can also be prepared by heating alcohols with catalyst such as anhydrous zinc chloride (ZnCl,) or
alumina (Al,O,).

CH,CH,OH —3-
hyd Al,0O.
OH; =CH, + H,O
Ethene
LEARNING PLUS
Rearrangement of Carbocations during dehydration of alcohols
During dehydration of alcohols, sometimes unexpected alkenes are formed. This is because of the
intermediate carbocations which have the property of rearrangement from a less stable to a more stable
carbocation. This occurs either by 1,2-hydride or 1,2-methy] shift. This means that the carbocation initially
formed can rearrange to form a more stable carbocation if the structure of the alkyl group permits. The
carbocation formed, then loses a proton to form unexpected or the rearranged alkene. For example, the
dehydration of butan-1-ol or butan-2-ol with conc. H,SO, gives a mixture of but-1-ene and but-2-ene in which
but-2-ene predominates. This can be explained by the rearrangement of the initially formed less stable 1°
carbocation to the more stable 2° carbocation by 1, 2-hybride shift, which then loses a proton to form but-2-
ene (major) and but-1-ene (minor) products.

CH,CH,CH,CH,OH —— CH,CH=CHCH, + CH,CH,CH =CH,


; x H* : : : : :

Butan-1-ol But-2-ene But-1l-ene


(major) (minor)
The formation of but-2-ene is because of rearrangement of carbocation, initially formed as:
_ + : + 1,2-hydride
Bikers datas 5
ae ll H"*
CH,CH,CH,CH,OH —— CH,CH,CH,CH,OH, —H,0” re, shift

H
1° Carbocation
(less stable)

CH,CH,CH CH, —=#*» CH,CH=CHCH, +CH,CH,CH = CH,


2° Carbocation But-2-ene (80%) But-1-ene (20%)
(more stable)

WWW.JEEBOOKS.IN
HYDROCARBONS 13/41 =>

Similarly,
CH, oH, CH,
|
CH,CH,CH CH,OH —H*» CH,CH=CCH, + CH,CH,C =CH,
2-Methylbutan-1-ol 2-Methylbut-2-ene 2-Methylbut-1l-ene
(major)

This can be explained as:

CH CH, CH;
: | oa | 1. 2-hydride
_y
CH,CH,CHCH,OH ~a7,0" CH3CH,G—CH, “aig » CH3Gi, CRQCHa7 >
—-H,0 shift

H
1° Carbocation 8° Carbocation
(less stable) (more stable)
oH CH,
CH,CH —C — CHé», + §CHgCH, C—CH,
2-Methylbut-2-ene 2-Methylbut-1-ene
(major)

Similarly,
7; ois CH,
CH, 9i ie _ CH, H*
“a0” a) / .
ia ed fie + bat
ihren ners

CH, OH CH, CH,


2, 3-Dimethylbut-2-ene
3, 3-Dimethylbutan-2-ol (major) 2, 3-Dimethylbut-1-ene
This can be explained as:

CH, CH, H CH;


[ps Ht | | 1,2-methyl | _H*
see tas 1 ate Cita, CF — CH, ee eT ie ee ea =e

CH, OH CH, CH,


3° Carbocation
2° Carbocation
(more stable)
(less stable)

CH, oH
CH,—C = C—CH, + CH,= C—CH CH,
CH, CH,
2, 3-Dimethy Ibut-2-ene 2, 3-Dimethylbut-1-ene
(major) (minor)

It may be noted that when more than one alkene can be formed, the preferred product is the more stable
one. It also follows Saytzeff rule.

Laboratory Preparation of Alkenes


In the laboratory, ethene is prepared by the acidic dehydration of ethyl alcohol. In this method, a mixture
of ethyl alcohol and concentrated sulphuric acid in the ratio of 1 : 2 by volume is taken in a dropping funnel
attached to a round bottomed flask. Asmall amount of aluminium sulphate and sand is also added in the round
bottomed flask (Fig. 18).
Aluminium sulphate catalyses the reaction and sand prevents the frothing of the reaction mixture. The
flask is heated on a sand bath when the reaction takes place and ethylene gas is produced. The gas produced is
collected over water by the downward displacement of water.
le! 43/42 MODERN'S abc + OF CHEMISTRY-XI

Purification of ethene. The gas


CoH,OH +
obtained contains some impurities of WY Conc.H,S80,
sulphur dioxide (obtained by the
reduction of sulphuric acid) and .
carbon dioxide (obtained by the
oxidation of ethyl alcohol). Therefore,
it is purified by passing the gas Al,(SO4)3+
through a wash bottle containing |© "2SON__
concentrated solution of KOH which ‘Se
absorbs these gases. ee
SO,+ 2KOH ——> K,SO, + H,O NaOH solution

CO,+2KOH —— K,CO,+H,O
The pure gas is then collected over
water.
Fig. 18. Preparation of ethene in the laboratory.

5. By electrolysis of potassium salts of dicarboxylic acids. The electrolysis of potassium salts of


dicarboxylic acids gives alkenes. This is called Kolbe’s reaction.
CH,COOK C
("2 + 2H,0 —ectroves, |7, 4 2CO, +H, + 2KOH
CH,COOK CH,
Potassium succinate Ethylene
The reaction is believed to occur by the following steps :

CH,COOK ,.4,,,
On1ZaTLlon |
GHaCOO a 9K
.
CH,COOK CH,COO-
2H,O 2OH- + 2H*
CH.COO- CH,COO CH,
At anode | = —2e° |: —» a + 204

CH,COO™ ON Ethene
At cathode 2Ht + 2e*- ——> [2H) > H,
Since the electrode potential of Ht ions is higher than that of Kt ions, therefore, H* ions are preferentially
reduced to H, while K* ions remain in the solution.
Properties of Alkenes
The important physical and chemical characteristics of the alkenes are discussed below :
Physical Properties
1. Physical state and smell. The first three alkenes (ethene, propene and butene) are gases at ordinary
temperature. The alkenes containing 5 to 18 carbon atoms are liquids while the higher members of the
family are solids. Ethene is a colourless gas with a faint sweet smell. All other alkenes are colourless and
odourless.
2. Melting points. In general, alkenes have higher melting points than the corresponding alkanes.
This is due to the reason that z-electrons of a double bond are more polarizable than o-electrons of single
bonds. As a result, the intermolecular forces of attraction are stronger in alkenes than in alkanes. Thus,
the melting points of alkenes are higher than the corresponding alkanes. The melting and boiling points of
alkenes, in general, increase with the increase in the molecular mass. However, these do not show regular
eradation. Amongst isomeric alkenes, trans-alkenes have higher melting points than their corresponding
cls-lsomers.
For example,
CH CH CH H
3. 3 3

\ C=—— “ SS == C 4

a” Na a Neu,
cis-But-2-ene trans-But-2-ene
(m.p. = 134.2 K) (m.p. = 167.4 K)
This is due to the reason that the molecules of trans-alkene are symmetrical and therefore, they can pack more
closely in the crystal lattice than the molecules of cis-alkene. Therefore, larger amount of energy is required to
break the crystal lattice of trans-alkene than the corresponding lattice of cis-alkene. Hence, trans-alkenes have
higher melting points than the corresponding cis-isomers.
3. Boiling points. The boiling points of alkenes show a regular gradation with the increase in number
of carbon atoms like alkanes. In general, for each added —CH, group the boiling point rises by 20-—30°. The
branched chain alkenes, however, have lower boiling points than the corresponding straight chain alkenes.
Amongst the geometrical isomeric alkenes, cis-alkenes have higher boiling points than the corresponding
trans-isomers.
For example,

CH, CH, CH, H


*% C—
P ne
Yo a
Y C — A
ee
H H H” CH,
cis-But-2-ene trans-But-2-ene
(b.p. = 277 K) (b.p. = 274 K)
This is due to the reason that cis-alkene is more polar than trans-alkene. Therefore, trans-alkene has
stronger dipole-dipole interactions and its boiling point is high.
4, Dipole moments. Alkenes are weakly polar. The m-electrons of the double bond can be easily polarized.
Therefore, their dipole moments are higher than those of alkanes.
The dipole moment of alkenes depends upon the position of the groups bonded to the two double bonded
carbon atoms. The symmetrical trans alkenes are non polar and hence have zero dipole moment. In these
alkenes, the dipole moments of individual bonds are equal and in opposite direction. Therefore, these get
cancelled resulting zero dipole moment for the molecule.
However, unsymmetrical trans-alkenes have small dipole moment because the two dipoles oppose each other
but they do not cancel out each other exactly since they are unequal.

CH H CH,CH, H
\ — a No =— Oo

H CH, H CH,
trans-But-2-ene trans-Pent-2-ene
p= 0 p>0
(small dipole moment)
On the other hand, both symmetrical and unsymmetrical cis-alkenes are polar and hence have finite dipole
moments. This is because the two dipoles of individual bonds are on the same side and hence have a resultant
dipole moment.

CH, CH, CH,CH, CH,

ran ON *s
lal H H H
cis- But-2-ene cis-Pent-2-ene
(u= 0,25 D) (u > 0)
Similarly, unsymmetrical terminal alkenes such as propene and but-l-ene have some dipole moments
because the polarity due to the alkyl group is not cancelled by the corresponding polarity in the opposite
direction. For example,

CH, H CH,CH, H

Noa
rsi,
No 7= 0%
Zz ~
H H H H
Propene But-1-ene
(u= 0.35D) (u = 0.87D)
— 13/44 MODERN'S abc + OF CHEMISTRY-XI
5. Solubility. Alkenes are lighter than water. These are insoluble in water because they are non-polar.
However, they readily dissolve in organic solvents like alcohol, benzene, ether, carbon tetrachloride, etc.
Chemical Properties
Alkenes are characterized by the presence of double bond in their molecules. They are quite reactive
chemically. This is due to the presence of m-bond. The n-electrons form an electron cloud which lies above
and below the plane of o-bonded carbon atoms. These 1-electrons are therefore, more exposed and less tightly
held between the two carbon atoms. Therefore, alkenes are attacked by reagents or compounds which are
in search of electrons. These reagents are called electrophilic reagents. Because of rich source of weakly
held n-electrons, they show addition reactions in which the electrophiles add on to the C = C double bond to
form addition products. The a-bond easily breaks during the chemical reactions of alkenes and results in the
formation of two strong o bonds. Hence alkenes have a tendency to undergo addition reactions which are of
electrophilic nature.

v = ia

—- A B
Alkene Attacking
molecule Addition product

Apart from addition reactions alkenes also participate in the oxidation, polymerization and some replacement
reactions of the alkyl group.
1. Addition Reactions
In these reactions, the attacking reagent is an electrophile i.e. either positively charged or electron deficient
species. The negative centre for the electrophilic attack is made available by the high electron density of the
double bond of alkene.

Ure NdRNO :
Electrophilic addition to alkenes
The electrophilic addition to alkenes takes place in the following two steps :
Step 1. In the first step, the attacking molecule (A — B) gets partially polarized. As this molecule
approaches the t bond of the double bond, it begins to repel electron cloud of the attacking molecule away from
it. As a result, the nearer end of the molecule acquires partial positive charge and farther end acquires partial
rial
negative charge (A — B).
At the same time, the double bond also undergoes electromeric effect resulting in the shift of the n electron
pair to one of the carbon atoms making it negatively charged while the other carbon atom becomes positively
charged. The positively charged atom (A*) of the attacking molecule attacks the negative carbon of alkenes and
forms a new C —Abond.
This leaves the other carbon atom with a +ve charge. The intermediate formed is called carbocation.

Sf eS»
cs a
F
me £—c— —> —C—6— +B
7 || | |
Alkene Carbocation
Step 2. In the second step, the anion B~ attacks the +ve carbon atom of the carbocation to form the
addition product.
A —~ A B
| | |
—C—C@— +k —-> —(—6—
Carbocation Addition product

WWW.JEEBOOKS.IN
HYDROCARBONS 13/45 =>

The first step may be shown as :


Ee

For example, a specific example of addition of HBr to ethene may be shown below :
Step 1

Ethyl carbocation

i 1

+ r Hig —i
H fH
The important addition reactions of alkenes are :
(a) Addition of halogens. Halogens (particularly chlorine and bromine) react with alkenes in the presence
of an inert solvent (e.g. CCl,) to form dihalogen derivatives :

CH, =CH, + Br,—“4 / CH, —CH,


Ethene 4 ‘Ss

1, 2-Dibromoethane
CH,-CH=CH,+ Br, ——45 CCl
CH,— CH — CH,
Propene i” Br
1, 2-Dibromopropane
CH, -CH=CH,+ Cl, “t+ CH,— CH—CH,
Propene by dy
1, 2-Dichloropropane
(Propylene chloride)
The reaction with fluorine is explosive whereas iodine does not react under normal conditions.
This reaction can be used as a test for alkenes because the brown colour of bromine will get decolourised
by alkenes.
Pil
Seep /+ Br, ———> —C—C—
Alkene Brown ee |
Si

colour Colourless
Note. It may be noted that the above test is also given by alkynes (triple bonded compounds). In fact, this test is used
to detect the presence of double bond or triple bond in a compound (i.e., unsaturation).

l Wicchctsii - Addition of Br, to alkenes ll


The addition of Br, to an alkene (e.g., ethene) occurs in the following steps :
(i) Polarisation of bromine molecule. Bromine molecule is a non-polar molecule because the electron
pair between two bromine atoms is equally shared between two atoms. As the bromine molecule approaches the
ethene molecule, the m-electron cloud of the carbon-carbon double bond repels the electron cloud of the bromine
molecule. Consequently, the electrons of the bromine molecule get displaced away from the bromine atom
_— 413/46 MODERN'S abc + OF CHEMISTRY-XI

nearer to the ethene molecule. Therefore, the bromine atom nearer to double bond becomes positively polarised
while the other atom gets negatively polarised. The positively polarised bromine atom attacks the z-electron
cloud of the double bond giving rise to a carbocation and a negative bromide ion.
5+ 5 Nucleophilic
(\ Re
attack H,C—CH,
+ Br— Br )
iy = CH,
Polarised ia
| ene
Eth
Bromoethyl! carbocation
(ii) Attack of bromide ion. The Br ion then attacks the positively charged carbon atom to form the
addition product.
2
Br- + a aaa —— tags — ear:

Br Br Br
1, 2-Dibromoethane (Ethyl bromide)
Evidence in favour of Ionic Mechanism
The mechanism of addition reaction is ionic in nature. This can be supported by adding some CI ion to the
reaction mixture. The product obtained is a mixture of 1, 2-dibromoethane and 1—bromo—2-chloroethane. The
formation of 1—-bromo—2—chloroethane occurs due to attack of Cl-ion on the carbocation and also establishes the
ionic nature of the mechanism. The formation of chloro derivative along with 1, 2-dibromoethane from the
intermediate carbocation is shown below :
_ D
cH_cH, <CXo. ueocwH, — 5 4c-0cn;
— | | | |
Br Cl Br Br Br
1-Bromo -2-chloroethane Carbocation 1, 2-Dibromoethane
Trans bromination
It is interesting to note that addition of bromine leads to trans addition product. The frans addition
product can be explained with the help of a cyclic intermediate called bromonium ion in the initial step. In
the first step polarised bromine atom attacks the ethene molecule forming cyclic bromonium ion as :

ee Bei 28 “ee /
| Br—Br —“@ | ee
gate af
Cyclic bromonium ion
The attack of the Brion on this cyclic ion can take place from the opposite side only resulting in the
formation of trans addition product.
ne

No" a eee
Br” NN Zea. f/— BrC_H
C2, ]
HY HA H
Trans product

(6) Addition of halogen acids (hydrohalogenation). Alkenes react with halogen acids (HCl, HBr or
HI) to form alkyl halides.
SS stg, 6.6) 1 |
—C=C— + HX —> —C—C—
Halogen acid
(X = Cl, Br, D H XxX
Alkyl halide
The order of reactivity of hydrogen halides is :
HI > HBr > HCl
The actual product formed depends upon whether the alkene is symmetrical or unsymmetrical as discussed
below.
(i) Addition reactions of HX to symmetrical alkenes.
In these reactions, only one product is theoretically possible. For example,
H,C =CH, + HCl ——~> H,C—CH,Cl
Ethene Chloroethane
HYDROCARBONS 13/47 —_—

H,CHC = CHCH, + HBr ——> en ani


But-2-ene
Br
2-Bromobutane

(zt) Addition reactions of HX to unsymmetrical alkenes


When the alkene is unsymmetrical, then two products are possible. For example, when propene is treated
with HBr, both 1-bromopropane and 2-bromopropane may be formed as the products.
CH,—CH,—CH,Br 1-Bromopropane
CH,— CH = CH, + or]
Propene ie ee a 2-Bromopropane

Br
In these cases, the addition is governed by Markovnikov rule proposed by a Russian chemist Markovnikov.
It states that
reel. the addition across unsymmetrical multiple bond, the negative part of the addendum
(attacking molecule) joins with the carbon atom which carries lesser number of hydrogen atoms
while the positive part goes to the carbon atom with larger number of hydrogen atoms.
For example, in case of propene, 2-bromopropane is major product.
CH,CH=CH, + HBr —> CH,—CH—CH, + CH,—CH— CH,
Propene | | |
Br H Br
2-Bromopropane (90%) 1-Bromopropane (10%)
However, a small percentage of the isomer in which halogen atom (negative part) is attached to carbon
with more number of hydrogen atoms is also formed.
Exception to Markovnikovy rule
Kharasch and Mayo (1933) observed that when HBr is added to an unsymmetrical double bond in the
presence of organic peroxides (R — O — O — Re.g. benzoyl peroxide, C,H,CO OO CO C,H,), the reaction takes
place opposite to the Markovnikov rule. This is known as anti-Markovnikov rule or peroxide effect. It is
also called Kharasch effect. According to anti Markoynikov rule, during the addition of HBr to an unsymmetrical
alkene the Br atom will join to the carbon carrying more hydrogen atoms while H atom will go to the other
carbon atom. For example,
(R-—O-O-R)
CH,— CH=CH, + HBr —ji > GF
P ropene Ho OBr
1-Bromopropane
It may be noted that only HBr shows peroxide effect. HF, HCl and HI do not exhibit peroxide effect
and therefore, addition of HF, HCl and HI takes place according to Markovnikov’s rule even in the
presence of peroxides.
Thus, to sum up, the addition of HBr to propene gives 2-bromopropane in the absence of peroxides
and 1-bromopropane in the presence of peroxides as :
No
CH, CHCH, Markovnikov rule
Peroxides .
Br
CH, — CH = CH, + HBr 2-Bromopropane
Propene Peroxides
CH,CH,CH, Br Anti-Markovnikov rule
1-Bromopropane
Thus, according to Markovnikov rule, the main products in the reactions can easily be predicted. For
example, addition of hydrogen halides to higher alkenes give the main products as:
CH, v3
CH,—C=CH, + HCl ——> an

2-Methylpropene Cl
2-Chloro-2-methylpropane
‘a CH,
CH,—CH=C—CH, + HBr ——> a aes ea
Br
2-Methylbut-2-ene 2-Bromo-2-methylbutane
a) 43/48 MODERN'S abc + OF CHEMISTRY-XI

Hiceh aise . Explanation of Markovnikov rule


In order to understand Markovnikov rule, let us consider the attack of electrophile (H*) on the propene
molecule. The two intermediate carbocations which are possible are :
CH, — CH - CH, Joins with terminal C atom
2° Carbocation (I)
CH,—-CH
3 = CH, + H"
es ? |
(more stable)
Propene ‘6 CH, — CH,,— CH? Joins with central C atom
electrophile 3 : -
1° Carbocation (IT)
(less stable)
The carbocation that is formed with a large proportion is decided on the basis of its relative stability.
Now, we know that 2° carbocation is more stable than 1° carbocation, therefore, the attack of (H*) ion on
propene results in the formation of more stable carbocation (I). This ion further reacts with Br- to form the
addition product : ,
CH, CH” CH, + Bro ——_: CH Ub
Carbocation (1) 4

This product is in accordance with Markovnikov rule. Thus, Markovnikoy rule may also be stated in the
alternative form as :
The electrophilic addition to an unsymmetrical alkene always occurs through the formation
of a more stable carbocation intermediate.
Thus, the complete mechanism for the addition of HBr to propene may be explained as :
Br Ionization iH . Br

CH,—CH =CH,+H —-> CH,—CH’—CH,


Propene 2° Carbocation
CH,CHCH,+Br ——= CH,—CH—CH,
Br
2-Bromopropane
Note : 2° carbocation will be formed instead of 1° carbocation because it is more stable.
Peroxide effect
Kharasch and Mayo discovered that the addition of HBr to unsymmetrical alkene in the presence of organic
peroxides (such as benzoyl peroxide, C,H, CO-O—O-COC,H,) takes place opposite to the Markovnikov rule.
This is known as anti-Markovnikov e or peroxide effect. According to this rule, the Br atom will join
to that carbon atom which carries more hydrogen atoms, while the H atom will go to other carbon atom.
Thus,
CH,—CH = CH, + HBr) Sexes, CH,—CH,—CH,Br + CH,—CH—CH,
Br
1-Bromopropane (90%) 2-Bromopropane (10%)
Explanation for anti-Markovnikov rule.
The addition of HBr to alkenes in the presence of peroxides occurs by free radical mechanism. In
this mechanism, the first two steps result into generation of free radical Br*. Then, it adds to double bond
in such a way to give more stable free radical. Then this free radical abstracts a H* free radical from HBr
to complete the addition. The various steps of the mechanism are :
O O
(1) | : Homolysis | z ee |
(1) C,H; -C—O—O—C—C,H, —“"*> 2c,H; C—O — > 2C,H, + 2CO,
Benzoyl peroxide ae Free radical
. lomolysis :
(2) C,H, + H+-Br —————> C,H, + Br
‘ Free radical
Oh a) a CH, Br
(3) Br Ppt els (eres CH 2 Free radical (I) (more stable)
3 r + CH,—CH-———CH,
se ARE
a OR.
Br
1° Free radical (II) (less stable) _
HYDROCARBONS 13/49 >=

Because the 2° free radical is more stable than 1° free radical, it will be preferably formed.

(4) CH,—CH —CH,Br + H+-Br ——» CH,—CH,—CH,Br + Br


J |

Preferred free radical 1-Bromopropane


(Major product)
Then steps (3) and (4) are repeated again and again to form the desired product.
Exceptional behaviour of HBr. It is interesting to note that peroxide effect is applicable only to
HBr and not to HF, HCI or HI. This can be understood on the basis of energy changes in the propagation
step using different halogen atoms :
Step 3: X+CH,- CH=CH, —~> CH,-
CH — CH, X x AH = — 209 kJ/mol
| | x I Qh AH = — 101 kJ/mol
X= Le AH = — 42 kJ/mol
A= AH = +12 kJ/mol
Step 4: H - X + CH, - CH - CH,X —> CH, - CH, — CH,X + X xX= AH = + 159 kJ/mol
ASS AH = + 27 kJ/mol
xX= = AH = — 87 kJ/mol
x SQ AH = — 106 kJ/mol
From the above data, it is clear that only with HBr both the steps are exothermic and hence, free
radical mechanism is possible. With HF or HCl, the attack of these do not follow free radical mechanism
because the second step involving the attack of carbon radical with HF or HCI is endothermic and not
favourable. In other words, homolysis of HF or HCI does not take place. In case of HI, the homolysis of HI is
possible as evident from step 4 but the attack of I to propene does not take place (Step 3 is endothermic).
So, the free radical I will combine with similar free radical to form I, molecule. Thus, peroxide effect is
observed only in case of HBr.
LEARNING PLUS
Rearrangement of Carbocations
We have learnt that addition of HBr to alkenes occurs through the formation of carbocation. Sometimes
itis observed that the product formed is different from the one expected. This is because of the rearrangement
of carbocation. The carbocation initially formed can rearrange to form a more stable carbocation if the
structure of the alkyl group permits. For example,
CH CH, CH,
3 : |
CH eS
3 2 ae a CH a
3 ee el
3 Rearra gement_.
>H Shit CH —C—CH.—CH.
ry 2 3

3-Methylbut-1-ene H 3°carbocation
2°carbocation (more stable)
[Br fast

CH
CH,—(—CH,—CH,
Br
2-Bromo-2-methylbutane (major product)
We observe that 2° carbocation rearranges to 3° carbocation because 3° carbocation is more stable than
2° carbocation. This is called 1, 2-hydride shift. Similarly,
CH, CH,
I re Rearrangement | ae
CH,—C—CH —CH, aaCH Shift
hte hr aN mS

CH CH,
: 3°carbocation
3, 3-Dimethylbut-1-ene 2° Carbocation (more stable)
(less stable) |Br-, fast

CH,
ST Aloe 3
|

2-Bromo-2, 3-dimethylbutane
WWW. JEEBOOKS.IN
_— 13/50 MODERN'S abc + OF CHEMISTRY
-XI

This is called 1, 2-methyl shift. So, if the structure permits, the rearrangement occurs in such a way that
a less stable carbocation is converted into a more stable carbocation. This rearrangement can occur only if
a 1, 2-shift of hydrogen or alkyl can form a stable carbocation.The products obtained are those formed from
the rearranged carbocations (major product) and not from the original ones. Thus,
CH; H. CHs
CH,—CH—CH =CH, —#8, CH3;—C—CH2— CH, (Rather than a
|
Br Br
(Major)

CH, | CH,

CH,—C—CH=CH, —#, CH;—C—CH— CH, (Rather than CH,—CH—CH—CH,)

CH, . CH; Br
(c) Addition of water (hydration of alkenes). Water adds to alkenes in the presence of mineral acids.
In this case also, the addition occurs in accordance with Markovnikov’s rule. For example,
CH,=CH, + H,O —#., CH,CH,OH
Ethene Ethanol

CH,CH 7 CH, 7 H,O ts CH,CHCH,


Propene |
OH
Propan-2-ol
(major product)
OH
CH,—C=CH, + H,O —" > cH, O64,
|
H, CH,
2-Methylpropene 2-Methylpropan-2-ol
(Major product)
This reaction is also known as hydration reaction.

Wicehcisii ; Addition of water to alkenes


H,O + H* ——— H,0*

cH,cHCH, +Fi_0'—H —— CH,—CH'—CH,+H,0


H Isopropyl carbocation
(Stable carbocation)
CH,—CHtCH,+H—O—H = —— CH,—CH—CH, _-H"_, CH,—CH—CH,
ae
O OH¢
A (iy Propan-2-ol

(d) Addition of hypohalous acid. Alkenes react with hypohalous acids (HOCI or HOBr) or halogen Cl, or
Br, in the presence of H,O to give halohydrins. In this reaction, the addition occurs according to Markovnikov rule
(halogen is positive part and —OH is negative part).
H,C=CH, + Hocl —“22*2 , CH—CH,
Ethene
H
Ethylene chlorohydrin
(2-Chloroethanol)
: Brg, H,0
CH, = CH, + HOBr —*—=> yo y

Ethene OH Br

Ethylene bromohydrin
(2-Bromoethanol)
Clo, H yO
CH,— CH =CH,+HOC] —*—*— CH, —CH — CH,
Propene
OH Cl
Propylene chlorohydrin
(1-Chloropropan-2-ol)
In the case of propene, the reaction occurs according to Markovnikov rule.
The order of reactivity of different hypohalous acids (HOX) or X, | H,O is
Cl, | H,O (HOCI) > Br, | H,O (HOBr) > I, | H,O (HOD)

Wiccnatsii * Addition of Cl, | H,O to alkenes


CH,—CH=CH,+ (j—@] —“> CH,—CH*—CH,Cl
CH,—CH'—CH,Cl+H,6: —™4 CH,——CH—-CH,Cl fie, cre/CH—CH,Cl

iT & HW hin
a an.-9.
1-Chloropropan-2-ol

(e) Addition of sulphuric acid. Cold and concentrated sulphuric acid adds to alkenes forming alkyl
hydrogen sulphate.
H,C = CH,+HOSO,OH — > H,C—CH,OSO,OH or CH,CH,. HSO,
Ethene Ethyl hydrogen sulphate
CH,—CH =CH, + HOSO,OH ———> CH,—CH—CH,
Propene |
OSO,0H
Isopropyl hydrogen sulphate

—— Wicehcsiia * Addition of H,SO, to alkenes —


The mechanism of the reaction may be written as :
H».Og—_—= H’ + Oso ,OH
CH,CH=CH, + Ht —> CH,—CH _CH,
Isopropylcarbocation
(Stable carbocation)
CH,—CH —CH, + 0 SOjOH ——— CH,—CH—CH,
OSO,0OH
In case of unsymmetrical alkenes, the addition also occurs according to Markovnikov rule and -SO,OH
group adds to the carbon linked to lesser number of hydrogen atoms.
Alkyl hydrogen sulphates on boiling with water undergo hydrolysis to form alcohols.
CH,CH,OSO,0OH+H,0 _*i_, CH,—CH,OH+H,SO,
Ethanol
CH,CH—CH, + H,O __Boil_, CH,—CHCH, + H,s0O,

OSO,0OH OH
Isopropyl! alcohol
Thus, alkenes can be converted to corresponding alcohols on treatment with sulphuric acid and subsequent
hydrolysis.
CH, = CH, —H2804 (col) , CH,CH,OSO,0OH —,2-—>
H95O0
CH,CH,OH
Ethanol
2. Oxidation Reactions
Alkenesco tb different products on oxidation under different conditions.
; ) — ustion. Alkenes burn in oxygen or air to give carbon dioxide, water and a large amount of heat
is evolve
C,H, +. 30, —> 2CO, + 2H,0O; AH=- 1411.0 kJ mol™
Ethene
nCO, + nH,O : AH =—ve
C,H, + a O, —?
Alkene
They form explosive mixture with air or oxygen and burn with luminous
— 13/52 MODERN'S abc + OF CHEMISTRY-XI
(6) Oxidation with potassium permanganate. Alkenes react with cold, dilute aqueous, neutral or
alkaline potassium permanganate solution to form 1, 2-diols called glycols. The glycols contain two —OH groups
on adjacent carbon atoms. This reaction of addition of two hydroxyl groups to each end of double bond is called
hydroxylation of the double bond.
2KMnO, + H,O ——> 2KOH + 2Mn0, + 3[0]
Alk. KMnO,
CH, = CH, + H,O +0 a = ms
Ethene from KMnO, OH OH
Ethane-1, 2-diol (Ethylene glycol)
Alk. KMnO,
CH, —— CH —= CH,+ H,O0 + O ee (cold) CH, — a— 2
Propene OH OH

Propane-1, 2-diol
(Propylene glycol)
Alk. KMnO 4
CH,CH=CHCH, - H,O + O aod CH, — CH —CH —CH,
But-2-ene
La
OH H
Butane-2, 3-diol
The alkaline potassium permanganate solution is known as Baeyer’s reagent. It has bright pink colour.
It oxidises alkenes to glycols which is colourless. Therefore, the pink colour of Baeyer’ s reagent is discharged
and brown precipitate of manganese dioxide is formed. This reaction is used as a test for the presence of double
bond in a molecule. This is also known as Baeyer’s test.

Sy sues + KMn0, H,0_, - /_ i + MnO,


Alkene Pink colour | Brown ppt
OH OH
Colourless
(c) Oxidation with hot potassium permanganate solution. When alkene is treated with hot acidic
potassium permanganate or potassium dichromate solution the alkene gets split up at the double bond forming
carboxylic acids or ketones. This is also called oxidative cleavage of alkenes. For example,
O
) I Hot KM
CH, + CH, —~~""45 H-C+oH >O 200, + 2H,0
Ethene Methanoic acid
(Formic acid)
With terminal alkenes (double bond present at the end of carbon chains), one of the products is always
methanoic acid (formic acid) which on further oxidation gives carbon dioxide and water.

| |
CH,—CH+CH, =" + CH,—C—OH + CO, +H,0
Bot hind, Ethanoic acid
Propene (Acetic acid)
CH, CH,
CH,— CECH / O
cH,—¢=0
|
+ CO,
2-Methylpropene . ~ 4 (Acetone)
Propanone
With non terminal alkenes, carboxylic acids or ketones or both of these are obtained depending upon
the nature of alkene. For example.

CH,CH4CHCH, —!_ , CH, COOH + CH, COOH


Hot KMnO,
But-2-ene 2 molecules of ethanoic acid

CH,CH,CH+CHCH, —l©! , CH,CH,COOH + CH,COOH


, Hot KMnO 4
Pent-2-ene Propanoic acid Ethanoic acid
HYDROCARBONS 13/53 a

CH, H CH,
| | | |
CH,—c-+C—cH, -—“> cH,—C=0 + CH,—C—OH
" -Hot KMnO, Propanone Eihanoig acid
—— (Acetone) (Acetic acid)
A: [0] CH we ee
_-CH. 3
ete Hot KMnO, =O+0=C
H, CH; Propanone (2 molecules)’!4s
2,3-Dimethylbut-2-ene (Acetone)
The reaction also occurs in the presence of sodium periodate (NalO,)
As a memory aid :
During treatment of alkenes with hot conc. KMnO,, the products formed are :
YR R
= C\. gets oxidised to ee: =i) (Ketones)
NR R’
A
= << gets oxidised to RCOOH (Carboxylic acids)

= CH, gets oxidised to CO, (Carbon dioxide)


(d) Catalytic oxidation. Alkenes react with oxygen in the presence of silver catalyst at 525 K to 675 K to
form epoxyalkanes or epoxides. (alkene oxides)
RCH= CH, += » ~~“
525 —675 K
RCH—CH, 2
O
| Epoxide
1 :
CH, = CH, + 5 O, SOR_eTBEK “He fH GH,

Ethene
| Epoxyethane

CH,—CH = CH, + =0; = CH,~CH—CH,


p
ropene O
1,2-Epoxypropane

(e) Oxidation with ozone. Alkenes are oxidised with ozone to form ozonides which are unstable
compounds. These are reduced with zinc and water forming aldehydes and ketones. The reaction is called
ozonolysis. For example, when ozone is passed through a solution of an alkene in some inert solvent such as
dichloromethane (CH,Cl,), chloroform (CHCl,) or carbon tetrachloride (CCl,) at a low temperature (195
— 200 K) it oxidises alkene to ozonide. Qzonides :are unstable and explosive compounds. Therefore, they are not
generally isolated but are reduced in situ with zinc dust and water to give aldehydes and ketones depending
upon the structure of the alkene.

7 A CHC, XK XM7~ mao N_C=0 A=


Ze = ( ta Y O, "T95-200k” mal —elaaeer + O=C + H,O,

Alkene QO: — —0 Aldehydes or ketones


Ozonide

It is clear that if the carbon forming the double bond carries a hydrogen atom attached to it, it gives rise
to aldehydes otherwise ketones are formed.
For example,

CH, = CH, +0, CB, CH’, (CH, 120 , 2HCHO + H,0,


Ethene | Fae Methanal
= ad a (Formaldehyde)
Rilede ees
_— 43/54 MODERN'S abc + OF CHEMISTRY-XI

CH,-CH =CH, +0, —S#2_» CH,CH’, “ CH, —22420 , CH,CHO + HCHO


Propene }‘ Ethanal Methanal
O.-—0 (Acetaldehyde) (Formaldehyde)
Propene ozonide

CH, CHS
3 Cc. / CH, CH,
weC=CH,+0, He
195-200K _, cHy | |.
070 | EE i C=O + HCHO
H.C C 3 Methanal
2-Methylpropene Propanone (Formaldehyde)

O
_ CH,Cl.
“se oe CH,
ac ee a + O, —suanK CH, —C y of 1
y Zn,
mS H,O
CH, | Ph I CH;
2-Methylbut—2—ene ~~ ~ CH,
|
CH.CHO + CH, —C=0+H,0,
Ethanal Propanone
(Acetaldehyde) (Acetone)

Ree ee . R~J R’
> iC
CR. Ses “XR”
ah
(ii) Zn, H,O R’ il
/—0+0=Cco
3 R”
(R may be H) Aldehydes or ketones
ie RN
eae = part gives Ze =O Ketones
R’ k’

Pm part gives RCHO Aldehydes


H
H,C = part gives HCHO Formaldehyde
€.g., CH, on - as (i) O CH, =a A ‘ eee

= + =f
ae
CH. J \ “CH, Ct) Zagyg CH,~ cH,
=S

2-Methylbut-2-ene i Acetone Acetaldehyde

Applications of ozonolysis.
The process of ozonolysis is quite useful to locate the position of double bond in an alkene molecule.
The double bond is obtained by joining the carbon atoms of the two carbonyl compounds. For example,
(1) Let the product of ozonolysis be two molecules of efhanal (acetaldehyde).
H H
A 1 2 3 4
CH,C = 0+ O=C— CH, — CH,CH =CH CH,
T Cas T But-2-ene
Ethanal
(it) Let the product of ozonolysis be a molecule of ethanal (acetaldehyde) and propanone (acetone).
CH, H
| | 4 Is 21
CH, —-C=0+0=C—CH, —> CH,—C=C—CH,
~ fe
____COLD® | i
Ethanal Propanone CH,
2-Methyl—but—2—ene
Thus, it may be noted
Bromine water test and Baeyer’s test are used for detecting the presence of double bond while ozonolysis
helps in locating the position of double bond.
HYDROCARBONS 13/55 =>

LEARNING PLUS
Hydroboration oxidation. This is another method of hydration of alkenes where hydration takes place
anti to Markovnikov rule. Alkenes react with diborane (B,H,) to form trialkyl boranes which on oxidation with
alkaline H,O, give alcohols. This reaction involves two reactions; addition of borane (BH,) to double bond
called hydroboration followed by oxidation and hydrolysis to give alcohol.
For example, |
H,C—CH = CH, + (BH,), —> H,C—CH,CH,BH, —S&:!*#:
_, (CH,—CH,CH,),BH
Propene Diborane n-Propyl borane Di-n-propylborane

CH,CH = CH,
0, OH7
B(OH), + 3CH,—CH,CH,OH <2" — (CH,—CH,CH,),B
Boric acid Propan-1-ol Tri-n-propylborane
The net addition of the reaction is the addition of a water molecule to a double bond opposite to that of
Markovnikov rule. This may be represented as :
H,C—CH=CH, —?=*s, H,C—CH,—CH,OH
Propene (i) H,05, OH” Propan-1-ol
, , , (t) BoH
Pent-1l-one pega tae Pentan-1-ol
The hydroboration reaction was discovered by Herbert C. Brown, who was awarded Noble Prize in 1980
for the synthetic uses of the reaction.
3. Reduction. Alkenes react readily with hydrogen in the presence of a catalyst to give alkanes.
This reaction is called catalytic hydrogenation. Alkenes add hydrogen in presence of Raney nickel (activated
form of nickel), palladium or platinum catalyst at room temperature or ordinary nickel at 523-573 K (Sabatier
Sendern’s reduction). —_ -
H,C =CH, + H, —Feu .SBCACH,
Ethene Ethane
CH,—CH = CH, + H, —=7 ailé, @/—CH.—CH,
Propene Propane
This reaction is used in the manufacture of vanaspati ghee from vegetable oils.
4, Polymerisation Reactions
Polymerisation is a process in which a large number of simple (same or different) molecules combine
to form a bigger molecule of higher molecular mass. The small molecules are called monomers
while the bigger molecules are called macromolecules or polymers.
Alkenes undergo polymerisation to give very important compounds. For example,
(¢) Polymerisation of ethene, When ethene is heated in the presence of traces of oxygen at 473 - 673 K
under high pressure (1500 atm) a large number of molecules combine to form polyethylene or polyethene.
rn CH, 7 CH, 473 — 673 K/Pressure o_ CH, — CH, —-.
traces of oxygen
Ethene Polyethene
Here n may have very large value varying from 600 — 1000 K. Polyethene is extremely useful in industry.
It is mainly employed to make toys, bottles, electric goods, plastic bags, refrigerator dishes, T.V. cabnets, etc.
(ti) Polymerisation of propene. Propene on heating at high temperature and high pressure in the
presence of benzoy! peroxide as initiator or by Ziegler-Natta catalyst, polymerises to polypropene.

n CH,—CH=CH, —High temp/pressure _, |


Catalyst CH,
i
CH—+ pn
Propene
Polypropene
Polyprene is harder and stronger polymer than polythene. Itis used for making milk crates, plastic buckets,
automobile mouldings, seat covers, carpet fibres, ropes, heat shrinkable wraps for records and other articles.
(zit) Polymerisation of vinyl chloride. Vinyl! chloride polymerises to give polyvinyl chloride commonly
known as P.V.C.
n CH, = a Polymerisation H,— ‘ia

Cl Cl
Hl

Vinyl chloride Polyvinyl chloride (PVC)


_— 13/56 MODERN'S abc + OF CHEMISTRY
-XI

It is commonly used for making P.V.C. pipes which are used in the underground electric fittings, in making
raincoats and also in special type of curtains, etc.
(tv) Polymerisation of styrene. Styrene (C,H, — CH = CH,) polymerises to give polystyrene.

H = CH, CH—CH,
nh Polymerisation
C,H; —- 7 oH;
i
Styrene Polystyrene
Polystyrene is used for making packing materials and also for the toys.
(v) Polymerisation of tetrafluoroethylene. Tetrafluoroethylene polymerises to teflon.
nF,C = CF, Polymerisation . [—F,C—CF,—],

Tetrafluoroethylene Polytetrafluoroethylene (Teflon)


Teflon is thermally stable and is very little reactive chemically. It is, therefore, used for coating the inner
surfaces of the non-sticking cooking pans.
The important reactions of ethene are summarized below :
H,, Ni
ae CH, CH, Ethane

Br,,
CCl ‘ii — vie 1,2-Dibromoethane
4
Br Br
HBr 1-Bromoethane
CH,CH,Br (Ethyl bromide)
CH, == CH, _| SO
50s CH,CH,HSO, —~>
H,O
CH,CH,OH Ethanol
Ethene warm
KMn0O,
= Ethane-1, 2-diol
H,0 He OH) (Ethylene glycol)
H OH
O
O, vA BS Zn, H,O Methanal
iC ,*———~+ 2HCHO (Formaldehyde)

0,
——___- Jie —)_ Polyethene
High pressure
600 K

SOLVED EXAMPLES
lL} Example 17.
The reductive ozonolysis of an alkene gave butanone and propanal. Write the structure of alkene and its
IUPAC name.
Solution: The products of ozonolysis are : N.C.E.RT.
H CH,
| |
CH,CH,C =O O = CCH,CH,
Propanal Butanone
The alkene will be (remove oxygen atoms and make double bond between the carbonyl carbon atoms).
CH,
6 5 4 Ig 2 1
CH, CH, CH = C CH, CH,
IUPAC name : 3-Methylhex-3-ene
lL} Example 18.
Write the products and their IUPAC names obtained when hex-1l-ene reacts with HBr
(t) in the absence of peroxide (ti) in the presence of peroxide
HYDROCARBONS 13/57 —_—

Solution:
CH,=CHCH,CH,CH,CH,+ HBr CH,—CHCH,CH,CH,CH,
Hex-1-ene :
r

2-Bromohexane
| STE CE aT : ___ Peroxide | ,
Hex-l-ene
Br
1-Bromohexane
lL} Example 19.
Ozonolysis of an alkene X’ followed by decomposition with water and a reducing agent gave a mixture of two
isomers of the formula C,H,O. Give the structure of the alkene and its IUPAC name.
Solution: The two isomers of the molecular formula C,H,O ;

Hy
CH,CH,C = O O=C— CH,
Propanal Propanone
The alkene is
CH,

IUPAC name: 2-Methylpent-2-ene


L} Example 20.
Propanal and pentan-3-one are the ozonolysis products of an alkene. What is the structural formula and
IUPAC name of alkene. N
Solution: The products are :
CH,CH,CH =O O =CCH,CH,
Propanal
CH,CH,
Pentan-3-one

Alkene is CH,CH, re «

C,H,
3- Ethylhex-3-ene

Q16. What is the formula of teflon ?


Q@17. A compound (C,H, ,) on ozonolysis gives ethanal and pentan-3-one. What is the structure of alkene?
Q18. How will propene react with HBr (7) in the presence of peroxide (z1) in the absence of any peroxide ?
Q19. Name two tests to test the presence of double bond in a compound.
@20. Name an alkene which on reductive ozonolysis produces only acetone.
@ 21. Complete the reaction :
CH, = CH, =alk.KMn04
Q 22. Complete the reaction:
H

HgP04
Heat

Answers to Practice Problems :


©16. Polytetrafluoroethene

WWW.JEEBOOKS.IN
_— 413/58 MODERN'S abc + OF CHEMISTRY-XI

©19. (7) decolourises brown colour of bromine.


(iz) decolourises pink colour of Baeyer’s reagent.
© 20. 2, 3-Dimethyl-but-2-ene
© 21. CH,(OH) CH,(OH)
©22. -~
+ H,O

~. CHEMISTRY OF ALKYNES _,

Alkynes are unsaturated hydrocarbons having carbon-carbon triple bonds in their molecules. Their general
formula is C_H,,_..The simplest member of this class is ethyne (C,H,) which is popularly known on acetylene.
STRUCTURE OF TRIPLE BOND
Ethyne is the simplest molecule of alkyne series. In this case, each carbon atom is sp hybridised forming
two sp hybrid orbitals. These two sp hybrid orbitals lie along straight line and the bond angle between them is
180°. The two unhybridised p-orbitals are perpendicular to each other and also to the axis of the hybridised
orbitals.
In the triple bond formation, one sp hybridised orbital
of one carbon atom overlaps axially (head on) with the similar
sp hybrid orbital of the other carbon atom to form o bond.
Each of the two unhybridised orbitals of one carbon overlaps
sidewise with the similar orbital of the other carbon atom to
form two weak t bonds. The remaining sp hybrid orbital of
each carbon atom overlaps with 1s orbital of hydrogen to
form C —H bond. The structure of the triple bond is being
illustrated by the orbital structure of ethyne shown in Fig. 19.
Thus, carbon to carbon triple bond is made up of
one 6 bond and two 7m bonds. If one m electron cloud is
visualised as lying above and below the internuclear
axis representing o bond, the other x electron cloud
lies in front and in back of the line [Fig. 20 (a@)|. The
overall orbital picture of the molecule is, that of a | ’ | : ’
cylindrical electron cloud symmetrically distributed S ae a.
around the internuclear axis [Fig 20 (6). | (b)
The electron diffraction studies have shown that pel eo eee Ce
the —C=C— bond length is 120 pm and C—H bond
length is 106 pm. While the H—C=C— bond angle is 180°, the strength of the
—C=C— bond 823 kJ mol.
Thus, the strength of the triple bond in alkyne is more than that of the
double bond (681kJ mol!) in alkene and C—C single bond (348 kJ mol-*) in
alkanes. In fact , greater energy is released in the orbital overlap when triple
bond is formed than in case of double bond.
It may be pointed out that carbon—carbon bond length in acetylene is 120pm in comparison to 134 pm in
ethene and 154 pm in ethane molecule. This means that C=C bond is shorter than that of C=C bond and
C—C bond. It has been explained on the basis of extent of overlapping of orbitals. The C=C bond involves the
formation of one 6 bond and two mz bonds. For the formation of additional m bond in acetylene as compared to
ethylene, the bonding orbitals of carbon atoms have to come close together which results in shortening of bond
distance. Moreover, the effective size of sp hybrid orbitals is smaller than that of sp? hybrid orbitals. This also
decreases the distance between the constituent carbon atoms.
The value of the bond angle and the orbital structure of alkynes clearly indicate that they are linear
molecules. Hence, there is no restricted or hindered rotation about the triple bond. Alkynes, therefore, do not
show any geometrical isomerism.
HYDROCARBONS 13/59 =>

NOMENCLATURE OF ALKYNES
In common systems, the alkynes are named as derivatives of acetylene (Unit 12). For example.
HC =CH CH,C = CH CH,CH,C = CH CH,C = C—CH,
Acetylene Methyl acetylene Ethyl acetylene Dimethyl acetylene
In IUPAC system, they are named as derivatives of corresponding alkanes replacing ‘ane’ by the suffix
‘yne’. As already discussed the following rules should be followed :
(t) The longest continuous chain should include both the carbon atoms of the triple bond.
(it) The suffix used for alkyne is —-yne.
(iui) The chain is numbered from the end which gives the lower number to the first carbon atom of the
triple bond.
(tv) The position of the substituents are indicated.
For example,
CH 3
4 3 2 1 5 A | a 2 1 iL 2 3 4 5 6 3 2 1
CH, CH,C = CH CH, CH CH,C = CH CH.CH,C = C—CH, CH, CH—CH,—C = CH
But -1- yne 4-Methylpent-l-yne Hex-3-yne | 3-Cyclopropylprop-1-yne
If both double bond and triple bond are present in the molecule such that both get the same number while
numbering from either side then double bond is given preference.
1 2 3 A 5 6
CH, —CH—CH,—_CH,—C =CH
Hex-1-en-5-yne
However, if double bond and triple bound are present at non-equivalent positions, then lowest set of locants
rule is followed. For example,
1 2 3 4 5 5 4 3 2 1
HC=CH—CH=CH—CH, and not CH= C—CH=CH—CH,
Pent-3-en-1l-yne Pent-2-en-4-yne (wrong)

ISOMERISM IN ALKYNES
Alkynes exhibit the following structural isomerisms :
(t) Chain isomerism, The isomers differ in the chain of carbon atoms. For example, the molecule having
molecular formula C,H, shows chain isomers as :
5 4 3 2 if 4 3 2 i]
CH,—CH,— CH,—C = CH CH,—CH— C = CH
Pent-1-yne |
CH,
3-Methylbut-1l-yne
(ti) Position isomerism. Alkynes having more than four carbon atoms show position isomerism. For example,

CH,—CH,—C = CH CH,—C = C_CH,


But-1-yne But-2-yne
The structural isomers of C,H, may be written as :
(z) CH,—CH,—CH,—-C = CH Pent-1-yne
(IL) a a = CH 3-Methylbut-1-yne

CH,
(LiL) CH,—CH,—C = C—CH, Pent-2-yne
Structures (Z) and (iz) are chain isomers while structure, (i) and (zz) are position isomers.
(zzz) Functional isomerism. Alkynes having four or more carbon atoms are functional isomers of dienes
l.e., compounds containing two double bonds. For example,
4 3 2 1 1 2 3 4 | 2 3 4
CH,—CH,—C=CH CH,=CH—CH=CH, CH,—C—CH—CH,
But -1- yne Buta-1, 3- diene Buta-1,2-diene
(tv) Ring chain isomerism. Alkynes are isomeric with cycloalkenes containing the same number of
carbon atoms. For example, |
H,C—C=CH and L\
Propyne Cyclopropene
_— 413/60 MODERN'S abc + OF CHEMISTRY-XI

H,C—CH,—C=CH or CH,.—C=C—CH, and |__| are ring-chain isomers.


But-l-yne But-2-yne Cyclobutene
No geometrical isomerism. It may be noted that the alkynes are linear molecules and —C = C—H bond
angle is 180°. Therefore, alkynes do not show geometrical isomerism.
PREPARATION AND PROPERTIES OF ALKYNES
Preparation of Alkynes
The alkynes are prepared by the following general methods :
1. Action of water on calcium carbide. Ethyne (acetylene) is prepared in the laboratory as well as on
industrial scale by the action of water on calcium carbide.
CaC, + 2H,O —— HC=CH + Ca(OH),
Ethyne
(Acetylene)
Calcium carbide required for this purpose is obtained by heating calcium oxide (from limestone) and coke
in an electric furnace at 2275 K.
Caco, —“> CaO -+ CO,
Calcium carbonate Calcium oxide

Ca0+3C —#SK_, Cac, +af Co


Calcium carbide

- Ethyne gas

Calcium
carbide

Fig. 21. Preparation of ethyne.

In the laboratory, ethyne is commonly prepared from calcium carbide. In this method, a few pieces of
calcium carbide are taken in a conical flask fitted with a dropping funnel and a delivery tube. The air present in
the flask is replaced by oil gas because acetylene forms an explosive mixture with air. Water is taken in the
dropping funnel and allowed to fall into the flask dropwise. The reaction takes place vigorously and ethyne gas
is produced which is collected over water.
Purification. The gas obtained by the above method generally contains the impurities of hydrogen sulphide
and phosphine due to the contamination of calcium sulphide and calcium phosphide in calcium carbide. These
impurities are removed by bubbling the gas through acidified copper sulphate solution.
H,S + CuSO, —— Cus + H,sO,
(Black ppt.)
2PH, + 3CuSO, ——> Cu,P, + 3H,SO,
Cupric phosphide
(Black ppt.)
2. By dehydrohalogenation of dihalides or vicinal dihalides. Alkynes are prepared by double
dehydrohalogenation of vicinal dihalides (containing two halogen atoms on the adjacent carbon atoms) with
strong base like alcoholic solution of potassium hydroxide or sodamide in liquid ammonia.
R—CH—CH—R’ ess «6 RCO SR
| | Alkyne
> =
vic—dihalide
HYDROCARBONS 13/61 —_—

CH,Br CH
| + 2KOH ——> | + 2KBr + 2H,O
CH,Br (alc.) CH
1,2-Dibromoethane Acetylene
(Ethylene dibromide)
CH,—CH—CH, + 2KOH ——> CH,C=CH + 2KBr + 2H,O
(alc.) Propyne
Br Br
1,2-Dibromopropane
(Propylene dibromide)
The reaction, in fact, occurs in two steps. The first step gives haloalkene and under suitable conditions
this may be isolated. For example, vicinal dihalides on treatment with alcoholic potassium hydroxide give
alkenyl halide.

H
|
+ “OH no
-KX, -H,O R—C 74 *

Strong base xX

The haloalkene on treatment with sodamide in liquid NH, gives alkyne.


ie,
R—CZC-—R + ‘NH, _lo™
—HX
. p o@@ rR
Alkyne

The reaction may be written as:


RCH—CH—R’ #0808
“KX, -H,O = RCH=C—R’ Sa
- > R—C=C—R’
| | Alkyne
xX xX xX
Alkenyl halide

e.g. Br CH,—CH,Br3% H,C =CH—Br S/o N_, HC =CH


1, 2-Dibromoethane Vinyl bromide Ethyne
It may be noted that second step is more difficult and a stronger base like sodamide is needed for the
second step which removes proton from alkenyl halides more easily. Depending upon the conditions, these
two dehydrohalogenation reactions may be carried out as separate reactions or they may be carried out in a
single mixture.
The strong base sodamide, is capable of effecting both dehydrohalogenations in a single reaction mixture.
For this two molar equivalents of sodamide must be used. However, if the product is terminal alkyne, three
molar equivalents of sodamide would be used. This is because terminal alkyne product will react with sodium
amide and one equivalent of base in addition to the two equivalents required for double dehydrohalogenation
is needed. Treatment of the sodium salt of alkyne with water or acid gives corresponding alkyne. For example.
(CH3)36CH,CHC], —“S**2_5(CH,)CC = CNa —#22-4 (CH;),CC = CH
1,1-Dichloro-3,5-dimethyl Sodium salt of 3,3-Dimethy]! but-1-yne
butane alkyne (55-60%)

Vicinal dihalides are prepared by addition of chlorine or bromine to alkenes and therefore, alkenes can
serve as starting materials for the preparation of alkynes as shown below :
(CH,),CHCH = CH, —B%°C4_, (CH, ), CHCH—CH,Br —~ Gyo > (CH), CHC = CH
3-Methylbut-1-ene | 3-Methy] but-1-yne
Br
1,2-Dibromo-3-methylbutane
3. By the action of zinc on tetrahalogen derivatives of alkanes (tetrahalides). On treatment with
zinc in methanol, tetrahalides get dehalogenated to give alkynes.
Br Br
H—C—C—H +2Zn —3°% , HC=CH + 2ZnBr,
| | | Ethyne
Br Br (Acetylene)
1,1,2,2-Tetrabromoethane
_— 43/62 MODERN'S abc + OF CHEMISTRY-XI

Br 7
| |
CH,—C—C—H
3
+22n —=32*>CH.C=CH
Heat 3
+ 2ZnBr 2
i I Propyne
Tr Tr

1,1,2,2-Tetrabromopropane
This method is mainly used for the purification of alkynes or protection of a triple bond because the
tetrabromides are themselves obtained from alkynes.
4. Higher alkynes from acetylene. Higher alkynes are prepared from acetylene by treating its sodium
salt with alkyl halide. The sodium salt is prepared by treating acetylene with sodium metal at 475 K or by the
action of sodamide (NaNH,) on acetylene in liquid ammonia at 196 K.
HC=CH + £4NaNH, __liq. NHg_. HC=CNa + NH i)
Acetylene Sodamide Sodium acetylide

HC=CNa + CH,—Br ——>» HC=C—CH, + NaBr


Bromomethane Propyne
HC =CNa + C,H,Br ———> HC=C—C,H, + NaBr
Bromoethane But-1-yne
HC =CNa + CH,CH,CH,Br ———~+ HC = C—CH,CH,CH, + NaBr
1-Bromopropane Pent-l-yne

_ 2 NaNHo a Pa 2CHgl be MA

Disodium acetylide But-2-yne


This method is mainly used to convert lower alkynes to higher alkynes.
R—C=CH + Na —**#3_, R-C=C-Nat —%s, R—C=C—CH, + Nal
Lower alkyne Sodium alkynide Higher alkyne |
5. By electrolysis of aqueous solution of potassium salt of fumaric acid. Acetylene is prepared by
the electrolysis of aqueous solution of potassium salt of maleic acid or fumaric acid.
a aide Noe

C| sad
C| 7
Sfp Electrolysis
——_., a
|
CH + 2CO, + H, + 2KOH
a
KOOC H H
“oN COOK
. Acetylene
Potassium fumarate Potassium maleate
(trans-isomer) (cis-isomer)

This reacton is called Kolbe’s electrolysis reaction. It is believed to occur as :


CHCOOK CHCOO-
Ionization | + 2Kt
CHCOO-
2H,0 == 20H- + 2H*
At anode CHCOO- CHCOO CH
| : | — | + 2CO,
CHCOO- SHCOO OF
unstable
At cathode 2H* + 2e-
——— [2H] ——> H,
unstable
6. By dehalogenation of haloforms. Chloroform and iodoform on heating with silver powder undergo
dehalogenation to form ethyne.
CH Cl, + 6Ag + Ci, CH _Heat_. HC=CH + 6AgCl
Chloroform Acetylene
CH Le +6Ag + 1,|CH __Heat . HC=CH + bAgl
Iodoform = Acetylene
7. Synthesis from carbon and hydrogen. Acetylene can be prepared by passing a stream of hydrogen
through electric arc struck between carbon electrodes at 3270 K.
Electric arc
2C + H, 370K HC = CH
Acetylene
no) Oi) BD
Electrolysis reactions for the preparation of alkane, alkene and alkyne.
e Electrolysis of aqueous solution of sodium or potassium salt of acetic acid gives ethane (alkane)

CH,COOK CH
4 (aq) __Blectrolysia. |
CH,COOK CH;
Potassium acetate Ethane
e Electrolysis of aqueous solution of potassium salt of succinic acid gives ethylene (alkene)

‘H,COOK
Pe Cor eee
a
_ CH
|
CH,COOK CH,
Potassium succinate Ethylene
e Electrolysis of aqueous solution of potassium salt of fumaric acid gives acetylene (alkyne)
CHCOOK . CH
|e ty __Electrolysis_. |||
CHCOOK CH
Pot. fumarate Acetylene

Properties of Alkynes
Alkynes contain carbon-carbon triple bond (C = C) in their molecules. Alkynes resemble alkenes in most
of their characteristics.
Physical Properties
1. Physical state and smell. The first three members (ethyne, propyne and butyne) of the family are
gases at room temperature, the next eight are liquids while the higher ones are solids. All alkynes are colourless.
However, ethyne has characteristic odour of garlic smell.
2. Solubility. Alkynes are weakly polar in nature. They are lighter than water and immisible with water
but are soluble in organic solvents such as petroleum ether, carbon tetrachloride, benzene, etc.
3. Melting and boiling points. The melting and boiling points of the members of the family are slightly
higher as compared to those of the corresponding members of the alkane and alkene families. This is due to the
fact that the alkynes have linear structure and therefore, their molecules are more closely packed in space as
compared to the alkanes and alkenes. The magnitude of attractive forces among them are higher and therefore,
the melting and boiling points are also higher. The melting and boiling points increase with increase in molecular
mass of the alkynes.
Hydrocarbon Ethane Ethene Ethyne
m.p. (K) 101 104 191
b.p. (K) 184.5 171 198
Chemical Properties
The alkynes have at least one triple bond (— C = C —) in them, therefore, they are quite reactive chemically.
They readily take part in addition reactions and can also be easily oxidised. A few important chemical
characteristics are discussed below :
1. Acidic character of Alkynes
Acetylene and other terminal alkynes (1-alkynes) are weakly acidic in character. They react with strong
bases like sodium metal at 475K or sodamide, NaNH, (in liquid ammonia at 196K) to form sodium acetylide
derivatives known as acetylides or alkynides with the liberation of dihydrogen gas.
H—C=C—H + Na ——-> H—C=CNa’ + <H,
Monosodium ethynide

H—C=C Na + Na ——>» NaC =C Nat + H,


Disodium ethynide
H—C=C—H + NaNH, ——> H—C=CNa + NH,
Ethyne Sodamide Monosodium ethynide
(Acetylene)
_— 413/64 MODERN'S abc + OF CHEMISTRY-XI
H—C=C—H+ 2NaNH, ——> Na C=C Na +2NH,
Sodamide Disodium ethynide
R—C=C—H+ Na‘'NH, —— R—C=C Na’ +NH,
(terminal alkyne) Sodamide Sodium alkynide
These reactions have not been observed in case of alkenes and alkanes indicating that alkynes are acidic
in nature in comparison to alkenes and alkanes.
Sodium alkynide is decomposed by water regenerating alkyne.
R—C =C Nat + HO —— R—C=CH + NaOH
Sodium alkynide Alkyne
This shows that water is a stronger acid than alkynes and therefore, displaces alkyne from sodium
alkynide.
e.g. HC =C Nat + H,O ——> HC=}CH + NaOH
Monosodium Acetylene
acetylide
Acetylides react with alkyl halides to give higher alkynes. For example,
HC =CNa + CH,Br ——~+> HC=C—CH, + NaBr
Methyl bromide Propyne
The acetylenic hydrogen of alkynes can be replaced by copper (1) or silver (I) ions. They react with
ammoniacal solutions of cuprous chloride [Cu(NH,),|/*OH and ammoniacal silver nitrate [Ag(NH,)o]° "OH solution
(called Tollen’s reagent) to form the earrespendiie copper and silver alkynides.
H—C+=C—H + 2[Cu(NH,),)" OH ——> CuC =CCu + 2H,O + 4NH,
Acetylene Diamminecopper (1) Dicopper acetylide
hydroxide (Red ppt)
R—C+=CH + [Cu(NH,),]'OH — , R—-C=CCu + H,O+ 2NH,
(terminal alkyne) Monocopper alkynide
(Red ppt)
H—C=C—H + 2[Ag(NH,),]}OH-
48, AgC=CAg + 2H,O + 4NH,
Acetylene Diammine silver (I) Disilver acetylide
nitrate (White ppt.)
R—C+=CH + [Ag(NH,),]* OH- ——> R—C=CAg + H,O + 2NH,
(terminal alkyne) Silver alkynide
(White ppt)
Unlike alkali metal acetylides, copper and silver acetylides are not decomposed by water. However on treatment
with dilute mineral acids, they regenerate original alkynes.
CuC=CCu + 2HC] ——> HC=CH+2CuCl
Dicopper acetylide Acetylene
AgC=CAg + 2HNO, ———> HC = CH + 2Ag NO,
It may be noted that heavy metal acetylides are highly explosive when dry. Therefore, these should be destroyed
while still wet by warming with nitric acid.
It may be noted that only the terminal alkynes (1-alkynes) react with ammoniacal solution of silver nitrate or
cuprous chloride. Therefore, this reaction can be used to distinguish between 2-alkynes and 1-alkynes.
1-Alkynes will give this test while 2-alkynes will not give this test.
CH,CH3C CH + [Ag(NH,),]O0H ——> CH,CH,C = CAg
But-1l-yne White ppt.
CH,C = CCH, +[Ag(NH,),JOH —M——> No reaction
But-2-yne
Explanation for the acidic character of alkynes. The acidic character of l-alkynes can be explained
on the basis of sp hybridisation state of the carbon atoms in alkynes. We know that an electron in s-orbital is
more tightly held than in a p-orbital eases s-electrons are more closer to the nucleus. In sp hybridisation,
s-character is more (50%) as compared to sp? (33%) or sp? (25%) hybrid orbitals. Due to very large s-character,
the electrons in sp hybrid orbitals are held more tightly by the nucleus and are quite electronegative.
Consequently, the electron pair of H — C= bond gets displaced more towards the carbon atom and helps in the
release of a proton (H* ion).
RC C vrrrrrttetetees A (Cleavage of bond is easy)
HYDROCARBONS 13/65 =>

The alkynes with triple bond in the non-terminal position do not show any acidic character as there is no
hydrogen atom attached directly to the triple bonded carbon atom (H.C — C = C — CH.,).
Comparison of relative acidic strengths of alkanes, alkenes and alkynes. The alkynes are
weakly acidic in nature while alkenes and alkanes do not show any acidic nature. This is due the fact that
the carbon atoms of the double bond are sp? hybridised (33% s-character) while those involved in the
single bond are sp* hybridised (25% s-character). Because of lesser s-character, these carbon atoms are
much less electronegative than the carbon atoms taking part in the triple bond. Therefore, the release of
H* ion from an alkene and alkane molecule is difficult and they do not show acidic character. The acidic
character decreases as :
HC = CH = CH. = CH, > CH, — CH,
K, ~ 10-5 K, = 10-85 K, ~ 10-1
The relative acidity of acetylene is more than that of ammonia but less than that of water.
H,O > HC = CH > NH,
The acidity among alkynes is
HC = CH > CH,—C = CH >> CH,—C = C—@ny
It may be noted that hydrocarbons are weak acids as compared to water (K, ~ 107"), alcohols
(K, = 10-1 to 10-8) and carboxylic acids (K, = 10~).
2. Addition Reactions
Like alkenes, alkynes undergo addition reactions due to their unsaturated nature. However, in alkynes,
the addition occurs in two steps. In the first step, one molecule of the attacking reagent is added to the alkyne
and its triple bond changes to a double bond. In the second step, a second molecule of the reagent is added which
changes the double bond into a single bond and a saturated compound is obtained.
¥ xX X
| |
—C=C— + X, os eC — of@ f+A2_ , —_C _ C0
Electrophile x | |
X XX
The alkenes formed by the addiction of X, to alkynes are generally trans-isomers. Some important
electrophilic addition reactions are discussed below :
(a) Addition of halogens. Halogens add to alkynes in two steps forming a dihalide and then tetrahalide. For example,
Cl Cl
3 Cl, : | Cl, | |
HC=CH cq,’ HC=CHy —Gqj? H-— °— °—H
Ethyne Ch Cl C] C]

1, 2-Dichloroethene
1, 1, 2, 2-Tetrachloroethane
1,1,2,2-tetrachloroethane is called westron. It is used as a solvent.
i r
__ Bro | 7 Bry
H.C — C=CH “(CGl,) H.C —C=CH Cal,’ H.C —C—C—H
Propyne | | | |
Br Br Br Br
1,2-Dibromopropene 1, 1, 2, 2-Tetrabromopropane

In case of addition of bromine, the reddish orange colour of solution of bromine in carbon tetrachloride
gets discharged as in alkenes. Therefore, this can be used as a test for unsaturation in alkynes also. Thus, the
discharge of brown colour of bromine indicates the presence of unsaturation (alkenes or alkynes).
Br Br
—C=C— + Br, —— —C—C—
Alkyne Reddish Le ol
orange colour Br Br
Colourless
_— 13/66 MODERN'S abc + OF CHEMISTRY-XI

The reaction of alkynes with iodine is very slow reaction. It is believed to stop after the addition of one
molecule of iodine to alkene stage as
I H
HC =CH +L, _C2Hs0H_, Neo=cd
| H~ Nr
trans- 1, 2-diiodoethene
Thus, the order of reactivity of halogens towards alkynes is :
Cl, > Br, > LI,
(6) Addition of halogen acids. The addition of halogen acids (HCl, HBr and HI) also take place in two
stages to form dihalogen derivatives. After one molecule of the acid has been added to a symmetrical alkyne,
the product is an unsymmetrical derivative of alkene (vinyl halide). The further addition takes place in accordance
with the Markovnikov rule to form alkylidene dihalides (gem dihalides) in which two halogens are attached to
the same carbon atom. For example,

Hc = cH —HC , H-c=c—H —#C.,


LA
H—C—C_—H
Ethyne | |
H Cl H Cl
Chloroethene 1,1-Dichloroethane
(Vinyl chloride) (Ethylidene dichloride)

Br
HB HBr |
H,C —C =CH > H,C —C=CH, ——— > H,C—C —CH,
Propyne | |
Br r
2-Bromopropene 2, 2- Dibromopropane

It may be noted that peroxides have the same effect on the addition of hydrogen bromide to alkynes as
they have on alkene.
The order of reactivity of halogen acids is HI > HBr > HCl>HF

Wiceh arise . Addition of halogen acids to alkynes.


The addition of halogen acids to alkynes is an electrophilic addition reaction similar to alkenes.
It occurs in two stages and the path is decided by the stability of carbocation formed.
HCl =——: Ht + Cl-
. H+ slow H,C — CHt
Stage I HC=CH
Vinylic carbocation
H,C—CHt /@, “SR/ —*, H,C = CHCl
1-Chloroethene
CH,CH*—Cl
Stage II H,C—@@(r ap / Ht —, 2° Carbocation (I)
(more stable)
CH,-CH,Cl
1° Carbocation (II)
(less stable)
2° Carbocation is more stable than 1° carbocation and therefore, the reaction occurs through carbocation (T)
forming 1, 1-dichloroethane as
CH,-CH’—Cl + Cr —®*, CH,—CH—Cl
2°Carbocation a
Cl
1, 1-Dichloroethane
HYDROCARBONS 13/67 =>

(c) Addition of hypohalous acid (HOX). Alkynes react with two molecules of hypohalous acids
(HOCI or HOBr) or halogen Cl, or Br, in the presence of H,O. For example,

— ce [OH ?Cl
Th LY Cl
H—c =c—H—“1!_.Hc = CH Cl-OH Hc ley) 2226 H-—C— or OHC — CHCl,
Acetylene ae || | 0 Cl
Cl Cl | OlHI Cl |
Dichloroacetaldehyde
(2, 2-Dichloroethanal)

OH (Cl
HOCI HOC! || -H,0
H.C — C = CH ——— CH,
—C =CH ———} |CH, —C—CH |—— H,C—C—CHCl,
3 3 Markovnikov 3 ; | 3 | -
e L4
Propyne OH Cl OH, Cl O
Dichloroacetone
(1, 1-Dichloropropanone)

(d) Addition of water (hydration of alkynes). Like alkanes and alkenes, alkynes are also immiscible
and donot react with water. However, alkynes react with water in the presence of mercuric sulphate (HgSO,)
and sulphuric acid at 333 K. The products are carbonyl compounds (aldehydes and ketones). In this reaction,
first enol is formed which contains a double bond (ene) and an alcoholic group (ol). But the enol formed is quite
unstable in the acidic conditions. It isomerises to a more stable (kefo) form as a result of rearrangement.
For example :

H»SO, (dil.), HgSO4 |


HC = CH + H,O — a H,C = CHOH __!somerisation _. H,C —C — H
Ethyne Ethenol Kthanal
(Vinylalcohol)
In case of unsymmetrical alkynes, the addition occurs in accordance with Morkovnikv’s rule. For example,

CH,— C =CH+H,O —H2804


(il),HeS04 HC
— C = CH, __lsomerisation_. HC —C — CH,
Propyne oy | ||

Prop-l-en-2-ol Propanone

The rearrangement of enol form into keto form is called tautomerism.

However, if the unsymmetrical alkyne is non-terminal, a mixture of two isomeric ketones is obtained in
which methyl ketone predominates. For example,

H9S04 dil.
CH,CH, — C=C— CH, + H,O —HgS0q, 333K” CH,CH,CH, — C—CH,
, | + CH,CH,—C—CH,—CH,
I 9

Pent-2-yne Pentan-2-one Pentan-3-one


(Non terminal alkyne) (Major product) (Minor product)

(e) Addition of hydrogen cyanide. Alkynes react with hydrogen cyanide (HCN) in the presence of
barium cyanide catalyst. Acetylene gives vinyl cyanide or acrylonitrile which can be used to prepare synthetic
fibre orlon.
CH=CH+HCN —“““~2>
Ba (CN
cH, =CH—CN
Vinyl cyanide
(acrylonitrile)
_— 13/68 MODERN'S abc + OF CHEMISTRY-XI

(f) Addition of alcohols and carboxylic acids. Ethyne adds a molecule of alcohol in the presence of
alkali to give vinyl ether.
KOH
HC =CH +CH,OH eae H,CO—HC = CH,

Methyl vinyl ether


Similarly, alkynes add acids in the presence of a Lewis acid catalyst, Hg** ions to give vinyl esters. For
example,

HC = CH+CH,COOoH #2". 4,c = CHOCOCH, —#£"_4 CH,—CH(OCOCH,),


a+ +

CH
Vinyl acetate : Ethylidene diacetate

Ethylidene diacetate on heating to 573-673 K gives acetaldehyde and acetic anhydride :


OCOCH,
a |
CH.—CH 8573 --678K
673 K
. CH.CHO ‘ (CH.CO),0
° ~~ OCOCH, cee Adie anhydride
(g) Addition of hydrogen (hydrogenation). Alkynes react readily with hydrogen in the presence of
finely divided Ni, Pt or Pd as catalysts. The reaction is called hydrogenation or reduction of alkynes.

H,, 23 Ni or Hg, Ni or
HC CH Pt or Pd CH, CH, Pt or Pd CH; CH;
Ethyne Ethene Ethane

CH,—C =CH 22. Nar ,cH;CH


Ho, Ni or Pt Ai an
cH, —2"eF
Ho, Nior Pt
,CH,CH,CH,
Propyne Propene Propane

It may be noted that the hydrogenation can be controlled at the alkene stage only. This is possible by
using a Lindlar’s catalyst which is a mixture of palladium and barium sulphate poisoned by quinoline.
It allows the hydrogenation of alkynes only to the alkene stage.
Pd, BaSO,
HC =CH Quinoline CH, _ CH,
(Lindlar’s catalyst) Ethene

It may be noted that catalytic reduction of alkynes with hydrogen in the presence of Lindlar’s catalyst
gives cis-alkenes.
CH, CH,
Pd, BaSO, ee
CH,—C=C—CH, Quifoline me
But-2-yne H H
cis - But-2-ene

However, chemical reduction (known as Birch reduction) of non-terminal alkynes with sodium
(or lithium) in liquid NH, at 195—200K gives trans alkenes.
CH,
CH,—C = C—CH, Na in lig. NH3 4
=
ee
196 - 200 K res
But-2-yne CH e H
trans - But-2-ene
3. Oxidation
Alkynes can be oxidised under different conditions to give a variety of products.
(a) Combustion. When alkynes are burnt in air or oxygen, carbon dioxide and water are formed.
The reaction is highly exothermic and a large amount of heat is produced.
2HC=CH +50, —_____—. 4CO, 4 2H,O; AH =— 1300 kJ
Ethyne
Under normal conditions, acetylene burns with a luminous yellow sooty flame due to the presence of
higher carbon content. However, with air or oxygen under high pressure, acetylene burns with a blue flame
producing high temperature of the order of 3000 K. The heat produced during the combustion of acetylene can
be used for welding purposes in the form of oxy acetylene flame.

(6) Oxidation with alkaline potassium permanganate


(t) With dilute cold KMnQO, solution
Alkynes react with dilute cold alkaline KMnO, solution and form diketones or carboxylic acids depending
upon the nature of alkyne.

In these reactions, =CH part of alkyne is oxidised to LG


| group while RC= part is oxidised to R—C—
part. For example,
CH,C=CH + KMn0O, + H,O —“23*
cH, c—c—OH
Propyne Baeyer’s reagent
O O
2-Oxopropanoic acid (Pyruvic acid)

CH, —C = C— CH, 298-303 K


|
dil. Alkaline KMnO,4
CH, —C—C— CH,
But-2-yne |
O
Butane-2, 3-dione (Biacetyl)

Acetylene under these conditions gives oxalic acid.


CH COOH
Alk. KMnO
| +4101 —sos303K* |
CH COOH
Ethane-1, 2-dioic acid
(Oxalic acid)

In the above reactions, the pink colour of alkaline KMnO, (Baeyer's reagent) gets discharged. Therefore, this
reaction can be used as a test for alkynes.

(ii) With hot KMn0O, solution


When alkynes are treated with hot KMn0O, solution, the triple bond is completely broken leading
to the formation of carboxylic acids and carbon dioxide depending upon the position of triple bond.
The cleavage occurs at the site of triple bond.

CH,C=CH——_~—___
!
+ >CH,COOH + CO,
Alkaline KMnO,

Propyne Ethanoic acid

In this case, =CH part of alkyne is oxidised to CO, and H,O while RC= part is oxidised to
RCOOH. For example,
Alkaline KMn0O,
HC =CH 373-383 K
2CO, +H,0
Ethyne

WWW.JEEBOOKS.IN
— 13/70 MODERN'S abc + OF CHEMISTRY-XI
Alkaline KMnO,
CH,CH,C =CH 374558 K CH,.CH,COOH + CO,
But —l—yne Propanoic acid

CH, — C=C
3
— CH, ———_—*
AlkalineKMnO
373-383K
> 2CH,
;
COOH
cue
But-2-yne Ethanoic acid

Alkaline KMnO,
CH.CH, C=C CH, 373888 K CH,CH,COOH + CH,COOH
Pent Seine Propanoic acid Ethanoie acid

The above reactions in which triple bond is completely broken are called degradation or cleavage
reactions. These can be used to locate the position of triple bond.
It may be remembered that
with cold KMnO, (at 298-303 K)
CH part is oxidised to — COOH group
= CR part is oxidised to R — c— group
O
with acidic KMnO, at high temperature (373-383 K)
= CH part is oxidised to CO, and H,O
= CR part gets oxidised to RCOOH.
e.g. ,
CH, CH, C=CH
Ox3 But-l-yne
sX
poe “uy,
FX. ~ o
a> 3 p=

Propanoic acid
O
2-Ketobutanoic acid

(c) Oxidation with ozone (Ozonolysis). The alkynes react with ozone to form ozonides. These ozonides
on decomposition with zine dust and water or H,/Pd (reductive cleavage) give dicarbonyl compounds.
O

CH,C=CH —2,/@gHmip/ cu —200


, cH,-c— C—H
Propyne i | Il I

Propyne ozonide 2-Oxopropanal

CH,C = CCH»,
O;
>> CH, — C — C— CH, Sa
Zn, H2O CH,— C — i— CH,
But-2-yne | | !
O— O O O
But-2-yne ozonide Butane-2,3-dione
Ethyne gives glyoxal on ozonolysis.

/ \ Zn, HO |
CH =CH +O, —— > HC—CH —=90—7 H—-C— C—
Ethyne | |
O —OQO
lI
1, 2-Ethanedial
(Glyoxal)
HYDROCARBONS

4, Polymerisation Reactions
Alkynes have larger tendency to polymerize than alkenes and therefore undergo two types of polymerization
reactions :
(a) Linear polymerisation
(¢) Acetylene undergoes linear polymerisation under suitable conditions to form linear polymer polyacetylene
or polyethyne. It is a high molecular mass conjugated diene containing repeating units of (CH=CH—CH=—CH)
and the polymer can be represented as —CH—=CH—CH=CH-—.. Under special conditions, this polymer conducts
electricity. Thin sheets of polyacetylene can be used as electrodes in batteries.
(iz) On passing acetylene through a solution of cuprous chloride and ammonium chloride at 343K,
vinylacetylene and divinylacetylene are formed.
2HC=CH ——~
Cu,Cl,,NH,Cl
7
343K
> H—C=C—CH=CH, Cu,Cl,/NH,Cl a oa H,C=CH—C=C—CH=CH,
Acetylene Vinylacetylene Divinylacetylene
Vinylacetylene is widely used in the manufacture of chloroprene which is a starting material for the synthetic
rubber neoprene.
CH,—CH—C=CH + HC] —““""""_,
addition
H,C=CH—C=CH,
Vinyl acetylene Cl
Chloroprene
(b) Cyclic polymerisation. Alkynes on passing through red-hot iron tube at 873K, undergo cyclic
polymerisation. For example,
(i) When ethyne is passed through red hot tube at 873 K, three molecules of it polymerise to give benzene.
HC x ya \

Red hot tube HC CH


ser (CH
(Catalyst) | |
HC ¥ \y HC a HC CH
HC O CH Ze
3 molecules of acetylene Benzene
This is a best route for entering from aliphatic to aromatic compounds. Benzene can be used as a starting
material for preparation of derivatives of benzene and other organic compounds.
Similarly, propyne trimerises under similar conditions to form mesitylene (1, 3, 5-trimethyl benzene).
CH,
|

wp| ©
C
SS
Red hot Fe

CH,—C yx, C_CHi


oc 4 ;
1, 3, 5-Trimethylbenzene
(Mesitylene)
(it) Four molecules of ethyne combine together in the presence of nickel cyanide as catalyst and under
high pressure to form tetramer called cycloocta-1,3,5,7-tetraene.
HC==CH 6 5
: pc CH,

He)
HC
oh
(cu
Ni(CN), catalyst
High pressure
HC
re |
HC
CH
[4
CH
or
q 4

:
\ f3 :
HC==CH HC=mCH 1 2
1 2
Cycloocta—1,3,5,7-tetraene
5. Isomerization of alkynes
When alkynes are heated with sodamide (NaNH,) in an inert solvent, such as kerosene oil or paraffin oil,
they undergo isomerization t.e., 2-alkynes isomerize to 1-alkynes. However, in the presence of alcoholic potash
reverse isomerization is effected.
NaNH@ in inert solvent
CH,C=C—CH, CH,CH,C=CH
ale - KOH
But-2-yne But-1-yne
CH,CH,C=CH —2S#2minet
alc - KOH
solvent. CH,C=C—CH,
But-1l-yne But-2-yne
_— 13/72 MODERN'S abc + OF CHEMISTRY-XI

Summary of Reactions of Ethyne

Br,, CCl, e
CHBr, — CHBr, 1, 1, 2, 2-Tetrabromoethane

CH,CHBr, 1, 1-Dibromoethane
CH, = CHOH —> CH,CHO Ethanal

CH, = CHCN Vinyl cyanide

COL + Ho Carbon dioxide


Combustion

HC =CH COOH |
Ethyne | Oxalic acid
COOH

il
O

HC=CNa —*>»HC=CCH, Sodium acetylide, Propyne

AgC =CAg Silver acetylide

CuC =CCu Copper acetylide


[(Cu(NH,),]?*

1 BW BO)

Effect of Hybridisation on Bond Lengths, Bond dissociation enthalpies (or bond strengths) and
Electronegativity
The bond length and bond strength of a bond depends upon the type of hybrid orbitals involved.
© Bond length. Since a p-orbital is bigger in size than an s-orbital of the same shell, therefore, as we go from
sp® to sp” and to sp, the percentage of p-character decreases from 75% (in sp*) to 66.7% (in sp) and to 50%
(in sp). Accordingly, the size of hybrid orbitals decreases in the order : sp? > sp? > sp. As a larger orbital forms
a larger bond, therefore, carbon-carbon bond length decreases in the order :
C (sp*—sp*) > C (sp?—sp”) > C (sp-sp)
154 pm 145 pm 138 pm
e.2., Shoei. 18 ae PS le CH=C—C=CH
Bond length also decreases with an increase in the multiplicity of the bond (or bond order) because the
sidewise overlap of p-orbitals to form z-bonds brings the two carbon atoms closer.

aa oe
“ a
—C=C—
=

154 pm 154 pm 120 pm


e.g., H,C—CH, H,C=CH, HC=CH
Similarly, a C—H bond may be formed by overlap of a sp*, sp? or sp-orbital of carbon with an 1s orbital
of hydrogen. As the size of the hybrid orbital decreases, the length of C—H bond also decreases.
Consequently, a C—H bond formed with a carbon orbital to high s-character will be shorter than the one
formed with a carbon of orbital of high p-character as:
HYDROCARBONS 13/73 =>

C (sp) -H > C (sp?)-H > C(sp}-sp


111 pm 110 pm 108 pm
eg., CH, CH,=CH, CH=C—H
As discussed above, pi bonds are weaker than sigma bonds because during the formation of t-bonds. The
extent of overlap is low. For example,
Carbon—Carbon (i bond) Carbon—Carbon (o bond)
284 k.J 348 kJ
However, a carbon-carbon double bond (in ethene) is stronger (681 kJ mol) than a carbon-carbon single bond
(348 in ethane). Thisis because a double bond consists ofa strong o-bond and a weak 1-bond. Similarly, a carbon-
carbon triple bond is stronger (823 kJ mol in ethyne) than a carbon-carbon double bond because it consists
of one 6-bond and two 1-bonds.
In may be noted that bond length also increases with increase in size of the bonded atoms with C-orbital of the
same type.
C—F C—Cl C—Br C—I
142 pm 177 pm 191 pm 213 pm
6 Bond strength (or Bond enthalpy). Bond strength increases with decrease in bond length. For example,
for C—H bond,
C (sp®) -H < C (sp*)-H < C (sp)-H
436 kJ 443 kJ 507 kJ
Similarly, for C—C bond;
C (sp)ay (sp?) < C (sp) Csp") < C (psp)
o Hisceeaeenieg The type of hybridisation also“effects the electronegativity of a atom. The more
the s-character of the hybrid orbital, more electronegative is the atom. Thus, a sp-hybridized carbon
atom having 50% s-character is more electronegative than a sp?- hybridized carbon atom with 33.33%
s-character and sp® hybridized carbon with 25% s-character. This is mainly because s-orbital is more
closer to the nucleus than a p-orbital and is more tightly hold by the nucleus and is, therefore, more
electronegative.

SOLVED EXAMPLES
lL} Example 21.
Write IUPAC names of the following ;
(i) CH, = CH—CH—CH = CH—CH=CH, (ii) CH =C—CH—CH =CH,
| |
CH, CH,
Solution:
7 6 a 4 3 2 1 1 2 3 4 2
(i) CH, = CH-CH/@M = CH —CH = CH, (ii) HC = een = CH,
CH, CH,
5- Methylhepta- 1, 3, 6-triene 3-Methylpent- a en-l-yne
lL} Example 22.
Write structures of different isomers corresponding to the 5th member of alkynes series. Write IUPAC names
of all the isomers and name the type of isomerism exhibited by different pairs of isomers.
Solution: 5th member of alkyne series has the molecular formula C,H,,. The different isomers are :
1 2 3 A 5 6
(i) HC = C—CH,—CH,—CH,—CH, Hex-1l-yne
1 2 3 4 5 6
(ii) CH,—C = C —CH,—CH,—CH, Hex-2-yne
1 2 3 4 5 6
(wt) CH,—CH,—C = C —CH,—CH, Hex-3-yne
1 2 3 4 )
(wv) HC=C—CH —CH,—CH, 3-Methylpent-1l-yne

H,
1 2 3 4 5
(v) HO=C—CH,—CH —CH, 4-Methylpent-1l-yne

H,
1 2 a 4 o
(vt) CH,—C==C—CH —CH, 4-Methylpent-2-yne

CH,
EZ MODERN'S abc + OF CHEMISTRY-XI

CH,
(vii) HCO = c_t_cu, 3,3-Dimethylbut-1-yne
|
CH,
These represent position and chain isomerism shown by different pairs.

lL} Example 23.


How would you separate propene from propyne ?
Solution: Pass the mixture through ammoniacal silver nitrate solution. Propyne will precipitate out as silver propynide
and propene will pass unchanged.
H,C—C =CH+Ag* —> H,C—C =C Agt
Precipitate
H,C—CH =CH,+Agt — > No reaction
Example 24.
How would you carry out the following conversions ?
(a)
(b) — H,C—CH,—CH(OH)CH,OH
(c) — H,C COCH,
(d) Br, CHCHBr, — HC =CH

Solution: (a) H,CCH,CH = CH, Belg, H202,0H _, CH,CH,CH,OH


(6) H,C CH,CH = CH, AK KOs _, H,C—CH,—CH—CH,

(c) HJCC = CH HyS04


A Rearrangement
one = CH, See CALCCH,
OH O
(d) Br, CHCHBr, —“5 HC = CH

How will you prepare acetaldehyde from acetylene ?


Which of the following alkynes react with sodium in liquid ammonia ?
(z) Propyne (71) But-2-yne (111) Pent-3-yne (iv) Hex-1-yne
@ 25. Name the process which may be used to locate the position of a triple bond.
oO 26. Which of the following is most acidic
(t) Butane (iz) But-1-ene (viz) But-1-yne (iv) But-2-yne ?
0 27. What happens when water is dropped on calcium carbide ?
@ 28. Name the reagent ‘X’ in the reaction :
1, 2-Dibromoethane = /, Acetylene
Q 29. Write chemical equation for the combustion of hexyne.

= Answers to Practice Problems


© 23. HC = CH a, 4c = CHon = CH,CHO
4
© 24. (z) and (iv)
© 25. Ozonolysis
© 26. (tL)
© 27. Acetylene is formed.
© 28. alc. KOH
Vv Heat
© 29. C,H, (ge)+ a: O, (g) ——— 6C0,(g) + 5H,0 (g)

WWW.JEEBOOKS.IN
HYDROCARBONS sms) 0

_ CHEMISTRY
OF AROMATIC HYDROCARBONS _»

ARENES OR AROMATIC HYDROCARBONS


The parent members of the family is benzene having the molecular formula C,H,. It has hexagonal ring of
six carbon atoms with three double bonds in alternate positions. It has a resorianics stability and the structure
may be represented as shown in (c).
H

Bee
H | H |
H
(a) (6) (c)
Most of these compounds possess pleasant odour (Greek aroma meaning pleasant smelling) and therefore,
these compounds were named as aromatic compounds. Aromatic compounds containing benzene ring are
known as benzenoids and those not containing a benzene ring are known as non-benzenoids. Some examples
of arenes are :
CH, CH,CH, CH, —CH— CH,

Benzene Toluene 1, 2-Dimethylbenzene QO | Isopropylbenzene


(or Methyl benzene) (o-xylene) (cumene)

Monocyclic arenes like benzene (C,H,) have the general formula C,,H,,,_, and bicyclic arenes like naphthalene
(C,,H,) have the general formula C,H,,_,.. In general an arene with ‘m’ rings has the formula C,H, ,,,. The
structures of monocyclic, bicyclic and tricyclic rings are given below :

Benzene
ouoy aay
Bote.
H H
Anthracene
H

Some arenes may contain two or more isolated rings. For example,

Diphenyl or Biphenyl Diphenylmethane

Triphenyl methane
NOMENCLATURE
The nomenclature of aromatic hydrocarbons have already been discussed in the previous unit. Some common
examples are :
CH,CH,

Toluene Ethylbenzene 1,2-Dimethylbenzene


(o-Xylene)
— 13/76 MODERN'S abc + OF CHEMISTRY-XI
CH, CH, CH, CH,
CH, 1

Cl Chk
6 2

CH, CH, H,C 4 CH,


CH, |
1,3-Dimethylbenzene 1,4-Dimethylbenzene 1, 2, 3-Trimethylbenzene 1, 3, 5-Trimethylbenzene
(m-Xylene) (p-Xylene) (Mesitylene)
STABILITY AND STRUCTURE OF BENZENE
According to the structures, arenes are expected to be unsaturated and highly reactive in nature. However,
they are stable. Because of unsaturated nature, benzene is expected to undergo only addition reactions like
alkenes and alkynes. However, it does undergo addition reactions but it mainly undergoes substitution reactions
in contrast to unsaturated hydrocarbons.
Br,
C,H;Br+ Br Substitution reaction
FeBr,

Benzene C,H,Br, Addition reaction


Br,,CCl, (Not formed)
The stability of benzene is explained on the basis of resonance as well as its orbital structure.
Resonance in Benzene
The stability of benzene can be explained on the basis of concept of resonance. Kekule in 1865 gave a ring
structure for benzene in which the positions of the three double bonds are not fixed. He suggested that the
double bonds keep on changing their positions and this is called resonance. According to Kekule, benzene is a

I —- Oso
resonance hybrid of the two structures (a) and (6) and the hybrid structure may be represented as (c).

(a) (6) (c)


Kekule structures of benzene Resonance hybrid
The resonance structure of benzene is supported by the following facts :
(1) The carbon-carbon bond length in benzene is 139 pm which is intermediate between bond lengths for
C—C bond (154 pm) and C = C bond (134 pm) and the value is the same for all the bonds.
(ii) Due to resonance the m-electron charge in benzene gets distributed over greater area i.e., it gets
delocalised. As a result of delocalisation the energy of the resonance hybrid decreases as compared to contributing
structures by about 50 kJ mol. The decrease in energy is called resonance energy. Therefore, it is stabilised
and behaves as a saturated hydrocarbon.
(iii) If the positions of double bonds are fixed, we expect two isomers of 1,2-dichlorobenzene as shown
below (one having Cl atoms attached to C—C bond and the other having Cl atoms attached to C = C bond) :
Cl C]

However, there is only one compound which can be explained only on the basis of equal type of bonds
between the two carbon atoms. This supported the Kekule’s proposed structure in which the double bonds in
benzene are continuously oscillating back and forth between two adjacent positions. Since positions of double
bonds are not fixed, only one product is formed.
Cl Cl
Cl
Cl | Same product

This structure was also known as Kekule’s dynamic formula in which he proposed dynamic equilibrium
between two structures.
Though Kekule formula for benzene was | | Padi
accepted, many alternative structures have also
been proposed from time to time by different -
workers. Some of these (I-III) are given :
I {I Ill
Dewar (in 1867) — Claus (in 1867) Baeyer (in 1892)
Modern Concept : Orbital Structure of Benzene
According to the orbital concept, each carbon atom in benzene is sp*-hybridised and one orbital remains
unhybridised. Out of the three hybrid orbitals, two overlap axially with the orbitals of the neighbouring carbon
atoms on both sides to form o-bonds. The third hybridised orbital of the carbon atom overlaps with the half-
filled orbital of the hydrogen atom resulting in C—H bonds. Thus, benzene has a planar structure with bond
angle of 120° each (Fig. 22).

ey7
H

Fig. 22. Overlap of the hybridised orbitals Fig. 23. Unhybridised 2p-orbital on each
of carbon to form o-bonds. carbon atom.

There is still one unhybridised 2p-orbital left on each carbon atom as shown in Fig. 23. Each one of these
orbitals can overlap sidewise with similar orbital of the carbon atoms on either sides to form two sets of
nt-bonds [Figs. 24 (a) and 24 (6)|. The resultant m-orbital cloud is spread over all the six carbon atoms and has
been shown in Fig. 24 (c). As a result, there are two continuous rings of t-electron clouds, one above and the
other below the plane of the carbon atoms as shown in Fig, 25.

Therefore, benzene is a flat molecule in which


the six carbon atoms are arranged at the corners of a
hexagon. The m-electron charge in benzene is not
confined to space between two carbon atoms as in
ethylene, but is spread over a greater area. This is known
as the delocalisation of the electron charge. This
delocalisation results in the stronger bonds than the
normal m-bonds and accounts for the stability of the
molecule Fig. 25. m-electron clouds; one lying above
and the other below the plane of carbon atoms.

Stability of benzene and enthalpy of hydrogenation


Benzene does not undergo addition reactions readily but 1t undergo substitution reactions like nitration,
sulphonation, halogenation, etc. This shows that benzene is more stable than the hypothetical cyclohexatriene
molecule. This unusal stability of benzene which had been puzzling the chemists for more than fifty years could
only be explained on the basis of resonance concept. This has been proved by the fact that the enthalpies of
hydrogenation and combustion of benzene are lower than expected on the basis of cyclohexatriene.
Enthalpy of hydrogenation ts the amount of enthalpy change when one mole of an unsaturated
hydrocarbon tis hydrogenated. It has been experimentally observed that enthalpy of hydrogenation for
_— 13/78 MODERN'S abc + OF CHEMISTRY-XI
disubstituted alkenes, R—CH=CH-R varies between 117—125 kJ mol-!. For example, the enthalpy of hydrogenation
of cyclohexene is found to be 119.5 kJ mol-!.

CD hh > C) AH =-119.5 kJ mol


Cyclohexene Cyclohexane
If the benzene contains three double bonds which do not interact (as in case of cyclohexatriene) then its
enthalpy of hydrogenation should have been three times the enthalpy of hydrogenation of cyclohexene.
Enthalpy of hydrogenation expected for cyclohexatriene :

CT +385 __Catalyst_, e AH =-119.5x3


Cyclohexatriene Cyclohexane = —358.5 kJ mol!
However, the experimental value of enthalpy of hydrogenation of benzene is —208.1 kJ mol

+ 3H, __Catalyst_, C) AH = -208:1 kJ mol


Benzene Cyclohexane
Thus, the actual molecule of benzene is more stable than hypothetical cyclohexatriene molecule by an
amount equal to :
358.5 — 208.1 = 150.4 kJ mol!
This is called resonance energy. Thus, resonance energy for benzene is 150.4 kJ mol7!. This can also be
confirmed by enthalpy of combustion data.

ISOMERISM IN ARENES
Arenes exhibit position isomerism. Monosubstituted benzenes do not show any isomerism because all the
six hydrogen atoms attached to the carbon atoms are identical. The disubstituted arenes exhibit position
isomerism. For example, there are three disubstituted benzenes namely 1,2 (or ortho), 1,3 (or meta) and
1,4 (or para) isomers as shown below :

CH, CH,

Cm “ H,

1,2 Dimethylbenzene 1,3-Dimethylbenzene 1,4-Dimethylbenzene


(ortho-Xylene) (meta-Xylene) (para-Xylene)
Similarly, three position isomers of trimethylbenzene are :
CH.
1 CH,

3
Hi,

1,2,3-Trimethylbenzene 1,2,4-Trimethylbenzene 1,3,5-Trimethylbenzene


(Mesitylene)

In the case of naphthalene, even monosubstituted compounds exhibit isomers as in 1-methyl and
2-methyl naphthalene.
CH,

IC On
1-Methylnaphthalene 2-Methylnaphthalene
(a-Methylnaphthalene) (B-Methylnaphthalene)
With higher arenes, the number of possible position isomers also increases.
HYDROCARBONS 13/79 —_—

AROMATICITY (HUCKEL RULE)


We have learnt that aromatic compounds are those which resemble benzene in chemical behaviour. These
compounds contain alternate double and single bonds in a cyclic structure. They undergo substitution reactions
rather than addition reactions. This characteristic behaviour is called aromatic character or aromaticity.
Criteria for Aromaticity
The modern theory of aromaticity was advanced by Eric Huckel in 1931. The aromaticity depends upon the
electronic structure of the molecule. A polynuclear compound, heterocyclic rings or cyclic ions may be aromatic
if these have a specific electronic structure.
The main essentials for aromaticity are :
1. Delocalisation. The molecule should contain a cyclic cloud of delocalized m electrons above and below
the plane of the molecule,
2. Planarity. For the delocalisation of m-electrons the ring must be planar to allow cyclic overlap of
p-orbitals. Therefore, for a molecule to be aromatic, the ring must be planar.
3. (4n + 2) x electrons. For aromaticity, the m-electron cloud must contain a total of (4n + 2) x electrons
where n is an integer equal to 0, 1, 2, 3 ........... This is also known as Huckel rule.
In a nutshell, for aromaticity, the molecule must be planar, cyclic system having delocalized
(4n + 2)n electrons.
This is known as Huckel rule.
Thus, according to Huckel rule, the aromatic compounds have delocalized electron cloud of 1 electrons of
2 or 6 or 10 or 14 electrons.
For example, benzene (67 electrons), naphthalene (107 electrons) and anthracene (147 electrons) are
aromatic compounds. 3eS

Benzene Naphthalene Anthracene Cyclooctatetraenyl dianion


6 t electrons 10 x electrons 14 7 electrons (Aromatic : planar, 107
(n =1) (n = 2) (n = 3) electrons, n = 2)

Similarly, cyclopropenyl cation, cyclopentadienyl anion and cycloheptatrieny! cation (tropylium ion) are also
aromatic because these contain 67 electrons.

H H
H

H =) H

Cyclopropenyl cation Cyclopentadienyl anion Cycloheptatrienyl cation


(Aromatic : Planar, 27 electrons, n = 0) (Aromatic : 67 electrons, n = 1) (Tropylium cation)
(Aromatic : Planar, 67 electrons, n = 1)

v 0 0 @
Heterocyclic compounds such as pyrrole, furan, thiophene and pyridine also behave as aromatic because all
are planar and have a cyclic system of 6n-electrons which are delocalised over the ring.

H Furan Thiophene Pyridine


Pyrrole

WWW.JEEBOOKS.IN
— 13/80 MODERN'S abc + OF CHEMISTRY-XI
However, the following molecules or ions do not satisfy Huckel rule and are not aromatic.
H

: @
H

Cyclopentadiene Cyclopentadienyl cation Cyclooctatetraene Cyclopropenyl! anion


4r electrons 4n electrons 8x electrons 4n electrons
H H

Cycloheptatriene contains 61 Cycloheptatrienyl anion


electrons but 7 electrons are not 8x electrons
spread over the entire ring

SOLVED EXAMPLES
lL} Example 25.
Predict which of the following systems would be aromatic ?

ofl # Of) «GB iw


Solution: Structures (ii) and (iv) are aromatic because they obey Huckel rule. Others are not aromatic because
(i) contains 4 electrons (iii) contains 6x electrons but is not conjugated system due to a sp® carbon.
lL} Example 26.
Is the following molecule aromatic or not ?

Solution: [t is not aromatic because it is non-polar and contains 87-electrons.


PREPARATION OF BENZENE AND ITS HOMOLOGUES
Benzene and its homologues can be prepared by the following methods:
1. From alkynes. Acetylene and other alkynes polymerise at high temperatures to give benzene and other
arenes. For example, benzene was first synthesized by Berthelot by passing acetylene through red hot iron tube.
CH CH

po CH er ~*~

Q ¢ _
Red hot tube | |
Hc CH 873 K, Catalyst Hc CH

CH HC
Benzene

Red hot tube


or snc =CH ——
ok Gui CF

Benzene
Similarly,

SCH C=O a
__ 873K,
K
By.
1, 3, Secatebe we
(Mesitylene)
2. Decarboxylation of aromatic acids. Benzene is prepared in the laboratory by heating sodium benzoate
with soda lime.

WWW.JEEBOOKS.IN
HYDROCARBONS 13/81 >

COONa

+ NaOH — Heat OC} + Na.COo


ay 3

Sodium benzoate Benzene

This reaction is called decarboxylation reaction.


3. Reduction of phenol. Benzene can be prepared by reduction of phenol by passing its vapours over
heated zinc dust. H

Zn, dust (7
oe + ZnO
Heat
Phenol Benzene
4. From diazonium salts. Benzene can be prepared from benzene diazonium chloride by reduction with
hypophosphorus acid (H,PO,) in the presence of Cu(I) ions.
N,*Cr
CF + H,O + H,PO, ——-» + N, + HCl + H,PO,

Benzene diazonium Benzene


chloride
5. From aryl halides. Benzene can be prepared from chlorobenzene by reduction with Ni—AlI alloy in the
presence of sodium hydroxide.
Cl
Ni—Al alloy
+ 2 |H] in + HCl

Chlorobenzene Benzene
Arenes can also be obtained by the action of sodium metal on a mixture of aryl halide and alkyl halide in ether.
CH,CH,

+2Na + CH,CH,Br _Dryether_, + 2NaBr

Bromobenzene Ethylbenzene
This reaction is called Wurtz—Fittig reaction.
6. From benzene sulphonic acid. Benzene can be prepared by heating benzene sulphonic acid with
super heated steam.
Cy F Pa 423-473
2 K , Cr H,SO,

Benzene sulphonic acid Steam Benzene


7. From Friedel Crafts reaction. Arenes can be obtained from benzene by treating with alkyl halide in
the presence of anhydrous aluminium chloride.
CH,
+CH,Cl “> C) + HCl
Benzene Toluene
8. From Grignard’s reagent. Arenes can be prepared by reacting aromatic Grignard reagent with alkyl
halide.
CH CH
So
gBr H

+ CH,—CH—Br—=_, + MgBr,
Phenyl magnesium lavptonsibenzaéne
bromide Isomboyl (Cumene)
(Grignard reagent) bromide
WWW.JEEBOOKS.IN
| 43782 MODERN'S abc + OF CHEMISTRY
-XI

PROPERTIES OF BENZENE AND ITS HOMOLOGUES


Physical Properties
(4) Benzene and its homologues containing upto eight carbon atoms are colourless liquids while the
higher ones are solids with characteristic smell.
(ii) Aromatic hydrocarbons are immisible with water but are soluble in organic solvents.
(iii) They are inflammable and burn with sooty flame.
(iv) The melting and boiling points of aromatic hydrocarbons increase with increasing molecular mass.
This is due to increase in magnitude of van der Waals’ forces of attraction with increase in molecular
size. Amongst isomeric arenes, (i.e. o-, m- and p- xylenes), the p- isomer has the highest melting
point because it is most symmetrical.
(v) They are toxic and carcinogenic in nature.
Chemical Properties
Arenes are expected to be quite reactive due to the presence of double bonds. However, benzene is quite
stable and does not undergo common addition reactions undergone by alkenes. For example, benzene
(1) does not decolourise bromine in CCl,
(tt) does not decolourise alkaline KMnO, solution.

Br,, CCl,
No reaction
Benzene
No reaction
alk KMnO,

The stability of benzene is due to resonance. However, benzene and other arenes undergo following types
of reactions :
1. Substitution reactions
2. Addition reactions
3. Oxidation reactions.
1. Substitution reactions. In these reactions, one or more hydrogen atoms in the benzene ring get
replaced by other atoms or groups. For example :
(a) Halogenation. The replacement of a hydrogen atom in the ring of an arene by a halogen atom
(F, Cl, Br or I) is called halogenation. The halogenation takes place in the presence of a Lewis acid such as
ferric halide or aluminium halide which acts as halogen carrier. The chlorination and bromination occur at a
temperature between 310 to 320 K.
Cl

Anhyd. AICI,
+ , Gl, 310-320K + HCl
Benzene Chlorobenzene

Br

Anhyd. FeBrg
+ Bry 310-320 K || + HBr
ee

=
Benzene Bromobenzene

The function of Lewis acids like AICl,, FeCl, or FeBr, is to carry the halogen to aromatic hydrocarbons.
Therefore, they are also called halogen carriers.
Reaction with iodine is reversible because HI formed reduces iodo derivative back to benzene. Therefore,
the reaction is carried out in the presence of an oxidising agent such as iodic acid (HIO,) or nitric acid (HNO,)
or mercuric oxide (HgQ).
|

a: tS Ci + HI
Benzene lodobenzene
HYDROCARBONS 13/83 >»

SHI + HIO, —“>3H,O + 31,


6HI + 2HNO, —#%! > 4H,O + 2NO + 31,
2H1 + HgO ——> Hgl, + H,O
Fluorination is very fast and it is very difficult to control the reaction.
(6) Sulphonation. The replacement of a hydrogen atom in the ring of an arene by a sulphonic acid
(— 5O,H) group is called sulphonation. It is carried out by heating benzene with fuming sulphuric acid or
oleum (concentrated sulphuric acid containing dissolved sulphur trioxide). For example,
SO,H
SO, |
+ H,sO, 330K
K + H,O

Benzene (Fuming) Benzenesulphonic acid


Sulphonation can also be carried by treating benzene with chlorosulphonic acid. For example,
S0,H

+ CISO,H ——____. ? + HCl


Benzené Chlorosulphonic acid Benzenesulphonic acid
(c) Nitration. The replacement of a hydrogen atom in the ring of an arene by a nitro (—NO,) group is
called nitration. It is carried out by heating benzene with the nitrating mixture consisting of concentrated
nitric acid and sulphuric acid to about 330 K.
NO 2

Ol nh.
Conc. HeS0
a + H,O
Benzene (Conc.) Nitrobenzene
(d) Alkylation or Friedel-Crafts alkylation reaction. On treatment with an alkyl halide in the presence
of anhydrous aluminium chloride, benzene forms an alkyl] benzene. The reaction is also called Friedel-Crafts
alkylation reaction. In this reaction, a hydrogen atom in the benzene ring is replaced by alkyl group.
For example,

CH,

+ coc / Qe + HCl
Benzene Methyl chloride Toluene
C,H,

O + e 9H,Br Anhydrous AICls + HBr

Benzene Ethyl bromide Ethyl benzene


(e) Acylation or Friedel Crafts acylation reaction. On treatment with an acyl halide or acid anhydride
in the presence of anhydrous AICl, (Lewis acid), benzene forms acyl benzene. This reaction is called Friedel
Crafts acylation reaction. For example,
COCH,

+ CH,coc =, + HBr
; 3
= Ethanoyl! chloride
Benzene (Acetyl chloride) OC
COCH,
CH,CO Fig, Anhyd. AIC]| |
+ YP nee + CH,COOH
: CH,CO :
Benzene Ethanoic anhydride Acetophenone
(Acetic anhydride)
— 13/84 MODERN'S abc + OF CHEMISTRY-XI
It may be noted that during electrophilic substitution reactions, if excess of electrophilic reagent is used,
further substitution reaction may occur in which other hydrogen atoms of benzene ring may also be successively
replaced by electrophile. For example, benzene on treatment with excess of chlorine in the presence of anhydrous
AICI, in dark gives hexachlorobenzene (C,C1,).
Cl
Cl
+ 6HCI Anhyd. AlCl, |
dark, cold
Cl | Cl
Cl
Hexachloro benzene
(C,Cl,)

2. Addition Reactions
Benzene and its homologous also undergo some addition reactions characteristics of alkenes and alkynes.
However, the reactions occur under more drastic conditions (higher temperature and pressure). This is due to
extra stability of the delocalised n-electrons. For example,
(a) Addition of hydrogen (catalytic hydrogenation). Benzene reacts with hydrogen in the presence
of a catalyst such as nickel or platinum at 473 to 573 K under pressure to form cyclohexane.

4 3H, Ni catalyst
A73-—573 K
Benzene Cyclohexane
(b) Addition of halogens. Benzene reacts with chlorine or bromine in the presence of sunlight and
absence of halogen carrier to form benzene hexachloride (BHC).
if

| ae Cl—H
_7 AY,
H—Cl
+30, —%Z
, Cl—-HC_ HO
Benzene \-

Cl
(Benzene hexachloride)
(BHC)
BHC is used as an insecticide. It is sold under the name Gammexane or lindane.
(c) Addition of ozone, Benzene reacts with ozone to form benzene triozonide which is decomposed by
zinc dust and water to form glyoxal (ethanedial).

HC O
Ho~ \, |
/\ O—CH CHO
CHoCl O O | __2n,H20_, 3 |
4 ES OOH Reeetive
cleavage ”” CHO
HCL / \ Ethanedial
Benzene Ozone HC. PP (Glyoxal)
O
Benzene triozonide
3. Oxidation Reactions
Arenes are quite stable and are not readily oxidised. Some of their oxidation reactions are:
(a) Combustion. On completely burning with oxygen, benzene gives carbon dioxide and water with the
evolution of a large amount of energy.
2C,H,+ 150, ——~ 12CO, + 6H,0; AH =— 6530 kJ

WWW.JEEBOOKS.IN
HYDROCARBONS

(6) Oxidation of benzene. Benzene on oxidation in the presence of vanadium pentoxide (V,O,) at 773 K gives
maleic acid and maleic anhydride.

¢ ) CHCOOH CHCO
+ 26, —¥205__, | Heat | ~*
9 T73 K . Hoo ee
| CHCOOH 2 CHCO~-
Benzene Maleic acid Maleic anhydride
(c) Oxidation of alkyl side chain. The alkyl side chain in arene can be oxidised by the following
methods :
(z) With hot acidic KMnO, or Na,Cr,O, or conc. HNO, the side chain is oxidised to carboxylic acid group.
CH, COOH

_ BMnO,,,H,0
50
sae
383 K
Benzoic acid
COOH
CH,CH,

Oxidation
Ss KMn0O,, 3873-383 K

Benzoic acid
Xylenes on oxidation give dicarboxylic acids.
CH, COOH
CH, OOH
Oxidation
KMn0O,, 3873-383 K

o-Xylene Phthalic acid


CH, COOH

Oxidation
KMnO,, 373-383 K |
|
CH, COOH
pmvlcse Terephthalic acid

It may be noted that the side alkyl chain is oxidised to —COOH group irrespective of its length
(—CH,, —CH,CH, or —CH,CH,CH,).
(ti) With mild oxidising agents such as acidic manganese dioxide (MnO,) or chromyl chloride (CrO,Cl,),
the side chain is oxidised to aldehyde group. This reaction is called Etard’s reaction.
CHO

ice
Oxidation

MnO», H*
or CrO,Cl, Benzaldehyde
Thus,
COOH

KMnO,
315-355 K

Benzoic acid
____ Oxidation _, CHO

Mn0O,, Ht
or CrO,Cl,

Benzaldehyde
13/86 MODERN'S abc + OF CHEMISTRY-XI

ll Micehaiisia ; Electrophilic substitution reactions of Benzene


The orbital structure of benzene makes it quite clear that the m-electron cloud lying above and below the

plane of the benzene ring is held loosely and is thus, available to the electron seeking reagents i.e.,electrophiles.
Therefore, the substitution reactions in benzene are electrophilic in nature. These reactions are characteristic
not only of benzene ring itself but also of the compounds where benzene ring is present. The various electrophilic
substitution reactions of benzene follow the same path. The electrophilic substitution reactions proceed via the
following three steps :
1. Generation of the electrophile
2. Formation of carbocation intermediate
3. Removal of proton from the carbocation intermediate to form the product.
Step 1. Generation of the electrophile
The attacking reagent may not be a strong electrophile. Therefore, first of all an electrophile is generated
by some preliminary reaction.

E th u —— gE 4 : Nu
Reagent Electrophile Nucleophile
For example, during chlorination of benzene, an electrophile (Cl”) is generated by reacting with anhydrous
AICI, used as catalyst.
Cl, + AICI, ——_ Cl” + AIC],
Chloronium ion

Similarly, during alkylation of benzene, electrolphile is generated as :


CH,—Cl + AICl —— CH,’ +AlCl,
Electrophile
Step 2. Formation of carbocation, (arenium) intermediate.
The electrophile E* attacks the m—electron cloud of the aromatic ring and forms a bond with carbon,
creating a positive charge on the ring. This results in the formation of a sigma complex or a carbocation (called
arenium ion)
H E

f +
+ KEK —->

Sigma complex
(arenium ion)
The arenium ion gets stabilized by resonance
H E H E
E + Pi
—— —-> —_——

Resonance stabilization of carbocation (arenium) intermediate.


The resulting carbocation has three important contributing structures which spread the positive charge
over the remaining carbon atoms, although more on the carbon atoms that are ortho and para to the position
attacked by the electrophile.
It may be noted that during the formation of sigma complex (arenium ion), the aromatic character of benzene
ring is lost. This is because in the carbocation, one of the carbon is sp® hybridised and delocalisation of electrons stops
at sp® hybridised carbon. Therefore this step is slow and hence is the rate determining step of the reaction.
| sp® hybridised
carbon

Step 3. Removal of proton


The carbocation formed loses a proton to the nucleophile (Nu~) present in the reaction mixture to form a
substitution product. During this step, the aromatic character of the benzene ring is restored and this step is fast.

WWW.JEEBOOKS.IN
HYDROCARBONS 13/87 =>

The loss of proton allows the two electrons from the carbon—hydrogen bond to move to regenerate the
aromatic ring and thus restoring the aromatic character.

fast

Monosubstituted
benzene
Thus, the electrophilic substitution in benzene involves addition followed by elimination and results in the
desired substituted product.
Role of Catalysts in Electrophilic Aromatic Substitution
In almost all the electrophilic aromatic substitution reactions of benzene, a catalyst is generally required
to generate a powerful electrophile i.e. the attacking reagent. The catalysts are usually Lewis acids or protonic
acids, For example, chlorination of benzene requires the presence of a Lewis acid catalyst, usually ferric chloride.
Ferric chloride being electron deficient in nature takes up a Cl ion from chlorine molecule to make it, polar
(Cl* — Cl*) and to generate a highly reactive electrophile i.e., chloronium ion (Cl*) as follows :
Cl Cl
+6 =-6
CI—Cl + Fe—Cl ——> Cl* + | Cl— Fe —C]
Chloronium ion
Cl (Electrophile) Cl
Lewis acid Complex ion

In the alkylation and acylation reactions of benzene, the catalyst employed is anhydrous aluminium chloride
which is also a Lewis acid . Similarly, sulphuric acid acts as a catalyst in the nitration and sulphonation.
It is interesting to note that no catalyst is required in the electrophilic addition reactions of alkenes.
In fact, there is no delocalisation of the 1-electron charge in alkenes. The t-electron cloud can easily cause
the polarisation of the attacking molecule to generate the electrophile. However, in benzene and other aromatic
compounds, there is delocalisation of electron charge . The electron clouds are incapable of causing any
polarisation of the attacking reagent. Hence, a catalyst i.e., a Lewis acid or protonic acid is always required
to generate the electrophile.
The mechanism of some important mono-substitution reactions of benzene are discussed below :
(1) Mechanism of Nitration of Benzene
In the nitration of benzene, a hydrogen atom of benzene is substituted by a nitro (NO,) group.
The attacking reagent is the nitronium ton (NO,*), whose formation is facilitated by the presence of
sulphuric acid (catalyst). The mechanism is illustrated by the following steps :
1. The electrophile nitronium ion is generated as follows :
H,sOo, — HH + HSO,

H* + H—O—NY — H-O-— \
7a +
Nitric acid oO 2 —)
Protonated nitric acid

a
H,0+O= N=0
Nitronium ion
or for simplicity (Electrophile)

H-O-NO, + H,SO, = H-O-NO, + HSO,


H
Protonated nitric acid

H-O: + NO,*
Nitronium ion
(Electrophile)

WWW.JEEBOOKS.IN
_— 13/88 MODERN'S abc + OF CHEMISTRY-XI

2. The electrophile attacks the benzene ring to form an intermediate carbocation.


H NO, H NO 9 H NO, H NO,

NO: + F

Nitrobenzene
(2) Mechanism of Sulphonation of Benzene
In the sulphonation of benzene, a hydrogen atom attached to the carbon atom of the ring gets substituted
by a sulphonic acid group (-SO,H). The attacking reagent is sulphur trioxide, SO, (neutral electron deficient
species). It can be formed by the dissociation of sulphuric acid.
The sulphonation of benzene follows the following steps :
1. The electrophile SO, is generated as follows :
2H,5O, —— H,0* + HSO, +50,
Electrophile
Sulphur atom in sulphur trioxide has only sextet of electrons. Therefore, the molecule is electron deficient
in nature and acts as an electrophile.
2. The electrophile attacks the benzene ring to form an intermediate carbocation.
H so, H so, H $0,Fa H 50,"
SO3 - +
: |
3. The intermediate carbocation loses a H* to HSO,- ion to form benzene sulphonate ion. This can take up
a proton from H,O0* ion to form benzene sulphonic acid.
SO,

+ H,s0,

SO.H

H,O* —— -- H,O

Benzene
sulphonic acid
(3) Mechanism of Halogenation of Benzene
The chlorination or bromination of benzene is carried out by treating with chlorine or bromine in the
presence of ferric salts t.e., FeCl, or FeBr,. The metal catalyst being a Lewis acid (Fe is electron deficient)
polarises the halogen molecule and thus, generates an electrophile . The mechanism is being illustrated with
the help of chlorination of benzene.
1, The electrophile (chloronium ion) is generated as follows :
6 &
Cl—Cl+ FeCl, M———> FeCl, + Cl’.
Chloronium ion
It has been suggested that free chloronium ion may not have actual existence. The electrophile is
supposed to be made available from a complex between FeCl, and Cl,.
]
—6 +06
a ee
Cl

WWW.JEEBOOKS.IN
HYDROCARBONS 13/89 >

2. The electrophile attacks the benzene ring to form an intermediate carbocation.

H Cl H (Cl H Cl

8-0
3. The intermediate carbocation loses a seston to FeCl, ion and results in the formation of a chloro
compound.

+ FeCl, —— + HCl +/ ¥Fe€l,

Chlorobenzene
(4) Mechanism of Friedel—Craft Alkylation of Benzene
In the alkylation of benzene, one hydrogen atom attached to the carbon atom of the ring gets replaced by
alkyl (-R ) group. The attacking electrophile is an alkyl carbonium ion (Rt) made available by alkyl halide
molecule. The release of electrophile is facilitated by the presence of anhydrous AIC1,. The mechanism is being
illustrated with the help of methyl chloride, CH,Cl.
1. The electrophile (methyl carbocation) is generated as follows :
_ +
H,C —Cl + AICI, eT AICI, +, CH,
Methyl carbocation
2. The electrophile attacks the benzene ring to form an intermediate carbocation.

= Cs og.O|-
7 H CH, H CH, H CH, H CH,

3. The intermediate carbocation loses a Boon to AICI, ion to give alkylated eS)
“1

fox:
.

Hy} CH, a
ae: 7

Dy+/Alcl-= —3 + HCl + AICl,


Toluene

Note. When the alkylation is i carried with higher alkyl halides such as n-propyl chloride, the electrophile
formed in the first step is a primary carbocation (CH, —CH,=— CH.) Being extremely unstable, it changes to
a more stable secondary carbocation by the ecameneemeut reaction.

ae Rearrangement +
CHA H, 1,2-hydride sift CH,—-CH-CH,

Primary carbocation (1°) Secondary carbocation (2°)


Thus, the electrophile is (CH,),CH* ion and the product formed is 2-phenyl propane and not
1-phenyl propane.

ee
H,—-CH,-CH, CH-CH,

1-Phenyl propane 2-Phenyl propane


(n-propyl benzene) (isopropyl benzene)
(not possible) (possible)
_— 13/90 MODERN'S abc + OF CHEMISTRY
-XI

(5) Mechanism of Friedel Crafts Acylation


In acylation of benzene, the hydrogen attached to the carbon atom of the ring gets replaced by acyl
(RCO-) group. The attacking electrophile ; an acyl carbocation (RC QO) is supplied by acid chloride (RCOCI) in
the presence of anhydrous AIC1,. The mechanism is illustrated with the help of acetyl chloride, CH,COCL
1, The electrophile (acetyl carbocation) is generated as follows :
O |

H,C—C+Cl +AlCl,
| —— > AIC], + cu,—t ©
1
Acetyl chloride | Acetyl carbonium ion
2. The electrophile attacks the benzene ring to form an intermediate carbocation.

I
C—CH a

+HCl + AICL,
Acetophenone
The reactions of benzene are summed up below.
a
Cl, FeCl,
- - Chlorobenzene

Br

a aia ae < Bromobenzene

CH,
: CH,Cl, AICI, : Tees

NO,
_HNO;, HOR < Nitrobenzene

SO,H

ie CO} Benzene sulphonic acid

CH,
CH,Cl, AICI,
Friedel Craft alkylation Toluene

CO
CH,COCI, AICI, “
Acetophenone
Friedel Craft acylation

O
2 CO, + H,O Carbon dioxide
Combustion

V,0, CHCOOH

770 ik CHCOOH
HYDROCARBONS 13/91 =>

DIRECTIVE INFLUENCE OF SUBSTITUENTS


AND THEIR EFFECT ON REACTIVITY
As we have learnt, all the six hydrogen atoms of benzene ring are equivalent. Therefore, replacement of
any one of these six hydrogen atoms by any substitution always forms a single mono substituted product.
However, when monosubstituted benzene is subjected to further substitution, the group present on the benzene
ring affects the incoming attacking groups.
The ability of a group already present in the benzene ring to direct the incoming group to a
particular position is called the directive influence of groups.
A substituent already present on the ring has two effects :
(i) Orientation effect - Substituents affect the orientation of the reaction. The three possible disubstituted
products t.e ortho, meta and para are not formed in equal amounts. The nature of the substituent already
present on the benzene ring determines the position of the second substitution. Some groups direct substitution
primarily to the ortho and para positions, while other groups direct substitution primarily to the meta position.
In this way, it has been found that every group can be put into one of the two classes : ortho and para director
or meta dtrector.
(zt) Reactivity — The substituents affect the reactivity of the aromatic ring. Some substituents activate
the ring and therefore make it more reactive than benzene. On the other hand, some groups deactivate the
ring and make it less reactive than benzene. For example, in aromatic nitration, the -OH group makes the ring
more reactive than benzene, whereas a —NO, substituent makes the ring less reactive.
The substituents can be classified into three groups :
1. Ortho and para-directing activators. These groups release electrons and activate the benzene ring.
These groups direct the incoming groups to ortho and para positions. The common examples are :
—OH, —OCH,,—OCOCH., —NH,,—NCHCH.,,—NHCOCH., —CH,, etc.
For example,
OH
OH
JH
NO, +
Nitration

(— NO, group)

ortho- he
para- o, p-directors

2. Meta directing deactivating groups. These groups withdraw electrons from the benzene ring and
deactivate it. These groups direct the incoming group to meta positions. The common examples are :
—-NO,, -CN, -CHO, -COOH, -SO,H, -COCH,,.
For example,
CN N

Nitration
ee
| (— NO, group)

meta- m-directors

3. Ortho and para directing deactivating groups. These groups withdraw electrons from the benzene
ring and deactivate it. However, these direct the incoming group to ortho and para positions. The common
examples are :
Halogens : -F, Cl.,Br, I.
For example, Cl

Nitration

(— NO, group)
ae 13/92 MODERN'S abc + OF CHEMISTRY-XI

Table 2. Effect of groups on electrophilic substitution reactions.

Ortho and para directing Meta directing Ortho and para directing
activators deactivators deactivators

~ NH, —NO, -F
— OH —CN -Cl
— NHR, —-NR, — N(CH,),* —Br
— OCH,, —C,H, — COOH =]
— NHCOCH, -S0,H
— CH, — CHO
— C,H, — COR
There is no meta directing activator group.

Explanation for different directive influence


Ortho and para directing groups. As an example, let us discuss the directive influence of —OH (phenolic)
sroup. The resonance structures of phenol are shown below :

+ O-H +O-H O.

I =
Resonance pay A. of ” hon
The resonance structures of phenol show that the overall electron density on the benzene ring increases
in comparison to benzene. Therefore, it is an activating group. The typical reactions of aromatic hydrocarbons
are electrophilic substitution reactions in which an electrophile attacks the benzene ring. The structures II, III
and IV show that the electron density is high at ortho and para positions and therefore, the electrophile will
preferentially attack ortho and para positions. Thus, —OH group is o- and p-directing group. Similarly, other
groups such as —NH,, —NR,, -OCH,, —CH,, etc. are ortho and para directing groups.
Meta directing groups. As an example, let us discuss the directive influence of nitro (-NO,) group. The
resonance structures of nitrobenzene are shown below :

Ke OO

VI Vil
_O- &- o
Resonance structures of nitrobenzene
It is clear from the resonance structures that the overall electron density on the benzene ring decreases
and therefore, this group is deactivating group. Moreover, the electron density on the o- and p-positions is
less than on m-position. Therefore, the two meta positions are rich in electron density, thus resulting m-
substitution. Other groups such as -COOH, —SO.H, — CN, -CHO and —COR are meta directing groups.
Directive influence of halogens. The halogens are highly deactivating because of their strong negative
inductive effect (—I effect). Therefore, the overall electron density on the benzene ring decreases. However, due
to resonance the electron density on o- and p- positions is more as shown below and therefore, these are o-and
p-directing. The resonance structures of chlorobenzene are shown below :

@ I: +Cl: *Cl: Cl
@'
—? *» oe C¢ OH i

XI Ls XIII V XV
Resonance structures of chlorobenzene
HYDROCARBONS 13/93 =>

Reactivity and orientation in electrophilic substitution can be explained by the combined effects of
inductive effect and resonance effect.
Orientation in Disubstituted Benzene
The presence of two substituents on the benzene ring makes the problem of orientation more difficult.
This can be decided on the basis of three rules :
1. If the directing effects of two substituents reinforce, then a single product is formed. For example, in
p-nitrotoluene, both the methyl and the nitro group direct further substitution to the same position (ortho to
methyl and meta to nitro)
we oup directs
| H,
3h A No,
Nitration

(— NO, group)

NO, NO,
eaeily )
directs
2. If the directing effect of the two groups oppose each other, strongly activating groups win over deactivating
or weakly activating groups. The sequence of directing power is
-NH, >—-OH >-—OCH, > —- NHCOCH, > —C,H, > —CH, > meta directors.
For example, during bromination of p—methyl phenol, -OH group is more powerful activator and therefore,
controls the product.

direc, A directs
Bro
SoC e.g.,
(FeBrs)

rae CH,
Hie Hs

3. There is normally little substitution when the two groups are meta to each other. Aromatic rings with
three adjacent substituents are generally prepared by some other routes.

CH, Too hindered


A position

Cl

POLYNUCLEAR HYDROCARBONS
Polynuclear hydrocarbons contain two or more benzene rings fused together. These are also called
polycyclic aromatic compounds. For example, naphthalene contains two benzene rings fused together.
Other common examples are anthracene, phenanthracene, etc.

Naphthalene Anthracene Phenanthracene

The major source of these polynuclear hydrocarbons is coal tar.


Positions in the naphthalene ring are indicated by numbers as :
8 1 Ol ol
Vo 2 B B
6 3 _ b B
a A CL ed

Two isomeric monosubstituted naphthalenes are differentiated by prefixes 1- and 2- or of - and B-


ae 13/94 MODERN'S abc + OF CHEMISTRY
-XI

OH

CFOs
OH

a-Naphthol $-Naphthol
or 1-Naphthol or 2-Naphthol
For highly substituted naphthalenes, the positions of the groups are indicated by numbers :

Or

1, 5-Dinitronaphthalene 2, 4-Dinitro-1-naphthylamine
Naphthalene and other polynuclear aromatic hydrocarbons show many of the chemical properties similar
to benzene.
The volatile arenes are highly inflammable and burn with a yellow sooty flame. Many of these polynuclear
aromatic hydrocarbons are toxic and some are carcinogenic.
Carcinogenicity and Toxicity of Polynuclear Hydrocarbons
Most polynuclear hydrocarbons are carcinogenic in nature. It has been observed that the people who
work under prolonged exposure to coal tar tend to suffer from skin cancer. A group of British chemists have
studied that high boiling fluorescent fraction of coal tar produces cancer. The major component of this fraction
was found to be 1, 2-benzanthracene.

1, 2-Benzanthracene
Later researches have shown that many other polynuclear hydrocarbons such as 3-methylcholanthracene,
1,2-benzpyrene, 9, 10-dimethyl-1,2-benzanthracene, etc. are carcinogenic in nature.

1, 2-Benzpyrene

9, 10-Dimethyl-1, 2-benzanthracene 1, 2, 5, 6-Dibenzanthracene


It may be noted that there is no general rule by which one can predict the carcinogenic activity of
hydrocarbons or their derivatives. However, the number and position of some groups such as —-CH., —OH,
—CN, —OCH.,, etc. have been found to influence the carcinogenic activity of polynuclear hydrocarbons.
Such polynuclear hydrocarbons are formed by incomplete combustion of organic materials such as coal,
petroleum, tobacco, etc. When these polynuclear hydrocarbons enter into human body, they undergo various
biochemical reactions and finally damage DNA and ultimately leads to cancer.
HYDROCARBONS

: SOLVED EXAMPLES

L} Example 27.
How will you convert ethanoic acid into benzene ?.
Solution:

CH,COOH—“*"_5 CH,COONa— 49> CH, —2"> CH,CI


Ethanoic acid Kcthanie Chicwonentails
Na |Wurtz
dry ether | reaction
CH,Br
¢
8 CH= CH SeEOE 1 Se Cn cH,
CH.Br Ethane
alc
KOH

CH,=CHBr—"2
5HC = CH— Redhat
Ethyne ron tube
_,
873K Benzene
(} Example 28.
How will you convert benzene into oo
(i) p-nitrobromobenzene (ii) m—Nitrochlorobenzene (iii) p—nitrotoluene (iv) acetophenone
Solution:

Oe Os GO
Br Br Br

Benzene (Minor product)


_ NO ;
Separation by (major*product)
fractional distillation
Br

NO 2
PON

(iz) __/Gone.HNOs,
/, Clg, anhyd AIClg
yoH»S04 Heat
Cl
m—Nitrochlorobenzene

CH, CH,
CH,

i wf CHSC] _, |
(uit) . anhyd AICl3 Ci +

Separation by N
fractional distillation | O»
CH,

NO,
p-N itrotoluene
a 13/96 MODERN'S abc + OF CHEMISTRY-XI

COCH,

wo CD ee O)
Acetophenone

@ 30. Complete the reaction :


(a ss AO ee
@ 31. Complete the reaction :
MnO,, Ht KMn0O,, H" _»
[5 fe Sosa LEM 8S3 41 wt A
@ 32. What is difference between alkylation and acylation ?
@ 33. What is halogen carrier ? Give one example.
@ 34, What is Huckel rule ?
@ 35. Which of the following are meta directors ?
— NO,, — SO,H, — Cl, - OH, - NH,, - CHO
036. Name an ortho-and para-directing deactivating group.

Answers to Practice Problems


© 30. Benzene triozonide, glyoxal
© 31. A=C,H,COOH, B = C,H,.CHO
© 33. FeCl, in halogenation of benzene
© 35. -NO,, —SO,H, -CHO
© 36. -— Cl

VHS
|.
J
|
Ct —s— SOLVED EXAMPLES

0) Example 29.

Calculate the percentages of all the monochlorinated products obtained from 2-methylbutane. The relative
reactivity of 1°, 2° and 3° hydrogen towards chlorination is 1 : 3.8 : 5.
(NCERT Exemplar Problem)
Solution: 2— Methylbutane is
ib 3° B® i
hes
CH, — CH — CH, — CH,
HH,

Since there are nine 1° H atoms (CH, groups) in the molecule, there are nine possibilities of compound A.

Cl — CH, — CH — CH, — CH, (A)

CH,
There are two 2° H atoms (one CH,) in the molecule and therefore, there are two possibilities of compound B.

CH, — CH — CH — CH, (B)

CH, 7
There is only one 3° H atom (one CH hydrogen) and therefore, there is only one possibility of compound C.
Cl

CH, — é — CH, - CH, (C)


CH,
WWW.JEEBOOKS.IN
HYDROCARBONS 13/97 ~~»

Relative amounts of A, B and C = No. of H atoms in molecule x Relative reactivity

A(1°) B(2°) C(3°)


Relative amounts 9x1=9 2x 3.8 = 7.6 1x5=5

Total amount of monohalogenated compounds = 9 + 7.6 + 5 = 21.6

9
Percentage of A =s16 * 100 = 41.7%

7.6
Percentage of B =316 * 100 = 35.2%

D
Percentage ofC = 316 * 100 = 23.1 %

(} Example 30.

How will you distinguish pent-1-ene from n-pentane ?


Solution: Pent-l-ene and n-pentane can be distinguished by the following tests :
1. Reaction with Br, in CCI,. Pent-1-ene will decolourise the brown colour of the solution of Br, in CCl,. But
n-pentane will not decolourise the colour of bromine at room temperature.
CH,CH,CH,CH =CH, + Br, —Ch , CH,CH,CH,CH — CH,
Pent-1l-ene Brown
Br Br
Colourless

CH,CH,CH,CH,CH, + Br, —C4 ,. No reaction


n-Pentane Brown
2. Reaction with cold dilute KMnO, (Baeyer’s test). Pent-l-ene will react with Baeyer’s reagent and its colour
will be discharged. n-Pentane does not react.
CH,CH,CH,CH = CH, —2#= "29 _, CH,CH,CH,CH — CH,
Pent-1l-ene || OH
OH

Colourless
CH,CH,CH,CH,CH, —™= "9° _, No reaction
n-Pentane
(J Example 31.
Complete the following reactions :
; , lc. KOH HBr
(i) Isopropyl bromide a Jae eee:
Propy Heat Peroxide

(ii) n-Propyl alcohol... ~ere-H28Q_, 4 —On48 , p


443 K Heat

(iii) 1, 1, 2, 2-tetrachloroethane —Z™ohe!_, 4 —trontube. p


Heat 675 K

(iv) Acetylene —NONHe , ~, _CHCH,Br , p


— H,, Pd,
Ot BaSO, l*F a()O
Ce Quinoline (ii) Zn, H,O
Solution: HBr
. = ,
(1) CH,CH CH, ee CH,CH = CH, peveutie CH,CH,CH,Br
(A) (B)
Br Propene 1-Bromopropane
Isopropyl bromide
. aol conc.H4S0,4 | O,, Ag |
(uw) CH,CH,CH,OH —"7BkK CH,CH = CH, Heat ? CH,CH — CH,

n-Propyl alcohol (A) O


Propene (B)

Propylene oxide

WWW.JEEBOOKS.IN
a 13/98 MODERN'S abc + OF CHEMISTRY
-XI

Cl Cl
_ i L #n, alcohol Iron tube
cic | | tet
eat OE
(A) ee
0

Cl Cl Ethyne (B)
1, 1, 2, 2-tetrachloroethane Benzene

(jv) Ho=cH =O § cm cNa SSB | c= CCH, CH,


Acetylene (A) (B)
Sodium acetylide But-1-yne

H,, Pd (1) O3 |
(v) CH,C =CH —— CH.CH = CH, —... ., ,” CH,CHOS + HCHO
‘ | BaSO, (ii) Zn, H,O 3
ropyne
(A) (B)
Propene Acetaldehyde Formaldehyde
(} Example 32.
An alkene on ozonolysis gives butan-2-one and 2-methylpropanal. What products will be obtained when it is
treated with hot conc. KMnO, ?
Solution: Let us first try to write the structure of alkene. The product of ozonolysis are :
CH, — CH, —C =O H CH,
||
CH, O = C — CH CH,

Butan-2-one 2-Methylpropanal
Alkene is CH, —CH, —C = CH—CH— CH,
| |
CH, CH,
With hot KMnO, it will react as : ‘4
CH, — CH, ~F = CH — CH — CH, hot KMnOq CH,CH,C = Q + CH,.CHCOOH

CH, CH, CH, CH,


Butan-2-one 2-Methylpropanonic acid

QO Example 38.
An alkyl halide CH, ,Br (A) reacts with ethanolic KOH to give an alkene ‘B’, which reacts with Br, to give
a compound ‘C’, which on dehydrobromination gives an alkyne ‘D’. On treatment with sodium metal in liquid
ammonia one mole of ‘D’ gives one mole of the sodium salt of ‘D’ and half a mole of hydrogen gas. Complete
hydrogenation of ‘D’ yields a straight chain alkane. Identify A, B, C and D. Give the reactions involved.
Solution: The reactions are : (NCERT Exemplar Problem)

Ale. KOH t, Bro dehydrobromination


C 5H,,Br —_—_ CH ——_—> C.H, Br. nea C,H,

Alkyl halide Alkene (C) Alkyne


(A) (B) (D)

: 1
C;H, ——*—3
Na,lig.NH
> C;H;Na + —GHe
(D) Sodium alkylide

The reaction suggest that (D) is a terminal alkyne. The possible structures are :
CH,CH,CH,C=CH CH,CH C = CH

CH,
(I) (II)

WWW.JEEBOOKS.IN
Since alkyne (D) gives straight chain on complete hydrogenation, therefore, only structure (I) is possible. Hence the
reactions may be written as :

CH,CH,CH,CH,CH,Br —“"*°" CH,CH,CH,CH = CH, —22 5 CH,CH,CH,CHCH,~Br


(A) (B) c BF
—alc. KOH. CH,CH,CH,C = CH “2 “"8_, CH,CH,CH,C =C Na’
(D)
and CH,CH,CH, C = CH —2-> CH,CH,CH,CH,CH,
Hence the structures of A, B, C and D are:

(A) CH,CH,CH,CH,CH,Br
(B) CH,CH,CH,CH = CH,
(C) CH,CH,CH,CH (Br) CH,Br
(D) CH,CH,CH,C = CH

QO Example 34.
An unsaturated hydrocarbon ‘A’ adds two molecules of H, and on reductive ozonolysis gives butane-1, 4-dial,
ethanal and propanone. Give the structure of ‘A’, write its IUPAC name and explain the reactions involved.
(NCERT Exemplar Problem)
Solution: Two molecules of hydrogen add on A and this means that ‘A’ is either an alkadiene or alkyne.
On reductive ozonolysis, ‘A’ gives three fragments and one of these is dialdehyde. Hence, the molecule has broken
down at two sites. Therefore ‘A’ has two double bonds. The three fragments obtained on reductive ozonolysis are :

CH,
OHC —- CH, -CH,-—CHO, CH,CHO CH,~ ~—

Butane-1,4-dial Ethanal Propanone


The structure of ‘A’ as deduced from three fragments is

ia Ns
CH,—C=O + O—=C—CH,—CH,—_C=0O + O=C.
Seg
H 3

The hydrocarbon can be written by removing oxygen atoms and writing double bonds between the carbonyl carbon
atoms:

8 7 6 5 4 3] 2 1
CH, - CH = CH —CH, - CH, —CH = C - CH,
|
CH,
IUPAC name : 2-Methylocta-2, 6-diene
Reactions are

7. laa y CAs
CH, CH=CH CH, —CH;—-Ca-—-C—CH, °3 , CH,— GH ‘CH-CH,-CH,— CH in
CH
CH, 5 —O °

CH
Zn,HoO
a, CH,-CHO + OHCCH,CH,CHO + O=_(~”
on :

WWW.JEEBOOKS.IN
a 43/100 MODERN'S abc + OF CHEMISTRY-XI

QO Example 35.
How will you convert :
(a) Ethane to butane (6b) Ethyne to methane
(c) Ethene to ethyne (d) Methane to ethane
(e) Propene to 2, 3-dimethylbutane (f) Ethane to ethyne
(g) Ethyne to but-2-yne
Solution: (a) Ethane to butane :

CHCH, “Suga?
Kthane
Cl,
CHSCHC! —“\ontereacion’ CHsCH,CH,CH,
Ethyl! chloride
2Na, ether

Butane
(b) Ethyne to methane

HC =CH H’, H,O


se, CH,CHO [0] >
——— |
CH,COOH1 NaOH
——~——» Ny,
CH,COONa
Ethyne gO, Ethanal oe as
NaOH, CaO
CH,
Methane
(c) Ethene to ethyne
Br,
H,C=CH, Gq.” CH,—CH, 2alc.KOH . CH =CH
Kthene ' | | Ethyne
Br Br
(d) Methane to ethane

cH,4
—%
Sunlight
, coa
es
==>
Wurtz reaction
cuca
gees
Methane Ethane

(e) Propene to 2, 3-dimethylbutane


CH,CH=CH,
ee
_ HBr. CH.CHC
“3 Hs
Ae es
Wurtz reaction
CHL.CH—CH—CH,
i
Propene ( i,
Br H, CH,
2, 3- iymetiiy batene

(f) Ethane to ethyne


Cl |
CH,CH, ——— cH,cHjo —*cX°F , cy,-cH, —2= > cH,—CcH,
Bihane Sunlight
Br r

|2 KOH alc.

HC =CH
Ethyne

(g) Ethyne to but-2-yne

HC=CcH —2NaNH, . NaC=CNa =—2CHBr , CH.C=CCH,


Ethyne Disodium But-2-yne
acetylide
(} Example 36.
A hydrocarbon containing two double bonds gave on reductive ozonolysis ethanal, glyoxal and propanone.
Predict the structure of the hydrocarbon and give its IUPAC name.
Solution: The products of reductive ozonolysis are (write the structures with their carbonyl groups facing one another).

WWW.JEEBOOKS.IN
HYDROCARBONS 13/101 —_—_—_

CH,C =O + = + ak
|
A H H CH,
Ethanal Glyoxal Propanone
The hydrocarbon can be written by removing oxygen atoms and writing double bonds between the carbonyl carbon
atoms:
6 5 4 3 2 1
CH,— C=C —C=C—CH
|

H
IUPAC name : 2-Methylhexa-2,4-diene
(} Example 37.
Assign structures for the following :
(a) An alkyne (X) has molecular formula C,H,. It reacts neither with sodamide nor with ammoniacal
cuprous chloride.
(6) Ahydrocarbon ‘YY decolourises bromine water. On ozonolysis it gives 3-methyl butanal and formaldehyde.
Give the name of the compound.
(c) A hydrocarbon (Z) has molecular formula CH jo. It does not decolourise bromine water and is oxidised
to benzoic acid on heating with K,Cr,O,. It can also have three other isomers A, B and C. Write the
structures of Z, A, B and C.
Solution: (a) Alkyne X is C,H,. Since it does not react with sodamide or ammoniacal cuprous chloride, the triple bond
cannot be terminal.
X = CH,CH,C =CCH, Pent-2-yne
(6) Hydrocarbon ‘Y’ is alkene because it decolourises bromine water. From the products of ozonolysis, the structure of
alkene can be predicted.

CH,CHCH,C=O + O=C—H
if —— £CH,CHCH,CH = CH,
Formaldehyde
CH, CH,
3-Methyl butanal 4-Methylpent-1l-ene
(c) Since it does not decolourise bromine water, it is arene. Its formula is
CH,CH, COOH

[O]
K,Cr,0., H*
a Benzoic acid
The other three isomers are :
CH, CH,
oF CH,

CH,
1,2-Dimethylbenzene 1, 3-Dimethylbenzene 1, 4-Dimethylbenzene
(o-Xylene) (m-Xylene) (p-Xylene)
(} Example 38.
The hydrocarbon A adds one mole of hydrogen in the presence of a platinum catalyst to form n-hexane.
When A is oxidized vigorously with KMnO,, a single carboxylic acid containing three carbon atoms is
isolated. Give the structure of A and explain.
Solution: H,
CH,CH,CH = CHCH,CH, _ CH,CH,CH,CH,CH,CH,
(A) n-Hexane

| KMn0O,
2CH,CH,COOH
Ais Hex-3-ene.

WWW.JEEBOOKS.IN
a 43/102 MODERN'S abc + OF CHEMISTRY-XI

(} Example 39.
A monosubstituted alkyl benzene of the formula C,H ,, resists vigorous oxidation to an aryl carboxylic acid.
Name the compound and write its various monosubstituted isomers.
Solution: An alkyl group on benzene nucleus undergoes oxidation if it has at least one benzylic hydrogen atom. Since
it resists oxidation, it means that it has no hydrogen on benzylic carbon. Therefore, it should be tertiary. So, the compound
is tert-butyl benzene as
C(CH,),

tert-butylbenzene
Other monosubstituted isomers are :
CH CH
CH,CH,CH,CH, (H-CH,CH, H,CHCH,

n-Butyl benzene sec-Butyl benzene 2-Methyl propyl benzene

(} Example 40.
A hydrocarbon ‘X° takes up two molecules of hydrogen and is converted into a saturated hydrocarbon. On
ozonolysis, X gives a mixture of three carbonyl compounds namely, acetaldehyde, acetone and propan-1, 3-
dial. Assign structure to compound X.
Solution: Since the compound takes up two molecules of hydrogen, it must have either a triple bond or two double bonds.
The formation of three products on ozonolysis indicates that the compound has unsaturation at two places. The equations
are i)

CH, —C = CH-CH,— CH = CH - CH, (ii) H,O. Zn CH.-C =O + OHCCH,CHO + OHCCH,


| fs | Propan-i, 3-dial Acetaldehyde
CH, CH,
Acetone
(} Example 41.
Suggest a method (a flow sheet) to separate a mixture of ethane, ethene and ethyne.
Solution: The flow sheet for separation of ethane (C,H,), ethene (C,H,) and ethyne (C,H,) is as follows :
C,H,, C,H,, C,H, mixture
Bubble through
ammoniacal AgNO,
solution

AgC = CAg (C,H, and C,H, gases]


(White ppt)
Treat with Bubble through
HCl Br,/CCl,
C,H, + 2AgCl (s)
Ethyne
BrCH,CH,Br C,H,
Ethane
Zn, ale. escapes as
gas
C,H, + £4nBr,
Ethene

WWW.JEEBOOKS.IN
HYDROCARBONS 13/103 —

(}) Example 42.


Write chemical equation for combustion reaction of the following hydrocarbons :
(i) Butane (it) Pentene (ut) Hexyne (iv) Toluene
Solution:

(1) C,H, () + — O,(g) —_—> 4CO, (g) + 5H,O (g)


Butane

(iz) C,H, fe) + 1 O0,(g) ——> 5CO, (g) + 5H,O (g)


rent

(iit) CH, ole) +i O,(g) —+ 6COQ, (g) + 5H,0 (g)


Hexyne
(iv) C,H(g) +90,(g) —> 7CO, (g) + 4H,0 (g)
Toluene
(} Example 43.

For the following compounds, write structural formulas and IUPAC names for all possible isomers having
the number of double or triple bonds as indicated :
(i) C,H, (one double bond) (ti) C,H (one triple bond)
Solution: (a)
(a)@) CH, = CH— CH, — CH, But-l-ene
(iL) CH, H,
ee _— cis-But-2-ene
a ‘N\

(iit) CH
ao < trans-But-2-ene
H,
(iv) CH,= C—CH, 2-Methylpropene
|
CH,
(b)@) HC=C— CH, — CH,— CH, Pent-l-yne
(ii) CH,—C=CH— CH,— CH, Pent-2-yne
(ii) CH,—CH—C = CH 3-Methylbut-1-yne

CH,

(} Example 44.

896 mL of a hydrocarbon ‘A having carbon 87.80% and hydrogen 12.19% weighs 3.28 g at STP. Hydrogenation
of ‘A’ gives 2-methylpentane. Also ‘A’ on hydration in the presence of H,SO, and HgSO, gives a ketone ‘B’
having molecular formula C,H ,.0. The ketone ‘B’ gives a positive todoform test. Find the structure of ‘A’
and give the reactions involved.
(NCERT Exemplar Problem)
Solution: 896 mL of GH, (A) weigh = 3.28 g

29400 mLomL of atl,


CH, ((A) weighweigh ==228x240
96

= 82 ¢ mol
“. Molecular mass of C, H, = 82 g mol!
Determination of empirical formula
Element To Atomic Relative Relative no. of Simplest
mass ratio atoms ratio
C 87.8 12 87.80/12 = 7.31 7.31/7.31 =1 3
H 12.19 1 12,19/1 = 12,19 12.19/7.31 = 1.66 498 = 5

Empirical formula of A = C,H,


Empirical formula mass = 3}x i2 +5x1=41

WWW.JEEBOOKS.IN
— 13/104 MODERN'S abc + OF CHEMISTRY
-XI

— “Molecular formula mass _ 82 _9


Empiricalformulamass 41
.. Molecular formula = (C,H,), =C,H,,
Now C,H,, __2mole_, 9_Methylpentane CH,—_CH—CH,CH,—CH,
(A) |
Cl
The molecule has a chain of 5 carbon atoms with a methyl group at the second carbon atom. Since A adds a molecule
of H,O in the presence of Hg?* and H* to give a ketone (B), it should be an alkyne. Two possible structures of ‘A’ are

CH a CH3~
CH
7 CH—C= C—CH, CH,~
C-—CH,—C = CH
a

(I) (IT)
Since the compound B does not react with AgNO, solution, the triple bond is not terminal and therefore, structure (1)
is the correct structure.

CH H,0
°SCH - C=C - CH. —_—> CHa. 4 We _ Von
s¢” CH—C=C—CH, g CH,
AgNO,
No reaction

(Juestions
Q.1. Why do alkynes not show geometrical isomerism ?
Ans. Alkynes have linear shape and therefore, do not show geometrical isomerism.
Q.2. Is it possible to isolate pure staggered ethane or pure eclipsed ethane at room temperature?
Ans. The energy difference between staggered and eclipsed forms of ethaneis only 12.5k.J mol-!. This energy
difference is very small and can be easily overcome by the collisions ofthe molecules at room temperature.
Therefore, itis not possible toisolate either pure eclipsed or pure staggered form at room temperature.
Q.3. What is the difference between isomers and conformers ?
Ans. Isomers cannot be changed into one another and therefore, these are not interconvertible. On the other
hand, conformers are interconvertible.
Q.4. What is the cause of geometrical isomerism in alkenes ?
Ans. Alkenes have a m-bond and the restricted rotation around the mt-bond gives rise to geometrical
isomerism.
Q.5. Draw the two geometrical isomers of but-2-en-1, 4-dioic acid. Which of these will have higher
dipole moment ?
Ans,
H NS00H Hi /COOH
C C
\| |
\ y
H COOH HOOC H
cis trans
Cis isomers will have higher dipole moment.
Q.6. How many isomers are possible for monosubstituted and disubstituted benzene ?

arn
xX
Ans. There is one monosubstituted benzene as

There are three disubstituted benzenes :

ortho meta
para
HYDROCARBONS

Q.7. Which of the two:


trans-but-2-ene or trans-pent-2-ene is non polar ?
Ans. In trans-but-2-ene, the dipole moments of the two C—CH, bonds are equal and opposite and therefore,
they cancel out each other. Hence, trans-2-butene is non-polar.

CHs~ a:
(* ———— f

Ho * Non,
=O
Q.8. Write the structural formulae ofall the possible isomers of C,H,Cl, and indicate which of these
is non-polar ?
Ans. H H. Cl H Cl
ee ae Sc=cm RCS
a~ Na c~ #H ee Ae
(i) ets-1, 2-dichloroethene (it) trans-1, 2-dichloroethene — (iii) 1,1-dichloroethene
Out of these (77) is non-polar.
Q. 9. Arrange the following alkenes in the decreasing order of stability :

(T) CH, (IT) CH, (IIT) CH,

CH,C = CHCH, CH,CHCH = CH, CH, = CCH,CH,


. I>T>Il
. Why are 1, 4 adducts of 1, 3-butadiene are more stable than 1, 2-adduct ?
- Because the alkene formed contains double bond which is more highly substituted.
Q. 11. Which of the following is acidic:
But-2-ene, But-2-yne, But-l-yne, But-l-ene
Ans. But-l-yne
Q. 12. Draw the structures of three cycloalkane isomers with molecular formula C,H,, each witha
different size ring.

(i) a Cyclopentane (iL) Methyl cyclobutane

(ii) 1, 1-Dimethyl cyclopropane 5, 1, 2-Dimethyl cyclopropane

Q.13. Arrange the following in increasing order of their release of energy on combustion :

»A\ A> (ii) Git) A\J\/J\ Ww DAKX>

Ans. Largest the number of carbon atoms having maximum hydrogens (1.e., CH, groups), greater is the heat
of combustion. Thus, the increasing order of heat of combustion :
(tit) < (iv) < (i) < (i).
Q.14. Arrange the following set of compounds in order of their decreasing relative reactivity with
an electrophile, E*.
(t) chlorobenzene, 2, 4-dinitrochlorobenzene, p-nitrochloro benzene
(ii) toluene, p-H,C—C,H,—CH,, p—H,C—C,H,—NO,, p-O,N—C,H,—NO,
. (t) Chloro benzene, p-nitrochloro benzene, 2, 4-dinitrochloro benzene.
(tt) p-CH,—C,H,—CH, > toluene > p-CH,—C,H,—NO,, p-NO,—C,H,—NO,,.
Q. 15. Write the structures of all alkenes which on hydrogenation give 2-methylbutane.
CH,

. (i) CH,—CH—CH = CH, (wz) CH,= C—CH,CH, (uit) CH,—C=CHCH,

CH, CH,

WWW.JEEBOOKS.IN
wy 13/106 MODERN'S abc + OF CHEMISTRY-XI
Q. 16. What effect the branching of an alkane has on its melting point ?
Ans, Branching of an alkane decreases the melting point.
Q. 17. Why is benzene extraordinarily stable though it contains three double bonds?
Ans, The extraordinarily stability of benzene is due to resonance. Due to resonance, the n-electron cloud gets delocalized
resulting stability of molecule.
Q. 18. Suggest name of another Lewis acid instead of anhydrous aluminium chloride which can be used for
ethylation of benzene.
Ans, FeCl 3"

Q. 19. Arrange benzene, n-hexane and ethyne in decreasing order of acidic behaviour.
Ans, Ethyne > benzene > n-hexane
This is because of maximum s-character in ethyne (50%) as compared to benzene (83%) and n-hexane (25%).
H

aca CH,(CH,),CH,
Ethyne
n-hexane
Benzene
sp carbon (50% s-character) sp” carbon (33% s-character) sp® carbon (25% s-character)
Q. 20. Which of the following compounds are aromatic according to Huckel rule?

(a) «) (b) no) CH, (c) (d) ()


y

Aw
. (d), (e), (g) are aromatic. (g) is aromatic because out of Sn electrons, it has 6n delocalised electrons in one six membered
planar ring, which obey Huckel rule and therefore, it is aromatic.

Q. 21. How will you prepare deuteriopropane ?


Ans, CH, —CH—I —“e“**"_, CH, —-CHMgl —®°_, cH,—CH—D + Mg(OD)I
CH, CH, CH,
Isopropyl iodide Deuteriopropane

Kate)
® Conformations are the different arrangements in a molecule which can be obtained due to rotation around carbon-
carbon single bond.
® Staggered conformations of ethane and propane are more stable than their eclipsed conformations.
® Geometrical isomers: The compounds which have same structural formula but differ in the spatial arrangement of
atoms or groups of atoms around a double bond.
® Markovnikov’s rule. During electrophilic addition across unsymmetrical double bond, the negative part of the
adding molecule goes to that carbon atom which has lesser number of hydrogen atoms.

WWW.JEEBOOKS.IN
HYDROCARBONS

QUICK CHAPTER ROUND UP


HYDROCARBONS

Saturated Unsaturated Aromatic


contain C-C single bond contain carbon-carbon multiple bonds contain benzene ring
e alkanes e Alkenes (double bond) » Benzene, toluene
e Alkynes (triple bond)

Alkanes: C_H,,_,, Alkenes: C_H,,


« Formed by dehydration of alcohols
e Prepared by » During dehydration of alcohols, the carbocation formed
RHC=CHR + H, —-~—> RCH, CH,R rearranges to stable carbocation.
« Wurtz reaction e.g., CH,CH,CH,CH,OH —#"_, CH,CH=CHCH,
But-2-ene (80%)
CH, I + 2Na + CH,I ———> CH,CH,
e Reduction of alkynes
Cannot be used for the preparation of alkanes with with Lindlar’s catalyst => cis-isomer
odd number of electrons. with Birch reduction > trans-isomer
« Halogenation Exhibit geometrical isomerism
CH, + Cl, ——> CH,Cl + HCl Similar atoms or groups on double bond
same side => cis
Reactivity of halogens opposite side > trans
Re Cl Bree For geometrical isomerism
Rate of replacement of H of alkane » the molecule must have a double bond and
the two atoms or group of atoms attached to the same
toe
carbon atom must be different.
Mechanism trans isomer is more stable than cis-isomer
« Halogenation is a free radical substitution reaction. Greater the number of alkyl groups attached to the
double bonded carbon atoms, the more stable is the
foe en alkene.
R,C=CR, > R,C = CHR > RCH = CHR > R,C=CH, >
(ii) Cl + CH, —> CH, + HCl RCH = CH, > CH, = CH,
(iii) CH,+ Cl, —> CH,Cl + Cl Addition to unsymmetrical multiple bond follows
(ii), (tii) repeated till Markovnikov’s rule.
Negative part of attacking reagent joins with C atom
cc — Cl,
carrying lesser number of H-atoms.
CH, + Cl —> CH,Cl In the presence of peroxides, anti-Markovnikov rule
CH, + CH, —> CH,CH, is followed.
CH,CH = CH,

Alkynes: C_H,_,
« Do not show geometrical isomerism
yy
e Alkynes are acidic
HC = CH > H,C=CH, > CH,—CH, aaa CH,CH,CH,Br
° Acidity among alkynes 1-Bromopropane
Br Anti-Markovnikov rule
HC = CH > CH,AC Seep .—C = CCH, 2-Bromopropane
e Terminal alkynes react with alkaline silver nitrate Markovnikov rule

solution to give white ppt. This is test for ¢ Markovnikov rule occurs through the formation of
distinguishing 1-Alkynes and 2-Alkynes, carbocation.
¢ Anti-Markovnikov rule follows free radical mechanism.
° CH,C=CH —g o> CH,nau e Ozonolysis indicates the position of double bond.
O 0 R Bk"

Age ae > oO +0 a
© 83CH=CH —“Sy
> CH, |
e CH,C =CCH, —@™_. CH,CH, C = CH
But- 2-yne But-1--yne
MODERN'S abc + OF CHEMISTRY-Al

Electrolysis reactions for the preparation of Aromatic Compounds


alkane, alkene and alkyne. » Benzene is resonance hybrid of
e Electrolysis of aqueous solution of sodium or potassium
salt of acetic acid gives ethane (alkane)
CH,COOK
+ (aq) Electrolysis
pectro.ysis _. . Aromaticity
CH,COOK CH 3
— planar
Potassium acetate Ethane
e Electrolysis of aqueous solution of potassium salt of — cyclic system
succinic acid gives ethylene (alkene) — delocalized (4n + 2) m electrons (Huckel Rule)
CH,COOK Benzene undergoes electrophilic substitution
| (aq) Electrolysis reactions.
CH,COOK Eu-Nu—> E++Nu
Potassium succinate H
e Electrolysis of aqueous solution of potassium salt of
fumaric acid gives acetylene (alkyne)
eae
Arenium ion
CHCOOK H E H E H E H E
Pot. fumarate Acetylene
* 1 _
= Ortho para directors (activators) Electrophiles in
-OH, -OCH,, -OCOCH, -NH,, - CH, |electrophilic 4

doa ty ae oe oo. |
H E E

ages a ‘a - aloe Nitration : NO,* + Nu —— + H—Nu

halogens F, Cl, Br, I Acylation : RCO*


Halogenation : X*
Alkylation : R*

CONVERSION ROUTES

CH POCa POF CH,COOH «—2—CH,CHO


O
H+, 443K Ea
CH,CH,OH
NaOH, CaO ]aq. KOH
CH,CH,Cl

Br, , CCl,

Na, ether
Br CH,CH, Br
CH,Cl

alc. KOH

cH,cHO —!, CH,COOH —


NaOH
es Ee

CH,COCH,
\ \ N
CERT
FILE \ Se
hue
d
Textbook Exercises iii
Q.1. How do you account for the formation of ethane during chlorination of methane ?
Ans. Chlorination of methane is a free radical reaction which occurs by the following mechanism :
(2) Chain initiation
ale —Homolyte C+ Cl
(iz) Chain propagation , CH, + Ccl——> ¢ H, + HCL
CH, - Cl—Cl — CH,Cl + Cl
(wii) Chain termination CH, + CH,——+ CH, — CH,
P ‘ Ethane
CH, + Cl——> CH,Cl
Cl + Cl—— Cl,
It is clear from the above mechanism, that the free radical CH, may combine with itself to form ethane molecule.
Write IUPAC names of the following compounds:
; Hy),
(a) CH,CH=C(CH,), (6) Sap
CH, =CH—C == C—CH (c) WU\
(d) ( \-cu,—cn,-cu—cu, (e) {\ OH (f) CH,(CH,), CH(CH,),CH,
(g) CH,—CH==CH—CH,—_CH=CH— CH—CH,—CH=CH, H,—CH(CH,),

2H;
4 ~~ 3 2 1 1 2 B74 = «6 - oe
Ans. (a) CH, CH= C—CH, (b) CH,=CH—C
==C —CH, oO WAY
bu, Pent-1 -en-3-yne Buta-1 3- diene

2—Methylbut-2-ene

7 3 2 1
9 /CH3 10 6-9 5 24 1
(d) —CH,—CH, CH=CH, _/ (e) 1_oy (f) CH,—(CH,),—CH(CH,), CHs
4-Phanylbut-1-ene CH, —CH(CHg),
2-Methy! phenol
5-(2—Methylpropyl) decane
10 9 8 7 6 5 4 3 2 1
(g) CH,—CH=CH—CH,—_CH—CH— (hy CH —CH=CH,

CoH;
4-Ethyldeca-1,5,8-triene
For the following compounds, write structural formulas and IUPAC names for all possible isomers having
the number of double or triple bond as indicated :
(a) C,H, (one double bond) (6) CH, (one triple bond)
Ans. Refer Solved Example 43. (Page 103).
Q.4. Write IUPAC names of the products obtained by the ozonolysis of the following compounds :
(4) Pent-2-ene (ii) 3,4-Dimethylhept-3-ene
(iit) 2-Ethylbut-1-ene (iv) 1-Phenylbut-1l-ene

Ans. (i) CH,—CH,—CH= CH—CH, —7>3"": 5CH;—CH,—CHO + CH, —CHO


Pent-2-ene Propanal EKthanal
(ii) CHs —CH,—CH,—C= C—CH,—CH, 2°88, oH .CH,CH,—C=O + O=C—CH,CH,

3,4—Dimethylhept-3-ene Pentan-?-one Butan-2-one

(iii) CH, —CH,—C=CH,


"5 _, oH.cH,—Cc—O + HCHO
Methanal
HCH; HCH,
2—Ethylbut-l-ene 5 io Pentan-3-one

(iv) CH, CH, -CH=CH—C,H, —22"°855_, CH,CH,CHO + C,H,CHO


1-Phenylbut-1-ene Propanal Benzaldehyde
ma 43/110 MODERN'S abc + OF CHEMISTRY-XI

Q. 5. An alkene ‘A’ on ozonolysis gives a mixture of ethanal and pentan-3- one. Write structure and IUPAC name
of ‘A’.
Ans. The products are : 4 5
CH.CHO
3
saan haa aed 1 2
Alkene is CH.CH=C
3 _~CH,CH,
Ethanal CH,CH, 3 ee CHCH
Pentan-3-one 2 a
3-Ethylpent-2-ene
Q. 6. An alkene ‘A’ contains three C—C, eight C—H o-bonds and one C—C 1 bond. ‘A’ on ozonolysis gives two
moles of an aldehyde of molar mass 44u. Write IUPAC name of ‘A’.
Ans. An aldehyde having molecular mass of 44 a.m.u. is ethanal, CH,CHO.
It gives two moles of ethanal
CH,CH==O O==CH—CH 3

Alkene is CH,CH=CH—CH, or i
Li ia
ai C— a

H H
But-2-ene has three C—C, one C=C and eight C—H bonds.
Q. 7. Propanal and pentan-3-one are the ozonolysis products of an alkene ? What is the structural formula of
the alkene ?
Ans. Refer Solved Example 20 (Page 57).
Q. 8. Write chemical equations for combustion reaction of the following hydrocarbons :
(¢) Butane (ti) Pentane (it) Hexyne (iv) Toluene
Ans. Refer Solved Example 42 (Page 103).
Q. 9. Draw the cis and trans structures of hex-2-ene. Which isomer will have higher b.p. and why ?
Ans. The structure of cis- and trans- hex-2-ene are :
CH CH,CH,CH Guns. _H
a a >. om
H~ NH H “SScH,CH,CH,
cis-Hex-2-ene trans-Hex-2?-ene
cis-Hex-2-ene has large dipole moment and therefore will have stronger dipole-dipole interactions and hence higher
boiling point.
Q. 10. Why is benzene extra ordinarily stable though it contains three double bonds ?
Ans. The extraordinary stability of benzene is due to resonance. In benzene, all the six m-electrons of three double bonds
get delocalised resulting stability of the molecule.
H- H delocalisation of six m-electrons

Q. 11. What are the necessary conditions for any system to be aromatic ?
Ans. The necessary conditions for a molecule to be aromatic are :
(1) The molecule should contain a cyclic cloud of delocalized m-electrons above and below the plane of the molecule.
(ii) For the delocalisation of m-electrons the ring must be planar to allow cyclic overlap of p-orbitals.
(iit) It should contain (4n + 2) n-electrons where n = 0, 1, 2, 3....... This is known as Huckel rule.
Q. 12. Explain why the following systems are not aromatic ?

(i) ()=cH, (ii) lJ (iii) @


Ans. (z) p< CH, contain one sp* hybridised carbon atom and therefore, the system is not planar. Though it

contains 67 electrons but the system is not fully conjugated because all the six 1-electrons donot form a cyclic electron
cloud which surrounds all the atoms of the ring. Therefore, the compound is not aromatic.
(iL) is not aromatic because it contains onesp* hybridised carbon atom and the molecule is not planar.Moreover,
it contains only 47-electrons and does not obey Huckel rule v.e., (4n + 2) m-electrons.
3
sp
(iit) is not aromatic because it is a non-planar system having 81-electrons. Therefore, it does not obey
Huckel rule z.e., (4n + 2)n-electrons rule.
HYDROCARBONS

Q. 13. How will you convert benzene into


(i) p-nitrobromobenzene (ii) m-nitrochlorobenzene (ii) p-nitrotoluene (tv) acetophenone ?
Ans. Refer solved Example 28 (Page 95).
Q. 14. Inthe alkane H,C—CH,—C(CH,),—CH,—CH(CH,),, identify 1°, 2°, 3° carbon atoms and give the number
of H-atoms bonded to each one of these.
Ans. Refer Solved Example 6 (Page 27).
Q. 15. What effect does branching of an alkane chain has on its boiling point ?
Ans. The boiling point of the alkanes decreases with branching. This is because with the increase in branching, the surface
area of an alkane approaches that of a sphere. Since sphere has minimum surface are, therefore, van der Waals
forces of attraction are minimum and hence boiling point decreases with branching.
Q. 16. Addition of HBr to propene yields 2-bromopropane, while in the presence of benzoy! peroxide, the same
reaction yields 1-bromopropane. Explain and give mechanism.
Ans. Addition of HBr to propene is an ionic electrophilic addition reaction which follows Markonikov rule. In this case,
H+ adds to alkene to give a more stable 2° carbocation. This is rapidly attacked by nucleophile Br- ion to give
2-bromo propane.
H—Br =~ 4Ht+Br
CH,—CH=CH, + Ht _Slow, CH,—CH'—CH,
2°-Carbocation
: (more stable)
CH,—CH —CH, + Br- ast, CH,—CH—CH,
r
2—Bromopropane
In the presence of benzoyl peroxide, the reaction follows free radical addition. In this case, Br free radical acts as
electrophile which is obtained from the action of benzoyl peroxide on HBr.
; . * *
C,H,CO—O—O—COC,H,.. _Homolytic fission , 2C,H,COO ——-> 2C,H, + 2CO,
; Heat
. Phenyl free radical
C,H, + HBr ——> C,H, + Br
= Benzene
Br free radical attacks propene in such a way to generate a more stable 2° free radical. This free radical obtained
rapidly abstracts a hydrogen atom of HBr to give 1-Bromopropane.

CH,—CH=CH, + Br =, CH,—CH—CH,Br
* *

2°-Free radical (stable)


* .
CH,—CH—CH, Br + H—Br _Fast , CH,CH,CH,Br + Br
1-Bromopropane
Q. 17. Write down the products of ozonolysis of 1,2-dimethylbenzene (o-xylene). How does the results support
the Kekule structure of benzene?
Ans. o-xylene may be regarded as a resonance hybrid of the following structures. Ozonolysis of each one of these gives
two products as shown below:
CH,
|

ae ; ”

2. (1) ()
(i) O,, CHCl, (i) O4, CH,Cl,
(zz) Zn, H,O (ii) Zn, H,O

CH; CH; CH=0O


é CH=0 ‘ 7 z
/ So ")CH=—O O
4~c I ag
yoxa
O=CH
Methyl glyoxal Glyoxal O
1,2-Dimethylglyoxal
Therefore, only three products are formed. Since all the three products cannot be obtained from any one of the two
Kekule structures, this shows that o-xylene is a resonance hybrid of two Kekule structures (I) and (ID)
Q. 18. Arrange benzene, n-hexane and ethyne in decreasing order of acidic behaviour. Also give reason for this
behaviour.
Ans. FRefer Conceptual Questions Q.19 (page 106).
ae 13/112 MODERN'S abc + OF CHEMISTRY-XI

Q. 19. Why does benzene undergo electrophilic substitution reactions easily and nucleophilic substitutions
with difficulty ?
Ans. The orbital structure of benzene shows that the m-electrons cloud lying above and below the benzene ring is loosely
held and is thus available to the n-electron seeking reagents i.e., electrophiles. Therefore benzene undergoes
electrophilic reactions readily and nucleophilic substitution reactions with difficulty.
Q. 20. How would you convert the following compounds into benzene ?
(4) Kthyne (zi) Ethene (zit) Hexane
Ans. (i) Ethyne is converted into benzene by passing its vapours through red hot tube at 873 K.
3CH = CH Red hot tube
875 K ?
Ethyne

| Benzene
(iz) Ethene is first converted into ethyne and then to benzene as :

CH, = CH, —2“ 5¢H CH, KOH9 — CH


Ethene | Heat Ethyne
Br r

Red hot tube


SHC =H H >a73 K

| Benzene
(111) n-hexane is converted to benzene as :
7 CAs
H,C H
"| [3 __©1203,V205_,
Cry »Vo0 i,
Wo0,
gO Cyclisation
Cyclisation’
H,C CH.
~~ CH a 7 Cyclohexane Benzene

n-Hexane
Q. 21. Write structures of all the alkenes which on hydrogenation give 2-methylbutane.
Ans. The product is 1 2 3 4
CH,— CH—CH,—CH,

Hs
2-Methylbutane
The different alkenes which give the product on hydrogenation are given below :
CH,—CH—CH = CH, CH,—C = CH—CH, CH, =C—CH,—CH,

3—Methylbut-1-ene 2-Methylbut-2-ene 2-Methylbut-1-ene


Q. 22. Arrange the following set of compounds in order of their decreasing relative reactivity with an
electrophile, E*. (a) Chlorobenzene, 2,4-dinitrochlorobenzene, p-nitrochlorobenzene.
(b) Toluene, p—H,C—C,H,—NO,, p—O,N—C,.H,—NO,
Ans. Refer Conceptual Questions, Q.14 (Page 105).
Q. 23. Out of benzene, m-dinitrobenzene and toluene which will undergo nitration most easily and why ?
Ans. CH, group is electron releasing group while —NO, group is electron withdrawing group. Therefore, the electron
density will be more in toluene than in benzene and the electron density in m-dinitrobenzene will be less than in
benzene. Therefore, the ease of nitration decreases in the order :
toluene > benzene > m-dinitrobenzene.
Q. 24. Suggest the name of a Lewis acid other than anhydrous aluminium chloride which can be used during
ethylation of benzene.
Ans. Anhydrous |FeCl,, SnCl,, BF, etc.
Q. 25. Why is Wurtz reaction not preferred for the preparation of alkanes containing odd number of carbon
atoms ? Illustrate your answer by taking one example.
Ans. For preparing alkanes containing odd number of carbon atoms, a mixture of two alkyl halides has to be used.
These two alkyl halides may react in three different ways producing a mixture of three alkanes instead of desired
alkane. For example, Wurtz reaction between bromoethane and 1-bromopropane give the following three alkanes
instead of a single alkane of odd number of C—atoms.
CH,CH,.Br_ + 2Na_+ Br, CH,CH, Dtvether, CH,CH,CH,CH,
Bromoethane Butane
CH,CH,CH, Br + 2Na + Br. CH,CH,CH, —Dryether, CH,CH,CH,CH,CH,CH,
1-ee n-Hexane

1-Boa epeeeane Epiariane ;Pantie (desired)


HYDROCARBONS gis}

Exemplar Problems //
Subjective Questions

Q. 1. Why do alkenes prefer to undergo electrophilic addition reactions while arenes prefer electrophilic
substitution reactions ? Explain.
Ans. Due to the presence of n-electron cloud above and below the plane of alkenes and arenes, these are electron rich
molecules and therefore, provide sites for the attack of electrophiles. Hence, they undergo electrophilic reactions.
The alkenes undergo electrophilic addition reactions because alkenes are unsaturated molecules. For example,

Domo +XY ——> — -


~—
xX Jay
Addition product
Arenes, on the other hand, cannot undergo electrophilic addition reactions. This is because benzene has a large
resonance energy of 150.4 kJ mol. During electrophilic addition reaction, two new o-bonds are formed but the
aromatic character of benzene gets destroyed and therefore, resonance energy of benzene ring is lost. Hence,
electrophilic addition reactions of arenes are not energetically favourable. Arenes, in contrast undergo electrophilic
substitution reactions in which o C — H bond is broken and new o C — X bond is formed. The aromatic character
of benzene ring is not destroyed and benzene retains its resonance energy. Hence, arenes undergo electrophilic
substitution reactions.
Q. 2. Alkynes on reduction with sodium in liquid ammonia form trans alkenes. Will the butene thus formed
on reduction of but-2-yne show the geometrical isomerism ?

Ans. CHiy—C= C—cH, —Sasian, “ced


CH, H

But-2-yne _ H CH,

trans but-2-ene
But-2-ene is capable of showing geometrical isomerism.
Q. 3. Rotation around carbon-carbon single bond of ethane is not completely free. Justify the statement.
Ans. Rotation around C — C single bond is not completely free and it is restricted due to repulsions between the electron
clouds of C — H bonds in the adjacent carbon atoms. Therefore, ethane exists in infinite number of conformations.
Out of these, two extreme conformations are staggered and eclipsed. Refer Text Page 23.
Q. 4. Draw Newman and Sawhorse projections for the eclipsed and staggered conformations of ethane. Which
of these conformations is more stable and why ?
Ans. Refer Text Page 24.
Q.5. The intermediate carbocation formed in the reactions of HI, HBr and HCl with propene is the same and
the bond energy of HCl, HBr and HI is 430.5 kJ mol"', 363.7 kJ mol and 296.8 kJ mol respectively.
What will be the order of reactivity of these halogen acids ?
Ans. The bond dissociation enthalpy decreases in the order. HCl > HBr > HI, therefore, the order of reactivity is in the
reverse order i.e., HI > HBr > HCl.
Q.6. What will be product obtained as a result of following reaction and why?

O + CH, CH, CH, cl “C5


| cn,

Ans. .
oO + CH, CH, CH, Cl>
Isopropyl benzene
ma 43/114 MODERN'S abc + OF CHEMISTRY-XI

Propyl chloride forms carbocation, CH, — CH, — CH5 with anhydrous AICI, which is less stable. This rearranges to
a more stable carbocation as: H.C CH
~~ ¢ 3
HC

CH, — CH— CH, =2uarides


CH,— CH— CH oO.
IN? el : —H

1° Carbocation 2°-carbocation
Therefore, it forms isopropyl benzene.
Q.7. How will you convert benzene into:
(z) p-nitrobromobenzene
(ti) m- icine tal
Br Br Br
NO, Separate by
Bra FeBr,
FeBr, Lt ei fractional distillation
[Sn ———s
Ans. (7) “so,”
minor
product NO, NO,
major -Bromonitro benzene
osthilict P

NO,
(ii) eae
Cone. hed aes,
anhyd.
HNO,” ——FeBr,
Br
m-Bromonitro
benzene
Q.8. Arrange the following set of compounds in the order of decreasing reactivity with an electrophile.
Give reason. NO, OCH; Cl

Ans. The +R effect of -OCH, group is more than that of -Cl, whereas —NO, group has —R effect. Therefore, reactivity of the
substituted benzene rings is |

Q.9. Despite their - I effect, halogens are o- and p-directing in haloarenes. Explain.
Ans. Refer Text Page 91.
Q.10. Why does presence of anitro group make the benzenering less reactive in comparison to the unsubstituted
benzene ring ? Explain.
Ans. Refer Text Page 90.
Q. 11. Suggest a route for the preparation of nitrobenzene starting from acetylene ?

SHC == CH red hot aR


Cone.
C )Gone. HNO, ..CT
— —_ ——»- ——— >
Iron tube, 873 K H,SO
Ans. Acetylene .
Nitrobenzene
Q. 12. Predict the major product(s) of the following reactions and explain their formation.
Ph-CO-
H,C—CH = CH, —“""0_,
H,C—CH = CH, —*>
Ans. (i) H,C—CH—CH 9 mncinchetals H,C—CH,)—CH)Br (Anti-Markovnikov rule)
1—Bromopropane
HYDROCARBONS 13/115 a

(ii) HgC—CH = CH» HBr, a lia (Markovnikov rule)


Br
2—Bromopropane

Reason for formation, Refer Text (Page 47).


Q. 13. Nucleophiles and electrophiles are reaction intermediates having electron rich and electron deficient
centres respectively. Hence, they tend to attack electron deficient and electron rich centres respectively.
Classify the following species as electrophiles and nucleophiles.

O
(i) H,CO- (ii) net (ii) Cl (iv) CI,C:
(v) (H,C),C* (vi) Br (vii) H,COH (viii) R-NH—R
O
Ans. Nucleophiles : (i) H,CO~ (iz) aoe oe (vit) CH,— O— H (vit) R — NH —R

Electrophiles :(iii) Cl (iv) C1,C: (v) (H,C),C*


Q. 14. The relative reactivity of 1°, 2°,3° hydrogen’s towards chlorination is 1 :3.8:5. Calculate the percentages of
all monochlorinated products obtained from 2-methylbutane.
Ans. Refer Solved Example 29 (Page 96).
Q. 15. Write the structures and names of products obtained in the reactions of sodium with a mixture of 1-iodo-
2-methylpropane and 2-iodopropane.
Ans. The following three products are formed :

@ CH, CH —CH,'i+9Na +1'CH,CH—CH, —““> CH, — CEiH 0H esoH— CH, + 2Nal


CH, CH, CH, CH,
1-lodo-2-methylpropane 2, 5-Dimethylhexane
(2 molecules)

0: CH, | CH; CH;


Gi) CH,
SoH —jT+ Nas —cHe CcH, reaction C H,
oH cH CH,
+ 2Nal
2=-lodopropane (2 molecules) 2, =Dimethylbutane

(iii)
— .
CH, —CH — CH,'1+2Na+1}—CH—CH,
an ee = ——"
———>
Wurtz
CH,—CH — CH, — CH — CH, + 2Nal
CH, CH, CH, CH,
1-Iodo-2-methylpropane 2-lodopropane 2, 4-Dimethylpentane

Q.16. Write hydrocarbon radicals that can be formed as intermediates during monochlorination of
2-methylpropane. Which of them is more stable ? Give reasons.
Ans. 2- Methylpropane has two sets of equivalent hydrogens marked as a and 6 and therefore, gives two radicals I and I
tL

CH, CH, CH,


a b a
CH,—C—CH, <———— CH,—CH—CH, —————> CH,—CH—CH,
I IT
Radical I is more stable than radical I] because it is tertiary while radical II is primary. It is also stabilized due to
hyperconjugation (nine structures).
Q. 17. An alkane C,H,, is obtained as the only product on subjecting a primary alkyl halide to Wurtz reaction.
On monobromination this alkane yields asingle isomer ofa tertiary bromide. Write the structure of alkane
and the tertiary bromide.

Ans. H,C — CH — CH,X —*#2#*_, H,C — CH— CH, — CH, — CH — CH,


H, H, H,
2, 5-Dimethylhexane

WWW.JEEBOOKS.IN
a 43/116 MODERN'S abc + OF CHEMISTRY-XI

Br

CH, — CH —CH, — CH, — CH — CH, sae CH, _b—cH, — CH, — CH — CH,

Hs H, Hs H,
2-Bromo- 2, 5-dimethylhexane (3°)
Q. 18. The ring systems having following characteristics are aromatic.
(i) Planar ring containing conjugated x bonds.
(4) Complete delocalisation of the n-electrons in ring system i.e. each atom in the ring has unhybridised
p-orbital, and
(tii) Presence of (4n + 2) t-electrons in the ring where n is an integer (n = 0, 1, 2, ............. ) [Huckel rule].
Using this information classify the following compounds as aromatic/non-aromatic.

a GO 8 O ONY
(A) (C) (i) (F) (G)
Ans. A = Planarring, all an (C and N) of the ring are sp” ee It has 6 delocalised 7 electrons and follows Huckel
rule. Therefore, it is aromatic.
B = Has 6 relectrons, but the delocalisation stops at sp* hybridised CH, — carbon. Therefore, it is not aromatic.
C = 6delocalised 1m electrons (4 1 electrons of the two double bonds and 2 unshared electrons on negatively charged
carbon) in a planar ring, follows Huckel’s rule. It is aromatic.
D = Has only 4 delocalised m-electrons. It is non-aromatic.
E = 6delocalised 1-electrons follows Huckel’s rule. nm electrons are in sp? hybridised orbitals, conjugation all over the
ring because of positively charged carbon. The ring is planar, Therefore, it is aromatic.
F = Follows Huckel’s rule, has 2 7 electrons 1.e., (4n + 2) t-electrons where (n = 0), delocalised 1-electrons. Therefore,
it is aromatic.
G = 8 T electrons, does not follow Huckel’s rule i.e., (4n + 2) n-electrons rule. It is not aromatic.
Q. 19. Which of the following compounds are aromgta according to Huckel’s rule ?

(A) (B) ( \ (C) nex ecu


H

(D) (E) gS (F)

Has 8r electrons, does not follow Huckel rule. The orbitals of one carbon atom are not in conjugation. Therefore,
it is not aromatic.
= Has 67 delocalised electrons. Therefore, it is aromatic.
= Has 67 electrons in conjugation but not in the ring. Therefore, it is not aromatic.
l Has 10 x electrons, all the C-atoms are sp? hybridised, the ring is planar. Therefore, it is aromatic.
OD = Has8telectrons, out of8 7 electrons it has delocalised 6 melectrons in one six membered planar ring, which follows
AO
Huckel’s rule, Therefore, it is aromatic.
F = Has 147 electrons which are in conjugation and are present in a ring. Therefore, it is aromatic if ring is planar.
Q. 20. Suggesta route to prepare ethyl hydrogen sulphate (CH,—CH,—OSO,—OH) starting from ethanol (C,H,OH).
Ans. Whehanol is heated with conc. H,SO, at 383 K, ethyl hydrogen sulphate is formed.

H,SO,> H* +0SO,0H
CH,CH,OH + H* ——> CH, — CH, — 0* —H
Ethanol
H
Protonated alcohol

HO—S0,—O° + CH,—cH,£O'—H —35 5 cy,cH,080,0H + H,0


H a or
Hydrogen sulphate ion
HYDROCARBONS

»> Long Answer Questions carrying 5 marks


<
Q. 21. An alkyl halide C,H,, Br (A) reacts with ethanolic KOH to give an alkene ‘B’, which reacts with Br, to
give a compound ‘C’, which on dehydrobromination gives an alkyne ‘D’. On treatment with sodium
metal in liquid ammonia one mole of ‘D’ gives one mole of the sodium salt of ‘D’ and half a mole of
hydrogen gas. Complete hydrogenation of ‘D’ yields a straight chain alkane. Identify A, B, C and D.
Give the reactions involved.
Refer Solved Example 33 (Page 98).
Q. 22. 896 mL of a hydrocarbon ‘A’ having carbon 87.80% and hydrogen 12.19% weighs 3.28¢ at STP.
Hydrogenation of ‘A’ gives 2-methylpentane. Also ‘A’ on hydration in the presence of H,SO, and HgsSO,
gives a ketone ‘B’ having molecular formula C,H,,O. The ketone ‘B’ gives a positive iodoform test. Find
the structure of ‘A’ and give the reactions involved.
Refer Solved Example 44 (Page 103).
Q. 23. An unsaturated hydrocarbon ‘A’ adds two molecules of H, and on reductive ozonolysis gives butane - 1, 4-dial,
ethanal and propanone. Give the structure of ‘A’, write its IUPAC name and explain the reactions involved.
Refer Solved Example 34 (Page 99).
Q. 24. In the presence of peroxide addition of HBr to propene takes place according to anti-Markovnikov’s rule
but peroxide effect is not seen in the case of HCl and HI. Explain.
Ans. Anti-Markovnikov rule or peroxide effect is followed only in case of addition of HBr and not in case of HCl and HI.
Refer Text Page (Page 47-48).

Memory TEST
W

wy Say True or False 6. The alkaline potassium permanganate solution is


known As .......... ;
1. The boiling point of 2, 3-dimethylbutane is higher
than that of 2, 2-dimethylbutane. 7. The chain isomer of pent-l-yne is .......... .
2. Staggered conformation of ethane is more stable than 8. Alkynes are .......... acidic than alkenes.
eclipsed conformation. 9. 2-Butyne on catalytic reduction with Lindlar’s
3. Cyclohexane mainly exists in boat conformation. catalyst gives mainly product.
4, The electrolysis of potassium succinate gives 10. Electrolysis of potassium fumarate gives ..........
acetylene. -
5. Bromine water can be used to distinguish between em) Choose the correct alternative
ethene and ethyne. 1. Cycloheptatrienyl cation obeys /does not obey Huckel
6. Moist ethene can be dried by passing it through vale.
sol ea, 2. The electrophile in sulphonation of benzene is SO./
7. Butan-1-ol reacts with cone. H,SO, to give but-2-ene. H,SO,
oO
+ Propyne is less acidic than atetylene. 3. Nitration of bromobenzene or bromination of
9. Nitration of nitrobenzene gives mainly m-dinitro nitrobenzene give the same/different bromonitro
benzene.
benzene.
10. Chlorobenzene on reduction with Ni-Al alloy/NaOQH
4, Nitration of benzene is electrophilic /nucleophilic
give toluene, substitution reaction.
5. Hydration of acetylene gives ethanal/ethanol .
1. All the carbon atoms in benzene are ....... hybridised. 6. But-2-yne can/cannot react with Tollen’s reagent.
2. When .......... is passed through red hot iron tube at 7. Chloroform on heating with silver powder gives
873 K, benzene is formed. ethyne /ethene .
3. Propyne on ozonolysis gives .......... 8. Ethyne an ozonolysis gives ethylene glycol /glyoxal.
4, The reaction of ethene with bromine is .......... 9. Cis-1, 2-dichloroethene has less /more dipole moment
reaction. than trans-1, 2-dichloroethene.
5. The addition of HBr to propene in the presence of 10. Sodium propanoate on heating with soda lime gives
peroxides follows .......... rule. ethane /propane.
Ll” 13/118 MODERN'S abc + OF CHEMISTRY
-XI

elie, Memory Test \\\


Say True or False 4, electrophilic addition 5. anti Markovnikov's
¥ ones 5 Twas 6. Baeyer’s seal A tenga
3. False : Cyclohexane exists mainly in the chair form. 5. more salts diniaaih Sina
4. False : It gives ethylene. Choose the correct alternative
5. False 6. False
7. True 8. True 1. obeys 2. SO, 8. different 4. electrophilic
9. True 10. False :Benzene is formed. 5. ethanal 6. cannot 7. ethyne 8. glyoxal
|Be Complete the missing links 9. more 10. ethane.

1. sp” 2. ethyne 3. 2-oxopropanal

HOTS. Higher Order Thinking Ski |


a Advanced Level
QUESTIONS WITH ANSWERS
Q.1. Explain : CH,=CH is more basic than HC=C-.
Ans. CH,=CH is a conjugate base of the acid H,C—=CH, while HC=C- is a conjugate base of acid HC=CH. We know
that stronger the acid, weaker is the conjugate base. Since HC=CH is stronger acid than H,C—CH,, therefore, its conjugate
base HC=C- is weaker than that of H,C—=CH . Therefore, CH,=CH is more basic than HC=C .
Q.2. Why is cyclopropane more reactive than propane ?
Ans. In cyclopropane, bond angle is 60° which is much less as compared to the normal tetrahedral bond angle of 109.5°
for sp® hybridised carbon. Therefore, the molecule is very strained and hence reactive.
Q.3. Which of the two: but-1-ene or but-1-yne has larger dipole moment ? Explain.
Ans. The dipole moments of but-1-ene and but-1-yne are shown below :

CH,CH, x wt
fur: CH,CH, +=? C=C“ H
H Sp H sp” sp sp
But-l-ene But-1l-yne

The resultant dipole moment of but-1-ene and but-1-yne is due to the dipole moment of CH,CH,—C and C—-H bonds which
oppose each other. Since a sp-carbon is more electronegative than a sp? carbon, therefore solarity ofCH,CH » (sp?)—C(sp) bond
in but-1-yne is more than that of CH,CH, (sp*)—C(sp?) bond in but-1-ene. Thus, the dipole moment of bat 1- -yne is more than
that of but-1l-ene.
Q.4.What alkyne would you start with and what reagents would you use to prepare:
(i) cis-but-2-ene (it) frans-pent-2-ene

Ans.s. (i)
(i CH,CH,C
Bute= =CH
3CH,C Lindlar's2 catalyst
pan ie
aNo<<
———

cis-But-2-ene

N CH.CH
a 2 H
(ii) CH;CH,C = CCH, —j— Soo
Pent-2- $ ae *
in | CH
trans-Pent-2-ene 3

Q.5. Show steps to prepare (E)-pent-2-ene from acetylene.


Ans. HC=CH —-> HC=C-Nat —"5, CH,CH,C=CH 2-> CH,CH,C=C"Na*
Acetylene

CH,Br
CH.CH, H
“Soo
|a a |
<—“_—
liq. NH,
CH,CH,C=CCH,
E-Pent-2-ene CH

WWW.JEEBOOKS.IN
HYDROCARBONS 13/119 SS

Q.6. When ethene gas is passed through an aqueous solution containing bromine and sodium chloride, three
products are formed. Predict the products.
Ans. CH,=CH, + Br—Br ———> Hc — CH, + Br

Br*
H,0
Br—CH,CH,OH; —{—-»> BrCH,CH,OH
+ -H* oF

H,C——CH, Br BrCH,CH,Br
Br*
“—» BrCH,CH,Cl
Q.7. When 3, 3-dimethylbutan-2-ol is treated with concentrated HI, a rearrangement occurs. Name the alkyl
iodide formed showing the steps of the reaction.
CH, CH, CH,
Ans. a ae al ot
-_
CHy—¢—FH—CHl, —H
Ty —s a

CH, :OH CH, :0H; CH,


4

3,0-Dimethylbutan-2-ol
1,2-methy] shift

I
CH,—C——CH—CHy +—— CH,—C——CH—CH,
a +

CH, CH, CH, CH,


2-iodo-2, 3-dimethylbutane
Q.8. Write the alkenes that give the following compounds on ozonolysis :
(a) CH,CO CH,CH, +CH,CHO (6) HC CH,CH,CH, C CH,CH,CH,

(c) CH,- :CH,CH,CH + nen +CH,CHO


O
Ans. 0
(a) CH,CH,C=O + O=C—CH, <~—2- CH,CH, C =CHCH,
| | An, H,O0 |
CH, H CH,
-CH 2
| O. cH
2
(b) HC CH,CH,CH,CCH,CH,CH, Tn, H,0
HO | ¢ —CH,CH,CH,
| | CH,
cu

O
(c) CH,CCH,CH,CH + HCH + H,CCH $7) CH,CCH,CH, CH=CH CH,
| | | | _ |
O O O O CH,
or CH,CH = CCH,CH,CH = CH,
CH,
Q.9. An organic compound ‘X’ of molecular formula C,H,, absorbs one mole of bromine to give C,H,,Br,.
On reductionit gives 2-methylpentane, while on oxidation yields a mixture of acetic acid and isobutyric acid. Derive
the structural formula of X and give its IUPAC name.
Ans. C,H,, is an alkene. Oxidation of C,H,, to acetic acid (CH,COOH) and isobutyric acid (CH, -CH COOH) establishes
its bond at C-2 as shown ahead: by
3

WWW.JEEBOOKS.IN
a 43/120 MODERN'S abc + OF CHEMISTRY-XI
id ‘i

CH,COOH + HOOC-CH-CH, ~2— CH,CH = CHCH-CH, —3®_, CH,-CH-CH-CH-CH,


Acetic
acid CH, CH, CH,
Isobutyric acid 4-Methylpent-2-ene
J Reduction
CH,-CH,CH,CH-CH,
CH,
2-Methylpentane
IUPAC name of X : 4-Methylpent-2-ene
Q.10. One mole of a hydrocarbon (A) reacts with one mole of bromine giving a dibromo compound, G.H,, Br.
Substance (A) on treatment with cold dilute alkaline KMnO, solution forms a compound CH,,0, . On ozonolysis
(A) gives equimolar quantities of propanone and ethanal. Deduce the structural formula of (A).
Ans. One mole of the hydrocarbon (A) adds on one mole of
bromine to form C,H, ,Br,, therefore, (A) must be an alkene having
molecular formula C,H,,. The position of double bond is indicated by ozonolysis as

CH, H CH, H
CH,-C=O + O=C—CH, ——> CH,—C = C—CH,
Propanone Ethanal CA)
2-Methylbut-2-ene
.. The compound (A) is 2-Methylbut-2-ene. With alkaline KMnO,, it forms a compound C,H,,0,.
CH,H CH, H

CH,—_C=C—CH, —2KMnQ, . (“Qf cH,

OH OH
(C5H,,0,)
Q.11. What product would you get from acid catalysed hydration of 1-methylcyclohexene ? Explain.
Ans. 1-Methylcyclohexanol will be formed because a 3° carbocation will be formed as an intermediate as :

CH H
3 3 CH _

‘ HCOOH =— (J + ‘0-H — + 0H
H H H
1-Methylcyclohexene

+ H,O*

1-Methylcyclohexanol

Q.12. Predict the major product in the following reaction :


C,H,+(CH,),CHCH,OH —224_5
Ans. In the presence of conc. H,SO,, isobutyl alcohol first forms 1° carbocation (I) which then undergoes rearrangement to
the more stable 3° carbocation (ID by 1, 2-hydride shift. Carbocation (IT) then reacts with benzene to form ¢ert-butyl benzene as :
CH; CH; CH,
CH,—CH——CH,—
a oy
-QO—H —>
#
CH,—CH——CH,— O*H, H,0” CH,—C—CH,
+ 1,2-hydride
shift
Isobutyl alcohol
H
1° Carbocation (I)
(less stable)
CH;
CH,—C—CH,
CH, :

3° carbocation (IT)
(more stable) tert-butyl benzene
Q.13. Predict the structure of a hydrocarbon which gives one mole each of ethanedial and butanedial on
ozonolysis.
O
\\
‘OH ao
""=O m ds Remove O atoms ia ‘ies
Ans. Thy and reconnect the |
CH=O | double bond HC ge CH,
Ethanedial CH, CH
O=CH Cyclohexa-1, 3-diene
Butanedial
Q.14. Predict the product of the reaction:

| (i) Mg, ether


Br < \-cu,cu, WD, P

Ans. B Mg, ether D,0


» br CH,CH, ——————» BrM CH,CH, ——————+ ]) CH,CH,
—Mg (OD)Br
Grignard reagent p-Deuteroethylbenzene

Q.15. Arrange the alkenes 2-methyl but-1-ene (I), 2-methyl but-2-ene (IT) and 3-methyl but-1-ene (IIT) in order
of decreasing reactivity towards bromine.
Ans. Il >I > ITI.

s+ Revision Exercises
»»> Very Short Answer Questions <4

1. Define conformations. 14. What is Lindlar’s catalyst ?


2. Name the stable conformation of cyclohexane. 15. What is the chemical composition of teflon ?
3. Can ethene show conformations ? Why ? 16. How will you convert benzene into nitrobenzene ?
4, What is the number of 6 and 1-bonds in a molecule of 17. Give a chemical test to distinguish between ethane
ethyne? and ethene.
5. Why do alkenes undergo electrophilic addition 18. What happens when a mixture of methyl bromide
reactions ? and ethyl bromide is treated with dry sodium in the
6. What are conjugated dienes ? Give one example. presence of anhydrous ether ?
7 What is CNG ? What is its use? 19. What happens when water is added to calcium

8. Name two
) |
conformations
)
of propane. oy
carbide
Hawaii
? - oe
9. Why are all C to C bond lengths in benzene equal ” eeeeaiis ghEeeh Rrtreces ptgheey an neta Urge gente
21. Complete the reaction :
10. Why do alkynes not show geometrical isomerism ?
CH, CH=CH, __ br,
11. What type of hybridisation of carbon is involved in
benzene ? 22. What does LPG represent ?

12. Which of the two can exhibit geometrical isomerism ? 23. Why are alkenes called olefins ?
ae eee ee ae 24. How does HBr react with 1- butene ?
18 ‘Whatda Markovnikove rule? 25. Out of ethyne and ethene, which is more acidic in
nature ?
MODERN'S abc + OF CHEMISTRY
-XI

26. Name the metal which is the constituent of Grignard 14, What are substitution reactions ? Give two examples
reagent. of substitution reactions of benzene.
27. What is Wurtz reaction ? Give one example. 15. Which of the following polymerises most readily and
Give a chemical test to distinguish between but-l-yne why?
and but-2-yne. (1) Acetylene (ii) Ethene
What happens when aqueous solution of (iit) Buta-1, 3-diene.
sodium acetate is electrolysed ? 16. Write down the products and give the mechanism of
How will you convert acetylene to but-1l-ane ? the following reactions :
ol. What is peroxide effect in electrophilic addition of (i) C,H,OH + H,SO, (cone.)
alkenes ? (wi) C,H,CH, + HNO, and H,s0, (conc.)
oo. Why do alkenes undergo electrophilic addition 17. Addition of HBr to propene yields 2-bromopropane
reactions ? while in presence of benzyl peroxide, the same reaction
What effect does branching of an alkane have on its yields 1-bromopropane. Explain and give mechanism.
melting point ?
18. How would you convert the following compounds to
How is isopropyl benzene prepared from Grignard benzene ?
reagent ?
(i) Acetylene (ii) Benzoic acid
How does benzene react with fuming sulphuric acid ?
(zit) Hexane (zv) Benzene diazonium chloride.
Give reaction.
19. How does ethylene react with :
(zt) bromine
Short Answer Questions <4
(ii) alkaline potassium permanganate
What are conformations ? Discuss conformations of (iii) ozone (iv) hydrogen ?
ethane. How does acetylene react with :
How do you account for the formation of ethane during (1) oxygen (iz) bromine
chlorination of methane? (1iz) ozone (tv) water
What is meant by (v) HBr.
(t) delocalisation 21. How will you convert benzene into :
(ii) resonance energy ?
(1) bromobenzene (ii) benzene sulphonic acid
Discuss the molecular orbital theory for bonding in (iii) acetophenone (iv) toluene
benzene.
22. Explain the mechanism of electrophilic addition
Account for the order of acidity :
reactions of alkenes.
Acetylene > Benzene > Hexane
23. Explain the mechanism of nitration of benzene.
Why does benzene undergo electrophilic substitution
24, How will you distinguish between (give one test)
reactions easily and nucleophilic substitutions with
(1) ethylene and acetylene
difficulty ?
(iz) ethane and ethylene
What are the necessary conditions for any compound
to show aromaticity ? (iit) But-1-yne and But-2-yne ?

How will you explain that there exists two varieties of 29. Complete the reactions :
Pd, BaSO,
1, 2- dichloroethene while there is only one variety of (i) CH,-C=CH+H, —gquinoline
1,2-dichloroethane ? (uz) CaC, +H,QO—
Explain the stability of alkenes. (iii) CH, CH = CHCH, nae
10. Give one method for the preparation of each of the
following : G@) HO =Cn ae
(7) Ethyne (it) Ethylene (v) CH, C = CH—2AKMnO%_,
(iit) Propyne (iv) Methane.
26. Give the main products of the reactions :
11. Why do alkenes show geometrical isomerism ?
12. What happens when HNOs
(a) C,H, ——>
({) ethyl alcohol is heated in the presence of H,SO, at HySO4
443 K?
(iz) ethyl bromide is heated with alcoholic KOH ? (b) C,H CH, —CO202_,
13. How will you detect the presence of double bond in a
hydrocarbon ? (c) CH ,CH,CH,CH, “9 KMn0,

WWW.JEEBOOKS.IN
HYDROCARBONS

(pense If one mole of HBr is added to the following compounds,


write down the structures of the product / products
(ce) C,H + CH,cocl —Ambye AMCs expected to be formed :
27. How will you convert : (i) Buta-1, 3-diene (it) Penta-1, 3-diene
(a) acetylene to acetaldehyde (iit) Penta -1, 4-diene (iv) But-1l-ene.
(b) ethylene to acetylene Write structures of all the alkenes which on
(c) isopropylbromide to n-propylbromide
hydrogenation give 2-methyl butane.
47. Which of the following polymeries most readily and
(d) acetic acid to methane ?
why?
Write brief notes on :
(i) Acetylene
(1) Markovnikov’s rule (it) Peroxide effect.
(ii) Ethene
Which out of ethylene or propylene is more reactive to
(iit) Buta-1, 3-diene
the addition of HBr ? Explain.
Why does benzene undergo electrophilic substitution
Explain the terms substitution and addition
reactions easily and nucleophilic substitution with
reactions ? Methane gives substitution reaction while
difficulty?
ethylene gives addition reaction. Justify the
49. Why do alkynes undergo addition reactions while
statement giving two examples in each case.
simple alkenes do not ?
ol. Why do alkenes undergo electrophilic addition
50. Why propane has only one eclipsed confirmation while
reactions ? Explain the mechanism of HBr to ethylene.
butane has three? Explain.
oe. How do you account for acidic nature of alkynes as
ol. Out of benzene, m-dinitrobenzene and toluene which
compared to alkenes ? Give two reactions in which
will undergo nitration most easily and why ?
acetylene behaves as an acid.
52. Why is Wurtz reaction not preferred or preparation of
Discuss the general mechanism of the electrophilic
alkanes containing odd number of carbon atoms ?
substitution in benzene. What is the role played by
Explain your answer by taking one example.
the catalyst ?
Discuss the structure of benzene in terms of reso-
nance and orbital concept.
Long Answer Questions <
Why does benzene undergo electrophilic substitution What are conformations ? Discuss the different
reaction instead of electrophilic addition reaction ? conformations of ethane. How will you account for their
Discuss the mechanism of chlorination of benzene. difference in their relative stability ?
How will you convert benzene into acetophenone ? Write short notes on :
Discuss the mechanism of the reaction. (i) Friedel Craft reaction
How is benzene converted into nitrobenzene ? (it) Markovnikov’s rule
Discuss the mechanism of this reaction. (iit) Wurtz reaction,
Explain the directive influence of substituents on What are alkenes ? Why do these show geometrical
benzene and their effect on reactivity. isomerism? Explain the mechanism of electrophilic
Explain the term polymerisation with two examples. addition reactions to alkenes.
Explain the following : Explain the structure of benzene and give its
(1) Alkynes are acidic in nature important chemical reactions.

(171) Alkenes show geometrical isomerism Explain the mechanism of sulphonation and nitration
of benzene.
Give four important reactions of benzene and explain
the mechanism of any one of these reactions. Explain the following :

Explain the mechanism of the following reactions : (1) Acidity of alkynes.


(i) Sulphonation of benzene (ii) Conformations of propane
(iz) Addition of HBr to ethene. (iit) Polymerisation reactions of alkenes.

Draw the structures of six isomeric pentenes, C;H,). Discuss the following characteristics of benzene :
Specify as E or Z to each geometric isomer. (1) Resonance
Write down the products of ozonolysis of (ii) Aromaticity
1, 2-dimethyl benzene (o-xylene). How does the result (iit) Directive influence of substituents
support Kekule structure for benzene ? (iv) Mechanism of electrophilic substitution reaction.

WWW.JEEBOOKS.IN
ee ee MODERN'S abc + OF CHEMISTRY-XI

(; | ton9lo Additional Useful Information and Objective Questions

ADDITIONAL USEFUL INFORMATION


> ALKADIENES
The name of alkadienes is often shortened as diene. Dienes are the compounds which contain two carbon—
carbon double bonds. Their general formula is similar to alkynes, C,,H,,_.. Therefore, dienes are isomeric with
alkynes. These are divided into three classes :
(i) Conjugated dienes : These contain two double bonds which alternate with single bonds.
(ii) Isolated dienes : These contain two double bonds that are separated by more than one single bond.
(iit) Cumulated dienes or allenes : These contain two double bonds that share a carbon atom. In these
dienes one carbon atom participates in two double bonds. These three types of dienes are shown below :

—-C=C-C=C- —-C=C-C-C=C- —-C=C=C-

conjugated double bonds isolated double bonds cumulated double bonds


CH, =CH-CH=CH, CH, = CH-CH,—-CH = CH, CH, CH=C=CH,
1, 3-Butadiene 1, 4-Pentadiene 1, 2-Butadiene
The stability of conjugated dienes can be explained on the basis of delocalisation of electron cloud between
carbon atoms. For example consider the conjugated diene 1,3-butadiene. This may be represented as :

In 1,3—butadiene the bonds C (1) C (2) and C(3)- C(4) are double bonds while C(2) and C(3) bond is single
bond. The orbital picture shows that the m bonds are formed by the overlap of the p orbitals of C (1) and C(2) and
p-orbitals of C(3) and C(4). The central carbon atoms in this molecule are close to each other and there is
possibility of overlapping between p-orbitals of C(2) and C(3). This gives the partial double bond character to the
central bond. In other words, the four 7m electrons of 1,3—butadiene are delocalised over all the four atoms.
This delocalisation of the x electrons makes the molecule more stable.
PARTIAL DOUBLE
BOND CHARACTER

(a)
(a) Overlap of p—orbitals in a conjugated (b) Delocalization of m-electrons.
diene to form two double bonds

The delocalisation of electron cloud may be ——— as below:

onan ee oene ee
t-Delocalised electrons
svomecanaone 15)

Electrophilic Addition to Conjugate Dienes


Conjugated dienes undergo electrophilic addition reactions. They show special behaviour when they react
with electrophilic reagents.
During addition to a conjugated diene, a reagent may attach itself not only to a pair of adjacent carbon
atoms (1, 2-addition) but also to the carbon atoms at the two ends of the conjugated system (1, 4-addition).
For example,

m, (CH -CH-CH=CH, + ‘aa = CH-


H Cl H Cl
1, 2-Addition 1, 4-Addition

CH, = CH -CH = CH, | a os=CH-CH, + a CH = CH- 7


Buta-1,3-diene Br Br Br Br
1, 2- Addition 1, 4-Addition
CH,CH-CH=CH, + CH,CH=CH-CH,
| | | |
H H H H
1, 2-Addition 1, 4-Addition

It may be noted that quite often, 1,4-addition product is the major product. This unusual behaviour
of 1,3-butadiene can be attributed to the stability of carbocation formed during the reaction.

As already discussed for electrophilic addition reactions of alkenes, the first step involves the addition of
hydrogen to C — 2 to form carbocation as :
+

CH, = CH-CH=CH,+ Ht -—+—>,, CH,-CH- CH=CH,


|
H Allylic carbocation

The carbocation formed is called allylic carbocation which means carbocation next to a double bond.
Allylic carbocation gets resonance stabilised and is more stable. In second step, the chloride ion can form bond
to either end of the allylic system and forms the 1, 2- and 1, 4-addition product.
+ Loar + 6+ O+
CH, -CH +CH = CH, <> CH, -CH = CH - CH,= CH,- CH —- CH —-CH,

H H H
Resonance stabilized allylic carbocation

CH, - veo CH=CH, 1, 2-Addition


O+ b+
CH, — CH = CH= CH, + Cl — >

CH,—CH =CH-—CH, 1, 4-Addition

Cl
The addition of HBr to 1, 3-butadiene also leads to a mixture of products. The relative amounts of
1, 2-and 1, 4-addition products depend upon the temperature of the reaction. At low temperatures, the major
product is 1, 2 addition product and at higher temperature 1, 4 addition product predominates. For example,
addition of HBr to 1, 3-butadiene at — 80°C gives a 80 : 20 mixture of 1, 2 and 1, 4-addition products. But at a
higher temperature (40°C) gives 80% 1, 4-and 20% 1, 2-addition product. At intermediate temperatures, mixtures
of intermediate compositions are formed.
MODERN'S abc + OF CHEMISTRY-XI

CH, = CH-CH = CH,


1,3-Butadiene

inateaslBB we
ns Oily OH CH «RE,
Br Br
1, 2 - (80%) 1, 2 - (20%)
+
+
CH, —CH=CH- CH, 40°C CH, — CH = CH —CH,

Br
1, 4 - (20%) 1,4 - (80%)
1, 2-Addition product predominates 1, 4-Addition product predominates
It is also observed that when the reaction mixture at 0°C is heated to 40°C, the composition of 80 : 20 for
1, 2 and 1, 4-product changes to 20 : 80 for 1, 2 and 1, 4 addition product.

<_ | OBJECTIVE TYPE QUESTIONS / 5


(a) CH,CHO (b) CH,
(c) C,H, (d) CH,COCHsg.
A6, An aqueous solution of compound A gives ethane on
Select the Correct Answers: electrolysis. The compound A is :
Alkanes (a) Ethyl acetate (b) Sodium acetate
(c) Sodium propionate (d) Sodium ethoxide.
Al. The correct [IUPAC name of the following alkane is
A7. Arrange the following in decreasing order of their
H,C— CH,— CH — CH, — CH,— CH — CH, + CH, boiling points.
(A) n-butane (B) 2-methylbutane
CH CH, (C) n-pentane (D) 2, 2-dimethylpropane
/\ | (a) A>B>C>D (6) B>C>DsA
(c) C>B>D>A (dq) D>C>Bs>A
(a) 3, 6-Diethyl-2-methyloctane A8. Isopropyl bromide on Wurtz reaction gives
(5) 3-Isopropyl-6-ethyloctane (a) Hexane (6) Propane
(c) 3-Ethyl-5-isopropyloctane (c) 2, 38-Dimethylbutane (d) Neohexane
(d) 5-Isopropyl-3-ethyloctane Alkenes
A2, Which of the following has least boiling point ? A9. Which of the following will not show geometrical
(a) n-Hexane (6) n-Pentane isomerism?
(c) 2-Methylbutane
(d) 2,2-Dimethyl propane (a) Cl
c= o) H;Cy
Sconce
CHg
A3. The chlorination of methane is an example of
(a) elimination reaction (6) substitution reaction ()_ HC
° Sco=cC
H,C~
CH
C,H;
8 @
_ &
Cl~oad “Np
H
(c) addition reaction (d@) oxidation reaction.
Al10. The ozonolysis of (CH,),C=C(CH,), followed by
A4, Arrange the halogens F.,, Cl,, Br,, I,, in order of their
treatment with zinc and water will give
increasing reactivity with alkanes.
(a) acetone
(a) I,<Br,<Cl,<F, (6) Br, <I,<Cl,<F,
(6) acetaldehyde and acetone
(c) Bro <Cl,<F, <I, (dq) F,<Cl,<Br,<IL,
(c) acetic acid
A5. In the reaction, (d) formaldehyde.
CH,COOH —s—-—>
eat
X NaOH,CaO Y: Y is 7 All. Ethylene reacts with Baeyer’s reagent to give

Aviswets
(a) glycol (b) acetaldehyde
(c) oxalic acid (d) ethyl alcohol.

Al. (a) A2. (d) A3. (6) Ad. (a) AS. (5) AG. (b) AZ. (c) A8s. (c) AY. (c) ALO (a)
All. (a)
HYDROCARBONS

A12, A gas on passing through ammoniacal AgNO, solution A22,. Arrange the following carbanions in order of their
does not give any precipitate but decolourises alkaline decreasing stability.
KMn0O, solution. The gas may be (A) H,C —C=C (B)H—C#C (C)H,C —CH,
(a) C,H, (b) CoH, (a) B>A>C (b)A>BsC
(c) C,H, (d) CyH, (c) C>A>B (d)C>BsA
A138. The compound which forms only acetaldehyde upon A23. Which of the following will not react with ammoniacal
ozonolysis is: solution of silver nitrate ?
(a) Ethene (6) Propyne (a) CH,C = CH
(c) But-l-ene (d) But-2-ene (6) (CH,), CH—C = CH
A114, In the reaction : (c) CH,C = CCH,
(dq) HC=CH,
AS, Bp, Cc—*» CH,CHO+ CO, A24, In the reaction :
nae

the compound A is Hc = cy @NaNHaee, SF! yn ., Z:


(a) Ethylene (6) Acetic acid “is:
(c) Propene (d) 1-Butene. (a) CH,CHO (b) CH,CH,CH = CH,
A15. When 3, 3-dimethyl-2-butanol is heated with H,SO,, (ec) CH,CO CH, (d) CH,CH,CHO
the major product is
A25. Number of acidic hydrogen atoms in but-1-yne is
(a) 3, 3-dimethylbut-1-ene
(a) 1 (b) 2
(6) 2, 3-dimethylbut-2-ene
(ec) 3 (d) 4
(c) 2, 3-dimethylbut-1-ene
A26._ The alkyne which will react with KMnO, to give pyru-
(d) cis and trans isomers of product named under (b) vic acid is
A16. Which of the following can exhibit geometrical (a) Ethyne (b) Propyne
isomerism ?
(c) Butyne (d) Pent-2-yne
(a) But-2-yne (6) But-2-ene
A27. Ethyne adds a molecule of methyl alcohol in the
(c) But-l-ene (d) Butan-2-ol.
presence of alkali to give
A17. Maleic acid and fumaric acid are the forms of (a) Acetone (6) Methyl vinyl ether
(a) Chain isomers (6) Conformations (c) Acetaldehyde (d) Acetic acid.
(c) Geometrical isomers (d) Optical isomers A28. Which reagent can be used to distinguish between
A18. Which of the following has smallest heat of but-1-yne and but-2-yne ?
hydrogenation per mole ? (a) ale. KOH (b) ale. KMnO,
(a) But-1l-ene (6) Trans- But-2-ene (c) Br, water (d) Ag*
(c) Cis- But-2-ene (d) Buta-1,3-diene
A19. Reaction of HBr with propene in the presence of
A29. CoH 1,504,H20
= HO
A —l0l, RB.
peroxide gives The compound B is
(a) 3-Bromopropane (6) alkyl bromide (a) an acid (5) an aldehyde
(c) n-propyl bromide (ad) isopropyl bromide (c) ketone (d) ethanol
A380. Products of the reactions are
. Oxidation of an alkene X gives a diol; further oxidation
gives a diketone. Which one of the following could be ‘X’? CH,C =CCH,CH, KMnO,, KOH
3739-5839K
(a) (CH,),C=C(CH,), (6) CH,CH=C(CH,), (a) CH,COOH + CO,
(c) (CH,),CHCH=CH, (d) C,H;CH=CHC,H, (b) CH,COOH + HOOCCH,CH,
(c) CH,CHO + CH,CH,CHO
Alkynes (d) CH,COOH + CH,COCH,
A21. Acetylene gas is obtained by the electrolysis of
Aromatic Hydrocarbons
(a) sodium fumarate (6b) sodium succinate
(c) sodium maleate (@) both (a) and (c) A31. Benzene molecule has
(a) 6o0 and 67 bonds (6) 160 and 67 bonds

Arisvets
(c) 12 0 and3 7 bonds (qd) 6o0 and 3 uz bonds.

A12. (6) Al13. (d) Al4. (c) Al15. (b) Al6, (5) A17. (c) Al18. (b) Al19. (c) A20. (d) A21. (d)
A22. (a) A23. (c) A24, (c) A25. (a) A26. (db) A27. (b) A28. (d) A29. (a) A30. (b) AS1. (ec)
MODERN'S abc + OF CHEMISTRY-XI

. Benzene reacts with acetyl chloride in the presence of (a) toluene (6) naphthalene
AICI, to give (c) acetophenone (d) benzophenone
(a) acetophenone (b) toluene A40. The attacking reagent in electrophilic sulphonation
(c) benzophenone (d) ethyl benzene. of benzene is
A33. An aromatic compound X, C.H.Cl on oxidation ig) 60> (b) SO,?
gives an aromatic compound Y. The sodalime (c) SO, (d) SO,
decarboxylation of Y gives benzene. X is Addl. In Friedel Craft reaction, anhydrous AICI, is used.
(a) o-chlorotoluene (6) p-chlorotoluene Its function is to
(c) benzyl chloride (d) m-chlorotoluene (a) absorb HCl (6) absorb H,O
. Which of the following is less reactive than benzene (c) produce electrophile (d) produce nucleophile
towards electrophilic substitution reactions ? A42, The electrophile in nitration of benzene reaction is :
(a) Nitrobenzene (6) Aniline (a) NO, (b) NO,
(c) Bromo benzene (d) Chlorobenzene
(c) NO,* (d) NO
A483. Benzene can be converted into acetophenone by
. In benzene, each carbon atom undergoes
treating it with
(a) sp (a) Acetone in the presence of HCl
(b) sp? (6) Acetyl chloride in the presence of AICl,
(c) sp? (c) Methyl chloride in the presence of AICI,
(d) sp* and sp? hybridisation (d) Acetaldehyde in the presence of Fe
. The number of m-bonds in naphthalene is Ad44, Which of the following is not o-and p-directing
(a) 5 (b) 6
group?
(ce) 3 (d) 4
(a) —Cl (6) —NH,
. Inthe reaction, ae
C,H, + RCOC] ———? > C,H, COR + HCl (c) BGH, (d) —NO,
the attacking electrophilic is : A45. Which of the following does not obey Huckel rule ?
(a) AICI,* (b) Rt
(c) RCO* (d) AICI,
. n-Propyl chloride and benzene react in the presence of
anhydrous AICI, to form.
(a) ethyl benzene (6) methyl benzene CH, |
(c) n-propyl benzene (d) iso- propyl benzene
A39. Benzene reacts with C,H.COCI in the presence of (c) @ (d)
anhydrous AICI, to give

Arisvets
A382. (a) A33.. (c) A34. (a) A35. (b) A36. (a) A37. (c) AS8. (dq) AS89. (2d) A40. (d) AA. (ce)
A42, (c) A43. (b) A44, (d)

CH,—CH,\_//H
b |
from competitive examinations e H~
\cH,CH,CH,
AIPMT & Other State Boards’
CH H
Medical Entrance ° ad H
Bl. Which of the following is not aromatic? (c) > =o
(a) Benzene H ‘atin
(b) Cyclopentadienyl cation
(c) Cyclopropenyl cation CH,
(d) Tropylium cation
CHiN,
(e) Cyclopentadienyl anion (Kerala PMT 2008)
© CH,CH,~ cu,
ad ;
B2. The most stable geometrical isomer among the
following is :
CH,—CH,, CH,—CH.: "7
a 7 (Kerala PMT 2008)
CH,~ cH,

(rc
u~ \cu,—cH,

B1. (5) B2. (a)

WWW.JEEBOOKS.IN
HYDROCARBONS

BS. The decreasing order of reactivity towards electrophilic _,OH Cl

2999
substitution reaction of the following compounds is :
(c) (d)

(CBSE PMT 2008)

B8. Inthe hydrocarbon


6 5 U4 3 S|
7 ! CH,—CH=CH—CH,_C=CH
(a) 1>3>4>2 (b)4>1>3>2
ik Aoieds3 (D4>2>1>3 The state of hybridisation of carbons 1, 3 and 5 are in
- | following sequence :
(e-) 2>4>1>3 (Kerala PMT 2008) a): Beep Ref, sp*, ap?
B4. When 2-chloro-2-methylbutane is heated with 6) mash pA sn*. sp?.s
alcoholic KOH, the possible product/s is/are D: SP Fe te = me |
(CBSE PMT 2008)
a. (CH,),C=CHCH, / Bee 3
| B9. Which of the following compounds will exhibit cis-
6b. CH,=C(CH,)CH,CH, trans isomerism ?
c. (CH,),CHCH=CH, (a) Butanol (b) But-2-enol
(a) a,b,c (b)a ande (c) But-2-yne (d) toluene
(c) bande (d) a only (C_B.S.E.-PMT 2009)
(e:) aandd (Kerala PMT 2008) B10. Order of reactivity towards nucleophilic substitution
aT ee a r reaction of the compounds
ee NMe, NO, Cl
A (Predominantly) is : wy 6S

° (i) (ii) (iii)


a (a) (ti) > (it) > (i) > (iv)
(b) CH, —-C— CH,—CH, (b) (ii) > (i) > Wii) > (iv)
HW (c) (iv) > (it) > (it) > (2)
= (d) (itt) > (tv) > (it) > (2) (A.M.UL.PMT 2009)
(c) —_CH—CH—
CH,;—CH a CH; Bil. State of hybridisation of C,, C,, C; and C, of the
Br CH, hydrocarbon
(d) CH,—CH —CH—CH, (CBSE PMT. 2008) ai Ws

B6.
bri, Be
Base strength of
of ise
GGG
a. CH,—CH, 6. Hj;,C=CH-, c. H—C=C is in the following sequence :
is in order of : (a) sp*, sp”, sp? and sp
(a) b>a>c (b)e>b>a (b) sp, sp*,
sp and sp*
(ec) a>c>b (dja>b>e : sp, P" ‘DpaneP" CRSP END
sp, sp”, sp“ and sp BLS.E,
| ae eee Oe) B12. The IUPAC name of the compound having the formula
B7. Which of the following is most reactive towards CH=C—CH=CH, is
electrophilic attack? (a) 1-butyne-3-ene (65) but-1-yne-3-ene
CH,OH NO, (c) 1-buten-3-yne (d) 3-buten-1l-yne
(a) (b) (C.B.S.E.PMT 2009)

Ariswets
B3. (c) B4. (ec) BS. (5) B6. (dd) ~~ BY. (c) B8. (6) B9.(b) B10. (c) Bll. (d) B12.(c)
MODERN'S abc + OF CHEMISTRY-XI

B13. Benzene reacts with CH,Cl in the presence of B18. In a set of reactions, ethyl benzene yielded a product
anhydrous AlCl, to form : D.
(a) Chlorobenzene (5) Benzyl chloride CH,CH, .
v4 Tg
(c) Xylene (d) Toluene —xon > ® re,
FeCl, >Y
(C_B.S.E.PMT 2009) Ht
loaon
B14. When subjected to acid catalysed hydration, the order
of reactivity of the alkenes; D
‘D’ would be :
(CH,),C = CH, (I), CH,CH = CH, (II), and
COOH
CH, = CH, (IIT) is
(a) IT >II>1 (6) I> Ul > II
(a) (b)
(eo). > Tl TT (dq) Il>I>UI
(A.M.U. Med. 2010)
OCH,CH,

Crt | CH,—CH—COOC,H;
B15. In the eclipsed conformation of ethane, the dihedral
angle between the hydrogen atoms of adjacent methyl
groups is
(a) 60° (b) 120°
(c) 0° (d) 180°
(e) 109° 28’ (Kerala PMT 2010)
B16. The reaction of toluene with Cl, in the presence of FeCl,
(d) | ‘ Br
gives ‘X’ and reaction in presence of light gives ‘Y’. Thus,
*X’ and ‘Y’ are:
CH,COOC,H; (CBSE PMT 2010)
B19. Which one is most reactive towards electrophilic
(a) X = o- and p- chlorotoluene, reagent?
Y¥ = Trichloromethyl benzene
Hg
(6) X = Benzyl chloride, Y = m- chlorotoluene OCH,
(c) X = Benzal chloride, Y = o- chlorotoluene (b)
(d) X = m- chlorotoluene, Y = p- chlorotoluene
(C.B.S.E. PMT 2010)
Hs
B17. In the following the most stable conformation of
n-butane is : (d)
CH,
(CBSE PMT 2010)
B20. Liquid hydrocarbons can be converted to a mixture
of gaseous hydrocarbons by :
(a) Distillation under reduced pressure
(6) Hydrolysis
(c) Oxidation
(d) Cracking (CBSE PMT 2010)
B21. A hydrocarbon contains 80% carbon. What is the
empirical formula of the compound ?
(a) CH, (b) CH, (c) CH, (d) CH
(e) C,H, (Kerala PMT 2011)
B22. When one mole of an alkene on ozonolysis produces 2
H ¢y,H moles of propanone, the alkene is
(a) 3-methylbut-1l-ene
(C.B.S.E. PMT 2010) (6) 2, 3-dimethylbut-1l-ene
(c) 2,3-dimethylpent-2-ene
(d) isobutene

Ariel
(e) 2,3-dimethylbut-2-ene (Kerala PMT 2011)

B13. (d) B14. (c) B15. (c) B16. (a) B17. (d) B18. (b) B19. (c) B20. (d) B21. (6) B22. (e)
HYDROCARBONS

B23. Which one of the following has the lowest boiling point ? (c) pentan-2-ol, nucleophilic substitution
(a) 2-methylbutane (d) pent-l-ene, nucleophilic substitution
(b) 2-methyl propane (e) pent-2-ene, nucleophilic substitution
(c) 2, 2-dimethyl propane (Kerala P.M.T. 2012)
(d) n-pentane B30. In the following reaction
(e) n-butane (Kerala PMT 2011) CH,
B24, Reaction of hydrogen bromide with propene in the |
absence of peroxide is a/an
H,C— C—CH
|
= CH,/#28Ss aA +
Major
B
Minor
CH product product
(a) free radical addition 3
(6) nucleophilic addition The major product is
(c) electrophilic substitution CH, CH,
(d) electrophilic addition |
(e) nucleophilic substitution (Kerala PMT 2011) (a) HJC+C—CH—CH, (b) CH,—C—CH,—CH,
B25. Which of the following compounds can yield only one
monochlorinated product upon free radical OH CH, OH CH,
chlorination ? CH,
(a) Propane (b) 2, 2-Dimethylpropane
(c) 2-Methylpropane (d) n-Butane (c) Hey CH CH
(e) 2-Methylbutane (Kerala PMT 2011)
B26. Among the three conformations of butane the order of
CH,OH
stability follows the sequence CH,
(a) eclipsed > gauche > staggered |
(b) eclipsed > staggered > gauche
(d) HsC—0—CH,—OHs (A.LPM.T. 2012)
(ce) staggered > gauche > eclipsed CH, OH
(d) gauche > staggered > eclipsed B31. The ozonolysis of an olefin gives only propanone.
(e) gauche > eclipsed > staggered The olefin is
(Kerala P.M.T. 2012) (a) but-1l-ene (6) but-2-ene
B27. The IUPAC name of the following compound (c) 2, 3-dimethylbut-2-ene
(d) propene (A.M.U. Med 2012)
H,C—CH, —CH—CH, —-CH—CH, —CH, is B32. Which of the following reactions would give
isopropylbenzene as the major product ?
hapa, CH,
(a) 3-ethyl-5-methylheptane 2, IT
(b) 5-ethyl-3-methylheptane oC AICI, a) Ss H,SO,
(ec) 3,5-diethylhexane
Cl
(d) 1, 1-diethyl-3-methylpentane
(e) 1,3-diethyl-l-methylpentane
(Kerala P-M.T. 2012)
Itt
OR H,S0, "|IV. 1g
=}
AICI,
A,

B28. Meta-directing and deactivating group in aromatic


electrophilic substitution is (a) Land IV only (6) I and III only
(a) —CH, (b) -OH (c) II, I, IV only (d@) All of the above
(c) —NO, (d) —Cl (A.M.U. Med 2013)
(e) —OCH, (Kerala PM.T. 2012)
B29. 2-Bromopentane is treated with alcoholic KOH B33. The radical, CH, is aromatic because it has
solution. The major product formed in this reaction
and the type of reaction respectively are (a) 7 p-orbitals and 7 unpaired electrons
(a) pent-2-ene, B-elimination (b) 6 p-orbitals and 7 unpaired electrons
(6b) pent-l-ene, B-elimination (c) 6 p-orbitals and 6 unpaired electrons

Answer
(d) 7 p-orbitals and 6 unpaired electrons
(NEET 2013)

B24. (d) B25. (b) B26. (c) B27. (a) B28. (c) B29. (a) B30. (a) B31. (ce)
B32. (d) B33. (c)

WWW.JEEBOOKS.IN
MODERN'S abc + OF CHEMISTRY-XI

B34. Some meta-directing substituents in aromatic B41. On ozonolysis, one molecule of a hydrocarbon produces
substitution are given. Which one is most two molecules of ethanal and one molecule of
deactivating? ethanedial. The hydrocarbon could be
(a) — COOH (6) —NO, (a) 1, 3-hexadiene (6) 1, 4-cyelohexadiene
(c) —C=N (d) —SO,H (c) 1, 4-hexadiene (d) 2, 4-hexadiene
(NEET 2013)
B35. Arrange the following molecules in the correct order (e) 1, 3-cyclohexadiene. (Kerala PMT 2015)
of decreasing C — C bond length: B42. 2, 3—-Dimethyl—2—butene can be prepared by heating
C,H,, C,H, CpH,, CH, which of the following compounds with a strong acid?
(a) C,H, > C,H, > C,H, > C,H, (a) (CH,),C = CH=GH, —€He
(b) C,H, > C,H, > C,H, > C,H, (6) (CH,),CH—CH,—CH = CH,
(c) C,H, > C,H, > C,H, > C,H, (c) (CH,),CH—CH—CH = CH,
(d) C,H, > C,H, > C,H, > C,H,
(e) C,H, > C,H, > C,H, > C,H,
CH,
(d) (CH,)gC—CH = CH, (AIPMT 2015)
(Kerala PMT 2014)
B43. In the reaction with HCl, an alkene reacts in
B36. Which of the following organic compounds has same
accordance with the Markovnikov’s rule, to give a
hybridisation as its combustion product (CQ,)?
product 1—-chloro—1—methylcyclohexane.
(a) Ethane (6) Ethyne
The possible alkene is:
(c) Ethene (2d) Ethanol
(Al PMT 2014) CH, CH,
B37. Which one of the following is an aromatic compound?
(a) Cyclopentadienyl cation (a) Aas (b) Se
(Bb) Cycloheptatrieny! cation
CH,
(c) Cycloheptatrienyl anion
(d) Cycloheptatriene (c) (CA) and (B) (d) (AIPMT 2015)
(e) Cyclopentadiene (Kerala PMT 2015)
The oxidation of benzene by V,O; in the presence of
B38. n—Hexane on heating to 773 K at 10—20 atmospheric
air produces:
pressure in the presence of oxides of vanadium
supported over alumina, yields (a) benzoic acid (6) benzaldehyde
(c) benzoic anthydride (d) maleic anhydride
(a) 1—hexene
(AIPMT 2015)
(Bb) 2?-hexene
B45. The total number of m-bond electrons in the following
(c) benzene
structure is
(dq) 2—methylpentane
(e) 2, 2-dimethylbutane (Kerala PMT 2015) HC. HH 4»
B39. Which one of the following is not an isomer of wo _L Ven,
3—methylbut—1—yne?
H.C H CH,
(a) Pent—1—yne
(a) 12 (6) 16 (ce) 4 (d) 8 (AIPMT 2015)
(b) Buta—1, 3—-diene
B46. Given:
(c) Pent—2—yne
(dq) Penta—1, 3—diene “oy “om “te
(e) 2—-Methybuta—-l, 3-diene (Kerala PMT 2015)
B40. The correct decreasing order of acidic character of CH, CH, CH,
hydrogen in ethane, ethene and ethyne is
(I) (IT) (IIT)
(a) ethane > ethene > ethyne The enthalpy of hydrogenation of these compounds
(b) ethene > ethane > ethyne will be in the order as
(ce) ethyne > ethane > ethene (a) Il>UI>I (6) W>I>Ul
(d) ethyne > ethene > ethane (c) I>U>l (qd) Ul >>I
(e) ethane>ethyne>ethene (Kerala PMT 2015) (AIPMT 2015)

Anaswets
B35. (a) B36. (db) B37. (b) B38. (c) B40. (d) B41. (d) B42. (d)
B43. (c) B44, (d) B45. (d)
HYDROCARBONS

B47. The correct statement regarding the comparison of B53. Which one is the correct order of acidity?
staggered and eclipsed conformations of ethane, is (a) CH=CH>CH,—C=CH>CH,=CH,>CH,—CH,
(a) the eclipsed conformation of ethane is more
stable than staggered conformation even (6) CH=CH>CH,=CH,>CH,—C=CH>CH,—CH,
though the eclipsed conformation has torsional
(c) CH,—CH,>CH,=CH,>CH,—C=CH>CH=CH
strain
(6b) the staggered conformation of ethane is more (d) CH,=CH,>CH,—CH,>CH,—C=CH>CH=CH
stable than eclipsed conformation, because (NEET 2017)
staggered conformation has no torsional strain
(c) the staggered conformation of ethane is less B54. With respect to the conformers of ethane, which of the
stable than eclipsed conformation, because following statements is true?
staggered conformation has torsional strain (a) Bond angle changes but bond length remains same.
(d) the eclipsed conformation of ethane is more (6) Both bond angle and bond length change.
stable than staggered conformation, because
eclipsed conformation has no torsional strain. (c) Both bond angle and bond length remain same.
(NEET 2016) (d) Bond angle remains same but bond length changes.
B48. Consider the nitration of benzene using mixed conc. (NEET 2017)
H,SO, and HNO,, Ifa large amount of KHSO, is added
B55. Predict. the correct intermediate end product in the
to the mixture, the rate of nitration will be
following reaction:
(a) unchanged (6) doubled
(c) faster (d) slower. (NEET 2016) —H,0,H,50,
_, intermediate > product
B49. In the reaction HgS0, (A) (B)
(i)NaNH,/liq.NHyNH .y
Aa py QNaNHgllig. yy[)NaNHp/liq-NHg
(i/NaNH,/liq.NH ,
H—C=CH~(Gcn,cn,Br >~~ GnCH,CH,Br >» (a) pHs —C = CH» a
(a) X = but—2-yne, Y = hex—2-yne
(6) X = but—l-yne, Y = hex—2—yne
SO, O
(c) X = but—l—-yne, Y = hex—3—yne B: HeC — C= CHo
(dq) X = but—2-yne, Y = hex—3-yne. (NEET 2016) |
B50. In which of the following molecules, all atoms are
OH SO,
coplanar?
(ec) A: HeC —C —CHs,

se
(a) Ss (b)
OH O

(NEET 2017)
B56. Which of the following molecule represents the order
CH3~ CN of hybridisation sp*, sp”, sp, sp from left to right
atoms?
“SCN
(a) HC=C—C=CH
(NEET 2016)
B51. The compound that will react most readily with (6) CH, = CH—C=CH
gaseous bromine has the formula (c) CH, = CH—CH=CH,
(a) CH, (b) C,H, (d) CH,—CH=CH—CH, (NEET. 2018)
(c) C,H, (d) CH, (EET 2016)
B52. Which of the following can be used as the halide JEE (Main) and Other State
component for Fridel-Crafts reaction? Boards’ Engineering Entrance
(a) Chlorobenzene B57. Reaction of trans 2-phenyl-1-bromocyclopentane on
(6) Bromobenzene reaction with alcoholic KOH produces
(c) Chloroethene (a) 2-phenylcyclopentene
(dq) Isopropyl chloride (NEET 2016) (b) 1-phenylcyclopentene
(c) 3-phenylcyclopentene

Ariswets
(d) 4-phenylcyclopentene (ALE EE, 2006)

B47. (6) B48. (d) B49. (c) B50. (a) B51. (a) B52. (d) B53. (a) B54. (c) B55. (d)
B56. (5) B57. (c)
MODERN'S abc + OF CHEMISTRY-XI

B58. The IUPAC name of ~~ is

(a) 4,4-Dimethyl-5,5-diethylpentane
(6) 5,5-diethyl-4,4-dimethylpentane
(c) 3-ethyl-4,4-dimethylheptane
es |
(d) 1,1-diethyl-2,2-dimethylpentane
(A.LLE.E.E. 2007) (c) (ie EK
B59. The compound formed as a result of oxidation of ethyl
(AIEEE 2008)
benzene by KMn0O, is :
B65. In the following sequence of reactions, the alkene
(a) acetophenone
affords the compound B
(b) benzoic acid
(c) benzyl alcohol CH,CH=CHCH, —2> A—“ 5B
(d) benzophenone. (A.D ELELE. 2007) The compound B is
B60. Which of the following reactions will yield 2, 2- (a) CH,CHO (6) CH,CH,CHO
dibromopropane ?
(c) CH,COCH, (d) CH,CH,COCH,
(a) CH,CH=CHBr + HBr —> (AIEEE 2008)
(b) CH=CH + 2HBr —> B66. The hydrocarbon which can react with sodium in
(c) CH,—CH=CH, + HBr —> liquid ammonia is
(a) CH,CH,C=CCH,CH,
(qd) CH,—C=CH + 2HBr —> (A.LE.E.E. 2007)
(6) CH,CH,CH,C=CCH,CH,CH,
B61. The reaction of toluene with Cl, in presence of FeCl, (c) CH,CH,C=CH
gives predominantly :
(d) CH,CH=CHCH, (AIFEE 2008)
(a) benzyl chloride B67. The treatment of CH,MgX with CH,C=C—H
(6) o-and p-chlorotoluene produces
(c) m-chlorotoluene (a) CH, (6b) CH,—CH=CH,
(d) benzoyl chloride. (ALBEE. 2007) (c) CH,C=C—CH, (d) CH,—CH=CH—CH,
B62. Presence of a nitro group in a benzene ring : (AIEEE 2008)
(a) renders the ring basic B68. In which of the following species, all three types of
(b) deactivates the ring towards nucleophilic hybrid carbons are present ?
substitution. (a) CH,—=C=CH, (6) CH,-CH=CH—CH,
(c) deactivates the ring towards electrophilic (c) CH,—C=C—CH,; (d) CH,-HC=CH—CH,
substitution
(e) CH,—CH—CH=CH, (Kerala Engg. 2009)
(dq) activates the ring towards electrophilic
substitution (A.LELE.E. 2007) . Aqueous solution ofsodium succinate on electrolysis gives
(a) CH, —CH, (b) CH,==CH,
B63. An organic compound with molecular formula C,H,,
upon ozonolysis gave only acetone as the product. The
(c) CH=CH (d) CO,
(Orissa JEE 2010)
compound is
B70. The ease of nitration of the following three
(a) 2,3-dimethylbut-1l-ene hydrocarbons follows the order
(b) hex-3-ene
(c) hex-2-ene
CH, CH, CH,

Cu
(d) 2,3-dimethylbut-2-ene
(e) 3-methylpent-l-ene (Kerala PET 2008)

B64. The electrophile, E* attacks the benzene ring to


generate the intermediate o-complex. Of the following, CH,
which o-complex is of lowest energy? (1) (11) (iT)
(a) Il=I1=I1 (6) I>IT>!

ATWets
(c) It>U>I (dq) [=TI>II (WB JEE 2011)

B58. (c) B59. (b) B60. (d) B61. (6) B62. (c) B63. (a) B64. (c) B65. (a) B66. (c) B67. (a)
B68. (c) B69. (db) B70. (d)
HYDROCARBONS

B77. When HBr adds on hex-1l-ene in the presence of


B71. The Z isomer among the following is
benzoyl peroxide, the product is
th - CH,CH, (a) 2-bromohexane (6b) 2, 3-dibromohexane
(a) —_ (c) 1, 2-dibromohexane (d) 2, 4-dibromohexane
Cl” “Br
(e) 1-bromohexane (Kerala P.E.T: 2012)
CH;_ _ CH,OH B78. 2-Hexyne gives trans-2-hexene on treatment with :
(b) = (a) Li/NH, (b) Pd/BaSO,
Hn” ~ CHO (c) LiAlH, (d) Pw/H,
(A.LELE.E, 2012)
eo ci B79. An optically active compound having molecular
formula C,H,, on ozonolysis gives acetone as one of
(c) N the products. The structure of the compound is
~ou H.¢ CH,
CH, _ COOH (@) Wom / Ul
H,C oN
(d) i A= ' (A.M.U. Engg. 2011)
H;C, H
B72. When 3-phenylpropene reacts with HBr in the
H.C H
presence of peroxide, the major product formed is
(a) 2-bromo-1-phenylpropane
(b) Yom CH;
(Bb) 1, 2-dibromo-3-phenylpropane
(c) 3-(o-bromophenyl) propene
H,c, 4H
(d) 1-bromo-3-phenylpropane
(e) 3-(p-bromophenyl) propene (Kerala P.E.T. 2011) (c) 7- NSO ‘3
B73. Ozonolysis of an organic compound gives formaldehyde
as one of the products. This confirms the presence of
/
Hc) 0H
(a) two ethylenic double bonds
(6) avinyl group
(c) an isopropyl group (d) Pe 7 CH, (W.B. JEE 2013)
(d) an acetylenic triple bond (ALBEE. 2011)
H,C yeN
H H
B74. The non-aromatic compound among the following is

oD oO
B80. The correct order of decreasing H— C — H bond angle
in the following molecule is

(c) l\
‘S)
(d) l \ (A.LE.ELE. 2011) De,
(I) (11) (IIT)
we,
OH O (a) I>U>
B75. The IUPAC name of : is (6) Il>I>Ul
(a) 4-Hydroxypentan-2-one (c) It>U>I
(6) 2-Hydroxypentan-4-one (dq) I>Ul>u (WB JEE 2014)
B81. The IUPAC name of the following molecule is
(c) 2-Oxopentan-4-ol
(d) 4-Ketopentan-2-ol (Kerala C.E.T: 2012)
B76. The number of sigma(o) and pi(m) bonds present in
1, 3, 5, 7-octatetraene respectively are
(a) l4and3 (6) 17 and 4 (a) 5, 6-dimethylhept-2-ene
(c) 16 and 5 (dq) 15 and 4 (b) 2, 3-dimethylhept-5-ene
(e) 16and3 (Kerala PE.T. 2012) (c) 5, 6-dimethylhept-3-ene
(d) 5-isopropylhex-2-ene. (WB JEE 2014)

Answers
B71. (a) B72. (d) B73. (6) B74. (d) B75. (a) B77. (e) B78. (a) B79. (6b) B80. (5)
B81. (a)
MODERN'S abc + OF CHEMISTRY-XI

VoO-, 500°C Clo. Av B88. The major products obtained during ozonolysis of 2,
B82.
T 16 10-20 atm. 3-dimethyl-1-butene and subsequent reductions with
What is B in the above reaction? Zn and H,O are
(a) methanoic acid and 2-methyl-2-butanone
(a) Benzyl chloride (b) Benzal chloride
(b) methanal and 3-methyl-2-butanone
(¢) Hexachlorobenzene (d) Benzene hexachloride (c) methanol and 2, 3-dimethyl-3-butanone
(AMU Engg 2014) (d) methanoic acid and 2-methyl-3-butanone.
B83. The products obtained by ozonolysis (WB JHE 2016)
of
2-ethylbut-1l-ene are B89. Identify X in the following sequence of reactions:
(a) propanone and ethanal CH; — i” 4 “s — CH,— CH, —CH,
(6) ethanal and pentan-3-one
(c) butanal and ethanal Br Br
(qd) methanal and pentan-2-one
(e) methanal and pentan-3-one. (Kerala PET 2014)
B84. When but-2-yne is treated with Na in liquid ammonia Br NH,
(a) cis-2-butene is obtained
(b) trans-2-butene is formed CH 7H
(b) Ze =C
(c) n-butane is the major product H CH,CH,CH,
(dq) it rearranges to but-l-yne
(@7 Chip, _H,CH,CH3
(d) there is no reaction. (Kerala PET 2014)
ee wy “~sz
B85. Ios HBr (1 equiv.) (d) CHg — , — ‘ie— CH, CH,CH,
H.C NH, NH,
The major product of the above reaction is (WB JHE 2016)
_ CH, H.C B90. Predict the product (B) in the following sequence of

(a)
CH, (b) H.C Br
reactions:

Ethylbenzene __KMn04-KOH_ , 4 30" Bp


Br
AC H.C (a) Benzaldehyde (6) Benzophenone
(c) Benzene (d) Acetophenone
(c) H,C :; (d) H.C ay Be (e) Benzoic acid (Kerala PET 2016)
r
B91. The reaction of propene with HOC! (Cl, + H,O)
(WB JEE 2015)
proceeds through the intermediate:
B86. Which of the following compounds will exhibit
geometrical isomerism? (a) CH,-CH*-CH,-OH
(6) CH,-CH*-CH,-Cl
(a) 2—-Phenyl—1—butene
(c) CH,-CH(OH)}-CH,*
(6) 1, 1-Diphenyl—1—propane
(d) CH,-CHCI-CH,* (JEE Main 2016)
(c) 1—Phenyl—2-butene B92. The isomerisation of 1-butyne to 2-butyne can be
(dq) 3—Phenyl—1—butane (JEE Main 2015) achieved by treatment with
B87. Which compound would give 5—keto—2—methyl (a) hydrochloric acid
hexanal upon ozonolysis (6) ammoniacal silver nitrate
| CH, CH, (c) ammoniacal cuprous chloride
CH, (dq) ethanolic potassium hydroxide (WBJEE 2017)
(a) (b) B93. The reaction of propene with HBr in the presence of
peroxide proceeds through the intermediate
HAG
(a) H,C —CH—CH,
CH, CH; (6) H,C —CH —CH,Br
| CHs | Br
(c) | (d)
CH, ©) H,C—CH—CH,
(JEE Main 2015) (d) H,C—CH,—CH,
(e) none of the above. (Kerala PET 2017)

B82. (a) B83. (e) B84. (b) B85. (b) B86. (c) B88. (5b) B89. (6) B90. (e) B91. (bd)
B92. (d) B93. (db)
HYDROCARBONS

B94, Which of the following molecule is least resonance B102 . CH,—C=CMgBr can be prepared by the reaction of
stabilized? (a) CH,—C=C—Br with MgBr,

a) O (b) (b) CH,—C=CH with MgBr,


(ce) CH,—C=CH with KBr and Mg metal
aN
O (d) CH,—C=CH with CH,MgBr. (W.B. JEE 2018)
(c) (d) y B1083. The correct order for acid strength of compounds
CH=CH, CH,—C=CH and CH,—CH, is as follows:

(JEE Main 2017) (a)CH,—C=CH > CH=CH > CH,—CH,


B95. The trans-alkenes are formed by the reduction of (b)CH,—C=CH > CH,—CH, > CH=CH
alkynes with: (c)CH=CH > CH,=—CH, > CH,—C=CH
(a) H, - Pd/C, BaSO, (b) NaBH, (d)HC=CH > CH,.—C=CH > CH,=CH,
(c) Na/lig. NH, (d) Sn-HCl (JEE Main 2019)
(JEE Main 2018) B104. The major product of the following reaction is :
B96. What final product will form when alcoholic KOH is CH, CH, So (i) KOH ale.
treated with 1,1-dichloroethane? (ii) NaNH, in
Br Br liq.NH,
(a) Ethane-1,2-diol (6) Ethene
(c) EKthyne (d) Acetaldehyde (a) CH,CH,C=CH (6) CH,CH,C
— CH,
(J.K. CET 2018)
| |
NH, NH,
B97. Identify the following compound which exhibits
gemetrical isomerism. (c) CH,CH=C=CH, (d) CH,CH=CHCH,NH,
(a) But-2-ene (6) But-1l-ene (JEE Main 2019)
(c) Butane (d) Iso-butane JEE (Advance) for ITT Entrance
(Karanataka CET 2018) B105. CH,—CH=CH, + NOC] ——- (A)
B98. The reagent ‘X’ used for the following reaction is
Which of the following is the structure of compound A?
R R’ (a) CH,CHNO—CH,Cl
R—C=C—R’ + H, > omc’
H ‘H (6) ON—CH,—CH,—CH,Cl
(a) Ni (b) Pd/C (c) CH,—CHCI—CH,NO
(c) LiAIH, (d) Na/liquid NH, (d) CH,CIL—-CHNO—CH,CH, (L.T) 2006)
(Karanataka CET 2018) B106. The reagents(s) for the following conversion :
B99. Number of constitutional isomers of alkane with
OT ——» H—=—H is/are:
formula C,H, , is Tr
(a) 3 (b) 2 (a) alcoholic KOH
(c) 5 (d) 10
(6b) alcoholic KOH followed by NaNH,
(e) 8 (Kerala PET 2018)
B100. Phenylacetylene on treatment with HgSO,/H,SO,, (c) aqueous KOH followed by NaNH,
H,O produces (d) Zn/CH,OH. (LT. 2007)
(a) acetophenone (b) phenylacetaldehyde B107. The synthesis of 3-octyne is achieved by adding a
(c) phenylacetic acid (d) 1-phenylethanol bromoalkane into a mixture of sodium amide and
(e) 2-phenylethanol. (Kerala PET 2018) an alkyne. The bromoalkane and alkyne respectively
B101. Which of the following compounds are aromatic?
are:
(a) BrCH,CH,CH,CH,CH, and CH,C=CH
(6) BrCH,CH,CH, and CH,CH,CH, C=CH
| = (c) BrCH,CH,CH,CH,CH, and CH, C=CH
H (d) BrCH,CH,CH,CH, and CH,CH,C=CH (I.1.T. 2010)
A B C D
B108. In allene (C.H,), the type(s) of hybridisation of the
(a) A,B (6) A,B,C
carbon atoms is(are)
(c) B,C (d) B,C,D
(a) sp and sp® (b) sp and sp?
(e) A,B,D (Kerala PET 2018)
(c) only sp? (d) sp? and sp®
(LT) JEL. 2012)

B94. (b) B95. (c) B96. (c) B97. (a) B98. (bd) B99. (c) B100. (a) B101. (a) B102. (d) B103. (d)
B104.(a) B105. (c) B106. (5) B107. (d) B108. (5)
MODERN'S abc + OF CHEMISTRY-XI

C10. Which of the following has/have aromatic character :

Cl. Alkanes can be obtained from carboxylic acids by


(a) > ol]
(a) LiAIH, (b) Decarboxylation
(c) Kolbe’s electrolysis
(a) Clemmensen’s reduction
C2. Which of the following on treatment with warm
dil H,SO, in the presence of HgSO, will give Cll. Which of the following have dipole moment.
butan-2-one ?
(a) 2, 2-Dimethylpropane
(a) But-l-yne (6) But-l-ene
(6) 2, 2, 3, 3-Tetramethylbutane
(c) But-2-yne (d) Pent-1-yne
C3. The molecule that will have dipole moment is (c) trans-Pent-2-ene
(a) 2,2,-Dimethylpropane (d) cis-Hex-3-ene
(6) trans-pent-2-ene C12. In the Newman projection for 2, 2-dimethylbutane
(c) cis hex-3-ene xX
H,C CH,
(d) 2, 2, 3, 3-tetramethylbutane
. Toluene when treated with Br,/Fe gives
p-bromotoluene as the major product because the
CH.,- group H H
(a) 1s p-directing (6) is m-directing X and Y respectively be
(c) activates the ring by hyperconjugation (a) Hand H (b) Hand C,H,
(d) deactivates the ring (ec) C,H, and H (d) CH,andCH, (IT 2010)
C5. But-1l-ene and but-2-ene can be distinguished by C13. Which one of the following exhibits geometrical
isomerism ?
(a) Baeyer’s reagent (b) hot alk. KMnO,
(c) Reductive ozonolysis (a) 1, 2-dibromopropene
(d) Tollen’s reagent (6) 2, 3-dimethylbut-2-ene
C6. Which of the following can be used in Friedel Crafts (c) 2, 3-dibromobut-2-ene
acylation reactions ? (d) 2-methylbut-2-ene
(a) CH,COOCH, (6) CH,CH,COCI (e) 2, 3-dibromobut-l-ene (Kerala PMT 2011)
(c) CH,CH,Cl (d) (CH,CO),O
C7. Which of the following undergoes electrophilic C14, Which of the following molecules, in pure form is(are)
substitution reactions faster than benzene ? unstable at room temperature ?
(a) Phenol (6) Aniline
(c) Chlorobenzene (d) Nitrobenzene
C8. Which of the following has almost zero dipole moment? (a) C) (b) |_|
, =
(a) trans-dichloroethene (5) cis-but-2-ene Oo
(c) cis-dichloroethene (d) trans-but-2-ene
C9. Which of the following alkenes on ozonolysis give a
(c) -) (d) C) (LLT. J.E.E. 2011)
mixture of ketones only?
CH C15. Among the following, reaction(s) which gives(give) tert-

| =
; a 3
(a) =C. butyl benzene as the major product is(are)

(c) CH, — C —CH=CH,


ta)
Ks
waar
NaOC,H, AICI

CH, | | OH
CH (c) =<. (d)
(d) (CH,),C=cC* ~ H,SO, | BF,.OEt,
CHs
(JEE Advance 2016)
Answels
Cl, (fc) C2. (a,c) C3. (b,c) C4, (a,c) C5. (b,c) C6. (5,d) C7. (a,b) C8. (a,d) C9 (a,d)
C10.(a,d) C11. (gd) C12. (bd) C13. (a,c) C14, (6, c) C15. (6, c, d)
HYDROCARBONS

Passage-II
Benzene and other aromatic hydrocarbons, though contain
mt-bonds, yet they behave as saturated hydrocarbons. They
are stable because of delocalisation of n-cloud. These undergo
Passage-I electrophilic substitution reactions as :
Alkenes undergo a variety of oxidation reactions. With cold C,H.—_H + YZ——> C,H.Y + HZ
and neutral or alkaline KMnO,, alkenes are oxidised to give The reactivity of aromatic hydrocarbons towards electrophilic
vicinal diols. Oxidation with hot KMnO, undergoes cleavage substitution depends upon the electron density in the
of C=C bond leading to the formation of carboxylic acids, benzene ring.
ketones and carbon dioxide depending on the nature of alkene. Answer the following questions
Reductive ozonolysis of alkenes give aldehydes or ketones. D5. Which of the following is not compatible with arenes?
Answer the following questions : (a) Greater stability
D1. But-2-ene on treatment with cold alk. KMnO, gives (6) Delocalisation of n-electrons
(a) Butane-1, 2-diol (6) Butane-2, 3-diol (c) Electrophilic addition
(c) Ethylene glycol (d) Glyoxal (d) Resonance
D2. An alkene ‘X’ on treatment with hot alkaline KMnO, D6. In the reaction of C,H,.Y, the major product is
gives acetic acid. Alkene ‘X’ is m-isomer. The group Y may be
(a) Hex-3-ene (6) But-2-ene (a) —COOH (b)—Cl (c)—OH (d) —NH,
(c) But-l-ene (2) Pent-1-ene D7. The electrophile in sulphonation of benzene is
D3. Reductive ozonolysis of alkene ‘A’ gives propanone. (a), SO,* (6)HSO,° (c) SO, (d) H,SO,*
The alkene ‘A’ is D8. Which reagent cannot be used in Friedel Craft
alkylation of benzene ?
(a) 2,3,-Dimethylbut-2-ene
(a), FeCl, (6) SnCl, = (ce) AICI, (d) LiAlH,
(6) 1,4-Dimethylpent-2-ene D9. The correct order of reactivity in electrophilic
(c) 1,3-Dimethylbut-2-ene substitution reactions is

9999
(d) 2-Methylpropene
D4, 2-Methyl propene on treatment with hot alkaline
KMn0O, gives
(a) (CH,),CO,HCOOH (6) (CH,),CO, CO,
(c) (CH,),CO,CH,CHO (d) CH,CHO, CH,CH,CHO (a) l>2>3>4 (6b) 3>4>2>1
(ec) 2>1>3>4 (d) 2>3>1>4 #£=°(LLT 2002)

Answets
D10. Nitration of benzene undergoes in the presence of
(a) HCl (b)H,SO, (c)NO,* (d) HONO
Passage-I D1. (b) D3. (a) D4. (5)
Passage-II D5. (c) D7. (c) D8. (d) D9. (c) D110. (db)
1. Assertion : Addition of Br, to but-l-ene gives two
Assertion Reason Type Questions optical isomers.
Reason =: The product contains one asymmetric
The questions given below consists of an Assertion and carbon atom.
the Reason. Use the following key to choose the appropriate 2. Assertion : Alkyl benzene is not prepared by Friedel
answer. Crafts alkylation of benzene
(a) If both assertion and reason are CORRECT and reason Reason : Alkyl halides are less reactive than acyl
is the CORRECT explanation of the assertion. halides.
(6) Ifboth assertion and reason are CORRECT, but reason 3. Assertion : Lactic acid shows geometrical isomerism.
is Not the CORRECT explanation of the assertion. Reason =: It has C =C bond.
(c) If assertion is CORRECT, but reason is INCORRECT. 4, Assertion : Trans-but-2-ene on reaction with Br, gives
(d) If assertion is INCORRECT, but reason is CORRECT. meso-2, 3-dibromobutane.
(e) Ifboth asseriton and reason are INCORRECT.
Reason : The reaction involves syn-addition of
bromine.
Araswets
lLt@ 2t) 3&3 4 @
MODERN'S abc + OF CHEMISTRY-XI

. Assertion : Acetylene is more acidic than ethylene. 10. Assertion : Buta-1, 3-diene and but-l-yne can be
distinguished with Tollen’ vent.
Reason =: Acetylene has sp character of carbon and, eenigeg err n tna omree tages
therefore, more s-character. Reason =: But-1l-yne gives ppt. with Tollen’s reagent
but but-1, 3-diene does not.
. Assertion : Propene reacts with HBr in the presence on ; |
of peroxides to give 1-bromopropane. 11. Assertion : Toluene on Friedel Crafts methylation
gives o-and p-xylene.
Reason =: Alkenes react with HBr in the presence of R CH fomteetaY to hen:
peroxides according to anti Markovnikov’s eso Sees Oo Denzene rng
ela increases electron density at o-and
p-position. (NCERT Exemplar Problem)
: SeBeTeO dad aces aeynee ans oxtayon wie 12. Assertion : Among isomeric pentanes, 2, 2—dimet-
Bayer’s reagent give a mixture of carboxylic Vifee tans Tele hichest boil; at
acid and CO, ylpentane has highest boiling point.
se Reason’ : Branching does not affect the boiling point.
Reason =: Terminal alkynes show acidic character.
(NCERT Exemplar Problem)
Ran eae neenon GP TAbrOpenZene ee ueerniy 13. Assertion : Nitration of benzene with nitric acid
m-dinitro benzene.
requires the use of concentrated sulphuric
Reason : —NO, group is electron donation group. acid.
. Assertion : I is aromatic. Reason | : The mixture of concentrated sulphuric acid
and concentrated nitric acid produces the
Reason : It contains 47 electrons. electrophile, NO3.
(NCERT Exemplar Problem)

Aresvets
5. (a) 6G (a) %%7(6) 8 (ce) 9 (d) 104a) 11 (a) 12. (e) 13.(a)

4, The number of tertiary hydrogens in 2, 3-dimethylbutane


Integer Type Questions is
Integer
we Type : The answer; to each of the following 5. Among the following, the number of aromatic
question is a single-digit-integer from 0-9. compound(s) is
1. The number of molecules having zero dipole moment is

OAA
trans-but-2-ene, cis-pent-2-ene, cis-but-2-ene,
propene, trans-1,2-dichloroethene, but-2-yne,
cis-1,2-dichloroethene
2. The number of cis-trans isomers with molecular formula
C,BrCIFI is

Gace
3. How many of the following on reductive ozonolysis will
give ethanal as one of the products?
2-Methylbut-2-ene, 2-methylpropene, but-2-ene,
propene, ethene, pent-2-ene, pent-3-ene, hex-3-ene

(JHE Advance 2017)

Answer
1. (3) 2. (6) 3. (4) 4, (2) 5. (5)
HYDROCARBONS

‘y,,)__NCERT
Objective Questions
6. Which of the following will not show geometrical
>> Multiple Choice Questions (Type-l) <a isomerism ?
1. Arrange the following in decreasing order of their
boiling points. (a) “NomeZ" (b) So=cZ"
(A) n-butane (B) 2-methylbutane cl~ Np c|~ “Nol
(C) n-pentane (D) 2,2-dimethylpropane
(a) A>B>C>D (6) B>C>DsA | H,C / Cals CH, Jods

(c) D>C>Be>A (d) C>B>D>A (c) ¥ ma (d) ZA ma


2. Arrange the halogens F,, Cl,, Br., I,, in order of their
HC, CH, CH, C.H,
increasing reactivity with alkanes. Arrange the following hydrogen halides in order of
(a) 1,<Br,<Cl,<F, (6) Br, < Cl, <F,<L, their decreasing reactivity with propene.
(c) F,<ClL, < Br, <I, (dq) Br, <I,<ClL<F, (a) HCl > HBr > HI (6) HBr > HI > HCl
3. The increasing order of reduction of alkyl halides with (c) Hl >HBr> HCl (c) HCl > HI > HBr
zine and dilute HCl is Arrange the following carbanions in order of their
(a) R-Cl<R-I<R-Br_ (6b) R-Cl < R-Br < R-I decreasing stability.
(c) R-I<R-Br<R-Cl (d) R-Br < R-I < R-Cl (a) H,.CeCe C- (B) H-C=eC
4, The correct IUPAC name of the following alkane is
(C) H,C—CH,-
cia i iliaineiiniade iad (a) A>B>=C (6) B> ASC
(c) C>BoA (d) C>A>B
CH CH Arrange the following alkyl halides in decreasing order
/\ ° of the rate of B-elimination reaction with alcoholic
CH, CH, CH, KOH.
(a) 3,6— Diethyl — 2 — methyloctane
(6) 5 — Isopropyl — 3 — ethyloctane
(c) 3— Ethyl — 5 — isopropyloctane
(A) ian
iiin (B) CH,—CH,—Br
(d) 3— Isopropyl — 6 — ethyloctane
5. The addition of HBr to 1-butene gives a mixture of CH,
products A, B and C
(C) CH,—CH,—CH,—Br
(a) A> B>C (6)
C> BoA
(c) B> CsA (id) A>C>B
10. Which of the following reactions of methane is
H Br , incomplete combustion :
(C) CH,—CH,—CH,—CH,—Br
The mixture consists of (a) 2CH, +0, —Cu/S23K/100
atm_ 50H OH
(a) A and B as major and C as minor products (6) cH, +0, “22°93 ,HCHO+H,0
(6) Bas major, Aand C as minor products
(c) Bas minor, A and C as major products (c) CH,+0,—>C(s)+2H,0 (J)
(dq) Aand B as minor and C as major products
(d) CH,+20,—> CO,(g)+2H,0 (J)

1. (d) 2. (a) 3. (5) 4. (a) 5. (a) 6.(d) TF. (ce) 8. (b) 9. (d) 10. (c)

12. Which of the following alkenes on ozonolysis give a


Multiple Choice Questions (Type-ll)
mixture of ketones only ?
11. Some oxidation reactions of methane are given below. (a) CH,—CH = CH—CH,
Which of them is/are controlled oxidation reactions ?
(a) CH,(g)+20,(g)—— CO,(g)+2H,0 () (6) CH,—C—CH = CH,
(6) CH,(g)+0,(g)—— C(s)+ 2H,0 (2) CH,
(c) CH,(g)+0,(g)— 2223-5 HCHO + H,O
(d) 2CH,(g)+0,(g)—2228
B00 atm_,9CH,OH 0 Da CH CEs
(d) (CH,),Cc= CCH

Answes
11. (c,d) 12. (c,d)
H,

WWW.JEEBOOKS.IN
MODERN'S abc + OF CHEMISTRY-XI

13. Which are the correct [UPAC names of the following 16. In an electrophilic substitution reaction of
compound ? nitrobenzene, the presence of nitro group
HC(CH,), (a) deactivates the ring by inductive effect.
(b) activates the ring by inductive effect.
(c) decreases the charge density at ortho and para position
of the ring relative to meta position by resonance.
H,C—CH—CH,CH,
(d) increases the charge density at meta position
(a) 5-Butyl — 4-isopropyldecane
relative to the ortho and para positions of the
(b) 5-Ethyl — 4-propyldecane
ring by resonance.
(c) 5-sec-Butyl — 4-iso-propyldecane IT
. Which of the following are correct ?
(d) 4-(1-methylethyl) — 5-(1-methylpropyl)-decane
14. Which are the correct [UPAC names of the following (a) CH,—O—CH * is more stable than CH,—CH?
compound ?
(b) (CH,), CHis less stable than CH, —CH, —CH®
il Minina Mc Wein: ca dei ide (c) CH, = CH — CH?® is more stable than
CH, CH, — CH, —CH®
HC,— C— CH,
(d) CH, =CH® is more stable than CH, — CHF
CH, 18. Four structures are given in options (a) to (d), Examine
them and select the aromatic structures.
(a) 5 —(2’, 2’-Dimethylpropyl)-decane
(6) 4— Butyl — 2,2-dimethylnonane
(c) 2, 2-Dimethyl — 4-pentyloctane (a) >: (b) C=]
(d) 5—neo-Pentyldecane

oOL of
15. For an electrophilic substitution reaction, the
presence of a halogen atom in the benzene ring
(a) deactivates the ring by inductive effect
(b) deactivates the ring by resonance 19. The molecules having dipole moment are
(c) increases the charge density at ortho and para (a) 2,2-Dimethylpropane
position relative to meta position by resonance (6) trans-Pent-2-ene
(d) directs the incoming electrophile to meta position
(c) cis-Hex-3-ene
by increasing the charge density relative to ortho
and para position. (d) 2, 2,3, 3 - Tetramethylbutane.

Answers
13.(c,d) 14.(a,d) 15.(a,c) 16.(a,ce) 17. (a,c) 18. (a,c) 19. (5, c)

Matching Type Questions


20. Match the reagent from Column | which on reaction with CH,—CH = CH, gives some product given in Column II as
per the codes given below :

Column I Column IT
(1) O,/Zn+H,O (a) Acetic acid and CO,
(iz) KMnO,/H* (b) Propan - 1 - ol
(iit) KMnO,/OH™ (c) Propan - 2 - ol
(iv) H,O/Ht (d@) Acetaldehyde and formaldehyde
(v) B,H,/ NaOH and H,0, (e) Propane - 1,2-diol

Answets
au, (t) = (ae); (zz) = (a); (ree) — (e); (7) — te); (u) — (8).

WWW.JEEBOOKS.IN
HYDROCARBONS

21. Match the hydrocarbons in Column I[ with the boiling points given in Column II.

Column I ColumnII
(2) n—Pentane (a) 282.5 K
(zi) tso-Pentane (6) 3809 K
(zit) neo-Pentane (c) 301K

22. Match the following reactants in Column I with the corresponding reaction products in Column II.

ColumnI ColumnI
(i) Benzene + Cl, —4!@s_ (a) Benzoic acid
(ii) Benzene + CH,Cl — (b) Methyl phenyl ketone
(iii) Benzene + CH,COCI Aes, (c) Toluene
Gey ‘Toluene a= (d@) Chlorobenzene
(e) Benzene hexachloride

23. Match the reactions given in Column I with the reaction types in Column II.

Column I Column II

(i) CH, = CH, +H,O —"— CH,CH,OH (a) Hydrogenation


Gi) CH, = CH, +H, —*> CH,—CH, (b) Halogenation
Git) CH, = CH, +Cl, 44 CI—CH, —CH, —Cl (c) Polymerisation
(iv) 3CH = CH—&* 5C.H,
7 Heat
(dq) Hydration
(e) Condensation

Answer
21, (¢) — (6); (ez) —(e); (it) — (a). 22. Kz) — (et); (it) —(c); (ez) — (5); (zu) — (a).
23. (i) —(d); (tt) — (a); (tit) — (6); (tv) — (e),

It is cyclic and has conjugated 81-electron system but


Assertion Reason Type Questions it is not an aromatic compound.
In the following questions a statement of Assertion Reason (R) : (4n + 2) m electrons rule does not hold
(A) followed by a statement of Reason (R) is given. good and ring is not planar.
Choose the correct option out of the choices given 20. Assertion (A) : Toluene on Friedal Crafts methylation
below : gives o—- and p—xylene.
(a2) Both A and R are correct and R is the correct Reason (R) : CH, group bonded to benzene ring
explanation of A. increases electron density at o— and p— position.
(b) Both A and R are correct but R is not the correct 26. Assertion (A) : Nitration of benzene with nitric acid
explanation of A. requires the use of concentrated sulphuric acid.
Reason (R) : The mixture of concentrated sulphuric
(c) Both A and R are not correct.
acid and concentrated nitric acid produce the
(d) Ais not correct but R is correct. electrophile, NO,”.
24, Assertion (A) : The compound cyclooctane has the 27. Assertion (A) : Among isomeric pentanes, 2, 2-
following structural formula : dimethylpentane has highest boiling point.

(=) Reason (R) : Branching does not affect the boiling


point.

Araswets
24. (a) 25. (a) 26. (a) 27. (c)
Ll” 43/144 MODERN'S abc + OF CHEMISTRY-XI

| ons for
Hints & Exp Difficult Objective Type Questions
i lanations . . . * .

A. meq with only one correct answer a A 7


A2.(d) : Branched chain alkanes have lower boiling point. A31. (c) Bee 126 and 3m bonds
A5. (b): CH,COOH 22>
NaOH
CH,COONa
2
NaOH
> CH, H COOCH,
AG. (b): CH,COONa —Electrolysis_, CH,CH,
a
Sodium acetate
A382. (a): Ol + CH,COCI alls, | ]
AB. (c): OH,CHE: SCH, on cn CH,
Acetophenone
CH, CH,CH, NaOH
2, 3-dimethylbutane A38S. (c) Cg H sCH, ae 3 C,H; COOH —c.9 ? Ge H,
AO. (a) (CH,),C = C(CH. ae ae Benzyl chloride

(CH3),COreal ii AS6. (a): 5n bonds. CH,

A11. (a): CH,=CH, __ale. KMnO4 _, CH,.—CH, bu_cn,

H H
Glycol A388. (d): Gi CH,CH,CH,C1 “>
| (i) 0:
A138. (d) :H,C CH=CH CH, aa 2CH,CHO ; Ficuse
Bad) -ane sopropy! benzene

Al4. (ec): ami alias a aa , —se. KOH, B. meq from competitive examinations
a ai Br B1.(6): Cyclopentadieny! cation has 4n electrons. Hence it
Og does not follow Huckel rule.
CH,CH=CH, Zn, H20 CH,CHO+CO, B2. (a): The steric repulsion of the groups makes the cis-
isomer less stable than the ¢rans isomer in which
CH, Hy é the bulky groups are far apart.
Al5. eae
(6) :CH,af = CHCH, ss
H’ BS. (c): —CH,
Bet: group is electron donating 7 group and
Rearrangement reip ¢et effect, -NO, group is electron withdrawing . shows +I
group as
CH,OH well as shows —M effect. —Cl group shows —I effect and
ie _ , a is weak deactivating group. So, the order of reactivity
AIG, (0) ac arm pai aay can exhiby aes —_— = reaction "ss

HBr
Al9.(c): CH, CH =CH, —sacmaee ? CH, CH, CH, Br
n-Propy! bromide
A23. (c): Only terminal alkynes react with ammoniacal OC >
solution of AgNO.
—NaNH2_ , HC =CNa _ CHa! B5. (6): FeO—CH —CH, +HBr is
A24. (c):HC = CH
HO. bu,
HC = C—CH,—2-8
ut H,C—CH— CH*—CH, a
1.2-Hydrid
H,C = Va CH,—= ye ie HW
3
OH O | | (2°-Carbocation)
A26. (b) :CH.C = CH —5Mn4_, CH.—C__C_OH B6. (d): Stronger the conjugate acid, weaker will be
3 ; corresponding base. Acidity of conjugate acids of
Propyne given carbanions is (c) > (6) > (a). Therefore, the
Pyruvic acid base strength is (a) > (b) > (e).
B7. (c): Hydroxy group is most activating group among the
A27. (b) :HC =CH + CH,OH _ Alkeli_, H,C =CH—O CH ‘ given options towards electrophilic attack.
Ethyne Methyl vinyl ether sp® sp? sp? sp sp sp
He2* B8. (5): CH,
oo CH—OH 3 c= CH
4 1
A29. (a) :SHC= CH at, Hpo CH: CHO —2> CH,COOH fop? CH,
sp?
Aga | ye ‘
A30.(b) : CH,C = CCH,CH, ee B11. (d): CH XS
7 *6) cnati
5 nS CH

CH,COOH + HOOCCH, CH, CH,


HYDROCARBONS

B14. (c) : Acid catalysed hydration of alkenes proceeds


through formation of carbocation.
B19. (c):
(CH,),C = CH, —“—>(CH,),C — CH,
I is most reactive due to the presence of electron releas-
CH,CH = CH, —*>CH,CH — CH, ing group.
II
0
CH, = CH, —4!—>CH,— CH, CH, CH i
IIT B22. (c): on ow cH, 2.0" is. aia
Propanone
Alkene which gives more stable carbocation will
2, 3-Dimethylbut-2-ene
be more reactive. Here alkene | gives tertiary
carbocation, II gives 2° carbocation and III gives
1° carbocation. Order of stability of carbocations B25. (b) : All hydrogen atoms are equivalent in 2, 2-dimethyl
is 3° > 2° > 1°. Therefore, order of reactivity of
propane, Therefore, it forms only one monochlorinated
alkenes is I > II > III.
product.
B15. (c) : In general, the hydrogen atoms attached to
sp-hybridised carbon atom are most acidic, followed 1 4 3 4 o 6 7
by those attached to sp? and sp® hybridised carbon B27. (a); CH; CH,—_CH—CH, -CH—CH,—CH,
atoms. Therefore, order of acidity of hydrogen atoms
is : alkyne > alkene > alkane CH,CH, CH,
3-Ethyl-5-methylheptane
Out of ethyne and propyne, propyne is less acidic due
to electron releasing —CH, group. Therefore, the correct
order of acidity is : B29. (a) : PH sCH, CHa TH CH
ale KOH
—HBr
CH = CH > CH= C —- CH, > CH, = CH, > CH, — CH,
Br
(IIT) (TV) (IT) (I) 2-Bromopentane
CH,CH,CH=CH—CH,
Pent-2-ene
The reaction is known as B-elimination.

B30. (a):
B16. (a):
| - ™

H, C I _ apy eee
:—CHEGH, tO aa: [
| ofH—CH,
CH, CH,
(2°Carbocation)

\orr
Hy,
B17. (d):
od coc

is most stable due to least repulsions between two


fe
du,O1
Minor product shift
2Methyl
1,
methyl groups.
B18. (6):

c)
CH,CH, COOH H,

H,C—C—CH—CH,
KMn0, )
H,
(3°Carbocation)

Br, |FeCl,
jour
COOC,H, . _,CO‘OH:

ae
; C,H,O°H: |
| } _
H.H,O OHCH,
Major product
Br
MODERN'S abc + OF CHEMISTRY-XI

B31.(c): H3C__ = _ CH, B40. (d) : HC=CH > CH, = CH, > CH,—CH,
Cc Oronolysis_5
Ozonolysis
9CH.C—=O

B41. (d) :2CH,—CH=0 + OHC—CHO
H,c~ CH, |
CH, <_Ozonolysis_ CH ,CH=CH—CH=CHCH,
Propanone
or H,C—C=0 + O=C—CH, HC;—C—CH—CH, 2,4—-Hexadiene

H, CH,CH,
B42.(d) :
2,3-Dimethylbut-2-ene CH, CH;
B33. (c) : 6 p-orbitals and 6-unpaired electrons contribute to Ht | +

aromaticity.
CH, 4 CH = CH,——> CH,-4—C8—-CH,
B34. (b) :—NO, group is the most deactivating group because CH, CH,
of strong —I and — M effect. 2°-Carbocation
(less stable)
B35. (a) : Bond length decreases as
C-C(C,H,) > C=C(C,H,) > C=C(C,H,) CH,
sp? sp” sp 1, 2-methyl |
CH, —C — CH — CH,
HF, (Benzene) has, bond length in between C — C shift + ;
and C = C and therefore correct order is CH,
C,H, > C,H, > C,H, > C,H,
3°-Carbocation
B36. (6) : Hybridisation in CO, : sp
(more stable)
Hybridisation in ethyne (HC = CH): sp
+ n
laa
B37. (b) (a) ‘2, (b) O
Planar,
4x electrons
Planar,
Gr electrons i"
CH, —C = C—CH,

~Q © O
, 7 (aromatic)
CH,
2, 3-Dimethylbut-2-ene

Planar, Non-planar,
Sr electrons 6x electrons a

»o O B43. (c):

Bo E
Non-planar,
1-Chloro-1-methylcyclohexane
4x electrons
/CHs
lr y V30;/Al;05 Ss
Se
B38. (c): CH,
(c) 7 CH, 773K, 10-20 at
atm |

Not Benzene

n-hexane 1-Chloro-1-methyl cyclohexane

B39. (b): CHy—PHACNSS | O

CH, ii
B44, (d):
:
+ O,.—>
Vo0;
C
3—Methylbut—l—yne
(a) CH,CH,CH,C=CH O
(b) CH,= CH—CH = CH, Benzene Maleic anhydride

(c) CH,CH,C=C—CH, CH, H H H


(d) CH, =CH—CH = CH—CH, B45. (d): ae CH,

(e) CH,= c— CH = CH, CH,


CH,
HC HCH,
Isomers have same molecular formula but different No. of m bonds = 4
structures, No. of t bond electrons = 4x 2=8
(5) is not isomer of 2-Methylbut—1—yne.

WWW.JEEBOOKS.IN
HYDROCARBONS

B46.(d) : The enthalpy of hydrogenation is inversely


CH, CH, CH;

OO" -O
proportional to the stability of alkene
Correct order : [II > II >I Cl |
B48. (d): HNO, + H,SO, = NO,* + HSO, +H,O B61.(d) :
Presence of KHSO, will decrease the concentration
of NO,* and hence reaction will be slower.
B49. (c): HC =CH _NaNHp
ligNH3 yHC = C-:Nat CHaCHbr, o-and p-chlorotoluene

But-1l-yne B62.(c): —NO, group shows —R effect and withdraws


(x) electron density from the benzene ring and hence
|NaNH, deactivates the ring towards electrophilic aromatic
liq.NH, substitution,

CH,CH, C = CCH,CH, , cHscHgBr Nat:C- = C-CH,CH, B63.(a):: CH,—C =C— CH, —%-,
Hex-3-yne
H; CH,
(Y) 2 LS / an
B50.(a) : In biphenyl, all carbon atoms are sp? hybridised
and hence all lie in same plane.
B53.(a) : Correct order of acidic strength is:
xKe
(two acidic hydrogens) (one acidic hydrogen) ca \ &“
4 Zn, —Z£nO0
> CH,=CH, > CH,—CH, 0; 50
B54.(c) : There is no change in bond angles and bond lengths

i
in the conformations of ethane. There is only
change in dihedral angle.
2CH,—C—CH,
-C =CH __H90,Hg"*
B55.(d) :CH, CH;—C=CHy Acetone
HaS04
OH B64.(c): In all other structures, electron withdrawing —NO,
(A) group is present which intensifies the positive
charge and destabilises the structures.
tautomerism , CH,— ¥i CH,
B65.(a) : CH,—CH=CHCH, —%3_,
O
(B) | 2
sp? spe sp
B56.(b) : we = CH—C = CH a i ——— 9? 2CH,CHO

O |
B66.(c): Sodium in liquid ammonia reacts only with
ale KOH,
KOH,
B57.(c) : terminal alkynes.

B67.) : CH,C=CH +CH,MgX 72", CH,C=CMeX +CH,


3- Icyclopentene
It is anti-elimination reaction in which H—atom sp" sp_—s sp °
from second carbon atom cannot be eliminated B68. (c): CH,—C= C—CH,
because it is in syn position. But H atom from 5th B69. (b):
carbon gets eliminated, CH,COONa CH,
CH,CH, COOH | + 2H,O ———> || + 2CO, + 2NaOH
CH,COONa CH,
KMn0O,H"*
Sodium succinate
B59.(b) : = peroxide
B72. (d): C,H,CH, —CH = CH, + HBr
Benzoie acid C,H,CH,CH,CH,Br
B60.(d) :CH,—C=CH —“*> CH; —¢= CH; B78. (6) : Vinyl group on ozonolysis gives formaldehyde as
1—Bromo-3-—phenylpropane

Br Br one of the products.


OH
HBr, CH, — CCH F all 1
B75. (a): GH, —C HCH, —C—CH,
Br ‘Ciaspatea 2-one
2,2-Dibromopropane
MODERN'S abc + OF CHEMISTRY-XI

aerate
aiita H H H H
octa-1,3,5,7-tetraene
H H H H
B83. (e):
H3C 7 CH, _ 1—

CH,CH,
CH, (i) Og (ozonolysis)

canis:
(u) Zn HO

o-bonds = 17, 1 bonds = 4 2-Ethylbut-1-ene


HBF _s, H,C—CH,—_C=0 + HCHO
B77. (ec): CH,—CH,—-CH,_CH, CH=CH, ___Peroxide | Methanal
Hex-l-ene

CH,CH,CH,CH,CH,Br Pentan-3-one
1-Bromohexane B84, (hb):
It is according to anti-Markovnikovy rule.
CH, — C = Q/40CH, +2Na as
liq.NH

B78. (2): CH,CH,CH,C = CCH, ———2—


But-2-yne
Hex-2-yne |
CH,CH,CH,._ ea CH H
H~
i —

CH, - —_(C +2NaNH,


trans- hex-2-ene
H ~ CH,

B79. (b) : H,C H trans-but-2-ene


No=cO CH; (i) O, CH,
Hc~ Cc
(ii) Zn, H,O
-_ | HBr (1 eq.) _
H,c, 4H
Optically active compound
H.C =
H,C\_
_C=0 + O=CHCHCH,CH, | CH, CH,
H,C |
CH,
H
C
a ck edBr C CH,Br|

ou > DX > OX
H H H
H,C “Awa H.C / VY
B80. (5):
HW H H
B86. (c):
sp” Aw Pas : | | _7CHs
Trigonal sp" 7)
planar Tetrahedral Tetrahedral C,H.H.C
ak CH, C,H.H,C
Pe H
(IT) (I) (IIT)
The bond angle in (II) is largest because ‘C’ is sp” cis-form trans-form
hybridised. Therefore, bond angle is about 120. In B87. (d):
both (I) and (IID, carbon is sp* hybridised. However,
the bond angle in (1) is larger than (IIT) because (1)
CH, CH,
contains a three membered cyclic ring, which is O, =O
under a high angle strain. Due to this, the Zn, H,O CHO
hydrogens in (I) are predisposed far away from 3 CH,
each other, making the bond angle larger. Thus 5-keto-2-methylhexanal
correct order of bond angle is II >I > II. CH, CHs
7 1
B81. (a): Me Me ~Me
B88. (b): cH,—¢cu—¢ —CH, —3>
2,3-Dimethylbut-1-ene
Q
5, 6-Dimethylhept-2-ene
CH,-CH—C
rok | xX! CH,
B82. (a):
C
|m3 CH Ht |? mato
O+—.0
AH, CHg
my yo V,0;, 500°C Cl
HC OF, 10-12 atm
CH, CH—C = 0+HC HO
Methanol
CH, Toluene 3
2-Methylbutan-2-one
n-Heptane
HYDROCARBONS

B89. (b): CH, —CH—CH—CH,—CH,—CH,


B98. (6): Catalytic reduction of alkynes with hydrogen in
| | the presence of Pd/C gives cis-alkenes.
Br Br
| R
NaNH» R—C=C_R’+H, Pale, “No acZa
H i H
Na, liq.NH,
cis — but-2-ene
Birch reduction
B99. (c): CH, CH, CH, CH, CH, CH, (Hexane)
CH, H CH,— CH CH,CH,CH, (2-Methylpentane)
No =o
H~ CH,CH, CH,
CH,CH,— way CH, (3-Methylpentane)
CH,CH, COOH
CH,
B90. (c): OC) _KMn0,KOH oy CH,
CH,— C—CH,—CH, (2,2-Dimethylbutane)
COOH
CH,
Fg—Va_fi—co, (2,3-Dimethylbutane)
a C) CH, CH;
Benzoic acid
é- & -
OH
B91. (6): CH,CH = CH, + HOCI—-+CH,CHCH,Cl B100. (a): C,H,—C=CH ae F< C.H,—C = CH,
(Intermediate)
Phenyl acetylene | O

OH tautomerises !
—— CH, CHCH, Cl
Acetophenone
OH
B101. (a): According to Huckel rule, the compounds having
B92.(d): On heating with alcoholic KOH or NaNH, in an (4n + 2)n electrons are aromatic.
inert solvent, the triple bond of but-l-yne gets
shifted to but-2-yne.

CH,CH,— C=CH —*““" CH,— C=C—CH cs N|


OS |
‘cco
OS O +

But-1l-yne But-2-yne
H
B93.(b) :CH,—CH=CH, + Br — CHj>-CH —CH,Br 6ne 6ne 4ne 4nme
O (A) (B) (C) (D)

B94.(b) : is non-aromatic and is least resonance


B102. (d): CH,;—C=CH + CH,MgBr —
~_) _
stabilized. CH,;—C=C MgBr + CH,

B104.(a) : i mea Ce,


CH H
B95.(c): CH, -C=C—CH, SHaNs, ” Yo=c Br Br Br
H CH, NaNH : :
trans-alkene ig NH CH,CH,C =CH
a
B105.(c) : NOC1 = NO* + Cr
B96.(c) : CH;—CH 5" >CH,=CH—Cl CH,—CH = CH, + NO — Cl—_>
i Vinyl chloride
CH,—CH—CH, NO
eee CSC CH,CH—CH,NO + Cl —> CH, CH—CH,NO
Ethyne
]
B106.(5) : The conversion is carried out first by treating
ao | “Noa
ln oN
B97.(a) with alc. KOH followed by NaNH,.
as H.C’ H
cis trans
But-2-ene exhibit geometrical isomers
MODERN'S abc + OF CHEMISTRY-XI

B107. (d): CH,CH,C =CH —eN#:_, CH,CH,C =CNat 3 (i)0


D3. (a): Pe = Sei
CH,CH,CH,CH,Br + NatC = CCH,CH, (i) Zn, HoO

—> CH,CH,CH,CH, C =C —-CH,CH 3


3-Octyne
sp af sp sp”
B108. (5) :CH, = C=CH,

C. mcq with more than one correct answer 2&2 H,—C = O


D4, (b) : CH,—C = CH, +4(O} 5Mn04,C
C2. (a,c): HC =C_CH,—CH, +H,O #25 Hg CH,
iu
+ HCOOH

[O]
OH é CO,, H,0
CH,—C = C—CH, +H,0 He“, _Integer Type Questions
But-2?-yne

CH,;—CH = ie —— CH yO, 1, (3); trans-but-2-ene, trans-1,2-dichloroethene,


but-2-yne
OH 2. (6): Six isomers :

Cl CH, a a
oe am
=C ‘i

H “Sc. HH SH,
C14.(b,c) : (6) and (c) are anti-aromatic and unstable. (a) is
Bis =C
i
a Ny
non-aromatic and (d) is aromatic and are stable
at room temperature.
C15.(6,¢,d) : al
=C
(a) + AmB 2 oy
NaOC,H,. I 7” Nel
3. (4): 2-Methylbut-2-ene, but-2-ene, propene, pent-2-ene
3° a
Ci: px + C 2H,OH + NaBr 4, (2): CH,—CH — a —UGHe
|
CH, CH,
(b) O: a: faa 5. (5):

oy.
C+ Xe
x NCERT Exemplar Problems : MCQs Type-I
A; ter: Boiling point decreases with increase in branching
@) CF ON + H,O and with decrease in total number of carbon
atoms. Thus, the correct order is
C (n-pentane) > B (2-methylbutane) > D (2, 2-
dimethylpropane) > A (n-butane).
D. mcq based on comprehension 2. (a): Reactivity decreases down the group.

cold alk HBr


D1. (b): CH q—CH=CH—CH, “KMn0,4” BACs CH,CH,CH=CH, Markovnikov CH,CH,ZCH—CH;
But-1-ene rule I
CH; —7H— CH—CH, 1(major product)
OH OH + CH,CH,CH,CH,Br
Butane-2, 3- diol C (minor product)
D2. (b): CH,—CH=CH—CH, + 4 (0) 2%
KMn0O
Since | contains a chiral carbon, it exists in two
CH,COOH + CH,COOH enantiomers which are mirror images of each other.
HYDROCARBONS

Mirror | (ce):
| CH

i OF Lg, ED? |_DmO + OmOK


ree {)03 Hs

H.C,“‘' : fo C,H, H,

(d):
CH, H;C Br

(CH;CH.),C (72%
\ cu, {i Zn,B20Le
(CHC Cm 0=O + +0 aa
ae (B)
Thus, the mixture consists of A and B as major ),€ Sop
a
and C as minor product.
13. (c,d): 1-methylpropyl and 1-methyl ethyl are IUPAC
8. (b): sp-hybridised carbon is more electronegative than
names of sec-buty! and isopropyl groups. But both
a sp® hybridised carbon and hence can
these names are recommended by IUPAC.
accommodate the negative charge better.
Therefore, B is more stable than C. Since —CH, 14. (a,d) : Both 2’, 2’ -dimethyl propyl and neopentyl groups
group has + | effect, it intensifies the negative are recommended names by IUPAC.
charge and therefore, destabilises A as compared
18. (a,c): Cyclopropenyl cation (27 electrons) and diphenyl
to B. Thus, the order of stability is: B>A>C.
(two benzene rings have 6 7m electrons each) are
9. (d): The larger the number of alkyl groups attached to aromatic.
the double bonded carbon atoms, the more stable
is the alkene and more reactive is the (6): Cyclooctatetraene is not planar and has 8 &
corresponding alkyl halide. So correct order is : electrons.
A>C>B. (d): Cyclopropenyl anion has 4n electrons. Therefore,
(b) and (d) are not aromatic.
NCERT Exemplar Problems : MCQs Type-II — 19. (b,c):
11. (c,d): In which CH, does not undergo complete Hy i CH,;CH, «, 2 CH.CH,
combustion to give CO, and H,O. C=C So=
12. (c,d): The alkenes which have two substituents on each H~ “ScHCH, a” a
carbon atom of the double bond give mixture of u> 0 p>O
ketones on ozonolysis, i.e., (¢) and (d). trans-Pent-2-ene cis-Hex-3-ene

WWW.JEEBOOKS.IN
a 13/152 MODERN'S abc + OF CHEMISTRY
-XI

Unit Practice Test EXamivatvon

Time Allowed: 1 Hr. Maximum Marks: 25

1. Complete the reaction : (1)

CH,CH, COOH + HI —>

2. Which of the following has the highest boiling point ?


2-Methylpentane or 2,2-dimethylbutane (1)
3. Write the IUPAC name of \/\ (1)
4, Will the compound CH,—CH=CCl(CH,) show geometrical isomerism ? (1)
5. Which of the two : cyclopentadienyl anion or cyclopentadienyl cation is not aromatic ? (1)
6. Give two chemical tests to distinguish 1-pentene from n-pentane. (2)
7. Write structural formulas and [UPAC names of all possible isomers having molecular formula
CH, and one triple bond. (2)
8. Why is Wurtz reaction not preferred for the preparation of alkanes containing odd number of
carbon atoms? Explain with one example. (2)
9. Addition of HBr to CH,—CH—CH, gives 2-bromopropane while in the presence of benzoyl
peroxide, the same reaction gives 1-bromopropane. Explain and give mechanism. (3)
10. Complete the following reactions :
(i) C,H, + CH,Cl, anhyd. AlC1,—>

) CH.CeCCH (i) Os
(it) CH;C=CCH; —(Gzn, H20

(ii) CH,CH,C =CH SS, (3)


11. What is meant by electrophilic substitution reaction? Explain the mechanism of nitration of
benzene. (3)
12. (a) How will you convert benzene to following :
(i) m-nitrochlorobenzene
(ii) acetophenone
(iii) p-nitrotoluene
(b) Why is benzene extra ordinarily stable though it contains three double bonds? (5)

To check your performance, see HINTS and SOLUTIONS to some questions at the end of
Part II of the book.

WWW.JEEBOOKS.IN
aes
a ENVIRONMENTALD,
POLLUTION
0) 5510 Os MAYA DN
Building on..... Assessing..... Preparing for Competition.....
@ Understanding Text 1 @ Quick Memory Test with ¢@ Additional Useful Information 36
Conceptual Questions 27 Answers 33g Topiewise MCQs 37
CHAPTER SUMMARY & QUICK REVISION EXERCISES 34 % Competitive Examination Qs
CHAPTER ROUND UP 28 aaa VAR AIPMT & Other State Boards’
NCERT FILE Medical Entrance 38
¢ In-text Qs & Exercises with >» JEE (Main) & Other State Boards’
Solutions 30 Engineering Entrance 39
@ NCERT Exemplar Problems with @ NCERT Exemplar Problems
Answers & Solutions (Subjective) 31 (Objective Questions) Al
@ Hints & Explanations for Difficult
Questions 43

T here are a number of substances (toxic as well as non-toxic) which


are being added to our surroundings or environment by natural events
and human activities. These are in fact, the undesirable consequences
of modern civilization, increased use of natural resources, increasing
population, advancement in industries, increase in number of
SOLAR RADIATION vehicles, etc. These substances which are continuously going
CO(50%) into air, water and soil bring about undesirable changes in
physical, chemical and biological characteristics of our
environmentand adversely affect the life processes of animals
and plants.
The branch of science which deals with the chemical
silos MESOSPHERE phenomena occurring in the environment is called
O, + hv (220-330 nm)+ 0,+ 6 environmental science.
This deals with the study of the origin, transport,
STRATOSPHERE
reactions, effects and consequences of chemical species in
N,,0.,
the environment. The contamination of any part of the
CO,,H,o TROPOSPHERE environment is called pollution and the substances which
cause pollution are called pollutants. The pollutants can cause
air pollution, water pollution or soil pollution.
ENVIRONMENTAL POLLUTION
Environmental pollution is the effect of undesirable
changes in our surroundings (air, water or land) that have
harmful effects on human, animal and plant life as well as on materials. Pollution may be natural or man
made. It can be classified according to the components of environment being damaged. These are :
(i) Air pollution
(ii) Water pollution
(iii) Soil and) pollution.
A substance which causes pollution is called pollutant. The pollutants can be solid, liquid or gaseous
substances. These can be degradable like discarded vegetables and fruits which rapidly break down by natural

ant
WWW.JEEBOOKS.IN
| 182 MODERN'S abe + OF CHEMISTRY-XI

processes. The pollutants can also be non-degradable so it is a contaminant. But because of its dangerous
such as aluminium pieces, mercury, DDT etc. which effect, it is also regarded as a pollutant.
remain in the environment in an unchanged form for Generally, highly toxic substances are considered
many decades. In general, as pollutants.
pollutants are the substances made by us,
3. Source
used by us and even thrown by us as waste
The site from which the pollutants or
products which pollute the environment
contaminants come is called source. Every
directly or indirectly in one way or other.
pollutant originates from a source. The source is
Do you know that an average human being needs particularly important because its knowledge helps to
about 12-15 times more air than the food. So, develop the methods to eliminate pollutants.
even a small amount of pollutants in the air 4. Sink
become significant compared to similar levels
The material or medium which consumes
present in the food.
or interacts with a long lived pollutant is
Can you believe, man is the principal source of called sink. For example, a marble wall acts as a sink
pollution ? for atmospheric sulphuric acid because of the reaction :
CaCO, + H,SO, ——~> CaSO, + H,O + CO,
SOME COMMONLY USED TERMS
Some commonly used terms in environmental The oceans are sinks for atmospheric carbon
chemistry are : dioxide and other water soluble gases.
1. Pollutant
Ground water and subsoil water also act as sinks
for pesticides used in agriculture.
When the concentration of a substance
already present in nature or of a new substance 5. Receptor
increases to undesirable proportions causing Anything that ts affected by the pollutants is
danger to human beings, other animals or called receptor. For example, human beings are the
vegetation and other materials, the substance receptor of photochemical smog causing irritation in
is treated asa pollutant. The pollutants spoil the eyes and breathing problems.
environment and are harmful to living organisms and 6. Speciation
other materials . The common pollutants are: Speciation means the identification of
(i) gases like carbon monoxide, sulphur dioxide, different chemical forms or species of an element
oxides of nitrogen, etc. or a compound present in the environment. For
(ii) compounds of metals like lead, mercury, zinc, example, mercury may be present in the environment in
cadmium, arsenic, etc. various forms such as elemental mercury, mercury salts,
(iit) pollen grains, dust organometallic mercury species; CH,Hg*, (CH,),Hg. It
(iv) pesticides and detergents is very essential to identify the chemical species of the
(v) sewage and pollutants because some species may be more toxic than
(vi) radioactive substances. others and need special care. For example, alkyl mercury
It may be noted that normally highly toxic derivatives [CH,Hg* (CH,), Hg] are highly poisonous as
substances are considered as pollutants. However, compared to other species of mercury.
the substances which are normally harmless can cause 7. Threshold Limit Value (TLV)
pollution if they are present in undesirable
This indicates the permissible limit of a
concentrations. For example, nitrate is added to soil
pollutant toxic in atmosphere to which a healthy
as a fertilizer to increase growth of the plants, but an
worker is exposed during 8 hours a day or 40
excessive concentration of nitrate present in drinking
hours a week for life time without any adverse
water can be harmful especially to young children.
effects. TLVs are determined by experimentation on
2. Contaminant animals, medical knowledge and experience,
A substance which does not occur in nature epidemiology surveys and environmental studies.
but is introduced by human activity into the
For example, TLV of CO is 50 ppm and that of
atmosphere affecting tts composition is called
CO, is 5000 ppm. But TLV for a poisonous gas phosgene
contaminant, It may or may not be harmful to the
is only 0.1 ppm.
living organisms or non-living components.
In an environment pollution process, a pollutant
The contaminant is classified as pollutant when originates from a source. It is transported by air or
it hassome harmful effect. For example, water or is dumped on land by man. Some of the
pyrosulphuric acid (H,S,0,) leaked from a defective pollutants may be absorbed (assimilated) or chemically
tank killed many persons and caused skin and changed by the sink. The remaining pollutants build
breathing problems to many persons in Delhi. Since up to harmful concentrations and affect the receptor.
pyrosulphuric acid does not occur in the atmosphere, This is shown in Fig. 1.
ENVIRONMENTAL POLLUTION
143
major gases (N,, O,, CO,, water-vapour and noble
gases). It has decreasing temperature with increasing
Pollutant Build up of =| altitude from the earth surface with a minimum around
pollutant
—56°C at the maximum height of the troposphere.
However, on certain occasions, the temperature may
increase to some extent with altitude instead of
“Damage to » decreasing. Such a situation is called temperature
Source of —————— belcaso inversion and is due to trapping of a layer of cold air
| Assimilation by
pollutants sink
by a warmer upper layer.
Fig. 1. A schematic representation of an environmental With increasing altitudes from the earth’s surface,
pollution process. the other regions are stratosphere (11— 50 km),
mesosphere (50 — 90 km) and thermosphere (90 — 500
CHEMICAL REACTIONS INATMOSPHERE km). The stratosphere contains nitrogen, oxygen and
The gaseous envelope surrounding the earth is ozone.
known as atmosphere. The two major components of In stratosphere, the temperature rises with
dry and clean air in the atmosphere (by volume) are increasing altitude showing a maximum of —2°C. This
nitrogen (78.09%) and oxygen (20.95%). Argon (0.934%) phenomenon is due to the presence of ozone which may
and carbon dioxide (0.034%) are the minor components absorb ultraviolet radiation and raises the temperature
of the atmosphere. Air can hold water vapour from 0.1 in this region. This act of ozone protects us from the
to 5% by volume and also contains trace of elements harmful effects of sun’s ultraviolet radiation. This
such as noble gases (neon, helium, krypton, xenon), region contains N,, O,, O, and O-atoms. Due to the
hydrogen, methane, sulphur dioxide, ammonia, ozone, presence of ozone layer, this layer is also called
etc. ozonosphere.
The atmosphere surrounding us may be divided The region beyond stratosphere is dominated by
into four regions : positive ions such as O3, O* and NO} in addition to
() troposphere N, and O, and is called mesosphere. This extends upto
(a) stratosphere
(ui) mesosphere Above 90 km, the region is called thermosphere
(tv) thermosphere. which contains ions such as Oj, O*, NO* and free
About 80% of the total mass of air and almost electrons.
all of the water vapours of the atmosphere is found in
The regions mesosphere and thermosphere are
the inner layer known as troposphere which extends
collectively called ionosphere because these contain
to 8 to 12 km above the earth’s surface. This is the lonized gases.
most important region of the atmosphere because it
is the domain of all living organisms including animals The different regions of the atmosphere are
and plants. This has homogeneous composition of shown in Fig.2.

SOLAR RADIATION
High energy
500 km ultraviolet
_ O,',0°",NO" ae A< 100 nm
THERMOSPHER penetrates to ~ 200 km

Infrared Ultraviolet
90 km" i. (200 — 300 nm)
visible, ultraviolet
O,",NO
+ +
A. > 330nm penetrates to
MESOSPHERE 50 km
penetrates to
earth's surface 50 knV’6, + hv (220-330 nm)> 0,+0 ——
The region beyond
stratosphere
O; STRATOSPHERE contains lon and is
| km N.,O, also called
ionosphere
_CO,,H,O TROPOSPHERE

Fig. 2. Major regions of atmosphere.


| 184 MODERN'S abe + OF CHEMISTRY-XI

TYPES OF ENVIRONMENTAL POLLUTION Air Pollutants


The environmental pollution may be of the There are two types of air pollutants :
following types : 1. Primary air pollutants
1. Atmospheric pollution 2. Secondary air pollutants.
2. Water pollution 1. Primary air pollutants
3. Soil or land pollution. A primary air pollutant is a harmful chemical
The atmospheric pollution is generally studied substance that directly enters the air as a result of
under two heads : natural events or human activities. The primary
(a) Tropospheric pollution pollutants are :
(6) Stratospheric pollution. (t) Carbon oxides (CO and CO,)
Tropospheric pollution is much more widespread (ii) Nitrogen oxides (NO)
and we all are concerned about it. Since only a few (vii) Sulphur oxides (SO,)
substances produced by human activity reach the (tv) Hydrocarbons
stratosphere, its pollution results in the depletion of (v) Suspended particulate matter.
ozone layer, which may result into very serious problems. 2. Secondary air pollutants
AIR POLLUTION A secondary air pollutant is a harmful chemical
Air is never found clean due to natural and that forms in the air due to a chemical reaction
man-made pollution. Gases such as carbon monoxide, between two or more air components or a primary
nitrogen oxides, sulphur oxides, hydrogen sulphide and pollutant and one or more air components. For
many volatile organic compounds are continuously example, sulphur dioxide is a primary pollutant in air.
released into the atmosphere due to natural and human It reacts with oxygen gas in the atmosphere to form
activities. Air pollution oceurs when the the secondary pollutant sulphur trioxide (SO,).
concentration of a normal component of the air or Bos + O, —— 250,
a new chemical substance added or formed in air Primary pollutant Secondary pollutant
builds up to undesirable proportions causing
Even, the sulphur trioxide formed may react with
harms to humans, other animals, vegetation and
the water vapour in air to form sulphuric acid.
materials.
20, Bw =<——> HU;
The chemical substance causing pollution is
called air pollutant. About 90% ofair pollution problems Sulphuric acid is, therefore, also a secondary
are caused by the pollutants such as pollutant.
carbon monoxide, sulphur oxides, volatile organic The common secondary pollutants are :
compounds, nitrogen oxides and suspended particulate SO,, H,SO,, NO,, N,O, HNO,, H,O,, O,, nitrate
matter (such as carbon, dust, pollen, metal, etc.) as and sulphate salts, etc.
shown in Fig. 3. Pollutants can also be classified as:
(t) Biodegradable pollutants and
(it) Non-biodegradable pollutants
CO(50%)
({) Biodegradable pollutants. These are the
materials which are easily decomposed by
particulate the microorganisms either by the nature itself
or by suitable treatment so that their degraded
products donot cause any serious effects on
Volatile
the environment. For example, domestic
organic compounds sewage, cow-dung etc.
Fig. 3. Major air pollutants (approximate bundance). (it) Non-biodegradable pollutants. These are
Sources of Air Pollutants the matertals which donot undergo
degradation or degrade slowly or partially
The air pollution is caused by natural activity
and human activity. The natural sources of air and their presence even in very small amounts
pollution are volcanic eruptions, forest fires (caused inthe environment ts very harmful for humans
by lightning), pollen dispersal, evaporation of volatile as well as plants life. These pollutants not
organic compounds from leaves, bacterial only accumulate in the environment but are
decomposition of organic matter, wind erosion of soil often biologically magnified because they
and natural radioactivity. move in biochemical cycles and along food
The main human activities responsible for chains. For example, aluminium, mercury,
polluting the air are burning of fuels (coal, oil, gasoline, D.D.T., plastic materials, many chemicals,
etc.) In power and industrial plants and in motor nuclear wastes etc.
vehicles and industrial processes and industrial The main air pollutants and their major sources
wastes. are given in Table 1.
ENVIRONMENTAL POLLUTION
145
Table 1. Some common air pollutants and their living beings because of its ability to block the delivery
major sources. of oxygen to the organs and the tissues.
Pollutant Major pollutant source
Sources of CO in air
The main sources of air pollution of CO are the
Transportation, autoexhaust, petroleum, automobile engines and defective furnaces. Some of
paper industries, iron and steel industries, the sources of CO in air are:
solid waste disposals.
(t) Incomplete combustion of carbon, fossil fuel or
Fuel combustion and industrial processes.
carbon containing compounds.
Metallurgical operations, combustion of
coal and oil, paper, manufacture of H,SO,.
2C +O, ——> _2CO
Fertilizer industries, explosive industries, For example, the complete combustion of butane
manufacture of HNQO,. requires 6.5 moles of O, per mole of butane. If only 6.0
Petroleum refining, coke ovens, natural moles of O, are present, then 1 mole of CO and 3
gas plants. moles of CO, are formed.
Industrial processes such as paper, 13 3 ee
oni: ee (g) + io {),, (o) — > 4CO, (g) + 5H,O (L)
plastics, dyes, chlorinated hydrocarbons.
Natural biological processes, industrial C,H,, + 60, (g) —> 3CO, (g) + CO®) + 5H,0 (2)
processes, ammonia works.
Hydrocarbons] Petroleum refineries, chemical processes, (ii) Reaction of carbon dioxide and carbon
rubber manufacture, transportation, solid containing compounds at high temperatures in
waste disposal. industrial processes such as blast furnace.
Particulates Volcanic eruptions, metallurgical operations
such as smelting and mixing, incomplete
CO, + C——>2CO
combustion in industrial processes. (itt) Dissociation of carbon dioxide at high
temperatures :
“Usually CO, is not considered as pollutant.
Other air pollutants are metals like Be, Ba, Cd, 2CO, == 2CO + O,
Fe, Mn, Pb, Hg, Ni, Zn (from mining and metallurgy), Some natural processes such as volcanic activity,
non-metals such as As, P, Se (from combustion of natural gas emission, electrical discharge during
industrial fuels, fertilizers) radioactive substances
storms, marsh gas production, seed germination, etc.
(from nuclear power plants).
release some CO in the atmosphere.
Industrial accidents can also play havoc for the The main contribution of CO in air pollution is
environment. For example, an industrial accident from human activity. Of the total carbon monoxide
occurred in the Union Carbide factory on December content in atmosphere, about 74% is contributed by
3, 1984 in Bhopal. It released a large scale of automobile exhaust (motor vehicles, aircraft, rail,
poisonous MIC (methyl isocyanate) and caused road). About 16% is contributed by forest fires and
excessive damage to all forms of life. agriculture burning (burning of forest debris, crop
residues, weeds and other vegetation). Industrial
TROPOSPHERIC POLLUTION
processes mainly iron and steel industries, paper,
The tropospheric pollution occurs because of
petroleum industries contribute about 9.6% of CO in
the presence of undesirable solid or gaseous particles
atmosphere.
in air. The pollutants may be broadly classified into
two major types : Sinks of CO
1. Gaseous air pollutants. These include oxides Though human activities are releasing CO in
of sulphur, nitrogen and carbon, hydrogen sulphide, atmosphere and it is expected to double its
hydrocarbons, ozone and other oxidants. concentration in surrounding atmosphere every
2. Particulate pollutants. These are dust, fumes,
5 years. However, the actual increase in CO in
mist, spray, smoke, ete.
atmosphere is less because of some natural sinks.
These are briefly discussed.
SOME COMMON AIR POLLUTANTS (i) Soil is the major sink for CO. Some
Although there are many sources of air microorganisms in soil remove CO from the atmosphere.
pollutants, the common pollutants come from (ii) Some of the species in atmosphere such as
combustion processes. Let us discuss the major air hydroxyl (HO) and perhydroxyl (HOO) radicals, atomic
pollutants. oxygen and ozone help in the oxidation of CO to CO,
A, GASEOUS AIR POLLUTANTS and consequently, remove it from the atmosphere.
1. Oxides of Carbon Harmful effects of Carbon monoxide
(a) Carbon monoxide (CQ)
1. Effects of CO on human health
It is one of the most serious air pollutant. It is a
colourless, odourless and tasteless gas above 81 K. It is Carbon monoxide is very toxic because it
heavier than air and is not soluble in water. It is one of combines with haemoglobin in the blood preventing
the most serious air pollutants. It is highly poisonous to
S fias,
its function as an oxygen carrier. It is particularly
MODERN'S abc + OF CHEMISTRY-XI

of internal combustion engine consists of a mixture


dangerous because it is odourless and therefore, is of CO, oxides of nitrogen (NO_), hydrocarbons
not easily detected. Haemoglobin (Hb) in the blood (H,) and particulate matter. The variation of
combines with oxygen and forms oxyhaemoglobin by fuel-air ratios has significant effect on the
a reversible process : constituents of exhaust gases. It has been observed
Hb + O, = HbO, that as the air-fuel ratio increases (leaner in
fuel), the concentration of CO decreases.
Oxyhaemoglobin formed in the lungs is carried to Therefore, CO-emissions may be lowered by using
the different cells in the body and gives oxygen. a cleaner air-fuel mixture. For example, when
However, carbon monoxide combines more readily air-fuel ratio (wt/wt.) is more than about 16: 1 in
than O, with haemoglobin of blood to form carboxy internal combustion engines, they emit virtually no
haemoglobin which is about 300 times more stable CO.
than the oxygen-haemoglobin complex. Therefore, Some of the important attempts which have been
when CO is enhaled along with air, it combines with made for the control of CO pollution are given
haemoglobin to form a stable compound carboxy below :
haemoglobin, (HbCQ). (i) Internal combustion engines are modified
Hb + CO ——> HbCO to reduce the amount of pollutants formed during
fuel combustion. The presence of excess air ensures
As a result, the amount of haemoglobin for complete combustion of CO and hydrocarbons to
transport of O, decreases and it reduces blood’s capacity CO,. Automobile carburettors are adjusted and
for carrying oxygen. When the concentration of cleaned regularly to the proper fuel-air ratios.
carboxyhaemoglobin in blood reaches about 3-4 (ii) Some exhaust system reactors are
percent, the oxygen carrying capacity of blood is greatly developed which will complete the combustion
reduced. process and decrease the amount of CO.
This deficiency of oxygen results into many (iit) Catalytic converters can be fitted into
harmful effects such as reduction in awareness and exhaust pipes to cut down the CO emission. These
judgement, dizziness, weak eye sight, headache, anti-pollution catalysts eliminate the pollutants
nervousness and cardiovascular disorders. from exhaust gases before they are discharged
At higher concentrations of CO, suffocation, loss into the atmosphere.
of consciousness or even death after several hours may (iv) Substituted fuels for gasoline which will
occur. This is the reason why pregnant women who give low concentration of pollutants upon
have the habit of smoking are advised not to smoke as combustion are being tried. Natural gas in
increased CO level in blood may induce premature compressed form (CNG) and in liquefied form
birth of the baby, spontaneous abortions or deformed (LPG) has been used as a fuel. Though there are
pollution free fuels, yet they are less economical
babies. It is so poisonous that within halfan hour, 1300
and difficult to store and distribute.
ppm is fatal.
(v) Attempts are being made to develop
— Because of harmful effects of carbon pollution free energy sources such as electric power,
monoxide, it is dangerous to sleep in a closed room gas turbines, solar cells etc. However, these are
with a coke fire burning inside on a cold winter less economical as compared to gasoline.
night. When coke burns in an insufficient supply
of oxygen (due to closed room), a lot of carbon (6) Carbon dioxide (CQ,)
monoxide is produced. This can cause poisoning Carbon dioxide is a normal and essential
and may lead to death. component of the atmosphere. Animals exhale it.
Therefore, it is vital to all forms of plant and animal
Effects of CO on Plants life. It is normally not a pollutant. It is not an
Carbon monoxide has detrimental effects on important contributor to acid rain. It can cause acidity
plants when exposed for longer times. It inhibits the when it dissolves in water, but it is relatively less
nitrogen fixing ability of bacteria. It also affects leaf soluble in water than the oxides of nitrogen and
drop, leaf curling, decrease in leaf size and premature sulphur. Normally, it forms about 0.03% by volume of
ageing of the plants. the atmosphere. However, human activities are
changing the balance established by nature's carbon
Control of CO cycle. With the increased use of fossil fuels, a large
The major sources of CO air pollution are amount of carbon dioxide gets released into the
human activities. The internal combustion engine atmosphere. As a result, the concentration of CO, in
is the primary source of CO emission. The exhaust the atmosphere is increasing to an alarming extent.
ENVIRONMENTAL POLLUTION
47
Therefore, there is need to keep balance of CO, in the The irritant red haze in traffic and congested
atmosphere. places is due to oxides of nitrogen.
Sources Sources of NO M
The main sources of CO, in atmosphere are : Dinitrogen and dioxygen are the main constituents
(4) Carbon dioxide is released into the air by ofair. These gases donot react with each other at normal
respiration. temperature. But at high altitudes when lhghtening
(ii) Incomplete combustion of fossil fuels and strikes, they combine to form oxides of nitrogen.
carbon containing compounds.
The important sources of NO and NO, are given
C+0,——> Co, below :
(iit) Many industrial processes also produce a
(i) The two oxides of nitrogen NO and NO, come in
lot of carbon dioxide. For example, decomposition of
limestone during manufacture of cement releases lot atmosphere from the combustion at high temperature
of carbon dioxide into the atmosphere. (above 1483K) in automobile engines and thermal
(iv) Carbon dioxide is also produced by gvenerating stations. The basic reactions leading to the
biological decay of plants. formation of these oxides of nitrogen are :
(v) Carbon dioxide gas is also emitted during 1483K
volcanic eruptions.
N,(g) + O,(¢) == 2NO@)
Sinks of CO,
NO reacts instantly with oxygen to give nitrogen
The main sinks for CO, are natural processes.
Ocean is an important sink of carbon dioxide. It
dioxide.
contains most of the dissolved CO, in the form of 2NO(g) + Og) ——_ 2N0,,(g)
carbonates, bicarbonates and organic compounds. Rate of production of NO, is faster when nitric
The carbon dioxide of the atmosphere is also oxide reacts with ozone as in the stratosphere.
removed from the atmosphere by photosynthesis
NO(g) + O,(g¢) ——> NO,@) + O,(g)
process of plants. In a balanced ecosystem, the
additionally released CO, in the atmosphere is (it) Many industrial processes also release oxides
regularly removed by green plants for the of nitrogen in the atmosphere.
photosynthesis process and they in turn emit oxygen, (vii) The natural sources of oxides of nitrogen are
thus maintaining the cycle. bacterial action in soil, forest fire and lightning.
Effects of CO, Sinks of NO,
Normally carbon dioxide is harmless and is Many natural processes acts as sink for oxides of
not a pollutant. However, the increasing nitrogen. These oxides are inherently unstable and
concentration of CO, may affect the atmosphere, decompose to N, and O, after some time. Therefore,
causing undesirable change in climate. This effect is the concentration of nitrogen oxides in the atmosphere
known as greenhouse effect (discussed later). tends to remain low. Some natural processes and
Control of CO, photochemical reactions also take care of NO, in the
The major sources of CO, are vehicles, plants atmosphere and serve as sinks. The end product of
and industries while the sinks are ocean and these reactions is HNO,. The probable mechanism for
plants. The balance can be maintained by the the change of NO, to nitric acid are :
following steps : (i) 2NO +0, ——> 2NO,
(1) The release of CO, from various industrial 3NO, +H,O ——> 2HNO, + NO
processes should be controlled.
(ii) The production of CO, from vehicles
4NO,+ 2H,O +0, ——> 4HNO,
exhaust should be checked. (ii) NO, +0, ——> NO,+0,
(iii) To adjust the balance of CO, in atmosphere, NO, +NO, ——> N,0O,
the main sink of CO, i.e., forests should be developed. N,O, +H,O ——> 2HNO,
2. Oxides of Nitrogen (NO, ) The nitric acid acts as a temporary sink and it
A number of oxides of nitrogen (NO_) such as comes 1n the form of nitrate salts in acid rainfall or in
nitrous oxide (N,O), nitric oxide (NO), nitrogen dust.
dioxide (NO,), N,O, (dinitrogen trioxide) and N,O, Harmful effects of NO.
(dinitrogen pentoxide) are entering the atmosphere The irritant red haze in the traffic and congested
due to natural sources and human activities. Out of places is due to the oxides of nitrogen. The main
these, only two oxides, NO and NO, are considered harmful effects of oxides of nitrogen are :
as pollutants. 1. NO is biologically less active and less toxic
Nitric oxide (NO) is colourless, odourless gas and than NO, and therefore, does not have a significant
nitrogen dioxide (NO,) is reddish brown gas having adverse effect on human health even at high
pungent suffocating odour. concentrations. However, it is oxidised by oxygen or
a MODERN'S abe + OF CHEMISTRY-XI

ozone to nitrogen dioxide which is extremely toxic to 3. Oxides of Sulphur (SO_)


living tissue. Like CO, NO binds to haemoglobin and Oxides of sulphur are probably the most harmful
decreases oxygen transport efficiency of blood. of the common gaseous pollutants. Sulphur dioxide is
2NO(g) + O,(g) —> 2NO,(g) the main pollutant among sulphur oxides. It is a
NO(g) + O,(g) —> NO,g) + O,(g) colourless gas with a pungent odour. It is generally
accompanied by little of SO, obtained by partial
2. The oxides of nitrogen cause damage to the
oxidation of SO,,.
leaves of plants. Exposure of plants to NO, causes leaf
spotting and breakdown of plant tissues. Excessive 250,(g) + O,(g) ——_—- 250 ,,(g)
concentration (10 ppm) of NO causes decrease in the Under humid conditions, SO, reacts with water
rate of photosynthesis. vapour to form H,SO, droplets.
3. The sunlight reacts with nitrogen dioxide to §O.(g) + HOU) -_—
4a sOK(t)
produce highly active oxygen atoms. Sources of Sulphur dioxide
No, —s"t
, NO +0 Some of the important sources of sulphur dioxide
The active oxygen attacks traces of hydrocarbons in atmosphere are :
in the air and produces irritants which is called (1) Most of the sulphur dioxide in atmosphere
photochemical smog. This is a health hazard. originate through the combustion of fossil fuels which
contain some sulphur.
4, Acute exposure to NO, can also cause damage
(ii) The atmosphere contains H,S by decay of
to human health. Nitrogen dioxide is a lung irritant
organic matter. The H,S gets readily converted into
and can lead to acute respiratory disease in children.
SO, by the following reaction :
5. The nitrogen oxides are harmful to various 3
textile fibres and metals. These also cause fading of H,S + 50, ——> SO, +H,O
dyes used in textiles. (it) Large amounts of SO, are added to the
Control of Nitrogen oxides atmosphere during ore roasting processes of sulphur
The important factors in the control of oxides of ores. For example, SO, is produced during roasting of
nitrogen are : metallic sulphide ores such as pyrites (FeS,), copper
(i) NO in vehicular exhausts is converted into glance (Cu,S), galena (PbS), zinc blende (ZnS), etc.
N, and traces of NH, with the help of some catalyst 2Zn5 +30, ——> 22Zn0 + 280,
(platinum). Zinc blende
(it) Nitrogen oxides in fuel gases can be removed 38Cu,5 + 30, ——> 6Cu + 350,
by alkaline scrubbing solutions using Ca(OH), and
Copper glance
Mg(OH),.
(iv) Voleanic sources also contribute significantly
(iii) The nitrogen oxides can also be removed by
to the SO, in atmosphere.
acidic scrubbing. This process also removes SO,. This
involves the following steps : The combustion of any sulphur containing
material results in the formation of SO, and SQ,.
(a) The flue gases coming from power plants or
Sulphur dioxide is the major constituent (about 97%)
industrial units contain NO, NO, and SO,. These are
of the mixture.
introduced into an oxidizer in which the following
reaction occurs : Sinks of SO,
NO,+ SO, + H,O -——~+ H,SO, + NO Sulphur dioxide in air can undergo several
reactions and therefore, can be removed.
(b) The NO and NO, react to form N,O, which is A part of SO, combines with water droplets to form
scrubbed by H,SO, in a scrubber.
sulphurous acid.
NO, +NO —— N,O,
SO, + H,O ——> H,SO,
N,O, + 2H,SO, ——> 2NOHSO, + H,O
It can also be oxidised to sulphur trioxide in air
The scrubbed flue gas is released to the but uncatalysed oxidation of sulphur dioxide is slow.
atmosphere. But because the polluted air often contains particulate
(c) The reaction product, NOHSO, is then matter, it catalyses the oxidation of sulphur dioxide to
decomposed in a separate chamber called decomposer sulphur trioxide.
and the resulting H,SO, is recycled to the scrubber. 250, +O, ——> 280,
The reaction can also be promoted by ozone and
2NOHSO, + =O, + H,O ——> 2H,SO,+2NO,
hydrogen peroxide.
(d) NO, is then converted to HNO, in a reactor. 280,+0, ——> S0,+0,
3NO, + H,0 ———> 2HNO, + NO Sulphur trioxide combines with water vapour to
Excess of NO and NO, is recycled into the oxidiser. form droplets of sulphuric acid.
ENVIRONMENTAL POLLUTION
149
SO, +H,O ——\ H,S0, —— (H,5S0O,), the method is economical but it poses a problem of
Aerosol waste disposal.
droplet (6) Sulphur dioxide produced may be passed
The sulphuric acid comes down in the form of through a lime slurry. This process is known as lime
acid rain and gives rise to a phenomenon known as slurry scrubbing.
acid rain (discussed later on). Ca(OH), + 50, ——+ CaSO, + H,O
Effects of SO, The waste (CaSO,) from this method can be used
Sulphur dioxide and its compounds (formed from to prepare gypsum by oxidation of CaSQ,,.
SO, such as SO,, H,SO, or sulphates) are dangerous
1
air pollutants. These have dangerous effects on human CaSO, + 50,+ 2H,O> CaSO,. 2H,0
life. Some of the harmful effects are :
(c) Sulphur dioxide may be removed by first
1. SO, effects respiratory tract producing nose,
treating with sodium hydroxide and then with calcium
eye and lung irritation. It has been reported that
hydroxide as:
lower concentration of SO, causes respiratory diseases
e.g., asthama, bronchitis, emphysema in human 2NaOH + SO, ——> Na,SO, + H,O
beings. It also causes irritation to the eyes resulting Ca(OH), + Na,SO, ——> CaSO, + 2NaOH
in tears and redness. High concentration of SO, may In this method, the expensive sodium hydroxide
cause lung diseases and lung cancer. SO, has been solution is regenerated with inexpensive lime.
considered the most serious single air pollutant (iii) Alternative energy sources such as
causing many health hazards. hydroelectric plants and nuclear power plants may be
2. Atmospheric SO, is also harmful for plants. It used in place of thermal power plants using burning
damages vegetable crops and affects plant growth of coal (containing sulphur). However, these are not
and nutrient quality of plant products. The exposure economical.
of plants to SO, causes damage to leaf tissues and
causes chlorosis (a bleaching or yellowing of normally 4, Hydrocarbons
ereen portions of the leaves). In areas, having high Many hydrocarbons of low molecular mass are
levels of SO, pollution, and formation of H,SO,, may gases or volatile liquids and are present in air at
damage the plants. ordinary temperature. Among these, methane is the
most abundant hydrocarbon pollutant.
3. The marble and limestone, both forms of
calcium carbonate (CaCO,) are attacked by SO, and Sources of hydrocarbons
H,SO,. These react as : (t) Hydrocarbons are formed by incomplete
CaCO,+ H,SO, ——> CaSO,+ CO, + H,O combustion of fuel used in automobiles.
(it) Methane is released into the air in large
This causes deterioration of buildings, statues,
quantities by the anaerobic decomposition of
etc.
organic matter present in soil, water and
4. The oxides of sulphur cause fading and
sediments.
deterioration of fabric, leather and paper.
5. They affect the colour of the paints.
2CH,O —““— CH, + CO,
Organic matter
Control of SO, pollution
(iii) Evaporation of organic solvents such as
The various methods to control SO, pollution
benzene, toluene, etc. during industrial
are given below :
operations add to hydrocarbon pollution.
(4) A number of processes are being used to
(iv) Some hydrocarbon pollution also takes place
remove sulphur from fuels before combustion.
due to burning of fuels such as coal, wood,
(it) Sulphur dioxide may be removed from flue kerosene, etc.
gases. It can be easily eliminated by using chemical
(v) Some trees also emit large quantities of
scrubbers.
hydrocarbons into the air.
Some of the methods to remove SO, chemically
Effects of hydrocarbons
from the flue gases are :
(4) Hydrocarbons are carcinogenic and therefore
(a) The most widely used method is limestone
treatment in which finely divided limestone is
cause cancer.
directly introduced to the boiler. In this method, heat (ii) They harm plants by causing ageing,
calcines the limestone forming quick lime. Quicklime breakdown of tissues and shedding of leaves,
reacts with SO, in the flue gas forming calcium sulphate. flowers and twigs.
(iit) Due to their photochemical reactions with
CaCO, — > CaO +CO,(g) oxygen and oxides of nitrogen, they form
CaO + SO, + =0,—> CaSO, photochemical oxidants and photochemical
smog. This has very hazardous effect on the
Solid CaSO, is removed and disposed off. However, human being and plants.
Tian B. PARTICULATES IN AIR POLLUTION 3. Fumes
MODERN'S abe + OF CHEMISTRY-XI

1. Particulates These are condensed vapours. For example, fumes


The small sized solid particles and liquid are generally produced by the condensation of vapours
droplets which range in size from 2 x 10-!° m (0.0002 during sublimation, distillation, boiling and several
um) to 5 x 10-4 m are collectively called as other chemical reactions. Generally, organic solvents,
particulates. These particles are usually individually metals and nonmetallic oxides constitute fumes.
not visible to the naked eye. However, small particles 4, Dust
often collectively form a haze that restricts the The solid particles (over Inm in diameter)
visibility. The particulate matter in the range 0.001 suspended in the air are called dust. These particles
to 10 mm range is commonly visible as suspended in enter into atmosphere from natural, domestic and
the air near the sources of pollution. industrial sources. These enter the atmosphere by
These particulates include carbon (soot), metals, volcanic eruptions, blowing of dust by wind, mining
metal salts, cement dust, fly ash, hydrocarbons, operations, crushing and grinding, etc. The non-viable
soil dust, pulverized coal, asbestos dust, mineral dust particulates in the atmosphere consist of ground
particles, etc. limestone, sand tailings from floatation pulverised coal,
The particulates in the atmosphere may be cement, fly ash and silica dust.
viable or non-viable. The viable particulates are These above types of particulates consist of
the small living organisms which are dispersed in the different types of metals and their compounds and
atmosphere. These include bacteria, moulds, fungi, organic particulates as discussed below :
algae, etc. Some of these viable particulates cause (a) Metal particles. These are released into the
allergic reactions on human beings. Fungi can also atmosphere by various metal finishing processes. The
cause plant diseases. common metallic particles in air are lead, mercury,
Non-viable particulates are formed either by chromium, arsenic, zinc, cadmium, nickel, iron, etc.
the breakdown of large materials or by the
Although these metallic particles are present in air in
condensation of minute particles and droplets.
very small amounts yet some of these are very
Types of Particulates and their Sources
dangerous to living organisms.
There are four types of non-viable particulates
(b) Metal oxides. Metal oxides are released into
in the atmosphere injected into the atmosphere
the atmosphere during combustion of fuels containing
through human activity.
1. Smoke metallic compounds. For example, particulate Fe,O,
These are very small soot particles. These consist is produced by the combustion of coal containing iron
of solid or mixture of solid and liquid particles formed pyrite as :
during combustion of organic matter. Oil smoke, 3FeS, + 80, —— Fe,0O, + 6SO,
tobacco smoke and carbon smoke from burning fossil Power plants using vanadium rich residual oil
fuels, garbage and dry leaves are typical examples of are sources of V,O. pollutants.
this type of particulate materials. Their common
(c) Metal salts. Burning of coal gives CaCO, in
sources are :
ash which is partly converted into CaO. Sulphuric
(4) Carbon particles are produced in the
acid droplets in the atmosphere combine with basic
atmosphere when fossil fuels (coal, fuel oil, natural gas,
pollutants CaO or NH, to form salt particulates.
wood, etc.) are burnt in an insufficient supply of oxygen.
(it) Forest fires, coal refuse burning and H,SO, + CaO ——> CaSO, + H,O
agriculture burning also produce a lot of smoke. H,SO, + 2NH, ——~ (NH,),SO,
(iii) Many industries also release carbon particles (d) Tetraethy] lead. It is a source of lead salts
in the atmosphere. in the atmosphere. Tetraethy] lead is added to gasoline
(iv) Carbon particles are also produced in the to improve its antiknocking property. It is oxidised to
atmosphere due to inefficient burning of fuel in vehicles. PbO which tends to deposit on spark plugs and values.
2. Mists This is undesirable and is a source of air pollution.
These are suspensions of liquid particles in air (e) Fly ash. These are the small ash particles
which arise because of chemical reactions and formed by the combustion of fossil fuels which are
condensation of vapours in air. For example, mists carried into the air by gases produced during
are produced by the particles of the spray liquids. The combustion. These enter into the atmosphere from
common examples are portions of herbicides and power plants, smelters and mining operations.
insecticides which miss their targets and travel
(f) Asbestos dust. This originates from industrial
through the air to form mists. Similarly, sulphuric
units manufacturing asbestos sheets, gaskets, ropes,
acid mist is produced when SO, present in the
etc. Asbestos insulation and asbestos flooring also add
atmosphere comes in contact with moisture.
towards asbestos dust in the atmosphere.
ENVIRONMENTAL POLLUTION

(g) Organic particulate matter. Organic Control of Particulates


particulates are released in the air when hydrocarbons Various methods are employed to reduce the
present in coal and oil are oxidised. Among these presence of carbon and other particulates in the
polycyclic aromatic hydrocarbons (PAH) containing atmosphere.
fused benzene rings are important components of (i) Effluent gases from the industries are led
organic particulate because of their carcinogenic nature. into a chamber where the velocities of these gases
These particles are readily adsorbed on the decrease so that dust or droplets get settled.
surface of soot particles present in the atmosphere This method is suitable for particles which are settled
and therefore are serious health hazards. readily and is not suitable for the fine particles which
Effects of Particulates require longer settling time.
The effects of particulate pollutants depend upon (it) The carbon and other dust particles are
the size of the particles. The coarser particles of size removed by electrostatic precipitator. This methodis
more than 5 microns are likely to lodge in the nasal based on the principle that the particles acquire
passages whereas the smaller ones are more likely to electrical charges when subjected to an electric field.
penetrate into the lungs. The rate of penetration is In this method, two electrodes are fitted against the
inversely proportional to the size of the particles. Some inside walls of the smoke stack (Fig. 5). When high
of these particles are carcinogens. Continuous inhaling voltage (30,000 — 100,000 V) is applied, an electric
of these small particles for long periods of time irritates discharge takes place across the stack and as a result,
the lungs and causes ‘scarring’ or ‘fibrosis’ of the lung air in the stack is ionized. The ionized air consists of
lining. This type of disease 1s very common in industrial ions and free electrons. The free electrons get attached
settings and is known as “pneumoconiosis”. to the gaseous particles moving up the stack. The
particles get negatively charged and are attracted to
Different types of lung diseases are caused by the positive electrode located on the inner wall of the
different types of particulates. For example, asbestos stack and settle down there. These can be dislodged
causes asbestosis, the dust containing silica (S10,) from the electrodes by water or vibration of the
causes silicosis, the workers working in coal mines electrodes and are collected in a reservoir. In this way
suffer from black lung disease, whereas textile about 99% of the particulate matter gets removed from
workers suffer from white lung disease, etc. the fuel gases.
Lead particulates are highly poisonous and have _ 30,000— 100,000 V
a serious effect on brain central nervous system and
cause cancer.
The suspended particulate matter in the _+VE ELECTRODE
atmosphere reduce visibility and influence weather
also. These reduces the amount of light rays reaching —-VE ELECTRODE
the surface of the earth and therefore, lowers the
temperature of the earth. By blocking the sunlight, CHARGED
PARTICLES
they cause drop in the earth’s temperature by
providing condensation nuclei, they contribute to
increase fog and rain in cities. ; FUEL GASES
The large concentration of these particles spoil Fig. 5. An electrostatic precipitator.
many articles and blackens buildings. These
particulates have large surface area and hence offer (iit) The solid, liquid or gaseous particulates can
good sites for adsorption of many harmful inorganic be removed by spraying water from spray chambers.
and organic matter. Gases like SO,, PAH particles, (iv) Large amount of particles can be collected in
toxic metals (Be, Pd, Cd, Cr, etc.) are adsorbed on the cyclone collectors.
surface of soot as shown below. SMOG
Smog is a mixture of smoke, dust particles and
small drops of fog. It is a major air pollutant in big
ADSORBED cities. This is the best known example of air pollution
¢ TOXIC that occurs in many cities throughout the world. The
small drops of smog contains poisonous gases produced
by burning of fuels in homes, factories and automobiles.
ey Smoke + Fos ——~> #£Smog
-
Containing harmful
goo
iifeaa
a cases
ak
CT
Smog is of two types :
&
pig?
bagd
ak
a
¥oe= agn
te eV
tld
"|te
Bh
Ft ili
ea
aa

(t) Classical smog, and


Fig. 4. Soot particle with toxic adsorbates. (it) Photochemical smog.
tana 1. Classical smog
MODERN'S abe + OF CHEMISTRY-XI

organic free radicals. The formation of organic free


This type of smog is formed by the combination radicals results into a number of chain reactions
of smoke, dust and fog containing sulphur dioxide from producing many undesirable compounds (such as
polluted air. It is formed in those areas which have formaldehyde, acrolein, organic peroxides, organic
cool and humid climate and where sulphur dioxide in hydroperoxides, peroxyacyl nitrates etc.) which
the air is very high. This is also called chemical smog. constitute photochemical smog. It also includes H,O,.
This type of smog was observed in 1962 in London. The brownish haze of photochemical smog is largely
The city remained covered with smog for 5 days and it due to brown colour of NO,
caused deaths of many persons. Chemically, it is a 3CH, + 20, —— 3HCHO~ + 3H,0
reducing mixture so it is also called reducing smog. Formaldehyde
Acrolein and peroxyacetylnitrate (PAN) are very
2. Photochemical smog
noxious substances.
This type of smog is formed by the combination
of smoke, dust and fog with an air pollutant in the CH,—=CHCHO CH.—C—OONO,
atmosphere as a result of photochemical reaction. Acrolein |
O
It occurs in those areas which have warm, dry and Peroxyacetylnitrate (PAN)
sunny climate. It has high concentration of
photochemical oxidants. These are produced when For example, peroxyacylnitrate (PAN) is a
some of the primary pollutants interact under the constituent of photochemical smog which irritates eyes.
influence of sunlight. It occurs in warm, dry and This is formed from aldehydes, in which the reaction
sunny climate. It develops in cities in which the is Initiated by hydroxyl radical, OH* formed by the
atmosphere is loaded with large quantities of reaction of excited atomic oxygen with water, or
automobile exhausts, stagnant alr masses and photolysis of nitrous acid.
intense sunlight. It has been observed in cities which
H,O +O ——> 2HOr
have sunny, warm and dry climates such as Los
Angeles and Denver in U.S.A. Therefore, it was HNO,
—“> HO* + NO
earlier called Los Angeles smog. The photochemical
smog is chemically oxidising because it has high The OH leads to the formation of PAN as:
concentration of oxidants. Itis characterized by brown
hazy irritating fumes. As a result of photochemical RcHO —22
. R—cos Oz
as RCOO*
smog having characteristic brown haze, the cities
Aldehyde I I
such as Los Angeles are sometimes called brown air
cities. (Acyl radical) (Peroxyacyl radical)
Formation of photochemical smog
The chemistry of formation of photochemical |
smog centres around nitric oxide (NO). The formation R—COONO,
of photochemical smog can be understood by the |
following steps : O
PAN
(i) During the early morning before the sun rise,
the automobile exhaust emits CO, hydrocarbons and Harmful effects of Photochemical Smog
oxides of nitrogen. The NO reacts with oxygen to The three main components of photochemical
produce NO,, a yellowish brown gas. smog are ozone, nitrogen oxides and organic
(ii) As the sun rises, ultra-violet and visible derivatives (such as acrolein, formaldehyde, PAN etc.).
radiations fall on the earth. The NO, absorbs energy Each of these have hazardous effects of smog.
from sunlight and breaks up into nitric oxide and free 1. Pungent smelling smog containing ozone is
reactive oxygen atoms. very toxic. It can cause coughing, wheezing, bronchial
constriction and irritation to the respiratory mucous
NO, —~» NO+ O
(Reactive oxygen) system.
Some of these oxygen atoms combine with O, in 2. Aldehydes and peroxyacylnitrate (PAN)
the air to produce ozone gas. - components of photochemical smog cause irritation of
+) == *), eyes and affects respiratory tract of human beings.
Ozone reacts with NO to form NO, and O, 3. It reduces visibility causing nose, throat and
eye irritation. It may lead to several chronic diseases
O,+NO ——> NO,+0,
of eyes, heart and lungs.
NO, again absorbs U.V. radiations and the entire
4. It affects human health and comfort.
cycle starts again. Both NO, and O, are strong
oxidizing agents and can react with unburnt 5. Photochemical smog affects plant growth and
hydrocarbons (from exhaust of automobiles) to form damages plants. Ozone is known to cause damage to
ENVIRONMENTAL POLLUTION

vegetation and reduction in plant growth and N,O, + H,0Q ——> 2HNO,
productivity of crop. PAN has also highest toxicity to or 4NO,(g) + O, + 2 H,OW) —~> 4HNO,(aq)
plants. It attacks younger leaves and causes bronzing
Similarly, sulphuric acid is formed as :
and glazing of their surfaces.
6. Materials are also adversely affected by 280,(g) + O,(g) + 2H,0() >
particles 2H,SO,(aq)
photochemical smog. For example, rubber has strong Thus, acid rain is the rain water containing
affinity for ozone and is cracked and aged by smog. acids such as nitric acid and sulphuric acid which
Control of Photochemical Smog
are formed by the oxides of nitrogen and sulphur
Efforts are being continuously made to control
present in air as pollutants.
or reduce the formation of photochemical smog. This
can be done by controlling the primary precursors of The presence of soot particles in atmosphere
photochemical smog such as NO, and hydrocarbons speeds up the oxidation process. Due to the presence
and the secondary precursors such as ozone and PAN. of HNO, and H,SO, in rain the pH of water decreases.
For this, efficient catalytic converters are designed Ammonium salts are also formed and can be seen as
and installed in the automobiles to reduce an atmospheric haze (aerosol of fine particles). Aerosol
photochemical smog because reduce the smog and particles of oxides or ammonium salts in rain drops
prevent the release of nitrogen oxides and result in wet deposition. SO, 1s also adsorbed directly
hydrocarbons to the atmosphere. Certain plants such on both solid and liquid ground surfaces and this is
as Pinus, Juniparus, Quercus, Pyrus and Vitis can deposited as dry deposition.
metabolise nitrogen dioxide and therefore, plantation Harmful effects of Acid Rain
of these plants could help to reduce photochemical
Acid rain is very damaging. It is toxic to vegetation,
smog.
human life and aquatic life. Some of its harmful effects
Photochemical smog can also be suppressed by
certain compounds which trap free radicals. These are:
are sprayed in the atmosphere where they generate 1, It causes extensive damage to buildings and
free radicals which combine with free radical sculptural materials of marble, limestone, slate, etc.
precursors of photochemical smog and consequently These materials react with rain water and get
reduce it. damaged.
ACID RAIN CaCO, + H,SO, ==
————>
CaSO, + CO, + H,O
Fossil fuels contain compounds of sulphur and As a result of acid rain, the invaluable statues
nitrogen in addition to carbon. The combustion of
and buildings deteriorate. In Greece and Italy, many
fuels contributes significantly to atmospheric
invaluable statues have been partially facaded. The
pollution. The burning of fossil fuels gives CO,. The
Taj Mahal of India is facing the same problem.
gaseous CO, dissolves in water droplets to form weak
acid, carbonic acid. 2. When acid rain falls and flows as ground water
CO,(g) + H,O(aq) ——> H,CO,(aq) to reach rivers, lakes, etc, it affects plants and animal
life in aquatic ecosystem. The acid rain has also caused
H,CO,aq) === H'*(aq) + HCO, (aq)
elimination of life from some fresh water lakes by
Rain water normally has pH of 5.6 due to the
destroying the living bodies.
formation of H* ions from the reaction of rain water
with carbon dioxide present in the atmosphere. When 3. The rain water also corrodes metals and
the pH of the rain water falls below 5.6, it becomes damages iron and steel structures.
acidic and is called acid rain, Thus, acid rain means 4, Acid rain is harmful for agriculture, trees and
the way in which acid from the atmosphere is plants because it dissolves and washes away nutrients
deposited on the earth's surface. needed for their growth. Therefore, it damages leaves
However, CO, is not the major component of of trees and plants and retard the growth of forests. It
acid rain because it is not soluble in water like the also retards the growth of certain crops such as peas,
oxides of sulphur and nitrogen. The oxides of nitrogen potato, raddish, carrot, beans, spinach, etc.
and sulphur undergo many photochemical reactions 5. Acid rain dissolves heavy metals from soils,
in atmosphere and form HNO, and H,SO, acids. rocks and sediments. The heavy metal ions such
During rains, these acids fall to the earth with rain.
as copper, lead, mercury and aluminium leached
This polluted rain is called acid rain. The main
from the soil enter well water and produce many toxic
reactions are :
effects.
Nitric acid is formed as:
6. Acid rain corrodes water conduit pipes
NO+0, —~ NO, +0,
resulting in leaching of some heavy metals such as
NO, + 20, — > NO,+0 3 iron, lead and copper into our drinking water which
NO, + NO, —> N,O, have toxic effects.
ae MODERN'S abe + OF CHEMISTRY-XI

Methods to reduce the formation of acid It is formed in the atmosphere by the decomposition of
rain oxygen by ultra-violet radiation from the sun having
Acid rain is due to emission of oxides of nitrogen wavelength shorter than 260 nm.
and sulphur in the atmosphere. Therefore, to reduce O, (g) thv —UY_, 0@) +0
the formation of acid rain, the emission of these
gases has to be controlled. For this, the following O (g) + 0, (g) “> 0, @)
steps should be taken :
Most atmospheric ozone is found in stratosphere
(i) By using fossil fuels having less sulphur
where its concentration is about 10 ppm (parts per
content in power plants and industries. For example,
million) by volume. Ozone is thermodynamically
natural gas should be used than coal or coal containing
unstable and decomposes into molecular oxygen. It has
lesser sulphur contents should be used.
the important photochemical property of absorbing
(it) By using less vehicles driven by fossil fuels. solar radiation between the wavelength of 200 nm and
(iii) By using catalytic convertors in cars so that 300 nm.
exhaust gases are converted into nitrogen.
O, (g) t+hv —> O(g)+0,@)
(iv) By using powdered limestone to neutralise
the acidity of the soil. The reactive atomic oxygen formed in the above
reaction recombines with molecular oxygen to form
Acid rain and Taj Mahal of India ozone. This completes the ozone cycle. Thus, a dynamic
The acid causes extensive damage to buildings equilibrium exists between the production and
and sculptural materials of marble, limestone, slate decomposition of ozone molecules
etc. The air in the city of Agra where Taj Mahal is The thick layer of ozone is called ozone
located contains very high levels of oxides of blanket because it is very effective in absorbing
sulphur and nitrogen. This causes acid rain and it harmful ultra-violet rays given out by the sun.
reacts with marble (CaCO,) of Taj Mahal Therefore, the ozone layer is also known as protective
CaCO, + H,SO, > CaSO, + H,O + CO, shield. Since living cells can be destroyed by ultra-
violet radiation, the ozone layer protects us from its
As a result, the marble becomes pitted and damaging effects. If there were no ozone in the
weakened mechanically because the soluble salts stratosphere, life on the earth would not be
are easily leached out by rain water. Therefore, possible. Even a small decrease in the concentration
the wonderful monument is being slowly eaten of ozone could lead to an increased incidence of skin
away and the marble is getting discoloured and cancer.
lustreless. The same problem is being faced by
Depletion of Ozone layer and its effects
Greece and Italy where the stone statues have
been partially dissolved by acid rain. Recently in 1980, scientists have observed a hole
in the ozone blanket covering the upper atmosphere
To protect Taj Mahal, the Government of
India has announced an action plan in 1995 to around Antarctica. Recent observations have also
prevent the disfiguring of this historical wonder. shown that the ozone layer diminishes over the south
This plan aims at clearing the air in the Taj pole in spring during August-September to a greater
Trapezium, an area which includes towns of extent year after year. This depletion of the protective
Agra, Firozabad, Mathura and Bharatpur. Under blanket of ozone will cause a damaging effect because
this plan about more than 200 industries lying harmful ultra-violet rays can come to earth through
within this trapezium would shift over the use of this hole. The increased level of ultra-violet rays will
natural gas or liquefied petroleum gas instead of result in damage to plants, animals, human beings and
coal or oil. A new natural gas pipeline would even inanimate matter posing great threat to
provide more than half a million cubic metres of ecosystem over the globe.
natural gas a day to this area. The people living Thus, the depletion of ozone layer is a serious
in this area will also be encouraged to use LPG in threat to mankind. Scientists believe that every 1%
place of coal or kerosene oil. Even the vehicles decrease in ozone leads to 2% increase in skin cancer
plying on highways in the nearby areas of Taj due to exposure to sun’s ultra-violet rays.
Mahal would be encouraged to use diesel of low
sulphur content. The depletion of ozone may be due to some natural
processes or industrial activities.
STRATOSPHERIC POLLUTION : FORMATION Nitrous oxide (N,,O) occurs naturally at very trace
AND DEPLETION OF OZONE LAYER levels in stratosphere. It can react with electronically
Ozone is an important constituent of the excited oxygen atoms obtained by the ozone cycle to
stratosphere at altitudes between 15 and 25 km. sive NO.
ENVIRONMENTAL POLLUTION

N,O + O°——>2NO Effects of Depletion of the ozone layer


Excited Nitric The depletion of ozone layer has the following
atomic oxide
harmful effects :
oxygen
This removes an oxygen atom needed by the (i) Depletion of ozone layer poses serious threat
ozone cycle and the product, NO can destroy a second to mankind. It results in ageing of skin, cataract,
ozone molecule which breaks up another cycle. sunburn, skin cancer diseases, etc.
(ii) Ultra-violet rays may damage immune
NO +0, ——> NO, +0, system which may lead to increased viral infections.
NO, +O ——> NO+0,
(iit) Increased exposure to ultra-violet radiation
0,+0 ——> 20, will damage marine plants, marine animals and fish
which form an important part of human food.
Therefore NO is very damaging because just
one NO molecule sets up a chain reaction which (iv) These radiations adversely affect the plant
removes ozone again and again. There are similar ozone proteins which lead to harmful mutation of cells.
depleting chain reactions involving other stratospheric (v) These also increase the evaporation of
cases. surface water through the stomata of the leaves and
Chlorofluoro carbons and Ozone layer decrease the moisture content of the soil.
Chlorofluoro carbons are compounds such as (vi) Increase in ultraviolet radiations damage
CFCl,, CF,Cl,, CF,Cl, commonly known as freons. paints and fibres causing them to fade faster.
These compounds are non-reactive, odourless, (vit) Freons trap the warmth of the sun and
non-toxic, non- flammable, and thermally stable. disturb greenhouse effect leading to higher
The large amount of these products are being temperature which could damage crops and pose
extensively used in air conditioners, refrigerators, fire dangers to low lying ocean port cities from increased
fighting materials, fast food packing materials, for melting of ice.
sterilizing surgical instruments in medicinal products,
Ozone Depletion over Antarctica
in the production of plastic foam and by the electronic
industries as cleaning solvents and are released in the The chain reactions described above can occur
atmosphere causing air pollution. all over the stratosphere. However, the depletion
of ozone layer leading to ozone hole has mainly
Once chlorofluorocarbons (CFCs) are released
been observed in the stratosphere over Antarctica.
in the atmosphere, they mix with the normal
In the other parts of the stratosphere, two
atmospheric gases and ultimately reach the
stratosphere. In stratosphere, they get broken
alternative reactions consume chlorine monoxide
down by powerful ultraviolet radiations releasing
and chlorine atoms as :
chlorine free radicals (Cl’). These released Cl free ClO’ + NO, —> CIONO,
radicals cause a catalytic chemical reaction and Chlorine nitrate
cause significant depletion of stratospheric ozone Cl’ + CH, —> CH, + HCl
layer. In other words, oxides of nitrogen in the
CF,Cl, + hv ——> Cl’ + CF,CI’ stratosphere scavenge the chlorine monoxide
radicals while methane molecules scavenge the
The free radical, Cl’ reacts with O, through a chlorine radicals. Thus, these destroy the chain
chain reaction : mechanism for the ozone depletion and no ozone
Cl’ + O, ——} ClO’ +0, hole is created.
However, in Antarctica, the conditions are
ClO’ + O ——> Cl’ +0, different. During winters, special types of clouds
These chlorine atoms are free to react with more called Polar Stratospheric Clouds (PSCs) are
ozone. As aresult, many O, molecules can be destroyed formed over Antarctica. These clouds are of two
for each chlorine atom produced. It has been shown types :
that over one thousand ozone molecules can be Type I Clouds. These contain some solidified
destroyed by one Cl. The net result of these reactions nitric acid trihydrate (HNO,.3H,O) form at about
is destruction of several molecules of O, for each Cl 196 K.
atom produced. Type II Clouds. These contain some ice form
Thus, CFCs are transporting agents for at about 188 K.
continuously producing chlorine radicals into the These two types of clouds play a very
stratosphere and hence damaging the ozone layer. important role in ozone depletion. On the surface
1416 MODERN'S abe + OF CHEMISTRY-AI

of polar stratospheric clouds, chlorine nitrate atmosphere can lead to global warming through
formed in the above reaction undergoes two ereenhouse effect as explained below:
transformations : Sun emits light of different wavelengths
(t) It gets hydrolysed as : consisting of ultra-violet, visible region and infra-red.
Of these the harmful ultra-violet radiations are
CIONO, + H,O —““““ > HOCl + HNO, absorbed by the ozone layer in the stratosphere. The
(it) It reacts with hydrogen chloride formed visible and infra-red radiation pass through the
during the reaction atmosphere and reach the surface of the earth
ClONO. + Hel —> Cl + HNO. (Fig. 6). However, some of the hight incident on earth
These HOC] and Cl, formed can be is reflected back. For example, of the total incoming
reconverted into reactive chlorine atoms even light that falls upon the Earth, about 50% reaches the
under mild conditions resulting ozone depletion. surface and is absorbed by it. About 20% of the
The ozone depletion over Antarctica occurs incoming light is absorbed by gases in the atmosphere,
during spring time (from early September to late such as ultra-violet (wv) light by ozone in stratosphere
October) and gets replenished after spring time. and infra-red (IR) by CO, and water vapour present
During the spring time, the sun begins to shine in air. The remaining 30% is reflected back into space
over Antarctic skies and HOC] and Cl, are
by particulates and other reflecting bodies without
photolyzed by sunlight according to the following
being absorbed.
reactions:
When the earth cools, the energy is re-emitted
HOC] + hv —> OH* + Cl
from the earth’s surface in the form of infra-red
Cl, + hv —> 2Cl°
radiations which are absorbed by CO, and water
These reactive chlorine atoms formed
vapour. After the absorption of infra- ced radiations
deplete ozone through the chain mechanism
by CO, and water vapour, these thermal infra-red
reactions. During winters, when Polar
radiations are re-emitted in all directions and
Stratospheric Clouds are formed over Antarctica
therefore, some are redirected back towards earth’s
skies, stable patterns of wind cover the continent.
This is called Polar Vortex which is a tight surface and heat up the atmosphere (earth and air).
whirlpool of winds. This is very rigid so that the This heating of the earth due to trapped radiations is
air within it is isolated from the sun and from the called greenhouse effect.
warmer air of the temperate region. Because of Thus, the warming of the earth or global
this polar vortex surrounding Antarctica, the ozone warming due to re-emission of sun's energy
rich air from the non-polar region is not able to fill absorbed by the earth followed by tts absorption by
the ozone hole. Therefore, Polar Vortex remains carbon dioxide molecules and water vapour present
intact during the spring. After the spring, the near the earth and then its radiation back to the
sunlight increases and the vortex breaks down earth is called greenhouse effect.
(some time in November) and the ozone rich air
The gases which can trap infra-red radiation given
from the temperate region fills the ozone hole. bythe sun to produce greenhouse effect leadingto heating
To control this harmful effects of CFCs, serious
efforts are being made to control the use of CFCs Heat radiated back
into space
and find alternative methods. The Montreal Protocol
signed by 24 countries in October 1987, was the first
step to bring down the world production of freons. At
some places, these are being replaced by
safe substitutes like hydrochlorofluoro carbons
(HCIFCS) which have only one tenth as destructive
effect as CFCs.
GREENHOUSE EFFECT AND GLOBAL
WARMING
Carbon dioxide is an essential component of
aa reflected back
atmosphere and in normal concentrations, it is earth (greenhouse
harmless. However, due to human activities such as effect)
burning of fossil fuels like coal, natural gas, petroleum, Surface of earth

etc., the amount of CO, entering the atmosphere is


increasing. The increase in proportions of CO, in Fig. 6. Greenhouse effect.
ENVIRONMENTAL POLLUTION

up the environment are called greenhouse gases. The are continuing to do so at an increasing rate. Due to
principal gases of the atmosphere N,, O, and Ar do not this large scale release of CO, in the atmosphere, its
absorb infra-red light. The common atmospheric gases level in the atmosphere has increased to about
that have predominantly produced green house warming 306 ppm.
are CO,, water vapours (H,O) and ozone (Q,). Out of Methane (a greenhouse gas) is produced
these, water vapours and ozone do not contribute much
naturally when vegetation is burnt, digested or rotten
to the greenhouse effect of the earth’s atmosphere
in the absence of oxygen. Large amounts of methane
because water vapours are found only near the surface
are released in paddy fields, coal mines, from rotting
of the earth whereas ozone is present only in the upper
gcarbage dumps and by fossil fuels.
atmosphere. On the other hand, carbon dioxide is
uniformly distributed in the atmosphere and hence Chlorofluoro carbons (CFC’s) are also responsible
makes major contribution towards the greenhouse effect. for global warming due to their greater efficiency of
Thus, carbon dioxide is regarded as the most absorbing thermal infra-red radiations. Each molecule
important greenhouse gas. of CFC has the potential to cause the same extent of
elobal warming as do tens of thousands molecules of
The other greenhouse gases whose concentration
CO,. CFC’s are being excessively used in insulating
has been small in atmosphere but have ability to warm
freezers, refrigerators and air conditioners and are
the air substantially are CH,, N,O, CFCl,, CF,CL,, ete.
posing threat of global warming. However, Government
Their concentration in atmosphere is increasing with
is now trying to control the production of CFCs.
the industrialization and air pollution.
The net effect of increased level of all these
Greenhouse gases ereenhouse gases is increased greenhouse effect or
elobal warming which may lead to increased global
CO,, CH, O,, H,0 vapour, CFCs
temperature. The global warming will have drastic
This phenomenon has been named greenhouse effect on the climate and may prove disastrous. If
effect because it is similar to a greenhouse for plants nothing is done to control the concentration of these
that is generally made to protect the plants from cases into the atmosphere, than average temperature
cold in winter. In cold places, flowers, fruits and will increase. It has been observed that the average
vegetables are grown in glass covered areas called temperature of the earth has increased by about 1°F
greenhouse. The sun’s radiations can enter through due to greenhouse effect. If CO, is continued to be
the transparent glass and heat up the soil and the released into the atmosphere at the present rate then
plants. The warm soil and plants re-emit infra-red by the end of this century the earth’s average
radiations which are partly, trapped by the glass temperature is expected to increase by 3.6°F. This
(opaque to infra red radiations) and therefore remain excessive heating of the earth would melt all the snow
on poles and different mountains. This would increase
inside the glasshouse. Hence, the glasshouse holds
the water level of the sea and as a result the cities
the energy of the sun trapped inside and keeps it
located on the coastal areas are likely to be flooded.
warm. Therefore, greenhouse protects the plants
from cold climate during winter in cold countries. Effects of increase in global temperature on
the climate of the earth
Greenhouse Effect in an Automobile The increase in average global temperature can
The heating due to greenhouse effect can also have the following effects on the climate of the earth :
be observed inside an automobile such as a car (a) In temperate regions, the summer will be
parked in the sun with all its windows closed. The longer and hotter and winter will be shorter and
glass windows of the car allow the sun’s rays to warmer. A warmer climate will make certain cities
enter the car. Some of these rays are reflected from extremely hot to live.
the inside surface of the car in the form of infra-red (6b) There will be increase in total amount of
light. These rays are trapped by glass windows global rainfall but some regions will receive less
(just like CO, layer) and cause the interior of the rainfall.
car to heat up. Hence, the inside of the car becomes (c) The number of days having intense showers
considerably hot. and high temperatures both will increase.
Implications of Greenhouse effect or Global (dq) The problem of desertification, drought and
Warming soil erosion will become more worse.
Modern human activities are releasing large (e) As aresult of rise in temperature of the earth,
quantities of CO, in the atmosphere. The increase in ocean will get warm up and sea level would rise
its proportion in the atmosphere can lead to global flooding low lying regions. The increase in sea level
warming through increased greenhouse effect. The would have profound effects on habitation patterns
CO, level in atmosphere was about 290 ppm in 1870. and will threaten to submerge many coastal countries
Over the past hundred years we have released about like Bangladesh, Indonesia, Maldives, parts of coastal
3,60,000 million tonnes of CO, in the atmosphere and India and many other island nations.
a MODERN'S abe + OF CHEMISTRY-XI

(f) Increase in greenhouse effect is expected to water pollution is a matter of serious concern for
cause cooling of the stratosphere. This is because most every citizen. The main culprits for water pollutions
thermal IR radiations will be absorbed at low altitudes are domestic waste from urban and rural areas and
and little will be left to warm stratosphere. industrial wastes which are discharged into natural
(¢) Tropical storms, hurricanes, etc. will be water supplies.
stronger and more frequent and will cause Water pollution may be defined as
devastation. the contamination of water by foreign materials
(hk) Some areas may become more humid or which make tt harmful for the health ofhuman
wetter or some other areas will become dry. The beings, animals, plants or aquatic life and
tropics may become wetter and the subtropics which make it unfit for domestic, industrial or
are already dry, are expected to be drier. agricultural uses.
(i) Due to global warming, human health will The degree of purity required for water depends
be affected. Increased number of hot days and extreme upon its use. For example, the water polluted for
weather may cause chronic respiratory diseases. This drinking purposes may be satisfactorily used for
also causes Increase in the incidences of infectious irrigation or producing electricity at hydroelectric
diseases like dengue, malaria, yellow fever, sleeping power plant or for cooling purposes.
sickness, etc. Pollution of water originates from human
(Gj) Insect carrying diseases such as malaria may activities. Through different paths, pollution reaches
also increase. surface or ground water. The easily identified source
or place of pollution is called point source e.g.,
(k) Animal health will also be affected due to
Municipal and industrial discharge pipes where
spread of diseases by parasites.
the pollutants enter the water source. On the other
(/) Ocean temperature change may also affect hand, non-point sources of pollution are those where
marine life adversely. a source of pollution cannot be easily identified.
Methods to prevent global warming For example, acid rain, storm water drainage (from
Some steps to minimise the release of green streets, parking lots, lawns etc.) agricultural runs off
house gases and prevent global warming into the (from farm, animals and crop lands etc.). The major
atmosphere are : water pollutants and their common sources are given
(1) By minimising the use of cars by car pools or in Table 2.
taking public transport in cities. This can also be
Table 2. Major Water Pollutants and their
done by preferring walking or using bicycles. Sources.
(ii) By planting more and more trees and
increasing the green cover.
(iii) By avoiding the cutting of trees. Micro-organisms Domestic sewage
(iv) By avoiding the burning of dry leaves, etc. Organic wastes Domestic sewage, animal
manures, food processing
(v) By not smoking in public places and work
wastes, decaying plants and
places. animals, paper discards, rags
(vi) By educating the public about greenhouse and other bio degradable
effect and the methods to save our environment. wastes
Plant nutrients Chemical fertilizers
WATER POLLUTION Toxic heavy metals| Industries and chemical
factories
Water is most plentiful of all the natural sources.
Sediments Insoluble particles of soil and
It covers nearly three-fourth of the earth’s surface in rock and inorganic and organic
the form of snow over mountains, as a liquid in the compounds which wash into
rivers, lakes, springs and oceans. Water is very water from mining, agricultural
essential for life. Without water there would be no life. activities, construction activities
It is very essential for the growth of human body and and soil erosion.
has significant role in industry, agriculture and Pesticides Chemicals used for killing
civilization. insects, fungi and weeds
Radioactive wastes Mining of uranium containing
The quality of water especially for drinking
minerals and accidental
purposes has been a vital factor for the human welfare. release from nuclear plants.
The pollution of drinking water has frequently Heat Water used by industrial plants
caused many water borne diseases and epidemics for cooling which is discharged
such as cholera, typhoid fever, etc. Therefore, as hot water.
ENVIRONMENTAL POLLUTION

SOURCES OF WATER POLLUTION TYPES OF WATER POLLUTANTS


The water pollution may be caused by domestic 1. Pathogens. The most serious water pollutants
sewage, industrial effluents, agricultural discharges, are the disease causing agents called pathogens.
sedimentation, radioactive discharges, thermal Pathogens include bacteria and other organisms
pollutants etc. The major sources of water pollution which enter water from domestic sewage and animal
are domestic sewage and industrial wastes. excreta. Human excreta contains bacteria such as
(¢) Domestic sewage. The domestic sewage escherichia coli and streptococcus faecalis which
contains oils, human excreta, dirt, paper, rags, sand cause gastrointestinal diseases. These bacteria can cause
erains, dissolved material such as detergents and creat epidemic diseases.
inorganic compounds such as sodium chloride, 2. Organic wastes. The organic matter such as
ammonium sulphate and ammonium phosphate,
leaves, grass, trash etc. are also common pollutants.
decomposed kitchen waste. The sewage also contains
These pollute water as a consequence of run off.
many disease causing bacterias called pathogens
Excessive phytoplankton growth within water is
which are most serious water pollutants.
also a cause of water pollution. These wastes are
(ti) Industrial wastes. Industrial wastes are
biodegradable. The large population of bacteria
also the major source of water pollution. The industrial
decompose organic matter present in water.
wastes polluting water are mainly from industries
such as coal or ore mines, textile industries, paper 3. Chemical pollutants. Water being an excellent
industries, food processing industries, dairies, solvent dissolves many water soluble inorganic
chemical industries, pharma-ceuticals, sugar and chemicals. Some of these pollutants are:
distilleries, oil refineries, tanneries, vegetable oil and (.) Heavy metals. These are metals such as
soap industries etc. These wastes may contain cadmium, lead and mercury which are present in
inorganic and organic suspended particles and industrial or mining wastes. These metals are poisonous
and can be dangerous to humans because our body
inorganic and organic soluble matter.
cannot excrete them. These metals can cause damage
(iit) Agricultural discharges. These include to kidneys, liver, central nervous system, etc. For
fertilizers, pesticides and insecticides, etc. entering example, cadmium and mercury can cause kidney
the water body. damage whereas lead poisoning can cause damage to the
(tv) Sediments. Water can also be polluted by kidneys, liver, intestines, brain and central nervous
sediments. These are water insoluble particles of soil system. Mercury poisoning causes a disease called
and rock and many inorganic and organic compounds minamata in human beings which weakens the muscles
which wash into water from mining, agricultural and results in weakness in hearing and vision power,
activities, construction activities and activities mental retardation and paralysis. All of these metals are
associated with soil erosion. It represents the most cumulative poisons because the body does not excrete
them and their concentration builds up.
extensive pollutants of surface water.
(iz) Detergents and fertilizers. These may
(v) Radioactive pollutants. Many radioactive
contain phosphates as additives. It may be noted that
isotopes enter the surface water from geothermal
these do not pose any threat to the aquatic life but serve
wells or accidental releases from nuclear power plants. as nutrients for plants leading to their excessive growth
(vt) Thermal pollutants. Many processes such in ponds, lakes and rivers. These encourages the
as combustion of fuel in boilers, electric power plants formation of algae which reduces the dissolved oxygen
and industrial plants produce a lot of heat. Therefore, concentration of water. This process of overnutrition is
to cool the various equipments, the water from the known as eutrophication. This impedes the
nearby rivers, lakes and ocean bays are circulated development of higher life forms such as fish.
through the heated parts. The circulating water (zit) Petroleum products. Petroleum products also
becomes hot and when this hot water is discharged pollute many sources of water. e.g., major oil spills in
back to the surface water, it can have many adverse ocean. This is because of wreckage of oil tankers in
effects. This is called thermal pollution. open sea or accidents of ships carrying oil in the sea.
The spreading of oil into sea is called oil spill and the
(vii) Oil. Water pollution in seas and rivers is
thick layer of 011 on the surface of sea water is called
caused by oil due to increased use of oil base
oil slick. In India an oil spill occurred in Bombay on
technology, excessive off-shore explorations and March 17, 19938 due to rupture of pipeline which
transportation of oil through water ways. This results damaged the ecosystem and marine life.
from discharge of oil from oil tankers and offshore (iv) Acid polluted water. The acid polluted water
drilling explorations and accidents, leakage of pipe having pH less than 3 is deadly to most forms of
lines over and under water ways. aquatic life. The water downstream from a mine may
S Ta0
get contaminated by acid mine drainge formed by
MODERN'S abe + OF CHEMISTRY-AI

green plants, the water becomes supersaturated with


microbial oxidation of discarded waste materials at oxygen during the hours of daylight due to
the nine site. The acid mine water mainly contains photosynthesis. However, after dark, photosynthesis
sulphuric acid produced by the oxidation of iron pyrites stops but the plants continue to respire and therefore,
(FeS,). Industrial wastes and acid rain also contribute the amount of dissolved oxygen decreases.
to the acidic nature of natural water. Consequently, during a period of 24 hours, some water
(v) Polychlorinated biphenyls (PCBs). samples have considerable amount of dissolved oxygen.
Polychlorinated biphenyls have very high stabilities The deoxygenation of water is carried out by
and therefore, are finding many applications these various processes. For example, the dissolved oxygen
days. For example, these are used as fluids in in water is consumed rapidly by micro-organisms to
transformers and capacitors, as cleansing solvents, oxidise organic matter of sewage.
detergents and fertilizers. PCBs are resistant to [CH,O] (aq) +0, (aq) Micro-organisms CO, (aq) +H,0( 1)
oxidation and their release into the environment has From organic
become a serious pollutant. These are carcinogenic matter
and cause skin disorders in humans. Now a days most
If the concentration of the dissolved oxygen is
of the detergents available are biodegradable. But
not restored either by the turbulent flow of shallow
their use can create other problems. The bacteria
water or by a reaction, it is not able to support many
responsible for their degradation feed on degradable
organisms. The dissolved oxygen in water is also
detergent and grow very rapidly. During their growth,
consumed by the biooxidation of the nitrogenous
the bacteria may take all the oxygen dissolved in
material.
water. As a result of lack of oxygen, other forms of
NH,,* (aq) + 20, (aq) —> 2H* (aq) + NO, (aq) + H,O()
aquatic life such as fish and plants may die. Fertilizers,
and by the chemical or biochemical oxidation of
as we know contain phosphates as additives. The
chemical reducing agents.
addition of phosphorus in water promotes algae
erowth. Such a large growth of algae may ‘cover the 4Fe** (aq) + O, (aq) + 10 H,O (1) —> 4Fe(OH), (s)
water surface. These algae growth often release toxins + 8 H* (aq)
in water. This bloom infested water will inhibit the 280,” (aq) + O, (aq) —> 280,7 (aq)
erowth of other living organisms in the water body. Biochemical Oxygen Demand (BOD) and Chemical
The process of nutrient enrichment of water bodies Oxygen Demand (COD)
and subsequent loss of biodiversity is called The organic waste can be broken down by both
eutrophication. aerobic and anaerobic bacteria which act under
Until recently, it was a common practice to dump different conditions. Aerobic bacteria are oxygen
the industrial wastes into nearby stream. These days, consuming decomposers which bring about the
this practice has been stopped to prevent stream oxidation of degradable organic matter into carbon
pollution and to recover some expensive chemicals dioxide, nitrates, sulphates, phosphates, etc. As a
from these wastes. However, it may be noted that the result of aerobic oxidation, dissolved oxygen in the
treatment of industrial wastes is very tedious problem water decreases. Due to short supply of oxygen, the
because of the large variety of chemicals present in aquatic organisms especially fish and shell fish die
black liquor coming from the industries. from suffocation. Complete oxygen removal kills many
The treatment of industrial wastes depend upon other forms of aquatic life except anaerobic bacteria
the nature of the pollutants present. which do not require oxygen.
On the other hand, anaerobic bacteria do not
IMPORTANCE OF DISSOLVED OXYGEN
require oxygen for the decomposition of organic wastes.
IN WATER
The decomposition of organic waste by anaerobic
The concentration of dissolved oxygen (DO) in bacteria produce toxic and foul smelling substances
water is very important to support the aquatic life. If such as hydrogen sulphide, ammonia, methane,
the concentration of dissolved oxygen becomes low, the phosphine, ammonium sulphide, etc. This kind of
water is said to be polluted. The lower the concentration oxidation of sewage is called anaerobic oxidation.
of dissolved oxygen, the more polluted is the water Dissolved oxygen is an essential requirement of
sample. Ifthe dissolved concentration of oxygen in water aquatic life i.e. animal and plant population in water.
is below 6 ppm, the fish growth is inhibited. The two The quantity of oxygen consuming wastes in water is
main sources of oxygen are : the oxygen dissolves at the usually determined by measuring the biological
surface of the water from the atmosphere and from oxygen demand (B.0.D). It is defined as the
photosynthesis carried by many aquatic green plants amount of oxygen required by microorganisms
during day light. It may be noted that still water takes (bacteria) to break down the organic matter
up oxygen slowly whereas turbulent water takes it up present tn a certain volume of a sample of
more rapidly because bubbles are commonly sumerged. water.
Moreover, at the places where there are many aquatic
ENVIRONMENTAL POLLUTION

The amount of BOD in water is a measure of are large, it indicates that water is heavily polluted.
the amount of organic material in water, in terms of In order to measure COD, the given water sample
how much oxygen will be required to break it down is treated with a known quantity of an oxidising agent
biologically. (potassium dichromate, K,Cr,O,) in acidic medium.
The carbon of organic matter is converted into This oxidises most of the polluting substances including
CO, and since 1 mol of C requires 1 mol of oxygen, those which are resistant to microbial oxidation.
BOD is directly related to the concentration of organic The excess of K,Cr,O, is determined by back titration
matter. Therefore, the high value of BOD. indicates with a suitable reducing agent such as Mohr’s salt.
that water is polluted. Therefore, the BOD is taken From the concentration of K,Cr,0, consumed, the
as a realistic measure of water quality. The clean amount of oxygen used in oxidation may be calculated
water would have a BOD value of less than 5 ppm using the following chemical equation :
whereas BOD of 17 ppm or more indicate highly K,Cr,O, (aq) + 4350, (aq) —>
polluted water. K,SO, (aq) + Cr, (SO,)3 (aq) + 4H,O (1) + 30 (aq)
In order to measure BOD, the water sample is The results are expressed in terms of amount of
first saturated with oxygen. It is then incubated oxygen in ppm that would be required to oxidise the
at constant temperature (usually 20°C) for 5 days. contaminants. This is called COD.
This allows time for micro-organisms in water
WATER QUALITY PARAMETERS AND
sample to oxidise organic matter. The remaining
oxygen is determined and the difference gives the INTERNATIONAL STANDARDS
amount of oxygen consumed by micro-organisms for The quality of water is of vital concern for
decomposition i.e., BOD. mankind because it is directly linked with human
There is another parameter called chemical welfare. The main requirements for drinking water
oxygen demand (COD). Certain chemicals other than are :
organic wastes also react with dissolved oxygen in The drinking water should be fit for human
water. This is referred as chemical oxygen demand
consumption having the following essential parameters
(COD). Thus, BOD measures the oxygen consumed
for water quality :
by living organisms assimilating organic matter
present in waste while COD is a measure of biological (1) It should be colourless and odourless.
oxidisable as well as biological inert organic matter (it) It should be pleasant in taste.
such as cellulose. Thus, if BOD or COD values of water

Table 3. Permissible Limits of Drinking Water.

Permissible limits (ppm) Ill-effects of pollutants


Dissolved
Oxygen
(pH)
Arsenic Cramps, paralysis
Barium Toxic to heart
Cadmium Anaemic, kidney dysfunction
Chromium Carcinogenic
Fluoride Skeleton disorder, nervous breakdown and mottling
of teeth enamel
Lead Anaemic, kidney dysfunction, nervous
Aluminium disorder, brain damage
Selenium Toxic at high levels
Copper Toxic to plants and algae
Mercury Highly toxic, mental disorders and death
Zine Toxic
Alkyl benzene- Irritation of the gastrointestinal tract
sulphonate
Chloride Harms agricultural crops
Sulphate Laxative and hypertension
Pesticides Toxic, carcinogenic, kidney dysfunction
ae MODERN'S abe + OF CHEMISTRY-XI

(iii) It should be clear and turbidity should be less Table 4. Maximum prescribed concentration
10 ppm. of some metals in drinking water.
(tv) Its pH should be between 5.5 to 9.5.
Maximum Concentration
(v) The total dissolved solids should not be more (ppm or mg dm)
than 500 ppm.
(vi) It should be free from disease causing micro- Iron, Fe
organisms. Manganese, Mn
Aluminium, Al
(vil) It should be reasonably soft.
Copper, Cu
(vill) It should be free from objectionable chemicals. Zinc, Zn
There are some international standards for Cadmium, Cd
drinking water which must always be obeyed if water
is to be used for drinking purposes. The parameters
| R.U. Curious... «>
and standards for water quality and harmful effects
of pollutants beyond permissible limits are given in 4) Certain newly born babies develop illness
Table 3 for some common chemicals. Some of these known as blue baby syndrome.
are also briefly discussed below : > Blue baby syndrome or simply known as blue
baby is a term used to describe the infants with
1. Fluoride. Soluble fluoride is normally added to blue tinted skin. This may be caused by excessive
drinking water and its concentration should be upto nitrate (NO,) ions in drinking water in some rural
1—1.5 ppm or 1-1.5 mg dm~. This limit is within agreed area having high levels of nitrates present in ground
safety limits and protects teeth against decay. The water. When consumed, the nitrates get reduced to
deficiency for this compound in drinking water is nitrite ions in the digestive system. The nitrites
harmpul to man and causes diseases such as tooth | react with oxyhemoglobin (the oxygen carrying
decay, etc. The F ions make the enamel on teeth blood protein) to form methemoglobin, which cannot
much harder, by converting hydroxyapatite carry oxygen. As a result, the oxygen carrying
[8Ca,(PO,), .Ca(OH),] the enamel on the surface of capacity of blood gets reduced and the organs, cells,
the teeth into harder fluorapatite [(3Ca, (PO,),. CaF]. tissues or skin may be deprived of oxygen, causing
However, F ion concentration above 2ppm causes infants to develop blue baby syndrome. This can be
brown mottling of teeth. Higher concentrations of | fatal if left untreated leading to even death. This
fluoride are poisonous and are harmful to teeth and condition is also known as methemoglobinemia.
bones at levels above 10 ppm (mg dm~). This problem
has been reported from some parts of Rajasthan. SOIL POLLUTION OR LAND POLLUTION
2. pH. The pH of the drinking water should be Soil receives large quantities of hazardous wastes
between 5.5 to 9.5. A decrease in pH of the water from different sources and gets polluted.
increases the solubility of the heavy metal ions. Any factor which deteriorates the quality,
3. Sulphate. Sulphate is harmless at moderate texture and mineral content of the soil and
concentrations but excessive concentration of sulphate disturbs the biological balance of the organisms
in tt and has lethal effect on the plant growth
above 500 pm produces laxative effects and
is called soil pollutant.
hypertension.
India being agricultural based economy gives
4. Nitrate. Excessive nitrate in drinking water high priority to agriculture. The production of
is also harmful and can cause methemoglobinemia (blue agriculture depends primarily on the quality of soil
baby syndrome). It is linked to stomach cancer. because it 1s the soil which provides nutrients, water
The maximum limit of nitrate ion in drining water is and minerals for the growth of plants. However, India
50 ppm. is also facing serious problem of deteriorating quality
of soil and resulting into soil pollution. The main
5. Lead. Lead pipes are very commonly used for
sources of soil pollution are dumping of industrial
water transport and is liable to get contaminated wastes, urban wastes, volatile organic compounds, radio-
with lead, particularly when water is acidic. active wastes and agricultural practices. Among these
The permissible limit for lead ions in drinking water is modern agricultural practices pollute the soil to a large
50 ppb (ug dm-). Excessive lead cause anaemia, kidney extent. With the increasing use of fertilizers, pesticides,
dysfunction, nervous disorder, brain damage etc. herbicides, weedicides and soil conditioning agents to
increase the crop yield, the soil pollution problems
6. Other metals. The maximum permissible
have also been increased.
limit recommended of common metals in drinking
Fertilizers contaminate the soil with impurities
water and their ill effects are given alongside.
which come from raw materials during their
ENVIRONMENTAL POLLUTION

manufactures. For example, As, Pb and Cd present Among these DDT (dichloro dipheny] trichloro ethane)
in rock phosphate mineral get transferred to is an organo chlorine compound. Organo chlorine
superphosphate fertilizers. Since the metals are not compounds are stable in the environment, toxic to
degradable, these get accumulated in the soil due to insects In small amounts but much less so to humans,
the excessive use of phosphate fertilizers. Similarly, because they are not very soluble in water. These are
some metal ions such as Co**, Zn**, Ni**, Mn** etc.
persistent and show their biological activity for long
are added to the soil in trace amounts as
periods of time. However, these insecticides by
micronutrients. However, excessive addition of these
micronutrients may lead to their accumulation above accumulating in the environment affect many non
the toxic level. These may also cause soil pollution. target organisms in addition to the target pests.
Different kinds of pesticides are used to control 2. Herbicides
pests in the crops. Though these pesticides help in These are used to kill weeds or undesirable
getting boost in food production, yet the excessive vegetations. For example : sodium chlorate (NaClO,),
use has been of great concern because these cause sodium arsenate (NaAsO.,),2,4 dichlorophenoxy acid,
soil pollution. These pose a potential danger to the triazines, etc. Most herbicides are toxic to mammals
human health because eating of food contaminated but are not as persistent as organo-chlorides. These
with these chemicals may cause serious problems. chemicals decompose in a few months. Like organo-
Consequently, many of these chemicals have been
chlorides, these also become concentrated in the food
banned or restricted in use.
web. Same herbicides cause birth defects.
Thus, the role of pesticides in protecting crops
3. Fungicides
and generating pollution is very significant and
discussed below : These are used to control the growth of various
types of fungus and check plant diseases. For example,
Role of Pesticides in generating Pollution thiram (CH,), NCSS, phenyl mercury acetate,
The chemicals which are used for destroying bordeaux mixture. Generally, organic compounds of
pests are called pesticides. These are used to kill or mercury have been used as fungicides. However, these
control unwanted dangerous species of plants and compounds break down in soil and these have
animals. Pesticides are basically synthetic toxic disastrous consequences. In Iraq many deaths were
chemicals with ecological repercussions. The repeated reported in 1971-72 resulted from the people eating
use of the same or similar pesticides make some bread made from grain that had been treated by
pests resistant to that group of pesticides, thereby, fungicide methyl mercury.
making the pesticides uneffective. For example, when
4, Rodenticides
insect resistance of DDT increased, other organic
toxins such as Aldrin and Dieldrin, etc. were These are used to kill rodents (rats and mice). For
introduced. Most of the organic toxins are water example : warfarin, zinc phosphate.
insoluble and non-biodegradable. Therefore, these In addition pesticides also include nematicides
high persistent toxins are transferred from lower (which inhibit nematodes), molluscicides used to kill
trophic level to higher trophic level through food molluscs mainly snails and slugs), piscicides (used to
chain. Over the time, the concentration of toxins in control undesirable fish species and other aquatic
higher animals reach a level which causes serious animals).
metabolic and physiological disorders. In response to
A major Air Pollution Accident—Bhopal
high persistence of chlorinated organic toxins, a new
Gas Tragedy
series of less persistent or more biodegradable
Amagjor air pollution accident took place on the
products have been introduced. These are organo-
night of December 2, 1984 in Bhopal (Madhya
phosphates and carbamates. But these chemicals
Pradesh in the Union Carbide Factory. This is known
are severe nerve toxins and hence are more harmful
as Bhopal gas tragedy. On the chilly night, when the
to human beings. There are some reports of pesticides city of Bhopal was virtually asleep, at about 11.30 pm
related deaths of agricultural field workers. a dense cloud of deadly methyl isocyanate (MIC) gas
The pesticides may be classified in following leaked from a storage tank of Union Carbide Ltd.
categories: plant. This caused intense aching in the eyes of
1. Insecticides people around the factory. This resulted into deaths
These are used to kill insects in agricultural fields. of thousands of people and many suffered permanent
During world war II, DDT [2,2 (bis-p-chlorophenyl)- medical disability. Many women were badly affected
1,1,1- trichloroethane] was found to be of great use in and the worst victims being pregnant women.
controlling malaria and other insect borne diseases. Methyl isocyanate (MIC) was used to
Therefore, after the war, DDT was used in agriculture manufacture the insecticide, carbaryl marked under
to control the damages caused by in sects, rodents, the commercial name sevin in this plant. It was
weeds and various crop diseases. Other common prepared by the reaction of methylamine and
examples of insecticides are: malathion, carbaryl, BHC. phosgene.
sus MODERN'S abe + OF CHEMISTRY-XI

Biodegradable wastes are generated by food


CH,NH, + COCI,—+ CH,N =C =O + 2HCl
processing units, cotton mills, paper mills and textile
Phosgene MIC
factories.
MIC is stored in refrigerated moisture free Non-biodegradable wastes are generated by
tanks in Union Carbide Factory. It is very sensitive many industrial processes such as:
to moisture and itself is a volatile liquid (_) thermal plants produce fly ash
(b.p. 43.45°C). It has been investigated that probably (ii) iron and steel plants produce blast furnace
pressure rose in one of the tanks and opened its
slag and steel melting slag.
release value thereby allowing methyl] isocyanate
(iii) Industries manufacturing aluminium, zinc,
to escape into the atmosphere. As the clouds of
copper, etc, produce mud and tailings.
methyl isocyanate rose over the city, it caused
immense loss of life and injury to people. It proved (iv) fertilizer industries produe gypsum.
to be a big tragedy. (v) industries dealing with metals, chemicals,
drugs, pharmaceuticals, dyes, rubber, goods, etc.
Monitoring and Control of Pesticide produce hazardous wastes such as inflammables,
Pollution composite explosives or highly reactive species.
Excessive use of pesticides pose a potential The disposal of non-biodegradable industrial
hazard to man, animal, fish, vegetations, etc. These solid wastes, is very essential. If it is not done by a
spread most widely and alarming danger to the proper and suitable method it may cause serious threat
environment. The use of many toxic pesticides have to the environment. New innovative methods have
been banned. India is one of the few producers of been designed to make use of waste materials. Great
DDT in the world, which has been banned by USA care has to be taken for the disposal of toxic wastes.
since 1972 for environmental reasons. For example, nowadays, fly ash and slag from the steel
industry are used in the cement industry. Large
STRATEGIES TO CONTROL amounts of toxic and hazardous wastes are destroyed
ENVIRONMENTAL POLLUTION by controlled incineration whereas small quantities
To protect human life and lives of plants and are burnt along with factory garbage in open bins.
other animals, scientists are working hard to plan Moreover, solid wastes if not managed effectively will
certain strategies for control of environmental affect the components of environment.
pollution. In addition to solid waste which we see in Management of Domestic waste.
our household garbage bins, there are medical, The best way to manage domestic wastes are to
industrial, agricultural and mining wastes. The collect these in small bins which are transferred to
improper disposal of wastes is one of the major causes community bins by private or municipal workers.
of environmental pollution. Therefore, the From these community bins, the wastes are collected
management of wastes is of prime importance. Let and carried to disposable sites. At these sites the
us briefly study the methods to control other types of wastes are separated into biodegradable and non-
pollution caused by different sources. This involves biodegradable wastes. Non-biodegradable waste
two common strategies : meterials such as glass, plastic, metal scraps, etc. are
sent for recycling whereas biodegradable wastes are
A: Management of waste
dumped in land fills and are converted into compost.
B : Green chemistry.
If the waste is not collected in garbage bins,
it goes into the sewers. Some of it is eaten by dogs
A. MANAGEMENT OF WASTE
and cattles. Non-biodegradable wastes (polythene
The production and improper disposal of bags, metals scraps etc.) choke the sewers and
domestic and industrial wastes are responsible for cause inconvenience and problems. Polythene bags,
environmental pollution. The domestic waste includes if swallowed by cattle can cause their death also.
sewage and municipal waste while industrial wastes The poor management of waste causes health
include inorganic and organic suspended particles problems leading to epidemics due to contamination
and inorganic and organic soluble matter. of ground water. It is specially dangerous for those
Management of Industrial Waste. who are in direct contact with the waste such as
Most industries produce one or more unwanted rag pickers and workers involved in waste disposal
by products which must be discarded. The problem of because these persons handle waste materials mostly
solid waste disposal is one of the most serious problem without protective devices such as gloves or water
facing our society today. proof boots and gas masks. We are responsible for
Industrial solid wastes are sorted out as their lives.
biodegradable and non-biodegradable wastes. Therefore, the normal practice is that all the
domestic wastes should be properly collected and
ENVIRONMENTAL POLLUTION

disposed. These wastes are treated in the following : Aerobic conditions


ways: Organic waste + O. CO, + H,O
2 1273
1. Recycling This method disposes of the relatively inert PCBs
This is a simplest method in which the waste is (polychlorobenzenes) and the high temperature
recycled. When the materials are recycled, it saves generated allows the endothermic reactions such as
on the cost of raw materials as well as reduces the destruction of C—Cl bonds in organochlorine
costs of waste disposals. Some common examples of compounds. The main disadvantage of this process is
recycling by industries are : that it causes air pollution. Moreover, the ash from
(4) the collection and recycling of glass the municipal incinerators is very finely divided and
(ii) the use of scrap metal in the manufacture of can be ingested into the lungs. Incomplete combustion
steel of PCBs can cause formation of highly toxic chloro
(iii) recovery of energy from burning combustible compounds such as polychlorodibenzodioxins (PCDDs)
waste. and polychlorodibenzofurans (PCDFs).
(tv) the use of waste paper (6) Digestion
In other words, recycling converts waste into The sewage sludge can undergo anaerobic
wealth. digestion when micro-organisms degrade wastes in
2. Sewage treatment the absence of oxygen. This process can also be used
The sewage treatment involves the following to degrade a variety of toxic organic wastes. The
three types of stages : products of digestion are carbon dioxide and methane,
1. The first stage known as primary treatment which may be used as a fuel.
involves the removal of large solid particles. This is 6 Organic waste
done by mechanical process consisting of screening —_—> CO, (g) + CH, @)
C, H, O
and sedimentation. For this, the waste is filtered
through different types of screens. The solids that are (c) Dumping
removed are disposed off in land fill sites. It is then This is widely used practice of dumping the
allowed to flow into sedimentation tanks. This process sewage sludge in the nearby seas. It is known as
allows the removal of solids that settle out (called ocean dumping. However, the application of sludge to
sludge). This process also allows the removal of grease the land is increasing. The sludge contains nitrogen
which floats to the surface and can be skimmed off. and phosphorus which may be useful as a fertilizer.
2. The second stage known as secondary Urban areas produce sludge which contains many
treatment involves the biological oxidation of organic toxic substances, so the amount of such sludge dumped
content of waste materials by micro-organisms in this way must be carefully controlled.
followed by filtration. B. GREEN CHEMISTRY AS AN ALTERNATIVE
3. The third stage also known as final treatment TOOL FOR REDUCING AIR POLLUTION
is aserles of specialized physical and chemical processes We have learnt that increasing population,
that reduce the quantities of specific pollutants left after increased use of fossil fuels and increasing number of
primary and secondary treatments. These improve the industries and vehicles are polluting our environment
quality of waste water. The processes commonly used to an alarming extent. The increase in pollution
in this treatment ore chemical removal of phosphate, adversely affects the health of humans and other living
coagulation, filtration, treatment with activated organisms. As we know self sufficiency in food has been
charcoal, disinfection by chlorine, reverse osmosis, etc. achieved in India since late 20th century by using
During these operations sludge is produced fertilizers and pesticides and developing improved
which also poses a problem for safe disposal. The methods of farming, good quality of seeds, irrigation
sludge is dried and then may be incinerated, digested etc. But over exploitation of soil and excessive use of
or damped as explained below : fertilizers and pesticides have resulted into deterioration
(a) Incineration of soil, water and air.
This converts organic materials into CO, and
With the growing awareness of the
water. This is used for household waste, chemical
environmental pollution, the international community
waste and biological waste (e.g. from hospitals). Such
is under increasing pressure to find methods to save
treatment is carried out under aerobic conditions in
our lives from the increasing pollution. This does not
the presence of plentiful supply of oxygen and at very
mean that we should stop the process of advancement
high temperature (above 1273 K). The exhaust gases
that has been set in but to search methods which would
are filtered. This process reduces the volume of the
help in reduction of pollution of our environment. The
waste and inorganic ash is left behind which is
scientists are searching new alternatives which do not
disposed of as land fill.
cause environmental pollution. This new approach is
a MODERN'S abe + OF CHEMISTRY-XI

called Green Chemistry which was introduced in (vi) The use of starting materials—reagents and
early 1990s. It means zero discharge of toxic, persistent solvents that pose less hazard to man and his
substances, into the environment guaranted by the environment.
fact that they are never produced. Green chemistry (vit) The synthetic reactions may preferably be
may be defined as carried out in aqueous medium because water
a strategy to design chemical processes has high specific heat and low volatility. Water
and products that reduces or eliminates is cost effective, non-inflammable and does not
the use and generation of hazardous have any carcinogenic effects.
substances. (viit) The raw materials and methods should produce
This would bring about minimum pollution or less waste to avoid their treatment and problem
deterioration to the environment. In other words, of disposal.
ereen chemistry aims to virtually eliminate toxic, (ix) The use of renewable rather than depleting
persistent substances from the environment by resources wherever possible.
ensuring their no further releases and destroying (x) The use of biotechnological alternatives.
existing deposits of these chemicals. So, green (xt) New methods and tools for evaluating
chemistry means environmental friendly or no environmental impact.
pollution. For example, we have been using organic (xit) The awareness among common man to use
solvents such as benzene, toluene, carbon tetra- ereen products.
chloride, etc. as a media for many reactions. These Green chemistry in Day-to-Day Life
are highly toxic. Now, scientists are planning greener Some common examples of green chemistry in
alternative to use water or non polluting solvents as a our day-to-day life are:
medium in place of organic solvents. Similarly, instead
(¢) Dry cleaning of clothes.
of using synthetic materials for packing, green
chemistry plans to use environmental friendly Tetrachloroethene (C1,C = CCl1,) has earlier used
recyclable or safely disposable materials. as solvent for dry cleaning. This compound is suspected
Green chemistry also requires change in our to be carcinogenic and contaminates the ground water.
habits and life-style. These days many green products The process of using this compound has now been
are available in the market in developed countries. replaced by a new process in which liquefied carbon
dioxide alongwith suitable detergent is used. This
BASIC AIMS OF GREEN CHEMISTRY causes less harm to ground water. Replacement of
Since the inception of green chemistry, scientists halogenated solvent by liquid CO, will result in less
from all over the world are using the creative and harm to ground water.
innovative skills to develop new synthetic methods, Similarly, hydrogen peroxide (H,O,) is used for
new processes, analytical tools, reaction conditions, the purpose of bleaching clothes in the process of laundry.
catalysts, etc. So, green chemistry prevents problems This gives better results and makes use of less water,
before they occur by designing new approaches. therefore, saving a lot of water. It is also not harmful.
Green chemistry considers the full life cycle impacts (it) Bleaching of paper
of a product at the initial design stage. The basic
aims of green chemistry are : Chlorine gas was used earlier for bleaching paper,
which is highly toxic chemical. Its use has been
(1) Reformulation of synthetic routes so that replaced by hydrogen peroxide with a suitable catalyst
hazardous substances do not enter into the which promotes the bleaching action of hydrogen
atmosphere. peroxide.
(it) During synthesis, care must be taken to select
(tit) Synthesis of chemicals.
starting materials that can be converted into
Kthanal (CH,CHO) is commercially prepared
end products with allmost 100% yield. This can
nowadays by one step oxidation of ethene in the
be achieved by arriving at optimum conditions
presence of ionic catalyst in aqueous medium:
of synthesis.
(iii) The methods used to obtain starting materials
CH, = =CH, =— +0,
Catalyst Pd(IT),CutIT)
e.g. mining, refining, etc. should have minimum > CH,CHO(90%)
in
l water
te Ftanal
impact on the natural environment. Ethene

(iv) New routes for the production of green The yield is about 90.0%.
chemicals and materials.
Thus, we can say that green chemistry is a cost
(v) The development of environmentally improved
effective approach which involves reduction in
methods for the existing industrial processes.
material, energy consumption and waste generation.
The material inputs should be of low or no
toxicity or at least reduced toxicity compared Always remember environment protection begins
with us.
with the traditional method.
ENVIRONMENTAL POLLUTION 14/27 =>

To save our environment, we all as human beings have to think seriously. If we follow some basic steps
as an individual to keep our environment pollution free, we can contribute significantly towards a better
quality of our environment and human life.
* Always set up a compost pit in your garden or any other place in your home and use it to produce
manure for your plants. This will reduce the use of fertilizers.
* Always use a cloth bag instead of plastic carry bags when you buy vegetables, fruits, groceries or any
other item.
* Take care that all newspapers, glass, aluminium and other items in your area are recycled.
* We must realize that though we donot have solutions for every problem but we can concentrate on
issues and try to find their proper solutions.
* We should always take care to put into practice whatever we preach.
Always remember, environmetal protection begins with us.

ade Op - {Queso SS

Q.1. Explain giving reasons" the presence of CO reduces the amount of the haemoglobin available
in the blood for carrying oxygen to the body cells".

Ans. CO is a poisonous gas and it binds to haemoglobin of the blood more readily (about 200 times)
than oxygen to form carboxy haemoglobin.
Hb + CO — HbCO
Carboxy haemoglobin
The presence of CO, therefore, reduces the amount of the haemoglobin available in the blood for
the transport of oxygen to the body cells.
Q.2. Fish do not always grow as well in warm water as in cold water. Why ?
Ans. The amount of dissolved oxygen in warm water is less than in cold water.
Q.3. Name four major Greenhouse gases.
Ans, (i) Carbon dioxide (ii) Methane (iii) Ozone (iv) Chlorofluorocarbon compounds.
Q.4. Why is acid rain considered as a threat to Taj Mahal ?
Ans. Taj Mahal is made of marble. The acid rain contains H,SO, which attacks the marble (CaCO,):
CaCO, + H,SO, —— CaSO, + H,O + CO,
As a result, the marble becomes pitted and weakened mechanically. Therefore, the monument is
being slowly eaten away and the marble is getting discoloured and lustreless.
Q.5. Why does the rain water normally have a pH of about 5.6 ? When does it become acid
rain ?
Ans. Normally rain water has a pH of about 5.6 due to the dissolution of CO, of the atmosphere into it :
H,O(/) + CO, (g) —> H,CO,
H,CO, ——> 2H* + CO,”-
When the pH of rain water falls below 5.6, it becomes acid rain.
Q.6. What is pneumoconiosis ? How does it occur?
Ans. Pneumoconiosis is a disease of lung which is caused due to inhalation of small particles (mist,
smoke, fumes and dust). These particles irritate the lungs and exposure of such particles for long
periods of time causes scarring or fibrosis of the lung lining.
@.7. Name the gas which caused Bhopal Gas Tragedy.
Ans. Methyl isocyanate (CH, N = C = O) or MIC.
a 14/28 MODERN'S abe + OF CHEMISTRY-XI
Q.8. Name four tropospheric pollutants.
Ans. The tropospheric pollutants may be gaseous or particulate. For example,
Gaseous : Oxides of sulphur, Oxides of nitrogen
Particulate : Dust, Smoke.
Q.9. What is the major cause of ozone layer destruction ?
Ans. The major cause of ozone layer destruction is the release of chlorofluorocarbon compounds (CFCs)
know as freons in the atmosphere.
Q.10.List main differences between classical smog and photochemical smog.
Ans. The main difference between classical smog and photochemical smog are :

Classical smog Photochemical smog


This is formed due to buildup of sulphur oxides . This is formed due to photochemical reaction of
and particulate matter from fuel combustion. sunlight on the nitrogen oxides and hydrocarbons
produced by automobiles and factories.
It involves smoke and fog. . It does not involve any smoke or fog.
It occurs in cool humid climate (in winter). . It occurs in warm, dry and sunny climate (in
summer).
This type of smog was first observed in London .. This type of smog was first observed in Los
in 1952. Angeles in 1950.
It has high concentration of SO, and, therefore, 5. Ithas high concentration ofoxidising agents and,
is reducing in character. therefore, is oxidising in character.
It causes bronchitis and irritation 1.e., problems _ It causes irritation in eyes.
in lungs.

@. 11. What is the importance of measuring BOD of a water body?


Ans. BOD is the measure of level of pollution caused by organic biodegradable material in a water
body. The clean water would have a BOD value of less than 5 ppm whereas higher values of
BOD indicate polluted water.
Q. 12. Define eutrophication.
Ans. The process of nutrient enrichment of water bodies and subsequent loss of biodiversity is called
eutrophication.

|~ Key Terms | }
Contaminant: A substance which does not occur in nature but is introduced by human activity into the atmosphere
effecting its composition.
Pollution: The addition of any foreign material or any physical change occurring in nature which may harm or
affect living organisms directly or indirectly.
@ Smog: A combination of smoke and fog laced with SO,
® Acid rain Rain water containing acids such as H,SO, and HNO, and has pH of 4-5.
@® Greenhouse effect: Global warming due to re-emision of sun’s energy absorbed by earth followed by its absorption
by CO, molecules and water vapour near the earth and then radiation back to the earth.
Green chemistry: A strategy to eliminate toxic, persistent substances from the environment.
ENVIRONMENTAL POLLUTION

QUICK CHAPTERET ITT TJ


UP
AIR POLLUTANTS

Primary pollutants Secondary pollutants


® harmful chemical substance that e harmful chemical substance that is
directly enters the air formed in the air by chemical reactions

Common air pollutants


e CO, 50,, NO, NO, H,5, Cl, NH,, hydrocarbons, particulates, chlorofluorocarbons
= CO is the major air pollutant

SMOG ACID RAIN


is a mixture of smoke and fog. When the pH of the rain water falls below 3.6, it becomes
Classical smog: formed by the combination of smoke, acidic and is called acid rain.
dust and fog containing sulphur dioxide from polluted air. It is rain water containing acids such as HNO, and H,s0,
Photochemical smog: formed by the combination of smoke, which are formed by the oxides of nitrogen and sulphur
dust and fog with an air pollutant in the atmosphere as present in air as pollutants.
a result of photochemical reaction.
Peroxyacylnitrate (PAN) is a constituent of photochemical Acid Tain reacts with statues made of marble, limestone,
smog. etc. Acid rain Teacts with marble (CaCO.).
Payor CaCO,/+ H,SO,—> CaSO, + H,O + CO,
As a result, the marble becomes pitted and weakened
mechanically because the soluble salts are easily leached
out by rain water.
Acid rain attacks marble of Taj Mahal of Agra.

OZONE BLANKET GREEN HOUSE EFFECT


The thick layer of ozone is called ozone blanket because The warming of the earth or global warming due to
it absorbs harmful ultra-violet rays given out by the sun. re-emission of sun's energy absorbed by the earth followed
Therefore, the ozone layer is also known as protective by its absorption by carbon dioxide molecules and water
shield. vapour present near the earth and then its radiation back
The ozone layer 1s getting depleted due to some natural to the earth is called greenhouse effect.
processes of industrial activities ’ Green house gases are
The depletion of ozone layer is because of its reactions CO,, CH, O,, H,O vapour, CFCs
with chlorofluorocarbons (CFC)

WATER POLLUTION

* Water can be polluted by pathogens, heavy metals and pesticides

BOD & COD — Contaminants in Water


Amount of oxygen required by microorganisms to break PH of drinking water should be between 5.5. to 9.5.
down the organic matter present in certain volume of a F- ion concentration should be upto 1—1.5 ppm.
sample of water is called biochemical oxygen demand, BOD F™ ions make the enamel on teeth harder by converting
Chemical oxygen demand (COD) is a measure of [3Ca,(PO.),-Ca/OH),] to [3Ca,(PO,),.CaF,]
biological oxidisable as well as biological inert organic matter Excessive concentration of sulphate above 500 ppm
Clean water would have BOD value <5 ppm while produces laxative effects.
BOD of +17 indicates highly polluted water. Excessive nitrate causes methemoglobinemia (blue baby
syndrome).
Pesticides Excessive lead causes anaemia, kidney dysfunction, brain
e Insecticides. DDT, malathion, carbaryl, BHC damage.
e Herbicides. Sodium chlorate, sodium arsenate,
2,4-dichlorophenoxy acid, triazines
Fungicides. Thiram, phenyl mercury acetate, bordeaux
mixture Green chemistry is a strategy to eliminate toxic, persistent
Rodenticides. Warfarin, zinc phosphate substances from the environment.

WWW.JEEBOOKS.IN
MODERN'S abe + OF CHEMISTRY-XI

A) NGS Solyed
Textbook Exercises //
Define environmental chemistry. statues and monuments because limestone reacts
Environmental chemistry 1s defined as the branch with rain water and gets damaged.
of science which deals with study of origin, transport, CaCO, + H,SO, ——+CaSO, + CO, + H,O
reactions, effects and fates of chemical species in the (From acid rain)
environment. It deals with social, economical,
As a result, these monuments are being slowly eaten
biological, physical and chemical interrelations with
away and the marble gets discoloured and lusterless.
our surroundings.
Q. 2. Explain tropospheric pollution in 100 words.
Q. 6. What is smog? How is classical smog different
from photochemical smog?
Ans. The tropospheric pollution occurs because of the Smog is a major air pollutant in big cities. It is a
presence of undesirable gaseous and solid particles
mixture of smoke, dust particles and small drops of
in the air. The main pollutants in the troposphere
fog.
are as follows:
Smoke + Fog ———> Smog
(i) Gaseous air pollutants. The gaseous air
pollutants include oxides of sulphur (SO,, SO,), Differences between classical and photochemical
nitrogen (NO, NO,), carbon (CO, CO,), hydrogen smog. Refer Conceptual Questions, Q, 10 (Page 28).
sulphide (H,S), hydrocarbons, ozone and other Q. 7. Write down the reactions involved during the
oxidants. formation of photochemicalsmog.
(iz) Particulate pollutants: These pollutants are Ans, Refer Text page 12.
dust, fumes, mist, spray, smoke, etc. Q. 8. What are the harmful effects of photochemical
Q. 3. Carbon monoxide gas in more dangerous than smog and how can they be controlled?
carbon dioxide gas. Why? Ans. Refer Text page 12.
Ans. Carbon monoxide Is a poisonous gas and it binds to Q. 9. What are the reactions involved for ozone layer
haemoglobin of the blood more readily (about 200 depletion inthe stratosphere?
times) than oxygen to form carboxy haemoglobin. Ans Refer Text page 15.
Hb + CO —— HbCO Q. 10. What do you mean by ozone hole? What are its
Carboxy haemoglobin consequences?
The presence of CO, therefore, reduces the amount Ans. Refer Text page 15-16.
of haemoglobin available in the blood for the Q. 11. What are the major causes of water pollution?
transport of oxygen to the body cells. The harmful Explain.
effects of enhaling increasing amounts of CO include Ans. Refer Text page 19.
reduction in awarness of judgement, dizziness, weak Q. 12. Have you ever observed any water pollution in
eye sight, headache, nervousness and cardiovascular your area? What measures would yousuggest to
disorders. At higher concentrations of CO, control it?
suffocation, loss of consciousness or even death Ans Please try yourself.
after several hours. However, CO, does not combine
with haemoglobin and hence is less harmful
Q. 13. What do you mean by Biochemical Oxygen
Demand (BOD)?
pollutant. CO, is main contributor towards, green
Ans Refer Text page 20-21.
Q.4.
house effect and global warming.
List gases which are responsible for greenhouse
Q. 14, Do you observe any soil pollution in your
neighbourhood? What efforts will you make for
effect.
controlling the soil pollution?
Ans. CO, is the main gas which causes greenhouse effect.
Ans Please try yourself.
Other greenhouse gases are methane, ozone, water
vapour, nitrous oxide, chlorofluorocarbons (CFC’s). Q. 15. What are pesticides and herbicides? Explain
giving examples.
Q. 5. Statues and monuments in India are affected by
acid rain. How? Pesticides are synthetic chemicals used for
destroying pests. These are used to kill or destroy
Ans. The air around statues and monuments in India
unwanted dangerous species of plants and
contains fairly high concentrations of oxides of
animals. These are basically synthetic toxic
sulphur and nitrogen. This is mainly because of a
chemicals with ecological repercursions. The
large number of industries and power plants in the
repeated use of the same or similar pesticides
nearby areas. The presence of these oxides cause
make some pests resistant to that group of
acid rain. This acid rain causes extensive damage to pesticides, thereby, making the pesticides
ENVIRONMENTAL POLLUTION

uneffective. For example, when insect resistance dumping but you find an abundance of phytoplank-
of DDT increased, other organic toxines such as ton. Suggest a reason for the fish kill.
Aldrin and Dieldrin, etc. were introduced. Most Ans, Excessive phytoplanktons (i.e., organic pollutants such
of the organic toxins are water insoluble and as leaves, grass, trash, etc.) present in water is
non-biodegradable. Therefore, these high biodegradable. A large population ofbacteria decompose
persistent toxins are transferred from lower this organic matter in water. As a result, they consume
trophic level to higher trophic level through food the oxygen dissolved in water and therefore, the
chain. Over the time, the concentration of toxins concentration of dissolved oxygen becomes low. If the
in higher animals reach a level which causes concentration of dissolved oxygen in water becomes
serious metabolic and physiological disorders. below 6 ppm, the survival of fish becomes difficult.
Herbicides are used to kill weeds or undesirable Hence the fish die and float dead on lake.
vegetations. For example, sodium chlorate Q. 19. How can domestic waste be used as manure?
(NaClO,), sodium arsenate (NaAsQ,),
Domestic wastes consist of two types of materials;
2, 4 dichlorophenoxy acid, triazines, etc.
biodegradable such as leaves, rotten food, etc.and
Q. 16. What do you mean by green chemistry? How non-biodegradable such as glass, plastic, metal scrap,
will it help decrease environmental pollution? etc. The non-biodegradable waste is sent to industry
Ans. Refer Text pages 25-26. for recycling. On the other hand, the biodegradable
Q. 17. What would have happended if the green- waste should be deposited in the land fills on the
house gases were totally missing in the earth’s nearby areas. With the passage of time, it gets
atmosphere? Discuss. converted into useful manure (compost).
Ans, The solar energy radiated back from the earth . For your agricultural field or garden you have
surface 1s absorbed by greenhouse gases such as developed a compost producing pit. Discuss the
CO,, CH,, O,, CFC’s, water vapour, etc. present processin the light ofbad odour, flies and recycling
near the earths surface. Therefore, they heat up of wastes for a good produce.
the atmosphere near the earths surface and keep it Ans. The compost is very useful for agriculture as a
warm. Therefore, they keep the temperature of fertilizer. But the compost producing pits may give
earth constant and help in growth of plants and bad odour and flies. Therefore, the compost produc-
existence of life on earth. If there are no greenhouse ing pit should be set up at a suitable place or in a tin
gases, there would have no vegetation on earth and to protect ourselves from bad odour and flies. It must
hence no life on earth. be kept covered so that flies connot enter into it and
. Alarge number of fish are suddenly found float- there is not much of bad odour.
ing dead on alake. There is no evidence of toxic

Exemplar Problems //
Subjective Questions
stratosphere is very important. Explain what
» Short Answer Questions Corning
Fordmark C4 would happen if ozone from this region
completely removed ?
Q. 1. Green house effect leads to global warming. Ozone is an important constituent of the stratosphere.
Which substances are responsible for green Ozone layer acts as a protective blanket or protective
house effect?
shield because it is very effective in absorbing harmful
Ans. The heating of earth due to trapping of radiation is ultra-violet rays given by the sun. Since living cells
called green house effect. The gas such as CO,, CH, can be destroyed by ultra-violet radiations, the ozone
N,O, CFCI1,, CF,Cl,, O, etc. trap these radiations
layer protects us from its damaging effects.
and are called greenhouse gases.
Dissolved oxygen in water is very important for
Q. 2. Acid rain is known to contain some acids. Name aquatic life. What processes are responsible for the
these acids and where from they come inrain?
reduction of dissolved oxygen in water?
Ans. Acid rain contains acids such as HNO,, H,SO, and The discharge of human sewage and organic waste
H,CO, (alongwith small amount of HC\).
from paper and pulp industries and leaves, grass,
HNO, is formed by the oxidation of NO present in air
trash etc. in water result in growth of phytoplanktons.
to NO, and NO, and subsequent dissolution in water.
These micro-organisms which decompose this organic
H,50, is formed by the oxidation of SO, present in
matter require oxygen. As aresult, amount of oxygen
air to SO, and subsequent dissolution in water.
H,CO, is formed by the dissolution of CO, of the air in water of lakes, etc. decreases.
in water. On the basis of chemical reactions involved, ex-
Ozone is a toxic gas and is a strong oxidising plain how dochlorofluorocarbons cause thinning
agent even then its presence in the of ozone layer instratosphere.
| 14132 MODERN'S abe + OF CHEMISTRY-XI

Ans. Chlorofluorocarbons are stable compounds. They Ans. BOD is the measure of level of pollution caused by
move to stratosphere by random diffusion. These organic biodegradable material in a water body. The
undergo decomposition in the presence of sunlight to clean water would have a BOD value of less than 5 ppm
release Cl atoms. These Cl atoms cause catalytic whereas higher values of BOD indicate polluted water.
chemical reactions and cause significant depletion
of ozone layer as shown below : Q. 11. Why does water covered with excessive algal
growth become polluted?
CF,Cl, _2v, Cl'+CF,cr’ Presence of excessive algal growth shows that water
contains a lot of phosphate due to inflow of fertilizers,
Cl+0O, —— ClO'+0,
etc from the surroundings. Hence, such a sample of
ClO’ +O — > Cl +0, water is polluted.
What could be the harmful effects of improper Q. 12. A factory was started near a village. Suddenly
management of industrial and domestic solid villagers started feeling the presence of irritating
vapours in the village and cases of headache, chest
waste inacity?
pain, cough, dryness of throat and breathing
Ans. If the domestic waste in a city is not properly problems increased. Villagers blamed the
managed, it may go into sewers or may be eaten up emissions from the chimney ofthe factory for such
by the cattle. problems. Explain what could have happened.
The non-biodegradable waste such as polythene Give chemical reactions for the support of your
bags, metal scraps, ete may block the sewers. The explanation.
polythene bags if swallowed by the cattle can result The symptoms observed in villagers show the
in their death. presence of oxides of nitrogen and sulphur in the
Sumilarly, if industrial waste 1s not properly managed, environment. These must be coming out of the
it will cause pollution of air, water and soil. chimney of the factory. This is due to the combustion
of fossil fuels such as coal, oil, natural gas, gasoline
During an educational trip, astudent of botony
ete. to produce high temperature at which oxidation
saw a beautiful lake in a village. She collected
of atmospheric nitrogen takes place forming nitric
many plants from that area. She noticed that
oxide (NO) and nitrogen dioxide (NO,) as:
villagers were washing clothes around the lake
and at some places waste material from houses 1200—1750°C
N, 2 +O, 2 == 2NO
was destroying its beauty.
After few years, she visited the same lake again.
She was surprised to find that the lake was
2NO +0, —=~= ono,
covered with algae, stinking smell was coming The sulphur present in coal and fuel oil on combustion
out and its water had become unusable. Can you gives SO,. Sulphur dioxide is also produced by the
roasting of sulphide ores such as copper pyrite
explain the reason for this condition of the lake?
(CuFeS,), iron pyrite (FeS,), copper glance (Cu,S) etc.
Ans. The inflow of domestic waste from houses and
Cu,
5 +0, ——> 2Cu + SO,
fertilizer of plants and the entry of soaps and
detergents result into accumulation of phosphate in 2FeS, +50, ——> 2FeO + 450,
water. As a result, formation of algae increases Q. 13. Oxidation of sulphur dioxide into sulphur
which covers the water surface and concentration of
trioxide in the absence of a catalyst is a slow
dissolved oxygen decreases. Hence a process called
process but this oxidation occurs easily in the
eutrophication occurs.
atmosphere. Explain how does this happen. Give
Q. 8. What are biodegradable and non-biodegradable
chemical reactions for the conversion of SO, into
pollutants?
sO,.
Ans. Biodegradable pollutants are those which can be
The conversion of sulphur dioxide into sulphur trioxide
decomposed by bacteria. For example, dust particles,
sewage, cow dung etc. is catalysed by the presence of particulates in the air.
Non-biodegradable pollutants are those which cannot 280, + O, Particulates 290
be decomposed by bacteria. For example, plastic 3

materials, mercury, aluminium, DDT, etc. Q. 14. From where does ozone come in the photochemi-
calsmog?
Q. 9. What are the sources of dissolved oxygen in
water? In the presence of sunlight, ultra violet rays convert
NO, present in air to NO and atomic oxygen. Some of
Ans. The main sources of dissolved oxygen in water are :
the oxygen atoms combine with O, in the air to form
(i) Photosynthesis (it) Natural aeration ozone.
(iii) Mechanical aeration
U.V.
NO, —2.> NO +0
Q. 10. What is the importance of measuring BOD ofa
water body?
0, +O ———__>0,
ENVIRONMENTAL POLLUTION

Q. 15. How is ozone produced in stratosphere? vehicles which have zero pollution should be used.
Ans. Ozoneinstratosphereis formed bythe photochemical Q.20.Green plants use carbon dioxide for
photosynthesis and return oxygen to the
reaction of oxygen as:
atmosphere, even then carbon dioxide is
Oo, =" 626 considered to be responsible for green house
effect. Explain why?
0+0, —~> QO, Ans. The amount of CO, produced due to human
Q. 16. Ozoneisa gas heavier than air. Why does ozone activities such as burning of fossil fuels (coal,
layer not settle down near the earth? natural gas, petroleum, etc.) and production of lime
Ans. Ozone layer is formed in the stratosphere at an from limestone is much more than that consumed
during photosynthesis. Moreover, consumption in
altitude of 25 — 30 km from the earth's surface. At
photosynthesis has further decreased due to
this altitude, the force of gravity is negligible and
deforestation.
therefore, ozone layer does not settle down. Q. 21.Explaim how does green house effect cause global
Q. 17. Sometime ago formation of polar stratospheric warming.
clouds was reported over Antarctica. Why were Ans. Refer Text (Page 16).
these formed? What happens whensuch clouds Q. 22.A farmer was using pesticides on his farm. He
break up by warmth ofsunlight? used the produce of his farm as food for
rearing fishes. He was told that fishes were
Ans. Refer text (page 17-18).
not fit for human consumption because large
Q. 18. A person was using water supplied by amount of pesticides had accumulated in the
Municipality. Due to shortage of water he tissues of fishes. Explain how did this happen?
started using underground water. He felt laxa- Ans. The pesticides travelled from the soil into the crops
tive effect. What could be the cause? produced by the farmer. From the crops, used as
Ans. The laxative effect is observed only when the food for fish, the pesticides entered into water from
concentration of sulphates in water is greater than where they finally entered into the bodies of the
fishes. Therefore, the fishes were not fit for human
500 ppm. Sulphate is harmless at moderate
consumption.
concentration but concentration above 500 ppm
produces laxative effects and hypertension. Q. 23.For dry cleaning, in the place of
tetrachloroethene, liquefied carbon dioxide with
» Long Answer Questions <4 suitable detergent is an alternative solvent. What
type of harm to the environment will be
Q. 19.How can you apply green chemistry for the prevented by stopping use of tetrachloroethene?
following: Will use of liquefied carbon dioxide with
(i) to control photochemical smog. detergent be completely safe from the point of
(a) to avoid use of halogenated solvents in view of pollution? Explain.
drycleaning and that of chlorine in bleaching. Ans. Tetrachloroethene (Cl,C = CCl,) has earlier been
(ai) to reduce use of synthetic detergents, used as a solvent for dry cleaning. This compound
(iv) toreduce the consumption of petrol and diesel. is suspected to be carcinogenic and contaminates
Ans. (i) Refer Text (Page 25). the ground water. This harmful effect has not been
(it) Refer Text (Page 26). prevented by using liquefied CO, alongwith suitable
(iit) Soaps should be used in place of synthetic detergent.
detergents because soaps are 100% biodegradable However, use of liquefied CO, with detergent will
and donot cause any pollution problems. not be completely safe because detergents also cause
Nowadays, soft detergents are available which are pollution, as most of the detergents are non-
biodegradable and hence these may be used. biodegradable. Moreover, liquefied CO, will
(iv) CNG (Condensed Natural Gas) may be used ultimately enter into the atmosphere and add to
because it causes less pollution and electrical greenhouse effect.

Memory TEST Ww

6. The clean water would have a BOD value of more


1. Acid rain is due to the presence of oxides of sulphur in than 17.
ar. 7. Fish grows better in warm water than in cold water.
2. Classical smog is reducing in nature. 8. The major cause of ozone layer destruction is the
3. Ozone is present in stratosphere of the atmosphere. release of freons in the atmosphere.
4, N, and O, are the two main greenhouse gases. 9. Mercury is a biodegradable pollutant.
5. Excess sulphate in water causes blue baby syndrome. 10. Mesosphere is higher than stratosphere.
MODERN'S abe + OF CHEMISTRY-AI

1:3] Complete the missing links OF Choose the correct alternative


1, Carbon monoxide combines with haemoglobin of red
1. Carboxyhaemoglobin is more/less stable than
blood corpuscles to form ............ .
oxyhaemoglobin.
. The abbreviation PAN stands for .............. .
2. Mercury is a biodegradable /non-biodegradable
. London smog is ................ IN nature. pollutant.
. The gas leaked in Bhopal gas tragedy was ........... .
3. When pH of rain becomes more/less than 5.6, it
bo
Go
Om. Acid rain contains .......... and ........... and has pH of becomes acid rain.
4. Photochemical smog is oxidising /reducing in nature.
=r). The loss of green colour in plants due to the presence
of even a low concentration of SO, is called ......... ; on . Major pollutant in air is CO/SO,,.
7. The amount of dissolved oxygen needed by aerobic 6. Natural decay of animal and vegetable matter is the
bacteria to break down the organic matter is called main source of H,S/NO air pollutant.
7. The main greenhouse gas is SO,/CO,.
8. The process of nutrient enrichment of water bodies
8. Chlorofluorocarbons cause damage /prevent damage
and subsequent loss of biodiversity is called .......... . of ozone layer.
9. The heating of earth due to trapped radiation is
called ........ .
9. Water having BOD value of 5 ppm is highly polluted /
clean water.
10. Anything which is affected by the pollutants is called
10. The region of stratosphere is above/ below
troposphere.

elle Memory TEST Ww

Say True or False 3. reducing 4, methyl isocyanate


5. sulphuric acid, nitric acid, 4 to 5.
1. False : Acid rain is due to the oxides of sulphur and
nitrogen in the alr. 6. chlorosis 7. biological oxygen demand (BOD)
2. True 3. True 8. eutrophication 9. greenhouse effect
4, False 0. receptor.
5. False: Excessive nitrate causes blue baby syndrome.
6. False 7. False
8. True
1. more 2. non-biodegradable
9. False : Mercury is a non-biodegradable pollutant
10. True. 3. less 4, oxidising 5. CO
6. H,5 7. CO, 8. cause damage
Complete the missing links
9. clean 10. above.
1. carboxyhaemoglobin 2. peroxyacyl nitrates

<
. Why is ozone called protective shield ?
. What is an ionosphere ? What does it contain ?
1. Define pollutant. 11. What is greenhouse effect ?
2. Why is carbon monoxide a pollutant although it is a 12. Name two oxides of nitrogen which are pollutants in
colourless and odourless gas ? the atmosphere.
3 . Name two gaseous air pollutants. 13. What is photochemical smog ?
4 . Which region of atmosphere contains ozone ? 14, What is PAN ?
5 . Define environmental chemistry.
15. Why is CO a potentially dangerous air pollutant ?
6. Name two greenhouse gases.
16. Name two highly toxic compounds of mercury.
7. Name four regions of atmosphere.
17. Why is CO a very harmful pollutant of air ?
8 . Name major air pollutant.
18. List two health problems caused by SQ,.
ENVIRONMENTAL POLLUTION

19. Name the compounds which cause the depletion of 27. The depletion of ozone layer occurs over Antarctica
ozone layer. during spring time and it gets replenished after spring
20. How is ozone formed in the atmosphere ? time. Explain.
28. Why is acid rain considered a threat to Taj Mahal ?
21. How does CO, cause environmental pollution ?
29. Why does the rain water normally have a pH of about
5.6 ? When does it become acid rain?
lei melts cM Carrying 2 or 3 marks
<4 oO. Give a brief account of chemical reactions occurring in
1. What is environmental chemistry ? Discuss its social atmosphere.
relevance. ol. Write down the reactions involved during the formation
2. What is the difference between pollutant and of photochemical smog.
contaminant? oo. What are harmful effects of photochemical smog and
how can they be controlled ?
a. Discuss the sources, sinks of CO as air pollutant. How
is this pollution controlled ? cbads What are the reactions involved for ozone depletion in
4, What is the difference between the primary and stratosphere ?
secondary pollutants ? Give examples. oA, What are pesticides and herbicides ? Explain giving
How can internal combustion engines be modified to examples.
make auto exhaust free from pollutants ? wba). What do you mean by green chemistry ? How will it
What is the major air pollutant ? Give its important help to decrease environmental pollution ?
sources and its harmful effects on human life. 36. What are major causes of water pollution ? Explain.
Carbon monoxide is more dangerous than carbon of”. How do chemical industries cause environmental
dioxide gas. Explain. pollution ?
Explain giving reasons "the presence of CO reduces 38. What pollutants are released by motor vehicles ? How
the amount of haemoglobin av ailable in the blood for can we minimize damage caused by them ?
carying oxygen to the body cells". 39. What do you understand by BOD and COD ? How are
9. Discuss the effect of particulate pollution on human these determined ?
life. 4(). What do you understand by
10. What is photochemical smog ? discuss the possible (i) Polar stratospheric clouds (ii) Polar vortex
mechanism of its formation.
41. Define an environmental pollutant. What do you
11. What is acid rain ? What are its harmful effects ? understand by an environmental pollution model ?
12. How does oxygen reach water ? How is the oxygen 42. What is the cause of acid rain ? How is it harmful to
content of a sample of water measured ? environment ?
13. What do you understand by the term eutrophication? 43. Why does greenhouse effect lead to global warming ?
How does it threaten the development of fish ? What could be the consequences of global warming ?
14, What is pneumoconiosis ? How does it occur ? qd, Write a short note on Bhopal Gas tragedy ?
15. What are smogs ? Distinguish between classical and 45. Fish do not always grows as well as in warm as 1n cold
photochemical smogs. water, why ?
16. How does SO, cause pollution ? Explain as to how 416 "Oxygen plays a key role in the troposphere while ozone
this can be checked. in the stratosphere" Explain.
1%. Define the terms : Pollutant, contaminate, source and 47. What do you understand by ozone hole ? What are its
sink. Give one example of each. consequences ? Why does it occur mainly over
18. What is acid rain ? Describe its role in environmental Antarctica ?
pollution.
19. Write brief account of air pollution caused by oxides
of nitrogen. »> Long Answer Questions <q
20. “Carbon dioxide is non-toxic but it is an important 1. Discuss the various regions of the atmosphere. Explain
environmental problem facing us at present.” the different chemical reactions occurring in the
Comment on the statement. atmosphere.
21. Giving equations explain how does nitrous oxide 2. What is air pollution ? Explain briefly the major
present in atmosphere cause depletion of ozone.
atmospheric pollutants.
22. Why is the use of chlorofluoro carbons being oe How is ozone formed in the atmosphere ? What are the
discouraged ? Explain.
causes of depletion of ozone layer ? Explain harmful
23. What do you understand by Greenhouse effect ? What effects of depletion of ozone layer.
are the major Greenhouse gases ?
4, What is greenhouse effect ? What are its effects ?
24, Explain the difference between primary and secondary
a. Write short notes on:
pollutants.
(a) Acid rain
25. What is green chemistry ? Explain.
(b) Green chemistry as an alternative tool for reducing
26. Give harmful effects of depletion of ozone layer.
pollution.
| 14136 MODERN'S abe + OF CHEMISTRY-XI

6. Discuss the sources , sinks of carbon monoxide as an 10. Explain the following :
air pollutant. How is this pollution controlled ? (z) Smoke (ii) Fumes (72) Dust (iv) Mist
7. Discuss briefly the pollution caused by oxides of 11. Explain how does green house effect cause global
sulphur and nitrogen. warming.
8. Write short notes on : 12. How can you apply green chemistry for the following :
(1) Smog (a) toreduce the consumption of petrol and diesel.
(ii) Depletion of ozone layer (b) to control photochemical smog.
(zi) Greenhouse effect. (c) to reduce use of synthetic detergents.
9, Discuss five major water pollutants giving their (d) to avoid use of halogenated solvents in drycleaning
sources. and that of chlorine in bleaching.

Additional Useful Iinforgop and Objective Questions

ADDITIONAL USEFUL INFORMATION

> CHEMICAL AND PHOTOCHEMICAL dissociation of oxygen by ultraviolet radiation (EK = hv).
REACTIONS IN ATMOSPHERE 0,+hv ——> O0+0
Due to the above photochemical dissociation of
Many chemical reactions are occurring in O,, molecular O, is virtually non-existent at very high
atmosphere in which oxygen plays an important role. altitudes. At altitudes exceeding 40 km only less than
Some of these reactions occur by the absorption of 10% of oxygen is present as molecular oxygen. The
solar radiation and these reactions are called atomic oxygen combines in the presence of radiation
photochemical reactions. These chemical and to form excited oxygen atom according to the reaction
photochemical reactions play a significant role in
O+0+0 _fY , 0,+0*
soverning the chemical species present in the
atmosphere. It may be noted that there are large The excited oxygen atom gives out visible light
variations in the atmosphere with regards to at 636 nm, 630 nm and 558 nm wavelength and is
composition, temperature, humidity and intensity of responsible for air glow. Oxygen ions (O*) are formed
sunlight. Therefore, different types of reactions are when ultraviolet radiations react with oxygen atom
observed under varying atmospheric conditions. O+hv — > Of¢+ &
The oxygen lon is present in some regions of the
Oxygen in the troposphere plays an important role
ionosphere. It may be noted that the region beyond
in the processes occurring on the earth’s surface. For
stratosphere is dominated by +ve ions such as O*, O,*,
example, oxygen is taking part in burning of fossil fuels.
NO* and free electrons. It is called ionosphere.
CH, + 20, ——-+_ CO, + 2H,O O* ion also undergoes a number of other reactions
(Natural gas)
such as
Atmospheric oxygen is also used by aerobic
O* + O, —— 0O,*+0
organisms in the degradation of organic material or
in oxidation of weathering processes. O* + N,—— NOt+N
(From atmosphere)
Organic matter + O, (Nu CO, + H,O
O,* may also be formed according to the reaction
4Fe + 30, ———> sp.
2Fe,O
Ne 0, ——=>. NFO
The carbon dioxide and water present in the
atmosphere are used by green plants in the presence Ozone is present in the stratosphere which acts as a
of sunlight in the process of photosynthesis as : protective radiation shield for living organisms on earth.
hv
It is formed by photochemical reactions of oxygen as :
6CO, + 6H,O chlorophyll CHO, + 60, O,+hv ——> O+0
During photosynthesis, oxygen is again returned OFO04 M -——> (24M
to the atmosphere. Where M is another species like a molecule of N,
In the upper atmosphere, oxygen exists in some or O,. It absorbs the excess energy liberated by the
forms which are different from those stable at lower above reactions and hence stabilize the O, molecule.
levels. Thus, in addition to O, (molecular oxygen), the Ozone strongly absorbs ultraviolet light in “the region
upper atmosphere also have O (atomic oxygen), of 220— 330 nm according to the following reactions:
O,* (excited molecular oxygen), O* (excited atomic
oxygen), O* (ionic oxygen) and O, (ozone).
O, +hv ——> 0,+0
The atomic oxygen is produced by photochemical
O, +O ——> 0,+0,
ENVIRONMENTAL POLLUTION

Atomic oxygen may also be eliminated by the Nitric oxide is produced in the stratosphere
reaction below 30 km by the reaction of nitrous oxide (N,O)
O+0+M ——> O,+M
with excited oxygen atoms. Nitrous oxide originates
Ozone in the stratosphere is destroyed by reaction
with reactive hydroxy] radicals or nitric oxide, etc. from the microbiological processes occurring in the
Oo+ HO’ ——> 0,+HOO' atmosphere.
(Hydroxyl radical) NO, + OF J@ 2NO
HOO +0 ——~» HO'+0O, Above 30 km in the stratosphere it is produced
Similarly, by ionizing radiation on nitrogen
O. + NO: -—> NO,+0, N, + hv $m NA N
Nitric oxide O, + Mee—— >» JNO+ O
NO,+O0 ——» NO+O,
OBJECTIVE TYPE QUESTIONS ¥
A11. Acid rains are produced by :
‘ cw (a) excess release of carbon monoxide by incomplete
<or" combustion.
(6) excess formation of carbon dioxide by combustion
Air Pollution and respiration.
Al. The major air pollutant is : (c) excess production of NH, by industries.
(a) CO (b) Oxides of nitrogen (d) excess NO, and SO, from burning fossil fuels.
(c) Soot (ad) Oxides of sulphur. A12, Oxides of sulphur and nitrogen are important pollut-
A2, The region closest to earth's surface is : ants of :
(a) stratosphere (6) mesosphere (a) Water (b) Air
(c) troposphere (d) thermosphere. (c) Soil (d) All of these.
A3. The major source of CO pollution is : A138. In upper stratosphere, ozone protects us from :
(a) industrial processes (6) vehicular exhaust (a) infra-red radiations (b) ultra-violet radiations
(c) forest fires (d) voleanic activity. (c) carbon dioxide (d) peroxyacetyl nitrate.
A4, Increased concentration of CO, in atmosphere is A14, Burning of fossil fuels is the main source of :
responsible for : (a) Nitrogen dioxide (b) Nitric oxide
(a) greenhouse effect (6) acid rain (c) Nitrous oxide (d) Sulphur dioxide.
(c) lack of photosynthesis (d) death of aquatic life. A15. Which of the following gases is not a green house gas?
A5. Which of the following oxides of nitrogen is not (a) CO (b) O,
common air pollutant ? (c) CH, (d) H,O vapour
(d) N,O,. A16. Taj Mahal is threatened by pollution from :
(a) NO, = (6) N,O (c) NO
(a) Oxygen (b) Carbon dioxide
A6. White lung cancer is caused by (c) Sulphur dioxide (d) Chlorine.
(a) asbestos (b) silica A17. Which of the following is the primary precursor of
(c) textiles (d) paper photochemical smog ?
A7. Which one of the following particulates is most toxic? (a) Ozone (6b) Peroxyacetyl nitrate
(a) fly ash (b) soot (c) NO, (d) CO,.
(c) Inorganic compound (d) smog. A18. The main contributor of acid rain is :
A8. Besides CO,, other greenhouse gas is
(a) CH, (6)N, (c) Ar (d) O,. (a) H,SO, (b) CaCO, (c) CH,COOH (d) PAN.
A19. The main culprit for the depletion of protective ozone
AS. PAN stands for
layer is:
(a) Oxygen (b) CFCs
(ce) CH COONO.) (cd) CH,CH,O —N=0O. (c) Oxides of S (d) Particulate matter.
| | A20. It is not advised to sleep in a closed room with a coke
O fire burning inside on a cold day because of harmful
A10. Depletion of ozone layer causes effects of :
(a) blood cancer (6) breast cancer (a) CO
(c) lung cancer (d) skin cancer (b) Carbon particulates
(c) CO,
Aaswecs (d) SO, present as impurity in coke fire.

Al. (a) A2. (c) AS. (6) Ad. (a) AB. (d) A6. (c) AZ. (6b) A8&. (a) A9% (c) A1O. (d)
All. (d) A1l2. (6) Al3. (6) Al4. (d) Al5. (a) Al16. (c) ALT. (ec) A18. (a) A119. (b) A20. (a)
eee 14/38 MODERN'S abe + OF CHEMISTRY-XI

A21. Bhopal Gas Tragedy was caused by : A28. Which of the following has least permissible limit in
(a) Methyl amine (b) Methyl isocyanate drinking water ?
(c) Phosgene (d) Chloroform. (a) Chlorides (b) Dissolved oxygen
A22. Which of the followingis responsible for depletion of the (ec) Zine (d) Lead.
ozone layer in the upper strata of the atmosphere ? A29. COD stands for :
(a) Fullerenes (6) Freons (a) Chemical Oxygen Demand.
(c) Polyhalogens (d) Ferrocene. (b) Controlled Oxygen Demand.
A23. Most dangerous metal pollutant of automobile (c) Clouds causing Ozone Depletion.
exhausts is (d) Chlorinated Oxygen Demand.
(a) Lead (b) Copper
(c) Mercury (d) Cadmium Soil Pollution
A24, Which is most abundant hydrocarbon pollutant ? A30. Marble acts as a sink for
(a) Methane (6) Propane (a) metallic pollutants (b) NH, pollutants
(c) Acetylene (dq) Ethane. (c) acidic pollutants (d) none of these.
A31. Herbicides are used to kill weeds. The common
Water Pollution, herbicide is
A25. Which of the following species containing mercury is (a) D.D.T. (b) Triazine
oe ? ae (c) Methyl mereury (d) PCBS.
rm He Zid : He _. tal A32. Soil salinity can be measured by :
A26. Which of the following metal has maximum (a) Fae netor (b) Calorimeter
recommended level in drinking water ? (c) Conductivity meter (d) All of these.
(a) Cadmium (b) Zine A383. D.D.T. is:
(c) Aluminium (d) Iron. (a) an antibiotic
A27. The clean water should have BOD value
(b) biodegradable pollutant
(a) less than 5 ppm (b) 10 ppm
(c) non-degradable pollutant
(c) 20 ppm (d) 100 ppm

Arse:
(d) not a pollutant.

A21. (b) A222. (b) A228. (a) A224, (a) A25S. (a) AZG6. (6) A2Z,. (a) AS. (d) A229. (a) ASO. (c) AS1. (5)
A382. (c) A33. (c)

(c) The reduction in concentration of the dissolved


MULTIPLE CHOICE QUESTIONS oxygen in water due to phosphate pollution in water
from competitive examinations (d) Attack of younger leaves of a plant by peroxyacetyl
nitrate
AIPMT . fa (e) Increase in concentration of radioactive substances
sisTheehoicsd aM ie E@ards in water. (Kerala PMT 2008)
eee Eee B5. Green chemistry means such reactions which :
B1. Ozone in the stratosphere is depleted by (a) Produce colour during reactions
(a) CF,Cl, (b) CF, (b) a i use and production of hazardous
, , chemicals
(c) CHCl, (d) oi (ALLIS. 2004) (c) Are related to the depletion of ozone layer
B2. Which one of the following is responsible for depletion (d) Study the reactions in plants. (CBSE Med.2008)
ofthe ozone layerin the upper strata of the atmosphere? B6. Which one of the following statement is not true?
(a) polyhalogens (b) ferrocene (a) pH of drinking water should be between 5.5-9.5
(c) fullerenes (d) freons (CBSE P.M.T. 2004) (b) Concentration of DO below 6 ppm is good for the
B3. The environmental friendly method of killing harmful growth of fish.
insects is through the use of (c) Clean water would have a BOD value of less than
(a) Insecticides (6) Sex attractants (a) ee ee 7 ak -
(c) Sex hormones (d) Pesticides Macon acid gea rt. saugil, emacs
<rica gga ering aton Ran laden
fey Antbiolice (Kerala PMT 2008) most widespread air pollutant. (AIPMT 2011)
B4. The process of ‘eutrophication’ is due to B7. Excess nitrate in drinking water can cause
(a) methemoglobinemia
(a) Increase in concentration of insecticide in water (b) liver damage (c) kidney damage
(6) Increase in concentration of fluoride ion in water (d) laxative effect (e) innecdomn:
(Kerala PMT 2011)
Answets

Bl. (a) B2. (d) B38. (5b) Bé4. (c) BS. (5) B6é. (5) BY. (a)
ENVIRONMENTAL POLLUTION

B8. Which of the following is not a greenhouse gas? (d) the interaction of oxygen and water vapour
(a) Methane (b) Ozone (WB JEE 2011)
(c) Carbon dioxide (d) Nitrogen B17. The maximum prescribed concentration of cadmium
B9. The pH of normal rain water is in drinking water in ppm is
(a) 6.5 (b) 7.5 (a) 0.05 (b) 3
(c) 5.6 (d) 3.5 (c) 2 (d) 5
(e) 4.6 (Kerala PMT 2012) (e) 0.005 (Kerala PET 2012)
B10. Which one of the following statements regarding B18. The gas emitted by supersonic jet planes that slowly
photochemical smog is not correct ? depletes the concentration of ozone layer is
(a) Carbon monoxide does not play any role in (a) CO (b) NO
photochemical smog formation. (c) SO, (d) O,
(b) Photochemical smogis an oxidising agentin character. (e) HF (Kerala PET 2012)
(c) Photochemical smog is formed through
B19. CCl, and freons
photochemical reaction involving solar energy.
(dq) Photochemical smog does not cause irritation in (a) are green compounds because they are green
eyes and throat. (A.L.P.M.T. 2012) coloured
B11. Which one of the following is not a common component (b) deplete ozone concentration
of photochemical smog? (c) cause Increase in ozone concentration
(a) Ozone (b) Acrolein (d) have no effect on ozone concentration
(c) Peroxyacetyl nitrate (d) Chlorofluorocarbons (J.K.C.E.T. 2012)
B20. What is D.D.T. among the following ?
(AIPMT 2014)
(a) A fertilizer
B12. Which of the following is a sink for CO?
(b) Biodegradable pollutant
(a) Microorganisms present in the soil
(c) Non-biodegradable pollutant
(b) Oceans
(d) Greenhouse gas (A.LE.ELE. 2012)
(c) Plants B21. The gas leaked from a storage tank of the Union
(d) Haemoglobin (NEET 2017) Carbide plant in Bhopal gas tragedy was
B18. Which oxide of nitrogen is not a common pollutant (a) Phosgene (b) Methylisocyanate
introduced into the atmosphere both due to natural (c) Methylamine (d) Ammonia (JHE Main 2013)
and human activity? B22. Metal ion responsible for the Minamata disease is
(a) NO, (b) NO, (a) Co** (b) Hg?*
(ec) N,O (d) NO (NEET 2018) (c) Cu** (d) Zn?* (WB JEE 2014)
B23. Among the following the one which 1s not a “greenhouse
JEE (Main) & Other State Boards’
gas”, 1s
Engin
i eering Entrance
(a) N,O (b) CO,
B14. The smog is essentially caused by the presence of ic) AG, (d) O, (WB JHE 2014)
(a) Oxides of sulphur and nitrogen B24, Green chemistry deals with
(b) O, and N, (a) study of plant physiology
(c) O, and O, (b) study of extraction of natural products from plants
(d) O, and N, (A.I_E_ELE.2004) (c) detailed study of reactions involved in the synthe-
sis of chlorophyll
B15. Identify the wrong statement in the following :
(d) utilization of existing knowledge base for reducing
(a) Acid rain is mostly because of oxides of nitrogen
the chemical hazards along with developmental
and sulphur
activities
(b) Chlorofluorocarbons are responsible for ozone layer (e) synthesis of chemical compounds using green light.
depletion
(Kerala PET 2014)
(c) Greenhouse effect is responsible for global warming B25. The wrong statement among the following is :
(d) Ozone layer does not permit infrared radiation (a) Acid rain is mostly because of oxides of nitrogen
from the sun to reach the earth. (AIEEE 2008) and sulphur.
B16. The ozone layer forms naturally by (6) Greenhouse effect is responsible for global
(a) the interaction of CFC with oxygen warming.
(b) the interaction of UV radiation with oxygen (c) Ozone layer does not permit infrared radiation
from the sun to reach earth.
(c) the interaction of IR radiation with oxygen

Answers
(ad) Chlorofluorocarbons are responsible for ozone layer
depletion. (JK. CET 2015)

B8. (d) B9. (c) B10. (d) Bll. (d) B12. (a) B13. (a) B14. (a) B15. (d) B16. (6) B17. (e) B18. (5)
B19. (6) B20. (c) B21. (6) B22. (b) B23. (d) B24. (d) B25. (c)
suo MODERN'S abe + OF CHEMISTRY-XiI

B26. The concentration of fluoride, lead, nitrate and iron in B30. Which of the following is present in maximum amount
a water sample from an underground lake was found in acid rain?
to be 100 ppb, 40 ppb, 100 ppm and 0.2 ppm, (a) HNO, (b) H,SO,
respectively. This water is unsuitable for drinking due (c) HCl (d) H,CO, (WB JEE 2018)
to high concentration of : B31. The recommended concentration of fluoride ion in
(a) Fluoride (b) Lead drinking water is up to 1 ppm as fluoride ion is required
(c) Nitrate (d) Iron (JEE Main 2016) to make teeth enamel harder by converting
B27. Which one of the following 1s not a common component
of photochemical smog? [8Ca,(PO,),. Ca(OH),] to :
(a) Ozone (b) Acrolein (a) [CaF,] (b) [3(CaF,).Ca(OH),]
(c) Peroxyacetylnitrate (d) Chlorofluorocarbons (c) [3Ca,(PO,).CaF,] (d) [3{Ca(OH),}.CaF,]
(Karnataka CET 2017) (JEE Main 2018)
B28. A water sample has ppm level concentration of B32. The molecule that has minimum/no role in the
following anions formation of photochemical smog, is
F =10; SO} =100; NO; =50 (a) NO (6) CH,=O
the anion/anions that make/makes the water sample
(ce) 0, (d) N, (JEE Main 2019)
unsuitable for drinking is/are:
B33. Water samples with BOD values of 4 ppm and 18 ppm
(a) only NO; (b) both SO, and NO; respectively, are
(ec) only F (d) only SO, (JEE Main 2017)
(a) Clean and highly polluted
B29, Green fuel is the fuel obtained from
(a) bio-waste (6) metal waste (6) Clean and clean
(c) plastic waste (d) chemical waste (c) Highly polluted and clean

Asus
(e) electronic waste. (Kerala PET 2018) (d) Highly polluted and highly polluted
(JEE Main 2019)

B26. (c) B27. (d) B28. (c) B29. (a) B30. (5) B31. ic) B32. (d) B33. (a)
(c) biochemical oxygen demand 10 ppm.
(d) amount of carbon dioxide in the atmosphere 1s 0.03%
with more than one correct answer C3. The acids present in acid rain are
C1. The gases causing greehouse effect is/are (a) H,CO, (b) Peroxyacetylnitrate
(a) CH, (b) NO, (c) SO, (d) O, (c) H,So, (d) HNO,
C2. Which of the following conditions show the polluted C4. The atmosphere surrounding us may be divided into
environment? the following regions.
(a) pH of rain water in 5.6. (a) mesosphere (b) hemisphere
(b) eutrophication. (c) thermosphere (d) catosphere
i rs C5. The ozone layer is depleted by
(a) SO, (®#)CH, (c)NO- (d) CFCs

Cl. (a,d) C2. (b,c) C3. (a,c.d) C4. (a,c) C5. (c,d)
1. Assertion : Photochemical smog is produced by oxides
Assertion Reason Type Questions of nitrogen.
The questions given below consists of an Assertion and Reason =: Vehicular pollution is a major source of
the Reason. Use the following key to choose the appropriate oxides of nitrogen.
answer. 2. Assertion : For greenhouse effect, presence of green
(a) If both assertion and reason are CORRECT and plants is essential.
reason 1s the CORRECT explanation of the assertion. Reason: Chlorophyll of the green plants causes
(6) Ifboth assertion and reason are CORRECT, but reason green house effect.
is NOT THE CORRECT explanation of the assertion. 3. Assertion : Normal rain water has a pH of about 5.6.
(c) Ifassertion is CORRECT, but reason is INCORRECT. Reason =: The pH is due to dissolution of CO, to form
(d) Ifassertion is INCORRECT, but reason is CORRECT. carbonic acid.

(e) If both asseriton and reason are INCORRECT. 4, Assertion : In India, an oil spill occurred in Bombay
in 1993.
Reason =: Spreading of oil into sea is called oil spill.
rastgees
1. (6) 2.(e) 3. (a) 4. (d)

WWW.JEEBOOKS.IN
ENVIRONMENTAL POLLUTION

5. Assertion : Higher concentration of CO, in 9, Assertion : Excessive use of chlorinated synthetic


atmosphere is causing global warming. pesticides causes soil and water pollution.
Reason — : This is due to greenhouse effect. Reason : These pesticides are non-biodegradable.
6. Assertion : The presence of CO reduces the amount (NCERT Exemplar Problem)
of haemoglobin available in the blood for 10. Assertion : The pH of acid rain is less than 5.6.
carrying oxygen to the body cells.
Reason : Carbon dioxide present in the atmosphere
Reason = : COcombines with haemoglobin about 200
dissolves in rain water and forms carbonic
times less easily than oxygen to form
acid. (NCERT Exemplar Problem)
complex.
7. Assertion : Photochemical smog is oxidising in nature. 11. Assertion : If BOD level of water in a reservoir is less
than 5 ppm it 1s highly polluted.
Reason =: Photochemical smog contains NO, and O,,
which are formed during the sequences of Reason : High BOD means low activity of bacteria
reactions. in water. (NCERT Exemplar Problem)
8. Assertion : Ozone is destroyed by solar radiation in 12. Assertion : Green house effect was observed in houses
upper stratosphere. made of green glass which are used to grow
Reason : Depletion of the ozone layer allows plants.
excessive UV radiations to reach the Reason 7
* Green house name has been given because
surtace of earth. glass houses are made of green
(NCERT Exemplar Problem) glass. (NCERT Exemplar Problem)

nswecs
5.(a) 6. (ce) 7. (a) 8.(d) 9.2) 10.(5) 11. (e) 12. (e)

NCERT Exemplar Problems //


Objective Questions
»>»> Multiple Choice Questions (Typ AD)
(b) Ozone can oxidise sulphur dioxide present in the
1. Which of the following gases is not a green house gas? atmosphere to sulphur trioxide.
(a) CO (b) O, (c) Ozone hole is thinning of ozone layer present in
(c) CH, (d) H,O vapour stratosphere.
. Photochemical smog occurs in warm, dry and sunny (ad) Ozone is produced in upper stratosphere by the
climate. One of the following is not amongst the action of UV rays on oxygen.
components of photochemical smog, identify it. . Sewage containing organic waste should be
(a) NO, (b) O, disposed in water bodies because it causes major
(c) SO, (dq) Unsaturated hydrocarbon water pollution. Fishes in such a polluted water die
3. Which of the following statement is not true about because of
classical smog? (a) Large number of mosquitoes.
(a) Its main components are produced by the action (b) Increase in the amount of dissolved oxygen.
of sunlight on emissions of automobiles and (c) Decrease in the amount of dissolved oxygen in water.
factories. (d) Clogging of gills by mud.
(6) Produced in cold and humid climate. 7 . Which of the following statements about photochemical
(c) It contains compounds of reducing nature. smog is wrong?
(d) It contains smoke, fog and sulphur dioxide. (a) It has high concentration of oxidising agents.
4, Biochemical Oxygen Demand, (BOD) is a measure of (b) It has low concentration of oxidising agents.
organic material present in water. BOD value less
(c) It can be controlled by controlling the release of
than 5 ppm indicates a water sample tobe____.
NO,, hydrocarbons, ozone etc.
(a) rich in dissolved oxygen
(d) Plantation of some plants like pinus helps in
(b) poor in dissolved oxygen. controlling photochemical smog.
(c) highly polluted. 8 . The gaseous envelope around the earth is known as
(d) not suitable for aquatic life atmosphere. The lowest layer of this is extended upto
5. Which of the following statements is wrong? 10 km from sea level, this layer is :
(a) Ozone is not responsible for green house effect. (a) Stratosphere (6) Troposphere
(c) Mesosphere (dq) Hydrosphere
Answers
1. (a) 2. (c) 3. (a) 4.(a) 5. (a) G.(c) 7.(6) 8. (0)
Sfsuez MODERN'S abe + OF CHEMISTRY-AlI

9. Dinitrogen and dioxygen are main constituents of air 11. Which of the following statements 1s correct?
but these do not react with each other to form oxides (a) Ozone hole is a hole formed in stratosphere from
of nitrogen because which ozone oozes out.
(a) the reaction is endothermic and requires very (b) Ozone hole is a hole formed in the troposphere
high temperature. from which ozone oozes out.
(b) the reaction can be initiated only in presence of (c) Ozone hole is thinning of ozone layer of
a catalyst. stratosphere at some places.
(c) oxides of nitrogen are unstable. (d) Ozone hole means vanishing of ozone layer
around the earth completely.
(d) N, and GO. are unreactive.
10. The pollutants which come directly in the air from 12. Which of the following practices will not come under
ereen chemistry?
sources are called primary pollutants. Primary
pollutants are sometimes converted into secondary (a) Ifpossible, making use of soap made of vegetable
pollutants. Which of the following belongs to oils instead of using synthetic detergents.
secondary air pollutants? (6) Using H,O, for bleaching purpose instead of
using chlorine based bleaching agents.
(a) CO (b) Hydrocarbon
(c) Using bicycle for travelling small distances
(c) NO (d) Peroxyacetyl nitrate instead of using petrol/diesel based vehicles.

Answers (d) Using plastic cans for neatly storing substances.

9. (a) 10. (d) 11. (c) 12. (d)

»> Multiple Choice Questions (Type-Ill) Matching Type Questions


13. Which of the following conditions shows the polluted In the following questions more than one option
environment. of Column I and Column II may match.
(a) pH of rain water is 5.6. 17. Match the terms given in Column I with the
(6) amount of carbon dioxide in the atmosphere is compounds given in Column II.
0.03%.
(c) biochemical oxygen demand 10 ppm. Column I Column IT
(d) eutrophication. (i) Acid rain (a) CHCl, —- CHF,
14, Phosphate containing fertilisers cause water pollution. (ti) Photochemical (b) CO
Addition of such compounds in water bodies causes smog
(ii) Combination (c) CO,
(a) enhanced growth of algae. with haemoglobin
(b) decrease in amount of dissolved oxygen in water. (iv) Depletion of (d) SO,
(c) deposition of calcium phosphate. ozone layer (e) Unsaturated hydrocarbons
(d) increase in fish population. 18. Match the pollutant(s) in Column I with the effect(s)
15. The acids present in acid rain are in Column II.
(a) Peroxyacetyl nitrate (6b) H,CO, Column I Column IT
(c) HNO, (d) H,SO, (i) Oxides of sulphur (a) Global warming
16. The consequences of global warming may be ____ (ii) Nitrogen dioxide (b) Damage to kidney
(tii) Carbon dioxide (c) ‘Blue baby syndrome
(a) increase in average temperature of the earth.
(iv) Nitrate in drinking (d) Respiratory diseases
(b) melting of Himalayan Glaciers.
water
(c) increased biochemical oxygen demand. (v) Lead (e) Red haze in traffe and
(d) eutrophication. congested areas

Answers
13. (c,d) 14.(4,6) 15. (6,¢d) 16. (a, d) 17.(z) — (c, d); (it) — (e, d); (tit) — (6); (iv) — (@)
18. (1) —(d); (it) — (e); (tit) — (a); (tv) — (e), (v) — (Bb)
ENVIRONMENTAL POLLUTION

19. Match the activity given in Column I with the type of .


pollution created by it given in Column II.
Column I (Activity) Column II (Effect) In the following questions a statement of
; Assertion (A) followed by a statement of
(1) Releasing gases to |(a) Water pollution Reason (R) is given. Choose the correct option
the atmosphere out of the choices given below each question.
after burning waste (a) Both A and R are correct and R is the correct
material containing explanation of A.
sulphur. (b) Both A and R are correct but R is not the correct
explanation of A.
(ii) Using carbamates (6) Photochemical smog, (c) Both
A and K are not correct
as pesticides damage 3 plant life, (2d) Ais not correct but R is correct.
corrosion to building 21. Assertion (A) : Green house effect was observed in
material, induce breathing houses used to grow plants and these are made of green
problems, water pollution glass.
(iii) Using synthetic (c) Damaging ozone layer Reason (R) : Green house name has been given
detergents for because glass houses are made of green glass.
washing clothes 22. Assertion (A) : The pH of acid rain is less than 5.6.
Reason (R) : Carbon dioxide present in the
(tv) Releasing gases (d) May ees MERGE Mes SEs atmosphere dissolves in rain water and forms carbonic
produced by auto- in human. ‘a:
mobiles and factories 23. Assertion (A) : Photochemical smog is oxidising in
in the atmosphere. nature.
(v) Using chlorofluoro- | (e) Classical smog, acid rain, Reason (R) : Photochemical smog contains NO,
carbon compounds water pollution, induce and O,, which are formed during the sequence of
for cleaning breathing problems, ion agtions. ee :
saiiputee ats aernnes tabiainee: 24. Assertion (A) : Carbon dioxide is one of the important
greenhouse gases.
corrosion of metals. Reason (R) : It is largely produced by respiratory
20. Match the pollutants given in Column I with their function of animals and plants.
effects given in Column II. 25. Assertion (A) : Ozone is destroyed by solar radiation
in upper stratosphere.
Column I Column II Reason (R) : Thinning of the ozone layer allows
(i) Phosphate fertilisers (a) BOD level of water excessive UV radiations to reach the surface of
in water increases earth.
(ii) Methane in air (b) Acid rain 26. Assertion (A) : Excessive use of chlorinated synthetic
(iii) Synthetic detergents (ec) Global warming pesticides causes soil and water pollution.
in water Reason (R) : Such pesticides are non-biodegradable.
(iv) Nitrogen oxides in air (dq) Eutrophication 27. Assertion (A): If BOD level of water in a reservoir 1s
less than 5 ppm it is highly polluted.
Reason (R) : High biological oxygen demand means
Answers low activity of bacteria in water.

19. (z) —(e); (i)


— (d); (ut)
— (a); (tv)
=(6); (v) -—(e) 20. (@) —(a, d); (ut)
—(e); (Ut)
—(a); (tv)
—(6) 21. (ce) 22, (6)
23. (a) 24.(b) 25. (d) 26. (a) 27. (c)

ations for
Hints & Explan Difficult Objective Type Questions
Bl. (a) : Ozone is depleted by chlorofluorocarbons R11, (d) : Chlorofluorocarbonsare not a common component
| (CF,CL,). of photochemical smong.
B2. (d) : Chlorofluorocarbons known as freons deplete
B12. (a) : Microorganisms present in the soil is the sink
ozone layer. of CO
B6.(b) =: Fish dies in water bodies polluted by sewage or
other pollutants due to decrease in dissolved B18. (a) : N,O, is not a common pollutant.
oxygen (D.O.) value below 6 ppm. B16. (b) : Ozone layer is formed in the atmosphere by
B7.(a) : Excess nitrate in drinking water can the decomposition of oxgen by ultra-violet
cause methemoglobinemia (blue baby radiation.
syndrome). B20. (ec) : D.D.T. is non-biodegradable pollutant.
B8. (d) : Nitrogen is not a greenhouse gas.
ns 14/44 MODERN'S abe + OF CHEMISTRY-XI

ion Fle
B21. (b) : The gas leaked in Bhopal gas tragedy was
methylisocyanate (CH,N = C = 0). NCERT Exemplar Problems : MCQs Type-I
B23. (d)
(d) : O,is
aleenot wammmmanain
> enh wis 3.: (a): Classical smog is
;
not produced by the action of
B26. (c) : The permissible limit of nitrate in water is sunlight on emissions of automobiles and
about 50 ppm. Hence the concentration of faerie.
NO, ions in a given water sample exceeds from 5. (a): Ozone is also one of the greenhouse gas.
the permissible limit. The higher concentration 6. (c) : Organic waste consumes oxygen and therefore,
of NO; ions in water may cause blue baby dissolved oxygen in water decreases and fish in
syndrome disease. such polluted water die.
B28. a) 2 The F° jek Goneeutention. da Abaee: thie 7. (6): Photochemical smog is oxidising in nature
permissible limit in drinking water. Therefore, because it contains large concentration of
it 1s unsuitable for drinking. Excess oxidising agents such as O, and NO,.
concentration of F ions (greater than 2 ppm) 10. (d): Peroxyacetyl nitrate is a secondary pollutant
which is formed from primary pollutants such
causes brown mottling of teeth enamel.
as NO,, O, and hydrocarbons.
B31. (ec) : F ions make the teeth enamel harder by
Convers NCERT Exemplar Problems : MCQs Type-II
3Ca,(PO,),.Ca(OH), —> 3Ca,(PO,),.CaF,
B32. (d) : NO, O, and HCHO are involved in the 15. (b,c, d): The acids present in acid rain are H,CO,,
formation of photochemical smog. N, has no role HNO, and H,SO,.
| in photochemical smog. 16. (a,b): The global warming may result into
B33. (a) : Clean water would have BOD value < 5ppm increase in average temperature of the
while BOD value of > 17 indicates highly earth and melting of Himalayan glaciers.
polluted water.

12 )

Unit Practice Test | Orr EXAMULatlon


Time Allowed: 1 Hr. Maximum Marks: 25

1. Fish do not grow as well in warm water as 1n cold water. Why? (1)
2. What do the abbreviations 'BOD' and 'COD' stand for ? (1)
3. What are herbicides? Give one example. (1)
4, What is the major cause of ozone layer destruction? (1)
5. Name the gas which caused Bhopal gas tragedy? (1)
6. What is pneumoconiosis? How does it occur? (2)
7. Why does rain water normally have a pH of about 5.6 ? When does it become acidic rain? (2)
8. How can domestic waste be used as manure? (2)
9. Statues and monuments 1n India are affected by acid rain. How? (2)
10. List main differences between classical smog and photochemical smog. (3)
11. Give a brief account of green chemistry. (3)
12. Explain
(1) Green house effect
(it) Tropospheric pollution. (3)
13. What are the reactions involved in ozone layer depletion in stratosphere ? What are its
consequences ? (3)

To check your performance, see HINTS and SOLUTIONS to some questions at the end
of Part II of the book.
According to Completely CBSE Pattern MOCK TEST — 2

Time allowed : 3 hours Maximum Marks : 70 dig sit Included:


General Instructions : Redox Reactions
(1) All questions are compulsory. Hydrogen
(it) Question nos. 1 to 5 are very short answer questions and carry I mark each. paws Sael one and
(tit) Question nos. 6 to 12 are short answer questions and carry 2 marks each. |. Some p-Block Elements
(iv) Question 13 to 24 are also short answer questions and carry 3 marks each. . Organic Chemistry : Some
(v) Question nos. 25 to 27 are long answer questions and carry 5 marks each. Basic Principles and |echniques
vl) UseUse log
. Hydrocarbons
(vi) log tables
tables iif necessary.
ssary. Use lculators
Use of calculators in not
in not allowe allowed. _ Enuinfacniel Pollution :

1. Name the compound formed by the oxidation of ethyl benzene by KMnO,,. (1)
2. Which of the following is acidic?
(i) But-l-yne (ii) But-2-yne (tii) But-l-ene (iv) But-2-ene (1)
3. Compare the behaviour of CCl, and BCl, with water. (1)
4, Which out of lithium or sodium forms nitrides? (1)
5. Name a compound which retards the decomposition of H,,Q,. (1)
6. Give two examples each of ionic hydrides and covalent hydrides. (2)
7. Why is Li,CO, decomposed at a lower temperature while Na,CO, is decomposed at higher temperature? (2)
8. Explain the higher stability of BCL, as compared to TIC1,. (2)
9. Give condensed and bond line structural formulae for the following :
(i) 2-(4-isobutylphenyl) propanoic acid (ii) Hexanedial (2)
10. Write the IUPAC names of the following compounds :

| CHO
(i) (CH,),C(C,Hx), (iz) CS (2)

HO |
OCH,
11. What is acid rain? What are its harmful effects? (2)
12. What is salt bridge? Give its functions. (2)
13. 0.35 g of an organic substance was Kjeldahlised and ammonia obtained was passed into 100 mL of
N ; ' N —e
To H,SO,. The excess acid required 154 mL of Io NaOH for neutralisation. Calculate the percentage of

nitrogen in the compound. (3)


14, Give one method of preparation of each of the following :
(i) Ethane
(ii) Propyne
(iii) Ethene (3)
15. Write short notes on the following :
(a) Wurtz reaction
(b) Markovnikov’s rule
(c) Friedel Craft reaction (3)
16. What do you mean by green chemistry? How will it help decrease environmental pollution? (3)
17. Give the chemistry of Lassaigne’s test for nitrogen and halogens. (3)
MODERN'S abc + OF CHEMISTRY-XI

18. Complete the following reactions :


D+ +
(i) CH,-C=C—Ph —£=* , (ii) pr —
CH Bry, H,O

(iii) CH CH, C—CHCHs —tnH,o


‘ae
O : 3
(3)
CH,
19. Write the structures and IUPAC names of different structural isomers of alkenes corresponding to the
formula C,H,,. (3)
20. Explain the following reactions :
(i) CO1is heated with ZnO.
(ii) Hydrated alumina is heated with aqueous NaOH.
(iii) Silicon is heated with methyl chloride at high temperature in the presence of copper. (3)
21. Explain the following :
(a) Boric acid is polymeric
(b) Lead (IV) chloride is highly unstable towards heat.
(c) Graphite is used as a lubricant. (3)
22. (a) Why do alkenes under go electrophilic addition reactions? Give one example.
(6) An organic compound ‘X’ on ozonolysis gives a mixture of ethanal and pentan-3-one. Write structure
and IUPAC name of ‘X’. (3)
23. (a) What is meant by disproportionation reaction? Give one example.
(b) Balance the following redox reactions :
(i) Br, +H,O,—> BrO,- + H,O (acidic medium)
(ii) P(s) + OH (aq) —> PH, (g) + H,PO,- (aq) (3)
24, (a) Write down the chemical reactions involved during the formation of photochemical smog.
(6) Which out of CO or CO, is more dangerous pollutant? (3)
295. (a) What is diagonal relationship? How does lithium resemble magnesium?
(6) Explain the structure of diborane.
(c) Give three important uses of aluminium.
(d) N(CH,), is pyramidal but N(SiH,), is planar. Explain.
(e) Like CO why its analogue $10 is not stable. (5)
Or
(a) What are fullerenes? Discuss their structures.
(b) What are silicones? How are they prepared? Give their important uses.
26. (a) Describe Carius method for the estimation of halogens.
(b) What is the difference between distillation and fractional distillation.
(c) What is the structure of a carbanion? Explain the stability of CH,, 1°, 2° and 3° carbanions. (5)
Or
Give a brief account of the following;
(i) hyperconjugation (ii) inductive effect (iii) electromeric effect
27. (a) Account for the acidity of alkynes.
(6) Explain the mechanism of sulphonation of benzene.
(c) How will you convert the following :
(i) Ethyne to but-2-yne (ii) Propene to 2, 3-dimethylbutane (iii) Ethene to ethyne. (5)
Or
(a) Explain the orbital picture of benzene.
(b) Give three methods of preparation of benzene.

WWW.JEEBOOKS.IN
According to Completely CBSE Pattern MOCK TEST — 3
Time allowed : 3 hours Maximum Marks : 70
General Instructions :
(t) All questions are compulsory.
(ii) Question nos. 1 to 5 are very short answer questions and carry I mark each.
(iit) Question nos. 6 to 12 are short answer questions and carry 2 marks each.
COMPLETE SYLLABUS
(iv) Question 13 to 24 are also short answer questions and carry 3 marks each,
(v) Question nos. 25 to 27 are long answer questions and carry 5 marks each.
(vi) Use log tables if necessary. Use of calculators in not allowed.

1. An aqueous solution of sodium carbonate gives alkaline tests. Why? (1)


2. What is the oxidation state of S in Na,S,O,? (1)
3. What will be the pH of a 0.1 M ammonium acetate if pK, = pK, = 4.74? (1)
4, Write the conjugate acids of C,H,OH and NH,NH,. (1)
5. The equilibrium constant of a reaction at 27°C and 127° C are 1.52 x 10+ and 1.26 x 10~ respectively. Is
the reaction exothermic or endothermic? (1)
6. Give molecular electronic configuration of peroxide ion and oxygen molecule. Which of the two has
larger bond length? (2)
7. How would you explain the fact that the first ionization enthalpy of sodium is lower than that of
magnesium but its second ionization enthalpy is higher than that of magnesium? (2)
8. Calculate the wavelength of an electron having mass = 9.1 x 10-*! kg and kinetic energy = 3.0 x 107° J. (2)
9. If 20.0 g of CaCO, is treated with 20.0 g of HCl, how many grams of CO, can be produced according to
the reaction :
CaCO,(s) + 2HCI (aq) —> CaCl, (aq) + H,OW) + CO,(g) (2)
10. For the water gas reaction :
C(s) + H,O (g) > CO(g) + H,(g)
the standard Gibbs energy for the reaction at 1000K is —-8.1 kJ mol. Calculate its equilibrium constant. (2)
11. What is the relation between kinetic energy and temperature? (2)
12. Describe the shapes of BF, and BH, and assign the hybridisation of boron in these species. (2)
13. Draw structures of cyclic and acylic isomers of molecular formula C,H,O. (3)
14. (a) Which of the two: O,N CH, CH,O- or CH, CH,O- is expected to be more stable and why?
(6) Why alkyl groups act as electron donors when attached to a m-system.
(c) Which of the following carbocation is more stable :
(i) (CH,),CCH,* (iz) CH,CHCH,CH, (ii) CH,CH,CH,* (3)
15. How will you convert :
(1) Ethane to butane (ii) Ethane to ethyne (iii) Ethyne to methane (3)
16. (a) Write down the products of ozonolysis of 1, 2-dimethylbenzene (o-xylene). How do the results support
the Kekule structure of benzene?
(b) Why is benzene extra ordinarily stable though it contains three double bonds? (3)
17. Give the reasons :
(i) Aluminium wires are used to make transmission cables.
(ii) Diamond is used as an abrasive.
(iii) Aluminium utensils should not be kept in water overnight. (3)
18. Complete the following reactions :
(i) Ca,N,(s) + H,O()—> (ii) MnO, +H*+H,0O,—> (iii) 1,+H,0O,+OH—> (3)
19. (a) The Mn** ion is unstable in solution and undergoes disproportionation to give Mn**, MnO,, and H* ion.
Write a balanced ionic equation for the reaction.
(b) Predict the product of electrolysis of an aqueous solution of AgNO, with Ag electrodes. (3)
MODERN'S abc + OF CHEMISTRY-XI

20. (a) The wavelength of first spectral line in the Balmer series is 6561 A. Calculate the wavelength of the
second spectral line in Balmer series.
(6) How many electrons in a given atom can have the following quantum number values :
g=4. (=f. m= 21 (3)
21. Commercially available sulphuric acid contains 93% acid by mass and has a density of 1.84 g mL*t.
Calculate (i) the molarity of the solution (ii) volume of concentrated acid required to prepare 2.5 L of
0.50 M H,S0,,. (3)
22. (a) Define the following
({) enthalpy of combustion (ii) enthalpy of formation
(b) The enthalpy change for the reaction :
2NH,(g) —> N,(g) + 3H,(g)
is 92.2 kJ. What is the enthalpy of formation of ammonia? (3)
23. (a) Name two phenomena which can be explained as the basis of surface tension.
(b) What is compressibility factor? How does it help to account for the nature of the gas? (3)
24, (a) Calculate the strength in volumes of a solution containing 30.36 g/L of H,O,.
(b) Complete the following reactions:
(i) CaOCl, + H,O, ——> (ii) Fe,(SO,), + H,O, + NaOH——> (3)
29. (a) On the basis of VSEPR theory, explain the shapes of following :
H,O and NH, molecules.
(b) Draw resonance structures for SO, and CO, molecules.
(c) Which out of NH, and NF, has higher dipole moment and why ? (5)
Or
(a) Explain the structures of the following molecules on the basis of hybridisation.
(i) PF, (ii) BrF, (iii) XeF,
(b) Ice floats over water. Explain.
(c) Out of MgO and LiCl, which has higher lattice enthalpy?
26. (a) What is solubility product? How is it different from ionic product.
(6) Calculate the molar solubility of Ni(OH), in 0.10 M NaOH.
K,,, of Ni(OH), = 2.0 x 10-*.
(c) What are acidic buffers? Give one example.
(d) Calculate the pH of 10° M HCI solution. (5)
Or
(a) How will you show that in a reversible reaction, equilibrium can be achieved from either direction?
(b) What is Le Chatelier's principle. With the help of this explain : ice melts when pressure is applied on it.
(c) Calculate the pH of a solution obtained by mixing 10 mL of 0.1 M HCl and 40 mL of 0.2 M H,S0,.
27. (a) Assign structures for the following :
(i) An alkyne (X) has molecular formula C.H,,. It reacts neither with sodamide nor with ammoniacal
cuprous chloride.
(ii) A hydrocarbon “Y decolourises bromine water. On ozonolysis it gives 3-methyl butanal and
formaldehyde. Give the name of the compound.
(iii) A hydrocarbon (Z) has molecular formula C,H,,. It does not decolourise bromine water and is
oxidised to benzoic acid on heating with K,Cr,O.. It can also have three other isomers A, B and
C. Write the structures of Z, A, B and C.
(b) What effect does branching of an alkane chain has on its boiling point?
(c) Draw cis and trans isomers of hex-2-ene. Which isomer will have higher boiling point? (5)
Or
(a) Give the basic difference between Duma's method and Kjeldahl's method for the estimation of nitrogen.
(6) How will you convert:
(1) Acetylene to but-1-yne (ii) Propene to propan-1-ol (iii) ethanoic acid to ethane
(c) Suggest a suitable technique for the separation of naphthalene from kerosene present in a mixture.
HINTS AND SOLUTIONS

for Unit Practice Tests


i n t s & S olutions
H & Mock Tests
UNIT PRACTICE TEST-8
1. +6 because there is one peroxide bond (-O—O-) ..e., 0,7"
oxidised while O, has been reduced. Hence, it is a
2. Oxidising agent : Cu, reducing agent : I. redox reaction.
3. Na,O, (-1), OF (+2) 8. In AgF,, oxidation state of Ag is +2 which is very
4. The reaction in which an element in one oxidation state unstable. Therefore, it readily accepts an electron to
is simultaneously oxidised and reduced. form a more stable oxidation state of +1 as
5. No, because the oxidation number of Cr has neither Ag? oF, 5 Agt

increased nor decreased but remains +6. Therefore, AgF, if formed will act as a strong oxidising
6. Since reduction potential of silver is more than that of agent.
hydrogen (E° +H? Pt = 0), silver vessel will be 9. The O.S. of S in 5,0,7 is +2, in S,0,?" it 1s +2.5
Z
suitable to store 1M HCl. On ther other hand, gnc (average) and in SO,” it is +6. Since Br, is a stronger
oxidising agent than I,, it oxidises S of S,O,7" to SO,7—
is less than that of hydrogen ane , Pt) so that (with higher O.S. of +6). However, I, being a weaker
2
oxidising agent, it oxidises S of S,O,” into a lower
hydrogen will be liberated if stored in aluminium O.5. of +2.5 in 5,0,?-. Therefore, 5,0,7" reacts
vessel.
. +3 2 42-2 0 +4 -2 differently with Br, and I,.
7. 1) Fe, Oz +3CO ——>2Fe(s)+ 3CO, 10. The cell may be depicted similar to Fig. 3. It may be
O.N. of Fe decreases from +3 (in Fe,O,) to 0 (in Fe) represented as :
and that of C increases from +2 (in CO) to +4 (in Zn(s) |Zn**(aq) ||Ag*(aq) |Ag
CO,). Therefore, Fe,O, is reduced while CO is (i) Zine electrode (anode)
oxidised. Thus, this is a redox reaction. (ii) At anode : Zn(s) —> Zn?*(aq) + 2e~
- —3 +1 0 . +2 -2— +1 2
(it) 4NH,(g)+50,(g) ——>4N O(g)+6H, O(g) At cathode : Agt (ag) + e7 —> Ag(s)
11. Refer Solved Example 30
In this case O.N. of N increases from —3 (in NH,) to
12. Refer Solved Example 20 (ii) and (iv).
+2 (in NO) and that of O decreases from 0
(in O,) to -2 (in H,O). Therefore, NH, has been 13. Refer Solved NCERT Exercise, Q.4 and 5.

PRACTICE TEST-9
1. Perhydrol
2. Conc. H,SO, on absorbing water from moist H., produces 4 x 20 = 75.9 5|
so much heat that H, catches fire. Strength of 25 volume H,O, = 75.9 g/L
3. Deuteroacetylene is formed :
CaC, + D,O—-> Ca(OD), + C,D, Amount of H,O, present in 10 mL = a x 10
4. 1:2:3
5. Heavy water is injurious to plants, animals and human = 0.759 g.
beings because it slows down the rates of reactions 9. Refer NCERT File Q. 7.
occurring in them. So, it cannot be used. 10. Refer NCERT File Q. 8.
6. Refer NCERT File Q. 24. 11. (a) Zn(s) + 2NaQOH(aq) ——> Na, ZnO, (aq) + H,(g)
8. 2H,0, —> 2H,O + D.
Sod. zincate
68 2 22.4L at N.TP.
(6b) 4LiH + AIC1L, ——-> LiAlH, + 3 HCl
22.41 of O, at N.T-P. is produced from H,O, = 68 g
(c) Ca,N, + 6H,0 ——-> 3Ca(OH), + 2NH,
25 L of O, at N.T-P. is produced from H,O,

UNIT PRACTICE TEST—10


1. Francium and radium 6. Potassium carbonate cannot be prepared by Solvay
2. Both Li* and H- have small size and their combination process because potassium bicarbonate being more
has high lattice energy. Therefore, LiH is stable as soluble than sodium bicarbonate does not get
compared to NaH. precipitated when CO, is passed through a
3. Lithium concentrated solution of KC] saturated with NH,.
4. C,H, 7. Beryllium and magnesium atoms in comparison to
5. +1
other alkaline earth metals are comparatively smaller
and their ionisation enthalpies are very high. Hence,

WWW.JEEBOOKS.IN
MODERN'S abe + OF CHEMISTRY-A4I

_ the energy of the flame is not sufficient to excite their stabilize only a small anion O*-. Sodium is large
electrons to higher energy levels. These elements, and it can stabilize large anion O,?- whereas
therefore, do not give any colour in Bunsen flame. potassium and rubidium being very large in size
8. Refer NCERT File Q. 19. can easily stabilize larger superoxide O,- ion.
9. Refer NCERT File Q. 27. (c) and (b) (c) Refer NCERT File Q.29 (ii).
10. Refer NCERT File Q. 15. 13. (a) Rb*t > K* > Na* > Li* because of lesser hydration of
12. (a) Refer Solved Example 7 ion as size increases. In other words, hydrated radii
of ions decrease as Li* , Nat, K*, Rb‘.
(b) Lithium ion is very small ion and therefore, it has
strong positive field around it. Therefore, it can (b) Refer NCERT File Q.12.

1. Sil, (ii) Refer Conceptual Questions|2]Q. 2.


2. Due to inert pair effect, lead shows an oxidation state (iit) Refer Solved Example 25.
of +2 and therefore, PbCl, is more stable than PbCl,. 10. Refer NCERT File Q. 28.
x ae 11. (i) BCL, + 3H,0 —> H,BO, + 6HCI
4. :0==C=0: ee :O=c—S: — :O—C=0: (ii) SiF, + 2HF —> H,SiF,
5. Boric acid is not a protic acid because it does not 1onise (iii) ZNO + CO—-> Zn + CO,
in water to give a proton. But it acts as a Lewis acid by
accepting electrons from hydroxide ion. (iv) Al,O,. 2H,O(W) + 2NaOH(aq) + H,O() —>
B(OH), + H,O —-> [B(OH),]- + H,0* 2Nal[Al(OH),|(aq)
. Refer NCERT File Q. 14.
. Refer NCERT File Q. 3. (v) Pb + 2NaOH + 4H,0 —> Na, [Pb(OH),] + 2H,
. Refer NCERT File Q. 24. (vi) 2NaH + B,H,—> 2Na [BH Al
G.
=~]
© (i) Refer Conceptual Questions[2]Q. 1.

ST-11A
1. N,O, has the structure But PBr, splits up into more stable tetrahedral
structure.

PR =—@§ [Pr |* el

ra “Nes Since Br atoms are large, these six Br atoms cannot


The covalence of N is four because it has four shared be accommodated around P atom to form octahedral
pair of electrons. structure.
+3 +5 -—d (iz) In NF., the lone pair on N atom is on the opposite
2. 4H,PO, ——— 2H,PO, + BES direction to N—F bond moments and therefore, it does
3. 3CuO + 2NH, —_beat_, 3Cu +N, + 3H,0 not have donor properties like NH.
4. Phosphine is formed :
()t
orUF
P,+3NaQ0H + 3H,@- “iltePH. + 3NaH,PO,
5. Due to small size of Cl atom, five Cl atoms can be
accommodated around P atom. But I is of large size F
and therefore, five I atoms cannot be accommodated (zit) HJ,PO, contains two P—OH bonds and therefore,
around P atom. As a result, P—-I bonds are weak and its basicity is two.
prefer to form PI, rather than PI...
7. Refer Text Fig. 16 and Fig. 17 page 114/380.
8. (1) Both PCl, and PBr, have trigonal bipyramidal
|
P
geometry. This is not regular structure and is not /\~oH
stable. Therefore, PCl, splits into more stable H OH
+ 3H,O ——_» XeO, + 6BHE
octahedral and tetrahedral structures. (z) XeF,

(ii) P,O,, + 6H,0 —> 4H,PO,


2PCl, == [PCI] [PCI J-
(111) 2Ca,(PO,), + 6510 + 10C —~+
Tetrahedral Octohedral
P, + 6CaSi0, + 10CO

WWW.JEEBOOKS.IN
HINTS AND SOLUTIONS
|
T_T
NIT PRACTICE TEST—12

ALA
sp? sp*—s sp?_—s sp
% of chlorine = > «100 = 48.48 %
8. Refer Solved Example 30,
10. Refer NCERT File Q. 20.
. CH,—CH=CH—CN 11. (i) Pent-3-en-1-yne
; (CHIC (ii) 4-(1,1-dimethylpropyl)-3-ethyl-4, 7-dimethyldecane
. Fe,[Fe(CN),|., (iii) 5, 6-Dimethyleyclohex-2-en-1-one
bo
OG. O,NCH,CH,O-is
mom more stable because —NO, group has
—I inductive effect and tends to disperse the —ve charge 12. (i) OHCCH,CH,CH,CHO
on the O atom. This results into stability. However,
O
I
CH,CH, group has +I inductive effect and tends to CH,CH—-C—OH
intensify the —ve charge and therefore, destabilizes it.
Refer NCERT File Q. 11.
AgCl = Cl
143.5 30.0 CH,—CH(CH,),
143.5 g of AgC] contain Cl] = 35.5
(iii) CH,CH=CHCOOH
g¢ of
0.628 g¢
.
of AgCl con tain Cl =
35.5
"TCT aad
.62 13. (a) Refer NCERT File Q. 29.
= 0.155 g (b) Refer Conceptual Questions 2 Q. 9.
(c) Refer NCERT File Q.10.

CH,CH,COOH + 6HI —> CH,CH,CH, + 2 H,O +31,


Propene ;
(ii) CH,C=CCH,
;
—Ys_,
7 O
CH,—C’——C—CH,
2-Methylpentane
CH,=—CH—CH=CH,, : Penta-2, 4-diene OO
Yes, 1t will show geometrical isomerism.
Cyclopentadienyl cation.
2 BO, op —C—C—CH
Refer Solved Example 30. O O
Refer Solved Example 46 (ii). Butane-2, 3-dione
Refer NCERT File Q. 25.
oe
eeRefer NCERT File Q. 16.
SS (iii) CH,CH,C=CH “5s _,cH,CH,COOH
f CH, Propanoic acid
+ CO, +H,O
IOs 12. (a) Refer Solved Example 28
(b) Refer NCERT File Q.10.
Toluen

. The amount of dissolved oxygen 1s less in warm water Methyl isocyanate (CH,N=C=0).
than in cold water. Refer Conceptual Questions, Q.6.
Biological oxygen demand, chemical oxygen demand. Refer Conceptual Questions, Q.5.
The chemical substances used to kill weeds or Refer NCERT File Q. 19
undesirable vegetation. e.g., sodium arsenate. Sea
oP Refer NCERT File Q. 5
Chlorofluorocarbon compounds. 10. Refer Conceptual Questions, Q. 10.
OOO
MODERN'S abe + OF CHEMISTRY-AI

MOCK TEST —2
benzoic acid CH3
No (i) (it) Sc=cu, —By HO , CH,—C_CH Br
CCL, does not get hydrolysed while BCI, readily gets CH,
YY | -
hydrolysed to boric acid. OH
BCl, + 3H,0 —> H,BO, + 6HCl ‘5
Lithium (iii) CHgCH, C=CHCH3 H,0
gr >
. acetanilide | |
6. (i) LiH, CaH, : ionic hydrides
CH,
(ii) NH, SiH, : covalent hydrides
TT. Lithium is less electropositive than sodium and there-
CH3CH, —¢ —0O+CH,CHO
fore, carbonate of lithium is less stable than that of CHg
sodium. Li,CO, is not so stable to heat and therefore,
decomposes at lower temperature. This is because Le. Refer Chapter 13 Solved Example 14.
lithium being very small in size polarises a large CO,” 20. Refer Chapter 11 Solved Example 20.
ion leading to the formation of Li,O and CO,. On the
other hand, Na,CO, is very stable and decomposes at etl Fi. CH
higher temperature.
22. (6) CHCH=Cc% 7 7° : 8-Ethylpent-2-ene
8. Refer Chapter 11 Conceptual Questions, |1| Q. 9
\cH,CH,

g
O 23. (b) Refer NCERT File Chapter 13. Q. 1.
9. (i)CH,—CH—COOH
24. (b) CO
2NaNH, 2CH,Br
27. (c) (1) HC=CH _———=— NaC=CNa
Ethyne
CH,C=CCH,
But-2-yne

(i) CH,CH=CH, —#®"_, cy, Na, ether

Ha , CH3 Wurtz reaction

(Gi) OCH—CH,—CH,—CH,—CH,—CHO Propene Br


O
: yT CH, cH— cH— CH,
H CH, CH,
O 2,3-Dimethyl butane
10. (i) 3, 3-Dimethylpentane
(ii) 4-Hydroxy-3-methoxy benzaldehyde. (1tL) CH,=CH,
Bry, CCl,
4 _» CH,.—CH,
——#i——
2 ale. KOH
———>
13. Refer Chapter 12 Solved Example 36.
Ethene a te
18. (i) CH,—C=C—Ph #87", CH, CH= C—Ph HC=CH
OH Ethyne
——"CH;__CH,—C—Ph
.

WWW.JEEBOOKS.IN
HINTS AND SOLUTIONS

MOCK TEST -3
1. In aqueous solution, sodium carbonate gets hydrolysed
to give OH ions and hence it gives alkaline tests. (iii) HC=CH —3-3> CH,CHO —° >CH,COOH
CO,” +H,O —> HCO, +OH-
2. +2. 20h +CH,COONa
NaO, CaO
1 1 1
we) pH = Sak, + 5PK,- 5pm =7
CH,
C,H,OH,*, NH,NH,* 16. Refer NCERT File Chapter 13, Q. (a) 17 and (b) Q. 10.
endothermic
18. (i) Ca,N, (s) + 3H,0W)—> 3Ca (OH), +2 NH,
Peroxide ion has larger bond length.
Refer Conceptual Questions, Ch. 3. Q. 11. (ii) 2MnO, + 6H* + 5H,0, —>2Mn** + 8H,0 + 50,
Refer Chapter 2 Solved Example 53. (wii) 1, + H,O0, +20H- —- 215+ 2H,0 +O,
OID
Refer Chapter 1 Solved Example 93. 19. (a) Refer Chapter 2 NCERT File Chapter 8, Q. 21.
_ __AG°
10. logK = (b) Refer NCERT File Chapter 8, Q. 27.
-2.303 RT
— 8.1 20. (a) Refer Chapter 2 Solved Example 39
2.308 x 8.314 «107? x 1000 (b) 4
= 0.423
21. Refer Chapter 1 Solved Example 119.
S K = 2.64
12. BF, is trigonal planar having sp* hybridisation. BH,- 22.(b) —46.1 kJ mol
is tetrahedral having sp? hybridisation. 24.(a) 10 volumes
H (6b) (i) CaOQCl, + H,O, ——> CaCl, + H,O + O,
(ii) Fe,(SO,), + H,O, + 2NaOQH ——>
2Fe SO, + Na, SO, + O, + 2H,O
25. (c) MgO
26.(b) Refer Chapter 7 Solved Example 108.
(d) Refe Chapter 7 Solved Example 65.
Or
(c) 0.468
27.(a) Refer Chapter 13 Solved Example 37.
13. Refer Conceptual Questions Chapter 12. Q. 14.

15. (i) CH,CH, —“2>CH,CH,Cl .- CHA. / CH)CH,CH;


Ethane c=C
2Na, ether HW” Nx
Wurtz reaction CH3CHjCH»CH3 cis—
Butane

(ii) CH3CHg — CHACHN, SRO


Ethane
CH Pa:
=O

CH,=CH, —82-» CH,—CH, HH \cH,CH,CH,


br |Br trans—

cis-hex-2-ene has higher dipole moment and therefore,


2 ale. KOH will have stronger dipole-dipole interactions and
HC=CH
hence higher boiling point.
Ethyne
OOO

WWW.JEEBOOKS.IN
WWW.JEEBOOKS.IN
ORGANIC CONVERSIONS

WHAT HAPPENS WHEN ?

SOME ORGANIC COMPOUNDS POPULARLY KNOWN


BY THEIR COMMON NAMES
ar MODERN'S abc + OF CHEMISTRY-XI

Appendix
ORGANIC CONVERSIONS
A:Aliphatic Compounds
How will you convert?
. Methane to ethane
. Methane to ethanoic acid
. Ethane to ethene
. Ethane to ethyne
. Ethane to ethylene glycol
. Acetylene to acetic acid
. Ethene to ethyne
. Ethyne to propyne
Re
OWN
amar
omy. Ethene to bromoethene
. Propane to ethene
. Ethyne to 2-butyne
. EKthyne to propanone
. Ethanoic acid to ethane
fk
fh
fet
fake
m-
OoWhe. Ethyne to methane
. Ethene to methanal
. Ethene to ethanal
. Propene to 2, 3-dimethylbutane
. Propene to 1-propanol
. Ethyne to benzene
. Propyne to 1, 1-dibromopropane
. Ethyne to dichloroethanal
. Ethyne to 1,2 - dibromoethane
. Acetic acid to methane
. Acetylene to acetone
. Ethyne to oxalic acid
oe
ee a . Acetylene to 1 - butyne
i Tr
ee
eeWO
kr
oO
wm
Oe
Nr

bo =~]. Ethene to methanoic acid


28. Propene to 1— bromopropane
B :Aromatic Compounds
How will you convert?
. Benzene to acetophenone
. Toluene to benzene
. Benzene to benzoic acid
. Benzene to maleic anhydride
. Benzoic acid to benzene
. Benzene to m—nitrotoluene
. Benzene to benzene hexachloride
. Benzene to benzaldehyde
ak
a . Benzene to m-nitrobenzoic acid
WwW
eoComH
10. Acetylene to cyclohexane
A: Answers
1. Methane to ethane
Cl,
CH, Sunlight
CH,Cl Na, Ether
Wurtz reaction CH,CH,
Methane Chloromethane Ethane

WWW.JEEBOOKS.IN
APPENDICES

2. Methane to ethanoic acid


| Clo, Na/Ether
SS
On Sunlight a Wurtz reaction a
Methane Ethane

Cl, |Light

CH,CHO<———*— CH,CH,OH “2 cH,.cH,Cl


KMnO “c)

Ethanol Chloroethane
| KMn0O,

CH,COOH
Ethanoic acid

3. Ethane to ethene

| Cl, KOH(ale) . ~
CH,CH, i? CH,CH,Cl ee CH, =
ad = CH,

Ethane Chloroethane Ethene

4, Ethane to ethyne

|
CH,CH, 3 Cl,
jg? CH,CH,Cl| KOH (alc) | = CH,
CH,
Ethane Chloroethane Ethene

Br,

CH=CH <S0#@ _prCH,CH,Br


Ethyne 1,2-Dibromoethane

5. Ethane to ethylene glycol


Cly 4 __KOH (alc) _ __Alk, |
CH,CH, Sunlight CH,CH,Cl 7
—“e,_ 9/CH,, = CH, MnO, HOCH,CH,OH
Ethane Chloroethane Ethene Ethylene
glycol

6. Acetylene to acetic acid

HC= cH #05,
T
HC=CH == CH,CHO sae
Jxidation
CH,COOH
Acetylene Hg a Kes Acetaldehyde ° Acetic acid
Vinyl alcohol

7. Ethene to ethyne

CH,=CH, —25 9 BrCH,CH,Br “C2 , cH =cH


Ethene Dibromoethane Ethyne

8. Ethyne to propyne
NaNH CHB
cH=cH —S*2 , cH=CNa ——*“> CH=CCH,
Ethyne Sod. salt of ethyne Propyne

9. Ethene to bromoethene

CH,=CH, — > BrCH,CH,Br—@@"’


>cH = cH —@®">cu, = CHBr
Ethene 1,2-Dibromoethane Ethyne Bromoethene

WWW.JEEBOOKS.IN
ae tre MODERN'S abe + OF CHEMISTRY-XI
10. Propane to ethene

CH,CH,CH, —~?>
Cl,
CH,CH,CH,Cl—~*">
| KOH (alc)
CH,CH = CH,
Propane 1-Chloropropane Propene
|
Ozonolysis |0,

CH, = CH, <—2__4— CH,CH,OH <~*—~—


H,SO, H, (Ni)
CH,CHO
Ethene Ethanol Ethanal

11. Ethyne to 2-butyne

CH=cH —*2 ,Nac=CNa ——*—>CH,C=CCH,


Ethyne Di-sod. salt of ethyne 2-Butyne

12. Ethyne to propanone

cH=cH —2“® , cH=cNa —“3®", cH=ccH, [omy CH,COCH,


Ethyne Sod. salt of Propyne ip’ a Propanone
ethyne

13. Ethanoic acid to ethane

cH,cooH —N2°#_,cH,coONa —N20H#/@0 , cH,


3 | Fuse 4
—“2> cHcl
Light
Ethanoic acid Sod. ethanoate Methane Chloromethane

Na |Ether

CH,CH,
Ethane

14. Ethyne to methane

H KM
cH =cH ——*
HgSO, /HySO,
cu.cre, 8
3
cu.coon
3
Ethyne E Ethanoic acid

|NaOH

CH, <NaGHiCa9
Fuse
_ CH COONa
Methane

15. Ethene to methanal


O
CH,— = CH, Ds>.
Ozonolysis
CH,_,
-
H,O/Zn , sHCHO
CH,- —'29/@"
Ethene | Methanal
O O
Ethene ozonide

16. Ethene to ethanal


H,0,Ht
|
CH, =CH, BYy
—*> | 17
BrCH,CH,Brp,__KOH (ale)
—~~*"-> CH _= CH —{7, > CH,CHO
Ethene 1,2-Dibromoethane Ethyne Ethanal
APPENDICES

17. Propene to 2,3-dimethylbutane

CH,CH
| = CH, ats
HBr
i
|
ear
ICH,
|
T A a Sl
Na/Ether
ial:
Propene
Br CH, CH,
2-Bromopropane 2,3-Dimethylbutane

18. Propene to 1-propanol

CH,CH = CH, CH,CH,CH,Br


2!“ >CH,CH,CH,OH
Propene (R—O—O—R) 1-Bromopropane 1-Propanol

19. Ethyne to benzene

(Polymerisation)
Ethyne Benzene
20. Propyne to 1, 1-dibromopropane
cH,c=cH —#8", CH,CH=CHBr CH,CH,CHBr,
Propyne (R—O—O—R) 1-Bromopropane (R—O—O—R) 1,1-Dibromopropane

21. Ethyne to dichloroethanal

, HC1O
CH =CH ———-> [CH(OH) = CH(Cl)]
HClO.
——— > CH(OH),CHCI1, (Unstable) CHOCHCI,
HO
Ethyne Intermediate - Dichloroethanal
compound

22. Ethyne to 1,2-dibromoethane

CH =_ CH eaHy et Br. |
Pa/BaSO, CH, -=CH, —/@> 2
CibSrCH,Pr
Ethyne Ethene 1,2-Dibromoethane

23. Acetic acid to methane


cH,cooH —S®! ,cH,cooNa We cH,
Acetic acid , Methane

24. Acetylene to acetone

HC = cH“, Hc= CNa. —“8 5HC = C ~ CH, "+ >.cH, —C—CH,


Acetylene g504
Acetone

25. Ethyne to oxalic acid


CH AO] COOH

ay een ine CooH


Ethyne | ‘ Oxalic acid

26. Acetylene to 1 - Butyne


NaNH CH,CH.B
Hc=cH Nt, yo=cNa SP, yo =C- CH, CH,
Ethyne 1 — Butyne

WWW.JEEBOOKS.IN
| 16 MODERN'S abe + OF CHEMISTRY
-XI

27. Ethene to methanoic acid

a aie
O10.
Alk. KMnO,
cH, —cH,—
Nes
>HcHo—2!> ncoon
Ethene H Methanoic acid
Ethylene glycol

28. Propene to 1— bromopropane


CH.—CH= CH, == CH,CH,CH,Br
Propene pokes 1-Bromopropane
B : Answers
1. Benzene to acetophenone
COCH,
Anhyd. AICI
+ CH,coc]l ————3>
Acetyl chloride
Benzene Acetophenone

Toluene to benzene
CH, COOH COONa
O KMn0, oO NaOH __NaOH/CaO_,

Toluene Benzoic acid Sod. benzoate Benzene

Benzene to benzoic acid

CH, COOH
CH,Cl , KMn0O, ie
Anhyd. AICls, Oxidation |
Benzene Toluene Benzoic acid

Benzene to maleic anhydride


. CHCOOH CHCO
O) V0; —H,Ow * \
| ~~ 7? |l O
773K CHCOOH CHCO~
Benzene Maleic fad Maleic anhydride
5. Benzoic acid to benzene

OO ©
COOH COONa
——
N O)

Benzoic acid Sod. i. ae Benzene

6. Benzene to m-nitrotoluene

NO, NO,
HNO3/H,SO, CHC!
Heat Anhyd. AICI, CH,

Benzene Nitrobenzene m-Nitrotoluene


APPENDICES

7. Benzene to benzene hexachloride

Cl

——_,:
cH
CI—HC CH—Cl
+3cl, —Sunlieht_, | |
7 CI—HC AHA!
enzene CH

|
Cl
Benzene hexachloride (BHC)

8. Benzene to benzaldehyde
CH, CHO

eq) Anhyd. AICI,


1.
Chromyl
Toluene chloride Benzaldehyde
9. Benzene to m-Nitrobenzoic acid
CH, COOH
CH,Cl 6 KMnO,
, Anhyd. AICI, Oxidation
Toluene Benzoic acid

HNO,

COOH

© NO,
m-Nitrobenzoic acid

10. Acetylene to cyclohexane

SHC = CH __Cutubes
tubes
—Cu tubes _4
, Olan
250)
Polymerisation
Benzene Cyclohexane
ae rr MODERN'S abc + OF CHEMISTRY-XI

Appendix 'D)

Ethylene is passed through alkaline potassium permanganate solution.


2-Butene is treated with ozone followed by zinc dust and acid.
Propene reacts with water in the presence of mineral acid.
Ethyl chloride is treated with alcoholic caustic potash.
Benzene is treated with methyl! chloride in the presence of anhydrous aluminium chloride.
Ethyne is bubbled into ammoniacal solution of cuprous chloride.
Propene is treated with HBr in the presence of peroxides.
Water is dropped over calcium carbide.
Water is added on ethyl magnesium bromide.
Benzene is heated with mixture of conc. HNO, and conc. H,SO,.
bEeS Toluene is treated with hot solution of potassium permanganate.
Pe
ee
ee
ee
Sodium acetate is heated with soda lime.
. Ethyl iodide is reduced.
Acetylene is heated to 773 K in copper tubes.
. Ethylene dibromide is treated with zinc dust.
onoO
bo
Oop
andl
eat
aul
Sell . Benzene is treated with Cl, in the presence of FeCl,.
17. Ethylene is treated with ozone.
18. Benzene is treated with chlorine in the presence of sunlight and absence of halogen carrier.
19. An aqueous solution of sodium salt of succinic acid is electrolysed.
20. Methane is mixed with oxygen and passed through heated molybdenum oxide.
bo jh. Solution of ethylene in carbon tetrachloride is treated with chlorine at room temperature.

. Propene is treated with hydrogen in the presence of Ni or Pt.


bo. Propene is treated with Baeyer’s reagent.
boQo
24. Styrene is polymerised.
25. Ethene is treated with excess of oxygen.
26. Propylene is treated with excess bromine in CCl,.
27. Ethyne is oxidised with alkaline KMnO,,.
boOo . Propyne is treated with water in the presence of sulphuric acid and mercuric sulphate.
. Acetylene is treated with ammoniacal silver nitrate solution.
. Benzene is treated with sulphuric acid in the presence of sulphur trioxide.
. Benzene is treated with acetyl chloride in the presence of anhydrous aluminium chloride.
. Benzene is treated with hydrogen in the presence of Ni or Pt at 473-573 K.
. Toluene is oxidised with chromy] chloride.
2-methyl -2-butene is treated with ozone followed by reduction with hydrogen in the presence of zinc.
by
w
wt
www
OO
AahRoONE
. Mixture of methane and oxygen is compressed to 120 atm and passed through copper tubes at 475 K.
36. Ethane is treated with nitric acid in vapour phase.
Answers
1, Pink colour of KMnO, solution gets discharged and ethylene glycol is formed.
KMn0,
CH, = CH, + H,0 + O————* CH,—CH,
Ethylene
OH OH
Ethylene glycol

2. Acetaldehyde is formed. O.
CH, —CH = CH—CH,—*~>CH,-CH CH—CH, —, 5? 2CH,CHO
O “. Z

2-Butene | | 2 Acetaldehyde

WWW.JEEBOOKS.IN
APPENDICES

3. 2-Propanol is formed.
H,O |
CH,CH = CH, —*, > CH,—CH—CH,
Propene
2-Propanol

4, Ethene is formed.

CH,CH,Cl + KOH (ale) ——-—-> CH, = CH, + KC] + H,O


Ethyl chloride Ethene
5. Toluene is formed.
CH,
O) + CH, C] Anhyd.
AICI,
Benzene nile

6. Ared precipitate of dicopper acetylide is formed.


CH=CH + 2 [Cu(NH,),]C1 ——-> CuC =CCu + 2NH,Cl + 2NH,
Ethyne Red ppt.

7. 1-Bromopropane is formed.
CH,CH = CH, + HBr __Peroxides_, CH,CH,CH,Br
Propene 1-Bromopropane
The reaction proceeds according to anti-Markownikoff 's rule.
8. Acetylene is formed.
CaC, + 2H,O ——-> HC=CH + Ca(OH),
Cal. carbide Acetylene
9. Ethane is formed.
C,H,MgBr + H,O ——> C,H, + Mg (OH)Br
Ethyl magnesium Fthane
bromide
10. Nitrobenzene is formed.

NO,
H,SO,
+HNO,—2 +> (O} +H,0
Benzene
Nitrobenzene

11. Benzoic acid is formed.

CH 3
COOH

+ 3(0) ——
KMnO,
+> HO
Heat

Toluene Benzoic acid

12. Methane is formed.

CH,COONa + NaOH —“°-> CH, +Na,CO,


Sodium acetate Methane
13. Ethane is formed.
re Na, alcohol
CH,CH, I + 2H —~———-> CH,.CH, + HI
Ethyl iodide Ethane
14. Benzene is formed.
733 K
3CH = CH —————— ©
Acetylene Copper tubes
Benzene

WWW.JEEBOOKS.IN
80 MODERN'S abe + OF CHEMISTRY-XI
15. Ethene is formed.
CH,BrCH,Br —“> CH,=CH, + ZnBr,
Ethylene dibromide Ethene
16. Chlorobenzene is formed.
Cl
O . cB oO
Benzene Chlorobenzene

17. Ethylene ozonide is formed.


O

CH,=CH, +0,» H,C CH,


Ethene | |
O O
Ethylene ozonide
18. Benzene hexachloride is formed. C]
|
CH
ica a 4

+3cl, et, CHC CH-Cl


NcH~
Benzene
#!
Benzene hexachloride
(B.H.C.)
19. Ethene is obtained.
CH,COONa . CH,COO- | CH,COO

CH,COONa 7
Naa —h™
Electrolysis —2e7 |
CH,COO- CH,COO

Disod. succinate
|-2co,

CH,
||
CH,
Ethene

20. Formaldehyde is formed.

CH, +0, —MeO,. HCHO + 4H,O


Methane | Formaldehyde

21. 1,2-Dichloroethane is formed.


CH, =CH,+Cl, bc J CH,-CH,
——*>
Ethylene Cl a
1,2-Dichloroethane

22. Propane is formed.


CH, - CH=CH, +H, ae CH,—CH,—CH,
Propene Propane
23. Propane-1, 2 diol is formed.
| _ Alk. KMn0,
CH, —-CH=CH, + H,O0+0O ae CH.—CH—CH,
Propene | |
OH OH
Propane - 1, 2-diol
APPENDICES

24. Polystyrene is formed.


CH=CH, Polymerisation —_ CH- CH, —

n
n

Styrene Polystyrene

25. Carbon dioxide and water are formed.


C,H, + 30, ——> 2CO, + 2H,O
Ethene Carbon Water
dioxide
26. 1, 1, 2, 2 - Tetrabromopropane is formed.
Br Br
Br Bro | |
CH, CH=CH —_— CH, HC=CH —__> CH C— CH
Propylene
cei, 3 | |
CCly 3 | |
Br Br Br Br
1, 1, 2, 2-Tetrabromopropane
27. Oxalic acid is formed.
CH Alkaline COOH
ll + 40 aro |
| “4 COOH
Ethyne Oxalic acid
28. Propanone is formed.
| H,SO,, 3
H,C—C i = CH 2 Rearr angem ent CH |
CH,C =CH + H,0— 7,
= —___£ #4 »
on wa Ae

Propyne
Propanone
29. Disilver acetylide is formed.
HC =CH + 2 [Ag(NH,),] NO, » AgC =CAg + 2 NH,NO, + 2 NH,
Acetylene Ammoniacal Disilver
silver nitrate acetylide
30. Benzene sulphonic acid is formed.
5O,H

+ H,SO ald ae H,O


Benzene _ Benzene
sulphonic acid
31. Acetophenone is formed.
COCH,

+ CH,COC] SS [ + HCl
3
Benzene Acetophenone
32. Cyclohexane is formed.
CH;

Ni ; or Pt , LA:
H, CH a
O) + 3H, 473-573 K oe | I ll
peneene Cyclohexane a se 2
CH,

33. Benzaldehyde is formed.


CH, CHO

Oxidation , O

Toluene Benzaldehyde

WWW.JEEBOOKS.IN
la MODERN'S abe + OF CHEMISTRY
-XI

34, Ethanal and propanone are formed.


O
CH, . Pa \, jo CHa H,
CH, CH= €CH, ae | <a =
O
——>CH,CHO +CH,COCH,
H~ | | CH, . Ethanal Propanone
CH, O O
2-Methyl-2-butene Peale

35. Methanol is formed.


2CH, 4 +0, - —~3
Cu tubes
> °CH,OH
Methane 475 K Methanol
36. Nitroethane is formed.
CH,CH,+HNO, —Heat_,
Heat
CH,CH,NO, + H,O
Ethane Nitroethane
APPENDICES

Appendix
SOME ORGANIC COMPOUNDS POPULARLY KNOWN BY THEIR COMMON NAMES

Acetylene Glycolic aldehyde HOCH,CHO


Acetone Glyceraldehyde HOCH,CHOHCHO
Crotonaldehyde CH,CH = CHCHO
Formic acid HCOOH
Acetaldehyde
Acetyl acetone CH,COCH,COCH,
Formaldehyde
Valeric acid C,H,COOH
Neo-pentane
Malonic acid a _- COOH
“COOH
Oxalic acid COOH
Vinyl! chloride |
COOH
Crotonic acid CH,CH = CHCOOH
Suecinic acid CH,COOH
|
Tert-amyl alcohol CH, - C- CH, - CH, CH,COOH
| H- ‘~ COOH
OH Maleic acid

Iso-butyl bromide CH, - CHCH,Br H-C—COOH


(cis)
CH,
Acetonitrile CH,CN Fumaric acid H-C—-COOH
|
Allyl alcohol CH, = CH - CH,OH HOOC -H
Glycerol CH, - CH- CH, (trans)
| | | Malic acid CH(OH)COOH
OH OH OH |
Allyl bromide CH, = CH — CH,Br CH,COOH
Allene CH, = C =CH, Tartaric acid CH(OH)COOH
Propynyl iodide CH = CCH,I |
CH(OH) COOH
Crotyl bromide CH,CH = CHCH,Br
Citric acid |HOOCCH,C(OH)\COOHCH,COOH
Acrylic acid CH, = CHCOOH
Glycine NH,CH,COOH
Acrolin CH, = CHCHO
Lactic acid CH,CH(OH)COOH
Ethylene glycol CH,—CH,
Glycolic acid CH,OH
OH On |
COOH
Propylene glycol CH,CH——CH,
| | Glyoxalic acid CHO
OH OH |
COOH
Diazomethane CHN, Acetamide CH,CONH,

WWW.JEEBOOKS.IN
WWW.JEEBOOKS.IN
LOGARITHMS Log,,N
iy
MEAN DIFFERENCES
_

boaOo
Bo
bo oo
=]
Gy
th
ff

boBo
ho
BOOo
ta

ean

SS

MW
fk.
mk
Hon
UW
UOMO
HoOMUoMOn
oOo
WnmW
Wn
WO
m&
nM
Oh
&
>

Oo
no
bo
bBo
Bo st
eo
rs
=
ee
ss —y="
we
wm
eo
2
©

tReet
Ze
2c
eAero
2
2

= in
cr

2 an

2
1 Oo
OF
09 &
2
ao
=]

oe
oe
ee oo
a ee
wowaa &nana
wo
Oo
oO
ao
rm
©

AZAR
OC
FQ
EBs
e
aBo
bo
be mi
:¢6 on
om
ch
of
e =~
=~}
=)

=

ee
a en
oeee
Ge
Go
a te
fe
fm
fet ch
naan

ete
he
fe

6902 oo
a ob
oN
obo
bo co
oF
oo
6 aa anaan
i bo 3 =] oo =
MEAN DIFFERENCES

Example : Log 473.7 = 2.6749 (for 473) Log 0.004516 = 3.6542 (for 451)
+6 (mean diff. for 7) +6 (mean diff. for 6)
2.6755 3.6548
[Le Viantissa
haracteristic

(Z)
CCH ERESER EIENKSESEEER bs
MEAN DIFFEREN —
we oo

NN bo
ee oe
eo
bo aacn
ch
cm
en os
mM
mao4nowy
2w]e
1JI4

so
ts
Bo

FCaRes
SaUIG8R ee
en ht bo
Mab aa ff
cn
Ch
oh
a cn
oom
cm
choc
nAana
es

Soy
OAR tt
Seer aa Bo
BS
ba
bobo GO
G0
Oo
if
i ee
ee cn
oi
ti
cm
ot oh
Ga
G3
tn
Ot

FA
aaee
ee be
bh
ob
bo ttf
Jot
Join aa
Yotinffeatinf cn
Cn
me
f. oth
ch
cn
en So
ch
Oo
G&
&

on
om
ot
Z2rasot
i
et el
aoe
eel
a a
se co
C8
fo
OF
Oo oo
ol fe
ee
ee ah
a
or
ot
Cr ot

D
I
F et
ee
stel
el a Go
Co
fo
Oo oo
OF
09
Oo | cn
Ch
oe
ee oom
or
on
cn
tm

F
EK
R |

EK
tt
ol
oe
i
oe Ge
A OF
Oo
Ge
OG
& oo
fo
Oo aa ee
eeon
in
N
C
EK

eocrer
ol
oe
el
a aS
be
ho
& ce
oh
be me
Go
Oo
oFfe
Go
co Hes
ee fe
pe
ly
i

Seo
&
&& oe
ee
ee ee
a ob
be oo
Wo fa
Go
fo
Oo ee
ee aa

i? a A Hes Zaal\\ode edd os


i 245, 4 ae lier oe Ue
oar
ell
Ae
ea i. a7 4 hes aei| | Rake ade! <a
S44 “4 BS 9 al|| “alae de oe
9965 a Aa =3. 4
aS eae a Seas eS
MEAN DIFFERENCES

Some Rules

log (a) x (b) = log a + log b log (a)" =n log (a)


log (a) | (b) = log a—log b In x = 2.303 logx

(it)
ANTI-LOGARITHMS
MEAN DIFFERENCES
1 2
o)6OU0 2 2
o)6hOUO 2 2
A o)6hOUO 2 2
D o)6hOCUo 2 2
D o.61 2 2
o 61 2 2 2
N o)61 9 2 2
o 61 2 2 2
U m4 2 2 B
M o 61 9 2 8
B of 1 FSD Se aa ee
E o 1 je ee | |e a me
7 4 Te 285 Re, |e, tae
R Tel 12 2/2 83 8
S o 661 ee So GPs eae ae
Cheer ¥ i2 232/2 8 8
I 006061 ee ae |||” Aes One
o 1 dps! 3h es |e Pee et
N 061 1. 99° 98. oe oe
o 61 fee ae |||| ee ae Oe
M o 61 yA el |e es ae
E o 61 a. Sy) a. line Waa: a
o 061 BO oo Gee ore. Sue a
A o 61 rN el ee ee er
N o 61 Di Ne ieee bap aoe
te * ik a. Oo” ee lie, “ale Oe
D o 61 Bs gp, “Say Woes Wo: we
o 61 geome (aE ll cae oeakS ° sot
I Tee Bo Gry Gat leg’, tere
F o 61 re cme a lees ee ee
F o 61 oo ue onde liars va 28
E o 061 Be Su” ea linea” ae a
o 061 itm Eg 2A, Mince, eee) cot
R dpe pal BO Ge eet |ieae) ey Sg
E ae | a. ae. a) lina dae 7G
N i 9 8 3 ]4 4 5
C ie = 428 Beg) || a. oa? OG
i it a a, Saat linet, py pe
E i, 4 Be 2a = SATs || Sa? oe Pi
1 1 rae: eee a) |e eee ae
C i) ae eee || ee eee
O i" 8 Oe ae. thle oa, OG
L i Bo > SN a Ge ke
1 ee eee eee On| Wer ee eer
U i 8 ee ee ee 6
M ie “J = es caer Oa |e 6
N 1 vi a ee oe ee 6
S os | =eae ee a) [eee 6
ie 8 4 || 6
iy 38 ae cee ay Ulee G

Peer? t ti
SSS eS ae SS ae ae ae ee

Example: Antilog 1.3377 = 21.73 (for .337)


+3 (mean diff. for 7)
21.76 (decimal after 2 s.f.)
Indicates that the decimal point must be placed after the first 2 figures.

(iti)
ANTI-LOGARITHMS
MEAN DIFFERENCES
3 4 5 6 7 8 9

Hy
eS
He
ee be
ee
ie ot
te
Bo o
a
cn
con
ce

a Bo
Ba &o
fo
6bo
© oe
Brae
Broce

bo
obo

aa
i oo
co
eo ee
a

aee
ob
obo
Bo
ND Go
fe
Ga
G5 om
co
4m

& ee

& Heam

9 11
9 a jt

he
Zerne
HH
Zam
z
oD a Of
CO
ba
bo fe
fa
ef 9

1 rt
re
1 fo

1 mH
bso
co
bo

1 j=Oo

1 fo
Oo
RY fe
a
ee a Oo

oo
OF
Oo abe
tm
om
chor

AZAR
OC
Ae
FO
see
co
Co
C3
Fm
Bm oem
tm
chor

be
obo
bh
oM
ho i
i oH
ci
na
arm

ee
ee
A
ie ee &
37
yo |
i

=etots {ets t«+ts tetrts |e MEAN DIFFERENCES


Example : Antilog 2.6459 =0.04416 (for .645)
+9 (mean. diff. for )
O.004425
Asimple rule:
(7) Ifthe characteristic is positive, then put the decimal after (n + 1) digits from the left
(iz) Ifthe characteristic is negative, then put (n — 1) zeros before the first significant digit

(iv)
Useful Relations

T= 3.1416 InX = 2.3038 log X 2.303 RT/F = 0.05916 V

e = 2.71828 2.303 RT = 5709 J mol (at 25°C)

(at 25°C)

Selected Greek letters Some Useful Conversion Factors

alpha as 1000 gram


beta 10 decigram
camma A 100 centigram
delta 1000 milligram
epsilon M 1litre = 1dm'?= 10? cm
eta 1eV = 1.6022«10"J
theta oI ileal = 4.184J =4.184 x 10’ erg
kappa 1f) “em 10'£re
lambda 1 L. Airs YA.217 cal = 101.52 J
Mul i. <—e#m=
SS
fe
oops
2a 1A = 10%cm=10"m=0.1nm
latm = 760nmHg
1.013 x 10° pascals
1 bar 10° pascals

Quantity

Speed of light 2.9979 x 10°ms?


Atomic mass unit 1.6606 x 10°’ kg

Avogadro constant 6.022 x 107? mol


Planck constant 6.6262 x 10-* JK?
Faraday constant 9.64846 x 10°C mol
Boltzmann constant ae) eee
Charge-to-mass ratio of electron fos «eo ke
Electronic charge e 1.60219 x 10°" C
Gas constant 7 8.314 J mol K"
Molar volume (STP) 22.414 x 10“ m* mol
22.414 dm? mol
Mass of electron 9.10953 x 107%! ke
Mass of proton 1.67265 x 10°" kg
Mass of neutron 1.67495 x 10°’ kg
Rydberg constant 1.096 x 10’ m?
WWW.JEEBOOKS.IN
The book in your hands is strictly based upon the syllabi prescribed by C.B.S.E., New Delhi and Educational
Boards of other Indian states. It has been written according to N.C.E.R.T. pattern keeping in view the changing
trends of different examinations. Thus this book has been very popular among the teachers and students all
over India and praised for its clear presentation, effective approach of solving numerical problems and attractive
figures.

Simple language and easily reproducible diagrams.


Large variety of SOLVED NUMERICAL PROBLEMS.
Additional numerical problems under the heading PRACTICE PROBLEMS for self assessment and
practice.
Advanced Level PROBLEMS with solutions to accelerate
the potential for solving Numerical Problems.
REVISION EXERCISES in the form of "Very Short Answer Type’, ‘Short Answer Type' and ‘Long Answer
Type Questions’ with HINTS and SOLUTIONS to some questions Is also included.
CONCEPTUAL QUESTIONS are solved at the end of each chapter.
Additional information is given under ‘Learning Plus’,'Key Note’ and "Watch Out’.
For Quick Revision of the chapter objective type questions in the form of ‘True or False’, ‘Complete the
Missing Links' and ‘Choose the Correct Alternative’, under the heading Memory Test along with their
answers, are given at the end of each unit.
COMPETITION FILE which includes ‘Additional Information’, "Higher Order Thinking Skills’ (HOTS),
Advanced Level (with answers) and ‘Multiple Choice Questions’, is also given. This will prepare students for
various competitive entrance examinations for entry to Medical, IIT and other Engineering colleges.
COMPLETE coverage of previous years’ questions from all the board examinations and competitive
examinations such as AIPMT/NEET, JEE (Main), JEE (Advance) and other State Boards.
Ina NUTSHELL the book provides Excellent Guidance to students for Boards’ examinations as well as
for competitive examinations for entrance to professional colleges.

ISBN O9YESS5184121 q

9N7 893510841210

You might also like